88% found this document useful (24 votes)
55K views853 pages

GRB Physics For Competitions Vol 1

This Book is an excellent e-book for those appearing in various competitive examinations including the Joint Entrance Examination (JEE) and other entrance examinations to various other reputed engineering institutes, colleges, and universities.

Uploaded by

Shubham Gupta
Copyright
© © All Rights Reserved
We take content rights seriously. If you suspect this is your content, claim it here.
Available Formats
Download as PDF, TXT or read online on Scribd
88% found this document useful (24 votes)
55K views853 pages

GRB Physics For Competitions Vol 1

This Book is an excellent e-book for those appearing in various competitive examinations including the Joint Entrance Examination (JEE) and other entrance examinations to various other reputed engineering institutes, colleges, and universities.

Uploaded by

Shubham Gupta
Copyright
© © All Rights Reserved
We take content rights seriously. If you suspect this is your content, claim it here.
Available Formats
Download as PDF, TXT or read online on Scribd
You are on page 1/ 853

G.C.

Agrawal
A New Pattern Text Book of

Competitions
Vol. I

A New Generation
Book for
Competitive Entrance
Examinations
G. R. BATH LA PUBLICATIONS PVT. LTD.
DELHI·MEERUT
A New PaUern Text Book of

PHYSICS
for
Competitions
Volume -I

Thoroughly Revlsfnl Edition

G.C. Agrawal
PREFACE to the Eighteenth Edition ===========

Ever since the first volume of this book appeared in 1996, it has been so well accepted by the students and
teachers that its eighteenth edition is now in your hands with its present updated form.
We are indebted to Dr. KK. Chopra, Dr. A.K. Garg, Dr. P.K. Sharma, Er. Anurag Misra, Er. RK. Verma (Kota),
Er. s,c. Pandey (Allahabad), Mr. Raj Kushwaha (Kanpur), Mr. Virendra Yadav (Ghaziabad), Mr. B.K. Mataria (Kota), Mr.
Chandra Shekhar Sharma (Kota), Mr. Sudeept ChatUlvedi (Kotal. Mr. Amit Gupta (Kota), Mr. Ashish Srivastava (Kanpur) ,
Dr. D.V. Sardesai (Kanpur), Mr. K.K. Tyagi (Nagpur), Prof. H.S. Outta (Nagpur), Mr. Mohit Mehta (Nagpur), Mr. K.L. Vara
Prasad (Guntur), Mohd. Shahjahan Basha (Guntur), Mr. P. Mahesh (Nellorel, Mr. M. Ramesh Kumar (Vijayawada),
Mr. Vikas Rampal (BhiJai), Dr. K.v. Rao (Bhilai), Er. Rahul Srivastava (Gwalior), Mr. Rakesh Ranjan (Gwalior),
Mr. Shashi Bhushan (Jabalpur), Er. Surendra Singh (Bokaro Steel City), Dr. Saurabh Kumar (Ranchi), Prof. Pratyush Kumar
Singh (Ranchi), Mr. J .K. Bhardwaj (Ranchi), Er. Vikas Tank (Jamshedpur), Dr. K. Srinivasachary (Hyderabad) , Mr. Aditya
Sachan (Hyde rabad), Mr. S·.P. Sharma (Hyderabad), Mr. M. Srikant (Hyde rabad) a nd Er. J .B. Padhee (Bhubaneshwar) for
the useful suggestions given from time to time for improvement o f this book. We are highly grateful to Dr. Anupam Arora,
Dr. O .P. Tane;a and Mr. R.K. Singh for the invaluable labour in revising the book and thereby further enhancing its
usefulness.
We are thankful to S hri Manoj Kumar Bathla and Shri Vishvnath Bathla, Directors, G .R. Bathla Publications Pvt.
Ltd. a nd staff members for the whole-hearted co-operation in the preparation a nd publication of this book. We have
put our best efforts to make this book up-ta-date for its readers.
Suggestions for further improvement of the book will be gratefully received.

M ay, 2011 SHASHI AGRAWAL


PRATEE KAGRAWAL
PRAROOP AGRAWAL
PREFACE

During the last thirty years of his 'competition career' the author found it difficult to recommend a 'text hook' on physics
for competitions which might provide adequate theoretical knowledge along with proper numerical problems and standard
multiple choice objective questions all arranged systematically. This book "Physics for Competitions" is an attempt in Ihis
direction and meant for students preparing for various entrance examinations like lIT, Roorkee. MNR. CPMT, CSSE.
EAMCET, etc. In this book :
.:. graphical analysis along with integral and differential calculus is used freely, as in the opinion of author physics cannot be dealt
properly without their use .
•:. SI units are adopted throughoullhe book. However along with 5 1 units other systems of units or practical units have also been
frequently used in the book as the author finds it more convenient to use them (e.g., calorie instead ofjouleorcm instead of m) .
•:. subject matter(theory) is logically and thoroughly developed and discussed along with derivations and proofs so that students
may understand the basic concept properly.
•:. to make the subject matter up-to-date and advanced, a large number of central notes and foot notes havc been incorporated
where necessary. These contain either higher and lower level subject matter in brief or extension and alteration possible in a
given question or problem .
•:. here and there new techniques which are short and sharp have been used to solve the ex isti ng problems. Also whenever and
wherever possible many alternative solutions to the same problem are provided to make the subjcct well-understood .
•:. large number of carefully and critically selected questions and problems from different competition papers and books wilh
proper solutions, answers or hints are arranged systematically so that students may develop the aptitude of attacking and
attempting new questions himselfby applying the basic principles of physics logically.
The authorshall feel amply rewarded if the book is able to qucnch the thirst of the discerning readers in some measure.
It shall also be highly appreciated ifreaders either point out misprints (which inspite of best possible efforts might have crept in)
or suggest precise alternative solutions to any of the problems and questions given in the book.
In the end the author expresses his gratitude to all those who have in anyway helped him in bringing out this book in this fonn.

5th February, 1996 C.C. Agrawal


CONTENTS
Chapters
"' 0 Subjective Type Questions
UNITS AND DIMENSIONS 1-24
§ 1.1 Physical Quantities II FORCE AND TORQUE 53-125
§ 1.2 Magnitudes I
§
§ 3.1 Force 53
1.3 Fundamental and Derived Units 2
§ 1.4 Standards of Length, Mass and Time § 3.2 Translatory Equilibrium 56
IAI Length 3 § 3.3 Strings 57
[BI M." 3 § 3.4 Springs 65
[C] Time 4 § 3.5 Friction 70
§ 1.5 Systems of Units 4 § 3.6 The Basic or Fundamental Forces 80
§ 1.6 Dimensions and Dimensional Formulae 5 § 3.7 Impulse 81
§ 1.7 Appli9utions of Dimensional Analysis § 3.8 Torque 82
[In Mechanics] 6 0 Misce ll aneous Solved Problems 86
§ 1.8 Limitations of Theory of Dimensions 10 0 Exercise :
§ 1.9 Theory of Dimensions Applied to Heat 10 • Only One Choice is Correct 91
§ 1.10 Theory of Dimensions Applied to Electricity 11 • More thall aile Choice is Co rrect 107
0 Exerc ise : • Asserlioll-Rea.wn Typ e Questions 109
• Ollly Qlle Choice is Correct 15 • hlfeRl!J" Type Questiol1 s 1\0
• More tIll/II Olle Ch oice is Co rrect 19 • Match the COIIfIllIl.\' l\l
• IlITeger 7)'pe Questions 20 0 Linked Comprehension Type 1\4
• M(I{cli the CO/UfIIIlS 20 0 Subjective Type Questions 117
0 Li nked Comprehension Type 22
0 Subjectivc Type Questions 22 II CONSERVATION OF ENERGY 126-165
D VECTORS 25-52 § 4.1 Work and Power 126
§ 4.2 Energy 132
§ 2.1 Vectors 25 § 4.3 Kinetic Energy 133
§ 2.2 Addition of Vectors 27 § 4.4 Potential Energy 133
§ 2.3 Resolution of Vector into Components 30 § 45 Mechanical Energy 134
§ 2.4 Multiplication or Division of a Vector by § 4.6 Conservation of Energy 135
a Scalar 31 § 4.7 Modem Views about Energy
§ 2.5 Scalar Product of 1\vo Vectors 32 [AI Mass-Energy Conservation 140
§ 2.6 Vector Product of 1\vo Vectors 33 fBI Quantisation of Energy 140
§ 2.7 Scalar Triple Product 36 0 Miscell aneous Solved Proble ms 141
0 Miscellaneous Solved Problems 36 0 Exercise :
0 Exercise : • Ollly Olle Choice h Co rrect 146
• Dilly Olle Choice is Correct 39 • Mo re thall Olle Choice is Correct 156
• More than Dlle Choice i,l' Correct 47 • A ~'se rtiOIl-Re(/,I'OIl Type Questiolls 157
• AssertiOfl -Re{/ ~'OI1 Type Questions 48 • llIteger Type Questioll s 158
• Integer Type Questions 48 • Match the CO/UIIIIIS ISS
• Match the Columns 48 0 Li nked Comprehension Type 160
0 Li nkcd Comprehension Type 49 0 Subjec tive Type Questions 162
El CONSERVATION OF MOMENTUM 166-221 • Asserlioll-Reasoll Type Questions 288
• Integer Type Quesliolls 289
§ 5. 1 Momentum 166 • Match tIle Columl/s 290
§ 5.2 [AI Law of Conservation of Linear-Momentum 168 0 Linked Co mprehe nsio n Type 293
[B] Some Applications 169 0 Subjective Type Questions 296
§ 5.3 Collision of Impact 172
§ 5.4 Collision in One Dimension D ROTATORY MOTION 303-382
{A] Elastic collision 174 § 7. 1 Kinematics of Circu lar and Rotational Mot ion
[BI Inelastic collision 175 [A] Variables of Motion 303
§ 5.5 Centre of Mass 180 (a) Angular Displacement 9 303
§ 5.6 Motion of Centre of Mass 182 (b) Angu lar Velocity 0) 303
0 Miscellaneous So lved Problems 186 (c) Angular Acceleration ex 304
0 Exerc ise: [B] Relations among Angular Variables 304
• Dilly Olle Choice is Correct 197 [el Relations among Linear and Angu lar
• More Illall One Choice is Correct 210 Variables 305
• Assertion-Reason Type Questions 212 § 7.2 Dynamics of Uni form Circu lar Motion
• Imeger Type QuesJioTls 213 [AI Banking of Rails and Roads 309
• Match the Columns 214 [B] 'Death well' or Rotor 309
0 Linked Comprehension Type 215 [C] Conical Pendulum 309
0 Subjecti ve Type Questions 217 § 7.3 Dynamics of Non-Uniform Circular Motion 314
Condition for Oscillation or Leaving the Circle 315
I I TRANSLATORY MOTION 222-302 § 7.4 Rotational Dynmllics
§ 6. 1 Motion and Rest 222 IAI Moment of Inertia (or Rotational Ine rtia) 321
§ 6.2 Variables of Translarory Motion [B] Kinetic Energy, Work and Power in
[A] Distance and Displace ment 222 Rotational Motion 323
[BJ Average Speed and Velocity 223 [el Rolling Motion 323
[C] Instantaneous Speed and Velocity 224 § 7.5 Rigid Body Dynamics 325
[0] Average and Instantaneolls Acceleration 225 § 7.6 Angular Momentu m and its Conservation 333
§ 6.3 Relative Velocity § 7.7 Centre of PerCllssion 335
228
§ 6.4 Relations among Variables of Motion 231 0 M iscellaneous Solved Problems 343
§ 6.5 Motion Under Gravity 0 Exercise:
[AJ Ideal Motion 236 • Dilly Olle Choice is Correct 349
[B] Effect of Medium on Motio n under Gravity 238 • More t"all One Choice is Correct 365
• Al'sertion-Reasoll Type Qllesliolu 367
§ 6.6 Projectile Motion (on a Horizontal Plane) 243
• Integer Type Qllestiolls 368
§ 6.7 Applications of Calcu lus in Deriving
• Match the Co Il/mil S 369
Kinematic Relationships 255 372
0 Linked Comprehensio n Type
§ 6.8 Newton's Laws of Motion 256
0 S ubjective Ty pe Questions 375
§ 6.9 Mass and Weight
[A] M,ss 259 B SIMPLE HARMONIC MOTION 383-433
[BJ Weight (True) 260
[el Apparent Weight § 8.1 Simple Hanno nic Motion
260
0 Miscellaneous Solved Problems 263 [A] Harmonic, Oscillatory and SHM 383
[B] Equation of Simple HamlOnic Motion 384
0 Exerc ise :
266 [e] Characteristics of SHM 384
• Only One Choice i.f Correct
• More thall Dlle Choice is Correct 286 [OJ Some Physical Quantities of Intercst in SHM 385
(a) Displacement 385 (a) Orbital Velocity Va 445
(b) Velocity 385 (b) Time Period T 445
(c) Acceleration 385 (c) Angular Momentum L 446
(d) Energy E 386 (d) Energy 446
§ 8.2 Simple Pendu lum 390 (e) Weightlessness 447
§ 8.3 Spring Pendulum 395 0 Miscell aneous Solved Problems 451
§ 8.4 Some Other Examples of SHM 400 0 Exercise :
[A] When RestOling Force is due to • Ollly Olle Choice is Correct 453
Tension or Elasticity 400 • More thaI! One Choice is Correct 468
[BJ When Restoring Force is due CO • Assertion-Reason Type Questions 470
Hydrostatic Pressure or ThlUst 401 • Integer Type Qllestioll s 470
(C] When Restoring Force is due to Gravity • Match the Coilimns 470
or Gravitation 402 o Linked Comprehension Type 471
[D] When Restoring Force is Electrical 403 o Subjecti ve Type Questions 474
§ 8.5 Superposition of Two SHMs of Same
Frequency in the Same Direction 404 III FLUID MECHANICS 478-522
o Miscellaneous Solved Problems 407 § 10.1 Density and Pressure
o Exercise: [A) Density 478
• Oilly One Choice is CO/Tecl 4 11
[B I Pressure 479
• More Ihall Dlle Choice is Correct 423
§ 10.2 Archimedes' Principle and Buoyancy 482
• Asserlion-Reason Type Questions 425
§ 10.3 Floatation
• Inleger Type Que.stions 426
LAJ Translatory-Equilibrium 485
• Malch Ihe Columns 426
o Linked Comprehension Type 427 [BI Rotatory-Equilibrium 486
o Subjective Type Questions 429 [C] Applications 486
§ 10.4 Hydrodynamics
II GRAVITATION 434-477 [AJ Principle of Continuity 491
[BI Bernoulli's Theorem 491
§ 9.1 Newton's Law of Gravitation 434
[Cl Applications of Bernoulli's Principle
§ 9.2 Gravitational Field
(a) Action of Atomiser 493
[AJ Gravitational Intensity I 436
(b) Working of Aeroplane 493
[B) Gravitational Potential V 437
§ 9.3 Gravity
(c) Velocity of Efflux 493
[A] Acceleration due to Gravity 'g' 439 o Exercise :
[BI Variation in 'g' • Dnly One Choice is Correct 497
• Mo re t/wn One Choice is Correct 510
(a) Due to shape of earth 440
• Assertion-Reason Type Questions 513
(b) Due to change of planet 440
• Illleger Type Questions 513
(c) With altitude 440
• Match the Columns 51 4
(d) With depth 440
o Li nked Comprehension Type 515
(e) Due to rotation of cUlth 440
o Subjective Type Questions 520
§ 9.4 Potential Energy and Escape Velocity
[Aj Potential Energy
[B] Escape Velocity v"
442
442
III PROPERTIES OF MATTER 523-565

§ 9.5 Planets and Satellites § 11.1 [AJ Elasticity 523


[A I Planets 444 [BJ Stress 523
[B] Satellites [C] Strain 523
ID] Relation between Stress and Strain 524 IE] Characteristics of Sound
IE] Hooke's Law and Moduli of Elasticity (a) Loudness 583
(a) Young's Modulus Y 524 (b) Pitch 583
(b) Bulk Modulus B 525 (e) Quality (or Timbre) 583
(e) Modulus of Rigidity 11 525 o Exerc ise :
IF] More about Elasticity 526 • Gilly Olle Choice i.~ Correct 584
§ 11.2 IA] Viscosity and Newton's Law of • More IIIlIII Qlle Choice is Con"eel 592
Viscous Force 533 • Illfeger Type Questions 594
• MalcI, ,he Colllmlls 594
IB] Stokes Law and Tenninal Velocity 534
0 L in ked Comprehension Type 595
[C] Critical Velocity and Reynold's Number 535
0 S ubject ive Ty pe Questions 597
§ 11.3 IA] Surface Tension 537
IB] Surface Energy
lc] Excess Pressure
538
538
lEI SUPERPOSITION OF WAVES 601-660
[D] Capillarity 539 § 13.1 Reflection and Refraction of Waves 601
[E] Molecular Theory of Surface Tension 540 § 13.2 {A] Principle of Superposition 604
Exerc ise : (B] Complex Waves 604
0
• O/liy Qlle Choice is Correct 544 § 13.3 Interference of Sound Waves 604
• More ,hall Olle Choice is Correct 557 § 13.4 Beats 607
• Assertion-Rea.\'()11 Type Que.~ti()l1s 559 § 13.5 Stationary Waves 610
• integer Type QllestioJ/.~ 559 § 13.6 Vibrations of Strings 614
• Ma rcil tile CO/UIII IIS 559
§ 13.7 Vibrations of Organ Pipes
0 Linked Compre hension Type 561
(A) Closed End Organ Pipe 620
0 Subjective Type Questions 563
[B) Open End Organ Pipe 62 1

lEI VIBRATIONS AND WAVES 566- 600


[C] Discussion
§ 13.8 Doppler Effect
621
626
§ 12.1 Vibrations
0 Exercise:
IA] Damped Vibrations 566 • Dilly Olle Choice is Co rrect 634
IB] Maintained Vibrations 567
• More IIwlI Olle Choice is Correct 649
lc] Forced Vibrations 567
• Auerlioll-Re(/son Type QI/ e.~tioll.\· 65 1
[D] Resonant Vibrations 567
• Intege r Type Questiolls 652
§ 12.2 Waves • Match the CO/llmlls 652
[A] Physical Concept 0 U nked Comprehension Type 653
(a) What a Wave is 568 Subjcctive Type Q uestions 656
0
(b) Mechanical and Non-mechanical
Waves 569 THERMOMETRY AND EXPANSION
(c) Transverse and Longitudinal Waves 569 661-685
(d) Mechanical Waves in Different Media 569
IB] Analytical Concept 570 § 14.1 Temperature 661
lc] Velocity of Transverse Mechanical Waves 574 § 14.2 Scales of Temperature 661
§ 12.3 Sound Waves § 14.3 Thermometers
[A] Audible, Infrasonic and Uhrasonic [A] Liquid Thennometers 663
Waves 576 [ B] Gas Thermometers 663
[B] Velocity of Sound 577 IC] Resistance Thcnnometers 664
[C) Displacement of Pressure Waves 580 [D] Thcnnoclccliic Thermometers 664
ID] Energy, Power and Intensity of Sound 580 IE] Pyromctcrs 664
[F] Vapour-Pressure Thermometers 664 o Miscellaneous Solved Problems 719
§ 14.4 Thermal Expans ion 665 o Exercise:
o Exercise : • Ol//y One Choice is Correct 720
• Ollly Olle Choice is Correct 672 • More thall Olle Choice is Correct 730
• More tlum Olle Choice is Correct 679 • Assertioll-Reasoll Type Questiolls 732
• A.\'sertio/l-Reasoll Type Questions 679 • Integer 7j'pe Questiolls 732
• Integer Type Qlle<~tiOlu' 680 • Match tile Coillmns 733
• March 'he COIIlIIIIIS 680 o Linked Comprehension Type 733
o Linked Comprehension Type 681 o S ubjective Type Questions 735
o Subjective Type Q uestions 683
iii PHYSICS OF GASEOUS STATE 738-779
lEI HEAT AND CALORIMETRY 686-704 § 17.1 Macroscopic Concept
§ 15.1 Heat 686 [A] Gas Laws 738
§ 15.2 Specific Heat Capacity c 687 (B 1 Ideal Gas Equation 739
§ 15.3 Heat of Transformation 689
[C] Discussion 739
§ 15.4 Principle of Calorimetry 69 1 § 17.2 Hygrometry 748
o Exercise : § 17.3 Kinetic Theory of Gases 750
[Microscopic Concept]
• Ollly Olle Choice is Correct 696
• More th(1II Olle Choice i.f Correct 701 § 17.4 {AJ Degrees of Freedom F 754
(B] Law of Equipartition of Energy 755
• IlIIeger Type Qlle.wiolls 701
• Match "ie Coillmli s 702
[C) Molar Heat Capacity 755
o Linked Comprehension Type 702 o Exercise:
o Subjecti ve Type Qu est i o ll ~ 703 • Ollly Olle Choice is Correct 759
• More th(1II Qlle Choice is Correct 77 1
III HEAT TRANSFER 705-737 • Assertion -Rea.wlI Type Questions 772
• Illteger 1)'pe Qllestions 773
§ 16.1 Heat Conduction • Matcll fhe Coil/111m 773
[AJ Process 705 o Li nked Comprehension Type 775
[B] Steady State 705 o Subjecti ve Type Questi ons 777
[C] Law of Conduction 705
[D] Discussion
[E] Applications
705
706
II!I THERMODYNAMICS 780-823
§ 16.2 Convec tion 7 11
§ 18.1 rAJ Some Definitions 780
IB] Zeroth Law 780
§ 16.3 Radiation 712
§ 18.2 First Law of Thermodynamics
§ 16.4 Kirchhoff's Law
rA1 Physical Quantities Involved in the Law 781
[AJ Some Definitions Related to the Law 713
(81 The Law 782
(BJ The Law 713
[C] Applications of First Law 782
CC) Applications 713
§ 18.3 Adiabatic Process 789
§ 16.5 Stefan'S Law
§ 18.4 Second Law of Thennodynamics
Applications of the Law
(A) Cooling by Radiation 714
[A] Heat Engine 794
(B) Newton's Law of Cooling 714
[B] Refrigerator (or Heat Pump) 794
(C) Temperature of the Sun 715
[C] The Second Law 795
§ 16.6 Wien's Displacement Law § 18.5 Carnot Heat Engine
717
[AJ The Engine 795
§ 16.7 Planck's Radiation Law 718
[BJ Carnot Cycle (or Working of the Engine) 796
Ie] Efficiency of the Engine 796 0 Linked Comprehension Type 819
0 Exercise : 0 Subjective Type Questions 821
• Only One Choice is Correct 802
815
0 Appendix 824-832
• More thall One Choice is Correct
• Assertion-Reason Type Questiolls 817 0 Useful Mathematical Relations 833-840
• Integer Type Questions 817 and Tables
• Match the ColI/mils 817
0 Log and Antilog Tables 841-842
Units and Dimensions
[Physics is that branch of science in which we observe. then as I is greater than (1/10), ampere is a smaller
measure and describe natural phenomena related to matter and unit than emu of current (called biot).
energy. These natural phenomena can be classified as
mechanics, properties of matter, sound (acoustics), light § 1.2 Magnitudes
(optics), heat (thermal physics\ electricity, atomic physics, In physics we have to deal from very small (micro) to very
nuclear physics, etc.1 large (macro) magnitudes as on one side we talk about the
§ 1.1 Physical Quantities atom while on the other side of universe, e.g. , the mass of an
The quantities by means of which we describe the laws of electron is9.l x 10- 31 kg while that ofthe sun is2 x 10 30 kg. To
physics are called physical quantities. A physical quantity is express such large or small magnitudes simultaneously we use
completely specified ifit has- the following prefixes:
TABLE 1.1
I I I
Power of10 Prefix S mbol Power of 10 Prefix S mbol
(A) (B) (C)
101& E 10-1 deci d
Numerical value or
only
Magnitude'" only or Magnitude and
direction lOIS
'"
pet. p 10-2 centi c
Ratio Scalar Vector"'* 1012 tera T jQ-3 milli m
e.g., refractive index, e.g., mass, charge, e.g., displacement, giga G 10- 6 micro
dielectric constant, etc. current, etc. torque, etc.
10'
10' mega M 10- 9 nano "
n
Note : There arc also physical quantities which are nOI completely 10' kilo·" k 10- 12 pico P
specified even by magnitude, unit and direction. These physical 10' hecto h IO- IS fernto f
quantities are called TENSORS, e.g. ,Moment of Inertia .
101 deca d. 10- 18 alto ,
In general, in expressing the magnitude of a physical While using prefixes:
q"uantity we choose a unit and then find physical quantity how
(a) The use of double prefi xes should be avoided as far as
many times that unit is contained in the given physical
possible, e.g., instead of JiJiF we must write pF and
quantity, i.e.,
instead ofkMW we must write GW.
Magnitude of Physical Quantity "" Numerical value x Unit (b) When prefix is placed before the symbol of unit, the
From the above it is clear that for a given physical combined prefix and symbol should be considered as
quanlity- one new symbol which can be raised to a positive or
(a) As the unit will change, numerical value w ill also negative power without any bracket, e.g., km 3 means
change, e.g., (10 3 m)3 but never 10 3 m 3 oq .l.s - I means (l0-6 s)-1
densityofwaler= I g/cc""10 3 kg/m 3 and not I kg/m 3 .
but not IO--6 s - l.
(b) Larger the unit smaller will be the magnitude and vice--
Using the table of prefixes we can express magnitudes of
versa, e.g., if
many physical quantities more compactly, e.g., 5 x 10- 6 F will
1 ampere c: (1Il O) emu of current

• Magn itude means numerical va lue and uni l.


•• These are discussed in delail in Chapter 2 .
... In many books K is used for prefix kilo, which is not proper, as according 10 Ihe recommendations of lhe General Conference on Weights and Measures it
must be k.
be written as 51lF or1.3 x 10 6 eVas 1.3 MeV. While expressing Now as the radius of moon isl.738 x 10 6 m ~I06 m, while that
the magnitude ofa physical quantity, keep in mind that: of earth is 6.4 x 10 6 ~ 10 7 m, so moon is the required physical
1. Normally decimal is used after first digit using powers object.
often, e.g., 3750 m will be written as3.750 x 10 3 m--7 § 1.3 Fundamental and Derived Units
3.750 km. This helps in finding the 'order of Nonnally each physical quantity requires a unit or
magnitude' of a physical quantity. standard for its specification, so it appears that there must be as
many units as there are physical quantities. However, it is not
Note: The order of magnitude of a physical quantity is expressed in
so. It has been found that ifin mechanics we choose arbitrary
powers of iO and is taken to be 1 if::::(iO)!/2:= 3.16, and 10 if>
units of any three physical quantities, we can express the units
3.16, e.g., speed of light := 3 x 108 mls ~ 108 mls and mass of of all other physical quantities in mechanics in tenns of these.
electron = 9.1 x 10-31 kg ::::10-30 kg. Arbitrarily the physical quantities mass, length and time are
chosen for this purpose. So any unit ofmass, length and time in
2. Number of 'significantfigures' in the magnitude of a mechanics is called a fundamental, absolute or base unit.
physical quantity can neither be increased nor Other units, which can be expressed in terms offundamental
decreased, i.e., if we have 3.10 kg then it cannot be units, are called derived units. For example, light year or km
written as 3.1 kg or 3.100 kg. is a fundamental unit as it is a unit of length while S-I, m2 or
3. While making mathematical operations in the final kglm are derived units as these are derived from units of time,
result, proper number of significant figures· are mass and length respectively.
retained by 'rounding off'.
Note: (i) It is worthy to note here that only four additional
Note: In rounding off if the digit next to one rounded is more than 5, the fundamenta l quantities temperature, current, luminous
digit to be rounded is increased by I; if the digit next to the one intensity and amoul/t a/substance arc needed to deal all other
rounded is less than 5, the digit to be rounded is left unchanged; if branches of physics.
the digit next to the one rounded is 5, then the digit to be rounded (ij) Apart from fundamental and derived units we also
is increased by 1 if it is odd and i~ left unchanged if it is even. sometimes come across practical units.
(a) Thesc may be offundamental or derived quantities, e.g.,
4. The last digit in the magnitude of a physical quantity light year is a practical (fundamental) unit of distance
is called doubtful figure, e.g., in 3.2 m, 2 is doubtful while horse power is a practical (derived) unit of power.
figure as the true value is > 3.1 m but < 3.3 m, so its (b) These mayor may not belong to a system but can be
value is (3.2 ± 0.1) m. expressed in any system of units, e.g .•
5. The last digit in the magnitude of a physical quantity I mile = 1.6 km - 1.6 xl0 3 m = 1.6 x 105 cm
gives the least count or accuracy, (smaller is the least
count more accurate will be the measurement), e.g., While defining a base unit or standard for a physical
3 kg is less accurate than 3.0 .kg as least count in the quantity the following characteristics are considered:
fonner is I kg while in the later 0.1 kg. (a) Well defined (b) Invariability
Problem 1. The radius ofproton is about 10-9 microns (c) Accessibility (d) Convenience in use
and the radius of the universe is about 1028 cm. Name a As it is often difficult to satisfy all these requirements, a
physical object whose size is approximately haffway between compromise has to be made among these. While writing a unit
these two on a logarithmic scale. the following conventions are usually adopted:
Solution: As radius of proton = 10-9 microns = 10- 15 m I. Even if a unit is named after a person the unit is not
written with capital initial letter. Thus we write
and radius of the universe = 10 cm = 10 m, the size yhalf~
28 26
newton (not Newton) for unit of force.
way between these two on a logarithmic scale will be-
2. For a unit named after a person the symbol is a capital
loga + 10gb loglO-15 + log 10 26 letter. Symbols of other units are not written in capital
log y= =
2 2 letters. For example N for newton (and not n) while m
logy= -15 +26 =5.5 for metre (not M).
or
2 3. The symbols or units are not expressed in plural form.
Thus we write 50 m or 7 erg and not 50 ms or 7 ergs.
so
• Proper number of significant figures :
(a) In multiplication or division the number of significant figures in the fimil answer is the same as the minimum number of significant figures in the
physical quantities being operated.
(b) In addition or subtraction the number of decimal places in the result should be equal to the smallest number of decimal places of terms in the operation.
4. Not more than one solidus is' used. For example Problem 2. A man seeing a lighting starts counting
poise should be written as t poise = 1 gls em orIgs- 1 seconds until he hears thunder. He then claims to havefound
em -1 and not 1 gis/em. an approximate but simple rule that if the count of sec is
divided by an integer, the result directly gives, in km, the
S. Fullstops are not written after the abbreviations and distance of the lighting source. What is the integer if the
units, e.g., 1 litre = 1000 cc (and not c.c.), emf, amu, velocity ofsound is 330 mls?
etc.
Solution: If n is the integer then according to the given
§ 1.4 Standards of Length, Mass and Time problem
[A[ Length tins . km
- - = d·lstance 10
Length is the measure afintervals in space. So to specify a n
distance we must use some unit of length. Most common unit tins
or - - = [vm
. kmi][ .. ]
s thmems
of length is metre. Originally metre was defined as (1/10 7 )th n
part of the distance between pole and equator along a meridian ·1
through Paris. But this is not a very convenient definition for
or n= = ~3
[v in kmls] 330xlO 3
practical use.
[B] Mass
In 1889 the standard metre was redefined as the distance
between two scratches engraved on a platinum-iridium bar Mass is a basic property of maUer. The scientific definition
preserved at a constant temperature of273.16 K and at I bar of mass is rather complicated" · and we have no need to go
pressure in the International Bureau of Weights and Measures into it in detail. It will be sufficient for our present purpose to
at Sevres near Paris, France. All other metallic metres had to be define mass as the charactenstic ofa body that relates the forces
calibrated against this metre and has an accuracy of 1 part in acting on the body to the resulting acceleration.
10 7 parts. The most conunon unit of mass is kilogram. Originally
kilogram was defined as the mass of 1 litre, i. e., 1000 cc of
Since 1960, the standard metre is defined in terms of the water at 4 degree celsius, the temperature at which the water
wavelength of light and is called atomic standard of length. has maximum density. Now the standard kg is the mass of a
The metre is the distance containing 1650763.73 wavelengths platinum-iridium cylinder stored in a special vault ill the
in vacuum of the radiation corresponding to orangc~red light International Bureau of Standards in Sevres, France. The
of krypton-86. An accuracy of I part in 10 9 parts can be accuracy ofthis standard is I part in lOS parts.
achieved with it. To measure mass of atoms or molecules we use the unit
At present, metre is defined as the length ofthe path travelled 'atomic mass unit' abbreviated as amu (or now u). Initially this
by light in vacuum in (11299,792,45 8)lh part ofa second. unit was defined as the mass of a hydrogen atom and is the
We also have some other practical units which are most fundamental unit of mass. Later on, for certain reasons,
frequently used for small and large lengths. They are: the basis of atomic mass unit has been shifted from the
hydrogen atom to oxygen and now to carbon-1 2 atom. At
(a) I Fenni = 1 fIn = 10- 15 m present atomic mass unit is defined as (1112)th the mass ofall
(b) 1 X-ray unit = 1 XU =to- 13 m atom of Carbon- I2 isolope. Now as the mass of a Carbon-12
atom is,
(c) 1 Angstrom = I A=1O- JO m
12 = 12 kg
(d) 1 Micron·=IfJ.m=IO-6m
Avogadro's No. 6.02 x 10 26
(e) 1 Astronomical unit "" I AU = 1.49 x lOl l m
so I amu(oru) = .!..x( 12 )= 1.67 x 10- 27 kg
[Average distance between sun and earth, i.e, radius 12 6.02 x 10 26
of earth's orbit]
(f) 1 Light year = Ily =9.46 x 10 15 m Note: In Astrophysics sometimes we come across a term cha"drashekhar
limit. This term refers to 1.4 times the mass of sun (.. 2 )( I030 kg),
[Distance that light travels in 1 year in vacuum]
i.e, 1 chandrashekhar limit .. 2.8)( lolO kg. (Chandrashekhar has
(g) I Parsec" = I pc ::: 3.08 X 10 16 m = 3.26 light year shown that ifthe mass of an object becomes 1.4 times the mass of
[The distance at which a star subtends an angle of the sun, under gravitational collapse, it turns to a while dwarfJ
parallax of I sec at an arc of I AU]
• Micron is a unit oflcngth meaning)lm and should not be confused with the prefix Il standing for 10-6 .
•• Parsec is an abbreviation of PARallactic SECond.
U. In the general theory of relativity mass emerges as a constant ofintegmtion while considering the dimensions of density in Einstein's field equations. Synge
showed that I g of mass is equivalent to 2.476)( 10-39 S.
Question 1. 5 iitre o/benzene will weigh more in slimmer (ii) At a given instant oftime a physical object cannot be
or wi;tter? present at more than one position in space.
Answer: We know that Question 2. Is the time
Weight = W= mg = pVg [as density = mass/vol.] variation of position, shown in
Her( volume V(=5litre) ~nd acceleration due to gravity
Fig. 1.1, observed in nature? f
Answer: No, as according ~
g (=9.8 mls 2 ) are constant. to the given Fig.I . I,
So Wocmccp (i) The object is present at
Now with rise in temperature due to thennal expansion two positions at the same
density will decrease; so the weight (or mass) of a given instant of time which is 0 L _ _~p~'~"~t"-"-_---
volume will be less in summer than in winter. not physically possible. Fig. 1.1
(CJ Time (ii) Time first increases and then decreases which is also
Time is one of the phenomena which we cannot define in not physically possible as time cannot flow back.
the dictionary sense. It is what we know it to be and what we Note: If we interchange the position find time axes, the graph is possible
perceive it to be. The perception of time arises from the motion as (i) at a given position the body may be present at two different
of something, that is, by the changing position of a body in instants o f time, and (ii) its position will first increase and then
space ..Wecan tell the time by the position of lhe sun in the sky decrease, i.e.. the body fi rst moves away from the origin and a fter
some time comes back towards the origin.
during the day and by the position of the moon in the sky
during the night or by the position of the moving hands of a
clock on a dial. As a matter of fact any periodic phenomenon
§ 1.5 Systems of Units
can bc used to measure time, the measurement being through A complete set of units, both fundamental and derived for
the counting of repetitions. The Italian physicist Galileo was all kinds of physical quantities, is called a system of units.
There are several systems of units which have been employed
the first to develop a simple method of measuring time. He
for describing measurements. A few common systems are
'timed' the oscillations of a lamp hanging by a long chain in a
given below:
church using 11is own pulse as a walch.
(AI CGS System
The motion of earth around the sun, the motion of moon
a round the earth and the motion of earth on its own axis have The syste m is also called Gaussian system of units. In it
been used for a long time to define year, month and day length, mass and time have been taken as the fundamental
respectively. However, careful observations have revealed quantities, and corresponding fundamental units are centimetre
(em), gram (g) and second (s) respectively··. The unit afforce
that these astronomical stand~rds of time have limited
in this system is dyne while of work or energy is erg.
accuracy due to irregularities in the motion of the earth.
Furthennore as tidal fri ction is also slowing down the (BI FPS System
rotational motion of earth on its own axis·, invariability of It uses foot, pound and second for the length, mass and
these standards is also questionable. time measurements respectively. In this system force is a
To overcome these difficulties now-a-days the periodic derived quantity with unit poundal.
motion of astronomical objects has been replaced by periodic Note: There is a lso FPS ellgilleerillg system in which length, force and
motion of atoms or molecules which a re absolutely regular and time are laken to be fundamental quantities wi th units foot,
pure. This standard of time is called atomic standard. At pound and second respcctively. In this system mass is a derived
present second is defined as the time interval of 919263 1770 quantity with unit slug. It can be shown that:
vibrations of radiation (corresponding to the transition I slug '" 14.59 kg
between two specific hyperfine levels) in cesium 133 atom.
With this clock it is possible to achieve an accuracy of I part in Ic] MKS System
10 12 parts, f.e.', a cesium clock will go wrong hy I s in In this system the length, mass and time have been taken as
the fundamental quantities, and thc corresponding fundamental
(10 11 13.16 x 10 7 ) "" 30000 years.
un its are the metre, kilogram and second. The units of all other
A n obsole te unit is shake. In micro physics it s tands for mechanical quant ities like force, work, power, elc., are derived
8
1O- s. Regarding time here it is worth noting that: in tenns of these fundamental units. For example, the unit of
(i) Time can never flow back, i.e., it cannot decrease, as force is that force wh ich will produce a n acceleration of I rrJs 2
from present, future will come and not the past. in a body of mass I kg, and is called newton. The unit of work or
energy is joule, while of power is watt.
~ It is estimated lhqt due to tidal friction the length of day is increasing at th e ralc of7 ~ spcr year.
n For gram and second the symbols gOl and sec alongwilh g and s are used in this book.
TABLE 1.2 Problem 3. In British engilleering system the IInit a/mass
U nIt.0 r some pIh ' l quantlt
YSlca '1 es n d"I~
I erent systems is 'slug ' while the unit afforce is p ound. How many kg are in a
System slug?
Physical
quantity eGS MKS FPS Solution: We know that, I slug is that mass of a body
which moves with an acceleration of 1 ftls 2 under the action of
Length em m ft
a force of one pound, i. e.,
Fundamental Mass g kg Ib
Time s s s I slug = Ib force/(ftls 2 )
Force dyne ncwton----) N poundal but Ib force == f.g] x Ib (mass)
Derived Work or Energy "g joulc~ J ft-poundaJ
or Ib force = [32.2 ftlsl] x Ib (mass)
Power er;"s watt----) W ft-poundll.Vs
so I slug ~ [32.2 flIs' ]lb (mass)/[flIs']
Note: Here it is worthy to note that when MKS system extended to
electricity- or I slug =: 32.2 Ib (mass) = 32.2 x 0.4536 kg
(a) with currenl as fundamental quantity [AJ and ampere as its I . Iug ~ 14.59 kg
unit, is called MKSA system. § 1.6 . Dimensions and Dimensional Formulae
(b) with charge as fundamental quantity [Q] and coulomb as ils
A ll physical quantities can be expressed in tenns of the
unil, is called MKSQ syslem.
seven fundamental units. We call these seven physical
ID) International System of Units [SI Units]* quantities as seven dimensions of the physical world. Thus
In 1971 the International Bureau of Weights and Measures taking mass as {iv1], length [L] and time [T] in mechanics all
held its meeting and decided a system of units which is known physical quantities can be expressed in terms of [M] , [L] ·and
as the International system of units. It is abbreviated as SI units [TJ. So, dimensions of a physical quantity are the powers to
from the Frenc h name Le Systeme International d'Ullites and which the fundamelllal quantities must be raised to represent
is the extended MKS syste m applied to the whole of phys ics. the given physical qllantity. For example,
Now-a-days, most of the engineers and physicists use this densilF mass = [M] ~[ML-' ]
systemofunits. Table 1.3 gives the fundamental qua:1tities and volume [L3 ]
their SI units.
So the dimensions of density are 1 in mass, - 3 in length
TABLE 1.3 and 0 in time, while dimensional formula for density is,
Physical quantity Name of unit Symbol [ML-'] or [ML-' T')
I. length metre on
Now if we write the above as,
2. Mass kilogram kg
3. Time second s [p] ~ [ML-'T' )
4. Electric current ampere A
,. Temperature kelvin K
It is called dimensional 'equatioll for density. Here it is
worthy to note that constants such as (112), 1t or trigonometri(;al
6. Luminous intensity candela cd func tions such as sin rot, (ratio of two sides), etc., have no units
7. Amount of substance mole mol and dimensions. The Table J.4 gives the dimensional fonnu lae
Besides the above seven fundamental units, two and S1 un its ofsome physical quantities of interest in mechanics.
sllpplementary units are also defined, viz., radian [rad] for
plane angle and steradian [Sf] for solid angle.
TABLE 1.4
Pbysical qua ntity Dimensional formula SI unit
I. Velocity - displacementltime [L]~T] [MOLr' ] mi.
2. Acceleration a velocity/time [LT- ' ]/ [T] [MOLT'] m/s2
3. Force = mass x BCC. [M][Lr ' ] [MLT-' ] kg ml s2 -)0 newton --+ N
4. Work = Fscos 9 [MLT'][L]
,. Kinetic energy" (1 12) Imi [M][LT- ']'
) kg m 2/s 2 --+ joule --+ J
6. POIemial energy ~ mgh [ML'T-' ]
[M][LT'][L]
7. Torque .. Fr sin 9 [MLT-'][L] N-m

* Most of the books and authors are usi ng thc phrase Sf System wh ic h in the opinion of the author is not proper (as Sf stands for System Intern ational) and it
must be Sf lI11its .
Ph~slcaJ guanti!}: Dimensional formula SI unit
8. Power '" work/time [ML''''-')/[lJ [ML''''-' ) 2
kg m I 53 -+ Jls --+ watt --+ W
9. Momentum - mass x velocity
10. Impulse = Fllt
[M][L"'-')
[ML"'-'][T) ) [ML"'-') kg mls or N-s

11 . Angle'" arc/radius [L) / [L) [M'L"") radian -+ rad


12. Strain=WLor6V/V [L)I [L) Dimensionless No units
13. Stress:: restoring force/area
14. Modulus of elasticity '" stress/strain
[ML"'-') /[L')
[ML- 1T- 2]
} [ML- 'T-') ) N/m2

15. Pressure'" force/area [ML"'-')I[L') N I m2 or pascal --+ Pa


16. Frequency '" IItime period II[T) hertz --+ Hz

17.
A I I'
"gu ar ve oeLty '"
angular displacement
.
lime
II[T) ) [MoLoT- 1]
radls
18. Moment of intertia, / '" tmr2 [M)[L)' [ML'T') kgm 2
19. Angular momentum'" 100 [ML']["'-' ) [ML"r' ) kgm 2 /s orJ-5
20. Surface tension III force/length
21. Spring constant, K '" FIx
[ML"'-') I [L)
[ML...-') I [L)
} [MLo.,.-' ) ) N/m

22. Surface energy - energy/area [ML2r 2]/(L2 ] J I m2


energy [ML"r') J w
23. Intensity '" area x time [MLo.,.-') --->-
[L' ][T) m~ m2

Problem 4. If a composite physical quantity in terms oj replacing M, Land T by the fundamental units of the required
moment ojinerria l,Jorce F. velocity v, work Wand length L is systcm we get the unit of physical quantity. However,
defined as, sometimes to this unit we further assign a specific. name, e.g. ,
Q = (IF,'IWL'), as Force"" mass x acc.
find the dimenslons ojQ and identify it. so [F) - [M)x[Lr')=[MLr']
Solution: As [1)= [ML'], [F) = [MLr'] So its units in eGS system will be gcm/s 2 which is called
[v] = [Lr') and [W)=[ML'T-'] dyne while in MKS system will be kg m / S2 which is called
newton (N).
[ML' )[MLT- ' )[LT-')' 2. To find dimensions of physical constants or
[Q) = =-=~,.-"':;--'-
[ML'T ')[L)' coefficients :
[Q) = [MT- ') As dimensions of a physical quantity are unique, we write
any fonnula or equation incorporating the given physical
As [MT- 2 ] are dimensions of surface tension, force constant and then substituting the dimensional fonnu lae of all
constant or surface energy. i.e., energy per unit area, the other quantities, we can find the dimensions of the required
physical quantity may be anyone of these. constant or coefficient. To make this more clear, consider the
following examples :
Note: From this problem il is evident ilial if dimensions are given, the
physical quantity mayor may not be unique. rAJ Gravitational Consta nt G
I Approach II Approach
§ 1.7 Applications of Dimensional Analysis From Newton's Jaw of From the relation between
[in Mechanics I gravitation, we have: G and 'g'. we have:
Following are the main applications of theory of m1m2 GM
dimensions in physics: F =G g=-
R'
F,'"
1. To find the unit of a given physical quantity in a
given system of un its: ' gR'
or G =-- or G=--
Writing the definition or fonnula for the physical quantity m]rt/2 M
we find its dimensions. Now in the dimensional formula Substituting the dimensions of all physical quantities:
[ML,'][L'] [L,'][L'] So when the unit changes, numerical value will also
[G] = [M][M] [G ] = [M] change. To convert a physical quantity from one system to the
other, findin g the dimensional formula of the given physical
[G] =[M-'L',' ] quantity, we write its unit in terms of the units 'o fmass, length
So its SI unit is m 3lkg 52 or Nm 2/kg 2 . and time in that system and then using the rehitions,
I Ib = 0.4536 kg; and I ft = 0.3048 m
IB] Planck Constant"
if needed. convert the units into the required system. This
I Approach II Approach III Approach
will be clear from the following examples:
According to Planck: de Broglie: Bohr's II (A] T he newton into dyne
postulate:
The newton is the SI unit of force and has dimensional
E = hv A =~ AM:=n-
h
formula [MLT - 2 ];
mu 2.
E h=')..mv 2. '0 I newton = 1 kg m/s 2
oc 10= - h=-xAM
v n lkg = 10 3 g
Substituting the dimenlions of known P~Sica l quantities :
but
, ,
and Im=102cm
I N=(IO g)(I0 cm)=IOHcm = IO'dyne
[ML' r ' ] [10] = [L][M] [10] = [AM] as 2. '0
[10 ] = -I d s2 s2
[T-1] [LT J a~ .n are
dImensIOnless [B) Gravitational Constant G from CGS to MKS System
0' [h] = [ML','] The value ofG in eGS s·ystem is 6.67 x 10- 8 eGS units
while its dimensional fonnula is [M- 1L3T- 2 ];
So SI unit or Planck's constant is kg m 2,<:, which can also
be written as (kg m 2Is 2) x s. But as kg m 21s 2 is joule, so unit so G=6.67x I0 - 8 cm]/gs 2
of" is joule x sec, i. e., J-5. but l cm=10-2 mand l g=1O-3 kg
Ie) Coefficient of Viscosity 11 3
I Approach II Approach III Approach so G = 6.67 xlO- 8 (to-2 m)3 = 6.67xI0-1l 01
(10- 3 kg)(,') kg,'
According 10 Stoke's Law: Poiseuille '$
Newton's Law: Formula: i.e.. G = 6.67 x 10- 11 MKS units (or SI units)
dv dV npr4 [C) Density from a Given System to a New System
F=~A - - = --
dy F = 6mvv dt 8~/ Suppose we have a new system of units in which unit of
F F npr4 length is 5 cm and unit of mass 20 g, i.e.,
'0 ~= ~= -
A (dv/dy) 6.rv ~ = 8/(dV/d/) 5 cm"" 1 La (say) and 20 g = 1 Ma (say)
Substituting the dimensional formulae of all other known then density ofa substance, which is (say) 8 glcc, in this new
physical quantities : system will be.

["] = [ML.'] 8_ g_ =8[ IMaI20] = 50 Ma


[L')[LT-'/Lj cm 3 [lLal5]3 La 3
i.e., in this new system the value of density will be 50 units.
4. To check the dimensional correctness of a given
So 81 or MKS unit of coefficient of viscosity is kglm s (or physical relation:
g/cm s called poise in CGS system).
This is based on the 'principle of homogeneity'.
Note: ( i) From examples (A), (8) and (C) it is clear that if a physical According to this principle the dimensions of each tenn on
quantity is given, its dimensions fire unique. both sides of an equation must be the same. (The simple reason
Oi) There are also physical constants and coefficients which are for this principle is that mass can be added to mass to give mass
dimensionless. For example, mechanical equivalent of heat J and not to length or time.)
or amplification factor of a valve IJ..
If the dimensions of each tenn on both sides are same, the
3. To convert a physical quantity from one system to equation is dimensionally correct, ot~erwi se not. A
the olher: dimensionally correct equation mayor may not be physically
This is based on the fact that for a given physical quantity, correct. To understand this ali: /
Numerical value x unit = constant [A] Consider the fonnul a,
F=mv 2 /f2
Dimensionally, where K is a dimension less constant. If the above relation
[MLr'] = [M][Lr ' ]' / [L]' is dimensionally correct,
[ML'T-' ] = [Mj'[Lr' jY
i.e, [MLr']= [Mr' ]
As in the above equation dimensions of both sides are not or [ML'T-'] = [M 'L''r- Y]
same; this fonnula is not correct dimensionally, so can never Equating the exponents of similar quantities
be correct physically. x=l y=2
and
[B] Consider the fonnula, So the required physical relation becomes
s = ut - (1I3)at 2 ... (i) E = Kme 2
Dimensionally. The value of dimensionless constant is found unity
[L] = [Lr'][l1 - [Lr'][T'] through experiments, so

i. e., [L] = [L]- [L] E=mc 2


As in the above equation dimensions of each tenn on both This is the famous Einstein mass-energy relation!
sides are same, so this equation is dimensionally correct. IB) Stoke's Law
However, from equations of motion we know that When a small sphere moves at low speed through a fluid,
$= ut + (l/2)at 2 the viscous force F, opposing the motion, is found
experimentally to depend on the radius r, the velocity of the
So the given equation is physically wrong though it is sphere v and the viscosity TJ of the fluid.
correct dimensionally.
So F=/(~,r,v)
(C] Consider the fonnula,
Ifthe function is product of power functions of TJ . rand v,
T =2n,JI/mgL
... (i)
Dimensionally,
,
[ML] =[f]
where K is dimensionless constant. If the given relation is
dimensionally correct.
[T]=
[M][LT '][L] [MLr'] = [ML-'r' j'[LjY[LT -']'
As in the above equation the dimensions of both sides arc or [MLT-2]=[HXL-x+y+zT-X- Z]
same, the given formula is dimensionally correct. It mayor
Equating the exponents of similar quantities
may not be physically correct. However, from the theory of
physical pendulum we know thatT =2n../11 mgL. So the given x= l; -x+y+z= l and -x-z=-2
fonnula is both dimensionally and physically correct. Solving these for x, yand z, we get
S. As a research tool to derive new relations: x = y=z= l
One of the aims of scientific research is to discover new So Eqn. (i) becomes
laws relating different physical quantities. The theory of F =KTl1'V
dimensions in the light of the principle of homogeneity On experimental grounds, K = 61t; so
provides us with a powerful tool of research in the preliminary F =61tTJrv
stages of any investigation.· To illustrate this let us consider
This is the fa mous Stoke's Law which in the opinion of
the following examples:
the author still awaits fo r a theoretical justification!··
(AJEinstein Mass-Energy RelaUon
Ie) Plan ~ k 's Length
When mass is converted into energy let the energy
If we try to construct a new physical quantity having
produced depend on the mass (m) and speed of light (e). i.e.,
dimensions of length in tenns of well known universal
£ =/(m, c) constants G, e and h. then in accordance with the theory of
Assuming the function to be product of power functions of dimensions,
m and c,
... (i)
'" It mllst be emphasized here thnt mere dimensional correctness of an equation does not ensure its physical correctness, e.g., the equation, work " torque, is
dimensionally correct but physically absurd.
"'''' Similarto viscous-drag, for large objects moving at high speed (such as aeroplane), air.drag is given by D _ (112)CaAv 2, whereC is the dimCRsionless drag
coefficient. See Question 13.
If the function is the product of power functions afG, cand h, Solution: Given that,
QL =KGxcYh z ... (i) TocpQdbEc
where K is a dimensionless constant of proportionality. Equating both sides dimensionally,
If the above relation is dimensionally correct, [T]~ [ML- 1T-' ]'[ML- 3 ]b[ML'r']'
ell = [M - 1L 3T-' ]'[Lrl V[ML'r l ], [MOLOT] = [Ma+b+cL-a- 3b+2cT- 2a-2c]
i.e., [L] = [M - x+zL3x+y+2zr,.2x- y-z] Equating the exponents of similar quantities,
Equating the exponents of similar quantities a+b+c "" O, - a - 3b+2c "" O and - 2a - 2c =1
-x+z=O, 3x+y+2z =1 and -2x-y-z=O Solving these for a, band c, we get
Solving these for x, y and z, we get a =- ~. b=~ and c ::. ~
1 -3 1 6' 2 3
x=2"; Y=2 and z=-
2 Problem 7. If velocity, force and time are taken to be
fundamental quantities find dimensionaljormulafor (a) Mass,
So Eqo. (i) becomes
and (b) Energy.
QL =KGII2c-3I2hI /2
Solution: Let the 'q uantity be Q, then,
If the constant K is assumed to be unity Q=f(v,F,T)
3 Assuming that the function is the product of power
QL = .JOh/c
If we substitute the values of G, c and h functions of v, F and T,
Q=KvxFYT z ... (i)
(6.67 x 10- 11 )(6.63 x 10-34 ) 10-35
QL = ffil':: m where K is a dimensionless constant of proportionality.
(3xlO')3
The above equation dimensionally becomes
This physical quantity is called Planck's length and its
[Q]= [LrIJ'[MLr'J'[T]'
importance is still somewhat speculative!
Problem S. If 1 hp is 746 watt, comment on the statement i.e., [Q] =[M Y][I!+YT - x- 2y+Z] ... (ii)
"I hp is 550ft Ibis". Now
Solution: hp is a unit of power, so its dimensional fannula (a) Q = mass i. e., [Q] = [M]
will be [ML2T -3 1
So Eqn. (ii) becomes
So 1 hp = 746W= 746kl;m 2/s 3 [MJ:= [MYL'l'+ YT-x--2y+z]
but 1 Ib = 0.4536 kg and 1 ft = 0.3048 m its dimensional correctness requires

So 1 hP~746( _I_lb) (_I_ ft )' 1,3 y = l, x + y = O and - x - 2y+z=O


0.4536 0.3048 I' which on solving yields
i.e., 1 hp=746 x 23.73Ibft 2 /s 3 x=-l, y=l and z=1
Substituting it in Eqn. (i), we get
i. e., 1 hp = 17702.55 ft poundal [ as lb~ = ft poundal], Q Kv - 1FT
s s3 s
(b) Q energy i.e., [Q]= [ML'r']
or 1 hp=17702.55 ft-Ib
So Eqn. (ii) becomes
32 .2 s
[ML2y-2] = [MYI!+YT- x- 2y+Z]
or I hp!l::i 550 f't-lb/s
So the given statement is true. which in the light of principle of homogeneity yields
Problem 6. A gas bubble from an explosion under water, y=l, x + y=2 and -x-2y+z=-2
oscillates with aperiod Tproportional to pa db E C where pis the which on solving yields
static pressure, d is the density ofwater and E is the total energy x=y=z=l
ofexplosion. Find the values ofa, band c. IMNR 1990] So Eqn. (i) becomes
Q = KvFT
.. To convert poundal to lb we divide by 'g'. It is worthy to note here that ft Ibis is improper unit of power but is in common use.
§ 1.8 Limitations of Theory of Dimensions 4. Specific Heat c
Although dimensional analysis is very useful it cannot As Q <= melle
lead us too far as: Q I.e.,
. [ML' r'J
So e~-- [eJ~
1. If dimensions are given, physical quantity may not be m6.e [MJ[eJ
unique as many physical quantities have same
dimensions. For example if the dimensional fonnula i.e., [eJ~[L'r'e-lJ
of a physical quantity is [ML2T - 2 ] it may be work or So its SI unit will be J/kg K while practical unit caVg C".
energy or torque. 5. Latent Heat L
2. Numerical constants [K] having no dimensions such By definition
as (1/2), I or 21l', etc., cannot be deduced by the Q~mL i.e., [LJ~[ML'r'J/[MJ
methods of dimensions. [See § 1.7 (5)J
3. The method of dimensions cannot be used to derive i.e , [LJ~[L'T -'J
relations other than product of power functions. For SO its SI unit is J/kg while practical unit is callg.
example, 6. Coefficient of Thermal Conductivity K
s=ut+(IIZ)at 2 or y=asincot According to law of thermal conductivity,
cannot be derived by using this theory (try if you can). dQ ~ KAde or [KJ ~ [ML'T-'IfJ
However, the dimensional correctriess of these can be dt dx [L' ][elL J
checked [See § 1.7 (4) B].
4. The method of dimensions cannot be applied to derive i.e., [KJ ~ [MLr3e - lJ
formula if in mechanics a physical quantity depends Its SI unit is W ImK while practical unit is calls cm Co.
on more than 3 physical quantities as then there will 7. Mechanical Equivalent of Heat J
be less number (= 3) of equations than the unknowns
According to I law of thermodynamics
(>3). However, still we can check correctness of the
W~JH
given equation dimensionally. For example,
T = 2nJ flmgL cannot be derived by theory of
or J ~ W or[JJ~[ML'r'J
dimensions but its dimensional correctness can be H [ML'T 2 J
checked [See § 1.7 (4) C].
i.e.,
5. Even if a physical quantity depends on 3 physical
quantities, out of which two have same dimensions, i.e., Jhas no dimensions. Its practical unit is J/cal and
the fannula cannot be derived by theory of has value 4.18 J/cal.
dimensions, e.g., fannula for the frequency of a 8. Boltzmann Constant k
tuning fork f = (diL2 )v cannot be derived by theory According to kinetic theory of gases, energy of a gas
of dimensions but can be checked. molecule is given by
§ 1.9 Theory of Dimensions Applied to Heat [ML'r'J
[kJ ~ [eJ
1. Temperature
In heat it is assumed to be a fundamental quantity with i.e., [kJ~[ML'r'e-lJ
dimensions [e] and unit kelvin [K]. [However,
bK ~ bC'.J So its SI unit is JIK and value 1.38x 10- 23 JIK.
2. Heat 9. Gas Constant R
It is energy, so its dimensions are [ML2T- 2 ] and SI According to gas equation, for perfect gas,
PV~~RT
unit joule (J). Practical unit of heat is calorie (cal) and
1 calorie = 4.18 joule. [ML- 1r'][L 3J
3. Coefficient of Linear Expansion a
so
[RJ ~ [~][eJ
Itisdefinedasa = I1L i.e., [aJ - [L] or [RJ ~ [ML2r'e-l~-IJ
Me [LJ[eJ
So its SI unit is J/mol K while practical unit is cal/mol
i.e., [aJ~[e - lJ K. It is a universal constant with value
So its unit is (co) -l or K - l . 8.31 Jlmol K or 2 cal/mol K
10. van der Waals' Constants a and b 3. Electric Potential V
According to gas equation, for one mol of a real gas, It is defined as

[p+ ;, ] (V-b)=RT .
V=-
W
q
so [V] = [ML'y-']
[AT]
a ab i.e, [V]=[ML'T - 'A- ']
or PV +- - Pb+ -'=' RT
V V' So SI unit of potential is lie and is called volt (V).
As this equation is dimensionally correct, each tenn 4. Electric Intensity E
on either side will have same dimensions, i,e.,
It is defined as
[aIV] = [PV]
F
or [a] = [ML-'Y-'][L' ][L'] = [ML'Y-'] E =- so
q
and [P x b]=[PV]
i.e., [E] = [MLY-'A- I ]
or [b] = [V] = [L']
So SI unit of electric intensity is
Note: Actually van der Waals' equation for).l mol is newton N _ m [N-m =J] J
2 ==~-+ -- ,C- m -[J/OoVj V
- - » =-
[p + J.L 2U]CV - )lb) = IlRT
V
coulomb C- m m
5. Capacitance C
so that[l-Ib ] = [V], i. e, ,[ b ] = [L\L-'] with units m 3/mol It is defined as
and()l2a] = [ PV 2 ],i. e. ,[a 1= [ML5r 2Jl-Z } with unitsJm 3/mo12.
,
q = CV i.e., C = J.. = "- [as V = Wlq]
Problem 8. It is estimated that per minute each cm 2 of , V W
earth receives about 2 calorie of heat energy from the sun. so [c]= [An =[M - 'L- 2T'A']
This constant is called solar constant S. Express solar constant [ML'T ']
in Sf units.
and its unit coulomb/volt and is called farad.
Solution: Given that
S =2 caVcm 2 min Note: As W '" q V, joule/coulomb is volt -+ V

But as I cal=4.18J, 1 em = 10- 2 mand 1 min = 60s and as q '" CV, coulomb/volt is farad-+ F

s= 2x4.18J 6. Permittivity of Free Space &0


(10 'm)'(60s) According to Coulomb's Law
J . F= _ I _ qlq2
;::,;1.4xl03 _ _ or
m's 41t&o r2
§ 1.10 Theory of Dimensions Applied to Electricity so [, 0] =
[ ']
q
[A'T']
= -"-'-~'-o-
1. Current I [F][r]' [MLY-'][L']
While dealing with electricity we assume current to
be a fundamental quantity and represent it by [A] with
unit ampere(A). . . . coul 2 coul 2
and Its umt IS =-- [as N-m = J]
2. Charge Q N_m 2 J-m
AsI = (QIt) so [Q] = [I][t] i.e., [Q]=[AT]
The SI unit of charge is A x sand is called coulomb (C).
=
coul
volt- m [
joule
as--=volt
cow
1
Note: (i) In MKSQ system charge is assumed to· be fundamenta l = F/m [as couVvolt = farad]
quantity with dimensions [Ql and unit coulomb. So in Ihis 7. Dielectric Constant K or Relative Permittivity € r
system current will be derived quantity with dimensions
As K-Hr = (EI&O)
[QT- 1land unit coulomb/s which is ampere.
(ii) In CGS system there are two units of charge namely esu of
so it has no units and dimensions.
charge [called frankline (Fr)] and emu of charge. It is found 8. Resistance R
that, According to Ohm's law V = IR
I coulomb :: 3 x 109 esu of charge= (1/10) emu of charge
R=~=~
N J C-V volt-s weber
i, e., [asv=;] - - -;
Axm Axm2
-;
A_m 2
-;---;--
m2 m2

[ ML2y-2] and is called tesla (T).


i.e., [R] [ML2y-3 A -2]
= [AIl [A]
and its unit volt/ampere and is called ohm (0.).
Alternative: As!lJ = BAcos8 so B= -
A '"
9. Resistivity or Specific Resistance p [ML2r2A-l ] [MT-'A-' ] d . weber
[B] == 2 ~ an Unlt - ,-
I rrr2 R [L 1 m
As R=p- or p=--
1I:r2 I
13. Magnetic Intensity H
2
so [p] = [R][L ]-; [RL] From Biot-Savart Law
[L] dB=!:.Q.ldlsin9
m [M] 41t r2
p= - = ---,'-=-'--,---
2
ne , [L 3][AT]2[11 but B=JloH
-; = [ML3y-3 A -2]
so [H] = [:~I] = [AL_I ]
and its unit is ohm~m or ohm-em.
10. Coefficient of Self-Induction L and its unit is Aim.
According to definition of L, TABLE 1.5
EMF=L di or [L]=[Wlq][T] Physical Units
Dimensions Relations
dt [i] Quantity SI eGS

i.e., [L]= [ML2y-2 IAT][l1 = [ML2y-2 A - 2] Mag. flux $ [ML2T- 2A ~] weber maxwell 1 Wb = 108 Mx
[A] Mag. [Mr2A- 1] weber/m2
gauss IT=1040
induction B or tesla
and its unit volt x stamp --+ohm x s and is called henry
Mag. I AIm =
(H). [AL- I ] ampere/m oersted
intensity H 41tx 1O- 3 Oe'"
Note: (i) As energy stored in a coil is given by 14. Magnetic Dipole Moment M
U = (1I2)U 2, [L]=[U]I[if
As M=NiS [M] = [AL2 ]
i.e., L = [ML2r2] / [A2 ]--)o [ML2y-2A -2]
and so its unit is A-m 2.
(ii) Here it is worth noting that volt x s -+ weber (Wb) while ohm
x S -4 henry (H). 15. Permeability of Free Space Ilo
Force per unit length between two parallel curren
11. Magnetic Flux ~ carrying wires is given by
According to Faraday's law of electro-magnetic dF Ilo 21112
induction, -=---
dL 41t r
EMF = (d<Dldt)
so [<D] = [EMF][T] = [Wlq][T]
i.e., [<D] = [ML2y-2 IAT][T] = [ML2y-2 A-I]
So [~o] =[;, ]
and its unit will be volt x s known as weber (Wb). i.e., [~O]=[ M~;-2 1 =[MLT-2A-2]
12. Magnetic Induction B
As force on a current element in a magnetic field is . . N J volt-s ohmxs H
given by and Its umts - -» - - -; - - -; -; -
A2 2
A m A-m m rr:
F = Bilsin 9
Problem · 9. Finding dimensions of resistance R an
[B] = [F] = [MLy-2]=[MT - 2A - I ] inductance L, speculate what physical quantities (LlR) an
[i][n [A][L] (l/2)Li2 represent? (CBSE 1992
and unit
Solution: As
di dt But as it is given
e= L - i,e. , L=e
dt di X=3lZ' i.e., Y = .l'l(3Z')
so [L] =[ ;][1] -.[ ML~;-' x ~] so [Y] = [M - 1L- 'T'Q'] -. [M-'L-'T 4Q4]
[MT IQ - I]'
i.e., [L] = [ML'T -' A-']
Problem 11. A quantity X is given byeoL ~v, whereeo is
and as V =IR i.e., R = VII 6t
the permittivity offree space, L is a length, Il V is a potential
so [R] = [~]
qA
-. [ ML'T- ' ]
ATA
difference and M is a time interval. The dimensional formula
for X is the same as that of (a) resistance, (b) charge, (c)
[R]=[ML'T' A-'] voltage, (d) current.
i.e.,
Solution: As capacity ofparaUcl plate capacitor is given by
so L]_ [ML'T'A-']_[ I ]_
[ -R - ML2T 3 2 - -T - i -[T] C = coA
A d

and [!Li'] = [ML'T -' A-'][A '] = [ML'T-']


which has the same dimension as of coL,
2 .. X=coL . ~V =C~V =charge =current
Now as (UR) has dimensions of time and so is called time Ilt M time
constant of ~R circuit and (112)Lj2 has dimensions of work or Thus quantity X has the dimension of current.
energy, so it represents magnetic energy stored in a coil. Problem 12. Column I gives three physical quantities.
Note: (i) In case of objective questions the units and dimensions of
Select the appropriate units for these from choices given in
(UR), CR or Le can be calculated easily as follows: Column II. Some of the physical quantities may have more
L henry olun x s than one choice.
(a) - -+ - -= - - -+s-+[T]
R ohm ohm I II
coul volt Capacitance ohm x second
(b) CR -'t farad x oinn -+ - x- --)0 S -+ [T]
volt amp
Inductance coul 2 -joule-I
[as amp == coulls]
(c)
= -+ ~ henry x farad =
"Le ,ohrnxsx
--'':,=,u'
-
'l Magnetic induction coulomb (volt)-I
volt newton (ampere- m)-I
volt coul volt second (ampere)-1
--> - x-xs-+s.-..t[Tj
amp volt Solution: As
(ii) Similarly units and dimensions ofPRt,(lI2)CV 2 or (1I2)Li 2 (I) q=CV i.e, [C] = [q/V] so[C] = [M- 1L-'T 4A']
are of work or energy, i.e., [ML2T- 2 ].
_ -I L"I
U - i.e., [L]=[Uli'] so [L] = [ML'r'A-']
2
Problem 10. In the formula X = 3IZ 2, X and Z have
dimensions of capacitance and magnetic induction F = Bil sine i. e., [B] = [F/il] so [B] = [Mr' A - I]
respectively. What are the dimensions of Y in MKSQ system? (II) Now the dimensions from given units are

Solution: As q =CV, C = ~=~ [asv = ; ]


ohm x sec =[R][T]-.[ML'r3A-'][T]
= [ML'r'A -' ]
so [XH[C]=[ Q' ] = [M - 1L··'T'Q'] cou!2_joule-1 == L' ] . .-.)0 ::"-=-;=;-:
[A'T' ]
ML2T 2 [W [ML'T ']
and F = Bilsin9, [BJ=[~] = [M- 1L-'T 4 A']

so [Z]-> [B] = [ MLT-'] = [MrIQ - I]


QT IL
caul (volt) - I = [~]
W
-.
::,,
[A::-'-,:T=':,]e:
[ML'T ']
=[M - 1L-'T 4A']
newton (amp-m) - 1 ~[F ] --> [MLT -'] Problem 14. Four physical quantities are listed in colum,
I. Their values are listed in column II in random order.
iI [AL]
Column I Column II
=[Mr'A- 1]
[a] Thennalenergyo!airmoleculeatroomtemperature [e] 0.02 eV
volt sec (amp) - l
~[;: :]-->[M~;~'T1 [b] Binding energy 0/ heavy nuclei per nucleon [fj 2 eV
[c] X-ray photon energy [g] 1 keV
= [ML'r'A- ']
[d] Photon energy o/visible light [h] 7 MeV
Comparing dimensions of II with I, we find that:
Capacitance has unit coulomb 2-joule- 1• and coulomb Match the column correctly.
(volt)-I, inductance has units ohm-sec and volt-sec (ampere) -I , Solution:
and magnetic induction has unit newton (ampere m) -', 1. At room temperature thennal energy of a gas molecull
Problem 13. The terms in column X are somehow related is, E = ~kT =~ x 1.38 x 10-23 x300=6.21 x 10- 21 J
to the terms in columns Y and Z; match the following columns 2 2
correctly: or E""(6.2Ixl0- 21 1l.6xlO- 19 ) = O.03geV

x y Z So thennal energy of air molecule is of the order oj


0.02 eV,i.e.,
1. Planck A. Nuclear constant G. Angstrom
[a]-->[e]
2. Raoult B. Mass-wave H. Thresholdfrequency
2. Energy ofthe order of MeV is associated with nuclea
3. deBrog/ie C. Moderator 1. Heavy water phenomena such as fission, fusion, -radioactivity 0

4. Einstein D. Erg-second J. Molecular weight binding energy. So binding energy per nucleon is 0
the order of MeV, i. e.,
5. Nuclear E. Vapour pressure K. E = mc 2
reactor lowering [bH[h]
3. As X-rays are produced by kV potential differenc·
6. Mass defect F. Photoelectric effect L. 6.626x 10-27
across the X-ray tube, so energy of X-rays is of th
Solution Correct matching in tabular fonn is given order ofkeV, i.e.,
below: [c]--> [g]
x y z 4. As atoms get ionised by few eV so the energy ofligl1
I. Planck D. Erg~second L. 6.626x 10-27 emitted by atoms is of the order offew eV, i.e.,
2. Raoult E. Vapour pressure J. Molecular weight [d]-->[f]
lowering
3. de Broglie B. Mass~wave G. Angstrom
4. Einstein F. Photoelectric effect H. Threshold
frequency
5. Nuclear reactor C. Moderator 1. Heavy water
6. Mass defect A. Nuclear constant K. E = me 2
EXERCISE
•.
", '
(AJ Only One Choice is Correct 13. An atmosphere:
1. Proper symbol for kilowatt-hour is : (a) Is a unit of pressure
(a) kwh (b) KWH (b) Is a unit of force
(c) kWh (d) kWH (c) Gives an idea of the composition of air
2. The unit ofatomic mass is : ICBSEI9921 (d) Is the height above which there is no atmosphere
(a) 0" 16.0000 (b) 0 16 - 16.0000 14. The Sf unit of pressure is:
(c) e = 12.0000 (d) e" " 12.0000 (a) atmosphere (b) bar
3. Onemicronis: (c) pascal (d) mm ofHg
IMNRI9931
(a) 10- 9 m (b) 10- 12 m
15. The unit of impulse is the same as that of: ICDSE 19961
(a) Energy (b) Force
(c) W'm (d) 10- 15 m
(c) Angular momentum (d) Linear momentum
4. One nanometre is equal to :
16. Dyne·sec stands for the unit of:
(a) 10' mm (b) 10-6 em
(a) Force (b) Work
(c) 10-7 em (d) 10- 9 em
(c) Momentum (d) Angular momentum
5. Light year is :
17. The joule)( s is the unit of: ICPMT I 9901
(a) Light emitted by the sun in one year
(a) Energy (b) Momentum
(b) Time taken by light to travel from the sun to the earth
(c) Angular momentum (d) Power
(c) The distance travelled by light in one year, in free
18. One kilowatt·hour is equal to:
space
(d) The time taken by earth to go round the sun once (a) 3.6x 106 joule (b) 3.6x 105 joule
6. Light yellr is the unit of: (c) 10) joule (d) 107 joule
(a) Ene'llY (b) Intensity of lig ht 19. The unit of power is:
(c) Time (d) Distance (a) kilowatt (b) kWh
7. How many wavelengths ofKr 86 arc there in one metre? (c) dyne (d) joule
(a) 1553164.13 (b) 1650763.73 20. Hertz is a unit of:
(c) 2348123.73 (d) 652189.63 (a) Electromagnetic energy
8. Which of the following is not a unit of time? (b) Frequency
(a) Solar year (b) Leapyear* (c) Electric field strength
(c) Light year (d) Tropical year (d) Magnetic penneability
9. One sec is defined to be cqual lo: 2 1. Which one of the fo llowing quantities has not been
expressed in proper units? IMNRI9951
(a) 1650763.73 periods of krypton clock
(b) 652 189.63 periods of krypton c lock (a) Stress/Strain "" N /m2 (b) Surface tension '" N/m
(c) 1650763.73 periods of cesium clock (c) Energy = kg·mls (d) Pressure = N/m 2
(d) 9192631770 periods of cesium clock 22. Which of the following is dimensionally correct?
10. Nkg - I is the unit of: (a) Pressure = momentum per unit volume
(a) Velocity (b) Force (b) Pressure = momentum per unit volume per unit energy
(c) Acceleration (e) Pressure = energy per unit volume
(d) None of these
II . The SI unit of force is : (d) Pressure = energy per unit area

(a) walt (b) dyne 23. Indicate which pair of physical quantities given below has
not the same units and dimensions? IEAMCET 1993)
(c) newton (d) pounda l
(a) Momentum and impulse
12. Which ofthe following is nol the unit oflength? lilT 19981
(b) Torque and angular momentum
(a) micron (b) light year
(e) Acceleration and gravitational field strength
(c) angstrom (d) radian
(d) Pressure and modulus of elasticity
• Leap year is the year in which February is of29 days and is divisible by 4 whi le tropical year is the year in which there is total solar eclipse as in 1995.
24. The dimensional formula for impulse is : 34. The dimensional formula for surface tension is:
(a) [MLT-' ) (b) [MLr ' ) (a) [MT') (b) [MLor')
(0) [M -'L'T-') (d) [ML'T-' )
25. The dimensions of pressure are : (eBSE 1994) 35. The volume of a liquid of density p and viscosity '1 flowing
(a) [MLr' ) (b) [ML-'T') in time t through a capillary tube of length I !lnd radius R,
with a pressure difference P, across its ends is proportional
(0) [ML-'T-') (d) [MLT') to:
26. The dimensions of torque are: [CBSE 1990; MNR 19931 (a) p2R 2 tlTll 2 (b) PR' I~II
(a) [ML'r') (b) [MLr') (0) PR't1~1 (d) ~R4 I It
(0) [ML- ' r ' ) (d) [ML-'r') 2
36. Velocity ofa particle depends upon t asV '" A + Bt +Ct . If
27. The expression [ML2T- 2 ] represents: velocity is in mis, the unit of A will be:
(a) mI, (b) mis'
(a) Power (b) Kinetic energy
(e) Momentum (d) Pressure (c) mAS Cd) m 2 1 s
28. The dimensional fonnula for angular velocity is : 37. Given F =(alt)+bt 2 where F denotes force and t time.
(a) [M'LT-') (b) [MLor') The dimensions of a and b are respectively:
(a) [MLr')and [MLr4)(b) [Lr' ) and [T-' )

29. Planck's constant has the dimensions of: IEAMCET 19901 (0) [T] and [r') (d) [Lr') and [T- ')
(a) Energy (b) Momentum 38. The velocity v of a point at time t is given by:
(c) Frequency (d) Angular momentum b
v=at+-
30. The dimensions of gravitational constant G are: I+e

[CPMT 1990; CDSE 1992) The dimensions of a, b and care respectively:


[CPMT1990)
(a) [MLr' ) (b) [ML'r')
(a) [L'); [T]and[LT') (b) [LT'); [T) and [L)
(0) [Lr') ; [L) and [T) (d) [L); [L) ond [T')
31. E,I1I, I and G denote energy, mass, angular momentum and
gravitational constant respectively; E/2/ (m5 0 2) has the 39. Which of the following is true, given [q}=[ML- ' r i } ?
dimensions of: [Here v,A and t represent velocity, area and time
respectively]
(a) Length (b) Mass
dv
ee) Time (d) Angle (a) q c.A dl
32. The dimensional fonnula for modulus of rigidity is:
Idv
(a) [ML-'T-') (b) [ML-'T') (0) q. A dl (d) None of these

40. The time dependence of a physical quantity p is given by


33. Turpentine oi l is flowing through a tube of length I and p "'Po cxp( -01 2 ) [where 0. is a constant and t is time]. The
radius r. The pressure difference between the two ends of constant 0.: leDSE 19931
the tube is p; the viscosity of the oil is given by : (a) is dimensionless (b) has dimen..;ions (T-2 ]
p(,' -x' ) (c) has dimensions [T2] (d) has dimensions of P
11 '" 4vl
41. The velocity of water waves may depend on their
where v is the velocity of oil at a distance x from the axis of wavelengths A, the density of water p and the acceleration
the tube. From this relation, the dimensions of viscosity 11 due to gravity g. The method of dimensions gives the
are: leDSE 1993) relation between these quantities as: (MNR 19921
(a) [MoL'T' ) (b) [MLr') 2
(a) V «g - l).- l 2
(b) V «g).
(0) [ML'T-') (d) [ML-'r') (e) V 2 «g).p (d) V 2 a:g- 1).-3
42. From dimensional consideration which of the following 52. Unit of universal gas constant in SI units is :
equations is correct? (a) wattK- lmol- 1 (b) joule/newton K-1mol - i
. (a) T.2"'/R'/GM (b) T.2"'/GM/R' (e) jouleK-1moI- 1 (d) ergK-'mol- 1
~'---. 53. The equation of state of some gases can be expressed as;

43. The time period Tofa small drop of liquid (due to surface (p+ ;, r-b)'RT
tcnsion) depends on density P, radius r and surface tension
S. The relation is: [EAMCET 1993) where P is the pressure, V the volume, T the absolute
(a) T oc (pr' / S )112 . (b) T oc wS temperature and a,b, R are constants.
The dimensions of' a'are: ICBSE 1992, 96; MNR 19951
(c) T oc pr/S (d) T oc (Slpr)
(a) [ML' T- '] (b) [ML-1T-']
44. A gas bubble from an explosion under water oscillates with
a period T proportional to pQd bE e , where P is the static (c) [L'] (d) [L']
pressure, d is the density and E is the total energy of the 54. The dimensional formula for coefficient of thermal
explosion. The values of a,b and care: conductivity is: ICBSE 1994]
(a) a-O, b.J, c .2
(a) [MLTK] (b) [MLT-']
(b) •• J,b-2,c -3
(c) (MLTK- 1] (d) [MLT-'K- 1]
(e) a=SI 6.b=-1I2,c=1/3
(d) a·-5/ 6,b.JI2,c.J/3 55. The unit of Stefan's constant 0" is:
45. P represents radiation pressure, c represents speed of light watt 4
(a)
and S represents radiation energy striking unit area pcr sec. mJ('
The non-zero integers x,y and z such that P:XSYc z is
dimensionless are: (CBSEI992(
(a) x= l,y= l,z= 1 (b) x=- !,y =l,z =1
56. In case of diffusion the number of molecules crossing unit
(e) x=l,Y =-!,z =1 (d) x=I,y=J,z.-J area per unit time is given by:
46. A system has basic dimensions as density [D], velocity [VJ
N =_D(n2- nl)
and area (A 1 The dimensional representation of force in
(X2 -XI)
this system is :
(a) [AY' 0] (b) [A 'YO] where nl and n2 are the number of molecules per unit
volume at positions Xl and x2' The dimensions of
(c) [A YO'] (d) [A °YO] coefficient of diffusion D are therefore:
47. The Sl unit of temperature is: (a) [L'T- 1 ] (b) [Lr']
(a) Degree centigrade (b) Kelvin (c) [L'T'] (d) [Lr']
(e) Degree celsius (d) Degree fahrenheit
57. The ratio of the emu of charge to esu of charge is :
48. Celsius is a unit:
(a) 3x 10 10 (b) 4.8x 10- 19
(a) of electric potential
(b) of trigonometric angle (c) J/JO (d) 1/300
(e) equivalent to K 58. State which of the following is correct?
(d) equivalenllo degree centigrade (a) joule = coulomb x volt
49. Practical unit of he a! is: (b) joule == coulomb/volt
(a) calorie (b) horse power (c) joule" volt + coulomb
(e) joule (d) watt (d) joule == volt/coulomb
50. The dimensional fonnula for calorie is : 59. Electron volt is a unit of:
(a) [M1L' r ' ] (b) [M'Ur' ] (a) Potential difference (b) Energy
(c) Resistance (d) Electric charge
60. Which of the following quantities can be written in SI units
51. The dimensional formula for latent heat is : in kg m2 A -2s - 3? lilT 1993)
(a) [MOL' T-' ] (b) [ML'r'] (a) Resistance (b) Inductance
(c) [MLr'] (d) [ML'rl] (c) Capacitance (d) Magnetic flux
61. The dimensions of self-inductance are: (a) A +B (b) A-B
[CDSE 1992; CPMT 1992[ (e) Ale AS (d) AlB
(a) (MLT-' A -' ] (b) (ML' rIA-'] AV
70. A quantity X is given by £oL-. where £0 is the
(e) (ML' T- ' A -'] (d) (ML'T-' A'] At
pennittivity of free space, L is a length, .6.V is a potential
[Hint: Energy stored in a coil U '" (112)£;2J difference and .6.1 is a time interval. The dimensional
62. The dimensional formula for magnetic permeability 1.1 is : fonnula for X is the same as that of: (lIT 2001)
[CDSE 1991] (a) resistance (b) charge
(a) (MLr' A -'] (b) (MOL-IT] (c) voltage (d) current
(e) (MOL'r l A'] 7 1. In a particular system, the units oflength, mass and time are
chosen to be 10 cm, 10 g And 0.1 s respectively. The unit of
[
Hint : !!..~ =!:2. 2illl ] force in this system will be equivalent to: (CBSE 1993)
dL 41t d
(a) 0.1 N (b) I N
63. If L and R denote inductance and resistance respectively,
then the dimension of LlR is : (e) ION (d) lOON
72. The dimensions of the coefficient of viscosity are
fCPMT 1990, 91; Roorkee 19951
{ML- 1T-1l To convert the CGS unit poise (P) to the MKS
(a) (MOLOTO] (b) (MOLOT]
unit poiseuille (PI), the poise has to be multiplied by:
(e) (M'LOT'] (d) (MLT']
(a) Wi (b) 10
64. Ife and R denote capacity and resistance the dimensions of
(e) 10' (d) 10'
CR are: [CDSE 1992]
I 73. The dimensional formul a of magnetic flux is:
(a) (MOLOT- ] (b) (MILOT' ]
IKeraJa PMT 2005)
(e) (M OLOTI]
(a) (MILOr' A - I] (b) (MIL'T- I A - I]
(d) Not expressable in terms of [M]. [L] and [TJ
(e) (MIL'rIA -']
65. If C and L denote the capacity and inductance, the
dimensions of LC are: (e) (MIL'T-' A - I]
(a) (MOLOT'] (b) (MoL'T-'] 74. According to Bernoulli 's theorem,
(d) (MoLoTO] v'
-p + - + gh = C(lnstant
d g
66. The dimension of (112) EoE2 (co : permittivity of free
space; E: electric field) is : lilT 1000) The dimensional formula of the constant is :
(a) (MLrl] (b) (ML'r'] (P-pressure, d-density, v-velocity, h-height, g-acceleration
due to gravity) IEAMCET 20051
(e) (MLr ' ]
(a) (MOLOTO] (b) (MOLTO ]
[Note: (1/2) &0£2 represents energy stored per unit volume in an
(d) (MOL' T-']
electric field and has dimension [ML":' IT-2 land not [ML2r-l ]. In
the option (b), the dimension of energy is given, perhaps due to 75. 'Parsec' is the unit of IAIlMS 200S]
confusion in the mind ofthe paper setter.] (a) time (b) distance
67. The velocity of electr!>-magnetic waves in vacuwn is given (c) frequency (d) angular acceleration
by,
76. Ttle ratio of the dimension of planck's constant and that of
(a) ~ (b) '/"0/'0 the moment of inertia is the dimension of: (CBSE 2005]
(e) ./'0 ' "0 (d) 1/./"0'0 (a) frequency (b) velocity
(c) angular momentum (d) time
68. A pressure ofl06 dyne/cm2 is equivalent to:
77. The velocity v of a particle at time t is given by
(a) IO'Ni\n' (b) IO'NIm' ..
(e) 10' NIm' (d) 10' NIm' v =at +....!.... where a, b and c are constants. The dimensions
I +e
69. Which of the following functions of A and B may be ofa, band care respectively: (CBSE 2006]
performed if A and B possess different dimensions?
(a) (LT- ']. (L] and (T) (b) (L']. [T) and (LT']
; '1CP!\1T 19971
. " ~.' - (e) (LT']. (LT]and (L] (d) (L]. (LT] and (T']
78. A hypothetical experiment conducted to determine 81. The energy (E), angular momentum (L) and universal
cos9rX,t gravitational constant (G) are chosen as fundamental
Young's formula Y '-'- 3 . IfY = Young's modulus, quantities. The dimensions of universal gravitational
I
constant in the dimensional formula of Planck's constant
T= time period, or= torque and /= length. then find the value
Ch lis: [EAMCETCEngg.) 20081
ofx. (BHU (Mains) 20071
Ca) ,ew (h) 1 Ce) 2 Cd) 3 Ca) 0 Cb) - 1 Ce) 5/3 Cd) 1
79. The speed (v)ofripples on the surface of water depends on 82. Which one of the following pair of quantities has same
surface tension (cr ~ density (p) and wavelength 0.. ). The dimension? [DeE 2009]
square ofspeed(v)isproportional to: [AIIMS 20071 (a) Force and work done
a (b) Momentum and impulse
Ca) (h) 2.-
pI. aA (c) Pressure and force
(d) Surface tension and stress
Ce) J:..
ap
Cd) pAa
83. Which physical quantities have same dimen~ions?
4 [Orissa JEE 2009]
80. In the relation V = ttpr ,where the symbols have their (a) Force and power (b) Torque and energy
8~1
usual meanings, the dimensions of V are: (c) Torque and power (d) Force and Torque
84. TheSI unit of electron mobility is: [J & K 2009]
[Uttarakhand (Med.) 2007)
(a) m2 s- 1 V-I (b) ms V - I
Ca) [MOL'TO] (h) [MOL'ri]
(c) ms- I V
Ce) [MOL-'T] Cd) [ML'TO]

ANSWERS

1. (0) 2. (d) 3. (e) 4. (0) 5. Ce) 6. Cd) 7. (b) 8. Ce) •• (d) 10. (e) 11. (e) 12. (d)
13. (a) 14. (e) 15. (d) 16. (e) 17. (e) 18. (a) 19. (a) 20. (h) 21. (e) 22. (e) 23. (h) 24. (b)
25. (e) 26. (a) 27. (h) 28. (e) 29. (d) 30. (e) 31. (d) 32. (d) 33. (h) 34. (d) 35. (e) 36. (a)
37. (,) 38. (e) 3'. (d) 40. (h) 41. (b) 42. (a) 43. (a) 44. (d) 45. (e) 46. (a) 47. (h) 48. (d)
49. (a) 50. (a) 51. (a) 52. (e) 53. (a) 54. Cd) 55. (e) 56. (a) 57. (a) 58. (a) 59. (b) 60. (a)
61. (e) 62. (a) 63. (h) 64. (e) 65. (a) 66. None 67. (d) 68. (e) 69. (d) 70. (d) 71. (a) 72. (a)
73 . .(e) 74. (d) 75. (b) 76. (a) 77. (a) 78. (a) 79. (a) 80. (h) 81. (a) 82. (b) 83.(b) 84. (a)

[BI ,
More
. than One Choice is Correct 4. The unit of charge is:
}. Whi~h of the following pairs have same dimensions? (a) coulomb (b) frankline
, (~) Torque and work (c) faraday (d) ampere x sec
(b) .Angular momentum and work 5. Which of the following is a unit of permeability?
::(~) ' En~rgy and Young's modulus Ca) Wm (h) Wb/Am
(el) Light year and wavelength (c) ohmxs/m (d) Vxs/m2
2. Pressure is defined as : 6. The SI unit of inductance, the henry can be written as:
(a) Momentum per unit area [liT 1.981
(b) Momentum per unit area,per unit time (a) weber/ampere (b) 'volt-second/ampere
(c) Momentum per unit volume (c) joule/(ampere) 2 (d) ohm-second
(d) Energy per unit volume 7. Let [EO Jdenote the dimensional formula of the permittivity
3. The pairs of physical quantities that have the same of the vacuum and [JA 0] that of the permeability of the
dimensiop.s are: [liT 1995] vacuum. If M = mass, L = length, T = time and I = electric
(a) Reynolds number and coefficient of friction current: [liT 1998]
(b) Latent heat and gravitational potential Ca) ['0]=[M -' L-'T I]2 Cb) ['0]=[M- L-'T 4 12 ]
i

(c) Cune and frequency oflight wave 2


Ce) ["o]=[MLT- 'C'] Cd) ["0]=[ML r'l]
(d) Planck's constant and torque
8. If L,C and R represent the physical quantities inductance, 4. If length unit becomes doubled and mass unit becomes
capacitance and resistance respectively, the combinations halved then, find the factor by which the value of speed of
which have the dimensions of frequency are: light decrease in new system compared to old system.
(a) (lIRC) (b) (RlL) 5. The ratio of magnitudes of unit for viscosity in Sf to that in
(c) (1I..riC) (d) (CIR) COS isx. Then the value of(2X+2)/2is:

9. If dimensions of length are expressed as GXcYh z , where ANSWERS


G, c and h are the universal gravitational constant, speed of 1. 2. 1 3. 1 4, 4 5. 6
light and Planck's constant respectively, then: lIlT 19921
(a) x = (1/ 2), y = (1/ 2) (h) x = (1/ 2)" = (1/2) ID) Match the Columns
(c) y=(-3/2),,=(1/2) (d) y=(1/2),,=(3/2) 1. Match the physical quantities in Column-I with their
to. A reference frame attached to the earth: dimensional fonnula in Column-II
(a) Is an inertial frame by definition Column - I Column - II
(b) Cannot be an inertial frame because earth is revolving
(a) Angular momentum (p) ML2T 2
round the sun
(c) Is an inertial frame because Newton's laws are (b) Coefficient of viscosity (q) MOLOT 2
applicable (c) Torque (,) ML2 T- 1
Cd) Cannot be an inertial frame because the" earth is
rotating about its own axis (d) Angular acceleration (s) ML-1T- 1
[Hint: The frames of reference which are at rest or in unifonn 2. Match the physical quantities in Column-I with their
motion are called inertial frames while frames which are dimensional formula in Column-II
accelerated with respect to each other are non*inertial frames.
Spinning or rotating frames are accelerated frames.] Column - I Column - II
II. A train carriage moves along the x-axis with a unifonn (a) Pressure (p) ML2T I
->
acceleration a. An observer A in the train sets a ball in (b) Coefficient of friction (q) M- 1L3 T 2
motion on the frictionless floor of the carriage with a
-> -> (c) Planck's constant (,) MOLOTO
velocity u relative to the carriage. The direction of u
e
makes an angle with the x-axis. Let B be an observer (d) Universal gravitational (s) ML IT- 1
standing on the ground outside the train. The subsequent constant
path of the ball will be: (lIT (S) 1993) 3. Some categories of physical quantities are given in
(a) A straight line with respect to observer A Column*I. Match the physical quantities in Column-II with
(b) A straight line with respect to observer B the categories in Column-I
(c) Parabolic with respect to observer A Column - I Column * II
(d) Parabolic with respect to observer B
(a) Dimensionless quantity (P) Moment of inertia
(b) Scalar (q) Refractive index
1. (a) and (d); 2. (b) and (d); 3. (a), (b) and (c); 4. All; 5. (a), (c) Vector (,) Dielectric constant
(b) and (c); 6. All; 7, (b) and (c); 8, (a), (b) and (c); 9. (b) and (d) Neither a scalar nor a (s) Area
(c); 10. (b) and (d); 11. (b) and (c). vector

ICJ Integer Type Questions 4. Match the units/dimensions in Column-I with the physical
quantities in Column-II :
1. Ifmass length and acceleration is taken as base quantities in
a system then dimension of length in dimensional formula Column - I Column - II
of energy is : . .
(a) ML T- 1
2 (P) Impulse
2. Ifmass length and acceleration is taken as base quantities in
a system then dimension of acceleration in dimensional (b) Joule-sec (q) Planck's constant
fonnula of torque is : (c) MLT- 1 (,) Angular momentum
3. In SI system of unit the sum of dimension of all base
(d) Energy po. unit (s) Linear momentum
physical quantities in the dimensional fonnula of
frequency
co-efficient of viscosity is c. Find the value of c+ 2.
5. Match the units given, in Column-I with the physical (b) MOL2T- 2e- 1 (q) Mean squru:e velocity
quantities in Column-II :
(6 refers to the
[here N - newlon; kg - kilogram; s - second; m - metre] dimension of
Column - I Column -II temperature)

(a) N/m2 (P) Force constant (e) MOL2T-2 (r) Latent heat

(b) N/m (q) Surface energy of a (d) Joule/kelvin (s) Specific heat
liquid
9. Match the physical quantities given in Column-II ~ith the
(e) N-m (r) Stress categories mentioned in Column-I:
(d) kg/s 2 (s) Bulk modulus Column - I Column - II
6. Match Column-! with Co'tumn-II in regard to the units of (a) Axial vector (P) Current
the physical quantities mentioned in Column-! and the units
(b) Scalar (q) Stress
of expressions in Column-II:
(e) Polar vector (r) Angular mom.entum
Column - [ Column - II
(d) Rotational vector (s) Torque
(a) Frequency (P) &oE2
10. Referring to figures (a), (b), (c) and (d), match Column-I
(b) Energy density (q) B' with Column-II :
-
~o

(e) Pressure (r) I


-
CR
- T Ime -lime
(d) Energy of a particle per (,) -R
unit angular momentum L ~

[Here. EO - permittivity of free space; ~ 0 - permeability of


(e) ~1 (d) ~1
free spnce; E - electric field strength; B - magnetic flux J: "-

density; R - resistance; C - capacitance; L - inductance] _ Volume -TIme


7. Match Column-I with Column-II:
Column - I Column - II
Column - I Column - II (a) In Fig. (a), slope of the (P) Work
(a) Dimensionless quantity (P) Angle
graph represents
(b) In Fig. (b), the shaded (q) Force
(b) Young's modulus (q) kg m- I sec- 2
area represents
(e) Joule/cal (r) Mechanical equivalent (e) In Fig. (c), the shaded (r) Impulse
ofheat area represents
(d) Pascal (s) Thermal conductivity (d) In Fig. (d), the shaded (,) Torque
area represents
8. Match the units/dimensions in Column-I with the physical
quantities/expressions in Column-II . II. Match Column-l with Column-II:
Column - I Column -II
Column - I Column - II
(a) loulelkg k,T
(P) - - (a) GM, M J (P) (volt) (columb) (metre)
m
G- universal
[k, - gravitational constant
Boltzmann constant
Me --mass of the earth,
T-
Absolute temperature, M J -mass of the sun
m- mass]
3RT
(b) - (q) (kilogram) (metre) 2 3. Dimension of time in pressure is:
-
M (second) -2 (a) 2 (b) 4
R -universal gas (c) -2 (d) -4
constant
Paragraph. 2
T -absolute temperature
There is a concept that ifunit ofa physical quantity is large
M -molar mass then the multiplier of the unit becomes less while
(c) F' (r) (metre)2 (second) - 2 expressing any fixed magnitude. If in a new system of unit,
- -
q 2n 2
unit of time is 0.5 seconds and the unit oflenith is 4 metre,
then answer the following questions.
F -force 4. Unit of speed becomes (compare to 'its initial value):
q-charge, (a) one eighth (b) eight times
B -magnetic field (e) half (d) doubled
5. The value of speed of light becomes (c is initial value) :
(d) GM, (s) (farad) (voU) , (kg)-l
R, (a) c (b) c / 2

G - universal 00 ~4 ~) ~8

gravitational constant, 6. Ifuni! of time is doubled, the unit of length is halved and
unit of mass is increased by a factor n such that a particular
Me- mass of the earth, force retains its value in both the system of units then the
Re - radius of the earth value of n is :
(a) 8 (b) 6 (c) 4 (d) 2

ANSWERS
1. a--. r; b-+ s; c --) p; d-+ q
Paragraph·l 1. (b) 2. (a) 3. (d)
2. a-+s;b-+r;c-+p;d-+q
Paragraph·2 4. (b) S. (d) 6. (a)
3. a-+ q,r; b-+ q,r; c-+ s; d-+ p
4. a -+ q,r. h-+ q,l; c -+ P,S; d -+ q,r IF] Subjective Type Questions
S. a-+r,S; b-+ p,q; c-+doesnotmatch; d-+p,q I. (a) Can there be a physical quantity which has no unit and
dimensions?
6. a -+ T,S; h-+ p,q; c-+ p,q; d -+ T,S
(b) Can a physical quantity have unit without having
7. a-+ p,r; b-+q; c-+r; d-+ q dimensions?
8. a -+ p,q,r; b-+ s; c-+ p,q,r; d -+ does not match [Ans. (a) Yes, strain (b) Ves, angle with units radian]
9. 3-+1,S; b-+ p; c-)o doesnolmatch; d-+ T,S 2. Name the smallest and largest unit of length.
(Ans. fermi, Parsec]
10. a-+ s; h-+ T; c-+ p; d-+ p
3. In which system mass is derived physical quantity and what
JI. a-+p,q;b-+r,s;c-+r,s;d-+r,s is its unit?
(Ans. FPS Engg; slug]
(E) Linked Comprehension Type 4. The columns in (A) and (B) are already matched, (B)
should be jumbled:
Paragraph - 1
(A) (8) An •.
If force, mass and time are taken to be base physical
(a) Base unit N (0) kg
quantities with dimensional fonnula [F], [M] and [T), then:
I, Dimensional fonnula for energy is : (b) Derived unit hp (b) N
(a) F 2M- 1T - 2 (b) p2 M -I T 2 (e) Improper unit kgwt (c) kgwt
(d) Practical unit r&d (d) hp
(c) F'M-'T-' (d) F'M1T'
(e) Supplementary unit kg (e) rad
2. Dimension of mass in velocity is :
5. Express the power of I00 W bulb in CGS units with proper
(a) -1 (b) -2
prefix.
(e) 1 (d) 2
[Ans. 10 erg/s]
6. What is the weight of a body midway between the weights 16. A large fluid star oscillates in shape under tho influence of
of 1 kg and 16 kg'! its own gravitational field. Using dimensional analysis fmd
(a) on the ordinary scale the expression for period of oscillation in tenns of radius of
star R, mean density of fluid p and universal gravitational
(b) on the logarithmic scale
constant G.
[Ans. (a) 8.5 kg, (b) 4 k.g)
[Ans. T '" K(Gpr l'2 ]
[HI nt: log 1+2\ogl6 II 0g 16 cog
"''2 I 16'" - Iog 4J
17. (a) Construct an expression for a quantity having the
. dimensions of length starting' frpm velocity of light c,
7. The SI unit of energy is joule while CGS unit is erg. How
the mass of an electron m ~ci P.lanck's constant h.
many erg are there in a joule? .' -' i",~ .
. (b) . E,vall\ate the value of this Isngth<takJng constant of
[Ans. IJ = IO'ergJ , "~I' , ....i 31
., ".j p~op01:tiohality co be unitY" 'm.y= 9.lx 10- kg ,
8. (a) Find the dimensions of Planck's constant h, (b) If its , . . \·Y."I·"~"· 8 - 34
value in eGS system is 6.62x 10- 27 unit, what will be its . . { = 3x,1O , 'n1Js and II ... 6.63x 1O~.,J~s..i"'~'
value in MKS system? [This wavelength is calleq Compton wavelength and plays
(Ans. (a) (ML2r-I ], (b) 6.62 )(10-34 J·sJ
an important role i!1 atomic physics.]
[Ans, Ca) ~t = (h/mc), (bp..c = 2:43 )«0- 12 m)
9. Calculate the dimensions of universal gravitational
constant G. What is the value ofG in SI units ifits value in 18. Match tbe physical quantities given in Column I with
CGS system is 6.67x iO- 8 units? (MNR 1992] dimensions expressed in Column II in a tabular form:
[Ans. [M- 1L3T - 2 ]; 6.67 xlO- l! SI units] I IT
10. The CGS unit of viscosity is poise (P). Find how many
(8) Angular momentum (P) [ML'r-')~ "';I'
;'"
poise are there in 1 MKS unit of viscosity called poiseuille (b) Latent heat (q) [ML'Q-']
(PIJ?
(c) Torque (r) [ML'T- ' ]
[Ans. 10 poise]
II. In a new system of units cal led Star units 1 kg* = 10 kg; 1 (d) Capacitance (s) [ML'r-1Q-' ]
m+ = 1 km and I s· = 1 minute. What will be the value of I J (e) Inductance (1) [M-1L-'r'Q']
of energy in this new system?
(f) Resistivity (q) [L'r-']
(Ans. 3.6)(10-4 J+]
(AlIS. (8) Angular momentum'" [ML2-r- 1),
12. What will be the dimensional fonnula for mass if we choose
length L, time T and force F as fundamental quantities? (b) Lalent heat ~ [Ll-r-2 ]
[Ans. [FL- 1r 2]J (c) Torque --)0 [ML2y-2)
13. What are the dimensions and sr units of van der Waals' (d) Capacitance--)o [M - 1L- 2r 2Q 2]
constants a and b?
(e) Inductance ... [ML2Q-2)
3
{Ans. [aJ ::(MLs.r-2~ -2)and uni ts J m / mol 2 while
(f) Resistivity ... (ML3T- 1Q-2)).
Ibl= IL1j.l -1 l and units ml / mol)
19. If the speed of light c(=3xI08 mls), Planck's constant
h(=6.6)( 10-34 l os) and gravitational constant G(= 6.67x
14. Assuming that in case of motion of blunt bodies in air
aerodynamic drag depends on effective area A of the body, 10- 1! MKS units) are chosen as the fundamental quantities,
the speed of body relative to air v and density of air 0', show find out the dimensions and value of units of (a) mass, and
by method of dimensions: (b) time in Ihis system.
[Ans. Q", = (ch/G)"2 _ 10-1 kg, Q, = (Gh/~)1 12 _ 10-42s)
D =KcrAv 2
[Note: Experimentally K = (l12)C, where C is dimensio!lless 20, Write the dimensions of each of the following in terms of
drag coefficient and has value about 0.5 for spherical objects but mass, length, time and charge:
can be upto 2 for irregular bodies.) , (a) Magnetic flux (b) Rigidity modulus
15. The time of oscillation of a small dl'C'p of liquid under [Ans. (a) [ML2T- I Q- l ] (b) [ML- Iy-2)]
surface tension depends upon the density p, radius rand
21. Check the correctness of the relation:
surface tension S as:
I
T « paSbrc c="";~

Find out a, band c. ~~o'o


[Ans. a:: (1I2),b = (-112) and c '" (3/2)] (a) dimensionally and (b) numerically.
[Ans. Correct both dimensionally and numerically)
24 PHYSICS FOR COMPETITIONS - Vol. [

22, The frequency of vi bralion f of a mass m suspended from a (g) Dimensions of lime in power is .. .
spring of spring constant k is given by a relation ofthe type (h) Planck's constant has dimensions .. .
f == cm xk Y. where c is a dimensionless constant, Find the (i) The dimensional fonnula of the universal constal1t of
values of constants x and y. gravitation is... (EAMCET 1992)
I I G) Three physical quantities which have same dimensiclns
(Ans, x=-'2'Y='2 1
are... ,
23. State whether the following statement is true or false. Give '.
(k) Mention a scalar and a vector physical quantity having
,
very brief reason in support ofyoue answer. same dimensions. . . "
(I) ohm)( farad is ditrtensionally equal to ...
The quantity _e_ is dimensionless. Here e, hand c are
2s ohc (m) Dimensional analysis oflhe equation:
electronic charge, Planck's constant and velocity of light (velocity)X - (pressure difference) 3/2 )( (densltyr 312
respectively and eo is the permittivity constant of free gives the value ofxas. "
space.
(n) The unit of an electrical parameter whose dimensional
[Hint: As F '" _ ,_
41(£0 r
~ so [l..]
2to
'" [Fr21, {onnula is (ML2 T-3 A - 2 ] is ...
(0) The dimension o{(LIRCV)is. . . (Roorkee 1994)
i.e, ~ .[F"]~
2&(Jch ch
[M 'L,",')) ~~ (P) One hp "', . , watt ;;; . , . ft·lbls.
(q) There are ... erg in a kilowatt·hour.
[Ans. True1 (This quantity is a universal constant called fine (r) One electron volt is equal to ' , . joule.
SlnlCfltre COl/Slant (= 11137) and plays an important role in atomic
[Ans. (a) erg X5 (b) electrical conductance (c) weber Cd) P and Pa
physics.]
('J[ML'T- ' )(Q[M[;IT-' J(gJ -3(h)[ML' r l ] (')[M-ILJr ']
24. Fill in the blanks:
U) work, energy and torque (k) work and torque (I) second (m) 3
(a) The units of angular momentum in CGS system are ...
(ML2T-2 ) W V L
(b) Thesiemen is the SI unit for . . . [EAMCET 1991) (n) (ML2r 3A - 2 ] -+ 2 -+ - -+ - -+ohm(o)-C ~
[A T] qA A R V
(c) SI unit of magnetic flux is.. . (EAMCET 1990)
(d) Symbols for the units poise and pascal are respectively !:. . -,_ -+ [T]x _' ~ [~l -+ [ A- I] (p) 746 and 550
R CV [qJ AT
(q) 3.6 x 10 (r) 1.6 x 10- 9)
13 1
(e) The dimensional formula for potential energy is .. .
(f) Energy, density and pressure have dimensions .. .
[EAMCET 1993]
Vectors
[In this chapter we shall discuss only some basics of vector current-density are defined as vectors with specific direction,
analysis and its applications to physics without going in e.g., to area the assigned direction is outward nonnal to the
details. For details of vector analysis the interested reader may _ area while in case of dipole-moment it is alqng the line joining
go through any of so many books on the subject.1 the charges from negative to positive charge (Fig. 2.1),
§ 2.1 Vectors' Vectors can be divided into two types:
If a physical quantity in addition to magnitude (which we [AJ Polar Vectors
have discussed in Chapter I): These have starting point (as in case ·of displacement) or
I. has a specified direction (Question I), point of application (as in case of force),
2. obeys the law of paraJlelogram ofaddition. i,e.,
R =(A 2 +B 2 +2ABcos9)"2 $
3. and its addition is commutative. i.e.•
--t
A+B=B+A
--t -+ -+

then and then only it is said to be a vector. Ifany of the above


conditions is not satisfied the physical quantity cannot be a
$~$''f ,<1'
,f>.

Starting point Point of application


vector.
<a) <b)
Note: Here it is worthy to note thai if a physical quantity is a vector it Fig. 2.2
has a direction, but the converse mayor may not be truc, i.e., if a
physical quantity has a direction, it mayor may not be a vector, IBI Al:ial Vectors"""
e.g., time, pressure, surface tension or Cl frrent, etc" havc These represent rotational effects and are always along the
dircction but are not vectors.
axis of rotation in accordance with right hand screw rule.
Velocity, force, weight, momentum, impulse, torque, Angular velocity. torqlle and angular momentllm, etc., are
electric field or temperature gradient, etc., are examples of examples of physical quantities of this type.
some physical quantities which are vectors. Here, it must also
be noted that in phys ics certain physical quantities such as Radius Vectot

4;<;~'-~-="'¥ cb
area (more precisely surface element), dipole-moment and

Antlclockwise .1Il ClockWise


~ Vector
Axial vectors
la) Ib)
Fig. 2.3
Area Dipole moment
(8) (b) Vectors have their own algebra which is different from
Fig. 2,1 ordinary algebra, e.g., in ordinary algebra
3+4=7andA x B=B x A
• In most oflbe books vectors are represented either by bold face Ictters orwitb an arrow over a simple letter. In this book in order to minimise printing errors
or to reduce confusion we have represented vcctors by bold face lettcrs with an arrow over it.
on In lower classes or in some books these vectors are also referred as rOlalional veclors and the direction is specified as clockwise ot Qnficlockwise.


26 PHYSICS FOR COMPETITIONS - Vol, I

while in vector algebra -jo -jo -jo -jo 2 2 2 2


r = ix + jy+kzwithr =x +y +z
-to .............. -+-+-+-+
3 + 4= 1 107 andAxB:F-BxA 9. Angle between two vectors means smaller of the two
Regarding a vector it is worth noting that: angles between the vectors when they are placed tail
to tail by displacing either of the vectors parallel to
1. If a vector is displaced parallel to itself it does not
change. [See Fig. 2.4 Call
e
itself (i.e., O:s; S 1t).

L~--. '~
, ,
~
, B
0' _
A A A

Y
!(
(a) (b) (c)

A ~__, ____ B , Fig. 2.7

7--~ -. 7 9 10. Angle between collinear vectors is always zero or

A • B .. C
Translation 01 a vector
Parallel to Iisell
(a)
Fig. 2.4
--
A' B
Rotation of a vector

(b)
180",
-.
B

e.oo
- •-
(a)
A 0'

Fig. 2.8
-
-
B

6.180°
(b)
•- A

2. If a vector is rotated through an angle other than


multiple of21t (or 360") it changes, [See Fig. 2.4 (b)] Question I. We can order events in time and there is a
sense of time, distinguishing past, present and future. Is,
3. If the frame of reference is therefore, time a vector?
translated or rotated the vector
does not change (though its "
components may change). "" , / '
{/'5' Answer: Time always flows from past to present and then
to future, so a direction can be 'assigned to time. However, as
the direction is unique, it is not to be stated, i.e., specified. As
4. Two vectors (representing "8'./' the direction is not to be specified, time cannot be a vector
values of same physical 0'-- --"---5 though it has a direction.
quantities) are called equal if Fig. 2.5
their magnitudes and Note: In the same way pressure (force per unit area nonnal to the area)
or surface tension arc scalars. These have direction which is
directions are same.
unique and so need not to be specified.
5. Negative of a vector means a vector of same
magnitude but opposite in direction. (See Fig. 2.6) Q uestion II. Explain why current is not a vector though it

Fig. 2.6
• -B--A - appears to possess a direction .
Answer: Current is not a vector as:
(1) The direction associated with current is not a true
6. Null vector· or zero vector is defined as a vector direction but it merely indicates the sense of charge
whose magnitude is zero and direction indeterminate. flow.
It differs from ordinary zero in the sense that ordinary (2) Current also does not obey the law of parallelogram of
zero has no direction but it has a direction which addition as if currents of3 and 4 ampere are reaching a
becomes indeterminate by virtue of its zero point, the resultant is always 7 ampere (like scalar
magnitude. addition) whatever be the angle between the two.
7. Any vector can be written as the product of unit vector (3) Current is defined as scalar product of current density
in that direction and magnitude of the given vector ...j withareads,i.e.,
...
i. e.,
...A=A ...n ......
or n = AIA. ......
1=1 j ·d ,
8. Unit vectors along x, y and z-axes are usually
...... ...
represented by I, j and k respectively. So if the
Now as scalar product of two vectors is always a scalar. sc
current is a scalar and not a vector (See Question VI].
co-ordinates ofa point are (x, y, z), the position vector Question III. Discuss whether or not angula!
displacement is a vector.
• For more details see Question IV.
V ECTORS 27

Answer: Angular displacement represents rotational effect, vector. For example, moment o/fnertia is not a vector as by
changing the sense of rotation its value is not changed. It is

) also not a scalar as it has different values in different


go,
U90' ~ ,,,', directions (j.e., about different axes). Such physical
quantities are called tensors. *

Q uestion IV. Does it make sense to call a quantil'y vector


when its magnitude is zero? What are such vectors called and
) why needed ?
90' go' ->
a Answer: Imagine a vector A and let its magnitude and
hence length be reduced to zero by coinciding the initial and
terminal points. In this situation the modulus or length of the

u~
) 45'
vector will become zero and its direction indeterminate, i.e., it
may be regarded to possess any direction. So, it has a sense to
45' call a quantity a vector when its magnitude is zero.
9H92
Such vectors are called zero or nllll vectors and are
denoted by the clarendon symbol O. All null vectors are

~ ~,
regarded as equal and eliminate the need of stating exceptional
cases when writing certain relationships in vector fonn . For
U45' example, as the sum of any two vectors is a vector, the sum of
b -> ->
two equal and opposite vectors (A and - A) must be zero and a
vector, i.e., will be null vector, or area of sphere in vector form
is a null vector.
91+412 § 2.2 Addition of Vectors
The rule for adding vectors is illustrated in Fig. 2.10 (a).
-> ->
To add B to A shift any of the two vectors parallel to itselfuntil
-+ -+ -+ -+ -+
the tail of B is at the head of A . The sum A + B is a vector R
... ->
drawn from the tail of A to the head of B • i. e.,
-> -> ...
A+ B = R "." (i)
As the figure fonned is a triangle, this method is also
so, appears to be an axial vector. Furthermore as it obeys the law called' triangle method' of addition of vectors.
ofparal\elogram of addition so it may be a vector. However, it
If the 'triangle method' is extended to add any number of
has been found that large angular displacement does not obey
vectors in one operation as shown in Fig. 2.10 (b), the figure
the law ofcommutation while infinitesimal do, i.e.,
formed is a polygon.
c
This is self-evident from Fig. 2.9. So, angular
displacement is a vector if small and not a vector if large. "
... ,'"<> .~
~
Note: (i) From this example it is clear that certain physical quantities
can behave either as a scalar or a vector and how these
behave depends on the circumstances. This is somewhat like
dual nature of light or matter!
(ii) Here it is also worthy to note that there are also cenain
A
-
A
(8)
B A
(b)
physical quantities which behave neither as scalar nor as FIg. 2.10

'" Physical quantities which have no specified direction but different values in different directions are called tensors. Density p, refractive index II,
dielectric constant K, electrical conductivity a , stress and strain, etc., which are nonnally scalars in anisotropic media, assume different values in
different directions, so become tensors. Pulthennore if a physical quantity has only one component it is called a scalar or tensor of rank zero; if it
has components> I but:$ 4 it is called a vector or tensor of rank one and if it has components> 4 it is called a tensor of higher rank.
28 PHYSICS FOR COMPETITIONS - Vol. I

The addition of two vectors may also be understood by the 3. Vector addition is associative, i.e.,
-. -+ -+ -+ -+ -+ -+
law ofparallelogram. According to this law if two vectors P P+(Q+ R)=( P+Q) + R
-.
and Q are represented by two adjacent sides of a 4. As R =[P' +Q' +2PQcos9]"'
parallelogram both pointing outwards as shown in Fig, 2.11 so if out of R, P, Q and 9 if one is unknown, it can be
(a }/hen the diagonal drawn through the intersection oflhe two calculated analytically.
vectors represents the resultant (i.e., vector sum of P and Q).
-. -. 5. As R = ~rp'="-+-Q-"~+-2P-Q-C-O-S-9
-. so R will be maximum when cos a = max = I, i. e.,
Note here that if Q is displaced from position AD to BC by
displacing it parallel to itself, this method becomes equivalent e =0°, i. e., vectors are like or parallel
to the triangle method. and Rmax = P +Q
o o 6. The resultant will be minimum if, cos a = min = -I,
-
o --- ---- - - 0T
A - I
01
'I
I
~
c
Ui
i.e..9=180°,i.e., vectors are anti-parallel or unlike
andRmin =P- Q
I I 0
8~'.J E 1 7. If the vectors P and Q are orthogonal, i. e., 9 =90°
A
=~p2 +Q 2
A B
I '1 -I R [ascos90"=0}
Oeas.
(aJ (bJ
,,
A_ I;'+Q • r-:::+i
,,A_P-Q
Fig . 2.11
,, i3 , ,,,
0 0',, ,' >I -0 ,,
• ,,
In case of addition of two vectors by parallelogram
method in the light of Fig. 2.1 t (b), the magnitude of resultant
</-'
, ,
, , ,
will be given by P
., ' P P
~ ,

e_0° 9 .. 90° 9 _ 180°


(AC )' = (AE)' +(EC)'
Rm.... -P+Q Rom." P- Q
or R' =(P+Qcos9)' + (Qs;n9)' (aJ (bJ (eJ
Fig. 2.12
or R =:~p2 +Q2 +2PQcos9 .... (a)
8. From points 5, 6 and 7 it is evident that the resultant
-. of two vectors can have any value from (P - Q) to
The direction of resultant from P will be g iven by
(P + Q) depending on the angle between them and the
tan$ = CE = Qsin9 magnitude of resultant decreases as 9 increases frorr
AE P+Qcos9 0° to 180°.
i.e., ~ = tan~t[ Qs;n9 ] .... (b) 9. As Rmin=P-Q (when 9". ISO" from point 6)
P +Qcos9 so if P":#. Q, Rmin:F- 0
Regarding vector addition it is worth noting that: i.e., resultant of two vectors of unequal magnitudE
I . To a vector only a vector of same type can be added can never be zero.
and the resultant is a vector of the same type. For 10. So, minimum number of unequal vectors whose su"
example, to a force only a force and not velocity can can be zero is three.
be added and the resultant will be a force and not any ........ -.
other physical quantity. Let P+Q+ R =O
2. Vector addition is commutative, i.e., .... -. ....
i.e., R=-(P + Q)
-+ -+ -+ -+
P+Q=Q+ P This in tum implies that in case of three vectors th.
In Fig. 2.11 (a) in llABC, resultant may be zero and it will be only when on.
.... .... -. vector is equal to the negative of the sum of tho
P+ Q =R ····0) remaining two vectors, i.e., vectors are coplanar.
.... -. .... II. From the above it is also clear that the resultant oj:
while in MDC. Q + P = R nOli-coplanar vecJors can never be zero or minimun
So from Eqns. (i) and (ii), number of non-coplanar vectors whose sum can b
-+ -+ -+ -+
zero is four.
P+Q=Q+P
VECTORS 29

12. Subtraction a/a vectorfrom a vector is the addition of Solution: In this problem, P = Q = mv and a =90 0

negative vector, Le.,


so, R = [(mu)' + (mu)' +2(mu)(mu)cos900]tl2
-Jo -+ -+
P - Q=P+(-Q)
This is illustrated in Fig. 2,13.
-+
and ~=tan- I [ mvsin900
mv + mvcos90°
1
,, i.e., R = (Ji)mu
,, ,
-
p ,~,
and ~ = tan -I (I) = 45°
N
,,
,----- i.e., resultant momentum is of
.
(.)
V ,-'"

Fig. 2.13
o:..;
(b)
-0
- 180"- 9
magnitude (.Ji )mv directed along
N-E direction.
~
Now if V is the final velocity
m
.- -: ----.j<-_:--_~
, P _mv
I
R ,,,
,
,
.. E
~

In case of subtraction of a vector from a vector (2m)V =(Ji)muNorth-east Iv


,,
(i) R = [(p)2 + (Q)' + 2PQ cos (180' _ 9)] ln
i. e. , V=( ;,. ) North-east .m
i.e., R =~p2 +Q 2 - 2PQcos9
Fig. 2.15

[as cos(1800- 9) = -cos 9] Problem 3. A body is moving with uniform speed v in a


horizontal circle in anticlockwise direction startingfrom A as
(ii) Subtraction is not commutative, i.e., shown in Fig. 2.16 (a). What is the change in velocity in (a)
~ ~

[but= -(Q- P)] hal/revolution (b)first quarter revolution?


8 V; N North
(iii) Change in a vector physical quantity means
subtraction of initial vector from the final vector.
Problem 1. What is the displacement of the point of a AV,
-
:1' 1 ,
_
'3 '
, ,
"Cfj---East West I ~
vt-

,
lV ,

wheel initially in contact with the ground when the wheel roJis A'l I ___
lJ.v . -v,
.....
forward haifa revolution ? Take the radius a/the wheel as R s South
and the x-axis in the forward direction.
B
Solution: In ....... -r ...... (s) (b) (e)
. h F' I ,
aecor dance Wit Ig. /
I
: \ "Q Ag.2.16
2.14 during the half i---r----i ~ ~ ~

revolution of the wheel, \. : '.1.'I' Solution: Change in velocity means 6. v = VI - Vi


the point A covers
"
"'-A
.../

P c , (a) As shown in Fig. 2.16 (b), for half revolution:


1tR(= AC) horizontal
distance while 2R (= CB)
Fig. 2.14
if and
vertical distance, -+ -+ -+ -+ -+
So, here P = nR; Q = 2R and a = 900 so, tlV=v3-v \ = v - (-v)

D=~[(1tR)2 +(2R')]=R~.' +4
~ ~
so, i.e. , l1v=2v

and ~ = tant~l=tan-Hl i..e,


~

11 V = 2v directed south
(b) As shown in rig. 2.16 (c), for quarter revolution:
i.e., displacement has magnitude R~1t2 + 4 and makes an ~ ~ ~

angle tan""'l (2In) with x-axis. I1V = v2 - VI and a =90 0


Problem 2. A particle ofmass m moving eastward with a so 6V =~v' +v' = (Ji)u
speed v collides with another particle ofthe same mass moving
northwards with the same speed v. The two particles coalesce and ~ =tan -I(~ )= 450
on collision. What is the momentum and velocity of the new
particle? ~

tl V = (.Ji)v south-west
30 PHYSICS FOR COMPETITIONS - Vol. I

~ ~
Here it is worthy to note that once a vector is resolved into
Note: If the resultant of vI and "2 is to be calculated its components, the components themselves can be used to
~ ~ .... --+ specify the vector as:
V = vl + V2 I.e, V =.[iv(N-W) (try it)
->
so in case of two orthogonal vectors of equal magnitudes. I. The magnitude ofthe vector R is obtained by squaring
magnitudes of sum and difference are equal but their directions and adding Eqns.(ii) and (iii), i.e.•
are different.
.... (a)
-> ->
Problem 4. What is the property a/two vectors A and B, ->
if: 2. The direction of the vector R is obtained by dividing
-+ .... ' -+ -+ -+ -+ -+ -+ Eqn. (iii) by (ii), i.e.,
(a) 1M BI =IA - BI (b) A+B=A - B tan 9= (RyIRx) or e=tan- I (RylR,Y) .... (b}
Solution: (a) We know that
Note: IfR - Rx,Ry-OandifR " Ry. Rx "'O
1;\+ BI =~[Al +B' +2ABcos9] ' i.e., component ofa vector peIpendicular to itselfis always zero.

IA- BI =~[A' +B' -2ABcos9]


Importance of resolution of. vector into componeD'
and
"
lies in the fact that according to it the x-component 0:
According to given problem '.," ' . resultant will be equal to the algebraic sum 0 :
~[A' +B' +2ABc059] =~[A' +B' - 2ABc059]
x-components of all vectors (and same fo:
y.:component) and so we can calculate easily Rand (
which on solution gives cos 9 "" 0, i.e., e= 1tI2 using Eqns. (a) and (b).
-> ->
i,e., the vectors A and B are perpendicular to each other. Note: Significance of components
-+ --t -+ -+ Suppose an airplane moves in north·east direction with a veloci~
(b) Given that A+ B=A-B of V mls. Then the effective velocity of airplane along eal
-> -> direction is Vx - V cos 45° and along north Vy "" Y s~n 45.°.
j,e., 2B=0 or B =0 Displacement along east in I second - Vx t '
-> Displacement along north in t second - Vy t
i, e., B is a null vector. A component gives effective value of a vector in .a particuil
direction. Forexample the force Fin Fig. 2.18 (a) can be replace
§ 2.3 Resolution of vector into Components by its rectangular components. Similarly when a block slidf

.,...,.....'.. ,
-> down an incline [Fig. 2.18 (b)], component of weight alan
Consider a vector R in x-y plane as shown in Fig. 2.17. If a
incline is mg sin andnonnal component of weight is mgcos e.
-> ->
we draw orthogonal vectors R x and R )' along x and y axes , . ........,.'
,," ,,
respectively. by law of vector addition,
T ':
->
R=Rx+ R y
-> ->
::_ ________l..:,
vY"
,
V :,

Now as for any vector A = ;4 n


-> -> y 1 ~

I--V.-I
-+ -+ -+-l>
F,t
50

50
R;r=lRxand R y = jR y
-> ->
R = IR,+ JRy
But from Fig. 2.17
->

A,
.... (i)

-
-
R,

,
d •

(a)
.u- F
,

Rx =Rcos9 .... (ii) Fig. 2.17


and Ry =Rsin9 .... (iii)
mg sin 9
Since Rand 9 are usually known, Eqns. (ii) and (iii) give
->
the magnitude of the components of R along x and y-axes
(b)
respectively.
Fig. 2.18 • /
VeCTORS 31

Irlhe co-ordinates of a poil1tP are (.t. y, t), vector joining point P Solution: In tenns of components
to origin is called position vector. The position vector of point P
~ ~ ~
-+ -+ -+ -+ -+ -+
is r'" Ix+ J y+ kz with magnitude of position vector A+B=(IA,+ JAy)+(IB,+ JB y)
represented by ". - xl + I +;. -+ -+ -+ -+
V-axis Y i.e., A + B= I(A ,+B,)+ J(A y +By)
According to given problem,
Ax+Bx = IO and A y +B y =9
~
r, and Ax=4 and A y =6
'""'::::::"'_-_x So, (a) B x =1O-4=6mandB y =9 - 6=3m

Fig. 2.19 (b) B=~B; + li;=~62+32=.J45m=3.J5m


-+ -+
e=lan-I[~J,an-lm=tant]=266.
-+ -+
If a particle is displaced from position, rl =< x, 1 + YI j + zl k to
-+ -+ ....,. -+ -+-+
(c)
r2 '" x2 I + Y2 J + Z2k then rl- r, is the displacement of the
particle. § 2-4 Multiplication or Division ofa Vector by a
-+-+ ... -+ -+
rr r, '" (X2 -xl) I + (>'2 - >'I)j+ (z2 -z,)k Scalar
-> -> ->
-> If there is a vector V and a scalar S then as V = V n
3. In three dimensions, a vector R in components along
-> -> -> ->
x, y and z·axes can be written as: SV = (SV)n and VIS = (VIS)n
-> -> ->
OP~OB+BP i.e.,
(a) In multiplication of a vector by a scalar, the
-> -> -> z
=OA+AB+BP magnitude becomes S times while direction remains
, the same.
-> -> -> -> p ,
or R=Rx+ R y+ R : -- ._--- (b) In division of a vector by a scalar, the magnitude
-> -> ->
R becomes (liS) times and direction remains the same.
= iRx+ JR y+ kR z 0---__ y
Note: (;) It is worthy to note here that in vector algebra there is no law
with =~R2, +R2y +R2,
R x
A 8
of division (of a scalar or vector) by a vector, i.e., (SN)or
~

Fig. 2.20 ~~
R R (AN) are not defined, so a scalar or a vector, can neve,. be
and cos9 x = --!., case y = 2
R R divided by a veclor.
(ii) In case of multiplication of vectors, like scalars, vectors of
and cose r =R z different kinds can be multiplied to generate new physical
R quantities which may be a scalar or vector, If in
called direction cosines of the vector. mUltiplication of two vectors the generated physieal quantity
is a scalar, the product of vectors is called scalar or dOl
QuesHon V. Can a vee/or be zero ifany o/lts components product [dealt in § 2.5] and if a vector, the product is called
is not zero? vector or cross product [dealt in § 2.bJ.
Answer: In terms of components in three dimensions,
-+ -+ -+ -+
Problem 6. Apartic/e o/mass 3 kg moves underaforcc, of
A=iAx+jAy+kA z with -> -> ->
[4i +8j+ IOk] newton. Calculate the acceleration (as
so ifany ofAx,AyandA z is not zero, vector caMot ~e trro, i.e., vector) to which the particle is subjected. If the particle starts
for a vector to be zero (or nUll) all its components mus( vanish. from rest and was at the origin initially, what are its new
-> co-ordinates after 3s?
Problem 5. The x andy components 0/ vector A are 4 and
Solution: We know that according to Newton's laws of
-> ->
6 m respectively. The x andy components 0/ vector A + Bare motion
-> -> -> -> ->
10 and 9 m respectively. Calculate lor the vector B the F =ma i.e. , a = Flm
/oJ/owing: -+ -+ -+ -+
(a) its x andy components, (b) its length, and (c) the angle But ashere F=4i+8j+lOk and m=3kg
it makes with x-axis.
32 PHYSICS FOR COMPETITIONS - Vol. I

... ............-+ ~ ~ ~

• =(1/3)[41+Sj + IOk]mls' (A)' =A·A = AAcos6 =A'--- -


Further according to the equation of motion ~
............ -Jo 2
i. e., A =~A·A
S = ut + (1/2) al ~

-+ -+ -+ -+ 8. In case of unit vector n


,=O x3 +(II2)(1 /3)[41 +Sj+IOk]x3' ~~
n'n = lxlx cosOO= l
-+ ...... ...... -+
i.e., s = [61+12j + ISk] -) -) -) -) -+ -) -)-)
so n·n=i·i =j· j=k·k=l
so the co-ordinates of the particle after 3 s are [6, 12, 15].
~~ ~

§ '2.5 Scalar Product of Two Vectors 9. In case of orthogonal unit vectors i . j and k .
(A] Definition - ) -)0 -)0 -)0 -)0-)0

The scalar product (or dot product) aftwo vectors is defined as l·j = j·k = k·i = lxlcos900=0
the product of the magnitude of two vectors with cosine of 10. In tenus of components
angle ben.veen them. -+ -)0 -)0 -)0 -t -+ -)0 -)0

~ ~ A·B=(IA, + JAy+kA,)'(ID, + jB y+ kB,)


Thus, if there arc two vectors A and B having angle €I between
~ ~
which in the light of points (3), (8) and (9) becomes
them, then their scalar product written as A· B is defined ~~

~ ~
A·B =[AxBx + AyB y + AzB z ]
A · B=ABcos9 .... (i) ICJ Examples
(BI Properties (a) Work W
1. It is always a scalar which is positive ifang\e between In physics for constant force work is defined as
the vectors is acute (i.e.• < 90") and negative if angle W=Fscos9 .... (i
between them is obtuse (i.e., 900 < €I < 180<». But by definition of scalar product of two vectors
2. It is commutative,i.e., ~~

-+ -+ -+-+ F · s = Fscos9 .... (II


A·B= B·A So, from Eqns. (i) and (ii),
3. It is'pistributiw(f. e., ~~

-+ -+ -+ ........................ W=F · s
A·(B+C)=A·B+A·C i. e., work is the scalar product afforce with displacement
4. As by definition (b) Power P
~~
~ ~
A·B=AB cos9 As W =F·s
the angle between the vectors ~
~
dW =F.~ [as F is constan
6 -cos
- _'[A 0]
--
AD
dt " dt

Fig. 2.21
or
S. Scalar product of two vectors will be maximum when
cos9 = max = 1, i.e.,9 = 0°, i.e., vectors are parallel,
~~ i. e., power is the scalar product afforce wilh velocity.
and (A·B)~ =AD
(c) Magnetic Flux <I>
6. Scalar product of two vectors will be minimum when Magnetic flux through an area is given by
Icos91 = min = 0, i.e., 9 = 90°
d$ =Bdscos9 .... I
~ ~

(A· B)",o =0 But by definition of scalar product


~ ~
i.e., if the scalar product of two non-zero vectors B·ds =Bdscos9 .... (
vanishes, the vectors are orthogonal.
So, from Eqns. (i) and (ii), we have
7. The scalar product of a vector by itself is tenned as
selfdOl product and is given by
-+ -+ -+ -+ So from Eqns. (i) and (ii)t we have
d$ = B·d. or $ =!B·d. -+ .... -+ ....
dI= J ·ds or I=! J· d.
ds .., .., ..,
Problem 7. Under a force (101 - 3 j + 6k) newton a body
.., .., ~

a/mass 5 kg movesfrom position (6i + 5 j - 3k) 111 to position


.., .., ..,
Magnetic flux Current and CUffant density
(lOi - 2 j + 7k) m. Deduce the work done.
(al (bl
.., ~ ..,
Fig. 2,22 Solution: As displacement s· =r2 - r l
-+ -+.... -+ -+ -+ -+
(d) Current I i. e., s = (I0 i -2J+7k )-(6 i+ 5J-3k)

Current is defined as scalar product of current density J


~ ~ .., ~

= (4 i -7 J + IOk ) m
with area, i.e.,
.., .., so
-+ -+
W = F·s =(I0i-3j+6k)·(4i - 7j+IOk)
-+ -+ -+ -+ -+ -+
I=! J "ds
i.e., W = (40 + 21+60)= 12Ijoule
(e) Potentlal-energy of a dipole U
.., Problem 8. A particle moves in the x·y plane under the
..,
Iran electric dipole af mament p is situated in an electric
action ofaforce F such that the value o/its lineal' momentum
.., ~
..,
field E or a magnetic dipole of moment M in a magnetic field
.., p at any time t is Px = 2 cos t andpy=2 sin t. What is the angle a
of induction B, the potential energy of the dipole is given by:
.., ..,
.., .., between F and p at agivell time t? IMNR 19911
and UB=- M · B -+ -+ -+ -+ -+
Solution:As p= i px+ jp y= i (2cost)+ j(2sint)
Note: From the above it is clear that physical quantities work. power,
flux (elec/ric or magnetic) and Cllrrent can be ex pressed as scalar 1;1=~[(2 COSI)' + (2sio ,')] = 2
product of two vectors. Now as by definition, scalar product of
two vectors is always a scalar, so these physical quan tities are
..,
-+ d p -+ d -+ -+
scalar. Nowas F = - , F = - [ i (2cost) + j (2sin t)]
dt dt
.., ~ ~ ~
Question VI. If current density J is defined as a vector so F = i (-2sint )+ j(2cost)
with magnitude equal to current per unit area, area being
-normal to the current and direction in which currentj1ows,
.., .., with IFI= ~{(-2sin[)2 + (2cost)2] ==2
show/hat! =J J ·d s .., ..,
Now as F · p =FxPx+ FyP y
Answer: By defini tion
= (-2 sin t)(2 cos t) + (2eos t)(2sin t) = 0
1= dl k
..,
ds cos
~
a
9= cos _,[it.;]
F'i = cos -,[2 0]
x 2 = cos
-, 0
or k dl =J dscos a
~ ~ ~ .., ..,
k dl=J dscos ak [as J = J k] i.e.,
~

9 = 900 i.e., Fand p areorthogonal .


..,
Now taking scalar product of both sides with k, § 2.6 Vector Product of Two Vectors
-+ -+ -+ -+ IAJ Definition
k · k dl=Jdscos 9 k · k
.., .., The vector product or cross product of two vectors is
defin ed as a vector having a magnitude equal to the product of
or dl=J dscos a [ask·k= l] .. .. (i)
the magnitudes of two vectors wilh the sine of angle between
But by definition of scalar product o f two vectors, them, and direction perpendicular to the plalle containing the
~ .., t.vo vectors in accordance with right hand screw rule. *
J ·d s = Jdscos9 .... (ii)
• On looking fronl the back ifin clockwise rotation the s(;I"CW moves forward, then it is called R.H.S.R.
.........
- --
C=AxB
IB} Properties
C-AxB +c - -B
c::.b-A x I. Vector product of two vectors is always a vector
perpendicular to the plane containing the two vectors.

i.e., orthogonal to both the vectors A and B, though


... ...
... ...
the vectors A and B mayor may not be orthogonal.
, 2. Vector product of two vectors is not commutative, i.e.,
..... ...............
A x B*BxA
... ...
[but=-BxA}
(aJ (bJ Here it is worthy to note that
Fig. 2.23
... ...
Thus, if A and B are two vectors, then their vector product
-+..... .....-+
lAx BI=I Bx AI= ABsine
... ... ...
written as A x B is a vector C defined by
~ ..... .... ....
C=AxB ::: ABsin9n
...... ...
The direction of A x B, i.e., C is perpendicular to the plane
... ...
containing vectors A and B and in the sense of advance of a (a) (bJ
... ...
right handed screw rotated from A (first vector) to B (second Fig. 2.25
vector) through the smaller angle between them. Thus, if a ..... -+ ..........
right handed screw whose axis is perpendicular to the plane i.e., in case of vectors A x Band B x A magnitudes
.... .... -)0 ....
are equal but directions opposite [See Fig. 2.25).
framed by A and B is rotated from A to B through the smaller
angle between them, then the direction of advancement of the 3. The vector product is distributive when the order of
...... ...
screw gives the direction of Ax D, i. e., C [See Fig. 2.24 (a)].
the vectors is strictly maintained, i.e.,
..... ..... ..... -+..... ..... -+

--
Ax (B+C)=Ax B+AxC

tA X. ......
Ax. 4. As by definition of vector product of two vectors
... ...
Ax B = ABsin9
~

.1
so,
... ...
lA x BI = ABsin9
IIf-----.
1x--BI]
1

i.e., e =sm. _1[1


......
-
IAI I BI
(oj (bJ
5. The vector product of two vectors will be maximum
Fig. 2.24
when sin a = max = I, i.e., 9 = 900
Note: Right Hand Screw Rule: ... ... ...
... ...
Pial the vector A and B tail to tail (this defines a plane). Now
[Ax BJmax = ABo

place stretchc? fingers and thumb of right hand perpendicular to


i.e., vector product is maximum if the vectors are
... ... ...
the plane of A and 8 such that the fingers are along the vector A.
orthogonal.
If the fingers are now closed through smaller angle so as to go 6. The vector product of two non-zero vectors will be
.... ........ .... minimum when lsin 91 = minimum = 0, i.e., 9 = 00 or
towards B, the thumb gives the direction of A x B,i.e.,C {See
180Q and
Fig. 2.24 (b)).
... ...
[A x B]min = 0
i,e., if the vector product of two non~zero vectors --> --> -->
vanishes, the vectors are collinear. (d) Poynting vector S = Ex H
7. The self cross product, i.e., product of a vector by -->
(e) Force on a charg~d particle q moving with velocity v
itself vanishes, i.e., is a null vector -->
-+ -+ -+ -+ in a magnetic field B is given by
i.e., AxA=AAsinOon = 0 --> -->-->
F=q(vxB)
-->
8. In case of unit vector n from point (7) (t) Torque on a dipole in a field
--> --> --> --.)...... ..................
ox n =0 ~

tE = px E and tB =Mx B
-+ -+ -+ -+ -+ -+ -+
so that ix i = jx j =kxk= 0 --> -->
Question VII. If Land L' are two length vectors, what
--> --> --> --> -->
9. In case of orthogonal unit vectors i, j. k in physical quantity does [Lx L'] represent?
accordance with right hand screw rule:
r
Answer: According to
definition of vector product of
-- L)( L'

two vectors,
--> --> -->
Lx L' =LL'sin8 n
J---_i But we know that area of a
parallelogram
S =LL'sin 8
(a) (b) --> --> -->
FIg. 2.26 so LxL' = Sn
--> -->
--> --> --> --> --> --> So [Lx L'] represents the area of the parallelogram with side~
jx k = i and kx i = j
--> -->
And as cross product is not commutative, Land L' in vector form.
y
--> --> --> -+"7 -;+ --> --> --> Question VIII. If a particle
jx i =-k, kx J =- I and ixk= - j oj mass m is moving with m v
10. In terms of components constant velocity v parallel to T
--> x-axis in x-y plane as shown in b

--> -->
i 1 -->
k Fig. 2.28, calculate its angular
momentum w.r.t. origin at any
1IL_ _ _ _ _ _ _ _ ,
Ax B= A, Ay A,
time t. Fig. 2.28
B, By B,
Answer: We know that
meaning thereby --> --> -->
Angular momentum L = r x p in terms of components
-+ -+ -+ -+
becomes
Ax B = i (AyB z -AzB y )+ j(AzB x - AxB z )
--> --> -->
j k
-->
L= x y z
[C) Examples
p, Py p,
Since vector product of two vectors is a vector, many
vector physical quantities like -torque, angular momentum,
As motion is in x-y plane (i.e., z = 0 and pz= 0)
velocity and force on a moving charge in a magnetic field can
be expressed as the vector product of two vectors. It is --> -->
L = k(xpy - yp,) .... (i)
well-established in physics that:
--> --> --> Herex =vt; y=b;p;x =mv and Py =0
(a) Torque t = rx F ..., --> -->
--> --> --> so L = k[vtxO-bmv]=-mvbk .... (ii)
(b) Angular momentum L = r x p
Note: From this question, it is clear that:
--> --> -->
(c) Velocity v = (.Ox r (i) If motion is in ;x-y plane angular momentum is always
directed along z-axis [Eqn. (i)], i.e., angular momentum is
always perpendicular to the plane of motion.
(ii) Ifmotion is parallel to x-axis with constant velocity, angular ~ ~ ~
momentum w.r.t. origin is independent of time and is Now A·(BxC) = Acos9(BCsin~)
COllstant [as according to Eqn. (ii), its magnitude mvb and
direction (along negative z.axis) both are constant]. = volume of parallelopiped
Regarding scalar triple product,
~ ~ ~

Problem 9. Considering two vectors, F=(4i-IOj) It is worth noting that:


~ -+"""* -7-7 (i) It represents the volume ofparallelopiped represented
newton and r = (-5 i -3 j) m compute (rx F) and state what ~~ ~

physical quantity it represents? by edges A, Band C. (8-C)


~~~~,'
-7. -+ . -j. -+ -7 -7 -7 -7 ~ ~ ~ /~~~~----- //!
Solution: As F = (4i -10 j +Ok) and r= (-5 i - 3 j +Ok) (ii) A·(Bx C) = O implies "
, , ___
__ ~
I
I
I
" I __ - I I
that vectors are coplanar. ,_~--
__ I II II

~ ~ ~ ~ ~
• ,
,, ,, ,
,
~ ~
I j k i j k (iii) In scalar triple product
,,
, ,, ,,
rx F = x y z = -5 -3 0 dot and cross can be
.:
-A'I
~,
,
', ---;
r;I
,---- ,/'/__ - I,
I

interchanged provided 8: a
Fx Fy · F, 4 - 10 0
that their cyclic order is <:, "' '
,,
maintained.
• "
i. e.,
~ ~ ~

rx F =.k(50+12) =62k N-m


~
(iv) In component fonn,

•8
Fig. 2.29
~ ~
The given physical quantity r x F repre~ents torque (i.e.,
~ ~
moment of force) if F represents force and r is the position
vector.
§ 2.7 Scalar Triple Product (v) Four points A, B, C and D are coplanar if
-7-7-4 ..
~ ~ ~
A· (Bx C) is scalar and so is called scalar triple product. From AB·.(BCxCD) =0
the diagram,
~ ~
BxC,=BCsin$

MISCELLANEOUS SOLVED PROBLEMS

Problem 10. The sum o/Jh.e m,,agnitudes o/twojorees Substituting for Pin Eqn. (i),
acting at a point is 18 and the magnitude of their resultant is Q=13
12. If the resultant is at 900 with th¢ force of smaller SO the magnitudes of forces are 5 and 13 units.
magnitude, what are the magnitudes .a/lm·ces?
Problem 11. A swimmer who can swim in stillcwater at
Solution: Let P.be the smaller force, then it is given that speed, V wants to cross a river flowing at -speed v. In what
P+Q = 18 .. .. (i) direction .s hould he swim to cross the river (a) in least time·
R = [p2 +Q2 + 2PQcos 8]112 = 12 .... (ii) (b) along shortest distance?
and Qsin e/[p + Qcos8] "" tan$ = tan 90°= co Solution: (a) Let the swimmer start swimming in a
direction making an angle 8 with OB as shown in Fig. 2.30 (a).
i.e., P +Qcos8 =0 .... (iii)
If L is the width of the river, time ' taken by the swimmer to
Substituting the value of P from Eqn. (i) in (Hi), cross the river will be
Q(1-cos9) = 18 .... (iv) L
t = - - [as component of V alongOB =Vcos8]
Now subtracting square of Eqn. (ii) from (i), V cose .
2PQ(1-cos9)=18' _12 2 = 180 .... (v) This time will be minimum when
Dividing Eqn. (v) by (iv), cos8 = max = I, i. e.,
2P=IO, i.e., P = 5
• In this prublem usual1y one thinks that when distance is shortest the time will be least. However it is not so, as for~hortesl distance velocity in the direction of
shortest distance is V cos 9 « V) hence time (LtV cos 9) is not least.
37
i. e., to cross the river in shortest time the swimmer should swim But from Fig. 2.31,
perpendicular 10 the flow.
(::)=u .nd ( f ) =cose
A~
A B

V dy u
, L <l
90'
so - =--
dt cos9
o o ·i.e., upward speed of mass M is (u/cosS)
(Shortest time) (Shortest distance)
Note: As the mass goes up, Swill increase, so cos Owi ll·deerease and so
('J (bJ vertical speed of M will increase.
Fig. 2.30

Note: (i) In this situation lime taken to cross the river will be: Alternative Solutions
II '" (LIV) (A) As I and S both are changing with time, vertical
(ii) In this time the flow of water will take the swimmer from B component of velocity of mass M will be
to C such that
d
BC =. V XI. == (uLlV ) - (lcose) = v (say)
(iii) The resultant displacement of the swimmer from initial point dt
OC '" D J (say) == ~L2 + (vLlV)2 == L~l+ (v/V)2 or dJcosS....,/sina d9 =V .... (i)
dt dt
(b) As the shortest distance between two banks is the And as there is no horizontal motion of mass M,
perpendicular distance between them, i.e., L, so in order to
cross the river along shortest path OB(::= L) the swimmer .:'.(/sine)=O
dt
a
should swim at an angle to DB such that the horizontal
component of his velocity balances the speed of flow, i.e., i. e., d/ sinS+/cosSd9 = 0 .... (ii)
dt dt
Vsin9 ;;;;: v or sin 8 = (vlV) or 8=sin - 1(v/V)
Now taking (dl/dt) = u (given) and substituting (dS/dt) from
i. e., to cross the river along shortest path swimmer should Eqn. (ii) in (i), we get
swim at an angle (90°+9) to the direction of flow, with
. (-usine )
v = ucos9-/sm9
e = ~n - l (vlV). . Icose
Note: In this situation; u(cos 2 S +sin 2 S) u
or V= =--
(i) Time taken [0 cross the river 12 '" LIV cosec= tt !cose > II)' cos S cosS .
(ii) Resultant displacement of th~ swimmer D2 = L < D1, I.e,
(B) Suppose in time ~t, P _.dlA;;-_-';C'--_-..:~_
is
when palh Is shortest Ii~e not least alld when time is least
moves a distance ~y verticall y
path is 110 / shortest.
downwards while 0 moves.
Problem 12. vertically up a distance ~y' so p Q

arrangement shown in Fig. that 0 reaches 0' with &y l


2.3 1, the ends Pond Q of an 0' A = I', then p'!
Ul/stretchable string move
downwards with IIniform ~y=/-/' ; dY = u(given)
dl M
speed u. If the puJleys A and B " " d' Fig. 2.32
are fued, what is the IIpward and L =v(say)
M dr
.speed of mass M? Fig. 2.31
Now in 1100' A from cosine law
[CPMT 1990[
Solution: From Fig. 2.31 in ~O CA I' + (L1y') ' _ I"
cos S = '--'-''''-L--'_
12 = /+ 0 2 21L1y'

or 21
dl
dt
y
= 2y d +0
dt or I' = { H L1{ cos e +( LIn']'"
dy dl 1 (dl/ dO
or - =-x- =
dl dl y (y/0 or /' =-{1- .1.;' cose ) (using binomial theorem)
38 PHYSICS FOR COMPETITIONS - Vol, I

or 1-1'=6.y'c089 :.Total time taken by second swimmer =! +.!!I.


or 6y = 6y' cos 9, v vII
[as string is unstretchable (I-I') = .6.y] Since both the swimmers reach point B simultaneously,
6.y .6.y' we have
or -=-cos9
M III _1_=.!..+.!!I _ 1_ = I+li.
which in the limit b.t --t 0 reduces to vcos9 v vii cose Ii
d d' We have u = 2kmh - 1,v=2.5kmh- 1•
~ = Lcose or u =vcos e,I.e.,
' V = -"-
dt dt
C 8
cose
v
cose=1 V 'V' -"' V'2,5'_2' = _'_
15
Problem 13. Two --,---i--i'--,,--- v v 2.5 2.5
swimmers start at the same --!!-
time from point A on one bank 2,5 = 1+l
vy = vcose
of a river to reach point B on 1.5 "
the other bank, lying directly
opposite to point A. One of ~vx=vsine i. e., velocity of walking of the second swimmer = 3 km h- I
towards B.
them crosses the river along ,A Problem 14. Consider a collection of a large number of
Fig. 2.33
the straight line AS, while the particles each with speed v. The '-
other swims at right angles to the stream and then walks the direction of velocity is randomly ~ ~/ ; ;
distance which he has been carried away by the stream to get
. .to point B. Both swimmers reach point B at the same time.
What was the velocity (assumed uniform) of his walking
velocity of both the swimmers in still water is 2.5 km h -1 and
if ~~~f:~;tEi:~:!{;!'~:l:~%r:: ;'~':..:,~;;
averaged over all the pairs in the ...........
the stream velocity is 2 Ion h - \ ? collection is greater than v.
Fig. 2.34
Solution: One swimmer moving along AD must swim at Solution: Fig. 2.34 shows the
such an angle that his velocity along the stream is nullified by particles each moving with same speed v but in different
stream velocity. Ifv is the velocity ofthe swimmer and u is the directions. Consider any two particles having angle 9 between
velocity of the stream, then u + vsin9 = 0. His velocity along the directions of their velocities.
the direction AB = vcos9. Time taken by the swimmer to -> -> ->
Then Vrel = VB - VA
reachB = ~e where lis the width of the river.
vcos
i. e., vTel =~v2 +v2 - 2vvcos9
Second swimmer swims in a direction perpendicular to the
stream. or VTel =~2v2(I -cos9) =2vsin(912)
Time taken to reach the opposite bank =1.. So averaging vrel over all pairs
v
_ J'.
0 vrelde J'"
0 2vsm(912)d9
During this time the swimmer drifts by the stream and he vrel = ftt d9 = ftt d9
reaches the point C. o 0
Distance CB = velocity x time = uf. 2v x 2[- cos(9/2)]~rt
v =
If the swimmer walks this distance with uniform velocity
or
Ii, time taken =...!:!.i .
vII
EXERCISE
[AJ Only One Choice is Correct 7. A body is moving unifonnly on a circle with speed v. Find
the magnitude of change in its velocity when it has turned
-+ -+ 1t -+
1. If the angle between a and b is -, then angle between 2 a an angle 9.
3
~
(a) 2vsin 9 (hi 2vsin (9/2)
and-3bis: (c) 2vcos(9/2) (d) ,j2vsin(9/2)
(a) •3 (b) 2•
3
8. A plane mirror is moving with velocity4i + 5} + sk. A point
object in front of the mirror moves with a velocity
(c) •6 (d) 5•
3
3i + 4} + sf. Here k is along the nonna! to the plane mirror
-+ -+ -+ -+ and facing towards the object. The velocity ofthe image is :
2. If a and b are two units vectors such that a + 2 band (a) - 3i-4}+Sk (b) 3i+4}+lik
~ ~

5 a -4 b are perpendicular to each other, then the angle (c) -4i+S}+lik


~ ~

between a and b is : 9. Angle between the vectors (i +})and () +k) is:


(a) 1t (a) 60 0 (b) 90 0 (c) 1800 (d) 00
4 ~ ~ ~

(e) cos- 1 (1) 10. A parallelogram is fonned with a and b as the sides let d,
~

and d 2 be the diagonals of the parallelogram then


-+ -+ -+-+
3. Vector a is perpendicular to b components of a - b along a 2 +b 2 =:
~ ~

a+hwillbe: (d~ +di) (d~ -di)


(a) 2 (b) 2
(b) a-b

(d) -Jr
a,'--+- b"2 (c) d~ +di (d) d; -di
11. Which of the following quantities is a vector?
-+ --)0 ---)0 -+ (a) Displacement (b) Density
4. If Ax B = C+ D then select the correct alternative:
(c) Volume (d) Temperature
~ ~ ~

(a) BisparalleltoC+D 12. Which of the following is not a vector?


~ ~
(a) Linear momentum (b) Electric field
(b) A is perpendicular to C
-+ -+ -+-+ (c) Kinetic energy (d) Acceleration
(e) Component of C along A = Component of D along A 13. IdentifY the scalar quantity:
-+ -+ -+-+
Cd) Component ofC along A =-Componentof D alongA (a) Force (b) Acceleration
~ ~ (c) Work (d) Impulse
5. If a 1 and 02 are two non collinear unit vectors and if
14. Which of the following quantities is a scalar?

I~ +~ 1=.J3, then the value Of( ~ -;:}(2:1 +;; )is: (a) Electric field
(b) Electrostatic potential

(a) 2 (b) ~ (c) Magnetic moment


2 (d) Accleration due to gravity
1 15. Angular momentum is: [eDSE 19931
(c) 2 (d)
(a) A scalar (b) A polar vector
6. A ray of light is incident on a plane mirror along a vector (c) An axial vector (d) A linear momentum
i +J-k. The normal on incidence point is along i + }. The 16. Angular displacement is :
unit vector along the reflected ray is :
(a) A scalar (b) A vector
1 A " -1A A A A

(a) .j3(i+ J +k) (b) .j3(i+j+k) (c) Either (d) Neither


17. Moment of inertia is :
1 A A - 1 A A A

(c) ,j2(i+j) (d) .j3(i+ j -k) (a) A scalar (b) A vector


(c) Either (d) Neither
18. For thc Fig. 2.35: 27. The resultant of two equal forces is double of either of the
~

(a) A+B=C

(e) C+A=B
~

.~
~

~
/:l A
Fig. 2,35
forces. The angle between them is:
(a) 120'
(e) 60'
(b) 90'
(d) O'
28. The resultant of two forces acting at an angle of 1500 is 10
kg wt, and is perpendicular to one of the forces. The smaller
~ ~ ~ force is :
(d) A+B+C~O
(a) l0J3kgwt (b) 20J3kgwt
19. A vector may change if:
(e) 20 kg wt (d) (20/./3) kg wt
(a) Frame ofrcfercnce is translated
~ ~ ~~~-'jo

(b) Frame of reference is rotated 29. Two vectors A and B are such that IA + B 1=1 A - B I. The
(e) Vector is translated parallel to itself ~ ~

angle between the vectors A and B is :


Cd) Vector is rotated
~ ~
IEAMCET 1993; MNR 1994; eBSE 19961
20. When two vectors A and B of magnitudes a and bare (a) 0 (b) ,/3
added, the magnittldc of the resultant vector is always: (e) ,/2 (d) ,
IEAMCET 1993] 30. For the resultant of two vectors to be maximum, what must
(a) Equal to (a+b) (b) Lessthan(a+b) be thc angle between them?
(e) Greaterthan(a+b) (d) Not greater than (a + b) (a) O' (b) 60'
21. The rectangular components of force of 5 dyne are: (e) 90' (d) 180'
(a) 1 and 2 dyne (b) 2 and 3 dyne 31. Minimum number of unequal coplanar forces whose vector
(e) 3 and 4 dyne Cd) 2.5 and 2.5 dyne sum can be equal to zero is :
22. To get a resultant displacement of 10 m, two displacement .(a) Two (b) Three
vectors, one of magnitude 6 m and another of 8 m, should (c) Four (d) Any
be combined: . ~ ~ ~

(a) Parallel
32. Two vectors A and B lie in a plane, a third vcctor e lies
(b) Anti-parallel ~ ~ ~

(e) At an angle 60" (d) Perpendicular to each outside this plane, then the sum ofthese vectors A -r B + C :
~ ~ ~ (a) Can be zero
23. If the magnitudes of vectors A, Band e are 12,5 and 13
(b) Can never be zero
~ ~ ~

units respectively and A + B = e, the angle between ~ ~

~ ~ (c) Lies in a plane containing A + B


vectors A and B is: ~ ~

(a) 0 (b) , (d) Lies in a plane containing A - B


(c) rr./2 (d) ,/4 33. Two forces of magnitude 7 newton and 5 newton act on a
~~~ -)~ ~ particle at an angle €I to each other; €I can have any valuc.
24. If A = B+ e and the magnitudes of A, Band e are 5, 4 and The minimum magnitude of the resultant force is:
~ ~

3 units respectively, the angle bctween A and e is : (a) 5 newton (b) 8 newton
ICBSE 1990] (c) 12 newton (d) 2 newton
(a) cos- 1 (3/5) (b) eos - (4/5) l 34. Two forces of 4 dyne and 3 dyne act upon a body. The
resultant force on the body can only be:
(e) ,/2 (d) sin -I (3/4)
(a) More than 3 dyne (b) More than 4 dyne
25. If two wavcs of same frequency and same amplitude
(c) Between 3 and 4 dyne (d) Between 1 and 7 dyne
respectively on superimposition produce a resultant wave
of the same amplitude, the waves differ in phase by: 35. A force of 6 kg and anothcr of 8 kg can be applied together
to produce the effect of a single force of:
ICBSE 1990]
(a) 1 kg (b) 11 kg
(a) zero (b) "'4
(e) "'3 ~~,
(d) 2"'3
~ ~
(e) 15 kg (d) 20 kg
36. Out of the following the resultant of which cannot be 4
26. The angle betwccn (Ax B)and (Bx A)is: newton?
(a) zero (b) , (e) "'4 (d) "'2 (a) 2Nand2N (b)2Nand4N
(c) 2Nand6N (d) 2 Nand 8 N
37. In case of three vector quantities of same type, whose 46. I walked 4 mile, turned to my left and walked 6 mile, then
resultant cannot be zero? turned to my right again and walked 4 mile. Which of the
(a) 10,10,10 (b) 10,10,20 choice mentions the distance from the starting point to the
place where I stopped?
(0) 10,20, 20 (d) 10,20,40
(Hint: Ifi na set a quantit y has value in belWeenR,)l.'U and R lI1in of
(a) 15 mile (b) 10 mile
remaining two, resultant can be zero.] (c) 20 mile (d) 14 mile

38. Five equal forces of to N each are applied at one point and 47. A person moves 30 m north, then 20 m cast, then 30../2 m
all are lying in one plane. If the angles between them arc south- west. His displacement from the originel position is:
equal, the resultant of these force s will be: (CBSE 1993] (a) 14 m south-west (b) 28 m south
(a) zero (b) 10 N (e) 10 m west (d) lSmeast
~-+-+ -+
(0) 20N (d) I0J2N
48. A force F:::6i -8 j +l 0k newton produces an
~ ~
2
39. If n is a unit vector in the direction of the vector A, then: acceleration of I mls in a body. The mass of body would
be: ICBSE 19961
(a) 200 kg (b) 20 kg
~ ~ ~ ~ ~ ~

(0) n =IAII A (d) n = n x n (c) l0J2kg (d) 6-J2 kg


~ ~ ~
40. An aeroplane is moving in a circular path with a speed 250 49. The angle that the vector A =2 i + 3 j makes with y-axis
kmlhr; what is the change in velocity in halfrevolution? ;s: ICPMT 19931
(a) 500 kmlhr (b) 250 kmlhr 1
(8) lan- I (3/2) (b) 18n - (213)
(c) 125 km/hr (d) Zero
(0). s;n- l (213) (d) OOS-I (312)
41. A truck travelling due north at 50 km/hr turns west and
-+-+-+-+ ~ -+-+-+
travels at the same speed. What is the change in velocity? SO. If A =5 i +7 j -3 k and B=2i + 2j - ck are
tCBSE 19901 perpendicular vectors, the value of c is: IEAMCET 1991)
(a) 50 kmlhr north-west (b) s0J2 kmlhr north-west (a) -2 (b) 8
(c) 50 km/hr south-west (d) 50J2 kmlhr south-west (c) - 7 (d) - 8
42. A boat which has a speed of5 kmlhr in still water crosses a [Hint: Scalar product of perpendicular vectors is zero, I.e.,
~ ~
river of width 1 km along the shortest possible path in 15
A' 8 = A.. B.. + AyBy + AlB z '" 0,1. e., 10 + 14+3c"'Oorc '" - 8)
minute. The velocity of the water in knv'hr is:
~ ~
(a) I (b) 3 5 1. A vector perpendicular to (4 i - 3 j ) is :
(c) 4 (d) ,fl4 ~

43. A river is flowing from west 10 east at a speed o f 5 (b) 7 k


mJminute. A man on the south bank of the river, capable.of ~ ~ ~

swimming at 10 mlminule in still .water, wants to swim (c) 6; (d) 3 I - 4 j


~ ~ ~
across the river in the shortest lime. He should swim in a
direction:
52. The angle between the two vectors -2 I +3 j + k and
~ ~ ~
(a) Due north (b) 30° east of north i +2 j-4kis: IEAMCET 1990; CBSE 19941
(c) 30"westofnorth (d) 60" east of north (a) 0" (b) 90'
44. I started walking down a road to day-break facing the sun. (c) 180' (d) None of these
After walking for some time, I turned to my left, then f ~ ---+ ~---+

turned to the rig ht once again. In which direction was I (Him: e'" cos- I (A' BlAB); Here A' B '" 0, so 9", 90"]
going then? -+ -+ -+ -+
53. The angle between the vectors (i + j land (j + k lis:
(a) East (b) North-west
(e) North-east (d) South (a) 90" (b) 180"
45. A man travels 1 mile due east, then 5 mile due south, then 2 (0) 0' (d) 60'
mile due east and finally 9 mile due north ; how far is he 54. A body constrained to move in y-direction is subjected to a
from the starting point? -+ -+ -+ -7
force given by F =(-2 I + 15 j + 6k )N. What is the work
(a) 3 mile (b) Smile
done by this force in moving the body through a distance of
(c) 4 mile (d) Between Sand 9 mile 10m along ~axis?
(a) 1901 (b) 160 1 62. The resultant of two forces, one double the other
(e) 1501 (d) 201 magnitude, is perpendicular to the smaller of the two force
-> The angle between the two forces is :
5S. The torque on a magnet of magnetic moment Min a field of (a) 1200 (b) 6QO
->
induction B is: [EAMCET 1995J (e) 90 0 (d) 1500
-> -> -> -> -> -> ............. .....................
(a) 't =Mx B (b) t =D x M 63. If vectors2 1 +2j - 2 k ,5 i +y j+kand - I +2j+2
-> -> -> -> are coplanar, the value ory is:
(e) 't=M'B (d) t=-M'D
(a) 81 (b) 36 (e) 23 (d) 9
->
56. A vector Fl is along the positivex~axis. If its vector product 64. A projectile of mass mis fired with velocity v at an angle
-> -> 45° with the horizontal from point A. Neglecting !
with another vector F2 is zero, t~en Fz could be: resistance, the magnitude of change in momentum betwe,
-> the starting point A and the striking point B is :
(a) 4 i
-> -> -> (a) mv (b) 2mv (e).fl.mv (d).fl.
(e) (j+k) (d) -(4 j) ..fi mv
..... -+-+ -+ 65. Magnitudes offour pairs ofdisplacement vectors are g ive
57. What is the torque of the force F = (2 i - 3 j +4 k ) N Which pair of displacement vectors, under vector additic
-+ -+ -+ -+ fails to g ive a resultant vector of magnitude 3 em?
acting at the point r = (3 i + 2 j + 3 k )mabout the origin? [J&K200
[CDSE 1995) (a) 2 cm, 7 cm (b) 1 cm,4cm
-> -> -> -> -> ->
(a) 6i-6j+12k (b) 17 i -6 j - Ilk (c) 2cm,3 cm (d) 2 cm, 4 cm
-> -> -> -> -> -> 66. Two forces of magnitudes 30, 60 and P newton acting a-
(e) -6 i +6 j-12k Cd) - 17i+6j+13k point are in equilibrium. If the angle between the first two
58. In a clockwise system: [CPMTl990) 60", the value of Pis:
-> -> -> -> -> (a) 2sJ2 (b) 30.13
(a) jxk = i (b) j.j =0
(e) 3016 (d) 3M
-> -> -> ->
(c) j x j :=:l (d) k'i=1 .... A A A

67. A force F "" a I + bJ + ck is acting upon a body of mass' ;


59. The position vector ora particle is If the body starts from rest and was at the origin initial
-> -> -> find its new co-ordinates after time t :
r =(a cosCJ)t) i +(asin rot) j
al 2 bt 2 ct 2 at 2 2bt 2 el 2
The velocity of the particle is: [CDSE 1995) (a) 2m' 2m • 2m (b) 2m .--;;;- . 2m
(a) Parallel to position vector
(b) PerpendicuJar 10 position vector
al 2 bt 2 ct 2
(c) --;;; . --;;;, 2m (d) None of these
(e) Directed towards the origin
->
(d) Directed away from the origin 68. Let F be the force acting on a particle having positi
-> -> -> -> ->
60. The angle made bythe vector 4 i -3 j + 5 k with z-axis is: vector rand T be the torque of this force about the orig
Then: IAIEEE ZO(
(a) 300 (b) 45 0
-+-+ ......... ........ .... ....
(e) 90° (d) 1200 (a) roT ""OandFoT:;:tO (b) roT;t: OandFoT""O
[Hint: cose z ::: R/R] -+-+ -+..... ....-+ ........
(e) roT :;:tOandF·T:;:tO (d) roT =OandF oT=O
-+ -+ -+ -+ ....
61. Three vectors A, B and C satisfy the relation A ° B '" Oand
-> -> -> -+ -+ -+-+ -+-+
A ° C = 0. The vector A is parallel to: 69. If IAx BI = .J3 A ° B then the value o f lAx 8 1is:
-> -> ICBSE PMT/PDT (Screening) 20!
(a) B (b) C
-> -> -> -> (a) (A2 +B2 +.fjAB)1I2 (b) (A 2 +B2 +AB)1I2
(e) B· C (d) B x C
(c) (A2+B2+~)"2 (d) A+B
VECTORS 43
-> -> -> E
77. In the regular hexagon shown in Fig.
70. A , Band C are vectors each having a unit magnitude. If -+ -+ -+ -+ -+
--jo - t -+ -jo -+ -jo -jo -+ -+ 2.36, AB+AC +AD+AE+AF can
A+ B+ C - 0, then A ·8+ B ·C+ C·A will be: be expressed as:
(a) (b) ~ (a) 4AO
->

->
(e) (d) 0 (h) 3AD B ".
2 -> Fig. 2.36
(e) 8AD
71. Three forces act on a body. The body will certainly have an
acceleration if these are: (d) zero
-> -> ->
(a) 7 N, 8 N, 14 N (b) 10N,4N, 12N 78. In the regular hexagon shown in Fig. 2.36, AB + BC +CD
(e) 3N, ISN,8N (d) 2 N, 6 N, 7 N -> -> ->
+ DE+. EF+ AF can be expressed as:
72. If the sum of two vectors and their difference are parallel, it
->
can be made out that the two vectors are: (a) -2FA (b) zero
(a) perpendicular to each other -> ->
(e) 2FA (d) FA
(b) parallel 10 each other -> -> ->
(c) of same magnitude 79. Tn the regular hexagon shown in Fig. 2.36, AO+ BO+CO
-> -> ->
(d) of unequal magnitude + DO+ EO + OF can be expressed as:
-> ->
73. Vectors A and B are mutually perpendicular. Component ->
(a) zero (b) -20e
-jo -jo -jo -jo
-> ->
of A+ B in the direction of A- B will be: (e) -20F (d) OF
-+ -+ -+ -+ -+-+
(b) h' -B' 80. For vectors A and B, (A+ B)-(Ax B) will be:

(d)
, ,
A -B
(s) A'B' (b) (A+B)(AB)

(c) zero
h ' +B' 81. Minimum number of coplanar vectors of equal magnitude
-> -> whose vector sum could be zero, is:
74. Vectors Aand B are mutually perpendicular. Component
-+-+ -Jo-jo
(a) 2 (b) 3
of A+ B in a direction perpendicular to that or A- B can be (e) 4 (d) 6
expressed as: 82. Minimum number of coplanar vectors of unequal magnitude
(a) 2AB (h) A-B whose vector sum could be zero, is:
A+B (a) 2 (b) 3
.JA'+B'
(e) 4 (d) 6
->
83. In Fig. 2.37, D is the midpoint of BC A
75. A particle is given successive displacements. Which of the
Which of the following relations is
following sets of displacements could be capable of correct?
returning the particle to its initial position: -> -> -> ~

(a) 10 m, 8 m, 6 m, 30m (s) X +Y = Z y


-> -> ->
(b) 20m, lOm,6m,50m (h) X-Y =Z B D C
(e) 70 m, 20m,40m, 30 m -> -> -> Fig. 2.37
(e) X-Y=Z/2
(d) 100 m, 18 m, 22 m, 32 m
-> -> ->
76. An expression which cannot be defined meaningfuUy (d) X+Y=2Z
among vectors is : 84. Sum of magnitude of two forces acting on a body is 15 N.
-> -> -> -> ->-> The resultant force has magnitude 12 N and it is
(a) A- ( Bx e) (b) A x (Bx e) perpendicular to the larger force, magnitude of smaller
-+ -+ -+-+ -+ -+ -+ -+ force is:
(e) (Ax B)x (ex D) (d) (A'D) x (e -D)
(s) 2.7 N (b) 3.2 N (e) 1.8 N (d) 4.6 N

-> -> 91. In the parallelogram shown in
85. Resultant of two forces Fl and Fl has magnitude 50 N. The -> ->
-> -> Fig. 2.39, A and B are vectors
resultant is inclined to Fl at 60 0 and to F:z a130°, Magnitudes represented by sides of the
-> -> -> ->
ofF! and Fl , respectively, are: parallelogram and DI and Dz are
(a) 25 N, 2sJ3 N (b) 20N,2ohN diagonal vectors, then D~ + D~ is : Ii
Fig. 2.39
(c) 20 N, 30N (d) JON,40N
-+-+ -+ -+-+-+-+-+
86. A, Band C and are vectors such that C = A + Band C.l A
-> ->
and also C = A. Angle between A and B is:

(a) 1t (b) ~ ~ ~ ~~ ~~A'


2 4 92. A and B are vectors expressed as A = 2 i + j and B = i - j
(c)
3" -> ->
(d) " Unit vector perpendicular to A and B is :
4

~
87. A person walks along the path shown in i-3+k i+ J-k
Fig. 2.38. The path from B to C is
(a) .,fj (b) .,fj
semicircular and centred at O. If the
magnitude of displacement of the (d) "
person is 2 m, distance travelled by him D 4m C
is nearly: Fig. 2.38 93. In the vector fonn, the sides of triangle can be expressed, a:
-> -> ->
(a) 18m (b) 7m A, Band C, then the area of triangle can be expressed as
(c) 14m (d) 11m -t-t -t -t-t-t ~ ~~

-+ -+ -+ -+ (a) A'(Bx C)+ B'(Cx A) + C'(Ax B)


88. ABeD is a parallelogram A, B, C and D are the position
3
vectors of vertices A,B,C and Dwith respect to any origin,
-> -> ->
then: (b) A·(Bx C)
-+ -+ -+ -+
-t ~ ~~~~
(a) A+B=C- D
AxB+ Bx C+CxA
-+ -+ -+ -+ (c)
(b) C - B=A+D 2
--t -+ -+ -t -> ->->
(c) B - C = A-D (d) Ax(BxC)
-+ -+ -+ -+
(d) A + B+C+D :=: O 94. Area of .'the parallelogram formed by vector
-tAAA -tAA
89. A lion is at some instant ata position A (2 m, 6 m,- 1m) and A =\ +2j+4k and B = 3i -2jis:
a goat is at position B (Im, 12 m, 8 m). The lion is free to (a) 4.Ji7unit (b) 2.Ji7unit
move but the goat is unable to move due to some injury. The
lion runs towards the goat and reaches it in a time 2 sec. (c) 17,J2' unit (d) 17-13 unit
Average velocity of the lion can be expressed as: 95. Flight of a bird is an example of:

(a) (1 ' 9 ' ) mls


- '2+3j+'2k (b) (Iz - 2j'+'2k
5 ') mls .
(a) addition of vectors
(b) dot product of vectors

( ")
(c) cross product of vectors
(c) ,i - ]
'2+2' mls (d) none of the above
-t A ~ "
-> -> 96. Torque of a for~e F =-3i + j+5k acting at a poin
90. Vector A is in the vertically upward direction and B
-> -> -t " A "
towards north. Bx A is toward: described by position vector r = 7i + 3j+ k is :-
(a) east (b) west (a) 121 - 14] + 3k (b) - 141+38J-16k
(c) vertically downward (d) south (c) - 121+14J-3k (d) 141 - 38]+16"
~~ -lo"" 106. A particle is moving on a circular path
97. Vectors AandB can be expressed as A""81+6J and
with a constant speed ·v'. Its change of
-+"" -+ -+
B - 1- J. Component of A in the dire<:tion of B is : velocity as it moves from A to B is :
3 • • 5 • • (a) 2vsin9
(a) -(1-]) (b) "2(1-])
4 (b) vsin9 /2
(c) vcos 9
(c) i -] (d) ~(i+])
2 (d) 2usin9/2
98. Projectionof2i+6jonZ-axisis: -+
107. Linear momentum of an object can be expressed as P >=
(a) 2 (b) 6 (400s t)i + (4sin t)j. Angle between the force aetingon the
(c) 4.5 Cd) none of these object and its linear momentum is:
-+ -+ -+-+ --»
99. R is the resultant of Aand B. R is inclined to A at angle 9, (a) ~ (b) ~ (c) 3. (d) •
2 4 ' 4
-+
and to B at angl~ 9 2, then: -+ -+ -+ -+ -+ -+
108. Aand B are two vectors. (A+ D)x(A-D) can be
(a) 9 1 d)2 (b) 9, <8 2 if A >8 expressed as :
(e) 9, <9 2 if A <8 (d) 91 <9 2 if A =8 -+ -+ -+ -+
100. i, band care unit vectors. If a+ b= C. then the magnitude (a) B x A (b) 2(BxA)
ora-bis: (c) 2(AxB)
-+ -+
(d) zero
I (b) _I .
(a) - (c) .fi (d) ,f3 109. Vertices of a triangle are A (3,. 1, 2), B (I, - 1, 2) and C
,f3 .fi (2, I , I). Area of the triangle will be:
101. Diagonals of a parallelogram are represented by vectors (a) .fi unit (b) 2.fi unit
-+~~~ -+~""
A '- S i -4 j +3k and B=3i+2j - k. Area of the (c) .J3 unit (d) 2.J3 unit
parallelogram is :
110. A vector remains unchanged on :
(a) Jl7i unit (b) m unit (a) rotating it through some angle
(e) 171unit Cd) 72 unit
(b) taking itS cross product with a unit vector
102. A particle is moving on a circular path of radius' R', As it
moves through an angular displacement et its linear (c) taking its dot product with a unit vector
displacement will be : (d) shifling it parallel to itself
(a) Rsin a (b) 2Rcos 9/2 ' -+ ~ ~ A

Il l. The velocity of a pa~jcle is v=6f+2j - 2 k . The


(9) 2Rsin 9/ 2 Cd) Rcos 9
component of the velocitY ora particle parallel 10 vector
103. If a.,pand 'Yare the angles which a vector makes with x,r -+ A ~ A

and Z-axes respectively, then sin 2 a.+sin 2 p+sin 2 'Y is a = i + j+k in vector form is:
equal to: (a) 6i +2]+2 k (b) 2i+2j+2k
(a) 3 (b) 2 (c) (d) 0
(c) i+]+k (d) 6i+2] - 2k
-+-+ -+
104. Magnitudes of vectors P ,Q and R are equal, If 112. A particle is displaced from position (21 - k) to another J+
-+ -+ -+ -+ -+
P + Q + R = 0, then angle between Rand Q is a. while if position (3 1 +2j-2k) under the action of the force of
-+ -+ -+ -+ -+ (2i + j - k). The work done by the force in an arbitrary unit
P + Q = R, the angle between Rand P is J3,. then :
is:' IKerala PET200S)
(a) a =2~ (b) a =~ (a) 8 (b) 10
(c) a. = ~ (d) none of these (c) 12 (d) 16
2
113. A particle moves in the X -Y plane under the influence ofa
-+ -+ •
105 . A vector A is rotated through angle a. about its tail. Change force such that its linear momentum is p(t) = A (I cos (kl)-
of position vector of its tip is :
)sin (kt»). where A and k are constants. The angle between
(a) Asin9 (b) 2A cos ~ the force and the momentum is: li lT 2007)
2
(a) 0°
(c) 2Asin~ Cd) AcosO (c) 45°
2
-+AA -+AA (a) 0):-0,+01" (b) ax-o,
114. What is the angle betweenA =51-5j and B =51-5j 7
Ox - 0,
(BHU200S) (0) .f2 (d) 0x+o),+az
(a) 90' (b) 45'
(0) 0' (d) 60' 120. Theangle subtended by vector A=4i+ 3j + 12k with the
..... A A ..

115. A particle starting from the origin (0, 0) moves in a straight x -axis is: [WB (JEE) 20081
line in the (x, y) plane. Its coordinates at a later time are
(.J3,3} The path of the particle makes with the x·axis an
angle of: [AIIMS 20071
(a) Sin -
1 3
3) C(b) Sin -I(~)
(a) 30' (b) 45'
(0) 60' (d) 0'
~ ~
..... .....
121. A andB aretwovectorsgivenbyA=21+3jandB = i+j.
..... . .. ..... ....
116. The sum of two vectors A and B is at right angle to their
~ ~

difference. Then: [RPMT 20071 The magnitude of the component of A along B is :


(a) A =B )WB (JEE) 2009)
(b)A = 2B
(a)
5
.f2 (b) ~
(0) B =2A .f2
~ ~

(d) A and B have the same direction 7 (d) _I


(c) .f2 .f2
~ . J' ~ ...
11 7. IfA =21+ 3 - Kand B " - 1+3J+4k, thenprojection of ..... .......... ..... ..........
~ ~
122. Given C - Ax B and D = Bx A. What is the angle between
A on B will be: )Pb. PMT2007) ~ ~

Cand D ? )WB (JEE) 2009)


3 (b) _3_
(a) -
.m 56 (a) 30' (b) 60'

(0) g (d)
'113
(3
(0) 90'
~
(d) 180·

123. The resultant of two vectors P and Q is R. If the magnitude


~ ~

118. A particle acted upon by constant forces 4 i + j - 3 k and ~

of Q is doubled, the new resultant becomes perpendicular


3 i + ) - k is displaced from the point i + 2] - k to point ~ ~

5 f + 4j - 3 k. Total work done by the forces in SI unit is : to P . Then the magnitude of R is : [Kerala PET 20091
(a) P+Q (b) Q
[Kerala (Engg.) 2008)
(a) 20 (b) 40 (0) 50 (d) 30 (0) P (d) P+Q
2
(e) 35
(e) P-Q
-+ A A A

119. The component of vector A - ax I + a yJ + a z k along the


direction off - j is: IEAMCET (Engg.) 20081

ANSWERS

I. (b) 2. (b) 3. (c) 4. (d) 5. (e) 6. (b) 7. (b) 8. (b) 9. (a) 10. (a) 11 . (a) 12. (e)
13. (e) 14. (b) 15. (e) 16. (e) 17. (d) 18. (e) 19. (d) 20. (d) 21. (e) 22. (d) 23. (e) 24. (a)
2S. (d) 26. (b) 27. (d) 28. (e) 29. (e) 30. (a) 31. (b) 32. (b) 33. (d) 34. (d) 35. (b) 36. (d)
37. (d) 38. (a) 39. (a) 40. (a) 41. (d) 42. (b) 43. (a) 44. (a) <S. (b) 46. (b) 47. (e) 48. (e)
49. (b) SO. (d) 51 . (b) 52. (b) 53. (d) 54. (e) 55. (a) 56. (a) 57. (b) 58. (a) 59. (b) 60. (b)
61. (d) 62. (d) 63. (e) 64. (e) 65. (a) 66. (d) 67. (a) 68. (d) 69. (b) 70. (b) 71. (e) 72. (b)
73. (d) 74. (a) 75. (e) 76. (d) 77. (b) 78. (a) 79. (b) 80. (0) 81. (a) 82. (b) 83. (d) 84. (a)
85. (a) 86. (c) 87. (d) 88. (e) 89. (a) 90. (a) 91. (d) 92. (d) 93. (e) 94. (a) 95. (a) 96. (d)
97. (e) 98. (d) 99. (b) 100. (d) 101. (a) 102. (e) 103. (b) 104. (a) 105. (e) Hl6. (d) 107. (a) 108. (b)
109. (e) 110 (d) III. (b) 112. (a) 113. (d) 114. (e) 115. (e) 11 6. (a) 117. (b) 118. (b) 119. (e) 120. (e)
121. (a) 122. !dJ 123. (bJ 124. (a)
(BJ More than One Choice is Correct ~ ~

(a) Momentum p of the particle is always perpendicular to F


1. Which of the following will not depend on orientation of ~ ~

frame of reference? (b) Momentum p of the particle is always parallel to F


(a) A scalar (c) Magnitude of momentum remains constant
(b) A vector (d) Change in momentum of the particle is always parallel
~
(c) The magnitude of a vector
to F
(d) Component of a vector
9. The magnitude of the vector product of two vectors
2. The momentum of a particle is given by ~ ~
~ ~ ~ Aand B may be:
p = (2sin t i - 2eas t j )kg mls. Select the correct options:
(a) Greater than AB (b) Equal to AB
~

(a) Momentum p of the particle is always perpendicular to (c) Less than AB (d) Equal to zero
~ 10. The following sets of three vectors act on a body, whose
F resultant can be zero. These are:
~ ~
(b) Momentum p of the particle is always parallel to F (a) 10,10,10 (b) 10, 10, 20
(c) Magnitude of momentum remains constant (c) 10,20,20 (d) 10, 20,40
..... ..... ..... ..... .....
(d) None of the above 11. A+B = C. Vectors AandB if rotated by angle e in the
3. Which of the following relations are wrong? ~ ~

~ ~ ~
same sense to fonn A' and B' then (9;t 0):
(a) A+B =C (b) A+B=C ~ ~ -> ~ ~ ~

~ ~ ~ ~ ~ (a) A'+B' =C (b) A'+B' ;t C


(c) A+B=C (d) A+B=C ..... ..... ..... ..... ~ ~ ~
~ ~ (c) A'·B' =AoB (d) IA'+B' I= ICI
4. If a vector A has magnitude A and n is a unit vector in the
~ 12. Which of the following expressions have no meaning?
direction of A, then which of the following are correct?
~ ~ ~ ~
(a) (I. j)x j (b) ~;.,.
I --,
(a) A =AI n (b) n =AI A (Ixk)xj
~ ~ ~ 1
(c) I = IAI I A (d) A = A n (c) -;,-?:--,
(Ixk)·] (d) ""(]'x-jokc-).' j
~ ~
5. For two vectors A and, B which of the following relations 13. 2 vectors of the same physical quantities are unequal if
are not commutative? (a) They have the same magnitudes and same direction
~ ~ ~ ~
(a) A+ B (b) A - B (b) They have different magnitudes but same directions
~ ~ ~ ~ (c) They have same magnitudes but different directions
(c) AxB (d) A'B (d) They have different magnitudes and different directions
~ ~ ~ ~ ~ ~
6. The angle that the vector A = 2 i + 3 j makes with y -axis is : 14. Two vectors P and Q lie in one plane. Vector R lies in a
~ ~ ~
(a) tan -1 3/2 (b) tan - 1 2/3 different plane. In such a case, P+ Q+ R ;
(c) cos-I .lG 13
(d) sin -I 2/3 (a) Can be zero
(b) Cannot be zero
~ ~
~ ~
7. If a vector A has magnitude A and n is a unit vector in the
~
(c) Lies in the same plane as P or Q
direction of A, then which of the following are correct? (d) Lies in the plane different from that of any two of 3
~ ~
~
A vectors
(a) A=A (b) n =-
~
A ANSWERS
n
~
1. (a), (b) and (c); 2. (a) and (c); 3. (a), (b) and (d); 4. (b), (c)
I=~
~ ~

(c) (d) A=A n


A i
and (d); 5. (bj 'nd (c); 6. (b) and (c); 7. (h). (c) and (d); 8.
.1 (a), (c) and (d); 9. (b), (c) and (d); 10. (a), (b) and (c); 11.
8. The momentum of particle ,~ given by
~
(b), (c) and (d); 12. (a), (b) and (d); 13. (b), (c) and (d); 14.
p = (2sin t i - 2cos t J)kg mls. Select the correct options:
(b) and (c).
~ ~ ~
Ic] Assertion-Reason Type Questions regarding the magnitudes of A , B & C as A,B,(
(a) If both A and R rue. true and R is the correct respecti~ely, Column-II gives the angle between tht
explanation of A. ~ ~

vectors A and B , Match them.


(b) If both A and R rue. true but R is not correct
explanation of A. Column - I Column - II
(c) If A is true but R is false. (a) A2 +S2 =C 2 (p) 9 >90 0
(d) If A is false but R is true. •
(e) Ifboth A and R are' false.
(b) A2 + S2 >C 2 (q) e < 90 0

1. (A): A vector will not change when the frame of '(c) A2 +S2 <C 2 (r) 9= 90 0
reference in which it is existing is rotated.
(d) A2 =B2 =C 2 (s) 9=00
(R): A scalar quantity may (or) may not be independent
of orientation of frame of reference. ~ ~

2. Column-I gives operation of vectors A and B and Column


ANSWERS -+ -+ -+ -+
II gives the angle (9) between A and B ( A and B are no
zero vectors),
1. (c)
Column - I Column - II
(D) Integer Type Questions (a) ~ ~ ~ ~
(p) 00
!Ax BI=IA'BI
1. A point moves according to the law x=at,y=at(l-at)
where a and Ct are positive constants and t is time. If the (b) ~ ~ ~ ~
(q) n
AxB=BxA
moment at which angle between velocity vector and 2
acceleration vector is ~is given by A. Find the value of A. (c) ~ ~ ~ ~
(r) -n
4 a IA+ BI=IA-BI
4
2. If ABC is a right angled triangle with hypotenuse AB =P.
--» --» --» --» --Jo -t (d) ~ ~ ~
(s) -3n
A+B=CandA+B=C
Then ABoAC+SC·SA+CA·CB = mp2. Find m 4
~ ~ ~

3. Two forces P and Q are acting at a point. If P is reversed, 3. Match the Column-I with Column-II.
the new resultant becomes perpendicular to the initial Column - I Column - II
resultant. If the condition between magnitudes ofP and Q is
given by P = kQ. Find k (a) Triangle law of vectors (p) Addition of two vectors
-, (b) Parallelogram law of (q) Subtraction of two
-+ d2 r -+
4. Given r =A cOSfJii + BsinfJij. Then if _ _ =-u n r
A A

vectors vectors
dt'
(c) Polygon law of vectors (r) Addition of more than
find the value of 11.
two vectors
5. If vector A is perpendicular to vector Band
-+ -+ -+-+ (d) Component method (s) Lami's theorem
IA+ BI= nl A- BI then find the value of n.
-+ -+ -+ -+ -+-+
6. If 0.81 + 0.2c] represents direction then the value of cwill 4. Consider three vectors A = 4 , B = 5 and C = 3, Thes
be: vectors are of similar nature, e,g" these could be thre
displacement. Apply your understanding of vector algebr
7. If the angle between vector 21 + 2] - k and vector 1 + ck is to match Column-I with Column-II.
acute then the maximum value of cis
Column - I Column -II
(a) Maximum magnitude (p) zero
1. 2. 1 3. 1 4. 2 5. 1 6. J 7. 2 ~ ~

of A- B will be
IE] Match the Columns
(b) Minimum magnitude of (q) 12
~ ~ ~ ~ ~

1. The two vectors A and B are drawn from a common point A+ 8- C will be
~ ~ ~

and C = A+ B. In Column-I are given the conditions


(c) Maximum magnitude (,) 9 (c) -lOj -lO knewton-metre
~ ~ ~
of A·( B- C) will be (d) -12j-12k newton-metre
2. Angle between the position vector ofQ and the force vector
(d) Maximum magnitude (s) l2
~ ~ ~
is
of A+ B- C will be
(b) cOS- 1 (2.)"2
204
ANSWERS
cos -.(3)
(c) sin -I 200
"2
1. a~ r; b-+ p; c-+ q,s; d-+ p ( 204 ) (d) -
5
2. a-+ r,s; b-+ p; c-+ q; d -+ p
3. Work done for the motion of the point mass from P to Qis
3. a -+ p,q,Sj b -+ p.q; c-+ p,q,f; d -+ p,q,r (a) 21 (b) 3.21
4. a-+ r; b-+ p; c-+ Sj d-+ q (c) 1.6 J (d) zero
[F) Linked Comprehension Type Paragraph - 2
Consider a point object of Y'
Paragraph - 1
mass'm' moving in a circle
:or,'
Suppose that a point y
of radius a = 1m For any
mass 'm' is moving
instantaneous position ofthe
under a constant force
object, 9 is the angle that the --- -- X
-+ A A A P(x, y, -1)
F=2i - j + k newton. • radial line joining the object
At some instant, t = 0. and the centre makes with
point P(x m, y m, - I the positive X -axis of a
m) [m - metre] is the
AO__- - - -x cartesian coordinate system
with the centre of circle 0 as Fig. 2.42
instantaneous position
of the mass. We know the origin. i and] are unit
that torque can be Z vectors along X -axis and Y-axis. respectively. Suppose that
expressed as the cross- Fig. 2.41 the motion is a 'Unifonn Circular Motion' with a constant
product of position vector and force vector, i.e. , angular speed ..!!. radisec and that the sense cf rotation is
~ ~ ~ 36
rxF
t = counterclockwise with 9 = 0 at t = 0. For an object which
At P, torque can be expressed as moves in a circle, it is usually convenient to introduce two
~ " " mutually perpendicular unit vectors r rand f t' as shown in
't" =-4 j-4knewton-metre
Fig. 2.42. Here rr is the radial unit vector and rt • the
At some other instant, t = 3 sec, the point mass has another tangential unit vector.
instantaneous positionQ (x, y, z )such that the displacement Answer the following questions.
vector between points P and Q and the given force are
4. For any instantaneous position of the object P, the radial
mutually perpendicular. Also, x-componellt of torque at Qis
unit vector rr can be expressed as
zero and yand z-components are equal in magnitude and
directed along the negative directions of the respective (a) rr=isin9+Jcos9 (b) r=icos9+jsin9
axes. Using a definite scale. if we construct a parallelogram (c) i'=lsin9-]cos9 (d) i r=-lcos9-jsin9
~

with the position vector of Q and the given force F as its 5. For any position of the object P, the tangential unit vector
adjacent sides. area of this parallelogram is 2../2 m2 . Area can be expressed as
of the given parallelogram, in fact, represents a physical (a) it =lcos9-]sin9 (b) rt =isin9-jcos9
quantity whose magnitude in SI system can be expressed as
5 times the given area. (c) it =~icos9+jsin 9 (d) it =-lsin 9+jcos 9
Answer the following questions. 6. In tenns of i r' i f and 9, i can be expressed as
I. At Q. torque acting on the mass can be expressed as
(a) l=ircos9-itsin9 (b) 1=irsin9+ f,cos9
(a) -5] - 51< newton-metre
(c) i - - rsin9 + rtcos9 (d) i- - rrcos9+i,sin9
(b) -8] - 8k newton-metre
~ ~ ~
Paragraph - 3 [Hint: C is the resultant of A and B, i.e, C 2 = A2 + B2 + 2AB
-+-+-+-+ -+
Five vectors A, B, C ,D and E have magnitude 10. 12,,[2, cose. Now substituting the given va lue ofC in it calculate e.]
20, 20 and to units resp,~~tively they are directed as shown [Ans. (a) p~rallel (9 = 0°), (b) perpendicular (9 = 90°)]
in the Fig. 2.43. " ' "" .' . 3. Can the resultant be zero in case of:
Y
(a) two unequal vectors
(b) three coplanar vectors
(c) three non-coplanar vectors
/
'{- [Ans. (a) No, (b) Yes, (c) No]
[Hint: See points 10, 11 and 12 in § f.2]
....D ....E 4. Under what condition:
Fig. 2.43 (a) resultant of two vectors will be zero
(b) sum of two vectors is equal to their difference
Answer the following questions.
-+ -+ -+ (c) the magnitude of sum of two vectors is equal to the
. 7. The X and Y components of(A+ B+ C)are respectively: magnitude of difference between them
[Ans. (a) if the vectors are equal and opposite, (b) ifone of the
(a) 22-IM and 22 (b) 22 and 22-IM
vectors is a null vector, (c) iflhe vectors are orthogonal]
, (0) 17-IMand22 (d) 22'hd:!7~IM 5. State whether the following statements are true or false
-+ -+ -+ giving reason in brief:
8. The X and Y components of(A+ C+ D) are respectively:
(a) as addition of vectors IS commutative so subtraction
(a) 1M and 1M (b) 10 - 1M and 1M must also be
(e) -IMandlO-IM (d) ~ IM';'dIM (b) component of a vector perpendicular to itself is zero
9. The X and Y components of sum of all the five vectors are (c) angle between two vectors can never be greater than
respe(:tiveiy : 1800
(a) -1M and 1M (d) a vector cannot be divided by a vector
[Ans. (a) False, (b) True, (c) True, (d) True]
(b) 12-IMand 12 - IM
6. A room has dimensions 3 m x 4 m x 5 m. A fly starting at
(e) 12 - 1M and 12 + 1M
one comer ends up at the diametrically opposite comer.
(d) 12 + 1M aJjd 1:1-IM (a) What is the magnitude of its displacement?

ANSWERS (b) Could the length of its path be less than this distance?
(c) Choosing a suitable co-ordinate system find the
Pa'ragraph-l 1. (c) ," t. (e) , C (d) position vector.
Paragraph-2 4, (b) 5, (d) 6. (a) (d) If the fly does not fly but walks, what is the length of
the shortest path it can take?
Paragraph-3 7. (a) 8. (e) 9, (b) ~ ~ ..." ~

[Ans. (a) Eo:::7 m, (b) Never, (c) r = 3 1+4 j + 5 k, (d)


[G) S'ubjective Type Questions '.,- . 3+.J4i = 9.4m]

1. Give an example of a physical quantity which: 7. A 5 kg object with speed of 30


(a) has neither unit nor direction mls strikes a steel plate at an
(b) has direction but not a vector angle of 45 0 and rebounds at the
same speed and same angle (Fig.
(e) can be either a scalar or a vector
2.44). Calculate Fig. 2.44
Cd) is neither a scalar nor a vector
[Ans. (a) refractive index, (b) pressure, (c) angular displacement, (a) the magnitude of the change in momentum of the
!
(~) moment of inertia]
object and
~ -+
2. What are the properties of two vectors A and B such that: (b) the change in the magnitude of the momentum of the
-+ -+ ~ object.
(a) A+B=CandA+B =C [Ans. (a) 300J2 kg mis, (b) zero]
-+ ~ ~
(b) A+B=CandA2+B2 =C 2
8. A body is moving unifonnly on a circle with speed v. Find of the angular momentum of the particle about the point 0 at
the magnitude of change in its velocity when it has turned position:
an angle e. y

[Ans.2vsin(9/2)]
9. If a vector is added or subtracted from a vector, the resultant
is always a vector. Is this also true in case ofmuitiplication
of two vectors?
[Ans. Mayor may not be] o'~~~------~,
[Hint: See note 2 in § 2.4.] Fig. 2.45
~ ~ ~
10. Give a few examples of physical quantities which can be
r "'(0.7v~ Ig) i +(02v~ I g) j
expressed as (a) scalar product and (b) vector product of
two vectors. when the velocity of the particle is
~ ~ ~
[Hint: See § 2.5 (c) and § 2.6 (e).)
v = (O.7v o ) l-(O.3vo) j
11. What is the condition that two non-zero vectors are (a)
orthogonal and (b) collinear? [Ans. Magnitude = 0.35 mv~ 1g and direction along negative
-t ---+ -t --t z.axis]
[Ans. (a) A' B =0 and (b) Ax B =0]
17. A river flows at 3 mls and is 300 m wide. A man swims
12. Can you think of a unit vector which is perpendicular to across the river with a velocity of 2 mls directed always
~ ~
perpendicular to the flow of current. (a) How long does it
each of two vectors A and B having an angle e between take the man to cross the river? (b) In what direction does he
them.
~ ~
actually move relative to the shore? (e) How far down the
(Ans. Ax HI AB sin 9J river (from the starting point) does he reach the opposite
-). -). -). -). bank? [MNR 1991)
13. Two constant forces Fl =(2 i +3 j+3k) newton and
[Ans. (a) 150 s, (b) tan-' (2/3), (e) 15M m]
-+ -+ -+ -).
F2 =(51 - 6 j-2k) newton act together on a particle 18. A river is flowing from west to east at a speed of 5 mls. A
~ ~ man on the south bank of the river capable of swimming at
during its displacement from the position (20 i + 15 j )m to 10 mls in still water, wants to swim across the river (a) in
~
the shortest time and (b) along the shortest path. In which
8 k m . Calculate the work done.
direction should he swim?
[Ans. -87 joule]
[Ans. (a) due north (i.e, .L to flow) (b) 300 west of north or at an
-). -+ -). -+
14. Find the moment of force F = i + j + k acting at point (-2, angle 300 + 90 0 = 1200 from the direction of flow.]
3,4) about the point (I, 2, 3). 19. The width ofariver is 25 m and in it water is flowing with a
-+ -+ -+ -+ -+ velocity of 4 mlmin. A boatman is standing on the bank of
[Hint: Moment of force = r x F with r '" r2 - r1. the river. He wants to sail the boat to a point at the other
-+ -+ -+ -+ -+ -+ -+ -+ bank which is directly opposite to him. In what time will he
Here r2 = (- 2 i + 3 j+4 k)and r, =(1 + 2 j+ 3k).]
cross the river, ifhe can sail the boat at 8 m/min. relative to
~ ~

(Ans.4(j - k)] the water? IRoorkee 1995]


[Ans. 3.6 minute]
15. Compute the force (in vector notation) on an electron
~ ~ 20. Fill in the blanks:
moving with velocity v = 2.5x 106 i mls in a magnetic (a) The resultant of two forces acting at a point each equal
~ ~ ~ .
field B=(IOi-6k)xI0 2
Wb/m 2, if charge on an to P is also P. The angle between the two forces is . ..
~ ~
electron e = 16x 10- 19 coulomb. (b) A boat is moving with a velocity 3 i +4 j w.r.t.
~ ~

[Ans. F = 2.4 x 1010 j newton] ground. The water in the river is moving with a
-+ ~
~ ~ ~ velocity - 3 i - 4 j w.r.t.ground. The relative velocity
[Hint: F =q(v x B)]
of the boatw.r.t. water is... IEAMCET 1991)
16. The particle of mass mis projected at t = Ofrom a pointOon ~ ~ ~

the ground with speed Vo at an angle 45 0 to the horizontal as (Hint: vr~l =vb-vr ]
shown in Fig. 2.45. Compute the magnitude and direction
[Hint: As magnitude ofa _ ~3Z + 4 2 = 5so a unic vector parallel
~ ~ ~

(c) The angle made by the vector A =2 1+2 J with the ... -+ -I' -t -+
x·axis is ... to • will be n ::: al 5::: (3 I + 4 j ) / 5. Now as magnitude ofb is
[lIlat: 0,.. tan-I (A y I A. .) '" tan-I (212) ::: 45"] J72+ 242 _ 25, so a vector parallel to -: and having magnitude of
..... ........ -+ ...
-+-+ ....... -+ ... bwill be - 25 n =25(31 +4 J) /5 =15 t +20 j]
(d) If a - 3 I +4 j and b =7 1 +24 j, the vector having
~ ~
[Ant. (a) 120·, (b) 6 i
...+ 8...J. (c) 45°, (d) 151...+ 20 ...Jl
the same magnitude as b, and parallel to a is . . .

.'.
Force and Torque
§ 3.1 Force
.... dp
....
Force is a push or pull which tries to change* or changes F=- .... (a)
dt
the state a/rest oro/uniform motion ofa body, i.e., force is the
cause of translatory motion. It arises from the interaction of
i. e.,
....
F=-(mv)
d .... ........
[asp=mv]
bodies either due to contact (e.g., reaction or friction) or from a dt
distance (e.g., gravItational or electric force between two ....
charged particles). d v --+-dm
--+-
i.e., F=m-+v- .... (i)
dt dt
Body at Rest
Now for a body as m = constant, (dmldt) =0
.... ....
so F =m(d vMt)
If body remains al rest, force is If body moves, force Is changing .... .... .... = dvldtl
....
trying to change the slale of rest. the state of rest.
or F =ma ** [as a .... (b)
In physics we also have situations (such as rocket motion)
Body In Motion
in which mass of the body changes but velocity of escape of
(a) When force Is parallel or anliparallel to motion, it changes the
magnitude and not the direction of motion. mass is constant. In such situations as -: = constant (d-;/dt) =0,
,
so Eqn. (i) reduces to
--) --+-dm
F=v- .... (e)
Magnitude increases Magnitude decreases dt
(b) When force Is always (e) When force acts al an
Further, in case of conservative fields, force is defined as,
perpendicular to motion. angle to the mollon.
....
F = :-gradU
If potential energy U is assumed to be a function of
position r only, the above result can be written as
dU
F =--=-(slopeofUlrcurve) .... (d)
dr
So, depending on the situation any of the fonnulae (a), (b),
Direction of motion only Both magnltude and direction (c) or (d) can be used to compute force.
changes and not the of motion change and motion
magnitude and motion is non-uniform circular, elliptic, Regarding force it is worthy to note that:
is uniform circular. parabolic or hyperbolic. 1. It is ~ polar vector as it has a point of application.
Fig. 3.1 Action of force on 8 body 2. It has dimensions,
When force changes the state ofrest or motion of a body,
by Newton's II law, [Fl=[:]
,
'" When force is trying to change [he state of rest ofa body and cannot, then it may produce deformation in the body.
"'''' This formula is valid only ifforce is changing the state of rest or of motion and mass of the body is constant and finite.
[FJ=[maJ [FJ =[MLr2J in an atom), we neglect gravitational force in
comparison to electric force, i.e.,
[FJ = [<7] [FJ=[LT-
1
~]
F =FE +FG =FE

[FJ =[ ~~]
7. All the forces according to their dependence on
[FJ=[ MLt2 ] position or motion can be divided into the following
four types:
or [FJ =[MLr2J (a) Constant force: If the direction and magnitude
3. It has units: i of a force is constant, it is said to be a constant
MKS System CGSSystem FPS System force. Mechanical forces of constant magnitude
2 are produced by machines. Tension in a string can
kg mls g cmfs2 1b~ftls2
be a constant force.
newton dyne poundal
(b) Time dependent force: If a force depends on
with 1 newton = 10 5 dyne time, it is said to be time dependent, e.g., in case
Improper unit offorce is kg-wt or g-wt, of impulse or motion of a charged particle in an
i.e., lkg-wt = g newton = 9.8 N alternating electric field forc~ is time dependent.
1 g-wt = 1 g dyne = 980 dyne (e) Position dependent force: If a force acting on a
4. When force is written
without direction
then positive force
I body depends on the position of body, it is said to
be position dependent force. Gravitational force
between two bodies (= Gm lm2/?) or force
means repulsive +F Repulsive between two charged particles
while negative force I [=QIQ2/(41t'Bo?)]are examples of position
dependent forces.
means attractive,

~~~:~~ two ~~~!~~~


(both positive or
Jj ~~~~~~~;;~~~~;;;~i~~~i~i~~
==========================
r
(d) Velocity dependent force: Ifa force acting on a
body depends on the velocity of the body, it is
said to be velocity dependent. In whole physics,
negative) electric :::::::: Attractive ::::::::: in the opinion of author, only two forces,
force is posi.tive Fig. 3.2 viz. ,viscous force (= 6ml'v) and force on a
meaning repulsive charged particle in a magnetic field (qvBsin9)
while between dissimilar charged particles is negative are velocity dependent.
meaning attractive (See Fig. 3.2) 8. Aposition dependent/orce is always directed towards
5. Force can be applied by muscle (man or horse); or away from a fixed point and said to be central
machine (crane or tractor) or nature (gravitational or otherwise non-central. The motion of earth around the
friction). When force is applied by muscle or sun, motion of an electron in an atom or scattering of
machine, there is an upper limit to it. This limit is a-particles from a nucleus are few examples of central
different for different muscles or machines. forces. (See Fig. 3.3.)

6. Out of so many natural forces, for distance =1O-15 m, ~-


a-particle
~\
'
nuclear force is strongest while gravitational force Nuceu
I I

weakest, i.e.,
+ \,~~~~
Fnudear > Fclcclromagnelie > Fgravilational ~- .
For two electrons at any separation r,
lal Ibl Ie)
FE = (e 2/4m:or2) = ·9xI09x(1.6xIO- 19)2
Fig. 3.3
FG (Gm 2 /r2) 6.67xlO 11 x (9.lxlO 31)2
In case of central force as force passes through a fixed
=4.17xIQ42 = 10 43 point, torque about the fixed point is always zero and
so angular momentum is conserved, i.e., in case of
i.e.,FE>FG
central force,
This is why, in the situation where both electric and
Torque = 0 and Angular momentum = constant
gravitational forces are collinear (e.g., on an electron
9. Force can also be classified as conservative or (e) Thrust: It always acts opposite to the weight and
non-conservative. Ifunder the action ofa force, work is equal to the weight of fluid displaced by the
done in a round trip is zero or work is path body. (It is also known as buoyancy force). So, if
independent, the force is said to be conservative V' volume ofa body is inside a fluid of density a ,
otherwise not. Gravitational, electric and elastic
forces are conservative while frictional and viscous Thrust = V'ag
force s arc non-conservative. In presence of non- From this, it is clear that thrust is independent of
conservative force there is loss of mechanical energy all faclors of the body such as its shape, density,
which is usually converted into heat. etc., except its volume inside the medium.
10. Force can also be classified as internal and external. (d) Electric force: If a charged particle having
Internal forces are those which arise from the ~

interactions with other particles that are parts of the charge q is in a uniform electric fi eld E, the
system while external forces are those which originate electric force on it is given by:
beyond the system under consideration. Same force ~ ~

can be external or internal depending on the system. If F =q E


we consider a body as the system the force of gravity while if there are two charges separated by a
of earth is external while, if we consider body and distance r in free space, the el~tric force between
earth as the system the forc e becomes internal. Total them is given by:
internal force acting on a system is always zero as
these are parts of action-reaction pairs and cancel. F= _ 1_ qlq2 -; [Coulomb's law]
11. Force can also be classified according to its origin
41teo r3
such as reaction, thrust, tension, elastic, frictional, 12. If there act number of forces on a body we find thE
gravitational, electric. magnetic and nuclear. Though resultant force using law of parallelogram ofaddition, i.e.,
these forces are dealt in detail at appropriate places in ~ ~ ~

the book, here to solve problems related to force we F = FI + Fl +.... [as force is a vector)
describe some of these forces in brief. and then calculate acceleration of the body usin~
(a) Weight W: The weight of a body is the force by
which it is pulled by the: gravity of a nearby
astronomical body (usually earth). If a body of
mass 'm' is located at a point where acceleration
Newton's II law, i.e.,
~
0 =
.
(Flm)

due to gravity is g, the weight If acceleration ""it is constant, equations of motion


viz.,
~ ~
W = mg or W :::; mg v=u+at; s= ut + '!'at 2 andv 2 '= u 2 +2as
2 • .
(b) Reaction R: When a body is pressed against a
rigid surface, the body experiences a fo rce which can be applied to deal the motion. However, i'
is perpendicular to the surfaces in contact. This acceleration IS not co~tant, then integration o'
force is called 'normal force' or ' reaction'. Free acceleration will give velocity v and integration o'
body diagram* for fo rce of reaction In some velocity will give displacements, i.e., we use the"mos
situations is shown in Fig. 3.4. fundamental definitions
R R ~ ~
R F ~
,F sin e ~
a= -
dv ds
V= -
dl ' dr
" ~, __~,,,0

~-}~rtJ ~g COS e....""....F.-.C'O. '.; Problem 1. A block ofmetal weighing 2 kg;s resting on I
frictionless plane. It is struck by ajet releasing water at a ral.
O_
LJ" --",m,,---,
of I kg/s and at a speed of 5 mls. Calculate the initia
m, m, acceleration of the block.
(0) Ib) Ie) Solution: The water jet striking the block at
the rate 0 :
Fig. 3.4 I kgls at a speed of 5 mls will exert a force on the block,
Here, it is worthy to note that if the pressing force F= v dm =5x l=5 N
vanishes (or the body loses contact with the dl
surface), reaction will not exist.
• A diagram showing all externa l forces acting o n an object is ca!!ed 'free body diagram'.
§ 3.2 Translatory Equilibrium
When several forces act on a body simultaneously in such
a way that the resultant force on the body is zero, i.e.,
~

F =0 w:th
Flg.a.s
the body is said to be in translatory equilibrium.
Undcrthe action of this force ofS N, the block of mass Here, it is worthy to note that
2 kg will move with an acceleration given by 1. As if a vector is zero all its components must vanish,
F=ma, i.e., o""Flm=512=2.Smls2 Ans. i.e., in equilibrium as--
Problem 2. Calculate the volume 0/ the balloon filled ~ ~
F = ~F,=O
with hydrogen gas, which will be sufficient to lift a load of
2S kg in air. Given thaI the densities of air and hydrogen are IF;c =0'' Y
LF = 0 and LF~"-
=0
0.00129 glee and 0.00009 glee respectively. so in equilibrium forces along x-axis must balance
Solution: The weight of hydrogen in the balloon and load each other and same is true for other directions.
25 kg will act vertically downwards while thrust of air -) -+ -+-+
vertically up. So the balloon will lift the weight if, 2. As for a body F = 0 means rna =0 [as F = rna ]
Th 2: weight of hydrogen + load
or (d ~ldt) =0 (as m is finite and fi = d-:ldt]
or VPair g=VPHg+ Mg
~
or V(Pair-PH) = M or v = constant or zero
3
or V= 25x I0 g :::::25 x 10 7 i.e., if a body is in translatory equilibrium il will be
(0.00 129 - 0.00(09) 12 either al rest or in uniform mOlion. If it is at rest, the
eqUilibrium is called static, otherwise dynamic.
or V = 2.083x10 7cc
3. If the forces are conservative, then as for conservative
=20.83 m ' Ans. force (F'" - dUldr) and for equilibrium (F= 0)
Problem 3. Two blocks of so F=-(dU/dr)=O
massm= 1 kgandM=2kgare i.e., dUldr=O
in contact on a frictionless lable.
i.e., in conservative fields at eqUilibrium potential
A horizontal force F{=3 N) is
applied to m. Find the force of _-'-_ energy is optimum, i.e. , in equilibrium potential
energy is maximum or minimum or constant.
cOlltact between the blocks. Will
the force ofcontact remain sCl.me 4. Static equilibrium can be divided into the following
ifF is applied to M ? three types:
Solution: All the forces t.1g (a) Stable Equilibrium: If on slight displacement
acting on the two blocks are Flg.3.G from equilibrium position a body has tendency to
shown in Fig. 3.6. As the blocks are rigid under the action ofa regain its original position, it is said to be in
force F, both will move together with same acceleration, stable equilibrium. In case of stable equilibrium
potential energy is minimum (d 2U/dr2 = +ve)
a= - F - 3
=--=lmls 2
m+M 1+2 and so centre of gravity is lowest. Fig. 3.7 depicts
Now as the mass of larger block is M and its acceleration some examples of stable eqUilibrium.
a, so force of contact, i.e., action on it, M
-2..
AM=Ma= MF =2x3=2N @------@=-@
M + m 2 +1
If the force is applied to M, its action on m will be
Am =ma=~= l x3 = I N Ans.
M +m 2 +1
(.) (b) (0)
Note: From this problem it is clear that acceleration does not depend on
Fig. 3.7
the fact that whether the force is applied to 11/ or M but force of
contact does.
Neutral
(b) Unstable EqulUbrlum: If on slight
displacement from equilibrium position a body t t E, Unstable
moves in the direction of displacement, the .F .U E,
equilibrium is said to be unstable. In this situation
potential energy of the body is maximum
OhE",,-+':-r'--1.,-' 0 ~-\-,--I-IS~t!i.~2~~.
-F -U
(d 2U /dr2 = - ve) and so centre of gravity is j j E,
highest. Fig. 3.8 depicts some examples of Stable
Wlstable equilibrium, (.) (b)

~""
Fig. 3.11
~
M
U =max., i. e., (d 2Uldr2) = negative, equilibrium is
~
$"----~
unstable and
,, V = constant, i. e., (d 2V ldr2) = zero, equilibrium is
,
(a) (b) (e) neutral
Flg. 3.8 This all is shown in Fig. 3. 11 .
(e) Neutral Equilibrium: If on slight displacement Problem 4. The potential energy for a conser;vative
from equilibrium position a body has no tendency system is given by:
to come back to its original position or to move in V=ax 2 -bx
the direction of displacement, it is said to be in where a and b are positive conslants. Work out (a) the law of
neu/ral equilibrium. In this situation, potential force (b) equilibrium position and (c) potential energy at
energy of the body is constant (d 1U/dr2 =0) and equilibrium position. Discuss also whether the equilibrium is
so centre of gravity remains at constant height. stable or unstable.
Fig. 3.9 depicts some examples of neutral Solution: (a) In a conservative field as F = - (dUldr)
equilibrium.
F =-!!... (ax 2 - bx)=b-2ax .... (.)
____ uu ___ _ dx
----
" '" B-"y F""
---- (b) For equilibrium, F = 0
i.e., b-2ax=0 or x=(b/2a) =x, (say) .... (b)
(a) (b) (e)
(c) Substituting x = bl2a in the given expression for
Fig. 3.9
potential. energy,
5. In case of stable equilibrium lesser the potential
energy or lower the centre of gravity, i.e., greater the (U)x=,<> = a [!>...]2
2a
_b[!>"'
20
] = _lC
4a
....(c)
base area more stable is the equilibrium. (Fig. 3.10)
Further as from Eqn. (a),
,,
,
:,
,,
,,, . (dU/dx) =2ax - b
2

~---.-----~
d -U = 2 a -t PoSIUve
"
:, • i,
so -
dx 2
) ._---------- ,J--------
i.e., Vis minimum and so equilibrium is stable. Ans.
(a) (b) (e)
§ 3.3 Strings
Fig. 3.10
While dealing with force problems where strings are
6. If we plot graphs F vs r and U vs r, at equilibrium F involved, always remember that:
will be zero while U will be max. or min. or constant. I. String is assumed to be inextensible (perfectly elastic)
If: unless stated. This is why the magnitude of
U = min., i.e.,(d 1Uldr 2 ) = positive, equilibrium is accelerations of any number of masses connected
stable through string is always same. (Fig. 3.12)
the middle may break if the length of the string
shortened.
(b) As T= (W/2 cosS) so tension in the string willi
greater, equal or lesser than load W, as cosS
lesser, equal or greater than (112). i.e., as e
greater, equal or lesser than 60° or (n/3) radia
So, the tension in the string can be lesser, equ
or greater than load depending on e.
(.) (b)
Fig. 3.12
(c) Tension in the string will be minimum when,
cos9 = max=1
If the string is extensible, the acceleration of different i,e., e = 0°
masses connected through it will be different until the
and in this situation [Fig. 3.14 (b)]
string can stretch.
2. String is assumed to be massless unless stated. This is T"", = (W/2)
why, the tension in it everywhere remains the same
and equal to applied force. However, if a string has a
mass, tension at different points will be different
being maximum (= applied force) at the end through
which force is applied and minimum at the other end
connected to a body. [Fig. 3.13 (a)]
T sin e~.-~~~~-~
~;I?i' m ;Ov. F ~ ,,:;,J,,,, ~~
i~ ~ w W
(.) (b)
(.) (b)
Fig. 3.13 o p
3. In order to produce tension in a string two equal and -T--T-
opposite stretching forces must be applied. The
tension thus produced is equal in magnitude to either e =900
applied force (i.e., T = F ) and is directed inwards Tis
opposite to F[Fig. 3.13(b)]. Here it must be noted that Horizontal
a string can never be compressed like a spring. Wis
4. Every string can bear a maximum tension, i.e., if the Vertical
tension in a string is continuously increased it will w
(e)
break if the tension is increased beyond a certain limit.
Fig. 3.14
The maximum tension which a string can bear without
breaking is called breaking strength. It is finite for a
string and depends on its material and dimensions.
This is why tension in a string loaded at I
5. When a string is fixed horizontally (by clamping its middle can never be lesser than half I
free ends) and loaded at the middle [as shown in Fig. suspended load.
3.14 (a)], for equilibrium of point P*,
(d) Tension in the string will be maximum wh
2TcosS = W,i.e., T = W/(2 cosS) cosS = min =O,i.e.,S =90° and then Truax =
From this, it is clear that: However, as every string can bear a maximl
(a) As length of string is shortened, S will increase finite tension (lesser than breaking strength) tl
situation cannot be realised practical
and so cosS will decrease and hence tension in the
string will increase. This is why a string loaded at
e
Furthermore as = 90° means that the string
horizontal [Fig. 3.14(c)], we conclude that

'" If the Siring is not loaded at the middle for horizontal equilibrium of P, 1j sin at = T2 sin a2, while for vertical equilibrium of P,11 cos at + T2 cos 02 =
For middle point by geometry of problem,
at = a2and so, 1j = T2 = T
string can never remain horizontal when loaded move in a direction opposite to acceleration, We
at the middle however great the tension be or usually represent this fact by stating that mass m
tension required to completely straighten a rope experiences a psudeo-force ma opposite to
horizontally (without a sag) is infinite. acceleration.
So as a result of this, the string will incline to the
Note: a'" 90 0
means that the string at P becomes horizontal, so at P the
only force is weight (as T balance each other) which is acting vertical in a direction opposite to the acceleration
vertically downwards and remai ns unbalanced. This is also why of the carriage. Now as the mass m is in
this equilibrium cannot be achieved physically. equilibrium inside the carriage,
T=sin8=ma
6. When a mass is suspended from a vertical string in a
carriage, following situations are possible: and Tcose = mg
(a) If the carriage (say lift) is in translatory which on solving for 8 and Tyields
equilibrium (i.e., at rest or moving unifonnly), tan 8 "" (a/g) and T ~ m~g2 + a 2 (>To)
the forces acting on the system are as shown in
[Fig. 3.15(a)]. For translatory equilibrium at i.e., the tension in the string becomes more than
point 0, R = To and for equilibrium of mass m at To and so the string may break, Also the string
point P, To = mg. So in this situation does not remain vertical but inclines to the
R ~ To ~ mg .... (a) vertical at an angle 8 "" tan-! (a/g) opposite to the
acceleration (which mayor may not be the
0 R 0 0
direction of motion of the system).
T,

p
T,
m, [[]Jra OJla 0'

lal Ibl
Fig. 3.15
101
ma
P' m,
a

(b) If the carriage is accelerated up, the mass m is not


in equilibrium but it has an upward acceleration
a. So, the upward force T must be greater than the o
lal
downward force mg such that o
T-mg =ma
or T ~ m(g + a)
i.e., tension in the string will be more than To and rna P TsinO
the string may break if acceleration is such that mg
T> Breaking strength.
(c) If the carriage is accelerated down, the mass m is
not in equilibrium but it has a downward
Ibl Fig. 3.16
101

acceleration a. So, downward force mg must be


Note: (i) This arrangement is called 'accelerometer' and can be used
greater than upward tension T such that
to detennine the acceleration of a moving carriage from
mg - T =ma inside by noting the deviation ofa plumb line suspended in it
or T~m(g- a ) [< Tol from the vertical and using the relation a = g tan a
(ii) If the carriage is moving with unifonn speed on a circular
i.e., tension in the string will be lesser than To. It track [(Fig. 3.16 (c)l then as centripetal acceleration is
is also interesting to note here that if the carriage directed radially inwards and is orthogonal to g, the problem
begins to fall freely, i.e., a = g, tension in the becomes same as case (d) with a=: (v 21r) =: mi. So, the
string becomes zero! string will incline to the vertical away from the centre by an
(d) If the carriage is accelerated horizontally, the angle a such that
motion of point 0 will be accelerated translatory a =: tan - ] [a/ g] =: tan -1[v2/rg 1
while due to inertia mass at P will try to maintain ~~-.

its position. So relative to P, 0 will move in the


and tension in the string wi!! be: T =: m~[g2 + (v 2/rll
direction of acceleration or relative to 0, P will
7. Tension in a string passing over a fixed pulley'" and TB ~3(g-a) ~ 3(g-0.3g) ~ (2.1g) N
loaded at the two ends such as Atwood's machine and if we consider the motion of A,
[Fig. 3. 17(a)] can be dealt easily, quickly and
T. ~mx a ~7 x O.3g ~ (2.1g)N
correctly using properties 1, 2 and 6 of strings
discussed above. To understand this all, consider the i.e., TA ~TB ~T~(2.1g)N=2.1kg

system shown in Fig. 3.17 (a). The force (7g) N will Problem S. A mass M is hung with a light inextensible
pull the string down while (3g) N (on the other side) string as shown in the figure. Find the tension ofthe horizontal
will oppose it. So, net pulling force = 7g-3g= (4g) N, string.
while the mass in motion is 7 +3 = I 0 kg. So, the
magnitude of acceleration of A or B = (Flm) = (4g/10)
= O.4g m/ s2, NowasA is accelerated down whlleBup,
so, TA < (7g) Nand TB > (3g) N
(as tension increases when body is accelerated up and
decreases when accelerated down). But as tension in
the string must be same everywhere,
(3g)N< T< (7g)N
However, if we consider motion of D,
TB ~ m(g+a)~3(g+0.4g)~(4 . 2g)N
and if we consider the motion of A, Fig. 3.18
TA ~m(g-a) ~ 7(g - 0.4g)~(4.2g)N Solution: As there is a load atP, so tension in AP andPB
From this, it is clear that both A or B yield the same will be different. Let t,here be TJ and T2 respectively. For
tension = (4.2g)N, i.e., 4.2 kg which is more than 3 kg vertical equilibrium of p,
and less than 7 kg.
T2 cos 60° = Mg, i. e., T2 = 2Mg .... (i)
And for horizontal equilibrium of P
T} = T2 sin 60°= T2 = (J3 /2) .... (ii)
Substituting the value ofT2 from Eqn. (i) in (ii),
I T\ ~(2Mg)x(-./3/2)~(-./3)Mg
7, ADS.

Problem 6. Two particles, each ofmass m, are connected


ja by a light string of length 2L as shown in [Fig. 3.19(a)]. A
A=7kg continuous force F is applied at the mid point of the string
ja (x =O) at right angles to the initial position of the string. Show
that acceleration of m in the direction at right angles to F is
Ib) given by
Fig. 3.17
F x
a ~-r=i=="",
Now if we place A (7 kg) mass on a frictionless table x 2m ~L2 _ x2
and B (3 kg) hangs through a string from the pulley as
shown in Fig. 3.17(b). The forces acting on the system where x is the perpendicular distance of one of the particles
are as shown in same figure. In this situation the from the line ofaction ofF. Discuss the situation when x = L
pulling force is (3g) N and as no force is opposing it, Solution: Let the mid point of the string Obe displaced by
so the system will move whatever be the masses of A y downwards alongy~axis, so that at a given instant each string
andB. Here the mass in motion is3 + 7 = lOkg, so ace. makes an angle e with they~axis as shown in Fig. 3.19(b). For
of A or B =(3gI10) =0.3g m! 52. Also, the tension in motion of point C,
the string will definitely be less than (3g) N as 3 kg is F-2Tcos9=Oxa y , i.e., T = F / (2cos9) .... (i)
acceleration down. Actually if we consider the motion
ofB,
.. (I) Single fixed pulley changes the direction of force only and in general is assumed to be massless and frictionless. For details of pulley and system of
pulleys interested reader is advised to consult other books.
(2) The treatment given here is somewhat different from conventional method used to solve the problems of pulleys.
m~ -);;;;;;;;;;;;~2L~;;;;;~ m each o/length 1 m. The upper wire has negligible mass and the
A 0 B lower wire has unifonn mass 0/0.2 kg/m. The whole system 0/
blocks. wires and support have an upward acceleration 0/0.2
ml;. Acceleration due to gravity is 9.8 mI;.
Find the tension
at (a) the mid point 0/ lower wire. and (b) mid point o/upper
wire.
F F
y-axis Solution: As the system is accelerated
(s) (b) up, tension at any point will be B
Fig. 3.19 T =m(g+o) [See§3.3.6(b)]
1.9kg
Now consider the motion of mass m at A towards B or Now as for point A, 1
vice-versa" Then as component of T in the direction of motion m =2.9+0.212 = 3kg a
will be Teas (90°-0) = Tsin9, so if ax is the acceleration ofm T, =3(9.8 +0.2)= 30N
along x-axis, then from F = ma, 2.9 kg
And for point B,
Tsin9 = ma x or ax =Tsin9lm .... (ii) Fig. 3.21
m = 2.9+0.2+ 1.9+0 =5 kg
Substituting the value of T from Eqn. (i) in (ii).
so Ts =5(9.8 +0.2) =50N ADS.
F sinS F F x
ax =-2c-o-,-e x -m- = -2m- tan = 2
-m e
- -~FL"i,_'=,=,::" Problem 9. What is the tension in a rod 0/ length Land
mass Mat a dislanceyfrom Fl . when the rod is acted on by two
This is the required result. From this, it is clear that if unequal/orces and F2 « F,) as shown in Fig. 3.22?
x =L, ax=oo, j,e. , this situation cannot be realised in practice as
when Fis applied, the mid point of smog will be depressed in
the y direction and ~ x cannot remain equal to L.
Fig. 3.22
Nole: (i) Acceleration of particle in the direction of force will be,
Solution: As net force on the rod = (F, - F 2 ) and its mass
a, =- - =-2m =constant
Tease F
In is M, so acceleration of the rod wili be
a
(ii) As particles move towards each other x or will decrease and o = (FJ -F,)/M ....(i)
so also the acceleration ax and for x- O,(G=OO),O;. .. 0.
Now considering the motion of part AB of the rod [which
Problem 7, Two blocks of masses 6 kg and 4 kg has mass (M/L)y and acceleration a given by Eqn. (i)]
connecfzd by a rope ofmass 2 kg are resting on a frictionless assuming that tension at B is T,
floor as shown in Fig. 3.20. If a constant force of 60 N is FJ- T=(MIL)y xa [fromF = ma]
applied to the 6 kg block, find the acceleration of the system Substituting a from Egn. (i)
and the tension in the rope at points A, Band C.
Solution: As the FJ -T=(~yrJ :' )
mass of the system is
(6+4+2) =12 kg and
or Ans.
applied force is 60N, the
acceleration of the system,
Fig. 3.20
F 60 , Note: To calculate tension at B we can also consider the motion of the
a =- =-=5m1s other part ofrod, i.e., Be. However, then equation of motion win be,
m 12
T -F2 := (MIL)(L- y)xa
Now at point A as tension is pulling the rope and block B
r. M(L - y) (Fj-F2)
of mass 4kg, i.e., T :=r2 + L x M
T, = (2+4)x5 = 30N which on solving gives again,
Similarly for Band C,
TB = (1+4)x5 = 25N and TC =(O + 4)x5=20N Ans.
T~ Fj (1 "" Z) + Fz(i)
Note: In this problem as rope is not massless, tension is different at Problem 10. A small mirror 0/ area A and mass m is
different points of the string, being maximum at Ihe end c losesl lo suspended in a vertical plane by a weightless string. A beam 0/
the app lied force and minimum at the end farthest from the force. light 0/ intensity 1falls normally on the mirror and the string is
deflected/rom the vertical by a very small angle e. Assuming the
Problem 8. Two blocks of masses 2.9 kg and 1.9 kg are
suspended from a rigid support S by two inextensible wires mirror to be perfectly reflecting. obtain a~ expression for e.
Solution: If light exerts a force F on the mirror, then for the rope with a constant downward force F = 2mg. In which
vertical equilibrium of the mirror, case the acceleration of 'm' is more?
Solution: In case (A), the pulling force =2mg-mg =mg
and the mass is
2m+m=3m
p So, Acceleration a = (mgl3m)=g13
IT5=:,=''=
p
light While in case (8), the pulling force = 2mg - mg =mg
Mirror but The mass in motion = m + 0 == m
mg
so Acceleration a' = (mg/m) = g
Fig. 3.23 i.e., acceleration in case (8) is more (actually 3 times) than in
case (A). Ans.
Tease = mg .... (i)
Problem 12. Three equal weights of mass m each are
and for horizontal equilibrium,
hanging on a string passing over a rued pulley as shown in
Tsin9 =F .... (ii) Fig. 3.25. What are the tensions in the string connecting
Dividing Eqn. (ii) by (i), weights A to Band B to C?
F Solution: In this problem, as the pulling force is 2mg
tan9 = - .... (a)
mg while opposing force is mg, so net force =2mg - mg = mg, and
as the mass in motion = m + m + m = 3m,
Now for light radiation,
E=hv=h(d).) [as c = vA] so The acceleration = Force = mg = K
Mass 3m 3
or E=pc [as). =hlp] .... (b)
Now as A is accelerating up while ~=:i'E""'­
So, change in momentum of light due to reflection
Band C down, so tension TJ is such that
t:.p = p, - P, = p-(-p)=2p
mg< T\ <2mg
So, force due to light radiation on the mirror
Actually for the motion of A
F = I1p =2p = 2E [as from (b), p =Elc] T, =m(g+a)=m(g+g/3)
!:J.t {).t cD.t
= (4/3)mg Ja
or F =2IA
c [ as I=...!!...-]
AM
.... (e) Now to calculate tension in the
string BC we consider the downward
Substituting the value of Ffrom Eqn. (e) in (a), motion ofC, Fig. 3.25

i.e., T, = meg - a) = m(g - g/3) =(2/3)mg Ans.


tanS =e = 2!A [as9 is small] Ans.
mgc Problem 13. An elevator and its load weigh a total of
1600 kg. Find the tension T in the supporting cable when the
Note: This example is l!n i!lustration of the fact that like malter, elevator, originally moving downwards at 20 mis, is brought
radiation (light) also exerts pressure (force).
to rest with constant acceleration in a distance of 50 m.
Problem 11. The pulley arrangements of Fig. 3.24 (a) Solution: As the lift, moving downwards, stops after
2
and (b) are identical. The mass a/the ropes is negligible. In travelling a distance of 50 m, from v 2 = u +2as
we get 0 2 =20 2 +2a x50
a
i. e., a = -4 mls 2
1 ;~/fll i.e., lift is accelerated up with an acceleration 4 mls 2 .
m Ia m I

.. W = m(g + a) = 1600(!O+4)
= 22400 newton Ans.
2m F ", 2mg Problem 14. Consider the situation shown in Fig.
(al (bl 3.26(a). Both the pulleys and the strings are light and all the
Fig. 3.24 surfaces are frictionless. Calculate (a) the acceleration of
mass M, (b) tension in the string PQ and (c) force exerted by
(a) the mass m is lifted up by attaching a mass 2m to the other the clamp on the pulley.
end of the rope. In (b) m is lifted up by pulling the other end of
For mass M, T =Mao .... (i)
For monkey, mg-T=maO .... (ii)
From Eqns. (i) and (ii),
m Mm
ao= - - g;T= - - g Ans.
m+M M+m

Ib) (ii) The monkey's actual T


acceleration is superposition of M
FIg. 3.26
acceleration of rope and • • •111• •~
Solution: As pulley Q is not fixed, so if it moves a acceleration of monkey relative
distance d the length of string between P and Qwill change by to rope. Let the monkey move
2d Cd from above and dfrom below), i.e., Mwill move 2d. This
in turn implies that if a (-t 2ei) is the acceleration of M, the
acceleration ofQ and of 2M will be (aI2) (-t d).
upwards with respect to the rope
with an acceleration a MR = a.
The weight moves with an mg
r
acceleration ao , which is the Fig. 3.27
Now if we consider the motion ofmassM, it is accelerated
downward; so acceleration of rope relative to
T ~ M(g - o) .... (i) fixed pulley.

and for the motion of Q,


'2T-T'=Ox(al2)=O, i.e., T' =2T .... (ii)
~ ~ ~
and for the motion of mass 2M, aM = a MR + a R = (ao - a)
T' =2M(aI2), i.e., T' = Ma .... (iii) [as monkey moves upwards and rope slides downwards,
From Eqos. (ii) and (iii) as T = (1I2)Ma, so Eqo. (i) reduces to overall the monkey will be accelerated downwards]
(1/2)Mo~M(g-o) or o~(2/3)g The equations of motion are:
So, the acceleration of mass Mis (Z/3)g while tension in For mass M, T = Mao .... (i)
the string PQ from Eqo. (i) will be, Formonkey, mg-T=m(ao -a) .... (ii)
T ~ M[g - (2/3)g] ~ (1/3)Mg Solving Eqns. (i) and (ii), we will get
Now from Fig. 3.26 (b), it is clear that force on pulley by ao = meg + a) and T = Mm(g + a) Ans.
the clamp will be equal and opposite to the resultant ofTand T M+m M +m
at 90° to each other, i.e.,
(iii) For downward motion of the monkey, we have to
(R2)~~"T';-2+-T-;;-2 = ..fiT =(..fi13)Mg Ans. change sign of a to get

Note: In this problem the acceleration of pulley Q or mass 2Mwill be


°0 ~ "m"'(g'-.---'o:!.) and T = Mm(g - a) Ans.
M+m M+m
al 2 = (1I3)g and tension in the string connecting 2Mto the pulley
Q, T' will be 2T = (2/3)Mg. Problem 16. A block ofmass m is placed on the inclined
surface ofa wedge ofmass M as shown. Find thp. acceleration
Problem 15. A monkey of mass m climbs up to a rope of the wedge if the frictional forces are negligible.
hung overafixed pulley. The opposite end ofthe rope is tied to
Solution: When the block m is released from rest, its
a weight ofmass M lying on a horizontal plane. Neglecting the
relative motion is along the inclined surface of wedge M.
/riction,find the acceleration ofboth the bodies (relative to the
plane) and the tension of the rope for the three cases: Let a be the acceleration of block m relative to the wedge
(i) The monkey does not move with respect to the rope. along the incline and A be the acceleration of wedge relative to \
ground towards left.
(il) The monkey moves upwards with respect to the rope
with acceleration a. The block exerts a normal reaction R on the wedge. The
(iiI) The monkey moves downwards with respect to the horizontal component R sine ofthis normal reaction causes the
rope, with acceleration a. wedge to accelerate towards left.

Solution: (i) Ifthe monkey does not move on the rope, the Rsin9=MA .... (i)
acceleration of both bodies will be the same, equal to ao. The
equations of motion are:
Applying Newton's law, Fresultanl =rma in componer

M
m

, - A
fonn:
Alongx-axis Rsin9=ma x
Alongy-axis mg-Rcos9=ma y
.... (ii
.... (lll
mg
where ax =arcos9-A .... (h
a y =a r sin9 ....(,
A:~S'
.(, Solve the above equations to get the result.

-A

, AsinG
ax .. ar-Acos9
a,
Problem 17. Two 100 g blocks hang at the ends ofa fig}
flexible cord passing oyer a small jhctionless pulley. A 40 .
block rests on the block on right and removed after 2 sec.
(a) How far will each block move in thefirst second afte
N mg ay . Asln9
the 40 g block is removed?
Fig. 3.28
(b) What was the tension in the cord before the 40 g bloc
The net acceleration or the absolute acceleration of block was removed?
a = Qbw + all' is the vector sum of its relative acceleration a r (c) What was the tension in the cord supporting th
and the acceleration of wedge A. pulley after the 40 g block was removed?
abw =ab -aw Solution: When two blocks of masses ml and m2 ar
or Qb =abw+ow hanging from the ends of a string passing over a fixed puUe)
Applying Newton's law, Fresultant = :Ema in component their acceleration is:
form.
a =-'(;m-,,2~-_m-'.1)",g
Sum of all the forces along x-axis 'LFx = max T'
(m2 +ml)
Sum of all the forces along y-axis EFy = rna y
We choose x-a::is along incline and y-axis nonnal to it.
T
D T
'EFx = max:;:;> mgsin9 = mea, - Acos9) .... (ii)
EFy =rna y:;:;> mgcos9 - R = m(Asin 9) .... (iii) From t =Oto t =2(M =100g,m=40g), M

We can solve (i), (ii) and (iii) to get


[(M +m)-M]g g M
a= =-
[M+m+M] 6 Fig. 3.30
A = mgsin9cos9
[M + msin 2 €I] v = u + at = 0 + (g/6)(2) = 980/3 ,mls
(m + M)gsin9 Hence, at t = 2s, left block is moving up with velocit
and ar = ADS. 980/3 cmls and right block is moving down with velocit
[M +msin 2 9] 980/3 cmls. Tension in the string during this interval is:
Alternative method T = 2M(m + M)g 1.143N
From the force diagram of the wedge, m+2M
Rsin9=MA .... (i) From t =2 to t =3, when m is rell).oved, blocks move with
For the block, let us take the horizontal and vertical constant velocity.
components of acceleration as a x and a yo The distance covered = s = (98013) x 1= 326.6cm = 3.2261l!
A cos e
Tension in the string =weight of each block.
T = Mg =0.98NandT' =2T = I.96N Am
Problem 18. Find the acceleration ofrod A and wedge,
in the arrangement shown in the Fig. 3.31. The entire surface
y ay
are smooth.
mg
Fig. 3.29
..JR~
Regarding springs it is worth noting that:
L I. Springs are assumed to be massless. This is why the
restoring elastic force in a spring is assumed to be the
same everywhere.
2. Springs can be stretched or compressed and the
M
stretch or compression is always taken to be positive.
Wedge B CI.
(A string cannot be compressed!)
.£.. ..£. ..£. £..
N Rsin a
~, ~,
~ Stretching a spring Compressing a spring

a
Rcosa
-
A
(a)
Fig. 3.33

To produce extension or compression in a spring two


(b)

equal and opposite forces are to be applied and in


m, equilibrium restoring force developed due to
FIg. 3.31 elasticity of spring is equal to either force, i.e.,
F =F'
Solution: Accelerations of wedge and rod are different. In
this problem, applying Newton's law will provide just two and is always opposite to applied force.
useful equations, one in the direction of motion of rod and 3. For small stretch or compression, springs obey
other for the motion of wedge. The force diagram suggests Hooke's law. i.e., for a spring,
four unknownsN, R, a andA. Hence, we will have to establish Force 0: stretch (or compression)
relationship between accelerations of rod and wedge. This i.e., F=ky .... (a)
relationship is referred as kinematic relationship. It can be seen
i.e., restoring force is linear. This force in a spring is
from the free body diagram that if the wedge moves a distance
not constant and depends on stretch (or compression)
S horizontally then the rod moves distance s vertically.
y. Greater the stretch (or compression) greater will be
Relationship between them is, the force and viee·versa.
§. "" cotCl
S t
~

1
Ij
A
or -=cota .... (i) 1
a F k ,"-
Equations for rod: r.F y= ma y. mg - R cos Cl = rna ... (ii) I f
Hyperbola
Equations for wedge: r.F.... = MaxR sin a. = MA .... (Hi) k .. tanO
Solve Eqns. (i), (ii) and (iii) to get A ", geota ADS. y (Stretch or compression) L~L.-ng--';Ih-O~f~s~p~ri~ng":'f=
Mcot 2 a+1 (a) (b)
m Fig. 3.34
§ 3.4 Springs 4. k is called theforee constant of the spring and is equal
Springs can be of many types such as helical [Fig. 3.32 to the slope of force versus stretch curve. It has
(a)J or spiral [Fig. 3.32 (b)] and are stretchable or dimensions [FIL] = [MT- 2 ] and units N/m. Greater
compressible. the force constant of a spring lesser will be the stretch
(or compression) for a given force and morc stiffer is
said to be the spring. The force constant k of a spring
depends on wire (its length, radius r and material)
used to make the spring, radius of spring R and length
(a) (b) I of spring.· It is well established that for a given
Fig. 3.32 spring

• Actually k '" (111'4/4nlR 3) where" is the modulus of rigidity and /I is the number of turns per unit length.
koc(IID .... (b) spring by y, the other will be compressed by the
i.e., smaller the length of the spring, greater will be the same amount; so
foree constant and \lice~versa. [Fig. 3.34 (b)] .... (i)
S. Force constant ofcomposite springs: If a number of Now as F =kyand k's are not equal, so F, * F 2 ,
springs are connected to a body and we want to reduce but F =F1 +F2• i.e., ky = k,y, +k2 Y2 [asF=ky]
it to a single spring: following three cases of conunan
which in the light of Eqn. (i) reduces to
interest are possible:
k=k, + k, .... (e)
.<a) Springs in Parallel: This situation is shown in
Fig. 3.35(a). If the force Fpulls the mass m by y, 6. When a spring is stretched (or compressed) work done
the stretch in each spring will be y. i.e., on i, is stored as elastic potential energy. Ifat any time
a spring is stretched by an amount x, the force acting
Yt = Y2 = Y .... (i) on it F = kx. So the work done in changing the stretch
fromxtox+dx,
dW=Fdx = lo:dx [asF=Io:]
So, work done in stretching the spring from 0 to y,

W= I;lo:dx=k[x ' /2]t = &Iy'


lal
Parabola

o y-
lal Ibl
Fig. 3.36
Now as for a spring F = Icy and as k's are not
So, elastic potential energy U stored in the spring,
equal, so FI '*
F2 but for equilibrium,
F=F1 +F2 • i.e., ky=k lYt +k2Y2 (asF = ky] U = W=!Iy' ....(a)
2
which in the light ofEqn . (i) reduces to
i.e., work done or elastic potential energy stored in a
.... (a) spring varies non-linearly with stretch (or
This is like capacitors in paraliel or resistances in compression). From this it is clear that if the stretch in
series. a spring is doubled, the force is doubled but energy
(b) Springs in Series: This situation is shown in Fig. stored becomes 4-times.
3.35(b), as springs are massless, so force in these
Note: Work done can also be calculated graphically from force versus
must be same, i.e.,
stretch cUlVe. The area of strip shown dA ==F dx '" dW. so work
.... (i) done in stretching the spring from 0 to y, W ",JdA "' A== area of
Now as F =ky and as k's are not equal, so triangle III (1I2)yky== (lnyif.
stretches will not be equal,
i.e., YI *" Y2 but Y= y, + Y2
7. If there are two springs of force constants k , and k2

-F = -+
F, -F, [ asforF = ky,y= -FJ with k , >k 2• then work done :
or
k k, k2 k
Wi (l/2)k,x,
,
(a) When they are stretched by same amount (Xl =X2),
k,
which in the light ofEqn. (0, reduces to -= = - >1
1 1 1 W, (1 /2)k,x; k,
- =- +- +.... .. ..(b)
k k, k2 i.e., Wj >W2
(e)" Spring' in 'erie, with a mass betw'en them' i.e., work done is more on the spring of greater
As shown in Fig. 3.35 (c), if force F stretches a force constant.
Z=7[1±~1+~: 1
(b) When they are stretched by same force,
.... (c)
2
~ _ (1I2)k 1x 1 _ xI
- - 2
Wz (In)k 1x x2 However, as stretch can never be negative so
2
negative sign is inadmissible, i.e.,

z=mg[I+~1+2hk
k mg
l .... (d)

Now if h -to, Z --t (2mglk)


i.e., WJ. < Wz or w2 > wI
9. If a spring is horizontal and a mass m is attached to it
i.e., work done is more in the spring of smaller
as shown in Fig. 3.38(a}, in equi librium it is
force constant. unstretched or uncompressed and so its length will be
8. Elongation or compression produced in a spring: equal to its natural length as mg is balanced by
(a) When a i;ody: is placed on it and equilibrium is reaction R.
achieved. lf ~ the spring is stretched (or
compressed)'tjy x, then in equilibrium
TI
mg=kx or X = :g .... (a)

(b) When a body is just dropped on it, due to change


in momentum on collision an impulsive force will
act and so here, F > mg. If y is the maximum
stretch of the spring, by conservation of
mechanical energy,
lal Ibl lei
g
2"Iky2 =mgy, .
'.e., y=2m
k =2x .... (b) Fig. 3.38
However, if the system is made vertical as shown in
i.e., when the body is just dropped on a spring the Fig. 3.38 (b) and (c) then in equilibrium its length f>
maximum stretch (or compression) is double of
or :< /(= / ± Yo) such that mg=kyo, i. e., Yo = (mgl k).
when the body rests on it in equilibrium. This is also why in some problems related to springs,
(e) When the body is dropped from a height h, then if sometimes we replace (mlk) by (yflg) which is
the spring is compressed (or stretched) by z, constant for a given mass and spring.
:potential energy lost by the body is mg(h + z) 10. If two masses connected by a spring are pulled by a
[and not mgh] while elastic energy stored in the
force F (or one mass is given a velocity vol, then
spring is (112#. So, by conservation of initially the acceleration (or velocity) of the two
mechanical energy, masses will be different and stretch in spring will

I
change [Fig. 3.39(0)].
m m mth After some time both the masses will move with

tF.~ v1 same acceleration (or velocity) and the streich in the


E E 'I )tm spring will remain constant. The problem then can be
C ~ C
treated like t!tat of strings with T =ky [Fig. 3.39(b)].
C
~ iP:\ T
"n, Iv I.
lal Ibl lei
v or a vora
Fig. 3.37
lal Ibl '.
!!z2 =mg(h+z) Fig.. 3.39
11 . If in case of simple hannonic motion, restoring force
0' lkz2 -mgz - mgh=O is provided by a spring, then
2
(a) If the spring has a mass M and a mass m is
which on solving for z yields, suspended from it, the problem becomes
equivalent to that of massless spring if instead of (c) As the spring is stretched by Yo,
suspended mass m we consider effective mass,
me=m+M/3
gain in elastic energy = 4ky~ = ~ mgyo
(b) If two masses ml and m2 connected by a spring
are oscillating, the problem reduces to the [as from Eqn.(a)k = : : ]
oscillation of a single mass m called reduced
mass if we define mas, Note: Here it is worth noting that:
1 1 1 · (i) The mechanical energy lost by the body is mR}t) while the
- =-+- energy gained by the spring is (Il2)mm which contradicts
m m) m2
the law of conservation of mechanical energy. Actually in
Question J. If a spring afforce constant k is cut into two this process mechanical energy is not conserved but energy
pieces o/length I) and 12 ,find is conserved. The difference (1!2)mm is first converted into
, theforce constant ofeach part. KE of mass m (due to which it oscillates) and then lost as heat
Answer: As for a spring; force constant k IX (Ill) to the surroundings due to viscous force between air
1 1 1 molecules of the surroundings and the mass.
so k,oc- k 2 C( - and koc-- (ii) This problem can be viewed as mechanical analogue of
II' 12 /) + /2
charging a capacitor by a battery as also in charging a
k, 1\+/2 k2=h--
- + 12 capacitor by a battery. Energy lost by the battery is qVwhile
so - = -- and energy gained by the capacitor is (1!2)qV, i.(' . SOOIo of the
k I, k I, electrical energy is wasted as heat in charging a capacitor!

i. e., kl=k(l + ~~) and


*2 ={l+:~) Ans. Problem 19. A dynamometer Dis atlached to weights of
mass M= IOkgandm = 10 9. Force F=20 N andf= IO N are
applied to the weights. What will happen to tile weights and
Note: Ir I. : 12 : : I : n. i.e.. 12 - n/,.k. '" k(l+ n)andk2= [1 + (IIn)]k.
what will the dynamometer show if:
a
Q uestion n. If/he stretch in spring offorce constant k is (a ) The force F is applied to larger weight and f to
doubled, find: (0) the ratio affinal to initia/force in (he spring. smaller weight.
(b) the ratio of elastic energy stored in the two cases and (c) (b) The force f is applied to larger weight and F to
the work done in changing the stretch from y to 2y. . smaller weight.
Answer: (a) As for a spring F = Icy (c) What will the dynamometer show if the mass
M=m=Skg?
F2 =1cy2 =2y ~' 2
F, ky" y .
(b) As fora spring V = (1/2)ky' _
,PI' 2 ' .j.. 2 ',.'
V 2 _ (1 /2)ky, _. (Zy) _ Fig. 3.40
-- ----4
V, (l/2)ky~ (y)' Solution: In all the cases as net pulling force is F - f.
while the mass involved is m + M
(c) As work done is stored as elastic potential energy of
. force F- f m
spring AcceleratIOn (a) = - - " -- = 1- ... (i)
mass m+M s2
W=V, -V , = ~k (2y)' -~k(Y)' =~ky' [= 3V,] An,. So. the whole system will move in the direction of larger
force with acceleration given by Eqn. (i).
Q uestion III. If a body ofmass m suspended by a spring
comes 10 rest after a downward displacement Yo.find (a) the (a) IfTt is the reading of dynamometer then for motion of
force constanl oflhe spring, (b) loss in gravitational potential M and m respectively. we have
energy and (c) gain in elastic potential energy. F-TJ= Ma and T1 - f =ma
Answer: (a) For equilibrium i.e., TJ =F -Ma and TJ =f+ma
kyo=mg, i.e., k =mg/yo ".(a) Substituting a from Eqn. (i) in any of the above, we get
(b) As the mass nJ has descended a distance Yo,
TI = mF+Mj
m+M =j=10N .... (")
11
Ans.
loss in potential energy = mgyo
(b) For this case, equations of motion for m andMwill be Now the lower disc will leave the table only and only if
respectively, 10:2 > m2g i.e., x2 > m2glk .... (111)
F-T2 = ma and T2 -/=Ma Substituting the values ofxJ andx2 from Eqns. (i) and (iii)
Either of these equations in the light ofEqn. (i), yields in (ii),

T2 = MF + mf =- F =20 N
m+M
... (iii) ADS. F +mlg =2mlg
k k
+(> m2g)
k
(e) Substituting m = Min Eqn. (ii) or (iii), we get or F>(ml+m2)g ·i.e., F min =(ml+m2)g
T3=F +f =20+10=15N So the lower disc will spring back and rl'se offthe table if
2 2 the spring is pressed by aforce greater than the weight of the
The tension in the spring is independent of mass in this system. Ans.
case. ADs. Problem 21. Two blocks A and B are connected to each
other by a stri1,lg and a spring; the string passes over a
Note: If !=F,a=O and T=F=/ frictionless pulley as shown in Fig. 3.42. Block B slides over
i,e. , if two equal forces are applied to two different masses the horizontal surface of a stationary block C and the block A
connected by a spring the system remains at rest and the tension slides along the vertical side of C, both with same unifbrm
in the spring is equal to eitherforce. The stretch in the spring will
be(Flk)asF=ky.
speed. The coefficient of friction between the surface of the
blocks is 0.2. Force constant of the spring is 1960 Nlm.If the
Problem 20. A disco/mass m2 is placed on a table. A stiff mass of block A is 2 kg, calculate the mass of the block Band
spring is attached to it and is vertical. The other end of the the energy stored in the spring.
spring is attached to a disc a/mass ml [Fig. 3.41(a)]. What Solution: In this problem:
minimum force should be applied to the upper disc to press the (1) As masses A and B are moving with constant velocity,
spring such that the lower disc is lifted off the table when the this is a problem of dynamic equilibrium, i.e. , forces acting on
external force is suddenly removed? mass A (or B) balanc~ each other.
F C , ----------------.
-----------------1" (2) As string and spring are weightless and no mass is
m, =
involved between them, Tstring Tspring T =
A:,'
, ,, :1 (3) Force of friction on block B,fB =~B =j.lmBg (as
, , RB =mBg) while on block A,fA =j.lR A = 0 (as
, , RA = mAgcos90o=O),
I,I 1,I'1' In the light of above for horizontal equilibrium of B,
T=/n = ~mBg .... (i)
B b======_ --======J
While for vertical equilibrium of A,
(a) (b) T=mAg .... (ii)
Fig. 3.41 So from Eqns. (i) and (ii),
Solution: In Fig. 3.41(b), position A of the upper disc
shows undefOlmed spring. Let the external force applied be F. i. e., w".", = 10kg
If on applying the force F, the upper disc of mass ml is pressed
downwards by xl (as at b),
.... (i)
Now if on releasing the upper disc the extension of the
spring is x2 (as at C ), then by conservation of mechanical
energy
I 2 I 2
2'kxl =2'kx2 +mlg(xl +x2)

1 2 2
i.e., 2'k(x l -x2 )=mlg(XI +X2)

2mlg
i.e., xl =--+x2 .... (ii) Fig. 3.42
k
Now as for spring T = ky, in the light ofEqn. (ii), becomes the direction of intended motion. Regarding friction it is worth
ky=mAg i.e., y = (2 x 9.8) 11960 = 10- ' m noting that:
1. If a body is resting on a rigid horizontal surface and no
So the energy stored in the spring,
force is applied, the forces acting are weight and
U =(1/2)ky' =(1/2)x 1960x(lO-')' =0.098 J
Problem 22. On applying a force F the mass M is
"
ADS. reaction which balance each other. The force of
friction is zero (and not IlR). As if it is assumed to act
displaced vertically down by y from equilibrium position. Find on the body, the body will not be in equilibrium and
the force F in terms of the force constant k of the spring and the direction of force of friction cannot be decided,
displacement y,for the cases (A) and (B), as shown in Fig. 3.43. i.e., if a body is at rest and no pulling force is acting
on it,force offriction on it is zero.
Solution: Case (A) For the equilibrium of mass M,
2, Now if a force is applied to pull the body and it does
F=T .... (i)
not move, the friction acts which is equal in
and for the equilibrium of pulley, magnitude and opposite in direction to the applied
2T = F' .... (ii) force (again not IlR), i.e., friction is selfactingforce,
But as due to shift of mass Mby y, the spring stretches by Further as the body is at rest the friction is called static
(y/2), so friction.
F' = k(y/2) .... (iii)
So substituting F' from Eqil. (iii) in Eqn. (ii) and then T
from Eqn. (ii) in Eqn. (i), we get •
F = (k/4)y .... (a)

, I I, Friction = 0
mg
f=F
mg
f .. ma
S S
F' T IF' {.I {bl lei
Fig. 3.44
I P
I I 3. If the applied force is increased, the force of static
friction also increases. If the applied force exceeds a
T certain (maximum) value, the body starts moving.
M F This maximum force of static friction up to which the
Y body does not move is called limiting friction, i.e.,
static friction is a selfadjusting force with an upper
{al (bl limit, called limitingfriction.
Fig. 3.43 4, The maximum value of force of friction up to which
Case (B) As tension in massless string and spring will be the body remains at rest, i.e., limitingfriction is found
same, experimentally to depend on nonnal reaction,
T=F' limiting
.... (i)
Friction Dynamic
And for equilibrium of pulley, T + F' = F .... (ii) f max . ., !lsR - - --- - - - ;-- Friction

So from Eqns. (i) and (ii), F = 2F'


.... (iii) t
-;;
"il
~v x
Now if the mass M shifts by y the spring will stretch by 2y (as o 1> ~

string is inextensible) .-
'13 0'>: 'v.
~~ _ __ _ _ _ _ _ _ _ _

Applied Force F _
L~ _ _~~ _ __
cos e

F' =k(2y) .... (iv)


So, substituting F' from Eqn. (iv) in Eqn. (iii), (al (b)
Fig. 3.45
F = (4k)y .... (b) An,.
§ 3.5 Friction
i. e., fL ocR or fL =lJ.sR .... (.)
where ).ls is a dimensionless constant called
Ifwe slide or try to slide a body over a surface the motion
coefficient of static friction which depends on the
is resisted by a bonding between the body and the surface. This
nature of surfaces in contact and force of adhesion
resistance is represented by a single force and is called friction. between them,
The force of friction is parallel to the surface and opposite to
5. If the applied force is increased further and sets the Lubrication reduces friction as it prevents
body in motion, the friction opposing the motion is interlocking of elevations and depressions.
called kinetic or slidingfriclion. Experimentally. it is 9. It is a general misconception that friction always
also well established that kinetic friclion is lesser than opposes the motion. No doubt friction opposes the
limiting friction and is given by motion of a moving body but in many cases it is also
iK =~KR .... (b) the cause of motion. For example:
where J.1 K is called coefficient of kinetic friction . (a) In moving, a person or vehicle pushes the ground
Ifwe compare Eqn. (a) with (b) as!L>!K, /J- S> ilK . bachyards (action) and the rough surface of
i. e., we require more force to start a motion than to ground reacts and exerts a forward force due to
maintain it against friction. friction which causes the motion. If there had
been no friction there will be slipping and no
Note: In addition 10 static and sliding there is also another kind of motion.
friction called rolling friction. When a body (say wheel) rolls on
a surface the resistance offered by the surface is called rolling (b) In cycling, the rear wheel moves by the force
friction. In rolling the surfaces in contact do not rub each other. communicated to it by pedalling while front
The velocity of the point of contact with respect to the surface wheel moves by itself. So, when pedalling a
remains zero all the time although the centre of the wheel moves bicycle, the force exerted by the rear wheelan
forward. The rolling friction is negligible in comparison to static ground makes the force of friction act on it in the
or kinetic friclion which may be present simultaneously, i.e.,
forward direction (like walking). Front wheel
IlR<I-IK<I-IS
moving by itself experiences force of friction in
6. If we consider a body on an inclined plane which is backward direction (like rolling of a ball).
just on the verge of sliding, then as shown in Fig. [However, if pedalling is stopped, both wheels
3.45(b) for equilibrium along and perpendicular to the move by themselves and so experience force of
plane, friction in backward direction.]
It = mg sina and R = mg cosa (c) If a body is placed in a vehicle which is
accelerating, the force of friction is the cause of
But as by definition It = j.l s R,
motion of the body along with the vehicle (i.e.,
i.e., mg sina = )..t.smg cosa the body will remain at rest in the accelerating
i.e., ll S = tan9 or 9 = tan- I (J.L)* vehicle until ma <,.., s mg). If there had been no
friClion between body and vehicle the body will
Angle a is called angle offriction or angle ofrepose.
not move along with the vehicle.
In practice it is usually found that
0< j.ls < I Note: ' From these examples, it is clear that without friction motion
cannot be started, stopped or transferred from one body to the
7. It has been observed that force of friction is
other.
practically independent of macroscopic area of
surfaces in conlact and relative velocity between them 10. Malian of two bodies one resting on the other can be
(if it is not too high). However, it depends on the dealt easily using properties of friction and equations
nature of material ofthe surfaces in contact (i.e., force of motion. To understand this all, consider the
of adhesion) and their roughness or smoothness (i.e., following two cases:
elevations and depressions). Nonnally, with increase (A) When a body A of mass m is resting on a body B
in smoothness friction decreases. However, if the
of mass M and a force F is applied to the upper -
sUlfaces are made too smooth by polishing and body as shown in Fig. 3.46 (a). Following three
cleaning, the bonding force of adhesion will increase
situations are possible:
and so, the friction will increase resulting in 'cold
welding'. (a) When there is no friction the body A will
move on body B with acceleration (Flm) and
8. Friction is a non-conservative force, i.e., work done
body B will remain at rest, i.e.,
against friction is path dependent. In its presence
mechanical energy is not conserved as it converts, a ... =(Flm) while as =0
energy of motion (i.e., kinetic energy) of a body into
heat. Thus, fric tion reduces efficiency of a machine .

.. Ifinclination of plane 0 < tan 1).1, force offliction '" clttcrnai force (.. /IIg sin 0) and if 0 ~ Ian - ] ).I, frictional force .. ).IR "').I mg cos e
rLB~F
aB =(FIM ), aA =0

a m~:r
M,:iljSt Note: As relat ive to B, A will move backwards with acceleration (F/M)
and so will fall from it after time,
Frictionless
(.)
FOc1lonless
(b)
t.J¥ =JU;L
Fig. 3.46
(b) If friction is present between A and B only
Note: If L is the length of B as shown in Fig. 3.46, (a) will fall from B and both are moving together, then
after lime t F
a=---
m+M
V-;;
t -_ f2LaL -_ J2ML
F
I
[as s= '2at2 and a = Flm]
So, the force on the body A will be
(b) If friction is present between A and B only. F' = ma=~
the body A will not slide on B till m+M
F<fL i.e., F<llsmg Now as this force to the upper body A is
provided by the friction between A and B, so
[ash =~sR = ~smgl
both will move together only and only if
But both will move together with common
F'<h,
acceleration,
i.e" ~q.1 smg or F < ~s(m+M)g
aA=aB=~and F' =Ma= MF m+M
M+m M+m
So, both bodies will move together Wilh
(c) If in case (b) F > fL, the two bodies will acceleration
move in the same direction (i.e., of applied
aA =aR =FI(m +M) if F<~ s(m+ M)g
force) but with different accelerations. Here
force of friction ~Kmg will oppose the motion (c) From case (b), it is clear that both will move with
of A while will cause the motion of B. So, the different acceleration if F>~s (m+M)g and
equation of motion of body A will be. then equation of motion of A will be
F-fK=maA maA =~Kmgi.e.,aA =~Kg

i.e., Qj( =(F- IlKmg)lm [asJK =I..lKmg] while for B,


while for body B, F-/K =MaB j,e.,aB = (F-J.1Kmg)IM
JK=Ma a • i. e., QB=/lKmgIM Note: (0 So, accelerat ion of A relative to B will be

Note: (i) As both the bodies are moving in same direction,


acceleration o f A re lative to B will be,
a-aA-aB = _[F -I!K~m + M)J
_ _ MF - ).lKmg(m+M) negative sign implies that relative to B, A will move
o - oA - as- backwards and will fall from it after time
mM
So, A will fall from B after time,
T - {U _ 2mML
t=J¥=~F _~::C~+M) [as s . ~at2J
V-;; MF - ).lKmg(m+M ) (ii) If there is fri ction between B and floor, the system will move
only if F > Il(m + M)g(= Ii). then replacing F by F - fi.
(ij) If lhere is friction between B and floor, B will move-only if
The whole (B) case wi ll be valid. However, if F < Ii the
IK > Ii ":,,ith Ii '" ", '(m + M)g and then Ix - Ii = MaB' system will not move and friction between B and floor will
However, If B does not move, force between B and floor w ill
be F while between A and B ZCfO.
be f' '" Ix a nd not ii·

(B) When a body A of mass m is resting on a body B Problem 23. A block of mass I kg lies on a horizontal
of mass M and a force F is applied to the lower surface in a truck, the coefficient ofstatic/riction between the
body as shown in Fig. 3.46 (b). Again there are block and the surface is 0,6. What is theforce o//riction on the
three posibi lities: block if the acceleration o/the truck js 5 mls2?
(a) When there is no friction, B will move with Solution: The force on the block due to acceleration of
acceleration (FIM) while A will remain at rest the truck will be opposite to the acceleration of truck and will
(relative to ground) as there is no pulling be
force on A, i.e., F=ma= lx5=5N .... (i)
while the limiting friction,
o
fL =~sR = ~smg =0.6x 1x 9.8 =5.88N
As the applied force F is lesser than limiting friction fL,
,
the block will remain at rest in the truck and force of friction R Y
will be equal to applied force 5 N (and Dot 1£) in the direction
of acceleration of the truck. ----------- Bt h
Problem 24. Assuming the length of a chain to be Land
coefficient ofstatic friction f.l.. compute the maximum length of mg
"'"";
. . <:====~
_
the chain which can be held outside a table without sliding. Fig. 3.48
Solution: If y is the A
maximum length of the chain
which can hang outside the table
,,
So
h=r- ={- ~]
y

without sliding, then for


I
TY Problem 26. A block placed on a
equilibrium of the chain, the W'
weight of hanging part must be
balanced by force of friction on
1 horizontal surface is being pushed by
a force F making an angie 9 with the
the portion on the table: vertical as shown in Fig. 3.49. The
W
Fig. 3.47 coefficient offriction between biock
W = fL .... (i) and sUiface is 1-1-. (a) Find the force
But from Fig. 3.47, required to slide the block with
uniform velocity on the floor. (b)
W = ~ yg Show that if 9 is smaller than a
Fig. 3.49

certain angie9 0 , the block cannot be made to slide across the


and R = W'= ~ (L- y)g floor, no matter how great the force be.
Solution: (a) Forces acting on the block are shown in
so that fL=~R=~ M (L-y)g Fig. 3.49.
L
Substituting these values of Wand fL in Egn. (i), we get For vertical equilibrium of the block,
R =Fcos9+mg .... (i)
M yg=~M(L_y)g
L L Wl)ile for horizontal motion,
which on simplification yields y == ).tIJ(1 + ).t). Fsin9 - ).tR = ma
Problem 25.lfthe coefficient olfriction between an insect or Fsin9 =).tR [as v = constant a = 0]
and bowl is ).t and the radius ofthe bowl is r,find the maximum Substitutin'g 'R from Eqn. (i) in the above,
height to which the insect can crawl up in the bowl. Fsin9 = )l(Fcos9 +mg)
Solution: The insect will crawl up the bowl till the i. e., F= Mmg
component of its weight along the bowl is balanced by (sin9 )lcos9)
limiting frictional force. So, resolving weight perpendicular to (b) As friction is present, for motion F must be + ve. This
the bowl and along the bowl, we get is possible only if
R = mgcos9 .... (i) sin9 -/lcos9>0
fL = mgsin9 .... (ii) i.e., tan9>/1 or 9> tan -I (/l)
Dividing Eqn. (ii) by 0),
or 9>9 0 with 9 0 = tan - I (/1)
tane =(fLlR)
So, for angle9< 9 0 [= tan -I ()l)] no motion will take place
0' tan9 =)l [aslr =f'IIl
~~ however great the force F be.
0' ~(r2 - Y')/y=~
Problem 27. A body ofmass m rests on a horizontal floor
y= r with which it has a coefficient of staticfriction It. It is desired
~1+~2 to make the body move by applying the minimum possible
force F. Find the magnitude ofF and the direction in which it
has to be applied.
Solution: Let the force Fbe applied at an angle 9 with the h =~ =~mg=0.25x20=5N
horizontal as shown in Fig. 3.50. For vertical equilibrium, (a) When F = 2N, F<!I
R +Fsin9=mg i.e., R =mg-Fsin9 .... (i) So, both will move together with acceleration
while for horizontal motion 2 1 2
aB = ac = 20+2 =IT=O.09m1s
Fcos9'<!/L
i.e., Fcose~)J.R [asfL=}lR] .... eii) and force of friction will be equal to applied force (and not
20
~) but = Ma = TI = 1.8 N« F). Ans.

(b) When F = 20 N, the block and cart will move with


different acceleration as F > fL' For the-motion of block,
F-fr=maB
2
i.e., 20-5=2aB' i.e., aB =(1512)=7.5m1s Ans.
and for the motion of cart,
Fig. 3.50 2
fL=MaC, i.e., 0c=(5/20)=0.25m1s
Substituting value of R from Egn. (i) in (ii), we get and in this situation force of friction will be fL = 5 N.
Feose ~ Il(mg -Fsin9) Problem 29. Block A weighs 4 Nand block B weighs 8 N.
F;;:: Jlmg The coefficient ofkinetic friction is 0.25 for all surfaces. Find
i. e., .... (iii)
(cosS + )lsinS) theforce F to slide B at a constant speed when (a) A rests on B
and moves with it (b) A is held at rest and (c) A and Bare
For the force F to be minimum (cosS + )lsin9) must be connected by a Ught cord passing over a smooth pulley as
maximum, i.e.,
shown in Fig. 3.52 (a), (b) and (c) respectively.
de (cosS + j.lsin 9) = 0 A

- sin9+j.lcos9 = O i.e., tan8=)J.


.,

p
or •.•. (IV)

so that, sinS = p-
1+112
and cosS=
1+112
(a) (b)
Substituting these values in Eqn. (iii),
F~ F>
1
)lmg
+ J.l2
.
I.e., - W)J.mg

~1+1l2 ~1+}J.2
sothat, Fruin = ~ with 9 = tan-I (11) Ans. (e)
1+ III Fig. 3.52
Problem 28. A cart weighing 200 N can roll without Solution: (a) When A moves with B the force opposing
friction along a horizontal path. The cart carries a block the motion is the only force of friction between Band S the
weighing 20 N. The coefficient offriction between the block and horizontal and as velocity of system is constant,
the cart is 0.25 and g = 10 mls 2• Find the force of friction
F =!! = ~R! = 0.25(4+8)=3N
between the block and cart and their acceleration when aforce
(b) When A is held stationary, the friction opposing the
of(a) 2 N is applied to the block, (b) 20 N is applied to the block.
motion is between A and Band Band S. So,
Solution: In this
problem the mass of block F =~! +~2 =3+0.25(4)
is (2011 0) = 2 kg, while of i.e., F = 3+1 = 4N
cart (200/10) = 20 kg. The (c) In this situation for dynamic equilibrium of B
force of limiting friction F=~!+~2+T .... (i)
between block and cart, while for the unifonn motion of A,
Fig. 3.51
.... (ii)
Substituting T from Eqn. (ii) in (i), we get Problem 31. A body o/mass 5 x 10- ) kg is launched up on
F=flRl +2flR2 = l+2xl=5N Ans. a rough inclined plane making an angle of 30° with the
horizontal. Obtain the coefficient offriction between the body
Problem 30. A block of mass 2 kg slides on an inclined
and the plane ifthe time ofascent is halfofthe time ofdescent.
plane which makes an angle 0/300 with the horizontal. The
Jefficient of friction between the block and the surface is Solution: When the body is projected up the plane there
(3/2) (a) Whatforce should be applied to the block, so that will be retardation aA such that

the block moves down without any acceleration? (b) What mgsin9 + fL = maA
force should be applied to the block so that it moves up without or QA "" g(sin9 + Ilcos 9)[asfL = Ill? = Ilmgcos6] .... (i)
any acceleration? (c) Ca/cuiate the ratio ofthe powers needed Now from equation of motion,
in the above two cases, in tne speed a/block is same in both the v = u+at and v 2 = u2 +2as
cases.
Solution: Here fL =J.IR =llmg cos9
= .J3i2 x 2 x g x (,fJI2) =lg l.fi
and component of weight along the plane
mgsin 9 =2 x g x (1 /2) = g
~~
(a)
..
(a) As mgsin9 < fL . the block will remain at rest on the Fig. 3.54
plane and so to move it down the plane force F must be applied
parallel to the plane downwards as shown in . 3.53 (a). For the motion up the pl ane,
2
O=u - aA1A and 0 =u -2aAs [asv=OJ
Eliminating u between these, we get
.... (ii)
• fA = )(2slaA)
Now when the body slides down the plane, there will be
(b) acceleration an such that
Fig. 3.53
mgsin9 - IL = maD
So, equation of motion down the plane will be or an =g(sin9 - llcos9) [as/L =Ill? =llmgcos9] .... (iii)
(FD +mgsin9) - IL =ma and from equation of motion,
I 1
or FD +2g - =g- [asv =constantso,a = O] s = uf+ 1 at 2
2 .fi 2
fD = ~(2s laD) [asu=OJ .... (iv)
or
(Ji g=IIN
F D = l-.fi) ADS.
According to the given problem,
(b) If Fu is the force required to move the block up the I
fA = '2tD
plane, the equation of motion will be
Fu -(jL +mgsin9) =ma which in the light of Eqns. (ii) and (iv) becomes

:~ =~:: =~
3
or Fu =2g1 + g .fi [asv = constantso,a = O]
2 2
Now substituting the values of aA and an from Eqns. (i)
or Fu = ( I t ) g=30.6N Ans. and (iii) in the above, we get

(c) As power = (force) x (velocity) :;si::n"'e_--'~::.c::.o:.:s-:-9 =!


sin9 + llcose 4
so PD =FnvD = Fn
[asvD =vu l which on simplification gives
Pu Fuvu Fu
~=~tane=~ x_l =,fJ =0.l46 Ans.
i.e., PD =[l -.fi] =0.36 Aus. 5 5,fJ 5
Pu l + .fi
~B~~
Problem 32. A 60 kg block is pushed up an inclined plane
by means of a horizontal push P as shown in the figure. The
coefficients of/riction between incline and block are ~s == 0.6
and ~k = 0.4 and the ramp makes an angle 0/30 0 with the
horizontal.
~~
A 0
(a) What value of P is required to move the block at a lal Ibl lei
constant speed 0/0.20 mls along the incline? Fig, 3,56

(b) If/he person pushing should stop for rest and leI P =0, Now for equilibrium in a direction perpendicular to AB,
does the block slide back on the incline? R' = mgcos9
~Np x y Peos. which in the light ofEqn. (i) gives

~~~eo~N W Psin 9 W WsinO


R = (llfi)mgeose
And for the motion along the trough.
mgsin9-JJR I -jJR 2 =ma
.... (ii)

Fig. 3.55 or a =gsin e - (2IlRlm) [asR, =R, =RJ


Solution: From the force diagrams as shown in figure: Substituting R from Eqn. (ii) in the above.
x·components: Ffric + W sin e - P cos 9 =0 .... (i) . e 2jJ mgcos9
a=gsm - - x
m fi
y·components: N-Wcos9-Psin8==O " .. (H)
where Ffric =J.1kN i.e., a =g[~ne - fi~eoseJ Ans,
From Eqns. (i) and (ii), Problem 34. The friction coefficient between the board
Ilk (Weose +Psin8) + Wsin8 -PeosS =0 and the floor shown in Fig. 3.57 is Il' Find the maximum/orce
or P [Il k sin S - cosS] == -W[J.l k cosS + sin 9]-
that the man can exert on the rope, so Ihat the board does nol
slip on thef/oor.
. . sin8+).1kcos8
Henceweobtam. p= W=747,24N Ans. Solution: The forces acting on the system are shown ill
o cos9 -Ilk sin 9
Fig. 3.57
If force P is removed. the block will have a tendency to For vertical equilibrium of the point P
slide down due to Wsin 9 (component of weight along incline),
Maximum possible force of friction is ~sN ,
T=F ..
.... (
(i) If Wsin9 > ~ sN, the block slips down,
(ii) IfWsin9< ~sN, the block will stay in equilibrium in
this situation.

As here Wsine= W x .!.< ~sN =0.6x wcose =0.6Wx ..fj,


2 2
[from(ii),pu'P =OJ
therefore, the block will stay in equilibrium. Ans,
Problem 33. A block a/mass m is sliding 011 an inclined
(m+ 9
right angle trough as shown in Fig. 3.56 (a) and (b) . IffJ is the
Fig. 3.57
coefficielll ofkineticfriction,jind the acceleration ofthe block.
Solution: As here reactions are from two surfaces and the And for vertical equilibrium of the system'"
body is a cube so RI =R2 =R. The resultant of these reactions
will be perpendicular to line AB and will be given by
R,t tT tR
eMan eBoard
R' =2R cos 45°= fiR .... (i)
t, R.! t,
Fig. 3.58
• While for the vertical equilibrium oflllan
Rl + T .. mg .... (iii)
and for board R ,., Mg + Rl .... (iv)
Adding Eqns. (iii) and (iv), we again get Eqn. (ii), which is written directly.
R+T=(m+M)g, ;.e.,R=(m+M)g-T .... (ii) direction opposite to intended motion, i.e., on A towards the
Now the system will not move horizontally till pulley while on B downwards so that equations of motion for A
and B will become
T<fL
T+fL=ma and T =mg +fL
;.e., T<~[(m + M)g-TJ (fL=~)
or T =ma - flmg and T=mg+ll ma
which on simplification gives
Eliminating Tbetween these,
T<~(m+M)g
rna - Ilmg = mg + flma
(I +~)
or a =1 + ll g Ans.
So in the light of Eqn. (i), we get max l-J.l
F = ~(m+M)g
Ans. Problem 36. Two blocks of masses
""" (I + ~ ) m and M are connected to the ends of a
Problem 35. Consider the situatlon shown in Fig. 3.59. string passing over a pulley. M lies on
The horizontal surface below the bigger block is smooth. The the plane inclined at an angle 8 with the
coefficient of friclion between the blocks is J.1. Find the horizontal alld m is hanging vertically as
minimum and maximum accelerations with which the system
should move in order to keep the smafler blocks at rest with
shown. The coefficient of static friction
between M and the plane is 11". Find the • Fig. 3.60
respect to the bigger block. minimum and maximum values of m, so
R, that the system is at rest.
Solution: In the present problem motion will depend on
the relative values of m and M. If m is heavier than M, then the
blockM will move upwards.lfrn is much lighter than M, then
the block M will move downwards.
Consider following situations:
c Case (I) M is about to start sliding upwards (Motion
impending upwards). This will happen if m is much heavier
than M.
As M is just at the point of sliding up. frictional force on M
Fig. 3.59 is J.l sN acting down the plane.
Solution: Ifthe system is not accelerated, the blockB will Balancing forces alongxMaxis andY M 3xis:
pull the block A. Let a be the minimum acceleration, so that A N =Mgcos8, T =Mgsin8+flsN, T=mg
and B are at rest relative to C. As A has a tendency to move Hence, mg=Mgsin8 + flsMgcos8
towards the pulley, while B downwards, so force of friction on Y ,
T
A will act away from the pulley while on B upwards. N V T
Now as A is at rest relative to C, so its equation of motion
is
T -fL= ma
or T=ma+flmg .... (i) mg cos 0
mg
[asfL = ~Rl =~mgJ Fig. 3.61 (8)
and as block B is at rest relative to C. so for its vertical and
horizontal equilibrium, we have Maximum value ofm = M(sin6 +lls COSO)
T+fL =mg and R2 =ma Case (iI) M is about to start sliding dowr,wards (motion
or T=mg - flma [asJL =flR2 = flma] .... (ii) impending downwards). This will happen if m is much lighter
Eliminating Tbetween Eqns. (i) and (ii), thanM.
ma + flmg = mg -flma As M is about to start sliding down, the frictional force is
I -~
IlsH acting upwards.
i.e., amin = 1+ J.l g .... (iii) Balancing forces along xMaxis andy-axis:
N=Mgcos9, T = Mg sin9-flsN, T=mg
Now, if the acceleration of the system is continuously
increased, there will comc a situation when A will have a Hence, mg = Mgsina - llsMgcos8
tendency of slipping away from the pulley P and B upwards. In Minimum value of In ,= M(sina -fl s cos9)
this situation the force of friction on A and B will act in a
N'V'T
T Problem 38. In the
figure, masses m" m2
__
P" fiic:::=~m~,1.
and Mare 20 kg, 5 kg
F
and SO kg respectively. M
The coefficient of
mg cos e
mg
friction between M and ,.J:,j;;;~ ______L.._
ground is zero. The
Fig. 3.61 (b) coefficient of friction Fig. 3.64
between m l and M and that between m2 and ground is 0.3. The
Therefore the blocks are at rest if, pulleys and the string are massless. Tlte string is perfectly
M(sin9-l.l scos 9)< m< M(sin9+l.l s cos9) Ans. horizontal between PI and ml and also between P2 and m2' The
Problem 37. Two blocks in contact with each other are string is perfectly vertical between PI and P2. An external
sliding on an inclined board. The mass offirst block is ml and horizontalforce F is applied to the mass M. (Take g =10 mIl)
that of second block is m2' (a) Draw a free body diagram for mass M, clearly
The coefficient of/riction between the ~' showing all the forces.
block and the board ;S III for the block I m (b) Let the magnitude oftheforce offriction between m,
and }l2 for the block 2. The angle of Cl and M be fl and that between ml and ground be h. For a
inclination of the board is a. Determine: Fig. 3.62 particular F it isfound thatfl "'" 212. Findfi andf2, Write down
(a) acceleration a/blocks and equations of motion of all the masses. Find F, tension in the
string and accelerations of the masses.
(b) force R with which the blocks press against each other.
Solution: (a) The free
Solution: Consider the force diagrams of rn l and m2.
body diagram showing all
R represents contact force between ml and m2' forces acting on M is shown
N 1, N 2 are Dannal reactions between blocks and the in Fig. 3.65. F
M
inclined plane. (b) The maximum static
sliding frictional force on tnl, T --"'I--'N~tL_+- __
f l =Il m,g
Mg
"'':.10 = O.3x20xlO Fig. 3.65
Acceleration
=60N and on m2,
12 =ll tn 2g=0.3x5 x lO = ISN
m19 sin a For given F, f1 =212
Fig. 3.63 .'. Maximum value of fl can be 30 N as
Balancing forces along y-axis: (r,)~ =15N
N 1 = mlgcosa. .... (i) This implies that ml cannot slide on Mbut moves along with it
Forces along x-axis: with same acceleration, as pseudo force is less than maximum
N2 = m2gcosa. .... (ii) static frictional force. So, aU three blocks are moving with
m,gsino: - R - !lIN I =ml a .... (iii) common acceleration a. The forces acting on ml, m2 and M in
the direction of motion are as shown in Fig. 3.66.
m2gsina +R -!l2N2 =m2a ....(iv) -0 -a - a
(m, +m2)gsina -!l)m,gcos o: - J.12m2gcosa =(ml +tn2)a T. I m, I I m21 •T I MI· F
a = g[(m, + m2 )sino: - (J.1,m, + J.12m2 )cosa]1 (m , + m2); ' l- 212-30N '2-15N f l _30N
g=9.8m1s 2 Ans. Fig. 3.66

Substituting the expression for a, in Eqn. (iii) or (iv), we get Applying Newton's 2nd law of motion,
R =0.t 2 -J.1,)m,m2gcos a /(m, +m2) Ans. 30-T=20a .... (i)
T-15=5a ....(ii)
Note: IfjJ, > jJ 2 then R will eome OUi to be negative. Reaclion between
F -30 =500 .... (iii)
two bodies can never be negative. Least value of the reaction
between two bodies is zero, which implies that the blocks will Adding Eqns. (i) and (ii), 15 =25a
slide down without touching each other. They will get separated. a = 0.6 mls 2
Substituting for a in Eqns. (i) and (iii), planes ofa wedge and the peg isflXed to the top ofthe wedge.
T =30-20xO.6=lB N The planes ofthe wedge supporting ml and m2 are inclined at
30° and 60° respectively, with the horizontaJ. Calculate the
and F = 30 +50 x 0.6 =60 N Ans.
acceleration of the masses and the tension in the string.
Problem 39. A man pushes a cylinder ofmass ml with the p
help of a plank of mass m2 as shown. There is no slipping at

F_:IE'
any contact. The horizontal component of the force applied by
the man is F. Find:
(a) the accelerations of the plank
and centre of mass of the cylinder and m,
(b) the magnitudes and directions
a/frictional/orees at contact points. Fig. 3.67
Fig. 3.69
Solution: Fig. 3.68 (b) shows the plank moving to right
with acceleration Qp towards right under the action of external Solution: As mjgsin30"> m2gsin600, so m1 will be
force F and frictional force f l - In Fig. 3.68 (c), the cylinder is moving downward and m2 upward with the same acceleration
rotating due to frictional forces II and h producing a torque as the string is inextensible. The forces and their resolved
about the centre of mass. components along and perpendicular to the incline are shown

F_~.m, F (
""+- (If. in Fig. 3.69. The equations of motion are:
mlgsin300-T =mla .... (i)

~
T - m2gsin 60°= m2a ....(ii)
f,
Adding and solving,
(a) (b) (e) mlgsin300-m2gsin 60°
Fig. 3.68 a=
(ml+m2)
For linear motion of plank and cylinder, equations of
motion are, 3X! __1 x J3)X1O
....(i)
( 2 J3 2
F- f l =m2ap
I I + h =mlac .... (ii)

where a. =~ , a c being the acceleration of centre of mass


= 2.8 mls 2
of cylinder having radius R and a. is its angular acceleration. Substituting for a in Eqn. (i),
For rotatory motion of cylinder, T=mjgsin300-mla
2 . 1
(jl_ h)R =(mIR2)a = m1R . a c .... (iii) =3x lO x- - 3x2.8
2 2 R 2
=6.6N Ans.
Further, a p =2a c' Solving equations 0), (ii) and (iii),
Problem 41. A rod of length J and mass M is
4F
ac = '(3OCm
-,-'+'-;;-Bm-,') fIXed to one end of a string. The string passes over
a mass/ess.frictionless pulley. A small baJJ ofmass
a = 8F m « M) is attached to the other side of the string. T
p (3m l + 8m 2) The ball has an opening through which the string
passes alld the baJJ call sJide alollg Ihe Siring with F
f l = 3mlac = 3m1F somefric/ioll. Initially, the ball is held at level with
4 (3ml +8m2) the lower end of the rod. Whell set free. both the
masses move with constant acceleration. What is Mg
and Ans. mg
the force offriction. if the ball rises to the level of
FIg. 3.70
tile IIpper end of the rod ill time t?
Problem40. Two blocks ofmasses ml =3kgandm2 = ~ Solution: When the system is set in motion, let aM, am
be the accelerations of rod and ball respectively and Fthe force
kg are connected by a light inextensible string which passes
of friction applied by the string on the ball. The equations of
over a smooth peg. The blocks rest on the inclined smooth
motions are:
Mg-T~MaM .... (i) (ii) It is long range force as it acts between any two
F -mg=ma m " .. (ii) particles in the universe.
The string is pulled up and the ball slides down due to its (iii) It is independent of the nature of medium between the
weight. Friction between the ball and string tries to hold it but particles.
ultimately it moves with acceleration am' As the string is IB] Electromagnetic Force
,
massless, the net force on any portion of it must be zero. So, Force exerted by one particle on the other because of the
F~T electric charge of the particles is called electromagnetic force.
Eqn. (i) can be written as: The description of electromagnetic force is more complicated
and is given by Lorentz force equation-
Mg-F=MoM .... (iii)
... ...... ... ... -+
Adding Eqns. (ii) and (iii), FEM = FE+ FM =qE+q(vx B)

am +aM=F(~ - ~) .... (iv) For simplicity we divide it into two part, viz., eiectri(
which makes an electrically charged balloon stick to a walJ an<
As the rod moves downward while ball upward, so, relative magnetic with which a magnet picks up iron nails. Followinl
distance of the rod and ball in time t. are the main characteristics of electromagnetic forces:
(i) These can be attractive or repulsive.
l=~(am +OM)t
2
(ii) These are long range forces.
Substituting for (am +0 M ) from Eqn.(iv), (iii) These depend on the nature of medium between th,
charged particles.
1=!F(.!. __1 )12 (iv) All macroscopic forces (except gravitational) whic!
2 111 M
we experience as push or pull or by contact ar
or F = 21mM ADs. electromagnetic, i. e., tension in a rope, the force of frictior
(M-m)t 2 reaction, muscular force or force experienced by a compresse
spring are electromagnetic forces. These are manifestations (
Note: In case F <mg,om is negative, i.e., ball slides down. The relative the electromagnetic attractions and repulsions between atom:
diSPlacement",! (aM - Om)f2 and so the final result remains e.g., (a) When a body is placed on a surface. the surface due t
electromagnetic repulsion pushes it up producing nonru
unchanged.
reaction, (b) Tension in a stretched string is maintained onl
because the atoms of the rope attract one another.
§ 3.6 The Basic or Fundamental Forces Ic) Nuclear Force
All the forces observed in nature such as muscular force,
It is the strongest force that keeps nucleons (neutrons ar
tension. reaction, friction, elastic, weight. electric, magnetic or
protons) together inside the nucleus in spite of large electr
nuclear, etc., can be explained in tenns of only the following
repulsion between protons. Radioactivity, fission and fusio
four basic interactions:
etc., result because of nuclear force. Regarding nuclear force
(AJ ~ravitational Force is worth noting that:
The force of interaction which exists between two (i) It is charge independent as it exists between tv
particles due to their masses is called gravitational force. This neutrons, a neutron and a proton or two protons. (Yukawa gao
force is given by the well known Newton's law of gravitation, his 1t-meson theory to explain it).
i. e., (ii) It is a short range force, i.e., effective only over
distance of few fenni .
(iii) It shows saturation properties, i.e., force required
or remove a nucleon from a nucleus is approximately the sar
whether the nucleus contains 10 or 100 nucleons.
Falling of an apple to the earth or motion of moon around (iv) It can be attractive, repulsive or zero. For r< 0.5 fer.
the earth are the most familiar examples of gravitational force. it is repulsive and for r > 40 fenni it approaches zero.
This is discussed in detail in Chapter 9 'Gravitation and (v) It is spin dependent. The force between two nuclec
Gravity'. Here it will be sufficient to remember that: having parallel spins is stronger than the force between l'
(i) It is the weakest force and is always attractive. nucleons having anti-parallel spins.
IDI Weak Force (4) Now as according to Newton's II law:
It acts between any two elementary particles. Under its ~

action a neutron can change into a proton emitting an electron ~dp


F=- i. e.,
and a particle called antineutrino. This process is called dt'
negative ~ -decay. Never think that a neutron is composed ofa
proton, an electron and an antineutrino. The force responsible which in the light ofEqn. (0 reduces to
for this change and many other radioactive decay reactions is
.. ,(iv)
different from gravitational. electromagnetic or nuclear forces.
Such forces are called weak forces. Tile range of a weak force i.e., the action of impulse is to change Ihe momentum ofa body
is very small, in fact much smaller than the size ofa protanor a or particle and the impulse ofaforce is equal to the change in
neutron. momentum. This statement is known as impulse-momentum
It has be:en found that for two protons at a distance of I theorem and is analogous to work-energy t"eore~ with the
fermi, differen6e that the fonner is related to vector .physical
quantities while the latter to scalars. '
F N''FEM''FW''FG "1'10
'" - 2'10-
.
7'10- 38
. ,
(5) Examples of impulse are hitting, kicking, catching,
Physicists have long believed that all these fundamental
jumping, diving and collision, etc. In all these cases an impulse
forces are different aspects of a single ·super-force'. Einstein
acts. Now for a given effect,
spent most of his life in trying to unify these forces but failed .
However, in 1979, Glashow. Salam and Others showed that J =IF dl =Favllt "" 6p ""constant
weak force and electromagnetic force are different aspects of a
So, F. = constt.
single 'electro weak force'. The quest for further unification of av Ill'
fundamental forces still continues.
So if time of contact 6t is increased, average force is decreased
§ 3,7 Impulse (or diluted) and vice-versa. This is why:
Ifa large force acts on a body or particle for a small time, (a) In hitting or kicking a ball we decrease the time of
the product offorce with time is defined as impulse, i.e., contact, so that a large force acts on the ball producing greater
~
J = J'2 ~Fdl .... (i)
acceleration. . .
(b) In catching a ball a player by drawing his hands
"
Regarding impulse it is worth noting that: backwards increases the time of contact and so, lesser force
acts on his hands and his hands are saved from getting hurt.
(I) It is a vector quantity having dimensions (MLT- 1]
and unit N-s. Its direction is same as that of force. (c) In jumping on sand (or water), the time of contact is
increased due to yielding of sand or water, so force is diluted
(2) From Eqn. (i) it is clear that the impulse is equal to the
and we are not injured. However, if we jump on a cemented
area under F(t) curve (Pig. 3.71).
floor, the motion stops in a very short interval of time resulting
p' • ••
in a large force due to which we are seriously injured.

I Q uestion IV. A force exerts an impulse J on a body


changing jt.~ speed from II to v. The force and object's motion
are along the same line. Show thaI the work done by the force
is J(u +v)/2.
----,----,. Answer: According to work-energy theorem we have
W = M< =.!.mv 2_ .!.mIl 2
2 2
j. e.,
1
W =-m(v - u)(v + u) .... (i)
(3) In case of impulse usually force is time dependent, so 2
if Fav is the average magnitude of the force, we can write the But accordfng to impulse-momentum theorem
magnitude of the impulse as J = m(v - u) .... (ii)
J = 1'2 F dt = Fav It2 dt = Fa~/).t .... (ii) So, eliminating m between Eqns. (i) and (ii),
I, I.

in which /).t is the duration for which the force acts. The value w = ! _ J_(v-u)(v + u)
2 (v- u)
of Fav must be taken so that the area within the rectangle of
Fig. 3.71 is equal to the area under the F(r) curve, i.e., i. e., w = J(v + u)/2
I,; = (JIM) .... (iii) This is the required result.
Problem 42. A cricket ball a/mass 150 g is moving with a
velocity 0/12 mls and is hit by a bat so that the bali is turned
back with a velocity of 20 mls. If the duration of contact ,
.'Rot5'"
of/
.:;~/
,0
r
0,
,
Rotation \\\
0 0

between the ball and bat is 0.01 s, find the impulse and the
averageforceexerted on the ball by the bat. [EAMCET 1992]
<6;.~<
$o,~~~
0,,;,: ~C"k
e ( p
e
\

p
p F P
Solution: According to given problem, change in 0"'0$ o~ , , e ,,
F e, ,
momentum of the ball ,,:e F
,,• F ,,
f.p= PI - Pi = m(v - u) = 150x10- 3 [20-(-12)] lal Ibl lei 1'1
Fig. 3.73
So, by impulsewmomentum theorem,
J = lip = 4.8 N-s Further from Eqn. (ii) it is also clear that torque t will be:
And by time a:v~raged definition of force in case of impulse, minimum maximum
F =2..= f.P = 4.80 = 480N Ans. When Isin el= min = 0 When Isin 91= max = I
av llt III 0.01 i.e., 9 = OoorI80 o i.e.,9 = 90°
§ 3.8 Torque " ~ ~ ~ ~

i.e., F is collinear with r. i.e. , F is orthogonal to r.


If a pivoted, hinged or suspended body tends to rotate and tmin = 0 [Fig. 3.73 (c)] and 'max =Fr [Fig. 3.73 (d)]
under the action of a force, it is said to be acted on by a torque:
(4) Torque is the cause of rotatory motion and in
............
l' = rx F .... (i) rotational motion it plays the same role as force plays in
Rotation
translatory motion, i.e., torque is rotational analogue afforce.
....--. o
This all is evident from the following correspondencies
between rotatory and translatory motion:
F Rotatory motion Translatory motion
Rotation
~
../ O~ ~
1 t =Ia. -'l-F=m1
~ ~ ~ ~
F (ii) w=I ,,·de -'I- w =IF·d s
(al (b) (e) ~~ ~~
Fig. 3.72 (iii)P='t·(O -+ P = F·"
~
~
Regarding torque it is worth noting that: ~ dL ~ dp
(iv) 't =- -+ F = -
(1) It has dimensions [ML 2T - 2] and units N-m, same as d, dt
that of work or energy. However, in case of torque N-m cannot
be written as joule, because joule is used specifically for work (5) Rotation ofa door about a hinge, rotation of grinding
or energy. wheel about a pivot or unbolting a nut by a pipe-wrench can be
(2) It is an axial vector, i.e., its direction is always cited as examples of torque. In these for producing a desired
rotational effect-
perpendicular to the plane containing vector -; anq in F
accordance with right hand screw rule. It is customary to t = constant i. e. , Frsin8=constant i.e., F = co~e'
rsm
ascribe a positive sign to all torques acting to tum a body
anticlockwise and a minus to all torques tending to tum it i.e., longer the ann and greater the sin 8, lesser will be the force
clockwise. required for producing the desired rotational effect. This is
why it is much easier to rptate a body about a given axis when
(3) The magnitude of torque will be:
the force is applied at maximum distance from the axis of
........ rotation and nonnal to the ann.
t = rFsin8 [asIAxBI=ABsin8] .... (ii)
(6) On tilting, a body will restore its initial position due to
But from Fig. 3.73 (a) or (b) rsin8 = p = 1 distance
torque of weight about the point 0 till the line of action of weight
between axis of rotation and line of action of force.
passes through its base. On tilting, a body will topple due to
So, t = pF =Momentofforce .... (iii) torque of weight about 0, if the line ofaetion of weight does not
This is why torque is synonymous with moment afforce, pass through the base. This is self-evident from Fig. 3.74:'
and p is called moment or lever arm.
Question V. Discuss whether or not a body is in
equilibrium and what type 0/ motion it executes when two
oppositely directed/orces are applied to it.
Answer: There are four possibilities depending on the
magnitude and point of application of the forces:
(A) Wh en the forces are equal and act along the same line
As shown in Fig. 3.77 (a) in this situation:
W
r.F=O and I"t=O
F ~
(a) (b)
Fig . 3.74

(7) A body is said to be in rotational equilibrium'" if


resultant torque acting on it is zero, i.e.,
->
1>t =0
...L ..t-

F
I

j
+-
F, --- F,

F,
e.g., in case of beam balance or see-saw, the system will be in Rest Spinning Slipping Rolling
rotational equilibrium if (a) (b) (e) (')
-> -> Fig. 3.77
'tl+'t2=O
So the body is in both translatory as well as in rotatory
or Fill -F,!, =0 equilibrium, i.e., body will remain at rest, if initially it was at
or Fill = F2 1z rest. (A dipole situated in a uniform field with its axis
collinear with the field is an example of this situation.)

F+ ~
+ \
I,) .! F,
(B) When the forces are equal and do not act along the same
line
As shown in Fig. 3.77 (b) in this situation:
Fl R.H.S. EF=O but );t*O [=FI]
LH.S.
Ag.3.75 So, the body is in translatory equilibrium but not in
rotatory equilibrium, i.e., it wi ll not translate but will rotate,
i.e. , will spin. (A dipole situated in a unifonnfieldwith its axis
However, if t; >';; L.H.S. will move downwards and if
at an angle to the field is an example of this situation.)
~ < 't-; R.H.s. will move downwards and the system will not (C) When the/orces are unequal and act along the same line
be in rotational equilibrium. As shown in Fig. 3.77 (c) in this situation:
IF*O but I"t=O
Note: A special combination offorces even when the entire body is free
to move can rotate it. This combination of forces is called a So the body is in rotatory equilibrium but not in translatory
couple. A couple is defined as equilibrium, i.e., it will not rotate but will translate, i.e. , will
combination of two equal but F slip or skid. (A dipole situated in a non-unifonn field with its
oppositely directedforees not acting axis collinear with the field is an example of this situation.)
along the same line. The torque
exerted by a couple is also given by F
(D) Wh en the/orees are unequal and do not act along the same
line
t= 1xF Fig. 3.76
As shown in Fig. 3.77 (d) in this situation:
and is independent o f the location of the ax is of rotat ion. The EFt;O and E"t~O
basic difference between torque and couple is the faet that in ease
of couple both the forees are externally applied while in case of So, the body is neither in translatory nor in rotatory
torque one force is externally applied and the other is equilibrium, i.e. , it will rotate as well as translate, i. e., will roll.
reactionary. (A dipole situated in a non~ulliform field with its axis at an
angle to the field is an example of this situation.)

• As for translatory equilibrium as EF :0, so equilibrium ofa body or system implies:

Eqns. (a) and (b) are called conditions of eqUilibrium.


....(a)
,,' .... (b)
o
Question VI. A
triangular plate oj uniform a
p
a
p

''--.I.thickness
and density is made
to rolate about an axis A
perpendicular /0 the plane of
t~ W b h WI ,,
, ,
c',,
, ,
b h"p W,,
,,
b

the paper and (a) passing


through A (b) passing
through B by the application
(/12)
F
Fig. 3.78
(1/2)

(a)
j
(b)
- j
(e)
Fig. 3.80
of the same force F at C (mid point of AB) as shown in Fig.
From moment summation about the point of application of
3.78. In which case is the angular acceleration more?
the nonnal force N,
Answer: We know that in case of rotational motion . We
I>'C =0, -Ph + We == 0, h=-
't =Ia p
i.e., a=tl/ .... (i) As the force P is applied at larger values of h, the location of
Here as force is applied at mid .point, the line of action of the nonnal force N moves to the right. A
'elf = 'ta =F x (1/2) limiting condition occurs when e == ~. For this case, the nonnal
. And as moment of inertia depends on the distribution of
force acts at the rightMhand edge of the body. The body is in a
mass with respect to the axis of rotation I A > I B, 50,0. A < Cl B,
condition of impending tipping motion about the right-hand
i, e., angular acceleration in cas~ (b) is more.
comer and the corresponding value of h is designated hmax.
Problem 43. A uniform cube ofside a and mass M rests
For moment eqUilibrium about this comer, for the case of
on a rough' horizontal table. A horizontal force F is applied
impending tipping,
norma/to one ofthe faces at a-point that is directly above the
a
ce~Jlre of the face, al a height (3aI4) above the base. What is !t =0, -Phnw. + W"2 =0
the minimum value ofF for which the cube begins to tip about
an edge? h =Wa .... (i)
max 2P
Solution: As shown in
Fig.. 3.79 the ~ube wU,l tip about F The condition for sliding will be determined next. If
the edge through 0 if.torque ofF sliding motion of the body is assumed to be impending, the
.
exceeds the torque of mg (torque friction force Fmax is given by
of R about 0 is zero) _ __ Fmax = flN
The equilibrium requirements of the body are
i.e., Fx1a>MgE..
4 . 2 . rng ~Fx=O, P - ~N=O
Fig. 3.79 tFy=O, -W+N=O
or F>~(Mg) The force P is then found to be
P=~W .... (ii)
i.e.,
P in the above equation is the value of applied force which
is required to cause impending sliding motion of the body.
Problem 44. /n many sitltalions an external force is
a
applied to body to cause it to slid.e along a surface. In certain If sliding and tipping are assumed to be equally likely to
cases. "the ·bQdy may tip over before sliding ensues. Show the occur, then W may be eliminated between Eqs. (i) and (ii), with
criteria that may be usd to predict whether sliding or tipping the result
occurs. ·a
hmax = -
. Solution: Fig. '3.80 (a) shows a body which rests on a 2~

horizohtal plan~. The weight of th~ body is W, and it has the The result in the above equation is independent of the weight
dimensions a and b, 1.1 is the coefficient of friction. A force P, Wand height b of the body and of the applied force P and is a
at a distance h above t,he plane, is applied to the body. function only of the dimensions a and the coefficient of
The freeMbody diagram of the body is shown in Fig. 3.80 friction.
(b). The nonnaJreaction force N is assumed to act at distance e The five possible distinct regimes of motion are:
to the right of the line of action of the weight force. For this 1. 11< hmax' P < fl W: Neither tipping nor sliding occurs
first case, the body is assumed not to slide. and the body remains at rest.
2, h< hmax,P =I..lW: The body does not tip and sliding Problem 46. A table has a heavy circular top ofradius 1m
motion is impending. and mass 20 kg. It has four light legs of length I m fIXed
3. h< hmax. P > I..IW: The body does not tip but sliding symmetrically on its circumference.
motion. with increasing velocity, occurs. (a) What is the maximum area o/the table top over which
4. h=hmax,P=I..lW: Both sliding and tipping motion any weight may be placed withuut toppling it?
are impending and the occurrence of either situation is (b) What is the maximum mass that may be placed
equally likely. The results for this case are anywhere on the table without toppling the table?
independent of the weight and height of the body. Solution: (a) Ifa weight is placed anywhere on the table
5. h> hmax: The body will tip over for any value of IJ.. over the area ABeD enclosed by the lines drawn by joining the
with one or the other of the following conditions. If top of four legs, the table will not topple as there is no torque
).1 ~ : or P :s; IlW, the onset of tipping motion will
for turning the table.
a
occur with the tipping edge remaining stationary with
respect to the surface. Otherwise, with 1..1 < ~ or
p> IJ. W, the onset of tipping motion will be
accompanied by the onset of sliding motion of the mg
edge along the surface.
Problem 45. A cylinder ofmass M and radius R is resting Mg
on a horizontal platform (which is parallel to the x-y plane)
with its axis fixed along the y-axis and free to rotate aboul its FIg. 3.82
axis. The platform ;s given a motion in the x-direction given by
x = A cos rot. If there is no slipping. what is the maximum Now as i.e.,
torque acting on the cylinder during its motion? so Area ABCD=a x a =2r2 =2m2
Solution: We know that in rotational motion So t!:e area over which any weight can be placed without
t =la. toppling the table is 2m 2.
But as tangential acceleration aT = No. and I cylinder= ~ MR 2 (b) If m is the maximum mass which when placed at
maximum distance (i.e., at E) from the line joining any two
legs say CD, the table will not topple if
so .... (i)
MgxGF >mgxEF or m< M[GFIEF]
Now as the cylinder But from Fig. 3.82,
is moving without GF =rcos45°==...!:.... and EF=GE-GF =r-...!:....
slipping with the fi fi
platfonn, so, for it also
Fig. 3.81 so m< M[ (rffi) ]
r(1 - (Iffi)l
x = A cosrot
which on simplification yields
or dx =- A rosmrot
V=- .
dl m< 2.4M
so (m)max =2.4 x20=48kg
or ar=dv =-Aro 2 cosrot .... (ii) i.e., 48 kg is the maximum mass which can be placed anywhere
dl
on the table without toppling it.
Substituting ar from Eqn. (ii) in (0, we get
Problem 47. A steel drill making 180 RPM is used to drill
t=~MRAro2cosrot .... (iii) a hole in a block of steel. The mass of the steel block and the
drill is 180 g. If the entire mechanical work is used up in
Now [cosrot]max = 1 producing heat and the rate 0/rise in temperature oflhe block
and the drill is OS Cis. find: (a) the power of the drill in wall.
So, I tlmax= ~MRAro2 (b) the torque required to drive· the drill.
[sp. heat o/steel = 0.1 and J =4.2jou/elcalJ
Solution: (a) As the whole work is converted into heat, (b) NowasP='t"co i,e., t=P!ro
W=JH = Jmc6 [asH =mc9J but here, ro = 21tn =21t x 180 - 61t rad/s
t 60
so p=dW = Jmc d9
dt dt so ,= (37.8/6.) =2.01 N-m
i.e., P = 4.2 x 180 x 0.1 x 0.5 = 37.8 watt

MISCELLANEOUS SOLVED PROBLEMS

Pro~lem 48. At wood machine is attached below the left (as accelerated down), while of 111 will increase (as accelerated
hand pan ofa physical balance while equal weight (M + m)g is up). As M> 111, decrease in weight will be more than increase. So
place.d in the right hand pan. The beam is horizontal when the net weight on L.H.S. will decrease and, hence, it will move up.
masses M and m on the left hand side are braked. What will
Problem 49. Find the mass of the hanging block in Fig.
happen if the masses M and mare unbraked on the left hand
3.84, which will prevent the smaller block of mass m from
side?
slipping over the triangular block ofmass M. All the surfaces
Solution: Initially the system is in'equilibrium, so are frictionless and the strings and the pulleys are light.
~ = ~ = ~+M~·M R
When the L.H.S. is unbraked, mass M will move
downward while m upwards with acceleration T
(M -m)
a= g
M+m
So, the tension in the string,
m'
T=m(g+a) [orM(g-a)]= 2mMg Fig. 3.84
(M+m)
Solution: Let the mass of hanging block be m', then as
This in turn implies that force on L.H.S. when the masses
are in motion will be pulling force is m'g, while mass in motion (m + M + m'l,
m'g
F' ",, 2T = 4mMg ( .. ) so a = "(m-+-"Mo-!'-+-m"'7,",) .... (i)
L (M+m) .... H

The mass m will not slide over M if


aeos9 =gsin9 .... (ii)
Substituting the value of 'a' from Eqn. (i) in (ii)"

Flg.3.83
(m+M)g
m'geos9 =gsin9
(m+M+m)
Problem 50. In the Fig. 3.85 apainter
of mass 100 kg standing in a crate C of
mass 25 kg, pulls himselfup with the crate
with an acceleration.lfthe painter exerts,
or

-.-
m- [.:::...:m+,.::.:..-M]
eot9-1

I
Now from Eqns. 0) and (ii), it is clear that
an effective force of 450 N on the floor of T
FL-Fi=(m+M)g - 4mMg the crate, find (a) the acceleration of the
(M om) painter and (b) the tension in the string, IT
F F' (M-m)' 0 taking g:: 10 m/s 2 . B
i.e., L- L= (M+m) g> Solution: Suppose the man pulls the I
F
i.e.,
....
FL>Fi or Fi<FL rope with constant force F downwards;
i.e., the force on the left side decreases when the masses are then equation of motion for point B will be
unbraked and hence L.H.S. will move up while R.H.S. down. F +mBg - T=mBa
Note: Physical reasoning: When the masses ofL.H.S. are unbraked, M i.e., F =T [asmB =0] .... (i)
will move down while In up with same acceleration. Now as in while for painter,
accelerated motion W "" meg =1= a), the weight of Mwill decrease R+T-mg::ma
i.e., R +T=m(g+a) .... (ii) Now to calculate F2 consider element dy at a height y
And for the system above the table so that its mass will be (MIL) dy and it will hit
2T - (m+ M)g = (m + M)a the table, with a velocity v = J2gy. So, the momentum
i.e., 2T=(m+M)(g+a) .... (Hi) transmined by it to the table,
Here m= 100 kg, M =25 kg, R =A =450N dp = (dm)v = (MIL)dy~2gy
andg=10mls 2 so that, F, = dp = M dy ~2gy
So, from Eqns. (ii) and (HO, we have dt L dt

dt =v=~2gyl
450+T=100(10+a) and 2T = (100 +25)(10 +a) M
or F, =-2gy [as dy .... (,.,.,.)
L
i.e., T=550+1000 and 2T=125(10+a)
which on solution yields Substituting FJ and F2 from Eqns. (ii) and (iii) in (i), we get

a=2m1s 2 and T=F=750N[asfromEqn.(i)F=T] F =3{ ~ yg} = 3 [weight of chain on the table]


Note: (i) The equation of motion for the crate will be
T - A - Mg _ Ma,I.e.,T -R '" M(g+ a)[as A =-RJ .... (iv) Note: Here as y _ 4gt2 ,F=3(Mg 2/2J2L) , i.e., force is time dependent
and ifwe add Eqns. (ii) and (iv), weget2T-(m+ M)(g+a),
and increases non· linearly with time.
t.e., the equation of motion of system (iii) which consists of
painter and craie. Problem 52. One end (say B) ofa massless spring having
(ii) Free body diagram is: force constant k ;s altached to a block ly ing on a smooth
Point B Painter Crate System
suiface while the other end A is pulled by an external force. At

Ma~_of: ,a Rlf: la A~f' la ltT


rn+M ta
some instant the velocities ofends A and B ofthe spring are v A
and v B respectively. /fthe energy ofthe spring is increasing at
the rate olP Jls, find the instantaneous tension mid stretch;n

mg M, +
(m+M}g
the spring.
Solution: Let I be the
Fig. 3.86
natural length of the spring
(iii) From Eqns. (ii) 2nd (iii), R - {m - M)(g + ayZ. so the problem and XA and XB be the
is physically possible only if Fig. 3.88
position of ends A and B at
R:i!:O i.e. , m:i!:M time t; then stretched
length of the spring will be
Problem 51. A very flexible
uniform chain of mass M and length L is
suspended vertically, so that its lower and so the stretch,
end just touches the surface of a table. x=I'-I=(XA.-XB-I )
When the upper end of the chain is and so, F =Ia = k(xA. -xB-1) .... (i)
released, itfalls with each link coming to
rest the instant it strikes the table. Find and 1 , =-k(XA
U=-Ia 1 -XB- I)' .... (i i)
the force exerted by the chain on the 2 2
table at the moment when y part of the ._.iirlii:: Differentiating Eqn. (ii) with respect to time,
chain has already rested on the table.
Solution: In this problem, force on
Fig. 3.87 -dU =k (xA-xB-1) -A - -
dt dt dt
(dx dx
B ) ....(iii)

the table will be due to the weight oflhe chain on the table (say
F J ) and also due to the momentum of chain being transmitted But as dU
- =P --""VAA dx and dx B = VB (given)
dt • dt dt
at each instant when the chain strikes it (say F2 ), so that
So Eqn. (iii) in the light of(i) reduces to
F = F,+F, .... (i)
Now as mass per unit length of the chain is (MIL), so, P=F(VA -VB)
when y length of the chain is on the table, SO, Tension F= P
F, =( ~ y)gx ....(ii)
and Stretch x =F
(VA -VB)

= P
k k(VA -VB)
Problem 53. A small block starts sUding down an Problem 54. A uniform cylinder ofheight h and radius r
inclined plan,e subtending an angle €I with the horizontal. The is placed with its circular face on a rough inclined plane and
coefficient offriction bet}Veen the block and plafle depends on the inclination of the plane to the horizontal is gradually
the distance covered from rest along the plane as ).l=Mox increased. If)l is the coefficient offriction, then under what
where Ilo is a constant. Find (a) the distance covered by the conditions will the cylinder (a)sJide before toppling (b) topple
block down the plane till it stops sliding and (b ) its maximum before sliding?
velocity during this journey. . Solution: Consider the
Solution: If a be the cylinder on inclined plane as shown
acceleration of the block sliding in Fig. 3:90; then .
down the plane when it is at a
distance x from the starting point,
, i.e.,
(A) it will slide ifmgsin €I >}.tR
mgsin8>}.tmgcos8
the equation of motion will be
[as R = mgcos8] Fig. 3.90
ma = mgsin9 -)lR
i.e., tane>1l .... (a)
But R =mgcos9andfl = ).loX
(B) it will topple if
so, a = gsin€l - Moxgcos9 .... (i)
Now as' a'= dv "" dv. dx = v dv (mgSina)~> mgcosa x r (replacing €I by a)
dt dx dt dx
or tana '2:. (2r1h) .... (b)
so, Eqo. (i) becomes
So, from Eqns. (a) and (b), it is clear that:
v: = gsin9 -lloxgcos9 (a) The cylinder will slide before toppling if
8< 0.
or vdv = (gsin 9 -lloxgcos9)dx
i.e., tane< tan a or ).l <- (2rlh)
The above equa!ion on integration yields
(b) The cylinder will topple before sliding if
v2 . I 2
T= gx sm9 - z-1l0x gcose+A 0.<8
i. e., tane>tana or )l>(2r1h)
Now as atx ; O,v = O, so A =0;
so v 2 = 2gx sin €I -1l0x2gcose .... (ii) Note: For a cube ' h = (a) andr'" (a/2)
so, for sliding before toppling 1.1 < 1 i.e., e < 45°
From Eqn. (ii) it is clear that: and for toppling before sliding 1.1 > 1 i.e. , 8> 45°
(a) v will be zero when
Problem 55. Block A of mass m and block B ofmass 2 m
2gx:sine - ).lox 2gcose =0
are placed on afixed triangular wedge by means ofa massless,
i. e., gx(2sine - lloxcos8) =0 as
inextensibfe string and a frictionless puliey shown In Fig.
i.e. , x ='Oorx =2tan8/flo .... (iii) 3.91. The wedge is inclined
(b) v will be maximum when at 45° to the horizontal on
both sides. The coefficient of
(dvldx) =0 friction between block A and
But from Eqn. (ii), i.e., the wedge is 2/3 and that ..........._ _ _ _....
2 . 2 I
v =2gxsin8 - ).lox gcose between block B and the Fig. 3.91
wedge is 113. If the system of
i. e., 2V(:) = 2gSin8 - 2~OXgCOSe A and B is released from rest, find:
(i) Th'e acceleration ofA
For (dv/dx) = 0, it yields x ::: tan e/fl 0 .:.. (iv) (ii) Tension in the string, and
Substituting this value of x in Eqn. (ii), we get (iii) The magnitude and direction of the force of friction
v~x := 2gsin €I _1_ tan €I _ )1ogcose _1_ tan 28 acting on A.
)10 Il~ Solution: In this problem as (mB - mA )gsin9 (= mg/..fi)
which on simplification gives is lesser than ()l BmB + ~A mA )gcos9 (= 4mg/3..fi)the masses
will not move and hence,
v max =sine r---g .... (v)
V~ (i) Acceleration of B = acc. of A = 0
(ii) For equilibrium of B 3 Rooo
or t=---
4 ~g
2mgsinS =T + ~B2mgcos9 Solving problems involving variable forces
i.e., 45'
In some situations force is not constant. In such situations

r<¥[l-i]= ;~ fig . 3.92 acceleration may be function of time, displacement and


velocity.
(iii) As for mass A, T =(4mg/3..fi.) up the plane and 1. The Acceleration is a given function oft, i.e., a=/(1).
mg sin e =(mgIJi) down the plane, for its equilibrium, friction We write dv=adt;
4mg mg mg or du=/(t)dt
of value f = ~ - ~ = ~ must act down the plane.
3,,2 ,,2 3,,2 Integrating both sides, we obtain the equation
Idu=I/(t)dt
Note: ForbodyA,/L =1..1 mgcos9= 2m~ > f
3,,2 which defines v in terms of t.
i.e., here the friction is lesser than limiting friction and hence the Replacing the indefinite integrals by definite integrals
system will not move. with lower limits corresponding to the initial conditions t =0
and v=vo and upper limits corresponding to r=to and v = v,
Problem 56. A uniform disc ofradius R is spun about its
we write
axis to an angular velocity 000 and then carefully placed on a
rough horizon/al surface of coefficient offriction ~. Find the IV du = I/(t)dt
v,
time taken by the disc to stop rotation, if it is placed with its
plane on the table. or V - Vo =1' /(t)dt
Solution: When the disc spins "
which yields v in terms of t.
with its plane on the table as shown
We shall now solve the above equation for dx:
in Fig. 3.93, to calculate frictional
torque consider the disc to be made dx =vdt
of large number of concentric rings. and substitute for v the expression just obtained.
The mass of a ring of radius rand Both sides are then integrated, the left hand side with
thickness dr wi ll be respect to x from x =0 to x =xo. and the right hand side with
Fig. 3.93
dM = (MhtR')(2rrrdr) respect to t from I =0 to ~ = 10'
The position coordinatex is thus obtained in teons oft; the
so that force of friction on the ring will be
motion is completely determined.
d/ =~(dM)g = ~g (2rrrdr) 2. The Acceleration is a given function ofx,i.e., a=f{x).

and so, frictional torque on the ring


nR 2
Rearranging equations a ~~ =v:
=

d, =r(df) =2~Mg r'dr and substitutingf(x) for a, we write


R' vdv=adx
So, the total frictional torque on the disc, vdu= /(x)dx
't=J R 2}JMg r2dr=~IJ.MgR Since each side contains only one variable, we integrate
o R2 3 the equation. Denoting the initial values of the velocity and of
so 10. =j IJ.MgR [as l' =In J the position coordinate by Vo and xo. we obtain,
IV vdu=I/ (x)dx
v,
! MR2(_ dOl)=~~gR[asl =! MR2 andu =_dfiJ]
or
2 dt3 2 dt ~v' -H =I/(x)dx

or I, dt =- -3R
- I
o
dOl which yields v in teans of x. We now solve the above equation
° 4 ",g 6)0 for dt:
<Iv <Iv
dl::: -<Ix f(v)= - or f(v)=v-
V dt <Ix
and substitute for v the expression just obtained. Both sides <Iv du
dt= - - and dx=v--
may be integrated to obtain the desired relation between x f(v) f(v)
and!. Integration of the first equation will yield a relation
3. The Acceleration is a given function ofv, i.e" a =f(v). between v and t; integration of the second equation will yield a
We may then substitute f(v) for a either in a ::: dv ld! or relation between v and x. Either of these relations may be used
a =vdvldx to obtain either of the following relations: to obtain the relation between x and t which characterises the
motion of the particle.
EXERCISE
(A] Only One Choice is Correct to the rope at the mid point which now no longer remains
horizontal. The minimum tension required to completely
I. When a body is stationary:
straighten the rope is:
(a) There is no force acting on it
(.) 15 kg (b) 15/2kg
(b) The forces acting on it are not in contact with it
(c) 5 kg (d) Infinitely large
(e) The combination of forces acting on it balance each [Hint: See § 3.3(5) (d)]
other
10. A boy of mass 40 kg is hanging from the horizontal branch
(d) The body is in vacuum
of a tree. The tension in his arms is minimum when the
2. A particle is in straight line motion with uniform velocity. angle between the aons is:
A force is not required: (.) 00 (b) 9QO
(a) To increase the speed 0
(c) 120 (d)
(b) To decrease the speed [Hlnl : See § 3.3(5) (e))
(e) To keep the same speed J I. A block of mass M is pulled along a horizontal frictionless
Cd) To change the direction surface by a rope of mass m If a force F is applied at one
3. Essential characteristic of equilibrium is : end ofthe rope, the force which the rope exerts on the block
(a) Momentum equals zero is: leBSE 1990)
(b) Acceleration equals zero (.) FI(M+m) (b) F
(e) K.E. equals zero (c) FM I (m+M) (d) Zero
(d) Velocity equals zero 12. A string of length Land mass M is lying on a horizontal table.
A force F is applied at one of its ends. Tension in the string at
4. When a constant force is applied to a body, it moves with
a distance x from the end at which force is applied is:
unifonn:
(.) Zero (b) F
(a) Acceleration (b) Velocity
(c) F(L-x)1 L (d) F(L-x) 1M
(e) Speed (d) Momentum
13. A chain of mass M and length Lis held vertical by fixing its
5. An object will continue accelerating until:
upper end to a rigid support. The tension in the chain at a
(a) Resultant force on it begins to decrease distance y from the rigid support is:
(b) Its velocity changes direction (.) mg (b) Mg(L- y)1L
(c) The resultant force on it is zero (c) MgL/ (L - y) , (d) Mgyl L
(d) The resultant force is at right angles to its direction of 14. Find the tension T2 in the system shown
motion in Fig. 3.94: T,
"g
6. When a force of constant magnitude always acts (.) Ig N T,
2 kg
perpendicular to the motion of a particle, then: T,
(b) 2g N
(a) Velocity is constant (b) Acceleration is constant 3 kg
(c) 5g N Fig. 3.94
(c) K.E. is constant (d) None of these
(d) 6g N
7. A body of mass 2 kg moving on a horizontal surface with an
15. Three blocks are connected as shown in the Fig. 3.95 on a
initial velocity of 4 mls comes to rest after 2 sec. If onc
horizontal frictionl ess table. If m1 = I kg, m2 = 8 kg,
wanls to keep this body moving on thc same surface with a
m) = 27kg and T3 =36N,T will be:
velocity of 4 mis, the force required is: 2

(.) 8 N (b) 4 N
(c) Z"o (d) 2 N
8. Two blocks of masses 2 kg and 1 kg are in contact with each Fig. 3.95
other on a frictionless table. When a horizontal force of3.0
(.) 18 N (b) 9N
N is applied to the block of mass 2 kg, the value ofthe force
of contact between the two blocks is: (c) 3,375 N (d) 1.75 N
(.) 4 N (b) 3 N 16. Two bodies of masses 5 kg and 4 kg arc arranged in two
positions as shown in Fig. 3.96 (a) and (b), if the pulleys
(c) 2 N (d) I N
and the table are perfectly smooth, the accelerations of the 5
9. Two pcrsons are holding a rope of negligible weight tightly kg body in case (a) and (b) are:
at its ends so that it is horizontal. A 15 kg weight is attached
the other end at the same time. The reading of the balanct
will be:
(a) 2000 N (b) zero
(e) 1000 N (d) 500 N
22. A spring obeying Hooke's law has a force constant k. NO\1-
(a) if the spring is cut in two equal parts, the force constant o·
(0)
each part is:
Fig. 3.96
(a) k (b) kl2
(a) g and (5/9) g (b) (4/9)g and (1/9) g (c) 2k (d) zero
(e) glS and glS (d) (5/9) g and (1/9) g 23. A spring of force constantk is cut into two pieces such tha
t 7. If x, F and U denote the displacement, force acting on and one piece is double the length of the other. Then the lonl
potential energy of a particle: piece will have a force constant of: [lIT 1999
dU (a) (2/3) k (h) (3/2) k
(a) U c F (b) F =+-
dx (e) 3k (d) 6k .
(e) F=- -
dU
dx
(d) F =~('~) 24. A spring obeys Hooke's law. When loaded with 12 g it
extension is 2 cm. Which ofthe following will produce a:
18. Three equal weights of mass 2 kg each -"I!!!~!!!I'­ cmextensionifg""10m/s2 ?
are hanging on a string passing over a (a) 12 x(3/ 2)xg (b) 12x(2/3)xg
fixed pulley as shown in Fig. 3.97. What
(e) A force of 1.8 N (d) A force of 0.12 N
is the tension in the string connecting B
weights Band C! 25. T wo bodies A and B
each of mass Mare
(a) Zero
c fixed together by a
(b) 13 N Fig. 3.97 massless spring. A Fig. 3.99
(e) 3.3 N force F acts on the
(d) 19.6 N mass 8 as shown in Fig. 3.99. At the instant shown the mas
19. Two blocks each of mass M are resting on a frictionless A has acceleration Q. What is the acceleration of mass B?
inclined plane as shown in Fig. 3.98. Then: [CPMT 1993
(a) (F I M) - a (h) a
(e) - a (d) (FIM)
26. A block of mass 2 kg is placed on the floor. The coefficier
of static friction is 004. Ifa force of2.8 N is applied on th
. block parallel to floor, the force of friction between th
Fig. 3.98 block and floor (taking g = 10 m/s 2) is: (CPMT 1993

(a) The block A moves down the plane


(a) 2.8 N (b) 8 N
(c) 2 N (d) zero
(b) The block B moves down the plane
27. A body of mass 2 kg rests on a rough inclined plane ruakin
(e) Both the blocks remain at rest
an angle 300 with the horizontal. The coefficient of stati
(d) Both the blocks move down the plane friction between the block and the plane is 0.7. Th
20. A small sphere is suspended by a string from the ceiling ofa frictiona l force on the block is:
car. lflhecar begins to move with a constant acceleration a. (a) 9.8 N (b) 0.7 x 9.8 x.J3 N
the inclination of the string to the vertical is:
(e) 9.8 x .J3 N (d) 0.7 x 9.8 N
(a) tan - \ (alg lin the direction of motion
[Hint: If the body is at rest, friction = applied force and not pJi
(b) tan -I (alg) opposite to the direction of motion 28. A block has been placed on an inclined plane. The slop
(c) tan -I (gla)in the direction of motion e
angle of the plane is such that the block slides down th
plane at a constant speed. The coefficient of kinetic frictio
(d) tan - I (gla) opposite to the direction of motion
is equal to: [CBSE 1993
21. A stretching force of 1000 newton is applied at one end of a (a) sine (b) cose
spring balance and an equal stretching force is applied at (e) g (d) tan e
29. A body of mass 60 kg is dragged withjusl enough force to Ca) 2.S N Cb) 0.98 N
start moving on a rough surface with coefficients of static (e) 4.9 N (d) 0.49 N
and kinetic frictions 0.5 and 0.4 respectively. On applying 35. A lineman of mass 60 kg is holding a vertical pole. The
the same force. what is the acceleration? IEAMCET 1993) coefficient of static friction between his hands and the pole
Ca) 0.98 mis' (b) 9.8 mis ' is 0.5. If he is able to climb up the pole, what is the
(e) 0.S4m1s ' (d) S. 292 m1s' minimum force with which he should press the pole with
[Hint: F - f = mQwith F = fL - J.1 pg and f .. Ii =J.1.tmg) his hands? (g=1O m /s 2) ,

30. Pushing force making an angle eto the horizontal is Ca) 1200 N (b) 600 N

*
applied on a block of weight W placed on a horizontal table.
If the angle of friction is the magnitude of force required
to move the body is equal to:
Ce) 300 N .. Cd) ISO N
36. In a cricket match the fielder draws his hands backward
after receiving t~e ball in c;>rder to take a catch because:
Ca) Weos~/eosce-~) Cb) Wsi n~ /eos ce+~) (a) His hands will be saved fr.om getting hurt
Ce) Wtan~/sin(e-~) Cd) Wsin e/g'an ce- ~) (b) He deceives the playe.r ,
[Hint: Substitute in Fcos9= j.tR ,R - (W + Psin 9)andJ.l - t8n,] (c) It is a fashion
31. A block A of mass 2 kg rests on another block B of mass 8 (d) He catches the balJ finnly
kg which rests on a horizontal floor. The coefficient of 37. A ball weighing 10 g hits a hard surface vertically with a
friction between A and B is 0.2 while that between Band speed of 5 mls and rebounds with the same speed. The ball
floor is 0.5. When a horizontal force of 25 N is applied on remains in contact with thesurface for 0.01 sec. The
the block B, the force of fricti on between A and B is : average force exerted by the surface on the ball is:
[lIT 1993[ (Roorkee 19931
(a) zero Cb) 3.9 N (a) 100 N Cb) ION
Ce) 5.0 N (d) 49 N (e) I N (d) 0.1 N
[Hint: Friction between A and., B will act only if F > fB with 38. In a legend the hero kicked a baby pig so that he is projected
fB " f.1B(m A +mB)gJ with a speed greater than that of his cry. If the weight of the
lOON baby pig is assumed to De 5 kg and the time of contact 0.0 I
32. A 40 kg slab rests on a 10 kg Block
frictionless floor. A 10 sec, the force with which the hero kicked'him was:
kg block rests on top of 40 kg Slab Ca) Sx 10-' N (b) 2'x 10' N
the slab (Fig. 3. 100). _ L_ _ _ _ _ _L _
The slatic coefficient of
Fig. 3.100 (c) 1.65x 10 N5
(d) 1.6Sx 103 N
fricti on between the block and the slab is 0.60 while the 39. A force--time graph for the motion ofa body is shown in Fig.
kinetic coefficient is 0.40. The 10 kg block is acted upon by 3.102. Change in linear momentum between 0 and 8 sis:
a horizontal force of 100 N. If g = 9.8 m/s 2 the resulting
acceleration of the slab wiil be:
(b) 1.47 In /s 2
(a) 0.98 m/s 2
(c) 1.52 m/s2 (d) 6.1 m /s2
I
F
33. A mass m rests on a
t (in s)
horiwntal surface. The
coefficient of friction Fig. 3.102
between the mass and the
(a) zero (b) 4 N-s
surface is J.l. If the mass is Fig. 3.101
pulled by a force F as shown in Fig. 3.101, the limiting Ce) 8 N-s . (d) none of these
fricti on between the mass and the surface will be: 40. Torque is the cause of:
(a) ~"'g (b) ~[",g - (J3 / 2)FJ (a) Translatory motion
(e) ~[mg -(FI2)J Cd) ~ [mg +.(FI2)J (b) Rotatory motion
34. A block of mass 0.1 kg is held against a wall by applying a (c) Oscillatory motion
horizontal force of 5 N on the block. If the coefficient of (d) Combined translatory and rotatory motion
friction between the block and the wall is 0.5, the 41 . The dimensions of torque are:
magnitude of the frictional force acting on the block is:
Ca) [MLr' J Cb) [ML'r'J
[liT 1994J
Ce) [ML'T-'J (d) [MLr'J
42. When a steady torque or couple acts on a body, the body: (a) is at rest
(a) continues in a state of rest or of uniform motion by (b) experiences a torque
Newton's first law (c) experiences a linear motion
(b) gets linear acceleration by Newton's II law (d) experiences a torque and also a linear motion
(e) continues to rotate at a steady rate 50. A weightless rod is acted on by upward parallel forces of2
(d) gets an angular acceleration Nand 4 N at ends A and B respectively. The total length oj
43. In case of torque if the axis is changed in terms of position the rod AB = 3 m. To keep the rod in equilibrium a force oj
only, torque will: 6 N should act in the following manner: IMNR 1992]
(a) increase (b) decrease (a) downwards at any point between A and B
(e) remain constant (d) none of these (b) downwards at the mid point of AB
44. It becomes easier to open or close a door turning about its (c) downwards at a point C such that AC = 1m
hinges if the force is applied at: (d) downwards at a point D such that BD = 1m
(a) Two-thirds of the door 51. According to the Kepler's laws a planet moves in an elliptk
(b) The free edge of the door orbit with the sun at a focus ofthe ellipse. Taking the plane!
(e) The middle of the door and the sun to be point masses, the torque on the planet witt
(d) The point near the hinges respect to sun: lIlT 19931
45 . A small magnet is placed perpendicular to a constant (a) is zero at all times
magnetic field. The forces acting on the magnet will result (b) varies with time
in: (c) can be non-zero but constant in time
(a) Rotation
(d) cannot be defined for an elliptical orbit
(b) Translation [Hint: As force is central 't = 01
(e) Rotation as well as translation
52. A constant torque acting on a unifonn circular whee'
(d) No motion at all changes its angular momentum from Ao to 4Ao in 4 sec
46. A false balance has equal anns. An object weighs WI when The magnitude of this torque is:
placed in one pan and W2 when placed in the other pan. The (,) 4Ao (b) Ao
true weight W of the object is: IAFMC19941
(e) 3Ao 14 (d) 12A o
(a) (WI x W2 )112 2
(b) (W1 +Wi)1I2
53. Which of the following is non-conservative force?
(e) (WI + W, )/2 (d) 2WIW,I(WI+W,) (a) Electrostatic force (b) Gravitational force
47. The beam and pans of a balance have negligible mass. An (c) Viscous force (d) Interatomic force
object weighs WI when placed in one pan and W2 when
54. A rocket is fired from the earth's surface to put the pay loa(
placed in the other pan. The weight W ofthe object is:
in the required orbit. The motion of the rocket is given by
(a) ~WIW, (b) ~(WI +W,)

(c)
, ,
WI +W2 (d) (WI-I +Wi' )/2
(a) F=m(:') (b) F=:'
48. A body weighs 8 g when placed in one pan and 18 g when
placed on the other pan of a false balance. If the beam is
(c) F =v( ~7) (d) F = constant

horizontal when both the pans are empty, the true weight of 55. A box weighing 20 kg is pushed along the floor at ,
the body is: constant speed by applying a horizontal force. If th!
(a) 13g (b) 12g (e) 15.5g (d) 15g coefficient of friction is 0.25, then force applied is:
49. Four equal and parallel forces are acting on a rod (as shown (a) 5 N (b) 10 N
in Fig. 3.103) at distances of20 cm, 40 cm, 60 cm and 80
(e) SON (d) 200N
cm respectively from one end of the rod. Under the
influence of these forces the rod: 56. An elevator starts from rest with a constant upwaf(
F F acceleration. It moves 2 m in the first 0.6 second. J
passenger in the elevator is holding a 3 kg package by :
o 20 140 60 180 vertical string. When the elevator is moving, what is th'
c;:gl~l~~~f~~ tension in the string?
(a) 4 N (b) 62.7 N
F F
Fig. 3.103 (e) 29.4 N (d) 20.6 N
57. A unjform thick string of length 5 m is resting on a 62. A block of mass 20 kg is suspended through
horizontal frictionless surface. It is pulled by a horizontal two light spring balances as shown in Fig.
force of 5 N from one end. The tension in the string at 1 m 3.105; then:
from the force applied is: (a) Both the scales show 10 kg reading
(a) zero (b) 5 N (b) Both the scales show 20 kg reading
(c) 4 N (d) 1 N (e) The upper scale will read 20 kg while the
58. A body of mass M rests on a horizontal plane having lower zero
~

coefficient of friction .... At t = <\ a horizontal force F is (d) Their readings are in between 0 and 20 kg
~ ~ ~
with their sum equal to 20 kg
Flg.3.10G
applied that varies with time F(t)=Fo·r. where Fo is a
constant vector. The time instant to at which motion starts 63. The line ofaction of the resultant of two like parallel forces
and distance moved in t seconds will be: shifts by one-fourth of the distance between the forces
Mg Fo 2 when the two forces are interchanged. The ratio of the two
(a) - and -(1-1 0 ) forces is:
»Fo M
f.l,Mg 1 Fo 3
(h) - and - - ( l - I o )
Fo 6M
64. A block takes twice as much time to slide down a 45° rough
(c) J.1Fo and FOU_tO)2 inclined plane as it takes to slide down a similar smooth
Mg M plane. The coefficient of friction is:
F · F (a) 3 (h) J3
(d) ~ and _ 0 (1_1 )'
Mg 2Mg 0 4 2
59. In the adjoining diagram, the ball A is (c) (d) ~
released from rest when the spring is at its 4 3
natural length (neither stretcbed nor 65. A block of mass 2 kg rests on a rough horizontal plank, the
compressed). For the block B of mass M coefficient of friction between the plank and block is 0.3. If
to leave contact with the ground at some the plank is pulled horizontally with a constant acceleration
time, the minimum mass of A must be: of 4 m I s 2 • the distance moved by the block on the plank in
M 5 second slarting from rest (in m) is: (take g = 10 m I s 2)
(a) -
2
(a) 5 (b) 10
(b) M
(c) 25 (d) 50
(c) 2M
Fig. 3.104 66. If a particle is compelled to move on a given smooth plane
(d) a function of M and force constant k ~ ~

of spring curve under the action of given forces in the plane F = x i


~
60. A closed compartment containing gas is moving with some + Y j, then:
acceleration in the horizontal direction. Neglect the effect ~ ~ ... I 2
-t
of gravity. Then the pressure in the compartment is:
(liT 1999(
(a) F·d r =xdt'+ ydy (b)
sF.dr.;e'21nV
(d) ~ml)2 J(xdt'+ y4y)
~ ~
(a) same everywhere (c) F·d r i/:xdt+ ydy "'"
(b) lower in lhe front side
67. A rough vertical board has an ~
(e) lower in the rear side
acceleration' a' so that a 2 kg
(d) lower in the upper side block pressing against it does
61. A block of mass m is p laced on a smooth wedge of wedge not fall. The coefficient of
angle e.
The whole system is accelerated horizontally so friction between the block
that the block does not slip on the wedge. The force (nannal and the board should be:
reaction) exerted by the wedge on the block has a (Pb PMT 2007(
magnitude:
(a) >g/a
(a) mg tan e (h) mg
cos 8
(b) <g / a
Fig. 3.106
(c) =g / a
(c) mg (d) mgcos8
Cd) >alg
68. Ifa man ofmassMjumps to the ground from a height hand 75. In a conservative field, at stable equilibrium, potential
his centre of mass moves a distance x in the time taken by ~nergy is:
him to hit the ground. the average force acting on him is: (a) maximum (b) minimum
(a) Mgh (b) Mgx (e) constant Cd) maximum or minimum
x h 76. Two masses m, and m2 are connected to the ends of a

(c) Mg(~)' (d) Mg(i)' massless string that passes elVer a pulley fixed at the t.op ofa
double incline as shown. Assuming m, > m2 , acceleration
of the system is:
69. A bullet moving with a speed 'v' can just penetrate two
planks of equal thickness. Number of such planks that the
bullet will penetrate if its speed is doubled, is :
(a) 8 (b) 6 (c) 9 . (d) 4
70. In the system shown in Fig. 3.107, the mass 30 kg is pulled 0, e,
by a force 210 N. At the instant when the 15 kg mass has
Fig. 3.11 0
acceleration 6 mt s 2• the acceleration of30 kg mass will be:
(Assume the spring to be massless) mlgsin 9 1 + m2gsin6 2
(a)
~~ ~N ml +m2
m,gsin 9, -m 2gsin 9 2
15 kg 30 kg (b)
Fig. 3.107 m,-m2
In,gsin 9, -rn 2gsin 9 2
(a) 2m/5 2 (b) 3m/ 52 (c)
m, +m2
(e) 3.4 m / s 2 (d) 4 mi s'
7 1. In the system shown in Fig. 3. 107, when both masses are (m, +m2 )(sin 9, +sin ( 2 )g
(d)
moving with the same acceleration, the extension in the m j -1II 2
spring, if its spring constant is 100 N/m, is:
77. Length ofa spring is I, when it is loaded by 5N. Its length is
(a) 40 em (b) 50 cm (c) 70 em Cd) 62 cm 12 when loaded by 7N. Its length, when loaded by 9N, will
72. In the system shown in Fig. 3.108, the N be:
pulley is frictionless and the string U (a) 21, + 212 (b) 21, +'2
massless. If In,
== m2 , thrust on the
(c) 211 -'I (d) 21, -/2
pulley will be :
78. In Fig. 3. 111 (a), mass mis lifted up by anaching a mass 2m
(a) less than (m , + m2 )g m, to the other end of the string while in Fig. 3.111 (b), m is
(b) greaterthan(m, +m2 )g lifted by pulling the other end of the string with a constant
(e) equal to (m, + m2 )g m2 force F = 2 mg, then: ' .. ,
(d) none of the above Fig. 3.108
73. In Q. 72, if Inl == m2 , thrust on the pulley is :
(a) less than (m , +m2 )g
(b) greater than (m! +m2 )g
(c) equal to (m, + m2 )g m m
(d) none of the above
2m 2mg
74. In Fig. 3.109, tension in the string
that connects the masses A and B is Ca) Cbl
T! and that in the string connecting Fig. 3.111
T
Band C is f2' then...!. is : (a) acceleration of III in the two situations is same and
T, non-zero
A
(a) 2 (b) acceleration ofm in situation (a) is more than that of in
(b) 0.5 B s ituation (b)
(c) 4 (c) acceleration of m in situation (a) is less than that of in

(d) 0.25
c situation (b)
Fig. 3.1 09 (d) acceleration of 111 in the two situations is same and
equals to zero
79. Potential energy of a body of mass' m' in a conservative 85. A person wishes to slide down a rope whose breaking load
is ~ Qf the weight of the person. Minimum acceleration by
force field can be expressedU:::z <xx-~yas wherexand yare
position coordinates of the body. Acceleration of the body
can be expressed as : which the person should slide down without breaking the

[ 'm~~ ']'
0;2 _ ~l rope is:
(a) (b) (l (.) 0,8 g (b) 1.2 g (0) 0,6 g (d) 0.4 g
m
86. A gun is mounted on a vehicle which is at rest on a
a-p (d) a+p frictionless road. Mass ofvehic1e+ gun is 1600 kg. The gun
(0)
m m fires 20 bullets per sec, each with a velocity 400 mls. Mass
80. A 30 kg body is pulled by a rope 4 m long on a frictionless of each bullet is 20 gm. Acceleration produced in the
horizontal surface by a force 96 N. If mass of the rope is 2 vehicle is:
kg, force acting on the body is: (a) 12cm/s2 (b) '14cm/s 2
SO kg
(c) 8cm/s2 (d) JOcm/s2
87. An open knife edge of mass M is dropped from a height' h'
on a wooden floor. lethe blade pcnetrates a distance'S' into
Fig. 3.112 the wood, average resistance offered by the wood to the
blade is:
(a) 96 N (b) 94 N
(0) 92 N (d) 90 N (a) Mg (b) M~I+*)
81. A block of mass' m' is placed on a frictionless inclined
plane of inclination ewith horizontal. The inclined plane is
accelerated horizontally so that the block does not slide
(d) M~I+;)'
down. In this situation, vertical force exerted by the 88. Referring to Fig. 3.11 4 (a) and (b),
inclined plane on the block is: (CBSE (PMT) 2004} -.p-
(a) mgsina (b) mgcose
(c) mg (d) none of these Spring balance
82. In Q. 81, if the inclined plane is not accelerating and the
block is then sliding down the plane, force exerted by the
block on the plane is:
(a) mgtana (b) mgsina
(c) mg (d) none ofthese
83. A block released from rest from the top of a smooth 2kg 4 kg
inclined plane of inclination 45° takes time' t' to reach the 4 kg
6 kg
bottom. The same block released from rest, from top of a
rough inclined plane of same inclination, takes time '2t' to (0) Ib)
reach tI.e bottom, coeflic1.cnt of friction is: Ag. 3.114
(PMT (Ke r~l a) 2006) (a) reading of spring balance in (a) is 8 gand in (b) is also 8 g
(a) 0,75 (b) 0,5 . (b) reading of spring balance in (a) is 8 g and in (b), it is
(0) 0.25 (d) 0.4 less than 8 g
84. On the floorofan elevator, a block of (c) reading in (a) is less than 8 g and in (b), it is 8 g
mass 50 kg is placed on which (d) reading in both (a) and (b) 1s less than 8 g

ur
another block of mass 20 kg is also 89. Consider a system of

il~:'J I
placed. The elevator is moving up masses M, and M,
with a constant acceleration 1.5 M 2 (M 1 >M 2 ) IJ i I
ml s 2. Force exerted by 20 kg block 1 connected by a (0)
@J

on the 50 kg block is nearly: massless string. In

~
(a) 250 N Fig. 3.113 Fig. 3,115 (a), the
(b) 230 N
(0) 170N
system is pulled by a
force F from the side
of mass M I and in (b)
i
M,
J
(d) 150 N Fig. 3. 115 (b), the Fig. 3.115
system ispulled by the same force F from the side of mass 95. In Fig. 3.117, spring constant of the
M 2• then: spring is 100 N/m. Extension
(a) acceleration in the two cases is same and tension in the" produced in the spring i~:
string connecting M 1 and M 2 is also same (g"lOm/s')
(b) acceleration of system in (a) is more than in (b) while (a) 10 cm
tension in the string is same in both cases
(b) 20 em
(e) acceleration of system in both cases is same but tension
(c) 30 cm
in the string in (a) is more than in (b) 4 kg
(d) 40 em
(d) acceleration of system in both cases is same but tension Fig. 3.117
in the string in (a) is less than in (b)
90. The horizontal acceleration that should be given to a 96, Coefficient of static friction between a block of mass M and
an inclined surface is 0.8. The inclined surface is inclined to
smooth in~1ined p~ane of angle sin - 1( 1)to keep an object the horizontal at 30°. Force of friction acting on the block
is:
stationary on the plane relative to the plane is:
(a) 0.68 mg (b) 0.50 mg
IUPCPMT 20061
(e) 0.74 mg (d) 0.40 mg
(a) - g - (b) g~ ' 97. A block of mass 6 kg is
~/' -1 being pulled by force 24 '" F=24N

(e) N-I N as shown. If coefficient IJi!lL I


(d) - g- of friction between the Fig. 3.118
g ~/' +1 block and the surface is 0.6, frictional force acting on the
91. A train. having 60 compartments is pulled by engine with a block is:
4
force 6x 10 N. If mass of each c.ompartment is 4000 kg, (a) 36N (b) 32N
the . 'tension in the coupling between 40th and 41st
compartlpentJs:
., (e) 12 N (d) none of these
98. Coefficient of static friction between the two blocks in Fig.
(al 4x 104 N (b) 3x 104 N 3.119 is 1.1. and the table is frictionless. Maximum horizontal
(c) '1.5x 10'1 (d) 2x 104 N force which can be applied on the lower block such that the
blocks move together, is:
92. A monkey of mass 30 kg climbs a rope which can withstand
(a) Il mlg
I m, I
a maximum tension of 360 N . The maximum acceleration
which this rope can tolerate for the climbing of monkey is: (b) Il m2g
(g"IOm/s') ICBSE20031 (e) Il(m, -m 2 )g
Fig. 3.119
(d) Il(m, +m2 )g
(a) 2m/s2 (b) 3 m/s2
99. In Fig. 3.120, coefficient of friction
(c) 4m/s2 (d) 5 m/s2
between ml and m2 is Il and that
93. In Fig. 3.116, a sphere of mass 2 kg is suspended from the between ml and the wall is zero. A
ceiling of a car· which is initially at rest. Tension in the F m,
force F is pressing the system
string in this situation is T\ . The car now moves to the right against the wall.
with a unifonn acceleration and the tension in the string is Minimum value offorce required to
nowT2 • then.: (Takeg "" 10m/ s 2)' hold the system in equilibrium is:
(a) TI =T2 =20N (a) Ilmlg Fig. 3.120
(b) TI =20N,T2 >20N (b) Ilm2g
(c) T, =20N,T, < 20N Fig. 3.116 (c) Il(m j + ~12)g
(d) T! <20N,T2 = 20N (d) system cannot be held in equilibrium
94. In Q. 93, iLmass of the sphere be denoted as 'm' and the 100. If,in Q.99, coefficient of friction between m l and the wall is
a~('eleration of car be·' a', then T2 is given by: 1..1, and II and12 are, respectively, the force of friction on
(a) mg m l and m2 , then:
(b) m(g2 +a 2 F2 (a) fi and12 both are downward

(c) m(g2 ·_ a 2 F2
(b) fi and 12 both are upward
(c) fi is upward and 12 downward
(d) m(g+a) (d) fi is downward and 12 upward
101. In Fig, 3.121, coefficient of F ri 106. In Fig. 3.123, coefficient of
friction between the block and
" '
"'<c--i-,---, kinetic friction between the 4
the floor is 'I.l." Force F that ~'.,; M kg block and the inclined
will move the block on the • • • •
floor with a unifonn speed is:
-1..... surface is ~3' Here' m' is such
m

, j ""-"'3"'0"'---_ __
(a) (l.l.sin9+cos9)mg Fig. 3.121 a mass that the 4 kg block is Fig. 3.123
(b) (j.uin e-cos 9)mg moving up the plane with a
constant speed, then m is:
(e) _-:o~:cm",g--:-: (d) -;--f~::::m,,-g----:o
cos 9-l.l.sin 9 sin 9-l.l.cos 9 (a) 2 kg (b) 2.S kg
(e) 3.4 kg (d) 4 kg
102. In Q.IOI. the block can be made to slide on the floor only if
€I is more than a minimum value. Otherwise, the block can 107. A uniform rope of length I lies on a table. If the coefficient
not be made to slide by any value of F however large. of friction is ').1', then the maximum length I, of the part of
Minimum value of e for less than which sliding cannot this rope which can overhang from the edge of the table
happen, is: without sliding down is:

Sin-'(~ J
(a) I (b)
(a) ~ "+ I
(e) ~ .(d) ~
Ian-,( ~ 'J ~ +I ~ -I
(e)
w Cd) zero

103. As you walk on ice, you need to take smaller steps to avoid
lOS. Two balls of mass m, and m2 are separated from each other
by a powder charge placed between them. The whole
system is at rest on the ground. Suddenly the powder charge
slipping because: explodes and the ma~s are pushed apart. The mass m,
(a) it results in smaller friction travels a distance S 1 and stops. If the coefficients of friction
(b) it results in larger friction between the balls and the ground are same, the mass m2
stops after travelling the distance: (BY (Pune) 2~061
(c) it does not change friction but makes you more alert
m, m2
ahout slipping (a) S, =- S,
m2
(b) s, =-S,
m
(d) none of the above l
104. A block of mass M slides down a rough inclined surface of 1 1
m m
inclination • 9' and reaches the bottom at speed •v'.
However, if it slides down a smooth inclined surface of
(c) Sl = -
' SI
m1 , (d) Sl =---1..S 1
1
m ,
same length and same inclination, it reaches the bottom 109. In Fig. 3.124, coefficient of
A
with speed lev. Coefficient of friction between the block and static friction).1 $ between block
the rough incline is: A of mass 2 kg and the table is
0.2. Maximum mass of block B
(a) sin O(k' -I ) so that the two blocks do not
move, is: (g = lOm l 52)

(c)
[
k' +
~ cose
I) [DPMT2006[
Fig. 3.124

(a) 2.0 kg (b) 4.0 kg


105. In Fig. 3. 122, coefficient of friction bE:tween mz and the (e) 0.2 kg (d) 0.4 kg
horizontal swface is ' J.l ', 110. In Fig. 3.125, a cubical block is held
minimwn value of
so that the system has
m, r'-;;:1-----
m2
.stationary against a rough wall by
applying a force F, then the incorrect
zero acceleration, is: a
statement among the following is :
(a) ).t(rn, +m z ) F
[lIT 200S[ a
(b) ).t(rn, -m 2 ) m, (a) frictional force f = mg
(e) m1 - -
m, Fig. 3.122
(b) F = N ,N is nonnal reaction
Fig. 3.125
~ (e) F does not apply any torque
m,
(d) --m1
(d) N docs not apply any torque

"
Ill. A particle is placed at the origin and a forceF =Kxis acting
on it, where K is a positive constant. IfU(O) = 0, the graph
ofU(x) versus x will be: (where U is the potential energy
116 .. A bar of mass m1 is placed on a plank:
of mass m2 which rests on a smooth
horizontal plane. The coefficient of
m1
m2
& F
function) [liT 20M} friction between the surfaces of bar .
U(x) U(x)t and pla~ is k. The plank is su~jected Fig. 3.126
to a honzontal force F dependmg on
time t as F = at, where a is a constant. The moment of time
(a) -____,-+....--x (b) ---"k---x to at which the plank starts sliding is:
akg (h) (ml +m, )kg
(a) ~:":::-
m1 +m2 a

U(x) U(x) (e) .'-(m,..,I-;+:::-m.£.,)go:.


ka
117. A balloon of gross weight W newton descends with an
(e) -....:::.-J-'''----- x (d) --,fL-~ x acceleration f m/ s 2 . The weight that must be thrown out
in order to give the balloon an equal upward acceleration
will be:
112. A machine gun fires a bullet of mass 40 gm with a speed (a) Wj (b) 2Wj
1200 mls. The man holding it can exert a maximum force of g g
144 N on the gun. How many bullets can h.e fire per second (e) 2Wj (d) W(g+ j)
atthemost? [AIEEE2004) g+j j
(a) One (b) Four 118. A load W is to be raised by a rope from rest to rest through
(e) Two (d) Three a height h. The greatest tension the rope can safely bear is
113. The upper half of an inclined plane with inclination cjl is nW. The least time in which the ascent can be made will
perfectly smooth while the lower half is rough. A body be:
starting from rest at the top will again come to rest at the
bottom, if coefficient of friction for the lower half is given
by: (AIEEE 2005(
(a)
2h ]112
[(n l)g (b) f!
(a) 2 tan ~ (b) tan ~
(e)
. 2nh ]112 (d) F(n-I)h
(c) 2sincf! (d) 2eos~ [(n-l)g ng
114. Two particles of mass' ' m' each are tied at the ends of a 119. In the given Fig. 3.127, two masses 2 kg each are attached
light string of length '2a';. The whole system is kept on to a string which passes over a smooth massless pulley P.
frictionless horizontal surface with the strhig held tight so The surface PQ is smooth and LPQR = 45°. The surface PR
that each mass is at a distance' a' from the .centre P (as
shown in figure). Now, the midpoint of the string is pulled is rough, with coefficient of kinetic friction ~ and
vertically upwards with a small but constant force F. As a
result, the particles move towards each other on the L PRQ = 300. The acceleration of the system is:
p
surface. The magnitude of acceleration, when the
separation between them becomes 2xis: lIlT 2007J
(a) . F a (b) F x
2m ~a2 _x 2 2m ~a2 _x2

F ~a 2
F x 2
_x Fig. 3.127
(e) Cd) ~
1m. 2m x
g (,/2-1)
115. A particle moves in the x- y plane under the influence of a (a) along PR
~ ,
2,/2 .
force such that its linear momentum is ~(t) = A[i cos (kt)
(b) zero because the masses are equal
-j sin (kt)J, where A and k are constants. ' Angle between
the force"and the momentum is: [liT 2~071 (c) f alongPR
2
0
(a) 0 (b) 30 0
g( ,/2-1)
(e) 45 0 (d) 90 0 (d) alongPQ
2,/2
FORCE AND TORQUE 101,

120. Two steel balls A and Bare 12S. A string of negligible mass going over It
placed inside a right circular · clamped pulley of mass m supports a block
cylinder of diameter ' 54 em P 'A of mass M as shown in the Fig. 3.131. The
making contacts at points P, Q force on the pulley by the clamp is given
and R as shown. The radius R by: (liT 2001(
rA = 12cm and rB =1 8cm. The
(.) .JZMg
masses are mA =15 kg and
rna = 60 kg. The forces exerted I - - - 54 em------j (b) .J'hng
M
~r(M"':"'+-m"")'-+-m-;'-g
by the floor at the point Q and Fig. 3.128
(c) Fig. 3.131
the wall at R are, respectively (takingg = 10 m/ s2):
'(a) 600N.ISON (b) 7S0N, ISON (d) ~(M +m)' +M'g
(c) 600 N, 200 N (d) 7S0 N. 200 N 126. An insect crawls up a hemispherical
121. A block lying on a long horizontal conveyor belt moving at surface very slowly (see the Fig.
a constant velocity receives a velocity Vo m 5mfs relative to 3.132). The coefficient of friction
the ground in the direction opposite to the direction of between the insect and surface is
motion of the conveyor. After t = 4 s, the velocity of the 1/3. If the line joining the centre of • Fig. 3.132
blOCk becomes equal to the velocity of the belt. The the hemispherical surface to the insect makes an angle Q.
coefficient of friction between the block and the belt is ~ = with the vertical, the maximum possible value of Q. is given
0.2. The velocity of the conveyor belt is: by: (liT 2001(
(a) 3 mI, (b) S mI, (a) cota :::3 (b) tano.:::3
(c) 4 mI, (d) 7 mI, (c) secQ.:::3 (d) cosec c. = 3
122. Two persons of equal weight are hanging by their hands 127. A cart of mass M has a block of
from the ends of a rope hung over a frictionless pulley. mass m attached to it as shown in
They begin to climb the rope. One person can climb twice the Fig. 3.133. The coefficient of
the speed of the other (with respect to the rope). Who will friction between the block and
get to the top first? cart is Il. The minimum
(a) The faster climber acceleration 'ofthe cart so that the F~. 3.133

(b) The slower climber block m does not fall is:


(c) Both will get to the top simultaneously .~

(b)
(d) Nothing can be said as it is indetenninate g
123. A cubical block of side Lrests on ---,F=-..==~_",
a rough horizontal surface with
coefficient of friction ·fl . A
"
'!ftt"""
J I
'"'?oi
(d) M~g
m
128. A block is placed on a rough surface and an external
horizontal force F is applied on
the block as shown. If the
coefficient of friclion is _ _":::---::-::::---'
1 L
horizontal force F is applied on it. The force of friction I
exerted by the floor on the block is measured for different
sufficiently high so that the Fig. 3.129 values of F . The graph between F and lis: .
block does not slip before toppling, the minimum force (a) Straight line parallel to F·axis
required to topple the block is : (liT 2000) (b) Straight line inclined at 45 0
(a) Infinitesimal (b) mg 14 (c) Straight line inclined at 45 0 for small values of F and
(c) mg / 2 (d) mg(I-~) to
straight line parallel F·axis for large F
124. The pulleys and strings shown (d) None of the above
in the Fig. 3.130 are smooth 129: If the coefficient of friction between an insect and bowl
and of negligible mass. For the surface is Il and the radius of the bowl is r, the maximum
system to remain in equilibrium, height to which the insect can crawl in the bowl is:
the angle ashould be:
r
(liT 2001( (a) ""'r= (b) r [ l - 1 ]
(a) 0° ~1+1l2 ~I+~ ,
(b) 30"
(c) 4S'
Fig. 3.130 (c) rW (d) r(W -I]
(d) 60"
130. The potential energy of a particle of mass 5 kg moving in 137. A block of mass •m' is connected to another block of mass
thex-yplanc is given byU = -7x+ 24yjoule,xand ybeing •M' by a spring (massless) of spring constant' k'. The blocks
i~' metre. Initially at t :: 0 the particle is alme origin. (0, 0) are kept on a smooth horizontal plane. Initial ly the blocks are
moving witha velocity of6[2.4i + O.7j)m1s. The magnitude at rest and the spring is unstretched. Then a constant force
of force on the particle is: •F' starts acting on the block of mass •M' to pull it. Find the
(a) 2S units (b) 24 units
force on the block of mass •m': [AIEEE 20071
MF mF
(e) 7 units (d) None of these (a) (m+ M) (b) M
131. A block rests on a rough inclined plane making an angle of
300 with the horizontal. The coefficient of static friction (c) (M +m)F (d) mF
between the block and the plane is 0.8. If the frictional force m (m+M)
on the block is 10 N, the mass of the block (in kg) is (take 138. A boy is hanging from a horizontal branch of a tree, The
golOm/s'): (AIEEE2004( tension in the arms will be maximum when the angle
(a) 2.S (b) 4.0 (c) 1.6 (d) 2.0 between the arms is: [BHU (Matns) 20071
132. A player caught a cricket ball of mass 150 gm moving at the (a) 0' (b) 60'
rateof20 ms -I, If the catching pr,ocess be completed in 0.1 (c) 90' (d) 120'
s, the force of the blow exerted by the ball on the hands of 139. A bullet of mass 0,05 kg moving with a speed of 80 mls
the player is: [Kenls PET 2005j AIEEE 2006] ent~rs a wooden block and is stopped after a distance of

(a) 0.3 N (b) 30 N (c) 300 N (d) 3000 N 0.40 m. The average resistive force exerted by the block on
the bullet is: [Kerala (Engg,) 20081
(e) 3 N
(a) 300 N (b) 20 N (c) 400 N (d) 40 N
133. The upper half of an incline plane with inclination ~ is
perfectly smooth while the lower half is rough. A body (e) 200 N
starting from rest at the top will again come to rest at the 140. Two blocks, 4 kg and 2 kg are sliding down an incline plane
bottom if the coefficient: of friction for the lower half is as shown in Fig. 3,136. The
given by: [AIEEE 2005] acceleration of 2 kg block is:
(a) 2tan~ (b) tan~ [Orissa JEE 2008]
(c) 7sin~ (d) 2cos~ (a) 1.66m/s 2
134. A mass of M kg is suspended by a weightless string. The (b) 2.66 mfs 2
horizontal force that is required to displace it until the string Fig. 3.136
(c) 3.66 ml s'
makes an angle of 45° with t~e initial"vertical direction is:
.j;..',. (AIEEE 2006( (d) 4,66 mfsl
"'>¥- .
141. Sand is being dropped on a conveyer belt·-'at 'the rate of M
(a) Mgl.f2 (b) Mg(.f2-I)
kg/s, The force necessary to keep the belt movingy,itlf a
(c) Mg(.f2+I) (d) Mg.f2 constant velocity v mls wilt be: [CBSE (PMt) 2008]
135. Two blocks A and B of masses 2m and m,
respectively, are connected by a massless and
(a) 2Mv newton (b) zero
inextensible string. The ~hole system is (c) Mv newton (d) 1M)) newton
suspended by a massless spring as shown in 142. A steel wire can withstand a load upto 2940 N. A load of
the figure. Thc magnitudes of acceleration of 2m
150 kg is suspended from a rigid support. The maximum
A and B, immediately ancr thc string is cut, angle through which the wire can be displaced from the
are respectively: lIlT 20061 m mean position, so that the wire does not break when the load
(a) g,g / 2 (b) g / 2,g Fig . 3.134 passes through the position of equilibrium is:
(c) g .g (d) g / 2,g / 2
136. A block B is pushed momentarily along
a horizontal surface with an initial 2 B
CIt: v
(a) 30' (b) 6QO
(EAMCET (Engg,)2008(
(c) 800 (d) gso
143. A mass of 10 kg is suspended from a spring balance. It is
velocity v, if IJ. . is the coefficient of FIg. 3.135 pulled aside by a horizontal string so that it makes an angle
sliding friction between B and the 60 0 with the vertical. The new reading of the balance is:
stlrface, block B will come to rest after a time:
[Karnataka CET 20081
(UPSEE 2007, CBSE (PMT) 2007(
v (a) IOJikgwt (b) 2Mkgwt
(a) (b) g" (c) II. (d) v
(c) 20kgwt (d) 10kgwt
v v g
144, Three concurrent co-planar forces IN, 2N and 3N acting
(a) ."'L (b)
m,
along different directions on a body: m, m,
(Karnataka CET1009)
(a) can keep the body in equilibrium if2 N and·3 N act at (e)
(m] +m2 )
(d)
m,
right angle
m, (m ] +m2)

(b) can keep the body in equilibrium if 1 Nand 2 N act at 151. A block of mass m[ '" 5 kg on a smooth table is pulled by a
right angle block of mass m2 = 2 kg through a .uniform rope 1BC of
(e) cannot keep the body in equilibrium length 2m and mass lkg. The pulley is smooth and massless.
As the block of mass m2 falls from BC '" 0 to BC -: 2m, its
(d) can keep the body in equilibrium if IN and 3N act atan
acute IIngie acceleration (in ml s 2) changes from: (Take g '" IOm / s2)
145. A.mass of 1 kg is just able to slide down the slope of an m,
inclined rough surface when the angle of inclination is 60 0 ,
The minimum force necessary to pull the mass up the
inclined plane is: (g = 10 ms- 2 ) (Kerala P~T'20091
(a) 14.14 N (b) 17.32 N (e) 10 N(d) 16.(.'. :'.
(e) 0.866 N Fig. 3.138
146. Assuming earth to be an inertial frame, an example for
inertial frame observer is: IJ & K 2009] (a) 20 to 30 (b) 20 to 30
6 5 8 8
(a) a driver in a train which is slowing down to stop
(e) 20 to 30 (d) 30 to 20
(b) a person in a car moving with uniform velocity . . 5 6
5 6
(c) a girl revolving in a merry-go round
152. A block of mass m is on the smooth horizontal surface of a
(d) a passenger in an aircraft which is taking off plank of mass M . The plank is on smooth horizontal
147. Which of the following is not an illustration of Newton's surface. Now, a constant horizontal" force F acts on M.
third law? (UPSEE 2009( Now, for a person standing on the ground:
(a) Flight ofajet plane . r;;;-1
L..:.J
L.I--:_F IN. +-
(b) A cricket player lowering his hands while catching a
cricket ball -,-__M__
(c) Walking on floor Fig. 3.139
(d) Rebounding of a rubber ball
148. A rectangular block of mass 5 kg is kept on a horizontal (a) The acceleration of'~ is!... towards west
M
surface. The coefficient of friction between the block and
(b) The acceleration of m is zero
the surface is 0.2. Ifa forceof20 N is applied 10 the block at
an angle of 30° with the horizontal plane, what is the force (c) The acceleration of mis F towards east
of fri ction on the block? (Takeg '" 10 m / s2 ) (UPSe 20091 m

(a) 5 N (b) 10 N (d) The acceleration of m is ~ towards east


M+m
(e) 12N (d) 20N
153. A 45 kg box starts from rest and moves on a smooth plane
149. The velocityofa body of mass 20 kg decreases from 20 mls the force that varies w ith time as shown. The velocity of the
to 5 mls in a distance of 100m. Force on the body is: boxatt ; 8secis:
(Orissa JEE 20091 F
(a) - 27.5 N (b) - 47.5 N
(e) -37.5 N (d) - 67.5 N 1BO N
150. Two masses In] and m2 are accelerated uniformly on a
T or2!;--:-4--,':. - - t (sec)
frictionless surface as shown. The ratio of the tensions - '
T,
Fig. 3.140
is:
13 T1 &=T2
(a) 6 mi.
(e) 12 mi.
(b) 8 mi.
(d) 24 mi.
Fig. 3.137
154. Three forces are acting on a particle of mass m initially in 159. Two blocks A and B connected K
~
equilibrium. If the first two forces (R] and R2) are per-
~
by an ideal spring of spring ~F
~
constant K = 1000 N are Fig. 3.144
pendicular to each other and suddenly the third force (R3 ) m
moving on a smooth horizontal plane·due to the action ofa
is; removed, then the magnitude of acceleration of the
horizontal foreeF. Mass of A is 5 kg, mass of B is 2 kg and
~ .. particle is : F = 35 N. Find the extension of the spring at an instant
(a) ; IR21 (b) ~I R1+R2 1 when both A and B move with constant acceleration:
(a) 1 em (b) 2.5 em
(c) 1.5 cm (d) zero
160. Two small balls of the same size and of masses m, and m2
(m, > m2 ) are tied by a thin weightless thread and dropped
155. The . system shown in the Fig. 3.141 is from a balloon. The tension T of the thread during the flight
released from rest. (Neglecting friction and towards the ground after the motion of the balls has become
mass of the pulley, string and spring). The steady state is (steady state means that the balls are coming
spring can be elongated: down with unifonn velocity due to the forees weight, air
(a) ifM>m friction and upthrust). Consider that aids still without any
m m wind blow, during the motion of the balls through air:
(b) ifM>2m
(c) ifM>m (a) zero (b) (m, -m2 )g
2 2
(d) for any value of M (M should be M (m, +m2 )g
greater than zero) Fig. 3.141 (e) 2

156. A particle moves ion the x- y plane under the influence of a 161. Block A and C start from rest and move to the right with
~ accelerationa A =12t m!s2 andac = 3m / s 2 .Heretisin
force such that its linear momentum is p(t)=
seconds. The time when block B attain comes to rest is :
A[i cos(kt)- ]sin(kt)] , where A and k are constants. The
angle between the force and the momentum is :
(a) 0' (b) 30' (e) 45' (d) 90'
157. A ball of mass 3kg, moving with a speed of 100 mis, strikes a
wall at an angle 60° as shown. The ball rebound at the sarp.e
speed and remains in contact with the wall for 0.2 sec. The .
force exerted by the ball on the wall is: (Takeg = 10 m! s2)

Ball Fig. 3.145

60' (a) 2, (b) 1,


- •••••• - •••••••••• -.. Wall (e) 3/2, (d) 1/2,
60' 162. In the arrangement shown in Fig. 3.146 m, = 1 kg, m2 = 2
kg. Pulleys are massless and string are light. For what value
of M the mass mJ moves with constant velocity? (neglect
friction)
Fig. 3.142

(a) 1500v'3 N (b) 1500 N


(e) 300v'3 N (d) 300 N
158. A 2 kg block is on 5 kg block. The system
of blocks falls freely due to gravity as
shown, The net force on 5 kg block is : Aj;
(Takeg=lOm!s2)

(a) zero (b) 50 N


~ Fig. 3.143 Fig. 3.146
m,

(e) 70 N (d) 20 N (a) 6 kg (b) 4 kg


(e) 8 kg (d) 10 kg
163. Assuming the gravity to be in negative z-direction, a force 168. The graph describes an airplane's acceleration during its
-> -> ->
F = v x A is exerted on a particle in addition to the force of take-off run. The airplane's velocity when it lifts off at
-> -> t=20sis:
gravity where v is the velocity of the particle and A is a
constant vector in positivex-direction. With what minimum a(m/s 2 5)f
speed a particle of mass m be projected so that it continues 3
to move undeflected with constant velocity ?
A ~ A ~
(a) - - J (b) - J
mg mg
(c) mg J mg, o 10 20
(d) - - J -t(s)
A A Fig. 3.150
164. System shown in Fig 3.147 is in equilibrium
and at rest. The spring and string are massless (a) 40 mI, (b) 50 mI,
now the string is cut. The acceleration of mass (c) 90 mI, (d) 180 mI,
2 m and mjust after the string is cut will be :
(a) g/2 upwards, g downwards
169. In the arrangement shown in
figure, there is friction between ~~
the blocks of masses m and 2m
(b) g upwards, gl2 downwards
which are in contact. The ground
(c) g upwards, 2g downwards is smooth. The mass of the m
(d) 2g upwards, g downwards Fig. 3.147 suspended block is m
165. In the Fig. 3.148, the wedge is pushed with an acceleration The block of mass mwhich is kept
2m
of 1M ml s 2 .It is seen that the block starts climbing upon on mass 2m is stationary with Smooth ground
the smooth inclined face of wedge. What will be the time respect to block of mass 2m The Fig. 3.151

0-
taken by the block to reach the top? force of friction between m and 2m
is (pulleys and string are lig frictionless) :
/'
a • 1O ,J3m/,2 (a) mg (b) mg
2 .fi
~Fig. 3.148
(c) mg
4
(d) mg
3
2 170. A block of mass M = 4 kg is
1 .[5,
(0) .[5' (b) .[5' (c) .[5, (d) kept on a smooth horizontal

166. The pulley shown in the diagram , is


2
plane. A bar of mass m =
kg is kept on it. They are
I tk~=~,~o~oo~N~/m£~~1"_=_o_.2_
frictionless. A cat of mass I kg moves up connected to a spring as
on the massless string so as to just lift a shown and the spring is Fig. 3.152
block of mass 2 kg. After some time, the compressed. Then what is the maximum compression in the
cat stops moving with respect to the string. spring for which the bar will not slip on the block when
The magnitude of the change in the cat's released if coefficient of friction between them is 0.2 and
acceleration is : spring constant k = J000 N/m.
(0) g (b) II 2kg (a) 1 em (b) 1m
3
Fig. 3.149 (c) 1.25 cm (d) JOcm
(c) 2g (d) 4g 171. lfthe mass of block is 1 kg and 1-1 .. 0.5
3 3
,
a force of 101Jj N is applied
167. A particle is moving in a circular path. The acceleration and horizontally on the block as
momentum of the particle at a certain moment are shown in the figure. The
-: = (4i+3})m/s 2 and; =(SI-6})kgm/s. The motion frictional force acting on the
block is: 0
of the particle at the mcment is :
Fig. 3.153
(a) unifonn circular motion (a) zero (b) ~N
.fj
(b) accelerated circular motion
(c) decelerated circular motion (c) ~N (d) 5N
.fj
-> ->
(d) we cannot say anything with a and p only
172. Two blocks of equal mass 2 kg are placed on a rough (a) 12 N (b) 13 N
horizontal surface as shown and a force F is applied on the (c) 17 N (d) 16 N
upper block. The system is initially at rest. Find
176. A particle of weight W rests on a rough inclined planl
acceleration of the lower block in mt s 2. which makes an angle a. with the horizontal. If tho

).1"
2kg
O.6,~~;-F
2 kg
-+ .. 25 "1 + 10 1
t-x coefficient of static friction 11 =:z
Ian a., find the horizonta
force H acting transverse to the slope of the plane when thl
particle is abou~ to slip:
,---r---,
).I_ 0,1
Fig. 3.154
(a) 3m/s2 (b) 5 mis'
(e) zero (d) none of these
Fig. 3.157
173. A physics text book of mass mrests flat on a horizontal table
of mass M placed on the ground. Let N a-b be the contact (a) wJ3coso. (b) W.J3sin a
force exerted by body 'a' on body 'b', According to
W sin Ct Wcos ex
Newton's 3rd Law, which of the following is an (c) .J3 (d)
action-reaction pair of forces? .J3
177. A rhombus ABCD is shown in the Fig. 3.158 The sides 0
(a) mg and Ntable-+book
. the rhombus can rotate about vertex A,B,CandD. Th.
(b) (m+M)gandNtable-+book
vertex C is moving with a velocity of 6 mls in honzonla
(e) N ground-Hable and Mg + N book- Hable direction. Detennine the velocity of vertex A :
(d) N ground- Hable and N table-+groWld
A
174. A side view of a simplified fonn of vertical latch B is as
shown. The lower member A can be pushed forward in its
horizontal channel. The sides of the channels are smooth,
but at the interfaces of A and B, which are at 45 0 with the 8
horizontal, there exists a static coefficient of friction 11 = Fig. 3.158
0.4. What is the minimum force F (in N) that must be
applied horizontally to A to start motion of the latch B (a) 4.8 mls (b) 4.5 mls
upwards if it has a mass m = 0.6 kg?
(cj 5 mls (d) none of these
178. An elevator is accelerating upwards with an acceleration 0
6 m/s 2 . Inside it a person mass 50 kg is standing on ;
weighing machine which is kept on an inclined plan.
8
having angle of inclination 600 • The reading of tho
weighing machine is :
F

Fig. 3.155 la - sm/5 2

(a) 10 N (b) 0
(c) 14 N (d) 22 N
.... -"'_ weighing machine
175. A block of weight 5 N is pushed against a vertical wall by a
force 12 N. The coefficient of friction between the wall and
block is 0.6. Find the magnitude of the force exerted by the Fig. 3.,1 59
wall on the block:

12~
(a) 40 kg (b) 160 kg
(c) 80 kg (d) 50 kg

Fig. 3.156
ANSWERS

I. (0) 2. (0) 3. (b) 4. (a) 5. (c) 6. (0) 7. (b) 8. (d) 9. (d) 10. (a) 11. (c) 12. (c)
13. (b) 14. (c) IS. (b) 16. (b) 17. (c) 18. (b) 19. (,) 20. (b) 21. (c) 22. (c) 23. (b) 24. (a)
2~. (a) 26. (a) 27. (a) 28. (d) 29. (a) 30. (b) 31. (,) 32. (a) 33. (c) 34. (b) 35. (a) 36. (a)
37. (b) 38. (b) 39. (,) 40. (b) 41. (c) 42. (d) 43. (c) 44. (b) 45. (a) 46. (c) 47. (a) 48. (b)
49. (b) SO. (d) 51. (a) 52. (c) S3. (0) 54. (c) 55. (c) S6. (b) 57. (0) S8. (,) 59. (a) 60. (b)
61. (b) 62. (b) 63. (b) 64. (a) 65. (c) 66. (8) 67. (a) 6S. (a) 69. (,) 70. (d) 71. (c) 72. (0)
73. (a) 74. (a) ?S. (b) 76. (0) 77. (c) 78. (c) 79. (b) 80. (d) 81. (0) 82. (d) 83. (,) 84. (b)
... (d) 86. (d) 87. (b) 88. (0) 89. (d) 90. (a) 91. (d) 92. (a) 93. (b) 94. (b) 9S. (d) 96. (b)
97. (d) 98. (d) 99. (d) 100. (b) 101. (d) 102. (a) 103. (b) 104. (b) lOS. (d) 106. (d) 107. (c) 108. (c)
109. (d) 110 (d) 111. (a) 112. (d) 113. (a) 114. (b) lIS. (d) 116: (b) 117. (c) I1S. (c) 119. (a) 120. (d)
121. (a) 122. (c) 123. (c) 124. (a) I2S. (d) 126. (a) 127. (a) 12S. (c) 129. (b) 130. (a) 131. (d) 132. (b)
133. (a) 134. (b) 13S. (b) 136. (a) 137. (d) 138. (d) 139. (c) 140. (b) 141. (c) 142. (b) 143. (c) 144. (c)
14S. (b) 146. (b) 147. (b) 148. (0) 149. (c) 150. (d) lSI. (b) 1S2. (b) 1S3. (d) 1S4. (b) ISS. (d) 1S6. (d)
157. (a) 1S8. (b) 159. (a) 160. (b) 161. (d) 162. (e) 163. (d) 164. (a) 16S. (b) 166. (c) 167. (b) 168. (c)
169. (c) 170. (a) 171. (a) 172. (b) 173. (d) 174. (e) 175. (b) 176. (b) 177. (c) 178. (a)

[BI More than One Choice Is Correct 5. Five identical cubes each of mass m are on a straight line
with two adjacent faces in contact on a horizontal surface
1. A body is in translatory equilibrium if:
as shown in the Fig. 3.161. Suppose the surface is
(a) resultant force on it is zero
is
frictionless and a constant force P applied from left to
(b) it is at rest right to the end face of Ai which of the following
(e) it is in uniform motion statements are correct?
Cd) it is in accelerated motion ~rA"I""B~I-:c~1"'0T'1"'E1
2. In which of the following cases the net force is zero? Fig. 3.161
(a) A drop of rain falling down with terminal velocity
(a) The acceleration of the system is 5P
(b) A cork of mass 109 floating on water m
(e) A car moving with constant speed of 30 kmJhr on a
rough road (b) The resultant force acting on each cube is ~

(d) A charged particle moving parallel to the magnetic field


3. A body will not be in equilibrium if:
(c) The force exerted by C and D is ~
---+ ---+ ---+ ---+
(a) 2:F=Oandl:-c;tO (b) 1:F""ObutL"=O (d) The acceleration ofthe cube D is ~ m
-+ -+ -+ -+
ee) 1:F = ObutL. *0 Cd) EF;tOandL. ¢O 6. A reference frame attached to the earth
4. A metal sphere is hung by a string fixed to a wall. The (a) is an inertial frame by definition
forces acting on the sphere are shown in Fig. 3.160. Which (b) cannot be an inertial frame because the earth is
of the following statements are correct?
revolving around the sun
(c) is an inertial frame because Newton's law are
applicable in this frame
(d) cannot be an inertial frame because the earth is rotating
about its axis
7. Action and reaction:
(a) act on two different objects
(b) have equal magnitude
~ ~ ~ (c) have opposite directions
(a) R+T+W=O (b) T' =R' +W,
(d) have resultant zero
(0) T=R+W (d) R=WtanO
8, The system shown in Fig. 3.162 is in (a) Acceleration of m1 is g I (1+'I'\cot 2 9) where
equilibrium and at rest. The spring and string
'I'\=m 2 / m1
are massless. Now the string is cut. The
acceleration of mass 2 m and m just after the (b) Accelerationofml is gtan~ ,where'l'\=m2 Im l
string is cut will be: ~[ l+1an e]
(a) Acceleration of2 mis g/2 upwards (c) Acceleration of m2 is gl(tan9+'I'\cot9), where
(b) Acceleration of 2 m is g upwards 'I'\=m2lml
2
(c) Acceleration ofm is g downwards Fig. 3.162 f m ts
.
1 .
(d) AcceeratlOno 2
gtan 29 ,were'l'\=m2
h / m!
Cd) Acceleration of m is gl2 downwards . ~[l+1an e]
9. Two blocks of masses m1 and m2 (ml > m2 ) are connected 11. A painter is applying force himself to raise him and the box
by a massless threads. that passes over a massless smooth with an acceleration of 5 m/ s 2 by a massless rope and
pulley. The pulley is suspended from the ceiling of an pulley arrangement as shown in Fig. 3.165. Mass of painter
elevator. Now the elevator moves up with unifonn velocity is 100 kg and that of box is 50 kg. If g = 10ml s 2. then:
vO' Now, select the correct options:

m,
Fig. 3.163

(a) Magnitude of acceleration of m1 with respect to ground Fig. 3.165


(ml - rn 2 )g
is greater than ~~:-'= (a) tension in the rope is 1125 N
m1 +m 2
(b) tension in the rope is 2250 N
(b) Magnitude of acceleration ofm! with respect to ground (c) force of contact between the painter and the floor is
(m! -m 2 )g 375 N
is equal to ~~-=
m! +m2 (d) none of the above
(c) Tension in the thread that connects mJ and m2 is equal 12. Which of the following statement(s) is/are incorrect?
to 2m
__I,"-m~2cog (a) Friction force always acts opposite to the net applied
m 1 +m2 force
(d) Tension in the thread that connects m1 and m2 is (b) If net force on a body is zero, its acceleration will be
2m m g same in all frames
greater than ! 2
m 1 +m2 (c) A person is pushing a box. The force exerted by box on
. person is less than the force by person on box .
10. A horizontal bar of mass m! and Prism of mass m2 can (d) A moving object can move with a constant velocity by
move as shown. There is no friction at any contact point. a push less than the limiting static friction.
During the motion, the length of the rod is always
13. Block A is placed on cart B as shown in figure. If the
horizontal. Now, magnitude values of:
coefficients of static and kinetic friction between the 20 kg
block A and lOCI kg cart B are both essentially the same
m,
Smooth Wall 1=====:::;1 value of 0.5 :[g = lOm/s2]

A20kg
, I! = O
8100 kg

Smooth Surface
Fig. 3.164 Fig. 3.166
(a) The blocks A andD will have a common acceleration if (d) If A is false but R is true.
P=60N (e) If both A and R arc false.
(b) Acceleration of cart B is 0.98 m/s 2 if P "" 40 N I. (A): In the given Fig. 10kg
.kg

,,0 as.cl:::,F
(c) Acceleration ofcartB is greater thanthal of A !fP- 60N 3.167, tension in the
(d) The common acceleration of both the'blocks is 0.667 string that connects
&
m/s 2 ifP - 40N the two blocks is 10
Fig. 3.167
N. so that the force
14. A 3 kg block of wood is on a level surface where!! s =0.25 F will be 35 N.
and).l k ",0.2. A force of7 N is being applied horizontally to
(R): Forces of tension, each of magnitude ION here.
the block. Mark the correct statement(s) regarding this
which the two blocks exert on each other through
situation: [Takeg ""10m/5 2]
the string, balance each other.
(a) If the block is initially at rest, it will remain at rest and 2. (A): Force offriction applied by ground on the wheels of
friction force will be 7N a bicycle is always in the direction of motion of the
(b) If the block is initially moving, then it will continue its bicycle.
motion for forever if force applied is in direction of (R) : Force ofmction always opposes the relative motion
motion of the block, between the surfaces in contact.
(<!) If the block is initially moving and direction of applied 3. (A): In the given Fig. 3.168, a
force is same as that of motion of block then block
moves with an acceleration of ~ m / s2 along its initial
force 20 N pulls a block
of mass 10 kg on a rough
! 10kg t;:'ON
3 flo or. If coefficient of Fig. 3.168
direction of motion. friction is ).l = 0.4, force
(d) Irthe block is initially moving and direction of applied of friction acting on the block will be 20 N.
force is opPosite to that of initial motion of block then (R) : Force of kinetic friction is slightly less than the
block decelerates, comes to a stop and starts moving in limiting value of force of friction.
the opposite direction.
4. (A): A body of mass 10 kg is placed on a rough inclined
~

IS. A force F (larger than the limiting friction force) is applied surface ij.l "" 0.7). The surface is inclined to
to the left to an object mov ing to the right on a rough horizontal at angle 30". Acceleration of the body
horizontal surface. Then: down the plane will be zero.
(a) the object would be slowing down initially (R) : Work done against friction always converts to heat.
~ S. (A) : A cloth covers a table. Some dishes are kept on it.
(b) for some time F and friction force will act in same The cloth can be pulled out without dislodging the
direction and for remaining time they act in opposite dishes from the table.
directions.
(R) : For every action there is an equal and opposite
(c) the object comes to rest for a moment and after that its reaction. (lIT 2007)
~

motion is accelerating in the direction of F 6. (A): A block of mass m starts moving on a rough
(d) the object slows down and fina lly comes to rest. horizontal surface with a velocity v. It stops due to
friction between the block and the surface after
ANSWERS moving through a certain distance. The surface is
now tilted to an angle of3O" with the horizontal and
I. (a), (b) and (e), 2. All, 3. (b), (e) and (d), 4. (a), (b) and the same block is made to go upon the surface with
(d), 5. (b), (e) and (d), 6. (b) and (d), 7. (a), (b) and (e), 8. the same initial velocity v. The decrease in
(a) and (e), 9. (b) and (e). 10. (a) and (e), II. (a) and (e), 12. mechanical energy in the second situation i!> smaller
than that in the first situation.
~OO~~I~OO~~~~OO~OO I ~ ~OO
(R): The coefficient offriction between the block and the
and (c).
surface decreases with the increase in the angle of
Ie) Assertion-Reason Type Questions inclination. (liT JEE 2007)
7. (A): The acceleration ofa body down a rough inclined
(a) If both A and R are true and R is the correct plane is greater than the acceleration due to gravity.
cltplanation of A.
(R): The body !s able to slide on a inclined plane only
(b) If both A and R are true but R is not correct when its acceleration is greater than acceleration
explanation of A. due to gravity. (AIlMS 20071
(e) If A is true but R is false.
8. (A): Angle of repose is equal to the angle of limiting (D) Integer Type Questions
friction.
I. The pulleys and string are massless. No friction on the
(R): When the body is just at the point of motion, the inclined plane. The ir.clined plane is fixed. The mass of
force of friction in this stage is called limiting block A is 0.4 kg. The mass ofblock B is 0.3 kg. The angle
friction. IAIIMS 20081 of inclination of the inclined plane is 60°. Find the nonnal
9. (A): Kinetic mcti<ln force is non-conservative in nature. reaction on A offered by inclined surface during its motion.
(R): Net work done by kinetic friction on a block is
always completely converted into heat.
10. (A): If a body is trying to slip over a surface then friction
acting on the body is necessarily equal to the
limiting friction.
(R) Static friction can be less than the limiting friction
force.
tl. (A) It is easier to maintain the motion than to start it on Fig. 3.169
rough surface. 2. Two blocks of masses m1 and m2 are connected by massless
(R): Dynamiclkinetic friction is less than the force of . threads, The j)ulleys are massless and smooth. If a 1 is the
limiting friction. magnitude value of acceleration of m1 and a 2 is the
12. (A): Polishing a surface beyond a certain limit increases a
magnitude value of acceleration of"'2. find the ratio - ' .
rather than decreases the frictional forces. a,
(R) : When the surface is polished beronc:. a certain limit,
the molecules excn ::trong attractive force. This is
called surface allb,·.;ion, to overcome which
additional force is n:quired. Hence frictional force
increases.
13. (A) : Friction force does not depend on visible area of
contact.
(R) : Limiting friction force is· dependent on normal
reaction.
14. A frame of reference A is moving rectilinearly and
~

uniformly with a velocity u with respect to an


~ Fig. 3.170
inertial frame B. A body is mov!ng with velocity v
~
3. Fig 3.169 shows a unifonn rod of length 30 cm having a
and acceleration a in an inertial system. mass of3.0 kg. The strings shown in the figure are pulled by
(A): When we use Newtons second J.aw in frame B we constant forces of20 Nand 32 N. The force exerted by the
~ . ~ 20 cm part of the rod on the 10 em part is found to be 8T,
write IFI'IC\ :m a, Now when we use the same in Find but value of T (al1 the surfaces are smooth and the
~ ~
frame A we will write eltactly same F net and a. strings and the pulleys are light).
(R): All inertial systems are equally suitable for the 10em 20cm
description of physical phenomenon.
15. (A): Centrifugal force is reactionary force of centripetal
~
120 ~*h
J;$ 2
I
:\81;(0 )0
force.
(R) : A simple pendulum is oscillating in vertical plane Fig. 3.171
then at mean position net force on pendulum is zero. 4. A 20 kg block is originally at rest on a horizontal surface for
which the coefficient of friction is 0.6. A horizontal force F
ANSWERS
is applied such that it varies with time as shown in the figure
I. (c) , . (d) 3. (b) 4. (b) S. (b) 6. (c) 7. (e) (a) and (b). If the speed of the block after 10 s is 8vthen find
8. (b) 9. (c) 10. (d) 11. (a) 12. (a) 13. (b) 14. (a) v.(Takeg: IOm /s 2 )
15. (d)
F(N) aA
45'
......."'."""'--;:
200
ae
1L.l"'-- - _ t(s)
o1L-c5:--e,00-,(,)
(a) (. )
(a) (. ) FIg. 3.175
Fig. 3.172
11. A bfock A of mass 2 kg .rests on another block B of mass
S. Three blocks A, Band C of masses 5 kg, 10 kg and 15 kg 8 kg which rests qn a horizontal floor. The coefficient of
respectively connected by two ideal strings are present on a friction between A and B is 0.2 while that between Band
smooth horizontal surface. An external horizontal force of floor is 0.5. When horizontal force F = 25 N is applied
30 N acts on the block A to pull the system, The difference
on the block B. the force of friction between A and B is
in the tensions in strings connecting A and B, and Band C, is
found to be 5Tthen find Qut value ofT. (in N).
12. Coefficient of friction between two blocks shown in Fig.
6. A 6 kg block is moving with a speed ~.24.:~
3. 174 is J.I ::::: 0.4. The blocks are given velocities of 2 mls
of 10 mls over a rough surface with • ..l.~......I.."".".~••" and 8 mls in the directions shown in figure. The time when
coefficients of friction f.t s = 0.6 and p
I.l. /r = 0.4 as shown in figure. A time 1-1, - 0.6 relative motion between them will stop is (in sec.)

~
varying force F c 4t (F in Newton 0.4
IJk ..
and I in second) is applied on the Ag.3.173
block as shown. Find the acceleration lft9"""s
of block at t =5 sec. (Take g = 10ml s 2) Fig. 3.176
7. Two blocks A and B of mass 2 kg and 4
A
kg are placed one over the other as
shown in figure. A time varying 1. 2. 4 3. 3 4. 3 5. 2 •• 0 7. ~

horizontal force F '" 21 is applied on the B •• 2 •. 2 10. 3 Jl. 0 11. I


upper block as shown in figure. Here t is
in second and F is in Newton. Coefficient IE) Match the Columns
Fig. 3.174
of friction between A andB isl.l '"' Mand
the horizontal surface over which B is placed is smooth 1. In the diagram shown, match the following (g '"' 10m! s 2 )
(g = 10m! s2 ) If acceleration of block A as a function of
Blocks are on ~moolh incline. Ft and F2 are parallel to the
time is given by aA '"'!.. then find value of c. (I s: 7.5s) inclined plane. The motion ofthe blocks is along the incline
e the surface.
8. Two blocks of masses M 1 and M 2 connected by a light
spring rest on a horizontal plane. The coefficient of friction
between the blocks and the surface is equal to ~ . The
minimum constant force that has to be applied in the
horizontal direction to the block of mass M I in order to shift
the other block is(Mt+ :2 }g. then (l is: .
Fig. 3.177
9. A car of mass M is accelerating on a level smooth road
under the action of a single force F acting along the
direction of molion. The power delivered to the car is Column -I Column -II
constant and equal to p. If the velocity of the car 'at an
) (a) Acceleration of 2 kg (p) 39 SI unit
instant is v, then after travelling a distance of 7Mv the block
3p
velocity becomes /ro then k is : (b) Net force on 3 kg block (q) 25 SI unit
10. Two blocks A and B of mass 2 kg and 4 kg are placed one (e) Nonnal reaction (,) 2 SI unit
over the other as shown in Fig. (a). A time varying
between 2 kg and 1 kg
horizontal force F '"' 2t is applied on the upper block. Here t
is in second and F is in Newton. A graph drawn between (d) Nomlal reaction (,) 6 SI unit
accelerations of A and Bon y ·axis and time on x -axis is between 3 kg and 2 kg
shown in Fig. (b). Then tan e =..!.. , K =...
K
2, Block of mass mis sliding up on the smooth inclined plane (d) The value oftensionT2 (s) -40 SI umts
.
with some initial velocity. Now on the block, match the 19
magnitude value of the forces on • m' in the given
directions (Take the component of all the forces in the (t) None of the above
given direction). 4. The system shown below is initia!ly in equilibrium. Masses

~.m0'1 b'"k:a~ direction


of the blocks A,B,C,D and E are respectively 3m,3m,2m.
. 2m and 2m. Match the conditions in column-I with the
effects in cloumn-ll.
~ Horlzonlal dlrectfon
Fig. 3.178

Column - I Column - II
(a) Net force component in (P) Zero
Spring 1 c
horizontal direction B D
. (b) Net force component in (q) mg sin 9cos 9
vertical direction
A E
(e) Net force component (r) mgsin9
FIg. 3.180
along the inclined p lane
(d) Net force component (s) mg(l-cos 2 9) Column - I Column - II
perpendicular to the
inclined plane (a) After spring 2 is cut, (P) Increases
tension in string AB
3. Two blocks of masses In] = 5 kg and m2 "" 2 kg are
connected by threads which pass over the pulleys as shown (b) After spring 2 is cut, (q) Decreases
in the figure. The threads are massless and the pulleys are tension in string CD
massless and smooth. The blocks can move only along the
(e) After spring between C (r) Remains constant
vertical direction. Tt and T2 are the tensions in the string as
and pulley ;s cut,
showri. Now match the following: (fakeg = IO m/ s2)
tension in string AB
(d) After spring between C (s) Zero
and pulley ;s cut,
tension in string CD
5. A block of mass m IS kept on a smooth movlDg wedge. Ifth{
acceleration of the wedge is equal to a,

.. JL i

Fig. 3.179 Fig . 3.181

Column - I Column - II
Column - I Column - II (a) Acceleration of m (P) Ma+ m(gcos 9+asin 9)
(a) Magnitude value of (P) -500 SI umts
. sin e
acceleration of mt with t9 (b) Acceleration of m(q) (gsin a-acos 9)
respect to ground relative to the wedge
(b) Magnitude value 0 (q) -250 SI umts
acceleration of m2 with 19
. (e) Force on the wedge (r)
exerted by ground
(J
g 2 + a 2)sin 9
respect to ground
(d) Force on the wedge (s) (M+mcos 2 9)g+
(e) The value of tension Tl (,) 60 SI . exerted by external
- umts ma sin a cos 9
19 agent
6, Lift can move in y -axis as well as along x -axis. A ball of 8. Two blocks of mass ml and m2 (m2 > ml ) are placed in
mass m is attached to ceiling of lift with inextensible light contact with each other on an inclined plane as shown in
rope and box of mass mis placed against a wall as shown in figure. The coefficient offriction between ml and surface is
figure. Neglect friction everywhere.
III and between m2 and surface is 1.12' .
IY
T
m -x
Fig. 3.184
L-_cr;Tm _ N
Fig 3.182 Column - I Column -II
Column -I Column - II (a) III =0.3,112 =0.2 (P) Acceleration of both
-. blocks is different
(a) In figure lift is moving (P) Zero
along x -axis then value (b) !AI =0.2,1l2 =0.3 (q) Acceleration of both
ofT may be blocks is same
(b) Lift moving toward (q) >mg (c) III =0.3,112 :=0.3 (r) Nonnal reaction
right along x -axis with between both the
. decreasing speed then blocks is zero
value of N may be
(d) II I =0.3,!A2 =0.2 and (s) Nonnal reaction
(c) Lift ;5 moving in (r) <mg the inclined plane starts between both the
upward direction moving up with blocks is non-zero
(y-axis) then value of acceleration g 12
T may be
(d) Lift ;5 moving in (s) =mg 9. Match Column-I with Column-II
downward direction
Column - I Column -II
with constant velocity
then value ofT may be (a) Gravitational force (P) Central

7. Figure shows a block pressed against a rough vertical wall (b) Viscous force (q) Non conservative
with a force F as shown in side view. Column-I shows angle (c) Electric force (r) Conservative
at which force F is applied and Column-II gives infonnation
about corresponding friction force, match them. (d) Frcition (5) Mechanical energy ;5
conserved

F .. 100 N 10. Fig. 3.185 (a) shows a system of two blocks of masses m
m=7.5kg
and M (> m) plac~d on a surface; there is no friction
m between the blocks and the'surface. A force F is applied to
/--1< 1-\=0.1
the system as shown resulting in the motion of the system.
F ••• ~ ••••

Fig. 3.183 m
F
Column - [ Column - II M
(a) 9=37° (P) friction by wall on l&
block is upwards . (a) (b)
(b) 9 =45° Fig. 3.185
(q) friction by wall on
block is downwards Fig. 3.185 (b) also shows a system of blocks of masses lIZ
(c) 9=53° (r) friction by wall on andM. Masses mandMhave, respectively, the same values
block is static as in Fig. (a). There is no friction between the lower block ·
and the surface on which the system is placed but friction is
(d) 9=0° (5) friction by wall on present at the contact of the two blocks. A force F, having
block is kinetic the same value as in Fig. 3.185 (a), is .applied to the lower
block resulting in the motion of the system such that both ff'00
blocks move together with a common acceleration. Match ~1J.~'1:i
Column~I and II . Decelerating
~

~
i .. - 1m/s2 N\"'~
Column ~ I Column ~ II ~o "'~ ~'#)
E'9~1
<:,.~e~ ~ ~, . ~ .
(a) For the system in Fig. (P) Work done by the force force 5 kg
ce O'·
\0'
(a) does not convert to heat (d)
(e) 30"
(h) For the system in Fig. (q) AcceIeratton
. l. SF- -
(b) M+m Column - I
Column - II
(e) In Fig. (a), instead of (,) Force acting on m is Among the four situations
applying the force onM Fm (a) Force offriction is 50 N (P) Fig. (a)
--
towards left, if this M+m in
force is applied on m . ;
towards right (b) Force of friction is (q) Fig. (p)
maximum in
(d) In Fig. (b), instead of (s) AcceIeratlOn
. I. SF-
applying the force onM (e) External force is (r) Fig. (e)
M ;
towards left, if this maximum in
;
force is applied on M (d) After sometime, force (s) Fig';(d)
towards right of friction will be 45 N .
in
11. Coefficient of friction between the block and thl;( surface in
each of the given figures being 0.4, match Column-I and II.
ANSWERS
(takeg = IOm!s2)

(a)
as
(b) El::5N
2 kg
1. a-* r; b-4 p; c-* q; d-4 S
2. a-4 q; b -4 s; C-4 r; d-4 p
3. a-* s; b-4 r; C-4 q; d-* P
4. a-* r; b-4 q; c-* q,s; d -4 q

~5N
F",5N
5. a -* r; b-+- q; c-+- s; d-+- p
60"
(e) 2 kg (d) . 2 kg ---- 6. a -* q,s; b-* q,r,S; c-+ p,q,r,s; d-+ s
7. a-* p,s; b -* p,r; C-4 q,r; d-+ p,s

- Column ~ I 8. a-+- q,s; b-+- p,r; c-+- q,r; d-+ q,s


Column - II
9. a-4 p,r,s; b-+ q; c-+ p,r,sj d-+ q
(a) Force of friction is zero (P) Fig. (a)
. in 10. a-+ p,q,r; b-+ p,q,r; c-+ p,q; d -+ p,q,r
11. a-+- p,rj b-+- q,Sj c-+ p,q,r,s; d -+- does not match
(b) ~o:e 'of \,~~:i~n is 2.5 (q) Fig. (h)
.\ U. a-4 p.q,r; b-4 p,q,r; c-+ p; d-+- r

Accelei"atio~
, the (,) fig. (c)
.(e) .
block is zero-in
of
. [F) Linked Comprehension Type
Paragraph - 1
(d) Acceleration of the (s) fig. (d) Two blocks m1 and m 2 are allowed to move without
'b lock is 1 m!s2. in
. friction. Block m1 is on block m 2 and m2 slides on smooth
fixed incline as shown. The angle of inclination of inclined
12. Consider the'"situilti.o ns given in the following figures and
<" match Column-I andH: Take coefficient of kinetic friction plane is e.
as 1 andg =,IOm/s 2 :
Accelerating
-; = 1m/s2
~
Moving at constant speed
-
a _O
,
E":Jamal ~I
"C " , ·

External
~_ (a) for~ . 5 kg (b) for=-- 5 kg , Fig. 3.186
" 3 .6 £!.d
1. The acceleration of m1 with respect to ground is : m 2 g 3 cose 2 3
m g cos 9
(a) (b)
(ml +m2 )gsin2 9 (m l +m2 )gsin2 e 6k 2 sin 3 9 3k 2 sin 3 9
(a) "9 (b)
m2 +m1 sm
(c)
m 2g 2 cos9
(d)
m 2g 2 cos e
(m l +rnl)gsln2 a 6k 2 sin 3 9 3k
2
sin 3 9
(c)
Paragraph - 3
2. The acceleration of m2 with respect to ground is : Two smooth blocks are placed at a smooth corner as shown.
Both the blocks are having mass m We apply a force F on
(m l + m2 )gsin Z e (m 1 +m2 )gsin9 the small block m Block A presses the blockB in the normal
(a) . '9 (b) =--'-"~'C:';::9":- direction, due to which pressing force on vertical wall will
m2 +rn1 sm m ] + m2 sm
increase, and pressing force on the horizontal wall
(m l +m 2 )gsin 2 a (m J +m 2 )gsin 2 9 decrease, as we increase F (9 = 370 with horizontal). As
(c) . '9 (d) -'--.!--'-"'--."'-9- soon as the pressing force on the horizontal wall by block B
m2 -m1 sm m1 - m2 sm
becomes zero, it will loose the contact with the ground. If
3. Nonna! reaction on mj is : the value of F is further increased, the block B will
(a) mig accelerate in upward direction and simultaneously ·the
block A will move towards right.
(b) (m! +m2 )g
y
(c)
m,m2 gcos 2 e B
m
smooth
A

, """,,=~~
Fig. 3.188
Paragraph - 2
At the moment t "" 0 the force F = kt is applied to a small 7. What is the minimum value of F, to lift block B from
body of mass mresting on a smooth horizontal plane (k is a ground?
constant). The direction of this force always fonns an angle 25 5
9 with the horizontal as shown (a) ~ mg (b) - mg
12 4
F
3
r--(l'~---
(c) - mg
4
(d) "34 mg
m 8. If both the blocks are stationary, the force exerted by
ground on block A is :
Fig. 3.187 3F 3F
(a) mg+- (b) mg--
4. Find the time t at which moment, the body breaks off the 4 4
plane: 4F 4F
(c) mg+ - (d) mg - -
mg 3 3
(a) (b) ...!!'L
2ksin 9 ksin9 9. If acceleration of block A is a rightward, then acceleration
of block B will be :
(c) mg (d) 2mg
ksin 29 ksin 9 3a 4a
(a) - upwards (b) - upwards
4 3
5. The velocity of the body at the moment of its breaking off
the plane is : 3a 4a
(c) - upwards (d) 5
5
mg 2 cos 9 (b) mgcos 9
(a)
2ksin 29 2ksin 2 9 Paragraph - 4

(c) 02m
..=g,..
' ""co:::s",9 (d) 2mgcose I! = O.1 10kg ~
ksin 2 9 ksin 2 e 5 kg
6. The distance traversed by the body upto the moment of its , 0.3
).1 ..
breaking off the plane is : Fig. 3.189
10. When F =2N, the frictional force between 5 kg block and hanging container H has mass m. You are to measure the
ground is: magnitude a of the acceleration of the system in a $~ries of
(a) 2N (b) 0 experiments where mvaries from experiment to experiment
by M tot does not; that is, you will shift sand between the
(e) 8N (d) ION
11, The acceleration of 10 kg block when F = 30 N containers before each triaL ~ is taken on the horizontal
M tot
(a) 2m/s2 (b) 3m/s2
axis for all the plots.
(c) 1m/s2 (d) none
12. Find the minimum value ofF for which Skg block being to ,
,.;2
slide if the coefficient of friction between blocks is changed .'
to O.S: -----/ ---------3
"
(a) ION (b) 30N -----------.,5 ,:
(c) 45 N (d) No value exist o _.!!!...;:.' 1.0
! Mt~t
Paragraph. 5
(a) (b)
Two bodies A andB of masses 10 kg Fig. 3.191
and 5 kg are placed very slightly
separated as shown in figure. The 16. The plot in figure (b) which gives the acceleration
coefficients of friction between the A magnitude of the container (taken on y-axis) as a function
B
floor and the blocks are ~ s =!l k = F - 10kg 5 kg
004. Block A is pushed by an external
of ratio (~)
M
is :
tot
force F. The value of F can be
Fig. 3.190 (,) 2 (b) 3
changed. When the welding between
block A and ground breaks, block A will start pressing (e) 4 (d) 5
block B and when welding of B also breaks, block B will 17. The curve which gives tension in the connecting string
(~)
start pressing the,vertical walL
(taken on y -axis) as a function of ratio is :
13. IfF = 20N, with how much force does block A presses the . M tot
block B?
(,) ION (b) 20N (,) I (b) 2
(e) 4 (d) 5
(e) 30 N (d) "'0
14. If F = 50N, the friction force acting between block Band 18. The curve which gives the magnitude of net force, on
ground will be : container H (taken on y -axis) as a function of ratio (~)
M
(a) 10 N (b) 20.N tot
(e) 30 N (d) None is:
15. The force of friction acting on B varies with the applied (,) (b) 2
force F according to' curve : (e) 4 (d) 5

Paragraph - 7
(a) FBb (b) FBLe A block of mass M slides on a frictionless surface with an
initial speed ofv o. On top of block is a small box of mass m.
The coefficients of friction between box and block are ~ s
and).l k' The sliding block encounters an ideal spring with
force constant k. Answer the following questions:
(e) FBL (d) FBli
F F m
Paragraph - 6 M
Two containers pf sand Sand H are arranged like the
blocks figure~(a). The container alone have negligible Fig. 3.192
mass; the sand in them, luis a,--total- mass .M tot; ,the sand in the

19. Assuming no relative motion between box and block what 3. An iron sphere weighs 10 N and rests in a V-shaped trough
is the maximum possible acceleration of block and box at whose sides fonn an angle of 60°. What are the nonnal
the instant of maximum compression? forces exerted by the walls on the sphere in cases as shown
(h) ~,Mg in Fig. 3.1931
m
(e) ~,(M+m)g Il .r mg
(d)
m M
20. What is maximum value of k for which it remains true that
box does not slide?

(a) (~,g)2 M
Vo (M+m) (al Ibl lei
Fig. 3.193
(e) (~,g )2 (M+m)2 (d) None.ofthese
. 2vo M [Ans. (a) R,4 = 10 Nand RB = 0, (b) R,4 = RB = 10 N, (c)
R,4 :::20/.J3"NandRB = (10/.J3) N]
21. Suppose the value of k is just slightly greater than the value
found in Q.20, so that the box begins to slide just as the 4. T~eeblocksofmassesml :;2kg,m2 =3kgandm 3 =4kg
spring reaches maximum compression. The accelerations are in contact with each other on a frictionless horizontal
of box and block are respectively: . ' surface as shown in Fig. 3.194 (a). Find (a) horizontal force
m(1-' s -Il k )+)..L sM F needed to push the bl6ck as one unit with an acceleration
(a) oBox =l-tkg, °Block ~ M g ofi m/s2 (b) the resultant force on each block and (c) the
, magnitude of contact forces between blocks.
(b) 0Box =(Ils-Ilk)g, aBIOCk\ =(lls-llk):;:
. \ mg
(e) a~ox =l-tkg, °Block = (Il s -Ilk)/i

(d) none of the above


('I Ibl
ANSWERS Fig. 3.194

Paragrtlph-l 1. (a) 2. (d) 3. (e) [Ans. (a) 18 N; (b) 4 N, 6N, 8 N; (c) 14 N between ml andm 2 and
Paragraph-2 4. (b) 5. (a) 6. (a) 8 N between m2 and m31
Paragraph-3 7. (e) 8. (e) 9. (,) 5. A string is wrapped around a log of wood and it is pulled
with Ii force F as shown in Fig. 3.194 (b). How does the
Paragraph4 10. (a) . 11 •. (a) 12. (c) - value of tension T in the string change with e? When will
Paragraph-S 13. (d) 14. (,) 15. (b) the tension T be greater than applied force?
Paragraph-6 16. (c) 17. (d) 18. (b) [Ans. T increases as aincreases; a > 1t 13]
Paragraph-7 19. (a) 20. (b) 21. (a) 6. Three blocks are connected by strings as shown in Fig.
3.195 and are pulled by a force T) = 60 N . If ml :; 10 kg,
fG] Subjective Question m2 = 20 kg and m) ::: 30 kg, calculate the acceleration of
'th« system and Tl and T2 .
1. An empty plastic box ofmassM is found to accelerate up at
the rate of (g/6) when placed deep inside water. How much
sand should be put inside the box so that it may accelerate
down at the rate of (g16)?
[Hint: Initially Th - mg = m(g16) and finally (m + x)g - Th Fig. 3.19~
2 . , .
= (m + x)(g16); solve these to find xl [Ans. a=lm / s ;Tl = ION artd T2 =30N]
[Ans. 0.4 Ml 7. A unifonn rod of length L and density p is being pulled
2. If the potential energy between electron and proton at a along a smooth floor with horizontal acceleration a. What is
distance r is given by U "" -(Ke 2 / 3r2 ) obtain the law of the magnitude of the stress at the transverse cross-section
force. through mid p.ointofthe rod? [lIT 1993)
[Hint: F = -dU I dr] [Hint: Repeal .s olvedProblem 9withF2 =0, fl = Fand y= LI2

[Ans. F = -K; I r 4] togetT = Fl2


Now Stress =!.... =..!.... = Ma :: pALa == ..!.pLa ) pendulum and (b) the angle it makes with the vertical.
A 2A 2A 2A 2 (g=lOm/s 2 1
[Ans. (1/2)pLa)
[Ans. (a) 8.94 xlO- 4 N, (b)e:= tan- l (0.50)]
8. A block of mass M is pulled 13. A trolley A has a simple pendulum suspended from a frame
along a horizontal frictionless fixed to its deck [Fig. 3.199 (a)]. A blockB is in contact on
surface by a rope of mass mas its vertical side. The trolley is on horizontal rails and
shown in Fig. 3,196. A accelerates towards the right, such that the block is just
horizontal foreeP is applied to FIg. 3.196 prevented from falling. If the value of coefficient of friction
one end of the rope. Find (a) the acceleration of the rope and between A and B is 0.5, find the value of the inclination of
block (b) the force that the rope exerts on the block (c) the the pendulum to the vertical.
ten'sian in the rope at its mid point.
[Ans.(a)(_F_),(b) MF ,(c)(2M+m)F]
M+m M+m 2(M+m)
9. A block of mass 2 kg is kept at rest on a smooth inclined A
plane as shown in Fig. 3.197 (a) with the help ofa string.
Find the tension in the string and reaction on the block. If
the string is cut, find the acceleration of the block
neglectJng friction. (al (bl
Tree Branch Fig. 3.199

[Ans. e=tan - l (2) "' 63.4°]


14. What force F must be applied so that ml and m2 are at rest
onm3 in Fig. 3.199 (b)?
[Ans. F '" (ml + m 2 + m3 )m2g I md
15. A unifonn flexible chain oflength L .....
with weight A. perunit length passes T
I'f!!"'' ' 'T
over a small frictionless, massless I I
(al
Fig. 3.197
(bl
pulley. It is released from rest y
position with length of chain y
hanging from one side and (L - y)
1 1
(L-y)

from the other side. (a) Under what


[Ans.T = 9.8 N; R = 17 N anda = 4.9 m/s2] condition will it accelerate? (b)
to. A light rope fixed to a peg on the ground passes over a tree assuming these circumstances find Fig. 3.200
branch and hangs on the other side [Fig. 3.197 (b)], A man the acceleration as a function of y.
weighing 60 kg wants to climb up the rope. Ifthe peg comes
[Hint: The ehain wi!! accelerate jf Fl ¢ F2 and acceleration wi!!
out of the ground by a vertical force greater than (360.J3) N,
be = (Fj -F2 )/m]
with what maximum acceleration can the man climb up
safely? (g = IOm/s2) [Ans. (a) y¢ (Ll2), (b) a=g(l-2ylL»)

[Ans. 2 m I s2] 16. Calculate the tension in the string shown in Fig. 3.201 (a).
The pulleys and the string are light and all surfaces are
11. A vessel containing water is given a constant acceleration
frictionless. (g = 10m/s2)
'a' towards the right along a straight horizontal path. Which
of the following figures, represents the surface of the
liquid?

~~~~~
~~~
(al (bl (el
Fig. 3.198 (bl
Fig. 3.201
[Ans. (c)]
12. A pendulum bob of mass 80 mg and carrying a charge of [Hint: As pulleys are fixed a1 '" az := a. So, for A, I xg - T
2x 10-8 coulomb is at rest in a uniform horizontal electric := lxaand for B,T = 1 xa)
field of20 kV/m. Find (a) the tension in th.e .thread of the [Ans. T = 5N and a:= 5m/s2]
17. A block A of mass mis tied to a fixed point C on a horizontal
table through a string passing round a massless smooth pulley
B. A forceF is applied by the pulley B as shown in Fig. 3.201
(b). Find the accelemtions of the pulley and mass A.
[Hint: As acceleration of Bwill be halfthat of A, so, for motion of
A,T = rna and for B,P - 2T '" Ox(a/2)]
[Ans.a A ",Fl2rnandan =F I 4rnJ
18. Fig. 3.202 shows a man of mass
60 kg standing on a light [Ans. (a) 333.33 N /m, (b) 8.33 m l s2]
weighing machine kept in a I 21. From three identical springs (each,having force constimt k)
cabin of mass 40 kg. The cabin is using all at the'same time the following four combinations
hanging from a pulley fixed to are possible (Fig. 3.205). Calculate the equivalent force
the ceiling through a light rope, constant in each case.
the other end of which is held by
the man himself.. If the man
manages to keep the cabin at rest, B k k k
what is the weight shown by the
machine? What force should he
j
F
exert on the rope to get his k
correct weight on the machine?
Mg mg
[Hint: See solved Problem 50]
FIg. 3.202
[Ans. Weight ;:: R '" 109 N Si 10 kg
and F==T ", 300gN.]
19. The systems shown in Fig. 3.203 are in equilibrium. If the
spring balance is calibrated in newtons,what does it record (a) (b) (e) (d)
in each case?(g = lOml s2 ) Fig. 3.205
[Ans. (a) k 13, (b) 2k 13, (c) 3k 12, (d) 3k]
22. Consider the situations shown in Fig. 3.206 (a) and (b).
Initially the spring is unstretched when the mass m is
released from rest. Assuming no friction 'in the pulley, find
(a) the maximum stretch in the spring, (b) stretch in the
spring when the system is in equilibrium.
~~
k
10 kg 10 kg
t
(a) (b)

mg

(a) (b)
Fig. 3.206
(e)
[Hint:' For maximum' stretch loss in P.'E,. of mass '" gain in elastic
Fig. 3.203
energy of thc 'spring while for eqUilibrium in (a) mg '" T and (b)
[Ans. (a) lOa N, (b) 200 N, (c) 50 N] mg '" 2T, Further in c6se (a) stretch of spring'" displacement of
20. A 2 kg box rests on a frictionless incline of angle 30" mass while in case (b) stretch of spring '" 2 xdisplaeement of mass.
See also solved Prob.IS (Chapter 8)] ,
supported by a spring. The spring stretches by 3 cm. (a)
Find the force constant of the spring. (b) If the box is pulled [Ans. (a) Ymax ", ( 2~g)and, yeq =,:g,
down the incline 5 cm from its equilibrium position and
released, what will be its initial acceleration? (g = 10 ml s 2 ) mg mg
(b) Ymax '" k and Ycq == '2k]
23. Two identical blocks, each of mass m 28. A 40 kg slab rests on a frictionless floor. A 10 kg block rests
are connected through a massless
111, ""'- on the top of the slab [Fig. 3,209 (a)]. The coefficients of
spring of force constant k and static and dynamic friction are 0,60 and 0.40 respectively.
arranged as shown in Fig. 3.207. Spring The 10 kg block is pulled by a horizontal force of 100 N.
The spring is compressed by an What are the resulting accelerations of (a) the block (b) the
amount y by applying an external
force and released. (a) What is the m
velocity of upper block when the
C-'-= ~
~
slab? (g = IOm/s2)

spring is relaxed? (b) What should Fig. 3.207


be the initial compression of the
spring so that the lower block is just raised above the floor?
101 . (bl
(Hint: (a) By conservation of ME, . !.mvl + mgy=.!kj
. "
(b) From solved Problem 20 F> 2mg(mJ = m2~ [Hint: See § 3,5, 10 (a)]
Fig. 3.209

so,Fnct > 2mg+ mgor=kj = 3mgJ [Ans. aD =6m! 82 and as =1m ! s2]
k ,)]12 andj=m
3 g 29. A 4 kg block is put on the-top of a 5 kg block as shown in
(m y
[Ans.v=y - - ....!
k
]
Fig. 3.209 (b). To cause the top block to slip on the bottom
one, which is held fixed, a horizontal force of at least 12 N
24. A block of mass 2 kg is pushed by a horizontal force of 2.5
must be applied to the top block. The assembly of blocks is
N on a floor. What is the force of friction between the block
now placed on a horizontal, frictionless table. Find (a) the
and the floor if coefficient of static friction is 0.4?
maximum horizontal force F that can be applied to the
(MNR1995( lower block so that the blocks will move together and (b)
[Hint: f = F as F < ~R] the resulting acceleration of the blocks.
[Ans. 2.5 N] [Hint: See § 3.5, 10 (b)]
25. A piece of ice slides down a 4SQ incline in twice the time it [Ans. (a) FII'IIIX = 27 N, (b) 3 m! s2]
takes to slide down a frictionless 45° incline. What is the 30. A block of mass M rests L
'
coefficient of kinetic friction between ice and incline? on a smooth horizontal
(Ans. ~ ~ 0.75J surface over which it can M
j
26. A block slides down an inclined plane of slope angle €I with move without friction. A ,
constant velocity. It is then projected up the same plane
body with mass m lies on a
the block as shown in Fig. 3.210
with an initial velocity vo. How far up the incline will it
Fig. 3.210. The coefficient of friction between the body and
move before coming to rest?
the block is}l. (a) For what force F applied to the block in
[Ans. v~ / (4gsine)] horizontal ~direction, the body begins to slide over the
27. In Fig. 3.208 shown, the blocks A,B andCweigh 3 kg, 4 kg block? (b) In what time will the body fall from the block if
and 8 kg respectively. The coefficient of sliding friction the length of the block is L?
between any two surfaces is 0.25. A is held at rest by a [Hint: See § 3.5, 10 (b)]
massless rigid rod fixed to the wall while Band Care
connected by a light flexible cord passing around a fixed [Ans: F > )l(m+ M)g and t = .1-=,--.::'M
=L=c-.J
F - )l(m +'M)g
pulley. Find the force F necessary to drag C along the
horizontal at constant speed. 31. A rock-climber (49 kg) is
managing to be at rest between
two vertical rocks pressing one
rock A by his feet and the other A
B by his back. If the coefficient
of friction between his shoes
and wall A is 1.2 while between
his back and wall B is 0.8 and
limiting friction acts at all the
contacts, then: (a) What is his
Fig. 3.208 push against the rock? (b) What
Fig. 3.211
[Hint: See Solve Problem 29J per cent of the weight is
supported by the frictional force on legs?
[Ans.8gN]
[Ans. (a) 240 N, (b) 60%]
32. A box 1.5 m in height, 1 m wide and 0.7S mdeep containing 37. Detennine the minimum coefficient of friction between a
some material, sits vertically at the back ofa truck. The total thin homogeneous rod and a floor at which a person can
weight of the box can be assumed to be unifonnly slowly lift the rod from the floor without slipping to the
distributed throughout the volume of the box. The box is vertical position applying to its end a force perpendicular to
tipped over by an acceleration of the truck. What is the it.
minimum value that this acceleration must have? .fi
[Ans.gI2] [Ans.).l ="'4]

33. A wheel of radius 40 em rests against a step of height 20 em 38. Masses mj , m2 ,m) and m4 are arranged in a system as
as shown in Fig. 3.212. What is the minimum horizontal shown in Fig. 3.2 14 wherem, +m2 >m) +m4 . The lower
force which if applied perpendicular to the axle will make string keeping the system in equilibrium is burnt. The
the wheel climb the The mass of the wheel is 2 kg. masses stan moving. Find the acceleration of masses if the
threads are weightless and inextensible. The springs are
also weightless and the mass of the pulley is negligible.
f---+F

Fig. 3.212
lU
W.j h("'2R'
- "'h) 96,f)Nl
[Ans:Fmin= (R -h) =1.

34. A man of weight W stands on the right hand side of a large


beam balance and is just counterpoised by a load on the
other side. A rope is fastened to the mid point of the arm on Fig. 3.214
the right hand side of the balance as shown in Fig. 3.213.
Will the balance be disturbed if the man remaining wherc eml + m2 - m3 -m4)gl
[Ans. a!=az=~=O,a4=
he is, begins to puU the rope with a forceF( < W)at an angle m,
ato the vertical? 39. The three flat blocks in the figure are positioned on the 300
incline and a force parallel to the inclined plane is applied to
the middle block. The upper block is prevented from
moving by a wire which attaches it to the fixed support. The
coefficient of static friction for each of the three pairs of
contact surfaces is shown in the figure. Detennine the
maximum value which P may have before any slipping
takes place.
Ag.3.213
[Ans. The pan containing the man will go up]
35. A force P acts on a body of mass In initially at rest and
initiates its vertical upward motion at t "" 0 The force ~",0.3 -+' ~ = 0.45
decreases at a unifonn rate and is proportional to the
vertical displacement of the body. Find the velocity of the
body at a heighth from its initial position ifa. is the decrease
in force for unit displacement.
Fig. 3.215
f 211 .
[Ans.v=r;;:;-(P) - mg -
olJ}"2 ]
2 [Ans.P =93.8 N]
36. Four bricks each of length I are put one on top of the other, 40. A solid body moves through air, at very high speed, V,
such that a portion of each brick extends beyond the one faster than the velocity of molecules. Show that the drag
below it. Find the maximum length of the overhanging part
force on the body is proportional to AV 2 , where A is the
ofeach brick, when the entire system remains in equilibrium.
frontal area of the body.
[Ans. 112,114,116]
41. In the Fig. 3.216 shown a ).Is" 0.5 ).Is" 0.3 minimum values of F for which the block does not slip.
constant force is applied 0.4 ,l-,:;5::,kg!....i;
).Ik .. F ~",O.1 Take (g=lOm/s2)
on the lower block, just 15 kg
[Ans. 64 N. 31.S NJ
large enough to make this
Fig. 3.216 46. An empty cylinder of mass M 0 is moving on a smooth
block sliding out from
between the upper block and the table, Detennine the horizontal surface with velocity VO' The rain is falling
acceleration of each block. vertically at a rate of n drops per second per square metre.
Each rain drop has a mass m and is falling with a tenninal
velocity VR .
42. A trolley accelerates down a hill,
(a) Calculate the speed of the cylinder as a function of
going from rest to 30.0 mls in 6.00 s.
time.
During the acceleration, a pendulum
bob hangs from the ceiling of the (b) Calculate the nonnal force of reaction of ground on
trolley. Acceleration is such that the Fig. 3.217 cylinder as a function of time.
Siring remains perpendicular to the ceiling. Delennine (a) MoVo
IAn$. Vet) = ;FR = Mg + nAm(VR + gt)up- wards]
e
the angle and (b) tension in the string. (Mo + mAnt)
(ADS. (a) 30.7°; (b) 0.843 N] 47. A mass m1 is connected by a weightless cable
43. One end of an unstretched spring of over a frictionless pulley to a container of
stiffness constant kl is attached to the waler which initially at (/ = O)had a mass mo'
ceiling of an elevator and a 1.50 kg mass Now the system is released, water is ejected
is connected to the other. The ends of r.==;=~==; in the downward direction at a constant rate k
m,
another unstretched spring of stiffness kgls with a velocity Vo relative to container.
constant k2 are attached to the bottom of k, Find the acceleration of nit as a function of
time.
the mass and to the floor of the elevator as :~~~ m(t)
In] -m + kt)g + kv
shown in Fig. 3.218. When the mass is ( Ans. -o o
-] I,,~ k kg/s
released the top spring stretches 40 cm (1/1] +mo-kl)
Fig. 3.221
and the bottom spring is compressed by
this same amount. When the elevator 48. A very flexible chain of mass M and length I is suspended
accelerates upward, the top spring vertically in a lift so that its lower end is just touching the
stretches an additional 8.00 cm. (a) What Fig. 3.218 surface of the floor. When the upper end of the chain is
is the elevator's acceleration? (b) What released, it fa lls with each link coming to rest
must be the acceleration of the elevator to stretch the instantaneously. Find the force exerted by the floor of the
springs to a total length of 52.0 cm from their unstretched lift on the chain at the moment when one-fourth of the chain
lengths? has already landed on the floor. Assume the lift is moving
with acceleration g 12.
(Ans. (a) 1.96 m I s2; (b) 2.94 m I s2]
(An$. 9Mg/8]
44. A IOkgbiockrestson a5 kg
bracket as shown in Fig.
3.219. The 5 kg bracket rests 5k9~ 49. A solid homogeneous cylinder of height II and base radius r
is kept on a conveyor belt moving horizontally with an
on a horizontal frictionless . ! increasing velocity, v = a+ bt 2 . The coefficient of friction
surface. The coefficients of Fig. 3.219 between the cylinder and belt is~.
friction between the 10 kg
block and the bracket on which it rests are j.1 s = OAO and
J.l k = 0.30. (a) What is the maximum force F that can be
applied if the 10 kg block is not to slide on the bracket? (b)
What is the corresponding acceleration of the 5 kg bracket?
[Ans. (a) 24 Nj (b) 1.6 m I s2]
45. A block of mass 0.5 kg rests on a
~S-i; Fig. 3.222
wedge of mass 2 kg as in Fig. 3.220. 9
2 kg (a) Find the time when the cylinder starts slipping.
The wedge is acted on by a horizontal
force F and slides on a frictionless L _ _ _~30''!:L'''' (b) If the cylinder is not allowed to slip, find the time t
surface. If the coefficient of static Ag.3.22O when it is on the verge of tipping over.
friction between the wedge and the block is Il = 0.8, and [Ans. (a) )lgj (b) gr ]
the angle of inclination is 30°, find the ma~imum and 2b bh
50. A brick slides down an inclined plank at constant velocity 54. A block B has mass m, cart C has ~Fa .B '.
when the plank is inclined at an angle 9 0, If the angle is mass M, and the coefficient of C
increased to 9 1, the block accelerates down the plank with static friction between the cart and ~
acceleration a. The coefficient of friction is same in both block is Il s ' Determine an Fig. 3.226
cases. Given 9 0 and 9,. calculate Q. expression for the minimum value
[Ans.g sin (91-9!1)) of Fa such that the block will not slide.
cos So
[Ans. Fll min = mg
• 1-1,
(1 + ~l]
M
51. In the Fig. 3.223 shown, m2 =2 .5 kg, h = 1.5 m, the system
is released from rest at I = Oand the mass m2 strikes ground 55. A b lock of mass 2.20 kg is
at l =0.825. The system is placed in its initial position and a accelerated across a rough
1.2 kg mass is placed on top of the block of mass m], surface by a rope passing over a T
Released from rest, the mass m2 now strikes the ground 1.3 pulley as shown in Fig. 3.227. ...,1"'--
M
x
s later. Detennine the mass ml and the coefficient of kinetic The tension in the rope is 10.0
friction between In, and the shelf. N, and the pulley is 10.0 cm Fig. 3.227
above the block. The coefficient
of fri ction is 0.4.
(a) Determine the acceleration of block when x "" 0.4 m
(b) Find the valueofx at which the acceleration becomes
zero.
[Ans. (a)O.8Sm/s 2,(b)7.3S]
Fig. 3.223 56. There is a rectangular plate of - b_
mass M kg of dimensions (a x b). /
(Ans. 1.22 kg; 0 .672J
The plate is held in horizontal a
52. Two blocks of masses nil = 2 kg and m2 = 5 position by striking n small balls ~ rI
kg hang over a massless pulley as shown in each of mass m per unit area per I'/r ---''-''"-'c-J I
the Fig. 3.224. A forceFo = lOON acting at
the axis of the pulley accelerates the system
unit time. These are striking in the tf f
shaded half region of the p late. Fig. 3.228
upwards. Find (a) the acceleration of each The balls are colliding e lastically
mass, (b) the tensio n in the string. with velocity v. What is v?
5k.
[Ans. (a) Uz = l S.2m/s2 upward,os a O.2m /s2 Itis g ivenn= IOO,M=3 kg,m = O.OI kg;b =2 m;a = 1m;
Fig. 3.224
upward; (b) SON] g",;lOm/s 2 . IJlT2006)
53 . Two blocks I and 2 are sliding down a ~ [Ans. 10 mls]
plank, see Fig. 3.225. Each block has ~:---..
57. A circular disc with a groove
the same mass ml but the coefficients v
a long its d iameter is placed a _ 25 m/s2
of kinetic fri ction between the blocks 9 '
horizontally. A block of mass 1
and surface are different with Fig. 3.225 kg is placed as shown. The 0
J.l2 > lll' The system accelerates
coefficient of friction between
down the slope and the string remains taut. Find (a) the
the block and all surfaces of cos e .. 4/5
tension in the string, (b) the magnitude of the system' s
groove in contact is ~ = U5. sin e ... 315
acceleration, (c) the angle 9 for which the blocks slide down
The disc has an acceleration of FIg. 3.229
with constant velocity.
25 m /s 2 . Find the acceleration
1
[Ans. (a) T = "2(J.l2 - J.1] )mg cos 9; of the block with respect to disc. [lIT 2006]

(b) a '" g[ sin a-~(i.i I J


+ ).l2)cos{)
[Ans.10m/s 2]
58. Cite an example of (a) constant force, (b) position
dependent force, (c) velocity dependent force and (d) time
a
(c) = tan- [1
1 ().I 2 + J..l 1)]J
dependent force.
[Ans, (a) Tension in a massless string, (b) electric or gravitational
force between two charged panicles, (c) viscous force, (d) force
acting in a collision.]
59. (a) Does equilibrium mean that a body is at rest? Now as the spring is massless, F' = F and as only one-third pan o'
(b) Can a body be in equilibrium when onlyane force acts
it is considered, y - (yI 3}
on it? So Eqn. (ii) becomes F - k ' (y I 3)
(e) What are conditions of equilibrium?
which in the light ofEqn. (i) yieldsk' = 3k]
[Ans. (a) No; body may beat rest or in uniform motion, (b) No; as 66. For a spring what will be the graph between (a) force am
for equitibrium r.F " 0, (c) IF - 0 and r.t '" 0.] stretch and (b) potential energy stored and stretch?
60. A hollow pipe has one or two extra weights attached to its (Ans. (a) Straight line, (b) Parabola]
inner surface as shown in Fig. 3.230. Find which of these 67. An object is attached to a vertical spring and s lowl~
represents stable, unstable and neutral equilibrium, lowered to its equilibrium position. This stretches thi
[Ani. (a) Stable, (b) Neutral, (e) Unstable; considering potential
spring by an amount d. If the same object is attached to thi
energy or position of centre of gravity.] same vertical spring but permitted to fall instead, througJ
what maximum distance does it stretch the spring?

m~m
(Hint: See 8 (a) and (b) in § 3.4]
(Ans.2d]
68. Two springs have their force constants as kl and k
(k 1 > k2 } In which spring is more work done (a) when thei
(a) (b) (e) lengths are increased by the same amount? (b) when the:
Fig. 3.230 are stretched by the same force?
[Hint: See (7) in § 3.4]
61. If a match box is placed on a table in different possible
positions, in which case will the equilibrium be most (Ans. (a) *1' (b) *2J
stable? [Fig.l .IO) 69. Is it unreasonable to take the coefficient of friction to b
[Ans. When centre of gravity is lowest or base area maximum] greater than unity?
62. A clothes line is hung between two poles and a shirt is hung (HIDt: See § 3.5J
near the centre. No matter how tightly the line is stretched it (Ans. No; 1.1 can be greater than 1)
always sags a little at the centre. Explain why? 70. 11 is known that polishing a surface beyond a certain limi
[Hint: See 5 (d) in § 3.3] increases rather than decreases the force of frictior
6l. (a) Can the tension in a string be different at different Explain.
points? (b) Can the accelerations of two masses connected [Hint: See 7 in § 3.5]
by a string be different? (c) Can the tension in a loaded 71. State whether the following assertion is tcue or false givin
string be greater or lesser than the load? reason in brief-'When a person walks on a rough surfac!
[Hint: See (2), (I) and 5 (b, c) in § 3.3] the frictional force exerted by the surface on the person i
[Ans. (a), (b) and (c) all yes] opposite to the direction of his motion.
64. Four boxes each weighting 100 N are suspended from a [Hint: See 9 (a) in § 3.5]
beam as shown in Fig. 3.231. What is the tension in each of (Ans. False]
the strings? 72. Can the force of friction on a body be zero even if it is on
[Ans. (a) 57.7 N, (b) 100 N, (c) 50 N and (d) 100 N] rough surface?
(Ans. Yes; When it is at rest and forces acting on it (excludin
friction) balance each other]
7l. A block rests on a rough inclined
plane as shown in Fig. 3.232. A F
horizontal force is applied to it.
(a) Find out the force of reaction . ••_ . B .• •_ __
Fig. 3.231 (b) Can the force of friction be
zero? If yes, when? (c) Assuming Fig. 3.232
65. A spring of force constant k is cut into three equal parts.
that, friction is not zero, find its magnitude and the directio
What is the force constant of each part?
of its limiting value.
[Hint: Consider the spring to be made up ofthrce equal parts; Ihen
[Ans. (a) R "" mgcos8+ Fsin8, (b) Yes, if mgsin8 "" Feose, (,
for full spring,
f"').1R" Il(mgcos8+F sin8); up the plane if the bOdy hi
F=ky ... (i)
tendency to slide down and down the plane if the body hi
while for one part of ii, F' =k'y ... (ii) tendency to move up.]
74. A body A r~sts on B and friction exists between A and B 77. Fill in the blanks:
only. Find the maximum value of F in case (A) and (8) as (a) A proton and an alpha particle enter into a region of
shown in Fig. 3.233 so that both move together. uniform electric field. The ratio of the forc e on the
A~ F m A proton to that on the alpha particle is ....
(EAMCET 1990]
B M M B F
(b) f.1 s ,ll k and Ilr are coeffi cients of static, kinetic and
rolling fric tion between two surfaces. In their
(a) (b) increasing order of magnitude one can write ....
Fig. 2.233 (EAMC ET 1992]
(Hint: See § 3.5 (IOA·II, B·II)] (c) A cube of 10 N rests Qn a rough inclined plane of slope
(An"(A) Fmax =f.UI1B, (B) Fmax =Jl(m+M)g] 3 in 5. The coefficient of friction is 0.6. The minimum
force necessary to start the cube moving up the plane is
75. A cube is resting on an inclined plane. If the angle of (EAMCET 1990]
inclination is gradually increased. what must be the
[Hlnl: F = mgsinO+ flmgcosO]
coefficient of friction between the cube and the plane so
that (a) the cube slides before toppling and (b) topples Cd) An impulse is supplied to a moving object with the
before sliding? force at an angle of 1200 with respect to velocity
[Ans.(a)p < ~(b)Il > 1] vector. The angle between the impulse vector and the
change in the momentum vector is ....
76: Explain why;
(EAMCET 1990]
\ (a) A cricketer moves his hands backwards when holding -+ -+ -t -t
aeatch. [Hint: J = 6 p , so, directions of J and 6 p are same]
[Hint: See § 3.7, 5 (b)] (e) The magnitude of the force (in N) acting on a body
(b) A man who falls from a height on a cemented floor varies with time t (in J.1S) as shown in Fig. 3.234. AB;
receives more injury than when he falls from the same BC and CD are straight line segments. The magnitude
height on a heap of sand. of the total impulse of the force on the body from
[Hint: See § 3.7, oS (c)] t = 4IlSIO t = l6f.1s .... N-s. lilT 199'41
(c) It is easier to open or close a door when force is applied
at maximum distance from the hinge and normal to the
plane of the door. t 800 C

(Hint: See § 3.8, (5)] ~ 600


~
(d) A logger makes a cut on the side facing the direction in
which he wants the tree to fall. (Would it be safe to ~ 400
200 A B
stand directly behind the tree on the opposite side of
the fa ll?) "' D
[Hint: Due to cut, weight of the tree will exert a torque about the 2 4 8 10 12 14 16
point of con tact (which acts as hinge) and so the tree will tum in Time (fls) -
the direction of cut However, while turning, the tree may slip at Fig. 3.234
the point of contact and so, it is not safe to stand directly behind
the tree.] [Ans. (a) 1 ; 2, (b) fl T 'fl.\' Rnd I-Is' (c) 10.8 N, (d) 0°, (e) oS x 10- 3
N·s)
Conservation of Energy
[There can be no doubt that over a period of time the ........
W= F· s =Fscos €l .... (i
objects around us do change in such respects as form and
position. How these changes occur is described by certain laws However, if the force is not constant
of physics . The human mind, however, has always sought to ........
find those properties of physical objects that do not change. In W=!F·d s .... (ii
the development of physics this search has been rewarded by Regarding work it is worth noting that:
the discovery of conservation laws. A conservation/aw states ( 1) Work is a scalar (as it is dot product of two vectors
that the value of some quantity such as energy, linear
having dimensions [MLl-r-2 ] and SI unit N-m whic:
momentum, angular momentum or electric charge does not
change with time, i.e., it remains constant.] is more specifically called joule (J). The COS unit 0
work is erg and is related to joule through the relation
The advantages of studying physics in terms of
conservation laws are: IJ =107 erg
(i) Conservation laws are efficient tools which enable us A practical unit of work in atomic and nuclear physic
to predict significant and useful general consequences is electron volt (abbreviated eV) with
of the equations of motion even without an exact · leV =1.6x lO- 19 J
knowledge of the trajectories.
(ii) Conservation laws can be efficiently used even when
the forces involved are not known, e.g., though we do
not know the exact nature of nuclear forces, we solve
problems of nuclear forces by applying conservation (a) (b) (e)
laws. ~~
W ", F · 8. Fs cos a Posltive work Negative work
(iii) Conservation laws can also predict the impossibility Fig. 4.1
of certain happenings. For example, the laws of
conservation of linear momentum and energy ensure (2) Work can be positive or negative as €I is acute « 90
that y-rays can never produce an electron positron pair or obtuse (> 90°). Positive work means that force (I
III vacuum. its component) is parallel to displacement whi'
negative work means that force (or its component)
Here, it is worthy to note that conservation laws are
opposite to displacement. Thus:
empirical and our confidence in them rests on their
having proved true through various tests. (a) When a person lifts a body fro m the ground, It
work- done by the (upward) lifting force
§ 4.1 Work and Power positive, but the work done by the (downwar,
In our daily li fe 'work' implies an activity resulting in force of gravity is negative.
muscular or menta:! exertion. However, in physics the term
(b) Also when a body is pulled on a fixed rou~
'work' is used in a specific sense which involves the
surface, the work done by the pulling force
displacement of a particle or body'" under the action of a force.
positive while by frictional force (kinetic)
.... negative as pulling force is always in tl
Ifunder the action ofa force F, a particle or body is displaced
direction of displacement whi le frictional force
by 1, the scalar product offorce Wilh displacement is defined opposite to displacement.
as work, i.e.,
• In this chapter body means an object which can be treated as point mass.
Note: Ifa body is on a body and moving with it, the force of friction on sO,networkW=W! +W2 +W3 =O+Fs-fs
the upper body is in the direction of displacement while on the : (F-f)s
lower body is opposite to the displacement. So, work done by
friction (which is non-conservative) on the upper body relative to Now, ifthe body is in dynamic equilibrium I = F,
ground is positive. so,W=O.
-> -> ->
(3) Ifa number offorees act on a body orparticie then: (4) As W = IFods so, if ds =0, W=O i.e., if the
W=Wj +W2 +W) + ... displacement ofa particle or body is zero whatever be
-+ -+ -+ - -+ the loree, work done is zero. This is why:
or W: IF, ods+ IF, ods (a) When a person tries to displace a wall or stone by
applying a force and it (actually its centre of
Of ,
mass) does not move, the work done is zero.
or ....(iii) (b) A weight lifter does work in lifting the weight off
the ground but does not work in holding it up.
Work done in displacing a particle under the action of
a number offorces is equal to the work done by the (S) As W = IF ds cos e, so W =0, ife =90°, i. e., if force is
resultant force. Further if the body is in equilibrium always perpendicular to motion, work done by the
(static or dynamic) W=I:W=O as for equilibrium force will be zero though neither force nor
FR =0. Here work done by individual forces mayor displacement is zero. This is why:
may not be zero, but work done by some forces is (a) When a porter moves with a suitcase in his hand
positive while by others negative, and positive work on a horizontal level road, the work done by the
is equal to negative work so that net work becomes lifting force or force of gravity is zero. [Fig. 4.3
zero, e.g., (a)l
(a) If we lift a body from rest to a height h (b) When a body moves in a circle the work done by
[Fig. 4.2 (,)1 the centripetal force is always zero. [Fig. 4.3 (b)]
Work done by lifting force F. Wi =Fh cosoo=Fh (c) When the bob ofa simple pendulum swings, the
Work done by force of gravity mg. work done by tension in the string is zero.
W2 = mgh cos1800= - mgh (d) In case of motion of a charged particle in a
-> ->->
So, net work magnetic field as force F[ = q( vx B)] is always
W : Wj +W2 : Fh -mgh:(F-mg)h perpendicular to motion, work done by this force
Now, if the body is in equilibriumF =mg, w=o is always zero.
B

! /' ---fr'
.
,I F ",,
ds
9=90°

,,, ,
,, ,
1
A
F ,,
,,
..... _--; , ,
(b)
(.) (b) W. C as 9 .. 900
Fig. 4.2 Fig. 4.3

(b) If a body is moving on a horizontal rough road


Note: In circular motion IVOrk done by centripetal (i.e., radial or
[Fig. 4.2 (b)l ccntral)!o/,ce is always zero (as 9= 900); so, ki net ic cnergy and
Work done by reaction and gravity speed remains constant (whi le velocity and momentum changes
due to change in direction) and motion is unifonn. However, ifin
Wj = 0 as force is .1 to -; add ition to radial force there is also a ulIIgentlal!orce, lVork dOlle
by tmlgen/la/force will /lot be zero and so, KE will not remain
Work done by pulling force F, constant (but ~ = ll.KE) and motion will not be unifonn .

W2 = Fs cosoo= +Fs
(6) Work depends ol/frame ofreference. With change of
Work done by frictional force/,
frame of reference (inertial) force does not change
W3 = ftcos1800 = -fs
p
while displacement may change; so, the work done by Expansion
a force will be different in different frames, e.g., w. Positive
(a) If a porter with a suitcase on,his head moves up a A
staircase, work done by the upward lifting force
relative to him will be zero (as displacement p p
relative to him is zero) while relative to a person ! V,-dV,..--tV', + V
'---+'
-> ...
on the ground will be (F · s)=mgh(as F=mg
la) Ib)
and s= h).
(b) If a person is pushing a box inside a moving train, Compression
. -> -> w. Negative
the work done in the frame of train will be ( Fo s)
while in the frame of earth will be F. (t + ;;)
where ~ is t~e displacement of the train relative
to the ground.
Ie)
(7) In a conservative field work is path independent, e.g., Fig. 4.5
,-
if we shift a body in equilibrium from A to C in -a
gravitational field via path AC or ABC, the work done the work done in a small displacement dy oftbe piston
...
by the force F for both paths will be same and equal to
dW=Fdy=pAdy=pdV ["A dy=dV]

mgh. [Fig. 4.4 (a)J


so, W =fV pdV
2
v,
Note: (i) For path AC ,WAC =. Fscos (90°_9) '" mgh but pdV represents the area' of a strip under pv curve,
[as IFI"" mg and scos(900-9) '" ssin 8 = hJ so, W= tlv,
pdV =Area under p I V curve .... (v)
While for path AB, WAS =Fa cos90o= 0
work is positive if V2 >V, .i.e.. gas expands and
and for path BC, WSC '" Fh cosO"", mgh [as IFI= mg)
negative ifV2 < VI. i. e., gas is compressed.
sotha!, WAB +WBC",O+mgh=mgh", WAC
(10) By definition,
(ii) As WA C =mgh, work is also independent of the slope of the

F-ds-> =1ma-ds
inclined plane and depends on height h.
W= 1-> -> ->

-> -> -> ->


(8) In F versus 5 curve [Fig. 4.4 (b)] as [as fro m Newton's II law F = mal
-> -> ->
F-ds =dA -> -> dv
dvo d,->
W= 1m-
... -> or [as a =- J
dt dt
so, W = f F-d s =f dA =Area under Flscurve .... (iv)
IJF -+ -+ -> ->
i.e., area under Fls curve with proper algebraic sign or w= Jv, mv·dv [as ds/dt = v]
represents work dOlle by the force.

C~
I 2 I 2
or W= - mv F - - mv J =KF -K J = 11K .... (Vi.
2 2

~. '~+;
i.e.. work dOlle all a particle by the resultant forcl
acting 011 it is always equal to the change ill kinetic
d, , energy of tire particle. This theorem is called work·
-F energy theorem. Regarding work-energy theorem i
is worth noting that:
la) (b) (a) If work is positive, KF > K 1 • so, the kineti.
Fig. 4.4 energy will increase while if work is negative
K F < 'K I , so, the kinetic energy will decrease.
(9) As the force exerted by a gas (due to its elasticity) in a
(b) This theorem is also valid for a system in preseno
cylinder on a piston of area A is
of aU types of forces (ex.ternal or internal
F = pA ["p =F1AJ conservative or non-conservative).
(11) By definition, work ,done by a force does not depend rowing and moves down with the stream, is any work being
on the time taken for a given displacement, i.e., work done on him?
is independent of time. The time rate ofdoing work is Answer: (a) As the displacement of the boat relative to
defined as power and is given by the shore is zero (static equilibrium), so work done by his force
(Power) ilut. == d; and (Power) avo = ~ .... (vii)
(or force of flow of stream or by net force) is zero, i.e., he is
doing no work (though he is applying a force).
Regarding power it is worth noting that: (b) If he stops rowing, the force of flow will produce
~ ~
displacement relative to the shore in the direction of flow of
~ ~ water. So, work will be done by force of flowing water and will
(a) Power P = dW = F·ds [asdW =F·ds )
dt dt be positive (and so, kinetic energy will increase).
~~ Question III. Mountain roads rarely go straight up the
i, e., P = F-v [asd1/dt =~] .... (viii) slope but wind up gradually. Explain why?
i,e., power is equal to the scalar product offorce Answer: As gravitational field is conservative, the work
with velocity. done in a gravitational field is path independent, i.e., the work
(b) Power is a scalar with dimensions [ML2T - J ]. done in going from bottom to the top of a hill is the same
The SI unit of power is Jls and is called watt (W). whatever be the path followed, i.e.,
Practical unit of powel is horse-power with W=constant
1 hp = 746 watt =550 ft-Ib/s 0' Fs = constant (assuming8 = 0)
Work Power So, if we increase the length oftbe path by decreasing the
slope, the force needed will be small. On the other hand, if we
decrease the length of the path by increasing the slope, the
force needed to do the same work will be large. However the
magnitude of force that can be applied by man o~ machine is
, limited. So, it is more practical and economical to increase the
e: length of the patb rather than force to do a given amount of
TIme 11m,
(a) (b) work. Actually tbe increase in path length dil)Jtes the force
Ffg.4.6 needed. Thi~ is why the mountain roads, generally wind up
gradually and rarely go up straight.
(c) As power = (workftime), ally unit a/power multi-
Question lV. What is the stopping distallcefor a vehicle
plied by a unit of time gives unit of work (or
oj mass m moving with speed tI alollg a level road, if the
energy and not power), i.e., kilowatt-hour or
coefficient oJfriction between/he lyres and the road is Il?
watt-day are units of work or energy (and not of
power) with Answer: When the vehicle of mass m is moving with
velocity v, the kinetic en~rgy of the vehicle K = (1I2)mv 2 and
1 kWh =lO'x (lIs) x (60 x 60s) =3.6 x 106 J
if s is the stopping distance, work done by friction
(d) The slope of Wi t curve gives the instantaneous
W = Is coss = ~mgs cos I SOo=;.amgs
power (as P =dWldt = tan e) (Fig. 4.6 (a)] while
the area under PIt curve gives the work done as So, by Work-Energy Theorem, W=I1K = K F - K j
P=dWldt which means W =IPdt= area under 1 2
i.e. , -llmgs =O--mv
P I I curve) [F;g. 4.6 (b»). 2
Question I. Suppose thai the earth revolves around the
sun in a perfectly circular orbit. Does the slln do any work on Note: (i) Alternate proof: As fric lion is the reta rding force, the
retardation
the earth?
f IlR Ilmg
a=/ij "'m = Ii/''''Jlg",constant .... (i)
Answer: When the earth revolves around tbe sun in a
perfectly circular orbit the force of the sun on tbe eartb is along So equations of motion are valid. Now if s is the stopping
tbe radius wbile displacement of the eartb is along tbe tangent distance, then from equation of motion v} ;. v~ + 2as, we
to the orbit. Now in a circle radius and tangent are always hn ve,
orthogonal, i.e., angle between them at any point on the orbit is
90" and so, work done by the sun on the earth is zero.
Question II. A man rowing a boat upstream is at rest with i.e. • [as from Eqn. (i) a '" Il,gj
respect to the shore. (a) Is he doing allY work? (b) Ifhe stops
(ii) As s" L2p.g '" (1I2)mv .. K [as K .. _,I mv 2 and F .. ~mg]
Ilmg F table:
So, if a light and a heavy body are moving with same kinetic o mgydy =mgL2
energy and same retarding force is applied bOlh will stop W=-) --
after travelling same distance.
Un 2112
or W=MgLl2n 2 [asM =m1.]
Question V. A pump motor is used to deliver water at a
certain n.tefrom a given pipe. To obtain 'n' times water from Problem 2. A 100,000 kg engine is moving up a slope of
the same pipe in the same time by what amount (a) the force gradient 50 at a speed oj 100 mlhr. The coefficient offriction
and (b) power of the motor should he increased? between the engine and rails is 0. 1. if the engine has an
Answer: If a liquid of density p is flowing through a pipe efficiency of4%Jor converting heat into work, find the amollnt
of cross-section A at speed v the mass coming out per sec will oj coal lire engille has to bllm lip in olle hOllr (bllrning oj
be: I kg ofcoaf yields 50,000 joule).

(dm I dt) = Avp Solution: The forces acting


on the train are as shown in Fig.
So to get n times water in the same time
4.8. As the train moves along the
(dm/dl)' = n(dmldl) plane, the work done by Rand mg
i. e., A' 11 p' = nAvp cos 8 is zero (as these forces are
but as pipe and liquid are same p' = p and A' = A perpendicular to s). Now as the L""-_-:::--:=:-_ _ _
train is moving up, force offriction Fig. 4.8
lI=nv .... (i)
will act downwards (opposite to motion) and will be
(a) Now as F=v(dm/dl)
f=).1R=).1mgcos8 [asR=mgcos8]
F' _ 11 (dm / dl)' _ nv(n dm / dl) ,
so - - - = 11 Further as the velocity of the train is constant, so there is
F v(dm/dl) v(dm/dl)
dynamic equil ibrium.
i.e., .... (ii) i. e., F = mgsin9 + ).1 mgcos9
(b) And as or F = lOS x 9.8 [0.087 +0.1 x 0.996]
P' F'v'
-=--
P Fv
i.e., F ",,1.83 x lOs N
Now as the displacement
i.e., p'=,,3 p .... (iii)
s = vx t =100(mJhr) x I hr =100 m
So to get n times water, force must be increased n 2 times The work done W=FscosOO
while power n 3 times. =1.83 x lOs xlO 2 =1.83x 10 7 J
. Problem 1. A chain is held on a Jrictionless table with
if the chaill has a
(I / n) th of its length hanging over the edge.
But as efficiency of engine TJ = Output I Input WIH =
length 'L' and a mass 'M', how much work is required to pull So, Input H =1.83x I0 7 x l()QI4=4.56 x I0 8 J
Ihe hQJlgingparl backollthe table? (MNR 19901
Now as 5 x I 0 4 J is produced by burning of I kg coal,
Solution: If m is the mass per unit length of the chain,
4.56 x 10 8 J is produced by burning of (4.56xI0 8/5xI0 4 )
=9.l x I0 3 kg coal.
Problem 3. A point charge q movesfi'om point P to point
S along the path PQRS in a uniform electric field E pointing
parallel 10 the positive direction of the x-axis. The
co-ordinates ofthe points p. Q. R alld S are (a, b, 0). (2a, 0, 0).
(a, - b, 0) and (0,0,0) respectively. Calculate the work done by
Fig. 4.7 thefield ill the above process.
the mass-uflhe chain oflengthywill be ymand the force acting Solution: Electric fi eld is conservative, so work done in
on it due to gravity will be mgy(assuming thaty is the length of moving the point c harge q along PQRS will be equal to the
the chain hanging over the edge). So, the work done in pulling work done along the path PS. But as F =qE, so for path PS
the dy length of the chain on the table W=(qE)s cos(n - 9) = qEsx (- als) =-qEa
dW = F( - dy) [asy is decreasing] [as cos (rr - 8) = -cos8 = -(als)]
i.e., dW= (mgy)(-dy) [asF = mgy] negative sign means force is opposite to displacement.
p
y
,/,,' Note: Alternative solution: As x .. (I - 3)2 is Ihe equation of a
,/ , parabola with vertex al t '" 3 as shown in Fig. 4 .10,
,/ ,
-------r:------- (a) From the graph it is clear that at t =3, .t"'O
S
,,,T Q
x
and v=(dxldl) "" slope:: °
,, ~I. _ __ _ a -----0<.1
(b) Fromequationx=(1 - 3)2
at t "" 0 Xl = 9 and all=6
R So s"x2- x\-9 -9=O
And so W - Fs::.F )(O = O
la) Ib)
Fig. 4.9 Problem S. The earth circles the sun once a year. Ho w
much work would have to be done on the earth to bring it to
Note: Alternative solution: As electric field is conservative, rest relative to the sun (ign ore the rotatioll of earth about its
Wps =WPT +W~ (Fig. 4.9 (b)J own axis)? Given that mass of the earth is 6)( 10 24 kg and
But WpT:(qE)bcos9O"=!)andWrs =qEacos I 80"=-qEa
distance between sun and earth is 1.5)( 10 8 km.
so WPS .. 0+ (-qEa)= -qEa
Solution: As T = (21t I ro)
Problem 4. The displacemellf x of a particle moving in 7 7
so OJ = [2 n1 (3.1 5 x I0 )J = 1.99 x 10- radls
one dimension, under the action ofa constantjorce is related
to time t by the equation:
so V =rro = 1.5 x 1011 x 1.99 x 10- 7 :::: 3 x 10 4 mls
x=(1-3)'
Now by Work-Energy Theorem, W =KF -K/
where x is in metre and t in sec. Calcillate: (a) The
displacement o/the particle when its velocity is zero; (b) The i.e., W = 0 - lmv2 =-! x 6 x 10 24 )( (3 )( 10 4 )2
2 2
work done by the force in the first 6 sec.
= _2.7x 10 33 J
Solution: As x = (t - 3)2 .... (i)
(negative sign means force is opposite to the motion).
So v=(dxldl) =2(1-3 ) .... (ii)
Problem 6. Two idelltical 5 kg blocks are moving with
(a) v will be zero when, same speed of2 mls towards each olher along a frictionless
2(1-3)=0, i.e., 1=3 horizontal surface. The two blocks collide, stick together-and
Substituting this value of tin Eqn. (i), come to rest. Consider the two blocks as a system, calculate
the work dOlle by (a) extemal forces and (b) intemalforces.
x= (3-3)' =0
IMNR 19911
i,e., when ve!oc;ty is zero, displacement is also zero.
~ ->
(b) From·Eqn. (ii), Solution: (a) AsFe~t =0 so Wc~t = Felt! • S = 0
(V)," o = 2(0 - 3) = -6 and (v), .6 = 2(6 - 3) = 6
, (b) By Work-Energy Theorem, W=K F -K/

x-9 ··----------r------------ i. e., We~t +1tjnl =o - ( ~mv2 +~mv2)


(asW= ffinl +Wcxt1
i. e., Winl =_mv 2 = _5)(2 2 = - 20J
(as Well! =0 from part (a)]
9=~O----~~I_~3~----~I_~6~ (negative sign means internal forces on each block arc
Fig. 4.10 opposite·to its motion).
Problem 7. A person decides to use his bath tub water to
So, by Work-Energy Theorem,
generate electric power to run a 40 W bulb. The bath tub is
W=M( = .!.m[V~ - V7 1 located at a height of 10 m fro m Ihe ground and it holds 200
2 litre a/water. He illstalls a water-driven wheel generator on
the groulld. At what rate should the water drain from the bath-
=lrn[6' -(-6)'J=0
2 tub 10 light the bulb? How long call he keep Ille bulb all, if the
i.e., work done by the force in the first 6 sec is zero. bath tub was full initially? The efficiency of the generator is
90%. (g=9.8mls 2)
Solution: If m kg water fa lls from a height h As 2 kg uranium is consumed in 30 days, i,e,,1.514x l0 14 J
W=mgh=Vpgh [as m =Vp] of energy is produced in the reactor in 30 days, i.e.,
So, rate of doing work, i.e., power P 2.592 x lO 6sec. .
dW d dV l
P;, = dl = dl (Vpgh) = pgh{j/ So, Power output of reactor = ddW = 1.514 x : 1. ro
t 2.592 x 10 s
Now as efficiency II = (Pout/Pin), i.e., Pout =llPin
=58.4 MW
3
i, e., 40 = 90· x pgh dV or dV = 40 x l0 m Problem 10. An autbmobife 0/ mass 111 accelerates,
100 dt dt 9xl0 3 x9.8xlO sec starting/rom rest, while the engine supplies constant power P;
dV =~ litre =0.453 .litre [a51m 3 =I03litre] ' show that:
i.e.,
dt 9 x 9.8 sec sec (a) The velocity is given as a/unction a/time by

Further as V=(dV) x t. so t = 200 =441sec v=(2Ptlm)IIZ


dl' 0.453 .
(b) The position is given as a/unction a/time by
Problem 8. The human heart discharges 75 cc of blood s= (8PI9m)liZ 13/Z
through the arteries at each beat against an average pressure
0/10 em a/mercury. AssI4ming that the pulseJrequency is 72 Solution: (a) Given that Power = Fv = P = constt.
per minute. calculate the rate of working of heart in watt.
(density o/mercury =13.6 glee.and g= 9.8 mls 2) .
i. e., m~v ='p [ as F=ma=m~J
Solution: By definition,
or
Powe,(p)=dW = pdV [asdW=pdV]
dt dl which on integration gives:
Here p= hdg =10 x 13.6x 980= 1.3328 x 10 5dyne/em 2 v' P
and (dVldt) =(pulse frequency) x (blood discharged per pulse) . -=
2 -m t+C,
i. e.. (dV/dl) = (72 / 60)x 7S=90 eclsee Now as initially the body is at rest, i.e., v =O at t = 0, so
C, =0.
So, Power of heart = 1.3328 x 10 5 x 90 :~ Z
V=( 2=tr .... (i)
::1.19x10 ~;~ = l.19W
7
(b) By definition v = (dsldt), which in the light ofEqn. (i)
Problem 9. What is the power output ofa 92 U 235 reactor if becomes
it takes 30 days to use up 2 kg offuel and if each fission gives
. .• Ids=1 (2PI)"'
ds =(2PI )"' I.e
185 MeV a/usable energy? (Avogadro number =6xl0 23 Imol) dr m '
--;;; dl

Solution: 1 mole, i.e., 235 g of uranium contains 6x10 23 which on 'integration gives:

.. s=(2P)U2 ~tJ/2+C2
atoms.
So, 2 kg, i.e., 2)( 10 3 g of uranium will contain 1/1 3
3 23 Now as att =0, S=O,50, C 2 =0.
=2 )(10 x6)(10 atoms=5. 106 xI0 24 atoms
235
s= (8P)"2 , 3f2
Now, as in each fi ssion only one uranium atom is 9m .
consumed, i.e., Energy yield per uranium atom::: 185 MeV
::: 185 >< 1.6)( 10-23J =2.96 x 10":'11 J § 4.2 Energy
Energy is defined as capacity 0/doing work. When we say
So, energy produced by 2 kg uranium that a body has energy we mean that it can do work. This is
= (No. of aloms) x (energy/atom) why encrgy is a scalar and has same dimensions [ML 2T - 2]
= 5.1{)6x 10 24 )(2.96)( 10- 11 and units joule (J] as of work. Some practical units of energy
::: 1.514)( 10 14 J and their equivalcnce to joule is given in tabular fonn ahead:
CONSERVATION OF ENERGY 133

S.No. Unit Symbol Equivalent in Joule (3) For a body or particle as by definition kinetic energy
2
I. Kilowatt hou r kWh 3.6 x 106 J K = (U2)mv
Z. Calorie ""I 4.2 J and linear momentum p = mv
3. E,. "g 10-1 J
4. Electron volt ,v J.6 x 10- 19 J So, K=!m(!'.)2,
2 m
j.e., K=J!....
2m
.... (ii)

The most remarkable fact about energy is in its diversity. This is an important relation between kinetic energy
Like a clever actor who can assume many disguises, energy and momentum of It particle or body and shows thaI a
appears in many roons such as . mechanical, electrical, body cannot have kinetic energy without having
chemical, thennal (heat), optical (light), acoustical (sound). momentum alld vice*vcrsa.
molecular, atomic, nuclear, etc., and can change from one fonn (4) Ifa force (conservative or non-conservative, externa'l
to the other. Because of this richness of [onn, energy appears or internal) acts on a body, by Work-Energy
in nearly every part of the description of nature either Theorem
macroscopic or microscopic. Conversion of electrical energy
!ll( = W
to other fanns and vice-versa along with the devices which do
it is illustrated in Fig. 4.11. If kinetic energy of the body increases work is
~-- positive, i.e., body moves in the direction afforce (or
MeChanical,~::;M:o:lo:r=;( He<'.ter Thermal field) and if kinetic energy decreases work will be
Energy Dynamo Thermo- Energy negative and object will move opposite to the force
Energy
couple (or field) this is why:
(~ (a) In case of vertical motion of body under gravity
~,,~ ~ when the body is projected up, force of gravity
90...' 1 ~+.&,. (conservative) is opposite to motion and so
0
". 'J'-----, kinetic energy of the body decreases and when it
Optical Chemical Acoustic falls down, force of gravity is in the direction of
Energy Energy Energy motion and so kinetic energy increases.
Flg_ 4_11 " (b) When a body moves on a rough . horizontal
surfa~e, as force of friction (non-conservative)
§ 4.3 Kinetic Energy acts -opposite 10 motion, kinetic energy will
Kinetic energy is"a scalar property that is associated with decrease and the decrease in kinetic energy is
state ofmotion ofa" object. An aeroplane in straight and level equal to the work done against friction,
flight has kinetic energy oftralls/ation and a rotating wheel on
a
a machirie has kinetic ellergy ofrotation. If particle of mass In Note: In case of a system of particles KE = I:~ miv; can be written as
is mov'ing with velocity v «<
c)'" the kinetic energy K is given
the sum o f t\"o terms ( I) the energy associated with the centre of
by
mass motion ~MV~m ' where M is the total mass of the system
I 2
K=-mv .... (i)
2 and (2) the energy of motion relative to the eentre of mass

Regarding kinetic energy it is worth noting that: r1m/1I1, where IIi is the velocity ofa particle relative to centre
of mass, i.e"
(I) As mass m and v 2 (~.1) are always positive, kinetic 1 2 1 2 1 2
energy is always positive scalar, i.e., kinetic energy K =r"2l11iVi '" "2 Mvcm -+- r"2m;U{
call never be negative.
(2) The kinetic ellergy depends all the frame ofreference, § 4.4 Potential Energy
e.g., the kinetic energy of a person of mass In, sitting The .energy stored ill a body or system by virtue of its
in a train moving with speed v, is zero. in the frame of configuration or its position in a field is called potential
train but ( 112)mv 2 in the frame of earth. . energy.

W If v is comparable to c(speed nflight in free space::: 3 ,, 10 8 nlls), according to Einstein theory o f relat ivity

K.m'C'[J
. 1-(vl c) .
1, -I]
Where 1110 is the rest mass of the body.
134 PHYSICS FOR COMPETITIONS - Vol. 1

In case of conservative force as: while for a point charge q at a point in an electric field where
F=-(dUldr)*, i.e., dU=-Fdr the potential is V
U=qV ... (V)
So,
As charge can be positive or negative, electric potential
energy can be positive or negative.
i.e., .... (i) (e) Gravitational Potential Energy: It is the usual form
of potential energy and is the energy associated with the state
Whenever and wherever possible, we take the reference of separation between bodies that interact via gravitational
point at 00 and assume potential energy to be zero there, i.e., if
force. For two particles of masses mj and m2 separated by a
we take 1) = co and U I =:0, then dropping suffix 2* distance r, it is given by:'
I" ~ --+
U=- J00 F·dr= - W .... (ii) mjm2
U =_G •.•. (.)
VI
r
In case of conservative force (field) potential energy is
equal to negative of work done in shifting the body from some which for a body of mas~ at height h relative to surface of
reference position to given position. This is why in shifting a earth reduces to
particle in a conservativefield (say gravitational or electric), if U = mgh .... (vu.. )
the particle moves opposite to the field, work done by the field
will be negative and so change in potential energy will be r
(+ )
;
positive, i.e., potential energy will increase and when the
particle moves in the direction of field, work will be positive where R is the radius of earth. If h« R, (hIR)« I.
and so change in potential energy will be negative, i.e., So, U = mgh .... (viii)
potential energy will decrease. Furthermore, if V is the gravitational potential at a point,
Regarding potential energy U it is worth noting that: the potential** energy ofa particle of mass m at that point will
(1) Potential energy can be defined only for conservative
be
forces. It does not exist for non-conservative forces. U=mV .... (ix)
(2) Potential energy can be positive or negative. Gravitational potential energy can be positive or negative.
(3) Potential energy depends on frame of reference. Note: Positive 'gravitational potential energy means that the body will
(4) A moving body mayor may not have potential do w~rk in returning to its reference position while negative
energy.
(5) Potential energy should be considered to be a property
of the entire system, rather than assigning it to any
specific particle.
In physics, we usually come across of following types of
potential energies:
(a) Elastic Potential Energy: It is the energy associated
with state of compression or expansion of an elastic (spring
like) object and is given by: Fig. 4.12

1 2 potential energy means work will be done on the body to bring it


U =- k y .... (iii) back to the reference position. Gravitational potential energy
2
also depends on frame of reference, i.e., reference position. (See
where k is force constant and y is the stretch or compression. Fig. 4. 12)
Elastic potential energy is always positive.
(b) Electric Potential Energy: It is the energy associated § 4.5 Mechanical Energy
with state of separation between charged particles that Mechanical energy E of a particle, object or system is
interact via electric force. For two point charges q j and q2 defmed as the sum of kinetic energy K and potential energy U,
separated by a distance r, i.e.,
E = K+U
U =_1_ qjq2 (. )
•.•• IV
411:1;:0 r Regarding mechanical energy it is worth noting that:

* Potential energy can also be defined as a scalar function of position whose negative derivative (gradient) gives force.
** Gravitational potential at a point is defined as potontial energy per unit mass.
CONSERVATION OF ENERGY '35

(I) It is a scalar having dimensions [ML 2T -2 J and SI 1 , K


KE =- mv = - .... (i)
units joule. 2 2,
(2) It depends on frame of reference. Now as F=-(dUldr), i.e., U=- -f~ Fdr
(3) A body can have mechanical energy without having

f' ,
either kinetic energy or potential energy. However, if -(Klr)d,=-~
K
[1=- ... (ii)
both kinetic and potential energies are zero, 00 ,

mechanical energy will be zero. The converse mayor . K K


may not be true, i. e" if E :::O either both PE and KE are So, Total energy E=U +K =- - +-
r 21'
zero or PE negative and KE positive such that
i.e., E=-~
K .... (iii)
KE + PE=O. 2,
(4) As mechanical energy E""K + V, i. e., E - V=K. Now
Negative energy means that particle is in bound state.
as K is always positive, E - U:<! 0, i.e., for existence of
a particle in the field, E :<! U. § 4.6 Conservation of Energy
(5) As mechanical energy E = K + U and K is always For a body or an isolated system n by WorkMEnergy
positive, so, if U is positi ve E will be positive. theorem we have,
However, ifpotenlial energy V is negative,
E wil l be positive, if K>IU Jand
Ewill be negative, ifK<IU I But according 10 definition of potential energy in a
conservative field
i. e., mechanical ellergy of a body or system call be
U 2 - U t = - I F·dr
~ ~
lIegative and negative n:echan ical energy means that ... (ii)
potential energy is ilegative and in magnitude it is
more than kinetic energy. Such a state is callcd boulld So, from Egns . (i) and (ii), we have
state, e.g., electron in an atom or a satellite moving K,-K, =- (U,-U ,)
around a planet arc in bound state. i. c., K2 +U 2 = K 1 +U t
Question VI. (a) Call ( I body I/(Ive energy withollt having i.e., K + U = constant ...(iii)
momelltum? (b) Call a body have momentum without havillg Thus, we see that for an isolated system or body ill
energy? Explain. presence of conservative forces the sum of kinetic and
Answer: (a) Yes; if momentum p=O, KE =( p2I2m) = 0 potential energies at any point remaills constant throughout
So, for a body E = [(KE + PE)] will reduce to E = PE which the motion. It does not depend upon time. This is known as the
mayor may not be zero. So, a body mayor may 1I0t have law of conservation of mechanical energy.
energy withollt having momentum. From this law it is clear Ihat:
(b) Yes; if for a body E =O,K +U =0; so, either both arc ~(K +U) = IlE=O
zero or PE is equal to KE but negative, i.e., KE mayor may nol [as E is constant in a conservative field]
be zero. Now as p = ~2m(KE), so, a body mayor may lIot have i. e., 6.K +6U =0 K U"'O
momentum without having energy* . i.e. , if the kinetic energy of K", E
the body increases its
Note: If instead of energy we consider kinetic energy then ;IS
KE '" Ci12m111 body cannot h:we momentum without having
potential energy
decrease by an equal amount
wiH 1
kinetic energy and vice-versa. KE
and vice-versa. (Fig. 4.13)
Question VII. A particle of mass m is moving in a However, in addition to
horizontal circle ofradius r, ul1der a celilripetal force equal to 'he above if some K_O
- (Klr2), where K is COlis/alit. What is the total energy of the non-conselVative force like 0 L _ _ ...,.,--_~,-,~U~-~E:.
particle? friction is also acting on the PE - U
particle, the total mechanical Fig. 4.13
Solution: As the particle is moving in a circle
energy is no more constant. It changes by the amount of work
~ = -;, [as fm eireula, ,oo'ion IFI= m~'l done by the frictional force, i.e.,
.... (iv)
• In an atom electron has momentum even when its encrgy is lesser than l!:cro, i.e., ncgativc .
.. A systcm which has no intcnlction with Ihe surrou ndings i.e., F, x1 .; O.
136 P HYSICS FOR COMPETITIONS - Vol. I

where WI is the work done against friction. The lost energy is Note: )nlemal energy is the microscopic mechanical energy. Atoms
transfonned into heat and the heat energy developed is exactly and · molecules in a body or system have translational and
equal to mechanical energy d i s~i pated . We can, therefore write vibnl.lional ki netic energy due to their random motion. Also they
have potential energy due to mUlual attra.clions and repulsions.
6£ +Q = 0 ....(v) The .111111 a/kinetic and potentia/ energy 0/ molecules alld atonts
where Q is the heat produced. This shows that if the forces are ill a system is called Internal energy.
conservative and non-conservative both, it is not the
mechanical energy alone which is conserved; but it is the total Problem II. A simp le p endulum o/mass m and length I
energy, may be heat, light, sound or mechanical, etc., which is swings back and fo rth upto a maximum angle 9 0 with the
conserved. vertical. Wh en at an angle 9, what is its (a) potential energy.
(b) kinetic energy and (c) sp eed?
III other words, energy lIIay be transformedfi'om one kind
to (lnoth er but it C(llIlI o t be created or destroyed. The totai Solution : Taking the reference level at the lowest point C,
energy ofall isolated system i.~ constant. This is known as the we have
law of consel'vation of ellergy.
Most important of all the laws of nalure is the law of
conservation of energy. Although the law has been stated in
many different ways, they all have the same meaning; the
following three cxamples are typical statements:
(a) III transforming ellergy fro m olle / orm lo another, in an
isolated system energy is always conserved.
(b) Energy call neither be created nor desiroyed.
(c) The total Sl/~Il 0/ all energy in the ~niverse remaills Fig. 4.14
COllstant.
Regarding the law of conservation of energy it is worthy to hit = 1-/cos e o = J(I -cos So)
note that: hs =1 - /cos S = /(l - cosS)
( 1) In an isolated system total energy is always conserved So (a) Potcntial energy at B relative to C will be
whilc the mechanical energy is conserved only when PE = mg" s
thc forces are conservativc. i.e., PE =mg l(l - cos 9) ....(a)
(2) ' The law o f conservation of energy links mechanics to (b)PE at A = mghA =mgl(l-cos9 0 )
all othcr branches of physics. Th is is why the energy . 1
concept has become one of the unifying ideas of KEatA = -mv 2 =O [asat A,v =O]
2
physics.
so ME at A = mgl(l - cosSo) .... (i)
Thc law of conservation of cnergy so fa r has not been
Now if KB is the KE at B, then in the light ofEqn. (a),
contradictcd by any laboratory experiment or observation of
riaturc. Occasionally in thc history of physics this law seemed MEatB = KB + mg/(I - cosS) .... (ii)
to fail, but its apparcnt fa ilure stimulated the search for some But by conservation of ME between A and B
other fonn of encrgy that could be included in an even more Ko +mg/(I - cos9) = mg/(I- cos9 0 )
genera l law that would c'xplain the observation. For example, ....(b)
i.e., Ko = mg/(cas9 - cos S o)
in 1920 experiments on p-decay showed that electrons were
(e) If v is the speed at point B. fro m Eqn. (b),
emitted with less en'ergy than requ ired by conservation of
energy. To account for the missing energy Pauli in 1930 ! mv 2 = mg/(eos e -casS o )
proposed·' neutrino'. So, the disco very ofneutrino is lhe result 2 '
ofOil/faith ill the law of cOl/servation 0/ energy! i.e., v = ~2gl(eos S - cosSo) .. ..(e)
Questioll. VIII. .A spring is compressed by tying its cuds Eqns. (a), (b) and (e) are the required results.
togetber tightly. /t is then placed ill acid and dissolves. Wh at
happens to its stored potential energy? Note: (i) PE, KE and ti at C will be obtained by putting 9 .. 0 in Eqns.
Answel·:· When the spring dissolves in acid, its stored (a), (b) and (e) respectively and are
elastic potential energy also passes to the acid and is converted (PE)c", 0, (KE)c= IIIg/(I - cos So)
into intcrnal energy of the acid and so, the internal energy and and v '" ~2gf( 1 casSo)
hence the temperature, of the acid will increase. (ii) At A all the mechanical cnergy is potential. at C it is wholly
kinetic while at B it is partly kinetic and partly potential.
Problem 12. A string with one end fixed on a rigid wall will be mg(h + y) (and not mgh) while gain ,in elastic potential
passing over a fixed frictionless pulley at a distance of 2 m energy of the spring will be (1I2)kl; so by conservation of
from the wall has a point mass M = 2 kg attached to it at a mechanical energy
distance of 1 mfrom the wall. A mass m = 0.5 kg attached on
the free end is held at rest so that the string is horizontal mg(h+y)=iky'
between the wall and the pulley and vertical beyond the pulley.
What will be the speed with which the mass M will hit the wall As here particle and spring remain same,
when mass m is released? (g = 9.8 mls2)
Solution: Let the mass M strikes the wall at D, so that the
hi + YI
h2 +Y2
=(21)'
Y2
string supporting the mass m is pulled up by a height h given by
h = DC - BC=~AD'+AC' - AD [asAD=AB=BCj i.e., 0.24 +0.01 = ( O.oI)'
h, +0.04 0.04
j.e.,h = ~I' +2' - 1= .,[5 - 1= 1.236m 0' h2 = 3.96 m
Problem 14. In a spring gun having spring constant 100
Nlm, a small ball of mass 0.1 kg is put in its barrel by
compressing the spring through 0.05 m as shown in Fig. 4.17.
(a) Find the velocity of the ball when the spring is released.
(b) Where should a box be placed on the ground, so that the
ballfails in it, if the ballleayes the gun horizontally at a height
of2m above the ground? (g = 10 mls2)

(a) (b)
Fig. 4.15

Now as the string is inextensible the speed of both the


masses will be equal at all times (but will be different at
different times). If V is the sPlJed when Mhits the wall, gain in
KE of the system will be !(M +lfl)V 2 , gain in PE mgh and
2 Fig. 4.17
loss in PE Mg x AD = Mg (as AD = 1 m). So by conservation
of mechanical energy: Solution: (a) When the spring is released its elastic
1 , potential energy (1I2)h 2 is converted into kinetic energy
2(M + m)V +mgh=Mgxl (1I2)mv 2 of the ball; so, by conservation of ME,
Gain in mechanical energy =loss in ME l. mv 2 = 1.h 2
2 2
i.e., i(2+0.5)V' =9.8(2 - 0.5 x 1.236)
v=xg
m
V = 9.8xI.382 :::: 3.3m1s
1.25 OO
So, v = O.05 - mls
0.1
Problem 13. A massless platform is ,
,0.1 kg
kept on a light elastic spring, as shown in ,,,
Fig. 4.16. When a sand particle of 0.1 kg
mass is dropped on the pan from a height : i (b) As vertical component of velocity of ball is zero, time
of0.24 m, the particle strikes the 'pan and
the spring compresses by 0 ..0 1 m. From taken by the ball to reach the ground,

t=f! = ll~2 = Jfs


what height should the particle be
dropped to cause a compression of 0.04
m?
So, the horizontal distance travelled by the ball in this time
Solution: If the particle is dropped
from a height h and the spring is
compressed by y, loss in PE of the particle
FIg. 4.16
d=vt=-HX~=lm
138 PHYSICS FOR COMPETITIONS - Vol.

Problem 15. One end of i.e. ,


A Ring
a light spring of natural
.. 0
length d and spring constant k After starting from B the particle will reach C and then wil
isfIXed on a rigid wall and the rise up till the remaining KE at C is converted into potentia
other is fixed to a smooth ring h
energy. It will then again descend and at C will have the sam(
of mass m which can slide value as it had when ascending, but now it will move from Ctc
without friction in a vertical v B; the same w ill be repeated and finally the particle will comt
d B i.
rodfixed at a distance dlrom to rest at E such that,
the wall. Initially the spring Rod
BC +CB +BE =7.5
makes an a,}gle of 37' with FIg. 4.18
the horizontal as shown in or 3+3+BE=7.5
Fig. 4. 18. When the system is released/rom rest,find the speed or BE = 1.5
ofthe ring when the spring becomes horizontal. (sin 37°"" 3/5) So, the particle comes to rest at the centre of the flat part.
Solution: If I is the stretched length of the spring, then Problem 17. In Fig. 4.20 (a) and (b), AC, DG and GF are
from Fig. 4.18, fixed inclined planes. BC =EF = x and AB = DE = y . A smail
!{ "" cos37o =i i.e., l = ~d block ofmass m is released/rom restfrom the point A.It slideJ
down AC and reaches C with a speed vc. The same block iJ
I 5' 4
released from rest from the point D; it slides down DGF ana
SO, theslretch Y = l-d=% d -d=~ reaches the point F with speed v F. The coefficient oj kinetic
friction between the block alld both the surfaces AC and DGF
and is ",. Calculate Vc and v F'
0
Now taking point B as reference level and applying law of s,
conservation of ME between A and R, T---P __ ,,
Y
G
EA =E B ,,, s,
I 2 I
mgh + "2ky +O = O+O+'2/11V
2 mg
mg

,
1 , .'. ,
E
, ,
F
[as for B, h = Oand y = Oj I' '1 I' ·1
(b)
~mgd+ l2 k(:!)Z
(a)
or = .!.mv Z Fig. 4,20
4 4 2
Solution: (a) ME at A = mgy+Oand if vC is the velocity
[as for A. h =~d and y=~d] atC,
ME at C = 0 + (I/2)mv~
or v=d 3g +..l.....
2d 16m so, Loss in ME = mgy - (1 /2)mv~
Proble m 16. A particle This loss in ME is equal to the work done against friction,
slides along a track with
;.e.,
elevated ends and a flal
central part as show/l in Fig. mgy - (I/2)mv~ = ",mgcos o. x s
4.19. The flat part has a (1I2)mv~ =mgy -).1mgx [ascoso. =x/s]
length I = 3.0 m. The curved
portions of Ihe track are or Vc =~2g(y-IU) ····O}
frictionless. For the flat part the coefficient ofkineticjriction
(b) In this situat ion,
is Ilk =0.20. the particle is released at point A which is at
height h = 1.5 '" above theflatpartofrhe track. Where does the mgy - (I I2)mv~ = fls, +/zsz
particlefinally come to rest?
or (1I2)mv~ = mgy-).1mgcos i3s] -Ilmgcos ysz
Solution : As initial ME of the particle is mgll and fina l is
=
zero, so loss in ME mgh. This ME is lost in doing work or (I12)v~ = g[Y - IUI - 1U2J
against friction in the flat part, so
[ascos~ = (xl/SI )andcos y =(x z/sz)]
loss in ME = WD against friction
or vF =.Jig(y -!JX) =vc [asx\ +xz =x] .... (ii)
mgh = ",mgs
Note: Force of friction is non-conservative, i.e., work done against =
which on solution gives x 0.1 m (asx = -I. 1 is inadmissible).
friction is path dependent, so Vc should not be equal to vF if After moving the distance x = 0.1 m the block comes to rest.
force of friction remains the same. In Ihis problem force of Now the compressed spring exerts a force
friction and path both have been cleverly changed in such a way
that (jxS)A = (fxs)B,so that Vc",vF' F=kx = 2 xO. I =0.2N
----- on the block while limiting frictional force between block and
Problem 18. TlVO blocks track IL = fl ,r1ng = 0.22 x 0.5 x IO = I.l N. Since, F < fL . the
are connected by a string as block will not move back. So. the total distance moved by the
shown in Fig. 4.21. They are block
released from rest. Show that
= 2+2.14 +0. 1 =4.24m
after they moved a distance L.
(h eircon/mon speed is given by: Problem 20. Two
fIXed equal positive
2(m2 - ).1m] )gL
V= Fig. 4.21 charges each ofmagnitude -q
(m[ + m2) 5 x 10- 5caul, are located o ,, c 0: •
where}.1 is the coefficient offrictioll. at poillts A and B
:
Solution: In moving a distance L by In, and m2 the PE separated by a distance of
6 m. An equal and
s.+q
will be lost by nl2 only and will be m2g4 while work done by Rg. 4.23
opposite charge moves
frict ion wi ll beonlyon m, and will beJ.WIlgL. Now as the string
towards them along the line COD, the perpendicular bisector
is inextensible v I = v2 = v and so, the gain in KE will be
2 olthe line AB. The moving charge, when it reaches the point C
= (1I2)(rnJ + nl2 )v . By conservation of energy: at a distance of 4 m from 0 has a kinetic energy of 4 J.
Loss in PE =Gain in KE + WD against friction Calculate.the distance offarthest point D which the negative
I 2 charge will reach before returning towards C.
i.e.• m2gL =2: (ml +nl2)v +flmlgL
[1/(4.00) = (9 x 10') mlF]
v = F2("m,,''----''-~m=
, )"g",L Solution: The electrostatic potential energy of two point
(»11+ 111 2)
charges separated by a distance r is (1/4mo) ( q l:2 ) ; so
Problem 19. A 0.5 kg block slides from the point A on a
horizontal (rack with an initial speed 3 mls towards a energy at C = KE + electrostat ic PE.
weightless horizontal spring of length 1 m andforce constant
EC =4 +2x_
l- (q)(- q) =4 - -2- xq' -
2N 1m. The part AB ofthe track isfriction/ess and the pari BC
has the coefficient of static and kir. :Jtic friction as 0.22 and 4rreO ~32 + 4' 41t1:. 0 5
.0.20 respectively. If/he distance AB and BD are 2 m and 2.1 4 At farthest point D, KE =0 and if it is at a distance x from 0
m respectively.jind the total distance through which the block
ED =0+2x_l- (q)(-q)
moves before it comes to rest completely. (g = iO mls 2)
Solution: As the
'rnck AB is fricl ionless'
~
!li
[t::J
~
41t£0 ~32 +x2
Now by conservation of energy EC =ED. i.e.,
the block moves this _ .... _ _ _ -ill' 2q'
dishmce (= 2 m) without A B2 2! D C -4-'-"-0-.J73~':=+=X~" = 4 - -4'-'-0-
5
loss in its initial KE ' Fig. 4.22
=~ mu2 = ~ x 0.5 x3 2
= 2.25 J. In the path BD ("" 2.14 m) as or [~ _ I ]= 4x4,"0
friction is present, so work done against friction = fl K mgs 5 ~9+x' 2q'
"" 0.2 x 0.5 x lOx 2.14 = 2.14 J.
So at Dthe KE of the block is = 2.25 - 2.14 =0. 11 J.
Now if the spring is compressed by x,
or
I ,
0.11 ="2xkxx +J.lKmgx
or 9+X2 =8 1,
i. e., 0.11 =~x2 xx 2 +0.2xO.5xIOx
i. e., x =m = 8.48 m
or x 2 +x -0.ll=0
Problem 21. A drop a/water a/mass 18 x 10-6 kg/aUs Solution: In going up KE will be converted into PE and in
away from the bottom of a charged conducting sphere of coming down PE will be converted into KE and a~
radius 20 em, carrying with it a charge of 10- 9 coulomb and gravitational field is conservative the bullet will hit the grounc
leaving on the sphere a uniformly distr'ibuted charge of with same ICE with which it was projected up.
2.5 x 10-6 coul. What is the speed ofthe drop after it has/alien mv 2 1
I 2 =--ca
so MElost=-mv
30 em? [eo := l/(41tx 9 x 10 9 ) F/m and g = 9.8 mls 2 ] 2 2J
Solution: Taking reference level at B + + + + + and heat lost by the bullet to come to O°C =m.::Ll8 cal
and keeping in mind that for an external + and if M gm ice melts, heat gained = ML cal
point, a charged sphere behaves as whole of
its charge is concentrated at the centre,
q
EnergyatA=mgxO.3+-I_ ,Q2 +0
+
• . . !
a.2m
, +
+A: +
+q1
Now by conservation of energy
ME lost + HE lost = HE gained
,,, mv' 8 = ML
4"0 (0.2) i.e., --+mcf:..
and Energy at B
,i, a.3m 2J
: 4
or 50x(8.4xI0 )' +50xO.02x(30-0) = Mx80
:=0+ S'Q2
Fig. 4.24
2x.4.2xI07
4rr£o
or 4200+30 =-SOM, i.e., M=423CV80 = 52.S75g
So by conservation of energy,
Note: If a student misses mct..9 = 30 cal, the result does not chang~
mg(O.3)+ _ I_q,Q2 =~mv2 +_I_ Q,Q2
appreciably; however, the solution will be treated wrong.
4m::o 0.2 2 4m::o 0.5
or lmv 2 =O.3mg+ Q,Q2 [_I___I_J § 4.7 Modern Views about Energy
(AI Mass-Energy Conservation
2 4m:o 0.2 0.5
In classical physics, mass _and ene'i'gy are two separate
or v2 = 0.6g + 6ql q2
physical quantities each having its own conservation law, i.e.,
4m:om
for an isolated system mass = con~tant and energy = constant.
9 6 9
=O.6x9.8 + 6x9x 10 x2.5 x 10- x 10- In 1905, Einstein made the incredible discovery that energy
18 x 10 6 can be transformed into mass and vice-versa through the relation

or v 2 =5.88+7.5=13.38, i.e., v =3.65m1s E=mc 2


So at present mass and energy are not conserved
Problem 1.2. The height ofa water fall is 50 m. Calculate
separately but ' a.re conserved as a single entity called
the difference between temperature of water at the top and
mass-energy. So, the conservation law that describes any
bottom ofthefall. (J:::: 4.2 x 10-7 erg/cal and g = 9.S m/s2) transformation or process states that the mass-energy of an
Solution: Loss in ME of the water = mgh = (mghlJ) cal isolated system remains constant.
ras W =JH] The table below gives energy equivalent to different
Now if Ll9 is the rise in temperature, masses
heat gained, Q = (meLl9) cal Particle Mass Energy (equivalent)
Now by conservation of energy, Electron me == 9.lx 10- 31 kg 0.53 MeV
Loss in ME = Gain in HE Nucleon lamu == 1.67x 10-27 kg 931 MeV
mgh
i.e., - - =mcLl8 or Any I gram 9x 1013 J = 25GWh
J
[B] Quantisation of Energy
so 88=980X50 x 10' =0.49=0.1167C'
Planck has shown through his study on 'radiations emitted
4.2x107xl 4.2 by black body' that ener'gy of a particle cannot have all values
Problem 23. A 50 g lead bullet, sp. heat 0.02, is initially but only certain discrete values, i.e., energy is quantised.
at 30°C. It isjired vertically upwards with a speed ofS4'O mls The quantum nature of energy is specially revealing in the
and on returning to the starting level strikes a cake of ice at atomic and subatomic world. Though the energy of
O°C. How much ice is melted? Assume that ail the energy is macroscopic objects, say a satellite, is also quantised, yet
spent in melting only. (L = 80 cailg) energy levels are so close to each other that energy appears to
be continuous.
Problem 24. The energy supplied to Calcutta by the state Solution: Given that n ~ p.J. e + v
electricity board during an average November week day was
40 GWh. If this energy could be obtained by the conversion of Now as mn = 1.6747xlO-27 kg
matter, how much mass would have to be annihilated? and mp + me = 1.6725 x 10-27 kg + 9 x 10-31
(c=3 x IOSmls)
=1.6734xiO- 27 kg
Solution: E = 40 GWh = 40 x 10'(J/s) (60 x 60s)
so,massdefect llm=mll-(mp +me)
=1.44 x 10 14 J = (1.6747 - 1.6734) x 10-27 kg
14
But E=mc2;som="£' 1.44xlO = 13 x 10- 31kg
c2 (3xlO B)2
so, E = !:J.~C2 =13xI0 - 31 x (3xI08)2
= 1.6 x 10-3 kg
=1.l7xI0- 13 J
i.e., m=1.6g
1.17 x 10- 13
Problem 25. A neutron decays into a proton, on electron or E
and a neutrino. Calculate the energy produced in this reaction 1.6x10- 19
in MeV. (me =9x 10- 31kg, mp =1.6725xIO-27kg, =0.73 x 10 6eV
mn = 1.6747 x 10 - 27 kg, c = 3 x 10 8 mls; mass of neutrino may = 0.73MeV
be considered to be zero).

MISCELLANEOUS SOLVED PROBLEMS

Problem i6. Two identical cylindrical vessels with their Alternative Solutions:
bases at the same level each contain a liquid ofdensity p. The (A) We know that in terms of pressure and volume,
height of the liquid in one vessel is h I and that in the other is
W=!PdV=P(flV) [ifP=constt.]
h2 . The area of either base is A. What is the work done by
gravity in equalising the levels when the vessels are As hydrostatic pressure P = hpg, so initial and final
connected? pressure difference will be,
Solution: As the total volum~ of water is constant, the PI =(h] -h2 )pg and P2 =0
height h in each vessel after interconnection will be given by: So, p," = (PI + P, )/2 = (hi - h, )pg/2
h]AI +h2A2 = h(A I + A 2 ), i. e., h = (h] + h2 )/2 [as P oc h, so Pay = (PI + P2 )/2]
[as A] =A 2 (given)] And volume transferred from one vessel to the other
The level in the left flV=(h l -h,)AI2
vessel shown in Fig. 4.25 A
drops from A to C and that in
Tc-~- --------
the right vessel rises from B to h,
D
So, W=P(flV)=.!.pgA(h,-h,)'
4
[as(h, - h,)' = (hl - h,)' ]
D. Effectively, the water in
the part A C has dropped
down to BD.
1 ___ _ _=~= "Ih' (B) If h is the common height when they are connected, by
conservation of mass
Fig. 4.25
The mass of this volume pAlh l + pA2h2 =ph(A I +A 2 )
of water is or
m=pA[h l -(hi +h,)I2] As (h ll2) and (h2/2) are heights of initial centre of gravity
= pA(hl - h,)/2 of liquid in two vessels, the initial potential energy of the
The height descended by this water is
AC=hl -h=(h l -h,)12
system
hi h, (h, + h,)
, ,
So, work done by the force of gravity in equalising the U; = (h,Ap)g- + (h,Ap)-=pgA .... (i)
2 2 2
level
When vessels are connected the height of centre of
1 ,
W = mgxAC=-phA(hl-h,) gravity of liquid in each vessel will be h /2, i. e., (hi + h2)/ 4
4 [as h = (hI + h2 )/2]; final potential energy of the system
~ ~ ~

-1
U F _J(h'+"')A
2 p
}(hl+h')_A
-4- - pg [(hl+h,),]
4 .... (")
11
Now as here, F = Fr + F /
dW~~ -) -" ~
So, P=-=F·v = (Fr+F,)· v
Now as gravitational fi eld is conservative, dt
F =-dUldr, i.e., - dU=Fdr ~ ~ ~

Now in circular motion F r is perpendicular to v while F,


or - fFdU =!'Fdror U/ - UF=W [as !Fdr=WJ
U, 0 parallel, so
~ ~
so W =U/-U F =.!. pgA[2(h,' +hi)-(h, +h,)'J P = F,v [asF, ·v=O·.
4
1 , Which in the light 10 Eqns. (i) and (ii) becomes
=-pgA(", - ",)
4 p = mk 2 r 2 t
Problem 29. A particle y
Problem 27. Water is flowing continuously from a tap
having a bore a/internal diam eter 8 x 10-3 m. Calculate the of charge +q and mass m
cJiameter ofthe water stream at a distanee2 x 10- 1 m below the moving under the influence
~
tap. Assume that the water velocity as it leaves the tap is of uniform electric field E i
4 x lO- 1 ms- I , and Q uniform magnetic jield

, ,
~
Solution: By conservation of mass
B k follows a trajectory
A1v , = A2v2 i.e., d 1 vI = d 2 v2 from P to Qas shown in Fig.
4.26. The velocity at P and Q
And by conservation of mechanical energy ~ ~
2v
Fig. 4.26
are v i and-2v j . Which 0/
~mvi =~mv~ +mgh i, e., v2 =~(v: +2gh)
the/ollowing statement(s) is/are correct?
so v, = ~(O.4)' + 2 x 10 x 0.2 = ,/4.16 = 2.03 mls
(a) E =~[ mv' )
~ =8xIO-3 ~ 0.4
4 qo
SO d2 = d 1
V-;;; 2.03
(b) Rate of work done by efectricjiefd at P iS ~[ m~3 )
=3.55 x 10-3 m
Problem 28. A particle a/mass m is moving in a circular (c) Rate of work done by the electric field at P is zero.
path ofconstant radius r such tkat its centripetal acceleration
2 2 where k is a constant. (d) Rate o/work done by both the fields at Q is zero.
Q c is varying with time t as Q c = k rt
Solution: Electric and magnetic fi elds artl co nservativ~
What is the power delivered to the particle,by the forces acting and in 'a magnetic field force is always perpendicular Ie
on it? motion; so, work done in a magnetic field is always zero. Nov.
Solution: As Q = (v2/ r) so (v 2/ r} = k 2rt 2
c by Work·Energy Theorem as W:=. change in KE. i.e.,
I 2 I 2
Kinetic energy K= .!mv 2 =lmk 2r 2t 2 - m(2v) - -mv = WPo+WOQ
2 2 2 2
Now by Work·Energy Theorem W = 11K =~mk 2r2t2 - 0 i.e., (3/2 )mv 2 =:(qE)acos90o+(qE )2acosOo
[asW = Fscos9]
[asal t = O, K = O]

So, p =dW = !!"'(!mk2r2t2) = mk2r2t


dt dt 2
i.e. , or E = ~[ "~:' )
Further rate of work done by the electric field al
Alternative Solution: Given that a c = k 2 rt 2 , so that
P= l'v =qEv . [as FE =qE]

H[ ~3
2
Fr =ma r = mk 2 rt 2mv 2
Now as a r = (v 2/ r) . so (v2/r) = k 2rt 2 = qvx %[ ";.' ) [asE = --J
4qa
i.e., V = krf .... (i) And rate of work done by both fields at Q is zero as both
so that, a, = (dvldt)=kr the forces are along x-axis, perpendicular to motion .
i.e., F, =ma,= mkr .... (ii) So, answers (a), (b) and (d) are correct.
Problem 30. A ring of mass m = 0.3 kg slides over a
smooth vertical rod. Attached to the ring ;s a light string
k Xo = IJm2g or =--
'0 .,.(.)
}Jm2g
k
.

passing over a smooth fixed pulley at a distance 0/0.8 mfrom


the rod as shown in Fig. 4.27 (a). At the other end ofthe string
there is a mass M = 0.5 kg. The ring is held;n level with the
Using
theorem
Work-Energy
'::i5J:::f5J • $

Work done by force F ""


pulley and then released. (a) Determine the distance by which
work done against friction +
the mass m moves down before coming to rest/or theftrs! lime.
(b) How far below the initial position is the equilibrium point energy stored in the spring ~ E.f-m.:,-::._
located? I 2
orFxo = )lmlgxO +-kxo '~~, ~N2
Solution: (a) As tension acting on the ring initially is 2 m,g
horizontal, the ring will go down by its own weight till it stops, .,. (ii) Fig. 4.28
i.e., its kinetic energy becomes zero. Solving equation (ii)

~I,=
with equation k Xo = Ilmlg,
m
m
F = Ilmlg + j.l 2g
c' 2
Problem 32. Two blocks A and B each having mass of
0.32 kg are connected by a Jight string passing over a smooth
pulley as showlI in Fig. 4.29 (a). The horizontal surface on
which the block A slides is smooth. The block A is attached to a
spring offo rce constant 40 Nlm whose other end is fixed to a
support 0.40 m above the horizontal surface. Initially, when
(a) (b) the system is released to move, the spring is vertical alld
Ulls tretched. Find the velocity of the block A at the instant it
breaks offthe surface below it. (Take g "" 10 Nlkg)
Soludon: Let the block A start loosing contact with ttoe
surface below it at A' after travelling a distance x as shown in
Fig, 4,29 (b),

s
0' Jh=5(~(DB)2 + h2 - DB1Le,. h = l.5m
(b) For equilibrium of mass M. T", Mg while for ring,
= =
Tcose mg (and Tsin 9 R) so that,

cos8=(mgl=(~)=~ 4
i.e., tan e =-
) 8
\ T / M 5 B

Now if h is the distance of equilibrium position of the ring


below its initial position, then from geometry of the problem,
(a)
!:.. = tane=~i.e. , h=0.8x~=0.6m FIg. 4.29
h 3 4
In this process the block B will shift from B 10 B' such that
Problem 31. Two blocks ofmasses ml and m2 connected BB' = AA' =x (as string is inextensible) and so there is a loss
by a lIoll-def ormed Jight spring rest on a horizontal piane. The of gravitational potential energy mgx. =
coefficient offriction between the blocks alld the swface is This energy is partly stored as elastic potential energy in
equal to f.L What minimum constant force has to be applied ill the spring which is stretched by I1L and partly appears as
the horizollfa{ directiolllo the block ofmass ml ill order to shift kinetic e:1ergy of blocks A and B. So, by conservation of
the other block? mechanical energy, we have
Solution: The phrase " in order to shift the other block" I 2 I 2 I 2
means m2 is just about to move and that will happen when the mgx = - mlJ +- mv + -k(l1L)
2 2 2
spring force acting on 1112 just ovcrcomes the maximum value
2 k 2
of frictional force on m2' This happens when the compression 0' v ~gx - -(AL) ",(i)
in the spring is xo. 2m
Now for vertical equilibrium of block A at A', Dividing Eqn. (ii) by (i), we have
R+Fcos8=mg
But as for spring F = kM. and for breaking off R -t 0, the ~: =
(::;) i.~, W -
v=vo ( w+J
1)"'
above equation reduces to o
k MeasS = mg ... (ii) Problem 34. (0) A charge Q coul is uniformly distribute
Further from geometry of the problem, over a spherical volume a/radius R m. Obtain an expressio
for the energy of the system. (b) What will be II.
AL=~-L=L(_I_- I) .,,(iii) corresponding expression/or the energy needed to cemplete,
cosS cosS disassemble the planet earth against the gravitational pu
"-' in (ii) and
So substituting the value of M from Eqn. (iii) amongst its constituent particles? Assume the earth to be
solving for cos e, we get sphere!'oj imifonn mass density. Calculate this energy. giv~
the product of the mass and the radju~ of the earth to hI
cosS = 1- mg =1- O.32x 10 =~
kL 40 x 0.40 5 2.5 x 10 31 kg-m and g .. 10 mls 2 . (c) If the same charge of

so that, AL=(~
cosS
-L)= 0.4 x 5 -O.4=O.lm
4
caul as if! part (a) above is given to a spherical conductor,
the same radius R what will be the energy of the system?
Solution: (a) If r is the radius of the sphe~ at any tim
and x = LtanS =O.4x 0.3 =0.3 m then as charge on the sphere will be q := (4/3) nr3p, its surfa!
0.4
potential
Substituting these values of M and x in Egn. (i),
V_I · q _ 1 [4nr'p) ... (
[
v= IO xO.3 - 40x(0.1)
2xO.32 ']'" =./3 -0.625 = l.54m1s E - 4nEo -; - 41t" Eo - 3 -

So, work done in charging the radius of sphere from


Problem 33. A slone with weight w is thrown vertically tor+dr,
upward into the air with initial speed vo. Ifa constant force!
due to air arag acts on the stone throughout its flight,
(a) Show that the maximum height reached by the stone is : .

11 =
v'0
2g[1 + (J7w)] i. e.. U = _ 1_ (41tp) 2 JR r4dr
. 41tEo 3 0
(b) Show that the speed of the stolle upon im..,act with the
ground is: 1 (4.p)' R'
U =-
4'-&-O~3C'-- 5
i.e.,
(W_/)"2
v =vo , w+J
i.e., UE = _l_ ~.Q' [asQ=~3!tR3p] ...(
Solution: (a) By conservation of energy, for upward motion, 41(EO 5 R
Loss in ICE ;; Gain in PE + WD against drag (b) As in case of gravitation V := -OM I r so, in the light
i, e., !~v~
2g
= :::'gh+/h
g
[asm=~l
g
... (i)
Eqn. (i)

dr
0' v~ =h[1+ I]
2g w
v2
or h= 0
2g[1 + (f/w)]
(b) By conservation of energy, for downward motion, Fig. 4.30

(_I_)-+-G
Loss in PE = Gain in KE + WD against drag
w Iw 2 andq-+M
-gh=--v +Ih 4nEo
g 2g
UG=_~GM
,
i.e., !~v2 = :::'gh- f h ... (ii) so
5 R
... (
2g g
but as U <I) =0 so, work done in disassembling the earth (c) When the ctiarge is distributed over the surface only
W=UQ()-U . 1 q
dU =Vdq = - - -dq (r = R z:: : constant)
.3 GM' 3(gR2) M'
=-----'= - - - x--
4m:o R·
5 R 5 M R
U= . -1- fQ1 -
qdq=-- Q'- ... (c)
.41t8 oR 0 41ttO 2R
W=~gMR =~xlOx2.5xl031
5 5 Note: The (self) energy ofa body is defined as the potential energy ofthe
... (ii) body itself. It is equal to the work done in assembling the body
from infini~simal fragments having no initial potential energy.
i.e., distributed initially at infinite separetion.

"

", !.-

,
EXERCISE
(AI Only Once Choic~ is Correct (a) 20 h' (b) 15 h,
1. A body moves a distance of 1O.m along a straight line under (e) 7.5 hr (d). 3.75 h,
the action of a force of 5 newton.. If the work .d9ne is 25 9. A mass of 2 kg falls from a height of 40 em on a'spring of
j01.!ie, the angle which the force makes with the direction of force constant 1960 N/m. The spring is eomp.ressed by:
motion of the body is: . (a) 10 em (b) 0:4 em
· <a) <1' (b) 3<1' (c) .0.Otcm (d) ,'0.04 em
(ej 60" (d) 9<1' [Hint: See S (e) § 3.4 )
2. A body of mass m is moving in a circle of radius r with a 10. Two spring~ ha~e ·their force con'stants as K J and K 2
constant speed v. The force -on the b~dy is mv 2 Jr and is (K J > K 2 } The. work done, wh,en both are stre.tched by th.e
directed towards the centre. What is the work done by the same amount of I.ength, will qe:
force in moving the body half.the c'ircumference of the (a) Equal (b) GreateiforK J
circle? [CPMT 1993] (c) Creater for K 2 (d) Given data is incomplete
· (a) (mv2 /r)x 1tr (b) zero [Hini : See (7) i~ § 3.41.

3.
.
An object is moving in a circle with constant speed. If Fbe
II . In. the above problem if the springs 'are
same force, work done will be:
str~!ched by the

the resultant force acting on the object, then the amount of · (a) Equal (b) Greater for'K.1
work done on the object by this force is zero: (c) Greater for K2 (d) Given data is incomplete
(a) Sometimes (b) Frequently (Hint: See (7) in § 3.4)
(c) Always (d) Never 12. A chain is held oh a frictio nless tab.1e with one-third of its
4. A partIcle moves along the x-axis from x = 0 to x = 5 m length hanging over the edge. Ifthc chain has a length Land
mass M, how much work is 'required to pull th~ hanging
under the influence of a force given by F = 7 - 2x + 3x 2 .
part back on the table? IM NR 1990J
T~ e ~ork done in the process is: ICBSE 1994)
(a) MgL (b) MgLlJ
(a) 70 (b) 270 (e) 35 (d) 135
~ ~ . (e) MgLl9 (d) MgL/l8
. 5. A force F = - K(y .i +x j )(wherc K is a positive constant) 13. Under the action of a force, a 2 kg body move's such Ihat its
acts on a particle moving in the x~p l ane. Starting from the position x as a fun ction of time is given'by;
origin, the particle is taken along the positive x-axis to the , ' x=(3 .
/3
point (a, 0) and then. parallel to the y-axis to the point (a,a).
. . ~
·where x is in metre and f in second. The work done by the
The' total work done by the force F on the·particle is: force in the first two seconds is:
lIlT 19981 (a) 1600joule (b) 160 joule
(b) 2Ka' (c) 16jQuie (d) 1.6 joule
-t ---) ---) -+
(d) Ka' '1 4. Work done when a force F =( i +2 j +3 k)N acting on a
6. .A spring of force constant 10 N/m has initial stretch 0.2 m. -+ ---) ---) -+
'In changi~g th~ stretch to 0.25 m, the increase of PE is part.icle takes it from the point r I "" ( i + j + k )to the point
about: -+ -t -t -+
r 2 =( i ~ j +i k )is: liAS 19961
(a) 0.1 jou'le (b) 0.2joulc
(e) 0.3 joule (d) 0.5 joule (aJ - 3 J (b) - I J
7. 'A longspring.is stretched by xcm its PE isU. If the spring is (c) zero (d) 2 J
~tretched. by Nx em·the PE stored in it will be: 15. A vchicle needs an engine of7500 watt to keep it moving
IEAMCET 19931 with a constant velocity of20 mlsec on a horizontal surface.
The force resisting the motion is : _ .
·(a) UIN (b) NU
(a) 375 dyne (b) 375 newton
· (e) N'U (dj U IN'
(c) 150000 dyne (d) 150000 newton
8. It requ.ires twenty turns of the stem of a watch to wind the
· .main spring and this stores sufficient energy to keep the 16. An electric motor creates a tens ion of 4500 newton in a
watQh runnin.g for 30.0 hour. If ten turns are given to the hoisting cable and reel s it in at the rate of2 mls. What is the
steml it will nm for: . 'powcr o f this motor? .
(a) 25 kW (b) 9 kW (e) 225 watt (d) 9000 hp
17. From a waterfall, water is pouring down at the rate of.1 00 27. There is a hemisphe;rical bowl of radius R. A block of mass
kg per sec on the blade of a turbine. If th,e height of the fa U In slides from the rim of the bowl te the bottom. The
be 100 m, the power delivered to the turbine is approximately velocity of the block at the bottom will be:
equa l to : (a) .JRi . Cb) .)2Rg
Ca) 100 kW Cb) I W
Co) .)2rrRg Cd) .)rrRg
Ce) I kW Cd) 100 W
28. When a body falls from an acroplane there is increase in its:
18. An engine develops 10 kW ofpowe~. How much time wi\1
it take to lift a mass of 200 kg to a height of 40 m? (a) Acceleration (b) Potential energy
Cg=I Oni /s' ) ·(CPMT I992 ( (c) Kinetic energy (d) Rest mass
29. Water is fa lling from the top ofa high waterfall . On falling:
Ca)4s (b)5s
(a) It evaporates
Ce) 8 s Cd) lOs
(b) It freezes
19. A man weighing 60 kg lifts a body of 15 kg to the top of a
building 10m high in 3 minutes. His efficiency is: (c) There is no change in its condition
Ca) 10'10· Cb) 20'10 (d) It gets slightly warmed up
(e) 30010 Cd) 40'10 30. The PE stored in a long spring stretched by I cm isU. Uthe
spring is stretched by:3 cm instead, its PE will be:'
20. A 3 hp motor requires 2.4 kW to drive it; its efficiency is
about Ca) 9U Cb) 3U Ce) Vl3 Cd) Vl9
(a) 90% (9) 75% 31 . The PE of a simple pendulum is maximum when it is:
(e) 60% (d) 50% (a) At the turning points of the oscillation

21. Atomic power station a! Tarapore has a generating capacity (b) At the equilibrium
of2oo MW. The energy generated in a day by the station is: (c) In between the 'above two cases
Ca) 200 MW Cb) 200 joule (d) At any position it has always a fixed value
(e) 4800x 10 6 joule Cd) 1728 x 10[0 joule 32 . If a simple pendulum of length thas maximum angular
d isp l acemen~ 9 then the maxi~num KE of its bob of mass TIl
22. A body is moved along a straight line by a machine
is:
delivering constant power. The distance moved by the body
in time 1 is propo rtional to: IBHU 19951 Ca) CI/2)mClIg) Cb) CI/2)m (g /I)
(a) /112 (b) 1 314 (c) mgJ(l - cos9) (d) ( 1/ 2)mglsin e'
(Hint. See solved Problem 11)
(c) t 3f2 (d) (2
33. A stone tied to a string of length L is whirled in a vertical
[Hint. See solved Problem 10]
circle with the other end of the string at the centre. At a
23. A bent bow used for shooting an arrow possesses: certain instant o f time, the stone is at its lowest position and
(a) Potential energy . (b) Kinetic energy has a speed II. The magnitude of the change in its velocity as
(c) Heat energy (d) Chemical energy it reaches a position where the string is horizontal is:
24. A moving body need not have: (liT 1998(
(a) Potential energy (b) Kinetic energy (a) ~u2 -2gL Cb) .)2gL
(c) Momentum (d) Velocity
(c) ~1I 2 - gL Cd) )~2(~u,C-_-g-L)
25. The KE ofa body moving with a speed of 10 mls is 30 J. If
its speed becomes 30 mis, its KE will be: 34. When a spring is wound, a certain amount ofPE is stored in
(a) JO joule (b) 90 joulc it. If this wound spring is dissolved ill acid the stored .
energy:
(c) 180 joule (d) 270 joule
(a) In completely lost
26. Fig. 4.31 shows a body of mass M
sliding from rest, down a (b) Appears in the form ofelectfomagnetic waves
friction less track ofradiusR in time (c) Appears in the form of heat raising the tempe~ature of
t. Assume -that the body started the acid
from the top of the track A and (d) Appears in the fonn of KE by splashing acid drops
slides to the bottom B. The change
35 . These diagrams represent th~ potential ~nergy U of a
in the gravitational potential
Fig. 4.31 diatomic molecule as a function of the inter-atomic distance
energy is:
r. Which diagram corresponds to stable molecules found in
(a) gtl2 Cb) M Co) MgR Cd) MR( nature?
'Ong

~~
(a) 3mg (b)
k k
(a)
(0) mg (d)
k
41 . . If a particle slides from Test at the highest point down the

(0) ~~ outside of the arc of a smooth vertical circle, the particle


will [eave the curve after travelling a vertical distance equal
to:
(a) Half the radius of the circle
[Hint: For stable equilibrium U - min]
(b) .! rd the radius 'of the circle
36. A ball of mass m is suspended on a weightless elastic thread 3
whose stiffness factor is k. The ball is lifted so that the
thread is not stretched and let fall with zero initial velocity. (c) ~ th the radius of the circle
4
The maximum stretch of the thread in the process of ball's
motion is: (d) ! rd the radius of the circle
3
(a) mg / k (b) 'Ong I k
42. A unifonn, long, thin chain of length I, with one end
(c) mg/2k (d) mg l 4k attached to a horizontal axle of cross-sectional radius r
37. A stone of mass mis thrown from earth's surface at an angle (r« f), is hung vertically down initially. The chain is then
a. to the horizo ntal with an initial velocityvo' Ignoring the :wound up slowly on the axle, doing an amount of work WI '
air drag, the power developed by gmvity force t second
after the beginning of motion is:
The chain is then cut into two equal ~rts (with length !..
2
(a) mg(gt - vo sina ) (b) mgvosina ·t each) and the two parts are wound one afie{ the other on the
(e) mg(g - vsin (to t) (d) zero same ax le doi ng an amount o ( ~ork W2 and W)
W
38. A ball possessing kinetic energy T collides head-on with an respectively. The ratio I is:
initially stationary elastic dumb-bell (shown in Fig. 4.32) . W2 + W)
and rebounds in opposite direction with kinetic energy T'. (a) I (b) 112 (0) 114 (d) 2
The mas~es of all the three balls are equal. The energy of the
43. A particle moves in a circular orbit of radius r under the
dumb·bell oscillations after collision will be:
m_ m m inDuence of an attractive force F '" A), where A is a "
.------------~ - .- .. -.- 2,·
Fig. 4.32 constant. If the potential energy of the particle is zero at a
T -T' - 2.JTT' very large distance from the centre of the circular orbit, the
(b) T-3T' - 2&
(a) , "--'-2
: ='-'- total energy of the particle in the circular orbit is:
. 2
A
(0) T-T' - 2.JTT' (d) T-3T'-2.JTT' (a) (b) zero
21-'
39. A unifonn elastic string has length a 1 when tension is TI
aiwhen tension is T2 . The amount of work done
A
and length (0) (d)
~n stretching it from its natural length to a length (a l + a 2 )
IS:
"
44. A particle is moving with kinetic energy E, straight up an
inclined plane with angle a.. the coefficient of friction being
~ . The work done against friction before the particle comes
down to rest is:
(a) ..,..:E",~,-,e"os
,,-,o.O­ (b) ~==­
E cos Ct
sin Ct+J.lcos Ct sina + J.l cosa
E
40. A system consists of two identical slabs (e) ..,----=-- (d) --;c---,E"--------:
g (si na + ~cosa)
each of mass m linked by compressed sin a+l-1cosa
weightless spring of stiffness (k ) as 45 . A man whose mass is m kg jumps vertically into air from
shown in Fig. 4.33. The slabs are also sitting position in which his centre of mass is hi from the
connected by a thread which is burnt at a
ground. When his feet are just about to leave the ground his
certain moment. Find at what value of 111,
CClltre of mass is h2 from the ground and finally rises to h3
the initial compression of spring, the
when he is at the top of the jump. What is the average
lower slab will bounce up after the thread Fig. 4.33
upwa rd force exerted by the ground on him?
is burned th rough?
(a)
mg(h) ~ hl) mg(h3 -hi J
(b)
(h3 - h, ) h3 p
mg(h) -h 2 ) mg{h3 - hi) (a) (b)
(0) (d)
(h , -h i ) (h, -hi) ,~ .
46. A small sphere A of radius r "'A'--'--' : --~-,
rolls without slipping inside a :
large hemispherical bowl of iR
,,
radius R as shown in Fig. 4.34. : (oJ Pr--r'---_
If the sphere starts from rest at
the top point of the hemisphere Fig. 4.34
B ,- (d) Pf--f-,-----
,-
find the 'n onnal force exerted
by the small sphere on the hemisphere when it is at the 52. A block of mass m slides along the
bott,om B of the hemisphere: track with kinetic friction fJ.. A man T
17 2 pull the block through a rope which e
(a) -mg (b) - mg
7 5 makes an angle 8 with the horizontal
5 7 as shown in the Fig.4.35. The block Fig. 4.35
(c) - mg (d) -mg moves with constant speed y:' Power
7 5 .
deliveredCby man is :
47. A body starts from rest and acquires a velocity v in time t.
(,) TV (b) TV cos 8
The work done on the body in time T is proportional to:
., (c) (Tcos8-)lmg)V (d) zero
(a)
,
!:r (b) "-- T 53. A 'particle free to move along x-axis is acted upon by a
" force F = -ax + bx 2, where a and b are positive constants.
For x ~ 0, the correct variation of potentia l energy function
U(x)is best represented by:
48. If the force acting on a body is inversely proportional to its U U
speed, then its kinetic energy is: . (J & K 2009)
(a) OH---r- (b) O' r - -f - -
(a) linearly related to time ,
(b) inversely proportional to time
(C;) inversely proport i ~nal to the square of tiqle
(d) a constant u u
49. Spring A and B are id~ntial except that A is stiffer thanB. If
work expended in spring A and Bare WA and WB , when (0) 0 r'---\-- (d) 0 f"----L.
they are stretched by same amount, while work expend ip
spring A and Bare WA and Wn when they are stretched by
"
the same force then:
54. Two blocks are connected to an ideal spring of stiffness
(a) WA >WB (b) WB >WA
200N/m. At certain moment, the two blocks are moving in
(c) WA >Wn (d) WB >W~
opposite directions with speeds 4 ms- 1 and 6ms - I , and the
50. A small block of mass mis kept on a rough inclined surface
instantaneous elongation of the spring is 10 em. The rate at
of inclination e fixed in a lift. The lift moves up with a
unifonn velocity v and the block does not slide on the which the spring energy [ ~2 ) is increasing is :
incline. The work done by the force of friction on the bloek
in time t will be :
(a) 500 Jls (b) 400 Jls
(a) zero (b) mgvtcos Z 8 (0) 200J/s (d) 100 J/s
(c) mgvtsin Z e (d) mgvtsin 28 55. 'The potential energy function of a particle in the x - yplane
51. A particle is projected at t = 0 from a point. on the ground is given by U = k(x+ y), where k is a constant. The work
with certain velocity at an angle with the horizontal. The done by the conservative force in moving a particle from
(l,l)to (2,3) is,
power of gravitational force is plotted against time. Which
of the following is the best representation? (a) - 3 k (b) +3k
(c) k (d) none of these
56. Two equal masses are attached to the two ends of spring of Minimum horizontal force on M \ so as to move M 2 is :
spring constant k. The masses are pulled out symmetrically (,) " M , g ( b ) "(M,-M,)g
to stretch the spring by a lengthxoverits natural length. The
work done by the spring on each mass is : (c) "CM, +M 2 )g (d) ~('M, +M,)g
(,) ~kx2 (b) _~kx2 61. IfW\, W2 and W3 rep resent the work
2 . 2
done in moving a particle from X to
(d) Y along three different paths 1, 2 and ·
3 respectively in the gravitational 2
57. A self-propelled vehicle of mass m, whose engine delivers field of a mass· M, then: 3
a conslant power P, has an acceleration a"" (P / mv)
(a) WI:;: W2 = W) X .'-_~
(Assume that there is no friction). In order to increase its FIg. 4.39
(b) WI <Wz <W3
velocity fromv \ tov 2 , the distance it has to travel will be:
(c) WI >Wz >W3
(a) 3P (v 2 ~v2) (b) ~(v3 _ v 3 )
m 21 3P2! (d) W2 >W3 >Wl
62. Two particles of masses ml and m2 repcl each other by a
(c) !!!...(v 2 _v 2 )
3P 2 1 force F ;, ~ where C is a constant and r the separation
58. In Fig. 4.36, AB is a
between nil and m2 . Mass m l is fixed and m2 is released

11
massless rod of length '21'.
The rod is connected at end . from rest from on initial separation Ii froJ!! mI. Speed of m 2
Ala a spring of spring fl-<=M:!.I=:;:"'=!.=:;! when it is at extremely large distance from m], while
moving away from ml , is:

[r
constan~' k' and it is fixed at
o such that it can rotate
B (,
about 0. A mass' M' falls on (a)
C
(b) ~
the other end from a height 1112 Ii 2m2 Ii
, H', Assuming that the Fig. 4.36
spring is initially unstretched and also that there is no loss
of energy, maximum elongation of the spring is:
(,) Mg
k
(b) Mg~"
k
(c)
[r.2C
lnzli .
(d)
[r
~
'hill Ii

63. A mass 10 kg moving with a speed 10 emls on ·a smooth


horizontal surface hits a spring as shown in Fig. 4.40. Work

(d) Mg~l- 2kH done by the spring force oir the mass is :
Mg

~
tOcm/s . e::;Jk=40
N/m _
tOkg . _ _-
59. In Fig. 4.37, a block is pulled by a
constant horizontal force on a
ULSea: _. .
Fig. 4.40
smooth surface. Work done on the &.£
block as a function of speed (a) 0.05 J (b) -0.05J
acquired by the block can be described by: (c)-O.IJ (d)O.IJ

W~, W~,
64. An object of ll)ass 4 kg' is dropped from a height 20 m. It
(a) (b) reaches the ground with speed 16 mls. Work done on the
body by air friction is :
(,) 288 J (b) -288 J
(c)
.
WI /
~,
... (d)W
L
60. TW9 blocks are connected by a spring as shown in Fig.
. ,
(c) 312J (d) -312J
65 . Initially one half of a uniform ehain of length' I' and mass
'M' overhangs a frictionless table with the other half of it
lying on the table. Kinetic energy of the chain when it
4.38. The blocks are placed on a horizontal rough surface
finally slips off the tableis:
with').1' as the coefficient offriction between the blocks and
the surface. 4 2
(aJ - Mgl (b) -Mgt
5 7
2 3
(c) - Mgl (d) -Mgt
9 8
Fig. 4.38
66. Three objects are thrpwn -from-the top of a tower_with the 70. A particle' A' is moving along a circle with unifonn speed
samc_speed---object A thrown vertically upward,_object B and another particle B is in circular motion with variable
verticallr downward and C alo~g the horizontal, 'then: speed. II WA and WB are the work done in thfi:ir circular
(a) object A hits the.ground with maximum speed motions:
(b) object B hits the ,ground with maximum speed ' (a)" WA =WB =·0 (b) w..4 =O,WB ;to ·
(c) object C hits the ground:with maximum speed (c) WA ;t 0, WB =0 (d)WA=WB;tO
(d) all the objects -hit the ground with. equal speeds . 71. A particle ofma·s5' m' is moving along X -axis.under a force
67. Two masses, each 5 kg, are attached to the two ends of a F= k-Jx.
Work dondor the motion of particle from x = 4m
spring 'of spring constant 40 N/m. The masses arc -pu!led tox=9misnearly:
away from each other and th~ spring gets stretched by a (,) 19.2 k (b) 8.6 k (e) 32.3 k (d) 12.7 k
length 20 em. "Work done by the spring on each mass is : 72. An electric pump is used to fill an overhead tank qf capacity
(,) -0.4J (b) -0.81 9m 3 kept at a·height 10 m.above ground. If the p·ump takes
(e) -0.61 (d) 0.81 5 minutes to fill the tank by consuming 10 kW power, ·
A person of mass 75 kg carries a block or' mass 15 kg from efficiency of the pump should be": (Takeg. = 10mi s~)
ground to the top of a lower 10m high in 150 sec. The [PET (Kerala) 2007)
persoo"exerts"a constant' power throughout··this duration and
(a) . 60% (b) 40% (c) 20% (d) ·30%
his power-time graph is as shown in Fig. 4.49 . lfthe person
carries the samc object in a time 300 sec instead of 150 sec 73. A pump motor is used to deliver water at a certain rate from
by exerting a constant power, the correct power time graph a given pipe. To·obtain, twice as much water from the.same
will be: pipe, in the samc time, the power of motor has to be
increased to: [EAMCET (Engg.) 2006)

I P'oj~·..~~---:c,:,-i-
(a) 2 times (b) 4 times
(c) 8times (d) 16 times
74. A bullet when fired at a target has its velocity decreased·to .
150 sec 50% after penetrating 30 cm into it. Additional thickness it
-I will penetrate incm Qefore coming to rest is: ·IAIEEE 2005J
.Flg·.4.41
(a) 10 em (b) 30 cm
(c) 40 cm (d) 60 em
75 . . An object of mass 4 kgis moving along X -axis such that its
3
position x varies·with time t as x 2 = 16/ , with x in metre.
arip t in sec. Work done as a function of time· can bc shown
as:

-I
76. In Q..75, work done from t= 3 sec to t = 5se.o is:
(a) 1441 (b) 1761 (e) 2961 (d) 3121
77. In Fig. 4.42, a block of mass 4 kg falls on a
spring fro~ a height 15 cm. If the spring
o
(d) none of the above
69. Mass of bob ofa simple pendulum is 10 gm. It makes 10
constant of the spring is 2000 N/m,
maximum compression .in the spring will
. 1'5 em

1
oscillations in .(5 second passes the· mean position with a Be : (Takc g = I Oml s2)
speed 4 cm/sec, wurk done by the tension in the suspension
· -3 OSCI·11· . ·(a) 14 em (b) 18.2 cm
tIlrca d m atlOn IS:
4 (c) 12.6 cm (d) -1"0 cm
FISiJ. 4.42 .
(a) 5.6 J (b) 6.8 erg (c) 12.4 erg (d) zero
78. Two cars A and B with mass of A being three times the mass 84. An object A of mass 2 kg and another B of mass 8 kg are .
of B are moving such that kinetic energies are equal. They connected to each other by a spring of spring constant 2000
are brought to rest by applying brakes using the same N/m. .The spring is compressed by 10 em and released.
braking force. After applying brakes, if distance travelled Immediately after release, the 8 kg mass moves with speed:
by A before coming to rest is SA and that by B is S B' then: I
(a) 2 mI, (b) - mI,
S, 2
(,) S • • 2S, (b) S " 2
I
(o) . 12m1, Cd) -ml'
12 ..
79. Two cars A and B with the maSS of A being three times the 85. A body of mass 2 kg is projected at 20 mls at an angle 600
mass of B are moving with equal speeds. Their engines are above the horizontal. Power due to the gravitational force
turned off at the same time and each comes to rest due to
on the block at its highest point is : (Take g = I Om/ s 2)
friction between the tyres and ground. The distance
travelled by A while coming to rest is SA. and the time taken ICET(J &K)20061
by it to come to rest after turning the engine off is t A and the (a) 200 walt (b) 120 watt
corresponding quantities for Bare S Band t B' then: (c) 180 watt (d)' zero
(a) SA ""Sa,t A >'s (b) SA =Ss.t A =ts 86. A body stans from rest and acquires a velocity V in time T.
(c) SA <S8,t A <IS (d) SA. >SB,t A >ts The instantaneous power delivered to the body in time' I' is
80. Kinetic energy of a particle moving along a circle of radius proportional to: (AIEEE 2005]
'R' depends on the distance as K.B. = CS 2 , then the force y y'
acting on the particle can be expressed as ; (,) - I (h) _ I '
T T
(,) 2CS (b) 2 CS v' y' 2
R (0) , I (d) - . 1 ·
T T'
(d) 2S 87. A car of mass 800 kg is being driven with a constant
R acceleration 1.5 m/ s 2 on a straight road against a constant
81. An object of mass 6 kg is moved with unifonn speed of 8 . resistive,force 72 N. When the speed of the car is 72 ktnIhr,
mls for 8 sec on a surface. Coeffi~ien.t of friction between the rate at which engine is doing work will be : .
the body and the surface is 0.6, work done for the given (a) 25440 watt (b) 28920 watt
2
motion 'oflhe body is: (Take g "" 10m / 5 ) (c) 27590 watt (d) 26325 watt
(,) 2212) (b) 2304) 88. A massless spring when
. stretched by a force 150N 10 kg
(0) 1192) (d) 3222)
82. Two springs A and B have force const~nts k A and k B' shows an extension of 1.5
respectively, with k A > k B . Both springs are stretched by m. This spring is arranged.
the same force, then the work done is: as shown in Fig. 4.44. A
block of mass 10 kg is
(a) same in both cases
released at rest on a Fig. 4.44
(b) greater in case of spring A
frictionless .incline. It
(0) smaller in case of spring A slides down and compresses the spring by 3 m before
(d) none of the above coming to rest momentarily, distance by which the block
A B
83 . . Two objects A and Bare slides before hitting the spring is : (Take g "" IOm/ s 2)
connected 10 each other by a
spring as shown in Fig. 4.43, [t:j] (a) 4m
(0) 9 m
(b) 6 m
(d) 12 m
and placed on a horizontal Fig. 4.43
frictionless table. The spring 89. A human heart pumps 60 cc of blood through the arteries at
is compressed by 8 em and released. If rnA and rn B are the each beat against an average pressure 10 cm of mercury. If
masses of A and B, respectively. then the ratio of their the pulse beats 72 times i~ a minute, the pumping power of
· · . ·"b (K.E.), heart will be neaTly: (Density of mercury '" 13.6 gmlcc,
kmetlc energies WI e : 2
(K.E.). g "'10m / 5 )

(,)
m, (0) F,; (a) I W (h) 2.2 W
(0) 0.4 W (d) 9.6 W
V;;;;
90. An objec:t is dropped on sand from a height and it penetrates 9S. In Q. 94, maximum kinetic energy of the object is:
some distance into the sand before coming to rest. Average (a) 241 (h) 321
resistive force exerted on the object by sand is: (e) 121 (d) lSI
(a) equal to the weight of .the object 96. The force acting on an object varies with the distance
(b) more than the weight of the object travelled by the object as shown in Fig. 4.46. Work done by
(c) less than the weight ofthe object the force in moving the object from x:= 2 m to x = 4 m is:
Cd) none of the above
91. Consider two incline rough surfaces A and B with angles of F(N) 4
inclination 45° and 60°, respectively. Height of the top edge
of each surface from ground is same. A particle placed on t
top of the incline' A' rea~hes the bottom with a speed v A •
When the same particle is placed on top of the incline B, it o 6 10 14
reaches the bottom with speed vB' Assuming the same -x(rn)
coefficie'1t of friction in the two cases: Fig. 4.46
(a) vA = V B (b)VA>V B (a) 41 (h) 61
(c) v~ <VB (d) it can .not be decided
(d) 21
(e) S 1
92. A particle of mass 'M' is moved under'a constant power Po.
97. Velocity-time graph ofa particle of mass 0.5 kg is shown in
At some instant after the start, its speed is v and at a later
Fig. 4.47. Work done on the particle from t = 0 tot = 12s is:
. instant, the speed is 2v. Neglecting friction, distance
, travelled by the particle as its speed increases from v to 2v
is: v (m/s) 8
l2Mv 3
(a) - -- (b)
8P0 v' t
SPo M
4 8 12
9Mv 3 7Mv 3
(e) - - (d) - - -tis)
5PO 3PO Fig. 4.47
93. A particle of mass 1 kg is moving along a straight line. Its
velocity-time graph is as shown in Fig. 4.45, Work dorye ~y (a) 121 (b) IS 1
the resultant of all forces acting' on the particle from t ::: Oto (e) 10.61 (d) none of these
1 := 8sis: 9s. A particle is moving along a straight line under the action of
v(m/s) a force. Its kinetic e,n ergy varies with time as K.E. = 7t.
5 ,F orce acting on the paiticle :
(a) is increasing with time (b) is decreasing with time
(c) is constant , (d) is zero
f--'-T'---'-- tis)
8 99. Force acting on a particle varies with the distance travelled
by the particle as F ~_ 1_ , then power due to this force
(x) l/ 3
-5
varies with x as:
Fig. 4.45 1l3
(a) x
(a) 251 (b) 501 1
(c) zero (d) 12.51 (c) x 3 (d) xO
94. An object of mass 2 kg is placed at rest on a frictionless 100. Force acting on a particle moving along a straight line
horizontal surface. A horizontal force given by
~ , .
F := (x - 9) i N acts on it along X -axis. Kinetic energy of
varies with the speed of the particle as F oc _1_. If the
v'
motion of particle starts from rest at t = 0, work done in time
the object will be maximum at :
, t' depends on t as :
(a) x := 3m (h) x~ - 3m '
(a) ' (2 / 3 (b) ,0
(c) x:=+ 1.8 m (d) x:=-1.8m
(c) t 5/3 (d) t - 1I3
101. A body is moving along a straight line under a force that 107. If the potential energy ofa gas molecule is U = M ~~, ft,
delivers constant power. Distance travelled by the body in 1'6 1'12
time 't' is proportional to: [BV (Pune) 20061 and N b.eing positive constants, then the potential energy a
(a) (2 (b) t lf2 equilibrium must be,: [PET (Kerala) 2006
(c) t (d) (3/2 M'
(a) zero (b) 4N
\J2. A bullet loses..!. of its speed as it passes through a plank.
N' .MN 2
Number of such" planks required to stop the bullet are:
(e) - (d) - -
4M 4
(a) n (b)
,,' (ej
NM2
--
2n-\ 4
(0) "
n- I
(d) ,,+.rn lOS. A simple pendulum is released from A
as shown in Fig. 4.'48. If' m' and' l' ,
103. A bullet of mass' m' is fired with certain velocity from a gun represent the mass of bob and the ,,:30"
.
of mass' M', Gun, which is attached with one end of spring, length of pendulum respectively, the ,,
compresses it by distance' d'. If' k' is spring constant,
,,
gain in kinetic energy at B is: ,, B
velocity of the bullet is: IAMU (Medical) 2006) . [PET (Kerala) 20071 Fig. 4.48
(a) ~.JkM (b) ~.[k;, (a) mgt (b) mgt
m M

J;J;.
2 .fi
(e) dm~ MkI Cd) mk -
. . dM (0)
..f3
-mgt (d)
2
..f3 mgt
2
104. A ball of mass 0.2 kg is thrown veltically upwards by
109. A block of mass 10 'kg 'is 'moving in x -direction with
applying a force by hand. If the hand maves 0.2 m while
constant speed 10 mlsec. It is subjected to a retarding forc
applying the force and the ball goes upto 2 In height further,
F =~0.1 xjou!e/metre dur'ing its travel fromx =20 metre t,
force exerted by hand on the ball is: (Take g =: 10m! s 2) x =30 metre. Its final kinetic energy will be: [A-IIMS 2005
[AIEEE 2006[ (a) 475joule (b) 450 joule
(a) 22 N (b) 4 N (c) 275joule (d) 250 joule
(e) 16 N (d) 20 N 110. When a ,ball is thrown up ve~ically"with velocityv o' :
105. Potential energy of a I kg particle free to move along X a
reaches inaximum height of h. If one wishes to triple ~h
-axis is given by . . maximum height therl.' the . ball should be thrown wit:
x4
V(x)'" 4~x2
(
2] joule
v~ l odty:
(a) v.o..f3
'
(b) 3vo
[AIIMS ~005

3
Total mechanical energy of the particle.is 2joule. Then the (c) 9vO (d) ZV o
maximum speed in mls is: [AIEEE 20061
III. When the kinetic . energy of a. body is doubled, it
(aj 2 (b) ~ momentum increases' by ..... times. [Kerala PET 2005
.fi
(a) .fi (b) 2
(0) .fi (d) _I
.fi (e) 4 (d) 2.fi
I
106. A 2 kg block slides on a horizontal floor with a speed 4, mls. (e) .fi
It strikes an uneompressed spring and compresses it till the
block is motionless. The kinetic friction is 15 N and spring 112. The potential energy of a long spring when stretched by
constant is 10,000 N/m. The spring compresses by: em is U; If the spring is stretched by 8 cin the potentic
[AIEEE 2007[ energystor~d in it is: [CBSE2006
(a) 8.5 cm (b) 5.5 em (a) 4U (b) SU
(e) 2.5 cm (d) I I.Oem (0) 16U (d) U
4
11 3. ,' A particle of mass 100 g is thrown vertically upwards with a 120. A force of(5+ 3x)N acting on a body of mass 20 kg a long
. speed of 5 mls. The ,:,,",ork ,d one by the forc e qf gravity the x -axis displaces it from x '" 2 m to x:: 6 m. The work
. during the time the particle goes up is : done bylhe force is: [UPSEE 2009J
(AIEEE 2006; UPSEE 2007) (a) 20J (b) 48J
(a) 1.25 J (b) 0.5 J (0) 68J (d) 86J
(e) -O.5 J (d)-1.25 J 121. A rock of mass III is dropped to the ground from a height h.
114 .. Two springs P and Q of force constants kp and A second rock w ith mass 2 m is dropped from the same

kQ(kQ ' ~ k; )are s~lched ~y ap~IYing forces of equal


height. W hen second rock stri.kes the ground, what is its
kinetic energy ? IUPSEE 20091
(a) twice that of the first rock
magnitude. If the energy stored in Q is E, then the energy
stored in Pis:· (Kerala PET 20091 (b) four times that of the first rock
(a) E (b) 2E (c) the same as that of the first rock
(e) E 18 (d) E 14 (d) half that of the first rock
(e) E 12 122. If the kinetic energy of a body changes by.20% then its
momentum would change by: tWB (JEE) 20091
11 5. A particle moves in a straight line with retardation
proportional to jt~ disphicemenl. ,Its loss o f kinetic energy (a) 20% (b) 24%
for anydisplacementx is proportional to : (UPSEE 20071 (c) 4001c. (d) nonc of these
(a) x2 (b) eX 123. The height of a waterfall is 50 m. If g =: 9.8ms - 2 , the
(e) x (d) logex difference between the temperature at the top and the
11 6. An engine pumps up 100 kg ofwat"er through a height of 10 bottom of the waterfall is: [WB (JEE) 2009[
min 5 s. Given th!!l, the -efficiency of engine is 60%. If ' . (a) I.I7°C (b) 2.I7°C
g = 10m/52 , the power ofthis engine is: [RPMT20071 (e) O.II7°C (d) 1.43°C
(a) 3.3 kW (b) ' 0.33 kW 124. T wo masses of M and 4M are moving with cqual kinetic
(e) 0.033 kW (d) 33 kW cnergy. The ratioofthcir linearmomenla is: IDCE 20091
{aj 1 ,8 (b) 1 , 4
117. .A n engine develops 20 HP whcn rotating at a speed o f 1800
rev/min, The torque that it delivers is: (e) 1,2 (d) 4, I
[Uttarakhand (Med.) 20071 125. The amount of energx released when on~ microgram of

(a) 400 N-m (b) 60 N-m matter is annihilated is : [Karnataka CET 20091

(e) 40 N-m (dj 80 N-m (a) 0.25x 10 5 kWh (b) 9x 1010 kWh
11 8. A body is dropped from height 8 m. After striking thc (c) 3xlO lO kWh .(d) none of these
surface it' rises to 6 m, what' is fractional loss in kinetic 126. The work done by d force acting on a body is as shown in
energy during .impact? Assuming air resistance to be the graph. 'J!lc total work done in covering an initia l
negligible. [BH U (Mains) 20071 distance of20 m is: IKarnata ka CET 20091
(a) 2/5 (b) 1/4
20
(e; 3/4 (d) 115
15
119. Which of the following statement::: is wrong? F (in N)
10
(UPSEE 2009(
5
(a) KEofabody is independent of the direction of motion
(b) .In an clastic collision of two bodies, the momentum o.5 10 15 20 25 30
and enel'gyof each body is conserved S (inm) -
(e) If two protons are brought towards each other, the P.E. Fig. 4.49
o f the system increases (a) .225 J (b) 200 J
(d) A body can have energy w ithout momentum (e) 400 J (d) 175 J
ANSWERS

I. (e) 1. (h) 3. ee) 4. (d) 5. (e) 6. (a) 7. (e) . 8. (e) 9. (a) 10. (b) 11. (e) Il. (d)
13. ee) 14. (d) IS. Cd) 16. (h) 17. (a) 18. (e) 19. (h) ZOo (a) 21. Cd) 22. ee) 23. (a) 24. (a)
15. (d) 16. (e) 17. (b) 28. ee) 19. (d) 30. (a) 31. (a) 32. (e) 33. (d) 34. ee) 35. (a) 36. (b)
37. (a) 38. (b) 39. (a) 40. (a) 41. (e) 4l. (d) 43. (b) 44. (a) 45. (e) 46. (a) 47. (e) 48. (a)
49. (a) 50. ee) 51. (e) 51. (b) 53. (e) 54. (e) 55. (a) 56. (d) '7. (b) 58. (e) 59. (b) 60. (d)
61. (a) 62. ee) 63. (b) 64. (b) ... (d) 66. (d) 67. (a) 68. (a) 69: Cd) 70. (b)' 71. Cd) 72. (d)
73. (e) 74. (a) 75. (e) 76. (a) 77. (d) 78. Cd) 79. (b) 80. ee) 81. (b) 82. (e) 83. (b) 84. (d)
85. (d) 86. (0) 87. (a) 88. (e) 89. (a) 90. (b) 91. (e) 91. (d) 93. (e) 94. (b) 9'. (d) 96. (a)
97. (d) 98. (b) 99. (d) 100. (a) 101. (d) 101. (b) 103. (a) 104. Cd) 105. (b) 106. (b) 107. (b) 108. (e)
109. (a) 110 (a) Ill. (a) 112. ee) 113. Cd) 114. (e) 115. (a) 1J6. (a) 117. (d) 118. (b) 119. (b) 120. (e)
121. ~d) m. (dl 123. (c2 124. (c~ 125. Cd) 126. (b)

(B] More th an One Choice is Correct overhanging part equals 11(11 - 1) part of the chain length. I
the total work perfonned by the friction forces acting on th
I . Jf you lift a suitcase from the floor and keep it on the table,
chain by the moment it slides completely of the table is f
the work done by you does not depend on:
, and the friction coeffi cient between table and chain is ~
(a) the path taken by the suitcase Then:
(b) the time taken by you in doing so
(c) weight of suitcase
(a) ~= - ".
1+"
(b)
.-
~=~
1- 11
(d) frame of reference
2. If force is a lways perpendicular to motion:
(c) W =1-'( ' '''':11 )2 mig
2
(a) Kinetic energy remains constant
9. A body of mass III was slowly hauled
(b) Work done is zero up the rough hill by a force F which at
(c) Velocity is constant each point was directed along tangent
(d) Speed is constant to the hil,l. Work done by the force:
3. When a body is moving vertically up with constant velocity: (a) is independent of shape of
(a) Work done by lifting force is positive trajectory '-----=,--.,.",---
. 1 Fig. 4.50
(b) d epen ds upon vertlca
(b) Work done by lifting force is negative
components of displacement but independen.t (
(c) Work done by forc e of gravity is positive
horizontal component
(d) Work done by force of gravity is negative
(c) depe'nds upon both the components of displacement
4. Work done by fo~ce of friction:
(d) does not depend upon coefficient of friction
(a) can be positive (b) can be negative
10. Block i has no realtive motion with respect to wedge fixe
(c) can be zero (d) any of these to the lift as shown in fi gure during motion ~ l ormotion~2
5. Whcn work done o n a particle is positive: Direction
of motion-2
(a) KE increases (b) KE decreases .--------'.,
(c) KE remains constant (d) Momentum increases Direction
6. Which of Ihe following can be negative? of motion-1
(a) Kinetic energy (b) Potential energy
A
(c) Mechanica.1 energy (d) Energy
7. For an isolatcd system in the absence of any dissipative
effect: Fig. 4.51

(a) KE is conserved (b) PE is conserved (a) work done by gravity on block A in motion-2 is Ie:
(c) Energy is conserved (d) ME is conserved than in motio n~ I
8. A chain of mass' /II' and length ' l' rests on a rough surface (b) work done by normal reaction on block A in both tI
table so that one of its ends hangs over the edge. The chain motions will be positive
starts sliding off the table all by itself provided the
(c) work done by force . of.friction in motion-I may be (R) : In a conservative field, change of potential energy
positive
- of a system is always equal and opposite to the
(d) work done by force of friction in motion-I may be
negative
II. The potential energy of a particle due to certain system is
2. (A):
change of its kinetic energy.
An object A is dropped
from the top of an incline
B ? A

given by formula U = 100-Sx+ 1O(h:2 ,U and x are in SI at t = 0, as shown. It will


fall under gravity as
units. Ifmass of the particle is 0.1 kg then magnitude ofit's
acceleration: indicated by the arrow. At L.l!'' -:::-::-:c::--'
the same time, i. e., t = 0, Fig. 4.52
(a) at O.OS m from the origin is 50ms-2 another object B begins to
(b) at O.OS m from the mean position is 100ms-2 slide down the frictionless incline. The two objects
during their motion to the ground level, will move
(c) at O.OS m from t~e origin is ISOms- 2 with different accelerations, will reach the ground
(d) at O.OS m from the mean position is 200ms-2 level at different times but they will reach the
ground level travelling at equal speeds.
12. The potential energy U in joule of a particle of mass I kg
(R) : A constant force acts on body A but a variable force
moving inx-yplane obeys the lawU = 3x+ 4y, where (x, y)
acts on body B.
are the co-ordinates of the particle in metre. Ifthe particle is
o
at rest at (6, 4) at time t = then : 3. (A): A crane P lifts a car up to a cetlain height in 1 min.
Another crane Q lifts the same car up to the same
(a) the particle has constant acceleration
height in 2 min. Then crane P consumes two times
(b) the paI1icle has zero acceleration more fuel than crane Q.
(c) the speed of the particle when it crosses y -axis is 10 (R) : Crane P supplies two times more power than crane
mls Q.
(d) co-ordinate of particle at / = Isec is (4.S, 2)
4. (A): In the formula P = ~ E, then term E represents
13. An engine supplies a constant power' P' to automobile of 3
mass m starting from rest. At an instant of time I. translational kinetic energy per unit volume of gas.
(a) Velocity is inversely proportional to.JP (R) : In case of monatomic gas translational kinetic
(b) Velocity is proportional to Ji energy and total kinetic energy are equal.
5. A block is suspended from spring and block is in
(c) Displacement is proportional to II
V;;; equilibrium.

(d) Displacement is proportional to /3/2

1. (a) and (b); 2. (a). (b) and (d); 3. (a) and (d); 4. All; 5. (a) m
and (d); 6. (b). (e) and (d); 7. (e) and (d); 8. (b) and (e); 9. (a)
Fig. 4.53
and (e); 10. (b). (e) and (d); II. (a). (b) and (e); 12. (a). (e)
(A): Force acting at two ends of spring will be action
and (d); 13. (b). (e) and (d).
reaction pair.
[C) Assertion-Reason Type Questions (R): Action and reaction force are of same type and act in
opposite directions.
(a) If both A and R "e true and R is the correct
explanation of A. 6. CA): All surfaces are smooth.
Block A comes down along
(b) If both A and R "e true but R is not correct A
the wedge B. Work done by
explanation of A.
normal relation (between A
(e) If A is true but R is false. and B) on B is positive while
(d) If A is false but R is true. on A it is negative. B
(e) If both A and R are false. (R) : Angle between normal Fig. 4.54
I . CA): Mechanical energy ofa system is always conserved reaction and net displace-
irrespective of nature offorces acting on the system. ment of A is greater than 90° while between normal
reaction and net displacement of B is less than 90°.
ANSWERS Column - I Column - II
1. Cd) 2. (e) 3. (d) 4. (e) 5. (d) 6. (a) (a) By gravity (P) 1441

IDJ Integer Type Questions (b) By nonnal-reaction (q) 321

1. A car of mass M is accelerating on a level smooth road (c) By friction «) 561


under the action of a single force F acting along the (d) By all the forces (s) 481
direction of motion. The power delivered to the car is
constant and equal the p. If the velocity of the car at an (1) none
3
instant is v, then after travelling a distance of 7Mv the 2. System shown in Fig. 4.57 is released from rest at t = 0.
3p Friction is absent and string is massless. In time t = 0.3 s.
velocity becomes k:u then k is : Takeg=lOm!s2.
2. The potential energy U in joule of a particle due to certain
gravitational system obeys the lawU = ax + 4y, here x and
yare in metres. Assume there no other forces acting on the
particle aparts from these potential field forces. If mass of
the particle is 1 kg and it is at rest at (6m, 8m) at time t = 0,
then work done by the forces on the particle from the initial
positions to the instant it crosses the x ~axis is 50 J. Find the
value of a (which is constant).
3. A spring is connected between two blocks as shown in the . 2 kg
diagram. Initially the blocks dre held stationary and Fig. 4.57
suddenly they are released. Find the ratio of maximum
kinetic energy for the block of mass mto that of mass 2m.
Column - I Column - II
(a) Work done by gravity (P) - 1. 5 J
on 2 kg block
Fig. 4.55
(b) Work done by gravity (q) 2 J
4. Ifan ideal linear spring is stretched by x then energy stored on 1 kg block
in it is E and when it is stretched by a further]x then energy (c) Work done by string on «) 31
stored adds a further kE. Find the value of k. 2 kg block
5. A rope having unifonn mass distribution over its length can
(d) Work done by string on (s) - 21
be over hanged by half of its length without sliding at the
1 kg block
edge of a rough table. If three quarter of the rope is over
hanged and released, then acceleration of the rope, 3. A body is displaced from x = 4m tox = 2m along thex~axis.
immediately after release, is xg. Find the value ofa. For the forces mentioned in column-I, match the
corresponding work done is colunm-II.
ANSWERS
Column -1 Column - II
I. 2 2. 3 3. 2 4. 8 S. 1 .
~
(a) (p) Positive
F =41
[EJ Match the Columns
~
(b) (q) Negative
I. A block of mass m is stationary with F = (4! -4])
respect to a rough wedge as shown in
Fig. 4.56. Starting from rest the system (c) ~

F =-41
«) Zero
Fig. 4.56
is going. upward with acceleration a ,
then in time t, (m = 1 kg, 9 =30°, a =2m/s2 , / '=- 4 s). work (d) ~ ..
F=(- 41- 4j)
(s) IWI=8units

done on block.
4. A pm:ticle can be moved in the xy-plane from Oto P along (d) If the block is (s) Electromagnetic
different path's, as shown -in the Fig. 4.58. A force that disconnected from the energy
continuously acts on the particle during its motion along spring ,nd released
-> • •
any path is F ::: 4 i + 6 j newton. Consider action of only the from some height
given force and match Columns~1 and II : above the earth, so that,
it falls freely towards
y
earth, what formes) of
energy will be there
Mr-->-=."p(8m, 8m) during its motion?

6. Fig. 4.59 (a) shows potential energy as a function ofx for a


particle that is moving in the x -direction under a
conservative force.
,"-----+-i---X
L Fig. 4.59 (b) shows potential energy as a function of x for
Fig. 4.58
another particle that is l110ving in the x -direction under a
conservative force.
Column -I Column - II
Fig. 4.59 (c) shows interaction potential energy as a
(,) Work done is 80 J (p) For the path OP function of x for a system of two particles that could move
in the x -direction under the mutual conservative forcc.
(b) Mechanical energy is (q) For the path OLP Matr.h Columns-I and II :
conserved
u
(e) Linear momentum is (r) For the path'OMP
not conserved ~o

(d) Angular momentum (s) For the path ONP ~~ 0:


'~.~ ../ ! '"
about 0 is not <L~O,L~_--'-'--o---X
conserved . X2 X1 X3
. - Positive x direction
5. A block of mass' m' is connected to the lower eQd of a (,) (b)
massless vertically suspended spring. The ball is displaced
slightly downward and released; it oscillatcs up and down. u
. Match Columns-I and II : . ~o
~.g
Column - I .
Column - II :0:>0
.~.~
0.. 0 Xl X3 - Positive x direction
(,) In the system of the (p) Kinetk energy Or-~~f-~--~~----x
block, the spring and
the earth, what formes)
of energy will be there (e)
during the motion of Fig. 4.59
the block?

(b) In the system of the (q) Elastic potential energy Column - I Column - II
block and the spring,
what fonn -(s) of energy (,) For the system in Fig (p) Equilibrium position
will be there during the (,) will be x:::x J ,nd
motion? equilibrium is unstable

(e) While oscillating, (,) Gravitational potential (b) For the system in Fig. (q) Equilibrium position
the block passes its " energy (b) will be x ::=X j ,nd
equilibrium position, equilibrium is stable
I
what formes) of energy
will be there in the
systcm of block, spring
and earth?
(c) For the system in Fig. (r) Force on the particle for 3. Tension in the string at t =3.2 s,is:
(c), let one of the x <Xl is in the negative (a) lOON (b) 24N
particles be at x = 0 and x -direction with x > xl' (c) 48 N (d) zero
consider different it is in the positive
4. Height of the ~ kg object from the floor when the 6 kg
positions of the other x -direction
object is moving at 4 mls is :
particle, say, Xl ,x2 and
x). Keeping in view the (a) 5.2 m (b) 1.6 m
force acting on the (c) 2 m (d) 4m
other particle at ~1 ,x2 5. Maximum height above the flooi to which theA kg object
and x) rises is :
(d) Consider a particle that (s) Force on the particle for (a) 8.2 m (b) 10.8 m
performs SHM along x=x) will be in the (c) 9 m (d) 11.4m
the x -direction with negative x -direction
Paragraph - 2
x=x" as the mean
position x 2 and x) are Force acting on a particle moving y
tWo other 'values of x -+ " 2"
'x;
such that" '< Xl and
in theX-Y plane is F=(yi+x j)
N, x and yare in metre. As shown M1--+--;,A(10m, 10m)
x) >x\; then
. in Fig. 4.61, the particle moves
from the origin' 0' to point A (10
ANSWERS
m, 10m). The figure shows three O~+---;LL--- X
1. a~ t; b-+ p; c-t s; d·-} q paths, OLA, OMA and OA for the Fig. 4.61
2. a -+ r; b -+ p; C--i: s; 4:4:..q motion of particle from 0 to A.
- ' ;"~/';~~ -' > ,', Answer the following questions:
3. a-t q,s; b-4 q,~; 'c~'p,s; d-)o p,s
6. Which of the following is correct?
4. a-+ p,q,r,s; b-+ p, cj,r,s; c-+ p,q,r,S; d -+ p,q,r,s
(a) The given force is conservative
5. a-+ p,q,r; b-+ p,q; c ~ p,r; d-l> p,r (b) The given force is non-conservative
6. a-+q,s; b-+ p,r; c-t g,s; d-+q,s (c) Conservative or non-conservative nature of force can
not be predicted on the basis of given infonnation
[F] Linked Comprehension Type (d) There is . equal probability for the force being
Paragraph - 1 conservative or non-conservative
Consider the arrangement 7. Along which of the three paths is the work done maximum?
shown in Fig. 4.60. An object of (a) OMA
mass m l "" 4 kg is connected to (b) OA
anotherobjectofmassm 2 =6kg (c) aLA
by a massless string that passes
(d) Work done has the same value for_all th~ thr_ee paths
over a massless and frictionless
pulley. Initially, the 4 kg object T
9m
8. Work done for motion along the path OA is nearly:
(a) 1000 J (b) 629 J
is in contact with the floor and
the 6 kg object is held at rest at a (c) 383 J (d) 437 J
height 9 m above the floor. At Now consider another situation
t = 0, the 6 kg object is released.
Fig. 4.60
-+ " " y
(Takeg=lOm/s 2 ). A force F = (4i + 3j) N acts ona

Answer the following questions: particle of mass 2 kg. The particle L


under the action of this force, olc-+--ir----X
1. Speed of the 4 kg object just when the 6 kg object hits the
moves from the origin to a point A
floor is :
(4 m, - 8 m). Initial speed of the M'r -<>-" A (4m, -am)
(a) 6 mls (b) 8 mls particle, i. e., its speed at the origin
(c) 4.5 mls (d) 7.2 mls is 4 mls. Fig. 4.62 shows three
2. Tension in the string at t =2.5 s is: paths for the motion of particle Fig. 4.62
(a) 100 N (b) 18 N (c) 48 N (d) zero fromOto A.
9. Which of the following is correct? Paragraph - 4
(a) The force is conservative In the shown diagram two point m
(b) The force is non-conservative masses m are joined by a massless
(e) Conservative or non-conservative nature of the force rod of length 5.5 m. The lower end
L
cannot be predicted on the basis of given information of the mass system is dragged with
constant velocityv o ' rightw·ards.
Cd) There is equal probability for the force being
Initial value ofa is 53°.
conservative or non-conservative '
10. Speed of the particle at A will be nearly: Fig. 4.64
Answer the following questions:
(a) 5.6 mI, (b) 3.6 mI,
18. The initial kinetic energy of the system will be:
(c) 2.8 mI, (d) 4.4 mI,
11. Ifpotential energy atO is 32 J, potential energy at A will be:
2 2Smv~
(a) mvo (b) - 1-6-
(a) 401 (b) 241
(c) 32 I (d) 14.5 I 25mv 2 2Smv~
(c) 32 0 (d) -~
18
Paragraph ~ 3
19. The kinetic energy of system when a is 37° will be:
A particle is moving along X -axis under a conservative
force and its potential energy U varies with x co-ordinate as 2 2Smv~
(a) mvo (b) 16
shown in Fig. 4.63. It is evident that the variation has an
oscillating behaviour. P,Q,R,S,X,Y and Z are points 25mv~ 2Smv~
indicated on the curve. (c) 32 (d) ---
18
U (xl in Joule
20. The work done by the dragging force during the angle
10 p z change from 37 0 to 300 ifvo' = 3m/s :
2 2
(a) 25mv o (b) 2Smv o
18 32
17Smv~
(c) 288 (d) zero
- 10
S
Fig. 4.63 Paragraph - 5
Answer the following questions: In the shown Fig. 4.65, a _10m/S 2
12. Force is positive at :
(a) QandR (b) XandY
block of mass 1 kg is dragged
by a force 10 N. It is observed
from three reference frames
o
1 kg
(c) Qonly
13. Force is negative at :
(d) P,Q,R and S ground, A and B. A is an
inertial frame moving with G .10N

(a) Yonly (b) S , X,YandZ constant velocity 10 mls and


_10m/s
(c) XandY (d) Qonly
14. Force has a maximum magnitude at;
B is a non-inertial frame
moving with constant
acceleration 10 ml s 2. The
o Fig. 4.65
(a) P,SandZ (b) QandY frame B and mass m of I kg
(c) PandZ (d) RandX starts simultaneously at t = 0 from rest. Based on above
15. Equilibrium is indicated by which point (s)? data, answer the following questions. (All move in
horizontal direction)
(a) RandX (b) P,SandZ
Answer the following questions:
(c) Sonty (d) PandX
21. Net work done on mass m in 1st sec as observed from
16. Equilibrium is stable at :
ground:
(a) P (b) S
(a) 50 I (b) - 50 I (c) 100 I (d) zem
(c) R (d) X
22. Net work done onmassmin 1st secas observed from A is:
17. Equilibrium is unstable at :
(a) 50 I (b) -501
(a) P (b) S (c) R (d) X
(c) 1001 (d) zero
23. Network done on mass min lstsec as observed fromS is: 4. Can kinetic energy of a system be increased or decreased
(a) SO J (b) -50J without applying any external force on the system?
(c) 100 J Cd) zero [Ans. Yes; By doing work through internal forces, i.e., converting
other foons of energy into KE, e.g., in blasting of a bomb KE
Paragraph - 6 increases while in inelastic collision it decreases]
Only a conservative force f(x) acts on a m = 1kg particle 5. Is it possible to exert a force which does work on a body
.constraint to move along x -axis. The potential energy U(x) without changing its kinetic energy? Ifso, give example .
is given by U(x) = 20+ (x_2) 2, where x is in metres. At [Ans. Yes; When a spring is compressed or when a body is
dragged on a rough horizontal surface with constant velocityJ
x = 5 m a particle has kinetic energy of 20 J. The sum of
potential and kinetic energy of the particle remains 6. A lorry and a car moving with the same kinetic energy are
, constant brought to rest by the application of brakes which provides
at every position.
equal retarding forces. Which of them will come to rest in a
Answer the following questions:
shorter distance?
24. The minimum potential energy of the particle is : [Ans. Both will come to rest after travelling same distanceJ
(a) zero (b) 20 J 7. In case ofa moving body as force of friction iSl-lmg, can we
(c)29J (d)49J regard I-Imgx as potential energy similar to mgh?
25. The maximum speed of the particle is : [Ans. No; PE can be defined only for conservative forces.]
(a) 1 mls (b) 29 mls 8. (a) Can' mechanical energy of a body be negative? If yes,
(c) 59m1s (d) .Js8m1s where and what does it mean? (b) Is'mechanical energy
always conserved? If no, under what conditions?
26. The least value ofx(position of particle is) will be:
[Ans. (a) Yes; When PE is negative and> KE; bound state (b) No;
(a) zero (b) - 2 Only for isolated system; that too in presence of conservative
(c) - 59+2 (d) 59+2 forces.]
9. Is it physically possible to have situations where E -U < O?
ANSWERS [Ans. No; E-U represents KE which can never be negative]

Paragraph-! 1. (a) 2. (c) 3. (d) 4. (d) 5. (b)


10. Fill in the blanks;
(a) The work done in holding a 15 kg suitcase while
Paragraph-2 6. (h) 7. (c) 8. (c) 9. (a) 10. (c) 11. (a)
waiting for a bus for 15 minute is .....
Paragraph-3 12. (a) 13. (c) 14. (d) 15. (b) 16. (h) 17. (a) (b) The work done on a'particle moving round a horizontal
Paragraph-4 18. (c) 19. (d) 20. (d) circular path of radius r with unifonn speed under a
Paragraph-S 21. (a) 22. (h) 23. (d) centripetal forceF is equal to. . . .. [EAMCET 1992)
. (c) A man pulls a bucket of water from a well of depth h. If
Paragraph-6 24. (h) 25. (d) 26. (c)
the mass of the rope and that ofthe bucket full of water
are mand M respectively, the work done by the man is .
[G] Subjective Type Questions
I. Is work done by a non-conservative force (say friction) (d) A body of mass M tied to a string is lowered at a
always negative? If your answer is no, give examples. constant acceleration of (g / 4) through a vertical
[Ans. No; (a) Work done by friction is zero if the body is at res! distance h. The work done by the string will be .....
under the action ofa force (b) When friction causes a motion work (e) Two springs with stiffness constantk J and k2 (k2 > k J )
done by force of friction is positiveJ are stretched by the same force. More work is done on
2 Does kinetic energy depend on the direction of motion the spring having spring constt.... IEAMCET 19931
involved? Can it be negative? Does it depend on frame of [Hint: See in § 3.4J
reference? (f) An open knife edge of mass M is dropped from a
[Hint: See points 1 and 2·in § 4.3J height h on a wooden floor. If the blade penetrates's'
[Ans. No; No; Yes] into the wood, the average resistance offered by the
3. Does a single external force acting on a particle necessarily wood to the blade is.... . IEAMCET 1992]
change (a) its kinetic energy; (b) its momentum? [Hint: Loss in ME = Work done against resistance]
(Ans. (a) No, (b) Yes; When force is perpendicular to motion, KE (g) A body of mass 0.5 kg is moving with a constant
remains constant (as W = 0) while momentum changes (as velocity of 2 tn/s. In order to bring it to rest in a
4 distance of 2 m, the work to be done is .....
direction of v changes)}
[aSFama~m~~]
(h) A boy ofmass42 kg eats bananas of980 calorie. If this d,
[Hint: p:: Fv:: m-v
energy is used to lift him up from the ground, the dt
height to which he can climb is ..... m.
or -"'" v",-
p asdv =< dvdx ::vdv]
(i) The power of a pump which .. ise. 100 kg of water in dx m [ dl dx dt dx
10 s to a height 100 m is .. , .. kW.
orv1dv :: (P I m)dx which on integration gives the desired result.]
[Ans. (a) Zero (as s = 0), (b) Zero (as a=90,,), (e) (M + m / 2) gh,
16. A rope is uncoiled by pulling the end of the rope
,Cd) (-3/4) Mgh, (el kl' (f) mg(h + a)1 s, (g) 1 J, (h) 10 m, (i) 9.8]
-t ..y -Jo -t
horizontally along a frictionless surface at the steady speed
11. The force (F "" 2 i + 4 j + 5 k ) N acts on a body and of 2 mls. The mass of one metre of rope is 0.25 kg. How
-+ -+ -+-+ much work is done in uncoiling 12 m oirope?
produces a displacement (S '" 3 I + 2 j + k )m. What is the [Ans. 6 J)
work done by the force? 17. An object of mass 5 kg falls from rest through a vertical
[Ans.19J] distance of20 m and reaches ground with a velocity of 10 mls.
12. A man weighing 60 kg climbs up a staircase carrying a 20 How much work is done by air resistance? (g = 9.8 m/s2)
kg load on his head. The staircase has 20 steps and each step
has a height 20 em. Ifhe takes 10 sec to climb, calculate the [Hint: By Work-Energy Theorem, W '" I!J(£ or
power. (g :::9.8m / s 2 ) W, + Wr D ~nll? Here W, .. mgh ~ 5x9.8 x20 _ 980 J and
[Ans. 313.6 Wl (l/ 2)mI)2=(l/2)x5x(10)2 - 250 J; negative work means
13. An object of mass m accelerates uniformly from rest to a displacement is opposite to air resistance.]
speed v F in time t F . (Ans. - 730 J]
(a) Show that work done on the object as a functi on of 18. A spring obeys Hooke's law with a force constant k. It
time t in tenns of v F and IF is requires 4 J of work to stretch it through 10 cm beyond its
unstretched length. Calculate (a) value of force constant k
w= !2 m(~)'t'
'F
and (b) the extra work required to stretch it through
additional 10 cm.
(b) As a function oftime t what is the instantaneous power [Ans. (a) 800 N/m, (b) 12 J1
delivered to the object? 19. An ideal massless spring S
[Hint: By Work-Energy Theorem can be compressed 1.0 m by
a foreeof 100 N. This spring
W"'..!.m(v2 _ 02): ..!. /tw2
2 2 is placed at Ihe bottom of a
frictionless inclined plane
or [asv=O+llf] which makes an angle of e=
3D'
30° with the horizontal (Fig.
or W :: ..!.m (~)2 t'[asa:: vF -0] 4.66). A 10 kg mass is Fig. 4.66
2 l tF tF released from rest at the top of the incline and is brought to
rest momentarily after compressing the spring 2.0 m.
so p::dW ::m(~)'tl (a) Through what distance does the mass slide before
dt l IF coming to rest?
14. When a constant force is applied to a body moving with (b) What is the speed of the mass just before it reaches the
constant acceleration, is the power constant? If not, how spring? (g = 10 mls 2)
would the force have to vary with speed for the power to be
constant? [Ans. (a) 4 m, (b) 4.47 mls]
20. A particle of mass 1 g 0
[Hinl: As a .. dv '" f:. -= cons;ant on integration give v"'!...." so,
~ m m executes an oscillatory
motion on the concave
,1
!!.J
1
F'
power:: Fv:: - t,i.e.,P oc t. Now for power to be constant,
m surface of a spherical "l/ e
Fv '" constant i.e., F co: (1/1))) dish of radius 2 m placed
on a horizontal plane ! R _ 200 em
[Ans. No; F « (1Iv)J
(Fig. 4.67). If the motion /
15. Show that the speed v reached by a car of mass m that is of the particle begins / rf>
driven with constant power P is given by, from a point on the dish Ig '_ ~ ________ J
at a height of 1 cm from ":'~c~m~:.o"~"":"ii"""
v=e:J
/J
the horizontal plane and FIg. 4.67
the coefficient of friction
where x is the distance travelled from rest.
is 0.0 I, find the total distance covered by the particle before the mass is pulled 0.500 m to the right and released what is
it comes 10 rest. its speed when it reaches equilibrium point x = O?
[Hlnl: Treating the dish as inclined plane and using conservation
of energy. i.e., [A... (.JF. - 2k X [I-.J:x2 ]i;L
+ £2
Loss in ME - WD against friction, we get
mgn '" j.lmgcO$OS (b) W =- /(x2 + 2kL(L - J x 2 + L2) ; (e) V = J O.800 mls]
2
i. e. , s: _h_ '" 10- x 200 - 1.005 m] 27. A block slides down a curved
j.lcos9 ·O.Ol x 199 frictionless track and then up an
incline as in Fig. 4.69. The
1h
21. An alpha particle with KE 10 MeV is heading towards a
stationary point nucleus of atomic number 50. Calculate the
distance of closest approach. (MNR 1995)
coefficient of kinetic friction
between the block and incline is _, ______J
FIg. 4.69
lllt.. Find the maximum height
[Ans.1 4.4 x 1015 m]
reached by block.
22. A very small sphere of mass 80 g having a charge q is held
at a height 9 m vertically above the centre of a fixed
conducting sphere of radius 1m, carrying an equal charge q.
(Ans. Ym&ll '"
"
l+).Ik cota
]

When released it falls until it IS repeUedjust before it comes 28. An obj ect of mass mis suspended from a
in contact with the sphere. Calculate the charge q. post on top of a cart by a string of length
L as in Fig. 4.70. The cart and object
(g :9.8m /s' )
move to the left al constant speed v. The
(Am.28t.t coul] cart collides with a bumper and comes to
23. A block of wood is pressed against a wooden surface with a rest. Find the maximum angle to which e Ag.4.70
force of ION and then rubbed on it to sweep a distance of the bob swings.
10 em. This is done 30 times. If the coefficientoffiiction be
0.4, calculate the heat produced. Can wood bum?
(Ans. 9= 2sin-l ( J~L}
(Ans, 2.8 cal; No, as this heat is very small]
29. A ball having mass m is connected by a strong
24. A lead bullet of specific heat 0.032 strikes against a target string o f length Lto a point and held in place in
with a velocity of 300 mls. If the bullet is completely vertical position. A horizontal wind exerts a
stopped by the target, find the rise in temperature of the
constant force of magnitude F. Find the
bullet assuming that the heat produced is equally shared by
maximum height to which the ball can reach. Fig. 4,71
the target a~d bullet. Given that J "" 4.2 joule/cal.
[Ans. 167 CO] [Ans,H = 2L 2]
1+(mgIF )
25. (a) How much heat energy is released in the explosion ofa
30. A heavy ball of mass m slides without friction down an
fi ssion bomb containing ' ·kg of fissionable material?
Assume that 0.1 % of the rest mass is converted into inclined chule whj~h forms a loop of radius R (see Fig.
released energy. (J = 4.2 J/cal) 4.72). At what height will the ball leave the chute and to
what maximum height will it rise afterwards if it begins to
(b) What mass ofT.N,T. would have to explode to prov ide
run down the chute without initial velocity from a height
the same energy release? Assume that. each mole of
T.N.T. liberates 8,20,000 calorie on exploding and the h '" 2R ? Assume the size ofthe ball to be negligible.
molecular welgnt of T.N.T. is 227 glmol:
----------------------------_.,,--,..,-----
,,
[Ans. (a) 2.14 x lOB cal, (b) 5.93 x 106 kg] :R
,,
,
26. Apanicle is attached between two rr==r=;.,===' h •
identical springs on a horiwntal l !""'a.
Iinctlon
· . Iess la " Ie. Both springs
have svring constant k and are l
i: / .,
! ",,-m
initial1y unstressed. (a) If the
1b==!;F~1±~6=8==,!j
Fig. 4.72
particle is pulled a distancex along g.4. 4 50
a direction pcrpendicular to the (Ans. 27R' 27R ]
initial configuration of the springs, find the force exerted on
the particles by the springs, (b) Determine the potential 31. A small body of mass m is located on a horizontal plane, at
energy of the system, if the mass is pulled a distance x. (c) If the point O. The body acquires a horizontal velocityv. Find
(a) the average power required to overcome friction , during
the whole motion, if the coefficient of friction ~ = 0.27,

en.
m = l.Okg andv = 1.5ms- l ; (b) the maximum instantaneous
power required to overcome friction, if the coefficient
Il = ax, where 0; is a constant and x is the distance from the
pointO.
mu2 -fiJi Fig. 4.74
[Ans. (a) -2 watt, (b) 1
2 [Ans. v =(2rg 13f2 ,<\I = 17 0 (for a =g)]
32. A system consists of two springs connected in series and
35. A unifonn string of mass M and length 2a is placed
having stiffhess coefficients K 1and K 2' Find the minimum
work to be done to stretch this system by Ill. synunetrically over a smooth peg and has particles of
masses m and m' attached to the extremities. When the
I 2
[Ans. 2" [KIK1 1 (Kl + K 2)](M) ] string runs off the peg, what is'its velocity?
33. A small body of mass m slides along the frictionless loop- M+2(m m') I
[A DS. ga
the-loop track. The body is released from rest from the M + m + m'
point A. Find the resultant force acting on the body at the a,
36. A unifonn elastic spring has length when the tension is
point C,ris the radius of the circular portion. Also find the T} and length a2 when the tension is Tz. What will be its
minimum value of'h' so that the body is on the verge of length when the tension is TJ ?
losing contact with the track, at the top.
[ADs.Length=) TCa! -~>"+al:-aT! !2 J
A T[ T2
m
37 . . Two blocks of masses m, = I kg and m2 = 2 kg are
connected by a flexible, elastic cable of equivalent force
constant of 1 N/m. Block m, is connected to the wall by a
h spring having spri,ng constant of 3 N/m. Coefficient of
c
ft:iction ·be~een ,block and surface is J.L = 0.3. Initially the
cable ane( sp..rJ.ng ~re unstretched, What is the velocity of
bloc~ 2 after"rele~se when block 1 has moved by 10 cm and
Fig. 4.73 block 2 by 20 em?
k2 = 3N/m
[Ans. (mg I r)[r 2 + 4(h - r)2 1'2,II = 5r 12]
34. A body is placed on top of a smooth hemispherical surface,
of radius r. The hemispherical surface is given a unifonn
~

acceleration a to the left. Find the velocity (v) of the body


relative to the surface, at the time oflosing contact. Find the
angle $ifa = g. P is the point where the body loses contact. Fig. 4.75
[Ans. 1.8 mlsJ

".
Conservation of Momentum
§ 5.1 Momentum v .. Constt.

!C
m .. Constt.
Linear momentum of a particle is a vector physical
quantity associated with state of motion and is defined as
product of mass of the particle m with its velocity v (not
--> 1
p
I
p
speed), i.e.,
--> --> v_ m_ m_
p= mv ... (i) Fig. 5.1

The adjective linear is usually dropped but it serves to This is an important relation connecting momentump
distinguish from angular momentum. Regarding linear to kinetic energy K and implies that a body cannot
momentum it is worth noting that: have momentum without having kinetic energy or
(1) Linear momentum is a vector physical quantity vice-versa. Further from this equation it is clear that:
(Fig. 5.2)
having direction of velocity, dimensions [MLT- l ]
(a) If P = constant (b) IfKE = constant (c) If m = constant
and units kg mls or N-s.
(2) Linear momentum depends on frame of reference,
K oc(l/in) p oc ..r;;; p cr.; fK

e.g., the linear momentum of a body at rest in a i.e. ,if different bodies i.e., if different bodies i.e., if different bodies
have same momentum, have same KE, have same mass, the
moving train, is zero relative to a person sitting in the KE will be max. for heaviest body will body having greatest
train while is not zero for a person standing on the lightest body. have max. momentum. KE will have max.
ground. momentum.
(3) Two bodies of srme mass and moving with same p _ Constt. KE = Constt. m .. Constt.
speed will have different momenta unless their
directions of motion are same.
-->
(4) As for a. body or particle I pl= ml v i so, linear
--> I
K
I
P
I
K
momentum will change if either mass or velocity or
both changes so: (Fig. 5.1) m_ m_ p-
~ --> Fig. 5.2
(a) If In = constant (b) If v = constant (c) If p = constant
pocv pocm vocl/m (6) In tenns of momentum Newton's II law is
i.e. ,ifdifferent bodies i.e., if different bodies i.e., if different bodies --> -->
have same mass, the are moving with same have same momentum
F =(d pldl) ... (iii)
mOl!Jentum will be velocity momentum the lightest body will i.e., if a force changes the momentum ofa body, the
greatest for the body will be greatest for have greatest rate of change of momentum is equal to the force
moving fastest. the heaviest body. velocity.
acting on it [Fig. 5.3 (a)]. From Eqn. (iii) it is evident
~ ~~~ ~ ~
that:
(5) As p =m v, p. p=(mv)·(mv),i.e.,p2 =m 2v 2
(a) The slope of pit curve will give the force, i.e.,
so K=2
~mv22=~m[LJ
m2
slope of (pit) curve = tane = (dpldt) = F
(b) The area under Fit curve will give the change in
... (iv)

momentum [Fig. 5.3 (b)], i.e.,


i.e.,K = (p2 12m) or p=.,/2mK ... (ii)
CONSERVATION OF MOMENTUM 167

P2 Consider two boats of masses mA and mB' initially at rest.


fPI dp=f I)' 2Fdt [.sF = dp/dt]
A person sitting in the boat A pulls the other through a rope. In
the absence of resistance of water there is no external force
Now as r2dp=P2 - PI = I:::.p
P, acting on two-boat system. Pull of the person is internal force.
Initial momentum of system = 0
and f I)t2Fdt=fI2dA
I)
=A

so I1p = area under Fit curve .. ,(v)


P F
For the system of boats A and B lenslon
of the siring Is Inlernal force. We
neglect forces exerted by waler
F Fig. 5.5

For system of boat A and boat B no external force is


11 dt 12 exerted.
F=lanG t.p .. Area
(a)
Fig. 5.3
(b)
f ->Fdt =0
Applying impluse-momentum equation,
(7) For a photon as E = hv and also E = me 2
so mass of photon m = (hv/c 2 ) and as its speed is c, its mV
I
+
f"'
2Fdt = mV ' 2
momentum so, initial momentum of system = final momentum of system
hv
h
p=mc =-xc=- [aSC=VA] This is called law of conselVation of momentum which
c'
1- states that in the absence of external forces momentum of
h hv E system is conserved.
i, e., p=- = - =- .. ,(vi)
A AV C -> ->
or VB =-m A VA 1mB
Impulse momentum equation
Newton's law may be expressed in the form which implies that ifboat A moves to right, boat B will move to
d left.
F =-(mv) or F dt=d(mv)
dt Note: Students are advised to apply impulse-momentum equation to
applications of conservation of momentum, given in next section.
or fl2Pdt=mv2 -mv 1
" Question I. (a) Two masses, one n times heavier than the
Above equation is referred to as impulse-momentum other are dropped from same height. How do their momentum
equation. compare just before they hit the ground? (b) Two masses, one

d
F~v:,J-F dt n times as heavy as the other have equal kinetic energy. How

o mV';
do their momentum compare?
Answer: (a) By definition P = mv

J',
mV1 + my, so, PH1PL = (mHv H )/(mLv L )
Initial momentum II F dt Final momentum
of system Impulse of force of system
But mL m; = niH nm = [given]
Fig. 5.4 and v H =v L =~2gh [asinfreefallv=~2gh]
It reveals that: "The impulse exerted on an object equals so, (PH1PL)=(nmlm)=n
its change in momentum," i.e., the momentum of heavier body is n times that of
lighter one.
Note: If the motion of bodies takes place in a piane, resolve the
momentum/forces along x aria y-axis. In the scalar fonn this (b) WeknowthatK=(p 2 /2m); p=.J2mK
equation can be Written as·
so, PH = ~mHKH
PL mLKL
But given thatKH ='KL and mH = nm and mL =m
168 PHYSICS FOR COMPETITIONS - VOl. I

so, PH =~nm =..In Solution: Change in momentum of mass tlm of liquid as it


passes through the bend
PL m
D.P = PF "': Pi = t'J.mv-fi
i. e., the momentum of heavier body is -.In times that of
lighter one. so th.t F = Ap = (.Ji)v Am
Problem 1. Calculate the percentage change in kinetic tlt M
energy of a body ifits momentum is increased by (a) 5% (b) or F = .Jiv (PAM)
50%. A,
2 [as Am = pAM]
M 26p
Solution. (a) As K =L, - =- -
2m K P or F =.JipAv'
M [.sMIllI=V] Fig. 5.7
so, -=2x5=lO%
K So the force to be applied at the bend to hold it in position
is -fipAv 2 in the direction as shown in Fig. 5.7. (Otherwise
(b) As
K = ;~,~ =(~r pipe will move in opposite direction.).

But p' = p + (50 /lOO)p = 1.5 P § S.2 [AI Law of Conservation of Linear Momentum
K' K' Through experimental observations it has been found that
so, _=(1.5)' or - - 1=2.25-1 if no external force acts on a system (called isolated) of
K K
constant mass (called closed) the total momentum of the
i. e., 11K = 1.25 =125% system remains constant (with time). This simple but quite
K general law is called law of conservation oflinear momentum.
Note: Ifwe try (b) as (a) (6J(jK)= 2(50%) == 100% which is not correct
Regarding this law it is worth noting that:
as 50 cannot be taken as differential element of 100. Differential (1) According to this law for a system of particles
calculus is applicable only when change is small of the order of ~ ~

less than 5%. PI + P2 + ... = constant


Problem 2. A disc of mass 0.1 kg is kept floating This equation shows that in absence of external force
horizontally in mid air by firing bullets of mass 0.05-kg each for a closed system the linear momentum of
vertically at it, at the rate of 10 per sec. If the bullets rebound individual particles may change but their sum remains
with the same speed, what is the speed a/the bullets with which unchanged with time, e.g., in case of a gas contained
these are fired? in ajar at rest the linear momentum of the system (jar)
always remains zero, though according to kinetic
Solution: If a bullet of mass m F
theory of gas, each gas molecule has momentum that
moving with speed v strikes nonnally
changes with time due to collisions with other
at some surface and rebounds, change
molecules.
in momentum will be
(2) As linear momentum depends on frame of reference,
mv - (- mv)=2mv v
observers in different frames would find different
Now if n bullets strike per sec, the Mg values oflinear momentum of a given system but each
change in momentum per sec m
would agree that his own value of linear momentum
Fig. 5.6
i.e., dp = 2mvn does not change with time provided the system is
dt isolated and closed, i. e., law ofconservation oflinear
But as rate of change of momentum is equal to the force momentum is independent of frame of reference
and this force will balance the weight of disc so that it appears though linear momentum depends on frame of
floating, i. e., reference.
2mvn=Mg (3) Conservation of linear momentum is equivalent to
Newton's III law of motion. For a system of two
lxlO- 1 x 9.8
or v = 0.98 mls particles in absence of external force by law of
2x5xlO 2 xl0 conservation of linear momentum
Problem 3. A liquid ofdensity pjIows along a horizontal ~ ~

pipe of uniform cross-section A with a velocity v through a PI + P2 = constant


right angled bend as shown in Fig. 5.7. Whatforce has to be ~ ~

exerted at the bend to held the pipe in equilibrium? i.e., m1 VI + m2 V2 = constant


CONSERVATION OF MOMENTUM 169

Differentiating above with respect to time p' -t -tKa m .


~ ~
(3) Now as K = - and I PBI=I PGI, - =- I.e.,
2m KB M
d vI dV2
m1--+m,--=O [as m is constant] kinetic energy of bullet and gun will not be equal and
dt dt
as M» m, the KE of gun will be much lesser than
~ ~ ~ ~

or m1 81 +m2 a2 =0 [as(d v Idt)= a] bullet.


~ ~ ~ ~
(4) Initial KE of the system is zero as both are at rest and
or F, + F2 =0 [asF=ma] final KE of the system [(l/2)(mv 2 + MV2 )]> O. So,
~ ~ here KE ofthe system is not coltstant but increases. If
or F2 =-Fl PE is assumed to be constant, ME = (KE + PEl will
i.e., for every action there is equal and opposite also increase. However, energy is always conserved.
reaction which is Newton's III law of motion. Here chemical energy of gun powder is converted into
(4) This law is universal, i.e., it applies to both KE.
macroscopic as well as microscopic systems. It holds (b) Motion of a ball under gravity
good even in atomic and nuclear physics where (i) If the ball is the system, the force of gravity will be
classical mechanics fails. Further it is more generally external and so the linear momentum of the ball will change.
applicable than the law of 'consen--ation of This is not violation of law of conservation of linear
mechanical energy' because 'internal forces' are momentum as linear momentum is conserved only if F ext = 0.
often non-conservative and so mechanical energy is
(ii) lfball and the earth is the system, all the acting forces
not conserved but momentum is (if F ext = 0).
will become internal and so,
Principal applications of conservation of linear
~ ~ ~
momentum are in the field of collisions (discussed in Ps = PB + PE = constant
§ 5.3 and § 5.4).
Now as initially both ball and the
IB] Some Applications earth are at rest
(a) Firing a bullet from a gun ~ ~

(i) If the bullet is the system, the force exerted by trigger PB = P"E =0
Earth
will be external and so the linear momentum of the bullet will From this it is clear that:
change from 0 to mv. This is not the violation of law of ~ ~

conservation of linear momentum as linear momentum is (1) PE =- PB ,i. e., ifball acquires
conserved only in absence of external force. some momentum, the earth also

is the system, the loree........


VI....
(ii) Jfthe bullet and' gun - _ A
M
-iiIII.
- - - --- m - v
Bullet
n -
acquires equal and opposite
momentum.
exerted by trigger will be --jo --jo --jo --jo --jo --jo
Goo
internal so, (2) As p=mv,mv+MV=O,i.e.,V =-(mIM)vi.e.,
Flg.5.S
~ ~ ~ the velocity of earth is opposite to that of ball in
Ps = Ps+ Po:= constant direction and much smaller in magnitude (as
Now as initially both bullet and gun are at rest M» m). So, if ball moves away from the earth, the
~ ~
earth also moves away from the ball and ifball moves
PB+PO = O towards the earth, the earth also moves towards the
ball but the speed of the earth is much lesser than that
From this it is evident that:
of the ball (as its mass is much greater than that of
~ ~

(1) PO=-PB' i.e., if bullet acqUires forward ball).


momentum, the gun will acquire equal and opposite p2 --jo --jo KE m.
(3) Now as K = - and I PE l =! PBI, - =-I.e., KE
(backward) momentum. 2m KB M
---) ---) ---) ---) :-+ --t
of ball and earth will not be equal and as M» m the,
(2) As p =m v,m v+ MV =O,i.e.,V =-(mIM) v i.e.,if
KE of earth will be much lesser than that of the ball.
the bullet moves forward, gun 'recoils' or 'kicks'
(4) Initially KE of the system is zero (as both are at rest)
backward and heavier the gun lesser will be the recoil
velocity. and after some time KE of the system is not zero (as
both are in motion). So, KE of the system is not
constant but changes. However, (KE + PEl, i.e.,
170 PHYSICS FOR COMPETITIONS - Vol. I

mechanical energy a/the system is conserved. When the momentum, e.g., in case ofunifonn circular motion of a
the ball and earth approach each other KE increases body KE is constant while momentum changes.
and PE decreases and when they recede from each Question III. A meteorite burns in the atmosphere before
other KE decreases and PE increases. it reaches earth's surface. What happens to its momentum?
(c) Motion of two masses connected by a spring Answer: The momentum of meteorite is transferred to air
Consider two blocks,

::S~i~~~:e~!~t~Oynl:S~Sa~:;~~:
spring as shown in Fig. 5.10. If
the spring is stretched (or
l
V2 -

. :::O::M:O:JJ m,•• __
. Y L sa
l. 2
molecules by air drag and so, the momentum of meteorite plus
air molecules (system) always remains constant.
Question IV. Can a sail boat be propelled by air~blown at
the sails from a/an attached to the boat?
Fig. 5.10 Answer: As sails and fan both are attached to the boat,
compressed) and then released
force due to the air~blown on the sails by the fan is an internal
from rest,
force. Now as by an internal force momentum of the system
Then as F cxt =0 cannot be changed, so the system (boat + fan + sails) cannot be
--> --> --> propelled by blowing air at the sails from a fan attached to the
Ps = Pl+ P2 = constant
boat.
However, initially both the blocks were at rest so, Question V. A bird is held in a light container which is
--> --> completely closed. Can we tell when the bird is resting or
PI+P2=O flying by weighing the container?
From this it is clear that: Answer: According to the law of conservation of linear
--> --> momentum we know that the linear momentum of a system of
(1) P2 = - PI' i.e., at any instant the two blocks will have
constant mass remains unchanged in the absence of any
momentum equal in magnitude but opposite in external force. Now treating the bird, container and the air in
direction (Though they have different values of the container as the system of constant mass we find that the
momentum at different positions). force which the bird exerts in flying is an internal force. But as
-+ -+ -+ -+ -+ -+ by an internal force the momentum of a system cannot be
(2) As P =m v , m! v! + m2 v2 =0, i. e~ v 2 =-(mJm2 ) vI changed, so the weight of the system will not change, i.e. , we
i.e., the two blocks always move in opposite cannot tell when the bird is flying or sitting by weighing the
dir~ctions
with lighter block moving faster. container.
p2 -+ -+ 'K m
(3) Now as K = - andl pd = I P21, _1 = _2, i.e., the Note: However, if in the above problem the container is of wire gauge,
2m K2 m! the momentum of the system will not be conserved (why?) and
due to th is the weight of the system will be lesser when the bird is
KE of two blOCKS will not be equal but in the inverse flying as compared to the weight of the same system when bird is
ratio of their masses and so lighter block will have resting, i. e., we will be able to tell when the bird is resting, or
greater KE. flying by weighing the container.
(4) Initially KE of the blocks is zero (as both are at rest)
Problem 4. Two balls A and B of mass 0.10 kg and 0.25
and after sometime KE of the blocks is not zero (as
kg respectively are connected by a stretched spring of
both are in motion). So, KE is not constant but
negligible mass and placed on a smooth table. When the balls
changes. Here during motion of blocks KE is
are released simultaneously the initial acceleration ofball B is
converted into elastic potential energy of the spring
10 cmli west-ward. What is the magnitude and direction of
and vice~versa such that
the initial acceleration of the ball A?
KE + PE = ME = constant
Solution: As the force 0.10 kg
Question II. ra) Can kinetic energy of a system be provided by the spring IS
changed without changing its momentum? (b) Can momentum -->
ofa system be changed without changing its kinetic energy? internal, P s = constant
Answer: (a) Yes; In explosion of a bomb or inelastic -->
collision between two bodies as force is internal, momentum is +mB VB
Fig. 5.11
conserved while kinetic energy changes. =: constant
(b) Yes; Ifa force acts perpendicular to motion, work done
will be zero and so kinetic energy will remain constant. =0
However, the force will change the direction of motion and so,
CONSERVATION OF MOMENTUM 171

0.25 2 (b) As force of explosion is


or = - - x (lOcmls west-ward) internal and system is initially at rest
0.10

or
...aA 2 directed east-ward
... -> ...
P I+ P 2+ P 3 =0
== 2S cmls
[as - West-ward "" East-ward]
... ... ...
or P 3 =-( PI + P2)
Problem 5. AU 238 nucleus initially at rest emits an a -
particle and is converted into Th 234 , If/he KE o/a.-particle be or P3 = ~P~ + pi
4.1MeV. calculate the kinetic energy and recoil velocity ofthe Fig. 5.12
[as 9=90~
residual Th 234 nucleus. (Given mp -= mil =1.67 x 10-27 kg and or P, = (.fi)mv [as PI =P2 =mv given]
19
l eV= 1.6 x lO- J). So the final KE of the system
Solution: As the force is internal 2 2 2
...P Th+ ...P PlP2
K =-+ P3
- +--=-mv
2m 2m 2(2m) 2
1 2 1 2 3
1 2 +-mv
2
+-mv =- mv 2
2 2
a =0 [asinitiallysystemwasatrest}"
Now as initial KEofthe syste m is zero as it was at rest, the
~ 2 ..... 2· 2 2 "'z -+ ... 1 energy released in explosion
or (PTh) = (- P o.) ,1.e"PTh=Pa{asA =A· A= A]
3 2 3 2
or KThmTh =Ka ma [a sK= p2 / 2m] E::.KF -K/ ::.-mv - O=-mv
2 2
4 Problem 7. (a) A rail road car of mass M is moving
or KTh = 234 x 4.1=O.068MeV
witholltfric/ion on a straight horizontal track with a velocity u.
Now as K =! mv 2 A man ofmass m lands/rom a helicopter on it. What will be the
2 velocity of the car now? (b) Now li the man begins to run on it
with speedvre/ with respect to car in a direction opposite to
so ! mTh vh ='0,068 x 10 6 x 1.6 x 10- 19 J the motion ofcar, what will be the velocity ofthe car?
2

or
2
V Th =
2xO.068xI.6)(10- 13 v",- ~
234 x 1.67 x 10 27
which on simplification gives v Th =2.3x lO S mls ge)iL"(@} <@¥(@e-Y o

M![J£§#AO i4 ~ D w . so awiD a;sx o · 3 0&2 ; '0"


Problem 6. (a) A shell is fired from a cannon with a
Fig. 5.13
a
velocity v m/s at an angle with the horizontal direction. At the
highest point of its path it explodes into two pieces of equal Solution: (a) Considering the car and man as system, as
masses. What is the speed of other piece immediateiy after there is no external force in the horizontal direction, linear
explosion, if one of Ihe piece retraces its paih to the cannon ? momentum along horizontal is conserved. Initially the
(b) A particle of mass 4 m which is at rest explodes into horizontal velocity of man is zero w hile that of car is /I SO, Pi =
three /ragm ents. Two of the fragments each of mass mare m x 0 + Mu. Finally when man has landed on the car both
found to move with a speed veach in mutually p erpendicular moves with same common velocity (say Vol so, that p f =
directions. Calculate the energy released in the process of (m + M)Vo ' Hence, from conservation of linear momentum
explosion., IEAMCET 19911 along horizontal
Solution: (a) In case of projectile motion as at the highest Mil = (m + M)Vo
point (v) vertical = Oand (v)horizontal =vcos9, lhe initial linear
momentum of the system will be mucos9. i.e., Vo = (~)"
m+M
Now as force of blasting is internal and force of gravity is
i. e., the velocity of car decreases as [MI(m + M)J< 1
vertical so, linear momentum of the system along horizontal, is
conserved, i.e. , (b) Here initial momentum of the (car + man) system will be
PI + P2 == mu cosO or m1v 1 + m2v2 = mv cosO ~ =~+M ~ .~
But it is given that rnl = m 2 = (m/2) and as one part If the velocity of ear whe n man stans running on the car is
retraces its path v l == -vcos~, Ve , the velocity of man relative to ground will be Vc - v re ]'
I I So, the final momentum of the system (car + man) relative to
- m(-vcosO) + - mv 2 =mvcosO ground will be
2 2
i, e., Pj =MVC + mcYC - vrcl)= (M +m)Ve - mv rel
172

Now as force of running is internal so linear momentum of Now substituting the given data in this equation we have
the system must be conserved
150xlO- 2 x 10 3 =1O-2 +! x lO- 3 x10 - 4 x 2
x
i.e., (m + M)VC -mvre! = (m + M)Vo 2
V =V +
mv re1 or x2 5
+ZxI0 x-3xl0 10
=0
or
C 0 (m + M)
[- 2xl0' ±J(2xlO')2 -4xlx(- 3xlO IO)]
i. e. , the velocity of car will increase and will be > Vo' or x ~

2
Note: (i) The velocity of man relative to ground will be Here negative sign is inadmissible as x cannot be negative
_ _ MU,d
VM -VC - vrel -VO So, x=(1 / Z)[-ZxI0 5 +4 x l0 5 ]=lxl0 5 m
m+M
(ii) If the man runs in the direction of motion of train then vrc1 § 5.3 Collision or Impact
will be replaced by -v tel in all the above discussion Collision is an isolated event in which a strongforce act.
resulting in
between two or more bodiesfora short time. The basic idea 0
mv 1 Mv ,
V. =v _ ro '«V)andV = V + re, (>V) collision is that the motion of colliding particles (at least one 0
C I) (m+M)' 0 M 0 (I1I+M) 0
them) changes abruptly. Regarding collision it is worth notinl
Problem 8. A cylindrical solid of mass 10- 2 kg and that:
cross~sectionai area 10-4 m 2 is moving parallel to its axis (1) In collision particles mayor may not come in rea
(x-axis) with a uniform speed of 10 3 mls in the positive touch, e.g., in collision between two billiard balls or;
direction. At t = 0 its front face passes the plane x "" O. The ball and bat there is physical contact while in collisiOl
region to the right of this plane is filled with stationary dust of alpha particle by a nucleus (i. e., Rutherfor(
p~rticles a/uniform density 10- 3 kg/m 3 . When a dust particle scattering experiment) there is no physical contact.
collides with the face of the cylinder, it sticks to its surface.
Assuming that the dimensions of the cylinder remains Alpha-Particle
practically unchanged and that the dust sticks only to thefront

~
face of the cylinder, find the x-co-ordinate of the front of the
cylinder at t "" 150 s.
Solution: Considering cylinder and dust particles as the Ib)
I')
system let AB be the front face of cylinder at t "" 0 (and x "" 0). Fig. 5.15
So the initial momentum of the system Pj = MVo + 0 (as dust
particles are at rest). Now in time t the front face has shifted a (2) In a collision we consider the situation before ani
distance x so that its new position is A'B' and so, the mass of after the collision. The tenn 'before collision' an'
the particles sticked to the cylinder will be Axp and if v is the 'after collision' refers to the conditions whel
velocity of cylinder at this instant interaction force between particles become
PI ~ (M + effectively zero. The duration of collision Ilt i
negligible as compared to the time for which w
observe the event. During collision strong intema
forces act between the colliding particles.
(3) In a collision if the motion of colliding particle
before and after the collision is along the same lim
the collision is said to be 'head on' or on
1=0 . dimensional. Collision of two gliders on an air trac:
Fig. 5.14
can be cited as an example of this type.
Now as in this process no external force is acting on the system, Velocity of Velocity of
the linear momentum of the system must be conselVed, i. e.,
MVo = (M + Apx)v 1F --
Approach
Ul- Ul!
Recession

i.e., MVo=(M+PAX(:) [asv=: ]

or MVo f
0150 dt = f"oeM +pAx)cb:
OJ • After collision
or 150MVo ~ Mx+(l/2)pAx2 Before collision •
Fig. 5.16
Note: However, if two particles collision is 'glancing', i.e., such that (10) If in a collision, kinetic energy after collision is not
their directions of mati on after collision are not along the initial equal to kinetic energy before collision, the collision,
line of motion, the collision is called 'oblique. If in oblique is said to be "inelastic." Here KE appears in other
collision the particles before and after collision are in same plane, forms. In some cases (KE)Final < (KE)lnitial such as
the collision is called 2·dimensionai otherwise 3-dimensional.
when initial KE is converted into internal energy of
In case of 2·D or 3·D collisions usually it becomes difficult to
solve the problem unless some experimental data are provided as
the product (as heat, elastic or excitation) [See
in these situations more unknown variables are involved than Problem 18] while in other cases (KE)Final > (KE)Jnitial
equations fonned. such as when internal energy stored in the colliding
particles is released.
(4) In a collision the effect of external forces such as
gravity or friction are not taken into account as due to (II) If in a collision, two bodies slick together or move
small duration of collision (Ill) average impulsive with same velocity after the collision, the collision is
force responsible for collision is much larger than said to be perfectly inelastic, e.g., the collision
external forces acting on the system. between a bullet and a block of wood into which it is
(5) In a collision, interacting particles mayor may not fired is completely inelastic when the bullet remains
change. If they do not change the collision is termed embedded in the block. The tenn 'completely
'scattering' and if they change 'reaction', However, inelastic' does not necessarily mean that all the initial
in a collision charge is always conserved, i. e., KE is lost, it implies that the loss in KE is as large as it
can be, consistent with momentum conservation.
ql + q2 = qi +qi
(6) In case of collision as the impulsive force acting (12) Actually collisions between all real objects are neither
during collision is 'internal', the total momentum of perfectly elastic. nor perfectly inelastic. These are
system always remains conserved, i. e., called imperfect or semi-elastic collisions. To
---) --)0 --)0 --)0 describe such situations we define a dimensionless
ml ul +m 2 u2 =m l vI +m2 v2 physical quantity called coefficient of restitution as
~ ~
(7) In a collision, 'total energy' is also always conserved.
e = I relative velocity of recessionl = ,-Iv--,-2_
- _v-",-I
Here total energy includes all forms of energy such as
mechanical energy, internal energy, excitation I relative velocity of approach I --)0 --)0
lUI - u21
energy, radiant energy or even mass-energy.
~ ~
(8) In a collision, the internal forces act for a short time For perfectly inelastic collision as v 2 = v l '
and therefore over a short distance, while we observe
the particles only at much greater relative separation,
so, the effects of their potential energy can be
~ ~

--)0 --).
°
v 2 - VI "" 0, e "" while for perfectly elastic collision
--)0 --)0

neglected and we can assume safely that in a collision as I v 2 - vI I==I U 1 - U 21*, e= 1. For all other collisions
potential energy before collision is equal to potential 0< e< 1.
energy after collision. So the change in mechanical
energy means change in kinetic energy or vice-versa. Note: Here it is worth to note that:
(9) Ifin a collision, kinetic energy after collision is equal (i) Classification of collision
as elastic, imperfect or
to kinetic energy before collision, the collision is said
inelastic is indcpendcnt of
to be 'elastic', i.e., in elastic collision frame of reference though
12121212 momentum and energy
-mlu l +-m 2u2 =- miv i +-n12V 2 depends on frame of Fig, 5.17
2 2 2 2
reference.
According to kinetic theory of gases the collisions (ii) The common nonna l to the surfaces in contact during impact
between gas molecules are elastic. Most of the is callcd the line of impact.
collision of atoms, nuclei or particles like electron, (iii) If the velocities of the two particles are directed along the
proton, alpha, etc., are elastic. line of impact, the impact is said to be a direct impact. If, on
the other hand, either or both particles move along a line
Note: During collision as large impulsive force acts, so, kinetic energy other than line of impact, the impact is said to be oblique
changcs and hencc it is not very proper to say that "KE is impact.
conserved in elastic co!lision but it will be more appropriate to
say that in elastic co!lision KE after collision remains same as it
was before."

• Called Newton's law for elastic collision and is discussed in § 5.4.


(iv) It is postulated that the F ->
total period of impact ps = constant A
61 is divided into period
Now resolving the
of deformation and
period of recovery. The momentum of particles at Band C
. impulse of defannation horizontally and vertically we find
and recovery are finite that Ps = 0 so, that ,
although the time D~formatio~ , Recove~ -> -> -> GI,
interval of impact is PA+ Po+ Pc =0 ,
period period B~-----''-------''\oC
extremely small. FIg. 5.19
Fig. 5.18 Now after collision,
-> ->->
(v) Coefficient of restitution (e): The coefficient of restitution PA = 0 and Po = - Po (given)
is defined as the ratio of the impulses of recovery and
defannation of either body. -> -> ~ ~ ~ ~

Impluse of recovery
So O+(-PB)+PC=O i.e., Pc = Po or Vc = Vo
e= -
Impluse of defonnatior. i.e., the particle C after collision will move with velocity
Velocity of separation along line of impact of B before collision, i. e., it will have magnitude v ana
Velocity of approach along line of impact direction along BG.
Value of e is 1 for elastic collision, 0 for perfectly inelastic § 5.4 Collision in One Dimension
collision, 0 < e< 1 for inelastic collision. (AJ Elastic collision
(vi) During collision following scalar equations may be set up: Consider two bodies moving initially along the line
I. Conservation of momentum for pair of bodies will joining their centres as shown in Fig. 5.20.
provide two scalar equations, for conservation of
Assuming initial direction of motion to be positive and
momentum along line of impact called nonnal direction
and along tangential direction. uj > u2 (so that collision may take place) and applying law of
2. The definition of coefficient of restitution e provides
conservation of linear momentum, we get
one more equation. It e>..presses th.e loss of energy of the
system (Always velocities along line of impact are
used.).
3. For elastic collision conservation of energy equation Before collisIon After collisIon
may be used. Fig. 5.20
Problem 9. If a body falls normally on a surface from m,u, + m2u2 = mjVj +m2v2
height h, what will be the height regained after collision if i.e., mj (uj - v j ) = m2(v2 - u2 ) ... (i)
coefficient of restitution is e? Also since the collision is elastic, kinetic energy before
Solution: Ifa body falls from height h, from equations of collision must be equal to KE after collision, i. e.,
motion we know that it will hit the ground with a velocity say 12121212
u = ~2gh which is also the velocity of approach here. 2"m,u1 +2"m2 u2 =2"m,v 1 +2"m2v 2
2
Now if after collision it regains a height hj then again by or mj(u j -Vj2 ) = m2 (v 22 - u2)
2
... (ii)
equations of motion v = ~2ghj , which is also the velocity of
Dividing Eqn. (ii) by (i)
separation. So, by definition of e, uj +Vj =v 2 + u2
e=~2ghj or or (u j -u2 )=(v 2 -vJ) ... (iii)
2gh Thus, in I-D elastic collision 'velocity of approach'
before collision is equal to the 'velocity of recession' after
Problem 10. Three particles A, Band C of equal
collision, no matter what the masses of the colliding particles
masses move with equal speed v along the medians of an
be. This law is called Newton's law for elastic collision.
equllateral triangle as shown in Fig. 5.19. They collide at
the centriod G of the triangle. After the collision A comes Now if we multiply Eqn. (iii) by m 2 and subtracting it
from Eqn. 0)
to rest, while B retraces its path with the speed v. What is
the velocity ofe? (mj - m2 )u j +2m 2u2 = (m\ + m2 )v j
Solution: As in collision momentum is conserved or
v, =(:: :::}, +( m~:~2}' ... (iv)
Similary mUltiplying Eqn. (iii) by nil and adding it to (c) I/projeclile is massive: i,e.,m, » nil so that
Eqn. (i) m -m, 2m, 2m,
I All; ::::0 and =2
2mJuJ +(m2 - m1)uz = (m J +m2 )v 2
~+~ ~+~ ~ +~

or v2 -
_( 2m, } I +
ml +m2 ml +m 1
(m, -m, } 2 ... (v) So Eqns. (iv) and (v) reduces to
v I :::: u, and v 2 ::::2u, - u2
Eqns. (jv) and (v) give the final velocities in tenns of the If the target is initially at rest (i.e., u2 ~ 0)
initial velocities and masses of the particles and are the VI AI u, and v 2 =2u I ... (ix)
required results.· From these it is clear that in case ofCl - D)
i.e., when a heavy projectile collides head on elastically with a
elastic collision:
light target at rest, the motion of heavy body is almost
(a) Ifmasses are equal: If the colliding particles are of unaffected while light body flyaway at speed twice that of
equal masses, i, e., ml =m2 =m heavier.
-=2"m",,--- = I This is why in a.-particle scattering experiment the motion
of a-particles due to collision with electrons of the target
So Eqns. (iv) and (v) reduces to atoms remain practically unaffected.
vI =112 and v 2 = III ... (vi) (d) KE transferred from projectile to target: As KE of
i. e., when two bodies ofequal mass collide head on elastically,
,
projectile before collision is (l12)m, ur
while after collision is
their velocities are mutually interchanged. Further if u2 = 0 (1I2)m,", .
VI =0 and v 2 = ul •.. (vii)
i.e.• ifa body collides head 011 elastically with another body of
same mass at rest, 'he moving body SlOpS and body at rest
So or M( = l-(~)'
K ",
starts moving with same velocity. where VI is given by Eqn. (iv). However, if the target is at rest,
(b) If target particle is massive: If nl2 »nl i i.e.,112 =0
m, - m2 AI -I ' 2m, :=::0 and 2m2 Al2
+ m2
fill ' n i l +nl 2 m, +m2
So that Eqns. (iv) and (v) reduces to
v l =-u l +2u2 and v 2 =u2 ... (x)
If the massive target is moving slowly or is at rest, i.e.,
u 2 ~O
VI :=:: - u, and v 2 :=:: 0 ... (viii) or
i.e., when a light projectile collides head on elastically with a
very heavy particle at rest, the light projectile recoils with The transfer ofKE will be maximum when
almost same speed while heavy target remains practically at
rest.
So the transfer of KE in head on elastic collision (when
This is why:
target is at rest) is maximum when the masses ofparticles are
(i) A light moving ball rebounds from a wall with almost equal, i. e., massi'atio is I and the transferofKE is I, i.e. , 100%.
same speed while wall remains practica lly at rest.
IBI Inelastic collision
(ii) In case of head on elastic collision of a~particles (or
In case of inelaslic collisiofJ, after collision two bodies
electrons) with a heavy nucleus, the nucle us remains
(proje<:tile and target) move with same velocity (or stick
practically at rest whi le a-particles (or electrons) rebound with
together).
almost same speed.
~ In Clse of head on collision, if coefficient ofrcstitution is e. then after collision
_ (lilt - 111 2 ) (J + e)m 2
Vt - II[ + 112
(m t + 1//2) (III] + 111 2 )
(I + elm] (1112 - III)
v 2 ", 11[+ 112
(m) + 111 2 ) (111 [ + 1112 )
~. Greater the difference in masses lesser will be the transfer of KE.
Question VI. Explain why heavy water .... which contains lots
of I H2 is more effective in slowing down neutrons in a reactor
than heavy elements such as Pb 206 ?
Answer: In a nuclear reactor newly produced fast
Before collision After collision neutrons are slowed down before they can participate
Fig. 5.21 effectively in the 'chain reaction'. This is done by allowing
If two particles of masses rn 1 and rn 2 , moving with them to collide head on elastically with nuclei of atoms in a
velocities u1 and u2 « u1) respectively along the same line 'moderator' .
collide 'head on' and after collision they have same conunon lfuand vI are the velocity of neutron of mass mbefore and

.
velocity V, then by conservation of Hoear momentum,
mju1 +m 2 u 2 =mjV +m2V
after collision then
1 2 1 2
K] =-mu and KF =- mvl
V*=m,u j +m Zu2 2 2
or ... (i) So fractional loss ofKE of neutron
(m, +m2)
Now as the KE of the system before collision is f=M{ =K] -KF =1-(~J2
I 2 1 2 K K] u
K J = -rn1u\ + - m2 uZ
2 2 But for head on elastic collision when target is at rest.
And after collision is m-M)u. [See Eqn. (vii) in § 5.4]
1 2 vI=m+M
(
KF =- (m! +rn2 )V
2
So loss in KE During collision So !~I_(mm-+MM)2 ~ 4mM
(m+M)2
1 u 2] --em}
K J -KF = [ -I m1u 12 +-m 1 +m2 )V 2 ... (ii)
2 2 2 2 2 For lead f = 4x lx206 ""0.02 = 2%
Substituting the value of V from Ego. (i) (I +206)2

-,-(m..J1,:-U,-1_+_n-,'2~u,,-2,---
4x2
1 [(mju 2 )2] While for 1 H2 f= =0.89=89%
. v =-
llI\.
1 +m2 u22) - - (I + 2)2
2 (m! +m2 )
i. e., in collision with lead fast neutrons loose only 2% of its KE
2
_1[m 1m2 (U j +ui -2U 1UZ )] while with 1H2 it loses about 90% ofits initial KE. This is why
or M--
2 (mj + m2) light elements are better moderator than heavy ones.
Problem 11. Consider a one dimensional elastic collision
or ilK=.!. m1mZ CUI - u )2
z ... (iii) between a given incoming body A and body B, initially at rest.
2 (m] +m 2 )
How would you choose the mass of B in comparison to the
However, if the target is initially at rest mass ofA in order that B should recoil with (a) greatest speed
i.e., u2 =O and u1=u (b) greatest momentum and (c) greatest kinetic energy?

I::.K = m1mZ u2
Solution: As in a collision momentum is always conserved
2(m 1 +m 1 ) rnA u = mAv A + mBv B
or u=V A +kvB (withk=mBlm A ) ... (i)
or ... (iv) Now as collision is elastic
I 2 I 2
-mAu =-mAv A
Now if target is massive, i.e.,m 2 »m l 2 2
M ~ 1 (~100%) or u2 =v~ +kv~ ... (ii)
K
Substituting the value ofv A from Eqn. (i) and (ii)
i.e., if a light moving body strikes a heavy target at rest and
sticks to it, practically all its KE is lost . u 2 =(u-kv B )2 +kv~

• Ifm!u! + mzuz - 0, V _ O,i. e. ,ifin inelastic collision colliding particles have equal and opposite momenta, loss in KE is 100% and particles come to rest.
** Though ordinary water which contains lots of 1HI is better moderator [J= 100%] than I H2, it is not used in a reactor formoderation as it absorbs neutrons.
CONSERVATION OF MOMENTUM 177

which on solving gives Substituting this value Ofv2 in Eqn. (i)


2u u-v 1 =2(u+v 1), i.e., VI = -(u I 3)
vB =-- ... (iii)
I+k
(a) So far vB to be max, k must be min, i, e., so KF =~nw~ =~m( -~r =~mu2 X~=~K/
m
=~mu2 J
k =1--+0
mA [asK I
(b) Now Pa =mSvB
or K/ -/(F =-
8
But as
Ins
k=-andv s - --
rnA
2u
l+k
(from Eqn. iii)
or KF
K , =!9 ' i.e.' 1-K, =l-!9
KF
K, 9
Problem 13. A bullet 0/ mass m moving with a horizontal
2kmA u
2m Au veloCity v strikes a stationary block 0/ mass M sllspended by a
Pe = l+k = I + (Ilk) string o/Iength L. The bullet gets embedded in the block. What
So PB will be max when (I lk) is min or k is max is the maximum angle made by tTle string after impact?
m Solution: If V is the velocity of the block~bul1et system
so k = -.!!.... --+ 00 or ma » rnA just after collision, then by conservation of linear momentum
mA
mv=(M + m)V
I ,
(e) KB = - mB v 8 m V =[Mm+m ]v .. ~
2
m
But as k =1 and vB =-2u- (fromEqn . iii) So the KE of the block~bullet system just after collision is
rnA l+k
2
mv
! (m + M)V 2 (which is less than ! 2 as collision is
_ Ikm (2U)2 _ 2mAu k
2 2
K. - - A - -~'-:c inelastic). Now due to this remaining KE if the system rises
2 I +k (l+k) ' upto a height h, conservation of ME for this part of problem
yields,
or K. = 4(KA )k [asKA = (l /2)m u' ]
A
(l - k)' +4k ~(m+M)V' =m(m+M)gh, i.e., V =~2gh ... (ii)
2
SoKE will be max when (1- k)2 = min = 0
Substituting this value of V from Eqn. (ii) in (i). we get
m
i. e., k=--L = I
mA
or ' v =(M; m~ ... (iii)

Problem 12. A moving particle a/mass m makes a head


on elastic collision with a particle of mass 2 m which is or v' ( m )' ... (iv)
h=Zg M+m
initially at rest. Show that the colliding particle losses (8/9)th
ofits energy after collision.
Solution: Let u be the initial velocity of body of mass m
and v"v2 be the velocities of bodies of mass m and 2m
respectively after the collision. As in collision momentum is
always conserved
mu =mv 1 +2nw 2
i.e., VI +2v2 =14 or (u-v,)=2v 2 ... (i)
And as the collision is elastic
I 2 I 2 I 2
- mu = - nw, + - (2m)v2
2 2 2
Fig. 5.22
i.e., 2
v,+2v "
2 =u or (14 ' -v 2l )=2v 2
, ... (ii)
Now from Fig. 5.22, .
Dividing Eqn. (ii) by (i), we get
u+v, =v 2 L-h( 1- £h)
cos9*= - L- . ... (v)

• Here we hove assumed that 9 < 9QO so that II < L


Substituting the value of h from Eqn. (iv) in (v) And by conservation of momentum

cose=1-_·1_(,~ ) 2, or V2
mu=mv l + mv2
+V I =U ... (ii)

8--cos ~~[Ll -m;M(..inv )'J Solving Eqns. (i) and (ii) for v I and v 2' we get
or - --- VI = u(l - e)/2 and v 2 =u(l+ e)12
2gL m + M
So ~= [l - el
Note: This arrangement is c~lied .'ballistic penduiu~' and is used to
v2 1+ e
measure the speed offas! moving projectiles (such as bullet). By Problem 16. A simple pendulum is suspended from a peg
using Eqn. (iii) the speed vofthe projectile is 'calculated from the
on a vertical wall. The pendulum is pulled away Fom the wall
of
experimental value of h (as the masses projectile m and block
to a horizontal position and released. The ball hits the wall, the
M are known). • . '.
'" ., 1" '-';:'-1 coefficient of restitution, being (21J5). What is the minimum
Problem 14. A '20· g . " .'., M,
number of collisions after which the amplitude of oscillation
bullet pierces through.. , a .' ":

I
becomes less than 60° ?
plateo/massM J =lkga. nd,.. I- L .1
Solution: When simple p /~
then comes to rest'inside a ' m .~
pendulum released from position A -- - ----~-~--~~-~--~'"Il)
second plate oj mass " :::c;w..-:
strikes the wall with velocity V then 9 '
M 2 = 2.98 kg as shown in
Fig. 5.23. It isfound that the . " by conservation of mechanical ,I
energy L //
two plates, initially at rest,
now move with equal Fig. 5.23
velocities. Find the percentage loss in the initial velocity oj the
2
mgL + 0 = mv + 0 ~ 1--------~~~::,
bullet when it is between M J and M 2' Neglect any loss of i.e., v = ~2gL "'/
material of the plates due to the action ofbullet. B
'r . . Now as coefficient of Fig. 5.24
~olution: If VI is the.'Yelocity of the b':l.llet after passing restitution is e, so speed of
through M j (when it is between 1111 and M 2 ) and v 2 is the pendulum after first collision will be
velocity of the plates M ! ·and M 2 due to interaction of the VI = ev = e~2gL
bullet with them.
By conservation ofmomeptiIm for the '&ystem consisting Now after completing oscillation in accordance with
of bullet and plate ofmasstM~ ' " conservation of mechanical energy it will strike the wall with
same velocity and so its velocity after second collision will be
mv + M] xO=mv l +M}v 2
i.e., O.02v=O.02iii' + l x v'2 ... (i)
v, = eiJ l = e(e~2gL) = e' ~2gL
and for the system consisting ofbuliet and plate of mass M 2 So the velocity of the pendulum after n collisions will be
mv l ;= (M,it ni)v 2 .vn =ellv=ell~2gL
i.e., O.02v 1 =3v 2 ';"':':"'~' ... (ii) Now ifitrises to a height n, by conservation of mechanical
Substituting the value ofv2 from Eqil'. (Ii) In (i) energy
",: '(O:?~~ I )<j.; 'i.e·; . :.vl
O.02v = O.02v I = (3 /4 )v
~ m(v ni =mgh, i.e., ~e2112gL = gh
So dV% = v -V ] x IOO = v"- (3 l 4)V xIOO=25% 2 2
~ ~ -
v ~ , v 211 h L(l-cose)

r4)"
or e = - ~
Problem l~l A SPherl}. of mqss. m moving with velocity u L L
hits another statlon,ary'. 'sihel:e, of same ., mass. If e -is the
coefficient of restitution, whclt is th'e ratio' of velocities of two
spheres after the colliSiC! f/?."'lt:L..:!,:. ;;"::;e · ',,:,···'-~
or (1 =1-cos8 [ase= 1]
Solution: By definiiion coeffi~ient of restitution
.of,.,
.

e= ur - u2
V 2 -v I v 2 -vI
or (5" =I·-cose

u- O
Now fore to be lesser than 60°,cose> (1/2), i.e., 1 - cose< !
i.e., v 2 - vI = eu ... (i) 2
' 50, (H <~ (U or >2 i.e.,
2mv~
k = --
3x~
or n(log10 - 310g2) > log2 Problem 18. A man ofmass m is standing in a stationary
0.301 lift ofmass M which is counterpoised bya mass (M + m). If the
or n>-- [as lag 10= 1 andlog2=O.3010]
0.097 man jumps suddenly generating a velocity ~2gh, find the
or n>3.l velocity of the man relative to lift immediately after jumping.
As n(na. of collisions) must be integer so forS < 60°, n = 4. Show that the height to which the man rises relative to floor is
Problem 17. Two bodies A and B of masses m and 2 m 2h(M +m)
respectively are placed on a smooth floor. They are connected 2M+m
by a spring. A third body C of mass m moves with velocity Vo Solution: Given velocity of man is his
along the line joining A and B and collides elastically with A. absolute velocity. When man jumps upwards lift
At a certain instant a/time to after collision it isfound that the is jerked downwards. Consider (man + lift +
instantaneous velocities of A and B are same. Further at this counterpoise) as system. For this system
instant the compression a/the spring is xO' Determine (a) The momentum is conserved. M
m
common velocity ofA andB at time to (b) The spring constant. m~2gh = (m + 2M)V
Solution: Initially there will be collision betweenC and A
which is elastic so, by conservation of momentum we have m.J2ih
v
mvo =mvA +mvc (m+2M)
Fig. 5.26
i.e., VO = VA +ve ... (i) Vman,lift = Vman - Vlift = Vman + Vlift
m = ~2gh + m.J2ih
(m+2M)

or V = 2(M +m) 'Fh


Fig. 5.25 rei 2M + m vL.gn
And as in elastic collision KE after collision is same as After man jumps in air, the acceleration of lift and
before collision counterpoise
12 1 2 1 2.222
- mvo =-mvA +-mve I.e.,v o =V A +Ve ... (ii) a = ~[(,;-M;-:+_m
_)'c-_M~lg mg
2 2 2 (M + m)+M (J.M + m)
Subtracting Egn. (ii) from the sguare ofEgn. (i) Downward distance moved by lift
2v AVe =0
h2 = ~=m2.2gh(2M+m)= mh
so either VA =0 or ve = 0
2a (m+2M)2 x2mg (m+2M)
V A = 0 corresponds to no interaction between A and C so
only physically possible solution is ve = 0 which in the light of . (~2gh)2
Upward distance moved by man = =h
Egn. (i) gives v A = v O' i.e., after collision C stops and A starts 2g
moving with velocity v.
Now A will move and compress the spring which in turn h =h +h = 2(M +m) h
rei I 2 2M+m
accelerate B and retard A and finally both A and B will move
with same velocity (say V). In this situation the compression in Problem 19. Suppose a moving hydrogen atom makes a
the spring will be maximum (=x o )' head on inelastic collision with a stationary hydrogen atom.
Before collision both atoms are in the ground state and after
(a) As external force is zero so, momentum of the system
collision they move together. What is the minimum velocity of
(A + B + spring) is conserved, i. e.,
the moving hydrogen atom if one ofthe atoms is to begiven the
mvo =(m+2m)V minimum excitation energy after the collision? (m H = 1.0078u,
i.e., V = (vo/3) lu = 1.66 x 10-27 kgand ionisation energyofhydrogen is 13.6 eV)
(b) And by conservation of mechanical energy Solution: As collision is inelastic, momentwn is conserved,
I 2 1 '2 1 2 i.e., mu+O=(m+m)V or V = (u/2) ... (i)
- mvo =-(m+2m)V +-kxo
22 · 2 i. e., the velocity of hydrogen molecule after collision will be
or mv~ -3m(vo/3)2 =kx~ half that of hydrogen atom.
12 12112 For a discrete system of particles centre of mass is defined as
NowasKr= - mu andKF = -(2m)V =- (2m)(- u)
2 2 2 2
1 2 ---) m ] f 1 +m 2 f2 + m3 f 3 ··· 1 ---)
=- mu RCM = =-l:.mj fi " ,(i)
4 . m1 +m2 +m3 ". M
So in this collision th(!'e is loss ofKE. So that the co-ordinates of centre of mass will be
- 12 1 212 1 1 1
l!.KE=KJ -K F =-mu - - mu =-mu ... (ii) X CM = M l:.m,x j ; YeM = M 'Zm,Yj and ZCM = M l:.m,z,
244
Now according to Bohr's theory, energy of an electron in a However, if the system has continuous distribution of
hydrogen atom in nth state is given by ~

mass, treating the mass element dm at position f as a point


E =_ 13.6 eV
mass and replacing summation by integration
n2 "
~ 1 ~
As atom will be excited from ground state which for RCM =-f
M ·
f dm ... (ii)
hydrogen atom is n = 1 so,
EG + E ext = E Il , i.e., Eex! =En - £1 So that

i.e., E Cl'.t=I3.{1-n1z ]ev ' (withn>l) X CM = ~J xdm; YCM = ~J ydm and ZCM = ~J zdm;
Regarding centre of mass, it is worth noting that:
And it will be minimum when
(1) There mayor may not be any mass present physically
n=2 at centre of mass. (See Fig. 5.28)
i.e., ... (iii) (2) It may be within or outside the body. (See Fig. 5.28)

According to given problem


dKE = (Eex! )min for one H·atom
'"

Substituting the values of 6.KE and (Eex! )min from Egos.


(ii) and (iii) in the above
lal
L() Ibl (el
1 2
Fig. 5.28
-JnU =IO.2eV
4
19 (3) Its position depends on the shape of the body. (See
or u2 = 4xlO.2x1.6xlO- =39.02 x 108 Fig. 5.28) .
1.0078 x 1.66 x 10 27 (4) For a given shape it depends on the distribution of
or u = 6.25 x 10 4 mls mass within the body and is closer to massive part.
(See Fig. 5.29)
§ 5.5 Centre of Mass m,


m m
Through observations it has been revealed that every
physical system or body has associated with a certain point
whose motion represents the motion of the system or body.
0 eM
This point is called centre of mass, i.e., centre of mass of a lal (bl
Fig. 5.29
body or system is a point where the whole mass of body or
system is supposed to be concentrated for dealing its (5) For symmetrical bodies having homogeneous
translatmy motion. distribution of mass it coincides with centre of
y m, symmetry or geometrical centre. (Fig. 5.28)

m, m,
• • (6) Ifwe know the centre of mass of parts of the system
m, and their masses, we can get the combined centre of

mass by treating the parts as point particles placed at
their respective centre of masses. (See solved
Problems 23 and 24)

, , (7) It is independent of the co-ordinate system, e.g., the


centre of mass ofa ring is at its centre whatever be the
lal (bl
Fig. 5.27 co-ord inate system.
CONSERVATION OF MOMENTUM 181

(8) If the origin ofco~ordinate system is at c~ntre of mass, Note: From this example it is clcarthat in case oftwo particles system.
~

i.e., R CM :: 0, then by definition (i) Ifrn1 '" rn 2;XCM .. (Ll2),i.<!: ,centreofmass is at the middle
of the line joining Ihe two particles.
~

(l/M):Emj Ti ",,0 (ii) A s X CM ://12L/(ml + mz ),i.e.,XCM (m l + 1Il2 )'" m2L


or XCMIIl1 :mz(L-XCM )
i. e., the slim of the moments ofthe masses of a system
or RlI'i '" nl 2rZ wi th 'i = X CM and 'i + r2 '" L
about its centre ofmass ;s always zero.
i.e., moment of masses about centre of mass is equal and
Question VII. Are eenlre a/mass alld centre of gravity, centre of mass is closer to mnssive particle, i.e., ccntre of
two words/or one alld the same pOint? mass divides internally the line joining the two particles in
Answer: No; Centre of mass of a body is defined as the the inverse ratio of their mass.es.
point at which the whole of its mass is supposed to be 3 kg
Problem 21. Locate the Y
;}"
concentrated while centre of gravity ofa body is defined as the ,~,C
point at which whole of its weight is supposed to be centre of mass of a system of , '
particles of masses 1 kg, 2 kg , \'
concentrated, i. e. , ~,' \'
~
and 3 kg situated at tire ,, '
...... Lm . r·
ReM = __' _ L; and
Lm l
comers of an equilateral
" "
triangle of side b as shaWl! in A '"-. -.--- ~- -- -- ••\~""----
B
. ,
1 kg 2 kg
Fig. 5.3 1.
So jf the body is in a unifonn gravitational field, i. e., g is Fig. 5.31
constant then centre ofgravity coincides with centre of mass, i. e.,
~ ~
Solution: The co-ordinates of points A, Band Care (0,0,0)
~
R CG =
gLm , T, =--
:Em . r(
' - :::::R
-t (b,O,O) and (b/2, MI2,O) respectively. Now as the triangle is in
CM
gEm, Tom , x· yplane, i.e., zco~ordinat e ofal l masses is zero. So, ZCM =0.
However, as the force due to gravity decreases with Now by definition of centre of mass
altitude and lower portion ofa body is somewhat closer 10 the X _ l x O+ 2 x b+3(b/2) 7b
surface of the earth than the upper, the centre of gravity is CM - 1+2+3 12
slightly below the centre of mass . Therefore, while the
Y _l xO +2 x O+3Jl(bl2) _3Jlb
difference under ordinary conditions is so small as to be easily
CM- 1+2 +3 -~
neglected it is better not to interchange the two phrases. For
example, the centre o f gravity and centre of mass of a house are so the co-ordinates of centre of mass are
coincident but not so for a mountain.
7b 3Jlb 0]
Problem 20. Calculate the position ofcentre ofmass ofa [ 12' 12 '
system consisting of two particles of masses m l and /11 2
separated by a distance L apart, frOnl 11/1' Problem 22. If tile lillea,. density of a /"Od of lengrh L
Solution: Treating the lin e joining the two particles as varies as')... =A + Bx, compute its centre of mass.
x-axis, and m ] at the origin, the co-ordinates of the two Solution: Let Ihex-axis be along the length of the rod and
particles will be (0, 0, 0) and (L, 0, 0). So, origin at one of its end as shown in Fig. 5.32 . As rod is along
x·axis, for all points on it y and z will be zero so,
m@;A----------~------nB.~ x-axis r eM =0 and ZCM = 0
(0, 0, 0) (L, 0, 0) y
Fig. 5.30
m] xO+ ln 2 L In j xO+ ln 2 xO
X CM = ; rCM = ; '->I
m 1 +ln 2 nil + m 2 ,
_'"21_x_O_+_'_"2,,-x_O 0
and ZCM =
d'--+I i -
nil +m 2 , I' L ·1

/11 2 L Fig. 5.32


i.e., X CM . - -'--
nil +/112 i. e., centre of mass wil l be on the rod. Now consider an element
of rod of length dx at a distance x from the origin, then
i.e., centre of mass is at a distance [11I 2 L I (/Ill + 1112 )] from
m 1 internally on the line joining the two particles. d/ll =')...dx = (A + Bx)dx
J:xdm J:X(A + Bx)~ Solution: Let the
circular disc of radius a is
so, X CM = - - = - c - - - -
made up of the circular
f:dm J:(A+BX)dx section of radius band
AL2 BL3
remainder. Further let the
-- +-- line of symmetry joining
or, X _ 2 J = L(JA +2BL) the centres 0 and 0\ be
CM - BL2 J(2A + BL) the x-axis with 0 as Fig. 5.34
AL+ - -
2 origin. The centre of mass
of the disc of radius a will be given by
Note: (i) If the rod is of unifonn dens ity.
m]x\ +m2 x 2
i.e, A= colIstant := A, andB",O X CM = ... (i)
m +m l 2
X eM '" Ll2 (which is expected)
(ii) If the density of rod varies linearly with x,i.e., A '" Bx and while YCM and ZCM will be zero (as for all points on x-axis, y
A ::: 0 and z =0).
Ifcr is the density of the material of disc:
. 2
Problem 23. A caJpenter has m]""nba and xl""c
constructed a loy as shown in 2
m2 "" n(a 2 -b )cr and x2 = ?
Fig. 5.33. If the density of the
material afthe sphere is 12 times M = ml +m 2 = na 2 cr and
thalo/cone, compute the position
So substituting these values in Eqn. (i)
of the centre ofmass of the toy.
1tb 2crc+1t(a 2 - b2 )crx
Solution: If the density of. o= --~o----~~2
cone be p, then its mass will be na 2cr
1 2 16 3 _cb 2
m1 = -rt(2R) (4R)p=-.R P
J J ... (ii)
x 2 = -( a' 2:"_"-b'2-)
and its centre of mass 0] will be Fig. 5.33
at a height (hI4) = (4RI4) = R from 0 on the line ofsymmetry, i.e., centre of mass of the remainder (say O2 ) is at a distance
i.e"Yl =R cb 2/ (a 2 _ b 2 ) to the left of 0 on the line joining the centres 0

Similarly the mass of the sphere m2 = ~ nR 3 (l2p)


andO[.
J § 5.6 Motion of Centre of Mass
= 16n:R 3p =3m 1 and its centre of mass will be at its centre 02 As for a system ofparticies, position of centre of mass is
i.e., Y2 =5R given by:
Now treating sphere and cone as point masses with their
masses concentrated at their centre of masses respectively and
taking the line of symmetry as y- axis with origin at 0, for the
centre of mass of the toy
_
mCcl,-Yl,-
+_m-,2"Y~2
YCM '" so,
nI ] +m 2
"" m] x R + 3m] x 5R ""
m] +3m]
4R
i.e., centrc of mass ofthe toy is at a distance 4R from 0 on the
i. e., =
....
ml v\ + m2 v 2 +···
....
[ ....
d r -t ]
..
as dt = v ... (11)

line of symmetry, i. e., at the apex of the cone.


Eqn. (ii) is the desired result and gives velocity of centre of
Problem 24. Find the centre ofmass ofa umform disc of mass ofa system ofparticies. The velocity ofparticies in the
radius a from which a circular section of radius b has been centre of mass frame will be given by
removed. The centre of the hole is at a distance c from the .... .... ....
centre of the disc. (v,,)'=vi-v CM ... (iii)
Nowas /1/\ + /1/ 2 + ... == mass of the system
So Eqn. (ii) can be written as Question VIII. Two, particle.s. of masses m, and m 2
. '-':'
," ',', " " .'
~ ~ ~
separated by a distance d 'are ,'at.'re#-initially. If they move
M VCM =m 1 vI +m 2 V2 +...
towards each other under, m~tiiarlriteraction
",
(say electric,
...,. ...,. --t ...,. ~ ~
gravitational or elastic), where will/hey meet?
MV CM = Pl+ P2+ P3 + '" [asp := mv]
Answer: Initially the particl~s' a~e'~t rest sQ~he velocity of
~ ~
centre of mass . . , " . -',- ,
or [as~ Pi = P] ... (iv)
i,e., linear momentum a/a system a/particles is equal to the
product 'o f mass of the system with velocity of its centre of m, +m2
~ ~
~ ~
mass. Furthermore if VCM = 0, P = 0 i.e., in the frame of As here Fext := 0, so v CM will remain unchanged.
reference ofcentre a/mass the momentum ofa system is zero. ~ ~
This is why centre ofmassframe is called zero momentum/rame.
i.e.,
m, vI+m2 v2 =0 at all instants
Further if we differentiate Eqn. (iv) with respect to time,
we get
~ ~

dVCM dP ...,. or [asm l +m2 = M = finite]


M =- = Fcxt

=Tt~]
l
dt dt
~ Ar
So above equation reduces to or as v
~ ~

M aCM = Fext ... (v)


~ ~
This is the so, called equation of motion of centre of mass or ml .6.rl = m 2 .6.rZ ",, 0
and according to it centre of mass of a system of particles
~ ~ ~ ~
moves as though all the mass ofthe system is concentrated at it or ml d l +m2 d z = 0 (with.6. r ::.: d)
and all the exlemal forces were applied to it.
From Eqn. (v) it is clear that or midi - m 2 d 2 = 0
~ ~
~ ~ ~

(1) IfFeI:/ = O, aCM =0, i.e., vCM = constant


{as dire~fion ·'cif d z is opposite to d l ]
.", :r"
i.e., ifno external force acts on a system (or resultant external or mldl' = '~2d2
force acting on a system is zero) the velocity ofiis centre of But given that
mass remains constant, i. e., velocity of centre of mass is
d, +d2 =d , "',"
unaffected by internal forces. So, if the centre of mass of a
m2 d
system is at rest it will remain at rest unless acted on by an so that
external force.
~ ~ ~

(2) If Fcxt =0, aCM =0, i. e., vCM = constant d _ mid


and
~ ->
2:- m1 +' m 2'
so that P = M VCM = constant
Now as d l and d 2 represents the position of centre of mass
i.e., equation of motion of centre of mass leads to conservation relative to m[ and m2 respectively, so the particles will meet at
of linear momentum and as also in § 5.2 (A) (3) we have seen centre of mass of the syste~ .
that conservation of linear momentum leads to Newton's III
Question IX. State whether the statement given below is
law of motion so, 'concept of motion of centre of mass',
true or false giving reas~n iii brief '''Tlyo particles ofmasses 1
'conservation of linear momentum' and' Newton's III law of
kg and 3 kg move towards each ollter under their mutualforce
motion' are different approaches of dealing the same physical
of attraction. No oth'er for~e a~ts on them. When the relative
situation.

IF'
M eM m

I-- r2 ___1+- r,--l


-- Reaction Action
velocity of approac'h of the two particles is 2 mis, their centre
ofmass has a yelocity of0.5 mls. When the relative velocity of
approach becomes'3 mis, the velocity oftheir centre ofmass is
0,75 mls".
-> -> -> ->
m~+ Mr;=o Mv+mv .. o A+ R=O
~ ~

Answer: As F ext =Oso, VCM = constant, i.e., velocity of


la) (b) Ie)
FIg. 5.35 centre of mass does not change by mutual interactions. So, it
will remain unchanged, i.e., 0. 5 mls whatever be the relative ~ -:+ ~ ~
or md l + M d 2 =0 [as6.r= d =displacement]
velocity of approach. So, the given statement is false.
Problem 25. Two blocks of masseS .5 kg and 2 kg are ~ ~
or ... (i) [as d 2 is opposite to dd
placed on ajric/ionless surface and conllected by a spring. An
ex/ema/kick gives a velocity 14 mls to the heavier block in the Now as cannon
direction o/lighter one. Deduce (a) the velocity gained by the balls cannot leave the
celltre ofmass and (b) the separate velocities o/the two blocks car, so the maximum
in the centre of mass frame just after the kick. displacement of balls
~ ~
relative to car is L. And
--t m v +m v as in doing so the car
Solution: (a) By definition vCM = I I 2 1 will shift a distance d 2
m] +m2
relative to the ground opposite to the displacement of balls, the
Here vI =14m1s,v 2 =O,m j = 5kgandm 2 =2 kg displacement of balls relative to ground will be
v 5x14+'2xO
- :::.c,-,::..:.,::.:..c:: d l =L-d2 [i.e.,dl +d2 =dre1 =L] .... (ii)
so,
CM- 5+2 Substituting the value of d l from Eqn. (ii) in (i), we get
= 10 mI, m(L-d, ) = Md,
(b) The centre of mass reference frame is one in which
centre of mass is at rest. So the velocity of the heavier block in i. e., d2 =(m:LM ) =(I + ~/m)
this frame just after the kick is
Now as M and m are both > 0 and finite
v'l =v t - vCM =14-10=4m1s so d2 < L
and that of lighter block is
i. e., no matter how the cannon balls are fired, the rail road car
vi = V 2 -VCM =O-IO= - JOmls cannot travel more than L.
i. e., in the centre of mass frame the blocks of5 kg and 2 kg will Problem 27. A dog weighing 5 kg is standing on aflat
approach each other with speeds 4 mls and 10 mls boat so that he is 10 metrefrom the shore. He walks 4 metre on
respectively. the boat toward shore and then halts. The boat weighs 20 kg
Problem 26. A callnon and a supply of cannon balls are and one call assume that there ;s no frictioll between it alld the
iI/side a sealed rail road car. The cannon fires to the right. the water. How for is the dog from the shore at the end of this
car recoils to the left. The cannon balls remain in the car after time?
hilling the for wall. Show that no matter how the cannon balls Solution: Given that initially the system is at rest so
are fired, the rail road car cannot travel more than L, ~
v CM =0
assuming it star/sfrom rest.
Solution : Given that initially the system is at rest so, Now as in motion of dog no ex.ternal force is applied to the
~ system
YCM =0 ~
so, Y CM = constant = 0
Now as in firing the balls no external fQrce is applied to the
system ~ ~
m v l + M" 2
~ i.e. , =0
so, vCM = constant = 0 m+M
~ ~
or mY 1+M" 2 =0 [as (m + M) = FiniteJ
i.e., =0

[as ~ = ~;]
~ ~
6. r , 6.r
or (asm+ M =finiteJ
or m- - +M--2 =O
dt dt
~ ~
6.r 6. r') .
or m--l + M -'--'- =0
~
~ ~

6.1 6.t or m6. r l +M6. r 2 =0 [aS6.1"" d = dispJacement}


~ ~
or mdl - Md2 =0 (as d 2 is opposite to dd
or [as 6.t * ooJ i.e., mdl = Md2 .... (i)
Substituting the value of~ from Eqn. (ii) in (i), we get
-> -> ->

Fig. 5.37
Now when dog moves 4 m towards shore relative to boat, 0'
MV=-m(v ",+ V) .

v= .
mV rel .... (iii)
(m+M)
the boat will shift a distance d 2 relative to shore opposite to the
displacement of dog so, the displacement of dog relative to This is the desired result and from this it is clear that the
shore (towards shore) will be direction of motion of balloon is opposite to that of climbing
dj =4-d2 (i.e., d 1 +d2 =drd =4) ... .(ii) (V",el)' i.e., vertically down.
Substituting the value of d 2 from Eqn. (ii) in (i)
md, = M(4-d,) Note: (i) The velocity of man relative to ground

d - Mx4 = 20x4 = 3.2m -. -. -. M~


0' v= v + V =--"-' .... (iv)
, - (m + M) 5+20 reI m+M
As initially the dog was 10m from the shore, so now he (ii) In case of motion of two body systems if velocity . of III
will be 10 - 3.2 = 6.8 m from the shore. relative to M is 1T~I' the velocities of two bodies relative to
the ground will be respectively
Note: (i) The displacement of boat relative to shore (away from the
shore) -. mv
~

and V = - - -"-'
d '" md rc1 =~=O.8m m+M
2 m+M 5+20
(i j) In case oflwo body system ifm is displaced relative toM by (b) From Eqn. (i), we have
d rct • the displacement of individual bodies relative to the -> ->
ground are: mv+MV=O
Mdre1 d _ mdrcl
dl=m+ M and 2- m + M
respectively and are always in opposite directions to each
other.

Problem 28. A man of mass m climbs a rope of length L 0' [as .6.1* 001
suspended below a balloon of mass M. The balloon is
s tationary with respect to ground(a) Ifthe man begins to climb -> ..
up the rope at a speedv re/ (relative to rope) in what direction
0' [il' =dJ
and with what speed (relative to ground) will the balloon -> ->
0' [as d z is opposite to d 1]
move? (b) How much has the balloon descended when the man
reached the ballooll by climbing the rope? (c) What is Ihe state ~ ~-~ .. ~
of motion after the mall stops C/imbi'ng? Now as the man climbs up L towards the balloon (relative

.vCM =0
Solution: (a) Given that initially the system is at rest, i. e., to balloon), the balloon will descend a distance d 2 downwards
relative 10 the ground, so tbat upward displacement of man
relative to ground will be

so
.
Now as force of climbing is im internal force

V CM = constant = 0
d, =L-d, (i.e., d, +d, = L)
Substituting the value of d, from Eqn. (vi) in (v)
.... (vi)

-> -> m(L - d,)=Md,


mv+MV = 0 mL
Le., i.e., d 2 -_ - -- .... (")
VII
m+M m+M
-> -> i.e., the balloon will descend by mL/ (m + M) relative to the
or mv+MV =0 [as (m + M) = finite]
ground when the man climbs up a distance L (relative to
-> -> balloon) .
i. e., MV=-mv .... (i)
Furthennore here it is given that Ag.5.38 Note: The upward displacement of man relalive 10 ground will be
-> ->-> dI =L - d
2
. ML
(m+M) ....
( ... )
VIII
v rei = v- V .... (Ii)
(c) When man stops climbing ~rei = 0
or
~

so that from Eqn. (iii) V'=o.i.e., balloon will also SlOP


descending and will become stationary relative to the ground. ~ ~ ~ ~
or rn6r l +Mtlr2 =0 [astl r = d)
Problem 29. A block of mass
M with a semicircular track of or mdJ - Md 2 =0 ····Oi)
radius R rest on a horizontal ~ ~

frictionless surface. A uniform [asd 2 is opposite to dd


cylinder of radius r and mass m is Now when cylinder reaches from A to B it is displaced
released from rest at the top point (R - r) horizontally to the right relative to the block. So, if the
A (See Fig. 5.39). The cylinder Fig. 5.39 displacement of the block relative to ground is d 2 to the left,
slips in the semicircular the displacement of cylinder relative to ground to the right will
frictionless track. (a) How far the block moved when the be
cylinder reaches the bottom (point B) a/the track? (b) How .... (iii)
d, = (R - r)-d,
fast is the block moving when the cylinder reaches the bottom
of the track? Substituting d] from Eqn. (iii) in (ii), we get
Solution: Given that initially the system is at rest so "~(R-r)-d21-Md, =0
~ d _ m(R-r)
v eM =0. Now in slipping as no horizontal force is applied, the i. e. , (to 'he left)
2 - M+m
horizontal component of velocity of centre of mass of the (b) Applying conservation of ME between points A and B
V
system must remain constant. So, if -: and are the velocities I
mg(R -r)=-mv + -MV
2 I 2
of cylinder and block at B relative to the ground 2 2
~ -> Substituting the value of v from Eqn. (i) in it
IIIv+MV
"'-'-'--';'0-'- = constant = 0
m+M mg(R -r)=lm( - M v)2 +.lMV2
2 III 2
~ ~
i. e., mv + MV = O .... (i)
Which on simplification"g:;-iv-;,-;;s,-,,--
[asm+M=finite]
V = rn 2g(R r) (to 'he left:
M(rn+ M)

MISCELLANEOUS SOLVED PROBLEMS

Problem 30. A block ofmass m rests on a wedge of mass VEl = vcos9 - V (towards right)
M which in lurn rests on a horizontal table (Fig. 5.40). All the .... «(
v'v = vsin9 (vertically down)
Now as no external force acts along the horizonta
direction so, by conservation of linear momentum alon1
horizontal

.l. L _ __ _o =,________. vcos9


••
MV = mV'H = m(vcos9 - V)
Now by conservation of energy, we have
.... (ii:

t.
• ________ _
•• mgh = .lMV 2 +.lmv,2
2 2
vsin9 v
Fig. 5.40 or mgh =2'i MV' + '2I m ("v H + Vy.')
slIrfaces are smooth and the system is at rest initially. Find the
velocity ofthe wedge at the instant when block after sliding on [as v' =~v'J +vV]
the wedge tal/cites the table.
Substituting the values ofvli and v'v from Eqn. (i)
Solution; Ifvis the velocity of block relative to the wedge
and V that of wedge relative to table at the instant block mgh =.lMV 2 +.lm[(vcosB_V)2 + (vsin9)2 J
touches the table. The horizontal and vertical component of 2 2
velocity of block relative to table will be
i.e., mgh =!MV 2 +.!."i[v2 ·~V2 -2vVcosSY -- Adding tlW,s,e all total change in momentum ,
2 2 • .. ·.6~ .=:hi'[vo + 2vl.'+. :.. + 2~~::'1: "'" VII] . ' .
i.e., mgh = ~(M +m)V 2 +kmv2 -mvVcos8 or .6p =mv o[t'+2e+2e 2 +..... ]
Now substituting the value of v from Eqn. (ii) and
simplitying for V, we get
or bp = rnvo [1 +2e C~ e l] = m~2gh[: ~ :J .... (ii)
,

2 2 . ~ d~
V=[ 2m gcos 9h ]112 (c)"Nowas F=~ so, F = bp
., dt au IJ.T
(M+m)(M+msin 2 9)
Subs:t"ltuting the values of IJ.T and IJ.pfrom Eqns. (i) and(ii)
Problem 31. A ball is. dropped from a height h on to a
floor. If in each collision its speed becomes e times of its
striking value (a)find the time taken by bali to stop rebounding
F
au
. =[I+eJ
= m...;2gh - - x
l-e
~[I
- --eJ
- = mg
2h l+e
.... (iii)
(b)find the total change in momelltum in this time (c)find the
average force exerted by the ball on the floor using results of Note:. (i) This is the same force which a ball at rest will exert on the
pa,' (a) and (b). floor, a result which is correct but by no means evident!
Solution: (a) When the ball is dropped from a height h, (ii) The total distance covered by the ball
time taken by it to reach the ground will be H=ho +2h[+2h 2 + .. ··

to = J¥ and its speed Vo = ~2gh But II =h


o
and ase=.!!.=~2g~
2110lJ
II

i.e., h l = e"lh o ::= e"lll

similarly, 112 = e"lh l = e"l(;h) = e4h


2 : 4 6
H = h+ 21Th + 2e h+ 2e h+ .. ..
H =h[ 1+ 2;(1 +e"l+ e4 + .... )]

I-to-t-- tl~ t2--+l


Fig. 5.41
H=h[I+2;(_1,ll =h(I+~l I· · e l-e
Now after collision its speed will become e times, i.e., Problem 32. Prove that in case of oblique elastic
v I = ev 0 = e.J2gh and S0, it will take time to go up till its speed collision of two particles of equal mass if one is at rest, the
becomes zero = (vI/g). The same time it wilt take to come recoilingpartic!es always move offat right angles to each other.
down. So total time between I and II collision will be Solution: In elastic collision momentum is conserved. So,
'I =2vi/g· Similarly, total time between II and III collision conservation of momentum along x~axis yields
'2 = 2v2/g· mu=mv1cos9 1 +mv 2 cosS 2
So total time of motion
i.e., u=v 1 cosS I +v2 cas9 2 ... .(i)
IJ.T = to + II + t2 +.... and along y -axis yields
2v I 2v2
or IJ.T=t o + - + - 0=v l sin9 1 -v 2 sin9 2 .... (li)
g g
Squaring and adding Eqns. (i) and (ii), we get
2
u =vr +v~ +2v 1v 2 cos(9 1 +9 2 ) .... (iii)
or
As the collision is elastic y-axis IVj

121212 t,, ,i![i


Zmu =2"mv l +Zmv 2
,, ,,,
.2 u =2 2 ....
(.tV ) , '
_________..
J¥[I + C~ e l] =J¥[: ~ :J
I.e., V I +V2
~\e~
= 2e
In the light of Eqn. (iv), m
~ m .... .( 92 x-axIs
............ Y-
(iii) reduces to
(b) Change in momentum in I collision = mv 1 - (-mv o ) 0&.,
2v l v 2 cos(9 1 +8 2 ) =0 ....',
= m(v l +vo) Fig. 5.42
As it is given that VI 7; 0
Change in momentum in II collision = m(v 2 + vI)
andv 2 7;O
Change in momentum in n th collision = m(v /I + V 11-1)
so cos(9! +9 2 )=0, i.e.,
Problem 33. A nucleus ofmass M emits a y·ray photon of Solution: According to theory of relativity
frequency Va. Show that the loss of internal energy by the mo rn o 2 mo ( c)
nucleus is not hvo but hvo (l + hv o/2Mc 2 } m= ~1-(vlc)2 = ~1 - (l12)2 = -J3 as " =2
Solution: Initially the momentum of nuc\eus is zero and 2 c I
as in emission no external force acts on the nucleus the final p=mv =- mox-=-moc
momentum must also be zero. -J3 2-J3
-+ -+ -+ -+ E =me2 =(21.fi)moc2
i.e., ( P)pholon + (P}nuclcus =0 o r ( P )nllCleus =-(p)pboton
2 2 2 0.27 2
i.e., nucleus and photon will move in opposite directions. The KE =E - Eo = ..,fjIllOC - moc = ..fj mOc
K.E acquired by the nucleus due to decay will therefore be
Problem 36. Two identical particles a/mass III carrying a
K= (P)~ucleus. ( kv o/e)2 charge q each initially one is at rest on a smooth horizontal
2M • I. e., K= 2M
plane and the other is projected along the plane directly
towards the first particle/rom a large distance, with speed v.
hVo]
[as (P)N = (P)photon = -c- Find the distance 0/ closest approach.
Solution: As here
i,e., V
CM
=mx u+mxO =!!.
m+m 2
So the total internal energy lost by the nucleus So the velocity oftwo particles in the centre of mass frame will
f::..E =Energy of photon + K be

i, e., h2v22
l1E=hvo + _ _0 = /zvo 1+ __ [hV°_2 ] VI =u--u=-
u an d v
2 2 2
= O"
--= --U
2 2
2Mc 2Mc i. e., in the centre of mass frame both particles will approach
Problem 34. Assuming that according to 'theory of each other with a velocity (u/ 2).
relativity' the ellergy E and momeJltum p of a particle a/rest So the energy of the system when the particles are fa r apart
mass IIlO are related to each other through the relation
(2" )' +'2 m('2 )2 +O='4 mu
=
p 2 c 2 +m~c4, show that pair production cannot take I II I u I 2
E2
E I = 2: m .... (i)
place in vacuum i/momentum and energy are to be conserved.
And energy of the system when thc particles are closest to each
Solution: Let the pair production be possible in vacuum.
The momentum (hv l c) of the photon is equally shared other (When u = 0 at a separation R).
between electron and the positron so that each will have a I q2 I q2
E2 =::.0+0+ - ---= - - - - .... (ii)
momentum hvl2c. Thus, the total energy of the electron- 41tEo R 41tEo R
positron pair will be
Now as energy is conserved
E =::. E er+-,;-E~p,-----,,~ I 2 I q2
E =~ p 2 c 2 + m~c4 Et =E 2 i.e., - IIIU
4
=-- -
41tEo R
0'

hV)2 2 Alternative Solution


i. e., £=2 ( 2c;' e + mo2c 4 [as p = hv/2e]
When one particle approaches the other, due to electric
repulsion other will also start moving and so, the velocity of
i.e., £ =~(hv) 2 +4m5 c4 i.e., E> hv first particle will decrease whi le that of other will increase and
Now as the energy of the photon was hv, the energy of the at closest approach both will move with same velocity. So, if v
pair produced cannot be greater than hv if the energy is to be is the velocity of each particle at closest approach, by
~
conserved. T his in tum implies that jf a pair production takes conservation of momentum (as F exl =0)
place in vacuum both energy and momentum cannot be
mu ~ mv + mu, i.e., v = (u/2)
conserved, i. e., the process of pair production cannot take
place in vacuum. Now when the particles are far apart
Problem 35. A particle 0/ rest mass 1110 1II0ves with a I 2
EI =2:mu +0+0
speed el2. Calculate its mass, momentum, total energy and
kinetic energy. IMNR 19961 And when they are closest to each other
£2 :::!m(!:)2
2 2
+!m(!:)'
2 2
+_,_cC
41t6 R
So eliminating 9 between these we get trajectory of the
pointP as

So by conservation of energy
0
x'
--"--cc+ L ~ ,
,
[(Ll2) - r]' "
1 2 I 2 I q2
- mu =-mu + - - - which is an ellipse.
2 4 41t6 0 R
Which on solution yields Note: (i) If the rod rotates with its lower end fixed on the floor, for any
point P on the rod (even centre of mass) [Fig. 5.43 (b)]
4q' x=rcos 9 and y=rsin9
R ~ --"'---;-
2 so that the trajectory will be x 2 + I
47t&omu = r2 , i.e, cirde
(ii) If the rod slips on the horizontal with its one end always
Note: The usual way'ofsolving Ihis problem, I.e., equating the initial resting on the vertical wall, for any point P on the rod (even
KE ::: (l12)lIIu 2 with final PE", (q2{ 4negR) is wrong because the centre of mass) [Fig. 5.43 (c)l
particle which is initially al rest also starts moving as the other x'" (L-r)cose and y = rsi n e
one approaches it. so that the trajectory will be

Problem 37. A uniform thin rod a/mass M and length L is -


x';
-2 +""2 '" I,
.
i. e , ellipse
standing vertically along the y-axis on a smooth horizontal (L-r) r

surface, with irs lower end at the origin (0, 0). A slight
Problem 38. Two blocks of masses m l and 1n2 are
disturbance at I::: 0 causes the lower end to slip on the smooth
surface along the positive x-axis and the rod starts/alling. (a) connected by a light ;nextensible string passing over a smooth
What is the path followed by the centre of mass of the rod fixed pulley of negligible mass. Find the acceleration of the
during its fall? (b) Find the equation a/trajectory ofa point on centre ofmass ofthe system when blocks move under gravity.
the rod located at a dis tance r from the lower end. What is the Solution: The equation of motion of centre of mass is
shape of the path of this point?
-> ->
Solution: (a) As the floor is frictionl ess, no horizontal MacM == F cxt
force-acts on the rod and as equation of motion of eM is And as there is no external force in horizontal direction, so
-> -> centre of mass of the system does not change along horizontal
F ext == Ma w
direction.
-> -> ->
For vertical motion of centre of mass
--I> F ext ~M,--,!g:---,R
~
so 3CM == -- ~-
M M (a ) == F ext
So, centre of mass moves vertically downwards with
CM Y M
acceleration 'g' or the path followed by centre of mass is a (m, + m2 )g -2T
vertical line along y-axis, i. e., x == 0 (m l +ln2 )
(b) At any instant for a point P on the rod as shown in Fig.
5.43 (a) Further, aCM -- _n~"_m
a~,_+_m--,,,_a-,-,
+m I ,
x ~ [(LI2) - r ]eosa
and y == rsin 9 -> ->
[asa l =a, a 2 == -a]
.t.
::
Y
::
.t.
Y
,t
,,
:, :• :r :. :'
:'
CM':.:, ,- : ' eM
eMI.: :
eM: .:' CM
C
:
' 1

: :
' ........ ~
p~ ~
r"
ifi
"

Yl; olP
I .. : : :

yI: :
• P.:
-j-- ----
:e "-
~,
However, the equations of motion of two blocks are
m2g - T =m 2a
x x
, ;i"':.L- x O~X--l
--L);oc,,,,",:; I
Ol+-X-l
,
and T - mlg "'/lila
(a) (b) (e) Which on eliminating' a' between them gives
FIg. 5.43
2m l m2g
T ~ .... (ii)
(m 1 +m 2 )
Substituting the value ofT from Eqn. (1:::+:::e"--)c~o-,-s-,-
,-,-I,,
.m e-,-I___+--,C:::m-,-,_-:::e.em-,-I,-,-)u,",:::c~
os=--e",-,
vzcosP z =-
(ii) in 0), we get ml + ml
[em! +m2)2 -4m\m 2 ]g
=~(Vl sinPI )z +(VICOSP I /

I
(aCM)y "'" 2 VI
(m, +m 2 )
P. V II Final tangential component of velocity
tanpi = _ 0
2T .. vln Final normal component of velocity
eM ..
. ~imilarlywe can find V z and Pz..
This is the desired result and from
(ml + m2l 9 m2£! Note:
this it is evident that acceie;oation of eM Flg:,5.44
is always vertically downwards
i!Tespectiye of whether m1 is heavier or m2 •

Note: Here't'M should not be confused with a. OeM is the ac~~l eratjon
of centre of mass while albat or m! or m2' Problem 39. A ball ofmass m hits afloor with a speed v
making an angle . .of incidence S with the normal. The
Oblique impact coefficient ·of restituti.on is e. Find the speed of the reflected
Oblique impact is the impact in which relative velocity of ball and ·the angle ofreJl?ction of the ball.
approach ofthe colliding bodies is not alongthe'line of impact. Solution: Suppose the angle
Tangential of reflection is a' , and the speed

cfG~'{38r efG:·'
after .the ·collision is v' (see Fig. v'
5.46). Th.e ·floor . exerts a force on
the hall along.the normal during the
.' : Normal ' . collision. There is no force parallel ,.-vsinO tV'cosO'
Before collision . : axis After collision' tv cos a l-.v'sin 0'
to t4e ' ~urface. Thus; the parallel Initial velocity Final velocity
Fig. 5.45
comp.onent of the velocity of the ·Fig.5.46
Procedure for solvirig the problems ball remains unchanged. This gives
Step 1. Draw n-axis and t-axis at the point of impact. v'sinS'=vsinS .. ..(i)
Step 2. Conserve the momentum of the system, along and For the components normal to the floor, the velocity of
perpendicular to the line of impact, i. e., along n~axis and t~axis. separation is v' cosa' and the velocity of approach is vcosS.
We obtain, Hence, v' cosS' = evcosS .. ..(ii)
mlll i cosS I + mzllz cosS z :::: miv i cosPI .+ mzv z cosP z From (i) and (ii), v' = v~sin 1 a + e l cos Z a
and ml III sin S I + mllll sinSl :::: miv i sin ~I + mzv z sin P z
Hence, tan9, = tana
.... Ci) e
If the colliding bodies are smooth, no force is acting on ml For elastic collision, e = I, so that S' = a and 11 = v.
and ml along the tangent; the momentum of m] and ml
remains conserved along t~axis; Note: (i) Resultant velocity of particle v : u~ sin 2 0 + e'2 cos20
i.e., mlll i sinS I ::::mlv i sinP I .... (ii) tanO
(ii) Angle ofrebound tan 8': -,-
and mzllzsinSz=mzvzsinpz .... (iii)
Coefficient of restitution is defined along line of impact (iii) Impulse of the blow: Change of momentum of the body
only. -. -. -. -.
'" {mv'sinO' i + (mv'cos8' j)} - (mvsin 8 I - mvcos8 j)
e = _ ~vLIc"O-,-S"P-,-I_-_V
" ,,--c,,o-,-s,,P--,-, ~ ~
····Civ) : m(v'sin8'- vsinO) i + m(v'cos8' + vcosO) j
ulcosa l- uzcosS z
~

Now we have four equations and four unknowns v I' v z.P I Since v'sin8': vsinO=> Impulse: m(v'cos8'+vcos8) j
and pz. Solving four equations for the four unknowns, we Puttingv'cosO'= evcos8,
obtain Magnitude of the impulse : m(1 + e)vcos8
(m +ml)uI cosS I +mz(l+e)uzcosa z (iv) Change in kinetic energy
VI cosp] = l .... (v)
ml +m z t.KE: -1 mv' 2--mv
I 2
, 2
Putting the value of v' we obtain Solution: Impact takes place along the line of impact
= ~m[v2(Sin2 8+ ;. cos:2 S) _ v2 ) nonnal to the inclined plane at P. The velocity of approach of
the ball = (vcos8). The ball moves horizontally just after tlfe
'" ~mv2[sin2e + ;' cos 2 8 - 1] collision with a velocity say v'. Therefore the velocity of
separation of the ball is (v' sin 8). Using the definition of
= -~mv2(cos2e _ ;'cos2 8] coefficient of restitution,
I . (velocity of approach) e = velocity of separation'
6KE = --(1 - ;')mv 2 cos 2
2
e -> ->
=:> -{vcosS(-j)}e = (v'sinS)j
- ve sign indicates the loss ofKE.
tf = evcot S .... (i)
Problem 40. A particle ofmass m m,,,,_-,~~:::,.-c­ Since the time of impact is very small, the change of
strikes elastically with a disc a/radius momentum of the ball due to the non-impulsive gravitational
R, with a velocity v as shown in the R12 Q force is negligibly small. Therefore, the momentum of the ball
Fig. 5.47. If the mass of the disc is just before and after the impact remains constant, along the
equal to that of the particle and the Fig. 5.47 inclined plane.
suiface of contact is smooth, find the (mv)sinSi = mv' cosS i
velocity of the discjust after the collision.
Solution: Impact takes place ~,

along the line of impact, which is Line of', /


normal at the point of impact. impact
jlY sin e

. , ,, ' ' "


,
'Q'
y
<' vsinS = v'cosS
Using (i) and (ii), we obtain
vtanS=evcot8 =;. e=tan 2 S
.... (ii)

Hence, the momentum of particle Problem 42. A ball of mass m strikes the fixed inclined
and the disc changes along line of "-.weDs e plane after falling through a height h. /fit rebounds without
FIg. 5.48
impact. However, no external force losing energy, find the impulse on the ball.
acts on the system along the normal line. Hence we can em v cos 0
:,,
conserve the linear momentum of the system (disc + particle)
along the nonna!. Since the masses of the disc and particle are vsln 9
equal, so the exchange of momentum takes place along the
nonnal. The particle completely delivers component of its v Initial velocity

Final velocity
momentum (mvcose) along the nonna!. Fig. 5.50
Velocity of the disc = VI = (vcosS)j Solution: When a body strikes a surface, impact takes
since; sin S = R~2 = ~ [from figure] place along the line of impact. Line of impact is perpendicular
to surfaces in contact. Since impulsive force acts along line of
impact, the velocity of the ball changes along this line.
COSS=~I-m2 =-
~ The change in velocity of the ball along line of impact
= Llv =2vcos8.

v ! =-J
.J3v, If inclined plane is smooth there will be no change in its
2 velocity along the tangential direction (inclined plane).
Impulse = mLlv = m(2vcos8) = 2mvcos8
Problem 41. A ball after falling through a distance h
collides with an inclined plane of inclination 8. /f it moves Since the ball falls through a height hjust before collision
horizontally just after the impact, find the coefficient of v = ~2gh; Impulse = 2mcos8~2gh.
restitution? Problem 43. A ball ofmass 50 g is droppedfrom a height
n-axis h = 10m. It rebounds losing 75 per cent of its total mechanical
p ,/"n energy. /f it remains in contact with the ground for Llt =0.01
;
~
sec., find the impulse of the impact/orce.
t-axis

• vcos~ -L ~"tvsin9 Solution: We use impulse and


momentum equation. The change in
->
Before collision After collision momentum 11 P of the ball during
Fig. 5.49 M = 0.01 sec is the impulse (FLlt) of the FIg. 5.51
force of impact.
Solution: Let the ball strike the plane at A. We assign a
co~ordinate system with origin at A. The velocity of the ball
while striking the 30 0 plane = gt = 2g. The components of ~
along and perpendicular to the plane are usin 300 and ucos 30°
• --)0 ' --)0
since m vI and m Vz are vectors at 180°, respectively. After impact, the velocity component u sin 30°
along the plane remains unaffected while the component u cos
Hence .... (i) 30° perpendicular to the plane becomes e(ucos30 0 ) on
rebound.
Since the ball falls through a height h, VI ' =: ~2gh. It is
given in the problem that the ball losses 75 per cent of its total Thus velocity of rebound = e· 2g . .J3 = .J3eg
2
mechanical energy, i.e., final kinetic energy,
1 2 1 2 1
'2mv2 ""' 'imvi x '4
A

Hence, v = 1!..l = ~2gh = {gh


<i.. ...
" " ,\~. 2 2 2 f2 30'
Substitution ofthe ~values ofv\ and v 2 in (i) yields Fig. 5.52

Fl!.t = 3m (gh = 3xO.05 (9.8)(10) =1.05Nsee The acceleration perp'endicular to the plane =- g cos 300

" ' :,
V:i 2 =-g-!3 12
Problem . 44. Ra4iations of wavelength 200 tim, If t be the time of striking the plane again, then using
propagating in the form ofparallel beam, fall normally on a
I 2
plane. me...t{i{it~,#Tirface. The intensity of the beam is 5 mWand s= ut +2gt
" " " ', " ~ .". . 2
its cross·sec.tional area is 1.0 mm . Find the pressure exerted
by the radiation on the metallic surface if the radiation is r:;
0=v3egt--~gt
I -!3 ,
completeIY 1~fi.ected. 2 2
S'Olu~:~:
. Energy of each photon E = he , 5
t=4e=4x-=2.5sec
8
If P is the power of squrce, the number of photons per
second incident on the metallic surface
i
The distance AB = usin 30 0 t + gsin 30 0 t 2

p ,p
= -= ~

E he =(2g)x2.5x -1 + -I xlOx-1(5)'
-
2 2 2 2
Momentum of each photon =.ft , =25+ 250 = 40.63m
16
Change in momentum due to reflection = 2h
, Problem 46. Two wedges, each of inclination 45 0 and
mass M, lie on a smooth horizontal plane. A ball of mass m is
Total momentum imparted to the surface per second
dropped from a height. After falling through a distance H it
2h U 2P = -' ~ =~ strikes one wedge and then the other and finally bounces
').. hc c
vertically upwards. If both the impacts are elastic thenjind the
Pressure =: Force = 2P height to which the ball bounces.
Area cA ."
Solution: The ball strikes the wedge A i,'ith velocity
2 x 5 x 10-3 v = ~2gH .... (i)
=
3xl08xlO 6
Since ball falls vertically therefore the horizontal
= 3.33 x 10-5 N/m2 momentum is zero. After the impact the ball moves
Problem 45. A ballfaUs vertically for2s and hits a plane horizontally towards B. If v I is the velocity of the ball after
inclined at 30 0 to the horizontal. If the coefficient ofrestitution impact and VI that of the wedge, then the law of conservation
be 5/8, prove that the time that elapses before it again hits the of momentum gives
plane is2.5sec. Find also the distance along the plane between mV I + MVI =0 .... (ii)
thejirst and second impact. (g = IOmls2)
1. The component along
T T t-axis of the momentum of
H V2 h ball B is conserved since
FOI
"
friction force is absent. Fextl!.t
v, A B
V2 Hence the t component of .~~~F;O.Ijl;;i~:""
the velocity of ball B
Fig. 5.53 remains unchanged, Fig. 5.55
Impact is elastic, hence the kinetic energy of the system is i.e., (VB),=(V'B)'
conserved. 2. The component along the horizontal x-axis of the total
1 2 I 2 1 2 momentum of block A and ball B is conserved,
Zmv =2"mv] +'2MV1 .... (iii)
i.e., mAVA +mB(VB)x =mAVA+mB(ViJ)x
V2 _v 2 _ M v,2 3. The component along the n-axis of the relative
or 1 -m t .... (iv)
velocity of block A and ball B after impact is obtained
by definition of coefficient of restitution,
Egn. (ii) gives '""I .... (v)
VI =- M i.e., (V;'), -(VA)" =e[(VA ), -(VB),]
Problem 47. Consider a mass moving along x-axis with
From Egns. (iv) and (v), we get
velocity v, hits mass m2 and sticks to it. Masses m2 and m) are
2 M 2
VI =--_·v
M+m
.... (vi) connected through a massless rigid rod. Ail the three particles
have same mass. Find·the motion o/particles after impact. Ail
According to condition of problem after hitting B, the ball particles can move without/riction on the horizontal x-y plane.
moves up with velocity v 2 vertically. Let B move with Solution: Since all m3
velocity V2 . Then the law of conservation of momentum gives particles can move without
mV I = MV2 .... (vii) friction on the horizontal x-
y plane. We can apply law
The law of conservation of energy gives
of conservation of momen-
'21 mv 2, =:2mv2
1 2 I
+ZMV2
2
.... (viii) tum. A careful analysis will
show that the rod can
Egns. (vii) and (viii) give v~ = M - m
M
vi .... (ix)
transmit axial forces only. Fig. 5.56
Equation for conservation of momentum in axial direction
From Egns. (vi) and (ix) v~ = M - m v 2
mvcos450
mv =3mv'

-E
.... (x) .... (i)
M+m .fi ' mv
If h is the height to which the ball rises, then mv sin 45 0
v' =--.!L.. Fig. 5.57
v~ =2gh .... (xi) , 3.fi
(Vel0city component of all the three masses is same in axial
Egns. (i), (x) and (xi) give h = M - m H direction.]
M +m
Conservation of momentum in transverse direction
Application of oblique impact
Consider the collision between -mv = 2
mV'I or VI,=
v -- .... (ii)
block A and ball B. Block A is v, .fi 2.fi
constrained to move in horizontal
direction and ball B is free to move
" because no external force acts
on system of three masses in
Axial direction
v /
V',
in the plane of the figure. All the transverse direction. 3-/2/
m3
surfaces have been considered In the second equation Transverse
smooth. At the instant of collision, v ,VB are Initial velocity of mass m) in direction
action and reaction force F is veloc1t1es of block and transverse direction is zero ~ ___ ~ ___ _~ __ ___ ~_<!.t!l_o!fM
internal for block A and ball B. ball respectively and VA' just after impact because it 113..f2.
External force is exerted on block A
VB are final velocities. receives impulse in axial ml + m2
Fig. 5.54 direction only. Throughout
in vertical direction by ground.
Following equations can be set up: the whole problem centre of
mass moves with a constant velocity ~ along a line Y= Ir;;.
3 3,,2
After impact system rotates at a constant angular velocity co Problem 49. A 10 kg package drops
which is found by dividing difference of transverse component into a 25 kg cart with a velocity of3 mis,
of velocity at the two ends of rod by the rod length I. Knowing that the cart is initially at rest and
may roll freely, determine (a ) the final
ro=~ velocity of the cart, (b) the impulse exerted
2Jil by the cart on the package, (c) the fraction
Fig. 5.60
Notice that total angular momentum about the centre of mass is ofthe initial energy lost in the impact.
constant and equal to
Solution: We first apply the principle of impulse and
L= mvl momentum to the package~cart system to determine the
3Ji velocity v 2 of the cart and package. We then apply the same
Problem 48. Two identical buggies 'I' and '2' with one man principle to the package alone to determine the impulse R6.t
in each move withoutfriction due to inertia aiongparallei rails exerted on it.
towards each other. When the buggies get,opposite each other, (a) Impulse~Momentum PrInciple: Package and Cart
the men exchange their places by jumping in the direction mpv l +IFdt=(m p + m c )v 2
perpendicular to the motion direction. As a consequence, buggy
'1' stops and buggy '2' keeps moving in the same direction, with x~components; mpvl cos 300 +0 = (mp + m c )v 2
its velocity becoming equal to v. Find the initial velocities a/the (10 kg) (3mJs) cos 300 = (10 kg + 25 kg)v,
buggies v 1 and v2 if the maS$ of each buggy (without a man) v 2 =0.742m1s
equals M and the mass of each man is'm.
Solution: Before applying law of
conselVation of momentum one must ~YL~
+--- ~Rd1I A~
+,e X
... mpvl
"
+ _
d1 (m,+mpi"
decide the system first. For problems
involving mass ejection and addition, the
ejected mass as well as added mass both Fig. 5.61
are parts of the system. At the instant of Note that the equation used expresses conselVation of
jumping out action~reaction pair is internal momentum in the x~direction because forces between cart and
force for buggy and man jumping out. Fig. 5.59 package are internal.
Similarly action~reaction force between
(b) Impulse~Momentum Principle: Package
man jumping in alld buggy is also internal force.
Let the buggy' l' move to the right and buggy '2' to the
left. When the men jump, man in ' 1' brings momentum mv\ ~Lx ('--...,---- + I I·
(which is directed to right) in the buggy '2' and man in '2' +ve ~mpvl
R~ll.t·tRll.t
y
brings momentum mV 2 in 'I' (which is directed to left).
Fig. 5.62
Now, net momentum of '1' to the right at the time of
jumping of men, mpvl + IRdt=m p v 2
Pinilial = (M +m)v 1 -mv 1 -mv 2 = Mv 1 - mv 2 x~co"!pone n ts:
mpvl cos300+Rx6.t = (10 kg) (0.742 mls)
Since buggy' 1' stops, so Pfinal = O. R x 6.t = -18.56 N-s
By conservation of momentum MVI = mV 2 .... (i)
y~components; - mpv] sin300+R y6.t = 0
Similarly momentum of ' 2' to the left at the time of
- (10 kg) (3 mls) sin 300 +R y6.t =0
jumping of men,
Rllt=+15N~s
Pinilial = (M + m)v 2 - mV 2 - mV I
The impulse exerted on the package is R6.t = 23.9 N~s at
Pinitia] = MV 2 - mV I 38.9°.
P fina1 = (M + m)v (c) Fraction of Energy Lost
So, by conselVation of momentum,
The initial and final kinetic energies are
(M+m)v = Mv 2 -mv 1
So, combining equations (i) and (ii), we get .... (ii) KE.='m
I 2 p
v', ='(IOkg)(3mJs)2=45J
2
v _ mv
' -[M-m ] KE f = ~(mp + m,)v; = t(IOkg+25kg)(0.742mJS)'

and v2 Mv
= ,--'=~ = 9.63 J
[M -m]
The fraction of energy lost is Conservation of Energy
KE,-KE[ =45 -).63 Position t. Potential Energy: PE! =rnA gl
KEI 45 I ,
Kinetic Energy: KE l ='2rnBvO
~ 0.786
Position 2. Potential Energy: PE 2 =mA gl+mBgh
Problem 50. Ball B, a/mass mB, is I ,
suspended from a string of length I Kinetic Energy: KE 2 =2" (m A + mB)(v A )2
attached to cart A, ofmass mA' which may . . . . . . . . . ..
roll freely on a frictionless horizon/aJ
surface. If the ball is given an initial
horizontal velocity Vo while the cart is at B
rest, determine (0) the velocity of B as it
reaches its maximum height, (b) the
mB
Fig. 5.63 Solve for h. Substituting for (v A )2' h = mAv 0 lv'
.....Q.
[ (rnA +rnB) 2g
maximum height II through which B will rise.
Solution: We shall apply the impulse-momentum Note: FormA » niB_ the an~wers obtainedredoce to (vB h .. (v Ah :: 0
principle and the principle of conservation of energy to the and h =v~ / 2g; Boscillates as a simple pendulum with A fixed.
cart-ball system between its initial position 1 and the fina l
FormA «Ins they reduce to (v S )2 = (vA)z :vo and h--O ; A
position 2 when B reaches its maximum elevation.
and B move with the same constant velocity vo'
Position 1: (VA) ' =0;
(V.)I =v o .... (i) Problem 51. A rod AB ofmass M and length L is lying on a
horizontal ji-ictionless surface. A particle of mass m travelling
Position 2: When ball B reaches its maximum elevation its
along thesutface hits the end A ofthe rod with a velocityv o in a
velocity (VB /A h relative to its support A is zero. Thus, at that
direction perpendicular to AB. The collision is completely elastic.
instant its absolute velocity is
After the collision the particle comes to rest.
(v B )2 = (VA )2 + (v BIA h = (v A )2 (a) Find the ratio mlM.

czp--'
Position 1 Position 2 (b) A point P on the rod is at rest immediately after the
:~q: (vAh! collision. Find the distance AP.

~L"~"O (c) Find the linear speed ofthe point P at a time rr.LI (3vO)
after ,fie collision.
(VOh:· (v"b
Soh,tion: (a) Let the linear and angular
(vs)! - vo
velocities of centre of mass of rod just after
Fig. 5.64
collision bevCM and ro respectively. Applying
conselVation of linear and angular momentum,
Note: At the extreme position velocities of A and B are same.
nwo = MV CM ....(i)
Impulse-Momentum Principle
x-components: mBvO = (mA + rn B )(v A h
mVOL =( ML' )___ ~ ....(ii)
2 12 •
which expresses that the linear momentum of the system is P ,./00
conserved in the horizontal direction. Solving for (v A )2' As the collision is elastic, so applying
B
conservation of energy, Fig. 5.66 (a)
mBv O
(v.), = (VA)' = -,--"-'''-;-
(rnA + rnB) I 2 =-I MvCM
-mvO 2 ML') ,
+-I ( - - 0> .... (111)
2 2 2 12
A --...
"--,---'(V,v, - 0 mvo .. 6mvo
From Egn. (i), v CM = - - and from Egn. (II), 00 = - - ,
M ML
Substituting in egns. (iii), and simplifying, ~ ="4
Refl.'!~~~!t.l!IJ~J~ (ve)l - Vo
(b) For the velocity of point P immediately after collision
Fig. 5.65 W.r.t. centre of mass to be zero
=(X-~)ro-vCM =0 Problem 52. A cart loaded with sand having total mass
rna =1800 kg moves on a straight horizontal road starting
from rest under the action of a/oree 0/120 N. The sand spills
X_f.)6mVo_mVo="O j,e. x=2L through a smail hole in the bottom at a rate of K =0.5 kg/so
( 2MLM" 3 What will be the velocity a/the cart after20 min?
(c) Angle turned by rod in time [ 7tL ), Solution: In this case, sand falls on the ground vertically
3vo and comes to rest. Hence, horizontal velocity of ejected sand
with respect to ground is zero; so
9=00/='6:0 x3~ =~(; }n=~x~~=~ d
F ext = dt (mv) or Fextdt = d(mv)

The rod takes the position after '0 which on integration gives
time (1tL 13v o ) as shown in Fig.
5.66 (b). So,linear velocity of Pin F~t I;'t dl = 1d (mv)
y~direction
F ext (t 2 -f l )=(m2v 2 -mlv l ) ,,,,(i)
L L 6mvo va 4 The rate of leakage of sand, K = (-dm I dt) is constant which
=-ro=-x--=-
6 6 ML ,4 Fig. 5.66 (a)
on integration gives
[': CP =L/6Cj m
J 2 dm = -Kf2 dt
The velocity of the ·rod due to linear motion (in horizontal ml II

direction), is, (m 2 -m l )=-K(t2 -II) ",,(ii)

vCM~ (-my"0=·-
M
;:' Vo
' 4
Here, 11 =0, 12 =1, VI =0, v2=v
From Eqns. (i) and (ii), F ext ' t = (mo - Kt)v
v = -,--F-"",,_',,'-,-
So, speed of P = (rno -KI)
Substituting for Fext' t,K and mo' we get
V= 120x20x60 =120mls
(1800 - 0.5 x 20 x 60)
CONSERVATION OF MOMENTUM 197

EXERCISE
(AJ Only One Choice Is Correct 8. In case of rifle shooting the kick will be minimum when:
1. A particle of mass M is moving in a horizontal circle of (a) A light right rifle is held loosely against shoulder
radius R with unifonn speed Y. When it moves from one (b) A light rifle is held tightl~ against shoulder
point to a diametrically opposite point its: ICBSE 1992) (c) A heavy rifle is held loosely against shoulder
(a) Momentum does not change (d) A heavy rifle is held tightly against shoulder
(b) Momentum changes by 2MV 9. ~o unequal masses are tied together with a compressed
(e) KE changes by MV 2 spring. When the cord is burnt with a malch releasing the
spring the two masses fly apart with equal:
(d) KE changes by (1I4)MV'
(a) Kinetic energy (b) Speed
2. A lead hall strikes a waH and falls down. A tennis ball
having the same mass and velocity strikes the same wall (c) Momentum (d) Acceleration
and bounces back. Which is the correct statement? 10. Ifa ball is thrown upwards from the surface of earth:
(a) The tctulis hall suffers a greater change in momentum (a) The earth remains stationary while the ball moves
upwar~s
(b) The lead ball suffers a greater change in momentum
(b) The ball remains slationary while the earth moves
(e) Both the balls suffers equal change in momentum
downwards
(d) The momentum of lead hall is greater than that of
(c) The ball and earth both move towards each other
tennis ball
(d) The ball and earth both move away from each other
3. A light and a heavy body have equal KE. Which one has a
greater momentum? 11. A particle at rest suddenly disintegrates into two particles
of equal masses which start moving. The two fragments
(a) The light body will:
(b) The heavy body
(a) Move in the same direction with equal speeds
(c) Both have equal momenta
(b) Move in any directions with any speed
(d) It is nol possible to say anything without additional
(c) Move in opposite directions with equal speeds
information
(d) Move in opposite directions with unequal speeds
4. A light and a heavy body have equal momenta. Which one
12. A shell explodes and many pieces fly off in different
has greater kinetic energy?
directions. The following is conserved:
(a) The light body (b) The heavy body
(a) Kinetic energy
(c) Both have equal KE (d) Nothing can be said
(b) Momentum
5. Two bodies of masses rnA and nto have equal KE. The ratio
(c) Neither momentum nor KE
of their momenta is: ICPMT 1990; CBSE 1993)
(d) Momentum and KE
(a) mil : m H (b) ma: mA
13. The spacecraft of mass M moves with velocity V in free
(c) ~mA '~mB (d) m~: mi space at first. then it explodes breaking into two pieces. If
6. Two trains A and B are running in the same direction on after explosion a piece of mass m comes to rest, the other
parallel rails such that 'A' is faster than 'B', packets of equal piece of spacecraft will have a velocity: . [CPMT 1992]
weight are transferred between them. What do you think (a) MVI(M-m) (b) MV/(M+m)
will happen due to this? (c) mV/(M-m) (d) mV/(M+m)
(a) A will be accelerated but B will be retarded 14. A bomb of mass 9 kg explodes into two pieces of masses ?
(b) B will be accelerated but A will be retarded kg and 6 kg. The velocity of mass 3 kg is 16 mls. The KE of
(c) There will be no change inA but B will be accelerated mass 6kg injoule is: IEAMCET 1993]
(d) There will be no change inBbulA will be accelerated (a) 96 (b) 384
(c) 192 (d) 768
7. A bullet weighing 50 g leaves the gun with a velocity of 30
mls. If the recoil speed imparted to the gun is I mis, the 15. A free body of mass 8 kg is travelling at 2 metre per second
mass of the gun: in a straight line. At a certain instant, the body splits into
two equal parts due to internal explosion, which releases 16
(a) IS kg (b) 30kg
joule of energy neither part leaves the original line of
(c) 1.5 kg (d) 20kg motion. Finally:
198 PHYSICS FOR COMPETITIONS - Vol. I

(a) Both parts continue to move in the same direction as 21. A bullet in motion hits and gets embedded in a solid block
that of original body resting on a frictionless table. What is conserved?
(b) One part comes to rest and the other moves in the same (a) Momentum and KE
direction as that of the original body , (b) Kinetic energy alone
(e) One part comes to rest and the other moves in the (c) Neither KE nor momentum
direction opposite to that of the original body
(d) Momentum alone
(d) One part moves in the same direction and the other in
22. A bullet of mass 'a' and velocity 'b' is fired into a large
the direction opposite to that of the original body
block of wood of mass 'c'. The final velocity of the system
16. A cannon haH is fired with a velocity 200 mls at an angle is: IEAMCET 1993]
60 0 with the horizontal. At the highest point of its flight it
cb b
explodes into 3 equal fragments, one going vertically (a) (b) -(a+b)
(a+b) c
upwards with a velocity 100 mis, the second one falling
vertically downwards with a velocity 100 mls. The third ab b
(e) (d) - (a + c)
fragment wHi be moving with a velocity: (a+c) a
(a) 100 mls in the horizontal direction 23 . A bullet weighing 109 and moving at 300 mls strikes a 5 kg
(b) 300 mls in the horizontal direction block of ice and drops dead. The ice block is sitting on
(c) 300 mls in a direction making an angle of 60° with the frictionless level surface. The speed of the block, after the
horizontal collision, is:
(d) 200 mls in e direction making an angle of 60° with the (a) 6 em sec- l (b) 6 msec- l
horizontal (c) 60cmsec- 1 (d) 60 msec- l
17. A body of mass I kg initially at rest, explodes and breaks 24. A bag of mass Mhangs by a long thread and a bullet (mass
into three fragments of masses in the ratio 1: I: 3. The two m) comes horizontally with velocity v and gets caught in the
pieces of equal mass fly off perpendicular to each other bag. Then for the combined system (bag + bullet):
with a speed of 15 mls each. The speed of the heavier
(a) Momentum is mMvl(M +m)
fragment is: (CBSE 1992]
(b) KE is (l12)Mv 2
(a) Wi mI, (b) 45 mI,
(c) Momentumismv(M+m)IM
(e) 5 mI, (d) 15 mI,
2 2
(d) KEism v /2(M+m)
18. A body of mass M at rest explodes into three pieces, two of
which of mass (MI4) each are thrown ofT in perpendicular 25. A bullet of mass m hits a target of mass M hanging by a
directions with velocities of 3 mls and 4 mls respectively. string and gets embedded in it. If the block rises to a
The third piece will be thrown ofT with a velocity of: height h as a result of this collision, the velocity of the
(CPMT 19901 bullet before collision is:
(a) 1.5 mls (a) v ~.j2gh (b) v ~ .j2gh[I+(mIM)]
(e) 2.5 mI, (e) v~.j2gh[(l+Mlm)] (d) v ~.j2gh[I - (ml M) ]
19. When two bodies collide elastically, then:
26. A body of mass m moving with a constant velocity v hits
(a) KE of the system alone is conserved another body of the same mass at rest and sticks to it. The
(b) Only momentum is conserved velocity of the compound body after collision is:
(c) Both KE and momentum are conserved (a) v/ 2 (b) 2v
(d) Neither KE nor momentum is conserved (c) v (d) zero
20. Which one ofthe following statement is true?
27. A block of mass 2M, moving with constant velocity 3~
(a) Momentum is conserved in elastic collisions but not in
collides with another block of mass Mwhich is at rest and
inelastic collision
stick to it. The velocity of the compound block after the
(b) Total KE is conserved in elastic collisions but collision is:
momentum is not
(a) 3 ~
~
(c) Total KE is not conserved but momentum 'is conserved (b) v
in inelastic collision
(c) 2 ~ (d) 3 ~12
(d) KE and momentum both are conserved in all types of
collisions


199

28. Two balls each of mass 0.25 kg arc moving towards each 35. Six identical balls are lined up along a straight frictionless
other in a straight line, one at 3 mls the other at 1 mls grove. Two similar balls moving with speed v along the
collide. The balls stick together after the collision. The grove collide with this row on extreme left side end. Then:
magnitude of the final velocity of the combined mass is: (a) One ball from the right end will move on with speed
(a) 4 mI, (b) 2 mI, 2v, all the other remains at rest
(e) 1 mI, (d) (112) mI, (b) Two balls from the extreme right will move on with
29. A body of mass 2 kg moving with a velocity of 3 mls speed veach and the remaining balls will be at rest
collides head on with a body of mass 1 kg moving with a (c) All the balls will start moving to right with speed v I 8
velocity of 4 mls. After collision the two bodies stick each
together and move with a common velocity: (d) All the six balls originally at rest will move on with
(a) (1/4) mI, (b) (113) mI, speed v I 6 each and the two incident ball will come to
(e) (213) mI, (d) (314) mI, rest
30. A body of mass m moving with a constant velocity v hits 36. Two perfectly elastic particles A and B of equal masses
another body of the same mass moving with same velocity u travelling along the line joining them with velocity 15 mls
but in the opposite direction, and sticks to it. The velocity of and 10 mls respectively collide. Their velocities after the
the compound body after collision is: elastic collision will be (in mls) respectively: ICBSE 1994)
(a) v (b) 2v (a) Oand25 (b) 5and20
(c) zero (d) vl2 (c) lOandl5 (d) 20 and 5
~

31. Two bodies having masses m] and m2 and velocities It] and 37. A ball of mass m moving with a velocity v collides head on
elastically with another of the same mass m but moving
~
u2 collide and form a composite system of ~
with a velocity - v. After collision: IEAMCET 1992]
~ ~
m j u]+m 2 u 2 =O(m l ",m 2 )· The velocity of the (a) The velocities are exchanged between the balls
composite system is: (b) Both the balls come to rest
(c) Both of them move at right angles to original direction
(a) 0 of motion
(d) One ball comes to rest and the other ball travels back
with velocity 2v
32. Two solid rubber balls A and B having masses 200 g and 38. A massive ball moving with speed v collides with a tiny ball
400 g respectively are moving in opposite directions with having a mass very much smaller than the mass of the first
velocity of A equal to 0.3 mls. After collision the two balls ball. The collision is elastic, then immediately after the
come to rest when the velocity of B is: impact, the second ball will move with a speed approximately
(a) 0.15 mI, (b) 1.5 mI, equal to:
(c) -0.15 mls (d) None ofthese (a) v (b) 2v
33. A particle of mass m moving east-ward with a speed V (c) vl2 (d) "'
collides with another particle of the same mass moving 39. A mass mj moves with a great velocity. It strikes anothel
north-ward with the same speed V. The two particles mass m2 at rest in a head on collision. It comes back along
coalesce on collision. The new particle of mass 2m will its path with low speed after collision. Then:
move in the north-east direction with a velocity:
(a) m] >m 2
(a) V (b) VI2 (b) mj <m 2
(e) VI.fi (d) v.fi (c) mj = m 2
~
34. A steel ball moving with velocity v collides with an (d) There is no relation between m] and m2
identical ball originally at rest. The velocity of the first ball 40. Fast neutrons can easily be slowed down by: (lIT 1994(
after the collision is: (a) The use oflead shield
~ ~
(b) Passing them through water
(a) v (b) - v
~
(c) Elastic collision with heavy nucleus
(e) (- V2) v (d) zero (d) Applying a strong electric field
41. A bal! is dropped on the ground from a height of 1 m. The 48. Two particles of masses M and 2M are at a distance D apart.
coefficient of res lit uti on is 0.6. The height to which the ball Under their mutual gravitational force they start moving
will rebound is: (CPMT 1993) towards each other. The acceleration of their centre of mass
(a) 0.6 m . (b) 0.4 m when they areDn. apart is: ICPMT 1993J
(c) 0.36 m (d) 0.16m (.) 20M/D' (b) 40M/D'
42. A sphere of mass In moving with a constant velocity u hits (c) 80M/D' (d) zero
another stationary sphere of the same mass. If e is the
coefficient of restitution, then ratio of velocities of the two 49. A bomb travelling in a parabolic path under the effect of
spheres after collision will be: gravity, explodes in mid air. The centre of mass of
fragments will: IEAMCET 1993J
(.) ( :::) (b) ( :~:) (a) Move vertically upwards and then downwards
(b) Move vertically downwards
(c) (:~:) (d) (:::) (c) Move in irregular path
(d) Move in the parabolic path the unexploded bomb
[Hint: See wIved Problem 9]
would have travelled
43. The centre of mass of a body:
50. TwQ heavenly bodiesS l andS 2 not far from each other are
(a) Lies always at the geometrical centre seen revolving in orbits:
(b) Lies always inside the body
(a) Around their common centre of mass
(e) Lies always outside the body (b) Which are arbitrary
(d) May lie within or outside the body
(c) With S 1 fixed and S 2 moving round S I
44. A system consists of mass M and m« M). The centre of (d) WithS 2 fixedandS , movingroundS 2
mass of the system is:
5 1. The variation of momentum with time of
(a) AI the middle
one of the body in a two body collision
tI ;,:s
P~
(b) Nearer to M is shown in Fig. 5.67. The instantaneous
(e) Nearer to III force is maximum corresponding to
(d) At the position of large mass point:
1-
45. Two skaters A and B of masses 50 kg and 70 kg respectively (a) P Fig. 5.67
stand facin g each other 6 m apart. Then they pull on a rope (b) Q
stretched between them. How far has each moved when
(c) R
they meet?
(a) Both have moved 3 m (d) S
(b) A moves 2.5 m and B 2.5 m 52. An object of mass m moving with a velocity v is
approaching a second object of same mass but at resc. The
(c) A moves 3.5 m and B 2.5 m
total KE of the two objects as viewed from the centre of
(d) Amoves2mandB4m mass is:
46. Two carts on horizontal straight rails are pushed apart by an
explosion of 1\ powder charge Q placed between the carts. (b) .!mv 2
2
Suppose the coefficient of friction between carts and rails
are identical. If the 200 kg cart travels a distance of36 m (d) ..!. mv 2
and stops, the distance covered by the cart weighing 300 kg 8
;" IEAMCET 19951 53. Consider the head on elastic collision of a neutron moving
(a) 32 m (b) 24 m respe.-;tively through a sample of lead, carbon, paraffin and
(c) 16 m (d) 12 m tin . Which one of the above sample is most efficient in
47. A man weighing 80 kg is standing on a trolley weighing slowing down the neutron?
320 kg. The trolley is resting on frictionless horizontal rails. (a) Lead (b) Carbon
If the man starts walking on the trolley along the rails at (e) Tin (d) Paraffin
speed 1 mfs, then after 4 s his displacement relative to the 54. The coefficient of restitution (e) for a perfectly elastic
ground will be: collision is:
(a) 5 m (b) 4. 8 m (,) (b) zero
(c) 3.2 m (d) 3.0 m (d) -I
(c) '"
55. A cart of mass M is tied to one end of a massless rope of
length 10 m. The other end of the rope is in the hands of a
man of mass M. the entire system is on a smooth horizontal
surface. The man is atx=Oand the cart atx=10m. If the
man pulls the cart by a rope, the man and the cart will meet
at the point: leDSE 19971
(a) x=O (b) x=lm 62. The distance of a man standing in a boat is 20 metre from
(c) x=lOm (d) They will never meet the shore. If he walks 10 metre towards the shore, find the
56. If a ball moving with velocity u overtakes another ball of distance from the shore at the moment he finishes walking.
twice its own mass moving with one-seventh of its velocity The weight of the man is 75 kg and the weight of the boat is
125 kg.
and if the coefficient of restitution between them is ~, the (0) 3.71 m (b) 13.75 m (0) 6.25 m (d) 10 m
4
velocity of the first ball after striking the second ball is: 63. A body of mass 3 kg is acted on by a force which varies as
(a) u/2 (b) Zero shown in the graph. The momentum acquired is given by:
(0) u/ 4 (d) 3u/ 4
57. Two equal balls are in contact on a table and 8re in
equilibrium. A third equal ball collides with them
I 10

simultaneously, symmetrically and remains at rest after ForcelN


impact. The coefficient of restitution is:
(0)2 (b)~ o 2 4 6
3 2 Tlme/s -
Fig. 5.68
(0) 1 . (d) 1
3 2 (a) Zero (b) 5 N-s
58. A 3 kg bomb explodes into 3 equal pieces A , Band C. A flies (0) 30 N-s (d) 50 N-s
with a speed of40 mls andB with a speed of30 mls making 64. Two different bodies ofmassesM I andM z are dropped from
an angle of900 with the direction of A. The angle made by same height. The ratio of their momenta at the ground is:
the direction of C with that of A is:
cos-I(~) 1t-COS-I(~) (a) Mi (b) (M' )"2
(a) (b) Mi Mz

(c) cos-1GJ (0) M, (d) M,


Mz M\
59. On a smooth plane surface, making an angle of 60° with the 65. Particles x (of mass 4 kg) andy(ofmass 9 kg) move directly
horizontal, marbles, each of mass 2 g, fall at the rate of 40 towards each other, collide and then separate. If .6t1x is the
per second. If the height from which they fall is 20 m and change in the velocity of x and l1/ly is the change in the
the collisions are elastic, the force on the surface (taking 6v x
g=10m/s z )is: velocity ofy, the magnitude of the ratio- - is :
6v y
(0) 2.24 N (b) Zero
(0) 3.2 N (d) 1.6 N (a)
4
2 (b) ~
60. A ball impinges directly on a similar ball at rest. The first
(d) ~
2
ball is brought to rest by the impact. If half the kinetic (0) -
3 9
energy is lost by impacl, what is the value of the coefficient
of restitution? 66. A bomb bursts into three fragments. Two fragments of
1 equal mass move with equal speed ofv' at right angles to
(0) - (b) _I each other, while the third fragment, having mass twice that
2.Ji J3 of either of the small fragments, moves:
1
(0) - (d) J3 (a) With a speed of ~ at an angle of 1350 to either
.Ji 2
61 . Two loads PI and Pz ( PI > Pz ) are connected by a string (b) With a speed of-fiv' at an angle of 1350 to either
passing over a fixed pulley. The centre of gravity of loads
are initially at same height. Find the acceleration of the (c) With a speed of2v' at an angle of 1350 to either
centre of gravity of the system: (d) With a speed of -fiv' at an angle of 45° to either
~"==-\L==i"
67. Fig. 5.69 shows a rectangular plate of i- 73. A small disc of mass mslides down a smooth hill of height }
length L, the half of which is made of from rest and gets onto a plank of mass M lying on tht
material of density d 1 and another
o d, horizontal plane at the hilL Due to friction between the disc
half of density d 2 . The distance of L _ _-'-_ _...J and the plank the disc slows down and after a certair
moment, moves in one piece with the plank. Then the wod
centre of mass from the origin 0 is: FIg. 5.69
perfonned by the friction force in this process is (lgnon
friction between plank and plane) :
(d,+2d,) Cd J +3d z )L m
(a) L (b)
2(d, +d2 ) 4(d l +d z )
(d, +3d2 )L (3d l +dz) L
(e) (d)
2(d J +d2 ) 4(d l +d z ) h

68. The masses of five balls at rest in a straight line are in


geometrical progression with ratio 2 and their coefficients
of restitution are each 2/3. If the first ball be started towards
the second with velocity u, then the velocity communicated Fig. 5.71
to 5th ball is:
(a) Zero mM
(b) - - gh
(a) (5/9)u (b) (5/9)' u m+M
M
(e) (5/9)3 u (d) (5/9)4 u (e) -gh (d) -mM
- gh
m M--m
69. A projectile is moving at 20 mls at its highest point, where it
breaks into two equal parts due to an internal explosion. 74. A small block of mass m is released from rest from point l
and slides down DGF and reaches the point F with spee(
One part moves vertically up at 30 mls with respect to the
v F _ The coefficient of kinetic friction between block an<
ground. Then the other part will move at: ICET 1.0021
both the surface DG and GF is)l, the velocity v F is :
(a) 20 mls (b) 10m mls
(e) 50 mls (d) 30 mls
70. A car of mass 400 kg and travelling at 72 kmlhr crashes into
f m S,

a truck of mass 4000 kg and travelling at 9 kmlhr in the y G


same direction. The car bounces back at a speed of 18 S,
kmlhr. The speed of the truck after the impact is:
j
(a) 9 kmlhr (b) 18 kmlhr
(EAMCET 19981
~~=='='~~::~x~x~,::~Y~F
Fig. 5.72
(c) 27 kmlhr (d) 36 kmlhr
71. The end ofunifonn rope of mass m and length L that is piled (a) )2g(y~x) (b) )2g(y ,.x)
on a platfonn is lifted vertically with a constant velocity v
by a variable force F. The value ofF as a lifted position x of
(e) )2gy (d) ~2g(y' +x')
the rope is: 75 . A rocket of initial mass 5000 kg ejects gas at a constant ratt
of60 kg/s with a relative speed of2050 m/s. Acceleration 0
(a) !!!. (gx+v2) (b) m(gx+v2) the rocket 15 second after it is blasted off from the surfacl
L
of earth will be: (g = 10m/s 2 ):
(c) ~ (gx 2 +xv) (d) None of these
(a) IOmffi' (b) 20mls'
L
72. All surfaces are frictionless. The (e) 30mls' (d) 40mls'
speed of ball just before striking is 24 76. There are two events as shown in figure. In first even
mis, the coefficient of restitution e = sphere 1 kg collides head on elastically with sphere 2 kg a
0.8. The velocity of ball just after t = (1 In second event sphere 2 kg collides head 01
collision is: elastically with sphere 4 kg at t = O.
(a) 18m1s v=4 m/s v=O v=4 m/s v=2m/s

(b) 12.2 mls


(c) 17.2m/s
(d) None of the above
Fig. 5.70

8=8 8=8 Evenl1


Fig. 5.73
Evenl2
Force-time variation during collision may be represented as: 80. A block ' A ' of mass 2m is placed on another block' B' of
F F mass 4m which in tum is placed on a fixed table. The two
Event 2 blocks have the same length 4d and they are placed as
Evenl1 shown in figure. The coefficient offliction (both static and
kinetic) between block 'B' and table is ~. There is no
(.) (b)
friction between the two blocks. A small object of mass m
moving horizontally along a line passing through the centre
of mass (eM) of block B and perpendicular to its face with a
speed vcollides elastically with block B at a height d above
F the table. What is the minimum value ofvrequired to make
the block A topple?
,-----,
A 2m
(0) (d)

77. A small ball falling vertically downward with constant


o--'--r- __________ ~_ 4'" 1d

velocity 4 mls strikes elastically a massive inclined cart P----4d


moving with velocity 4 mls horizontally as shown. The Fig. 5.76
speed of the rebound of the ball is:
o (.) ~6"gd (b) ~3 "gd
14 mls (0) ~ ~3"gd (d) ~~6"gd
2 2
8 1. Two blocks A and B of masses m and 2m, respectively are
4 m/s connected by a spring of force constant k. The masses are
45'
moving to the right with unifonn velocity veach, the
Fig. 5.74 heavier mass leading the lighter one. The spring is in the
natural length during this motion. Block B collides head on
(.) 442 mI, (b) 4fj mI, with a third block C of mass nI, at rest, the collision being
completely inelastic. Calculate the maximum compression
(0) 4 mI, (d) 4.Js mI,
of the spring.
78. Two masses M and m are tied with a m 2m m
string and arranged as shown. The
velocity of block M when it losses the
contact is : ~ Fig. 5.77
(.) 2./ih (b)
(m+M) M _I ~3mv2 (b) ~mv2
Fig. 5.75 (,) 2 k
(0) 2m.Jgh (d) 2MJih 12k
(m+M) (m +M)
(c) ~3mv 2 (d) )mv2
79. A particle moving horizontally with a velocity u strikes a 4k 4k
fi xed frictionless sphere at a height !i above the centre of 82. Two identical discs initially at rest are in contact on a table.
2 A third disc of same mass but of double radius strikes them
sphere. After striking the sphere, velocity of particle symmetrically and itself comes to rest after impact. The
changes to vertically upward. If R be the radius of sphere, coefficient of restitution is :
the maximum height attained by the particle with respect to 9 1
centre of sphere is : (.) - (b) .3 (0) - (d) J.
16 4 2 16
.2 R 2
(.) -+ - (b) ~ + !!. 83. There are two masses M I and M 2 placed at a distance I
6g 2 2g 2 apart, let the centre of mass of this system is at a point
.2 R .2 R named C. If MI is di splaced II towards C and M2 is
(0) -+ -
3g 2
(d) -+ -
4g 2
'2
displaced by away from C. The distance from C where
the new centre of mass will be located is:
M 2'] +M]IZ MIl1 +M2i2
(a) (b)
M1+M2 M]+M2 1m M
M ]I]-M2'" M21] -M 1/ 2
(0) (d)
M1+M2 M] + M z
84. All the particles of a body are situated at a distance R from
the origin. The distance of the centre of mass of the body '----( ~
from the origin is : Fig. 5.80
(a) =R (b) <R
ML mL
(0) >R (d) >R (a) (b)
(M + m) (M+m)
85. In a vertical plane inside a smooth h91low thin tube a block
ML mL
of same mass as that of tube is released as shown in figure. (0) (d)
When it is slightly disturbed it moves towards right. By the m M
time the block reaches the right of the tube then the 88. If the density of material of square plate and a circular plat
displacement of the tube will be [R is the mean radius of shown in figure is same the centre of mass of the composit
tube. Assume that the tube remains in vertical plane] ; system will be :

Fig. 5.78 , .
(a) 2R/. (b) 4R/. Fig. 5.81
(0) R/ 2 (d) R
(a) Inside the square plate
86. A solid sphere consist of mass M has radius R and a small
(b) Inside the circular plate
spherical cavity of radius ( ~) is created from surface of (c) At the point of contact
sphere as shown in figure. The position of the centre of (d) Outside the system
mass of the remaining mass of the sphere from 0 point is: 89. A man weighing 80 kg is standing on a trolley weighin,
('0' is centre of 320 kg. The trolley is resting on frictionless horizontal rail!
sphere) If the man starts walking on the trolley along the rails ~
constant speed 1 mls. then after 4 · s his displacemer
relative to the ground will be :
(a) Sm (b) 4.8m
(0) 3.2 m (d) 3.0 m
90. The hero of a stunt film fires 50 g bullets from a machin
Fig. 5.79
gun, each at a speed of 1000 mls. If it fires 20 bullets in
(a) at ( ~ ) along leftward from centre of sphere seconds, what average force does he exert against th
machine gun during this period?

(b) at (:6) along leftward from centre of sphere


(a) 150 N
(0) 250 N
(b) 350 N
(d) 450 N

(c) at( :8) along rightward from centre ofsphere


~ ~
91 . Atoms of 02 are at positions r I and r 2' Position of thei
centre of mass can be expressed as:
(d) at centre of solid sphere ~ ~ ~ ~

87. A cart of mass M is at rest on a frictionless horizontal r 1+ r 2 r 1 -r 2


(a) (b)
surface and a pendulum bob of mass m hangs from the roof 2 2
of the cart as shown in the figure. The string breaks. the bob ~ ~

falls on the floor. makes several collisions on the floor and r2- r1 ~ ~
(0) (d) r 2 -r 1
distance between the string and the slot is L Find the 2
displacement of cart during this process.
92. Particles of masses 1 g. 2 g and 3 gare located at the comers 98. Assuming uniform mass distribution, distance of centre of
of an equilateral triangle of side' a'. Coordinates of centre mass from 0 is nearly:
of mass in em with respect to the particle of mass 1 g are: 1---10m ·1

(a) (.J3a,~,o) b (Nla..{la )


() 12'12,a I 11 2m

4m 4m
7a NJa 0) (d) (0,0,0)
(c) ( 12'12' 10m
93. Linear mass density of a rod of length' /' kept along ;x-axis
varies as, A. = a + bx.
Centre of mass of the rod is at x =
1(3a + lbl) (b) 1(20 + 3bl)
1 H
0

1m
7T1 m 11 2m

(a) 2 2 Fig, 5,85


(c) I(Ja+lbl) (d) I(Ja-lb/) (a) 6 m (b) 4m (c) 2m (d) 9.2 m
3(2o +bl ) 21a +b 99. Net external force acting on a system of particles is zero and
94. Mass is distributed unifonnly over a rectangular sheet such the internal forces obey the law of action and reaction.
that end points of a diagonal can be described by the Which of the following is not correct?
coordinates (3, 0) and (-3, 8). Coordinates of centre of mass (a) Position of centre of mass of system cannot change
ofthe sheet are: (b) Velocity of centre of mass of system cannot change
(a) (3,3) (b) (3, 4) (c) Linear momentum of system cannot change
(c) (4,3) (d) (0,4) (d) Kinetic energy of system cannot change
B
95. In Fig. S.82, density of material of sphere 100. Masses ml and ml are held a distance 'd' apart. Distance of
is 12 times that of cone. Centre of mass of centre of mass from mass ml is:
the system is at point: -----T (a) _ m-"_ d (b) _ m,,-,_ d
(a) A
4R
B
C'~A 1
(b)
(c) C (c) m l - ml d (d) m2 d
(d) D 1--4R--l ml + m2 "'I
Fig. 5,82 10 1. In Q. 100, If the masses are released and move towards each
other under mutual gravitational force and given that

-9--.'
96. Fig. 5.83 shows a disc of radius
12 em with mass distributed
ml >"'2 ' then:
unifonnly over it From this disc, (a) centre of mass begins to shift towards "'1
a disc of diam~ler 12 em is (b) centre of mass begins to shift towards "'2
removed. With 0 as the origin (c) position of centre of mass becomes undefined
and x-axis as shown, position Fig, 5,83
(d) position of centre of mass remains unchanged
vector of centre of mass of the
102. Particles of masses 12 kg al!d 6 kg are released from a
remaining part can be expressed as: separation 60 m and move towards each other under the
(a) 41 (b) -41 mutual gravitational force. They will hit each either at a
(c) 21 (d) -2 1 distance of:
(a) 20 m from the initial position of 12 kg
97. In Fig. 5.84, a spherical part of radius ~ is (b) 20 m from the initial position of6 kg
2
removed from a bigger solid sphere of (c) 30 m from the initial position of either mass
radius R. Assuming unifonn mass (d) none of the above
distribution, shift in the centre of mass will 103. Two blocks of masses 10 kg and 6 kg are connected by a
be, Fig. 5,84 spring 'and placed on a frictionless horizontal surface. A
hanuner strikes the block of mass 10 kg as a result of which
(a) !I. (b) R
this block is given a velocity 3.2 mls towards the lighter
7 i4
R block -immediately on being struck. Velocity imparted to
(c) (d) R the centre of mass of their system is:
9 3
(a) zero (b) 3 mls (c) 2 mls (d) 1.4 mls
104. In Q. 103. velocities ofthe blocks of masses 10 kg and 6 kg R 3R
(a) R ,2R (b)
with respe<:t to centre of mass, immediately whc:n the 2'2
hammer strikes the larger mass are:
(.) 2 mis, !.2 mls (b) 3.2 mls, - 2 mls (e) %,R (d) 2R,R

(e) !.2m1s, - 1.2m1s (d) 1.2m1s,-2m1s 111. A 400 kg boat is 6 m long. It is floating in still water and is
105. A man of mass 'm' climbs a rope of length I suspended at rest. A person of mass 80 kg moves from one end of the
below a balloon of mass 'M'. The balloon is stationary with boat to the other. Displacement of the boat relative to water
respect to ground. If the man begins to climb up the rope at is:
a speed ure ! relative to the rope, then the velocity with which
~
(a) 1min a direction opposite to the displacement of person
the balloon moves relative to the ground is v equal to: (b) 0.5 m in a direction opposite to the displacement of
m ~ m
--v
~ person
(a) - vreJ (b)
M M +m reI (e) 1.5 m in a direction opposite to the displacement of
m ~ M~ person
(e) - - - v J (d) - vrel
M+m re m (d) 1.5 m in the direction of displacement of the person
106. In Q. 105, when the OIBnreaches the balloon by climbing 112. A person ofmftSs 80 kg is standing on a boat of mass 320 kg
the rope, the balloon in this duration has descended by: which is at rest in still water. The person is initially 12 m
from the shore. The person walks on boat for 6 sec with a
(a) ~I (b) ~, constant speed I mfs towards the shore and then stops.
M +m M+m
Distance o f the person from the shore is finally:
(e) M I (d) "'-I
m M (a) 4.8 m (b) 6.4 m
107. Two objects of masses 8 kg and 6 kg are released from a (e) 7.2 m (d) 8.2 m
separation 10m and move towards each other under mutual 113. The masses ml and m2 with ml < m2 are connected to the
gravitational force. Acceleration of the centre of mass of ends of a compressed spring. When the masses are released
their system when they are 3 m apart is: they move away with:
(a) 4.8 mis' (b) 3.2 mis' (a) equal speeds (b) equal momenta
(c) 1.4 mls 2 (d) zero (c) equal accelerations (d) equal kinetic energies
108. Two objects initially some distance apart are reJeased from 114. An object explodes into many pieces which fly off in
rest. They move towards each other under mutual different directions. The quantity which is conserved is:
gravitational force. At some instant, speed of one of them is (a) linear momentum
u and that of other is u14. Then speed of their centre of mass
at this instant is: (b) kinetic energy
v (c) both linear momentum and kinetic energy
(a) 3v (b) 4
4 (d) none of the above
Sv liS. A 10 kg shell, initially at rest, explodes into three fragment!
(e) 4 (d) zero of masses 4 kg, 4 kg and 2 kg. If the two fragments ofequa:
mass move towards north and towards west with eqaa
109. An object of mass '2m' is projected with a velocity u at
speeds 4 mis, velocity of the 2 kg fragment will be:
e
angle with the horizontal. At the highest point of its path,
it explodes into two fragments of equal mass such that (a) 8 m/s along north-west
speed of one of the fragments immediately after explosion (b) 8f .f2 mls along south-west
is zero. Distance from the point of projection at which the (c) 8 mls along north-east
centre of mass hits the horizontal is:
(d) 8f .f2 mls along south-east
v 2 sin e v 2 sin 9
(a)
g
(b) ,,--;;=
"lg
116. An isolated particle of mass m is moving in a horizonta
plane (x - y), along the x-axis, at a certain height above the
2 ground. It suddenly explodes into two fragments of masse:
v sin 29 vsin9
(e) "--"= (d) ml4 and 3m14. An instant later, the smaller fragment is a
g "lg
y = + IS cm. The larger fragment at this instant is at:
v 2 sin 29 . lilT 1997
110. In Q. 109, if we write R , ,-~~~,then the distances
g (a) y =-5 cm (b) y=+20cm
from the point of projection at which the two fragment hit
(c) y=+5cm (d) y~ -20em
ground are:
117. An empty box is put on the pan of a physical balance and 3v 4v
(a) (b)
the scale is adjusted to zero by counterpoising. A stream of acos8 acos9
small identical beads each of mass m are then dropped into 5v
the box from a height h at a constant rate of il beads per (e) (d) 2v
acos 9 acos 8
second. If the collision between the beads and the box is
\22. Two masses m and M(> m) are joined by a
completely inelastic, find the reading of the scale, I second
after the beads begin to fill the box: light string passing over a smooth light
pulley. The centre of mass of the system
f2ii:
(a) nmV-g1 (b) IIm t
g moves with an acceleration:
h
(a) g(MM+m
-m)downward
(d) nmght

g (- - )' downward
(b)
M-m
11 8. A pendulum consists of a wooden bob of mass mand length I. M+III
Fig. 5,87
A bullet of mass m] is fired towards the pendulum with a
velocity vI' The bullet comes out of the bob with speed ~
(c) g(MM+m
_m)2 upward if M<m
3
and the bob just completes motion along a vertical circle. (d) zero
The velocity VI is: 123. A uniform solid right circular cone of
(a) ~(-",- I rsg;
2 m )-'5'
(b) -"'-
m[
.J5ii base radius ris joined to a uniform solid
~emisphere of radius r and of the same
l
density, so as to have a common face.

(e) ~(,~ ~ (d) M~ ).J5ii The centre of mass of the composite


solid lies on the common face. The
height of the cone is:
119. A block of mass 2 kg slides along a frictionless table with a
speed of 10 mlsec. Directly in front of it and moving in the (a) 2r (b) 4r
same direction is a block of mass 5 kg moving at 3 rnIsec. A (e) 3r (d) ,,j(, Fig. 5,88
massless spring of spring constant k'" 1120 N/m is attached 124. Two particles of masses tnl and /11
2 in projectile motion
to the back side of 5 kg mass as shown in figure . When the ~ ~

blocks collide the maximum compression in the spring (if have velocities VI and VI respectively at time t = 0. They
the spring does not bend) will be:

- VI
-V2 collide at time to' Their velocities become
time 2t 0 while still moving in air. The value of
~ and ~ at

Ds=D Fig, 5.86


1(1111

(a) zero
... .... ........
vI +m2 VI )-(m l VI +1112 v1 )Iis: (IIT2001)

(a) 0.25 m (b) 0.4 m (c) 2(m! +ml )gt o


(e) 0.33 m (d) 1.12 m
120. The centre of mass ofa non-uniform rod of length Lwhose J25. A block of mass M lying on a smooth horizontal surface is
. k·x 2 . rigidly attached to a light horizontal spring of force
mass per unit length p vanes as p = - - where k IS a constantk. The other end of the spring is rigidly connected
L
to a fixed wall. A stationary gun fires bullets of mass In each
constant and xis the distance of any point from one end is:
in horizontal direction with speed Vo one after other. The
3 I
(a) ; / (b) ;;L bullets hit the block and get embedded in it. The first bullet
hits the block at t :::: 0. The second bullet hits at
(e) Z (d) ~ t = 21t~M+m . at t = 21t~M+m
. buUet hIts
-k-' the thud -k- +
121. A hemisphere of mass 4 m is free to slide with its base or. a
smooth horizontal table. A panicle of mass mis placed on the 21t~M +k 2m an d so on. The maXImum
' . .m Ihe
compressIOn
top of the hemisphere. The hemisphere has now acquired a
velocity v. Then the angular velocity of the particle relative spring after the n th bul1ct hits is:
10 the hemisphere at an angular distance 8 is !
131. Consider a two particle system with panicles having
massesml and m2' If the first particle is pushed towards the
centre of mass through a distance d, by what distance
should_the second particle be moved so as to keep the centre
Fig. 5.89 of mass at the same position? IAIEEE 2006)
nmvoJk (M +nm)3/2
(a) ~d (b) d
(a)
(M + nm)3/2
(b)
nmvo.fk m,
~nmvok (d) nmvo (c) m1 d (d) _m-'.,_ d
(c)
(M +nm)3/2 ~k(M +nm) "',
132. A ball which is at rest is dropped from a height h metre. As
126. The block of mass m moving on the
it bounces off the floor its speed is 80% of what it was just
frictionless horizontal surface collides
. before touching the ground. The ball will then rise to nearly
with the spring of spring constant k
and compresses by length L. The Fig. 5.90 a height: (West Bengal JEE 2007J
maximum momentum of the block (a) 0.94 h (h) 0.80 h
after collision is: [AIREE 20051 (c) 0.75 h (d) 0.64 h
ML' 133. Three identical spheres, each of mass 1 kg are kept as
(a) . Zero (b) R shown in fi gure below, touching each other, with their
centres on a straight line. If their centres are marked P, Q.R
kL'
(c) LJMk (d) 2M respectively, the distance of centre of mass of the system
from P is: (West Bengal JEE 2007)
127. A bomb at rest explodes into 3 parts oflhe same mass. The Y
1
momentum oflhe 2 parts are-2p i and p The momentum
of the third part will have a magnitude of:
-~-+--~--~- x
ISEE (UPTU) 20051
(a) p (b) ,fJp P Q R

(c) p.Js (d) zero


Fig. 5.91
128. Which of the following is not an inelastic collision? PQ+PR+QR PQ+PR
IBHU200S] (a) (b)
3 3
(a) A man jumps on a cart PQ+QR PR+QR
(c) (d)
(b) A bullet embedded in a block 3 3
(e) Collision of two glass balls 134. A circular disc of radius R is removed from a bigger circular
Cd) Noneoflhe above disc of radius 2R, such that the circumference of the discs
129. A body A of mass M while falling vertically downwards coincide. The centre of mass of the new disc is aR from the
under gravity breaks into two parts, a body B of mass MI3 centre of the bigger disc. The value of Cl is:
and body C of mass 213 M. The centre of mass of bodies B IUPSEE 2007, AIEEE 2007]
and C taken together shifts compared to that of body A I
towards: IAIEEE 20051 (a) 3 (b) I
2
(a) BodyC I
(b) Body B (c) '6 (d)
4
(c). Depends on height of breaking 135. A particle of mass m. strikes on ground with angle 0 1
(d) Does not shift incidence 45°. If coefficient of restitution e::: 11..fi, the
130. A bomb of mass t 6 kg at rest explodes into two pieces of velocityofrefleclion is: (BHU (Mains) 20071
masses 4 kg and 12 kg. The velocity of the 12 kg mass is
,fJ
4 ms -I. The kinetic energy of the other mass is: (a) - v (b) ,fJv
2
IAIEEE 2006] I
(c) - v (d) -"-
(a) 192J (b) 96 J 2 ,fJ
(c) 144J (d) 288 J
136. For inelastic collision between two spherical rigid bodies: (a) Only momentum is conserved
(RPMT2007( (b) Only kinetic energy is conserved
(a) the total kinetic energy is conserved (c) Neither momentum nor kinetic energy is conserved
(b) the linear momentum is not conserved (d) Both momentum and kinetic energy are conserved
(c) the total mechanical energy is not conserved 142. A rocket of mass 6000 kg is fired from earth with speed I
(d) the linear momentum is conserved km/s, What should be the rate of ejection of gas, so that it
137. A shell of mass 20 kg at rest explodes into two fragments moves with initial acceleration 20 m/ s2?
whose masses are in the ratio 2 : 3. The smaller fragment (Takeg = 10m/ s2) [Orissa JEE 2009)
moves with a velocity of6 ms - I, The kinetic energy of the (a) 60kgls (b) 180kgls
larger fragment is: [Karnataka CET 2007) (e) 200 kgls (d) 120 kg/s
(.) 961 (b) 2161 (e) 1441 (d) 3601 143. A bullet of mass 109 moving with 300 mls hits a block of
138. A small disc of radius 2 em is cut fromadisc of radius 6 em. ice of mass 5 kg and drops dead, The velocity of ice is:
If the distance between their centres is 3.2 em, what is the (Orissa JEE 2009)
shift in the centre of mass of the disc? IJCECE 2007]
(a) 50 cmls (b) 60 cmls
(a) 0.4 em (b) 2.4 em
(c) 40 cmls (d) 30 emls
(c) 1.8 em (d) 1.2 em
144. A thin unifonn rod oflength 1and mass m is swinging freely
139. A body of mass 2m is split into two equal parts by an about a horizontal axis passing through its end, Its
internal explosion which generates a kinetic energy E, If, maximum angular speed is (0, Its centre of mass rises to a
after the explosion, the parts move in the same line as
maximum height of: [AIEEE 2009)
before, then what is theirrdative speed? [UPSC 2009)
I 12 (02 I leo
(a) .JElm (b) .J2Elm (a) - - (b)
3 g 6g
(e) .J4Elm (d) 0
140, A particle moves in the x-y plane under the influence of a
force such that its linear momentum is
~ " " 145. A bullet fired into a wooden block loses half of its velocity
p(t):=: A [I eos(kt) - j sin (kt)]
after penetrating 40 cm. It comes to rest after penetrating a
where A and k are constants. The angle between the force further distance of: [Kerala PET 20091
and momentum is: {AMU 2009] 22 40 20 22
(a) - cm (b) - cm (c) - cm (d) -cm
(a) 0° (b) 30° (e) W (d) 90° 3 3 3 5
141. A bullet hits and gets embedded in a solid block resting on a 26
frictionless surface. In this process which one of the (e) - em
5
following is correct? (DCE.2009(

ANSWERS

1. (b) 2. (a) 3. (b) 4. (a) 5. (e) 6. (b) 7. (e) 8. (d) 9. (c) 10. (d) 11. (c) 12. (b)
13. (a) 14. (c) 15. (b) 16. (b) 17. (a) 18. (e) 19. (c) 20. (c) 21. (d) 22. (c) 23. (e) 24. (d)
25. (e) 26. (a) 27. (e) 28. (e) 29. (c) 30. (c) 31. (a) 32. (c) 33. (c) 34. (d) 35. (b) 36. (c)
37. (a) 38. (b) 39. (b) 40. (b) 41. (c) 42. (a) 43. (d) 44. (b) 45. (e) 46. (e) 47. (c) 48. (d)
49. (d) SO. (a) 51. (e) 52. (e) 53. (d) 54. (a) 55. (b) 56. (b) 57. (a) 58. (b) 59. (d) 60. (e)
61. (e) 62. (b) 63. (b) 64. (e) 65. (a) 66. (a) 67. (b) 68. (d) 69. (e) 70. (b) 71. (a) 72. (b)
73. (b) 74. (b) 75. (b) 76. (e) 77. (d) 78. (e) 79. (a) 80. (d) 81. (b) 82. (a) 83. (b) 84. (b)
85. (c) 86. (a) 87. (b) 88. (b) 89. (e) 90. (e) 91. (a) 92. (c) 93. (e) 94. (d) 95. (d) 96. (d)
97. (b) 98. (a) 99. (d) 100. (b) 101. (d) 102. (a) 103. (e) 104. (d) 105. (e) 106. (a) 107. (d) 108. (d)
109. (e) 110 (b) 111. (a) 112. (c) 113. (b) 114. (a) 115. (d) 116. (a) 117. (e) 118. (a) 119. (a) 120. (a)
121. (e) 122. (b) 123. (b) 124. (e) 125. (d) 126. (e) 127. (e) 128. (e) 129. (d) 130. (d) 131. (a) 132. (d)
133. (b) 134. (a) 135. (a) 136. (d) 137. (a) 138. (a) 139. (c) 140. Cd) 141. (a) 142. (b) 143. (b) 144. (d)
145. (b)
[Bl More than One ,Choice is Correct 4 mls. If the collision is one dimensional and inelastic, the
velocity of composite mass after the collision is:
1. Internal forces can change:
(a) (3/2) mls (b) (2/3) mls
(a) Kinetic energy (b) Mechanical energy
(0) 4 mls (d) (10/3) mls
(e) Energy Cd) Momentum
10. Two blocks A and B each of mass m are connected by
2. In case of explosion of a bomb which of the following massless spring of natural length L and spring constant k.
changes? The blocks are initially resting on a smooth horizontal floor
(a) Kinetic energy (b) Mechanical energy with the spring at its natural length as shown in Fig. 5.92. A
(e) Chemical energy Cd) Energy third identical block C also of mass m moves on the floor
with a speed v along the line joining A and B, and collides
3. When two blocks connected by a spring move towards each with A then: [lIT 1993)
other under mutual interaction:
(a) Their velocities are equal and opposite
(b) Their accelerations are equal and opposite
(c) The force acting on them arc equal and opposite Fig. 5.92
::::::: B

Cd) Their momentum are equal and opposite (a) The KE of the AB system at maximum compression of
4. When a bullet is fired from a gun; the spring is zero
(a) Kinetic energy of bullet is more than that of gun (b) The KE of the AB system at maximum compression is
(b) Acceler~tion of bullet is more.than that of gUn (1I4)mv 2
(c) Momentum of bullet is more than that of gun (c) The maximum compression of spring is v-Jmlk
Cd) Velocity of bullet is more than that of gun (d) The maximum compression of spring is v-Jml2k:
5. A ball hits the floor and rebounds after an inelastic [Hint: See solved Problem 171
collision. In this case choose the correct alternative: II. Which oflhe frllow ing are not correct about centre of mass?
(a) The momentum of the ball just after the collision is the (a) It depends on frame of reference
same as that just before the collision (b) In centre of mass frame momentum of a system is
(b) The mechanical energy of the ball remains the same in always zero
the collision (c) Internal forces may affect the motion of centre of mass
(c) The total energy of the ball and earth is conserved (d) Centre of mass and centre of gravity are synonymous
(d) The total momentum of the ball and earth is conserved 12. If no external force acts on a system:
6. About a collision which is not correct? (a) Velocity of centre of mass remains constant
(a) Physical touch is a must (b) Velocity of centre of mass is not constant
(b) Particles cannot change (c) Velocity of centre of mass may be zero
(c) Effect of external force is not considered (d) Acceleration of centre of mass is zero
Cd) Momentum mayor may not change 13. Two balls shown in Fig. 5.93 are identical, the first moving
7. In inelastic collision kinetic energy after collision : with speed v towards right and the second staying at rest.
(a) Can be equal to before collision The wall at the extreme right is fixed and smooth.
Assuming all collisions to be elastic, which of the following
(b) Can be lesser than before collision
statements are correct?
(c) Can be greater than before collision
Cd) Any of the above -v
8. A body moving towards a body of finite mass at rest
collides with it. It is possible that:
(a) Both bodi~s. come to rest
(b) Both bodies move after collision Fig. 5.93
(0) The moving body stops and body at rest start moving (a) There are only three collisions
Cd) The stationary body remains stationary and the moving (b) The speed of first ball is reduced to zero finally after all
rebounds collisions
9. A body of mass 2 kg moving with a velocity of 3 mls (c) Only two collisions are possible
collides with a body of mass 1 kg moving with a velocity of
Cd) The speeds of balls remain unchanged after all
collisions have taken place
14. Two particles A and B of equal size but of masses mkg and (d) A anb B move with unequal speeds, making an angle of
2m kg 8re moving directly towards each other with speeds 30° and 600 with the x ~ax:is respectively
21 mls and 4 mls respectively. After collision, A moves 18. Choose the correct s.tatement(s) regarding inertial fra mes:
with a speed·of 1 mls in the original direction, then : (a) Impulse is independent of frame of reference
(a) After collision B moves with a speed of 6 mls in a
(b) Momentum depends upon frame of reference
direCtion opposite to its motion before collision
(e) Work done by a force depends on frame of reference
(b) After collision B moves with a speed of 6 mJs in the
original direction (d) Change in kinetic energy is dependent of frame of
reference
(e) The coefficient of restitution is 0.2
19. Choose the correct statement(s) :
Cd) The impulse of the collision is 20 m N-s
A (a) The locations of centre of mass and centre of gravity
15. The particles A and B each of mass
may be different for an object
, m' is attached by a light inextensible
(b) Internal forces can change the momentum of a
string oflength 2/. The whole system
non-rigid body
lies on a smooth horizontal table
with B initiaHy at a distance 1from A. (c) If the resultant force on a system of particles is (initial
B is given velocity u a shown in Fig. C.O.M. is at rest) zero, then the distance of the centre
5.94. Then just after the jerk: of mass of system may remain constant from a fixed
Fig. 5.94
point
u,f3
(a) vA = - (d) If net extemal force on a two body system is always
4
zero, then direction of velocity of the centre of mass of
u,f3 given system may change
(b) vB =-
4 20. A bob of mass 10m is suspended
through an inextensible string of m m
(c) component ofv B along AB is u-fj
4 length I. When the bob is at rest in
equilibrium position, two
(d) component of v B perpendicular to AB = ~ particles each of mass mstrike it as
2
shown in Fig 5.96. The particles
16. A block of mass Misplaced on ~
stick after collision. Choose the
a smooth horizontal floor. The .j~M~~~==:::lO m correct statement from the
block has massless rod of - Fig. 5.96
length L pivoted on it at 0. The Fig. 5.95 following:
rod has a point mass In attached to its end. The whole (a) Impulse in the string due to tension is 2 mu
system is released from the position shown in Fig 5.95. · 0 f the system Just
. aft er co11··· u,f3
(b) V e1oClty ISlOn IS v = - -
Then at the instant when rod becomes vertical (assume rod 12
does not hit the floor) :
(c) Loss of mechanical energy of the system is 137 mu 2
(a) ratio of the velocities of M and In =!!!.. 24
M
(d) Loss of mechanical energy of the system is 137 mu 2
(b) velocity of M is m~
M(m+M)
48
21. A ball is projected from a point in a horizontal plane with a

~
velocity of 20 mfs at 300 with horizontal. Coefficicnt of
(e) velocity of In is M restitution is 0. 8 then:
M(m+M)
(a) duration for which ball will rebound is 20 s
(d) both m and M will move lell
(b) duration for which ball will rebound is 10 s
17. In an elastic collision between spheres A and B of equal
(c) horizontal component of velocity of ball is 17 Illfs
mass but unequal radii, A moves along the x ~ax is and B is
stationary before impact. Which of the following is (d) horizontal distance before the ball ceases to rebound is
possible after impact? 17'/ m
(a) A comes to rest 22. A block of mass 111 moving with a vf?locity Vo collides with
a stationary block of mass M at the back of which a spring
(b) The velocity of B relative to A remains the same in
of spring constant K is attached, as shown in the Fig 5.97.
magnitude but reverse in direction
Select thc correct alternative:
(c) A and B move with cqual speeds, making an angle of
45° each with the x-axis
26. Two particles of equal mass are projected simullaneously
from the top of a tower of height 20 m with same speed
20ms - I, one horizontally and the other vertically upwards.
Fig. 5.97
Choose the correct altemative(s).
(a) The velocity of centre of mass is ~ Vo (a) The acceleration of centre of mass is g 12downward
m, M
(b) The acceleration of centre of mass is g downward
(b) The initial kinetic energy of the system in the centre of
maSSframe iS .!. ( mM
4 M+m
)vl (c) Maximum height ofcenrre of mass from the ground is
25 m
(d) Maximum height of centre of mass from the ground is
(c) The maximum compression in the spring is
40m
mM I
v -
o (",+M) K ANSWERS
(d) When the spring is in the state of maximum I. (a) <nd (b), 2. (a), (b) and (c), l. (c) and (d), 4. (a), (b) and
compression the kinetic energy in the centre of mass
(d), s. (c) and (d), 6. (a), (b) and (d), 7. (b) and (c), S. (b)
frame is zero
and (c), 9. (b) and (d), 10, (b) and (d), II. (a), (c) and (d),
23. A hopper drops gravel at a rate
of 75 kgls onto a conveyor belt 12. (a), (c) and (d), Il. (a) and (d), 14. (a). (c) and (d) IS.
moving at a constant speed v = (a), (c) and (d), 16. (a), (b) and (c), 17. All, IS. (b), (c) and
2.20 mls. Select the correct (d), 19. (a) and (c), 20. (a) and (b) 21. (b) and (d), 22. (a),
altemative(s) : (c) and (d), 23. (a), (c) and (d) 24. (a), (b) and (d), 25. (a), (c)
(a) The force needed to keep and (d), 26. (a) and (c)
the conveyor belt moving is Ag.5.98
165 N
Ic) Assertion-Reason ':'ype Questions
(b) The force needed to keep the conveyor belt moving is
llON (a) If both A and R are true and R ;s the correct
(e) Output power must motor have in order to drive the explanation of A.
-conveyor belt is 363 W (b) If both A and R are true but R ;s not correct
(d) The rate at which energy is lost is 181.5 W explanation of A.
24. Which of the following statements is/are correct about (c) If A is true but R is false.
location of centre of mass? (d) If A is fa lse but R is true.
(a) The centre of mass of three particles of equal mass (e) If both A and R are false.
placed at the vertices of a triangle ABC coincides with 1. (A): A bullet is fired from a gun which is initially at rest.
the centroid of triangle ABC Total kinetic energy is shared by the bullet and the
(b) A uniform wire is bent to form the sides of a equilateral gun in the inverse ratio of their masses.
triangle ABC. The centre of mass of wire coincide with (R) , Linear momentum of the system (bullet + gun) is
centroid of triangle ABC conserved.
(c) The centre of mass of four particles of equal weight 2. (A) , A body is moving along a straight line under a
placed at the vertices of a quadrilateral coincide with variable force. Area under the force-time graph on
centroid of quadrilateml the time axis will be the ' Impulse'.
(d) The centre of mass of a body can exist outside the (R) , Impulse is the total change of linear momenrum.
material of the body
l. (A) , In centre of mass reference frame, after perfectly
25. Two toy cars with different masses originally at rest are inelastic collision between two bodies, they will be
pushed apart by a ideal spring. Which of the following at rest.
statements would be true?
(R) , Net momentum of bodies is zero, before and aftcr
(a) Both toy cars will acquire equal but opposite momenta collision, in centre of mass reference frame.
(b) Both toy cars will acquire equal kinetic energies
(c) The tlJ.ore massive toy car will acquire less speed ANSWERS
(d) The smaller toy car will experience an acceleration of 1. (a) 1. (a) 3. (a)
greatest magnitude
[DJ Integer Type Questions its length has fallen on the table is 5n N. Find n. (The fallen
part does not fonn heap).
1. An inelastic collision takes place
between two spheres as shown in 7. In the Fig. 5.104 a 19 kg cart is ~38 kg 19 kg _
Fig. 5.99. The coefficient of connected by means of a coiled ~ ---- ~
(rest) spring to a 38 kg cart. The two • ••
restitution is 0.2. For what
minimum mass ratio m] I m2 will Fig. 5.99 carts are moving to the right at a Fig. 5.104
the velocity of m2 after collision be greater than v] ? speed of 25 mls when the spring suddenly becomes
uncoiled and propels the 19 kg cart forward with a speed of
2. A wedge of mass m] = 8 kg with its
upper surface hemispherical in shape
m, 27 mls. Find the speed of the second cart (in mJs) in centre
of mass frame.
(with radius r= 5m), as shown in Fig.
5.100. rests on a smooth horizontal 8. An object is projected with a speed 10 mJs at an angle of30°
surface near the wall. A small block of with the horizontal. The object breaks down into an equal
mass m2 =2 kg slides without friction fragments during its motion. One fragment in found to
on the hemispherical surface of the strike the ground at the distance of(n x .J3m)from the point
wedge. The maximum velocity of projection :n the same azimuthal plane, in which the
attained by the wedge is (in mls) ; Fig. 5.100 object is projected. If the centre of mass of the remain
3. A ball B is suspended from a string of fragments strikes the ground at the distance of7.J3 m from
length / attached to a cart C, which
carte Jmc the point of projection. Find the value of n. [g = 10mi S2]
may roll on a frictionless surface.
9. A block of mass M is placed on the top
Initially the cart is at rest and the ball
of a bigger block of mass 10M as
is given a horizontal velocity vo' The
shown in Fig 5.105. All the surface are
velocity of B as it reach the maximum
B >--vo frictionless. The system is released
height is (in mls) ; m, from 'rest. The distance moved by the
[Take mass of cart me = 20 kg, mass Fig. 5.101 bigger block at the instant the smaller
ofballmB =5kg,v o =5m1s] block reaches the ground is xx 10- 1 m. 2.2m
4. A bead of mass 2 m can
slide on a smooth rod. A
particle of mass m is m
2m

'
-" Find the value of x.
10. A circular arc (AB) of thin wire
from of radius R =.fin em and
Fig. 5.105
Y
attached to the bead by a ..-
light string of length / =
,','
..- mass M makes an angle of 90 0 at A
the origin. Find the y coordinate of
B
3m. Initially the particle
the C.M. taking 0 as the origin in
is held horizontally in
em, Fig. 5.106
level with the bead and FIg. 5.102
the string held just taut. 11. Fig. 5.107 shows a unifonn disc
The distance through which the bead will move when the of radius R from which a hole of
a
string has turned through an angle = 37 0 with the radius R / 2has been cut out from
horizontal is..!. m. Find n. left of the centre and is placed on
n right of the centre of disc. The
5. A particle of mass m moving with a m centre of mass of resulting disc is
speed v hits elastically another at a distance R I n from centre of
stationary particle of mass 2 m on a unifonn disc. Find the value of11.
smooth horizontal circular tube of Fig. 5.107
l2, The mass per unit length ofa rod
radius r. The time in which the next
collision will take is equal to nnr/ v,
find 11.
varies as m=( ~o )x where M 0 is a ·constant and xis the
Fig. 5.103 distance of any point on rod measured from one end. Centre
KL
6. A unifonn chain of mass m = I kg and length / hangs on a of mass of the rod from the given end is at a distance of
thread and touches the surface of a table 'by its lower end. 3
The force exerted by the chain on the surface when half of on the rod. Find the value of K.
13. A carpenter has constructed a toy as shown in Fig. 5.108. If ANSWERS
the density of the material of the sphere is 12 times that of
1. 5 2. 4 3. 4. 5 5. 2 6. 3 7.
cone. The C.M. of the system is at a distance xR from O.
what is x? •• 3 9. 2 10. 4 11. 4 12. 2 13. 4 14.
15. 5 16. 3 17. 6 18. 3

[E] Match the Columns


I. In a two blocks system shown in Fig. 5.110. Match the
following:
Rough
o ci
" q;50mla
"2R' 5ml~ 2kg
Fig. 5.108

14. One end of a massless spring of spring constant 100 N/m smooth/
md natural length 0.5 kg lying on a frictionless horizontal Fig. 5.110
table. The spring remains horizontal. If the mass is made to
rotate at an angular velocity of2 rad/s. Find the elongation
Column - I Column - II
of the spring. In em [Take nearest integer]
15. A particle of mass' m' having collided with a stational)' (a) Velocity of centre of (p) Keep on changing all
mass the time
particle of mass' M' deviated by an anglc 2: rad where as the
2
(b) Momentum of centre of (q) First decreases then
particle of mass' M' recoiled at an angle of 30° to the
mass becomes zero
direction of the initial motion of the particle' m', If after the
collision there is 40% loss in kinetic energy of the two (e) Momentum of 1 kg (e) Zero
particle system, find Mlm. block
16. A block of mass In is
(d) Kinetic energy of 2 kg (s) Constant
attached with a massless
block
spring of force constant k.
The block is placed over a 2. A particle of mass 1 kg has velocity VI = 21 i and another
rough inclined surface for M
particle of mass 2 kg has velocity v2 :=: 2,2 1. Match the
which the coefficient of
".. . p :=:-.3 The following for system of the two particles. Assume that
lfictlOn IS Fig. 5.109 motion is being observed only after t = O.
4
minimum value of M required to move the block up the
plane is km, when the system is released from rest in Column - I Column - II
equilibrium and spring unstretched. Find the value of 5 k.
(a) Net force on system at (p) 20/9 unit
(Neglect mass of string and pulley and friction in pulley) ( :=: 2sec
17. A sphere of mass In collides elastically with another
stationary sphere of mass In /2 obliquely. Both the spheres (b) Velocity of centre of (q) ,J68 unit
are smooth and there are no external forces acting on them. massatt = 2sec
Maximum angle through which sphere of mass In can be (e) Displacement of centre (e) .J86/3unit
deflected, w. r. t. its initial direction of motion is 21:. Value of of mass at' :=:2sf'c
x
xis (d) Magnitude of linear (s) J86 unit
18. Hailstones falling vertica!ly with a speed of 10 mis, hit the momentum of centre of
mass at , :=: 2 sec
wind screen (wind screen makes an angle 30 0 with the
horizontal) of a moving car and rebound elastically. The 3. In the arrangement shown in Fig. 5.111. Match the
velocity of the car, if the driver finds the hailstones rebound following:
vertically after striking, is I ifx mls find the value of x.
-
2ms- 1 3. Maximum displacement of block A in left direction is

~~
approximately: (g = lOm/s2)

3!.Q~ (a) 22 m (b) )2 m


Smooth surface (c) 4,2 m (d) 52 m
Fig. 5.111
Paragraph ~ 2
Column -I Column - II A U shaped tube of mass 2 m is placed on a smooth
(,) Velocity of centre of (p) 2 SI unit horizontal surface. Two identical spherical balls each of
mass mass maud of diameter slightly less than the inner diameter
of tube enters into the tube with a velocity v as shown in Fig
(b) Velocity of combined (q) 1 SI unit
5.1\3. (Assume no loss of energy anywhere and all
mass when compression
collisions to be elastic).
in the spring is maximum
(0) Maximum compression (,) 4 SI unit m0--"
B " "- -==:::::::::
in the spring
(d) Maximum potential (s) 0.5 S1 unit
energy stored in the
spring m0--"
A "0:===
Fig. 5.113
ANSWERS
Answer the following questions.
I. a-+ f,S; b-l> r,S; c-+ q; d-+ q
4. Speed of each spherical ball, just before their collision:
()
~ ~
2. a-t q; b-+ r; c-+ p; d-+ s
(a) ,.':., (b) (c) -/3u (d)
3. a-+ q; h-+ q; c-+ q; d-+ P -/3 3-/3 2 2
5. At the time of collision, angle between direction of motion
[FJ Linked Comprehension Type of spherical ball A and B is, as observed in the ground
Paragraph - 1 frame:
Block A (1 kg) is placed on a smooth horizontal surface and (a) 2tan - l ..fi (b) 2tan - t },.
connected with a block B (2 kg), as shown in the Fig. 5.112,
by an inextensible string. A bullet of mass 250 gm, strikes (c) tan-1..fi (d) "
the block A horizontally with speed 200 mls. The bullet
penetrates through the block A and comes out with velocity
6. In the given situation the maximum velocity that the U-tube
100 mls. can gain (choose one of the given options):
A m=2S0gm !!. when spheres are about to collide
--~
1kg
---.- speed =200m/s (a)
2
'0' (b) 2u when spheres move in backward direction with
speed u
(c) u, finally when sphere emerges from U-tube
(d) some extra information 'are required to calculate it
Br;~
Fig. 5.112 Pa,ragraph - 3
Two blocks of equal mass m are connected by an un-
Answer the following questions.
stretched spring and the system is kept at rest on a
1. Velocity of2 kg blockjust after bullet comes out of block A : frictionless horizontal surface. A constant force F is '
(a) 8 mls (b) ~ mls applied on the first block pulling it away from the other as
3 shown in Fig. 5.114.
(c) 25 mls (d) 25 mls m K m

~~w
3 7
2. Impulse produced by string on block B :
(a) 50 N-s (b) 25 N-s Fig. 5.114
50 50
(0) - N-s (d) - N-s Answer the following questions.
3 4
7, Then the displacement of the centre of mass after time t is: (a) 43 mls (b) 5.6m1s
B2 B2 (e) 1.8 mls (d) 3.4 mls
(a) (b)
2m 3m 12, Final speed of the platfonn in situation (ii), j, e. ,just after B
B2 B2 has jumped, will be nearly:
(e) (d) (a) 3.7 mls (b) 5.3 mls
4m m
(e) 4.4 mls (d) 6.2 mls
8. If the extension of the spring is Xo at time t then the
displacement of the first block A at this instant is : 13, Final speed of the platfonn in situation (iii), i. e., just after A

(a) ~[: +xo 1 (b) -H: l· +xo


has jumped, wilt be nearly:
(a) 4.4 mls
(e) 5.7 mls
(b) 3.2 mls
(d) 4.6 mig

(e) H~2 1 -xo (d) [ : +xo 1 Paragraph - 5


In Fig, 5,116, a block A of mass 2 kg is moving to the right
9, If the extension of the spring is Xo at time t then the with a speed 5 mls on a horizontal frictionless surface,
displacement of the second block B at this instant is : Another block B of mass 3 kg with a massless spring of

~[: +xo 1
spring constant 222 N/m attached to it, is moving to the left
(a) [ : -xo 1 (b) on the same surface and with a speed 2 mis, Let us take the
direction to the right as the positive X -direction, At some
instant, block A collides with the spring attached to block B,
I 2
(e) z[2Ft -xoJ (d) ~[: - xo1 At some other instant, the spring has maximum
compression and then, finally, the blocks move with their
final velocities, Assuming that (i) the spring force is
Paragraph - 4 conservative and so there is no conversion of kinetic energy
Two persons, A of to internal energy and (ii) no sound is made when block A
mass 80 kg and B of hits the spring,
mass SO kg are 3 kg
standing on
horizontal platfonn
of mass 20 kg, The
platfonn
a

is
2 :::::::::::008
Fig. 5.116
supported on wheels Fig. 5.115
Answer the following questions,
on a horizontal frictionless surface and is initially at rest.
Consider the following situations, 14. Velocity of centre of mass of the system of blocks A and B,
before collision, is :
(i) Both A and B jump from the plalfonn simultaneously (a) zero (b) - 0.61 mls
and in the same horizontal direction,
(e) o.81m1s (d) 1.41m1s
(ii) A jumps first in a horizontal direction and after a few
seconds B also jumps in the same direction, 15. In the collision process, while the spring is getting
compressed:
(iii) B jumps first in a horizontal direction and after a few
seconds A also jumps in the same direction, (a) both linear momentum and kinetic energy are
conserved
In all the three situations above, just after the jump, any
person (A or B) moves away from the platfonn with a speed (b) both linear momentum and mechanical energy are
5 mls relative to the platfonn and along the horizontal. conserved
Answer the following questions, (c) linear momentum is. conserved but mechanical energy
is not conserved
10. In situation (i), just after both A and B jump from the
platfonn, velocity of centre of mass of the system (A ,B and (d) neither the linear momentum nor the mechanical
the platform) is : energy remain conserved
(a) 1.4 mls (b) 3.2 mls 16. Final velocity of block A will be :
(c) 2,8 mls (d) none of these (a) 2.5 i mls (b) -1.8 i mls
II. Final speed of the platfonn in situation (0, j,e.,just after (c) 3.6 i mls Cd) - 3.4 i mls
both A and B have jumped will be nearly:
17. Final velocity of centre of mass of the system of blocks A 5. When a ball is thrown up the magnitude of its momentum
and B will be: decreases and then increases. Does this violate the
(a) zero Cb) 0.6 i mI, conservation of momentum principle?
Ce) 0.8 i mI, Cd) - 1.4 i mI, [Hint: See § 5.2 B (b) ~ and liz]

18. When the hlocks are yet to attain their final velocities, in [Ans. No]
this situation, at any instant when block A is moving with a 6. Why can momentum conservation be applied to the
velocity 4 i mis, velocity ofbJock B will then be: collision oftoVo cars even when there are other forces acting
Ca) - 1.33 i mI, Cb) -2.67 i mI, such as friction?
Ce) 1.67 i mI, Cd) 3.77 i mI, [Hint: See § 5.3 (4)]
19. In Q. 18, at the given instant, compression of the spring is 7. Is the statement given below is logically correct "The
nearly: collision between two hydrogen atoms is perfectly elastic,
so that momentum is conserved?"
(a) 16 em (b) 24 em
[Ans. No; As momentum is always conserved the statement is not
(e) 33 em Cd) 52 em logically correct]
20. Maximum compression of the spring in the collision will be
8. A body of mass ml makes a head on perfectly elastic
nearly: collision with a body of mass m2 initially at rest.
Ca) 30 em Cb) 50 em (a) What fraction of initial kinetic energy of mass m ] is
Ce) 72 em Cd) 36 em lost in collision?
ANSWERS (b) For what value of(m2 1m]) the fraction of energy loss is
maximum?
Paragraph-l 1. (e) 2. (e) 3. (d) [Hint: See § 5,4 (A) e]
Paragraph-2 4. (0) 5. (.) 6. (0)
[Ans.(a)f= ml m22 ,(b)m 2 =1]
+ m2 )
Paragraph-3 7. (e) 8. (.)
•• (d) (ml ml

Paragraph-4 10. (d) 11. (a) 12. (d) 13. (e) 9. If a ball of mass m moving with velocity v strikes head on
Paragraph.S 14. (e) ... (b) 16. (d) 17. (e) 18. (a) 19. (e) elastically with number of balls of same mass at rest in a
line, only one ball from the otherside moves with same
20. (a) velocity. Explain why not two balls can move
simultaneously with velocity (vI2)?
IGJ Subjective Type Que,tioa, [Ans. In the fonner case both momentum and KE are conserved
1. (a) A light and a heavy body have the same kinetic energy, while in later case momentum is conserved but not KE]
which one will have greater momentum? (MNR 1990] 10. What conclusion can you draw about the masses of
(b) A light and a heavy body have same momentum, which projectile and target in case of head on elastic collision? (a)
one will have greater kinetic energy? If projectile rebounds, (b) If projectile stops, (c) If target
flies ahead of projectile.
[Ans. (a) Heavy body as p = ../2m(K)
[Ans. (a) mp <"T, (b) mp ="T, (c) mp > "T]
(b) Light body as K =l l 2m]
11. If two objects collide and one is initially at rest, (a) Is it
2. (a) Is linear momentum ofa system always conserved? possible for both to be at rest after collision? (b) Is it
(b) Can Newton's III law be treated as a consequence of possible for one to be at rest after collision? Explain.
conservation of linear momentum? [Ans. (a) No; As momentum will not be conserved, (b) Yes; when
[Hint: See § 5.2 (a) 3] their masses are equal and collision is head on elastic]
[Ans. (a) No; Only if the system is isolated and closed. (b) Yes] 12. (a) Is kinetic energy always lost in, inelastic collision?
3. Does a light rifle 'kick' more strongly than a heavy one? (b) Is all the kinetic energy always lost in, inelastic
[Hint: See § 5.2 (B), (a) (2)] collision if there is loss of kinetic energy?
[Ans. Yes] [Hint: See § 5.3 (10) and (11)]
4. A bullet is fired from a rifle. If the rifle recoils freely, [Ans. (a) No, (b) No]
detennine whether the kinetic energy of the rifle is greater 13. Is it possible to have a collision in which all the kinetic
than, equal to or lesser than that of bullet. [MNR 1990) energy is lost? If so, cite an example.
(Hint: See § 5.2 B (a) 3] ~ ~

[Ans. Yes, in inelastic collision when ml U I + mz U z = 0; Inelastic


[Ans. KE of rifle is lesser than ofbuiiet]
collision of two bodies moving towards each other with equal
momenta]
14. (a) Does the centre of mass ofa solid body necessarily lie 18. A box is put on a scale balance marked in
within the body? units of mass and is adjusted to read zero
(b) Must there necessarily he any mass at the centre of when the box is empty. A stream of
mass of a system? marbles each of mass m is then poured into
[Hint: See § 5.5 (1) and (2)] the box from a height h above its bottom at
a rate R. If the collision between the
( 4.ns. (a) No, (b) No)
marbles and box are completely inelastic,
15. If only external forces can cause the centre of mass of a find the scale reading at time t after the
system to accelerate, how can a vehicle be started or marbles begins to fill the box.
stopped? [Ans. (Fig) = mR[t+ ~(2hl g)]
[Ans.ln case of starting or stopping a vehicle, the force of friction FIg. 5.117
between wheels and road is the cause which is external to the 26
19. A sodium atom (mass = 3.8 x 10- kg) initially at rest emits
system (vehicle») a photon of yellow light (1...= 5890 A) and recoils in the
~
opposite direction to the direction of emission. What is the
16. A particle of mass m and velocity v collides elastically with
recoil speed of sodium atom?
a stationary particle of mass m Calculate the angle between ~ ~

the velocity vectors of the two particles after the collision. [Hint:pp =hn...and Pp+ PA =0]
IRoorkce 19971 [Ans. 2.9xI0- 2 mls]
[Ans.91 + 92 = 90 6 ; See solved Problem 32]
20. A body of mass 1 kg initially at rest, explodes and breaks
17. Fill in the blanks: into three fragments of masses in the ratio I : I : 3. The two
(a) Kinetic energyofa body of mass I kg and momentum pieces of equal mass fly off perpendicular to each other
2 N-s is. . . . . IEAMCET 19901 with a speed of IS mls each. What is the velocity of the
heavier fragment?
(b) A 2 kg and 3 kg body have equal momentum. If the
kinetic energy of 3 kg body is 10 J, the kinetic energy [Ans. (5-./2) mls opposite to the resultant momentum of equal
of2 kg body will be... IEAMCET 19931 masses]
(c) A gun fires 50 g bullets with velocity 1,000 mls each. A 21. A block of mass mmoving at a speed v collides with another
person holding the gun can exert an average force of block of mass 2m at rest, the light block comes to rest after
180 N against the gun. The maximum number of the collision. What is the coefficient of restitution?
bullets he can fire per minute is ..... [EAMCET 1993) [Ans. eo= (112)]
(d) A particle of mass 4 m which is at rest explodes into 22. A body of mass 3 kg collides elastically with another body
three fragments, two of the fragments each of mass m at rest and then continues to move in the original direction
are found to move with a speed of veach in mutually with one-half of its original speed. What is the mass of the
perpendicular direction. The total energy released in target body?
the process is ..". . . - IEAMCET 1991) [Ans. 1 kg]
(e) A ball is dropped from a height II to the ground. If the 23. (a) Steel ball of mass 0.5 kg is
coefficient of restitution is e, the height to which ball fastened to a cord 70 cm long
goes up after it rebounds fourth time is ..... and fixed at the far end and is
IEAMCET 1993) released when the cord is
horizontal. At the bottom of its
(f) A 6 kg fish is swimming at 2 mls. He swallows a (2/3)
path the ball strikes a 2.5 kg
kg fish swimming towards him at 3 mls. The velocity
steel block initially at rest on a
of the larger fish immediately after his lunch will be ... Fig. 5.118
frictionless surface. The
(g) A railway truck of mass 2x 104 kg travelling at 0.5 mls collision is elastic. Find (a) The speed of the ball and (b)
collides with another of half its mass, moving in the The speed of the block, both just after the collision.
opposite direction with a velocity of 0.4 mls. If the [Ans. (a) -2.47 mis, (b) 1.23 mls]
truck couples automatically on collision, their common
24. A small bucket of mass M(= 10-2 kg) attached to a long
velocity after collision is ..... mls. IEAMCET 1991J
inextensible cord of length L(= 5x 10- 2 m). The bucket is
(h) The centre of mass of two particles with masses 4 kg
and 2 kg located at (1,0, 1) and (2, 2, 0) respectively released from rest when the cord is in a horizontal position.
has co-ordinates. .. . IEAMCET 1990) In its lowest position the bucket scoops up m(= 10-3 kg) of
[Ans. (a) 2 J, (b) 15 J, (e) 216, (d)(3/2) nw 2, (e) he8, (f) 1.5 mis, (g) water, what is the height of the swing above the lowest
0.2, (h) {(4/3), (213), (213)}) position? IRoorkee 1995)
[Ans. L{M I (m+M)}2 =4.13 x 10-2 m]
25. A bullet of mass 0.0 1 kg and travelling at a speed of500 mls
strikes a block of mass 2 kg which is suspended by a string c
of length 5 m. The centre of gravity of the block is found to
raise a vertical distance of 0.1 m. What is the speed of the
Fig. 5.122
bullet after it emerges from the block? (g '" 9.8ml 8 2 )
[Hint: First consider inelastic col!ision between Band C and
IRoorkee 1990] apply conservation of momentum to get u B = (2u/3), then again
[Hint: By conservation of momentum nUl '" mu + MV and after consider inelastic collision between A and (B + C) to give
collision V "" .figh.] V '" (3v 14} Now apply law conservation of mechanical energy
[Ans. 220 mls] before and aftercol!ision between A and (B + C)to get:
I 2 I 2 I 2 I 2
26. A rifle bullet of mass 0.1 -//IV +-3mvs =-(4m)V + - kx
2 2 2 2
kg strikes and embeds m v
itself in a block of mass ~ ~~~~::i On substituting Vs and V and simplifying we get the desired
result]
0.9 kg. The block rests on ~
a horizontal frictionless Fig. 5.119 [Ans. (mv2/12k)1I2]
table and is attached to a 30. (a) A neutron of mass I amu and speed 10 5 mls collides
spring as shown in Fig. 5.1 19. The impact compresses the with a stationary deutron of mass 2 amu. The particles do
spring by 0.2 m. If a force of 1 N compresses the spring by 2 not stick together and no kinetic energy is lost in the
em, calculate the initial velocity of the hu!let. collision. What is the subsequent speed of each particle? (b)
[Hint: By conservation of momentum mu = (m + M)V and after If the above two particles stick together, what is the speed
collision by conservation of ME, ( 1/2)(111 + M)V2 = (l/2)kx 2] of the composite particle called triton?
[Ans. (a) UN '" -3.33 x 104 mls and v = 6.66 xlO4m1s
D
[Ans.lO(.J2) mls]
27. A ball of mass 111 is (b)VT =3.33xl0 4 m/s]
projected with speed v 31. In case of a-particle scattering by gold calculate the
into the barrel of a percentage loss in kinetic energy of a-particle due to its
spring~gun of mass M collision with (a) orbital electrons (b) gold nucleus. [mas"
initially at rest on a FIg. 5.120 of electron is (1/1840) amu and mass of gold = 197 amu]
frictionless surface. The mass m sticks in the barrel at the [Hint: See § 5.4 (c)]
point of maximum compression of the spring. (a) What is [Am. (a) 0.05% (b) 7.8%]
the speed of spring-gun after the bhll comes to rest in the
barrel? (b) What fraction of the initial KE of the ball is Note: In both the cases percentage loss of KE is very small. In case
stored in the spring? Neglect friction. (a) projectile (a~particle) is very heavy than the target
[Ans. (a) IIIvl (m + M), (b) M / (m + M)] electron while in case (b) projectile (a-particle) is very light
as compared to target (gold nucleus).
28. Two blocks of masses ml = 2 kg and 1112 = 5 kg are moving
in the same direction along a frictionless surface with 32. Two unifonn solid spheres composed of the same material
speeds 10 mls and 3 mls respectively, 1112 being ahead of and whose radii are 6 and 3 em respectively are firmly
m! . An ideal spring with k = 1120 Nim is attached to the united. What is the distance of centre of mass of the whole
back side of m2 . Find the maximum compression of the body from the centre of large sphere?
spring when the blocks collide.
(Ans. I cm]
33. Four particles P,Q,R and S of masses m,2m,3m and 4m
respectively are placed at comers of a square of side' a' as
Fig. 5.121 shown in Fig. 5.123. Locate the centre of mass.
y
[Ans. 0.25 m]
-a->l
29. Two blocks A and B of masses m and 2m respectively are
connected by a spring of force constant k. The masses arc S 4m 3m R
moving to the right with uniform velocity veach, the
heavier mass leading the lighter one. The spring is of
natural length during this motion. Block B collides head~on
with a third block C of mass m, at rest, the collision being
.
Im'!C._-,,2~m~_
P Q
,
completely inelastic. Calculate the maximum compression Fig. 5.123
of the spring.
[Ans. al2, 7a/1O]
34. A circular plate of unifonn 41. A proton moves with a speed of 7.4Sx 10 5 mis direct!)
thickness has a diameter 56 em. A towards a free proton originally at rest. Find the distance 0:
circular portion of diameter of 42 closest approach for the two protons. (Given that 1I41tf;~
em is removed from one edge of the =9xI09 mIF;m p =L67xlO- 27 kg and e = L6xlO- 1
plate as shown in Fig. 5.124. Find
coulomb)
the position of centre of mass of the
remaining portion. [Hint: See solved Problem 36]
[Hint: See solved Problem 24] Fig. 5.124 [Ans.1O- 12 m]
(Ans. 37 em from A towards B on the 42. A ball ofmassM = 0.2 kg rests on a vertical column, heigh
line AB] h =Sm. A bulletofmassm=O.Ol kg travelling with veloci~
35. An arctic explorer pulls a dead polar bear towards himself Vo = 500 mis, passes horizontally through the centre ofth(
by means of a rope. Both he and the bear are on perfectly ball. The ball reaches the ground at a distance s = 20 m
smooth ice. The explorer weighs 150 kg and is 12 m away Where does the bullet reach the ground? What part 0
from the bear. He gives the rope a quick pull $0 that he and kinetic energy (KE) of the bullet has been transformed a:
the bear start sliding towards each other, The explorer and heat?
bear collide 9 m from the original position of the explorer. [Ans. 100 m; 92%]
How heavy is the bear? 43. A uniform rod AB of mass m and length Sa is free to rotab
[Ans. 450 kg] on a smooth horizontal table about a pivot through P, :
36. A boat of mass 90 kg is floating in stilt water. A boy of mass point on AB such that AP = a. A particle of mass 2mmovinl
30 kg walks from the stem to the bow. The length of the on the table strikes AB perpendicularly at the point 2a fron
boat is 3 m. Calculate the distance through which the boat P with speed v, the rod being at rest. If the coefficient 0
will move. [MNR 1992) 1
restitution between them is - determine their speed:
[Hint: 90dl '" 30d2 with dl + d2 '" 3J 4'
immediately after impact.
[Ans. 0.75 m]
37. A block of mass misresting on the
top of a smooth prism of mass M
m
T 15v 83v
[Ans. ooroo = 37a' vp&rtic!c '" 148 ]
44. A hammer of mass M kg falls freely from a height of ,
which is resting on a smooth h
table. Calculate the distance M 1
moved by the prism when the '--:::::--:-::::::,-r:::.
metre on the top of an inelastic pile of mass m kg which i
driven into the ground to a distance of a metre. AssuminJ
block reaches the bottom (Fig. Fig. 5.125 that the resistance of the ground is constant, show that th,
5.125).
[Hint: Md, = md2 and d,

[Ans. mhcot91 (m + M)]


+ d 2 '" d rel = hcose] =: :[U:~l~flC;jn::l:~l:h:Sk::et:o:::g~ l::~:: ,:
M gh
38. A man weighing 80 kg is standing on a trolley weighing impact.
320 kg. The trolley is resting on frictionless horizontal rails. [Ans. ( 2Mmghll
What is the displacement relative to ground after 4s, if the M+m
man starts walking on the trolley along the rails at speed of
45. A nail of mass M is driven into a board against a constan
I mis? resistive force F by a hammer of mass m. which is allowe.
[Hint: v =Mv rei I (m + M), with u re! = I mis and x =vI] to fall freely at each stroke through a height h. The hamme
[Ans. 3.2 m in his direction of motion) does not rebound after striking the nail. Find the distanc
39. Two particles A and B initially at rest move towards each the nail is driven in at each blow. Show that the total energ
other under a mutual force of attraction. What is the speed expended in raising the hammer during the operation 0
of centre of mass at the instant when the speed of A is v and driving the nail fully into a depth of d is independent ofth
the speed of B is 2v? value of h, and can be decreased by making the hamme
[Hint: Fcxt = 0 so, vCM '" constant] more massive.
2
[Ans.Zero] [Ans. d = mghM 1
F(m+ M)
40. Two particles of masses 2 kg and 1 kg are moving along the
same line with speeds 2 mis and 5 mis respectively. What is 46. A particle of mass 0.05 kg moving along a straight line A,
the speed of centre of mass of the system if both the at 1.5 mlsec is given an impulse which doubles its spee
particles are moving (a) in same direction, (b) in opposite and causes the particle to move along a line at an angle c
direction? 60° to the original direction. Ifa particle of mass 0.02 kg ~
[Ans. (a) 3 mis, (b) (1/3) mis in,the direction of motion of I kg] rest were given the same impulse, in what direction would
move with respect to the line AS and what would be its 53. A ball of mass m is just disturbed from the top of a smooth
speed? circular tube in a vertical plane and falls impinging on a ball
[Ans. 3~ mlsec; at right angles to AB] of mass 2m at the bottom. The coefficient ofl'estitution is~.
2
47. Particle 1 experiences a perfectly elastic collision with a Find the height to which each ball rises after a second
stationary particle 2. Determine their mass ratio, if(a) after impact.
a head on collision, the particles fly apart in opposite [Ans. ~ and ~, where a is radius of tube]
directions with equal velocities, and (b) the particles fly 8 2
apart symmetrically, relative to the initial direction of 54. Two particles P of mass 2m and Qof mass marc subjected to
motion of particle I, with an angle of divergence e = 60°, mutual force of attraction and no other force acts on them.
m] 1 m]
(Ans.-=-,-=2] At time t = 0, P is at rest at a fixed point 0 and Q is directly
m2 3 m2 moving away from 0 with a speed Su. At a later instant
when t = T before any collision has taken place Q is moving
48. A heavy mass M resting on ground is attached to a small
mass m via a massless inextensible string passing over a towardsOwith speed u. (a) Find in terms ofmanduthe total
pulley. The string connected to M is loose. The smaller work done by the forces of attraction during the time
mass m falls freely through a height h and the string interval O:s; t :s; T. (b) At the instant t == T, impulses of
becomes taut. Obtain the time from this instant onwards magnitude J and K are applied to P and Q bringing them to
when the heavier mass again makes contact with the rest. Find the values of J and K.
ground. Also obtain the loss in KE when mass M is jerked [Ans. (a) Work done = 3mu 2; (b) J = 6mu, K '" mu]
into motion.
55. A particle P of mass m is projected from a point 0 with
[Ans. t =: 2m ffh .
- ; Loss m KE ==
Mmgh ] speed u at an angle of inclination a to horizontal. When P is
(M+m) g (M+m) at A (highest point of its path) it collides directly and
49. When a block of wood of mass 1 kg is held in a vice a bullet coalesces with the particle Q of mass 3m which is moving
of mass 10 g fired into it penetrates to a depth of 10 cm. If horizontally with same speed as P but in opposite direction.
the block is now suspended so that it can move freely and a (a) Find the speed of composite particle and (b) calculate
second bullet is fired into it, to what depth will the bullet the loss of KE. (c) If the composite particle meets the
penetrate if the retarding forces are same in both cases? horizontal plane passing through 0 at a point B, find the
distance DB.
[Ans. d== 9.9 cm]
u 3 1/2 .
50. Two men A and Beach of mass msit in loops at the end ofa [Ans. (a) -cosa ; (b) _1111/2 cos 2 a; (c) -smacosa)
light flexible rope passing over a pulley. A is h feet higher 2 2 2g

than B. In B's hands is placed a ball of mass.!!!.. which he 56. Two equal balls are lying in contact on a smooth table. A
10 third equal and similar ball moving along their common
instantly throws upto A so that it just reaches him. By the tangent strikes them simultaneously. If e is the coefficient
time A has caught the ball, what is the distance moved by
of restitution, find the fraction of kinetic energy lost by
him?
impact.
[Ans.2h/l9]
3(1-.,')
51. Two small spheres A and B of equal radius but mass 3m and [Ans. Fraction = - - - ]
5
2 m an: moving towards each other and impinge directly.
The speeds of A and B before collision are respectively 4u 57. A sphere A of mass mA = 2 kg is ®
and u. The collision is such that B experiences an impulse of released from rest in the position ,
magnitude 6mcu, where c is a constant. Determine (a) the shown. It strikes the frictionless B
coefficient of restitution, (b) the limits for the value of c for inclined surface of a wedge B of .1....__.....
mass mB = 6 kg with a speed of 3
which such collision is possible, (c) the value of c if ~ of mls. The wedge which may move
Fig. 5.123
16
total kinetic energy is destroyed. freely in the horizontal direction, is initially at rest.
Knowing that €I '" 30° and the coefficient of restitution
5
[Ans. (a) e == (c-I); (b) 15: c 5: 2; (c) c=-] between the sphere and the wedge is e = 0.8, determine the
4
velocities of the sphere and wedge immediately after
52. A ball of mass 50 g falls from a height 1.0 m, on to the impact.
horizontal surface of a massive slab. Find the cumulative
[Ans. VAl""' 2.3 m/s at 18.90 from horizontal, VBI = 0.72 mls to
momentum that is given to the slab by the ball, after a large
number of impacts, if every impact decreases the velocity left]
of the ball bYll =(= l.2S) times.
[Ans. 2 kg/sec]
Translatory Motion
§ 6.1 Motion and Rest
The concept of motion and rest depends on frame of
reference of observer and object. If the position ofan object in
B
"
::
:' :'
.::A, B' A'
----
~~

'I :

\ ! \ !
\ I
space changes with time (relative to an observer) it is said to " "
I\ '\ "
/!
be in motion, otherwise at rest. Motion can take place in one , '-~ ,
" (/
dimension (linear motion), two dimensions (planar motion) or
in three dimensions. ..

::

~•
. /~
t!
r Translatory Motion
(8) (b)
Rotatory Motion
(e)
Vibratory Mollon
: Ftg.6.2
(8) (b) (e) (C) Oscillatory and Vibratory Motion: If a periodic
1-0 Motion 2-D Motion 3·0 Motion motion is constrained, i.e., within certain limits or is to and fro
Fig. 6.1 or up and down, it is said to be oscillatory. If in case of
Broadly the motion can be divided into following three types: oscillatory motion amplitude is microscopic, it is usually said to
be vibratory. Further in case of oscillatory or vibratory motion
(A) Rectilinear and Translatory Motion: Ifa particle or
if restoring force (or torque) is linear, the motion is said to be
body treated as a point is moving along a straight line, the
sjmple harmonic. This motion is discussed in detail in Chapter 8.
motion is called rectilinear. However, if the body cannot be
treated as a point but moves in such a way that all its particles The motion, either translatory, rotatory or oscillatory, is
move simultaneously along straight lines (which are collinear divided into two branches, viz., kinematics and dynamics.
Kinematics describes the motion while dynamics relates the
or parallel to each other) by shifting through equal distance in a
cause of motion (force or torque) with the properties of moving
given time, the motion of the body is called translatory.
objects (such as momentum, kinetic energy, etc.). In this
Translatory or rectilinear motion can be uniform or
chapter we shall discuss kinematics of translatory motion
non-uniform. Motion on an inclined plane or under gravity are
which one usually reads in mathematics (in dynamics) also.
familiar examples of translatory motion. Projectile motion can
also be regarded as superposition of two orthogonal translatory § 6.2 Variables of Translatory Motion
motions (one horizontal and other vertical simultaneously) (A] Distance Dnd Displacement
giving rise to 2-D curvilinear motion. This all is the subject of Distallce is the actual path
this chapter. (length) covered by a moving
(8) Circular and Rotatory Motion : If a particle or
body treated as a point, is moving on a circle, the molion is
called circular. However, if the body cannot be treated as a
particle (or object) in a given
interval of time while displacement
is the change in position vector, i.e.,
-" B

point and all its particles move simultaneously along circles a vector joining initial to final
(whose centres lie on a line called axis of rotation) by shifting position. Ifa particle moves from A 0''''''-------- - -
through equal angle in a given time~ the motion is called to B as shown in Fig. 6.3, the Fig. 6.3
rotation about a fixed axis· or pure rotational or simply distance travelled is 6s while
rotatory motion. It is 2-D or 3-D motion and can be uniform or displacement is
non-unifonn. If uniform, it is periodic. This motion is
discussed in Chapter 7 .
• In case ofrouuory motion if the axis also moves, the motion is combined translatory Bnd rotatory called rolling motion. e.g., motion of wheel ofnlOl':aa
vehicle.
Regarding distance and displacement it is worth noting (I) Average speed is a scalar while average velocity is a
that: vector both having same units (mls) and dimensions
(1) Distance is a scalar while displacement is a vector, [Lr'J.
both having same dimensions [L] and SI unit m. (2) Average speed or velocity depends on time interval
over which it is defined.
(3) For a given time interval average velocity is single
valued while average speed can have many values
depending on path followed.
(4) If after motion the body comes back to its initial
(a) (b) (e) position.
~ ~
Distance", 1lr Distance., 2h Distance", S vav = 0 [asAr =OJ
IDlsplacement] '" 2r JDlsplacement] = 0 IDlsplacementl = S but vav>Oandfinite [as 11s> 0]
Fig. 6.4
(5) For a moving body average speed can never be
(2) The magnitude of displacement is equal to minimum negative or zero (unless t ~ 00) while average
possible distance between two positions; so velocity can be, i. e.,
Distance 2: IDisplacementl ~
Vav>O while vav>,=or<O
(3) Formation between two points displacement is single
valued while distance depends on actual path and so (6) In general, average
can have many values. speed is not equal to
magnitude of average
82 ------------ -------~i T
(4) For a moving particle distance can never decrease
with time while displacement can. Decrease in las
displacement with time means body is moving
velocity (as As 'l'IA11) .
However, it can be so
81 ------ - ___ _ ~ _
t--at-l
_ __ J1
towards the initial position. : :
if the motion is along a
(5) For a movingparticie distance can never be negative :, :'
straight line without
or zero while displacement can be. (Zero : i,
change in direction
displacement means that the body after motion has t, t,
~
come back to initial position.) (as lis =1'; r I). Fig. 6.5
Distance> 0 but IDisplacementl > :: or < 0 (7) If a graph is plotted between distance (or
(6) In general, magnitude ofdisplacement is not equal to displacement) and time, the slope of chord during a
distance. However, it can be so if the motion is along a given interval of time gives average speed (or
straight line without change in direction. velocity):
IB] Average Speed and Velocity
Vav::o As ::otan$ =slopeofchort
,;, .... (iii)
Average speed or velocity is a measure of overall
'fastness' of motion during a specified interval of time. (8) If a particle travels distances L 1, L 2 , L3 , etc., at speeds
The average speed of a particle for a given 'interval of v j ' v 2 ' v 3 ' etc., respectively, then as
time' is defined as the ratio of distance travelled to the time As = L I +L2 + .... +LII = ,£L j
token while average velocity is defined as the ratio of Ll L2 Ln L
displacement to time taken. and t'1t = - + - + .... +_='£_j
VI v2 Vn Vi
Thus, if a particle in time interval At after travelling
distance As is displaced by A1 [
as v = As, At = As]
'" v
Average speed = Dista.nce travelled, i. e., v = A.. .... (i)
Tune taken av At so .... (iv)

~
Displacement . ~ ';r
Average velocity -~"",==, I.e., v = - .... (ii)
Time av At If LI = L2 = .... :== L/I = L, Eqn. (iv) reduces to
Regarding average speed and velocity it is worth noting that:
_1_= ![2...+_1 +....l=.!. X~2... ....(v)
...v =constt. =...v av
vav n V, V2 n Vi
However, converse mayor may not be true, i.e., if
i.e., average speed is harmonic mean of individual
...
v = ...
vav the motton
' mayor may not be unhonn.
.,
speeds.
Note: If a particle moves a distance at speed vI and comes back with (3) lfvelocity is constant, speed (= Ivelocity!) will also he
constant. However, the converse mayor may not be
spced vl
true, i.e., ifspeed = constant, velocity mayor may not
1=2'I[ ~+
v.v
1 I] i.e., v
v
IV
=~
vI +v2
be constant as velocity has a direction in addition to
2 magnitude which mayor may not change, e.g., in case
~ 0 of un ifann rectilinear motion
while vIV - -Ilt .. 0 [as displacement = 0]
...v =constt. and so speed =1...vi =constt.
(9) Ira particle travels at speeds v,. v 2 • etc" for intervals
while in case of unifonn circular motion,
t I' t 2' etc., respectively, then as
As=V, /, +v2 /2 + .... +V"ln =I:vj/ j V c: constt. but 1 "*
constt. due to change in direction.

[as V= ~,ru=V!1t] (4) Velocity can be positive or negative as it is a vector


but speed can never be negative as it is magnitude of
and llt =', + 12 +..... + In = Itj velocity, i.e., v =111
As V lt, +V2/2 +.... Ev " j (5) If displacement is given as a function oftime, the time
so Vav =-: =--' - ....(vi)
III " + 12 +.... Lli derivative of displacement will give velocity and
Iftl = / 2 =... ='" :::/,Eqn. (vi)reducesto modulus of velocity will give speed, e.g.,
s=Ao -AI/+A2 t
,
VJ.I_+..:V:,'c. .+. :' :..' =-Lv.
C 1 .... {vii)
n n'
v = ds=-A 1 +2A2t
i. e., average speed is arithmetic mean of individual dl
speeds. So initially (t =0), velocity = -AI' while speed =.I- Ad
= AI
Note: If a panicle moves for two equal intervals of time at different
speeds, Note: It is a common misconception that:
d, ,
dl :r
[C) Instantaneous Speed and Velocity , Ao
Here, -, :---><.-A
, I +A,'
Instantaneous velocity ,or simply velocity is defined as rate of
~ which is totally different from the above value of (ds/dt)
change afparticle 's position with lime. If the position r of a
(6) As by definition v ::: (dsldt), the slope of displacement
particle at instant I changes by 111 in a small time intervall1t versus time graph gives velocity, i.e.,
~ ~
-+ .
v = hm - = -
l1r dr
....(viii) v:: ds = tan9 == slopeofsl tcurve ....(x)
6.1-+0 .6. t dt dl

The magnitude o/velocity;s called speed, i.e.,

Speed = lvelocitYI i.e., v=111


Regarding speed and velocity, it is worth noting that:
o
(I) Velocity is a vector while speed is a scalar, both L..."--J..:...-~-_Time
I
!:--- Time
having same units (m/s) and dimensions [LT- 1]. lal Ibl
(2) If during motion velocity remains constant Flg.6.G
throughout a given interval o/time, the motion is said (7) As by definition
to be umjorm and for unifonn motion, v = (ds! dt), i.e., ds=vdt
and from Fig. 6.6 (b), vdt = dA dv
or a=v-
so dA = d~ j.~. s=!dA= !vdt .... (xi) dx
i.e.. area under velocity versus lime graph and time
axis. with proper algebraic sign gives displacement (6) As acceleration it = (d1Idt). the slope of velocity
while withollt sign gives distance. versliS time graph gives acceleration, i.e"
[D) Average and Instantaneous Acceleration
Ifthe velocity ofa particle changes (either in magnitude or
-+
• =-
d-: =tan 9 =slopeof vlt-curve
-+
.... (xvi)
dt
direction or both) with time the motion is said to be accelerated
or non-uniform, In case of non-unifonn motion if change in (7) The slope of11t curve, i.e., ditldt is a measure of rate
velocity is t:J. ~ in time interval tJ.t, then of non-uniformity of acceleration. However, we do

--t 6v
... ...v 1 - ...v
J
not define this physical quantity as it is not involved
in basic laws or equation of motion .
• = - =- - - .... (xii)
av III 1 - / (8) Acceleration can be positive or negative. Positive
2 1
acceleration means velocity is increasing with time
Instantaneous acceleration or simply accelerqtion is while negative acceleration called retardation
defined as rate of change of velocity with time at a given (deceleration) means velocity is decreasing with time.
instant. So if the velocity of a particle -: at time t changes by Question I, Usually 'average speed' means the ratio of
lotal distance covered to the time elapsed. However,
1:11 in a small time interval tJ.1 d:en sometimes the phrase 'average speed' can mean the
... ... magnitude ofthe average velocity. Are the two same? Discuss.
--t
a = hm

-/1v =-
dv
.... (xiii) Answer: No, usually they have different meanings, as
61-+0 llr dl according to I-defin ition vav = [distance/time] while
Regarding acceleration it is worth noting that: according to II-defin ition vav = [ldisplacementVtime]. Now as
( I ) It is a vector with dimensions {LT - 2 ] and SI units distance O?: ldispiacementl, vav ~ I
...
Vav I, i.e. , usually average
[mI,']. speed is greater than magnitude ofaverage velocity, e.g. , if a
(2) If acceleration is zero, velocity will be constant and body returns to its starting point after some motion, then as
the motion will be uniform. However, if acc. is distance travelled is fi nite while displacement is zero, so
constt., acc. is unifonn but motion is non·uniform and
jf ace. is not constt. both motion and acc. are vav > 0 but 1 v-:v 1= O. However, in case of motion along a
non·unifonn. straight line without change in direction as (displacementl =
...
(3) If a force F acts on a particle of mass m, by Newton 's
distance, the two definitions will mean same .
Question II. When the velocity4s constant, can average
II law velocity over any time interval differ from instantaneous
...• = ( ...FI m) .... (xiv)
velocity at any instant? Ijso, give an example; ijnot, explain
why?
(4) As by defi nition 1 ;: : (d1ldt) Answer: No, by definition of velocity and average
velocity we have 1 = slope of tangent to -;1t curve at a g iven
'0 -; J~ =~(dt)= d'dt2t
dt dt dt
.... (xv)
instant of time and ~av = slope of chord of -;1t curve for a
g iven interval of time. Now for constant velocity, the slope o f
i.e" if 1 is given as a function of time, second time -;1t curve will be constant and so it will a straight line making
derivative of di splacement w.r.t. time gives a constt. angle with time axis. Now as in case of straight line,
acceleration.
=
Slope of tangent Slope of chord,
(5) Ifveloeity is given as a function of position, then by
chain rule i.e.,
......
v = vav
a=dv=dv.dx i.e., ifvelocity is COllstant, average velocity over any illterval
dt dx dl a/time is always equal to velocity at allY instant of time.
Question [J[. Explain with reasons which of the ~ ~
Question V. What does dl v iI dt and ldv l dtlrepresent?
following graphs can possibly represent the motion of a
particle observed in nature? Can these be equal? Can: (a) dJ 1 11 dt =0 while Id11dtlotO

(b) ~ V'II dl .Owhileld V'I dll = 01

(b) Answer : dI ~I I dt represents time rate of change of speed


11me I, TIme as I~I = v, while Id 11 dtlrepresents magnitude of acceleration.
If the motion of a particle is accelerated translatory
(without change in direction)
§
~ ~
as
11
0:4
" , Is

or [as ii\sconstant]
Answer; (a) This graph shows that with increase in time
distance first increases and then decreases. However, distance
can never decrease with time. so this graph is not physically or
possible.
(b) This graph shows that at certain instant of time (I,) the However, if the motk' is unifonn translatory, both these
body is present at two positions. Also it shows that time first will still be equal but zero.
increases and then decreases. Since both these conditions
cannot he realised.in practice, this graph is not physically (a) The given condition implies that:
possible.
Id~/dtl :;t:O, i.e., lacc.lotO while dl~l / dt=o.
(e) This graph shows that speed is negative for some
interval of time (/2 to I). Since speed can never be negative. i.e., speed = constl.
this graph is also not physically possible. This actually is the case of unifonn circular motion. In
(d) This graph shows that at,a given instant of time (t 4 ) the case ofunifonn circular motion
particle has two velocities. Also it shows that at time (15) the ~ 2
dv -+ v
acceleratio~ is infinite (= s!ope of11 t curve). Since both these -
dl
= 1al = - =constt.:;tO
r
,conditions cannot be achieved practically, this graph is also
, not possible for the mottons observed in nature.
Question. IV. Is it possible to be accelerating if you are while
tra'Velling at constant speed? Is it possible to round a curve,
. with zero acceleration? With constant acceleration? With ~ ~ . e., a =0
~
(b) Id v l dll= Omeanslal=O, I.
'Variable acceleration?
Answer: If speed is constant, velocity may change due to or (dV'l dl) =0 or -: =constt.
'"'thange in direction and 'as .acceleration is rate of change of
velocity so acceleration may not be zero when speed is And when velocity ~ is constant speed will be constt.,
constant. A-ctually in unifonn circular n:totion speed = constt.
. but acc. ~ O. d ~
i.e. , speed = 111= constt. or - I v l= O
For motion on a curve we at least have to change the dl
direction of motion, so we will require a force, and ' hence
acceleration, i.e., it is not possible to round a curve with zero
acceleration. However, in rounding a curve acceleration may
So it is not possible to have rvdt =Owhile~I~I
dt
:;tO.
be constant or variable. In case of projectile motion
Problem 1. A particle is mo'Ving in a plane with 'Velocity
acceleration is constant (= ft) while in case of circular motion
gi'Ven by:
acceleration ot constt. either due to change in direction or both
~ ~ ~
change in direction and magnitude. v = i Uo + j arocosrot
If the particle is at the origin at t = 0: (a) calculate the i.e., the law between remaining distance and time taken is
trajectory of the particle; (b) find its distance from the origin similar to that of radioactive disintegration, i. e., exponential;
at time (31t/2oo).
so s = soe- At withAasconstt. .... (i)
Solution: In tenns of components:
In this problem as for t = Is, s = (sol2)
-->
v = -->
iv x + 1Vy So -s O e-Ax!
so ~ - i.e., A" log, 2 "0.693 .... Iii)
So and vy = aro cos 001 2
Substituting this value of A in Eqn. (i), the position of
i. e., and dy = aro cos rot rabbit w.r.t. carrot at any time t wi11l)e given by
dt
which on integration gives s=soe- O.693t .... (a)

x=Juodt and Y= I aro cas 001 dt (a) For reaching the carrots = 0, which in the light ofEqn.
(a) gives
i.e., x=uot+A and y=asinrot+B
0 = soe-O.693t, i.e., eO.693t = 00
Nowasatt =O,x=O and y=O
So A =0 and B =0 which is possible only if t --t 00.
.. x=uot .... (i) and y = asinrot .... (ii) So physically the rabbit will never reach the carrot.
(a) Substituting the value of t from Eqn. (i) in (ii), (b) As by definition vav = (distance travelled/time taken)
y = a sin (m).:/ uo ) s
v =JL=O
which is the desired trajectory and is a sine curve as shown in
Fig. 6.7.
" '"
So the limiting value of average speed is zero.

.a (c) As s = soe-O.693t, ~: '" _ 0.693soe-O.693t


~ 2nuo
ro ro i. e.; v = _O.693soe-O.693t .... (iii)
~----"~----C:.J'--x.
Negative velocity here implies that with increase in time s
-a ------------------ - is decreasing, i.e., rabbit is approaching the carrot.
Now if t =0, v = v O' Eqn. (iii) gives
Flg.S.7
Vo =-0.693so e- O =-0.693so ... .(iv)
(b) For t =(311/200) from Eqns. (i) and (ii), we have
i.e., x = uo(3nI2ro) and y = -a So that Eqn. (iii) can be written as
So distance from the origin at t = (31[/200) will be v = voe-O·693t .... (b)

r=~x2 + i =~[a2 + (31tuO/2ro)2] The sit and vlt curves as given by Eqns. (a) and (b) are

Problem 2. Each second a rabbit moves half the


remaining distance/rom his nose to a carrot. (a) Does he ever
50
shown below in Fig. 6.8.

get to the carrot? (b) What is the limiting value a/his average
velocity? (c) Draw graphs showing his position and velocity as
1~ 1Vo 0.693 So
time increases.
Solution: If So is the initial distance between the rabbit
and carrot, the distance between them after 1 sec will be
5 2
.50t=tb:=- 2 3
v

0 L.-lL::,-:":':::::::;:=~
(so -so/2) = s o/2. Now in next sec the rabbit will move by tin sec.- tinsec. -
la) (b)
(112) s~ ; so the distance between rabbit and carrot after 2 sec Flg.6.S
Problem 3. (a) A particle is moving eastward with a
will be So _ So = So =~. Proceeding in the same way we velocity 0/5 mls. If in 10 s the velocity changes by 5 IIIls
2 4 4 22
northwards, what is the average acceleration in this time?
find that distance between rabbit and carrot after n sec will be
(b) What is the retardation 0/ a moving particle if the
s
s=....Q. 0'
s
~ " ~ =
relation between time and position is, t Ax2 + Bx? (where A
2" So . 2n and B are appropriate constants)
Solution: (a) As change in velocity -> -> ->
or vps,=vps-vs's ... (i)
-» -) -) -) .....
llv = V 2 - VI = V2 +(-v\) Thus, if the velocities of two bodies (here particle and
frame S') are known with respect to a conunon frame (here S),
A ~ =(~52 +5 2 )N_W
N
i.e., we can find the velocity of one body relative to the other. Thus

i.e., ..1 =S,[zN-W if1A· and 1B are velocities of two bodies· relative to earth, the
E
'---'-".>1._--,> " velocity of iJ relative to A will be given by
So
-+ !J. ~ 5./2
aav=-=--N-W
-',
Flg.6.9
M 10 .... (ii)
-> I , To make this concept more clear consider the following
i,e., 'ov=-,[z(m1s )N-W
examples:
(b) As t=AX2 +Bx (a) If two bodies are moving along the same line in the
same direction with velocities v A and vB relative to
dt or v=(2Ax+B)- I .... (i)
so -=2Ax+B earth, the velocity of B relative to A will be given by
dx
v OA =V B -VA
Now by chain rule,
If it is positive the direction of v BA is that of B and if
a=:=:.~ = v: [as : =vJ negative the direction of v BA is opposite to that of B.
However, if the bodies are moving to.wards or ~way
from each other, as directions of v A and VB are
so a=v!(2AX+B)- 1 [from Eqn. (i)]
opposite, velocity of B relative to A will have
magnitude
or a=v(-I)(2Ax+B)-'(2A)
v BA =v o - (-VA)=V B +vA
or a=-2A(2Ax+B)-3 [asv=(2Ax+B)-I] and directed towards A or away from A respectively.
So Retardation = -a = 2A(2Ax + B)-3 (b) If a satellite is moving in equatorial plane with
->
§ 6.3 Relative Velocity velocity V and a.point on the surface of earth with -;
When we consider the motion of a particle, we assume a relative to the centre of earth, the velocity of satellite
fixed point relative to which the given particle is in motion. For relative to the surface of earth
example, if we say that water is flowing or wind is blowing or a
person is running with a speed v, we mean that these all are
relative to the earth (which we have assumed to be fixed).
Now to find the velocity of a P
moving object relative to another //~
moving object, consider a particle
P whose positiCm relative to frame - .-
rps /
,rps'

S is r;,s while relative to S' is ",/,/ ...IS~·-----


" r;·s VSE .. V-v VSE - V + v
;:;'s" If the position of frames S' S
Fig. 6.10 (aJ (bJ

relative to S at any time is r; 's then from Fig, 6.10, Fig. 6.11
So if the satellite moves from west to east (in the
~ -> -> direction of rotation of earth on its axis) its velocity
fpS = fpS'+ fS'S
relative to earth's surface will be:
Differentiating this Eqn. with respect to time VSE = V - v
-> -> ->
dfpS df ' dfS'S and if the satellite moves from east to west, i.e.,
- - = -pS- + - - opposite to the motion of earth,
dt dt dt
VSE =V - (- v) = V +V
-> -> -> -> ->
or v PS = v ps,- vs's [as v =dr/dt]
229

Note: From eXilmples (a) and (b) it is clear that in dealing with the So, if the boy is running on the train in the direction of
~ ~ ~ motion of train,
motion of two bodies rela:ive 10 each other "rei (i.e.. Vru or VSE )
is the difference of velocities of tWo bodies if lhey are moving in v=v rel +V
the same direction and is the sum of two velocities if they are and if the boy is running on the train in a direction
moving in opposite directions. opposit~ to the.motion of Ira in,

(c) Ifa train is moving at equator on the earth's surface v=v rel - V
with a velocity VTE relative to earth's ·surface and a
Note: From examples (c), (d) and (e) il is clear that in case of motion ofa
point on the surface of earth with velocity VE relative body A on a moving body H, the velocity ofthe body A relative to
to its centre, then ground is the sum of two velocities if A is moving on B in the
~ ~ ~ same direction and is equal to difference of two velocities if they
V TE =Vr-Ve are moving in opposite directions.
"'+ -+ -+
or V T = VTE+V E ~

(f) If rain is falling vertically with a velocity VR and an


~
observer is moving horizontally with speed V M • the
velocity of rain relative to observer will be
~ ~ ~
VRM=VR -VM
which by law of vector addition has magnitude

VRM = ~R~ +V~


and direction a = tan - I (VM' VR ) with the vertical as
(a) shown in Fig. 6.13.
FIg. 6.12
So, if the train moves from west to east (the direction
of mati on of earth on its axis) , v,
Vr = VTE +V£
and if the train moves from east to west (i.e., opposite
to the motion of earth) Fig. 6.13
Vr =VTE - VE
~
Note: If speeds are comparable to the velocity of light c, according to
(d) If a man can swim relative to water with velocity V theory of relativity, velocity of B relative to A (when both are
and water is flowing relative to ground with velocity moving along the same line in opposite directions) is given by:
~ ~
VR', velocity of man relative to ground VM will be
(I+ (vAvB ,e )]
given by:
From this, it is clear that if v A or VB is equal to c
-+ -+ -+
V = VM-VR, i. e.,
-+ -+ -+
VM=V+VR
v., .,
[1+ (vi c)]
So, if the swimming is in the direction of flow of
water, i.e. , speed of light is independent of relative motion between
source and observer, the basic postulate of special theory of
VM=V+~R relativity,
and if the swimming is opposite to the flow of water,
Question VI. A steam boat goes across a lake and comes
VM=V-VR back: (a) on a quiet day when the water is still and (b) on a
~
rough day when there is a uniform current so as to help the
(e) If a boy is running with speed V rei on a railroad car
~
journey onward and to impede thejourney back. lfthe speed of
moving with velocity V relative to the ground, the the launch on both days was same, in which case will it
complete thejourney in lesser time?
speed of the boy relative to ground -; will be given by
Answer: If the length of the lake is L and the velocity of
-+ -+-+. -+ -+ -+ boat is V, time taken in going and coming back on a quiet day:
Vrel = v- V, l,e" v = Vrel +V
230 PHYSICS FOR COMPETITIONS - Vol. I

",,(i) Note: There is a misconception about this problem that as the four ·
persons are moving on sides of square with equal speed they will
never meet. Actually they are always moving towards each other.
Now if v is the velocity of air-current, then in going across so the square rotates and reduces in size and finally merge into a
the lake point at the centre.
I, =: L [as current helps the motion] Problem 4. Two cars are moving in the same direction
(V +v) ,
with the same speed30kmlhr. They are separated by a distance
and time taken in coming back of 5 /an. What is the speed of a car moving in the opposite
direclion if it meets these two cars aJ an interval of4 minuJe?
12 == L [as current opposes the motion]
(V -v) Solution: As the two cars (say A 2nd 8) are moving with
same velocity, the relative velocity of one (say B) with respect
2LV 2L
so tR = II + 12 = = - -""-7'
(V' -v') V[I-(vlV)']
",,(ii) to the other, A,
~ ~ ~

So from Eqns. (i) and (ii), v BA =v B- v A =v-v =O


So, the relative separalion between them (= 5 km) always
!..tL= I >1 [as 1- (vIV)' < I] remains the same.
IQ [1 - (vIV)']
Now if the velocity of the car (say C), moving in opposite
i.e., tR > IQ
direction to A and B, is -:c relative to ground, then the velocity
i. e., time taken to complete the journey on a quiet day is less
than on a rough day. of car C relative to A and B will be
-> -> ->
Note: It is a common-misconception thai on a rough day decrease in VIel ="C - v
lime due to helping currents will be equal to increase in lime due
to opposition and so the lime of journey on rough and quiet day
But as -: is opposite to vc
will be same.
VIC] =v
Question VII. Four persons K, L, M and N are initially at c - (-30)=(v c +30)kmfhr
So, the time taken by it to cross the cars A and B,
the four corners ofa square ofside d. Each person now moves
d
with a ulliform speed v in such a way that K always moves I ~-

towards L, L towards M, M towards Nand N always moves vrc!


directly towards K. Where, and at what time will the four 4 5
persons meet? or -= - - - - [as 4 minute = 6~ hr]
60 (v C +30)
Answer: It is obvious from considerations of symmetry
that at any moment of time the persons will be at the corners of or Vc = 45 kmlhr
a square whose side gradually decreases (Fig. 6.14) andso they Problem 5. A person walks up a stalled escalator in 90
w ill finally meet at the centre of the square O. The speed of second. When sJanding on the same escalator, now moving. lie
each person along the line joining his initial position andO will is carried up in 60second. How much time would it take him 10
walk up Jhe moving escalator?
be vcos 45° ", v /J2. And as each person has to walk a distance
Solution: Let the length of escalator be L If v is the
dcos 45°", dl..fi to reach the centre, the four persons will meet
velocity of man (relative to escalator) and V of escalator, then
at the centre of the square 0 after time according to given problem
I ~ d l.,fi~<{ K K' l f '~ 90s ",.(i) and L ~60s ",,(ii)
v l.fi v ~/<~:'-"~~~~~~~~~~~ ,l '
~ . ,'
v V
Alternative: The velocity /' / : Now if the person walks up on the moving escalator his
of K is v along KL while of L i s :, ,/ : velocity relative to the ground will be (V + ·u). So time taken by
/ '
him to move a distance L relative to the ground will be,
also v but along L M " :'
I
(perpendicular 10 KL). So the :;'/ i• t=
L or
1 V
-= -+ -
V
velocity of L towards K will be :/ ': (V +v) ILL
the N'.f.. \:
vcos90o=O. Thus
separation KL decreases at the
, -----.-
-~~-~- ->.:...-
", which in the light of Eqns. (i) and (ii) gives
1 1 1
rate v + 0 = v. So the time N M' M [=60 + 90' i.e., t = 36s
FIg. 6.14
taken in reducing the
separation KL from d to zero will be t dlv. = i. e. , the person will take 36 s to walk up to the moving
escalator.
Problem 6. Two trains, each having a speed 0/30 kmlhr § 6.4 Relations among Variables of Motion
are headed towards each other on the same straight track. A If a particle moves with an acceleration a, by definition,
bird that canfly at60kmlhrflies oJJone train, when they are60
<Iv
a=- or dv=adt
km apart and heads directly for the other train. On reaching
dt
the other train, it flies directly back to the first and so on.
If acceleration of the particle is constant and velocity of
(a) How many trips can the bird make from one train to the
body chnges from u to v in time I, the above equation on
other before they crash? (b) What is the total distance the bird integration yields:
travels?
Solution: The velocity of one train relative to the other,
I~' dv=aI~dt or [v]~=a[/]~
v IT = (v) - (-v) =2v =2 x 30 = 60kmlhr or v=u+at .... (i)
And as the distance between the trains is 60 kIn, the two Now again by definition of velocity, Eqn. (i) reduces to
trains will crash after 60 km/60 km hr- 1 = 1 hr.
Now the velocity of bird w.r.t. train towards which it is
dS=u+at
dt [asv=~;]
moving will be 60 + 30 = 90 kmIhr. or I~ ds = I~ (u + at)dt
So the time taken by bird for I trip, I I = (60/90) = 2/3 hr. I ,
In this time the trains have moved towards each other 0' s=ut+-at .... (ii)
2
(213) x 60 = 40 km, so the remaining distance = 60 - 40 = 20 Now substituting the value of t from Eqn. (i) in (ii), i. e.,
Ian. . eliminating t between Eqns. (i) and (ii), we get
So the time taken by bird for II tr:p, 12 =20/90 =(2/3 2)hr.
s = u (v - u) +.!. a [(V - U)J'
Proceeding in the same way ti.me taken by the bird for nth a 2 a
trip, In = (2/3 n)hr.
or
(a) Now if the bird makes n trips till the train 'crashes, i.e., v 2 =u 2 +2as .... (iii)
II +/2 +/3 + .... +tn =Ihr Eqns. (i), (ii) and (iii) are called' equations ofmotion' and
i. e., 2 2 2 are very useful in solving the problems of motion along a
3+Jf+ .... +y;-=lhr
straight line with constant acceleration. Regarding these
equations it is worth noting that:
or ~[1+!+_12 +.... +_I_]
n 1
=lhr (I) These equations can be applied only and only when
3 3 3 3 -
acceleration is constant. In case of circular motion or

~[I-(1/3)"l=lhr 1 simple harmonic motion as acceleration is not


or or 1- ( -3 )" .=1 constant (due to change in direction or magnitude) so
3 1- (113)
these equations cannot be applied. When acceleration
or i.e., n = oo is not constant, integration of acceleration will give
velocity and integration of velocity will give
So the bird will make infinite trips.
.displacement (See solved Problems 7, 53 and 54). In
(b) The distance moved by the bird in I trip =vt l :<:: 60x such cases velocity can also be calculated by using
(2/3) = 40 km and the distance moved by the bird in law of conservation of energy.
Illrip=v" =60x (2/3')=(40/3) km. (2) For constant acceleration"alt, vlt and sit curves in the
light ofEqns. (i) and (ii) will be as shown in Fig. 6.15
So the total distance moved by the bird,
(a), (b) and (e).

c ,~~,lL,
40[1+~+ _1 + .... 00]=401LI-(113)
3 3'
I ] = 60km.

Note: As the speed of bird is 60


kmlhr and the two trains crash after a
I hr, so the total distance travelled by the bird is the distance
travelled by the bird in 1 hr, i.e.,
(al (bl (el
d= 60(:) xlhr =60 km Fig. 6.15

which is same as above.


(3) Actually first and second equations of motion are we have 0= u 2 -2(Ffm)s
vector equations while third one is a scalar equation,
mu'
i.e., i.e., s=--
~ ~
v = u+ at
~
.... (iv)
2F
,
But KE = !mu 2 and alsoKE =L
~ -j. 1-+2 2 2m
and s=ut +- at .... (v)
2 mu2 KE p2
are vector equations while so s=--=-=--
2F F 2mF
-j. -j> -+ -+ -j> -+ 2 2 -t -t From this it is clear that:
v·v = u·u + 2a·s or v = U +2a'5
is a scalar equation. (a) Ifuissame:
2
soc (mu I2F), i.e., socm
(4) If the velocity and acceleration are collinear, we
conventionally ~'ake the direction of motion to be Now as mass of car is less than that of truck, so car will
positive and so variables in the direction of mation are stop in shorter distance.
taken to be positive while opposite to it negative and
(b) If KE issame:
so equations of mati on become simply
soc (KEfF)
v ou+at } {v =u- at so both will stop after travelling same distance.
s=ut+iat2 or s = ut - ~at2 ... (vi) (c) If pfssame:
v 2 =u 2 +2as v 2 =u 2 _2as s:=(p2/2mF), i.e., soc (11m)

as the acceleration (which is of course constant) is Now as mass of truck is more than that of car, so truck will
parallel or antiparallel to the motion. stop in a shorter distance.
(5) If acceleration and velocity are not collinear, v and s Note: . As s = (mu2/2F), so for a given body if braking force remains
can be calculated using Eqos. (iv) and (v) respectively unchanged,
and law of parallelogram of vector addition, e.g., from
(as m is consti.]
Eqn. (iv)
i.e., if the speed ofa moving body is made ntimes, the stopping
v := [u 2+ (at)2+2uatcosS]112 distance will become n2 times.

with tan~ := atsinS .... (vii) Problem 7. (a) Ifthe initial velocity ofa particle is u and
u+atcosS collinear acceleration at any time t is at, calculate the velocity
From this it is clear that if force (or acceleration) acts ofthe particle after time t.
on a particle at angle S to the direction of motion, the (b) A particle moves along a straight line such that its
particle neither moves displacement at any time t is given bys = «(3 _ 6t 2 + 3t + 4)m.
in its initial direction What is the velocity of the particle when its acceleration is
of motion nor in the v zero? (CBSE 1994)
direction of force (or
Solution: (a) By definition acceleration = (wid!)
ace.). It moves in a
direction other than so ·w := at (given)
these; that too changes Fig. 6.16 dt
with time as shown in or I~w=I~at
Fig. 6.16.
Question VIII. A truck and a car are brought to a hault or v - u ='! at 2
2
by application of same braking force. Which one will come to
stop in a shorter distance if they are moving with same (a) I ,
or v=u+ - at
velocity (b) kinetic energy and (c) momentum? 2
Answer: By applying brakes the body is brought to rest, (b) As according to given problem,
so v = Oand a := (- Flm) (as it is retardation). Ifs is the distance s=t 3 _6t 2 +3t+4
travelled in stopping (called stopping distance), from 3rd instantaneous velocity
equation of motion,
v=ds=3t 2 _12t+3 .... (i)
dt
and acceleration Solution: From 3rd equation of motion for take off,
2
dv d s
a=-=-=6/
dt dt 2
- 12 .... (ii) [sox sf
5
1
2
= 0 +2a x 100

So acceleration will be zerowhen6t -12 =0, i.e., t = 2sec i. e., o =(200/81)mI,' .... (i)
And so the veloc~ty when acceleration is zero, i. e., at t = 2 Now if F is the required force, for take off
sec from Eqn. (i), will be
F- /;?: ma
v =3x2 2 -12x2+3 =-9m1s or F;?: ma +).tmg [asf =fJmg]
[Negative velocity means that body is moving towards the
origin, i.e., as time increases displacement decreases.] or F;?: 10 4 [200 +0.2X9.S]
81 .
Problem 8. A particle starts moving/rom the position of
rest under a constant ace. If it travels a distance x in t sec, or F;?:4.43xI0 4 N
what distance will it travel in next t sec? So the minimum force required by the engine of plane for
Solution: As ace. is consu., from 2nd equation of motion, take off is 4.43 x 10 4 N.

i.e.,s=ut+kat2 we have Problem 10. A perticle moving with velocity equal to 0.4
mls is subjected to an acceleration of0.15 mli for 2 sec in a
1 , direction at right angle to its direction of motion. What is the
x=-at [a,u=O] .... (i)
2 magnitude ofresultant velocity?
Now if it travels a distance y in next t sec., the total Solution: In vector fonn, 1st equation of motion is
distance travelled in (t + t = 2/) sec will be x + y; so -> -> ->
v = u+ at
1 ,
x+Y=2a(2t) .... (ii)
so v = ~u2 + (al)2 +2u(at)cos9
Dividing Eqn. (ii) by (i), Here u =.0.4 mls; a = .0.15 rnls 2; t =2sand 9 =9.0 0
X+Y =4
x
or y=3x so u=~[(O.4)' +(0.15x2)' +0]=0.5",1,

Alternative Solution: From 2nd equation of motion, we Problem ll. At the instant the traffic light turns green a
have car starts with a constant acc. 2 mli. At the same instant a
1 ,
truck, travelling with a constant speed of 10mls, overtakes and
x=-al .... (Hi) passes the car. (a) Ho~ for beyond the starting point will the
2
car overtake the truck? (b) How fast will the car be travelling
The velocity of particle after time 1 from 1st equation of at that instant? (c) Draw sIt curves for each,vehicle.,
motion will be Solution: Let the two 120 -------t--------:----- --,,
, ,
v=O+at, i.e., v=at vehicles meet after time t. 1100 -------1-------+-- ---1
Now for next t sec it will be the initial velocity; so again Then from 2nd equation of : : : :
E 80 ------- ~------- - +---1
from 2nd Eqn. of motion, . 1 2
mOtion, i.e.,s=ut+-at , .!:
' ct'
: .. ~'::
, I

2 Q) : " , : :

i.e., s=ut+~at2 g
the distance travelled by car !!! : : : :
40 ------- ---- --..... ---..---.
III ' " I
1 , '- : v,.~:::
sc =-x2t
C ,
, , ,
, ,
we have, 'J= (at)! + lat 2 =l at 2 .... (iv) 2 : : :
2 2
[a,u = O] .... (i) o 4 8 10 12
Dividing Eqn. (iv) by (iii), llme in sec, _
And distance travelled
(ylx) =3, ;.~, y=3x Fig. 6.17
by truck
Problem 9. An aeroplane requires for take offa speed of sr=lOxt [as a =0] .... (ii)
8.0 kmlhr on a ground havinglOOm runway. The mass of the
But according to given problem
plane is 1.0 4 kg and coefficient of/riction between the plane
and the ground isO.2 What is the m!nimumforce required by Sc =sr' i.e., t 2 =IOt
the engine of the planefor take off? (g =9.8 mli) i.e., 1=0 or lOs
2
So, (a) The distance travelled by the car in overtaking the I v2 vlv
truck, =-a-+v-- - p -
2 a 2 ~ 2 ~2
2
Sc =10 =100m
(b) The speed of car at t = lOs, from equation v = u + at
or s+2[~ +~]
will be
2 2 2 2
v=O+2x!Q=20mls =!a p t x[a+~] = l apt .... (v)
(c) sit curves for car and truck, i. e., Eqns. (i) and (ii), are 2(a+~)2 a~ 2(a+~)
plotted in Fig. 6.17. Problem 13. A passenger is standing 'd' m away from a
Problem 12. A car accelerates/rom rest at a constant rate bus. The bus begins to move with constant acceleration a. To
a.for some time after which it decelerates at a constant rate pto catch the bus, the passenger runs at a constant speedv towards
come to rest. If the total time elapsed is t sec, evaluate: (a) the the bus. What must be the minimum speed of the passenger so
maximum velocity reached and (b) the total distance travelled. that he may catch the bus?
Solution: If the car accelerates for time 1\ and decelerates Solution: Let the passenger catch the bus after time t.
From 2nd equation of motion, the distance travelled by the
for time 12 then according to given problem,
bus,
1=/1 +1 2 .... (i)
I 2 .... (i)
Ifvrnax is the maximum velocity of the car, from vlt curve, sl =O + "2 at
we have
and the distance travelled by the passenger
v v
Ct. = max andp = max [as slope ofvlt curve gives ace.] s2 = ut + O ....(ii)
II 12
Now the passenger will catch the bus if
So that [! +.!.]P
a
= (1\ + (2 )
v m3x
d + s, = s2 .... (Hi)
Substituting the values of sl and s2 from Eqns. (i) and (ii)'in
Which in the light of Eqn. (i) gives (iii), we get
a~t I 2
d+ - at = ut
v=, (a +~)
.... (ii) 2

i. e., .lat 2 - ut+d =O


Vm~ tan 91 '" ct 2
1
v
tan 92-P
or t = ~[u:.:±,-~:.:u,-2_--=2=
ad::..!l
e, e, a
1<--1, -I- I, -I So the passenger will catch 'the bus if t is real, i. e.,
Fig. 6.16 u2 ;:::2ad or u2·had
Further as area undervlt curve gives the distance covered So the minimum speed of passenger for catching the bus is
in a given time so, ·had.
I apt 2 Problem 14. The driver of a train moving at a speed v 1
s=2"(v max )t= 2(a +P) .... (iii)
sights a goods train a distance d ahead of him on the same
track moving in the same direction with a slower speedv 2 . He
Eqns. (ii) and (iii) are the desired result.
puts on brakes and gives his train a constant retardation a.
Alternative: From I st equation of motion we have: Show that there will be no collision if
v=O+ai l
and O=v-Pt 2 2
d > (V I -v 2 )
which in the light ofEqn. (i) gives
2a
v = a~t/(a + ~) .... (iv)
Solution: For no collision the speed of the t:-ain must be
and from II equation of motion we have reduced to v 2 before the trains meet, i.e. , the velocity of train
I 2 I 2 relative to goods train VI -v 2 should become zero before the
sl ="2 ail and Sz =vt z -"2Ptz
trains meet.
1 2 1 2 Now ifsis the displacement of train relative to goods train
i.e., s=sl +s2 ="2ail +vt2 -2" Pt2 when it comes to rest relative to goods train due to application
of brakes, we have, from 3rd equation of motion, i. e.,
v 2 = U2 +2as, ltV, + 12v2
i.e., 1= .... (ii)
0 2 = (V\ -V 2 )2 -2as (v2I +v2)
1 ·
Substituting this value of t from Eqn. (ii) in (i) and
s= (VI -v 2 )2
or simplifying, we get
2a
llv z - lzv,
The trains will not collide jf
d~(VI - V2)
, L . =
nun ~(vr +v~)
.... (iii)

d:z. s, i.e.,
2a Eqns. (H) and (iii) are the desired results.
Alternative Solution: The collision will be avoided if the Problem 16. A body
velocity of first train v 1 just before touching the other is starting/rom rest slides on
reduced to v 2 (the two trains being at same speed can never
collide). Thus if a is the retardation, the time taken by first train
to reduce velocity from vI to v 2 will be given by
an inclined plane 0/ length
s as shown in Fig. 6.20.
Calculate the time 0/ h
I
v 2 = v I -at, i,e., . t=(v j -v 2 )la
In this time the train will move a distance d 1 such that
v ,2 =v ' 2' d1 = (vI' -v ')/
descent and speed at the
bot~om. Find also (a)
distance covered in halfthe Fig. 6.20
1
I - adl' I.. e., 2 2a time ofdescent and (b) time
During the same time, the other train will move a distance taken to cover halfthe distance.
d2 =v 2 t=v 2 (v\ - v 2 )la Solution: When a body slides on an inclined plane,
component of weight along the plane produces acceleration
If the initial distance between the two trains is d, the
mgsin9 .
collision will be avoided if a= = gsm e = constant
m
, ,
vI -v 2
d 1 <d2 +d
v2(v \ - v )
2
So, equations of motion can be applied. From 2nd
i. e., :5 d + ~-'-'-~ equation of motion, here
2a a
s=0+~(gSine)[2 [asu = O] .... (i)
which on solution gives d ~ (vI - v 2 )1 12a.
Problem 15. Two particles A and B move with constant i.e., ~ I f2h as s =-,!!-] .... (ii)
wlocities v J and v 2 along two mutually perpendicular t = V~=sin9Vg [ smB
straight lines towards the intersection point 0. At moment t = 0
While from 3rd equation of motion,
the particles were located at distances 'I and '2 from 0
respectively. Find the time when they are nearest and also this v 2 = 0 2 +2(gsin9)s
shortest distance. or v=~2gs(sine)=~2gh [ass sin 9 =h] .... (iii)
I· I, -10
Solution: As shown A

I
Eqns. (ii) and (Hi) are the desired result. Further as from
in Fig. 6.19, in time t, A
will move a distance v I t
A'
" Eqn. (i) s cc [2, so
L
while B,v 2t; so after
time t the distance of A
and B from 0 will be
B'
I, (a)lf
Its'
I =2:' '=(-1
tl2
)'
' i. e" s' = §..
4

1
i.e., in halfthe time ofdescent the body covers only one..Jourth
(II -vIi) and (12 -v 2t)
respectively. So the of the total distance.
"
distance between them
L at time t will be given
B (b)If s,=~, (S~) = C;y, i.e., t' =-'- =-0.7t
,f2
Fig. 6.19
by
i.e. , in about 0.7 times the time o/descent the body covers half
L2 =(/1 _Vlt)2 +(12 - V2t)2 .... (i) of the total distance.
Differentiating Eqn. (i) with respect to time, Problem 17. An inclined plane making an angle of300
dL with the horizontal is placed in a uniform horizontal electric
2L dl =2(11 - v 11)(- v 1) +2(1, -v,I)(--v,) field E of 100 Vim. A particle ofmass 1kg and charge 0.0 I coul,
is allowed to slide down/rom rest/rom a height of1 m. If the
Now for Lto be minimum (dLldt) =0 coefficient 0/friction is 0.2. find the time taken by the particle
so , +v2')t=l,v, +12v2
(VI to reach the bottom. (g ::;: 9.8mli )
Solution: The forces acting on the particle are:
18 = Ox3+~ax32
(a) its weight mg acting vertically downwards
(b) electric force qE horizontally in the direction of field i.e., a =4m1s2 .... (i)
(e) reaction R nomlal to the inclined plane Now if B is projected up the plane with a velocity U. it will
(d) friction 1(= 1-tR) along the plane upwards go up the plane till its velocity becomes zero (i.e., v =0) as for
So, resolving the foeces R it acc. will be 4 mls 2 down the plane. So from 1st equation of
perpendicular to the plane and motion, time taken to go up the plane,
along the plane, we have 0EC"_'' 'O' 'O"''Vc1:/rn T
1m
O= u - at, i.e., II = (u l a) .... (ii)
R =qEsin9 + mgcosS .... (i) and from 3rd equation of motion distance moved up the plane
and mgsiu9 - ~ - qEcos8
= rna .... (ii)
L-"3",O,--'_-::-_m-:-g:::----.J
Fig. 6.21
1 2
O=u -2as, i.e., s=(u 2 / 2a) .... (iii)
Now the body B will slide down the. plane and so time
Substituting the value of R from taken by it to slide a distance s down the plane starting from
Egn. (i) in (ii), and simplif)dng_for ace. ' a' rest at C, from 2nd equation of motion,
a = g(sin e - Jlcos8) - qE ().lsin8 + cos9)
m s=o+!ati I: e., t2 = (2,;
2 V~
So here, a =9.8[~ - 0.2x ~l so t2 = ~3.xu2 =~=tl .... (iv)
a 2a a
O.OI XIOO[O.2X!+ .J3l ~2.3m1S2
1 2 2 [as fromEqn (iii), = ;: 1
As ace. is constant, from 2nd equation of motion, i. e.,

s = Oxt+~at2,
t_ (2h _
i,e.,

'"'2""x-'-l-
t=t i.e. ,
But it is given that body B reaches the ground with A,

tl + t2 =3 or 2: =3

[as from Eqns. (ii) and (iv), t1 = t2 = ul a]


V~ 2.3 x (1/2)
or u = -3 a = -3 x 4 = 6m1s [as from Eqn. (i), a = 4]
2 2
i.e., t= ~ '::: 1.3s
'12.3 So the acceleration ofB is 4 mls 2 down the plane while its
velocity is 6 mls up the plane. [The distance travelled by B up
Problem 18. A mass 1. is released from the top of a
frictionless inclined plane 18 iii long and reaches the bottom 3 the plane ·from Eqn. (iii), i.e., s =-(u 2I2a) = 4.5 m] .
sec later. At the instant when A is released, a second ma~s B is § 6.5 Motion Under Gravity
projected upwards along the plane from the bottom with a IAI Ideal Motion
certain initial velocity. The mass B travels a distance up the
The most important example of motion in a straight line
plane, stops and returns to the bottom so that it arrives
with constant acceleration is motion under gravity. In case of
simultaneously with A. The two masses do not collide. Find the
motion under gravity unless stated it is taken for granted 'that:
acc. and initial velocity of B. (How far up the inclined plane
does B travel?) (l) The acceleration is constant, i.e.,
Solution:
inclined
As the
plane is
t = g:= 9.8 mls 2 and directed vertically downwards.
frictionless, the only (2) The motion is in vacuum, i.e., viscous force or thrust
force along the plane is the of the medium has no effect on the motion.
component of weight, i.e. , Now in the light of above assumptions, there are two
mgsin9. So acceleration possibilities :
down the plane (AI) Body falUng freely under gravlty*
=
a g sin 9 = constnat and Fig. 6.22 Taking initial position as origin and direction of motion
so equations of motion can be applied here. So, applying 2nd (i,e" downward direction) as positive, here we have
eqn. of motion, keeping in mind that mass A starts from rest,
u=O [as body starts from rest]
• If body is projected vertically down with velocity u, same theory will apply, but now u;t: O.
a = +g [as acc. is in the direction of motion] So, if the body is projected with velocity u and reaches the
So iftbe body acquires velocity v after falling a distance h highest point at a distance h above the ground in time I , the
in time I, equations of motion, viz., equations of motion viz.,

v =u+at; $=ul+~at2 and v 1 =u 2 +20s v =u +al, s=ut+(~}[z and v 2 =u 2 +2as

t
reduce to v =gt .... (i), h = gt 2 .... (ii) and v 2 = 2gh .... (Hi) reduce 10

O=u - gt, h = ut --I gt 2 and o = u2 -2gh


These equations can be used to solve most ofthe problems 2
of freely falling bodies as if:
Substituting the value of u from first equation in second
and rearranging these,
u =gl ....{i). h={II2)gl' .... {ii) and u 2 =2gh .... {iii)
tisgiven Ii isgiven v isgiven 1:hese equations can be used to solve most of the problems
From Eqns. (i) From Eqns. (ii) From Eqns. (iii) of bodies projected vertically up as if:
and (ii) and (iii) and (i)

v=gt I=ff t =E.


g
I
t isgiven h isgiven u is given
I 2
andh=-gt V = .J2gh v2 From Eqns. (i) From Eqns. (ii) From Eqns. (iii)
h=-

V
l £
2 2g and (ii) and (iii) and (i)

lL, I:CI
u=gt I=ff t=!!.
g

u = .J2gh u'
h=-
o 2g

~kl
(a) (b) (e) u
Fig. 6.23 + (u/g) (2u/g)
0 I
Here it is worth noting that: - g
-u
(i) If the body is dropped from a height H, as in time t it
has fa llen a distance h from its initial position, the (a) (b) (e)
height of the body from the ground will be Fig. 6.24

h' =H- h with h=(~)gt2 Discussion


From case (AI) and (A 2 )!1 is clear that:
2 (I) In case of motion under gravity for a given body.
(ii) As h = (1I2)gt , i.e., h oc , 2, distance fallen in time I,
mass, acceleration, and mechanical energy remain
21,3!, etc. will be in the ratio of12 :22 :3 2, i.e., square
constant while speed, velocity. momentum, kinetic
oi integers. energy a nd potential energy change.
(iii) The distance fallen in the n th sec,
(2) The motion is independent of the mass of the body. a.s
h() - hI I) =! g{n)2 _! g{n_ I) 2 =!g{2n- l) in any equation of motion, mass is not involved. This
n n- 2 2 2 is why a heavy and light body when released/rom [he
So distance fallen in 1st, 2nd, 3rd sec, etc., will be in same heighl, reaches the grou11d simultaneously and
the ratio of I : 3: 5, i.e., odd integers only. with same velocity, i.e.•
(A z ) Body is projected vertically up I = .J{2hlg) and v=.J2gh
Taking initial position as origin and direction of motion However, momentum, kinetic energy or potential
(i. e., vertically up) as positive, here we have v = 0 [as at highest energy depend on the mass of the body (all oc mass).
point velocity = 0]
(3) As from case (A 2 ) time taken to reach a height h,
a = -g [as acc. is downwards while motion upwards]
'U =.J{2hlg)
and from case (AI) time taken to fall down through a So the body will be in neutral equilibrium, i. e., it will
dislanceh, remain at rest or in uniform motion as the case may be.
'D =.J(2hlg) (c) If 0< p, then as thrust will be lesser than weight, net
force will be downwards and will be
so 'U ='D =.J(2hlg) F=mg=mg-Th

g'=g-~={l-~)
So in case oCmotion under gravity time taken to go up
is equal to the time taken to jall down through the or .... (Ui)
same height.
i. e. , now the body will be accelerated down with acc.
(4) If a body projected vertically up reaches a height h,
g'=g(t-(cr/p)] which is lesser than g, i.e., the
then from case A 2,
effect of thrust here is to decrease acc. due to gravity.
u=.J(2gh)
From Eqn. (iii) it is clear that:
and if a body falls freely through a height h, from case (1) Motion under gravity is still independent of shape,
Al size and mass of the body. Now
v=.J(2gh)
So in case of motion under gravity, the speed with which a
I=!! and v=.J2gh with g'={l-~)
body is projected up is equal to the speed with which it comes (2) Greater the density of body P, greater will be the
back to the point ofprojection. acceleration g' and so lesser will be the time of
[BJ Effect of Medium on Motion unojer Gravity descent and greater will be the velocity. So if a body
Medium affects the motion of a body falling freely under of high density (say stone, iron piece or solid body)
gravity due to thrust and visccus drag. and a body of low density (say wood, feather or
(B.) Effect of Thrust hollow body) are allowed to faU simultaneously from
the same height, the body of higher density reaches
If a body of volume V and density p is dropped in a
the ground first with greater velocity.
medium of density cr, thrust will be Vag and will oppose the
(3) Greater the density ofthe mediuma. JesserwilJ be the
weight. So there are three possibilities:
acceleration g' and so greater wiJi be the time oj
(a) Ifo < p, thrust will be greater than weight and so there descent and lesser will be the velocity. So if a body
'fill be a net force (Th - W) opposite to gravity and so falls through same distance in air and in water, it takes
the body will experience an acceleration longer and acquires lesser velocity in water than in
g' = Til - W opposite to gravity air.
m (B z) Effect of Viscous Force·
Th g = _
g'= __ V<1g_ g [asW = mgandm =Vp] When a spherical body of radius rfalls with velocity v in a
i.e.,
m Vp medium of viscosity fl, the viscous force opposing the motion,
according to Stock' s ~law is given by 6mVv** .
or .... (i)
This force is velocity dependent and non~conservative. So
when a spherical body falls freely in a medium (0< p) due to
So if a body is at rest it will move opposite to acc. due force mg, the velocity will increase and as velocity increases
to gravity (as in the case ofa hydrogen filled balloon) the viscous force (0::'1) will also increase. After some time a
and if the body falls through a liquid with an initial stage will be reached when the viscous force exactly balances
velocity, the motion will be retarded [with relardation the net downward force mg. Then the body will fall with a
given by Eqn. (i)] so the body will stop after travelling constant velocity called terminal velocity [See solved Problem
some distance and finally will move up [with acc. 31]. Thus, ifvr is the terminal velocity,
given by Eqn. (i)] (See solved Problems 29 and 30).
61t11rvr =mg, i.e., vr = mgl61t11r
(b) If a =p, the net force 3
But for a spherical body, as m = (4/3 )1tr p,
F =W - Th =Vpg - V<1g =0
or mg' = 0, i.e., g' =0 .... (ii) ur =~r2 pg .... (iv)
9 ~

• Discussed in detail in §1 !.2 (8).


2
** However. for large objects moving at high speed through air such as aeroplane or sky divers the drag force is proportional to v .
However, if along with v stone straight down with the same initial speed and from the
viscous force thrust is also same position. Find the ratio of the speed the stones would
VT ------------ --------
taken into account which is have attained when they hit the ground at the base ofthe cliff.
usually the case, by replacing Answer: As the stone thrown vertically up will come

g->g[I-~] TIm,
back to the point of projection with same speed, both the stones
will move downward with same initial velocity, so both will
Fig. 6.25 hit the ground with velocity
in Eqn. (iv), we get
v 2 =u 2 +2gh, i.e., v=~(u2 +2gh)
V
T+,[p~o ]g .... (v)
So the ratio of speeds attained when they hit the ground is
1 : 1.
This is the required result and from this it is clear that:
[However, the stone projected up will take (2u /g) time
(1) For a given density of body and medium more to reach the ground than the stone projected downwards.]
v T oc r2 Question XI. From the top of a building, a ball is
i.e., greater the radius ofthe body, greater will be the dropped while another is thrown horizontally at the same time.
terminal velocity. This is why very small objects such Which ball will strike the ground first?
as dust particles and water droplets have very low Answer: Both the balls will reach the ground
terminal speeds and appear to float in the sky. simultaneously as horizontal velocity does not affect the
Furthennore, as v IX: r2, ifradius of the body becomes vertical motion.
n times, the tenninal velocity will become n 2 times. i. e.,
(2) For a given radius and medium, greater the density of (However, for the ball dropped vertically VI = ~(2gh)
a body greater will be the terminal velocity as
while for the ball projected horizontally:
vr oc(p-cr)
This is why the terminal velocity of iron (or solid) (v H )2 = uand(v v )2 =~(2gh) sothatv 2 = ~(u2 +2gh)
sphere is more than a wooden (or hollow) sphere of i. e., on hitting the ground speed of horizontally projected ball
same size in the same medium. Furthermore, if the will be more than the ball dropped vertically]
density of a spherical body becomes n times, terminal
Question XII. A block slides down on a smooth inclined
velocity will become more than n times as
plane when released from the top, while another falls freely
11 (np-o) (np-no)+(n - l)<> from the same point. Which one ofthem will strike theground:
v (P-o) p 0 (a) earlier (b) with greater speed?
11 (n ·-1)<> Answer: We know that in case of sliding motion on an
i. e., -=n+ >n
v (p 0) inclined plane [See solved Problem 16]
(3) For a given spherical body (i. e., rand p constt.),
greater the density of medium lesser will be the
IS = si~ef!- and Vs = ~(Zgh)
terminal velocity. This is why a given body has while in case of free fall
smaller terminal velocity in water than in air.
Question IX. The statement given below is true or false? tF = f!- and v F =~2gh
Give reason in brief
"Two balls of different masses are thrown vertically (a) (t F /t s )=sin9<1, i.e., tF<ts,i.e., falling body
upwards with the same speed. They pass through the pOint of reaches the ground first.
projection in their downward motion with the same speed. " (b) (v Flv s ) =1, i.e., v F = V s' i.e.,both reach the ground
Answer: The statement is true as motion under gravity is with same speed (not velocity, asfor falling body direction is
independent of mass of the body and as body comes back to the vertical while for sliding body along the plane downwards).
point of projection with same speed Problem 19. A ball is projected vertically up with an
i.e., VI = u 1 and ';v 2 = u2 initial speed of20 mls on a planet where acceleration due to
Here u1 = u2 = u (given) so vI =: v 2 =u gravity is 10 mls 2 . (a) How long does it take to reach its
Question X. A man standing on the edge ofa cliffthrows highest point? (b) How high does it rise above the point of
a stone straight up with initial speed u and then throws another projection? (c) How long will it take for the ball to reach a
point 10 m above the point ofprojection?
Solution: As here motion is vertically upwards, finished in 1minute and it continues to move up. (a) What is th~
a=-g am! v=O maximum height reached? (b) After how much timefrom ther,
(a) From 1st equation of motion, i.e., v = u + at will the maximum height be reached? (Take g = 10 mls2)

0=20 - 10/, i.e., t =25 Solution: (a) The distance travelled by the rocket durin~
2 burning interval (I minute = 60 s) in which resultant acc. il
(b) From 3rd equation of motion, i.e., v2 = u +2a8,
vertically upwards and 1(I mls 2 will be
0=(20)2 -2xlOxh, i,e., h =20m
hi = Ox60+012)xlOx60' = 18000m .... (1.
(c) From 2nd equation of motion, i. e., s = ut + 1.. at 2 , And velocity acquired by it will be
2
v=0+10x60 =600mls .... (ii:
10 =20/ -(112) x 10 x /'
Now after 1 minute the rocket moves vertically up witt
i.e., ,2 -4/+2=0 initial velocity of 600 mls and acc. due to gravity opposes itl
motion. So, it will go to a height h2 till its velocity becomel
or t =2 ± J2, i.e., t = 0.59 s or 3.41 s
zero such that
i.e., there are two such times, as the ball passes twice through
h = 10m once when going up and once when coming down. 0 = (600)' -2gh,
Problem 20. Ajugg/er throws balls into air. He throws 2 ,... (m
or h2 = 18000m [asg = lOmls ]
one whenever the previous one is at its highest point. How high
So from Eqns. (i) and (iii) the maximum height reached b~
do the balls rise if he throws n balls each sec? Acceleration
the rocket from the ground,
due to gravity is g.
H=h 1 + h2 =18+18=36km
Solution: As the juggler is throwing n balls each second
and 2nd when the first is at its highest point, so the time taken (b) As after burning of fuel the initial velocity from Eqn
by one ball to reach the highest point, (ii) is 600 mls and gravity opposes the motion of rocket, s(
from I st equation of motion time taken by it to reach thl
t = 01n)5
maximum height (for which v =0)
and as at highest point v = 0, from 1st equation of motion,
0 = 600-g/
O=u-(g)(lIn), i.e., u=(gln) .... (i)
i. e., t =60s
2 2
Now from 3rd equation of motion, i.e., v = u +2as, i.e., after finishing fuel the rocket goes up for 60 s, i.e.,
0=u 2 -2gh, i.e., h = (u 2/2g) minute more.

or [as from Equ.(i), u = -!] Problem 23. A body is released from a height and fall.
freely towards the earth, Exactly I sec later another body i,

Problem 21. A pebble is thrown vertically upwards from .


after the release of the second body, if g =9.8 mls ?
,
released. What is the distance between the two bodies 2 se.

a bridge with an initial velocity of4.9 mls. It strikes the water


Solution: According to given problem 2nd body falls fa
after2s. If ace. due to gravity is 9.8 mls2(a) what is the height
2 s so that
ofthe bridge? (b) with what velocity does the pebble strike the
water? h, =!g(2)'
2
Solution: Taking the point of projection. as origin and
downward direction as positive, while I st has fallen for 2 + 1 = 3 s; so

(a) By 2nd equation of motion, i.e., s = ut+(~}t2 we


hi =!g(3)'
2
.... (n

:.Separation between two bodies 2 s after the release of 2",


have
body,
h =-4.9X2+(~)9.8X22 =9.8m d=h l -h, =~g(32 -2')=4.9x5 = 24.5m
(u is taken to be negative as it is upwards.) Problem 24. Ifa body travels halfits total path in the la~
(b) By 1st equation of motion, i. e., v = u + at second ofits fallfrom rest, find: (a) the time and (b) the heigJ
v = -4,9+9.8 x2=14,7m1s of its fall. Explain the physically unacceptable solution of th
Problem 22. A rocket is fired vertically up from the quadratic time equation. (g :::: 9.8 mls2) [lAS 199~
ground with a resultant vertical acc. oflO mls 2. Thefuel is
241
Solution: If the body falls a height h in time t, from 2nd Now, as after impact the velocity ofthe body is horizontal,
equation of motion we have so time taken to reach the ground
h=~gt2 [u=Oasthebodystartsfromrest] ... (i)

Now, as the distance fallen iri (t -I) s will be


I, =J¥
So total time of motion
h'=!g(t-I)' ",,(ii)
2
1=11 +1, =H(.Jh+~(H-h)]
So from Eqns. (i) and (ij) distance fallen in the last second

h-h'=i gl ' -ig(t-l)' For t to be maximum (dtldh) =0

ie, h-h'=ig~l-l)
i.e., ~[H{hll2 +(H _h)II2)]=O
But according to given problem as (h - h' )=hI2 or 1 h- l12 +! (H - h)-112 (-I) =0
2 2
i.e., O}=G)g(21-1)
or h=H - h, i.e.,
or O)g12 =g(21-1) (as from Eqn. (i)h=O)gl']
Problem 26. A stone is dropped into a well and the sound
or (2 -4t+2=0 of impact of stone on the water is heard after 2.056 sec of the
release of stone from the top. If acc. due to gravity is 980
or I = (4±-f(4' -4x2)]/2 cm/sec 2 and velocity of sound in air is 350 mis, calculate the
or 1= 2±.J2 depth ofthe well.
or 1=0.59s or 3.41s Solution: Ifthe depth of well is h and time taken by stone
to reach the bottom t l , then
0.59 s is physically unacceptable as it gives the total time t
taken by the body to reach ground lesser than one sec while I 2
h='lgll .... (i)
according to the given problem time of motion must be greater
than 1 s. and time taken by sound to come to the surface
so t = 3.41s h
I, =350 .... (ii)
and h =~ x (9.8) x (3.41)' =57 m
But according to given problem
Problem 25. A body fallingfreely from a given height H tl + 12 = 2.056 .... (Hi)
hits an inclined plane in its path at a height 'h', As a result of
Substituting h from Egn. (i) in (ii) and then t2 from Egn.
this impact the direction of the velocity of the body becomes
(H) in (iii), we get
horizontal. For what value of (hlH) the 'body will take
maximum lime to reach the ground? IMNR 19901 gIl
,
~iH-h
t,,
r----r or
'I + 700 = 2.056

98tl2 +7000/1 -14392=0

~~~~------- ' Hi
or 14tl2 +10001] -2056=0
////¥ h or 14/]2 -281 1 +1028/] -2056=0
......~ _______i ___ _ or 14/1(1 1 -2)+1028(11 -2)=0
Fig. 6.26 or (141 1 +1028)(1 1 - 2)=0
Solution: In accordance with Fig. 6.26, time taken by the so 11=2 or 11 = - (1028/14)
body to strike the inclined plane Now as negative time is not physically acceptable, so tl =2s

'I =l(Hg-h) The depth of well h = ~ x 9.8 x 22 =19.6 m


":,
Note:' Ifwe substitute v~lUes oft! and ti fto'PJEqns. (i) and (ii) in (iii) in time another identical ball is dropped from a height of 98
tenus of hand try to solve the qU,adratic equati~n for h, it becomes
metre to fallfreely along the same path as thatfollowed by the
very difficultto fitid:h. Trr; if you can! first ball. After some time the two balls collide and stick
together andfinally fall to the ground. Find the time offlight of
Problem 27; A pj~de ofwo~d OImass 0,03 kg, is ;dropped the masses. (g = 9.8 mls2)
from the top ala buildinglOOm high. At th~same time, a bullet Solution: Let the two balls collide after time t (Fig. 6.27).
a/mass 0.02 kg isfired vertic,ally upward with d've/oeity of 100 The distance fallen by 1st ball
m/s from the ground. The bullet gets "embedded in ~he wooden 1 ,
piece after striking :"i/:" ,Pinc( the :' he~gh~ i9 which the hi =2g1 .... (i)
combination risesa,bove the: puil{ijng be/lj re,Jalling./rt!ely.
while the height reached by the 2nd ball
(g=9.8ml,') . ,' .' .' ,. ' . .· • . ' · 'MNR1994'
1 ,
Solution:
Let the two -::. . hz =ut-'2gt .... (ii)
;bodies collide after -'time t _
(Fig. 6.28) the distance fallen
so hI +hz =ut ,
by the wooden piece, by 2nd But according to given Problem
equatio.n -of motion, hi +h z =98m and u=49m1s
1 , so 98 = 49t, i.e., t =2s
hi =2g1 .... (i)
i,e., the two balls will collide after 2 s. So at the time of
and height reached by~bullet, B collision,
1 2 .; .-
h2 =ut--gt ..... (11)
2
But according to given' problem ' . For 1st ball For 2nd ball
hi +h z =1~9m : , and u = lQOmls moving down moving up
so . loo=ioOx' t; , i.. e., t=ls .- v = gt =19.6 mls v = u - gt =49 -19.6 =29.4 mls
i.e. , the two wilt't6llide after ·1--sec.'-So at the' time of collision and hi =igtZ = 19.6mandhz=ut-4gt2=98-19.6=78.4.m
.. I ' .
I -Now by conservation of linear momentum, taking upward
direction to be positive,
For wooden piece For bullet
0.1 x 29.4-0.1 x 19.6=(0.1 +O.l)V
moving down , moving up
i.e., V =4.9 mls
v=gt=9.~nys V=. Ii ~ gl ~ 100 ~ 9 : 8= 902 mI,
, ,- . -' 1 2 "" " ,
So after coll!sion the composite mass moves upward from
and hl=~gtZ '=4.9 rn., h,=ut- - gl =100 ~ 4.9 = 95.1 m a height 78.4 m. Taking the point of collision as origin and
." 2. " downward direction as positive, from 2nd equation of motion,
Now by c~nservation'~fi~~~niomcenhim,; taking,upward '
direction to be positive:. ' , ",' h=ut+(~}tZ
0.02 x 90.2 ~ 0.Q3 x 9.8 ~(0,Q7+0.03) V
i.e., .v = 30.2 mI, i.e., 78.4 = - 4.9t + (~)9.8t2

or [2 -t-16=,0

or 1= "1±:..-J,,,,(1:...:+..:64
2)
2
or 1/ =(3,o,2)'/ (2~ 9,&);;46.53 m '. As negative tiIlJ.e is not physically acceptable,
,- i ".- ' .' '- - _:, ",:, :,r ", ~,,; .- ~
Now as point of c.~ni!:~i91i'js ath l (=:,4: 9ip)"~eIo)y the top of 1=(1+8.06)/2, i.e.• 1=4.53,
building, so the height, to which cO!Dposite body, rises above Now as collision took place after 2s from beginning of
the building =46.53 -4.9 41.63 'm. = motion, so time of flight
Problem '28. A ball of ma;s 100 g is projected vertically T=2+4.53 =6.53,
upwardsjrom the ~round with a velocity of49 mls. At the same
Problem 29. A solid ball ofdensity halfthat a/waterfalls body to reach the floor t = (1/2)!1 and so the velocity with
freely under gravity from a height of 19.6 m and then enters which the body hits the floor (from 1st equation of motion) will
water. Upto what depth will the ball go? How much time will it be
take to come again to the water surface? Neglect air resistance = =
v 0 + g(li2) (1I2)gl, .... (i)
and viscosity effects in water. (g = 9.8 mls2)
Solution: When a body falls
freely under gravity from a height
Ball, U.. O
i
I (a) If d L > d, thrust will be greater than weight and so net
force will be upwards and will be
F =Th-W
h, it acquires a velocity: 9 l! h so mg' = VdLg-Vdg [as Th =VdLg and W =Vdg]
v = ~(2gh) i
I:
1 or g' = g[ ; -1] [asm=Vdj .... (ii)
So here the ball hits the water AIr
surface with initial velocity :-------_-:1-: j __-=--_-_-:::-: i. e., in the liquid acc. will be upwards and constant. So if (' is
U =~(2 x 9.8 x 19.6) =19.6 mls ;:-:-:---:-r-...t:-:::-:-:-:-:-:-
~::~gt:::::t:{::::::.~~~:: the time of motion in the liquid, from 2nd equation of motion

~::::~~:>~~~:~:::::~:::
Now inside water thrust (Vag) 0= vI'- (1/2)g' (t')'
will act opposite to weight
mg(= Vpg) and as density of water i.e., I' = (2v/g') [ast'~O]
Fig. 6.28
(0-) is more (double that of ball p), Substituting the value ofv andg' from Eqns. (i) and Oi) in
thrust will be greater than weight; so net force on ball in water the above
will be (Th - W) directed upwards. So in water, t'= gt1 = tid
mg' = Vcrg - Vpg g[(dL/d) - I] (dL -d)

i. e., g'=g[~-ll [asm=Vp] So the time taken by the ball to reach the point of
projection from the instant of release
1 I,d
i.e., g' = g and upwards [asp=-cr] t2 =1'+/1 = tl + (d -d)
2 L
So, (a) from 3rd equation of motion, taking downward tl dL
direction as positive i. e.,
(dL - d)
0=(19.6)' - 2gh
(b) Outside the water acc. = g and inside water acc.
or h=19.6m = g[(dLI d) - IJ both always directed towards a point on the
(b) From 2nd equation of motion air-water interface. So restoring force F = m x acc. = constt.

0 = 19.6t-(~}t2
And so the motion is oscUla/ory but not simple harmonic
(as for simple hannonic motion restoring force must be linear,
i. e., RF oc y) with time period t 2.
or t =O or 4s
(c) If d = dL,g' = 0, i.e., acc. of the ball inside the liquid
As zero is not physically acceptable, t = 4, i. e., after entering will be zero. So from 3rd equation of motion,
water the ball will come back to the surface in 4 s.
v 2 =u 2 + 2xOxh
Problem 30. A ball of density d is dropped on to a
horizontal solid suiface. It bounces elastically from the i.e.,
sUiface and returns to its original position in time t l • Next, the
ball is released and it falls through the same height before i.e. , the speed of the ball inside the liquid is independent of
striking the sUiface of a liquid of density dL' (a) If d< d L depth and remains constant.
obtain an expression (in terms ofd, tl and dL)for the time 12
the ball takes to come back to the position from which it was
§ 6.6 Projectile Motion (on a Horizontal Plane)
released. (b) Is the motion of the ball simple harmonic? (c) If Ifan object is given an initial velocity in any direction and
d = d L how does the speed ofthe ball depend on its depth inside then allowed to travel freely under gravity, the motion is called
the liquid? Neglect all frictional and other dissipative forces. projectile motion. In case of projectile motion it is assumed
Assume the depth of the liquid to be large. (g = 10 mls2) that acceleration due to gravity is constant and the effect of air
resistance is negligible. Projectile motion is a two-dimensional
Solution: As in time I, the body comes back to the point motion and can be regarded as simultaneous superposition of
of projection after rebounding elastically so time taken by the two motions one horizontal with velocity ucos9 and acc. = 0
and the other vertical with velocity usin9 and acc. = - g.
(~'\
, ' Substitu~ing the value oft from Eqn. (ii) in Eqn. (v), we get
y ,,: ,,: a .. -g
, ' y=(tan9)x- ,g , x' .... (vi)
= a .. 0 + :: t 2u cos S
, ' I
This is an equation of the type y= ax - bx 2 which
U cos e USlnet ! represents a parabola; so the path ofa projectil~'.is a parabola.
~~"L- ____ ,___ x
~, ----+-.-. -~
Horizontal Vertical Alternative equation of trajectory:
• motion motion 2 2 tanS
Fig. 6.29 y = x tan 9 - gx = x tan S - --,gx~-='-'--
Consider a particle projected with an initial velocity u 2u 2 cos 2 S 2u 2 sin9cos9
from origin 0 at an angle 9
with the horizontal. (see~ t
~
y=xtan9 [1- ~J
Fig. 6.30) ~~
General equations for O~'==:Xx===------'-
I where
2
R=2u sin9cos9
g
horizontal motion: Fig. 6.30
[BJ Time of Flight T
U
x = ucos9
vx=ux +0(t)=ux=ucos9 .... (i) The total time taken by the projectile to go up and come
i. e., horizontal component of initial velocity is constant at all down to the same level from where it was projected is called
instants. time of flight. From 1st equation of motion, i.e.,
v = 'I+at
x = uxt + ~ axt 2 = ucos9t + ~ (0)t 2
Time taken to reach the highest point (v =0),
= ucos9t .... (ii) O=usin9-gt, i.e., t=(usin9/g)
Horizontal displacement = horizontal velocity x time Now as time taken to go up is equal to the time taken to
General equations for vertical motion: come down, so
u y =usin9 T =2t = 2usin9 .... (vii)
ay = -g [considering upward direction +ve] g

vy =u y +a/ = usin9-gt .... (iii) [C) Horizontal Range R


v 2y = uy2 +2a y y=u 2 sin 2 9-2gy ,,,.(iv) It is the horizontal distance travelled by a body during the
time of flight. So by substituting u = ucos 9, a = 0 and t = T =
1 . 2 I 2 (2usin9/g) in 2nd equation of motion,
y=u/+'2a/ =usin9t-2"gt .... (v)
1 ,
i. e., s = ut+ - at
Note: (i) Vertical component of velocity is zero when particle moves 2
horizontally, i.e., at the highest point of trajectory. we get R = ucos9 x (2usinS/g)
(ii) Vertical component of velocity is +ve when particle is
moving up.
i.e., R ~ u2sin29
g
. 9 cos 9 =sm
[as2sm . 29J .. .. (Vlll
... )
(iii) Vertical component of velocity is - ve when particle is
coming down.
[D] Maximum Height H
(iv) Resultant velocity of particle, V = ~v} + vi at an angle
It is the maximum height from the point of projection a
V projectile can reach. The third equation of motion,
tan9=...L
v, v 2 = u 2 + 2as
(v) Displacement from origin, S = ~x2 + ; for vertical motion, i.e., with v = 0 and u == usin 9 and a = - g ,
becomes
Foliowing terms are used in relation to projectile motion:
[A] Trajectory
It is the path followed by the projectile, i. e., a curve joining i. e., .... (ix)
positions ofthe same particle in motion atdifferent instants of
time". ',.,-
" The curve joining position of different particles in motion at same instant is called profile.
Regarding projectile motion it is worth noting that: i.e., 29=90 0 0r 9=45°andR max = (u 2 /g) ",,(x)
(1) The three basic equations of motion, i,e"
i.e., a projectile will have maximum range when it is
v=u+at, projected at an angle of45° to the horizontal and the
maximum range will be (u 2 /g).
For projectile motion give:
When the range is maximum, the height H reached by
2
T =2usin8;R = u sin2e and the projectile
g g
u 2 sin l 9 u2 sin 2 45 0 ul R .
H= = =-=~ .... (Xl)
(2) In case of projectile motion, the horizontal component 2g 2g 4g4
of velocity (ucos9), acceleration (g) and mechanical
i,e., if a person can throw a projectile to a maximum
energy remains constant while, speed, velocity,
distance Rmax' the maximum height to which it will
vertical component ofveiocity (vsinG), momentum,
rise is (Rmax /4) and the angle of projection is 45° (Fig.
kinetic energy and potential energy all change.
6.31 (e)].
Velocity and KE are maximum at the point of
projection while minimum (but not zero) at the (5) For height H to be maximum,
highest point. ul sin l e = max
H=
(3) If angle of projection is changed from e to 2g ,
a' = (90°-9), the range:
2
i.e" sin l 9=max=1, i.e., 9=90°
2 2
R' = u sin2e' = u sin [2(90°-9)1 = u sin2e R
g g g so that H = u , sm
. '90' =_ u' .,.,(xii)
max 2g 2g
remains unchanged. So a projectile has the same
range at angles of projection e and (90°-9), though j,e., a projectile will have height H, maximum when it
time of flight, maximum height and trajectories are is projected vertically (i.e., 9 =90') and maximum
different (See solved Problem 32). This is also why height reached will be (u 2/2g). When the height H is
range of a projectile for 9 1 =(45°-a) and maximum, the range will be
9 = (45°+Ct) is same [equal to (u 2 cos2ct Ig)]. l
2
High
Trajectory
R =u sinlSO°
g
=0 ,.. (xiii)

Further, from Eqns. (x) and (xii),


H ~ = (R"";/21
j, e,.
if a person can throw a projectile to a maximum
(al (bl distance Rffiax[= 'u 2lg with 9 = 45,,], the maximum

Range same for angle of Range same for angle of height to ·:·which he can throw the projectile
projection 9 and 9' ., 90· - 9 projection 9 .. 45~ - Cf. H~ =(R~/ 2)[= (u2/2glwithe = 901.
,,.
,, Rmax. .,,,,
ande'=45·+Cf. (6) In case of projectile motion at highest point, potential
energy wil'! be max and equal to
,, " ,,
,,, Hmax.=Rmax.
,,
,, 2 . 29
,, 45" - -4 (PE) H = mgH = mg u Sill .- .!. mu 2 sin 1 9 .... (xiv)
2g 2
(01
while KE will be minimum (but not zero) and as at the
Range max. for an~le of highest point the vertical component of velocity is
projection e ., 45· zero,
Fig. 6.31
1212122
(4) For range to be maximum,
so,(KE)H = 2"mv =2"lnVf] =2"mu co:> 9 .... (xv)

dR
d9 =0, i.e.,
-,,-[u
d9
Sin2e]=0
2
g so,(PE) H + (KE) H =~ mu 2 (sin 29 +cos 2 9)=~ mu l
which is the ME at the point of projection. So in
i.e., cos 29 = 0 [as(u 2 /g)is fin ite) projectile motion mechanical ellergy is colI.\·c/w:d.
u
Furthennore, Eqn. of.horizontal motion:
~L
PE1 2
= (1I2)mu sin 2 €I = tan 2 a ,,,.(xvi)
[ KE H (l/2)mu 2 cos 2 €I
x=ucos9t .... (iii)
Eqn. of vertical motion:
+ve

So, if6=45 0 PE=KE=G)ME


-h=usin9t-~gt2 .... (iv)
1______
From Eqns. (iii) and (iv),
Fig. 6.33
i.e., if a body is projected at an angle of 45 0 to the gt 2 -2usin9t-2h::::O
horizontal at the highest point, half of its mechanical
energy is kinetic and half potential. or
(7) If in case of projectile motion range R is n times the
maximum height H, i. e.,
Case (iii) Projection at an angle 9 below horizontal:
R=nH
ux :::: ucos9
u siD2S = n u 2 sin 2 9
2
then u :::: -usin9
y
g 2g
ay::::-g
i.e., tan9=[4In] or 9 = tan-l [4In] .... (xvii) Similarly for projection at an angle 9
~L
(8) Weight ofa body in projectile motion is zero as it is a
freely falling body. If equation of trajectory is
downwards with horizontal, the
Eqns. are
• u
+ve

2 Eqn. of horizontal motion:


y =ax - bx
x:::: ucos9t .... (v)
Comparing with equation of trajectory
Eqn. of vertical motion:
then y=xtan9 - g x2 Fig. 6.34
2u cos €I 2 2 -h:::: -usinet - ~gt2 .... (vi)
and, angle of projection €I = tan - 1(a) From Eqns. (v) and (vi)
Some Applications of General Equations or gi 2 +2usin9t-2h::::O
(AI Projection from a height:
Case (i) Horizontal projection or t=
-2usine±~4u2sin29+8ghNegect-verooto
I f
t,
2g
U
x =u
uy = 0 as negative value oft has no meaning.
ay =-g
Note: In an the above three cases, we can calculate the velocity of
Horizontal motion x = ut .... (i) u ~L projectile allhe instant of striking the ground by using
Vertical motion +ve V :::: IV 2 + V 2
'/, Y
-h = o(t) _ ~gt2 ... ,(ii) h V

1~~
tan e:::: .....1:. [angle at which projectile strikes ground]
V,
From Eqns. (i) and (ii)

~ 'R~ [B) Projection from a moving body: Consider a boy


=:> t = fI who throws a ball from a moving trolley. Let the velocity of
Fig. 6.32
ball relative to boy is u.
Horizontal range (R) :::: u x t
-> ->->
V ball, troiley :::: V bal! - Vtrolley
= ufI
-> -> ->
Case (ii) Projection at an angle 9 above horizontal: V ball :::: V ball, trolley + Vtrolley

U;c = ucos9 Above equation shows that absolute velocity of ball is vector
uy =usin9 sum of its velocity with respect to trolley and velocity o'f
trolley. Apply this equation to horizontal as well as vertical
a y =-g
motion of the ball. Now consider following cases:
. . '. ' '. 2 . ~ "2
Hence, R sin p,= R cos", ~~~: Ct. :i_.:~~~,'.pos; p
Case (i): Ball is projected in direction of motion of trolley
Horizontal component of ball's velocity = ucosS"+v \ '" ,. ;2u ,',cos ' a.
. '~ . .' •.,: ' .. "';2 '". ' •... -', '.
Vertical component of ball's velocity = usin €I
. , ") . gll. cos~ i

~· v~
or R(1ana ' -tan.., :=
-' ', '
• , ,, >
2 2 ' 2, '\.
. i" U.,·cos,,·a;:;:·
sin(a-~) :. 'gl<c";~ :'
or
~.os a. cos p;' 2u~ c~.s ~:'Ci:::,
Horizontal component - u cos {} + v Horizontal component .. u COS 9 - v , R = 2~~ '~(~ :: p)~~i~:'
Vertical component .. u sin a Vertlca! component ", u sin e or ,.;. , 2 ' '' , .
Fig. 6.35 Fig. 6.36
-.. . -.
:gCQS p'""
.<. ' . ."
Case (U): Ball is projected opposite to direction of motion of Alternate methodj;',:,:We talCe':~es along "incline and
trolley perpendicularto;IDcline as' showh ::" '
Horizontal component of ban's velocity = uease -v in Fig. 6.40. In this co-o~~inate '. "~ .:.',: ,'+
system component ' .; of ~
Vertical component of ball's velocity = usin €I
accelerations al?ng , the incline : ' ,: x , X ~<:q
Case (iii): Similarly for a ball projected upwards from a and normal to the incline are ' ' " ;,<:!,C§.
upward moving platfonn
ax =-gsinp
Horizontal component of ball's velocity = ~cose
a y =-gcosp, Fig. 6.40
Vertical component of ball's velocity = usin8 + v
U
x =ucos(a - P)
uy=us,i n(q.-Pl " ':-":,

~
When projectile lands at A its y~c9-of(iinate is-zero, i. e.,
. I 2
o = u I+-a t .... (viii)
ass: 2L122 Motion Y ' 2. Y

Horizontal component .. u cos 9 Horizontal component .. u cos e u


Vertical component .. u cos e + v Vertical component., u sin e - v
Fig. 6.37 Fig. 6.38 A
or
Case (iv): For a downward moving platform
Horizontal component of ball's velocity = ucos9 is the time ,o f flight.
Vertical component of ball's velocity = usin9 - v For motion along inclined plane Fig. 6.41
[e) Projection on an incli~ed pla.ne: (x-axis) .. "
,
, . -, 1 2
(1) Range of a projectile on an inclined plane: OA is an x=u t+-a t .... (x)
inclined plane, inclined at Y u
x 2 x

angle p with the horizontal M


A
line OX. Projectile is
projected from origin 0 with R ie:;
,.-
velocity u at an angle a with i~
the horizontal. OA is range of "" ,
0',::':':~"::;,==~==t----'~ ,
projectile along incline. f- R cos ~
Point A where projectile Fig. 6.39
lands has co-ordinates
(Rcos~,Rsin~). "'-
The equation of trajectory is

y=xtana. - g x' .... (vii)


2u 2 cos 2 a
Point (R cos p, R sin p) must satisfY the above equation (vii).
X=UCOS (l )( t .... (xi) Y
A
,, y=xtana - -
gx2
2 (1 + tan a )
2
2u::.:s"in",(a:.,-:-~,,",)
= UCOS(l )( ::; IY
I 2"
gcos~
~:'::~==i'L..1.- x
~x--..j 2u 2 2u 2y
and R=-
x Fig. 6.43 or tan 2 a ---tana ---+1=0
gx gx2
COS~

2u 2 cosa sin(a - ~) If point P is within the range of projectile then roots of above
equation must be real. Complex roots imply that point P is out
gcos 2 ~ of range. By method of completing the square,
(1) Vertical height at which projectile strikes!
tan(l_~)2
( gx
2
=tan2a _2u tan a + 1= 0 .... (xiv)
From equation y= usin w - ~ gt 2 •.•. (xii) gx

, ) 2usin(a
= (uSllla -~) --1{2"Sin(a _ ~»)' from which we obtain
2
gcos~ 2 gcos~ tana =11- .... (xv)
gx
2u 2 cosa sin ~sin(a - ~)
which is the required angle such that the trajt'Ctory just reaches
gcos 2 ~
point P on the envelope of possible trajectories for a given u.
Alternate method :
Question XIII. A hunter aims his gun andjires a bullet
n ucosa)( 2usin(a -~) t " directly towards a monkey sitting on a distant tree. At the
y=xtan,..= )( anI"'
gcos~ instant the bullet leaves the barrel of the gun, the monkey
dropsfrom the tree freely. Will the bullet hit the monkey?
=_
2"_' c::o:::s::a.:::si:::n,<~=si::n("a,--.1~:!.)
Answer: Let the horizontal distance to the tree be x and
gcos 2 ~
original height of the monkey be H as shown in Fig. 6.44. The
(3) Angle of projection for maximum horizontal range: angle of projection 8 will be given by tane = (Hlx).
From the fonnula derived above. Ifthere were no gravity the bullet would reach height H in
2
R 2u sin(a-p)cosa = u2 [sin(2a.-p) - sinPJ the time t taken by it to travel the horizontal distance x
2
gcoS p gcos 2 p i.e., H=(us.ine) xt with t=(xl ucos8)
For R to be maximum sin(2a. - P) must be maximum. However, because of gravity, the bullet has an
acceleration 'g' vertically downwards; so in time t the bullet
Hence 2a - P= nl2
will reach a height
' e.,
R 'IS maximum
, •lor a = • + ~ '4 2
_ (~ ,.,2 ~ H :... ~ gl2
~ I.
y = (usin8) x t
, u2(1 - sin~)
The maxllnum range R = ':"""'--7=
2
gcos P
or ,,'
R malt = ---;,...::-,--:::-
g(1 +sin~)
[as cos' ~ = (1 - sin' P) = (I -sin P)(I + sinp)]
Similarly it can also be proved that range down the incline is
given by

R malt = -,-"-,.--,,,, ,,' ,


,,
g (l -sinp)
(4) Initial projection angle so that particle passes
• 'u cosx ••- ===~'---J;J---''-
01
through a given point P(x, y): From the equation of
traj ectory,
Fig. 6.44
gx'
y = x tan a - --;;"''-:;- .... (xiii)
2u 2 cos 2 a
4
This is lower than Hby gt 2 which is exactly the amount
Problem 32. At a harbour enemy ship is at a distance
ISO./3 mfrom the security cannon having a muzzle velocity of
the monkey falls in this time. So the bullet will hit the monkey 60mls (a) To what angle must the cannon be elevated to hit the
regardless of the initial velocity of the bullet so long as it is ship?(b) What is the time offlight? (c) Howfar should the ship
great enough to travel the horizontal distance to the tree before be moved away from its initial position so that it becomes
hitting the ground. However, for large u lesser will be the time beyond the range of the cannon? (g = 10m ls2)
of motion; so the monkey is hit near its initial position and for Solution: (a) For hitting the ship the range of cannon must
small u it is hit just before it reaches the ground. be equal to the distance of ship from cannon, i. e. ,
Note: Bullet will hit the monkey only and only if
y>O. i.e., H-!gt' >O

or H >4gp, i.e., H>!gu2~s'e


or II >~~ g
cos9 2H
or II> 1...L(x2 +H 2) .
V2H
(Uo)

If II < Ii> the bullet will hit the ground before reaching the
monkey. Fig. 6.46
2
Problem 31. A relief aeroplane is flying at a constant u sin 29 == IS0./3
g
height of 1960 m wiJh speed 600 kmlhr above the ground
towards a point directly over a person struggling in flood
or sin 29 ~ ISO,J) x 10 =./3
water (See Fig. 6.45). At what angle of sight, should the pilot 60 x60 2
release a survival kit if it is to reach the person in water? i.e., 29 =60° or 120°
(g=9.Sm/s') or 9 = 30° or 60°
i. e., to hit the ship the cannon must be elevated at an angle of
30" or 60".
a
(b) As T = (2usin9/g), depending on there are two times
of flight.

t, 2 x60 . 30. = 6 s
= lOxsm

and 12 = 2x60 xsin60" = 6.J3'=IOAs


10
Fig. 6.45 (c) The maximum range of cannon (when = 45°). a
Solution: As the plane is flying at a speed of 600 x (5/ 1S)
R =.!C = 60 x 60=360m
,i.e., (500/3) mfs horizontally (at a height of 1960 m above the max. g 10
ground), the time taken by the kit to reach the ground
And as initially the ship is ISM m, so to become out of
1= (2h = l x 1960 =20s maximum range of cannon, the ship should be moved away
'fi 9S from the harbour from its initial position by at least
360-1 8M =48.6m.
And in this time the kit will move horizontally by
x = ul = (500/3) x 20 = ( 10,000/3) m Note: From this example it is clear that for a given range and initial
velocity there are two angles of projection and two times of
So the angle of sight
flight, i.e. , two trajectories are possible .
Ian $ = o!. = 10000 = J!!.. = 1.7 0 ./3 Problem 33. During volcanic eruption chunks of solid
" 3xl960 5.SS
rock are blasted out of/he volcano. (a) At what initial speed
i.e. , $ =tan - I (./3) = 60· would a volcanic object have 10 be ejected at 37° 10 the
horizonralJi"om the vent A in order tofall at B as shown in Fig.
6.47? (b) What is the time offlight? (g = 9.8 mls" )
T=2usina=u,[i [asa=45~ .... (i)
g g

and range of projectile R


2
u sin 29 =u 2 .... (ii)
g g
According to given problem,
R =500+vT
Substituting the values ofT and R from Eqns. (i) and (ii) in
Fig. 6.47 the above,
Solution: (a) Taking the point A as origin, with upward £= 500 + uJi x20
direction of mation positive and using the equation, g g

s =ut +'!'at 2 or u' -20,[iu-4900=0


2 u = (1/2)[20,[i ± .;r;;(8""00'-+""C4c-x-:C49"'0:C-0)]
or
for horizontal motion: x=ucos8xt .... (i)
or u =IQ,[i ±,J5i]
and for vertical motion: y=usm
. e xt - lgt
1 , .... (ii)
As negative sign is physically unacceptable,
Substituting t from Eqn. (i) in (ii), ,,= lq1.414 + 7.141] = 85.56 mls

yc xtan9 _![_x_]'
2 ucosS
Substituting the above value of II in Eqn. (ii),
R = .' (85.56)' = 746.9 m
g 9.8
Here y = -3.3)( 10 3 m, x = 9.4 x 10 3 m and 9 = 37°
Problem 35. A particle thrown over a triangle from one
3 end ofa horizontal base falls on the olher end ofthe base after
so -3.3)(10 3 = 9.4XI0 3 tan37 0 -4.{9.4X I0 ]' grazing the vertex. /fa and ~ are the base angle ofprojectioll.
ucos37° show that:
tana. = tane + tan~.
or
ucos37 Q '[
9.4xl0 = 10.35 x lO
4.9 ']"' =46
Solution: The equation of trajectory is
,
J
y=x tana. - gx
or u =J.4 x lO = 47000:::255m1s 2u 2 cos 2 a
46 x (4/5) 184 y=xtana.(l-xIR) where R = range
(b) Substituting the above value of u in Eqo. (i),
3
[-: R = 2u' Sin; coso. ]
I=_x_= 9.4 x l0 =9400=468
ucosS 255 x (4/5) 204
The co.-ordinates of A are (hcotS,h) and range = OB
Problem 34. A gun, kept on a straight horizontal road, is = hcot a+ hcot~. Substituting the co-ordinates of A in the
used to hit a car travelling along the same road away from equation of trajectory,
the gun with a uniform speed of 72 kmlhr. The car is at a
distance of 500 m from the gun, when the gun is fired at an h = hoot9tano.[l- heota ]
hcota + hool.
allgle of 45° with the horizontal. Find (a) the distance of the
tan = =:'"an;;a"c"o;,~'T
car from the gun when the shell hits it; (b) the speed of
projection of the shellfrom the gun. (g = 9.8mls2 )
a '
cote +cot$
Solution: The speed of the car v = 72 x (5/18) = 20 mls = tan9cot9+tanacot4t=tanacot¢l
1+ tanS = tanCt = tan$ +tan9 = tan a
tan$ tanljl
, ~

~

i
ih

I. 500 m - I. vT--+l o B
Fig. 6.48
Fig. 6.49
For a particle projected with an initial velocity u at an angle e Solution: Due to inertia ball will share the velocity of
trajectory is platform at the instant of projection. Hence horizontal
2 components of ball's velocity =12 mls and vertical
y=xtana - gx components of ball's velocity = 3m1s.
2u 2 C08 2 a
Considering the vertical motion,
Problem 36. A gun is fired from a moving platform and
10 = 3t + ~ x 9.8t
the ranges of the shot are observed to be Rand S when 2
platform is moving forward or backward respectively with
velocity V. Prove that the elevation a/the gun is 49,2 +301-100=0
- 30 ± '/900 + 19600
t - I[ g(R - S)2 ] 1 =-=-:-=-:"'9;:;8:-'-'=:':'
an 4V 2 (R+S)
Rejecting - ve root, we have
Solution: Let u and v be the horizontal and vertical
components of velocity when the piatfonn is at rest. 1 =-30 +;~.J205 =1.15 s (approx.)
When the platfonn is moving forward, the resultant
u + V and vertical velocity is v. The
horizontal velocity is = (i) ThedistanceAB = 12xl.15=13.80m
range R is given by (ii) The time taken by platform to reach the ground = 10/3
= 3.33 sec. Thus, the time difference =3.33 -1.15 =2.18 s
R = 2(u+V)v
g
.... (i) (iii) Let the ball strike the ground with velocity v; then
horizontal component v h = 12 mls and vertical component
When the platfonn is moving backward with velocity V,
the horizontal and vertical components of the velocity of the
v~ =3 2 +2x9.8xlO
shot are u - V and v respectively. => vv=14.32m1s
The range is then
Then v=~v~ +v~ =.J144 +205 =.J349m1s
S = 2(u - V)v
g
.... (ii) = 18.68 ms - I

From Egns. (i) and (ii), R + S = 4uvlg and 14.32


tan9 = vv'v h =12=1.l9
and R - S=4Vvlg
9=tan - 1 1.l9 Answer
(R _ S)2 16V 2v 21g2 4V 2v
(R+S) = 4uv/g =-;;g Problem 38. A block of ice starts sliding down from the
top ofthe inclined roofofa house (angie ofinclination of roof
If a be the angle of projection, then tan a = vi u = 30° with the horizontal) along a Une ofmaximum slope. The
(R - S)2 4V2 highest and lowest points ofthe roofare at heights of 8.1 m and
Thus,
(R +S) =gtana 5.6 m respectively from the ground. At what horizontal
distance from the starting A
tana = g(R _ S)2 point will the -block hit the T
~.
4V2(R +S) ground? Neglect /riction. I
,~
Solution: When the ice E t 30' v cos 30·
or a=tan - ,[g(R-S)2 ]
4V2(R +S)
Problem 37. A rider on an open platform, which is
block slides down from A to ~
B, it falls vertically through
AC = 2.5 m
1
descending at constant speed of 3 m s-I, throws a bal/. If v is the velocity with "o'---;::.c,-:-::----'-'
Relative to platform, ball's initial velocity is horizontal at 12 which it reaches B, then Fig. 6.50
2 2
mls. The ground is 10m below the location where the ball is ·v = u +2as, where u=O, a = gsin300ands= AB.
thrown:
(i) Where does the ball hit the ground? v 2 =2g x AC x AB(sin300= AC)
AB AB
(ii) How long after the ball hits the ground does the
platform reach ground level?
=2x9.8x2.5=49

(iii) With what velocity does the ball hit the ground? v = 7m s -I along AB. Resolve this velocityvinto:
(i) a horizontal component While for the second gun

=vcos30o:::: 7x ~ =3.5x..J3 m 5- 1 x2 =5./3cos600t2 .... (iii'

and ....(iv·
(ii)a vertical component = vsin 30°= 7x 112 = 3.5 m s- I
Vertical Motion: For collision
Let t be the time taken by the block to reach the ground, xI =x2 and YI =Y2
1 , 1, IS 1,
Apply s =ut+-gt i.e., Sv'3I, =S.J3!I, and 1O - '2gt1 =10+'2'2 -'2gt2
2
Given $=5.6 m, u =3.5 ms- 1, g = 9.8 ms - 2 i.e., and
, ,
g(' , -', )=ISI,
5.6=3 5 t+~ X 9.8t 2 or 4.9t 2 +3 .51- 5. 6 =0 which on simplification gives I , =1 sec and t2 =2 sec
[ast 2 ¢O
Dividing this equation by 0,7, we get 7t 2 +5t -8 =0 So time interval between the firings = 2 - 1= I sec
- S±J(S)' +(4x7x8) and substituting II = 1 sand g = 10 mls 2 in Eqns. (i) and (ii
, = --'-'-'::--=----"-
2x 7 co-ordinates of point P will be (5./3, 5) m. Answe,
,= - S±IS.78 Problem 40. Particles P and Q of mass 20 g and 40 ;
14 respectively are simultaneously projectedfrom points A and j
Rejecting the -ve value, on the grormd. The initial velocities of P and Q make 45° an,
135° angles respectively with the horizontal AB as shown in Ih
_ - S + IS .78 = 10.78 =0.77 s
14 14 Fig. 6.52 Each parlide has an initial speed of 49 mls. Th
separation AB is 249 m. Both particles :ravel ill the sam
Horizontal Motion: vertical plalle and undergo a collision. After collision •
Horizontal distance travelled EF = Horizontal velocity x retraces its path. Determine the position of Q when it hits th
time ground. How much time after the collision does the particle!
= 3.S x -J3 x 0.77 = 4.668 m take to reach the ground? (Takeg =9.8ml s 2 )
:.Total horizontal distance = DF = DE + EF
= 4.33 + 4.668 = 8.998 m
(DE =BC = ACcot30o=2.5x v'3 = 4.33 m)
;{ ~1350
A B
Problem 39. Two guns, situated on the top of a hill of Fig. 6.52
height 10m, fire one shot each with the,same speed 5..J3 m s- I
Solut ion : As'the'horizontalspeed"oftWo partiCles toware
at some interval of time. One gun fires horizontally and other
each other is same (ucos45°), they will meet at the middle (
fires upwards at an angle 0/60 0 with the horizontal. The shots
AB, i.e" at distance (24512) = 122.5 from A towards B.
collide in airat apoint P.Find (i) the time interval between the
firings and (ii) the co-ordinates of the point P. Take origin of
the hill right below t~e muzzle and trajectories in x-y plane.
Nowas R = "-==,,,
'sin"'>ll
U
g
49 x 49 x l=245m
9.8
Solution: As shown in Fig. 6.5 1, from equation of motion So AB is the range and as the collision takes place at the middl
s= So + ut + ~ atl for fi rst gun, of AB, so it is at the highest point of the trajectory.

y Now applying conservation of linear momentum at tt


It highest point along horizontal direction keeping in min_
60" vp =-upcos 45°,
--------- -----
20 x 10-3 cos 45°-40 x 10-3 cos 45°= -20 x 10-3 cos 45°+v,
10 m] P

0
",, , This gives vQ = 0, i. e., after collision, the velocity of Q '
highest point is zero, So Q will fall freely llnci.Jf gravity ar.
Fig. 6.51 will hit the ground ill the middle of AB, i.e., 122.5 mfroll!
x, = S.J3I , .... (i) towards B.
1 . 2 2
and >'1 =lo - -gt
2
f .... (ii) Now as H = 1/ sin 8 = 49x 49x l = 490 = 61.25 m
2g 2 x 9.8x2 8
TRANSLATORY MOTION 253

lH
So time taken by Q to reach ground, and distance travelled by 2m towards m horizontally,

2x 490 ) 5 I 2 10
t= g~ Sx9.S = ~'=3.536s d2 =ucos9xt=1OX2'x ..Jj= .J3m

So distance between the towers AD and CD,


Note: Time taken by Q to reach ground can also be calculated as
d=d1 +d, =20 +IQ.=1Q.=IO.,fj~17m
t .. r",
2
usin9 =~:::+- =3.536s
g 9.8 x..,,2 ,,2 .,fj .,fj .,fj
(b) Applying conservation of linear momentum along
Problem 41. 1Wo towers AD and CD are situated a horizontal direction
distance d apart as shown in Fig. 6.53. AD is 20 m high and

11
mu-2mucos600=3mV, j,e., V=Q

~__ /~m
f. e., the velocity of combined mass after collision along
horizontal is zero, so after collision it will move vertically
{m _____ u down from the point of collision and wilt hit the ground at a
distance dl =20 / .J3 from AB or at a distance d 2 =1D / ..Jj

1 d1
- -- - --/
';,(.. d2
from CD between BD.
Problem 42. An object of mass 5 kg is projected with a
B d D velocity of20 mls at an angle of60° to the horizontal. At the
highest point of its path the projectile explodes and breaks up
Fig. 6.53 into two fragments of masses ) kg and 4 kg, The fragments
CD is 30 m high from the ground. An object of mass m is separate horizontally after the explosion. The explosion
thrown from the top of AD horizontally with a velocity 10 mls releases internal energy such that the kinetic energy of the
towards CD. Simultaneously another object of mass 2 m is system at the highest point is doubled, Calculate the
thrown/rom the top a/CD at an angle 0/60 0 to rhe horizontal separation between the two fragments when they reach the
towards AD with the same magnitude ofinitial velocity as that ground. (g = IOmls2)
of the first object. The two objects move in the same vertical
plane. collide in mid air and stick to each other. (a) Calculate Solution: At the highest point of trajectory applying
the distan ce d between the towers and (b) find the position conservation oflinear momentum along horizontal,
where the objects hit the ground. 5x20cos60o = l x v l +4v2' i.e., VI + 4v2 = 50 .... (i)
Solution: Let the two particles collide after time I; then And by conservation ofenergy at highest point, according
from 2nd equation of motion, i. e., to given condition,

s= ut+ 1. at 2 I ') 1 2 I 2
( '2 x 5xlO x2 = 2'x lv 1 +'2 4V2 '
2
for vertical motion of A, i.e. , vf+4v~=IOOO .... (ii)
I ,
h1=-gt .... (i) Substituting v 1 from Eqn. (i) in (ii),
2
while for D,
(50-4v 2 l +4v~ =1000, i.e., v~ - 201.1 2 +75 =0
which on solving gives
h2 =(usin600)t+4 gt2 (ii) v2 =S mls or 15 mI,
But as the collision will take place at same level, So from Eqn. (i),
for v2 =5m1s VI =30mls
30-h2 =20-hl' i.e., h2 -hi = 10
which, in the light of Eqns. (i) and (ii), gives andforv 2 = 15m1s v 1 =- IOmis
So if both the particles move in same direction,
10 X( ~}=IO, i.e., t= 3Js v re1 =30-S=25m1s
and ifboth move in opposite direction,
(a) Now as horizontal motion is independent of vertical v re1 = 15-(-IO)= 2Smfs
motion, so from 2nd Eqn. of motion, i. e., s = ut + (1I2)al 2 , i. e., fragments after explosion separate from each otber
distance travelled by m towards 2m horizonb\lIy, horizontally 25 m per sec,
2 20 Now as time taken by fragments to reach, ground
d 1 = IO x .,fj = .,fjrn
254

t=I=u,in9=20x,/3 =,/3,
2 g IO x2
i.e.,
[ '2'0,9
25 ] ' = 3; g

25 ease = ~l50
So the separation between two fragments when they reach
the ground or T """"3 = 1O.77;socos9 =0.86
d =v re1 x t=25x .J3:::'43.3 m
i.e.,
Problem 43. A bullet a/mass Misfired with a velocity 50 (b) As initially the bob is at the highest point of trajectory,
m/s at an anglee with the horizontal. At the highest point a/its
trajectory, it collides head on with a bob of mass 3 M
suspended by a massless string of length (10/3) m and gets
x=R
2
=!["
2
2
Sin2a) =50XSOX.J3 = 108.25 m
g 2xlOx2
emhedded in the bob. After the collision the string moves
, . '9 1
through an angle 120°, Find: (0) The angle 9, (b) the vertical y=H=u sm =50x50 x =31.25m
and horizontal co·ordinates of the initial position of the bob 2g 2xIOx4
with respect to the point affiring ofthe bullet. (g = 10 mls2)
Note: In this problem the most common error which one will commit i
Solution: (a) At the highest point of trajectory horizontal that "at highest point of bob like ballastic pendulum v", (1'
force is zero (as mgcos90 Q = 0); so by conservation of linear However, lhis is not correct as here angle of swing is greater thai
momentum in borizonlal direction, 90" so at highest point T = 0 but v *" O. (For details see conditiol
for oscillation or leaving the circle ill § 7.3.)
B
Problem 44. A particle of mass m is projected at I ='
T : from apointOon the ground wilh aspeedvo at an angleof45
1200 ..
~//' to the horizontal. Find the magnitude and direction of Ih,
~¥~_v angular momenlum of the particle about 0 at lime t =volg.

I~::~::::~:·~H__- _- _-'~'_"'_'_"_',_\~
Solution: Using Y
1st-equation of motion
o.• m
v =u + at along horizontal P

I' x -I
direction, 45' , ,
Fig. 6.54
v, =(vo/J'i) +Ox t o
Fig. 6.55
MxSOcos9+0=4MV =0.7vo [asa=Oj
and along vertical direction,
i,e., V =(50/4)cos9 .... (i)
Now equation of circular motion of the bob at B will be v y =(vo/J'i) - g(vo/g) =-03v o
4M' + 4Mgcos120° So that the velocity of particle at time t = (volg) in
T = _....£.... vector fonn will be
I
-> -> ->
ButatB, T=O and V;I!!:O [as angie > 90o:J v = i (0.7v o )- H03v o) .... (:
So that the above equation reduces to

v 2 = !/=~g [as 1= l~ m] . . (ii) Now from 2nd equation of motion = lit S +(k)0/ 2 alon.

Now by conservation of mechanical energy between A


,horizontal direction,
Vo Vo Vo
,
and B (after collision) x= - x-=0.7-
J'i g g
!(4M )V' = !(4M)v' + 4MJ10 + 10 ,inlOO] and along vertical direction,
2 2 6Ll l

or 1 2 1 2
- V = - v +5g
2 2
....(iii) y= 1z v; -kg[v; ]' V;,
x =0.2

Substituting the values of V and v from Eqns. (i) and (ii) in


(iii), we get So the position vector -: atthe l(=volg) will be

-1 [50
2 4
- cose ]' -g]
=-1 [5
2 l
+5g
• 1= 107v;]+ ~02v;] .. (i
255

Now as by definition L= (1x p) =m(1x ~) From Eqn. (Hi), 1.25 +5t 2 =lOt 2 -3 -0.75t 2
2
i.e., 4.25t = 4.25 or t = 1s, ignoring negative value oft.
1, -->j k From Eqns. (ii) and (i),
O.2v~
So L= ?7v o
-->
0
ucosa =3+0.75=3.75
g g and usina =lOxl - 3.75=6.25
o. 7v o - O.3v o 0 Squaring and adding, r---,-- - - , -
u=~(3.75)' +(6.25)' =7.3 m1s
=mk V'
-->[-0.21 : -0.14 : v'] and dividing, tana = 6.25 =~
3.75 3
1
--> --> a =tan - (5/3)=59°
i.e., L = -(O.35mv~/g) k
§ 6.7 Application of Calculus in Deriving Kinematic
i.e., angular momentum has magnitude O.35mv~/g and is Relationships:
directed along negative z-axis fA] In the system shown in Fig. 6.58 velocities and
Problem 45. An object A is Y A accelerations of block I and 3 at any instant are:
kept fIXed at the point x =3m and ' f------
y = 1.25 m on a plank Praised E
VI ::6m1s(upward) .""'l!!!!i!I!""'-
a l =2m1s2 (downward)
above the ground. At time t =0 ~ u
the plank starts movlng along the v3 =3 m/s(upward)
+x-direction with an acceleration L----_;;_;i_::_, a3 = 4 mls
2
(downward)
1.5 mls 2, At the same instant a 3.0 m
stone is projectedfrom the origin Fig. 6.56 We wish to find velocity and
acceleration of block 2 at the instant.
with a velocity u as shown. A stationary person on the ground
Length of string is constant. It is
observes the stone hitting the object during its downward 3
considered to be inextensible. By
motion at an angle of45 0 to the horizontal. All the motions are Fig. 6.58
choosing centre of the pulley as
in the x-y plane. Find u and the time after which the stone hits
reference point we assign position co-ordinates to all the
the object. Takeg=10mls 2 .
moving masses as shown in Fig. 6.58.
Solution: Let the projectile Y A B Length of cord between mass I and pulley A
be projected with velocity u at --------- ----- - ~
= XI + X = constant .... (i)
an angle a with horizontal. As E . I P
the projectile hits the object B : On-differentiating this expression we get
: dx, dx
after time t, where its velocity is :, - + - =0 or VI + v=O .... (ii)
at 45" to horizontal during, U~--_;;_;i'_::_ ' dt dt
downward motion, so, 3.0 m
Fig. 6.57
On second differentiation, we get
usina -gt =-1 2
ucosa d xI + d'x --0 or al+a =O .... (Hi)
dt dt 2
ucosa +usina =gt .... (i)
Similarly length of string between block 2 and 3 is constant.
Further, 3 + ~ x 1.5t 2 = ucosa. t .... (ii) i.e., (x 2 -x) + (x 3 -X) = constant
or x 2 + x3 - 2x = constant
and 1.25=usina.t-!gt 2 .... (iii) On differentiating above equation, we get
v 2 +v3 - 2v=0 .... (iv)
From Eqn. (i), usin a . t =gt 2 - ucosa , t
On second differentiation we get
Substituting for ucosa· t from Eqn. (H), a 2 + a 3 -2a =0 .... (v)
usina· t =gt
2
-(3 +ix J 1.5t
2 Assigning upward direction + ve and substituting numerical
values in Eqn. (ii) to (iv) we get
= lOt 2 -3 - 0.75t 2 v 2 = 2m1s
a2 =+8mJs 2
256

(B) Consider a ring which can ID) In the system shown


slide on a vertical wire, attached to ml ~---=,,( in the Fig. 6.62 we wish to
a block of mass M. System is find relationship between
released from rest. We wish to accelerations of masses
calculate ratio of velocities of ring m 1,m2 and m3' Since length "'_ __
and block after the ring has of string is constant, •
Fig. 6.62
descended through a distance h. x l +x 2 +x 3 =1
Choosing pulley as origin of our 2 2
co-ordinate system. Y, and Y2 Fig. 6.59 d xI + d x2 + d2x3 =0
or
represent co-ordinates of ring and dt 2 dt 2 dt 2
block respectively. As the length of string is constant or a l +a2 +03 =0
(/2 + yt) 112 + Y2 =constant O2 +a3 =-a 1
Differentiating above Eqn., we get § 6.8 Newton's Laws of Motion
1 2y, x dy, + dY2 =0 Newton gave three laws related with motion and are
2 (/' + yi )112 dt d, popularly known as 'Newton's Laws a/motion'. These are of
central importance in classical physics as a large number of
Taking downward direction + ve principles and results follow from these.
and substituting fA) First law
cosa = Y,
(12+y~)lf2' dt
dy, =V
I
y,
T
y,
1
According to it, "Every object maintains its state o/rest or
of uniform motion along a straight line unless acted on by an
external force. "
and
dy,
-
dt
= - v1
'
1• ,
o Regarding Newton's 1st law it is worth noting that:
(I) Thi.s law defines force as an agent which is capable of
wegetv j cos9 - v 2 = 0 producing any change in the state of rest or motion of
Fig. 6.60
VI I a body.
i, e.,
17;"=c059 (2) It is a qualitative law and not quantitlltive, i.e., it tells
Ie) In the system "I!____~I'_iiaiiiiiiii!'~
' _" us that to produce an acceleration in a body we need a
force but the law does not tell us how much force is
shown in Fig. 6.61 we wish
needed.
to calculate ratio of
m, (3) The frame of reference in which Newton' s 1st law is
velocities of and 111 2 ,
Taking ceiling as reference
point let positions ofml and
l' valid is called 'inertial frame', i.e., if a frame of
reference is at rest or in unifonn motion it is called
m2 be x and yrespectively. inertial, otherwise non-inertial.
Length of string (4) The property ofa body due to which it opposes any
Fig. 6.61
change in its state of rest or unifonn translatory
=2~a2 +x2 + y =constt. motion is called' inertia'. A body at rest has' inertia
On differentiating above Eqn., we get ofrest' while a body in motion has 'inertia ofmotion' .
Mass of a body is a measure of its inertia. This is why
2 )( !(a 2 +x2 ) - 112 .2x dx + dy =0
2 dl dt sometimes Newton's 1st law is also referred to as ' law
2x of inertia'.
or r=~=;" V I + V, =0 Following are the few ofso many familiar examples ofthis
~a2 +x2 law:
(a) When a fruit tree is shaken briskly, the fruits are
or detached from the branches because fruits tend to
continue in the state of rest (due to inertia of rest)
while branches go into motion. However, if the
or cosO =!L (in magnitude) motion of branches is slow, the inertia of motion will
2vI
be transmitted to the fruits also and so fruits will also
v 2 has - ve sign which implies that velocities of ml and m2 are move along with branches and will not be detached.
in opposite direction.
(b) A bullet fired on a window pane makes a clean hole
through it while a stone breaks the whole of it. The so
.-)0d -+ d1
F =k-(mv)=km--
bullet has a speed much greater than the stone, so its dt dt
time of contact with glass is small. Motion is
or
~
F=kma
~ ~
[as (dvl dt) = a]
....
communicated only to a small portion of glass in that
time, hence a neat hole. Time of COil tact of stone with Now the units of force are so selected that k becomes
glass is large. During this tip1e motion could be =
unity, i. e., if m I, a = 1 and F = I, so
communicated to whole 'ofthe window, thus cracking l=kxlxl, i.e., k=1
the whole of it.
i.e., if a unit force is chosen to be the force which produces unit
(c) A passenger sitting in a bus gets a jerk when the bus acceleration in a body of unit mass, then Newton's II law for a
starts or stops suddenly. When the bus starts particle becomes
suddenly, the lower part of the body acquires motion
with the bus while the upper part remains at rest due to .... d....
inertia. This causes a backward jerk. Similarly when a
F = --.!!. = m1 .... (ii)
dt
moving bus comes to stop suddenly, the lower part of
(2) As from Newton's II law of motion,
his body comes to rest with the bus while upper part
continues to be in motion due to i:1ertia. This causes a ....a = ....Flm
forward jerk.
(d) If we place a coin on a card board resting over a So if a constant force acts on a particle, "it = constt. Now
tumbler and kick the card board with a jerk, due to for constant acceleration we know that
inertia of rr.otion the card board will fly apart while
due to inertia of rest the coin will not move with the v=u+at; s = ut+(~}t2 andv 2 =u 2 +2as
card board, but by force of gravity will fall into the
tumbler. However, if we pull or push the card board i.e., equations of matt on can be regarded as a consequence of
steadily. the coin will also acquire inertia of motion Newton's II law.
and will move with it. (3) From Newton's II law we can derive I and III law as:
Question XIV. A block of mass M is (a) According to Newton's II law,
A
suspended by a cord A from the ceiling and ....F = (d ....pldt)
another cord B is attached to the bottom ofthe
block as shown in Fig. 6.63. Which string will M
.... ....
break ifwe pull the string B (0) with a sudden Soif F=O, dp = 0
jerk, (b) steadily? dt
Answer: (a) When the string B is pulled .... . ....
with a sudden jerk it will experience tension i.e., dv =0 [asm;tO] t.e., v =constt. or zero
while due to inertia of rest of the mass M this dt
force will not be transmitted to the string A and i. e., a body remains at rest or moves with unifonn velocity
so the string B will break. unless acted upon by an external force which is Newton's 1st
(b) When the string B is pulled steadily Fig. 6.63 law.
the force applied to it will be transmitted from (b) In case of two particles moving towards each other
siring B to A through the mass M and as tension in A will be under mutual forces, from Newton's II law,
greater than in B by Mg (weight of mass M), the string A will d -+ -+ -)0

break. dt (Pl+ P2) ~ F = 0


IB] Second law
According to it, "the rate of change of momentum of a
or
body is directly proportional to theforce applied to it", i.e.,
~ dp
~
~ d~ .... ....
Foc - or F =k--". .... (i) or [as (d pi dt) = F]
dt dt
.... ~

Regarding this law it is worth noting that: F2 =- F1

(1) For a body or particle, as p= m1 with m= constt. i.e., for every action there is an equal and opposite reaction,
which is Newton's III law.
[CJ Third law (e) In case of an orbital motion of satellite, the force
According to it, "For every action there is an equal and exerted by gravitational pull of earth on the satellite is
opposite reaction." This is the traditional statement of the law. action and the force which the satellite exerts on the
A more modern version of the law is, "When two bodjes exert earth is reaction [Fig. 6.64 (b)]. In accordance with
mutual/orees on one another, the two forces are always equal Newton's III law both are equal and opposite.
in magnitude but opposite in direction." Ii
Regarding Newton's III law it is worth noting that: <iF
(I) Whenever one force acts on a body it giv'es rise to
another force called reaction, i.e., A single isolated
'"
force is physically impossible. This is also why total
internal force in an isolated system is always zero.
(2) The two forces involved in any interaction between
two bodies are called 'action' and 'reaction.' But this
does not imply any difference in their nature, i.e., one
force is the 'cause' and the other 'effect'. Either force
may be considered as action and the other as reaction iol ibl
ofit. FIg. 6.64
(3) Action and reaction never act on the same body, but However, as mass of earth is much greater than that of
always on different bodies. If action and reaction had satellite, its acceleration is much lesser than that of
been acting on the same body, the body will be in satellite. (acc. = force/mass)
equilibrium and no motion on a curved path would
have ever taken p lace (as for motion on a curved path
(1) When a book is resting on a table, the earth pulls the
book downward with a force FBE(=mg). The book
acceleration is a must).
:t does not accelerate because this force is balanced by
(4) There is no time lag between action and reaction, i.e., contact force FBT(=R) exerted on the book by the
they act simultaneously and not action today with table, i. e.,
react jon tomorrow. Furthennore, reaction exists only --jo --jo --jo --jo
so long as action is present. FBE + FBT=O, i.e., mg+R=O
(5) Newton's III law, conservation of linear momentum ~ ~ ~ ~
and motion of centre of mass are different physical Even though F BE (= mg) and F BT (= R) are equal and
concepts to deal the same problem, i.e., they are opposite, they do not form an action-reaction pair, as
equivalent to each other. [See point (2) in § 5.6] they are acting on the same body, the book [Fig. 6.'65
Following are a few of so many familiar examples of this (a)]. They balance each other and thus account for the
law: fact that the book is at rest.
(a) When a bullet is fired from a gun (action), the gun A - FST
recoils due to reaction.
(b) In swimming or boating we push the water backward
(action) and water pushes us or boat forward
(reaction) giving rise to the required motion.
(c) In moving on earth we push the earth back (action)
and as reaction to it, the earth pushes us in the forward ~
direction. However, our mass is much smaller than ~AE~
that of earth, so acc. is much greater.
101 Ibl lei
Note: However, if the surface is friction less, pushing of earth will not FIg. 6.65
be possible and so no motion. In such situations we can still Each of these forces must then have a corresponding
produce motion through application of Newton's III law, i.e., ~ ~

through the reaction of some action, say of throwing shirt or by reaction somewhere. The reaction ofFBE is FEB' the
splitting or sneezing.
force with which the book attracts the earth [Fig. 6.65
~ ~
(d) The working ofrocket orjet is also based on Newton's (b)] and the reaction ofFBT is FTB [Fig. 6.65 (c)], the
III law as due to ejection of burnt gas at high speed
(action), the rocket or jet moves in opposite direction contact force on the table owing to the book. The
reaction. [Fig. 6.64 (a)] action-reaction pairs involved in this problem are
I pair (book-earth)
e

+
II pair (book-table)
f FCH
Quesdon XV. When a horse pulls a cart, by Newton's JJI
law the cart also pulls the horse with equal and opposite force; Mg
mg
then how does the motion 0/ horse, cart and system takes
place? (m+M)g
Answer: Here the following forces acts along the (a) (b) (e)
horizontal : Cart Horse System
(1) The force on the cart due to thepuU of the horseFcH ' Fig. 6.67

(2) The force on the horse due to the cart F HC' Note: Same argument is applicable in tug o/war. The team that pushes
(3) The frictional force on the cart due to ground f. harder against the ground wins. The tension or the rope pulls 1\
leam backward. II is the earth's reaction to the push against the
(4) The reaction from ground due to the push of horse ground which supplies the necessary force for pulling.
against the ground F HG'
The vertical forces on the cart and horse are their weights § 6.9 Mass and Weight
vertically down while reactions due to contact from ground IAI Mass
upward. This all is shown in Fig. 6.66 Jt is a conunon misconception that mass is the quantity of
matter contained in a body. Since 1971 ' quantity of substance'
has been adopted as the seventh fundamental quantity with
units mol and so mass is something else, other than 'quantity of
matter'.
At present mass ;s defined as Ihe characteristic of a body
which relates the force acting on the body to the resulting
acceleration, i.e., mass is a quantitative measure of the
Fig. 6.66 resistance of a body to acceleration for a given force.
So for the motion of the cart, Regarding mass it is worth noting that:
FCH>! .... (i) (I) It is a fundamental quantity which is scalar having SI
i. e., the force with which the horse pulls the cart (FCH ) is unit kg. However, in British Engineering system mass
responsiblefor the motion ofthe cart and the cart will move if is a derived quantity with unit slug [See Problem 3 in
this force exceeds the static friction (f) between the cart and Chapter IJ.
the ground. (2) It is intrinsic prop erty of a body which does not
And for the motion of horse change with place, position, planet or time, i.e., the
mass of a body on earth or moon, on hill or in a mine,
.... (ii)
today or tomorrow remains the same.
i.e., the/orce which the ground exerts on horse (i.e., F HG) is
(3) It is measured by a beam balance (also called physical
responsible/or the motion ofthe horse and the horse will move balance). However, beam balance works only if ace.
if this force exceeds the force with which the cart pulls the due to gravity exists.
horse (FHC)'
(4) In classical physics mass is a/two types, viz., inertial
And so for the motion ofthe system [adding Eqns. (0 and (mj) and gravitational (m, ). The ratio of magnitude of
(ii)l applied force (other than gravity) to resulting
acceleration is called inertial mass while ratio of
magnitude of gravitational force to acceleration that it
or .... (iii) produces in the body is called gravitational mass, i.e.,
i.e., the /orce which the ground exerts 011 the horse (due to its mj = F /a and mg =F,tg
push agains~ the ground) is responsible for the motion a/the However, experiments show that m j = mg and
system. Free body diagrams for cart, horse and system are therefore we use a common word ' mass' for both
shown in Fig. 6.67. inertial and gravitational masses.
(5) In classical physics mass is conserved, i.e. , can Now as:
neither be created nor destroyed. However, according (a) For a given astronomical body,
to modern views mass can be created
(pair-production) or destroyed (annihilation) in g=,-GM
, 2- or gocpR ( 4 1tR 3p)
asM=-3 .... (iv)
R .
accordance with the relation
so with change of astronomical body, weight of the
E=mc 2 .... (i) given body will change. This is why the weight of a
[See § 4.7 for details.] body of given mass is different on moon than on the
(6) In classical physics mass is independent of state of earth.
rest or motion. However, according to modem views (b) Due to oval shape of earth, radius of earth increases
mass depends on state afrest or of motion according from pole to equator [as shown in Fig. 6.68 (a)] an.d as
to the relation g oc (1/ R 2), weight of a body wiU decrease as it is
m= 1/10
shifted from pole to equator on the surface of the
.... (ii)
earth, i.e.,
~I- (vic)'
We lIsl!ally neglect this variation of mass unless
stated.
lB} Weight (True) T
A
The jiJrce Wilh which a body is attracted hy a near-by
astronomical body (usually earth) is called the weight of the
1
body. If in a body of mass m due to attraction of earth an

acceleration it is produced then by Newton' s II law, la) Ib) Ic)


~ ~ Fig. 6.68
W = mg or simply W = mg .... (iii)
(c) At height h above the surface ofa planet [Fig. 6.68 (b)}
Regarding weight it is worth noting that:
(I) It is a vector having dimensions [MLT -2} and Sl unit as g' g so W' W .... (v)
[I + (hi R)]' [I + (hlR)]'
newlon. Improper unit of weight is kg-wt
1 kg-wt = 1x 9.8 newton i. e., the weight of a body on a hill will be lesser than
on the surface of the planet.
(2) A spring balance detennines the weight of a body as
in equilibfium (d) Ata depthdbelowthe surface ofa planet [Fig. 6.68 (c)]

0'
W=F=kx
Woox
[asforspringF=kx]
"., .. a~. f:~§[lc- ;] so .. W'=,I-~] .. (vi)

(3) It depends on the mass ofthe body and also on 'g' i. e., i.e., tneweight of a body in a mine will be lesser than
astronomical body. So if mass of a body is changed, on the surface of the planet. Further, at the centre of
weight will change, but if 'g' is changed (say by the planet (d = R) the weight of a body will be zero.
changing planet) weight will change' but mass will (e) Due to spin motion of the planet at its equator,
not, i.e., mass of a body cannot be changed without
changing weight but weight can be changed without
changing mass [by changing g}.
as g' = g - R(j,' so W' = 11-R;'] ....(vll)
(4) The true weight of a body will be zero where the i.e., if the planet spins faster, weight will decrease and
gravitational effects are nil, e.g., at the centre of earth if spin motion slows down, weight will increase.
or deep in space far from any star or planet. In these ICJ Apparent Weight
siv.lations no force will be required to lift a heavy lead
Our psychological perception, i. e., feeling of weight
block (W = 0) but mass remains unchanged, so we (called 'apparent weight) comes due to the reaction of a
may still be hurt in kicking the block (m 0). *' supporting surface, i. e.,
(5) The weight of a body depends on 'g', so '?iil! change Wapp = Reaction of supporting surface
with place, position or planet·. For a given body:
To understand this more clearly, consider the following
W' mg' g'
-= -= - examples:
W mg ' g
* S~C ~ 9.3 for details.
261

(1) Weight of a body in a lift* Note: If a > g ,Warp will be negative; negative weight will mOlln Ihat
Consider a person standing on a spring balance (or a body the body is pressed against the roofofthe lin instead of floor (as
suspended from a spring balance), or in a lift. The following lift falls more than the bOdy) and so the reaction will be
situations are possible: downwards, the direction of apparent weight will be upwards but
(a) Lift is at rest or moving with constant velocity: The the body will not be illvelicd as sh~wn ill many books.
person or body [shown in Fig. 6.69 (a)] will be in translatory
(d) Lift is infreefall, i.e" a :: g in case (c) . So
equilibrium
so R =mg
mg-R =mg
i.e., R =0
.. =
~pp mg [as Wapp R] =
or Wapp =Wo [asWo =mg=truewcight] .... (viii) so Wnpp =0 .... (xi)

-~
T fb/~a '\
i.e., apparent weight (reading of balance) will be equal to true v ... - ...
weight.
/-S~.'\
a .. g \
, ,
I
,,' ,, .. g '\
\ , ,, ,,
,,
j \, Planet ,/ ,,, ,,
_-_ ........./ ,, ,
J. "9
(aJ
'... ... ...

(bJ (eJ
Freely failing 11ft Satellite motion Projectile motion
80 kg Fig. 6.70
mg i.e., apparent weight of a freely jalling body is zero (though
(aJ (bJ true weight always remains mg). This is why the (apparent)
Fig. 6.69 weight ofa body is zero, i.e., body is weightless if it is in a (i)
(b) Lift is accelerated up: This implies that the lift starts lift whose cable has broken, (ii) orbiting satellite and (iii) cabin
moving up or if moving upwards its velocity increases or if which is projectile.
moving downwards its velocity decreases. If a is the (2) Weight of a body in a liquid
acceleration of the body, from Newton's II law [as shown in If a body is weighed in a liq~id with the help of a spring
Fig. 6.69 (b)] we have I balance as shown in Fig. 6.71 (a), thrust of liquid will act
R -mg =ma opposite to its true weight. So in equilibrium:
or R=m(g+a) R +Th=mg
or . Wipp =m(g+a) [asWapp =RJ or R=mg-Th
or Wapp = WO[1 + ~J [as Wo = mgJ .... (ix)
so Wapp = mg -Th

or wapp = mg[l- mg
Th ]
i.e., apparent weight (reading of balance) will be more than
true weight.
Now if V is the volume of body and p its density while cr
(c) Lift is accelerated down.: This implies thap he lift starts
that of liqU-ii.d,~_
moving down or if in downward motion its velocity increases
or if in upward motion its velocity decreases. In t~is situation
[as shown in Fig. 6.69 (c)] from Newton's Ill.aw,
mg - R =ma
i. e., R =m(g-a)
or ~pp =m(g-a) [as .Wapp = R]
i.e., ~pp =Wo[l-~]
(aJ (bJ
[asWo =mg=trueweight] .... (x)
Sinking body Floating body
i. e., apparent weight (reading of balance) will be lesser than
Fig. 6.71
true weight:
'" See also § 3.5 and solved Problem 8 in Chapter 3 with T = R = w.w.
mg=Vpg and Th=Vcrg i.e., T=1500(9.S+I.S)

so Wapp =mg[I-~l
p = Wo[I-~l
p
.... (xii)
or
(ii) Att = 6s
T =17400N

[asWo = mg] a =3.6 - 3.6 "" 0 mls 2


10-2
i.e., apparent weight ofa body in a liquid will be lesser than its
true weight. i.e., lift ~as no ace,; so
Further, if the body is floating, mg = Th T=mg
so Wapp = R =mg-Th=O i.e., T=1500x9.8
i.e., apparent weight ofa jIoating body is zero. or T = 14700N
Question XVI. State whether the statement given below (iii)Att=lls
is true or false, giving reason in brief' "Two identical trains a=0-3.6=_I.Smls'
t ~,~,r ----;
are moving on rails along the equator on the earth in opposite 12 - 10 ~ I

directions with the same speed. They will exert the same
pressure on the rails. "
i.e., lift is ace. down; so ••>
T = m(g - a)
Answer: The weight of a body at rest on earth at equator
i.e., T = 1500(9.S -I.S)
o 2 4 6 8 10 12
considering spin motion of earth is given by Time in sec._
or T = 12000 N Fig. 6.72
W=m(g - Ro,'j
(b) The displacement ofthe passenger is equal to the area
under vI t curve, i.e.,
or
W=+< ]
[asV=Rro]

Now if the train is moving in the direction of spin motion


.... (i)
= t (2 - 0) x 3.6 + (10 - 2) x 3.6 + t
(12 - 10) x 3.6

of earth, i,e., from west to east with speed v relative to earth's = 3.6[I+S+I] =36m
surface, the velocity of train relative to the centre of earth [as . displacement 36
(e) Average velocIty =. k = -12 =3 mJs
explained in § 6.3 (c)] will be (V + v); so time ta en
. change in velocity 0 - 0
_ [ g _(V+V)']
WE -m .... (ii) and Average acceleration = = _ .- = 0
R time taken 12
Problem 47. With what minimum acceleration can a
Similarly for the train moving opposite to the motion of fireman slide down a rope whose breaking strength is (2/3) of
earth, i.e., from east to west his weight?

Ww =m[g (V ~V)' ] .. (iii) Solution: The rope will break if T > breaking stren%th
.... (i)
From Eqns. (ii) and (iii) it is evident that WE < Ww' So the Here, breaking strength = (2/3)mg (given) ... (ii)
pressure (oc weight) will not be equal and the train going west and as tension in the rope is due to the weight of fireman which
will exert more pressure on the rails. So the statement is false. is sliding down, i.e.,
Problem 46. A lift is going up; the total mass of the lift T = m(g - a) .. .. (iii)
and the passenger is 1500 kg. The variation in the speed of the Substituting the value of breaking strength and tension from
lift is as shown in Fig. 6.72 (a) What will be the tension in the Eqns. (ii) and (iii) in (i), we get
rope pulling the lift at; (l) I s (ii) 6 s (iii) II s (b) What is the m(g-a»(2/3)mg
height to which the lift takes the passenger? (c) What will be
the average velocity and acceleration during the course ofthe
or a> (gl3) so amin = (gI3) Answer
entire motion? (g "" 9.8 mls2) Problem 48. What should be the length ofthe day so that
the weight of a body on the equator of earth becomes zero?
Solution: (a) As slope ofvi t graph gives acceleration, so Given that radius of earth is 6400 km and acceleration due to
(i) Att=ls gravity on its surface is 10 m/s 2 .
a -_ -3.6-0-
--. ISmI s ' Solution: As, W = m(g - Rro 2 ), so the weight ofa body at
2
equator will become zero if
i. e., lift is ace. up; so
g _ Rro 2 =0 [as m is finite]
T=m(g+a)
263

or co- Jg - 10 =_1_ rad Solution: The force on the surface S will be due to the
'til 6400x 10' 800 5 weights of bodies A and B both; so
F=WA +WB
so T = 21t =21t fK Now as B is always at rest,
co Vii
W. =m,g=4.l02x9,8 = 40,2N
j. e., T = 6.14 x 800s :::84.6 min .::::.1.4 hr But A is executing S.H.M., so is in accelerated motion.
WA = meg ± roiy) [as in S.H.M. lacc. 1= 0)2 y]
Note: As the present length of the day is 24 hrs and T <.JC (l/ro),
So WA will be maximum or minimum when
..!E...=TO '" 24:::'17
rol) T 1.4 y= max = amplitude a
i.~ ,ifthe earth starts spinning about 17 times of its present value, so (WA)maxormin =m[g±oia]
the weight of a body on equator wi!! become zero.
so (a) As (WA)inax =m(g+co2a)
Problem 49. A body A of m1C=:;==::JA
mass m] = 1 kg and a body B of =Ix (9,8 + 25' x 1.6 x 10-') = 19,8N
mass m2 =4.102 kg are (when the body A is at its lowest position)
interconnected by a spring as
so Fmax =40.2 + 19.8=60N
shown in Fig. 6.73. The body A
performs vertical simple (b) As (W.)"" =m(g-co'a)=lx(9,8-10)~-0,2N
harmonic motion with amplitude (when the body A is at its highest position)
a = 1.6cm and angular frequency Fig. 6.73
Negative sign here means that the direction of WA is
w =25 rad/s. Find the maximum and minimum value a/the
upwards as downward acc. > g
force on the suiface S.
so F min =40.2 + (- 0.2) = 40N

MISCELLANEOUS SOLVED PROBLEMS

Problem 50. Two particles of masses m] and m2 are


initially at rest at infinite distance apart. Ifthey approach each
other under inverse square law offorce (F = Klr 2 )find their
or
:;' =~ [~1 + :J [by chain rule]

Now as the particles are approaching each other, with time


speed of approach at the instant when they are distance d
r is decreasing, so
apart.
(d,ldt) =-v
Solution: Let the particles be at a distance r apart at any
instant. Then as (F = Klr2), the accelerations of the particles so _vdv = ~ [_
I + _1_ ]
F K dr r m]m2
a] = - = --
m] m r2 Integrating the above equation
1

and a 2 =F- = - K- v'2 =K[_I


r
+ _I_ ]+C
m2
m2 m r 2 In]
2
As they are approaching each other, the acc. of one But as initially v = 0 at r = 00, C = 0

~[*+~]
relative to other
.. ,.(ii)
V=
.... (i)

so forr=d, V= 2K[_1 +_
1]
or d m] m2
264 PHYSICS FOR COMPETITIONS - Vol. I

Problem 51. A particle of mass 10-2 kg is moving along


Note: (i) If we write [.-!..
m 1n2
. . ~] = ..!.,m the above · result becomes
the positive x-axis under the influence of a force
1
v = ,j2Klmd, m is called reduced mass. F(x) =' _KI(2x 2 ) where K = 10- 2 N_m 2. At time t =0 it is at

(,',') Here a = -d Krll- + --


dv = 2"
t r m1 m2
I]. no conslao so equations a
IS t tj . f
x = 1.0 m and its velocity is v = 0.
(a) Find its velocity when ,jt reaches x = 0.50 m
motion cannot be applied to solve the problem. However, the (b) Find the time at which it reaches x ::;: 0.25 m
problem can be solved by applying conservation laws of
energy and momentum as given below in alternative Solution: (a) As F = ma ::;: m ~~
solutions.

Alternative Solution: (A) If the velocities of two = mv dv [ as dv =dv . dx = v dV] and according
dx dt dx dt dx
particles are vI and v 2 when they 8fe distance d apart, by
conservation oflinear momentum, to given problem F = -KI (2x 2 ), so
m1v\ +m 2v 2 =O[asinitially Ps =0] .... (i) <Iv K .
mv dx =- 2x 2 ' J.e"
I"oVdv=mII'1- 2xK2 dx
and as PE when the particles are distance 'd' apalt,

u=-IFdr=-t[-~ldr=-~ .. (ii) or V'=~U-I)=:~~:G-I)=G-I) .... (i)


So by conservation ofme.:hanical energy, So for x =0,5 m, v 2 =(2-1), i.e., v=±lmls

im,vr +~m2V~ - ~ =0 [as initially (ME)s =OJ And as here the force is opposite to displacement and
particle starts moving from x = 1 towards the origin, i. e., with
or 1
2
2 12K
- m1v, +- m2v 2 =-
2 d
.... (m)
increase in time, x decreases, i, e., v( = -dxldt) will be negative,
so
Solving Eqns. (i) and (iii) for vI and v 2 , 0.25 0.5 F x= 1
I • I X
2m,K v.O
IVII = .1-...,--'---,-, t. a
In!(m! +m2 )d
Fig. 6.74

and Iv, l= [ 2mJK v=-I(mls)


V;;2 (m! + m2)d (b) As from Eqn. (i)
So the velocity of approach (I-x) ie _ dx = (I-X)[asherev=_dx ]
- '1J=--,
r-;~-" X ." dt x dt
v=lvtl+lv, l= 2K[_1 +_1_] radt =_ JO.25 _x_ dx
d m! m2 so
I (I-x)
(B) The initial ME oftbe system is zero. Ifvis the velocity
of approach when they are distant d apart. To integrate the above expression let x = sin 2 9 so that
U = -(Kid) [from Eqn. (ii)l dx = 2sin Scos9 de and hence

KE =!).2 .lV 2 =!lr m!m2]v' t= - J'/6[


n12
sin2e ]112 .
. 2smScos9d9
2m! + m2 2
l-sm e
[See note (ii) in solved Problem 32 in Chapter 8] 1,,"6 2
= - nl2 sm
. 'e de
So by conservation of mechanical energy: ,/6
i.e., 1
t = - nl2 (l-cos2S)d9
![~!~]V2 _K = 0
2 m! +m2 d =[ 9 --I sin2S ]'1'
r-;~-" 2 nl6
i. e., v= 2K[_1 + _1 ]
d m! m2
Problem 52. An object thrown vertically upwards from
the top ofa building reaches the/oot in time t ,.It
takes time 12 X eM -_R_u2sin28
-
g
if thrown vertically down with same velocity. If the time offree
fall is t, show that t = N,. = (20)' x (,fJ )/2 = 20,fJ m
10
Solution: Taking downward direction as positive and
As the fragment P comes to rest after explosion, it falls
applying equation of mation $= ul +(~ }t 2
, we get down vertically and hits the ground at half of the range,
i. e., x p = 20-/312 = 10-/3 m. If the fragment Q hits the ground
1 ,
h = -ut 1 +zgt j •••• (i) atxQ' then
x _ mpxp + mQxQ
and .... (ii) CM- mp+mQ

Multiplying Egn, (i) by 12 and Eqn. (ii) by II and adding the 20,fJ = (mI2)1O,fJ + (mI2)xQ
two, we get i.e. ,
(ml2) + (ml2)
1
h(11 +I')=Zglll'(/1 +1,) or

or 1 t
h=-gt Alternative Solution : By conservation of linear
[as!, +t2 *0]
2 12 momentum at the hig~est point,
1 , 1 1 , m x 20 x cos600= (m I2) x 0 + (mI2)v Q
cr -gt =-gt1t 2 [ash= - gl 1
2 2 2 i.e., v Q =2x20x(l/2)=20mls
or r=N, Now time taken by P to reach ground,
Problem 53. A shell is fired from a gun with a muzzle
velocity of20mls at an angle of60° with the horizontal. At the
I=~
top of the trajectory the shell explodes into two fragments P
and Q ofequal masses. If the speed a/fragment P immediately 2u 2 sin 28
= g·2g
after tile explosion becomes zero, how far from the gun does
the other fragment Q land? (Take g = 10 mls2) = usin8
Solution: As shown in Fig. 6.75, at highest point, the g
shell has horizontal velocity and as fragmentP comes to rest, Q
will move with increased horizontal velocity. However, in
_20x(,fJ)l2=,fJs '. ,..
10 ·
vertical direction both have zero velocity, so will hit the
ground simultaneously. Now as internal force does not affect so in time t, Q will move horizontally = 20 x -/3 ~ 20-/3 Itl.
the motion of centre of mass, th~ centre of mass hits the ground so horizontal distance of Q from the gun, i. e., 0 will be
at the position where the original shell would have landed, i. e., x = (R I2) +20,fJ
the range of the original projectile. So that,
2
But as R = u sin2e
g

= (20)' x (,fJ)/2 =20,fJ m


10
so x = 10,fJ + 20,fJ = 30,fJ m
266 PHYSICS FOR COMPETITIONS - Vol. I

EXERCISE

(AI Only One Choice is Correct 8. If a particle is moving with uniform velocity, its
acceleration is:
1. A person travels along a straight road for the first half
length with a constant speed v I and the second half length (a) Infinite (b) Finite
with a constant speed v 2 . The average speed V is: ee) Negative (d) Zero
(CPMT19921 9. The velocity of a body depends on time according to the
(a) (v I +v2 )/ 2 (b) 2v Jv 2 I (VI +V 2 ) equation v = 20+ 0. 1t 2 • The body is undergoing:
(e) (v l v2 )112 (d) (va I v I )"2 (MNRI9951
[Hint: See nOle in § 6.2 B (8)] (a) Unifonn acceleration
2. A motorist travels from A to B at a speed of 40 kmlhr and (b) Unifonn retardation
returns back at a speed of60 kmlhr. His average speed will (c) Non-unifonn acceleration
be: (CBSE 19901 (d) Zero acceleration
(a) 40 km/l" (b) 48 km/l" 10. Thex and yco-ordi nates of a particle at any time I are given
(c) 50 kmlhr (d) 60 kmlhr by;
3. The velocity oflighl emitted by a source S, observed by an X=7/+4t 1 and y=51
observer 0, who is at rest with respect to S is c. If the where x and yare in m and I in s.
observer moves towards S with velocity v, the velocity of
The acceleration of the particle at 5 sis: (CPMT 1993)
light as observed will be:
(a) c+v (b) c - v (a) Zero (b) 8 m / s2
2
(c) [I -(c' I V' )]"' (d) c (e) 20 m /s (d) 40 m /s2
II. A point moves with uniform acceleration and v t ,v 2 andv 3
[Hint: See note in § 6.3]
denote the average velocities in three successive intervals
4. A ISO m long train is moving to north at a speed of 10 mls. of time t l,t 2 and t). Which of the following relations is
A parrot flying towards south with a speed of 5 mls crosses correct?
the train. The time taken by the parrot to cross the train
would be: (eBSE 1992)
(a) v j- v 1 : v 1 - v) = / 1 - t 2 : 11 +I J
(a) 30, (b) 15, (c) 8, (d) 10 , (b) vI - v 1 : v 2 -v J ='1 +1 2 :'2 +13
(e) VI -V 2 : V 2 -V3 "" I -1 2 : II - 13
5. A steam boat goes across a lake and comes back (i) on a
quiet day when the water is still and (ii) on a rough day (d) VI - V 2 : V1 -V 3 ='1 -'2 : t2 - t3
when there is a uniform current so as to help the joumey (Note: Think carefully about the difference between instant and
onward and to impede the journey back. If the speed of the interval.]
launch, on both days, was same, the time required for the 12. If the relation between d istance x and time 1 is of the fonn;
completejoumey on the rough day, as compared to that on
t =ax 2 +Jlx
the quiet day, will be:
(a) Less (b) Same
ex and pbeing appropriate constants, then the retardation of
the particle is:
(c) More (d) Cannot be predicted
(a) 2ew 3 (b) 2pv'
[Hint: See solved Question VI]
6. The displacement s crf a point moving in a straight line is (c) 2a~v' (d) 2P'v'
given by: [Hint: See solved Problem 3 (b)J
2
s=8t + 3t-5 13. The height yand distance x along a horizontal plane of a
s being in em and t in s. The initial velocity of the particle is: projectile on a certain planet are given by;
2
(a) 3 cml' (b) 16 cml' x = 6lm and y=(8/ - 5t )m
(c) 19cmls (d) Zero The velocity with which the projectile is projected is:
7. A travelling wave in a stretched string is described by the (a) 8 mI' (b) 6 mI' (c) 10 mI, (d) Zero
equation,
14. In the above problem the direction o f initial velocity with
y= Asin(kx-oot) x-axis is :
The maximum particle velocity is: (lIT 1997 (II») (a) tan - I (3/4) (b) tan - I (4/3)
Ca) Aoo (b) 001 k (c) drol dk (d) xl/
(c) sin -I (3/4) (d) cos - I (3/4)
15. In the above problem the acceleration due to gravity is : 24. A body of mass 2 kg has an initial velocity of 3 mls along
(a) -lOmis' (b) 5m1s' OE and it is subjected to a force of 4N in a direction
perpendicular to OE. The distance of the body from 0 after
(c) 20 mis' (d) 2.5 mis'
4 sec will be:
16. The initial velocity of a particle is u and acceleration f is (a) 12m (b) 28m
given by at. Which of the following relations is valid?
(c) 20 m (d) 48 m
IBHU 19951
25. If a force is applied at an angle to a body moving along a
(a) v =u+at 2 (b) v=u+at l l2 straight line:
(c) v=u+at (d) v = u (a) The body continues to move in the direr-tion of force
[Hint: See solved Problem 7 (a)] (b) The body continues to move in its initial direction of
17. An electron starting from rest has a velocity that increases motion
linearly with time, that is, v = Kt where K = 2 mls 2. The (c) The body moves in a fixed direction other than that of
distance covered in the first 3 sec will be: force and initial motion
(a) 9m (b) 16 m (d) The body moves in a direction other than that of force
(c) 27 m (d) 36 m and initial motion which varies with time
18. How long will it take to stop a car travelling at a speed of20 26. Tripling the speed of a motor car multiplies the distance
m/sec, if the uniform ace. during braking is ~5 mls 2 ? needed for stopping it by:
(a) 100s (b) 4s (a) 3 (b) 6
(c) (114) s (d) (11100) s (c) 9 Cd) Someothernumber
19. The initial velocity of a particle moving along a straight line [Hint: See note in solved Question VIII]
is 10 mls and its retardation is 2 mls 2 . The distance moved 27. A car travelling at a speed of30 kmlhr is brought to a halt in
by the particle in the fifth sec of its motion is: 8 m by applying brakes. If the same car is travelling at 60
(a) 1m (b) 19m kmIhr it can be brought to a halt with the same braking force
in:
(c) 50m (d) 75m
(a) 8 m (b) 16 m
20. A body starting from rest covers a distance of 9 m in the
fifth second. The acceleration of the body is: (c) 24 m (d) 32 m
28. A truck and a car moving with the same KE are brought to
(a) 2m1s' (b) 0.2m1s'
rest by the application of brakes which provide equal
(c) 1.8 mis' (d) 4 mis' retarding force. Which of them will come to rest in a shorter
21. A car is moving along a straight road with a unifonn distance? [CBSE 1990)
acceleration. It passes through two points P andQseparated (a) The truck
by a distance with velocity 30 krnlhr and 40 km/hr
(b) The car
respectively. The velocity of the car midway between P and
(c) Both will travel same distance before coming to rest
Q;" ICBSE 1993J
(d) The distance of travel will depend on the horse power
(a) 33.3 kmllu (b) 2M kmlhr of the vehicle
(c) 2s-i2 kmlhr (d) 35 kmlhr (Hint: See solved Question VlIlj
22. A particle starts moving from the position of rest under a 29. A body sliding on a smooth inclined plane requires 4 sec to
constant acc. It travels a distance x in the first 10 sec and reach the bottom starting from rest at the top. How much
distance yin the next 10 sec, then: (EAMCET 19931 time does it take to cover one-fourth the distance starting
(a) y =x (b) y=2x from rest at the top?
(c) y = 3x (d) y =4x (a) 1 s (b) 2 s
[Hint: See solved Problem 8] (c) 4 s (d) 16 s
23. A particle moving with a velocity equal to 0.4 mls is [Hint: See solved Problem 16 (b)]
subjected to an acceleration of 0.15 mls 2 for 2 sec in a 30. A body of mass m slides down an inclined plane making an
direction at right angles to its direction of motion. The angle of 45° with the horizontal. Ifthe coefficient offriction
resultant velocity is: between the body and the plane be 0.3, the acceleration of
the body is approximately equal to:
(a) 0.7 mls (b) 0.5 mls
(c) 0.1 mli (d) Between 0.7 and 0.1 mls
(a) 0.22 g (b) 0.30 g
[Hint: See solved Problem IOJ (c) 0.49 g (d) 0.70 g
31. Check up the only correct statement in the following: (c) Doth hits the ground with same velocity
(a) A body has a constant velocity and still it can have a (d) Which one will hit the ground with greater veloci~
varying speed depends on their shape
(b) A body has a constant speed but it can have a varying [Hint: Motion under gravity is independent of mass of body.]
velocity 38. Two bodies of different masses ma and mb are droppe<
(c) A body having constant speed cannot have any from two different heights, viz, a and b. The ratio of tim
acceleration taken by the two to drop through these distances is:
(d) A body in motion under a force acting upon it must (a) a:b (b) ma/mb : bla
always have work done upon it
32. An electron of mass me' initially at rest, moves through a
(e) .fa:.fb (d) a' :b'
certain distance in a uniform electric field in time t \' A 39. Two bodies one held 30 cm directly above the other ar
proton of mass mp' also initially at rest, takes time t2 to released simultaneously and faU freely under gravity. Afte
move through an equal distance in this uniform electric 2 sec their relative separation will be :
field. Neglecting the effect of gravity, the ratio t21tl is (a) 10 em (b) 20 em
nearly equal to: lIlT 1997) (c) 30 em (d) Zero
(a) 1 (b) (m/m e )112 40. A ball is released from the top of height h metre. It takes:
second to reach the ground. Where is the baU at the tim
(d) 1836 Tl2sec?
33. A particle moving in a straight line has velocity and (a) At(h/4)mfromtheground
displacement equation as (b) At(h/2)mfromtheground
v;"4.Jl+s, (e) At(3h/4)m from the ground
where v is in mls and s is in m. The initial velocity of the (d) Depends upon the mass and volume of the ball
particle is: 41. A body is released from a great height and faUs freel:
(a) 4 mI, (b) 16 mI, towards the earth. Exactly one sec later another body i
(c) 2 mls (d) Zero released. What is the distance between the two bodies 2 se
34. Starting from rest a particle moves in a straight line with after the release of the second body?
acceleration (a) 4.9 m (b) 9.8 m
a=(25_t 2 )1/2 mls 2 forO$;t$;5s (e) 24.5 m (d) 50 m
42. A body falls from rest freely under gravity with a
a=31t mls 2 fort>5s
8 acceleration of 9.8 mls 2 • Neglecting 'air resistance, th
The velocity of particle at t = 7 sis: distance travell~d by the body during the third second ofit
motion will be:
(a) 11 mI, (b) 22 mI,
(a) 14.7 m (b) 24.5 m
(e) 33 mI, (d) 44 mI,
(e) 19.6 m (d) 29.4 m
35. A body falling under gravity moves with uniform:
43. A stone is thrown upwards from the surface of the eart
(a) Speed (b) Velocity
with an initial speed of 5 mls. The stone comes to rest at
(c) Momentum (d) Acceleration height of (g = 1000 dyne/g):
36. Two bodies of different masses say I kg and 5 kg are (a) 1.25 m (b) 12.5 m
dropped simultaneously from a tower. They will reach the
(e) 125 m (d) 2.45 m
ground:
(a) Simultaneously 44. An object is projected upwards with a velocityof4.9 mls.]
wilt strike the ground in approximately:
(b) The heavier one arriving earlier
(a) 2, (b) 1,
(c) The lighter one arriving earlier
(e) 1.5, (d) 0.5,
(d) Cannot say, the information is insufficient
[Hint: Motion under gravity is independent of the mass of the
45. A boy throws balls into air. He throws one; whenever th
body.] previous one is at' its highest point. How high do th~ ball
rise if he throws one ball each sec?
37. If a light and a heavy body are released from same height:
(a) 19.6 m (b) 9.8 m
(a) Heavier bo~y hits the ground with greater velocity
(e) 4.9 m (d) 2.45 m
(b) Lighter body hits the ground with greater velocity
46. A pebble is thrown vertically upwards from a bridge with (a) Sliding block will reach the ground first with greater
an initial velocity of 4.9 mls. It strikes the water after 2s. speed
The height of the bridge is : (b) Freely falling block will reach the ground first with
(a) 19.6 m (b) 14.7 m greater speed
(e) 9.8 m (d) 4.9 m (c) Both the blocks will reach the ground at the same time
47. A bullet is fired with a gun from a tower horizontally with a but with different speeds
velocity 400 mls. At the same time a stone is dropped from (d) Both the bl~cks will reach the ground with same speed
the same tower: but the freely falling block first
(a) The stone will reach the ground first [Hint: Sec solved Question XII]
(b) The bullet wi!! reach the ground first 54. The time taken by a particle to slide down a smooth inclined
(c) Both will reach the ground at the same time plane is double the time it would take in falling down
(d) (a) and (b) according to the height of tower through a height equal to the vertical height of the plane.
The inclination of the plane with horizontal is:
[Hint: See solved Question XI]
(a) 30' (b) 45'
48. Two bullets are fired simultaneously, from the same level
and in the horizontal direction, over a lake. The speed of (c) 60' (d) 90'
one is 196 mls and ofthe other is 98 mls. Assuming that the 55. At a given place where acceleration due to gravity is g, a
air resistance is negligible and the lake is still the bullet sphere of lead of density d is released in a 'column of liquid
which is moving faster will, compared to the slower one, of density cr. If d > cr, the sphere will fall vertically with an
fall in the water: acceleration:
(a) Halftime before (b) At the same time (a) g (b) zero
(c) Twice the time after (d) Thrice the time after (e) g (d -a) 1d (d) g(ald)
49. A ball is thrown vertically upwards with a speed of 10 mls 56. A bomb is released by a horizontally flying aeroplane. The
from the top of a tower 200 m high and another is thrown trajectory of the bomb is a: [CPMT 19931
vertically downwards with the same speed simultaneously. (a) Straight line (b) Parabola
The time difference betv.ieen them in reaching the ground in (c) Hyperbola Cd) Circle
s(g=10m's 2)is: (EAMCET1993]
,.
57. A ball is thrown upwards and returns to the ground
(a) 12 (b) 6 describing a parabolic path. Which of the following
(e) 2 (d) 1 quantities remains constant?
[Hint: See note in solved Question Xl (a) Kinetic energy of the ball
50. A wooden block is dropped from the top of a cliff 100 m (b) The speed of the ball
high and simultaneously a bullet of mass 109 is fired from (c) The vertical component of velocity
the foot of the cliff upwards with a velocity of 100 mls. The (d) The horizontal component ofveloeity
bullet and wooden block will meet each other after a time:
58. Ifa body A of mass M is thrown with vclocityv at an angle
(EAMCET 1991( 30° to the horizontal and another body B of same mass is
(a) 10 s (b) 0.5, thrown at an angle of 60° to the horizontal, the ratio of range
(e) I, (d) 7, of A and B will be: (CDSE 1992]
51. A stone is dropped into a lake from a tower 500 m high. The (a) [:./3 (b) ./3:[
sound of the splash will be heard by a man on the tower (e) I: 3 (d) [: 1
after: (CPMT 1992]
59. It was calculated that a shell when fired from a gun with a
(a) 21 s (b) 10, certain velocity and at an angle of elevation 5rr. / 36 radians
(e) l1.5s . (d) 14, should strike a given target. In actual practice it was found
52. A lead sphere of mass 20 kg has the same diameter as an that a hill just intervened in the trajectory. At what angle of
aluminium sphere of mass 72 kg. The spheres are elevation should the gun be fired to hit the target?
simultaneously dropped from a tower. When they are 10 m (a) 5rr. radian (b) 7rr. radian
from the ground, they have identical: 36 36
(a) Kinetic energy (b) Potential energy [I"d·
()
e - r a Ian (d) 13rr. radian
(c) Momentum (d) Acceleration 36 36
53. A block slides down a smooth inclined plane when released 60 . A ball of mass M is thrown vertically upwards. Another ball
from the top, while another falls freely from the same point: of mass 2M is thrown at an angle 9to vertical. Both of them
stay in air for the same period of time. The heights attained (a) Zero (b) K
by the two are in the ratio: (c) KI2 (d) Ki../2
(a) 1:2 (b) 2:1 70. A man can throw a ·stone 80 m. The maximum height 1
(c) I: I (d) I :cosO which it will rise in metres is :
is
61. A particle projected at angle of 45" with a velocity of9.8 (a) 10 (b) 20
mls. The horizontal range will be: (c) 40 (d) 50
(a) 9.8 m (b) 4.9 m [Hint: See § 6.6 part (4)]
(c) 9.8/../2m (d) 9.s.J2m 71. The greatest height to which a man can throw a stone is .
62. The range of a projectile, when launched at an angle of 15" The greatest distance to which he can throw will be :
with the horizontal is 1.5 km. What is the range of the (a) hl2 (b) h
projectile when launched at an angle of 45° to the (c) 2h (d) 4h
horizontal?
[Hint: See § 6.6 part (5)]
(a) 1.5 !an (b) 3 !an
72. A bomber is flying horizontally with a constant speed (
(c) 6 !an (d) 0.75 !an 150 mls at a height of78.4 m. The pilot has to drop a born
63. An object is thrown along a direction inclined at an angle of at the enemy target. At what horizontal distance from fr
45° with the horizontal. The horizontal range of the particle: target should he release the bomb?
(a) Is equal to vertical height (a) Om (b) 300m
(b) Is equal to twice the vertical height (c) 600 m (d) 1000 m
(c) Is equal to thrice the vertical height 73. A box containing food supplies is released from a
(d) Is equal to four times the vertical height aeroplane moving horizontally at a height of 490 m with
64. At what angle to the horizontal should an object be velocity of 180 kmlhr. The box will move horizontall
projected so. that the maximum height reached is equal to while falling just before striking against the earth by:
the horizontal range? IEAMCET 199:
(a) tan€l=2 (b) tan€l=4 (a) 180 m (b) 98 m
(c) tanO~2/3 (d) 0~3 (c) 500 m (d) 750 m
65. Two projectiles, one fired from the surface of the earth with 74. A person standing at some distance from a high tree thro'>'
speed 5 mls and the other fired from the surface of a planet a stone taking aim at a fruit hanging from that tree. The fru
with initial speed 3 mis, trace identical trajectories. begins to fall freely at the time when the person throws tt
Neglecting friction effect the value of acceleration due to stone. Of the following statements which is correct?
gravity on the planet is: ICPMT1991
(a) 5.9 mis' (b) 3.5 mis' (a) The stone moves above the falling fruit
(c) 16.3 mis' (d) 8.5 mis' (b) The stone strikes the fruit if the stone is thrown with
definite velocity
66. A stone is thrown with a velocity V making an angle €I with
the horizontal. The horizontal distance covered by it, before (c) The stone moves below the falling fruit
it falls to the ground, is maximum when 9 is equal to: (d) The stone always hits the fruit
(a) 0 0
(b) 30 0
(c) 45 0
(d) 6QO [Hint: See Question XIII]
67. A body is projected with a speed u at an angle to the 75. A particle is thrown with the speed u at an angle 0: with It
horizontal to have maximum range at the highest point; the horizontal. When the particle makes an angle p with t~
velocity is: horizontal, its speed will be :
(a) zero (b) u (a) ucoso: (b) ucoso:secp
(c) ul../2 (d) u../2 (c) ucoso:cosp (d) useco:cosp
68. A grasshopper finds that he can jump a maximum 76. A body is dropped from a plane moving with constal
horizontal distance of 0.8 m. With what speed can he travel horizontal velocity. The path of the body as seen by
along the road ifhe spends a negligible time on the ground? person on the plane will be:
(a) 2 mI, (b) 2.8 mls (a) Straight line (b) Parabolic
(c) 104 mls (d) I mls (c) Hyperbolic (d) None ofthese
69. A cricket ball is hit for a six leaving the bat at an angle of 77. Two particles A and B are shot from the samc height at t =
45 0 to the horizontal with kinetic energy K. At the top the in opposite directions with horizontal velocities 3 mls and
kinetic energy of the ball is : mls respectively. If they are subjected to the same vertic:
acceleration due to gravity (g '" 9.8 m1s 2 ), the distance This result follows from:
between them when their velocity vectors become mutually (a) Newton's first law (b) Newton's second law
perpendicular is : (c) Newton's third law (d) None of these
(a) 1.059 m (h) 1.412 m 85. Ifin a hydrogen atom at rest the momentum of the e lectron
(c) 2.474 m (d) 9.8 m
78. Between two stations a train first accelerates uniformly,
at any given time be i, the momentum of the proton at that
time is :
then moves with uniform speed and finally retards
~
uniformly. If the ratios of the time taken for acceleration, (0) p (b) pll836
uniform speed and retarded motions are 1 : 8 : 1 and the
~
maximum speed of the train is 60 klTJ/hr the average speed (c) Zero (d) - p
of the train over the whole journey is :
86. The revolver recoils after a bullet is fired because:
(0) 25 kmlhr (b) 54 kmIhr
(a) Thc man draws it backwards out of fear
(c) 40 kmlhr (d) 50 kmlhr
(b) The trigger of the revolver moves backwards
79. When a projectile is projected with very large velocity from
earth's surface, its path will be : (c) The forward motion of the bullet imparts to the
revolver an equal and opposite reaction
(a) Parabolic (b) Hyperbolic
(d) The que3tion is irrelevant
(c) Elliptical Cd) Semicircular
87. Working of rocket or jet is based on:
80. A body is projected vertically up from the ground. Taking
air resistance into account, if t I is the time taken in going up (a) Newton's I law (b) Newton's II law
while t 2 in coming down 10 staning point, then: (c) Newton's III law (d) All the three laws
(a) I I > / 2 88. When a horse pulls a wagon, the force that causes the horse
(b) I ) => 12
to move forward is the force:

(c) I I < /2
(a) He exerts on the wagon
(d) 'Ican be greater or smal ler depending upon the initial (b) The wagon exens on him
(e) The ground exerts on him
velocity of the body
[Note: In upward motion, resistance of air acts downward while in (d) He exerts on the ground
downward motion it acts upwards, always opposite to velocity.] 89. A man is at rest in the middle of a pond on perfectly smooth
81. The property of a body due to which it resists any change in ice. He can get himself to the shore by making use of
its state of rest or of uniform translatory motion is called its: Newton's:
(a) Mass (b) Weight (a) First law (b) Second law
(c) Inertia (d) Moment of inertia (e) Third law (d) Law of gravitation
82. When a mango tree is shaken mangoes are detached from 90. A material body A of mass m] exerts a force on another
the tree (branches) because: material body of mass mz . If the acceleration of B be a 2 , the
magnitude of the acceleration of A is:
(a) There is no option to them except to fall down
(a) zero (b) mz Q 2 1m,
(b) Mangoes experience a net downward pull towards
ground (c) m,
Q I fI12
2 (d) Q 2
(c) Mangoes tend to continue in the state of rest and 91. There are three Newton's laws of motion namely I, II and
branches go into motion III; we can derive:
(d) Branches tend to continue in the state of rest and the (a) II and III laws from the I law
mangoes go in motion (b) III and I laws from Ihe II law
83. Newton's II law of motion connects: (e) I and lllaws from the III law
(0) Momentum and acceleration (d) All the laws are independent of each other
(b) Change of momentum and velocity 92 . The ratio of the inertial to gravitational mass is equal to :
(c) Rate of change of momentum and external force (0) 112 (b) 2
(d) Rate of change of force and momentum (c) I (d) Nofixednumber
84. The distance x covered in time t by a body having initial 93. When a body moves with high speed its mass:
velocity Vo and having a constant acceleration ais given by (a) Remains unchanged (b) Decreases

x=vot +(~)at2 (c) Increases (d) None of these


94. If the rest mass ora body is mo' its mass when it is moving (a) ma (b) mg
with speed O.6c will be: (e) (mg -ma ) (d) (mg +ma)
(a) mo (b) (5/3) mo 103. A parachutist of weight W strikes the ground with his"legs
(e) (5/4) mo (d) (4/3) mo fi xed and comes to rest with an upward acceleration of 3 g.
95. Which is the correct statement ? The force exerted by him on the ground during landing is:
(a) The mass of a body can bechanged without changing (a) W (b) 2W (e) 3W (d) 4W
its weight ~.' 1
104. A lift is moving upward with an acceleration of3 mls and
(b) The weight ofa body can be changed without changing g = 9.8 mls 1 . To carry a bag of mass 5 kg inhis hands a man
its mass in the lift has to exert a force of:
Cc) The weight of a body cannot be changed witho'ut (a) IS N (b) 49 N (e) 34 N (d) 64 N
changing the mass 105. A monkey is descending from the branch of a tree with
(d) One cannot be changed without changing the other constant acceleration. If the breaking strength is 75% of the
. '-', . .
96. If the force of gravity suddenlY ,disappears : weight of the monkey, the minimum acceleration with
(a) The mass or all bodies Will become zero , . which the monkey can slide down without breaking the
(b) The weight' of all bodies 'will bec~me zero ' . " branch is: (CBSE 1993]
" (e)' ,Both mass' and weight ~fall bodieirwill become ~ro (a) g (b) 3g 14

Cd) '~ Neilher mass nor weight of all bodies will become zero (e) g 14 (d) g 1 2

97. When 8' body is. taken from the equator to the Wles its
106. A 60 kg man stands on a spring scale of a lift. At some
weigh'I: ' . instant he finds the scale reading has changed from 60 kg to
.. ' 50 kg for a while and then comes back to the original mark.
(a) Remains ,cOnstant .(b) Incz:eases What should he conclude?
(c) Decreases (d) Becomes indeterminate (a) The lift was in unifonn motion upwards
98. A thief stoic a box full of valuable articles of weight W and (b) The lift was in unifonn motion downwards
while carrying it on his head, jumped down from a wall of
(c) The lift while in accelerated motion upwards suddenly
.height h to the ground . .Before he reached ground he
experienced a load of: ' stopped
Cd) The lift while in accelerated motion downwards
(a) 2W (b) W
suddenly stopped
(e) W 12 (d) Zero
107. A body suspended from a spring balance is executing
99, A weight is suspended from the roof of a lift by a spring simple hannonic motion in a vertical plane, The reading of
balance. When the lift is stational)' the spring balance reads the balance is :
W. If the lift suddenly falls freely under gravity the reading
(a) Maximum when the body is at highest position
on the spring balance will be:
(b) Maximum when the body is at lowest position
(a) Wg (b) 2Wg
(c) Max.imum when the body is in equilibrium
(e) W I2g (d) Zero
(d) Same for all positions ofth€.: body
100. The tension in the rope ofa lift is 1000 kg weight when at
rest. When it is in motion the tension remains unaltered; [Hint: See solved Problem 49]
from this we conclude that: 108. A body will not be weightless if it is:
(a) Lift is accelerated upwards (a) A projectile
(b) Lift is accelerated downwards (b) A satellite
(c) Lift is moving with uniform velocity (c) Floating in a liquid
(d) Lift is moving with uniform velocity or is at rest (d) In a lift fa lling with acceleration a > g
101. A man of weight W is standing on a lift which is moving 109. Velocity verS/ls time graph for a body projected vertically
upward with an acceleration' a'; the apparent weight of the upwards is: IEAMCET 19951
man is: (a) Parabola (b) Ellipse
(a) W(I +alg) (b) W (c) Hyperbola (d) Straight line
(e) W(I-alg) (d) W (I_a' I g') 110. A projectile is fired horizontally with a initial speed of 20
102. Consider an elevator moving vertically up with an mls. Its horizontal speed 3 sec later is :
acceleration a. The force exerted by a passenger of mass m (a) 20 mI, (b) 6.67 mI,
on the floor of the elevator is : (e) 60 mI, (d) 29.4 mI,
TRANSLATORY MOTION
273
Ill. Ifearth stands still what will be its effect on man's weight?
116. Which of the following figures represents the motion of a
IAFMCI9941 body moving in a straight line under constant acceleration?

!LL
(a) Increase (b) Decrease
(c) Remains the same (d) None of these
112. The displacement time graph for two particles A and B
(a)
are straight lines inclined at angles of 30° and 60° with the
time axis. The ratio of the speeds v A : vB is: (CPMT 1990)
t-
(a) 1, 2 (b) 1,-13
(e) -13, 1 (d) 1,3
113. The displacement of a particle as a
function of time is shown in Fig. 20 E
I
(e)!~
6.76. The figure indicates that;
10 ."
00
t-
117. A particle is thrown vertically upwards with a velocity v. It
(a) The particle starts with a certain tin sec._
velocity but the motion is 01020304050 returns to the ground in time T. Which of the following
retarded and finally the particle Fig. 6.76 graphs correctly represents the motion? ICPMT 19931
stops
(b) The velocity of the particle is constant throughout
(c) The acceleration of the particle is constant
(d) The particle starts with a constant velocity, the motion (a) >
~M
. !
~ ,
,: (b) v~ lime _
is accelerated and finally the particle moves with a T/2 T
another constant velocity TIms_ -v
114. The graph between the S
o

v~
displacement x and time t for a v
particle moving in a straight line
c
A B T
is shown in the diagram. During (e) (d)
T/2
the intervals OA, AB ,BC and CD
the acceleration of the particle O'~----- -v ------------- o T/2 T
is: Fig. 6.77
OA AB BC CD lIS. Velocity versus time graph of a
(a) + 0 + + body is shown in Fig. 6.7S. It t A
explains that v
(b) - 0 + 0
(e) + 0 + (a) At B force is zero
(d) - 0 (b) At B there is a force but t-
0 Fig. 6.78
115. Figure shows some velocity versus time graphs: towards motion
(c) At B there is a force but opposes the motion

I~
(d) Forces are equal at A,B and C
.~ 119 . The velocity versus time curve of a moving point is shown
(8) .l! in Fig. 6.79. The retardation is :
!Ii
TIme-- Time --
11"'-
sot
!:~~
o 10 20 30 40 50 60 70
t_
TIme-- Fig. 6.79
Only some of these can be realised in practice. These are :
(a) 1 cmlsec 2 (b) 2 cm/sec 2
(n) (A), (8) 'nd (D) (b) (A), (8) and (C)
(c) 3 cm/sec 2 (d) 4 cm/sec 2
(e) (8), (D) (d) All
PHYSICS FOR COMPETITIONS - Voi. I
274

120. The area of the shaded portion of 125. An object is dropped from rest. Its velocity versus
the graph represents: .:\ .: displacement graph is:

ca)!~
(a) The average acceleration
(b) The maximum KE 1:-+

.-
(e) The momentum FI~. 6.80 Cb)tL
(d) The displacement s_

!l==
121. A rocket is fired upwards. Its velocity vet:sus'time' graph is

ce)!~
shown in Fig.. 6.81. The maximum height reach'ed by the
rocket is:
Cd)
t 1200 s_
s_
~E 600 f. Two ,
126 . An aeroplane is rising vertically with acceleration
.5 . 140 stones are dropped from it at an interval of time t. The
>
o 20 40 60 80100120 ',: . \ distance between them at time t' after the second stone is
dropped will be :
12 Iinsec.-
Fig. 6.81:-
132
Ca) f)
~,(g+ (b) ~,(g+
: ; 2 _ 1+21'
f)
Ca) 7.1 km Cb) 79.2 km 2 "
(e) 72 kIn (d) Infinite' (e) ~,(g+ f) Cd) ~( gif)
122. In the above problem the acceleration of the rocket during · 2 1+1'· . .
2 t + t'

burning interval is : 127. A swimmer crosses a fl~wing stream of breadth b to and fro
(b) (12/1200) mh;2
in time TI , The time taken to cover the same distance up and
Ca) (1200/12) mis'
down the stream is T2 , 1fT) is the time the swimmer would
Ce) C1200x 12) um' Cd) (1200/132) mis' take to swim a distance 2b in still water, then:
123. The velocity versus time graph of a body moving in a (a) Tl =-T2 ·T3 (b) TI2 '= T2 ·T3
straight. line is as follows. The distance travelled 'by the
2
body in 5 sec is: (c) Ti =TI .T) (d) T3 = TI·T; '·

t 128. A street car moves rectilinearly from station A to the next


g +2 stop B ,with an acceleration varying according to the law
~ +1 17 1.5 f = a -bx, where a and b are positive constants and x is its
,. c a 2 3 5
tin distance from station A . The distance AB is then equal to:
';: -1 sec.
-2 Ca) ~ Cb) :
Fig. 6.82

Ca) 2m (b) 3m,_' ., Ce) -


a
11>
Cd) 2a - b
Ce) 4m (d) 5m
129. A particle is projected at an angle a. with the horizontal
[Note: In this problem dispiacement = 3 m] from the foot of an inclined plane making an angle p witil
124. The velocity versus time graph of a linear motion is shown horizontal. Which of the following expressions holds good
in the Fig. 6.83. The distance from the origin after 8 sec is: if the particle strikes the inclined plane nonnally?
Ca) eot~ " tanCa - ~) Cb) eot~"2tanCa - ~)
(c) cota.",tan(a. - p) (d) cota.=2tan(a.-p)
130. A particle starts from rest and moves with an acceleration l
8
which varies with time t according to the equation a = kt
t In sec.
where k is a constant. The displacement S ofthe particle a
time t is :
Fig. 6.83
Ca) -1 kt ' Cb) ~at2
Ca) 18 m (b) 16 m 2 2
1 ,
(c) 8 m Cd) 6 m Co) -at Cd) k/3
[Hint: Distance from origin means magnitude of the displacement1 6
131. A boy throws up a ball vertically inside an elevator, with a 137. If R is the range of a projectile on a horizontal plane and hits
velocity of 10 mls relative to the elevator. It takes I second maximum height, the maximum horizontal range with the
for the ball to reach the hands of the boy. The acceleration same velocity of projection is :
of the elevator, taking g = lOmls 2, is: R2 h2 R2
(a) 2h (b) - (c) 2R+- (d) 2h+ -
(a) Zero (b) 10 m/s 2 8h 8R 8h
(c) 30 mls 2 (d) 20 mls 2 138. A particle is projected upwards with a velocity of 100 m/sec
at an angle of 60" with the vertic~l. Find the time when the
132. A body of mass 111, moving along the positive x direction is particle will move perpendicular to its initial direction,
subjected to a resistive force F =Kv 2 (where K is a taking g = tomlsec 2 :
constant and v the particle velocity). lfm ::: 10 kg, v =: 10 mis
at t = 0, and K = 2N (ml s)-2 the velocity when t = 25 is:
(a) to second (b) 20 second
(c) 5 second (d) 10 ~ second
(a) 10 mls (b) 2 mls 139. A cannon ball has the same range R on a horizontal plane
3
for two angles of projection. If hi and h2 are the greatest
10 3
(c) - - mig (d) - mig heights in the two paths for which this is possible, then:
3 10
(a) R =h 1h 2 (b) R =4~hlh2
133. It takes one minute for a person standing on an escalator to
reach the top from the ground. If the escalator is not (c) R=~hlh2 (d) R = (h 1h2 )114
moving, it takes him 3 minute to walk on the steps to reach
the top. How long will it take for the person to reach the top 140. A tennis ball is released so that it falls vertically to the floor
ifhe walks up the escalator while it is moving? and bounces again. Taking velocity upwards as positive,
which of the following graphs best represents the variation
(a) 2 minute (b) 1.5 minute
of its velocity v with time t?
(c) 0.75 minute (d) 1.25 minute
134. For..!. of the distance between two stations a train is
m
unifonnly accelerated and 1. of the distance it is unifonnly
n
(a) ~~t (b) ~rt-t
retarded. It starts from rest at one station and comes to rest
at the other. The ratio of the greatest velocity to the average
velocity will be :

(a) (l+~-~) (b) h~ + ~)


(c) (~ +~-I) (d) (;+~) 141. Two p~icles A and B are thrown simultaneously from the
same point at the same angle of projection but with the two
Y different initial velocities (v+ u) and (v ;- u) respectively.
135. A rod AB moves towards the origin 0
Which of the following statements will be true in respect of
of a fixed rectangular co-ordinate A
their motions?
system, always perpendicular to the
bisector of the angle XOY, with a (a) The difference in their maximum heights is (2uv/ g)
velocity v. The speed of end B with (b) They reach their maximum heights at a time interval of
respect to 0 will be : (2usin 9)/ g

2uv~n 29)
(a) " (b) 2»
B X
Fig. 6.84 (c) They will be separated by the distance Of(
v
(c) (d) .J2v
2 when they reach the ground agai'n
136. Two plane, smooth surfaces are parallel to each other and (d) They are never in the same horizontal level during their
are initially a distance of 2 metre apart. The two surfaces flights
approach each other with a velocity of 1 cm/sec. A particle 142. Two trains take 3 second to pass one another when going in
starts with a velocity of 4 cm/sec from one surface and opposite directions but only 2.5 second if the speed of one
collides nonnally and elastically on the other surface from is increased by 50%. The time one would take to pass the
the time the two surfaces start moving. The collisions other when going in the same direction at the original speed
continue back and forth till the surfaces touch each other. is:
The total distance covered by the particle is : (a) 10 sec (b) 12 sec
(a) 2m (b) 1m (c) 4m (d) 3m (c) 15 sec (d) 18sec
143. Two bodies move in a straight line towards each other at ficld. What will be the minimum constant speed of the man
initial velocities v I and v 2 and with constant acceleration a J so that he may catch the dog before the bend in the track?
and a 2 directed against the colTesponding velocities at the (a) 1.50 km/h, (b) 40 mI,
initial instant. The maximum initial separation lmax
(c) 90 kmlhr (d) 20 mI,
between the bodies for which they will meet during the
motion is: 149. The speed of a body moving on a straight track varies
according tov =21 + 13forO ~ , ~ 5s, v = 31 + Sfor 5 < t 5. 7
v2 v 2 s and v = 4t + 1 for t < 7 s. The distances are measured in
(a) _I +--1.
al fl2 metre. The distance in metres moved by the particle at the
end of IO second is :
(a) 127 (b) 247 (c) 186 (d) 313
150. The speed of a car was 50 kmJhr for the first 900 s, then 40
144. A body moves from rest with a constant acceleration. kmlhr for the next 50 Jon and then the car decelerated
Which one of the following graphs represents the variation uniformly at 10 kmfhrl till it came to fest. The average
of its kinetic energy Ek with the distance S travelled? speed of the car was:

(a) J,U s_
(b) J,LL s_
(a) 50 kmIlrr
(c) 30 kmIlrr
(b) 7.2 mI,
(d) 9.0 mI,
15 I. From the top of a towef a stone is thrown up which reaches
the ground in time' I' A second stone thrown down with the

J,~ J,~
same speed reaches the ground in a time t 2' A third stone
released from rest from the same location reaches the
(c) (d) ground in a time 13 , Then :
s_ (a) -I=-
I+I- ()b '"
'3 =tl -1 2
145. A particle moving with constant acceleration, travels 10m '3 " '2
in the first 5 second and another 10m in the next 3 second.
The distance, it will travel in the nex.t 2 second will be :
(c) 13 =', +'2
2
(d) t3 =M
(a) 8.33 m (b) 5.67 m 152. The x and y displacements of a particle in the x· y plane at
(c) 9.37 m (d) 10 m any instant are given by x = aT2 and Y= 2aT where a is a
146. Two particles are initially located at points A and B distant constant. The velocity of the particle at any instant is given
d apart. They start moving at time t = 0 such that the by:
~

velocity u of B is always along the horizontal and velocity (a) 4a~T' +4 (b) "'~T' + I
~ ~

.v of A is
'"
continuously aimed at B. At
~ . t = 0, u is (c) 4a~r' + I (d) '!. Jr' + 4
2
perpendicular to v. The particles will meet after time:
153. A parachutist steps from an aircraft, falls freely for two
vd v 2 +u 2 second, and then opens his parachute. Which of the
(b)
ud following acceleration time (a ·, )graphs best represents his
downward acceleration a during the first 5 second?

a~1 a~ 1
vd
147. A ball rolls off the top of a staircase with a horizontal (a) (b)
velocity urn s - , . If the steps are h metre high and w metre
wide the ball will hit the edge of the nth step if:
gwl
(a) n~ --
2hu2

(c) n= -
2u'
-
2hu2
(b) n = - -
gw2 (c) all (0) a l = 1

gw'h g
154. With what speed should a body be thrown upwards so that
148. A man starts chasing his dog IO second after the latter runs the distances traversed in the 5th second and 6th second are
along a straight track at a unifonn acceleration of 0.5 m/ s 2 . equal? .
The track is 2 km long after which it bends away into the (a) 58.4 mI, (b) 49 mI, (c) 98 mI, (d).J9s mI,
TRANSLATORY MOTION
277
155. If co-ordinates of a moving point at time 1 are given by
162. The velocity ofa particle is v =v o + gl + ft2. Ifits position
x=a(t +sin t)and y = a(l - cos/), then:
is x = Oat 1 = 0, then its displacement after unit time (I = 1)
(a) The slope of acceleration time graph is zero
is: [AIEEE 20071
(b) The slope of velocity-time graph is constant
(a) vo+g/2+J (b) vo+2g+3J
(c) The direction of motion makes an angle 1 / 2withx-axis
(c) vo+g/2 + //3 (d) vo+g+J
(d) All of the above
163. A particle located at x = Oat time t = 0, starts moving along
156. A particle moves along the positive branch of the curve
2 the positive, x-direction with a velocity' v' that varies as
x t2 v = nJX. The displacement of the particle varies with time
y = - where x = - , where x and yare measured in metre
2 2 as: (AIEEE2006)
and t in second. At t = 2 sec, the velocity of the particle is: ll2
(a) t (b) /3
(a) (2f -4j)mlsec (b) (2f +4j)mlsec
(e) ,2 (d) ,
(e) (2i+2j)mlsee (d) (4i - 2])mlsee
164. A ball of mass 0.2 kg is thrown vertically upwards by
157. A particle moving with a unifonn acceleration along a applying a force by hand. If the hand moves 0.2 m while
straight line covers distances a and b in successive intervals applying the force and the ball goes upto 2 m height further,
of p and q second. The acceleration of the particle is: find the magnitude of the force. Consider g = 10 mls 2 .
(a) pq(p+q) (b) 2(aq-bp)
[A1EEE 20061
2(bp aq) pq(p_q) (a) 20N (b) 22 N
(e) bp-aq (d) 2(bq-aq) (e) 4N (d) 16 N
pq(p q) pq(p+q) 165. A man throws balls with the same speed vertically upwards
158. A particle's position as a function of time is described as one after the other at an interval of2s. What should be the
y(t) = 2t 2 + 31 + 4. What is the average velocity of the speed of the throw so that more than two balls are in the sky
particle fromt = 0 to t = 3sec?[CET (Kurukshetra) 2002) at any time? (Giveng =9.Sm /s2 ) [AFMC2007)
(a) 3 mlsec (b) 6 mlsec (a) Any speed less than 19.6 mls
(c) 9 mlsec (d) 12 m/sec (b) Only with speed 19.6 mls
159. A projectile is thrown in the upward direction making an (c) More than 19.6 mls
angle of 60° with the horizontal direction with a velocity of (d) At least 9.S mls
147 ms - 1. Then the time after which its inclination with the 166. A car moves from X to Y with a unifonn speed viland
horizontal is 45°, is: fJCECE 2007) returns to Y with a unifonn speed v d' The average speed for
(a) 15 S (h) 10.98 S this round trip is: IUPSEE 2007, CBSE PMT 2007)
(e) 5.49 S (d) 2.745 S (a) 2vdvll (b) ~vrlVd .,-
160. A coin is dropped in a lift. It takes time t 1 to reach the floor vd +v/l
when lift is stationary. It takes time 12 when lift is moving
up with constant acceleration, then: IAFMC 2005) (e)
(a) t1 >t2 (b) 12 >11
(c) t1 = / 2 (d) t1 » /
167. A point initially at rest moves along x-axis. Its acceleration
2
161. The v-I graph for a particle is shown. The distance varies with time as a = (61 + 5)m/ s2. Ifit starts from origin,
travelled in the first four seconds is: (SEE (UPTU) 2005) the distance covered in 2s is: [RPMT 2007]
mls (a) 20 m (b) 18 m
(e) 16 m (d) 25 m
16S. The height yand the distance xalong the horizontal plane of
a projectile on a certain planet (with no surrounding
atmosphere) are given by y = (St _ 5t 2 ) metre and x = 61
t (sec) metre, where 1 is in seconds. The velocity of projection is:
2 4
Fig. 6.85 [RPMT20071
(a) 12m (a) 8 mls
(b) 16m
(e) 20m (b) 6 mls
(d) 24m
(c) 10 mls
(d) not obtained from the da.ta
278 PHYSICS FOR COMPETITIONS - VOl. 1

169. A particle is projected at 60 Q to the horizontal with a kinetic 173. A boy is riding on a flat car of a train moving with velocity
energy K. The kinetic energy at the highest point is : 10 mls. The boy throws a ball which according to him
IAIEEE 2007; WB (JEE) 20091 moves an angle of 600 with the horizontal and in line with
the track. An observer on the ground observes the baH to
(a) KI2 (b) K
rise vertically. How high does he see the ball rise?
(c) zero Cd) K /4
(g '" IOm/s2)
170. The particle A is projected from point P with velocity u
along the plane and simultaneously another particle B with (a) 10 m (b) 15m
velocity v at an angle' a.' with vertical. The particles collide (e) 20m (d) 25m
at point Q on the plane. Then: 174. A lift is moving with a uniform down~cceleration of
2 m / s 2 . A ball is dropped frem a height2 m from the floor
.
'0 B
of lift. Find the time taken after which ball will strike the
floor? (Take g = 10m/ s2)
pi<""'-,
A (a) .J2 sec .' (b) 2 sec
I I
Q, (c) - sec (d) .J2 sec
2
Fig. 6,86 175. A river is flowing with a speed of I kmIhr. A swimmer
wants to go to point C starting from A. He swims with a
(a) -v sin (9 - a)=11 (b) vcos(e-a)=u speed of 5 kmlhr at an angle e w.r.t. the river flow. If
(c) v = u (d) '<m(6 - a) •• AB = BC = 400 m. At what angle with river tank should
171. A man hold s an umbrella at 30 g with the vertical to keep swimmer swim?
himself dry. He, then runs at a speed of l Omls and finds the 400m
rain drops to be hitting vertically. Study the following B C
statements and find the correct options.
CA) velocity of rain w.r.t earth is 20 mls . 400m
(8) velocity of rain w.r.t man is 1OJ3 mls
(C) velocity afrain w.r.t earth is 30 mls A

Fig. 6.88
(D) velocity of rain w.r.1 man is 10.J2 mls
(a) Statements (A) and (B) are correct (a) 370 (b) 53 0 (e) 0" (d) 900
(b) Statements (A) and (C) are correct 176. A juggler maintains four balls in vertically upward
(c) Statements (C) and (D) are correct molion. He attempts next ball after.! seconds. For the
4
(d) Statements «B) and (D)-are correct show to go one, what should be the height fo r which he
172. Two particlcs are projected from A and C simultaneously throws the ball ? (g = 10m/ s2)
towards each other and they collide in air. The ratio of
vertical velocities of particles at C and A, VCY and VAy is
(a) 1.25 III (b) 5 m
same as the ratio ofCO and AD. IfVAy = 2m/ s and VAx = I (e) 25 m (d) 1.0 m
mis, find Vex where VAX andVex are horizontal velocities 177. On an inclined plane two particles A and B are projected
of particles at A and C respectively: with same speed at the same angle with the horizontal,
particle A down and particle B up the plane. If the ratio of

A~:X
time of flight of A and B is cot e, where 9 is the angle al
which B is projected measured from inclined plane. Find
the angle at which particles are projecled.
(a) 90 0 (b) 60 0
y (e) 300 (d) 45 0
Fig. 6.87 178. A platform P is moving with a velocity v p over
x+ y 3x+2y hemispherical shell. A vertical rod AB passing through a
(a) (b) hole in the platform is moving Oil the shell and remains
2y x
vertical. There is sufficient friclion between rod and shell to
3x-2y lx- Y stop Ihe slip. C is the crown of the shell and 0 is its centre
(e) (d)
x 2x LBOe :=: €I at any instant. Find the velocity of point B in
downward motion at that instant.
iIi proponion to the
+-I~­
183. The current velocity of a river grows
distance from its bank and reaches the maximum value Vo
~Vp in the middle. Near the banks the velocitY is zero. A boat is
. :, ,/ 8 moving along the river is such a manner that it is always
:, ' ' perpendicular to current and the speed of the boat in still
~
:&/ water is u. The distance through which the boat crossing the
0:' river will be carried away by the current if the width ofriver
Fig. 6.89 is C, .will be :
(a) vp sine (b) vp cose (a) CVo (b) CVo
(e) vptan9 Cd) v p cot9 2" 4"
179. A: particle is projected vertically from the ground, takes (e) CVo (d) 2CVo
time f 1 upto point A, t 2 from point A to B <l:nd titl).e t 3 from
point B to highest point and back to the ground. Find the
., " u
"
184. A particle moving in a straight line covers half the distance
height of the middle point of A and B ~rom the ground. with speed' 3 mls. The other half of the distance is covered
~[tr +ti +2 (t,'2
in two equal time intervals with speed of4.5 rills and 7.S
(a) +( 2 / ) +/ 3 1,)]
mls respectively. Average speed of the particle during this

(b) "'g[t r ti +1112 +tlf) + ' 2'3


4 2+2" 1 motion is:
(,) 4.0 mi. (b) 5.0 mi.
(e) 5.5 mi. (d) 4.8 mi.
(~ ![tr +t i+ tlt2+t)')+t2t)1 185. A train moving with unifonn speed passes a pole in 10 sec
2 and a bridge of length 1200 m in 130 sec. Speed of the train
(d) none is:
180. Two particles A and B are projected simultaneously from (,) 90km/hr (b) 72 kmIhr
point O. Their maximum heights and ranges achieved are (e) 36 km/hr (d) 54 kmIhr
shown. Find correct option.
y' 186. Engine of a train that is moving with unifonn acceleration
passes a pole with speed' u' while the last compartmcnt
2H •••• Particle A passes the pole with speed ·v'. The middle point of the train
passes the given pole with speed:
H -- Particle B
(a) v - u (b) v+u
2 2

(d) ~v2 +2"'


R,
Fig. 6.90 187. For a particlc moving along a straight line, its velocity' v'
(a) necessarily R z = 2Rj (b) range may be same and displacement's' are related as v 2 ::= cs, here c is a
constant. If the displacement of the particle at t ::= 0 is zero,
(e) ranges cannot be same Cd) none
its velocity after 2 sec is :
181. The heat generated H in a circuit is given by ]2RT with
usual notations. The per<:en~ge errors in measuring I,R (a) £so (b) 08 0
and Tare 3%, 2%, 1% respectively. The maximum error in 2
measuring heat will be: (c) £s- I (d) 08- 1
(a) 6% (b) 7% 2
(c) 8% (d) 9% 188. A car accelerates from rest at a constant rate' a.' for some
time after which it decelerates at a constant rate pto come to
182. Three vectors P,Q and Ii. are such that IQI=A.J2 and the rest. If the total time elapsed is t, maximum velocity
angles between P and Q ,Q and Ii. ,Ii. and Pare 90°, 150°, reached by the car is :
120° respectively. Find the value of !P!::=
(,) ~~(a+~) (b) ~t
(,) ~ (b) A.J2 t (a-~)
.J2 .,f3
(e) ~, (d) ~+~,
(e) 2A A
(d) a+~ a~
.,f3 2
189. in Q. 188, time for which the car decelerates is: (a) 18 mI, (b) 15m1s

(a) -'=--' (b) -~-, (c) 12 mI, (d) 9 mI,


5
a+p a+p

(c) *' (d) ~,


190. In Q. 188, total distance travelled by the car is ;
196. A stone dropped from the lOP of a tower travels - th of the
9
height of tower during the last second of fall. Height of the
tower is: (Take g = 10 m /s2)
(a) 52 m (b) 36 m
(a) a+~,' (b) a-~
"
(c) 45 m (d) 78 m
(a'+~')2 (a'+~')2
197. Engine of a car can produce a maximum acceleration
ap t2 ap (2 2 m /s 2 and its brakes can produce a maximum retardation
(c) (a-~) 2 (d) (a+~) 2 3 m /s 2 . Minimum time in which the car can travel a
191. A person is standing at a distance's' m from a bus. The bus distance 6 km is:
begins to move with constant acceleration' a t rut s2 away (a) 120 sec (b) 100 sec
from the person. To catch the bus, the person runs at a (c) 82 sec (d) 64,oc
constant speed' v' mls towards the bus. Minimum speed of
198. Trains A and B are moving towards each other on the same
the person so that he can catch the bus is:
track with velocities 40 kmlhr and 20 kmlhr respectively. A
(a) .n;;s (b) ..r;;; sparrow which can fly at 30 kmIhr flies off from train A

(c) ..&; (d) fi when the trains are 30 km apart. The sparrow directly
moves towards the train' B' and on reaching there flies back
to ' A' and so on. Distance travelled by the sparrow till the
192. The driver of a train moving at a speed 'vl ' sees a goods
two trains will hit, is :
train a distance' d' ahead of him on the same track and
moving in the same direction with a slower speed 'v 2 " He (a) 60 kin (b) 45 kin
puts on brakes and gives his train a constant retardation' a', (c) 30 kin (d) 15 kin

(a) d < (V I +V 1 )
,
There will be no collision if: 199. A balloon is moving vertically up with a velocity 4 mls.
When it is at a height h. a body is gently released from it. If
it reaches ground in 4 sec, the height of balloon, when the
a
body is released, is: teET (Kar nataka) 2004]
(d) d < (V 1-
v)'
, (Takeg -9.8 m1s')
a a
(a) 62.4 m (b) 42.4 m
193. The deceleration experienced by a moving motorboat. after
(c) 78.4 m (d) 82.2 m
its engine is cut-off is given by dv "" _kv 3 , where k is
d, 200. A packet is dropped from a balloon that is moving upward
constanl.lfv o is the magnitude of velocity at cut~off, the when the balloon is at a height 60 m above ground. If the
magnitude of velocity at time ' t' after the cut~offis : speed of the balloon at .the moment of release of packet is

(a)
v, (b) v,
5 ml s, time taken by the packet to reach ground will be :
(Takeg = IOm /s 2 )
2

,
(c) v e- kl (d)
v, (a) 6 sec (b) 4 sec
(c) 2 sec (d) 3.2 sec
~2V~kt + I 20 1. When a motorcycle moving with a unifonn speed 11 mls is
~94 . A train is moving towards East with a speed 20 mls. A at a distance 24 m from a car, the car stans from rest and
person is running on the roofofthe train with a speed 3 mls moves with a unifonn acceleration 2 ml s 2 away from the
against the motion of train. Velocity of the person as seen motorcycle. If the car begins motion at I = 0, time at whieh
by an observer on ground will be : the motorcycle will overtake the car is ( =
(a) 23 mls towards East (b) 17 mls towards East (a) 8 sec (b) 6 sec
(c) 23 mls towards West (d) 17 mls towards West (c) 3sec (d) l.5sec
195. A packet is released from a baUoon which is moving 202. In Q.20 I , after the car is overtaken by the motorcycle, it will
upward when the balloon is at a height 200 m above again overtake the motorcycle at what time, from t = O?
ground. The packet reaches the ground in 8 sec. Speed of (a) 8 sec (b) 6 sec
the balloon when · the packet is released, is: (Take
(c) 3 sec (d) 1.5 sec
g:lOm/sl)
203. An object is thrown vertically upward with a speed u and it
1 210. Velocity-time graph for the motion of a particle along a
travels 8 m in the last second of its upward motion. If the straight line is as shown in Fig. 6.91.
object is thrown upward with a speed u2 which is twice of v{m/s)
u 1' the distance now travelled by the object during the Jast
second of its upward journey will be : 10
(a) 32m (b) 16m 1 21 ,
(e) 12m (d) 8m ~i======d~~
6'
" ~'1:'==;'1:'7'~1-> t{s)
204. A person is at a distance' d' from a bus when the bus begins
to move with a constant acceleration' a' away from the -10
person. At the same time, the person runs towards the bus
with a unifonn speed. Minimum speed of the person so that
he is able to catch the bus is: Fig. 6.91
(a) ad (b) .Jad Average speed of the particle for the whole motion is :
(e) -i2ad (d) .)sad (a) 9.6 mI, (b) 7.2 mI,
205. A long horizontal helt is moving from left to right with a (e) 8.3 mI, (d) 6.5 mI,
unifonn speed 2 mls. There are two ink marks A and B on 211. Average velocity of the particle for the whole motion in Q.
the belt 60 m apart. An insect runs on the belt to and fro 210 is:
between A and B such that its speed relative to belt is (a) 3.3 mI, (b) 6.7 mI,
constant and equals 4 mls. When the insect is moving on the
(e) 2.7m1, (d) 7.6m1,
belt in the direction of motion of the belt, its speed as
observed by a person standing on ground will be : 212. A particle is moving such that its position vector varies. with
~ ~ ~
(a) 6 mI, (b) 2 mI, time as r = (I-at)t A, where (l and A are constant
(e) 1.5 mI, (d) 4 mI, quantities. At 1 "" 0, the particle is at a position O. At some
206. In Q. 205, if A lies to the left of B, then: later instant' 10 ', the particle is again at O. Velocity of the
particle at the instant lois:
(a) time taken by insect to travel from A to B and time
~ ~
taken by it to travel from B to A are equal (a) 2 A (b) A
(b) time taken by insect to travel from A to B is less than ~ ~

time taken by it to travel from B to A (e) -A (d) -2 A


(c) time taken by insect to travel from A to B is more than 213. In Q. 212, total distance travelled by the particle from t "" 0
the time taken by it to travel from B to A tOI = t o is:

(d) none of the above (a) ~ (b) ~


207. In Q. 205, if A lies to the left ofB, time taken by the insect to 2a a
travel from B to A will be : (e) 2Ao (d) 4AO
(a) 12 sec (b) 15 sec a a
(c) '18 sec (d) 21 sec 214. A body is thrown vertically upward at t = a It is at a height
208. An object is dropped from the top of a tower. It travels a 'I
80 m at two instants and t 2 , then tl/2 ,is:
distance' x' in the first second of its motion and a distance (Takeg = 10 m/s2)
'7x' in the last second. Height of the tower is:
(a) 30 (b) 24
(Takeg = IOm/s2)
(e) 16 (d) 12
(a) 60 m (b) 70 m 215. A body is thrown vertically upward at t = 0. It is at a height
(e) 80 m (d) 90 m 80 m at instants t I and t 2' Also, it is at a height 60 m at
209. A bus starts from rest and accelerates at a unifonn rate 4 instants t; and 2, Then:
1

ml s 2 for certain time. It then moves with a constant speed (a) 11 +/ 2 =t; + / 2 (b) {J +t2 >t; +t2
for some time and finally retards at 4 ml s 2 to come to rest. (c) ' 1 +t2 <ti +/ 2 (d) none of these
Average speed of the bus during the total journey is IS mls 216. An object is projected so that it just clears two walls of
and the total time is 20 sec. Time duration for which the car height 7.5 m and with separation 50 m from each other. If
moves with constant speed is: the time of passing between the walls is 2.5 s, the range of
(a) 18 sec (b) 16 sec the projectile will be: (g = 1bml s 2)
(c) 12sec (d) IOsec (a) 35 m (b) 70 m
(e) 140 m (d) 57.5 m
217. A particle is projected from point A with velocity u at an (a) 2.[3 (b) 7.[3
angle (l with horizontaL In its parabolic path, at point P the g g
particle is moving at right angles to its initial direction of
projection. Its velocity at Pis: (c) 14.[3 (d) 313
g g
(a) u tan 0. (b) u cot a
(e) u cos a (d) u cosecCt. 223. A body is thrown up in a lift with a velocity u relative to the
lift and the time of flight is found to be t. The acceleration
218. A projectile can have the same range R for two angles of with which the lift is moving up is :
projection. If t I and t 2 are the times of flight in the two
u-gt 2u-gt
cases, then: (a) (h)
t
(c) u+gt (d) 2u+gt
t
224. Water drops fall at regular intervals
from a hole at the bottom of a vessel
219. A particle is projected with velocity 20 mis, so that it just placed at a high level. The ninth drop is
clears two walls of equal height 10m, which are at a about to fall when the first drop just
distance 20 m from each other. The time of passing between falls on the floor after being in the air
the walls is: (Take g = lOml s2) for 2 second. The distance between the
I 3rd and the 5th drop at this instant is:
(a) 2 s (b) - s Floor
2 (Taking g = IO~/ s2)
Fig. 6.93
(c) w[i s (d) 2v'iO s 40 (b) 25 m
A (a) -m
220. In 1.0 s, a particle goes from point A to S, 9 4
moving in a semicircle ofradius _l.O m (see 5
(c) 5 m (d) - m
Fig. 6.92). The magnitude of the average 16
velocity is : lIlT 1999) 225. A particle is thrown above, then the correct v - t graph will

n,
(a) 3.14m1s (h) 2.0mls be : [DCE 2007[
B
(c) 1.9 mls (d) Zero Fig. 6.92 V
VI
221. A ball is dropped vertically from a height d above the
ground. It hits the ground and bounces up vertically to a (a) (b) ~
height d / 2 Neglecting subsequent motion and air
resistance, its velocity v varies with the height h above the
ground as:

(a)
v

(b)
v

f-U,d-- h
(lIT 2000]

(c) 16, (d) V~,


226. The position x of a particle with respect to time t along X-
axis is given by x == 9t 2 _t 3 , where x is in metres and t in
seconds. What will be the position of this particle when it
v achieves maximum speed along +x direction?
[CBSE PMT2007[
(a) 54m (b) 81 m
(c) (d)
(c) 24m (d) 32m
227. Two balls are dropped to the ground from different heights.
One ball is dropped 2 sec after the other but they both strike
the ground at the same time. If the first ball takes 5 sec te
222. Two particles move in a unifonn gravitational field with an
reach the ground, then the difference in initial heights is:
acceleration g. At the initial moment the particles are
located at one point and start moving with velocities (Takeg = lOm/ s2) (Kerala PET 2007]
3 m ! sec and 4 mlsec horizontally in opposite directions. (a) 20m (b) 80m
The distance between them at the moment when their
(c) 170m (d) 40m
velocity vectors become mutually perpendicular will be :
(e) 160 m
228. Two electrons are moving in opposite directions with 233. Depict the shown v -xgrapb ina -x graph:' lIlT 2005J
speeds 0.8 c and 0.4 c, where c is the speed of light in
'0
vacuum. Then the relative speed is about: [VIT 2007)
(a) 0.4 e (b) 0.8 e
(e) 0.9 e (d) 1.2 e
229. Two trains are moving with equal speed in opposite
,
Fig. 6.95

aL=--'
directions along two parallel railway tracks. If the wind is

(a)~'
blowing with speed < u' along the track so that the relative
velocities of the trains with respect to the wind are in the
ratio 1 : 2, then the speed of each train must be: (b)
(Keraia PET 2007]
(a) 3u
(e) 5u
(e) u
(b) 2u
(d) 4u

230. A particle moving along X -axis has acceleration' F at time


t, given by
(e) ar-' (d) a~,
234. From the top of tower, a stone is thrown up. It reaches the
f=fo(I-~) ground in 1\ second. A second stone thrown down with the
same speed reaches the ground in 12 second. A third stone
where 10 and T are constants. The particle at t = Ohas zero released from rest reaches the ground in 13 second. Then:
velocity. In the time interval between t = Oand the instant (West Bengal JEE 20071
when f = 0, the particle's velocity (VX )is :
(/\ +/ 2 )
IUPSEE 2007; CBSE (PMT) 20071 (a) 13 =:< (b) 13 ""' M
2
I I 2 _ 2 2
(e) -= - - - (d) 13 - / 2 -11
') ,1
(d) for "
235. A ball is thrown from a point with a speed Vo at an angle of
231. A particle starts from rest. Its acceleration (a) versus time
e.
projection From the same point and at the same instant, a

(I) is as shown in the figure. The maximum speed of the person starts running with a constant speed ~ to catch the
2
particle will be: (lIT 2004]
2 ball. Will the person be able to catch the ball? If yes, what
a{m/s )
should be the angle of projection?
10 IUPSEE2007; JCECE 20081
(a) Yes, 60 0
(b) Yes, 300
(e) No (d) Yes, 45 0
236. Two stones are projected with the same speed but making
'----'--~ 1(,) different angles with the horizontal. Their horizontal ranges
11
Fig. 6.94 are equal. The angle of projection of one is ~ and the
3
(a) II0mis (b) 55 m1, maximum height reached by it is 102 m. Then the
(e) 550mls (d) 660 m1, maximum h~ight reached by the other in metre is :
232. An aeroplane is flying horizontally with a velocity of 600 IRPMTZ007; BCECE 20081
kmlh and at a height of 1960 m. When it is vertically above (a) 336 (b) 224
a point A on the ground a bomb is released from it. The (e) 56 (d) 34
bomb strikes the ground at point B. The distance AB is:
237. A point initially at rest moves along x-axis. Its acceleration
(West Bengal JEE 2007]
varies with time as a=:«6t+5)m/s2. If it starts from
(a) 1200 m (b) 0.331an IBCECE 2008J
origin, the distance covered in 2 sis:
(e) 333.3 Ian (d) 3.33 Ian
(a) 20m (b) 18m
(e) 16m (d) 25m
238. Three blocks of masses 2 kg, 3 kg and 5 kg are connected to (a) 22 dyne (b) 20 N
each other with light string and are then placed on a (e) 20 dyne (d) 100 N
frictionless surface as shown in the figure. The system is 245. A ball is dropped from a bridge al a heightof l76.4 m over a
pulled by a force F = ION, then tcnsion Tl is: river. After 2s a second ball is thrown straight downwards.
IBeECE 2008] What should be the initial velocity of the second ball so that

(a) IN
=
10 N 1":':1 T1 Cl T2

Fig. 6.96

(b) 5 N
r:-=l both hit the water simultaneously?
(a) 2.45 mi.
(e) 14.5 mi.
246. Which ofthe following are true?
(b) 49 mi.
(d) 24.5 mI,
rUPSEE 20091

[UPSEE 20091
(e) 8 N (d) 10 N (A) A body having constant speed can have varying velocity.
239. A ball is thrown up atan angle with the horizontal. Then the (B) Position time graphs for two objects with zero relative
total change of momentum by the instant it returns to velocity are parallel.
ground is: IBCECE 2008) (C) The numerical ratio of velocity, to speed of an object
(a) acceleration due to gravity x total time of flight can never be more than one.
(b) weight of the ball x half the time of flight (a) (A) (b) (B) and (C)
(e) weight of the ball x total time of flight (e) All (d) None of these
(d) weight of the ball x horizontal range 247. An artillary piece which consistently shoots fts shells with
240. A truck is moving on a frictionless surface with unifonn the same muzzle speed has a maximum range R. Tq hit a
velocity of 10 mls. A leak occurs in the water lank of the target which is R / 2from the gun and on the same level, the
truck at the rate of2 kg/so What is the speed of truck after 50 elevation angle of the gun should be: IUPSEE 20091
s if the mass of truck is 100 kg and mass of waler in the (a) J50 (b) 45'
truck initially was 100 kg? (UPSEE 2008) (e) 30' (d) 60'
(a) 20 mi. (b) 10 mi. 248. A constant power P is applied to a car starting from rest. Ifv
(e) 5 mI, (d) none of these is the velocity of the car at time t. then : IUPSEE 2009]
24 I. A body moving with a uniform acceleration crosses a I
(a) voct (b) voc -
distanceof65 m in the 5th second and 105 m in 9th second. I
How far will it go in 20s? (UPSEE 2008]
(e) voc.fi (d) voc ~
(a) 2040 m (b) 240 m
(e) 2400 m (d) 2004 m 249. R is the range on a horizontal plane for a shot with the same
242. A particle starting from the origin (O, 0) moves in a straight velocity at two different angles of projection. If hand h' be
line in the (x,y) plane. Its coordinates at a later time are the greatest heights attained corresponding to these angles
(.fj,3). The path of the particle makes with the x-axis an of projection, what is R 2 equal to? (UPSC 2009]
angle of: (JCECE 2008] (a) hh' (b) 9hh'
(a) 30' (b) 45' (e) 16hh' (d) 25hh'
(e) 60' (d) O' 250. Displacement {x)of a particle is related to time (r)as
x= at+bt 2 _et 3
243. A block B is pushed momentarily
along a horizontal surface with an S52C2! ~v where a, b and care constants of the motion. The velocity of
initial velocityv.lf~ is the coefficient Fig. 6.97 the particle when its acceleration is zero is given by:
of sliding friction between B and the .
IAMU2009)
surface, block B will come 10 rest after a time:
b' b'
IJCECE 2008] (a) a+- (b) a+-
v e 2c
(a) (b) g.
v b' b'
In' (e) a+- (d) a+ -
3c 4c
(e) ~ (d) v
v g 25 1. A bullet is fired with a velocity u making an angle of 60°
with the horizontal plane. The horizontal component oflhe
244. Diwali rocket is ejecting 50 g of gases/sec at a velocity of velocity oflhe bullet when it reaches the maximum height
400 mls. The accelerating force on the rocket will be : is: IWB (JEE) 2009]
IJCECE 20081
(a) • (b) 0 (a) R=16H (b) R=8H

(e) .J3. (d) . 1 2


(e) R =4H (d) R=2H
2 255. 2 stones arc thrown from top of tower, one vertically
252. The acceleration a (in ms - 2 ) ofa body. starting from rest upward and other downward with same speed. Ratio of
varies with time t (in s) foll owing the equation a ::: 31 + 4. velocity when they hit the ground is: (Orissa JEE 2009)
The velocity of the body at time t = 25 will be: (a) I : 2 (b) I : I
[WO (JEE) 2009[ (e) 2: I (d) I: 9
(a) 10 ms - 1 (b) 18.,,- 1 256. A particle has an initial velocity 31 + 4jand an acce leration
(e) 14111S- 1 (d) 26 ms- I of 0.41 + 03]. Its speed after lOs is: [AIEEE 2009[
253 . The acceleration aofa particle starting from rest varies with (a) 10 units (b) 7./2 units
time according to relation a "" at + ~. The velocity of the (e) 7units (d) 8.5 units
particle after a time t will be: [DCE2009[
257. A bullet is to be fired with a speed of 2000 ms- 1 to hit a
oJ ' oJ' larget 200 m away on a level ground. If g = 1Oms - 2, the
(a) - +P (b) - + pt
2 2 gun should be aimed: IKerala PET 2009)
(e) at 2 +..!.~t (d) (oJ' +P) (a) directly at the target (b) 5 em below the target
2 2 (e) 5 em above the target (d) 2 em above the target
254. For an object thrown at 45° to horizontal, the maximum (e) 2 em below the target
height (H) and horizontal range (R) are related as:
[DCE20091

ANSWERS

1. (b) 2. (b) 3. (d) 4. (d) 5. (e) 6. (a) 7. (a) 8. (d) 9. (e) 10. (b) II. (b) 12. (a)
13. (e) 14. (b) 15. (a) 16. (b) 17. (a) 18. (b) 19. (a) 20. (a) 21. (e) 22. (e) 23. (b) 24. (e)
25. (d) 26. (e) 27. (d) 28. (e) 29. (b) 30. (e) 31. (b) 3'. (b) 33. (a) 34. (b) 35. (d) 36. (a)
37. (e) 38. (e) 39. (e) 40. (e) 41. (e) 42. (b) 43. (a) 44. (b) 45. (e) 46. (e) A7. (e) 48. (b)
49. (e) 50. (e) 51. (e) 52. (d) 53. (d) 54. (a) 55. (e) 56. (b) 57. (d) 58. (d) 59. (d) 60. (e)
61. (a) 62. (b) 63. (d) 64. (b) 65. (b) 66. (e) 67. (e) 68. (a) 69. (e) 70. (b) 71. (e) 72. (e)
73. (e) 74. (b) 75. (b) 76. (a) 77. (e) 78. (a) 79. (e) 80. (e) 81. (e) 81. (e) 83. (e) 84. (b)
85. (d) 86. ee) 87. (e) 88. (e) 89. (e) 90. (b) 91. (b) 91. (e) 93. (e) 94. (e) 95. (b) 96. (b)
97. (b) 98. (d) 99. (d) 100. (e) 101. (a) 102. (d) 103. (d) 104. (d) 105. (e) 106. (d) 107. (b) 108. (d)
109. (d) 110 (a) Ill. (a) 112. (d) 113. (a) 114. (b) 115. (e) 116. (d) 117. (e) U8. (e) 119. (b) 120. (d)
121. (b) 122. (a) 123. (d) 124. (d) 125. (e) 116. (b) 127. (b) 128. (a) 129. (b) 130. (e) 131. (b) 132. (b)
133. (e) 134. (b) 135. (d) 136. (e) 137. (d) 138. (b) 139. (b) 140. (e) 141. (e) 142. (e) 143. (b) 144. (e)
145. (a) 146. (a) 147. (b) 148. (e) 149. (b) ISO. (b) lSI. (d) 152. (b) 153. (e) 154. (b) 155. (d) 156. (b)
157. (d) 158. (e) 159. (e) 160. (a) 161. (b) 161. (e) 163. (e) 164. (a) 165. ee) 166. (a) 167. (b) 168. (e)
169. (d) 170. (a) 171. (a) 172. (e) 173. (b) 174. Cd) 175. (b) 176. (a) 177. (d) 178. (e) 179. (b) 180. (b)
181. (d) 18'. (b) 183. (a) 184. (a) 185. ee) 186. (d) 187. (b) 188. (e) 189. (a) 190. (d) 191. (a) 191. (b)
193. (d) 194. (b) 195. (b) 196. (e) 197. (b) I .... (d) 199. (a) 200. (b) 201. (e) 201. (a) 203. (d) 204. (e)
205. (a) 206. (a) 207. (b) 208. (e) 209. (d) 110. (e) 211. (EI) 212. (e) 213. ea) 214. (e) 215. (a) 216. (b)
117. (b) 218. (e) 219. (a) 220. (b) 211. (a) 222. (e) 113. (b) 224. (b) 225. (a) 226. (a) 127. (b) 228. ee)
229. (a) 230. (e) 231. (b) 232. (d) 233. (a) 234. (b) 235. (a) 236. (d) 237. (b) 238. (e) 239. (e) 140. (d)
241. (e) 242. (e) 243. (a) 244. (b) 145. (d) 246. (e) 247. (a) 248. (e) 249. (e) 250. ee) 251. (d) 252. (e)
253. (b) 254. (e) 255. (b) 156. (b) 257. ~e~
(B) More than One Choice is Correct (a) HA >H B ' (b) HA <HB
(e) TA >TB (d) TA <TB
I. Which of the following statements ahout distance are lrUe?
10. Apparent weight of a body in an elevator is more than res'
(a) It cannot be negative weight if elevator is:
(b) It cannot be zero (a) Going up and slowing down
(e) It can never be lesser than magnitude of displacement
(b) Going up and speeding up
Cd) It can never decrease with time (c) Going down and slowing down
2. If a body after travelling some distance comes back to its (d) Going down and speeding up
starting point:
II . Weight of a body is zero if it is:
(a) Average velocity is zero
(a) At the centre of earth (b) Floating in. liquid
(b) Average speed is zero
(e) In a satellite (d) A projectile
(e) Distance travelled is zero
12. The velocity of a particle is at any time related to th,
Cd) Displacement is zero distance travelled by the particle by the relatiol
3. If the velocity ofa body is constant:
v (x) = ax + b. where a is positive and b is :<;; ~. Which ofth
(a) [Velocityl- speed 2
(b) IA verage velocity! =- speed following statements will be true for this motion?
(e) Velocity =- average velocity (given x = 0 when t = 0)
(d) Speed '" average speed (a) The displacement of the particle at time t i

4. If a particle travels a linear distance at speed


back along the same track at speed 1)2 :
v, and comes x=£.{e
a
Q
/ - I)

(b) The particle will experience a retardation if b < 0


(a) Its average speed is arithmetic mean (v I +v 2 )fl •
(c) The particle will be at rest at time t = 0
(b) Its average speed is geometric mean ~vlv2
(d) The motion of the particle is under constant acceleratiOl
(e) Its average speed is harmonic mean 2v IV2 , (vI +v2) 13. Two particles A and B are projected from the same poir
(d) Its velocity is zero with the same velocity of projection but at different angle
5. For a moving particle: a andp ofprojection, such that the maximum height of A i
two-third of the horizontal range of B. Then which of th
following relations are true?
~ ~ (a) 3(I-cos2a.)=8sin2j3
(c) vav =Obutvav;ol:O (d) vav ;l:Obutv av =0 (b) Range of A 0: maximum height of B
6. Which of the following statements are true? · 1 f"'
(e) M axunumvaueo pls -1sm
. - I -3
(a) A body can have constant speed but varying velocity 2 4
2
(b) A body can have constant velocity but varying speed
(d) Maximum horizontal range of A =!:... and this OCCUI
(e) A body can have acceleration without having velocity g
(d) A body can have velocity without having acceleration " = -sm
1. -I -3
whcnp
7. If a body is accelerating: 2 8
(a) It may speed up 14. Two particles are projected from the same point on lev·
ground simultaneously with the same velocity u but at tl
(b) Il may speed down
angles (0. +/3) and (0.-13) of projection (/3<45° ) re
(e) Il may move with same velocity
pectively. Which ofthe following statements will be true
(d) It may move with same speed
(a) They will have the same horizontal range if a. = 45°
~ ~
8. If two bodies are in motion with velocities VI and v 2 : (b) They reach their maximum heights at the time interv
of (2usin o.cos /3)1 g
(c) They will be separated by the distance
2
(c) vrcl = 0 Cd) vrel >c(speedoflight) 2u (sin 20. cos2p)when they reach the ground
9. In case of projectile motion if two projectiles A and Bare g
0
proj ected with same speed at angles 150 and 75 (d) They are never in the same horizontal level during thf
respectively to the horizontal then: fli ghts
B
15. Two projectiles following the s~me parabolic trajectory, are ,,
found to be in the same horizontal level, separated by a ,,
,,
distance of 1 m, 2 second after the second particle was
projected. Both the particles were projected with the same
, B:
500m

initial velocity from the same point on level ground. In


respect of their motion, which of the following statements A "
Fig. 6.98
will be true?
(a) To cross the river in minimum time, €I = 0"
(a) The horizontal range of the parabolic path is (l +4V),
where V is the (constant) horiwntal component of (b) To cross the river in minimum time, 9 = 30"
velocity in the path (c) For u =3 kmlhr and v =5 kmlhr, the time taken to cross

r
(b) The maximum height of the parabolic path the river in minimum time will be 6 min
(d) For u =3 kmIhr and v= 5 !cmlhr, the time taken to cross
=~g(2+ ~ the river in minimum time will be 3 min .
20. Two cities A and B are connected by a regular bus services
(e) The total time of flight in the parabolic path for each
with buses flying in either direction every T seconds. The
particle = 4 s
speed of each bus is unifonn and equal to Vb' A cyclist
(d) The particles will reach the ground in an interval of cycles from A to B with a unifonn speed ofVe (Vb >Vc } A
time=2s bus goes fast the cyclist in TJ second in the direction A to B
, 16. A train acceleraling unifonnly passes Ihree successive and every T2 second in the direction B to A. Then:
kilometre posts at times t = 0, t = 75 and t = 125 (all in
(b) T,.~
T
second). In respect of this motion, which of the following (a) T Vb
J
statements are true? Vb +Ve Vb -Ve
8 VbT Vb T
(a) The acceleration of the train is( 75)m/ s2 (c) TJ . -- (d) T, ' - - . -
Vb -Vc Vb +Ve
(b) The speed at the last of the three posts is 22.67 mls 21. A particle is projected from horizontal ground with speed
(c) The initial velocity of the train is 10 mls 5ms- J at 53" with horizontal. Find time after which
(d) The train will travel the next one kilometre in 15 velocity of particle will be 45° with horizontal.
second I 3
(a) ~ sec (b) ~ sec
17. Two particles A and B located at (0, 0) and (4, 4) 10 10
respectively start moving simultaneously with velocities 5 7
VA =-4 i andVs =-41 (c) ~ sec (d) ~ sec
10 10
(a) The shortest distance between them is 4.fi m 22. A particle projected from horizontal ground at angle '€I'
(b) The shortest distance between them first decreases and with speed 'u'. In same plane of motion a horizontal
then increases acceleration' a' exists so that projected particle returns back
(c) The distance between them increases from the to point of projection. Find time of flight.
beginning (a) 2usin 9 (b) 3usin €I
(d) The magnitude of relative velocity of A w .r.tB is 4 rrJs g g
18. Two stones are projected simultaneously with equal speeds (c) 02"u"co"s-,,8 (d) "3,:.:c",os,,,8
from a point on an inclined plane along the line of its a a
greatest slope upwards and downwards respectively. The 23. A particle is projected from horizontal XZ plane with
maximum distance between their points of striking the
velocity (uxl + u y1+ uzk)from origin (+ y-axis is upward).
plane is double that of when they are projected on a
horizontal ground with same speed. If one strikes the plane Find time when velocity of particle will be at 37" with
after two seconds of the other then: horizontal.
r;--o;-
(a) the angle of inclination of plane is 45" U _~u2 +u 2
y x , 3uy-4~u; +u;
(b) the speeds of their projection is 12.8 mls (a) (b)
g 3g
(c) the angle of inclination of plane is 60"
(d) the speeds of their projection is 128 mJs 3u y +4~u; +u; Uy+~u; +u;
(c) (d)
19. A swimmer wishes to cross a river 500 m width flowing ata 3g 3g
rate' u'. His speed w.r.t. to still water is v. For this, he makes
an angle €I with the vertical as shown. Then:
288 PHYSICS FOR COMPETITIONS - Vot. I

24. A carrom striker is given velocity on carraro based has right angle to the initial motion. Its velocity after 5s
always. Friction causes constant retardation. Striker hits has a magnitude nearly 14 mis.
boundry of carrom and comes to rest at point from where it ~ -+ ~ ~

started. Take initial velocity direction is positive, choose (R); The equation v = u + at can be applied to obtain v
~
correct graph descriting motion. (v-velocity, s-displacement, if a is constant.
'-time)
4. (A): A body is moving

(a)
'I~
" (b)
along a straight line
such that its velocity v
varies with time as
shown in fig. 6.99. 2 4 6 8 10 12 t(s)
, Magnitude of FIg. 6.99
displacement of the
=
body from I = 010 t 12s is the same as the distance
(c) travelled by it in Ihe given time duration.
(R) : For a unidirectional motion of a body,
Idisplacementl = distance
5. (A): A body is thrown with a velocity II inclined to the
ANSWERS
horizontal at an angle 9(9 > 0, < 90°} At the highest
• point. the angle ~etween instantaneous velocity and
I. (a), (c) and (d); 2. (a) and (d); 3. All; 4. (c) and (d); 5. (b)
~~~~OO~~~~OO~~&~OO~
acceleration is zero:-:'~~" ,"
(R); At the highest point velocity ofthe-ni:ojec'tile is zero.
(c); 9. (b) and (d); 10. (b) and (c); II.AII; 12. (a) and (b);
6. (A): A body X is dropped from the top ofa tower. At the
13. (a),(c) and (d); 14. All; 15. (a) and (b); 16. (a) and (d); same time, another body Y is thrown horizontally
17. (a) and (c); 18. (a) and (b); 19. (a) and (c); 20. (c) and from the same position with a velocity u. Both
(d), 21 , (a) and (d); 22. (a) and (c); 23. (b) and (c); 24. (b) bodies will reach the ground at the same time.
and ee) (R): Horizontal velocity has no effect on motion in the
vertical direction.
Ic] Assertion-Reason Type Questions 7. (A); A body is thrown with a velocity II inclined to the
horizontal at some angle. It moves along a parabolic
(a) If both A and R are true and R is the rorreet path and falls to the ground. Linear momentum of
explanation of A .
the body, during its motion, will remain conserved.
(b) If both A and R are true but R is not correct (R): Throughout the motion of the body, a constant force
explanation of A.
acts on it.
(e) If A is true but R is false.
8. (A): Two bodies of unequal masses In! and 1n2 are
(d) If A is false but R is true.
dropped from the same height. If the resistance
(e) If both A and R are false. offered by air to the motion of both bodies is the
1. (A): A body X is thrown vertically upwards with an same, the bodies will reach the earth at the same time.
initial speed 45 mls. Another body Y is also thrown (R) : For equal air resistance, accelerations of fall of
vertically upwards with an initial speed 27 mls. masses In! and m2 will be different.
During the last ~ sec of motion of each body, speed 9. (A): A body is released from a height. As it is falling
vertically downwards, at some position. it explodes
of each reduces by the same value.
into fragments under purely internal forces. Centre
(R) : Both bodies are moving with same acceleration. of mass of the system of fragments will keep
2. (A): A body is thrown vertically upwa rds with an initial moving along the original vertical line and also
speed 25 mls from a position I. It falls back to accelerate downwards with an acceleration g.
position I after some time. During this time (R): Whenever linear momentum of a system is
duration, total change of velocity of the body is zero. conserved. its centre of mass always remains at rest.
(R) : Average acceleration of the body during this time is 10. (A): In an elastic collision between two bodies, the
zero. relative speed ofthe bodies after collision is equal to
3. (A): An object moving with a velocity of magnitude 10 the relative speed before collision.
mls is subjected to a uniform acceleration 2 mIs 2 at (R): In an elastic collision, the linear momentum of the
system is conserved. tl1T20011
TRANSLATORY MOTION 289

11. (A); Two particles start from the rest simultaneously and 18. (A) : If two particles are neither approaching towards nor
proceed with the same acceleration in the same receding away from each other then their relative
direction. The relative velocity of these particles velocity is zero.
will be zero throughout motion . (R) : Relative velocity of I W.r.t. 2 is given by
At every moment the two particles will have the ~ ~ ~

same velocity. vI? :::v1-v2 •

12. (A): A river is flowing from east to west at a speed of 5


ANSWERS
mlmin. A man on south bank of river capable of
swimming 10 mfmin in still water, wants to swim I. (.) 1. (e) 3. (.) 4. (a) 5. (e) 6. (a) 7. (d)
across the river in shortest time. He should swim 8. (d) •• (0) 10. (b) II. (0) 12. (c) 13. (8) 14. (a)
due north throughout the motion.
IS. (.) 16. (b) 17. (d) 18. (d)
For the shortest time the man needs to swim
perpendicular to bank. I
[D) Integer Type Questions
13. (A): Rain is falling vertically downwards with velocity 6
kmIhr. A man walks with a velocity of 8 kmlhr. I. The displacement of a particle moving in straight line is
Relative velocity ofrain w.e.1. the man is 10 kmIhr.
Relative velocity is the ratio of two velocities.
.
gIVen as fu nctton
' 0 f hme
' 3t +
as s= 3 - 2 (t 3 2
2t}' ..m m
SIS

14. (A): Relative velocity of A w.r.t. B is greater than the and t is in sec. The particle comes to momentary rest n
velocity of either, when they 8rc moving in opposite times. Find the value of n.
directions. 2. A particle is moving up with balloon with constant
The relative velocity between any two bodies is acceleration (g/8) which starts from rest from ground and at
equal to the sum of the velocities of the two bodies. height H particle is dropped from balloon. After this event
time for which particle will be in air is ~ ~. Find the value
15. (A) : An object can possess acceleration even al a time
when it has unifonn speed
It is possible whe~ the direction of motion keeps ofk. -'f
changing. I ".,'

3. A particle has initial velocity (2i + 3j)ms- 1 when it was at


16. (A) : In a plane projectile motion, the angle between
instantaneous velocity vector and acceleration origin and has constant acceleration (3i + 2k) ms -2 . Find
vector can be anything between 0 and 1t (excluding angle made by displacement after 2 sec with Xl' plane
the limiting case)
In a plane t6 plane projectile motion, acceleration
{sin - I .J~} Find the valu~ of k.
vector is always pointing yertica~ downwards 4. Two particles were projected simultaneously in horizontal
(neglect air frict ion)
plane with same velocity u perpendicular to each other. The
17. Fig. 6.100 shows sequence of large number of time after which their velocities makes angle 600 with each
photograph of an object moving vertically under
gravity. A motion picture of this photograph is run other is k~. Find the value of k.
g
backward.
5. A particle is projected with velocity (61+5j+8k)ms- l.
§ from a vertical tower ofhight IO m. lfthe y·axis is vertical
up find time of flight (in sec).
o
o 6. Particle projected from tower of height IO m as shown in
Fig. 6. 101 . Find the time (in sec) after which particle will
o hit ground.
o
o
Fig. 6.100
45·

(A) : In time reversal sequence the gravitational 10m


acceleration will appear to be upward.
~ ~

(R): A time reversal operation changes every v to - v . Fig. 6.1 01


7. Adjacent graph is drawn for particle_along straight line Displacement of coin when it hits the ground is x metre (in
motion. Where a-acceleration &. v-velocity. The earth frame). Value ofx is (g = lOm/ s2)
displacement of particle from rest till it acquires velocity
.J2 ms - 1in s. Find the value of2s (in m).
1 I. 2 ,. 4 3. 2 4. I 5. 2 6. 2 7. 9
a (m-1s2)
5 8. S 9. I 10. 4 11. 12. 3
:1'
IE) ' Match the Column
3 I. A ball is projected from the ground with velocity v such that
its range is maximum
L _l:--,- : 2(m 2s-2)
4
Fig. 6.102 Column - I Column -II
8. A particle is thrown horizontally from the top of a tall tower (a) Velocity at half of the (P) -v
wi~ a speed of 10 mls. If radius of curvature of path maximum height in 2
followed is 4..r'2Jc m at t = 1sec, then find the value of k. vertical direction is ,
9. Five persons A,B,C,D & E are pulling a cart of mass 100 (b) Velocity al Ih. (q) v
kg on a smooth surface and cart is moving with acceleration maximum height J2
3 m / s2 in east direction. When person' A' stops pulling, it (e) Change in its velocity (,) vJ2
moves with acceleration I mt s 2 in the west direction. when it returns to the
When person ' 8' SlOPS pulling, it moves with acceleration ground
24 m/ s2 in the north direction. The magnitude of

¥~
(d) Average velocity when (s)
acceleration of the cart when only A & B pull the cart it reaches the maximum
keeping their directions same as the old directions, is (25/n)
height
m/ s2, value ofn is
2. Vx and V yare the horizontal and vertical compounds of
10. A ball is projected from the ground
velocity with x and y as the corresponding displacements
with speed 10 mls at an angle of 45 0 B
along horizontal and vertical at any time t in a projectile
with horizontal. It collides with a
motion in XY co-ordinate system, where g is the
walt at a distance 2 m from the point .,
acceleration due to gravity.
of projection and returns to its
original position. If the coefficient _--!-____--l Column - I Column - II
of restitution between the ball and 0 (a) (Vy - t) graph is a (P) Straight . line nOI
wall 'is 1, find x. , Fig, 6,103 straight line with passin!! throullh oriszin
x (b) (VX - t)graph is a (q) Straight line passing
through origin
11. In the arrangement shown in Fig. mA =lkgandm B =2kg,
while all the pulleys and strings are massless and (e) (x-t) graph is a (,) Straight line with
frictionless. At f '" 0, a force F '" I Ot starts acting over positive slope and
central pulley in vertically upward direction. If the velocity neeative intercept
of A is xx IOmls when B loses contact witJt floor, fmd x. (d) (y - t),nloh isa (s) None of these
F= lOt
3. For component of a veclor A =31+4j-5k, match the
fo llowin2 table :
Column -. Column - II
a l v-axis (0) 5 units
(b) A~on~ anA other vector (q) 4 units
Fig, 6.104 (21+ j + 2k)

12. A lift is moving up with a constant retardation of2 mt s 2. (e) Along (61 + S)-lOk ) (,) Zero
When its upward velocity is 5 mis, a boy in the lift tosses a (d) Along another vector (s) None
coin, imparting it an upward velocity of 3 mis, w.r.t. (-31-4)+sk)
himself. His fingers at the moment of toss are midway
between th~ floor and ceiling, whose totai height is 2m.
4. A ball is allowed free to fall from ,height ' H' which 6. Match Column-} with Column-II:
rebounds back to maximum height ' h' (H). Take upward as
the direction and initial position o n origin. . . Column -I Column -II
(a) A body is moving along (P) This will be a unifonn
Column -I Column - II
a straight line and linear motion
(a) Displacement accelerating uniformly
graph is
(b) A body is moving along (q) This will be a non-
a straight line. It covers uniform linear motion
a distance 72 m during
, the first six seconds of
(b) Distance from starting (q) "
its m'otion' an'd another
point versus time g}aph ,
72 m during the next six
is
" seconds
- -
(0) A b,ody is thrown (c) During the motion.
(c) Distance time g<aph is (c) ~ vertically upward: It linear momentum is not
rises to some height and conserved
then falls down along
the same line ,

(d) Velocity
displacement
versus(s) ~ (d) A bullet is 'tired into air (s) Position-time graph of
from a gun the motion will be a
straight line that is
pa~lIel neither ,.
x -axis nor the .v-axis.
(') Ii\ ' 7. Assuming one dimensional motion (along x -direction) in
,' all the cases , match Columns-I and II ,

IV' Column - I
,
Column -II

(.) uniform linear

ftL
5. Trajectory of particle in projectile motion is (P) is
y =(x_x 2 180), x and yare in metre. motion ':·
"
Projectile range is on horizontal plane
_ ,TIme ·
Column -] ! Column - II The motion depicted' in " ., ,r,' "
,,;-. ,

(a) Angle of projection S (P) 0'


this graph , ,
with horizontal is
(b)' Angle of velocity with (q)
horizontal at time I sec t)
'an -3
(b)
"E L
.,
~i I
"-~
_ TIme
(q) is non-uniform linear
motion

(c) Angle of velocity with (c) 45'


horizontal height The motion depicted in
"
20m from ground is this graph

IlL
positive
(d) Angle ,o f displacement (s)
with horizontal when
tan-'G) (c) (c) involves
aecel,eration
velocity is 0:
,
perpendicular
displacement
'0 ..... x (Distance)
"
The motion depictt:d in
, •
(') Not possible this graph
nt~
(d) (s) involves negative (0) Each ball will finally hit (r) Ball 3
acceleration the ground. Ball 3 hits
,_c
". the ground travelling at

- Unear
momentum
30 mls. Which of the
other balls will hit the
ground moving at the
The motion depicted in
this graph same speed (30 mls)?
(d) Which ball, as each ball (s) Ball 4
8. From some height above the ground at t = 0, a ball A of
finaUy hits the ground,
mass m is thrown horizontally with an initial speed 10 mls will have maximum
and another ball B of mass 2m is released. Both balls move vertical velocity ?
under gravity. Neglect the air resistance and match
Columns-I and II : 10. An object' X' ofmass '2Jn'is thrown with an initial speed 50

mls at an angle 60° with the horizontal from a spot on
Column -] Column - II ground. Another object' Y' of mass 'm' is also t~own from
the same spot, at the same lime, and with the same initial
(.) For the motion orball A (P) Same
speed, but at a different inclination with the horizontal such
(b) For the motion of bat! B (q) Different that the highest points in the trajectories of the two objects
are at the same horizontal displacement from the initial
(0) Time ~ken by ball A to (r) Mechanical energy ;s spot. Now suppose that the object X explodes at the highest
move from the given conserved point in its trajectory into two fragments of equal mass.
height to the ground is Neglect air resistance and match Colutruls-J and II :
'I'. Time taken by hall B
for its motion from the (Takeg = IOm / s2)
.
given height to the
ground will be Column - I Column - II

(d) Vertical speed of ball A (s) Angular momentum (a) 216.5 m (P) Distance of the centre
when ;t strikes the about the centre of of mass of the
ground ;s 20 mls. earth is conserved fragments of object X
Vertical speed of ball B from the initial spot
on striking the ground when the fragments
will be strike the ground

9. From some height above the ground, at t = 0, identical balls (b) 120.5 m (q) Time taken by the
1,2,3 and 4 are thrown with the same initial speed 10 mis, ""d
initially faster moving
as shown in the Fig. 6.105. Neglect air resistance and match fragment to strike the
Column-} with Column-II. ground from the instant
1 of its formation

T~3
(0) 4.3 s (r) Time taken , by the
initially slower moving
'h 4 fragment to strike the
ground from the instant
! of its formation
Fig. 6.105
(d) 2.7 s (s) Horizontal range of
object Y
Column - I Column· II
11. A person of mass 65 kg gets into an elevator at the 30th
(a) Linear momentum ;s (P) Balli floor of a building. The elevator begins to move at t = (1
not conserved for " Apparent weight of the person as a function of time is
(b) Torque due to (q) Bal12 shown in Fig. 6.106.. Take g = IOm /s 2. and match
gravitational fo= ;s Columns-l and II :
zero
App. wt IN)
Column - I Column - II
800
(a) Magnitude of torque (P) Variable
650 throughout the motion is
SOOI+-- (b) Parabola (q) Graph of angular
displacem=::nt as a
function of time for
o 3 5 7 9 1 1 1 3 t (sec.) 0!>:t!>:2 s
Fig. 6.106
(e) Radial force is (r) A non-zero constant
value
Column I R Column -II
(d) Straight line (s) Graph of linear
(a) The elevator is moving (P) t = Is distance travelled as a
down at function of time for
2st!>:4s
(b) Magnitude o (q) t = lOs
acceleration of the
ANSWERS
object is 2.3 mt s 2 at
1. a -+ p; b -+ q; c-+ fj d-+ s
(e) Speed of the object is (r) t = 125
2. a -+ s; b-+ s; c-+ q; d -+ s
2.3 mls at
3. a-+ qj b -+ r; c -+ 5; d-+ s
(d) Magnitude of (s) t = li s 4. a -+ q; b -+ S; c -+ r; d-+ t
acceleration of the 5. a-+ r; b-+ S; c -+ p; d -+ t
object is 1.1 S mt s 2 at 6. a-+ q,r; b-+ p,s; c -+ q,r; d -+ q,r
12. A particle is moving along a circle of a fixed radius and 7. a-+ q,r; b-+ q,r; c -+ q,r; d -+ q,r
gaining speed in a uniform manner, Match Columns-I and II. 8. a -+ f,S; b-+ ' ,5; c -+ p; d -+ p
9. a-+ p,q, r,S; b-+ p,q,r,S; c~ p,q,s; d-+ p
Column - I Column -II 10. a -+ P,$; b-+ does not match; c -+ q,r; d -+ does not match
(a) Tangential acceleration (P) 11. a-+ p,q,r,S; b-+ p,q, S; c -+ P,$; d-+ r
is """ Il. a-+ q; b -+ r; c-+ q; d-+ r
13. a -+ r; b -+ q,s; c-+ p; d -+ does not match
(h) Radial acceleration is (q)
• nOD-zero
value
constant
IF] Linked Compreh~n sio n Type -
(e) Angular acceleration is (r) variable Paragnph - 1
(d) Angular momentum is (s) g ( IOm / s2 ) A particle is moving along X -axis under a force such that
its position-time graph is as shown in Fig. 6. 108.
13. Consider the motion of a particle along a circle of a fixed
radius 10 em. Let us describe the clockwise motion as
positive and vice--versa. Variation of angular velocity with .
time is ~ shown in the Fig. 6.107. Match Columns-I and 11.
OJ (radls)

" r-=-~-:-7:-
o I (s)
8
-4.
Fig. 6.108
Fig. 6.107
294

Answer the following questions. (b) it is moving along positive X -direction with a spee<'
l. As the particle passes position (I) that is decreasing with time
(a) it is moving along negative X-direction with a speed (c) it is moving along negative X -direction with a spee(
that is increasing with time that is increasing with time
(b) it is moving along positive X-direction with a speed (d) it is moving along negative X-direction with a spee(
that is decreasing with time that is decreasing with time
(e) it is moving along negative X-direction with a speed Paragraph - 2
that is decreasing with time
A person standing on the roof of a building throws a bal
(d) it is moving along positive X~direction 'with a speed vertically upward at an instant t = 0. The ball leaves hil
that is incre¥ing with time hand with an upward speed 20 mls and it is then in free faU
2. , As the particle , position (2)
, passes The ball nses to ~ certain height and then moves down. Of
(a) it is movirig along positive X-direction with a its way down, tllc baU dust misses to hit the roof of the
maximum speed building and keeps falling towards the earth. ,The ball hit:
earth at t = 5 sec. Considering that (i) the vertically upwan
(b) it is moving along posiljv~ X -direction with a direction is the positive Y-direction (ii) the position ofbal
minimum speed at t ",,0 is the origin '(iii) the ball does not rebound anI
(e) it is moving along negative X -direction and its speed is comes to rest at the same place where it hits earth and (iv
zero here --- air resistance is negligible. (Take g = 10m/s2)
(d) it is moving along negative X-direction with a Answer the following questions.
minimum speed
7. Maximum displacement of the ball from the initial positiol
3. As the particle passes position (3) ;s
(a) it is moving along positive X-.direction with a (b) -45jm
(a) 45 jm
maximum speed
(b) it is moving along negative X-directi~n with a speed (e) 25 jm (d) -25 jm
that is increasing with time 8. Average velocity of the ball from t = Oto t = 5 sec is
(c) it is moving along positive X-direction with a speed (s) IOjrnls (b) -5 jrnls
that is decreasing with time
. (e) ~8 jrnls (d) -9jrnls
(d) it is moving along neg~tive X-direction with a speed
that is decreasing with time ' 9. Position-time graph for the given motion of the ball is
4. As the particle passes position (4) Ylm) Ylml~
(a) it is instantaneously at rest and will now move along (a) tis) (b) O . 5 tis)
1235 ' 1234
negative X-direction . _ ",,, .. -I,
,
,

(b) it is instanta~eously in~st and'will now move along


positive X -direction
Ylml~ ' Ylml~ ·
(c) it is moving along · positive X-direction with a (e) ' O ' 5 (s) t (d) 0 3 45 tis)
maximum speed . 1 234 I2
(d) it is moving along negative X-direction with a
maximum speed 10. ' Velocity of the ball will vary with time as
5. As the particle passes position (5) v(mls) v(mJs)

O~IIS)
(a) it is instantaneously at rest and will now move along
positive X -direction
(b) it is moving along positive X-direction with
that is decreasing with time
(c) it is moving along negative X-direction
maximum speed
a speed

with a
(a)

v(mJs)
(b)

°h
v(rn/s)
'IS
)

O~'IS)
(d) it is moving along negative X-direction with a
minimum speed
6. As the particle passes position (6) (e) (d) O r lis)
(8) it is moving along positive X-direction with a speed
that is increasing with time
II . Acceleration of the ball will vary with time as I S. The velocity with which particle strikes the planeOB :
a(mls2) ' a(m/s2) (.) 15 mls (h) 30 mls

O~I(S)
(e) 20 mls (d) 10 mls

(0) (h) 0u=t-'(S) 16. Time of flight ofthe particle:


(0) 8, (h) 6, (e) 4 s (d) 2 s
17. The vertical height' h' of P from 0 :
(.) 10m (h) 5m
(e) 15 m (d) 20 m
Paragraph 5 M

A particle A is projected with an initial velocity of60 mls at


angle 300 to the horizontal. At the same time a second
particle B is projected in opposite directions with initial
Paragraph - 3 speed of SO mts from a point at a distance of 100 m from A.
A particle is projected horizontally with a speed V = 5 mls If the particle collides in air,
from the top of a plane inclined at an angle 9 = 3T to the 50 m{s
horizontal. (g = 10m/5 2 ) 60 mls

'oA"'=6= - V & Fig. 6.111


:A
Answer the following questions.
_ _ _ _--'6,c:,. 8
18. The angle of projection of , a' of B is :
fig . •.109
Answer the following questions.
(0) m
Sin - 1 (h) gn-I m
12. How far from the point of projection will the particle strike
the plane?
(b) 65 m
(e) sin -I(~) (d) m
Sin- 1
(a) 75 m 19. Time of collision will be :
16
(a) \.09 sec (b) 9.01 sec
75 (d) 85 m
(e) -m (c) 2.09 sec Cd) 6 sec
16 9
13. Find the time taken by the particle to hit the plane. 20. The distance of point P from A where collision takes place
3 4
will be approximately:
(0) - s . (b) 3, (e) 4 s.. (d) - , (.) 100 m (b) 50 m
4 3
(e) 30 m (d) 63m
14. What is the ';elocity of the particle just before it hits the
plane? Paragraph. 6
(0) s,fj mI, (b) ~Ji3m1s Two enemy guns are placed at A and B at 10.J3 Ion apart
2 horizontally. A shell is ftred from A horizontally with
(e) IOJiJ mls (d) sJ'i6 mI, velocity 10 mls. At the same time a shell of double the mass
of shell at A is ftred from B at an angle 600 with horizontal
Paragrapb - 4 towards A with the same magnitu4e .of initial velocity as
Two inclmed planes OA and OB having inclination (with that of A. Moving in the same vertical plane, two shells
horizontal) 300: and 600 respectively. intersect each other at collide in air while sticking to each other and falling at the
O. A particle is projected from point • P' with velocity deepest point of the valley C (neglect air resistance).
u =lo.ff mfs aiong a direction perpendicular to plane OA. B
If the particle strikes pianeOB perpendiculary atQ, Calculate
A
B
Q

h
, , C
Fig. 6.112
21. Calculate how much above is the position B than position A 26. The resistive force suffered by a motor boat is proportional
(in km)? to V 2 , where V is instantaneous velocity. The engine was
(.) 5 (b) ;J3 shut down when the velocity of the boat was VO. Find the
average velocity at any time t.
(0) 10 (d) 1M
(a) Vo +V Wo
22. Find the time of collision of the two shells (in sec). (b)
2
(.) 200 (b) 20
J3 J3
3
(e) 2x 10 (d) 2xlO'
J3 J3
23. Find the magnitude of horizontal component of the
--+ ANSWERS
displacement vector (Be) (in kin):
Paragraph-I I. (d) 2, (a) 3. (0) 4. (8) 5. (0) 6. (d)
(.) ,J J.S5 (b) .5.7.7
Paragraph-2 7. (d) •• (b) 9, (a) 10. (a) 11. (c)
(0) 10.98 (d) 6.12
Paragrapb-3 12. (c) 13. (a) ... (b)
P.ragraph ~ 7 Paragraph-4 15. (d) 16. (d) 17. (b)
Consider a particle moving along x -axis. Its distance from
Paragraph-S 18. (b) 19. (a) ZOo (d)
origin 0 is described by the co-ordinate xwhich varies with
time. At a time t l' the particle is at point P, where its Paragraph-6 21. (c) 22. (c) 23. (b)
co-ordinate is xI and at lime '2' the particle is at point Q. Paragraph-?, 24. (8) 2S. (a) 26. (c)
where its co-ordinate is x2 . The displacement during the
time interval from II to 12 is'the vector from P to Q: the [GI Subjective Type Questions
x-component of this vector is (x 2 -xl) and all other
components are zero: 1. Can speed ever be negative? If so give an example and if
It is convenient to represent the quantity X2 - x I the change not explain why?
in x by means of 8 notation 6., thus Ax"=x1 -xI and [ADS. No; as speed" (distance/time) and both distance travelled
and time taken can never be negativeJ
111='2 -II
y 2. (a) Can a body have constant velocity but varying speed?
(b) Can a body have constant speed but varying velocity?
(Ans. (a) No, (b) Yes; See § 6.2 (C) 3J
3. (a) Is in case of un ifann motion, velocity always equal to
average velocity?
.x, (b) Is the motion always unifonn if velocity is ·equal to
Fig. 113
average velocity?
[ADS. (a> Yes, (b) No; See § 6.2 (C) 2]
The average velocity V- =---
x2 -xI __
.&x
4, Can average speed be lesser than magnitude of average
. t2 -t l l!t
velocity? If yes explain and ifnot explain why?
24. A particle moves halfthetimeofitsjoumeywith u. The rest ~

of the halftime it moves with two velocities VI and V2 such [Ans. No; v'v .. (distance/time) and " IV" (displacement/time).
"
that half the distance it covers with VI and the other half Now as distance ~ j<!isplacementl so v,v ~ I~ I V I]
with V2 , The net average velocitY is (Assume straight line ,
motion): . S. Can a body have zero velocity ·and still be accelerating? If
(a) u(VI +V2 )+2VIV2
yes give an example and if not explain why?
(b) 2u(V, +V,)
{Ads. Yes; if in case of moving body acceleration is opposite to
2(y, +V,) 2u +V, +V2 motion, the body will first stop and then will come back. At the
2V'Y2 turning point of motion velocity will be zero while acceleration
not, e.g~ if a body is projected vertically up, at highest pointv:::: 0
but a:::: g .. constant while in case of simple hannonic motion at
25. A particle moves according to the equation x =,2 + 3t + 4. extreme positions v:::: IJbut laccelerationl ~ (1)2A '" max.]
The average velocity in the first 5 s is : 6. Can velocity of an object reverse direction when its
(.) 8 mI, (b) 7.6 mI, acceleration is constant? If yes give an example and ifnot
(0) 6.4 mI, (d) 5.8 mI,
explain why?
[Aas. Yes; in case ofverticaJly upward monOD under gravity]
7. Can an object be increasing in speed as irs acceleration pulley remain balanced, that is, there is no tendency for the
decreases? lfso give an example and ifoot explain why? pulley to tum.
[Ans. Yes; in simple harmonic motion when body moves from [Ans. Masses are weightless, so lift is in free faU)
extreme position to equilibrium position 1J increases while 15. State whether the following statements are true or false
laccelerationl- 002 ydecreases] giving reason in brief:
8. Can there be motion in two dimensions with acceleration in (a) The speed acquired by a body when falling in vacuum
only one dimension? for a given time is dependent on the mass oftbe falling
[Ans. Yes; projectile motion is an example of 2-D motion with body. ' IRoorkee 1995)
acc. in l-D only] (Ans. False; See § 6.5, A, discussion]
9. If in case of a motion, displacement is directly proportional (b) A projectile fired from the ground foUc.ws a parabolic
to the square of time elapsed, what do you think about its (a) path. Tne speed of the projectile is zero at the'topmost
velocity and (b) acceleration? point.
[ADS. As s cc (2 ,i.e. ,s _ kt 2, v = (ds I Jt) - 2kt and acceleration ... [Ans. False; .speed is minimum " ucos9 but not zero]
(dv l dt) .. 2k. So (a) velocity varies linearly with time and (b) (c) For angles of projection which exceed or fall short of
acceleration is constant) 45 0 by equal amounts, the ranges are equal.
lO. Taking the rotation and revolution of the earth into account, [Ani. Trne; as for 91 '" (45°- a)
docs a tree move faster during day or during night?
RI_u 2 sin2(45°_a)lg,,",u 2 cos2alg

and for 92 .. (4So+a~

R2 - u2 sin 2(450 + a)1g "" ar COi2a I g '" Rtl


(d) A rocket moves forward by pushing the surrounding
Fig. 6.114 air backwards.
[Ans. As earth's rotation and revolution are in the same direction, (Ans: False; rocket moves by ejecting gases from inside it and not
relative to sun by pushing SUITOunding air. It can move even in vacuwn where
there is no surrounding air]
VD "'Vo-vs -Vo -Roo
(e) The mass of a moving body is greater than when it is at
and VN "'Vo +vs "'Vo+Roo
rest.
So VN > VD , i.~, objects on earth mcive faster iil nightthan in day.
However, V N - VD - 2Roo:::: 1kmls which is much lesser than Vo (Ans. TlUe; in motion a body has kinetic energy which is
.. 30 kmfs) equivalent to mass, i.e. ,m '" mo + (KE I c2).]
II. If a ball A is dropped while B is projected vertically down, (0 The weight ofa body in projectile motion is zero.
which ball will reach the ground (a) first (b) with greater (Ans. TlUe; projectile is a freely falling body and weight of a
velocity? freely
,.falling
.
body is zero]
[Ans. (a)B,(b)B ] 16. Fill in the blanks:
(u2 + 2gh)1I2 _ u
~g
[HID1: (a) t A - - while t8 -
(a) A particle moves in a circle of radius R, In half the
g period of revolution its Idisplacementl is : ~ ., ..... and
distance covered is ....... ,.
(b) vA =.J2ih whHevB :~<u2+2gh)J
(b) If the velocity ofa particle is given byv = ~(180-1~)
12. A man can throw a stone R m away:
mis, its acceleration will be ...... .. .
(a) What is the maximum height to which the stone will
rise? [Hlnl: Ace. = dv ... dv = dx '" v dv arid from the given relation
it fix it dx
(b) How high can the person throw the stone?
lADS. <a) H .. Rf4, (b) H .. Rf2J
v 2 (180 - 16.%), i.e., 2V: = -16 so that a _ -8 m/5 2 ]
::t

[Hint, S" § 6.6 (0) (4) on' (S)] (e) A boat takes two hours to travel 8 km and back in a still
13. According to Newton's 3rd law every force is accompanied water lake. With water velocity 4 kmIhr the time taken
by an equal and opposite force. How can a movement ever for going up stream 8 km and coming back is ..... .
take placo? minutes. [EAMCET 1990)
[ADS. As action and reaction act on different bodies) IHlal: vb = (8+ 8) / 2- 8 kmIhr; now asvup:::' 8- 4 .. 4 kmlhrso
14. What conclusion will a physicist draw if, while standing in tup - 8 14 =2hrandv d = ~+ 4 - 12, so td ",8/12 os ']J3 hr; i.e. ,
an elevator, he observes that unequal masses hung over a t .. tup + td - 2 + (2/3) hr - 160 minute]
(d) The velocityofa particle at an instant I is 10 mls. After 18 . Prove that for a projecti le fired from level ground at an
5 s the velocity is 20 mls. The velocity at 3 s before was e
angle above the horizontal, the ratio of the maximum
IEAMCET 1992) height H to the range R is given by
[Hint: Here 10= u + ot;20 .. u+ 0(1+ S} and V= u + a(I- 3). So H 1
-=~tane
subtracting 2nd Eqn. from 1st, we get a - 2 m/ s::!:, Now R 4
substituting (1) and a- 2 in 3rd Eqn.• v .. 10- 3 x2 "" 4 mls] (Hint: Writing expressions for Hand R find their ratio]
(e) A food packet is released from a helicopter which is 19. A projectile is fired from level ground at an angle e above
rising steadily at 2 mls. After 2 s the velocity of the the horizontal. Show that the elevation angle ~ of the
packet is ......... (g ,., 9.8m1s 2 ). highest point as seen from the launch point is related to e,
the elevation angle of projection, by
(f) Ifa splash is heard 4.28 s after a stone is dropped into a
well 78.4 m deep, then velocity of sound in air is ..... .
(g) A particle is projected with an initial velocity of 200
u
mls in a direction which makes an angle of 30° with the
vertical; the horizontal distance travelled by the H
particle in 3 s is ......... m. IEAMCET 1992}
[Hint: x = !looSe )( t] I" R
(h) A body of mass m thrown horizontally with velocity v Fig. 6.115
from the top of a tower of height h touches the level of 1
ground at a distance of 250 m from the foot of the tan,= - tane
2
tower. A body of mass 2m, thrown horizontally with
[Hlnl: From Fig. 6.115, tan+=H I (R I 2); but in case of
velocity (v 12) from the top of a tower of height 4h will
touch the level ground a distance ......... m from the projectile motion H = (u 2sin l O/ 2g) and R c (u 2 sio 20 1g); so
foot of the tower. (EAMCET 1991) substituting these values of H and R in the above we get the
desired result]
{Hint: For the fitst body as t .. ...f(2iiIij so 250 .. vx J2h l g .
20. When a particle is projected at an angle to the horizontal, it
Now for the second body as 10:: J(i )(4h l g) and v 0:: u / 2, so
has range R and time of flight t,. ·lfthe same projectile is
x = (v I 2) )( 2.j2h l g .. vJ2h l g " 250 m] projected with same speed at another angle to have the same
(i) A horizontal stream of water leaves an opening in the range, time of flight is t 2 . Show that
side of a tank. If the opening is h m above the ground 'I'> =(2Rlg)
and the stream hits the ground D m away and the 2 1. A car covers the first ,half of the distance between two
acceleration due to gravity is' g', the speed of water as places at a speed of 40 kmIhr and the second half at 60
it leaves the tank in tenns of g ,h and D is .... . kmlhr. What is the average speed of the car?
IEAMCET 1992) [Ans. 48 kmlhr]
[Hint: D .. ut, i.e., v = D 1 I with I" J2hl g] [Hint: See note in § 6.2 (B) (8)]
IAn •• (a) 2R and nR; (b) - 8 mis' (c) I~ (d) 4 mI, (0) 22. A particle moves along x-axis in ' such a way that its
- 17.6 mI, (f) 280 mI, (g) 300 (h) 250 (i) D g 12h 1 co-ordinatexvaries with time t according to the expression:
x=(2-5t+6t 2 )m
17. Prove that:
(a) The trajectory ofa projectile is a parabola. What is the initial velocity of the particle?
(Hial: See § 6.6 (A») (Ans. -5 mls)
(b) The path of a projecti le as seen from another projectile 23. Find the speed of two objects if, when they move uniforml}
is a straight line. towards each other, they get 4.0 metre closer each sec anc
when they move unifonnly in the same direction with the
[Ans. The co-ordinates of a point on one trajectory relative to
other will be original speeds, tbey get 4.0 m closer each lO s.
x =x2 - XI '= ("lcoss.: -lit COSOI)1 (Ani. 2.2 mls and 1.8 mls)
24. Snow is falling vertically at a constant speed of8 mls. (a) A
",d . 92/-'2gt
y= ( "lSID 1 ') - ( lit SIn'1
. A ·t -'2gt
1 ') what angle from the vertical and (b) with what speed do th(
snow flakes appear to be falling as viewed by the driver in!
'" ("I; sin 8l - Ii sin 01)t
car travelling on a straight road with a speed of21.6 km/hr?
So y ( ,; sinOl - UI Sino!) =constt.'"'m
-.. IAns. (a) tan -I (3/4)= 37° (b) 10 mls}
x U:2 cos 0l - Ii cos 01

0' =
Y mx whicb is a straight line]
25. Wind is blowinS west to east along two parallel tracks at a (Hlnl: Taking downward direction +ve, u =-9.8 mis, $ = 39.2 m
speed v. Two trains moving with the same speed in opposite and a =g =+9.8 m/ s2. Now apply II and III equations of motion
directions have the steam track of one double that of the 10 calculate lime and speed respectively)
olher. What is the speed of each train? 3\. A hot air balloon of mass M is descending vertically with
[Ans, 31)) downward acceleration Q. How much mass must be thrown
[Hint: If V is the velocity of each train, then for the train moving out to give the balloon an upward acceleration Q.
opposite to the wind steam track will be given by d1 '" (V -V)I (Ani. 2Mal (g + a)}
while for the other d2 '" (V + v)t.
(Hint: Initially Mg-Th = ma and when mass m is removed
=
Now as dz 2dJ (given), (V + v)t "" 2(v-v)t,
Th - (M - m)g '" (M -m)a. Solving these equations for m we gel
i.e., Y=3v] the desired resull.]
26. Three rats A,B and C 8re sitting at the vertices of an 32. In an eltperimcnt for measuring 'g', a body is thrown
equilateral triangle ABC of side d. If each of them starts vertically up in an evacuated tube and allowed to come back.
moving with speed v such that A always moves towards B, If c.TL is the time interval between the two passages of the
B towards C and C towards A, where and at what time will object across a lower level and c.TH the time interval between
they meet? two passages across an upper level and H the distance
[Ans. At the centroid of the triangle at time 2d 13v] between two levels as shown in Fig. 6.117, show that:
[Hint: See solved 'Question VII] 8H
g 0 -;:=:c.----c
27. A ship of mass 3x 101 kg initially at rest is pulled by a force c.Tl-!>.T~
of5>< 104 N Ihrough a distance 3 m. Calculale the speed of
the ship neglecting the resistance of water.
y
{Ans.O.J mls]
28. Two inclined planes are H
placed as shown in Fig.
6.116. A particle is projected
from the foot of the plane of
angle (l along its line with a 11m.
velocity just sufficient to Fig. 6.116
Fig. 6.117
carry it to the top after [Hint: Time taken b a body to go up and come down a heighth is
which the particle slides down the other inclined plane. giving by t =(2h 1g) = ~ (8h I g). So, if y is the height of highest
Find the total time it will take to reach the point C.
point above higher level, time taken to go up and come down
[A",.t .
g)
1 +-' 1
vlF2h)[_sma smj3
6TIf .. ~(8ylg) while fot lower level 6TL =~[8(y+h)lg).
Solving Ihese for g, we get the desired resull.]
29. A lawyer seeks your advice in one of his cases. The 33. A v.:ooden block of mass 109 is dropped from the top of a
question is whether a driver was exceeding a 45 kmIhr cliff 100 m high. Simultaneously, a bullet of mass 10 9 is
speed limit before he made an emergency stop, brakes fired from the foot of the cliff vertically upwards with a
locked and wheels sliding. The length of skid marks on the velocity of lOOmis. (a) Where and after what time will they
road was 5 m. The policeman, assuming that maximum meet? (b) If the bullet after striking the block gets
deceleration oftbe car would not exceed the acceleration of embedded in it how high will it rise above the cliff before it
a freely falling body (10 m / s2), arrested the driver for starts falling?
speeding. What is your opinion? (Ans. (a) 4.9 m below the top and I s after dropping,
[Hint: As stopping distance s '" (,J I ?a). (b) 82.4 -4.9 ""17.5 m)
u=..fiQS=·./2x IO x5 1Om/5) [Hlnc: See solved Problem 27J
[Ans. The policeman was wrong in arresting the driver as the 34. A bomb is dropped on an enemy post by an aeroplane flying
speed of car was - 10 mls - 36 kmIhr « 45 kmIhr) when brakes with a horizontal velocity of60 kmIhr and at a height of 490
were applied] m. How far the aeroplane must be from the enemy post at
30. A balloon is ascending at the rate of9.8 mls and is 39.2 m the time of dropping the bomb, so that it may directly hit the
above the ground when a package is dropped. (a) How long target? (g = 9.8ml s 2 )
does the package take to reach the ground? (b) With what [Ans. 500/3 m]
speed does it hit the ground? (g = 9.8 ml s 2 ) 35. In a detective story a body is found 5 m from the base of a
[Ans. (a) 4 s (b) 29.4 m/s] building and beneath an open window 25 m above. Would
you guess the death to be accidental or not? Explain your 44. At a picnic spot on a hill, a boy falls over the cliff. Suddenly
answer. (g = 10m! s2) Shaktiman arrives and dives off the edge 2.0s after the start
of the boy's fall. If the cliff is 100 m high, what must
[ADS. As t = ~ (2h / g) = (.J5) s, the horizontal velocity of body
Shaktiman's initial velocity be ifhe is to catch the boy just
would have been u = 5/..[5 =.J5 mls=2.23 mis (about 20% of before he reaches the ground?
world class sprint speed of 10 mls), As this horizontal la\lllch {Ans. 27.3 mls]
speed cannot be achieved by a standing person himself without a
45. A body is released from rest at to =O,x o =0 and moves
push from behind or by throwing him, so the case is Dot
accidental.] with constant acceleration. Its position is measured at the
equally spaced times t, =At,/2 =21./,13 =3M and so
36. A gun is pointed horizontally on a vertical wall a distance x forth. Prove that displacements lu j =x j - x '_I which
m away from the gun. If the bullet hit the target y m below occur in the successive time intervals i':J.t are in the ratio
the gun, show that the trajectory of the shot is y = Ax2 .
Ax, : lu2 : Ax 3 '" ""1:3:5· ..
37. A fireman 50 m away from a burning building directs a stream 46. On a cricket field, the batsman is at the origin of
of water from a firehouse at an angle of 30" above the ~ ~

horizontal. If the velocity of the stream is 40 mis, at what height co-ordinates and a fielder stands in position (46 i + 28 j )
will the stream of water strike the building? (g = 9.8m/s 2 ) m. The batsman hits the ball so that it rolls along the ground
~ ~
[Ans. 18.65 m] with constant velocity (7.5 i + 10 j) mls. The fielder can
38. An astronaut on a strange planet finds that he can jump a run with a speed of5 mls. Ifhestarts to run immediately the
maximum horizontal distance of 30 m ifhis initial speed is ball is hit, what is the shortest time in which he could
9 mls. (a) What is the acceleration of gravity on the planet? intercept the ball?
(b) What is the maximum height to which he can jump ifhe [Ans. 4 second]
starts with the same initial speed?
47. A particle of mass 3 kg takes 2 s to move from A toB under the
[Ans. (a) 2.7 ml 52, (b) 15 m] ~ ~ ~ ~

39. If the mass ofa body is mkg, what is (a) its mass in slug, (b) action of gravity and a constant force F == 12 i - 3 j + 21 k.
=
its weight in newton if g 9.8 m/ s 2 ?
~

where the unit vector k is in the direction of the upward


~ ~ ~
[Ans. (a) (ml14.6), (b) (9.8 m) N]
verticaL The position vector of B is 15 i + 7 j -6 k and the
[Hint: See solved Problem 3 in Chapter 1] ~ ~ ~

40. If the diameter of the earth becomes two times its present particle arrives at B with a velocity of 12 i + j - 4 k. Find
value and its mass remains unchanged, then how would the the position vector of A and the velocity with which the
weight ofan object on the surface of the earth be affected? particle leaves A.
[Ans. Weight will become one-fourth] -+ -+ -+ -+
[Ans,rl ",-1+3j-4k]
,41. Detennine the speed with which the earth would have to
rotate on its axis so that a person on the equator weighs 48. An aeroplane is observed by two persons travelling at 60
(3/5)th of his rest weight. Take the equatorial radius as km/hour in two vehicles moving in opposite directions on a
6400 Ian. (g 0 9.8ml ,2) straight road. To an observer in one vehicle the plane
appears to cross the road track at right angles while to the
(Ans. ro .. 7.8)( 10-4 rad/s]
observer in the other vehicle the angle appears to be 45°. At
42. A lift of mass 400 kg is hung by a wire. Calculate the
what angle does the plane actually cross the road track and
tension in the wire when the lift is (a) at rest (b) moving
what is its speed relative to the ground?
upwards with a constant velocity of 1.0 mls (c) moving
[Ans. 9= tan-I 2; v = 134.l6kmlhour]
upwards with an acceleration of 2.0 m/s2 (d) moving
downward~ with an acceleration of2.0 mls 2. 49. A stone is projected from the top of a cliff h high with
[Ans. (a) 3920 N, (b) 3920 N, (c) 4720 N, (d) 3120 N] velocity u in the horizontal direction. At what angle can
43. A car waiting at a traffic light starts to move with a constant another stone be projected with same velocity u to hit the
same spot on the ground hit by the first stone?
acceleration of 2.0 ml s 2 when the light turns green. Two
fI
Vgh ]
second later, a truck moving in the same direction passes [Ans.a.=tan-IU
the light at a constant speed of 25 mls and soon passes the
car. Soon the car passes the truck:
50. An electric train of mass M kg moves from rest along a
(a) How long after the car starts do the two overtakings straight line track. The traction force of the motors initially
occur? Pnewtons decreases with time uniformly to R newtons over
(b) How far from the light does each take place? a period of T seconds and then remains constant at R
[Ans. (a) 2.19 s and 23 s; (b) 4.8 m and 520 m] newtons. The total resistance to motion is R newtons. Show
that the acceleration of the train' a' at time t seconds after it
starts to move (t :'>: 1) is given by Ma =-P + (R +P)t -R . interception and (c) the position vector of point of
T interception from the buoy if distances are measured in
Find the maximum speed achieved by train, the distance it kilometres and speeds in kilometre per hour.
travels before reaching this speed and the power developed --+ -+ 9 -+ -+
[Ans. (a) 7 i + 2 J; (b) 12,JOp.m. ; (c)- i + J]
by motor at time T / 2. 2
(P - RlT 56. A particle moves from rest in a straight line with alternate
[Ans. u max ::= mlsec ;
2M acceleration and retardation of magnitudes f and .f during
(P_R)T 2 equal intervals of lime t. At the end of 2n such intervals
Distance travelled., metre
3M prove that the space it has described is
3(p2 _R2 )T nt 2
Power - watt.] -[(2n + III -(2n- ll/'1
16M 2
51. A submarine of mass m when moving in a horizontalline 57. A helicopter takes-off along the vertical with an
OX with speed v experiences a resistance Jro2 where k is a acceleration 3 mlsec 2 and zero initial velocity. In a certain
constant. The power P of the engine is constant and is just time t I the pilot switches off the engine. At the point of
sufficient to maintain a speed v. (a) Calculate the distance take-off the sound dies away in a time 30 second. If the
moved while the submarine accelerates from a speed vl4 to velocin' of sound in air is 320 mlsec, find the velocity of the
3v14. (b) Calculate the time for the submarine to reduce its helicopter at the moment when the engine is switched off.
speed from 3v14 to vl4 when moving against the resistance
with no power supplied by the engine.
(Ans. v = 80 mlsec]
mv3 63 8mv 2
58. A particle having a velocity v ::I. Vo at t = 0 is accelerated at
[Ans. (a) 3), 10837; (b) t::= J'j) J a rate lal= a../V, where a is a positive constant. Calculate.
52. A shell flying with velocity v = 500 mlsec bursts into three the distance travelled before the particle is brought to rest.
identical fragments so that the kinetic energy of the system 2
(Ans.S " --'-I
v" 2
increases 11 = 1.5 times. What maximum velocity does one 3 a
of the fragments obtain? 59. A small stone of mass mis thrown vertically upwards with
[Ans. 1000 mlsec) an initial speed v. If the air resistance at speed v is mkv 2 ,
53. A particle is projected from a point on the level ground and where k is a constant, show that the stone returns to the

V( ~
its height is h, when it is at horizontal distances a and 2a 2
from the point of projection, find the velocity of projection s1arting point with a spoed I+ ) -"2.
and the angle of projection.

[Ans.u='2 I (4.;;-2.... 9h1g;ct=tan _,(3h)


2a 1
60. A body is thrown from the surface of the earth at an angle, a
to the horizontal, with initial velocity vo ' There is no air
drag. Find (a) the time of motion, (b) the maximum height
54. Two particles are simultaneously projected in the same of ascent and horizontal range. At what value of ex, they are
vertical plane from the same point with velocities u and v at equal to each other, i. e., hmax =R? (c) the equation of the
angles a and j3 with horizontal. Show that: trajectory y(x) where yand xare the vertical and horizontal
(a) the line joining them moves parallel to itself, displacements. (d) the radii of curvarure of the trajectory at
(b) Ihe time that elapses when their velocities are its initial point and at maximum height (hfl1o'U }
parallel is uvsin (a-j3) [Ans. (a) 2vosinct/g; (b) hma:>l .. v~sin2ct / 2g;
g(vcosJ3-ucosa)
R""v~ sin(2a) l g;
(c) the time that elapses between their transits through the
ct=tan- 1 4; (c) y(x)=x(tanct) _ x2(g / 2v~C()s2a~ (d) at the
other common pomt . _.:'2uv==sin:-(,-,a:..-.!:~"-l...,
. IS
g (vcosJ3+ ucos a) highest point; R =(-v~ cos 2ct I g). At the initial point,
~ ~ R=(-v~ / g cosa) }
55. A motor boat set out at II a.m. from a position - 6 i - 2 j
61. A heavy particle projected with velocity u strikes at an
relative to a marker buoy and travels at a steady speed of
angle of 45° an inclined plane of angle p which passes
magnitude ..[53 on a direct course to intercept a ship. The
~ ~
through the point of projection. Calculate the vertical
ship maintains a steady velocity vector 3 i + 4 j and at 12 height of the point struck above the point of projection.
~ ~

noon is at a position 3 I - j from the buoy. Find (a) the [Ans. h = ,; 1+ cotJ3 ]
2
g 2 + 2eoll}"" cot 1}
velocity vector of the motor boat, (b) the time of
62. A man can row a boat at 4.0 kmIhr in still water. A river
flows at 2.0 kmIhr. (a) If he is crossing the river, in what
[Ans.v"'J~ - PKgL ]
direction the boat should go to reach a point directly 67. A girl throws a ball at a vertical wall 4 m away. The ball is
opposite to his starting point? (b) If the river is4.0 km wide 2m above ground when it leaves the girl's hand with an
how long will it take him to cross the river? (e) How long ~ .,
initial velocity of Vo = (IOi + IOj) mls. When the ball hits
will it take him to row 2.0 km down the river and then back
to his starting point? (d) How long will it take him to row the wall, the horizontal component of its velocity is
2.0 km up the river and then back to his starting point? (e) In reversed; the vertical component remains unchanged.
what direction the boat should go, if he wants to cross the Where does the ball hit the ground?

m/m~'~--l
river in the smallest possible time?
[Ans. (a) 30" to vertical; (b) 2/.fj hr; (e) 4/3 hr; (d) 4/3 hr; (e)
10
perpendicular to shore]
a,
63. A pilot has to fly from A to due east, and then back to A.
1---4 m---l
due west. The velocity of the plane in air is v. The velocity
Fig. 6.121
of air with respect to ground is u. The distance between A
and B is I, v is constant. (a) If u = 0 (i. e., still air), show that [Ans. 18.2 m from wall],
the time for the round trip,A ~ B ~ A, I = 211 v. (b) If the 68. A car is travelling on a highway at 25 mls; a passenger
velocity of the air is due east (or west), show that the time throws a ball at 45° angle of elevation in a plane
for the round trip'" t E "" t 1(I-(u 1v)2 ]. (c) If the velocity perpendicular 10 the motion of the car. The initial speed
of the air is due north (or south), show that the time for the relative to car is 10 mls. The ball is released at a height of
round trip =I N = 1I[ L_(u l v)2]112. (d) In parts (b) and 1.2 m above the road. (a) Write the initial velocity of the
ball (relative to the road) in tenns of the unit vectors i,j and
(e), u <v, why?
It (b) Where does the ball land?
64. A projectile is launched with a velocity u at right angles to
the slope, which is inclined at an angle e with the [An!. (a) (2Si + 7.D7h (b) 39.9 i + Il.33j ]
horizontal. Derive an expression for the distance R to the 69. A car goes out of control and slides off a
point of impact. steep embankment of height h at e to the
horizontal. It lands in a ditch at a distance
R from the base. Find the initial velocity
of the car when it left the slope.
(Ans. tlo "'~ I g ]
Fig. 6.122
cos9V 2(h - Rtan9)
70. At a certain instant two cars are each 10 km from the
Fig. 6.118
intersection of roads that are perpendicular. Car A is
2,' tan osec 0]
[Ans. R .. -
moving east at 30 kmIhr while car B moves north at 50
g kmIhr both toward the intersection. (a) Find their closest
distance of approach. (b) Where are A and B when they are
65. A shell is fired vertically from a cannon closest?
which is travelling at constant speed u =
30 kmlhr. The projectile leaves the u [An •. (al 3.42 km, (bl A, 2.95 km w,n, 1.75 km N.]
... --- . ...
,,... "
cannon with a velocity v r "" 20 mls 7 1. A stone at the end of a string is
relative to the cannon. Show that the whirled in a vertical circle of
shell will land on the vehicle at the gun' FJg.6.1 19 radiusr=I.20mataconstantspeed B f .A
location and calculate the distance stravelled by the vehicle u = 1.50mls.Thecentreofthestring I 30" 30" J
during the flight of shell. is 1.50 mabove the ground. What is \ "
[Ans . .s = 34m] the range of the stone ifit is released " ...... ___ ...... /
66. A heavy chain with a mass per unit when the string is inclined at 30" Fig. 6.123
length m is pulled by a constant force. with the horizontal: (a) atA (b) atB?
Initially the chain is at rest with x = U
Determine the velocity of chain when
'A'"oug'7hM'" :S::h
k
What is the acceleration of the stone: (c) just before release
at A (d) just after release at A?
x '" L The force P is greater than Fig. 6.120 [Ans. (a) 600 m; (b) 0.402 m; (c) 1.87 m/s2 towards centre; (d)
J.1 KmgL in order to initiate motion. 9.80 m I s2 down]
Rotatory Motion
§ 7.1 Kinematics of Circular and Rotational Motion
(A] Variables of Motion
If a particle is moving in a circular mo'tion or a body is
rotating about a fixed axis (rotatory motion) following
variables are needed to describe the motion-
(a) Angular Displacement e
The angle turned by a body rotating about a given axis or a (aJ (bJ
particle moving on a circle from some leference line is called Rigid body Non-rigid body
angular displacement. Regarding angular displacement it is e1 = e2 e2 >e1
worth noting that- Fig. 7.2
(1) It is dimensionless (asS =sIr) and has proper unit (SI (b) Angular Velocity ro
unit) radian while practical unit degree or revolution
If the angular position of a body or particle changes with
21trad = 360°= I rev time it'is said to have angular velocity. If9, and 9 2 are the
y
angular positions ofa particle at time t1 and t2 respectively, the

/
,//'" ftj"
,
~~- - --- ...
s average angular velocity wav for this time interval is defined as

Olav =
9, - 9 1 l!.9
=- 1,
I r 1 t2 t, I1t
\, 0 ,I
,, ,, and instantaneous angular velocity w
\ /
is defined as the limiting value of this
'''... ... _----_ .... ./
ratio
(aJ (bJ
Circular Molion Rotatory Molion i.e., m= lim 119 = de
61-+011t dt
Fig. 7.1 Fig. 7.3
In case of unifonn circular or
(2) It is an axial vector if small and not a vector if large as
rotational motion
addition of small angular displacements IS
commutative while of large not, i..e., Olav =<0

--)0 -+ -+ -+
Regarding angular velocity it is worth I),oting that:
d9 1+d9f=d9 2 +d9 1 (I) It is an axial vector with dimensions [T - 1] and SIunit
but 9, +9 2 ... 9 2 +9, radls. .
(2) For a rigid body as all points will rotate through same
Note: See Question III in Chapter 2 for details.
angle in same time, i.e., angular velocity is a
(3) If a body makes nrevolutions, its angular displacement characteristic of the body as a whole, e.g., angular
velocity of all points of earth about its own axis is
9= radian 21tn (2",24) rad/hr.
(4) Angular displacement of all points of a rigid body is
same. In case of non-rigid body greater the distance of (3) If a body makes n rotations in t seconds
the point from axis of rotation greater will be its
angular displacement.
Olav = [2;n ] r~
.'
So, if T is the time period and f the frequency of However, to make this concept clear, consider the
unifonn circular (or rotational) motion case when A and B are closest to each other and
ro*=21t xl=21if
T
[asf=t]
moving in same direction. In this situation,
..
(4) If an equation is given between and t, the time e ..., ...,
derivative of e will give instantaneous value of
rrc! =i rB - rA i::::rB -fA
angular velocity, e.g., if
_ Vre! _ VB -VA
e = a - hI + et 2 so, (Ore! - - - -
de rrc! rB -rA
co=-=-b+2ct
dt (c) Angular Acceleration a.
So that for t =0,00 = - band for t = 1,00= -b + 2e. If the angular speedofa body is not constant, the body has
(5) Angular velocity depends on an angular acceleration. Let (01 and (02 be the instantaneous
the . point about which angular speeds at times 11 and t2 respectively, then the average
rotation is considered, e.g., if angular acceleration a av is defined as
a particle is moving on a
a :::: 002 -(01 :::: 6.00
circle from P to P' in time t as av t2 - II 6.t
shown in Fig. 7.4, the angular
velocity with respect to 0 The instantaneous angular acceleration is the limit of this
will ~e 00 = (P/ t) while with
0 Fig. 7.4 ratio as At approaches zero, i. e.,
respect to A }Vill be
OJ A= (a / t). But from geometry of figure P= 2a, so a = lim 6.00 = dro
l!.HO!1t dt
000 =2ro A ,
(6) If two particles are moving on same circle or different It is also an axial vector with dimension [T -2] and unit
coplanar concentric circles in same direction with radls 2. Regarding angular acceleration it is worth noting
different uniform angular speeds <0 A and roB that-
respectively, the anguJarvelocity ofB relative to Afor (I) If a. = 0, circular or rotational motion is said to be
an observer al the centre will be lUlifonn.

(O=de,~=dro=d2e,
(Orei =(OB -(OA
SO, the time taken by one to complete one revolution (2) As i.e., 'second time
dt dt dt2
around 0 with respect to the other (i. e., time in which
. B completes one more or less revolution around 0 derivative of angular displacement gives angular
th~n A) , .. acceleration.
(3) If a torque 't' acts on a body of moment of inertia I,
, T= 21t = 21t = TIT2 [asT = 21tJ
about axis of rotation,
, rore! ro 2 - (01 TI - T2 (0
a= «II) [analogous to a =(F 1m)]
Note: If wB = WA 'W rel = 0 and so T = oo,i.e, particles will maintain (4) Angular accelera'iion is same for all points ofa rigid
their position relative to each other. This is what actually body.
happens in case ofgeostationary satellite (ws = wE lor in case of IBI Relations among Angular Variables
rotationofarigidbody(w] =00 2 =constt.). ,.
These relations are also referred as equations of rotational
If two particles are mo~ing
B
(7)
on tw~ different concentric
"
,/
//~ r---_
r "
motion and are-
, I e =coot +-2I at .2
circles '.-" with different
velocities then angular
,,
,,,
, VA
,/
,,~

r,
A
--, , r, " ,,,, 00=(00 +at;

,,, , ,
,,,
,, and can be derived as follows-
velocity of B relative to A
,, , ,
,, o . ,,
as observed by A will
,, , ,, By definition a = (dro/dt), i.e., dro = udt
depend on their positions ,,
/
'~---~" ,, So, jfin time I the angular velocity ofa body changes from
and 'velocities"" . The , ,,
general theory is beyond
,
------ ---~~",,/ 00 to 00
0
r", dco=J'adt
the scope of this book. Fig. 7.5 a
* A!so T = (2n I (0) "" (2nrlv) [asv = roo]
de,Olora
But (dsldt) is the linear speed of the particle along the
tangent while (d8Idt) is the angular speed about 0, so
v ~ roo .... (ii)
Differentiating Egn. (ii) with respect to time
dv ~i!...(rm)~rdw [as forP,r = constt.]
dt dt dt
But (dvldt) represents the magnitude of tangential
Linear and angular variables
acceleration of the particle while (dwldt) that of angular
in circular motion acceleration so
Fig. 7.6
aT ~ ra •... (iii)
If a is constant,
00 - 00 0 =01
aT is along the tangent.
In addition to tangential acceleration aT in case of circular
or (0 = 00
0 + ext .... (i)
motion there is also a radial acceleration a, given by
Now as by definition (0 = (d8Idt), Eqo. (i) becomes
(d9Idt) = roo +W, i.e., d8 = (ooo +W)dt a r == (v2Ir) ~ rm 2 ... ,(iv)

So, if in time t the angular displacement is e, Which is along the radius towards the centre as shown in
I: =1;(0)0 +at)dt
d9 Fig. 7.7 (b).
Egns. 0), (ii), (iii) and (iv) are the desired relations among
or e=coot + (I/2)at 2 .... (ii) linear and angular variables, From these' it is clear that:
Now eliminating t between Egos. (i) and (ii), we get (1) For a rigid body as angular ,variables (i.e.,a,w and ct)
ro 2=ooo+u
2 29 .... ( 111
...) are same for all points, the corresponding linear
variables (i.e.,s,v,a T and a r ) will depend on r.
Egos. (i), (ii) and (iii) are the desired equations. These are Greater the distance of point from the axis of rotation,
valid only if angular acceleration is constant and are analogous greater will be the linear variables. e.g., angular
to equations oftransJ.atory motion, i.e.,
displacement (or velocity) of PI or P2 [in Fig~ 7.2·(a)]
v=u+at; s = ut+(1I2)at 2 and v 2 =u 2 +2as is same while linear displacement (or velocity) of P2
[CJ Relations among Linear and Angular Variables will be greater than of PI as 1'2 > rl ."
This is also referred as kinematics of a particle in circular (2) In a circle as tangent and radius are always normal to
motion. For this consider a particle P moving on a circle of each other so will aT and a r . This in tum implies that
radius r. Tfio time dt its linear displacement along the arc is ds acceleration in case of circular motion
and its angular displacement about the centre is d8, by
.... (v)
definition of angle

d8 ~ ~, i.e., ds~1'de .... (i) Here, it must be noted that aT governs the magnitude of~
while ar its direction of motion so that
Dividing both sides ofEqn. (i) by dt
ifa, ~OandaT ~O;
[d']
dt
=, de
dt
a -t O~ motion is unifonn translatory
if a, ~ObutaT ,.,0;
Y
a -t aT :::;> motion is accelerated translatory
ifa r ,.,Obuta T =::0;
a -t a r ~ motion is .unifonn circular
ifa r ,., 0 but aT ,.,0;
a -t Egn. (v) ~ motion is non-unifonn circular
'---i-'--- x ~-x
Note: Proofforar : (v2/r)
If a particle moving with unifonn speed v on a circle of radius r
suffers angular displacement e in time At, then change in its
velocity
(al (b)
-t -t -t .~ -t
Fig. 7.7 Av = v2- VI : v z + (- VI)
.,
i.e., w=~ +2vl~s(l80"-9)
.:1v - vJ2{l- cosO)
or 1
f = 21t Vl 7 )
N)
or btl - lvsin (&12) ....(i)
Here,Q, < 109, so
< ..!..~(lOx 10) i. e.,
/ 2ft 4'
/ =[l.JID:
max 2ft sec
Problem 3. A uniform disc ofradius 'R' m and mass 'M
kg can rotate without friction on an axis passing through it,
centre and perpendicular to its plane. A cord is wound at thE
o,tl.---1 " rim of the disc and a uniform force ofF newton is applied 0'
the cord. Find the tangential acceleration ofa point on the rin
ofthe disc.
Solution: As shown in Fig. 7.9, the torque of force l-
Fig. 7.8 about the axis of rotation will be
It will be directed along the radius towards the centre. 't=FxR ....(i)
Now as
But t =/a R
,. , i.e, 6t_ r9v
o __,! __ Vl1t ....(ii)
and as here /=lMR 2 anda=a r
So in the light ofEqns. (i) and (ii) radial acceleration 2 R
a ... 6v _ 2vsin(Ql2) _ v2.[sin<0/2>] I R' Xjl=2'
r At ' (rOl v) r (612)
So t=2'M aT I"I'l'IUr R ....(11.. )
F

[ I· I]
~ Equating't from Eqns. (i) and (ii), we fig. 7.9
or Dr., .. gn-(On)
,m -
(Qf2}-+O
->
(9/2) gel

Problem 1. /fthe radius a/earth ls6400km. calculate (a) i.e., ar =2F


-
M
angular velocity (b) linear velocity and (e) radial acceleration Problem 4. A motor car is travelling at 30 mls on I
for a point on its equator considering its spin motion alone. circular road ofradius 500 m. It is increasing its speed at tit.
What will be the values ofabove quantities ifthe point is at the rate of 2 mls2, what is its acceleration?
pole?
Solution: In this problem
Solution: As earth makes one rotation on its axis every
<Iv ,
24 hrs- ar = dt =2m1s [given
(a) 0>, =(2"'T) =(2"'24 x 60 x 60) =7.3 x 10-' radls 2
(b) v ==
roo (6.4 x 10')(7.3 x 10- ') = 4.7 x 10' mls
a = v =30x30=1.8m1s 2
, r 500
r--co--cc
<::::: 0.5 km/s
So, a=~a; +af =~(l.8)2 +22 =2.7mJs 2
(e) or = rCi} =(6.4)( 10 6 )(7.3 X 10-5 )2 =3.4 x 1O- 2 m1s 2
Problem 5. The kinetic energy ofa particle moving alan.
Now if the point is at pole, r---ioO so v=(rro) and a circle of radius r depends on distance covered s as K = As:
a r (= Too 2 ) will bewme zero while Q) (characteristic of rigid where A is a conslant. Find the force acting on the particle as .
body) will remain unchanged. function o[s. IMNR 1992
Problem 2. An astronaut is rotating in a rotor ofradius 4 . Solution: According to given problem
m. If he can withstand uplO ace. of 109. then what is the
!mv 2 =Ai .... (i
maximum number of permissible revolutions? (g = 10 mls 2) 2
v2
Solution: In case ofunifonn circular motion
or = (v 2/r)=rro
2
[asv = rro]
So, a, =-=-- r
2Ai
mr .... (11

Further more as
or a r = (2ttf)2r [asw=2ttf]
a =du=dv.ds = v dv (by chain rul~
t dt dsdt ds
Which in the light ofEqn. (i), i. e., V =: s~(2Alm) yields

r
Solution: As according to Kepler's 3rd law T2 IX r3

at = hl~][f~} ~ .. (iii)
[~:r =[;J i.<, [~r =[1~4
So that a=M +a; = [2~:2r +[2~Sr or
So, (a) As in case of circular motion

i.e., a=2As[I+[slr]2 T =21tr v = 21tr


m v ' T
So, F = rna =2AsJr + (sl r)2
Problem 6. A planet P revolves around the sun in a
circular orbit, with the sun at the centre, which is coplanar and
concentric to the circular orbit of earth E around the sun. P
and E revolve in the same direction. The time required for the
revolution of P and E around the Sun are 3 years and 1 year (b) When they are closest to each other and moving in
respectively. What is the time required for P to make one same direction
revolution around the sun relative to £?
-> -> 4
Solution: As Tp > TE and T =2tc!ro so wp < roE and Ivred=lv2-v1 1=1txI0 kmIhr
hence with respect to sun the difference in their angular
displacement per unit time will be (co E - cop). So, they will be and rrel =1r;-r;I=(4x104 - lx10 4 )=3xI0 4 km
at same position with respect to the sun again for the first time 4
when their relative angular displacement becomes 21t So, ifT so ro )vred:::1t x 10 =~radlhr
is the required time rei Irred 3x104 3
(WE -oop)T=21t [asO =(Ot] Problem 8. A threaded rod with 12 turns/em and

or
U; -:;]=2; [as T=~J
diameter 1.18 cm is mounted horizontally. A bar 'with a
threaded hole to match the rod is screwed onto the rod. The
bar spins at 216 rev/min. How long will it take jar the bar to
I I move 1.50 cm along the rod?
or i. e.,
'T= TE - Tp' Solution: As the distance
between two consecutive threads
so T=3xl = 1.5 year on the rod, i.e., pitch is = (1112)
3-1
em, so the number of rotations to
Alternative Solution: As angular speed of P and E are be made in moving a distance of
different, they will be in same position with respect to sun for
1.5 em,
the first time when faster one, i. e., E completes one more
revolution around the sun than the other, i.e., P. So, if n is the n = distance = -.1L = 18
number of revolutions made by faster one pitch (1112) Fig. 7.10
T=nTE =(n - I)Tp So, angular displacement of
bar
So, that TE =[I-1.J, i.e.,
Tp n e = 2n:n = 2n:x 18 = 361t rad
TET And as angular speed of bar
T=nTE= p
Tp - TE (0 =-=21tn
t
21tx216
60
Problem 7. Two satellites 8 1 and 8 2 revolve round a
=7.21trad/s
planet in coplanar circular orbits in the same sense. Their
So, time taken by bar to travel a distance of 1.5 em on the rod
periods oj revolution are 1 hour and 8 hours respectively. The
4 (as8=rotwitha =0)
radius ojorbit ojS1 is 10 km. When S2 is closest to SI' Find,
(a) The speed 0[S2 relative to SI' (b) The angular speed oj8
a
36.
1=-= --= 5s
2 ro 7.21t
as actually observed by an astronaut in S l'
308

Problem 9. Wheel A has radius IOcm is coupled bya bell =


(2) As (I) constt., from ro = COo + oJ, we find that angular
B [0 wheel C of radius 30 acceleration ( l =0. And as aT =r<l, tangential acc.
em as shown in Fig. 7.1 J. or is also zero.
Wheel A increases its
(3) As aT = 0, 0= (a r' +aT')" 2 Ylc
. Ids a=ar,l.e.,
'
angular speed from rest
at a uniform rate of 1. 57 acceleration is not zero but along radius towards
radls 2 . Determine the centre and has magnitude
time for wheel C to reach 8 a=a r = (v2/r) = r(ji
Fig. 7.11
a rotational speed 0/ 100
This acceleration is called centripetal acceleration.
rev/min assuming that the belt does not slip.
(4) Speed and magnitude of acceleration are consll.. but
Solution : As the belt does not slip v A =V c
their directions arc always changing so velocity and
i.e., rAoo A = rcCl)C [asv = rro]
acceleration are not constant. Direction of -: is always
Here, 'A = 10 em, =30 cm rc
and roc =(2nx 100/60) rad/s along the tangent while that of tr along the radius,

so 1000 =30 X 21t X 100 ~ ~


i. e., v and a r are always orthogonal.
A 60
i,e., wA = 10nrad/s ~
(5) If the moving body comes to stand still, i.e., v -.0,
Now from I st equation of rotational motion, i. e.,
the body will move along the radius towards the
00 = 00 0 +01 centre and ifrad ial acceleration Q r vanishes. the body
, (j)
1= - [as<oo = 01 wi ll fly offaJong the tangent. So. a tangential velocity

'" "
. t =I Ox1t=20sec.
and a radial acceleration (hence force) is a mustfor
uniform circular motion.
i.e.,
1,57 (6) To provide centripetal acceleration, by Newton's 2nd
; • j

Problem 10. A spot light S rotates in a horizontal plane law a force


with a constant anglllor velocitY of 0.1 radls. The spot oflight F =mQ = (nnilr) = mrro 2
P moves along. the wall at a distance 3 m. What is the velocity
e
of the spot P when = 45°?
s
is required. This force is called centripetal force and
is of cunstallt magnitude but changing direction. In
Solution: Ifxisthe distance
different problems this force is provided by differe nt
of point P from 0 then from Fig.
7,12 , •1 h 3m
origins such as friction, reaction, tension, gravitation,
elastic or electric, etc. .
tan~ ~(x/h)
or x = htanq, P
, , 0 Wall
~
(7) As F = (mu 2/1' '# 0), so the body is not in equi librium
or dx = "(scc2 ~/~ Fig . 7.12 and linear momentum of the particle moving on the
dt dl circle is nOI conserved. However. as the force IS
i.e., v = " sec' ~ [as (dx/ dt) = vand (d~ /dt) =wj central,
,
I. e.,
~
't = 0. so angular momellfum is
=
Here h 3 m, +=180°_(45°+90°) = 4S andro=0. 1radls o
conserved. i.e.•
So, v =)x (Ii)' x O, 1 =0,6mis -+ ~
P ;I! consU. but L = constl.
§ 7.2 Dynamics of Uniform Circular Mo tion (8) The work dOlle by celltripetal/oree is always zero as it
If a particle moves with constant speed on a circle. motion is perpendicular to velocity and hence displacement
is called uniform circu lar. In case ofunifonn circular motion-
Further by Work-Energy Theorem as
(I) As speed is constant amI v == r~ angular velocity will
Work done = change in kinetic energy
be constant and so the motion will be periodic with
time period so 6K = 0 [asMV=Oj
i.e., K (Killetic energy) remains COIISIOIII.
T = 21C = 21t1~ [as V = rm]
W V
Motion ofa planet (or satellite) around the sun (or earth) of rotor person is at rest and the walls rotate. In ~oth these cases
and motion of an electron around the nucleus in an atom arc friction balances the weight of person while reaction provides
most familiar examples of uniform circular motion. To make
the concept more clear we now tonsidcr following examples
the centripetal force necessary for circular motion, i. e. >
I "" mg and
,
(mv Ir)=R
.
ofunifonn circular motion--
(A) Banking of Rails and Roads; When a moving
vehicle takes a tum, it travels along a nearly circular path. So, ~mmIDD]
there must be some force to provide the necessary centripetal f
acceleration (otherwise it will skid). If the vehicle is moving
on a horizontal circular path as shown in Fig. 7.13 (a), the
weight of vehicle is balanced by normal reaction R while force
t-B-tc
mg
of friction provides the necessary centripetal force, i.e.,
~--------- ----~,
R : =: mg and jo=(nw2/ r) '--
(a) (b)

R 'Death well' Rotor


Fig. 7.14

mg
so i.e., V = rrgf R1
vl~
mg I- b-i
(b) Now for v to be minimum! must be maximum, i.e.,
(a)
Fig. 7.13 v"" = )(rg/~) [asf~ =~RJ
However, friction is self adjusting force with nn upper
Note: This example should not be cOilfused with 'looping the loop'. It
limit, i. e., j :$ ).tR(= 'ling), so for a safe tum
is (Ill example of unifonn circular motion in a horiwntal plane
(mv 2/ r)< ).tmg while ' looping the loop' [discussed ill § 7..1 and solved
prob']cm I 9] is Mil example of 1I0n-unifoml circular motion il l a
i.e., v< ..,Nrg or 00< .Je).tg)l-;: [asv=rw] .... (i) vertical plane.
However, friction is not always reliable at circular turns if

li~
(C) Conical Pendulum: If
high speeds and sharp turns arc involved. In order that a vehicle a small body (say stone) of mass
may make a safe tum without depending on friction, roads and m tied to a string is whirled in a
rails arc banked on the outerside [as shown in Fig. 7.13 (b)] so horizontal circle, the string will
that vertical component of reaction R (which is normal to the not remain horizontal [as a
surfaces in contact) balances the weight white its horizontal vertical force (mg) cannot be hi T \
component provides the necessary centripetal force, i. e.,
Rcos9=mg

v'rg
and RsinO=(mv 2/r)
balanced by a horizontal force (
T)] but the string becomes
inclined to the vertical and (~ _____J1+--------\~i\
! ____ _ ~ _____
i.e., tan 0* = - sweeps a cone while the body "'~___ 0 __ --~
moves on a horizontal circle ------------
mg
The angle 0 depends on both the speed of the vehicle as with un ifor.m speed a,s shown. in Fig. 7.15
well as on the radius of the tum. Roads and rails are banked for Fig. 7.15 . Such an ~.rrangement
average expected speeds of vehicles. However, if the speed of is called 'conical pendulum' . c.' . . • .• •

a vehicle is somewhat less or more than this. the self adjusting So in 'c ase of conicai pen'dulU1~ the ~~rtic~; ~o~ponelit of
static friction will operate between the road and tyres 3Jld the tension balances the weight while its ~oriz6ntal c?m ~onent
vehicle willllO, skid. provides the necessary centripetal force, i.e. , lcose = mg and
(8) 'Death Well' or Rotor: In case of 'death well' a T sin = (mv 2Ir). Solving these for 0 and T , we get
e
person drives a bicycle on a vertical surface of a large wooden
well while in case of a rotor at a certain angular speed of rotor a tanO = - - ,,' ....(i)
person hangs resting against the wall without any floor. In ''g

'death well' walls are at rest while person revolves while in case
and T 0= m) g2 + (v 2 !rl ... ( ii)
• !f lhe widfh of the road is hand ouler end is raised h relative to inner one, Ian 0 = (hib).
From Eqn. (i) From Eqns. (i) and (ii) it is clear thatFB = (1I2)FT' SO it is
v = ~(rgtane), i.e., ro=~gtane/r [asv=rro] .... (iii) better to apply brakes than to take tum for avoiding crashing
with the wall.
So, that time period Problem 11. (a) Find the maximum speed at which a car
can turn round a curve of30 m radius on a level road if the
t = 21t =21t~ r =21t~LCOSe coefficient of friction between the tyres and the road is 0.4
co gtan9 g [ace. due to gravity = 10 mlsz]
[asr=Lsin9] .... (iv) (b) For traffic moving at 60 kmlhr, if the radius of the
This is the required result and from this it is clear that- curve is 0.1 km, what is the correct angle of banking of the
(I) Time period is independent of the mass of the body road? (g= IOmls2) fMNR 19931
and depends on Lcos9(=h), i.e., distance between Solutiou: (a) Here, for turning, centripetal force is
point of suspension S and centre of circle O. provided by friction so
(2) If €I =90°, i.e., pendulum becomes horizontal from
Eqns. (i), (ii) and (iv) [":']<!L' i.e. , m~' <~mg [ash=~mgl
v=oo, T=oo and t=O
However, as velocity and tension cannot be infinite so i.e., v< ~).lgr so that Vrnax = ~figr
this situation is not physically possible.
Here ).l = 0.4,r=30mandg = IOm/s z
(3) If 9=0°, v=O,T = mg and t=2Jt.j(L/g) i.e.,
pendulum becomes vertical and at rest so if made to so Vrnax =.J0.4x30xlO=llmls
oscillate it will act as a 'simple pendulum'.
e
(b) In case of banking, tan = (v2Irg)
Note: If gravity is not present or weight (= mg) is balanced by reaction
by placing the body on a frictionless horizontal plane, the string Here, v =60 km = 60x 2.. mls= 50 mls
hr 18 3
will become horizontal and body will move in a circle of radius
r=L. In this r:::O.1 km=IOOm and g=IOmls 2
situation tension in
the string will
provide necessary
So, tane= 50x50
3x3xl00xIO
. 9 -1[5]
I.e., 18= tan
centripetal force N
Problem 12. A car is moving along
i. e. , T= -mv'
,' Fig. 7.16 a banked road laid out as a circle of
radius r. (a) What should be the banking
.e. ,,
I. angle e so that the 'car travelling at
speed u needs no frictional force from
So, that time period the tyres to negotiate the turn? (b) The
t = 2M = 21t /mL [asr=L] coefficients offriction between tyres and L _ _ +."..-'''''
V fT road are fis = 0.9 and).lk = 0.8. At what Fig. 7.17
maximum speed can a car enter the
Question I. The driver ofa truck travelling with a velocity curve without sliding toward the top edge ofthe banked turn?
u suddenly notices a brick wall infroni afhim at a distance d.lt
Solution: (a) x and y~components of reaction are
is better for him to apply brakes or to make a circular turn
without applying brakes in order to just avoid crashing into N x = Nsin9
the wall. Why? Ny=Ncose
Answer: Ifthe driver stops the car by braking in a distance In our co~ordinate system we apply Newton's law in
d, the force required will be given by component form.
.
I.e., FB = 2d
mv' .... (i) r.Fx = max 'f.Fy = may
Nsin8i = rna) N cos8J-WJ = rnayJ
And if the driver takes a tum to avoid the collision, the
force required will be Nsin8=ma x Ncos8-W=ma y
FT = (mv'/d) .... (ii) Since the car travels along a circle a y =0, ax = v /r
2
. mv' Sum of forces in horizontal direction,
Nsm9=--
r mv'
FI +F2 = - - .... (ii)
v'rg ---1h,..-x
N,
r
Ncos9=mg or tanS = -
Taking moments about the centre of mass G,
W (FI +F2 )h+R l a=R 2 a
In fannulating the above equations we Fig. 7.18
have assumed friction to be absent (friction FI +F2 =(R 2 -R I )* .... (iii)
does not play any role in negotiating the tum).
N Combining this with Eqn. (ii) to eliminate FI + F2 gives
(b) If the driver moves faster
than the speed mentioned above, a .... (iv)
friction force must act parallel to the
road, inward towards centre of the Friction
We now have two simultaneous Eqns. 0) and (iv), for RI and
tum. Fig. 7.19 R 2. Solving these by adding and subtracting, we find that
(Ffric)x =Fliiccos9 hmv 2 hmv 2
(Ffrie) y =F frie sinG 2R J =mg--- and 2R2 =mg+--
ar ar
Now 'i.Fx = max From these expressions it is clear that the inner wheels will
N sinS + F frie cosS = rmJ21r leave the ground when R J will become zero and' the car begins
to overturn,
IFy =ma y
2
N cosS -Fmc sinS - w=o i. e., mg= hmv
ar
Since, the tyre rolls forward without slipping, the
tendency of slipping arises sideways. Hence, we get So the limiting speed is given by v 2 = g~a as required.
F fric = (Ffric)max =~sN
, Alternative Solution: We choose the frame of reference
Nsin9 + ~sN cosS = mv attached to the car, i.e., a fictitious centrifugal force of
r
magnitude mv2lr, acts through the centre of mass G and
N cosS - !lsN sinS =mg
directed away from the centre of the circle. At the instant when
On eliminating N. we get
Sin9+/l scose)
. =-
v' the car begins to overturn, the forces R j andFJ are zero, so the
[ cos9-lAssm9 rg only forces acting on the car are R 2 , F 2 the centrifugal force
and the weight mg.
sin9+!lscos9 R,
or V= rg If the resultant P of the centrifugal
cO$9-).1ssin9 force and the weight passes outside the
Problem 13. A car ofmass m travelling at speed v moves point of contact with the ground (the
on a horizontal track so that the centre of mass describes a outer wheels) as shown in Fig. 7.20 (b),
II
circle of radius r. Show that the limiting speed beyond which
the car will overturn is given by
the couple on the car will be clockwise,
causing it to overturn. The critical
condition, when overturning is just
F,
Fig. 7.20 (b)
'p
v2 = gra
about to occur, happens when the
h
resultant P passes through the contact point. By using similar
where2a is the separation ofthe inner and outer wheels and h triangles, we can see that the condition for overturning is thus
is the height of the centre of mass above the ground.
mv'
--+mg>-
a
Solution: We consider the rear Rl
, r+---h r h
view of the car; centripetal force mv G
r which gives v 2 > g~a as before.
to the left is provided by frictional
force acting on the tyres. From the Note: These analysis are only valid ifthe frictional forcesFI andF2 are
force diagram, F, mg large enough to provide the centripetal acceleration, if they are
Sum offorces in vertical direction, a a not, the car will skid instead of overturning. If the coefficient of
Fig. 7.20 (I) friction between the tyres and the road is).t,
····U) Fl+F2~).t(Rl+R2)
where the equality implies that the lyres are skidding. Since, Problem 16. A particle describes a horizontal circle Of
2
Fi + F2 = E.!!:...., the IimitiJ~g speed before the smooth inner surface ofa conicalfimnel as showl! in Fi&
R] + RZ = mg and , 7.22 If the height of the plane ofthe circle above the vertex i
skidding occurs is given by
9.8cm.find the speed ofthe particle. [MNR 1992
2
v = !1gr ,,
Solution: The forces acting on the ,
Jfwc want the car to skid rather than overturn, we must have particle are reaction R and weight mg as -------r-------
a shown in Fig. 7.22. So for vertical
i. e. , !1 <-,
h eqUilibrium of the particle in horizontal
mg
plane
Problem 14. A tube of length L isfilled completely with
an incompressible liquid a/mass M and closed at both ends. R cos 9 = mg .... (i)
The tube is then rotated in a horizontal plane about one of its And for circular motion of the
ends with a uniform angular velocity Ol Find the force exerted particle in horizontal plane
by the liquid at the other end. Rsin9 =-- (mv 2Ir) .... (ii)
Solution: In rotational motion Dividing Eqn. (ii) by (i)
the angular velocity remains same
for all points so the force required tanS=-,
v' r-~
i.e., v =.Jrg tan 9
Fig. 7.22

for the circular motion of element 'Z


shown in Fig. 7.21. or v = .Jrg x (hiI') = Jih [as from figure tan 9 = hlr
dF = (dm)xol
Fig. 7.21 v = ~9.8 x 9.8 x 1O~, = 0.98 m1,
[asF=mrro 2 ] Problem 17. Two blocks ofmasses m l = 10 kg and m 2 =
i.e., dF '~ [~ drJxro 2
[as dm= 1dx] kg, connected to each other by a massless inextensible strin.
oflength 0.3 mare placed along a diameter ofa turn table. Th
So, ,F =I~ (MIL)",'x dx =(I/2)M"" L coefficient of friction between the table and m 1 is 0.5 whil
there is no friction between m2 and the table. The table i
Problem,IS. A string qflength 1m isfixed at one end and rotating with an angular velocity oflO radls about a vertic(.
carries a mass oj1 00 g a'; the other end. The string makes (2/1t) axis passing through its centre 0. The masses are placed alan.
revol~iidns per second around vertical axis through the fixed the diameter of the table on either side of the centre 0 sue
e~d. Calculate - (a) the tension in the string (b) the a'n gle of that the mass ml is at a distance ofO.124mfrom 0. The masse
inclination of the string with the vertical alld (c) the linear are observed to be at rest with respect to an observer on lh
velocity of the mass. (g = 10 mli) turn table:
. .. ---,
Solution: As shown in Fig. 7.15, for vertical equilibrium (i) Calculate thefrictionalforce on mi'
of mass m (iO What should be the minimum angular speed of th
Tcos9 =mg .... (1) turn table so that the masses will slip from this position?
While for circular motion (iii) How should the masses be placed with the strin,
remaining taut, so that there is nofrictiollaljorce actillgon tit
TsinS =(:nv 2ir) =mro:i .... (ii) mass m l ?
(a) SofromEqn ..(ii), T = [~rro2/sin9] = mL0)2 Solution: 0) As for the c..!...J (Q

circular motion of mass m 2 ,


[asr=Lsin9]
centripetal force is provided m, T o T m,
[as 0) =2nf =2n x (2/1t) = 4]
by tension alone
T "" m r 0)
2 2
, ,_f:
~" - 7-r2----'<
. ,
1
(b) From Eqn. (i), cosS = mg = 10- x 10 5 = 5xO.176x(10)2 = 88N Fig. 7.23
Tl.6 =8'
And as for circular motion of mass m1 centripetal force i
i.e.•
provided by both tension a!ld friction, i.e.,
(c) v=rro=LsinSxO) [as r=LsinS] 2
T + f =mt1iro
or v=l xO.78x 4 = 3.12m1s [as sinS =0.78]
so, f=lOxO.124x 10 2 ·- 88-=124-88=36N
Note: As the limiting friclion IL "'j.tmJg '" 0.5 x 10 x9.8 '" 49N, here surface of the bowl is also rotating with the same 01 Th e
the foree of friction f(z 36N) is lesser than I L • particle is at a height h from the bollom ofthe bowl.
(a) Obtain the relation between h alld ro What is the
(ii) The masses will slip if minimum value of(;.)needed, in order to have a non-zero value
2
T + IL < m,r]w i.e., (ml'i - mZ'i)w2 > IL of"?

[asT"'m 2 rZoo
,J (b) It is desired to measure 'g' using this set up. by
measuring h accurately. Assuming that Rand (;.) are known
i. e., (lOxO.124 - 5 xO. 176)ro 2 > 49 or (02 >~ precisely and that the least-count in the measurement of h is
0.36 10- 4 m, wllat is the minimum possible error I!:.g ill the
i. e., 00> 11.67 radls measured value of g? (g = 9.8 mls 2)
So, for slipping wmin =11.67 radls t..l)0l N
Solution: The forces acting on the R"""""'"
(iii) In absence of friction, the masses will not slip if particle are reaction and weight, so for L-__-_~__~_-__-_b_~_~__-_-__-_
m]Ylw2 = mz rzoo 2 = T(~CPF) vertical equilibrium of the particle
," 0: R-h
Ncos8=mg .... (i) r ........ ----- -
i. e., mlr] ::: m Zr 2 with r1 +r2 =0.3m ,, h
And for circular motion in horizontal
i.e., lOr, = 5(0.3 - rl ) or rJ =0.1 m
plane mg
And so, r2 =0.3 -0.1 =0.2 m Fig. 7.25
.... (ii)
i. e., if the centre of mass of m, and rn z coincides with the centre
of the lable. the siring, will remain taught and masses will not (a) Dividing Eqn. (ii) by (i)
slip whatever ~ ro. tan 0 = ro)2 = R(sin8)(;.)2 [asr=Rsin S]
Problem IS: Water ofdensitypflows wilh a linear speed g g
v through a horizontal rubber tube having the/orm ofa ring of . R-h g
or
radius R. Ifthe diameter ofthe tl/be is d «<
R),jilld the tension
coso = L , I.e., --~--
R.,' R R():i
in the rubber tvbe.
or h ~ R-(g/.h .... (iii)
Solution: :'::onsider a small portion of the tube as shown in
Fig.7 .24(b). Centripetal force is provided by radial component So forh > O,R - (g/ro 2 »O
of tension in the tube. So, ifT is the tension in the tube
i.e., ", > ~(glR) so "'m>, ~~(9.8 /0.1)~7.J2rad/s
-~------o . ,--------- (b) From Eqn. (iii),g = ro 2 (R - h). So for a given roand R
T \, F,'
, T 2
dg = - ro 2dh, i.e., (tlg)min = -(ro )min (6h)min
\0: /
"'I',
~,

So, substituting the value of oomin and (6h)min


o
(6g)min =-98 x IO- 4 =_9.8xlO-3 m/s2
(a) (b)
Fig. 7.24 Problem 20. Three particle. each ofmass m are situated
at the vertices of all equilateral triallgle of side u. The ollly
F =2Tsin9
forces acting 011 the particles are their mutf/al gravitational
2
So that v :m = 2TsinS, i.e., forces. It is desired that each particle move in a circle while
mailllainillg the original mutual sepamtioll a. Find the initial
But dm =n(dl2)2 (2RS)P [asm =nr 2/p] .... Oi) velocity that should be given 10 each particle aJ/d also the time
period ofthe circIllar motioll. IMNR 19901
Substituting the value of dm from Eqn. (ii) in (i), we get
Solution: As the gravitational force between any two
2 2
T = npv Sd =.!.npul d 2 [ as lim sine =1] particles is F = G mml a 2, the resultant force on each particle
4sin9 4 e~o 9
due to the other two
Problem 19. A hemispherical bowl o/radius R = 0.1 In is FR. ~rF-;':-+-F
-:C
' -+-2-FC;'-co-s-6-0°
rotating abOllt its OWII axis (which is vertical) with an angular
velocity III A particle o/mass 10- 2 kg 011 thefrictionless inlier i. e., FR =.J3F=J3Gm 2/a 2 .... (i)
So if the particles were at rest they Which is the equation of a parabola. So the surface of
will move under the action of F R revolution is a paraboloid and for a point on the rim y = hand
acting on each and will meet at the x=rso
centreO. Now ifeach of the particle is
given a tangential velocity v so that h = (ror)'/2g .... (vi)
FR acts as centripetal force, they will Here, 0)= (21tx21l) = 41trad/s and r = 0.05 m
move in a circle of radiusr= (2/3)
a sin 60°= (al.J3) maintaining the So ( '
h=4tt)
,
x(0.05) =2xI0-2m=2cm
Fig. 7.26 , 2x9.8
original mutual separation a.
i.e., FR = (mv2/r) Alternative Solution: According to Bernoulli's principle

Which in the light ofEqn. (i) and above value of r yields p+~pv2 = constant
c: -
,,3 Gm'
- =-mv' c:.I,e.,
-'\13, V=
~Gm
- At. sides the velocity is higher [as v=rw], so the pressure is
a2 a a lower. Since the pressure at a given horizontal level must be
So that time of one revolution equal, the liquid rises at the sides to compensate forthis drop in

=~ ~;m =2~3~m
pressure i.e.,
T=2:r
!pv2 =pgh or [asv=rw]
2
Problem 21. A liquid is kept in a cylindrical vessel which
is rotated along its axis. The liquid rises at the sides. If the which is the desired result, i.e., Eqn. (vi).
radius oflhe vessel is 0.05 m and the speed afrotation is2 rev § 7.3 Dynamics of Non-Uniform Circular Motion
per sec, find the difference in the height of the liquid at the
If in case of motion of a particle moving on a circle a
centre of the vessel and at its sides. (g =9.8 mls2) torque acts, angular acc. (X (= TIl) will not be zero and so its
Solution: Consider a particle at angular velocity 00(= 000 +at) will not be constant and so the
position (x, y) as shown in Fig. 7.27. motion will not be uniform. Such a motion is called
For its equilibrium along y-axis non-uniform circular motion. In this situation as
RcosS = mg .... (i) l.:::=:j:;,:=l '* 0
't '* constt.
so Angular momentum' L
And for its circular motion in [ F '* 0 so Linear momentum p '* constt.
Kinetic energy '* constt.
horizontal plane
speed :# constt.,
R sin e = mx0)2 .... (ii)
So, dividing Eqn. (ii) by (i)
To understand
consider the motion of a small
this
/"......
-- -
_H__ _
...~"\,
tanS = (o>'lg)x .... (iii) body (say stone) tied to a string // \

and whirled in a vertical circle. I


:.
0 \
But from differential calculus we :
know that Fig. 7.27 If at any tin:~ the body is at
angular position S, as shown in \
1, T ~v
'
dy
Fig. 7.28, the forc!;;!' acting on '\ r "
slope=-=tanS
dx
.... (iv)
it are tension T in t:~:" string "'. . -J __ /
~
mg sine,
~
~ mg cose
So, from Eqns. (iii) and (iv) along the radius toward:. :he " /
dy 0)2
-
dx= g- x • j.e., ro'
dy=-xdx
centre and the weight of the
body mg acting vertically Fig.
mg
7.28
g
downwards.
which on integration yields
Resolving the forces along the radius and tangent at P we
0)2 x2 finrl [,Iat for circular motion
y=- - + C
g 2 ,
But as atx =0, y=O,C =0 !!El- = T - mgcos9
r
ro' , .... (v)
,
y= 2g x or T = mv + mgcos9 .... (i)
r
v"",,-z./fi
'*
Note: (i) Here T mgcos8 or Teose 0#. mg as the body is not in (6) In case of looping
T"",,-z.Q
equilibrium. the loop for hori~
(ii) Torque aboutO here will he mg rsin (} ¢ O(as torque ofT and zon1al position of /" ---:.----
H, ....
string (i.e.,S =90 0 ,/ :, '\,
mgcos9 about 0 are zero) so ro and hence v(= rro) will not , ,' ,,
for point M) by ,, ,
remain constant. Ifthe particle moves fromLtoH torque will
oppose the motion and so 0) (or v) will decrease and when
body moves from H to L torque will increase OJ (or v).

Eqn. (i) is the desired result and from this it is clear that-
conservation
mechanical energy
between Hand M
of
\
:

,,
,
"
'
:

°I~-~-~~~Mi
,,
, T -z. 3mg
/
.l..V"it..f3W

(1) At different positions (9) of the body, v will be


different and if out of €I, v and T any two are given
I 2 I 2
"2mvM ="2mvH +mgr
"...
"-
L
---
VmlX. "it.!5Qr
,,/

third can be computed. Which in the light of Tmox.;?:6mg


Eqn.(iii), Fig. 7.29
(2) T will be minimum when cos €I = min =-1
Le.,9=180o, i.e., at highest point. So, ifvH is the i. e., v H :z. -fir yields:
velocity at highest point v M ~~3gr
T.
mv'
= ----.!L - mg .... (ii) And on substituting this value of v M with S =90 0
in
~" r Egn. (i), we get
So, in case of circular motion in a vertical plane TM :z.3mg
tension is minimum at the highest point. This all is shown in Fig. 7.29.
(3) The body will move on the circular path only and only Condition for Oscillation or Leaving the Circle
ifTmin >0 (as ifTmin :S 0, the string will slack and the
In case of non-uniform circular motion in a vertical plane
body will fall down instead of moving on the circle).
if velocity of body at lowest point is lesser than ~5gr, the
So, for completing the circle, i.e., 'loofing the loop'
particle will not complete the circle in vertical plane. Now it
mv'
----.!:!.... - mg :z. 0 i. e., VH :z. -fir .... (iii) can either oscillate about the lowest point or after reaching a
r certain height may loose contact with the path.
(4) Now 8j:pJying conservation of mechanical energy If hi is the height where v becomes zero, by conservation
between highest point H and lowest point L of mechanical energy
1 2 1
2mvL = "2 mvH +mg2r
2 1 ,
2mvL = mgh, , i.e., h =...b.
v' .... (vii)
I 2g
Which in the light ofEgn. (iii) gives
And if h2 is the height where o
vL~~5gr .... (iv) tension T becomes zero, by Eqn. (i) 1i ;:o'--~---------)
i.e.Jor looping the loop, velocity at lowest point must mv'
be ~ ~5gr. _P- + mgcosS p =0 .... (viii)

' ,ll~--~--~--~~-
r
(5) Tension in the string will be maximum when And by conservation of mechanical p //

i.e.,
cosS =max = I,
e =0,
energy
1 , I, h
j iL ___ __
'h /
:::~::_~~ ~~~_
-'

-mv L =-mvp +mg 2 Fig. 7.30


i.e., at lowest position
mv' i.e.,
2
, , -2gh
2
Vp =V L 2 .... (ix)
i.e., T =-.--..f..+mg .... (v)
mru< r
Substituting the value of v p from Egn. (ix) and
And if the body is looping the loop, i.e., v L :z. ~5gr =
Tmax :z. 6mg .... (vi)
,
cosS p (r - h2 )lr [from Fig. 7.30] in Eqn. (viii), we get
[v L -2gh,l +g[r-h,l =0
So in case of motion in a vertical plane tension is r r
maximum at lowest position and in case of looping Which on solving for h2 gives
the loop Tmax :z. 6mg.
h _ vZ + gr .... (x)
,- 3g
From Eqns. (vii) and (x) it is clear that: Solution: Consider the body inside the loop at position E
(I) Ifvp becomes zero before T vanishes, the body will as shown in Fjg. 7.32. Then for its circular motion~
oscillate and will not leave the path, i, e., for 2
R _ mgcos9 = mv
oscillation hi < Il z· r
v 2 v L2 + gr 2
i, e., ~ < -"c 0-'''- or or R "" mv + mgcos9
2g 3g r
Now for v L = ~2gr, from Eqns. (vii) and (x), we get So R will be min, when cos 9 = min = - J,
hi = hz = rwhich implies 8 = 90°, So, if the velocity of i. e., 9 = 180"
the body at lowest point isO< V L < ~2gr (or 0< 8< 90,,), (i.e., at highest point). So ifvf{ is the velocity of the body a-
the body will oscillate about lowest position. highest point
(2) 1fT becomes zero before v p vanishes, the body will R min =(mv~/r) - mg
leave the circle with velocity v p along the tangent. So Now in order to 'loop the loop' body should not loosf
for leaving the circle h2 < hI contact with the track any where so that
2 2
i.e., vL+gr<v L or vL>~2gr R min <:! 0,
3g 3g i.e., (mv1Ir)-mg>O
Now from the theory oflooping the loop we know that
or [asm is tinite1 .... (1.
if v L 2: ~5gr, the body will loop the loop. So if the
Now applying conservation of mechanical energ)
velocity ofa body at·lowest point is such that:
behveen points A and H
.j2gr< v L < .jSgr
0+ mgh = ~ mv~ + mg(2r)
the body will move along the circle for > 90° and e
will not reach upto highest point but will leave the 2
VH
circle somewhere between 90°<9< 180°. Here it is i.e., h=2r+ - .... (i(
worth noting that at the point of leaving the circle 2g
T =0 but v p ;1:0. This aU is shown in Fig. 7.31. Substituting the value ofv If from Eqn. (i) in (ii),

~~ - -__ T= 0
v;tO (<:.
T<:.O
J9ri
h=2r+[~~l i.e., h<:!~r
/~pf--",
//" {-,\;t 0 ,
/

,,
,
i

''
\
,,
,
so hmin = (5/2)r
:,
,
0 e
,
I ~,, O· ,, ,~,
Problem 23. A mass 0/2.9 kg slIspendedfrom a string Q
length 50 cm is at rest. Another body 0.1' mass 100 g which i~
\, :, ' /
'---$---"'-
'
\ ,"
--<!>--'
:,
i ,
, " moving horizontally with a velocity of 150 mls strikes ant
sticks to it. \~) What is the tension ill the string when it make:
(a) (b) (e) all angle of60° with the vertical? (b) Will it complete a vertica
For Oscillation For Leaving the Circle For looping circle?
H
O<v L::;!2W !2gr < v l < ./5gr the Loop Solution: (a) Apt'lyjng the
0<e::;90° 90° < e < 180
0
vl ~ Isgr law of conservation or lin('ar
Fig. 7.31 momentum to the collision, the
velocity V ofthe composite mass
Problem 22. A small A
after collision wi!! be given by:
body is allowed to slide on H
0.1 x ISO = (2.9 +O.I)V
an inclined frictionless ,: V:..::5 mls
track from rest position as h i
shown in Fig. 7.32 What 0., R Now to calculate the
~,
must be the millimum i e velocity v of composite body
height, so that body may
Li P;;;;;~m;:,ii,~
..I.............;........ when it makes an angle of 60°
successfully complete the with the vertical we apply law cof conservation of mechanic a
Fig. 7.32 energy between Land P, i. e.,
loop? IEAMCET 19911
IMv 2 = lMv 2 +Mgh And as in circular motion
2 2 2
T = mv + mg' cosS
i.e., v 2 =V 2 - 2gL(l-cosS) [ash=L(l-cos6)] r

or 2 2
v =5 -2 x 9.8 X 2" 1- I[ 2I] so T will be maximum when cos = max = I e
i. e., e= 0 0
(at lowest position)
or v 2 =20.1 (m/s)2
so T
mv'
= __ L +mg' [as at lowest pointv=v L ]
Now as for circular motion of P - r
(Mv 1/L) = T - MgcosS [asr=L] or Tmax =6mg'
[as forjust looping the loop v L = ~5g'r]
so T = 3 ~1\~~.I) + 3 x 9.S x k= 120.6 + 14.7 = 135.3 N so Truax =6xO.Olxl09.8=6.59N
(b) For looping the loop Problem 25. A nail is located at a certain distance
vertically below the point ofsuspension ofa simple pendulum.
v L ~ ~5gr, i.e., VL >~5x9.Sx(1I2)
The pendulum bob is releasedfrom a position where the string
or vL ~ 4.93 mls makes an angle of600 with the vertical. Calculate the distance
But as here the velocity at lowest point V is 5 mis, the body ofnail from the point ofsuspension such that the bob wil/just
will be looping the loop. peiform revolution with the nail as centre. Assume the length
Problem 24. A unifonn electric field ofstrength 10 6 Vim of the pendulum to be one metre.
is directed vertically downwards. A particle of mass 0.0 I kg Solution: Ifris the radius of the loop about the nail, for
and charge 10-6 caul. is suspended by an inextensible thread just completing the revolution
of length 1m. The particle is displaced slightly from its mean v L = ~5gr
position and released. Calculate the time peri.od of its
Now this v L is provided by
oscillation. What minimum velocity should be given to the
conversion of potential energy of
particle at rest so that it completes a full circle in a vertical
the bob into kinetic energy in
plane without the thread getting slack? Calculate the
moving from A to L, so by
maximum and the minimum tensions in the thread in this
conservation of mechanical
situation. (g = 9.8mls 2 ) energy between A and L
Solution: Here the body in addition to force of gravity
(= mg) will also expcrience an electric force (= qE) and as
imvz = mgL(l-cosG)

both are directed vertically downwards so Fig. 7.34


[ash=L(I-cos6)] .... (ii)
F = mg + qE Now substituting the value ofv L from Eqn. (i) in (ii)
i.e., mg' ~ mg+qE 5gr =2gL(1-cosG)
g'~g + qE = 9.8+1O-6xI06
or
m 0.01
=109.8m/s2
or r=~x r[I-~] =i=o.2m
(a) Now as for a simple pendulum So, the distance between point of suspension S and nail N,
T = 21t{I;=2~
yg'
I -=0.6s
109.S
y ;;;L -r= I - 0.2 = 0.8 m
Problem 26. A small block of mass m slide~' along a
(b) And for looping the loop smooth frictional track as shown in the Fig. 7.35.
v L >~5g'r (i) If it starts from rest at P, what is the resultant force
acting 011 it at Q?
so (v L) min = .j"'5-'-x"'I0"9".Se-x"'l = 23.43 mls
(it") At what height above the bottom ofthe loop should the
(c) For looping the loop T > 0 block be released so that theforce it exerts against the track at
so Tntin =0 the top of tile loop equals its weight?
Solution: (i) By conservation of mechanical energy Substituting the value of v~ from Eqn. (ii) in (i)
between points P and Q
T =2mg(cos9 -cos9 0 ) +mgcos9
mg(5R)=mgR +~mv2 so T=mg(3cose-2cos9 0 )
i.e., v=~8gR
h .. 5A
1 (b) For a given angular amplitude9 0' T wiU be maximum
when cosS = max = 1, i. e.,S =0°, i.e., at lowest position
Now in case of circular
so Tmax = mg(3-2cosS o)
motion N
a Now for oscillation without breaking the string
mv'
N(orT)=T
mg
Tmax < Breaking strength
Fig. 7.35
or 40g(3 - 2eos 9 0) < 80g
+ mgcos9
or 2cose o > I, i.e., 9 0 < 60°
And as atQ,e =90°
Problem 28. A mass m is released from the top of a
N_
-T-
mv' _m(8gR)_8
R - mg vertical circular track of radius r with a horizontal speed vo'
Calculate the angle S with respect to the vertical where it
So, resultant force on m at Q leaves contact with the track.
H
F =J(8mg)' + (mg)' = (.J65)mg Solution: The forces ~-i'!'==~v, R
acting on the body are its Y :
(ii) At highest point weight mg and reaction R as •~-------- p
: ,/'e
mv'
N=---mg (as9=180")
shown in Fig. 7.37. h: /'
R So for circular motion of ~/ r mg

But according to given problem N = mg the body at any position 9 "---oi;o~---'"


mv' v=~2gR
mv'
- - =mgcos9-R
Fig. 7.37
so T=mg+m g , i.e., r

If for achieving it h' is the height, by conservation of or R=mgcos9--- mv'


mechanical energy again r
The body will leave contact where R -+ 0
mgh'=~mv2 +mg(2R)
2
i.e., mv'
mgcos9 - - - = 0
r
or 1{=2R + v =2R + 2gR =3R
2g 2g
i.e., cose = - v' .... (i)
Problem 27. A 40kg mass hanging at the end ofa rope of rg
length L. oscillates in a vertical plane with an angular Now applying conservation of mechanical energy
amplitudee o' What is the tension in the rope when it makes an
between Hand P, we get
angle e with the vertical? If the breaking strength ofthe rope is
80 kg, what is the maximum angular amplitude with which the I 2 1 2
'2mv ='2mvo +mgr(1-cos9) [asy=r(I-cos9)]
mass can oscillate without breaking the rope?
Solution: (a) When a body tied
" or v 2 =v~ +2gr(l-eos9) .... (ii)
to a string of length L moves in a
vertical circle
mv'
T = L P + mgcos9 .... (i)
I
,, ' '
eo'"
:, e '" "
L
,'
,
"
f----- T_______)!r 0
,"
Substituting the value ofv2 from Eqn. (ii) in (i)
v'
cos9 =~ + 2(1-cos9)

1
rg
/'~ H
,.------- .-v -
:
Now by conservation of mechanical
energy between P and H with
$-------P , i. e., eos9=[vi
3rg
+~]3
h = L(cos9-cos9 0 ) mg

~mv~ = mgL(cos9 -cos9 0 ) ... (ii)*


Fig. 7.36
or V2 + _
9 = eos - 1 _0
[ 3rg 3
2]
, Here we have asstuned that at extreme point v _ 0 and T :j! O(i.e.,9 < 90°) which is condition for oscillation.
0
Note: (i) If Vo = O,cos9 = (2/3) which in the light of Fig. 7.37 gives Problem 31. A small object slides without friction from
(hlr) =(2/3), i.e.,h =(2!3)r,i.e. ,ifthe body starts sliding from the heightH = 50cm and then loops the vertical/oop o/radius
rest it will leave contact with the track at a height (2/3)rfrom R = 20 cm from which a symmetrical section of angle 2a has
base or (rl3) from lop. been removed. Find ang/ea such that after losing contact at A
(ii) For e= 0", Vo = .[ii,i,e., if at highest point the velocity of and flying through the air, the' object will reach point B.
body Vo ~ .Jirthe body will leave the track at the highest Solution: Applying C
point flying horizontally without travelling any distance on law of conservation of
the track. mechanical energy between
point e and A :
Problem 29. A particle at rest starts rollingfrom the top
ofa large frictionless sphere ofradius R. The sphere isjixed on 1 2
mgH=ZmIJ A o
the ground. Calculate that heightfrom the ground at which the
particle leaves the surface of the sphere. +mgR(l+cosa)
Solution: As for rolling, friction is a must with v =rroand As here H = 0.5 m and Fig. 7.39
as here the body is a particle, i. e., r = 0 and the track is R=0.2m
frictionless, so rolling is not possible. So, assuming that so v~ =2g(0.3 -0.2cosa) .... (i)
particle slides from the top, this problem becomes Solved
Problem 28 in with H:;; R + h = R + (2./3)R = (S/3)R Now as at A(v A *" 0) the path of object from A to B will be
Problem 30. A particle of
H that of a projectile, with angle of projection a. so that
/ ............ --~------.,
mass m is moving inside a smooth Range = distance AB
R • '
vertical circle of radius r. If it is I ,I ,: or (v~ sin2a)/g=2Rsina.
"
I \

projected horizontally
velocity Vo from its lowest ,
with , :,'
'' , ' , mg,\
i. e., 2 - R
position, find the angle S (as \----------()~;----------1
vAcosa.-g .... (ii)
, , '
shown in Fig. 7.38) at which it will \ : / So substituting the value of v~ from Eqn. (i) in (ii)
lose contact with the circle. \ : /
", : " ~/
' 2g(0.3 -0.2cosa. )cosa. = 0.2g
Solution: If the body is at any "-." - _ _~o _ _ ,flo'" ...... /

position ~, then for its circular i.e., 2cos 2 a. -3cosa +1=0


L
motion- Fig. 7.38 or (cosa-l)(2cosa. -1)=0
.... (i) i. e., cosa. = I or cos a =112
Here ~ =90°+9 [as for leaving the circle 90 < ~ < 180°]
0 oc a. =0° oc a. = 60°
So, R = (mv2/r) - mgsin9 As according to problem a *" 0°, so a. = 60°
The body will leave contact where R ~ 0 Note: Here (HIR) = (50f20),Le,H = (5/2)R, so if the sector is not
i.e., v 2 = grsinS .... (ii) removed the object will just 'loop the loop' .

Now applying conservation of energy between lowest Problem 32. A particle is suspended vertically from a
position L and this position P with h = r(1 +sin9) point 0 by an inextensible massless string of length L. A
kmv~ =~mv2 +mgr(l+sinS) .... (iii)
vertical line AB is at a distance Ll8 from 0 as shown in Fig.
7.40. The object is given a horizontal velocity u. Atsome point,
Substituting the value of v 2 from Eqn. (ii) in (iii)
its motion ceases to be circular and eventually the object
passes through the line AB. At the instant of crossing AB, its
v5 = grsin 9 + 2gr(1 +sinS) velocity is horizontal. Find u.
o :,A
0' sinS =[vL~]
3gr 3
.. (iv)
Solution: Let the string be slackened at
point C. Then for circular motion,
US ,I
,,
T
mv' I
,,

1
T+mgsinS=T L :
Note: (i) At the point P where body leaves the circle R = 0 but v ¢ O. :
(ii) As for sin e!> 1,(v~/3gr) < (513), i.e.,vo < J5ir and for where v is the velocity of particle at point e, For :,
sin 8 > 0, (v~/3gr) > (2/3), i.e, Vo > ..p:g;, so for leaving the pointe, :
u-'B
circle..J2Kr < Vo <~. T=O Fig. 7.40
v = ,j'gLo--s'--in"S ....(i)
Now, applying law of conservation of energy, Solution: If the ball has to just complete the circle then
the tension must vanish at the topmost point, i.e.,T2 =0.
imu2 =kmv2 +mg(L+LsinS) From Newton's second law,
2 2 q2 = mv2 F

.
or u =v +2gL(1+sinS) " .. (ii) .... (i)
A, I T,
The particle will leave '
H":-~- v vcos8
, , mg
circular motion at C and now At the topmost point T2 =0,
: ' q
move as a projectile with initial o ~-----7'-<;;"--;~'C q2 mv2
velocity v at an angle e with : v Sin e
: ~T
'1>"
,...- mg - = -- .... (ii)
T,
vertical. As it passes line AB in i A~i).~ 1'1)9 Cl0o$'6' 4m>oP l
the horizontal direction, implying
that point H corresponds to
highest point in its projectile
O,e
Po"
'"
I :,
------------"---
From energy conservation,
Energy at lowest point = Energy at
--oJq-U
mg
F
Fig. 7.42
trajectory and CD equal to half topmost point
L
of horizontal range, .!.mu 2 =.!.mv 2 + mg21 .... (iii)
..

2
2
2
CD =: I v sin (180°-20)
g 1 ,u ,B
iU
2 2
v 2 =u 2 -4g1
2
.... (iv)

From Eqn. (ii), v 2 = gl-


~-~
or v sin29 = LcosS - f '8 ' q .... (v)
2g 8 Fig. 7.41 4m:oml

[asCD = A' N = (ON - OA')] From Eqns. (iv) and (v),


Substituting for v 2 from Egn. (i),
r-------''---
q2
u = Sgl

2g
gLsin€l .sin2e =,( cosS
\. _!) 8
=(2;Sr2
41tBoml

or sin 2 SeDsO = COS€l-t


= 5.86m/s
or COS0(1 - cOS2e) = cose - ~ Problem 34. A block is placed inside a horizontal hollow
cylinder. The cylinder starts rotating with one revolution per
1 second about its axis. The angular position 0/ the block at
i.e., or cos9= -
2 which it begins to slide is 30° below the horizontal level
e =60 0
passing through the centre. Find the radius a/the cylinder if
Substituting for v 2 from Egn. (i) and e in Egn. (ii), we have the coefficient offriction is 0.6 . What should be the minimum
angular speed of the cylinder so that the block can reach the
u2 = gLsin 9 + 2gL(1 +sin9) highest point o/the cylinder?

=g{~ + 2 + 2:l
Solution: Forces acting
on the block are shown in the
Fig. 7.43. The block begins to
slip when
u= [;L (4 + 3.J3) JII2 mls towards right. J.1N = mgcos300 .... (i) mg sin 30"
From Newton's second law,
Problem 33. A small ball of mass 2 x 10- 3 kg having a mg cos 30"
charge of 1 IlC is suspended by a string of length 0.8 m. N -mgsin30o=mroo 2.... (ii) Fig. 7.43
Another identical ball having the same charge is kept at the
point of suspension. Determine the minimum horizontal or N = mrro 2 + mgsin 30° .... (iii)
velocity which should be imparted to the lower ball so that it
or ll[mrro 2 + mgsin 30°] = mgcos300 .... (iv)
can make complete revolution.

,
or r = cm~g~cco~S~3cO_O-_~":im~g~s=in=3CO:...o So for a body made up of number of particles (discrete
2 distribution) of masses mt , m2 , •. . ete.,at posiiions Ii. , r2 , ..• etc.,
f..1mro
respectively from the axis of rotation:
_ gcos30o.........gsin30°
~ro2 ....(ii)

On substituting numerical values, we get while for a continuous distribution of mass, treating the
element of mass dm at position r as particle
r,,=O.25 m
Forces acting on block at topmost position dl::dmr2, i.e., I=Ir, 2 dm ....(iii)
are shown in the Fig. 7.44. From Newton's Regarding moment of inertia it is worth noting that:
second Jaw, I
mg (1) It has dimensions [ML 2] and SI unit kg m 2.
N + mg:= ln1''(f,2 (2) It is not a vector as direction CW or ACW is not to be
Block will just complete a circle when specified and also not a scalar as it has diflerent values
N =0; mg=mrai
Ag.7.44 in different directions 0. e. , ahout different axes). It is
a tensor as discussed in Chapter 2 [Note (2) after f'ig.
i.e., ro =.Jglr 2.9].

= ~ 10 = 6.32 rad/sec
(3) MOl11cnt of inertia for a givcn body depends on the
axis of rotation, So ifaxis of rotation changes, usually
0.25
moment of inertia will change.
The block can reach the highest point only if able to just cross
point A. For point A, from Eqn. (iv), (4) Moment of inertia ti)r a given axis depends on mass,
shape and size of the budy. If same mass is casted in'
ro=~ different shapes with same axis, moment of inertia
will be different (Sec Fig. 7.46).
no
='i~
(5) For a given shape, size, mass and axis it depends on
the distribution of mass within the body. Farther the
constituent particles of a body arc from the axis of
= 8.16radls
rotation larger will be its momcnt of inertia. This is
So, minimum velocity = 8.16 radls
why in casc of a hollow and solid body ~f same mass,
§ 7.4 Rotational Dynamics radius and shape for a given axis, moment of inertia of
(A) Moment of Inertia(or Rotational Inertia) hollow body is: greater than !,hat for the solid body.
It is the property of a body due to which it opposes any (6) Radius of Gyration: II is Ihe ·distance whose squ~re
change in its state of rest or oj uniform rotation. Analytically when multiplied with the mass of the body gives the
for a particle of mass m rotating in a circle of radius r, moment momenl of inertia ~f the body about the given axis.
of inertia ahout the axis of rotation is given by For (I hO((v ~f mass M if k is radius ofgyration

····0) I =Mk 2
or k= j(ji,i1) .... (iv)
Through this concept a real body (particularly
-----ml irregular) is replaced by a point mass for dealing its
r,
rotatiOJl(lllllotion, e.g., in ease ora .disc rotating about
an axis lhrough its centre of mass and perpendicular to
its plane.

(a)
Particle
(b)
Discrete system
(c)
Continuous body
k=g - -
2
Fig. 7.45 =JQI2)MR =R.
,M Ii
(ML'/ 12); so M.l. about an axis perpendicular to
W ."', .0, , UJ '
length and passing through one end by theorem of
parallel axis will be
R A C
A ,,C
" IG :
1 t . MA2 "
(.
Ring Disc
° \,. G ° ,'
- ~
,
,,
,,
" UJ ,,,
, ,
B 10 B '0
1' a io) ib)
' . I'" c' b
, Fig. 7.47
, J'" ,." i. e., I=I G +Ma 2

1= ML2 + M[~]2 ~_M_L_2


, .. .lMA2
,~: 'r I, 2 . ,. J 1 ,., <"il' • or
Solid Cylinder . Solid Box 12 2 3
" I ',1-' "

(8) Theorem of perpendicular axes for a lamina:


According to it the suin of moment of inertia of a plane
lam ina about two mutually perpendicular axes lying
.' in its plane is equal to its moment of inertia about an
axis perpendicular to the plane of lamina and passing

,-- Ml2 .
... 12
through the point of intersecti2.1J of first two axes.
Z Z
,, I,. ",I ori
Rod , "
Rod
,.
"
(.0 " <),OI----1--V Y
.'
"

"
x)'..-s- x

"
.. , la) Ib)
1.~MR2 Fig. 7.48
i.e., Iz=lx+l y .... (vi)
. Solid ~phere $pherical,Sheli
... , -' e.g., moment of inertia ofa ring about an axis through
its centre of mass and perpendicular to its plane is
Fig. 7.46 '
MR 2, so if the ring is in x-yplane, then by theorem of
so'instead "of disc we can assume a point mass M at a
perpendicu lar axes
" ,; distance (R 1../2) from the'axis of rotation for dealing
i.e., I z= l x+ l y ; MR 2 = Ix+ l y
the rotational motion oCthe disc.
(7) Theorem of parallel axes: According to it moment However. ring is a symmetrical body so for it
o/inertia ofa body about a given axis I is equal to the Ix =I ). =I D
moment o/inertia ofthe bodyabollt an axis parallel to 21 0 ~ MR 2 or I 0 ~ (112)MR-'
given axis and passing through centre a/mass of the
i.e., moment of inertia of a ring about a diameter is
body Jop/us Ma 2 where M is the mass o/the body and
(1I2)MR 2
a is the distance between the two axes.
i. e., 1= IG + Ma 2 .... (v) Note: [n case of symmetrical two-dimensional bodies as moment of
inertia for ali axes passing through the centre of mass and in rhe
e.g., Illoment of inertia of a rod about an axis through plane of body will be same so the two axes in the plane of body
its centre of mass and perpendicular to its length is need not be perpendicular to eaeh other.
(9) Physical significance of moment of inertia: (f we
p = !!..[..!./oo2] = foo doo
compare the expression for rotational motion with dt 2 dt
corresponding relations for translatory motion as
given in tabular form below, we find that moment of
oc P = faoo [as (dOli dt) ~ ct J
inertia plays the same role in rotatory motion as is oc p = 'tOO [as! :::: fu] .... (ii)
played by mass in translatory motion, Le. , if a body Furthennore as P = dW and ro = de Eqn. (ii) reduces to
has large moment of inertia, it is difficult to start dt dt
rotation or to stop it if rotating. Large moment of
dW ~ ,de
inertia also helps in keeping tlte motion uniform. This dt dt
is why stationary engines are provided with
fly·wheeis having large moment of inertia oc w~hde .... (iii)

Translatory motion Eqns. (i), (ii) and (iii) represents kinetic energy, power and
Rotatory motion
work respectively in rotational motion. Here it is worth noting
I. Force F = ma Torque .=Ia that as in rotational motion angular momentum L = fro (§ 7.6)
2. Work w",IFds Work W=hde so Eqn. 0) may also be written as

J.
4.
Power
KE
P=Fv
I nrv 2
Power
KE
P='r(U

1 2
KR =~/[7r =;; .. (iv)
Kr =2 KR =z.Iw which is analogous to (p2I2m) [kinetic energy in translatory
5. Momentum p=mv Momentum L = lro motion].

IB) Kinetic Energy, Work and Power in Rotational Motion


Ie) Rolling Motion
Ifin case of rotational motion ofa body about a fixed axis
A body rotating about a fixed axis
possesses kinetic energy because its (called spinning), the axis of rotation also moves, the motion is
constituent particles are in motion, even called combined translatory and rotatory. Motion of a rolling
wheel orflying saucer can be cited as familiar examples of this
though the body as a whole remains in place.
type of motion. In this situation kinetic energy of the body will
The energy which a body has by virtue of its
be given by
rotational motion is called rotational kinetic
energy. K =Kr +KR
In order to calculate this energy consider a oc 1 2 I 2
K =~ Mv + ~fro ....(i)
rigid body rotating about an axis 2 2
Fig. 7.49
perpendicular to the plane of paper through 0 where v is the velocity of centre of mass of the body.
with angular velocity ro. The speed of a particle at a distance r Here the two tenns translational and rotational kinetic
from the axis ofrotation will be given by v = rro. So if the mass energy are independent of each other. For example a flying
of the particle is m, its KE will be sauecr is set spinning about its axis (for orientation stability)
[as v = rro] and is independently given a translational velocity to boost it
into space.
Now the body actually consists of large number of
particles which need not to have same mass or be at same v
distance from the axis. However, all particles have same
angular velocity 00 (as all particles of a rigid body have same
angular velocity). Hence, the total kinetic energy of the body
(a) (b) (e)
KR = l:.! mr 2oo2 = .!oo 2 l:mr2
Fliction
2 2 =0 Spinning Friction *- 0
Slipping
KR "' ~1Q) 2
Rolling
[asl:mr2 = f] KT"' ~Mv2
Fig. 7.50
Now as in a rigid body palticlcs maintain a Hxed position
Now in case of combined translatory and rotatory motion
relative to each other and move only with the body as a whole
if the object rolls across a surface in such a way that there is no
so we can equate the rate at which work is done on the body,
relative motion of object and surface at the point of contact, the
i. e., power to the rate at which kinetic energy changes, i. e.,
motion is called Rolling without slipping.
In case of rolling without slipping-
(I) Friction is responsible for the motion but work done
or Mgh=.!.(3Mv 2 with
2
~" = [I+_I ] .... (iii)
Mr'
or dissipation of energy against friction is zero as
there is no relative motion between body and surface
at the point of contact.
So that V=~i(2gh) ....(iv)

(2) As rotation produces translation Now from figure h = s sin 9


21tr = 21t(=T) so v2 =(2gs sin9)/(3
v ro
i.e., v=rro* .... (ii) or 2v dv =(2g sina/~)ds
dt dt
So, that Eqn. (i) becomes
I, , or a=[!(gsina)][asdv =0 and ds=v] ... (v)
K = -00 [/ + Mr- ] ~ dt dt
2
2 As here acc. is constt. and body starts from rest, so from
i.e., K=&lpro with I p =I+Mr 2 2nd equation of translatory motion
i.e., rolling motion ofa body may be treated as a pure s(= hlsina) =0 x t + ~ (gsin a/~)t'
rotation about an axis through point of contact with
same angular velocity 00.
i.e., . ... (vi)
(3) In case of rolling all points of a rigid body have same
angular speed but different linear speeds. The linear
speed is maximum for the point H equal to 2v while Eqns. (iv), (v) and (vi) are desired equations for velocity,
minimum for the point L and is zero. This all is acceleration and time of descend respectively in case of body
illustrated in Fig. 7.51. rolling on an inclined plane. From these equations it is clear
v that-
(1) As factor (3 = [1 + (/I Mr2)] depends on the shape of
+ = body and is independent of mass, so ifbodies of same
shape but different masses are allowed to roll down an
incline plane, they will reach the bottom with same
speed and at same time.
(a) (b) (e) (2) Velocity, acceleration and time of descend (for a
Translation Rotation Rolling
given inclined plane) all depends on (3. Lesser the
Fig. 7.51
moment of inertia of the rolling body lesser will be (3
As an example of rolling (but> 1) so greater will be its velocity and acceleration
consider the motion of a body and lesser will be the time of descend. This is why-
rolling on an inclined plane as >; 1 (i) If a solid and hollow body of same shape are
shown in Fig. 7.52. As the body
rolls the inclined plane, it loses
h , allowed to roll down an inclined plane then as
(3 S < PH' solid body will reach the bottom first
potential energy. However, in
rolling it acquires both linear and
j with greater velocity.
(ii) Ifa ring, cylinder, disc and sphere runs a race by
angular speeds and hence, g~in Fig. 7.52
rolling on an inclined plane then as (3 sphere = min
kinetic energy of translation and that of rotation. So by
while (3 ring = max, the sphere will reach the
conservation of mechanical energy
bottom .first with greatest velocity while ring in
I
Mgh=-Mv 2 +-/00
I 2 the last with least velocity.
2 2
(3) The velocity is independent of the inclination of the
But as in rolling v = roo plane and depends only on height h through which the

Mgh =- Mv
2
I 2[1+--
I]
Mr2
body descends. Acceleration and time of descend
depends on the inclination. Greatcr the inclination,
grcater will be the acceleration and lesser will be the
time of descend.
* However, ifthcrc is slipping in rolling or a spinning body is thrown with certain velocity, v 'Ie- roo but motion is still combined translatory and rotatory and
[qn. (i) will still hold good .
.... P is a t1irnensionless conSII.grcater than I and depends only upon the shape of body and is independent of its mass and radius.
(4) If we consider different types of motion viz. rolling, fB] Rotation about a fixed axis, In motion under this
slipping and falling for a given body then. category, the particles fonning the rigid body move in parallel
R(Ai" " plane circles centered on the same fixed axis. If this axis passes
through the body, the particles located on the axis have zero
~ I
h
I
h
velocity and zero acceleration, If the axis passes through centre
of gravity of body, motion is said to be centroidal rotation, if

~'_-----Jj
axis is shifted motion is said to be non-centroidal rotation,
-'-"0'-------_ j !
~z tz
:
I') c-f...... :
;00 cbw
Sliding
,,
" (--- - ,-----,
---- ,,------
C'
,----- ~-I -- -- - -;.
---- -t-------
Q ----r-----,
, ~~""-,,,- ~- -- --'
° (0)
,I
,, ,
Falling
Fig. 7.53 (al Axis within the body (b) Axis outside the body
Fig. 7.55
Physical Falling
Rolling (~ > 1) Sliding p = I p"~ [C] General plane motion. A rolling wheel has general
quantity
9=90° plane motion, It translates as well as rotates, hence general
I. Velocity v R . ,j(IIP)2gh Vs .,j2gh v F .,j2gh plane motion can be throught of as combination of rotation and
translation.
2. Acceleration aR '" (l/p)gsin €I as :::gsinEl aF =g
3. Time of
descend tR=Si~e)~~) IS::: Si~e!! tF =!! [D] Three dimensional motion. The motion of top on a
rough floor fulls under this category,
Rotation of a representative rigid body
(i) Velocity of falling and sliding bodies are equal and is When a body rotates about a fixed axis, velocity of any
more than rolling.
(ii) Acceleration is maximum in case of falling and point P can be expressed as V= eok x 1 = reo (in magnitude)
minimum in case of rolling. Since point P under consideration is in curvilinear
, (iii) Falling body reaches the bottom first while rolling translation, it must have a tangential acceleration (a , ) and a
body in the last. nonnal acceleration (a,), given by:

§ 7.5 Rigid Body Dynamics -> '->


a/ = a.kxr;a/ = ra
Various types of rigid body motion can be classified as
follows: -+ 2-+ 2
a ll =-{() r;all ,=oo r
[A] Translation. In translation all the particles fonning a
body move along parallel paths. If these paths are straight y, Tangential axis
Vp= roo \
lines, . the motion is said to be 'a rectilinear transiation; if the Normal at = ret.
paths are curved lines, the motion is a curvilinear translation,

\
p

:\\-~J \
:.-~ .~~\-- ---.~'~
,) \~~i
:.'~ ~~~\ Q, (a) (b)
Q, Q' Q' Fig. 7.56

(al Rectilinear translation (b) Curvilinear translation Tangential acceleration is tangent to path traced by point
Fig. 7.54 and centripetal acceleration is directed towards axis of
rotation.
Case (i): Rigid body in pure rotation (axis passes through Instantaneous centre of rotation
CG): A' A'
F sin IX F
For centroidal motion, (Radial
+~a~Fcosa A" Final "
axis passes through centre of component)
(Transverse ,Qosition
gravity, hence acceleration ,,",
component) B' B" B"
of centre of mass (a( =ra = B B"
Oanda n =ro 2 r=0, because Fig. 7.57 General plane motion Translation + Rotation
Fig. 7.60
r=O)
Hence equations of motion are Consider a rod slipping along two walls. In order to find
out final position of the rod we may imagine a translation of
'iF/ = 0 [sum of forces along tangential axis] rod parallcl to itself so that it reaches the dotted position A' B'.
LFn =0 [sum offorccs along nonnal axis] Now imagine a rotation about end B' so that it achieves its final
L. = 10. [sum of torque of all the forces about 0] orientation. Hence a body in general plane motion may be
considered to be in translation as well as rotation.
For a body in pure rotation apply L. = 10.. The radial
component of force F does not affect the rotation of wheel. Similarly a sphere in pure rolling translates as wet! as

r=s::
The torque is exerted by tangential component only. rotates.

Case (ii): Rigid body in pure rotation, non-centroidal ~v " ~RW +

~ ~v ~~
motion (axis does not pass through CG):
For non~centroidal motion,
centre of mass moves in a circle of General plane motion Translation Rotation
radius r, hence it may have Fig. 7.61
tangential as well as centripetal For a body in general plane motion, a point can exist such
acceleration. that the body may be assumed to rotate about an axis through
,,
For non~centroidal motion the that point at the instant. Such a point is called the instantaneous
centre of rotation. Since the velocity of centre of rotation is
~~:--- -1 ---
following equation is applied,
zero, the velocity of the instantaneous ccntre of rotation is
L'C :;= 10. .... (i)
zero. Consider a rigid body which translates as well as rotates.
Resolving all the forces along Mg
Consider any two points on the body, P and Q. If it is in pure
nonnal and tangential directions, Fig. 7.58
translation, all thc points on the body must have same velocity.
"iF/ = mal .... {ii) In plane motion each point has two velocities, one due to
.... (iii) rotation and the other due to translation. Resultant velocity of
'iFn = man
any point is vector sum ofthese two velocities. Points P and Q
a / represents tangential acceleration of CM have different velocities and hence it is in general plane
an represents nonnal acceleration of CM motion.
Herc you can notice that CM moves along a circular path. Ifbody shown in Fig. 7.62 is assumed
Case (iii): Rigid body in plane motion: to be in pure rotation about any point
(which we call instantaneous centre of
For non~plane motion following equations are applied, rotation) then nonnals to Vp and VQ must
L'C =10. .... (i) N
, pass through point I, centre of rotation. As
Resolving all thc forces along x-axis normal to velocity is radius vector, so it
andy~axis, must pass through instantaneous centre of
.... (ii) rotation. Since at any instant angular
velocity of each point on the body is same,
LFy=ma y .... (m) mg we may write,
Fig. 7.59 Vp V
For solving problems on rigid body -oo=ro = Q
- Fig. 7.62
dynamics students should decide the type of motion the body IP IQ
perfonns. According to motion, the one of the above three V =0 IQ xro (perpendicular to lQ)
Q
cascs will arise. Vp =IPxro (perpendicular to IP)
B coefficient' of static fri ction. Hence in. case of ro lling motion
the magnitude of friction fo rce is independent of N, nonnal

A

-. reaction,
,
I>
+ V, -.
I>
When sliding , is impendi!1g, theJrictiol). fqrce ca~ be

.............-.
calculated from N, When the disc slides and rotates at the same
.J2 Roo I> +
time, the force of friction h~s !"ag?itude Fk~ '" I-lk"N, where Ilk
I> is coefficient of kinetic frictiort '1n this case acceleration of
Instantaneous centre of rotation
I centre of mass of disc and an~lar acceleration are not related,
A ·----------------1---
:: i, e" a is nOl equal to ra.. I
' l' " (\
VA , In brief follow ing three cases are possible: '
,,, ,,'
, R,911ing, no sliding , F :5'1;1- $!' a eM '" ra. I',
e +-V B Rolling, sliding impending F '" Il $N a CM 00; (J.)"
B Rotating and sliding ,F 00; fl k N a CM and 'u are
Fig. 7.63
'" . ", " intlepimdent
A body in pure rolling may be assumed to be in pure
Note: When it is not clear whether a dise slid~ it' is a~sumed that the
rotation about contact point as shown in Fig. 7.63. Point of disc rolls without sliding. Ifftietion force is found to be less than
contact is instantaneous centre of rotation. or equal to J.l.sN, our' assumption is If~frietion force is
cJrrect:
Velocity of any point on rolling disc is vector sum of found to be larger than Il sN, the assumption is' incorrect and the
problem should be started again, ~ssuming ro!a,l in;-a~~ sliding,
velocity oftranslalion (VT) and velocity of rotation (VR ).
Consider a rod slipping on two walls. Draw normals to Question II . State whether Ihe siaiement gi~n be/~w ,:;
velocities of ends A and B. Point of intersection of these true or false giving reason in brief I :'11- I

nonnals is instantaneous centre of rotation. "A ring of mass 0.3 kg and radius 0.1 ri, and a 'solid
V V cylinder of mass 0.4 kg and of the same radius are given the
_8_ =_ A
_ = 6) = constt
{sinS IcosO . same kinetic ellergy and released simuiro'neously on a flat
horizontal sUI/ace such that ,they begin to roll as sogn as , •• ' " , I •
Note: Consider a rolling sphere; we may write its kinetic energy as
released towards a wall which is at the saine distantejrom the
KE", KE translation + KErolalion ring and cylinder, The rolling fric.t!on in both the _cases is'
_ I MV' II ' negligible. The cylinder will reach the wallfirst, ""
-"2 em +2"(()
Answer: II), case ,of rolling as, WI,' \'"
'" lM(Rro)2 + .!('lMR20~l
,,) 1 II

225
K='tMV:, +~J(I} with 1! "" rro,\ ,
.,.2MR2fi)2
10
or we may assume that sphere is in pure rotation ab01:lt point of
so K ='1. Mu 2 [J 4- IIMr2] "
. 2
contact.
'. '. " 'I' '.. ' : ' " ,"
Now
KR = Mv~, i!e." vk 00; ~KR'/013 ;j~
where Ie is moment of inertia abou t contact point from parallel
. 2
ax is theorem And for cylinder, I '" (l12)Mr i. '

(314)"1"~' i.e., Vc .=)(4K~/3 x 0.4) = .J~K~


c""IO"7.3
, 7 ,
Ie ., Icc+ MR =SMR Kc =
,.
Now according to given problem ' K~ ~ Kc ' :s~' vR =ve
Rolling motion
and as the motion i~, unifonn, both , will reach the wall
When a disc rolls without sliding, there is no relative simultaneously, i,e., given statement is wrong,
motion between contact point of the disc and ground. As
Question III. A body o/mass M dHd'rdUiuij., roliingoll a
explained earlier a rolling disc may be assumed to rotate about
smooth Itorizqntaljlo0l," with vF,locity)vl roHs up an irrel:,1JIJar
contact point, which we call instantaneous centre of rotation.
A rolling disc may be compared with a block at rest on a inclined plane up to a vertic~l height (3v2/4g)" ICompute the
surface. The magnitude of friction force on static block can moment of inertia of the body alld commellt 011 its shape,
vary from 0 to maximum value F m3.X 00; 1-1'/";, where 1-l.I' 'is
This is the position of centre of
mass of the rod from MI' So the
required work is minimum when M1
:
the rod is rotating about an axis -x---J-
passing through its centre of
mass and perpendicular to the
:

,,,
,,
.! .
L - x-
""
c..p
M2

length of the rod. Fig. 7.65


Fig. 7.64
Answer: The total kinetic energy of the bOdy: NOll": As accord ing 10 Iheorem Or parallel axes
1 2 I 2
K=Kr+KR = - Mv + - /00 J =: 1(; + M(al ) so (/),nin = IG as (Ma 2 )min ... 0
, 2 2
Problem 37. Two point masses m l and m 2 arejoined by a
or K =!Mv 2 [1 +(/IMr')] [aS11:::; roo]
2 weightless rod a/length r. Prove that the moment o/inertia 0/
the system about all axis passing through the centre vf mass
When it rolls up an irregular inclined plane of height
and perpendicular to the'rod is
(h =3v 2/4g), its KE is converted into PE, so by conservation m1m2
of mechanical energy where ~=
m l +m2

!MV'[I+ _ I ]=~)3V'] and is called reduced mass ofsystem.


2 Mrl 'l 4g Solution: (a) Jf 1'1 and 1'2 are distances of th.e masses m,
2 and m 2 from the centre of mass tban the moment of inertia of
which on simplification gives I ::: (1/2)Mr . This result clearly
indicates that the body is either a disc or cylinder. the system about centre of mass will be
1 = 11/11'12 + m 1'2
2
Problem 35. Calculate the radius ofgyration ofa slender 2 .... (i)
rod of ma,fS M and length L ahoul an axis of rotation But by the definition of centre of mass
perpendicular to its length alld [lassillg through the centre. ....(i;)
mIl'l = m2'2
IMNR '9961 and 1', + r2 = r (given) .... (iii)
Solution: Here 1 :0: Mf.2t12 aud as I =. Mk 2 So from Eqns. ( ii ) and (iii)
m I' mIl'
Mk 2=: !~~.~, i.e. , k=2lJ 1'1= 2
(m l +m 2 )
and'2
(m l +m 2 )

Problem .16. Paim masses M!, and M 2 are placed at ihe This in turn mean that

opposite end~' ~fa rigid rod o/hmgr" L oml afnegligible mass.


nil.! rod h to be set rotating about all axis perpendicular to it.
Find the positiol! 01/ this 1'00 through which the axis should
pass ill order Ih{/t the work required 10 sci the Y(x/ rolalillg with i. e., with
QlIgu/al've/oci1rWo slwuld be minimum. IMNR 19961
1 , Problem 38. Two thin discs each of mass 4.0 kg and
Solution: (KE), =0 (KE)F =2(/' +I,)<no radius 0.4 m are attached as shown in Fig. 7 .66toform a rigid
body. What is the rolational inertia a/thiS body aboul all axis
So by Work-Energy-Thcorcm:
perpendicular to the plane 0/ disc B and passing through its
W=(KE)F -(ICE), = ~(/, +1')0: 6 centre?
Solution: Moment of
or W=~[MIX2 +M2(L - x}2](O~ inertia of each disc A and B
about the axis through their
For W to be minimum centre of mass and perpendi-
cular to the plane will be
'Z =0 i. e., 2M 1x-2M 2 (L-x)=0
1AA =188 =(II2)Mr
2

so Now moment of inertia of Fig. 7.66


disc A about an axis through B by 'theorem o/parqlle/ axes'
will be
lAB =;/AA + M(2r)2 = (9/2)My Z or
I 2
so 1 = IBS + / A8=2Mr +29 Mr 2 Ifv.is the linear velocity of the end hitting the floor, then
v=rw = J3gL [.,, = L]
i. e., / =;; 5My 2 = 5x 4 x(0.4)2 =3.2kg-m 2
Problem 41. A rigid body is made o/three'jdentical thin
Problem 39. Find the moment q( O2 M2 01 rods, each oflength Lfastened together in theform ofthe leiter
inertia of ,a square plate about a ", , ~/,,- H. The body is free to rotate about a horizontal axis that runs
diagonal (Fig. 7.67)and show thaI it is .' ')'., / along the length ofone ofthe legs of!he H. The bOdy is allowed
eqllal lo the rolalionol inertia about a " '. // to fall from rest from a position in which the plane of H is
median lille. -t~;;o~,,*,,--I- M, horizol/tal. What is the angular speed 0/ the body when the
/ \ ,.-
Solution: By symmetry I D = ,,' "" plane ofH is vertical?
I/) = I D (say), The two diagonals 'ofa ,"'""-+--""" Solution: Moment
of inertia of the system
A
sqoare intersect at right angles, so Fig. 7.67
using theorem of perpendicular axes about the given axis
c
1: = 210 I=I A +18 + I c
But for a square a '" b = L
,
Now as rod is thin '?J
(L' + L') ML' IA ::=LmXO =0
so / =M =-
z 12 6 Fig. 7.68
Rod B is rotating
/n =(1/2)/, =ML'/12 about one end

, ,
Again by symmetry 1M = 1 M :: 1M (say). Now as the I n = ML2/3
And for rod C all points are always at distance L from the
two medians also intersect at right angles, by theorem of
axis of rotation, so
perpendicular axes
I z =2I M = 21D soI M = 10 Ie = '£mL2 = ML2

Note: In this problem as I AI ':: I D so that / = 0 + (ML'/3) + ML' =4 ML'13

I; ""1M +ID So if ro is the desired angular speed, gain in kinetic energy


But median and diagonal are not orthogonal. Is it a violation of due to rotation of H from horizontal to vertical position
theorcm of perpendicular axes?
1 , =-'[4
KR ",, -/00 - ML , ]co,
2 2 3'
Problem 40. A thin rod of length L alld mass M is held
vertically with one end Oil tht! floor and is allowed to f all. Find And loss in potential e~ergy or the system in doing so
the velocity of the other end when it hils the floor. assuming
that the end onlhefloor does not slip.
=0 + Mg~ + MgL=~ MgL
Solution: Lei M be the mass and L the length of the slick. So by conservati?n of mechanical energy
When it is held vertically, its centre of mass is at a height (U2) (2/3)ML'",' = (3/2)MgL
from the floor, so Ihallhe potential energy of the stick is Mg
(U2). On releasing, the stick falls, i. e., it rotates about the end
on the floor and the pOtential energy is converted into co=~Jf
rotational kinetic energy ~lw2, where 1 is the moment o f Problem 42. A carpet 0/ mass M made of inextensible
a
material is rolled along its length in the form of cylinder of
inertia of the rod about the lower end and 00 the angular radius R and is kept on a rough floor. The carpet starts
velocity when it hits the floor. Thus by . conservation of unrolling without sliding on the floor when a negligibly small
mcchil.nical energy, push is given to it. Calculate the horizontal velocity ofthe axis
of the cylindrical part of the carpel when its radius reduces to
Mg!:.=!/ro 2 RI2. lilT 19901
2 2
Solution: If p is the density of the material of the carpet,
L I ML2 2 initial mass of the caipet (cylinder) M will be 7tR 2Lp while
or Mg-=---w
2 2 3 when its radius becomes half the masS of cylindrical part will be
MF = n(R I2)'Lp= M I4 ....(i)

So initial PE of the carpet is MgR while final Now as in rolling


(MI4)g (Rn) = MgRIS 1 2 I 2
K = K r +K R ="2 Mv +'2 [00
So loss in potential energy when due to unrolling radius
c hanges from R to RI2 And here [ = (2 /S)Mr2 with v '" roo

K=~MV2 + ~[~Mr2J[;r
= MgR - (1/S) MgR = (7/S)MgR .... (i)
r---'l~ so

i. e., .... (ii)

So in the light ofEqns. (i), (ii) becomes


7
K=-Mg(R - ,) .... (iii)
(a) (b) 10
FIg. 7.69 As this kinetic energy is provided by loss in PE, so
applying conservation of mechanical energy between P and H.
This loss in' potential energy IS equal to increase in
rotational KE which is 0+ Mgh =I~ Mg(R - r) + Mg(2R - r)
I 2 I 2
K = Kr +KR='2Mv +2 /00
or h = I~(27R- 1 7r]
Ifv is the velocity when half the carpet has unrolled then as
No te: In this problem due to rolling motion KE of sphere at any point i!
v=RID, M .... M and l = l [ M ][ R]' (7/10)mv 2 and not ( 1I2)mt? So if we ap ply conservation of ME
2 4 2 4 2 .J5ir
r
between P and L taking vL = the solut ion will becomE

~Jv' +H~~'][~
wrong as in deriving it from vH "'..fir we have taken KE at 1
K =H 1 2
equal to 2mvL'

i.e., K=!Mv2 +..!.Mu 2 =1.Mv 2 .... (ii) Problem 44. A small sphere rolls down without sJjppjn~
S 16 16
from (he top of a (rack in a vertical plane. The track has at.
So from Eqns. (i) and (ii) elevated section and a horizontal part. The horizo~tal part j~
(3/ 16)Mv' = (7/S)MgR 1. 0 m above the ground level and the top of the track is 2.4 n,
above the ground. Find the distance on the ground witl
i.e., v=~(I4gRI3)
respect to B (which is vertically below the end of the track:
Pro,blem 43. A small solid marble of mass M and radius where the sphere lands. During its flight as a projectile doe:
'r' rolls down along loop the loop track, without slipping. Find the sphere continue to rotate ahout its centre ofmass.'Explafl~ .
the height h above the base, from where it has to start rolling
Solution: Loss in PE .when the sphe:r~ rolls. fr,o m C to A
down the inc/ine such that the sphere just completes the
vertical circular loop ofradius R. " Mg(2.4 -I) = 1.4 Mg
This potential energy is converted into KE of rolling
Solution: Here the centre of mass of marble will move in
a circle of radius (R - r) so for just looping the loop, at H K= ! Mv 2 + 1[0)2
p 2 2
Now as for sphere [ = (21S)Mr 2 and for rolling v = roo
1h 50 K = .!.Mv 2 +~Mv2 =2Mv 2
o 2 5 10

j So by conservatio n of mechanical energy:


I.4Mg = (7/10)Mv 2 or v={ii
Fig. 7.70
c 2 4
0' v =-gh
3
Now differentiating both sides with respect to time
2vdv = ~gdh
2.4 m- dt 3 dt
"O i ·' A
-->---<e, 0'
2
a=-g asdV = a and dh=v]
3 [
\ dt dt
,""'1-
'-----"-"------,t:-"---- - ---- - -~-- - ­
\
Now as cylinder is accelerated down [W =: M(g - a)1
B
Fig. 7.71
2T = M(g - a), i.e., T "" ~ Mg[I-~] =iMg
Now at A as v is horizontal, time taken by the sphere to
reach the ground Alternative Solution: The equations of translational and

t=~ = l;l =H
rotational motion of the cylinder will be
Mg - 2T = Ma (Translational motion) .... (i)

SO the horizontal distance moved by the sphere in this time And 2TR = Ia (Rotational motion) .... (ii)

x =vt = J2ii x ~(2/g) = 2m Now as 1 = 1- MR 2 anda "" aIR, the equation of rotational
2
i. e., sphere falls at a distance of 2m from B. motion, i.e., Eqn. (ii) reduces to
Now before leaving the point A the sphere has an angular I
momentum (= 1(0) and as in flight torque ofmg will be zero (as 2T = -Ma .... (iii)
2
mg will pass through centre of mass of sphere), so angular
Solving Eqn. (i) and (iii) for a and T, we find
momentum will remain constant. This in tum implies that
sphere will continue to rotate about its centre o/mass during 2 I
a = -g and T=-Mg
flight. 3 6
Problem 46. A uniform A
Problem 45. A cylindrical rod o/mass M, length Land ,
radius R has two cords wound around it whose ends are circular disc has radius R and mass !
attached to the ceiling, as shown in Fig. 7.72. Tlte rod is held m. A particle, also of mass m, is ,IR
horizontally with the two cords vertical. When the rod is fixed at a point A on the edge ofthe cL-------- _J
released, the cords unwind and the rod rotates. Find the disc as shown in the Fig. 7.73. The ~!c-----___,hRl4
disc can rotatefreefy about aflXed P Q
tension in the cords as they unwind and determine the linear
horizontal chord PQ that is at a
acceleration of the cylinder as it falls.
distance RI4from the centre C ofthe Fig. 7.73
Solution: The cylinder ...... "'11...... '11'......... disc. The line A C is perpendicular
rotating under gravity has both 10PQ.
translational and rotational T T
Initially, the disc is held vertical with the point A at its
motions. Let v be the linear highest position. It is then allowed. to fall so that it starts
velocity of its centre of mass andro T T
rotating about PQ. Find the linear speed of the particle as it
its angular velocity about the axis reaches its lowest position.
of rotation, then in descending a
Solution: As moment of inertia of a disc about a diameter
distance h it will lose PE = Mgh RCL'>..L"-""--'----''---'--'''-"'
while gain in KE is ~(~mR2 J. the moment of inertia of the disc about the

=[!Iro +.!.MV
2 2),SO Mg
chord PQ by 'theorem ofparallel axes' will bc
2 ,2 Fig. 7.72
1
~4mR
2 JI
+"\"4 R
)2 5
=16 mR
2
Mgh ",, !...Iw 2 +.!.Mv2 (lD)PQ
2 2
and as particle of mass m is at a distance [R + (R I4) ~ (5/4)R]
Here 1 ",,!...MR 2 and v = Rro from PQ, the moment of inertia of the system about PQ
2
5 2 5 )2 15
1=(ID)PQ + (Ip)PQ =16 mR +"\'4 R =SmR
2 J
so Mgh=!...Mv
2
2
+ !...[!
2 2
MR2][.'C]
R2
Now ifrois the angular speed of the system when A reaches the i.e., so (,,) 2
. =-lan9
lowest point A' on rotation about the axis PQ, by 'conservation r nun 7
ojmechanical energy', Alternative Solution: In case of body rolling on an
kIm2 = mg(AA') + mgCC' = mg[2AD +2CD] inclined plane, its acceleration = g sin 9/P

Finally Here P=[l + m:2]=1+~=~ [asfsphere=~mr2]


so 5 . 9
a = "'7 gsm

Now as for translatory motion of sphere


Initially mgsin9 - f = ma
(a)
Fig. 7.74
(b)
f = mgSinB[I-~J = ~mgSine
i.e., ~x l~ mR 2ro 2 =2mg [( R +~R )+~R l Rest part of the problems have the same solution as above.
Problem 48. Two thin circular discs of mass 2 kg and
radius IOcm each arejoined by a rigid massless rod of length
i.e., ro=4{f;; 20 cm. The axis of the rod is along the perpendicular to the
planes of the disc through their centres (See Fig. 7.76). This
object is kept on a truck in such a way that the axis ofthe object
so (V)A,=rro=(R+lR )X4{f;;='/5gR is horizontal and perpendicular to the direction ofthe motion
of the truck. Its friction with the floor of the truck is large
Problem 47. A sphere of R enough so that the object can roll on the truck without
mass m rolls without slipping on o slipping. Take x-axis as the direction ofmotion ofthe truck and
an inclined plane of inclination 9. k~ " , z-axis as the verticaily upward direction. If the truck has an
j;'.'<l 0 ....
Find the linear acceleration ofthe ~~lJ acceleration of9 mls 2, calculate:
sphere and force offriction on it. m, mg coso (E) The force offriction on each disc.
What must be the millimum value
of coefficient of friction so that , (ii) The magnitude and the direction of the frictional
torque acting on each disc about the centre of mass o of the
sphere may roll without sliding? Fig. 7.75 object. Express the torque in the vector form in terms of unit
Solution: If a is the ~~ ~

acceleration of the centre of mass of the sphere and! the force vectors i, j and k in the x, y and z-directions.
of friction between sphere and the plane, the equations of - - " , " - - - 0.2 m-----;i..
translatory and rotatory rriotion of the sphere will be , Disc

y
mg sinS - f = rna [Translatory motion] .... (i) 0, 0, F' .. maT
00

And fr = fa [Rotatory motion] .... (ii) Y


, 4"
0.1 m
Now as a = aIr and here f = (2/5)mr 2 m m

Eqo. Oi) reduces to P, " P,


l = (2/5)ma .... (iii) (a) (b)
Fig. 7.76
Solving Eqns. (i) and (iii) for a and f, we get
Solution: (i) When truck accelerates along x-axis equation
a =~gsine and f =~mgsinB of translatory motion of a disc will be
7 7
F'-f=ma i.e., 9m-f=ma [asF'=ma T ] .... (i)
Now as static friction is self-adjusting, the sphere will roll
without sliding if While equation for rotational motion ofa disc will be
l<lL t = Ia i.e., fxr=~mr2(alr) i.e., 2f=ma ....(ii)
But as fL =J.iR =lJ.mg cosB
and f = (217) mg 'sin e So from Eqns. (i) and (ii)
~ ~

so ~mg sinB< I-lmg cose 3f=9m i.e., f=3x2=6N or. f=6i -N


(ii) Now as for wheels with centre 01 and 02 (b) When angular velocity of Y is to radls, the linear
-+ -+ -+ -+ -+ -+ -+ -+ velocity of a point on it and so of X will be
OP, = r, =-0.1 j - O.lk and OP2 =rz =0.1 j - O.lk v=nll=O.2 x 10 = 2 mls
-+ -+ -+ -+ -+ -+ -+ -+ Now as acc. of X is (g16) down the plane. the distance
so 't( =rlx r =(- 0.1 j-O.lk)x61 =-0.6 j+O.6k
moved by it, till it stops from 3rd equation of motion will be
-+ -+ -+ -+ -+
And '2 = r 2 x f =.(0.1 j-O.1 k) x 61 =-0.6 j-0.6 k
-+ -+ -+ 0 = 2 2 - 2(g/6)s
Le., s = (12/g) =1.22m
From this it is clear that 't I = 't' 2 :::: O.if2 N·m and makes
Alternative solution to part (b): When the block X
an angle of 45 0 with the rod as shown in Fig. 7.76.
moves up its translational KE and rotational KE of Y are
-J -+ -+ -+ converted into potential energy of X. So if X moves a distance
Note: Here, 't ='t
J +T 2 = - 1.2j N-m. i.e.,the resultant torque is of salong the plane, then by conservation of mechanical energy,
magnitude 1.2 N-m along negative y-axis. i.e., will rotate the
system anticloc:kwise and so the system will roll inside the truck, mgh =!nw 2 +.!./ai
i.e., . along negative x-a;t;is opposite 10 the direction of 2 2
acceleration ofthc truck.
or !ls.!.]=! x!(r(a)2 +.!.X [.!. X2r2](j,2
Problem 49. A block X vI v 2 ° l2 2 2 2 2
mass 0.5 kg is held by a long 2
or s=3x(O.2xl O) =12=1.22m
massless string on africtionless g g
inclined plane ofinclination 30°
to the horizon/aI, The siring is § 7.6 Angular Momentum and its Conservation*
wound on a uniform solid fA) Angular Momentum
cylindrical drom Y of mass 2 kg
Fig. 7.77 The angular momentum of a moving particle about a point
and radius 0.2 m as shown in
is defined as
Fig. 7.77. The drum is given an
initial angular velocity such that block X starts moving up the ....(i)
plane. (a) Find the tension in' the string during motion. (il)At a
certain instant oftime Ihe magnitude ofthe angular velocity of where pis the linear momentum of particle and -; its position
Y is I 0 rad s -I . Calculate the distance travelled by Xfrom that vector from the point.
instant. oftime until it comes to rest.
Regarding angular momentUm it is worth noting that:
Solution: (a) As X stops moving ·up, its acc. must be
(1) It is an axial·vector (i.e. , aiways perpendicular to the
downwards along the plane. So equation of motion of X will
plane of motion) having dimensions [ML 2T - I] and
be
Sl unit J·s (same as that of Planck's constant).
T - mgsin30 o=-ma,
i.e., T =~Gg-a) ....(i) L L

Now for rotational motion of cylinder


T xr =Ia p , 0
v
m
Butasa=a lr andhere I =!Mr2
2
(a) . (b)
so Txr=iMr2x(a lr), i.e.·, T= -1.x2a or T=a ... (ii) Fig. 7.78
2
Solving Eqns. (i) and (ii) for a and T, we get (2) As torque (= 1x F)
is defined as the ' moment of
a =K force' , angular momentum is also referred some times
6 as 'momel/t of iiI/ear momentum'.

and T= ~=9.8 = 1.63 N (3) In cartesian co.-ordinates angular momentum will be


6 6 given by:

~ Here we have limited ourselves to a particle but the results obtained are also valid for a rigid body.
~"""*"""* -+-+ ~ ~ about the axis of rotation. For angular velocity ro, it is
L=(rx p)=m(rx v) [as p =mv]
given by
c> ~ ~
L = fro
I j k
~
Fig. 7.79 (a) shows a disc rotating about its axis. The
i.e., L =m x y z
angular momentum is L "" fro.
vx vy v, Angular momentum of a body in non-centroidal
rotation: Fig. 7.79 (b) shows a disc rotating about an
axis perpendicular to disc but a distance h from the
symmetry axis. The angular momentum is
~
+ k(xv y - YV x )] .... (ii) L=LcM +'CM x MVCM
2
= IcMoo+ Mh w
(4) As the magnitude of angular momentum from Eqo. (i)
is: = l'ro
L = mvrsin9 I' is moment of inertia Axis of rotation
So, about the axis of ro~(!) eM
(a) It will be minimum (= 0). Whenlsin 91 = min = 0,
rotation. The angular c: < :::a b c::: P' <~
momentum about a n y ! !
~ ~
i.e.,a =co 0" or 180°, i.e., r and v are parallel or point in space 0 is the (a) (b)
antiparallel, i.e., if the point is on the fine of sum of the angular Centroidal Non-centroidal
motion angular momentum is minimum and zero. momentum about the rotation rotation
centre of mass plus thc Fig. 7.79
(b) It will be maximum (=co 11Wr) when Isin 81=co max ""
angular momentum
I, i.e.,e =co 90", i.e., angular momentum is
associated with centre of mass motion about the point
~ ~
maximum when r and v are orthogonal. This is 0'_
what actually happens in case of circular motion, (8) In case of rotational motion as
i.e., in case of circular motion of a particle
angular momentum is maximum and is mvr. L =/oo and

(5) As ILl =co mursin8 so if the point is not on the line of


motion, i.e.,9;;!:00 or 180 0 , L>O, i.e., a particle in ___ .(iv)
translatory motion always has an angular momentum
unless the point is on the line of motion. This result is rotational analogue of Kr =co p2/2m.
(6) In case of circular motion of a particle
(9) In case of circular motion for a particle
---)- ---)---) ~ -)­ ~ ~
L =co rxp =co m(rxv) [asp=comv] ~ ~
L =col ro
~ ~
i.e., L =co mvrsin 90 0 n [asS =901 ~ ~

so dL =col dro [as1 is constt.]


[where It is a unit vector perpendicular to the plane of dt dt
motion in accordance with right hand screw rule} ~

~ 2~ or dL = 1d
or L = mr ron [asv = roo] dt

L=/ro [asmr 2 = /andro"it= "fri] ....(iii)


or
i.e., in case ofcircular motion, angular momentum is or
~

dL ~
-=t
dt
-.
[as"t =fa]
~ ____ (v)
equal to the product of moment of inertia with
angular velocity. This result is rotational analogue of i.e., the rate ofchange ofangular momentum is equal
to the lIet (orque acting 011 the particle. This
~ ~
(p = mv). ~ ~
expression is rotational analogue of(dp / dt) = F and
(7) Angular momentum of a body in eentroidal
so also referred as Newton's 11 law for rotational
rotation: The body possesses the angular momentum
motion.
(10) Ifa large torque acts on a particle for a small time then
'angular impulse' oftlie torque is defined as ~v,
---)
Angular impulse J = 1'[ dt
---)

~
=;
-)-
't" a
. ~
"
t
I 2 dt
CM ,
J Y
h

, CM

or Angularimpulse='t av M = !J.L .... (vi)


~ ~
Fig. 7.80 After being struck by an Impulse the
[as t" = !J.LI!J.r] resulting motion can be thought of as the
superposition of translational and rotational
i.e., action of angular impulse is to change the motion about centre of mass
angular momentum. It has same unit, dimensions and
VCM "" VI' VCM ::o:hoo
direction as angular momentum. [See solved Problem
J net =hyJnet
59J
m I
(II) Like energy and charge, angular f!1omentum is also
I
quantised. * The qu<!ntisation of angular momentum y= CM
was first proposed by Bohr in order to explain the mh
spectrum of hydro£ ~n atom. According to him The point whose position is given by y is called centre of
angular momentum of an electron in an atom can be percussion.
only Problem 50. A rotating wheel is placed in contact with
ground. Atfirst the wheel remains stationary, spinning in its
L=II~ [where 11 is an integer]
2. place. After a short time it begins to move forward and
eventually reaches the pOint where it rolls. Find the final
However, later on it has been found that angular
velocity o/the wheel in terms ofthe initial angular velocity 000'
momentum of elementary particles (such as electron,
proton, etc.) is given by s(hI21t) where S '" 0, (II2) or Solution: From linear impulse momentum theorem
integer. The smallness of h(_ 10-33) makes +FfricM = MV - MVo (translational motion) ..... (i)
quantisation of angular momentum detectable only at From angul?r impulse angular
momentum equation
0(00L x
'atomic and subatomic levels.
§ 7.7 Centre of Percussion -RFfric M = loo-loo o vo",o
/00 - /00 0
Let us consider a bat which is stationary. The bat is struck F •. t.t ::: .... (ii)
IC -R
by an impulse J at a distance y from the cent(e of mass. From
Combining Eqns. (i) and (ii), Fig. 7.81
impulse~momentum equation, '
/00 - 100
J ::o: mv CM + 0 =MV -MV w
R o
Torque exerted by this impulse causes the bat to rotate
about the centre of mass. From angular impulse~angular MV = 100 _ 1000
momentum theorem R R
't!:J.t::o:f::,.L : O: /CMoo For pure rolling only V :::; Roo
Angular impulse is related to linear impulse through the or MV = /000 _ .IV
relation R R' W
't6t = Jnety Fig. 7.82
Let
yJ net 1 CMoo
v=(~
::0:

Any point on the bat has two velocities, the centre of I"Wo
l+kr
.... (iii)
mass velocity VCM and the tangcnti;li velocity VI caused due
to rotation about the centre of mass. As shown in Fig. 7.80, According to initial condition Rooo is initial tangenlial velocity
these two velocities are in opposite directions. If they are of imparted to wheel. Above fonnula tell us that the final
equal magnitude, the point x acquires no net velocity due to
impulse.
translational velocity of the wheel is the fraction (_k_)
1+ k
of

initial tangential velocity .

.. A physical quantity is said to be quantised if it t:an exist with certain discrete values such that all intennediate values are prohibited.
Question IV. A particle is moving along a straight line So if the net torque on a particle (or system) is zero
parallel to x·a.xis with constant velocity. Is its angular ~
momentum about the origin decreases with time or increases dL =0, i.e., L=conslant
with lime or remains constant? dl
y
Answer: Let a particle of i.e., angular momentum of a system (may be particle or body)
mass m is moving with speed v remains constant if resultant torque acting on it is zero. This
parallel to x-axis as shown in Fig. _ r,P,,-.. ,
_______rt:::=-"'
;/
principle is called 'Law ofconservation ofangular momentum '.
7.83. Then at any time t m
co-ordinate of P will be 1 _ i It is universal and holds good for both in relativistic limit

x =vt,y = b b
, '
! and quantum limit. No exception to this law has yet been
found. Regarding this law it is worth noting that:
and z=0 1 i (a) For a system consisting ofa single body rotating about
while components of velocity o"------~--, a fixed axis
will be Fig. 7.83
vx =V• v y = 0 and v z = 0
(as it is moving parallel to x-axis)
-7
• -.J k
~ ~ ~
So L=rxp = m v, b 0
v 0 0

~ ~ ~
i.e., L =mk[vt x O- vb] = - mvb k
(a)
i.e., angular momentum has magnitude mvb and is directed
along negative z-~xis ., i.e.; angular momentum remains
constant.
Question V. Show that if a particle moves in x-y plane,
the resultant angular momentum has only z-component.
Answer: By definition, angular mome~tum
~ ~ ~
i i k
~ ~ ~
L=rxp=m x y z
Vx vr v,
(b)
Now for motion in x~ yplane z = 0 and v z = 0

~
j
~
So L=m x y o = mk(xvy - YV x )
o
i. e., the resultant angular momentum has only z~component if
the ,motion is confined to x~y plane. This is tum implies that
angular momentum vector is always perpendicular to the
plane ofmotion.
[BJ Law of Conservation of Angular Momentum
(0)
Newton ' s 2nd law for rotational motion is
Angular velocity Is increased by changing posture from a position

l ciP -~l
~
of large moment of inertia to a position of small moment of inertia.
-dL
dt
=, ~
~ -- F
dt
Fig. 7.84
337
L=lw cylinder has a horizontal velocity Ve
Now ifno external torque is applied V Vb
and angular velocity co = .-£. Since N
IW=constt. r
So if I is changed by changing the distribution of mass cylinder is in pure rolling, the point of
w.r.t. axis of rotation, w will also change so that Iw remains contact is :nstantaneous centre of
constant. Decreasing the rotational inertia as a method of rotation. Just after impact point B B
increasing the angular velocity is quite familiar to dancers, becomes the new instantaneous centre.
mg
gymnasts, divers, acrobats and skaters (See Fig. 7.84). To In such problems during interval Fig. 7.86
understand it more clearly, suppose initially a person with his of impact very large impulsive forces
legs and anns stretched has some angular momentum. When act on cylinder at point B. In comparison to these forces
he pulls his anns and legs in, his moment of inertia about the gravity is neglected and assume that only impulsive reactions
axis through his centre of mass will decrease appreciably. are acting at point of impact. Since these forces pass through
Since angular momentum lwremains constant and I decreases, instantaneous centre, the moment of these forces vanishes. We
so angular velocity w will increase. conclude that angular momentum about impact point is
conserved.
Note: Here although L -/00 remains constant, duc to decrease in I,
Angular momentum before impact
kinetic energy KR == (L2/21) will increase. This increase in
kinetic energy is supplied from the work done by the am) and leg MR2 V.
muscles in pulling them in. L.I =--~+MV.cRcos.!..
2 R 'f .... (i)

(b) For a system of particles angular momentum Angular momentum after impact
-> -> -> MR2 V:
L=L j +L2 +····· Lf =-~.-£+MV.' R ....(ii)
2 R C
Hence, when the total angular momentum is conserved Equating Eqns. 0) and (ii), we get
-> ->
L 1+L 2 +····=constt. Vc = Vf (1+2cos41)
B
[BJ A slender prismatic bar AB of
length I and weight w is allowed to fall
vertically in an inclined position but
without rotation as shown in figure. If
the centre of gravity C of the bar has
velocity Ve at the instant when the end A 2A",,=:-=-c_ _
(b) strikes a smooth horizontal plane, we Fig. 7.87
Fig. 7.85 wish to find the new motion of the bar just after the impact
This shows that angular momentum of individual particles assuming no rebound at A.
may change, but their sum remains constant in the absence of Problem says that rod is dropped without rotation, i. e., it
an external torque. This is why in case of a two body system at has only translation.
rest if one part begins to rotate clockwise, the other part will Angular momentum of the rod about .A,
rotate anticlockwise, so that the resultant angular momentum
MVe l
may remain constant (= 0), e.g., in case of a spacecraft LA = - -cosa. .... (i)
provided with a flywheel (Fig. 7.85), if the flywheel begins to 2
spin in one direction, the spacecraft will rotate in opposite When the rod strikes at A, the reaction of the ground is
direction to conserve angular momentum. Now if the flywheel vertical. Note that ground is smooth. Point C will move in
stops, the rotation of the spacecraft will also stop (to conserve vertical direction with velocity Vc'Bar will also acquire an
angular momentum) but will have changed orientation. This is angular velocity co'.
how we change the orientation of a spacecraft in space. Angular momentum about A after impact
Some advanced applications of law of conservation of MV'I MI2
angular momentum L'-A = - 2_C- cos a + _12_ ",'
VJ ••••
(11.. )
[AJ Consider a solid right circular cylinder that rolls with On cquating Eqns. (i) and (ii), we get
constant velocity along a horizontal plane and suddenly strikes
an obstruction at B as shown in figure. Just before impact at B co' = ~(Vc -Vc)cosa.
338 PHYSICS FOR COMPETITIONS - Vo1. I

[C] A rigid lamina of T = Ar n , where A is a constant, ris the instantaneous radius of


arbitrary shape moves in its plane the circle and n.... tUT 1993)
with velocity components Vx and Answer: (a) By definition L = mvr
Vy of its centre of mass and
Butas v =rro =2nfj' [as 00 = 2:n:f)
angular velocity 0). If a certain
point 0 in the plate is suddenly
fixed by a pin. we wish to find the Fig. 7.88
so L=mvx(2~ )=~ [aSKE = ~mv2]
new angular velocity of the plate Now if the frequency is doubled and KE is halved the new
around the point. angular momentum will be
The angular momentum of plate just before 0 is fixed.
L z = M(rVy + k 2m) .... (i)
L' =KE' =(KE/2) =£
"I' 1f(2f) 4
where k is radius of gyration of plate w.r.t. axis passing (b) For circular motion ofstone(mv 2/r) == T .... (i)
through centre of gravity. Component of velocity V;c does not
appear in this equation because it has no moment arm. As soon
[asg=OJ
And as A.M. is constt. mvr = K. i.e., v = (KimI') .... (ii)
as point 0 is fiKed the large impulsive forces are generated al
O. The moment of these forces about 0 will be zero because Eliminating v between Eqns. (i) and (ii)
they pass through O. Hence angular momentum of the plate 2
will be conserved.
m[.E.]2 =T i.e., T=K r- 3
I'mI" m
L~ =J 20:l=M(k 2 +rz)ro' .... (jj)
0' .... (Hi)
V r+k2(j) comparing Eqn. (iii) with T =
Ar n we find n = - 3.
Fro91 Eqns. (i) and (ii), 00' ;=; -,y~---o;-

k 2 +r2 Problem 51. Two wheels A and C connected by a belt B


Question VI. A thin wheel can stay upright on its rim for as shown in Fig. 7.89. The radius ofC is three times the radius
a considerable length of time when rolled with a considerable ofA. What would be the ratio ofthe rotational inertias (I AlIc)
velocity, while il falls from its upright position at slightest if(a) both wheels have the same angular momentum (b) both
disturbance when ~tationary. Explain. wheels have the same rotational kinetic energy?
Answer: The rolling wheel has a certain angular
momentum. A torque perpendicular to the axis of rotation by
ch:mging direction of angular momentum will make the wheel p Q
fall. 'The larger the angular momentum of the wheel, the less
successful will be a given torque in changing the direction of
the axis of the rolling wheel, i.e., more stable will be the B
orientation. This is why the equilibrium of rolling wheel is Fig. 7.89
more stable than that of a stationary wheel (which is in Solution: As the belt does not slip v p = V Q
unstable eqUilibrium and falls at slightest disturbance).
i.e., rAffiA =rcffic [asv =rro] .... (i)
Note: This is also why before launching, a satellite is made to spin
According to given problem if rA = r, re = 3r.
about its ccntral axis. Otherwise orientation might be changed by
even a small external torque perhaps due to residual air resistance So that Eqn. (i) becomes
or radiation pressure of sun light. ffi,A =3ffiC .... (ii)

Question VII. Fill in the blanks-{a) A particle (a) If both the wheels have same angular momentum
performs uniform circular motion with an angular momentum IAffiA = Icffie
L. If the frequency of particle's motion is doubled and its IA ffiC 1
kinetic energy is halved, the angular momentum becomes ... 0' - =- =- [from Eqn. (ii)]
I e rolf 3
IMNR 19911
(b) Ifboth the wheels have same rotational kinetic energy
(b) A stone of mass m, tied to the end of a string. is
I 2
whirled around in a horizontal circle (Neglect the force due to '2/AroA = '12 Ieroc2
graVity). The length of the string is reduced gradually keeping
the angular momentum of the stone about tlte centre of the
circle constant. Then the tension in the string is given by
0' r
~; =[:: =[t]' =~
Problem 52. If the radius of the earth is reduced to half Problem 54. A thread is passing through a hole at the
of its present day value without change in its mass, what will centre ofa frictionless table. Atlhe upper end a block ofmass
be the length of the day? 0.5 kg is tied and a block ofmass 8.0kg is lied allhe lower end
Solution: If M is the mass of earth, R its radius and ro its which is jreely hanging. The smaller mass is rotated on the
spin angular velocity, the angular momentum of earth will be table with a constant angular velocity about the axis passing
through the hole so as to balance the heavier mlMs.lfthe mass
L, =fro ="52 MR 2co ' .... (i)
of the hanging block is changed from 8.0 kg to 1.0 kg. what is
the fractional change in the radius and the angular velocity of
Now when its radius becomes half without change in its the smaller mass so Ihat it balances the hanging mass again?
mass andro' spin angular velocity, the new angular momentum
Solution: For circular motionofa body tied to a string on
L, = ['co' = J,. M (Rfl)21if .... (ii) a horizontal plane
5
As no external torque is aCting, angular momentum must m
be conserved, i.e.,
L, =L,
which in the light ofEqns. (i) and (ii) yields
~MR2(J)
5
= !)(~MR2ro'
4 5 ,
i. e. ro'=4U'l
.
i.e.,angularve!ocitywill become 4 times ofi1S initial value so Fig. 7.90
T'
Tro'
=~ =.'.
4
[as T ",.'.]
ro
2
(mv /r) = T
Here as tension is provided by the hanging mass M, i. e.,
or T=Mg
so (mv'/r) = Mg
Problem 53. State whether the following s(aiemer;t is (me
or false giving reason in brief. According 10 given problem
A Jhin uniform circular disc of mass M and radius R is (mv~/rl) Mtg =~
rotating in a hon'zonlal plane aboul an axis passing through
ils centre and perpendicular to its plane wilh an angulaf (mv~ ' r2) M 2g I
velocityro. Another disc oflhe same dimension but ofmass MI4 2
VI r2 8
is placed gently all the first disc coaxially. The angular or - -=- .... (i)
2
velocity of the system now is'liiJl.J5. v 2r1
Solution: As moment of inertia of a disc through its centre Now as the force (T is central) so angular momentum is
of mass and perpendicular to its plane is (112)MR 2. its angular also conserved, i. e.,
momentum about the given axis when it is rotating with nrvlrl ::: ntU2r2 ....(ii)
angular velocity (I) will be So substituting the value of (vI' v 2 ) from Eqn. (ii) in 0)
L =fro =-1 MR 2 co
r, =~,I
.... (i)
2
[r,f, ]2 )( r2. i.e., r2=2 .... (iii)
When another disc of same radius but mass (MI4) is f,
placed on the first, the moment of inertia of the system
so that flr = r2 - rl = r2 - 1=2- 1=1
J'=!MR 2 +![M]R 2 =?.MR2 r rl rj
2 2 4 8
Furthermore as in circular motion v = rro
And so if 00' is the new angular velocity of the system
L' =J'oo' =?. MR 200,
8
.... (ii) so
[
as from Eqn. (ii) v 2 =
VI
1.]
f2
Now as no external torque has been applied, by
conservation of angulor momentum, we have or [as from Eqn. (iii) =;, =2]
L'=L, i.e., ~MR 2OO = ~MR2ro'
so
or 00' = (4/5) ro; so the given statement is false.
340 PHYSICS FOR COMPETITIONS - Vol. I

Problem 55. A satellite is pur in all orbit just above the are conserved in the collision? (b) What must be the mass of
earth's atmosphere with a velocity.J[5 times the velocityfor a the ball so that it remains at rest immediately after collision?
Circular orbit at that height. The initial velocity imparted is Solution: (a) (i) As external force M
horizontal. What would be the maximum distance 0/ the is zero so linear momentum is
satellite from the earth, when it is in the orbit? conserved. (ii) As torque is zero so
Solution: lfvI andv 2 arc s angular momentum is conserved. (iii)
As collision is elastic mechanical
the velocities of satellite at energy is also conserved. (KE after
closest point P and farthest P ~E A collision is equal to KE before
point A respectively, then as :W
• • collision). m v
force is celltral, angular L

__ .l-_~_-__-_-__-_-__~_-_-__-_____ ..J (b) By conservation of linear
Fig. 7.92
momentum will be conserved, r\ r2 momentum
i.e., Fig. 7.91
nw =mxO+MV .... (i)
VI r
0' - = 2- .... (i) By conservation of angular momentum
V2 rl
mud .= 16) ....(ii)
Also as gravitationalforce is cOllservative, mechanical energy By conservation of mechanical energy
is also conserved, i.e.,
1.mv 2=1.MV 2 +!1002 .... (iii)
1 2 GMml2GMm
- nwl - -- = -nw2 - - -
222
2 rl 2 r2 Substituting the value of V and 00 from Eqns. (i) and (ii)
respectively in (iii)

! mv2 =1. M [ mv ]2 + 1.1[mVd ]'


22M 2 I
Substituting the value of (v 2 !v l ) from Eqn. (i) in it 2
or 1=.E!. + md [as 1 = ML']

viH~J]=2GM[r'rl~,rl ] M (ML'1I2) 12
.... (ii)
m
ML'
0'
Now as according to given problem (L' + 12d')

VI = (vl.5)v o =~r.;(3;-;G"'M-;-;/::-
So Eqn. (ii) reduces to
2r-;-
l) [as Vo =~GM Ir] Problem 57.
uniform bar of length 6 a
and mass 8 m lies on a
A
r
:v c
3GM[;]
- -
2 fj
- rl' ] -2GM
ri
_ - -- rl ]
[r,
r l '2
Two point masses m and
2 m moving ill the same
horizOlltal plane with speeds
,a 'a,at
smooth hoNzolltal table. 1151H W"'lfr..thW ~""Zi"I.4:fH* ~ 1t< ~18m
I I ' " "

or 3(r2 +rl )=4r2 or r2 = 3rl


2v andv respectively, strike
So the maximum distance of the satellite from the centre
the bar (as showli in Fig. Fig. 7.93
of earth is 3R (with r l =R) and so maximum distancefrom the
7 .93) and stick to the bar after collision. Calclllate (a) velocity
surface will be3R -R = 2R.
ofthe centre ofmass (b) anglliar velocity about celltre ofmass
No te: In this problem for satellite as Fcxl DGMmt? '" 0. SO linear and (c)'total kinetic energy, just afler collision.
momentllm of th e satellite is 110t cOl/served. Further as torque is Solution: (a) As F ext = Dlinear momentum of the system
zero (as the force is central), angular momentum is conserved, is conserved, i.e.,
i.e., mvr = COIISIf. This in tum implies v 0: (!/r); so, kinetic
energy of sntellite [(1/2)mv 2] is not conserved. However, as -2m x v+ mx2v +0 = (2m + m +8m) x V
gravitational force is conservative, meeh:mical energy of satellite or V = 0 i.e., velocity of centre of mass is zero.
is conserved. (b) As 'text =0 angular momentum of the system is
Problem 56. A stick of length L and mass M lies 011 a conserved, i. e.,
frictiolliess horizontal suiface on which i/ is free /0 move in "'lv\r\ +m2v 2 r2 =(1\ +[2 +IB)ro
anyway. A balf ofmass m moving witlt speed v as shown in Fig. 2mva + m(2v)(2a) = [2m(a)2 + m(2a )2 + 8m x (60)2/ 12]00
7.92 collides elastically with the stick. (a) Which quantities
i.e., 6mva = 30ma 200 or C!)= (vI5a)
(e) As from part (1) and (2) it is clear thai, the system has
1/002 =I:J.U= O.08 Ix9.8 J
no translatory motion but only rotatory motion, 2

E= -1
2
!ro2=-
1
2
(30mo 2) V
-
Sa [ ]' =-3mv
2
5
Substituting the values of 1 and ro from Eqns. (ii) and (iii)
in the above, we have

Note: As initial KE oflhe system was- 1


2 x O.09x"3v[2 ]' = 0.08Ix9.8
E/ "'~ (2m)u2 + ~m(2V)2+0: 3mv2
v 2 =S.lx4.9, i.e., v = 6.3m1s
But as E '"' (3/5) mvl, there is loss ofKE (= 2.4~). Problem 59. A bllllet afmass m m u
f
~--~ ---,--~
So in this problem mechanical ellergy is nol consclVcd. moving with velocity u just grazes the
top of a solid cylinder of mass M and
Problem 58. Two uniform thin rods A and p
radius R resting all a rough horizolltal ·c
E, of length 0.6 In each and a/masses 0.01 kg suifaceas shown in Fig.7.95.Assuming
QlldO.02kg respectively, are rigidly joined, end that the cylinder rolls witholll slipping.
to end. The combination is pivoted at the lighter A A
find the angular velocity ofthe cylinder Fig. 7.95
end P as shown in Fig. 7.94 such that it can and the final velocity ofbullet.
freely rotate about the poim P in a vertical Solution: Let v and co be the velocity of bullet and angular
plane. A small object of mass 0.05 kg moving velocity of cylinder respectively. Applying conservation of
horizontally hits the lower end oj the B
angular moment about the point of contact of cylinder with
combination and slicks to it. What should be the
floor,
velocity of the object so that the system could m ,
~ mu·2R=mv·2R+(ICM +MR2)CO
just he raised to the horizontal position ? Fig. 7.94
I ,
Solution: As initially torque'" about P is zero, angular Here v = 2Rro and leM = -MR
2
momentum of the system is conserved, i.e.,
mv x 2L = lro .... (i) Substituting and solving, we get
V= Smu and ,,= _-,4"m",u,-_
Sm+3M (Sm+3M)R
Problem 60. A
homogeneous rod AB of
length L = I.S m and mass M
is pivoted at the centre 0 in
such a way that it can rotate

i.e., I =0.0720 +0.0168 +0.00 12 =0.09kg-m


. , .... (ii)
freely in the vertical plane
as shown in Fig. 7.96. The
So Eqn. (i) becomes rod is initially in the
0.05 x v x 1.2 =0.09 x 00, horizontal position. An Fig. 7.96
insect S of the same mass M
Le., ro=(2 / 3)v ... .(iii)
falls vertically with speed V all the point C, midway betweell
Now due to angular velocity 00 the system will rotate and the points 0 and B. Immediately after falling, the insect moves
kinetic energy ofrolation will be converted into PE. So gain in towards the end B such that the rod rotates with constant
PE when the system becomes horizontal, angular velocity 01 (a) Determine the velocity OJ in terms of V

I:J.U = mpghp + M8{LA + L; ]+ MAg L;


and L. (b) If the insect reaches the end B when the rod has
tllrned through an angle of900. determine V.
Solution: (a) Initially the insect has an angular momentum
i.e., /),U =O.05xl.2g+0.02xO.9g+0.0IxO.3g
about the point 0 and so when it comes in contact with the rod
= O.OSI x 9.8 J both will rotate aboutO. Ifro is the initial angular velocity, by
As this energy is provided by rotational KE, by conservation of angular momentum, i. e.,
conservation of mechanical energy (ane;' collision), we have MVr = lro
* As body is pivoted, i.e, not free to translate but free to rotate. In Ihis problem, in collision LM is not conserved but AM is conserved. Also as coUision is
inelastic, (KE) AC < (KE)gC' After collision whcn the rod rotates, neither LM nor AM remains constant but ME is conserved.
342

we have
M1~l =[~~' +Mm}
or MY ~ = }8 ML2ro i.e., ro=[,~ J~ ....(i)

(b) Since the weight of the insect will exert a torque, the mvo :::: MVCM .... (i)
angular momentum of the system will not be conserved. Let at •
any time t, the insect be at a distancex fromOon the rod and by mVOL =(ML2)ro .... (ii) P
.Jro
2 12
then the rod has rotated through an angle e. Then angular B
momentum ofthe system will be As the collision is elastic, so applying Fig. 7.97

J'f' :;. ML' + Mx 2


[12 1
0)
conservation of energy,
1 (ML2) ,
21 ntV o2 = "2MvC
I 2
M +"2 U 00
.... (iii)
so =2Mx dxro [asro = constt.given]
dJ .... (ii)
dl dl mvo
From Eqn. (i),
t =Mgx cos a = Mgx cos rot [as e = rot1
and .... (iii) vCM = - -
M
So the relation t =(dJldt) in the light ofEqns. (ii) and (iii) 6mv o
and from Eqn. (ii), ro=--
yields
ML'
Mgx cosrol =2Mx fix 0) Substituting in Eqn. (iii) and simplifying,
dl
.!!!. =!
or dx =[~}osoot dt M 4
(b) For the velocity of point P immediately after collision
According to given conditions, i. e., for x = L 14, t = 0 and w.r.t. centre of mass to be zero,
for.x = 112, t = 1t / 2ro[asrot = W21, the above equation becomes

I~! dx = [~]I~I2<O (cosrot)dt (x- ~ }O-V CM =0


i. e.,
U2 g .
[x l U 4 = - - [smrot]o
20>'
nl2w (x _ ~)6;;LO - :0 = 0, i.e., 2L
x=-3
(c) Angle turned by rod in
or I:. =J.....
4 2002'
i,e., ro=.{t lime [.!!!:...), B
Vo

A
3v o
'Substituting this value of ro iii. Eqn. (i),

fj- =I; ~ , i e, V= I~ ~2gL


6mvo
9=oo/ :::: - - x -
"TtL
ML 3vo
-2U3-
Fig. 7.98

So V =2 .J2 x lOx 1.8 -1::' 3.5 m ls


12 2
= ~(: } n = ~x~n=~
Problem 61. A rod AD ofmass M and length L is lying on a The rod takes the position shown in the Fig. 7.98. So, linear
horizolltal frictionless surface. A particle of mass m travelling velocity of P in y-direction
along the surface hits the end A ofthe rod with a velocityv o in a L L 6nwo Vo
direction perpendicular to AB. The collision is completely =-oo = -x - - = -
elastic. After the collision the particle comes to rest:
(a) Find the ratio ;;..

(b) A poim P on the rod is at rest immediately after the


So,speedofP =
6 6

V~M +(v: )' =


Vo
ML
r
4;-----,,__""""
(v: +(v:)'
collision. Find the distance AP. '
(c) Find the linear speed of the point P at time nU(3vo) =2.J2
after the collision.

• Here J is used for angular momentum as L is used for length of the rod.
MISCELLANEOUS SOLVED PROBLEMS

Problem 62. A particle of mass 3 kg is moving under the so from equation of translatory motion R
action of a central force whose potential is given by v = u + at, we have
U (r) = IOr 3 joule. For what energy and angular momentum v=v o - jJ.gt
will the orbit be a circle ofradius 10m? Calculate also the time [asu=v o anda= - Ilg] .... (i)
period of this motion. and from equation of rotatory motion
Solution: Given thatU(r) = 10,.3 and as F = - (dUldr), (0 = 000 + at, we have
ro =0+(5jJ.g/2r)t mg
F=-..'!.(10r 3 ) [aswo =Oanda =5jJ.gI2r] .... (ij) Fig. 7.99
dr
2 Now as in case of rolling without
= - 30r
sliding v = roo, so
So for circular motion, (vo - jJ.gt) = r(5IJ.g/2r)t, i.e., t = (2v0/7jJ.g) .... (iii)
mv 2
1F1=-=30r
2 Substituting the value of(t) from Eqn, (iii) in (i), we get
r
1 2 v=v o _jJ.g x
2vO
7~g
=vo[I - ~l=~vo
7 7
so KE=-mv
2
Note: (i) If during the period of transition from sliding to roi!ing, the
=l x 30r 3 = 15r 3 ball travels a distance s, then from equation of motion
2
v 2 "' 1I 2 +2as, we get (5vo/7l "' v~ -2 )lgs, i.e.,
i. e., KE=15x(l0)3 = 1.5 x 10 4 J s",(12v~/49)lg).
and PE=lOr 3 =10(10)3 = lx10 4 J (ii) If n is the number of revolutions made during the period of

so E = KE + PE=1 x 10 4 + 1.5 x 10 4 transition then from Eqn. of motion a", olaf + ~ al 2 ,


2
= 2.5xl04 J n = '(al2n) '" (al /4ft) (as Ole '" 0] or n '" (5v~/981t)lgr).

Further as (l/2)nw 2 = 1.5 x 10 4 ~


Problem 64. A umform
V = )(3x10 4/3)
= 100 m!s
disc ofmass m and radius R
is projected horizontally t - 0
3(d ~ D42& k~
t - to
with velocity Vo on a rough FIg. 7.100
so Angular momentum = mvr = 3 x 10 2 x 10 horizontal floor so that it
=3x10 3 Js starts offwith apurelysliding motion at t = 0. After to seconds,
it acquires a purely rolling motion as shown in Fig. 7,100.
and Time period T=2rtr =2rt x l0=E: s
v 100 5 (i) Calculate the velocity ofthe centre ofmass ofthe disc
at to '
Problem 63. A ball is thrown in such a way that it slides
with a speedvo initially without rolling. Prove that it will roll (iz)Assuming the coefficient offriction to be IJ., calculate to'
Also calculate the work done by the frictional force as a
without sliding when its speed falls to ~ vo' The transition function of time and the total work done by it over a time t
much longer that to'
from pure sliding to pure rolling is gradual so that both sliding
and rolling take place during this interval. Solution: With I = i Mr 2
Solution: Here the force of friction not only causes v to fr 2~g
decrease but also produces a torque which gives an angular so that a= -f =Ilg and 0. =-=--
m I r
acceleration a causing ro to increase.
As the force of friction f = IlR = 11 mg (as R = mg), the and hence v = Vo -lJ.gt and w=2j..lg t
r
deceleration of the ball a = f 1m = Ilg and as for rotational
motion of the baH "t = la, i.e., fr = 10. [as"t = fr] for rolling v = rw, so with t "" to above yields

or
fr5~g
ex. =j=2 r [asf=jJ.mgandI = Smr]
22 (Vo - jJ.gto) ={2~g}0 i.e.,
And substituting this value of to in v = Vo -Mgt (b) Now from equation of rotational motion,
va 2
v=v o -)lg-=-vO 0/ =005 +2ae
3~g 3
g
Now work done by frictional force will be Here 00 , =0+2[ 2 sintjlj e, 9= ,,',
.
r 4gsmtjl
W=K F-K j =[(~mv2 +~IO/ )-~mv~] So, work done in reaching angular velocity ro,
2
00 I' I
2 1(1 2)(2~g)2
2 2

[I ---;:- t 2-2'"v
1 o2] W=te=(Mgrsin~)x . ""- (Moo r )
Wo 2'"(va - ~gt) +2 2 mr 4gslll~ 4
(c) As in one rotation 21tr length of tape will unroll, so

w = m!lg{~!lgt-vo]
total length of tape unwound if the cylinder makes n rotations,
or

Total work done will be obtained by substituting t = to (as


L = 2nrn = 21tr x ..!
2n [asn
o
:n]
after this no work is done)

va -/lgx---vo
W=mj.lg--
3~g 2
va
3~g .
[3
=--mv o
6
112 So,

Problem 66. A rectangular rigid fixed block has a long


horizontal edge. A solid homogeneous cylfnder of radius r is
Note: Total work done by friction can also be evaluated as follows:
placed horizontally at rest with its length parallel to the edge
1 2 1 2 I 2
W ="2mv +"2/ro -Z mvo such that the axis ofthe cylinder and the edge of the block are
in the same vertical plane. There is sufficient friction present
W =~mv2+¥1mr2)(~;) - ~mv~ at the edge so that a very small displacement causes the
cylinder to roll of/the edge without slipping. Determine: (a)

~mv2 -1mv~ '" %mGvoy -imv~ '" -imv~


the angle 9 through which the cylinder rotates before it leave
i.e., W '" contact with the edge, (b) the speed afthe centre o/mass ofthe
cylinder before leaving contact with the edge and (c) the ratio
Problem 65. A tape is of translational to rotational kinetic energies of the cylinder
wrapped around a cylinder of when its centre afmass is in horizontal line with the edge.
mass M and radius r. The tape is Solution: When the
pulled as shown in Fig. 7.101 to cylinder rolls by an angle 9
prevent the centre of mass from about the edge its centre of
falling as the cylinder unwinds mass will go down by
the tape. (a) What is the angular h = r(1-cos9). So by
acceleration of the cylinder? (b) conservation of ME,
How much work has been done Fig. 7.101
mgr[l- cos9] = ~ mv2+~ /00
2
on the cylinder when it has reached an angular velocifyw? (c)
What is the length o/the tape unwound in this time?
Fig. 7.102
Solution: (a) As the ccntre of mass of the cylinder is not But here 1 = ~ mr2
changing its position there is no translational motion. 1fT is the
tension in the string, then for translational equilibrium of and v = roo
cylinder,
So, mgr[l - cos9] = .lmv 2 +.!x.lmr2(v 2Ir2)
T=Mgsintjl 2 2 2
and for rotational motion (i.e., spinning) of cylinder 2
i.e., gr[1-cos9]= (3/4)v .... (i)
t = la Now for circular motion of cylinder about the edge,
But here
mgcos9 -R = (mv2/r)
't=Txr=Mgsintjlxr and
The cylinder wiUleave contact when R 4- 0

a =.! = Mg sin tjl x r = =2g=sic.n.c~ i. e., gcose = (v 21r) .... (ii)


So
/ (l/2)Mr2 r
(a) Eliminating v 2 between Eqns. (i) and (ii) T= Mg ; a = _ -..c3"g"c-,o"s e;.-
g.-[I - cose]= (3/4)g,cose 1+3cos 2 e L[1+3cos 2 S]
i.e., e = cos - I(417) [- ve sign ofa shows that it is clockwise.]
(b) Now substituting this value of cos 8 in Eqn. (ii), Problem 68. A man ofmass 100 kg stands at the rim of a
v =.j (417)g, turntable 0/ radius 2 m, moment of inertia 4000kg_m 2
mounted on a verticalfrictional shaft at its centre. The whole
(c) At the time the cylinder leaves contact with the edge, system is initially at rest. The man now walks along the outer
K R =.!. Iro 2 =.! mv 2 = 1. mgr edge of the turntable with a velocity ofl mls relative to the
2 4 7 earth:
and then as T =O,a =0 and so ro :::: constt., i.e., after leaving the (a) With what angular velocity and in what direction does
I the turntable rotate?
edge K R = -::; mgr :::: constt.
(b) Through what angle will it have rotated when the man
So when centre of mass comes in horizontal line with the reaches his initial position on the turntable?
edge, by conservation of ME, (c) Through what angle will it have rotated when the man
I 6 reaches his initial positioll relative to earth?
mgr = K R +KT' i.e., KT = mgr- - mgr=-mgr
7 7 Solution: Let the man be moving anticlockwise.
KT = (617)mg, = 6 (a) By conservation of angular momentum on the
so mantable system
KR (117)mgr
L j =L or
Problem 67. A thin rod hangs/rom a ceiling by means of f w,
two inextensible cords.' Mass of the rod is M and length 2L. WI =-lmromIII
Rod is held at an angle8 with the horizontal. Ifthe string at the where ro = v/r=:l radls
right end breaks, determine the instantaneous angular m 2
acceleration of rod. ---- ,,
0), =-100(2)2 XG}4000
Solution: We assign x andy-axes "

as shown in figure. In this reference


frame let centre of mass has = _l.- radls
B 20
accelerations a x and a along x and
y-axes respectively. Th~ equations of
Thus, the table rotates clockwise FIg. 7.105

~
motion are (opposite to man) with angular velocity O.OS radls.
Since no force acts in x-direction (b) If the man completes one revolution relative to the
table, then:
,
I-F =0 .... (i) Fig. 7.103
8 ml =21t; 21t=9 m -8 1
:::::) ax = 0
21t "" (Om t - rolt (where t is the time taken)
Applying Newton's law in y-direction gives
t = 2Jt1(ro m -ro/)=2Jt1(O.S+0.OS)
"LFy =Ma y
Angular displacement of table is
T-Mg::::Ma y .... (ii)
T=O 9 t = wit = -O.OS x (21t10.SS)
LT=Ia
= -(21t11!) radian
-TLcos9=lea
The table rotates through 2n111 radians clockwise.
M(2L)2 ML2
where Ie = 12 =-3- .... (iii) (c) If the man completes one revolution relative to the
earth, then
From kinematics, acceleration of fY , 8 m =211:
eM (centre of mass) L- Time = 21t/wm =21t/O.S
ay = Lacose .... (iv) Fig. 7.104
During this time, angular displacement of the table
Substituting Eqn. (iv) in Eqn. (ii), we get
S t = w/(time) = -O.OS x (2tr10.S)
T - Mg= MLacosS
8 t = -rr/5 radian
and solving for T and a simultaneously, we get
8 t ;;::; 36° in clockwise direction
Problem 69. A thin uniform rod oflength L is initially at Applying the conservation of angular momentum about an
rest w.r.t. an inertial/rame a/reference. The rod is tapped at axis through C and perpendicular to the plane of the figure,
one end perpendicular to its length. How far does the centre of mvf+mvl=/ro where 1 =2m(f)2
mass translate while the rod completes one revolution about
ro=2mvl// = vii
its centre of mass? Neglect gravitational effect.
ro = 2013 rad/s

1 ~M
Solution: The impulse
(b) When the skaters reduce their separation no external
delivered perpendicular to the rod
torque acts on the system, hence we can apply conservation of
at one end gives some linear
angular momentum about CM. Due to redistribution of mass
momentum to the eM (centre of
about axis of rotation, moment of inertia changes.
mass) of the rod and also some Fig. 7.106
Since the separation reduces to 2f = I m,
angular momentum about the eM.
The rod will rotate about eM. Free rotation of any rigid body lro = I'ro' (conservation of angular momentum)
always takes place about eM. ro' =lrol I' =2mProl2ml,2 = 9ro = 60 rad/s
Translation of eM: Applying impulse-momentum equation, => angular velocity increases
EF ~ dPld, (J'ro' )ro' ro'
(c) KE IKE.
f
~lI'OJ"/1IOJ'
2 2
""':,:::- ~ - ~ 9
IFdt = M.6.v CM I (IOJ)00 OJ

JF dt = M(VCM -0) [initially eM is at rest] The kinetic energy increases because the skaters do positive
work in pulling themselves towards the centre of pole.
Rotation about the eM:
Problem 71. A uniform saUd
Ec=dL or cylinder of radius R = 15 cm rolls
d, over a horizontal plane passing
L ML' into an inclined plane forming an
-(Mv CM ) " ~(OJ - O)
angle a. = 30° with the horizontal N
2 12
(Fig. 7.108). Find the maximum
Let I1t be the time required for one complete revolution of
value of the velocity Vo which still - - -
21trad. Multiplying both sides by f1t gives permits the cylinder to roll on the //
[LI2]vCM'" ~ (L'112)oo", [':v eM '" ~ xl inclined plane section without a mg cos 0:
jump. The sliding is assumed to be mg
x ~L9/6
absent. Fig. 7.106
x ~ L1'i3 [9 ~ 2n",dl Solution: Situations of the cylinder just before and just
Problem 70. Two skaters, each of .i,===2~1===,. after passing on to the inclined plane without ajump are shown
T
mass 50 kg, approach each other along
parallel paths separated by3 m. They have
equal and opposite velocities of 10 mls.
1I: m,
in the figure.
By conservation of total energy, we have

The first skater carries a long light pole, 31mv

m long and the second skater grabs the '=>==~F=~'


A 1mv6 +1/ro 2 +mgR =lmv 2 +1/ro,2 +mgRcosa.
2 2 2 2
end ofit as he passes (assumefrictionless ~ t so,
Since the cylinder rolls without sliding,
va = Rro and v = Rro'
ice). (a) Describe quantitatively the Fig. 7.107
motion of the skaters after they are and also 1=~mR2
connected by the pole. (b) By pulling on the pole the skaters
reduce their distance apart to I m. What is their motion then? 3 2 3 2
Hence '4mvo +mgR ='4mv +mgRcosa. .... (i)
(c) Compare the KE's of the system in parts (a). Where does
the change comefrom? Considering the circular motion of centre of mass of the
Solution: (a) Initial linear momentum of the system cylinder about point 0, we get
(skater +pole) is zero; the centre of mass will be at rest before mgcosa - N=mv 2/R
and after the collision, as discussed in topic 'Centre of Mass' .
It will not jump so long as N 2 O.
The skaters and the pole will rotate around the centre of
mass (at the midpoint of the pole). Free rotation of a system or mgcosa. 2 mv 2/R
always takes place about CM. or gRcosa 2 v 2 .... (ii)
So, from equations (i) and (ii), we have Problem 73. An elastic spherical ball of mass M and
radius a moving with velocity v strikes a rigid surface at an
%mv~ + mgR:::; ~mgRcosa + mgRcosa angle S to the normal. Assuming it skids while in contact with
the surface, the tangential frictional force being a constant
or Vo :::;~~gR(7COSa - 4) fraction /J. of the normal reaction force, show that:
(a) The bali is reflected at an angie $ to the normal where
or (vO)max =..jgR(7cosa - 4) /3 {tane-tan~)=2~.
So, on putting the value of R,g and a, we get (b) The angular velocity of the rebounding ball changes
(vo)max = 1.0 mls 5~v
by an amount -case
Problem 72. A uniform cylinder of a
radius R is spinned ahout its axis with You may assume that the component of velocity
angular veJocitycoo and then placed into perpendicular to the surface is reversed in direction without
a corner (Fig. 7.109). The coefficient of change ofmagnitude.
friction between the corner walls and
the cylinder is equal to ).1. How many
1'-. Solution: (a) We cannot
assume that the ball is not

.
turns will the cylinder accomplish mg initially rotating because if the M
before it stops? Fig. 7.109 ball was not rotating, it could
not skid against the surface; so
Solution: The different forces acting on the cylinder are
shown in the figure . The initial kinetic energy of the cylinder is the angle e and $ would be
v
,
given by identical. Let us therefore
Fig. 7.110
assume that it has an initial
.... {i) angular velocity co! and a final angular velocity co 2 ' as shown
in Fig. 7.110.
Since there is no horizontal or vertical translation motion It is given that the perpendicular component of the
of the body, so velocity is reversed by the impact; so (with the notation of
N! +/J.N 2 =mg .... {ii) figure)
and N,=fLV l .... (iii) vcosS = ucos$ .... {i)
or, from equations (ii) and (iii), we get The change in the perpendicular component of the
mg momentum is 2Mvcos9, which is equal to the perpendicular
N! =- - .... {iv)
component of the impulse delivered to the ball by the surface.
1 + ).12
Since the frictional force is J..l times the perpendicular force, the
Now, if the cylinder performs n rotations before stopping, component of the impulse parallel to the surface must be2Mv/J.
then distance moved by this is 21tnR . Hence work done by cose.
friction till it stops is given by
If the ball is rotating clockwise as shown in the Fig. 7.110,
W={fLV l +fLV,)2.nR this impulse will be to the right, but it would be to the left if the
= (fLV l +~'Nl )2.nR ball were rotating anticlockwise.
The parallel component of the ball's momentum must thus
= fLV1{1+~)2.nR
change by 2Mv/J.cos9; so the parallel component of the
W = 2..R~(l+~)mg .... {v) velocity changes by 2v).1 cos 9. Thus,
1 +/J. 2
usin$ - vsin e = 2/J. vcos9 .... {ii)
So, applying Work-Energy Theorem to the equations (i) Equations (i) and (ii) give tan $ - tanS =2/J.
and (v), we get
but if the original angular velocity were antic10ckwise instead
- ..R~(l+~)mg
mR ' COo'/4_2 of clockwise, we could obtain
1 +/J. 2 tan/fl - tan9 =-2/J.
co~R(l+/J.2) so we may put Itane-tan$I=2/J.
or n=
8.~g{1 +~) (b) We have shown that the horizontal component of the
impulse delivered to the ball is2Mv/J.cos9 and since the ball's
radius is a the impulsive moment (angular impulse) is But, the mass and length of this part of the rod are reduced
2Mvllacos9. Since the impulsive moment is equal to the to half, so the moment of inertia is reduced to one-eighth of its
change in angular momentum and the angular momentum is 2
given by lro, where I is the moment of inertia, we must have fOffiler value, i. e., equal to mz~ .
/leo:=: 2~va cos €I The kinetic energy of this part of the rod is thus,
I
I ma 2 3g 1
= ,,2~=v:::a::c:::o::;-s,,9 ----=-mga
2 24 a 16
(2/5)Ma 2
If the rod now rotates through an angle 9, as shown in
=,-,5~=v~co::::scc9 figure, it3 centre of mass will rise through a distance
a
%(I- cos9); so it will gain potential energy
Problem 74. A uniform rod a/length a isfreely pivoted at
one end. It is initially held horizontally and then released from mga (I-cos9)
rest. What is the angular velocity at the instant when the rod is 8
vertical? When the rod is vertical it breaks at its midpoint? e
The maximum value of reached by this part of the rod
What is the largest angle from the vertical reached by the will occur when all the kinetic energy has been converted to
upper part a/the rod in its subsequent motion? Describe the potential energy. Thus
motion ofthe lower part a/the rod. (Assume that no impulsive
forces are generated when the rod breaks.) l~ mga =~mga(l-COSe)
Solution: Let m be the mass of the rod when it rotates
from a horizontal to a vertical position; the centre of mass falls l-COS9=~;
through a distance ~, so the loss of potential energy is m~a.
Hence
Since the rod is initially at rest, its kinetic energy as it passes
aJ2
T
through the vertical position must be given by mga from
. 2
cons.:rvation of energy. The moment of inertia of a rod of
rna 2 aJ2
t
length a and mass m about an end is - 3- ' hence

o
w=.J3gfa
Flg.7.112

Which gives
ro=J¥ Consider the lower half of the rod. At the instant the rod
breaks, shown in figure, the velocity of the upper end (OZa is to
According to condition of problem no impulsive forc"e acts
when the rod breaks, so that the angular velocity of the upper the left and the ve!ocityofthe lower end is (Oa also to the left.
Now velocity of centre of mass of this lower end of the rod will
part immediately after the break occurs is still
. f3i.
V-;; be 3roa to the left and an angular velocity (0 (clockwise) about
4

e
I
aJ2
its centre of mass. The centre of mass of the free fragment will
follow a parabolic path downwards and to the left, while it
rotates clockwise at a constant angular velocity.

Fig. 7.111
1
EXERCISE
[A) Only One Choice is Correct 9. Two particles P and Q are located at distances Yp and Y
Q
1. A mass m is moving with a constant velocity along a line
respectively from the centre of a rotating disc such that
Yp >Y : (EAMCETI992)
parallel to the x-axis, away from the origin. Its angular Q
momentum w.r.t. the origin : (lIT 1997) (a) Both P and Q have the same acceleration
(a) is zero (b) remains constant (b) Both P and Q do not have any acceleration
(c) goes on increasing (d) gocs on decreasing (c) P has greater acceleration thanQ
2. Two racing cars of masses m1 and m2 are moving in circles (d) Q has greater acceleration than P
of radii 'i and Y2 respectively; their speeds are such that they
each make a complete circle in the same length of time t. [Hint: a= ~a; + a? = ,.Jw4 +(1.
2
)
The ratio of the angular speed of the first to the second car 10. Two bodies of masses 10 kg and 5 kg are moving in
is: [MNR 1995) concentric orbits of radii r] and Y2 such that their periods are
(a) m]:m2 (b) Y, :Y2 same. The ratio of centripetal accelerations is :
(c) 1: 1 (d) m]'i: m'Y2 (a) 'i I Y2 (b) Y2 l'i "
[Hint:w :: 21tIT] (c) (YJ I r2 )3 (d) (r2 l'i )2
3. A particle P is moving in a circle of radius 'a' with unifonn
11. What happens to the centripetal acceleration of a revolving
speed v. C is the centre of the circle and AD is a diameter.
body if you double the orbital speed and halve the angular
The angular velocity of P about A and C are in the ratio:
velocity?
(a) 1:1 (b) 1:2
(a) Remains unchanged (b) Is halved
(c) 2: 1 (d) 4: 1
(c) Doubled (d) Quadrupled
[Hint: See § 7.1 (A) (b) (5)1
[Hint: ar = r002 = VO) (as v =: roo)]
4. A pulley one metre in diameter rotating at 600 revolutions a
minute is brought to rest in 80 sec by a constant force of 12. A body is moving in a circular path with a constant speed;
friction on its shaft. How many revolutions does it make its acceleration acts in a direction:
before coming to rest? (a) Along the tangent
(a) 200 (b) 300 (b) Along the circumference
(c) 400 (d) 500 (c) Along the radius towards the centre
5. The linear velocity of a particle on the equator is nearly (d) Along the radius away from the centre
(radius of the earth is 4000 miles) :
13. A motor cyclist going round in a circular track at constl.
(a) zero (b) 10 mile/hr
speed has:
(c) 100 mile/hr (d) 1000 milefhr (a) constant acceleration
6. A particle moves in a circle of radius 25 cm at two (b) constant velocity
revolution per sec. The acceleration of the particle in mls 2
(c) acceleration of constant magnitude with its direction
is: (MNR 19911
changing
(b) 8,' (d) constant force
(d) 2,' 14. For a particle in unifonn circular motion:
7. Certain neutron stars (extremely dense stars) are believed to (a) Both velocity and acceleration are constant
be rotating at about I rev/s. Ifsuch a star has a radius of20
(b) Acceleration and speed are constant but velocity
km, the acceleration of an object on the equator of the star
changes
will be :
(c) Both acceleration and velocity change
(a) 20x 103 mls 2 (b) l20x 10 3 mJs2
(d) Both acceleration and speed are constant
(c) 8x 10 5 mls 2 (d) 4x 10' """ 15. A particle is moving in a circle with uniform speed. It has:
8. A motor car is travelling at 60 mls on a circular road of
(a) constant kinetic energy
radius 1200 m. It is increasing its speed at the rate of 4 m/s 2 .
(b) constant acceleration
Thc acceleration of the car is:
(c) constant velocity
(a) 3 mls 2 (b) 4 Ill/s 2
Cd) constant displacement
(c) 5""" (d) 7m/s 2
16. A body whose speed is constant: 25. A particle moves in a circle of radius R with a constant
(a) Must always be accelerated speed under a centripetal force F. The work done in
(b) Might be accelerated completing a full circle is: ICPMT 1992, 931
(e) Must aiways be retarded (,) (Mu' / R)2rrR (b) rrR' F
(d) Must always have a constant velocity (e) 2rrRF (d) zero
17. Force in linear motion has its analogue in rotational 26. A piece of stone is being whirled in a circle in a horizontal
motion: plane with constant speed:
(a) Moment of inertia (b) Torque (a) Its kinetic energy goes on increasing
(e) Angular momentum (d) Weight (b) Its kinetic energy goes on decreasing
18. When a steady torque or couple acts on a body, the body: (c) Its kinetic energy remains constant
(a) continues in a state of rest or of unifonn motion by (d) Centripetal force will do work on it
Newton's 1st law 27. Railway tracks are banked at the curves so that:
(b) gets linear acceleration by Newton's 2nd law (a) The train may not fall down inwards
(e) gels an angular acceleration (b) The weight of the train may be reduced
(d) continues to rotate at a steady rate (c) The necessary centripetal force may be obtained from
19. A rigid body is rotating about an axis; to stop the rotation the horizontal component of the reaction of the train
we have to apply: (d) No frictional force may be produced between the
(a) Pressure (b) Force wheel and the track
(e) Torque (d) Momentum 28. An aeroplane is taking a tum in a horizontal plane:
20. In order to cause a body to move along a circular path we (a) It remains horizontal
must supply: (b) It inclines inwards
(a) Inertial force (b) Gravitational force (c) It inclines outwards
(e) Centripetal force (d) Centrifugal force (d) Its wings become vertical
21. Which of the following is the evidence to show that there 29. At a curved path of a road, the road bed is raised a little on
must be a force acting on the earth and directed towards the the side away from the centre of the curved path; the slope
sun? of the road bed is given by:
(a) Deviation of the falling bodies towards east 2
(a) tan9 = vg/r (b) tan9 "" v !rg
(b) Revolution of earth round the sun
(c) tan9=rg!v 2 (d) tan e=v2 r/ g
(c) Phenomenon of day and night
30. A cyclist turns around a curve at 40 kmlhr. If it rounds the
(d) Apparent motion of sun round the earth
curve at double the speed, its tendency to overturn is :
22. A satellite has mass m, speed v and radius of circular orbit r.
(a) Doubled (b) Tripled
The force F acting on it is :
2 (c) Halved (d) Quadrupled
(a) zero (b) mrv
31. A person with his hands in his pocket is skating on ice at the
(c) mr2 Iv (d) I1W2/r rate of 10 mls and describes a circle of radius 50 m. What is
23. Two particles of equal masses are revolving in circular his inclination to the vertical: (g = lOm / s2)
paths of radii r, and r2 respectively with the same speed.
(EAMCET 19901
The ratio of their centripetal force is:
(,) tan -I (1/2) (b) tan - I (1/5)
(a) r2 / Ij (b) ~r2 / r,
(c) tan - I (3/5) (d) tan - I (1110)
(c) (r, / r2 )2 (d) (r2 /Ij )2 32. A motor cyclist moving with a velocity of 72 km per hour
24. When a body moves with a constant speed along a circle: on a flat road takes a turn on the road at a point where the
(CBSE 19941 radius of curvature of the road is 20 m. The acceleration due
to gravity is 10 mls 2 . In order to avoid skidding, he must
(a) Its velocity remains const!.
bend w.r.t. the vertical plane by an angle 9 greater than:
(b) No force acts on it
(a) 8=tan- 1 (6) (b) 6 = t,n- 1 (2)
(c) No work is done on it
(d) No acceleration is produced in it (c) 9=tan- 1 (25.92) (d) 9=tan- 1 (4)
33. A car is moving in a circular horizontal track of radius 10 m (a) At the top of the circle
with a constant speed of 10 mls. A plumb bob is suspended (b) At the bottom of tile circle
from the roof of the car by a light rigid rod of length 1.0 m.
(c) Halfway down
The angle made by the rod with the track is: lilT 1992J
(d) None ofthe above
(a) zero (b) 30°
41. A stone is tied at one end of a 5 m long string and whirled in
(0) 450 (d) 600 a vertical circle. The minimum speed at the top required to
34. A motor cyclist rides around the well with a round vertical just cross this position is:
wall and does not fall down while riding because: (a) 5 mls (b) sJ7 mls
(a) The force of gravity disappcars
(0) 7 m/s (d) 7.Js mls
(b) He loses weight some how
42. In the above problem 41 the speed of stone at the lowest
(c) He is kept in this path due to the force exerted by position is:
surrounding air
(a) 5 mls (b) sJ7m1s
(d) The frictional force of the wall balances his weight
(0) 7 mls (d) 7.Js mls
35. A toy car travels in a horizontal circle of radius la, kept on
the track by a radial elastk string. Its period is T. The length 43. A stone is whirled in a vertical circle. In what position ofthe
of the unstretched string if; found to be a. When the car is stone is the tension in the string maximum?
speeded up, the string stretches until the car is moving in a (a) In the position when the string is horizontal
circle of radius 3a. Assuming that the string obeys Hooke's (b) In the highest position of the stone
law, the period of revolution is:
(c) In the lowest position of the stone
(a) T(3/4) (b) T5i4 (d) The tension remains same at all positions
(0) T.J4i3 (d) T 44. A stone of mass mtied to a string of length Lis whirled in a
36. The time period of a conical pendulum shown in Fig. 7.113 vertical circle. If the string remains just stretched when the
is: stone is at the top of the circle, the tension in the string when
"~
, , the stone is at the bottom of the circle is:
(a) T=2~
,/ e\, (a) 3 mg (b) 6 mg
(b) T=2,.,j(Loos9Ig) ,
,, / \L~
,,
,, (c) mg (d) 2 mg
(0) T = 2,.,j(Lsin9/g) ,, , ,, 45. The string of a simple pendulum is horizontal. Now it is
m released. The tension in the string in the lowest position is :
(d) T = 2,.,j(Ltan9/g)
[Hint: See § 7.2 (C)] Fig. 7.113 (,) 2mg (b) 6mg
(0) 3 mg (d) 4mg
37. If a body describes a circular motion under inverse square
field, the time taken to complete one revolution T is related 46. In a circus a rider rides in a circular track of radius R in the
to the radius of the circular orbit r as : vertical plane. The minimum velocity at the highest point of
the track will be:
(a) Tocr (b) Tocr 2
(a) ~2gR (b) 2gR
(c)T 2 ocrJ (d) Tocr 4
38. In an alom two electrons move round the nucleus in a (0) )3gR (d) .Jiiii
circular orbits of radii Rand 4R respectively. The ratio of 47. A particle rests on the top ofa hemisphere ofradiusR. Find
the time taken by them to complete one revolution is : the smallest horizontal velocity that must be imparted to the
(,) 114 (b) 4/1 particle if it is to leave the hemisphere without sliding
(0) 8/ 1 (d) 1/8 down:
39. A body of mass m attached to a thread is revolved along a (a) Ii- (b) .p.g;
vertical circle of radius r. If its velocity at the topmost point
of the circle is v, the tension of the thread at the instant will
(0) fig; (d) f5Y
be: 48. When a simple pendulum is rotated in a vertical plane with
constant angular velocity, centripetal force is :
(a) mg-mv 2 r (b) mv 2 1 r
IEAMCET 19901
(c) (mv 2 / r)-mg 2
(d) (mv 1 r)+mg (a) maximum at highest point
40. A 2 kg stone at the end of a string 1 m long is whirled in a (b) maximum at lowest point
vertical circle. At some point its speed is 4 mls. The tension
(c) same at all points
of the string is 51.6 newton. At this instant the stone is:
(d) zero
49. A stone of mass mis tied to a stringoflength Land rotated in (a) A disc of radius a
a circle with a constant speed v; if the string is released the (b) A ring of radius a
stone flies: (c) A square lamina of side 2a
(a) Radially outward (d) Four rods forming a square of side 2a
(b) Radia!1y inward 58. The moment of inertia of a thin unifonn eireular disc about
(e) Tangentially one of the diameters is I. Its moment of inertia about an axis
Cd) With an acceleration /tW
21L perpendicular to the plane of the disc and passing through
its centre is: ICBSE 1992; EAMCET 19931
50. Moment of inertia comes into play:
(a) (.fi)l (b) 2J
(a) In translatory motion
(b) In rotatory motion (0) II 2 (d) II.fi
(e) In vibratory motion 59. Two rings of the same radius rand mass mare placed sueh
that their centres arc at a common point and their planes arc
(d) When the body is permanently at rest
perpendicular to each other. The moment of inertia of the
51. Momcnt of inertia plays the same role in rotatory motion as system about an axis passing through the eentre and
in translatory motion is played by: perpendicular to the plane of one of the rings is :
(a) Velocity (b) Acceleration [PMT (MP) 19941
(e) Mass Cd) Force (a) (1/2) mr2 (b) mr2
52. The dimensions of moment of inertia arc: (c) (3/2) mr2 (d) 2 mr2
(a) [M i L2 T- i ] (b) [MoL- 2TO] 60. If the moment of inertia ofa rod of mass M and length L
(0) [MoL2T2] (d) [M i L2 To] about an axis passing through its centre of mass and
perpendicular to its length is (ML2 /12\ thcn moment of
53. The moment of inertia of a body docs not depend on:
inertia of the same rod about an axis passing through one of
(a) The mass of the body its ends and parallel to the given axis will be:
(b) The angular velocity of the body (a) (ML2 112) (b) (ML2/6)
(e) The axis of rotation of the body 2
(0) (ML2/3) (d) (13/12)ML
(d) The distribution of the mass in the body
61. A ring of radius r and mass m rotates about an axis passing
54. Four masses are fixed on a massless rod as shown in Fig.
through its centre and perpendicular to its plane with
7.114. The moment of inertia about the axis P is about:
angular velocity (U Its kinetic energy is: (CBSE 1992)
u:> 1 2
(a) mrco (b) - mrco
2kg 'kg p 'kg 2kg 2
O.2m _ _ O.2m_
-O.2m-........ O.2m
(d) 1. mr2(U2
2
Fig. 7.114 62. A wheel of mass 2 kg, having practically all the mass
concentrated along the circumference of a circle of radius
2
(a) 2kgm (b) 1 kgm 2 20 cm, is rotating on its axis with an angular velocity of I 00
(c) 0.5 kg 10
2
(d) 0.3 kg m 2 radls. The rotational kinetic energy of the wheel is:
55. The moment of inertia of a ring of mass mand radius rabout (a) 4 joule (b) 70 joule
an axis, passing through the eentre and perpendicular to the (c) 400joule (d) 800 joule
plane of the ring, is: 63. The moment of inertia of a wheel about the axis of rotation
(a) (1/2) mr2 (b) mr2 is 3.1 MKS units. Its kinetic energy will be 600 joule if the
period of rotation is :
(c) (1 /4) mr2 (d) (3/4) m,·2
(a) 0.05 s (b) 0.3148
56. Which of the following has the smallest moment of inertia
(0) 3.IS8 (d)
about the central axis if all have equal masses and radii?
64. A spherical solid ball of mass 1 kg and radius 3 em is
(a) Ring (b) Disc rotating about an axis passing through its centre with an
(e) Spherical shell (d) Sphere angular velocity of 50 radls. The kinetic energy of rotation
57. For the same total mass which of the following will have the IS:

largest moment of inertia about an axis passing through the (a) 4500 J (b) 90 J
centre of mass and perpendicular to the plane of the body? (0) (9/S)J (d) (9/20) J
65. A metal ring of mass 2.1 kg and of 10 cm radius is revolving (a) The cylinder will reach the bottom first
about its axis, making lOOltt revolution/so If this ring is (b) The disc will reach the bottom first
dropped in a viscous liquid the heat generated in the liquid
(c) The sphere will reach the bottom first
is equal to: (J =4.2joule/cal)
(d) All will reach the bottom at the same time
(a) 100 cal (b) 1000 cal
74. One hollow and one solid cylinder of the same outer radius
(c) 2100 cal (d) IOkeal
rolls down on a smooth inclined plane. The foot of the
66. A rod of length L is hinged from one end. It is brought to a inclined plane is reached by:
horizontal position and released. The angular velocity of
(a) Solid cylinder earlier
the rod when it is in vertical position is: lIlT 19901
(b) Hollow cylinder earlier
(a) .j2gIL (b) .j3g/L
(c) Simultaneously
(e) .jglU (d) .jg/L
(d) The heavier earlier irrespective of being solid or hollow
67. A metre stick is held vertically with one end on the floor 75. A solid sphere rolls down two different inclined planes of
and is allowed to fall. The speed of the other end when it the same height but of different inclinations:
hits the floor assuming that the end at the floor does not slip (a) In both cases the speeds and time of descent will be
2
andg=9.8m/s is: same
(a) 3.2 mls (b) 5.4m1s (b) The speeds will be same but time of descent will be
(e) 7.6 mls (d) 9.2 mls different
[Hint: See solved Problem 40] (c) The speeds will be different but time of descent will be
68. When a sphere of moment of inertia 1 about an axis through same
ils centre of mass and of mass m rolls from rest down an (d) Speeds and time of descent both will be different
inelined plane without slipping, its KE is: 76. Two identical solid cylinders run a race starting from rest at
I 2 1 2 the top of an inclined plane. If one cylinder slides and the
(a) - 100 (b) - mv other rolls:
2 2
1 2 I 2 (a) The sliding cylinder will reach the bottom first with
(c) 100+ mv (d) -1ro +- mv greater speed
2 2
(b) The rolling cylinder will reach the bottom first with
69. A ring of I kg mass and I m radius is moving forward with
greater speed
velocity I mls by rolling without slipping in a horizontal
plane. Its kinetic energy would be : (c) Both will reach the bottom simultaneously with the
(a) (1/2) joule (b) I joule same speed
(d) Both will reach the bottom simultaneously but with
(c) 2 joule (d) 4 joule
different speeds
70. If a spherical ball rolls on a table without slipping, the
fract ion of its total energy associated with rotation is :
77. Angular momentum of a body is defined as the product of:
(eDSE 1994( (a) Mass and angular velocity

(a) 3/5 (b) 217 (b) Centripetal force and radius


(c) 2/5 (d) 317 (c) Linear velocity and angular velocity
71. The speed of a homogeneous solid sphere after rolling (d) Moment of inertia and angular velocity
down an inclined plane of vertical height II, from rest, 78. The dimensional formula for angular momentum is:
without sliding is: [CBSE 19921
(a) [ML'Y-'] (b) [M ' L'Y- ' ]
(a) Iii; (b) )(g 15)gh
(c) (ML- 'y-2] (d) [ML'Y-' ]
(c) .j(4 / 3)gh (d) .j(IOI7)gh
79. Angular momentum is: (eDSE 1993(
72. An inclined plane makes an angle of 300 with the
(a) A polar vcctor (b) An axial vector
horizontal. A ring rolling down this inclined plane from rest
without slipping has a linear acceleration equal to : (c) A scalar (d) None of these
(eDSE 1992( 80. A fan of moment of inertia 0.6 kg m 2 is to be run upto a
(a) 2g13 (b) gl2 working speed of 0.5 rcv. per sec. Indicate the correct value
of the angular momentum of the fan:
(c) g/3 (d) gl4
2
73. Ifa solid sphere, disc and cylinder are allowed to roll down (a) (1t 1D.6)kgm 2/s (b) 6kgm /s
an inclined plane from the same height: (CBSE 1993J
81. A constant torque acting on a unifonn circular wheel 90. A thin circular ring of mass M and radius ris rotating aboll
changes its angular momentum from Ao to 4Ao in 4 sec; its axis with a constant angular velocity til Two object.!
the magnitude of this torque is: each of mass m, are attached gently to the opposite ends of
diameter of the ring. The wheel now rotates with an angula
(a) 4 Ao (b) Ao
velocity :
(e) 3Ao/4 (d) 12 Ao
(a) roM/ (M +m) (b) ro(M-2m)/(M+2m)
82. When torque applied on a system is zero which of the
(e) ooM/(M+2m) (d) oo(M+2m)/M
following will be constant?
91. A disc of radius b and mass m rolls down an inclined plan
(a) Force (b) Linear momentum
of vertical height h. The translational speed when it reache
(c) Impulse (d) Angular momentum .the bottom of the plane will be :
83. When there is no external torque acting on a body rotating
elliptically, which of the following quantities will remain (a) 4~h (b) .j2gh (e) t;h (d) ~5~h
constant?
92. A uniform rod oflength I, hinged at the lower end is free tl
(a') Kinetic energy (b) Potential energy
rotate in the vertical plane. If the rod is held vertically in tho
(c) Linear momentum (d) Angular rno.mentum beginning and then released, the angular acceleration ofth'
84. A.hoy is standing on a rotating table with heavy spheres in rod when it makes an angle of 45° with the horizontal is:
his extended hands. Ifhe suddenly withdraws his hands to 3 g 6 g
his chest the angular velocity of the table: (a) 2.[i I (b) .[i I
(a) Becomes zero (b) Increases
(e) .[iiI (d) 2g
(c) Decreases (d) Remains unchanged 1 1
85. A dancer (gymnast) is taking turns with her arms and legs 93. A table tennis bat has a
stretched. When she pulls her arms and legs in : flat rectangular handle
(a) The angular velocity decreases of mass m and length i ~_ _ _ _-1
(b) Her moment of inertia decreases and a flat circular disc 1 - - 1
(c) The angular velocity stays constant of radius r and mass m
attached to the handle.
(d) The angular momentum increases Fig. 7.115
The moment of inertia
86. A uniform heavy disc is rotating at constant angular of the bat about an axis perpendicular to its plane anI
velocity about a vertical axis through its centre; some wax passing through its centre of mass is :
is dropped gently on the disc; the angular velocity of the
mi 2 mr2 ml2 3mr 2
disc: (a) - + - (b) - +- -
(a) Does not change
12 2 3 2
(b) Increases
(c) Decreases (d) Becomes zero
87. If a running boy jumps on a rotating table which of the
following is conserved? 94. Out of the two eggs, both equal in weight and identical it
(a) Linear momentum (b) KE shape and size, one is raw and the other is half boiled. Thl
(c) Angular momentum (d) None of these ratio between the moment of inertia of raw to boiled one
about a central axis, will be :
88. A solid homogeneous sphere is moving on a rough
horizontal surface, partly rolling and partly sliding. During (a) one (b) greaterthanone
this kind of motion of the sphere : leBSE 19921 (c) less than one (d) none of these
(a) Total kinetic energy is conserved
(b) Angular momentum of the sphere about the point of
contact with the plane is conserved
95. A cylindrical dmm is F - I -
pushed along by a board - I
of length I. The drum
rolls forward on the
-~
8 '/--"
2 \ ,-- ,-
(e) Only the rotational kinetic energy about the centre of
ground a distance of ~.
Fig. 7.116
mass is conserved 2
(d) Angular momentum about centre of mass is conserved There is no slipping at any instant. During the process 0
89. If the radius of earth contracts to half of its present day pushing the board, the distance movcd by the man on tht
value, the length of the day will be: ground is :
(3) 48 hr (b) 24 he 1 31
(a) - (b) 4
(e) 12 hr (d) 6 hr 2
(e) 1 (d) none of these
96. The speed of a particle moving in a circle slows down at a 102. A point is moving in a circle. The ratio of its angular
2
rate of2 m/sec . At some instant the magnitude of the total velocity about a point on the circumference ofthe circle and
acceleration is 4 m/sec 2 and the particle speed is 12 mfsec. about the centre of the circle is:
The radius of circle will be: (a) 1,2 (b) 2, I
(a) r = 21 m (b) r = 42 m (c) 1t: 1 (d) I "
(c) r = iO.5m (d) ,=48 m 103. For the toppling of the shown Fig. 7.119
97. A unifonn, massless, long brass regular hexagon. The coefficient of friction
tube has two identical marbles at must be:
onc end. The other end is pivoted to Flg.7.119
(a) > 0.21 (b) < 0.21
a fixed horizontal bar. The time
m (c) = 0.21 (d) <0.21
period of oscillation of the tube for
this arrangement is found to be TI • 104. An annular ring with inner and outer radii R, and R2 is
m
when it oscillates in the vertical rolling without slipping with unifonn angular speed. The
m m
plane. One of the marbles is then 'G '--' ratio of the forces experienced by the two particles situated
moved to the center of the tube and Fig. 7.117 on the inner and outer parts of the ring F j lF2 is:
the time period of oscillation becomes T2 . The ratio TJ / Tz
IAIEEE 2005]
is :

(a) I
(a) I (b) !i
R,
2

(c)
4
(d) (6)"'
-
5
(d) [ : ; ) '

9S. A point moves along a circle with velocity v =at. The total 105. Two rings of radius Rand nR made up of same material
acceleration of the point at a time when it has traced I18th of have the ratio of moment of inertia about an axis passing
the circumference is: through centre is 1 : 8. The value of n is : (DCE 2005)
(a) ~ (b) "'~4 . .2 (a) 2 (b) 2J'i
Sa
(c) 4 (d) ~
(c) a (d) ". ~ 4 . . 2 2
2
106. The angular momentum of a rotating body changes from
99. The kinetic energy T of a particle of mass m moving in a Ao to 4Ao in 4 sec. The torque acting on the body is :
circle of radius r varies with the distance traced, S as
IBHU2005]
T = KS 2. The tangential acceleration of the particle is:
}(S ' (b) 4Ao
(a) (b) 2KS
m m
}(S' 2KS ' (c) 3Ao (d) ~2 A 0
(c) (d)
3m r 107. The moment of inertia of a uniform circular disc of radius
100. The winding radius of a tape on a cassette was reduced to 'R' and mass' M' about an axis touching the disc at its
half in 20 minute of operation. In what time the winding diameter and nonnal to the disc is: (CBSE 2006)
radius be reduced to halfagain?
(a) 15 minute (b) 10 minute (a) MR' (b) ~MR2
5
(c) 5 minute (d) 20 minute
101. A ball rotating with angular velocity (J) ~I\
(d) ~MR'
2
about its centre of mass strikes a rough v 1m
horizontal surfacc as shown in the Fig. lOS. Four point masses, each of value m, are placed at the comers
of a square ABeD of side i. The moment of inertia of this
7.118. Aftcr the collision the path of _,~g.(J_'~24~'.~(!~',,!,£1
centre of mass of ball will be: Rough system about an axis passing through A and parallel to ED
(a) A parabola to the right Fig. 7.118 is: (AIEEE 2006)

(b) A parabola to the left (a) 3 ml2 (b) ml'


(c) A straight line (c) 2 ml 2 (d) .fj ml2
Cd) A circle
109. Moment of inertia of a triangular lamina about its base is 113. Point C is the centre of the
(assume M is mass, h is altitude and b is base) : rigid body shown in Fig.
~
,~~. c /
,_\ ____ .' 5 cm
7.124. Find the total torque
acting on the body about "'~ 10cm \~5cm
,,
point C. (CW-clockwise,
ACW-anti clockwise) 10 N
(a) 1.71 NmCW Fig. 7.124
b
,Fig. 7.120 (b) 2.71 Nm ACW
(c) 270NmCW
Mb' Mh'
(a) --
6
(b) --
6
(d) 2.71 Nm CW

~
114. A uniform rod is kept on a
M(h' +b') Mbh frictionless horizontal table and
(c) 6 (d) - -
~
6 two forces FI and F2 are acted as
110. The moment of inertia of a 0 shown in Fig. 7.125, The line of F,
uniform rod about an axis actionofforceFR (which produces Fig. 7.125
passing through its centre and U2 U2 same torque) is ~t a perpendicular
perpendicular to its length is J. distance 'C' from O. Now FI and F2 are interchanged and
If the rod is broken into two FI is reversed. The new force FR (which produces same
equal parts kept as shown, then Fig. 7.121 torque in the present case) has its line of action at a distance
moment of inertia of system C
- fromO. IftheFR :FR is in the ratio 2: 1, then a: bis
about an axis passing through its point 0 and angular 2 ' ,
bisector of rods is : (assume F 2 a > FJb) :
I
(a) - (b) ~ 2F2 -FI F2 + 4FJ
2 8 (a) (b)
4F, F, 4F2 - F J
I
(c) - (d) ~ F2 - 3FI F2 +FI
32 4 (c) (d)
Ill. A solid sphere of radius R has moment F J + F2 2F2 +3FI
of inertia I about its diameter. It is 115. A wire of uniform linear density is bent into a circular wire
melted into a disc of radius rand , of radius r. Its moment of inertia about an axis along its
thickness t. If its moment of inertia tangent and perpendicular to its plane is II' Now same
ahout the tangential axis (which is material is used to form a square of side Q. Its moment 01
perpendicular to plane of the disc), is Fig. 7.122 inertia about an axis passing through one of the sides is J2'
also equal to I, then the value of rand t
Then I I : 12 is:
are (respectively) :
2 2 12nr 3r 3n
(a) (b)
(a) M R,5R (b) .[5 R,2R Sa Sa'
3 R l2>·6 n 12r3n
(c) - R , - (d)t RIi (c)
4
(d)
M2 15 ' 5 a Sa'

Bn'h
112. Figure shows a uniform rod' of length I 116. A thin rod of length 4/, mass 4111 is b(:nt at the points as
and mass M which is pivoted at end A shown in Ihe Fig. 7.126. What is the moment of inertia of
such that it can rotate in vertical plane. the rod about the axis passing point 0 and perpendicular to

A~
The free end of rod B is initially vertically the plane of the paper:
above the point and then released. As the o
rod rotates about A; its angular
acceleration and tangential acceleration Fig. 7.123
of centre of rod are (respectively) when it is inclined to
Fig. 7.126
horizontal at angle €I :
I €I 3gcos9 3g 3gcos €I ml' 10m/ 2
(a) 3gcos,
f\
4 (b) ~cOSu (a) (b) - -
21 ' 4 3 3
(c) 5gsin9 5gsin9 ml' ml'
(d) !sine 3gsin9 (c) (d)
41 ' I I ' 4 12 24
117. In the pulley system shown, if radii of the 122. A unifonn rod of mass M and length L lies
bigger and smaller pulley arc 2 m and 1m radially on a disc rotating with angular speed
respectively and the acceleration of block A win a horizontal plane about its axis. The rod
is Sm/s 2 in the downward direction, then the does not slip on the disc and the centre of the
acceleration of block B will be : rod is at a distance R from the centre of the
(a) 0 m/s2 (b) 5 m/s2 disc. Then the kinetic energy of the rod is : Fig. 7.131
A B
(c) IOm/s 2 (d) Sl2m / s2 Fig. 7.127 (,) !mro2(R2+L2)
118. An equilateral prism of mass m rests on 2 12
a rough horizontal surface with
coefficient of friction 11. A horizontal (d) none of these
force F is applied on the prism as
shown in the Fig. 7.128. If the 123. A mass m is moving at speed v ~M
coefficient of friction is sufficiently perpendicular to a rod of length d and
high so that the prism does not slide mass M = 6 m which pivots around a d
a
before toppling, then the minimum Fig. 7.128 frictionless axle running through its
force required to topple the prism is : centre. If strikes and sticks to the end of ru-~..... .
the rod. The moment of inertia of the rod Fig. 7.132
(a) mg (b) mg 2
.fj 4 about its centre is Md /12. Then the
angular speed of the system just after the collision is :
(c) ~ mg (d) ~mg
.fj 4 (a) 2v (b) 2v
3d d
119. A rectangular block has a square base measuring a x a, and
its height is h. It moves on a horizontal surface in a direction (c) v (d) lv
perpendicular to one of the edgcs of the base. The d 2d
coefficient of friction is It. It will topple if (choose the most 124. A uniform rod of mass 'M'and length' L'is bent in the form
appropriate option) : of a regular hexagon. Moment of inertia of the hexagon so
(a) l1>a/2h (b) 11>2a / IJ formed about an axis passing through its centre and
(e) l1>a/h (d) l1>h/a perpendicular to its plane is :
120. A disc is perfonning pure rolling 7ML2 9ML2
(a) (b) 272
on a smooth stationary surface 343
with constant angular velocity as
SML2 3ML2
shown in Fig. 7.129. At any (c) (d)
instant, for the lower most point of 216 109
the disc: Fig. 7.129 125. A uniform rod oflength' L' and mass' M' is.bent to make a
(a) velocity is v, acceleration is zero rectangle PQRS such that QR = ~. Moment of inertia of the
(b) velocity is zero, acceleration is zero PQ 2
2 rectangle about the side QR can be-expressed as:
(c) velocity is v, acceleration is ~
. R (a) .'!!.-ML2 (b) E..MIl
2 212 343
(d) velocity is zero, acceleration is ~
R (d) 2ML2
162
121. A small block of mass m is rigidly
attached to P to a ring of mass 3m 126. A uniform rod of length 2 m and mass 10 kg is placed on a
and radius r. The system is released table. It is made to rotate about an axis that passes through
from rest at e= 900 and rolls one of its ends and is perpendicular to the plane of the table
without sliding. The angular with an angular velocity 10 1t rad/s. Angular momentum of
acceleration of hoop after release the rod about the given axis is:
is: Fig. 7.130 lOOn 2
400n 2
(.) --kgm /sec (b) - kg m /sec
(b) g 3 9
8, 200 2 50 2
(c) - - kg m /sec (d) - kg m /sec
(d) g 7 9
u
127. A particle ofmassO.5 kg moves aJonga line y=x-2(x, yin 134. The driver of a bus lravelling with a speed 'v' suddenly
metres) with a constant speed 6 mls. Angular momentum of observes a wall in front of his bus at a distance , a'. Father of
the particle about the origin, in magnitude, is: this driver, who was also a driver, had advised him to take a
(a) 2.fikgm 2/s (b) 4kgm 2/s
circular tum to avoid hitting in such a situation. However,
the driver in 'luestion decides otherwise by using his own
(c) 4-12kgm'/s (d) 3.J2kgm'/s wisdom. He applies brakes as hard as possible without
128. A solid sphere of mass M and radius R is placed on a rough taking a circular tum, then:
horizontal surface while in rotation with an angular velocity (a) he is more likely to hit the wa!!
00 0 , It takes some time till it begins to roll without slipping. (b) he is less likely to hit the wall
Another sphere which is hollow but of same mass M and
(c) he is as likely to hit the wa!!
radius R is also placed on the rough horizontal surface
(d) none of the above
while it is rotating with angular velocity wo' It also takes
some time till its pure rolling motion starts. If v and v are i35. Particles A, Band C, each of mass 1 kg, are at the vertices of
.• "
the linear speeds of the solid and the hollow spheres when an equilateral triangle of side 2 m. The particles ar'! moving
in a circle maintaining a constant inter-particle separation
they arc rolling without slipping, then:
2 m under their mutual gravitational forces. Speed of each
(a) Vs >v h (b) Vs <v h particle will be:

..fr
(c) Vs =v/i (d) none of these
(a) J3Gmls (b) )G/8m1s
129. A ring is rolling on a rough horizontal B
surface without slipping with a linear
(c) )G/2m1s (d) .fa mls
speed v. Referring to Fig. 7.133, ratio A v [G = universal gravitational constant]
of speeds of points B and A is: 136. A mass 3 kg is suspended from a massless string of length
(a) I: I (b) I: 2 Fig. 7.133 50 cm. Initially, the mass is at rest with the string along the
vertical. Another object of mass 200 g and moving
(c)-I2:1 (d) 1:-12
horizontally at a speed 100 mls hits the suspended body and
130. A ring is rolling on a rough horizontal

.J:i1..
sticks to it, then:
surface without slipping with a linear
(a) they are able to complete vertical circle
speed 'v'. Referring to Fig. 7.134,
(b) they are unable to complete vertical circle
linear speed of point X is: Fig. 7.134
(c) their system begins to oscillate about the original
(a) vsin 0 (b) v
position of 3 kg mass
(c) 2vcos 01 2 (d) 2vsinO/2
(d) tension in the string remains constant
131. A person is rotating in a rotor of radius 4 m. If the person
137. Fig. 7.135 shows a
can withstand acceleration upto 8.1 g, then the maximum
smooth path, the section
number of permissible rotations per second will be: 8
A CB of which is part of a 4a
[Takeg=lOm/s2] vertical circle of radius c
(a) -
2n
5
(b) -
4n
9 (e)
,
.rr
6n (d) ~
5n
'a'. An object of mass' m'
is released from the A
highest point of path at Fig. 7.135
132. A uniform disc of radius 20 cm and mass 2 kg is fixcd at its height '4a'. Resultant
centre and can rotate about an axis through the centre and force acting on the object when it is at C, is:
perpendicular to its plane. A massless cord is round along
(a) ,J52mg (b) .J47mg
the rim of the disc. Ifa uniform force 2 newton is applied on
the cord, tangential acceleration of a point on the rim of the (c) mmg (d) Mmg
disc will be: 138. Fig. 7.136 shows a
(a) I mls 2 (b) 2"",2 smooth path, the section
8
(c) 3.2 mls 2 (d) 1.6 mls 2 ACBofwhich isa part ofa h
vertical circle of radius
133. An object is moving on a circular track of radius 450 m. At c

'a'. An object of mass' m'· ai
some instant, the object is moving at 30 m/s and gaining is released from the ......- - -.........A~......
speed at a uniform rate of 2 mIs 2. Its accelcration at this highest point of path Fig. 7.136
instant is nearly: which is at height 'h'
(a) 1.6 mls 2 (b) 0.8 mls 2 above ground. Complete range of values of' h' for which
2 the object will leave the circular path anywhere from Cto B
(c) 2.8 mls (d) 3.4 n"lls 2
IS :
(,) a<h< -5 a 5 9
(b) - a<h< - a 143. A disc' X'rolls clown an incline orheight II and inclinatimi.
2 2 2 8 while an identicar disc ' Y' rolls down an incline of same
a 7 height but of greater inclination fY, then, as they reach the
(0) ::<h<4a (d) - <h <-- a bottom of their inclines:
2 2 2
139. Fig. 7.137 shows a large frictionless m (a) speed ofbolh is same, time of descent for both is same
sphere of radius 'R'. The sphere is (b) speed of Y is more than X. time of descent for both is
fixed on the ground. A mass 'm' same
begins to slide on the sphere from the (c) speed of both is same, time of descent for X is more
top of it. Height from the ground (d) speed of both is same, time of descent for X is less
where the object leaves contact with ..- ................
the sphere is: Fig. 7.137 144. A solid sphere of mass M and radius R is placed on a rough
horizontal surface. It is pulled by a horizontal force F acting
(,) 2R (b) 2R through its centre of mass as a result of which it begins 10
S 4 roll without slipping. Angular acceleration of the sphere
(0) 12 R (d) ~R can be expressed as :
7 3 3F SF
(,) (b) 7MR
140. Fig. 7.138 shows , m 4MR
frictionless vertical circular SF

"'A,
(0) 7F
track fixed on ground. (d) 2MR
IIMR
Radius of the circular track 145. Three unifonn rods, each of mass ' M' and length· L' fonn
is 10 m. An object of mass the sides of an equilateral triangle. Moment of inertia of this
m= 2 kg is given a Fly. 7.138 system about an axis passing through a comer and
horizontal velocity 'v' at the top of the path. Minimum perpendicular to the plane of triangle is:
value ofv slich that the object inunediatcly leaves contact of
the path without at all sliding along the path is: [Take (,) ~ML' (b) ~ML2
2 2
g=lOm/s2]
(a) 8 mls (b) 10 m1s
(c) 2MLz (d) ML2
2
(0) 12 m1s Cd) 6 mls 146. Two particles of masses M 1 and M Z are connected by a
141. Fig. 7.139 shows a hemisphere p .... 3!'! ___ t, massless rigid rod oflcngth' rto foml a dumbbell. Moment
ofradius4R. A ball ofradiusR is ,, of inertia of this system about an axis passing through the
,
released from position P. It rolls centre of mass of system and perpcndicular to the length of
without slipping along the inner the joining rod is:
surface of the hemisphere. Q /'
Linear speed of its centre of Fig. 7.139
(a) (M l +M2 )'4-
mass when the ball is at position
M M 2
Q, is: (0) - -1_Z. / (d) (Ml +M 2 )/
2

(,) po;R (b) fEr M l +M2

r: r--+':P'--
147. Fig. 7.141, shows a uniform rectangular
sheet with Be = 2AB. Moment of inertia of D C

-g- - ----
R
(0) (d) )6gR the sheet is minimum about: ,
(,) AC -A ----:--- S-
142. Fig. 7.140 shows a system of two rings, ,,
(b) PQ
A'--b:a-B
each of mass ' M' and radius' R'. They have
a common centre but their planes are .
-"," ------/
(0) RS ,
Fig. 7.141
mutually perpendicular. Moment of inertia (d) AD
of their system about an axis through the 148. Four particles, each of mass II~ arc arranged at the comers
common centre and perpendicular to the Fig. 7.140 of a massless square of side' a'. Mornen: of inertia of the
plane of one of the rings is: system about a diagonal is:
(a) ma 2 (b) 2ma 2
(,) ~MR 2 (b) MR2
2
(0) 2MR' (d) ~MR'
2
149. Three point masses, each m, are placed at

~
m
155. In Q. 154, the cnd B of the rod will hit thc ground w ith a
the vertices of an equilateral triangle of C:, linear speed:
side' a', Moment of inertia of the system :
about the axis COD which passes 1
(a) fiii (b) .J5ii
through the mass at 0 and lies in the
plane of triangle and perpendicular to OA
is:
0
,,m
:
0:
mA
(e) f3ii (d) Pf
Fig. 7.142 156. A solid sphere of mass M and radius R is placed on top of a
(a) ma 2
2 2
(b) -rna rough inclined surface of height h. It rolls down thc incline
5 without slipping. Its kinetic energy on reaching the bottom
(c) ~ma2 Cd) ~ma
5 2 of the incline will be:
4 4 (a) Mgh
150. A square sheet of side' a' is of mass' M', Its moment of
(b) less than Mgh as the surface is rough
inertia about a diagonal is:
(c) more than Mgh because of additional rotational kinetic
Ma 2 Ma 2 energy
(a) (b)
8 6 (d) depending on the coefficient of static friction between
Ma 2 Ma 2 the sphere and the surface
(e) (d)
12 9 157. Fig. 7.145 shows an inclined
151. An object of mass' m' is projected with a velocity' u' at an surface. Half of the surface is
angle 45° with the horizontal. When the object is at absolutely smooth and the other
maximum height, its angular momentum about the point of half rough enough to prevent h
projection is:
mu 2 mu 2
slipping. Sphere A is placed at the
top on the rough part and sphere
,
(a) (b) B on the smooth part at the same Fig. 7.145
g 2g time. Both spheres move down the plane, then:
3
(c) ~
mu..[;; (a) A and Breach thc bottom at same time
(d) - -
4,{ig 2g (b) B reachcs bottom earlier
152. A spherical ball of mass 2 kg is rolling without slipping (c) A reaches bottom earlier
with a speed 4 mls on a rough floor. Its rotational kinctic (d) which sphere will reach carlicr cannot bc decoded
energy is:
158. In~. 157,
(a) 7.23 (b) 6.43 (a) A and B reach the bottom with equal kinetic encrgies
(e) 9.4 3 (d) 11.63 and with equal linear speeds
153. Fig. 7.143 shows a flywheel of radius 10 em. (b) A and B reach bottom with equal kinetic energies but
Its moment of inertia about the rotation axis with B having a greater linear specd
2
is 0.4 kg_m . A massless string passes over
(c) Breaches thc bottom with a greater kinetic energy and
the flywheel and a mass 2 kg is attached at its
with grcatcr speed
lower end. Angular acceleration of the mass
in rad! s 2 is nearly:
2kg (d) A and Breach boltom with equal kinetic energies but
with A having a greater linear speed
(a) 4.8 radls 2 (b) 6.2 radls 2 FIg. 7.143
159. Let I be the moment of inertia of a uniform square plate
(c) 3.2 rad/s 2 (d) 9.6 radls 2 about an axis AB that passes through its centre and is
154. Fig. 7.144 shows a unifonn rod of length I B parallel to two of its sidp.s. CD is a line in the plane of the
and mass M which is pivoted at end A such plate that passes through the centre of the plate and makes
that it can rotate in a vertical plane. The /-
/> an angle 9 with AB. The moment of inertia of the plate
free end of the rod' B' is initially vertically ,/;// about thcaxisCDis then equal to: fliT 1998]
above the pivot and then released. As the //
rod rotates about A, its angular acceleration • , ,~
_J ________ _ (a)/ (b)/sin 2 e
when it is inclined to horizontal at angle A e 001_ 2 0 001_2(O/~
is: FIg. 7.144 160. A satellite S is moving in an elliptical orbit around the
(a) 3g cos e (b) !l..tan e earth. The mass of the satellite is very smal l comparcd to the
2/ / mass of the earth: [lIT 19981

(e) 5g sin e (d) fsin 9


(a) The acceleration of S is always directed towards the
4/ centre ofthe earth
(b) The angular momentum of S about the centre of the 165. A uniform cylinder is placed on a horizontal board. The
earth changes in direction, but its magnitude remains coefficient of friction between them is k. The board is
constant imparted a constant acceleration w in a horizontal direction
(c) The total mechanical energy of S varies periodically at right angles to cylinder axis. Find the limiting value ofw
with time for which there is no slipping:
(d) The linear momentum of S remains constant in (a) wlim = 2kg (b) Wlim =3kg
magnitude 3
161. A uniform rod of mass m is bent p , (c) wlim := lkg (d) W
lim "-kg
2
into a semicircle of radius R. The 166. A stone of mass 0. 1 kg is projected with a velocity of2ofi
moment of inertia of the rod mls from the origin of an Xy co~ordinate axis system at an
about an axis passing through P 0
angle of 45 with the horizontal. The angular momentum of
and perpendicular to plane of Fig. 7.146 the stone about the origin after I second is:
semicircular ring is:
[Takeg=lOm/s 2 j
(a) mR2 (b) 2mR2
(a) 10k (kg m2Is) (b) - 10k (kgm 2 /s)
(c) ~mR2 (d)
3 (c) lOi(kgmis 2) (d) 10](kgmis2)
162. A thin wire of length Land uniform linear mass density pis 167. A ring of mass m and radius a rolls down an inclined plane
bent into a circular loop with centre 0 as shown. The of length I and inclination 9, starting from rest without
moment of inertia of the loop about the axis XX' is: slipping. When the ring reaches the bottom ofthe plane, its
[IIT2000[ angular momentum about its centre of mass is:
X- -"""7"1T"""-- X' (a) mgleose (b) ~ma~gisin9
2
o (c) ma~glsin 9 (d) ma~2glsin 9
168. A small mass m is rotating on a circular

(a)
pL'
8,2
Fig. 7.147

(b)
pL'
100 2
path of radius r on a smooth horizontal
table with speed v. The centripetal force
is providcd by a light inextensible
string tied to In and passing through a
hole in the table at the centre of the
circle and having a mass M hanging at
&! ,,
:
:
(c)
5pL3
(d)
3pL3 its lower end. The mass M is suddenly oM
161t 2 8,2 increased to 8M. Find the new radius of
Fig. 7.148
163. A uniform solid cylinder of radius r starts rolling without the circle and the new velocity of m
when the system attains equilibrium:
slipping down from the top of a fixed sphere of radius R.
r
The angular velocity of the cylinder at the moment it breaks (a) 2v and - (b) ~ and 2r
off the sphere will be: 2 2
(a) 3(RH)g 4(R+r)g (c) 2vand2r
v
(b) (d) -and4r
4r2 8
169. The greatest angle through which a man can oscillate on a
(R+r)g 2(R +r)g swing the ropes of which can support twice his weight
(c) (d)
2r2 ,2 when at rest is:
164. A solid unifom1 sphere rotating about its axis (with (a) 90' (b) 60'
rotational kinetic energy Eo) is placed on a rough (c) 120' (d) 30'
horizontal plane without any translational push. Friction 170. A coin is placed on the horizontal surface of a rotating disc.
coefficient j.l is not same everywhere on the plane and it The distance of the coin from the axis is' I m and coefficient
may differ even at each point. After sometime the sphere of friction is 0.5. If the disc starts from rest and is given an
begins pure rolling with total kinetic cnergy E. Then:
angular acceleration (ll..fi) radlsec 2 , the number of
(a) E = 2Eo/5
revolutions Iluough which the disc tums hefore the coin
(b) £"2E o17 slips is:
(c) E=5E ol7 5
(d) E cannot be expressed in terms of Eo
(a) 4 (b) 24, (c) -
4,
(d) 24,
171. A massmis attached to a string 176. A smooth sphcrc A is moving on a frictionless horizontal
of length I at one end. The ,
!,mmC---'----::1C7
a 0 planc with angular speed wand centre of mass velocity v. It
other end of the string is fixed \,, collides elastically and head on with an identical sphere B at
to a point O. The string is held ,,, rest. Neglect friction everywhere. After the collision, their
horizontally initially and then \ angular speeds are roA and ron' respectively. Then:
\

Yb
allowed to swing in the '\ lilT 19991
vertical plane. When the string (a) 0),( <ron (b) w,( =ro n
makes an angle a. with the
(c) w,( =w (d) wB =00
horizontal at a certain instant,
Fig. 7.149 177. A disc of mass M and radius R is rolling
the magnitude of the total
with angular speed (0 on a horizontal --::::--"00
acceleration of the mass at this instant is:
plane as shown. The magnitude of
(,) gsin a (b) )g+2gsin a angular momentum of the disc about the
originOis: lIlT 19991 0 x
(e) g~1+4sin2 a (d) g~1 + 3sjn2 Ct 2
(a) (1/2)MR ro Fig. 7.152
172. A metal wire in the fonn of a ring of radius R and mass m
(b) MR'",
placed on a smooth horizontal table is set rotating about its
own axis in such a way that each part ofthe ring moves with (e) (3/2) MR2",
speed v. The tension in the ring will be: (d) 2MR 2",

(,)
21tmv2 mv 2 178. A sphere of mass M and radius rslips on a rough horizontal
(b) plane. At some instant it has translational velocity Vo and
R 2nR
V
mv' rotational velocity about the centre~. The translational
(e) (d) 21tmv2 2r
R velocity when the sphere starts pure rolling motion is:
173. A metal wire of mass M and length / is rotated in a circle by
(a) Vo (b) 2Vo (e) Vo (d) 6Vo
holding it at onc end. The breaking stress of wire is F and its 5 7 3 7
area of cross-section is a. Calculate the maximum
frequency of rotation at which it breaks:
179. One quarter sector is cut-from a uniform '-f-.t
circular disc of radius R. This sector has
(a) {Fa (b) ~2Fa (e) ~ 2F (d) r,;l mass M. It is made to rotate about a line
V;;;; ml amI fPC;; perpendicular to its plane and passing
174. A cylinder of radius R is spinned through the centre of the original disc. lis
and then placed on an incline having moment of inertia about thc axis of
coefficient of friction ).L = tan e (9 is rotation is: lilT 2001]
o Fig. 7.153
the angle of incline). The cylinder (a) _~MR2
continues to spin without falling for Fig. 7.150 2
time:
(b) .!MR2
R",o _ Rwo <\
(a) (b)
3gsin 9 2gsin e (c) ~MR2
8
R",o 2R",o
(e) (d) (d) ,J2MR '
gsine gsin 9
180. A small block is shot into each of the four tracks as shown
[Hint: Frictional force = mgsin e, so net force along the plane oz 0,
below. Each of the track rises to the same height. The speed
torque t =mgsin{lxR =fo. opposing the motion. Use with which the block enters the track is the same in all
00 = 000 + otto find t.)
cases. At the highest point of the track, the nonnal reaction
175. A cubical block of side a is moving ismaximumin: IIIT200t]
with velocity v on the horizontal
smooth plane as shown in Fig. 7.151.
It hits a ridge at point O. The angular
speed of the block after it hits 0 is:
Fig. 7.151
lIlT 19991
(a)
~
f\ (b)~
(e)~ (d)~
(a) 3v (b) 3v (e) -/3v (d) zero
4a 2a ,J2a
181. A heavy disc is thrown on a horizontal surface ill such a 187. Angular momentum of a particle rotating under a central
way that it slides with speed Vo initially without rolling. It force is constant due to: {AIEEE 2007]
will start rolling without slipping when its speed reduces to:
(a) constant torque
(,) Vo (b) 2Vo (b) constant force
2 3
(c) constant linear momentum
(c) 3Vo (d) 5Vo
(d) zero torque
5 7
188. If a solid sphere, a disc and a hollow sphere are allowed to
182. A unifonn solid cylinder rolling
without slipping along a horizontal roll down an incline plane from the same height
plane suddenly encounters a plane
inclined at angle e
as shown in Fig.
"QcL'l6l
Fig. 7.154
(Orissa (JEE) 2007)
(a) hollow sphere will reach the bottom first
7.154. The value of9which could bring
(b) disc will reach the bottom first
the cylinder immediately to fcst after impact, is:
(c) sphere will reach the bottom first
(,) 90 0 (b) 60 0
(d) all will reach the bottom at the same time
(d) 300
189. A small object of unifonn ,,
183. A solid sphere of mass M, radius R and having moment of
inertia about an axis passing through the centre of mass as I, density rolls up a curved :,
:,
is recast into a disc of thickness t, whose moment of inertia
about an axis passing through its edge and perpendicular to
surface with an initial
velocity v. It reaches up to
, :,
,
2 - .--- - - ~ ---- ----~- .~ -- - ,
its plane remains I. Then, radius of the disc will be: .
a maxImum Ilelg
' ht 0 f -
3v- Fig. 7.155
(I1T2006( 4g
2R 4R R with respect to the initial position. The object is:
(,) - (b) R {2 (e) - - (d) 4
.fl5 1/iS .JI5 (liT JEE 2007(
184. A cylinder is placed on a rough inclined surface of (a) fing (b) solid sphere
inclinztion 'e'. Minimum value of coefficient of static (c) hollow sphere (d) disc
frictic"! j between the cylinder and the surface so that the 190. A body rolls down an inclined plane. Ifits kinetic energy of
cylinoer rolls without slipping is: rotation is 40% of its kinetic energy of translation, then the
()a -tan
' e - sm ·
(b)"O body is: (West Bengal JEE 20071
3 3
(a) solid cylindcr (b) solid sphcre
(d) ~sin 9
2
(c) -tane (c) disc (d) ring
3 3
185. A thin circular ring of mass M and radiusR is rotating about 191. A wheel has angularacccleration of3.0 radls 2 and an initial
its axis with a constant angular velocity' w'. Two objects, angular speed of 2.00 rad/s. In a time of 2s it has rotated
each of mass' m', arc attached gently to thc opposite ends of through an angle (in radian) of: IUPSEE 20071
a diameter of the ring. The ring rotates now with an angular (,) 6 (b) 10
velocity: (A IEEE 2006, HP (PMT) 20061 (e) 12 (d) 4
roM (b) ro(M -2m) Y
(,) 192. A particle of mass mmoves
M +m M+2m in the XY plane with a
roM (d) ro(M +m) velocity v along the straight B
(e) - -- A
M+2m M line AB. If the angular
186. A tube of length L is fiHed completely with an momentum of the particle
incompressiblc liquid of mass M and is closed at both ends. with respect to origin 0 is X
0
The tube is then rotated in a horizontal plane about one of LA when it is at A, and Ln
Fig. 7.156
its ends with a unifonn angular velocity' w'. Force exerted when it is at·B, then:
by the liquid at the other end is: (UPSEE 2007, AIIMS 2007, CBSE (PMT) 2007)
(CBSE (PMT) 2006( (a) LA >LB
(a) Mw 2L (b) ~ Mw2L (b) LA = LB
2 (c) the relationship between LA and LB depends upon the
2
(c) 2Mw L (d) none of these slope of the line AB
(d) LA <LB
193. A spherical ball of mass 20 kg is stationary at the top of a 196. A solid sphere of mass 2 kg rolls on a smooth horizontal
hill of height 100 m. It rolls down a smooth surface to the surface at 10 mls. It then rolls up a smooth inclined plane of
ground, then climbs up another hill of height 30 m and inclination 30° with the horizontal. The height attained by
finally rolls down to a horizontal base at a height of 20 m the sphere before it stops is: IUPSEE 2008J
above the ground. The velocity attained by the ball is: (a) 700 em (b) 701 em
IAIIMS 20011 (c) 7.1 m (d) nonc of these
(a) 40 m/s (b) 20 m1, 197. Three identical rods, each of length x, are joined to form a
(c) 10 m1, (d) 1050 m1, rigid equilateral triangle. Its radius of gyration about an axis
194. A round uniform body of radius R, mass M and moment of passing through a corner and perpendicular to the triangle
inertia 'I', rolls down (without slipping) an inclined plane is: IUPSEE 20081
making an angle 8with the horizontal. Then its acceleration x (b) :':
(,) Jj
is: IAIEEE 2007) 2
gsin 8 gsin 9
(,)
I-MR /12
(b)
1+// MR2
(c) ~x (d) -"-
.fi
gsin9 gsin 9 198. A solid sphere is rotating in free space. If the radius of
(c) (d)
1+MR2/J I-I I MR2 sphere is increased keeping mass same which one of the
195. For the given uniform square lamina ,, following will not be affected? [UPSEE 20081
F-
ABeD, whose centre is 0, IAIEEE 20011 D ,, ,,,, C (a) Angular velocity (b) Angular momentum
(,) lAC = -fiIEF
', (c) Moment ofincrtia (d) Rotational kinetic energy
,fO
(b) -fiIAC =IEF A
' / " ,' B
199. A solid sphere is rolling without slipping on a horizontal
E,, surface. The ratio of its rotational kinetic energy to its
(c) lAD =3I EF translational kinetic enerzy is: [UPSEE 20081
Fig. 7.157
(d) JA~ =IEF (a) 2/9 (b) 211
(c) 2/5 (d) 112

ANSWERS

1. (b) 2. (e) 3. (b) 4, (,) 5. (d) 6. (e) 7. (e) 8. (e) 9. (c) 10. (a) II. (a) 12. (e)
13. (e) 14. (e) 15. (a) 16. (b) 17. (b) 18. (c) 19. (e) 20. (e) 21. (b) 22. (d) 23. (a) 24. (e)
25. (d) 26. (c) 27. (e) 28, (b) 29. (b) 30. (d) 31. (b) 32. (b) 33. (e) 34. (d) 35. (b) 36. (b)
37. (e) 38, (d) 39. (e) 40. (b) 41. (c) 42. (d) 43. (e) 44. (b) 45. (e) 46. (d) 47. (a) 48. (e)
49. (e) 50, (b) 51. (e) 52. (d) 53. (b) 54. (b) 55. (b) 56. (d) 51. (d) 58. (b) 59. (e) 60. (e)
61. (d) 62. (e) 63. (b) 64. (d) 65. (a) 66. (b) 67, (b) 68. (-I) 69, (b) 70. (b) 71. (d) 72. (d)
73. (e) 74. (a) 75, (b) 76. (a) n (d) 78. (a) 79. (b) 80. (d) 81. (e) 82. (d) 83. (d) 84. (b)
85, (b) 86. (e) 87. (e) 88. (b) 89. (d) 90. (e) 91. (a) 92. (a) 9). (G) 94. (b) 95. (e) 96. (b)
97. (d) 98. (d) 99. (b) 100. (e) 101. (a) J02. (a) 103. (a) 104. (b) 105. (a) i06. (a) 107. (e) 108. (a)
109. (b) 110 (a) 111. (a) 112, (b) 113. (d) 114. (b) 115. (b) 116. (b) 117, (d) lIS. (a) 119. (e) 120. (d)
121. (b) 122. (a) 123. (a) 124. (e) 125. (d) 126. (a) 127. (d) 128. (b) 129. (e) 130. (d) 131. (b) 132. (b)
133. (e) 134. (b) 135. (e) 136. (a) 137. (e) 138. (a) 139. (d) 140. (b) 141. (a) 142. (d) 143. (e) 144. (b)
145, (b) 146. (e) 147. (b) 148. (a) 149. (d) 150. (e) 151. (,) 152. (b) 153. (a) 154. (a) 155. (e) 156. (a)
157. (b) 158. (b) 159. (a) 160. (a) 161. (b) 162, (d) !;:i:.J. (b) 164. (b) 165. (b) 166. (a) 167. (e) 168. (a)
169. (b) 170. (b) 171. (d) 172. (b) 173. (b) 174. (b) 175. (a) 176. (e) 177. (e) 118. (d) 179. (a) 180. (a)
181. (b) 182. (e) 183. (a) 184. (a) 185. (c) 186. (b) 187. (d) 188. (e) 189. (d) 190. (b) 191. (b) 192. (b)
193. (a) 194. (b) 195. (d) 196. (e) 197. (a) 198. (b) 199. ee)
[B] More than One Choice is Correct 9. If torque acting on a particle is zero:
1. For a particle ofa rotating rigid body v = rCA SO: (a) Linear momentum = constant
(a) rocc(l! r) (b) coocv (b) Angular momentum = constant
(e) vocr (d) mis independent ofr (c) Force is conservative
2. If ar and at represents radial and tangential acceleration, (d) Force is central
the motion of a particle will be circular if: to. A particle moves on a straight line with a unifonn velocity.
(a) a r =Oanda t = 0 (b) ar=Obuto,*O The angular momentum ofthe particle is:
(e) ar:;t:Obuta, = O (d) ar*Oandat*O (a) Always zero
3. In case ofunifonn circular motion which of the following (b) Zero about a point on the straight line
physical quantities remains constant?
(c) Zero about a point away from the straight line
(a) Speed (b) Momentum
(d) Constant always about a given point not on the line
(e) Kinetic energy Cd) Angular momentum
II. A particle of mass m is projected with a velocity v making
4. In case ofunifonn circular motion which of the following
an angle of 45° with the horizontal. The magnitude of the
physical quantities do not remain constant?
angular momentum of the projectile about the point of
(a) Velocity (b) Momentum
projection when the particle is at its maximum height h is :
(e) Kinetic energy (d) Torque
(liT 1990(
5. In case of uniform circular motion:
(a) zero (b) mv"(4.fi)g
(a) Torque is zero (b) Force is central
(c) Work done is zero (d) Acceleration is zero (c) nw"(.fi)g (d) mJ1gh'
6. A simple pendulum of length L and mass (bob) M is [Hint: L '" mvrsin9 '" m(vcos45<»h '" m(vl..J2) x(vsin45 o)21
oscillating in a plane about a vertical line between angular
limits --q, and $. For an angular displacement 9.[ 1 9 1 <~] 2g '" mv 3/(4..J2)g and as h '" (vsin45o)212g,L::: m~2gh3 also.] .
the tension in the string and velocity of the bob are T and v 12. A unifonn bar of length 6a and mass 8m lies on a smooth
respectively. The following relations hold good under the horizontal table. Two point masses m and 2m, moving in the
above conditions: same horizontal plane with speeds 2v and v respectively,
Mv' strike the bar (as shown in Fig. 7.159) and stick to the bar
(a) Tcos 9 =Mg (b) T-Mgcos9= - -
L after collision. Denoting angular velocity (about centre of
(c) Tangential acceleration-=gsin 9 mass), total energy and velocity of centre of mass by c:q E
and Vo respectively, we have after collision: lIlT 19911
(d) T=Mgcos9
[Hint: See § 7.3 Note 0) and (ii)]
7. The moment of inertia of a thin square A 4 1,8
T:m
, c
plate ABCDofunifoml thickness about
an axis passing through the centre 0
" ' _, :
' __: ,/
,/
s::~~~,~;.~:.i::=~=,~:~~,18m
2a . a' 2a 1~~
and perpendicular to the plate is : -- --O'T-
:--- --3
, , '
(a) I [ +12 /' : '-,
D' ',C Fig. 7.159
(b) 13+14 2
(c) 1] + 13 Fig. 7.158 (a) Vo =0 (b) ro=(3v/5a)
v 2
(d) I ] +12 + /3 +/4 (c) w=- (d) E = 3mv
where 1],12,13 and 14 are respectively the moments of Sa 5
inertia about axes 1, 2, 3 and 4 which arc in the plane of the [Hint: See solved Problem 57]
plate. lIlT 19921 13. A solid cylinder of mass mand radius ris rolling on a rough
[Hint: See soived Problem 39 with Note.] inclined plane of inclination 9. The coefficient of friction
8. In case ofa wheel roiling without slipping ifv r ,vc and v s between the cylinder and incline is~. Then: lIlT 2006)
represent the linear speeds of a point at the top, centre and
(a) frictional force is always Ilmgcos 0
bottom of the wheel respectively,
(b) friction is a dissipative force
(a) Vs ",, 0 (b) vn =vC ~ vT
(c) by decreasing a, frictional force decreases
(c) vr =2v C (d) Vc >vn
[Hint: See § 7.4 (C) (3)] (d) friction opposes translation and supports rotation
14. A ball moves over a fixed track as shown in the Fig. 7.160 (b) Moment of inertia about its centre of mass and
From A to B the ball rolls without slipping. Surface BC is perpendicular to rod is ~ (xi - x~ )
frictionless. K A ' K Band K c are kinetic energies of the 3
ball at A ,B and C, respectively. Then lIlT 2006] (c) Moment of inertia about x = Oand perpendicular to rod
is(ICM+2~6)kgm2
(d) Moment of inertia aboutx =Land perpendicular to rod

is(lCM+~~)kgm2
Fig. 7.160
18. Four identical rods, each of mass mand length I, are joined
(a) hA >hc;Ks > Kc (b) hA >hc;Kc >KA to fmm a rigid square frame. The frame lies in thexyplane,
(c) hA = hc;Ko ""Kc (d) hA <hc;Ks >Kc with its centre at the origin and the sides parallel to thex and
IS. A solid body rotates about a stationary axis so that its yaxes. Its moment of inertia about:
e
angular velocity depends on rotating angle as 00 = 00
0 - as. a
()th
. . 2/,
ex-aXISls-m
Where 010 and a are positive constants. At the time 3
t = O,8:=:Othen: . . 4/,
ro t ez-aX1Sls-m
(b)h
(a) Rotating angle at any instant is ~ (l _ e~at ) 3
a (c) an axis parallel 10 the z -axis and passing through a
ro . -10 m/'
(b) Rotating angle at any instant is ~ e - at comerls
a 3
(c) Angular velocity at any instant is woe-at (d) one side is ~ ml2
2
(d) Angular velocity at any instant is 00 0 (l _ e- at )
19. On a smooth horizontal circular plate a bead of mass' m'
16. A rod of length' r is pivoted smoothly at a is resting on a tied to a string, the other end of which passes through a hole
block of height h. If the block moves with a constant at the centre '0' of plate, moves with certain angular
velocity V, pick the correct alternatives. velocity. The string is pulled further to bring the radius to
one fourth of its initial value. Then:
(a) The angular momentum is conserved about vertical
V h
axis passing through center' 0'
(b) The angular velocity becomes 16 times its initial value
(c) The tension becomes 64 times its initial value
Fig. 7.161
(d) The tension falls to one fourth of its initial value
() · 0fd·Vcos9
a angu Iar ve IoClty ro IS - - 20. A man spining in free space changes the shape of his body,
h e.g., by spreading his anns or curling up. By doing this, he
. . 2V2 cos 3 9sin 9 can change his:
(b) angular accelerallon of rod IS = - = 'c'::"',,-"
h' (a) moment of inertia
2 (b) angular momentum
· 0 f ro d·IS Vco s
(c) angu Iar ve IoClty e (c) angular velocity
h
(d) rotational kinetic energy
(d) tangential velocity of free end of rod is IV cos 2 9 21. A disc is given an initial angular
h
velocity roo and placed on rough
17. Linear density of a rod of length L, is horizontal surface as shown. The
given as A. = AO+ax[AO = 2kgm- l , x 0 x o< l quantities which will not depend on the
Fig. 7.162 Fig. 7.163
a = lkgm 2 ,x I and x 2 are distance of coefficient of friction is/arc:
x = 0 and x = L from COM] consider L = 3 m (a) The time until rolling begins
(a) Moment of inertia about its centre of mass and (b) The displacement of the disc until rolling begins
2 x4 _ x 4 (c) The velocity when rolling begins
perpendicular to rod is _ (xi - x~)+ 2 I (d) The work done by the force of friction
. 34
22. A particle moving on a horizontal frictionless plane collides 6. (A) : A body is moving along a circle with a variable
elastically with a unifonn smooth rod lying on the same angular speed. Work done by centripetal force will
plane and which is hinged to rotate about a vertical ax is be zero.
passing through one of its ends. Then : (R) : In non-unifonn circular motion, net force on the
(a) angular momentum of the system about any point is body is not in the radial direction.
conserved 7. (A) : A body tied to an end of a string is whirled along a
(b) linear momentum of the particle is conserved vertical circle by giving some velocity at the lowest
perpendicular to the rod position. If the velocity becomes zero before the
(e) kinetic energy of the system (rod + particle) remains tension in the string is zero, the body will leave the
unchanged after the collision circular path at the position of its zero velocity and
(d) linear momentum of the system (rod + particle) is then fall vertically downward.
conserved (R) : In vertical circular motion, tension in the string at
the highest position is maximum.
8. (A) : A body tied to an end of a string is whirled along a
I. (c) and (d); 2. (c) and (d); 3. (a), (c) and (d); 4. (a) and vertical circle with such a velocity at the lowest
(b); 5. (a). (b) and (c); 6. (b) and (c); 7. (a), (b) and (c); 8. point that, at some position, tension in the string is
zero but the speed at the position is non-zero. The
(a), (c) and (d); 9. (b) and (d); 10. (b) and (d); 11. (b) and
body will leave the circular path at the position of
(d); 12. (a), (c) and (d); 13. (c) and (d); 14. (a), (b) and (d); zero tension.
15. (a) and (c); 16. (b), (c) and (d); 17. (c) and (d); 18. (a),
(R) : In vertical circular motion, so as to cross the highest
(b) and (c); 19. (a), (b) and (c); 20. (a), (c) and (d); 21. (c) point along the circle, speed at the highest point,
and (d); 22. (c). v H ;:>: 0
9. (A) : A rigid body is rolling without slipping. All the
Ie) Assertion-Reason Type Questions points of the body will have the same linear speed
(a) If both A and R ace true and R is the correct which is equal to the speed of centre of mass of the
explanation of A. body.
(b) If both A and R ace true but R is not correct (R) : Kinetic energy of a rigid body that is rolling can be
explanation of A. expressed as K.E .... 12 mv~, Vern being the linear
(c) If A is true but R is false.
speed of centre of mass.
(d) If A is fal se but R is true.
JO. (A) : A body is rolling without slipping on a surface.
(e) Ifboth A and R 8rc false.
There must be fri ctional fo rce for such a motion to
1. (A): In circular motion, centripetal and centrifugal forc es be possible.
act in opposite directions and balance each other.
(R) : In rolling without slipping, work done against the
(R): Centrifugal force is a pseudo force . frictional force is zero.
2 . (A): A body is moving along a c ircle with a constant II . (A) : A solid sphere and a ring of same mass and radius
speed . Its angular momentum about the centre oflhe are released simultaneously from the top of an
circle will remain conserved. inclined surface. The two objects roll down the
(R): A constant non-zero torque acts on the body. plane without slipping. They reach the bottom of the
3. (A): In uniform circular motion of a body, its linear incline with equallincar speeds.
speed remains constan!. (R) : Loss of potential energy for both is same.
(R) : Total acceleration o f Ihe body has no radial 12. (A) : A ring and a disc o f same mass and rad ius begin to
component. roll without slipp ing from the top of an inclined
4. (A): In non-uniform circular motion, linear specd of the surface at I = n The ring reaches the bottom of
body is variable. incline at time t] while the disc reaches the bottom at
timet 2 , then t ] > t 2 •
(R): Acceleration of the body is towards the centre.
(R) : Disc will roll dow n the plane with a large acceleration
S. (A): In uniform circular motion, acceleration is lime-
because of its smaller moment of inertia.
dependent.
13. (A) : If there is no external to rque on a body about its
(R): In unifom\ circular motion, tangential force acting
centre of mass, then the velocity of centre of mass
on the body is time-dependent.
remains constant.
(R): The linear momentum of an isolated system remains 23. (A) : There is no relative motion between any two points
constant. lilT 20071 of a rigid body along the line of seperation. but there
14. (A): A ladder is more apt to slip, when you are high up on may be a relative motion perpendicular to the line
it than when you just begin to climb. joining the points.
(R): At the high upon a ladder. the torque is large and on (R) : If there is a relative motil)n along the line of
just beginning to climb up the torque is small. separation, the body is no longer called a rigid body.
IAIIMS 20071 24. (A): The direction of torque is parallel to (i;x F).
15. (A): The velocity of a body at the bottom of an inclined (R) : Torque is a vector quantity.
plane of given height is more when it slides down
the plane, compared to, when it rolling down the ANSWERS
same plane. 1. Cd) 2. (e) 3. (c) 4. (c) 5. (c) 6. Cb) 7. Ce)
(R): In rolling down a body acquires both, kinetic energy 8. Ce) 9. (e) 10. Cb) 11. Cd) 12. (a) 13. (d) 14. (a)
of translation and rotation. IAIIMS 2008) IS. (a) 16. (a) 17. Cb) 18. Cd) 19. (c) 20. (a) 11. (b)
16. (A): A body of mass 1 kg is making 1 rps in a circle of
11. (d) 23. (a) 24. Cb)
radius 1 m. Centrifugal force acting on it is 41t2 N.
2
[D) Integer Type Questions
(R): Centrifugal force is given by F = mv .
, I. The wheel rolls on the flat slab
IAIIMS 20081 without slipping. If the centre 0
17. (A): When a body is rolling, its kinetic energy is equal to moves 200 mm 10 the right while Ihe
the sum of translational and rotational KE of all the slab moves tOO mm 10 Ihe left, Fig. 7.164
particles. calculate the angular displacement
(R) : Rolling is the combined motion of translatory as a.6 of the wheel.
well as rotatory motion. 2. A ring is rolling on a rough
18. (A): A solid sphere rolling on a rough horizontal surface. horizontal surface without slipping
Acceleration of contact point is zero. with a linear speed v. Referring to
Figure, if ratio of speed of points B
(R): A solid sphere can roll on the smooth surface.
and A is .fX then find the value of x. Fig. 7.165
19. (A) : When a cyclist is going at a speed von a circular
3. A point mass is tied to one end of a V1 I ~1
horizontal road of radius r which is not banked,
cord whose other end passes through
cyclist bends at an angle tan -1 [;; ) With vertical. a vertical hollow tube. caught in one
hand. The point mass is being rotated
(R): Total external force and total external torque in a horizontal circle of radius 2m
together must be zero with respect to ground with speed of 4 mls. The cord is then
pull down so that the radius of the
reference frame.
circle reduces to 1 metre. Computer
20. (A): If moment of inertia of a non uniform thin circular Fig. 7.166
ratio of kinetic energies under the
ring is same about two different axes parallel to each
final and initial states.
other and lying in the plane of ring, then both the
axes can be at same d istance from centre of mass of 4. A cubical block of side a moving
ring. with velocity v o n a horizontal
smooth plane as shown in Fig. 7. 167.
(R) : From parallel axes theorem, II = 12 is possible on ly It hits a ridge at pointo. If the angular
Fig. 7.167
ifleM +1II/~ =/CM +mli· speed of the block after it hilsOis 3V
21. (A) : Total torque on a system is independent of origin if n"
Ihen find the value of n.
the total external force is zero. Origin is point of
rotation about which the torque can be calculated. S. A uniform rod of mass I kg. length 1 m rests on a horizontal
smooth surface. An impulse I Ns is provided at one edge
(R): Torque due to couples is independent of origin.
normal to the rod. Angular velocity of rod just after
22. (A): Radius of gyration is independent of distribution of
receiving impulse (in radls) is :
mass of body relative to an axis of rotation.
6. The moment of inertia of a pair of solid spheres. each
(R): The radius of gyration is root mean square distance
having a mass m and radius r, kept in contact about the
of particles of the body about the axis of rotation.
ta[n!:~: }p'' ing through the point of contact is Neglect any rebounding of hemisphere. If sphere comes to
rest after the simultaneous collision with three hemispheres
n -5- Find value of n.

7. A uniform rod of mass mand length lis


suspended by means of two light
inextensible strings as shown in Fig.
D then ratio of KE of sphere before collision and KE of any
hemisphere after collision is p then find the value of p
12. In the figure shown a blockB of mass mean slide on a fixed
horizontal smooth plane. A solid sphere A of radius rofthe
same mass rolls without sliding on the block B. The angular
7.168. Tension in one string A B
acceleration ofthe sphere is (5n)g, find the value of' n'
immediately after the other string is cut Fig. 7.168 ,
is mg. Then the value of n is : 13. Uniform rod mass (2m), length L. is lying on a smooth
n horizontal surface. Two insect of mass m each moving
8. A wheel starting from rest is uniformly acceleration at 4 horizontally and perpendicular to rod hits the rod and stick
radls 2 for 10 seconds. It is allowed to rotate uniformly for it. After hitting the .rod, insect starts walking along the rod
the next 10 seconds and is finally brought to rest in the next away from centre with constant speed v, relative to rod. The
10 seconds. The total angle rotated by the wheel is n x 102 total angle rotated by the rod, till the insect reaches the ends
rad. Find the value of n. of rod is 1t -./3' Value of x is
(x)4 3
9. A string.is wrapped around the rim ofa wheel of moment of
inertia 0.20 kg~m2 and radius 20 em. The wheel is free to
rotate about its axis is fixed. Initially, the wheel is at rest. m ~··········l·~ c~ U6

m~'!--------- _ U6!
The string is now pulled by a force of 20 N. The angular
velocity of the wheel after 5.0 seconds is nx lO2 rad/sec.
Then value of n is : Fig. 7.171
lO. In the pulley system shown in Fig. 7.169, if radii of the 14. In the below situation 10 kg mass is balanced by M kg
bigger and smaller pulley are 2 m and I m respectively and mass. Drum radii ratio is R2 :R1 = 3 : ~ the value of Mis
the acceleration of block A is 5 mls 2 in the downward xx 20 kg. The value of x is (pulleys are light)
direction, then the acceleration of block B will be!!.. mlsec 2
2
then the value of n is :
Smooth
'---""-cs
10 kg

R,
Fig. 7.172

ANSWERS

1. 2 2. 2 3. 4 4. 4 5. 6 6. 7 7.4
Fig. 7.169
8. 8 9. I 10. 5 11. 9 12. 8 13. 3 14. 6
11. A ball of mass m falls symmetrically on a set of three
hemispheres placed in contact with each other on smooth [E] Match the Columns
horizontal ground from a height h. Mass of each
hemisphere is!!!. and radius of each body is R. 1. In Column-I description of force(s) acting on a body is
2 given and in Column-II the description of motion. Match
the entries of Column-I to that ofColumn-IJ
Column - I Column -11
(a) A single force whose (p) Body performs pure
line of action is passing translational motion
Top View Side View
through centre of mass
Fig. 7.170
is acting on body
(b) Two equal and opposite (q) Body perfonns pure (c) If there is relative (,) Zero
forces whose lines of rotational motion motion between the
action (not along same block and plank, then
line) are located work done by friction
symmetrically with force acting on block
respect to centre of plus work done by
mass friction acting on plank
is
(c) Two equai·and opposite (,) Combined rotational
forces whose lines of ,nd translational (d) If there is no relative (s) Is equal negative ofloss
action (not along same motion motion between the in mechanical energy of
line) "e located block and plank, then two block plus plank
unsymmetrically w.r.t.
centre of mass . work done by friction
force acting on block
system

(d) A single force whose (s) An~lar acceleration is plus work done by
line of action is not nonMzero friction acting on plank
passing through centre is
of mass
3. A uniform dise rolls' without slipping on a rough horizontal
2. A small block of mass m l lies over a long plank of mass mz . surface with uniform angular velocity. Point 0 is the centre
The plank in tum lies over a smooth horizontal surface. The of disc and P is a point on disc as shown in Fig. 7.174. In
coefficient of friction between nil and 1!12 is~. A horizontal each situation of Column-I a statement is given and the
force F is applied to the plank as shown in Fig. 7.173.
ColumnMJ gives four situations corresponding to the system
.
corresponding results are given in Column-II. Match the
statements in Column-I with the results in Column-II.

..8..
given above. In each situation given in Column-I, both
bodies are initially at rest and subsequently the plank is ·0
pulled by the horizontal force F. Take length of plank to be
. - . -
large enough so that block does not fall from it. Match the Fig. 7.174
statements on Column-I with results in Column-II
Column - I Column -II
(,) The velocity of point P (p) Changes in magnitude
on the disc with time
(b) The acceleration of (q) always directed from
Match the following: point P on the disc that point (the point on
disc given in Column-I)
Column -I Column - II towards the centre of
disc
(a) If there is no relative (p) Positive
motion between the (c) Thc tangential (,) Is always zero
block and plank, the acceleration of point P
work done by force of on the disc
friction acting on block
-
(d) The acceleration of (s) I, non-zero and
in some time interval is
point 011 the disc which remallls constant In
(b) If there is no relative (q) Negative is in contact with rough magnitude
motion between the horizontal surface
block and plank. the
work done by force of 4. In each situation of Column-I , a unifonll disc of mass mand
friction acting on plank
,I
radius R rolls on rough tixed horizontal surface as shown.
At I O(in itially) the angular velocity of the disc is Wo and
0:
is some time interval
velocity oflhe centre of1l1ass of the disc is 1) 0 (in horizontal
direction). The relmioll betwecn 'Vo and (00 for e<1ch
situation and ,liso initial sense of rotation is given for each
situation in Column-/. Then match the statements III
Column-I with the corresponding results in Column-no
Column -] Column - II 6. A solid sphere, a thin-walled 'hollow sphere, a solid
cylinder, a thin-walled hollow cylinder and a ring. each of
(a) '0-,"'" Vo (p) The angular
momentum of the disc
mass M and radius R, are si multaneously released at rest
from the top of an inclined plane, as shown in Fig. 7.176.
A ~ ./ (vO>RWol about point A (as The objects roll down the plane without slipping. Also we
shown in figure) may consider the objects and the surface on which they roll
remains conselVed. to be perfectly rigid. Match Columns-J and II. :

G"'~o(vo>""""
(b) The kinetic energy of
(q) the disc after it starts
A rolling without slipping
;. . 2 2 is less than its initial
kinetic energy.

(e) '0-,'" Vo (r) In the duration disc


rolls w ith slipping, the
A~ .J (vo<Rmol friction acts on disc
towards left.

(d) ~ "0 Vo (s) In the duration disc Column - [ Column - II


rolls with slipping, the
(a) Time taken to reach the (P) Solid sphere
A \. / (Vo<Rwol friction acts on disc for
bottom is maximum for
some time to right and ,
for some lime to left. (b) Angular acceleration (q) Hollow cylinder
maximum for
5. A solid sphere X of mass' M' and radius R and another
(e) Kinetic energy at the (r) Hollow sphere
sphere Y of mass '2M' and radius !i are simultaneously bottom is the same for
2
rekased at rest from the top of an inclined plane as shown in
(d) Rotational kinetic (5) Ring
the Fig. 7. 175. The spheres roll without slipping. Also we energy is maximum for
may consider the spheres and the surface on which they roll
to be perfectly rigid. Match Columns-l and II. 7. A solid spherical ball of mass M and radius R rolls without
slipping down a surface inclined to the horizontal at angle 9.
G' Considering that the ball is a unifonn solid sphere and that
T '~Al2 the ball and the surface are perfectly rigid. Match
Columns-J and II :
h
Column - I Col~mn- II

l~~.~~ Fig. 7.175


(a) Frictional
involved is
force (p) Zero

(b) Minimum value of (q) 217 Mg sin 9


Column· I Column -II coefficient of friction
for rolling without
(a) More for sphere X (p) Sp(.'Cds of the two slipping will be
spheres at the bottom of
the incline (e) Work done against the (r) Static friction
frictional force and
(b) More for sphere Y (q) Time taken by the hence loss of kinetic
spheres to reach the energy the object
bottom "
rolls down the plane is
(e) Same for both spheres (r) Angu lar acceleration (d) Forceofkioetic friction (s) 217 lao 9
(d) Zero (s) Kinetic energy at the is
bottom
8. Fig. 7,177 (a) shows two discs If and B, rotating about a (e) If the maximum tension (r) The object will not be
common axis with constant angular speeds IDA and fJ)B' in the rope is 15 N able to reach the
respectively. Moment of inertia of disc A is l.of and that of highest point on the
disc B, J B' The discs are then gently pushed towards each circle
other by forces that act along the axis. Finally. the discs rub
against each other and attain a conunon angular speed ~ as (d) If the maximum tension (s) Linear displacement of
shown in Fig. 7.177 (b). Match the Colunms-I and II : in the rope is 24 N the object from the
d,A ~fiJ
lowest position will be

----~----
about 2.8 m
-----{)-------&----
A B I. a "",* p; b-+ q,5; c-+ r,S; d -+ T,S
(01 (bl
Fig. 7.177 1. a-+ p; b-+q;c -+q;d-+r
3. a -+ p; b-l> q,5; C-I> r; d -... q,s
Columa - I Column - II
(al As the discs stick (P) will be zero 4. a -I> p,q,r; b-l> p,q,r; C-I> p,q; d -+ p;q,r
together, the final 5. a -+ does not match; b -+ r, s; C -I> p,q; d -I> does not match
angular speed, 6)
6. a -+ q,s; b -l> p; c-+ p,q, r,S; d -I> q,s
(b) Torque on each disc, (q) will be decreasing
due to the force F 7. a -+ q,r; b-+ s; c-+ p; d-+ P
applied on the disc.
•. a -I> r; b -I> Pi C -I> q,S; d -I> does not match
(0) When the discs are (r) l " m" +ISwB
rubbing against each 9. a -I> p,r, b -I> q, r; C-I> r,S; d -+ q,r
1,,+18
other, what happens to
the ~ine(ic energy of IFI Linked Comprehension Type
their system?
Paragraph - 1
(d) Angul!U' mom~ntum of (s) · there will be loss of Moment of inertia a body can be calculated using perpendi-
~e system, energy in doing work cular ues theorem J : = ! x + ! y (for a lamina) and also
, 2
against the non-
parallel axes theorem I =!c + mL for any body. We use
" conservative internal
forces perpendicular axes theorem if plane of lamina is xy -plane
and z -axis is perpendicular to lamina and we use parallel
9. A small object of mass 0.5 kg and attached to an end of a axes theorem, if the axis of rotation is parallel to the axis
massless 2 m long rope is rotated in a vertical circle with the passing through centre of mass and L is perpendicular
other end of the rope being at the centre of rotation. For any distance from the two axes. Based on these theorems.
given speed at the lowest point,consider that the motion
Answer the following questions.
happens only under gravity, take g = IOm/s2 and match
I. A circular hole of radius 'r/2' is cut from a circular disc of
the Columns·} and II : radius '~ from the edge. The disc lies in the xy -plane.
Column -I Column - II Dctennine the moment of inertia about an ax is passing
through the centre and perpendicular to the plane of the
(a) If the speed of the (P) There will be some disc. (m is mass of remainillg disc)
object at the lowest points on the circle at
13mr2 I3mr2
point is 3.5 nUs which speed of the (a) 2 (b) 24
object is zero but
tension in the rope is
(e)
2m,' (d)
4mr2
3
not zero 3
(b) If the speed of the (q) There will be some 2. A soild cylinder has length equal to two times its radius R. If
object at the lowest points on the circle for m is mass of the cylinder, then its moment of inertia about
point is 8 mls which tension in the an axis perpendicular to its own axis and passing through a
rope is zero but speed point which is at a distance !i. from centre is :
ofthe object is not zero 2
MR2 IlMR2 10 m
(a) - - (b) --- VA = 2m/s ~ A B
4 6 aA = 3 m/s2
5MR2 45 G
45·
(e) - - (d) 6MR 2 Fig. 7.179
6
3. A unifonn circular disc of radius R lies in the xy plane with Answer the following questions.
its centre coinciding with the origin. Its moment of inertia 7. The angular velocity of the rod at the instant shown is :
about an axis passing through a point on the x ~axis at a (rads - I )
distance x = 2R and perpendicular to xy plane is euqaJ to its
moment of inertia about an axis passing through a point on (a) 0.45 (b) 0.64
y-axis at a distance y = d and parallel to x ~axis in the xy (e) 0.28 (d) 0.57
plane. The value of d is : 8. Centripetal acceleration of A with ~espectS at the instant is:
4R (m s-2)
(a) (b) /!..Jl7
3 2
(a) 0.8 (b) 0.4
(e) /!..J15 (d) LID (e) 0.2 (d) 0.\ '
2 2
9. The moment ofinertia ofthe rod with·respect to instant axes
Paragraph. 2 of rotation (m = lkg) :
A rotating disc moves in the (a) 400 Kg~m2 (b) 100 Kg_m 2
positive direction of the x ·axis. 3 .
If at the initial moment the
centre C of the disc was located
, (c) 200 Kg_m2 (d) None
3
at the point 0 after which it
moved with constant velocity v Paragrapb - 4
while the disc started rotating Consider a stationary cricket bat being hit
Fig. 7.178 ~
counter-clockwise with a
constant angular acceleration ct. The initial angularvelocity by an impulse I as shown in Fig. 7.180.
is equal to zero.
From impulse-momentum equation, h
Answcr the following questions. ...(i) C
4. The co~ordinates of centre of disc at t '" t is : If y is the distance of point of application A y
(,) (O,VI) (b) (VI,O) of impulse from the centre of mass of the 1
bat, then from angular impulse· Angular
(e) (vt,vt) (d) ( "; ,VI) momentum theorem,
Fig. 7.180
~ ~ ~
5. Instantaneous co-ordinates of axis of rotation at t ::: t is : J =y1 =1CM 0:: ...(ii)
(,) (VI,O) (b) (O,~) Suppose the batsman holds the bat a~ point 0 ~t a height ~ h'
from the centre of mass. The pointOwill ha'.'e the velocities :

(c) (vt,:) (d) [v~2 ,011 (i) VCM ' due to translational motion
(ii) mh, due to rotational motion of bat
Two velocities acquired are in opposite directions, if
6. The equation y(x) describing the position of the
VCM = roh it acquires no ne~ velocity due to impulse, i. e. ,no
instantaneous axis of rotating is :
sting on hands.
(a) xy:::v 2 (b) xy:::-
v2 From equations (i) and (ii),
a ~ ~

(e) xy=v a 3
(d) xy ::: va I YI ICM
- = - - h '" y=- ,.. (iii)
M 1CM Mh
Paragraph ~ 3 This point whosc position is given by • y' is called "sweet
The rod AS shown in Fig. 7.179, is confined to move along spot".
the inclined planes at A and B. If point A has an acceleration Answer the following questions.
of3m / s 2and a velocity of2 mis, both directed down the 10. If the equation (iii) holds good. which point of the bat does
plane at the instant the rod becomes horizontal. not move after the impulse?
(a) Centre of mass
Paragraph - 6
(b) Points of impulse
A solid spherical ball of mass 0.5 kg and radius 10 em rolls
(e) Point a
to the top ofa hill, as shown in Fig. 7.182. Kinetic of the
(d) No such point exists in the bat ball at the bottom of hill is 140 J. As the ball moves up the
11. If JCM =: mil 2 , the kinetic energy of the bat is given by ; hill, it rolls wi~hout slipping. Height of the hill is 16 m. As
the ball reaches the top of the hill, it is moving horizontally.
(a) ~mh2w2 (b) J.. mh 2 Qi From the top of hill, it will move into air and fall freely
2 4 under gravity. Assuming that no work is done against
(e) mh 20i (d) 2mh oo
2 2 friction as the ball rolls and neglecting air resistance.
12. In the above case (ICM =mh2} (Takeg = lOm/s2)

(a) Centre of mass is midway between point of impact and


holding point 0
(b) Centre of mass coincide with point of impact K.E. = 140 J
(e) Centre of mass is (1I3)rd from point of impact and
(2/3)rd from the holding point
A B c
Fig. 7.182
(d) Centre of mass is (2/3)rd from holding point and
Answer the following questions.
(l/3)rd from the point of impact.
16. Angular speed of the ball at the bottom of the hill is :
Paragraph - 5 (a) 50 radls (b) 200 radls
(c) 350 radls (d) 150 radls
A unifonn chain of length 1tR and mass m held on the
2 17. Rotational kinetic energy of the ball at the top of the hill is
smooth sphere of radius R as shown. nearly:
(a) 22.5) (b) 37)
(0) 23) (d) 17)
R '+---\: • 18. Referring to the figure, when the ball strikes the ground at C,
its distance from the foot of the hill, i.e. ,BC is nearly:
(a) 23m (b) 36m
(0) 14m (d) 42m
Fig. 7.181 19. Just before the ball hits the ground atC, ils rotational kinetic
Answer the following questions. energy, is :

13. The gravitational potential energy of the chain is (Take (a) greater than its rotational kinetic energy at A
horizontal diameter as the zero level for G.P.E.) : (b) less than its rotational kinetic energy at A
(a) rngR (b) mgR (c) the same as its rotational kinetic energy at A
• 2. (d) either greater than or the same as its rotational kinetic
(c) 2mgR energy at A
(d) mgR
20. Just before the ball hit'> the ground at C, its translational
14. The chain is now released. Its gravitational potential energy kinetic energy is :
when the entire chain becomes vertical for the first time is : (a) greater than its translational kinetic energy at A
(a) _ mgR (b) - mg- nil (b) less than its translational kinetic energy at A
• 4 (c) the same as its translational kinetic energy at A
(0) _ 2mgR (d) -mgR (d) either greater than or the same as its translational

IS:

The speed of the chain at this moment is :
kinetic energy at A
21. Total kinetic energy of the ball as it is about to hit the
2 ground at C is :
8gR + 1t gR 8gR + 1t 2 gR
(a) (b) (a) 140) (b) 150)
27t 1t '
(0) 118) (d) 92)
8gR + 1tgR
(0) (d)
2.
Paragraph - 7 2. What is the weight ofa body of moss 111 at
Two discs A and B are mounted coaxially on a vertical axle. (a) the highest and
The discs have moments of inertia I and 21 respectively (b) lowest point if it is just 'looping the loop' in a vertical
about the common axis. Disc A is imparted an initial circle?
angular velocity 2(1) using the entire potential energy of a (IIint: Sce § 7.3 (3) and (5)]
spring compresscd by 11 distance XI' Disc B is imparted an [Ans. (a) zero (b) 6 mg]
angular velocity (') by a spring having the samc spring
3. A bucket of water is rotated in a vertical circle so that the
constant and compressed by a distance x 2' Both the discs
surface of water is at a distance r from the axis of rotation.
rotate in the clockwise dircction. lilT JEE 20071
What is the minimum angular velocity so that the water
Answer the following questions: does not spill out?
x [Ans • .j(ii0]
22. The ratio ...1- is :
x, 4. What is the maximum possible velocity at lowest position
(,) . 2 (b) ~ for oscillation of a simple pendulum of length L? What can
2 happen to the motion if the velocity exceeds this value?
, [Hint: Sec condition for oNcillation in § 7.3)
(0) .fj (d) ~

.fj [Ans. v'" 12K£: The pendulum after reaching · a certain height
23. When disc B is brought in contact with disc A, they acquire may kavc the circk with ccrtain velocity jf .J2iL<v < JSgL or
a common angular velocity in time t. The average frictional may loop the loop if v <! ..j5gL]
torque on one disc by the other during this !1eriod is :
5. A simple pendulum ofinass mand length Lis released from
(a) ~~ (b) 9;~ horizontal position [Fig. 7.183 (a)1. At the lowest position
what is
9/(0 3100 (a) the speed of the bob and (b) the tension ill the string?
(0) (d)
4, 2,
[AilS. (a)[iif (b) 3111g j
24. The loss of kinetic energy during the above process is:
6. What initial speed should be given to mass II! [Fig. 7.183
100 2 1{I)2
(,) (b) (b)] so that it may 'loop the loop'? '
2 3 [AilS. figf.]
2
(0)
100
4
.
ANSWERS
Paragraph- I I. (b) ,. (oj 3. (h)
l'aragraph-2 4. (h) 5. (oj 6. (b)
Paragraph-3 7. (e) H. (,) 9. (h) i'J (h) ioJ .
Flg ..7.183
Pal'agraph-4 10. (e) II. (e) 12. (,)
7. A particle of mass m is projected with velocity just 10
Paragraph.S 13. (e) 14. (b) 15. (a)
complete the loop. At position P [Fig.-7.183 (c)): what is (a)
I'aragraph-(, 16. (b) 17. ,d) 18. (a) 19. (b) 20. (3) 21. (a) the speed of mass mand (b) tension in the string?
[flint: Sc!.' § 7.3 (6)]
Pal'agraph-7 22. (cl 23. (a) 24. (h)
[Ans. (a) J3gL (b) 3mgJ
IG) Subjective Type Questions 8. You arc given <I ring, a cube, a cylinder, a prism and a
sphere, all having cqun!' masses and cross-scctions with
I. What is the approximate linear speed (in klll/hr) ora point
at the equator and pole due to lIlotion nfearth aboul its own equal heights [as shown in Fig. 7.184] and equal ma'ximum
axis if the radius of the earth is 6400 km? w idth. Which one has the largest and whieh onc ' has the
smallest Illoment of inertia about an axis perpendicular to
[Ans.1600km/ hra nd zc roj
cross-section and passing through the centro:: of mass?
[Ans. Ring; sphere)
o Ring Cube Cylinder
(b) Will it take longer to roll down one inclined plane than
the other? Explain.
[Ans. (a) Yes, as v is independent of 8. (b) Yes, greater the
inclination lesser will be the time of descent as toc; (l/sin 9).]
16. Can a body in translatoty motion have angular momentum?
Explain.
[H'"', § 7.6 (AI (5)]
[Ans. Yes]
Prism Sphere
17. Explain why spin angular velocity of a star is greatly
Fig. 7.184
enhanced when it collapses under gravitational pull and
9. In which of the following cases, it is most difficult to rotate becomes a 'neutron star'.
the rod? Explain why? [Ans. By conservation of angular momentum as J decreases, (£
[Ans. (e) as I is maximum] will increase.]

~=I'&+ ~
18. Fill in the blanks:
(a) A point on the rim of a wheel 3 m in diameter has a
linear velocity of 18 mls. The angular velocity of the
wheel is ....
(b) The length of seconds hand of a clock is 10 cm. The
(al speed of the tip of the hand is ...
(bl (el
Fig. 7.185 [Hint: ro:: (21t160) and then v:: roo]

10. In case of rolling bodies, static friction is needed to cause (c) Starting from rest the flywheel of a motor attains an
the body to rotate (otherwise it will slip). But as friction is a angular velocity of 60 radls in 5 sec. The angular
dissipative force, how can we justify the use of law of acceleration is ....
conservation of mechanical energy? (d) The acceleration of a train travelling at 40 mls as it
[~Int: Sec § 7.4 (e) (1)]
goes round a curve of 160 m in radius is ....
11. A body of mass m slides down an incline and reaches the IEAMCET 1992]
bottom with a speed v. If the same mass were in the fonn of (e) The banking angle for a curved road of radius 490 m
a ring which rolls down this incline, what will be the for a vehicle moving at 35 mls is equal to tan - I ....
velocity of the ring at the bottom? [EAMCET 19921
[Ans. vl.J2] (f) A vehicle is moving with velocity v on a curved road of
12. A cannon ball and a marble roll from rest down an incline. width' b' and radius of curvature • 1". For providing the
Which gets to the bottom first? necessaty centripetal force the diffcrcnce in elevations
[Hint: § 7.4 (e) (I)J required between the outcr and inner edges of the road
[Ans. Simultaneously] IS ....

13. You are given two spheres of the same mass, size and [Hint: tanG :: (v 2/rg) = (h/b)]
appearance, but one ofthem is solid while the other hollow. (g) A ball of I g released down an inclined plane describes
If they are allowed to roll down an incline which one wlll a circle of radius 10 cm in the vertical plane on
reach the bottom first? reaching the bottom. The minimum height of the
[HI"', § 7.4 (01 (31 (21 01] inclined plane is ... cm. IEAMCET 1991]
[Ans. Solid] [Hint: For looping the loop.'/;:: (SI2)r 1
14. Two identical cylinders 'run arace' starting from rest at the (h) A very small particle rests on the top ofa hemisphere
top of an inclined plane. One slides without rolling and the of radius 20 cm. The smallest horizontal velocity to be
oth:=r rolls without slipping. Assuming that no mechanical given to it, if it is to leave the hemisphere without
energy is dissipated as heat, which one will win? sliding down its surface taking g = 9.8m1s 2 is ....
[Hint: § 7.4 (c) (4) (iii)]
IEAMCET 19931
[Ans. Sliding one]
[Hint: For leaving the circle R -t 0, i.e., (mv2/r) = mg]
15. A solid wooden sphere rolls down two different inclined
planes of the same height but of different inclinations. (i) A ,"lid eylinde, of m"s M nnd cadius R colis down an
inclined plane of height h and inclination 9. The speed
(a) Will it reach the bottom with same speed in each case?
of ils centre of mass when the cylinder reaches the
bottom of the inclined plane is ...
(j) A solid cylinder at rest at the top of an inclined plane of 23. Two balls start moving from same position on a circle of
height 2.7 m rolls down without slipping. If the same radius 10m with speeds 201t mls and l61t mls respectively
cylinder has to slide down a frictionless incline plane in the same direction. After how much time will they collide
and acquires the same velocity as that acquired by the and how many revolutions does the slower ball make before
centre of mass of the rolling cylinder at the bottom of collision?
the inclined piane, the height of the inclined plane [Hint: See § 7.I(A) (b) (6) and solved Problem 6]
should be.... IEAMCET 1993] [Ans. 5 s and 4 revolutions]
[Hint: Vs '" vRl 24. A racing car is travelling along a track at a constant speed of
(k) A baUet-dancer stretches her hands out for slowing 40 mls. A T.V. cameraman is recording the event from a
down. This is based on the principle of conservation distance of30 m directly away from the track as shown in
of.... (EAMCET 1990( Fig. 7.187. In order to keep the car under view, at what
angular speed must the camera be rotated?
(I) A smooth unifonn rod of length Track Car
Land mass M has two identical
~40mls
beads of negligible size, each of
mass 111, which can slide freely . ,.. ..,., , ..,.
(
E
along the rod. Initially the two
beads are at the centre of the
-t -~- g
30'
rod and the system is rotating
Fig. 7.186
with an angular velocity (00
about an axis perpendicular to the rod and passing Fig. 7.187
through the mid~ point of the rod. There is no external
force. When the beads reach the ends of the rod, the [Hint: See solved Problem 10]
angular velocity of the system is .... [ADS. I rad/s]
[Ans. (a) 12 radls; (b) (ttl3) cmls; (c) 12 rad/s 2; (d) 10 mls 2; (e) 25. A disc revolves in a horizontal plane at a steady rate of 3
0.25, (f) l!1.hrg; (g) 25 cm; (h) 1.4 mls; (i) ~(4gh 13) (j) 1.8 m, (k) rev/so A coin just remains on the disc if kept ata distance of
2 em from the axis of rotation. What is the coefficient of
Angular momentum, (I) MoorJ(M + 6m)]
friction between the coin and the disc? (g ::: 10m/ s 2)
19. In a hydro,'!n atom when an electron is revolving in a
circular oro.t of radius 0.53 Aaround the nucleus it is found [Hint : Here centripetal force is provided by friction; so
that it makes 6.64x 101.5 rev/sec. What is (a) the angular mrci "" / ==)J.R = )J.mg]
speed (b) linear speed and (c) radial acceleration of the [Ans.0.725]
electron? 26. A mass m on a frictionless table is attached to a hanging
(Ans. (a) 4.17 x 10 16 radls, (b) 2.21 x 106 mis, (e) 9.2 x I 022 mls 2 ] mass M by a cord through a hole in the table (Fig. 7.90).
20. Consider the earth's orbit around the sun as a circle of Find the angular speed with which mmust spin for M to stay
radius 15x 10" m. Calculate (a) the angular velocity of the at rest.
earth (regarded as a particle) (b) orbital velocity of earth [Hint: See Note in § 7.2 (c) with 00 '" 2tt1J and tension T = Mg]
and (c) centripetal acceleration of the earth, w.r.t. the sun. [Ans. ()) = ~(Mg 1mr)]
[Ans. (a) 2 x 10-7 rad/s, (b) 3 x 104 mis, (c) 6 x 10-3 mls 2]
27. A small sphere of mass m is attached to a spring of force
21. A phonograph turntable rotating at 78 rev/min slows down constant k and unstretched length L as shown in Fig. 7.188.
and stops in 30 sec after the motor is turned off; then (a) Ifthe sphere is rotating at frequency fwhat is the:
Find its (unifonn) angular retardation. (b) How many
revolutions did it make ill this time? ~-----------

[Ans. (a) a = 0.27 Tad/5 2 (b) 9 = 391t rad, so n = 19.5] (~- t'''fifo~t~~~m
22. A fan having blades of length 0.5 m from the centre of a ' ......... _---- ----- ~~ ~~./
motor has a constant angular acceleration of 6 rad/s 2. At
the end of 5 sec find (a) angular speed (b) angular Fig. 7.188
displacement (c) linear speed (d) tangential acceleration (e)
radial acceleration of a fly sitting at the tip of the blade. Are (a) radius of circular path? (b) tension in the spring?
the results same if the fly is half~way in from the tip? [Hint; Here centripetal force is provided by elastic force of the
[Ans. (a) 30 rad/s (b) 75 rad (c) 15 mls (d) 3 mls 2 (e) 450 mls 2. If spring, so that mr(21t/)2::: k(r-L) and tension in the spring
the fly is half.way in from the tip, the angular variables (9 and (0) T:=kx:::k(r-l).]
will not change as these are characteristic of rigid body kL 41t2mkL/2
(independent of r) while linear variables, v , ar and a, will be [Ans. (a) 2 2; (b) 2 2]
halved as these are directly proportional to r.J (k - 4nm/) k - 4Itm/
28. A particle is placed inside a 33. A boy is seated on the lop of a A
hemispherical bowl which rotates hemispherica l lump of icc of
about its vertical axis with constant radius I: (a) If the boy begins to
c
angular velocity. It is just prevented slide without friction from the
from sliding down when OP is top, at what height from the base
inclined at 45 0 with the axis (0 is the win the boy lose contact with the Fig. 7.191
centre of the sphere and P is the Fig. 7.189 surface of the sphere? (b) What
position of the particle). The radius horizontal velocity be given to the boy at the top so that he
of the bowl is (lo.j2) m and coefficient of friction between leaves the sphere without sliding?
the particle and the howl is 0.5. Find the magnitude of co [Hint: See solved Problem 28 with Note 0) and (iOl
assuming g = 10m/5 2 . [Ans. (a) (2/3)r, (b) 11 ;:: fg7- J
[Ans. (I f..fj) radls] 34. A bullet of mass I'"
is fired
29. A large mass M and small mass mhang with a velocity 50 ntis at an
at the two ends of a string that passes angle Owith the horizontal. At
through a smooth tube as shown in the highest point of its
Fig. 7.190. The mass m moves around trajectory, it collides head on
a circular path which lies in horizontal with a bob of mass 3M
plane. The length ofthe string from the suspended by a massless
mass mto the top of the tube is Land e string of length (10/3) III and Fig. 7.192
is the angle which this length makes gets embedded in the bob.
with the vertical. What should be the After the collision the string moves through an angle 120°.
frequency of rotation of the mass III so Find (a) thc angle 0, (b) the vel1ical and horizontal
M co-ordinates of the initial position ofthc bob with respect to
that the mass M remains stationary?
FIg. 7.190 the point of firing of the bullet. (g = 10 m l s 2 )
[Hint: For circular motion of III,
mroi '" Tsin8with r '" LsinO and T '" MgJ [!lint: See solved Problem 44 in Chapler 6]
[Ans. f '" (l/21t)~(Mg I IIIL) J [Ans. (a) O~30o. (b) x = 108.25 m and Y'" 31.25 til)

30. A sphere of mass 200 g is attached to an inextensiblc string 35. Three thin iroil rods each of mass M and length Lare welded
of length 130 em whose upper end is fixed to the ceiling. so as to form an equilateral trianglc lFig. 7.193 (a)}. Find
Thc sphcre is made to describe 11 horizontal circle of radius the rotational inertia of the triangle abo\lt an axis
50 cm. (a) Calculate the time period vf one revolution. (b) pcq)cndicul ar to the planc of the tri,mgle and passing
What is the tension in the string? (g ~ 1Om/ s 2) through (a) its centre of mass, (b) onc orits vertices.

[Ails. (a) 2.2 see, (b) 2.12 NJ A y


31. A particle of mass 100 g is suspended from the end ofa ,
weightless string of length 100 em and is allowed to s\ving

L
in a vertical plane. The speed of the ma:~s is 200 cm/scl.:
,:,~/
°
when the string makes an angle of8 = 60"with thc vertical.
Detcnnine (a) the tension in the siring at ~ 60"and (b) thc
speed of the particle when it is in the lowest positioll. 8 -
__
.//-' i -----_ .
I

-~C
~

(g = ·9.8m/s 2 ) (a) (b)


[Hint: (a) T = (mv 2 I r)+ mg cosO(b) Apply conservation OfCllCrgyj
[Ans. (a) 0.89 N, (b) "\1!J:8 '" 3. 71 mlsj " 00
,\
32. State whether the following statement is true or false: "A \'0
" , h
simple pendulum with a bob of mass 111 swings with an """'\'\f
angular amplitude of 40". When its angular displacement is "\ \ "
\/
20", the tension in (he string is greater than mg cos 20"." /'
"
[Hillt: Prove as in solved Problem 27. T '" mg(3eos8-2eosOo)'
(e)
He re 00 '" 40 0 and e '" 20°, so T '" mg cos 20 0 + 2mg (.:os20°
--eos400» II/geos200 J Fig. 7.193
[Ans. True]
[Hint: (a) I '" 3(lG + Ma 2) with fG '" ML2/l2 and a = .!.Lsin60o [Hint: If the reel falls a vertical distance h (by
3 rolling), conservation of ME requires
2
(b) I:: 'I + 12 +13 with]l '" 12 :: ML2/3 and 13 = fG + Ma with Mgh "'!.. Mv 2 +!.. lro 2 with v '" Roo so that
2 2
fG '" ML2/12 and a "" Lsin600]
~v2[1 + (I / MR2)] '" gh; differentiating this
(Ans. (a) !...ML2, (b) ~ML21
2 2 equation with respect to t with (dh / dt) '" v and
36. Two uniform, thin identical rods each of mass M and length (du / dt) = a, we get the desired result.
Lare joined so as to form a cross as shown in Fig. 7.193 (b). Alternative Solution: For translatory motion of
Find the moment of inertia afthe cross about a bisector line reel
AC. Mg-T=Ma ... (i) Fig. 7.194
and for its rotation T '" Ia with T '" TR
(Hint:
and a '" aiR i.e.,TR '" I(a/ R) ...(ii)
[Ans. ML2/12l Eliminating T between Eqns. (i) and (ii), we get the desired
result.]
37. A thin rod of length Land mass M is suspended freely at its
end as shown in Fig. 7.193 (c). It is pulled aside and 40. A length Lofflexible tape is tightly wound on a cylinder. It
allowed to swing like a pendulum. When it passes through is then allowed to unwind as it rolls down a steep incline
its lowest position it has angular velocity (0. (a) Calculate its that makes an angle 8 with the horizontal, the upper end of
kinetic energy as it passes through its lowest position. (b) the tape being tacked down. Show that the tape unwinds
How high does its centre of mass rise above its lowest completely in a time,
position? t=~(3L1gsine)
[Hint: (a) R.KE '" !..I0)2 with I '" ML2/3; (b) By conservation of (Hint: This problem can be solved by two different approaches,
2 viz., (I) By cOl/servation ofmechanical energy: Repeat example of
ME loss in R.KE OIro 2 ) '" gain iu PE'" (Mgh)] rolling motion on an inclined plane dealt in § 7.4 (c) to prove
t '" (I / sin8),j(2I!PI g) Here Ii '" Lsin8andp '" I+!.. =~. (2) For
2 2
[Ans. (a)!..ML2ro 2 ,(b) '" (Lw)2/6g] translatory motion MgsinO- f '" Mawhile for rotational motion
6
38. A small h lid marble of mass m and radius r, rolls without 1=10., i.e., fr"'(~Mr2)(alr). Solving these for a, we get
slipping along a 'loop the loop' track. (a) From what
minimum height above the bottom of the track must the (j '" ~gSin 850 that 1 = .j(2L1 a) 1
marble be released in order that it just stay on the track at
the top of thc loop? (R »r) (b) If the marble is released 41. Show that <I cylinder wil! slip on an inclined plane of
from height 6R above the boltom of the track, what is the inclination e if the coefficient of static friction between
horizontal component of the force acting on it at point ([! plane and cylinder is less than ~ tan 8.
(Fig. 7.70) 3

[Hint: (a) In case of rolling sphere as KE '" .2. mv 2, So (a) for [Hint: Repeat solved Problem 47 with J '" !..mr2)
10 2
'looping the loop' as ull <':,fir, by conservation of energy 42. A string is wrapped around a cylinder of mass M and radius
R. The string is pulled vertically upward to prevent the
between P and H, mgh '" .!...m(.[iiii + 2mgR: (b) By
10 centre of mass of the cylinder from falling as the cylinder
unwinds the string. (a) What must be the tension in the
conservation of ME between P and Q mg6R ",.2. mUZQ + mgR so string? (b) How·much work has been done on the cylinder
10
that PQ'" mv~1R =(50/7) mg] until it has reached an angular speed oo? (c) What is the
length of the string unwound in tbis time?
[Ans. (a) 2.7R (b) =7mg]
[Hint: Repeat solved Problem 65 with <P = 90"]
39. The moment of inertia of a reel of thread of radius Rand
mass M about its axis is I . If the loose end of the thread is [Ans. (a) Mg, (b) .!.M{iR 2, (c){,iR 2 /4g]
4
held in hand and the reel is allowed to unroll itself while
43. A uniform solid cylinder of radius rand mass III is given an
falling down under the action of gravity, show that it fans
initial (clockwise) angular velocity 00 0 and then lowered on
down with an acceleration
to a flat horizontal surface. The coefficient of kinetic
g
a = - - - " -;- friction between the surface and the cylinder is 11. Initially
1+(J/MR2) the cylinder slips as it moves along the surface, but after
time t pure rolling without slipping begins. (a) What is the [Hint: By conservation of AM, 00 "" 000 /2, i.e., the angular
velocity of centre of mass at that time? (b) What is the value velocity oCthe spinning disc changcs from 000 to 00r/2.]
oft? [Ans. (3Roor/8~g)]
[Hlne: See solved Problem 64. Here rotation is converted into
translation, so angular velocity will decrease while linear velocity 48. A toy car of mass m can
will increase, i.e., travel at a fixed speed. It
v == 0+ at ... (i) moves in a circle on a
oo=coo-aJ .. ,(ii) A ,
--------- horizontal table. The string
is attached to a block of
However,
mass M that hangs as shown
a = (f I m) = (j.lmg I m) "').tg in Fig. 7.198. The coefficient
Fig. 7.198
And of friction is 11. Show that
't
u=-=-=--=-
Ir IJmgr 41g the ratio of maximum radius to the minimum radius is given
Fig. 7.195
I I .!.m? r by
2
M+ll m
Now for rolling v = reo, in the light ofEqns (i) and (ii) gives
M ~m
Ilgt '" r( 000 - 2/lgt / r)
1 49. A truck initially at rest with
i,e., t = (rmo /3j.1g)and sov = at:: '31'Wo] a solid cylindrical roll of
paper, moves forward with
[Ans. (a) ..!..rroo. (b) t = (rma 1311g)] a constant acceleration a.
3
The cylindrical roll is lying
44. A particle of mass mis projected at time t = Ofrom a pointP parallel to the edge of the Fig. 7.199
on the ground with a speed v 0' at the angle of 45 0 to the truck at a distance d. Find
horizontal. Find the angular momentum of the particle at the distance S which the truck travels before the roll moves
time t = (vo / g ) about point of projection P. offthe edge of its horizontal surface. Friction is sufficient to
[Hint: See solved Problem 45 in Chapter 6] prevent slipping.
[Ans. (0.35mv~ / g) aiong negative z-axis] [Ans.3d/2j
45. Two beads (each of mass m) can move 50. A unifonn thin rod oflength L, mass m is lying on a smooth
freely in a frictionless semicircular horizontal table. A horizontal impulse Fis suddenly applied
wire whose rotational inertia w.r.t. the perpendicular to the rod at one end.
vertical axis is I. The system is rotated (a) How far does the rod travel during the time it takes to
with an angular velocity roo when the make one complete revolution?
beads are at a distance rfrom the axis.
(b) What are the translational, rotational and total kinetic
What is the angular velocity of the
energies of the rod after the impulse'?
system when the beads are at a distance
mg n: 2 I F2 3F 2
(rI2)from the axis? (liT 1990] [Ans. s '" .- ; - - ; - _.j
Fig. 7.196 32m 2 m
[Hint: Apply conservation of angular
momentum] 51. A flywheel \'I'::h an initial angular velocity roo' decelerates,
due to forces whose moment, relative to the axis, is
[Ans. (/ + 2mr2)ooQ / [I + (mr 2 /2)]
proportional to the sq '!1re root of angular velocity. Find the
46. A small bead with a mass 100 g slides average angular velocity O'l'.::r the lotaitime ofdeceleration.
along a semicircular wire with a radius [Ans. wav "" 000 13]
of 10 cm that rotates about a vertical
52. Five identical cylindrical drums rotate around their axis
axis at a rate of 2 revolutions per
parallel to their lengths. A wooden board of mass M is
second, as shown in Fig. 7.197. Find
~!?:,;ed gently on these drums. The length of the board is
the values of e for which the bead will
enough to rest on all the five drums. If each drum is of
remain stationary relative to the
radius R and mass Me and rotates with injtial angular
rotating wire.
Fig. 7.197 velocity roo and if 11 is the coefficient of friction between
[Ans.51.6 G]
the drum and the board, (a) how far does the wooden board
47. A unifonn circular disc of radius R is placed on a friction- move before it starts slipping on the drum? (b) what is the
less horizontal plane. Another identical disc rotating with final velocity of thc board after slipping has stopped?
angular velocity roo is gently placed on top of the first disc.
Calculate the time in which both disc acquire same angular
2
R 002 (b) (MR
[Ans. (a) ~j;
2p.g
C
Mc+ 2M
1
(0)0 J

velocity. Coefficient of friction between the discs is fl.


53. A unifonn cylinder rolls A force exerted by the floor on the rod in this position and the
from rest down the side of coefficient ofstatic friction when slipping occurs at €I "" 30".
a trough whose vertical
dimension y is given by
T
y,
[Ans. N =: :g (4 - 3sin2 9),1! .= 0.4]

equation y = k · x 2 . The
cylinder does not slip
J ______ _ 58. A uniform rod of mass m and
length l is lying on a smooth .,
from A to B but the horizontal table. An impulse (P)
surface of the trough is is suddenly applied to one end r;;;;;;;;;;1 P
frictionless from B to C. of th~ rod, perpendicularly. '; I
How far will the cylinder ascend towards C! Under the How far does the rod travel, Fig. 7.203
same conditions, will a unifonn sphere of same radius go while it makes one complete
farther or less far toward C than the cylinder? revolution? Find the translational, rotational and total
kinetic energies of the rod, after the impulse.
[Ans. )12 = 3: y]> In the case of sphere J'2 = ~ Yt. Hence it travels
3 7 [Ans. S =: ft/3,p2 12n1,3p2 /2m and 2P2/ m]
further.}
59. A ball of mass mis thrown at an angle a to the horizontal,
54. Starting from rest at top a .... N
/ with an initial velocity u. Find the time dependence of the
,
small sphere rolls without
slipping off a fixed sphere.
At what point will the small
0
;(----
' / ',,
/
- -. magnitude of the ball's angular momentum vector, relative
to the point from which the ball is thrown. Find the angular
a/~ momentum M at the highest point of the trajectory, if
sphere leave the surface of A ,, m::: 130g, a = 45 0 and u ::: 25m/sec. Air drag is negligible.
the big sphere? // Mg
-+ Al 2 A 2
[Ans. When the line joining [Ans. L =: - k(- mugt CCl:la.), wherek=:unitvector; 37kgm Is]
2
centres of two spheres makes an Fig. 7.201
angle of 54" to the vertical.] 60. The angular momentum (M) of a particle, relative to a
55. (a) A marble of mass mand radius a rolls without slipping certain point (0), varies with time as M ::: a + bl 2 where a
on an inclined pIane, making an angle €I with the and b are constants and a is perpendicular to b. Find the
horizontal. Calculate the acceleration of the centre of moment of the force N relative to the point 0, acting on the
mass of the marble.
(b) If the marble starts with an • particle, when N makes 45° with M .
[Ans. u,../aib 1
initial velocity v directly up
the inclined plane, how long , 61. In the Fig. 7.204 a lawn roller of mass M
and radius R is being pulled by a horizontal
will it take for the marble to Fig. 7.202
force at its centre. Show that the minimum
return to its starting point?
5. 140
coefficient of friction needed to prevent Fig. 7.204
slipping is ~.
[Ans. (a) --g sm9; (b) - - .- ]
7 5gsm9
3Mg
56. A uniform rod AB of mass 2m is freely joined at B to a
62. A horizontal weightless rod of length 3/ is suspended on
second rod BC of mass m The rod lies on a smooth
two vertical strings. Two loads of masses ml and m2 are
horizontal plane at right angles to each other and the
suspended symmetrically at equal distances from each
impulse! is applied to AB at A in a direction panllel to BC.
other and from the ends of the strings. Find the tension T of
Find the initial velocity of Be and the kinetic energy
the left string at the instant when the right string is cut.
generated.
[Ans. Initial velocity of BC '" ~i; kinetic energy generated
3m
[Ans. T = ( m]m2
nil+ 4m2
19l
51 ' 63. A block of mass ml is
= -- ]
6111 attached to a cord of length
57. A tmiform rod of mass m and length 2a stands vertically on L 1, which is fixed at one end.
a rough horizontal floor and is allowed to fall. Assuming The block moves in a
FIg. 7.205
that the slipping has not occurred show that when the rod horizontal circle on a
makes an angle e with the vertical, (jJ2 = 3g (I-cos e) frictionless table. A second block of mass m2 is attached to
2a the first by a cord of length L2 and also moves in a circle, as
where (jJ is rod's angular velocity. Also find the normal shown in the Fig. 7.205. If the period of motion is T, find the
tension in each cord.
[Ans. Tl = [m1/1 + m2(L, + Lz)](~ r T2 ,. m2<L, + 1:z{~n:rl 69. A heavy particle of mass mmakes ncomplete revolutions in
a smooth circular tube in a vertical plane. Its greatest speed
is n limes Ihe least speed. Prove thai the pressure in the

n:n + I}
64. A hollow sphere of radius R '" 0.5 m revolves about its
vertical diameter with an angular velocity <.0= 55 -I. Inside tube when the particle is moving vertically is 2mg (
-I
the sphere at the height!!.. a small block revolves together
2 70. One end of a light inextensible string of length a is fastened
with the sphere (g = lOm/ s 2} What is the least value of to a fixed point and a particle is attached to the other end.
the coefficient of friction to realize the situation? The particle is held so that the string is taut and horizontal.
[Ans.~ =0.225) It is then projected vertically upwards with velocity u.
Given that the string slackens when it is inclined at an angle
65. A particle P is slightly displaced from rest at the highest of 30° to horizontal, find u in terms of a and g.
point A of a fixed smooth sphere of radius r and centre O.
Find (a) the angle between OP and the upward vertical at
the instant when the particle leaves the sphere, (b) the
[Ans. u= p~a]
distance between the lowest point B of the sphere and the 71. A simple pendulum is started to make complete
point where the particle strikes the horizontal plane through revo lutions. If WI and ~ be its greatest and least angular
B. velocities, T, and T2 the greatest and least tensions in the
string, find the angular velocity and tension when the
[ADs.9 ~40011'; 1.46rJ

66. A string oflength rhas one end attached to a fixed point and
a
pendulum makes an angle with the vertical.

the other supports a particle. When the particle is hanging [Ans.", -


freely it is given a horizontal velocity J(kgr) where k is a
72. (a) Find the greatest angle through which a man can
constant. Ifthe particle subsequently strikes the filC.ed point,
oscillate on a swing, the ropes of which can support
find k. Also the angle at which the string slackens.
twice his weight.
[Ans. k = 2+ ,J3,cos9 '" 11 J3]
(b) If the ropes are strong enough and he swings through
67. A bag is gently pushed off the top ofa wall at A and swings 180° and if v be his speed at any point, find the tension
in a vertical plane at the end of a rope of length /. in the rope at that point if m is the mass of man and Jis
(a) For any position B of the bag ......t,;A~-L-;;ij the length of rope.
determine the tangential e . 311w2
(Ans. (a) angle 120°, (b) tensIOn - 2t J
=0

component of its acceleration \\"'"


a l and obtain its velocity.
73. Four identical rods of mass M and ~I
(b) Determine the value of e for B ··· length L are joined to form a rigid Dr----.;C
which the rope will break square frame ABCD lying stationary
knowing that it can withstand Fig. 7.206 on a smooth horizontal floor. Two
maximum tension equal to particles each of mass m hit the
twice the weight of bag.
vertices A and C with velocities 2v and
[Ans. (a) (I, :: e e= 41 .8"]
glcose. ; v '" J2glsin (b) v respectively as shown in the Fig. A i----' 8
68. A point moved along a circle of radius r"" 10 cm with
constant tangential acceleration (I,. Find the tangential
7.207. Aftcrthe impact, these particles
stick to the frame. Find the velocity of
l~v
centre of mass of frame and its angular Fig. 7.207
acceleration (/1 of the point if its velocity is v =79.2 cmls at
the ~nd of the fiOh revolution after motion has begun. velocity after Ihe impact
[Ans. 0.1 mls 2) ImJ 9mll
IAns. VeM '" 4M + 2m ,W '" 2/(4M + 3M» )
Simple Harmonic Motion
§ 8.1 Simple Harmonic Motion that every oscillatory motion is periodic but every periodic
(AI Harmonic, Oscillatory and SHM motion is not oscilfatory, e.g., the unifonn circular motion of
a conical pendulum or the motion of electrons around the
Any motion which repeats itself after regular interval
nucleus in an atom are periodic but not oscillatory while the
of time is called periodic or harmonic motion and the time
motion of a liquid in a V-tube or that of the free end of a
interval after which the motion is repeated is called lime
clamped strip are oscillatory and hence periodic.
period. A periodic motion can be either rectilinear or closed or
-Q

""Q"";Q ~... ,
',~--- --
-'
(a) (b)
k

(c)
k
@
:( ~ ~:x
!
+ 0 + -q
+ ,: + ~

-
(d)
~

MOIlon 01 MOlion 01 Motion of Some oscillatory motions


hands of earth around a piston
Fig. 8.2
a clock the sun in a cylinder
(a) (b) (c) If ill case ofoscillatory motioll Q particle moves back alld
forth (or up and down) about afixed point (called equilibrium

V 'U
positioll) Ihrough a/oree or torque (called restoring/oree or
torque) which is directly proportional to the displacement but
opposite iff direction, the motion is called simple harmOffic and
abbreviated as SHM. Further if the displacement in case of
Motion of Motion of a SHM is linear, the motion is called 'Linear SHM' and if the
a ball in liquid in a displacement is angular it is known as angular SHM. The up
a bowl U-Iube and down motion of a partially submerged floating body or of a
(d) Some periodic motions (e)
mass attached to a spring are examples of linear SHM while
the motion of a simple pendulum or physical pendulum or
Fig. 8.1
torsional pendulum (twisting and untw isting motion of a body
In case of periodic motion , force is always directed towards a suspended from a wire) are examples of angular SHM .
fi xed point which mayor may nOI be 011 the path. of motion. o
If in cas\! of periodic motion , the particle moves back and
10rlh (or to <lnd n·o) on the same paTh, the motion is said to be
oscillatory or vibratory, i.e., oscilhllOf)' (or vibratory) motiOIl
/,0 /
, " 0\,
" ,,
\L
is a constrained periodic motion betweell certain preci.~ely
~~>_,'. . .r--~T.\ "
," T \,,
""
fixed limits. The world is ful! of osci llatory Illotions. The ,
moliOIl of the balance wheel ofa watch, the motion of piston in
an aUlOmob ile engine, the mot ion of the needle of a sewing
machine or the motion of a ball in a bowl are all oscillatory
(a) (b) (c)
motions. The motion of molecu les in a solid around fixed
Conicat Spring Simple
latt ice s ites. the iJi!ll>chlal motion of aloms withi n molecules, Pendulum Pendulum Pendulum
vibrations of the prongs of u tunillg fork and the motion of
electric <lnt! magnetic fields in all el ectromagnetic wave afC T. 2nj!f T .. 2R~
few othef examples of vibratory lllotion. 1t is to be noted here
which on integration gives [~ r = _00
2
i + C1

c Nowifaty=A,(dyldt) =O, C 1 =ro 2A2


Extreme Equilibrium Extreme
Position Position Position
e i~) EOJ'. ) < e-A----l F P A
@}----------.---<-<I>---€»
(dJ
-(eJ
i 1-Period i
: - - A -...:-- A--l
(aJ
, dyldt .. O °f-Y-m
,
(bJ
PhysIcal Torsional Fig. 8.5
Pendulum Pendulum
T-2nJ mgl I T - 21t fl So, the above equation becomes:

dy ] 2 =_ro2l+ro2A2 i.e"
[ dt
dY=~(A2_l) .... (ii)
Different types of Pendulum dt
Fig. 8.3
or dY/~(A2 - i) =rod'
Regarding SHM it is worth noting that:
which on integration becomes
(1) In case of simple harmonic motion (linear or angular)
the time period is independent of amplitude. sin-'(}iA) = (ro' + ~), i.e., y=A~n(ro'+~) .... (iii)
(2) The necessary and sufficient condition for a motion to Here it is worthy to note that Eqns. (i), (ii) and. (iii) give
' be simple harmonic is that the restoring force (or acceleration, velocity and displacement in case of linear SHM
torque) must be linear, i. e., respectively in terms of constants A, 00 and $.
F=-ky or 't=-c9
Note: Similarly for angular SHM as necessary and sufficient condition
(Linear SHM) (Angular SHM) is. = -c€land as. = 10. = Id 2a/dt2, the equation of ASHM will
(3) If a point P moves with uniform angular velocity in a b,
circle, the foot of the perpendicular drawn from P on a 2 ,
diameter executes linear SHM.
i. e., with ro =1
(4) Every periodic motion can be which on integration gives
N P
resolved into a number of A cia = roJa2 _ 2 a and
simple harmonic motions with dt 0

the help of 'Fourier Analysis' rot


and any number of simple o (C] Characteristics ofSHM
hannonic motions can be Here we shall discuss the physical meaning of constants
combined to give the resultant A, co and $ used to describe simple harmonic motion.
motion. [See solved Problems (a) Amplitude A: It is defined as the maximum (positive
FIg. 8.4
8 and 9.J or negative) value ofdisplacement from equilibrium position.
(B) Equation of Simple Harmonic Motion Since maximum and minimum values of sine functions are
The necessary and sufficient condition for a motion to be respectively +1 and -I, the maximum and minimum values of
simple harmonic (linear) is yin the equation y = A sin(oo! +$ )are respectively +A and -A;
A is called amplitude. Furthermore if S is the span of SHM,
d'y 2y amplitude A will be (812).
F =-ky, i.e., m -~ =-ky aSF =ma =md ]
d,' [ d,' (b) Time Period T: The smallest time interval after
which the oscillation repeats itself is called the time period T
d' 2 k
or ~=-o?y with 00 =- .... (i) of the oscillation. If y is the displacement of a particle
d,' m executing SHM at any time t,it will achieve the same position
2 again for the first time if time is advanced by (21t/oo), i.e.,
or 2[dY ] d y = - o? Y[2 dY ] [ multiPlYing both sides by2 dY ] [' = t + (21t/oo) so that
dt dt 2 dt dt
y'=Asin(oot'+$)
or Y]' =-ro'-"-(i)
-"-[d = ASin[oo(t + ~ )+$ ] = Asin (oot +$) = Y
dt dt dt
In other words, the displacement repeats itself after a time [D] Some Physical Quantities of Interest in SHM
interval of (2roro), so that T = (2n/ro). (a) Displacement: In SHM if time is considered from
(e) Frequency f: Frequency in SHM is the number of equilibrium position, displacement y at any time t is given by:
oscillations completed in a unit time interval. Therefore, by y=Asinrot with T=(21t1ro)=(l/f) .... (v)
definition, frequency is the reciprocal of time period, i,e., Regarding displacement it is worth noting that:

00= ~
lro (I) Its direction is always away from the equilibrium
f=-=- or 00 = 21[/ = -2 nwith
· - .... (iv)
T 21t T m position either particle is moving away from or is
coming towards the equilibrium position. (See Fig. 8.7)
Quantity co is called angular frequency of the motion. The
Equilibrium Extreme
SI unit ofT is s (second), of f is Hz (hertz) and afro is rad/s. POslt~~~ ___~~ __ ~-~~~ltlon
The period or frequency is the characteristic of the system o a P A
executing SHM and is determined from its elastic and inertial
properties. Furthermore out ofT.! and wonly one is usually ia)
given and the others may be calculated by using Eqn. (iv).
Equilibrium y Extreme
(d) Phase: The B A Phase Position _ v Position
r -- -------- -~ ~-- ------i- - 0 .- - --------~----
argument (rot + cjl) of the ,, ,, ,, o a P A
sine function is called the : : :
phase of the motion. The
,r-------------i-----------1>-
, , 7tI2
ib)
Fig. 8.7
phase of an oscillating : : :
system at any instant is its ,~----------+_$-----------+- (2) 1ft is given it can be calculated and vice-versa, e.g., if
, , 11: y = (AI2) , (Al2) = A sin(2.tIT), i. e., , = (Til 2) and if
state as regards its i ii
position and direction of *"------------+-----------+- 37t12 ,= TIS, y= A sin[2.(TIS)IT] =AI,,!2 =0.7 A, i.e., in
motion at that instant, i ii travelling half the amplitude from the equilibrium
e.g., ifthe phase is zero at L ___________ ~--------L 2ft position, the particle takes (1 / 12)th of its time period
a certain instant then Fig. 8.6 while in time (1I8)th of the period it travels
from equation ofSHM.
(AI-fi)'::.0.7 times of its amplitude.
i.e., y= Asin(rot +$), y =O
dy (b) Velocity: In case ofSHM when motion is considered
and v=-=Aro from the equilibrium position,
dt
y = Asinrot so v=(dyldt)=Arocosrot .... (vi)
This implies that the particle is crossing the equilibrium
position and if (rot + $) = (ro'2). or V= AorJl-sin 2 rot =onlA2 - i
y=A and v=o [as from Eqn. (i)sinro' = (ylA)] .... (vii)
i.e., the particle is at extreme position. Fig. 8.6 shows the state Eqns. (vi) and (vii) give the velocity of a particle
of a particle executing SHM in different phases. executing SHM at any instant and position respectively. From
(e) Phase Constant$: The constantq. in the equation of these equations it is clear that:
(1) v will be minimum when y=max =A andv rnin =0,
SHM is called epoch or phase constant or initial phase. This
i. e., in SHM velocity is minimum at extreme position
enables us to find the position from where time is considered in
and is zero.
SHM, e.g., if we consider the motion from equilibrium
position, i.e.,att = 0, y = 0, the equation y= Asin(rot + $ ) gives (2) v will be maximum when y = min =Oandv rnax = Am,
O= Asin(roxO+q.), i.e., sinq. =0 or $ =0 i. e.,in SHM velocity is maximum at equilibrium
position and is Am.
i.e., if time is considered from equilibrium position, initial
phase is zero and so equation of motion becomes y = A sin rot. (3) Direction of velocity is either towards or away from
equilibrium position. (See Fig. 8.7)
And if we consider the motion from extreme position, i. e.,
(c) Acceleration: In case of SHM when time is
at t =0, y = A. then from equation y= Asin(rot +$),
considered from equilibrium position as,
A=Asin(roxO + $), i.e., sin$ = l or 41 = 1t12 y=Asinrot i.e., v=(d;ldt)=Arocosrot
i.e., if time is considered from extreme position, initial phase is 2
dv d y 2 . 2
(1tI2) and equation of motion will become y= A sin [rot + (1t/2)] so a =- =--=-Aro smrot=--<o y
dt dt 2
= Acosrot.
[asy=Asinrot] .... (viii)
From this -it is clear that: (d) Energy E: In case of SHM as v=roJ(A 2 - i),
2
(I) In SHM as IAccelerationl (= 00 y) is not constant so kinetic energy
equation of translatory motion cannot be applied.
K=!mv 2 = l mro 2 (A2 -
2 2
i) .... (ix)
"(2) IAccelerationi will be minimum when l yl=min =O
and sol ace lmin =0, i.e., acceleration is minimum (= 0) while as F = -ky, potential energy
at equilibrium position where IvelocityJ is maximum
U =-Idw =-1F dy =1' kydy=lky' .... (x)
(=roA). o 2
(3) IAcceleration j will be maximum when Iyl = max = A So mechanical energy
and so I acc1max =(Jl A, i.e., acceleration is maximum
E=K+U=1. mro 2A 2 [asro2 =klm] .... (xi)
at extreme position where velocity is minimum (= 0). 2
(4) Acceleration is always directed towards the Eqns. (ix), (x) and (xi) are the desired Eqns. and from
equilibrium position and so is always opposite to these it is clear that:
displacement. (Fig. 8.7) ( I) With increase in y, i.e., when particle moves from
Fig. 8.8 and table below give comparative study of eqUilibrium position to extreme position, PE
displacement. velocity and acceleration in SHM. increases, KE decreases and ME remains constant.
(2) At equilibrium position as y = min = 0, so
Equilibrium
S.No. Physical Quantity Extreme Position
Position KE = max = !mro 2A2 =E and PE = min = 0
2
1. Displacement Minimum mO Maximum = A
y = A sin wt While at extreme position as y::: max = A
2. Vel$ ; Maximum = oM Minimum =0 KE = min =Oand PE = max =!mro 2A2 =E
v= A'-; 2
The variation of KE, PE and ME as functio n of
3. Acceleration Minimum =O Maximum ,.. 002A position are shown in Fig. 8.9 (a).
14" 002Y (3) As in SHM y= Asinrot
KE, K = !mro 2 (A 2 _ i)
=!mro 2 A2 coo 2 rot
2 2

.A =.! mro 2A 2 (I + cos200t)


4
t;;----'k-- --)';----C'k----;I;'" Displacement
,:2T y _ Asinwt PE, U =! mro 2 i = 1. mro 2 A 2 sin 2 rot
-A ,, --~ --.
2 2
,,
= ! mro 2 A 2 (l-cos2ro/)
4
Velocity
:2T v _ Aw cos WI so that K = ~E(1 +cosro' t)
,
,- Am
:,,
-- ----.~
,,, . . -
U =~ E(I - cosro' t)
....
,,,
-. - .~ -.-
, _.- :,....
,,
and

:, ,,
, Acceleration
4
with ro' = 2roand E = mro 2A2

:,, T :2Ta_ - Aw2CO$(I)I


,,, i,e. , in SHM, K.E and PE vary periodically· with
, , , double the frequency of SHM (i. e., with time period
.. i--- -.- .-. ~-.- _.J • • • _ • • • •• ! . . .

~T T ~'T 21 ___ Time T' = TI2). The varintions of KE, PE and ME as


Fig. 8.8 functions of time are shown in Fig. 8.9 (b).

* The variation ofKE and PE in SHM is periodic with periodicity (TI2) but itself it is not simple hannonic as (d 2Kld/ 2 ) or (d 2Uldt 2 ) is not <c -K or -U
respecti vely.
K,UorE
Answer: No, not at all. In the limit of small amplitudes
-,-_....,,.,.,,,_-.-cE .. ~ moo2A2 only those oscillatory motions can be treated as simple

,,L..PE", !moo y2
2 harmonic for which the restoring force (or torque) becomes
linear. For example, the oscillatory motion of a simple or
spring pendulum or motion of atoms in a molecule becomes
simple harmonic in the limit of small amplitude as restoring
force (or torque) becomes linear while in case of oscillatory
(a) motion of a ball between two inclined planes or two perfectly
K,UorE elastic walls, the motion does not become simple harmonic
even for vanishingly small amplitude.
Question III. What would happen to the motion of an
oscillating system if the sign of the force term in equation
F = -k x is changed?
Answer: If the sign is changed in the force equation,
!T T acceleration will not be opposite to displacement and hence
(b) the particle will not oscillate, but will accelerate in the
Fig. 8.9 direction of displacement. So the motion will become
(4) In SHM, E = (tl2)mw 2 A 2, i.e., energy is directly accelerated translatory. However, equations of motion cannot
proportional to the square ofamplitude and can also be applied to analyse the motion as acceleration (= ro 2 y) is not
be expressed as constant.
Mathematical analysis shows that in this situation both
E =K +U =!m[dy ]2 +.!.ki velocity and displacement wjJl increase exponentially with time.
2 dt 2
This equation, i. e., conservation of ME in SHM can be Note: IfF = ky, m~1 = ky, i.e, c;;: = ro2y, with 002 k
used to find time period, since E is constant (dEIdt) m
= 0 that the above equation reduces to dv dy 2
or v dV =ro2y, as dy=v
or dy· dt =ro y, dy dl
2 2
-d -y +-k d Y 2
y = O, i.e., - - +00 y=Owlthw =-
. 2 k
dt2 m dt2 m orvdv = ro 2 ydywhich on integration gives iv2 = iro2 ; + c.
[See solved Problems 32 and 33] Now if at Y'" O,V = O,C will be zero, so v = roy
Question I. Which of the following fUJlctions are Ca) or (dyldt) = ooy,i.e.,dyl y == mdt which on integration gives
logy= rot + D
aperiodic (b) periodic but not simple harmonic (e) simple
Now ifat t = O,y= yo,D '" 10gyo so that
harmonic: (i) sin2rot (if) 1 + cos 200t (iii) a sin rot + b cos rot
(iv) sin rot + sin 2mt + cos 2wt (v) sin 3 rot (vi) log (I + rot) (vii) log(y! Yo) = rot, i. e., y = yoeW1 and v = yoroeWl

exp (--{()/) ?
Question IV. In case ofsimple harmonic motion (a) what
Answer: (a) Functions (vi) log(l +oot) and (vii) exp (-<0/)
fraction ofto tal energy is kinetic and whatfraction is potential
increase (or decrease) continuously with time and can never when displacement is one half of the amplitude, (b) at what
repeat themself so are aperiodic. displacement the kinetic and potential energies are equal?
(b) Functions (ii) (l +cos2rot), (iv) sinrot+sin2rot+ (EAMCET 1993)
cos2rot and (v) sin 3 rot are periodic [i. e.,f(t + T) = f(t)] with Answer: In case of simple harmonic motion
periodicity (woo), (2wro) and (2wro) respectively but not
=lmw 2 (A2 =lmro2 i =lmro2 A2
simpiehannonic as for these functions (d 2 yl dt 2 ) is not oc - y. K
2 - i),U
2 andE
2
(c) Functions (i) sin 2rot and (iii) a sin rot + bcos rot, i. e.,
(a 2 + b 2 )112 sin[wt + tan -1 (bl a)] are simple hannonic [with Cal SolK =KE =[l _ L]=
A2
[l-~]=
4
.34 [asv. = Agiven]
2
time period (row) and (21[/ro) respectively] as for these
(d'yldt 2 )oc-y.
Ip = U =L=[CAI2l]2 =_
Question II. Can every oscillatory motion be treated as E A2 A 4
simple harmonic motion ill the limit of small amplitude?
(b) According to given problem
K=U, i.e., ~mro2(A2 _ y2)=~mro2y2
or 2:t=~ [asOl=2;=24
lt
J. i.e., t=~sec
Alternative Solution: As the time period is 4 sec, time
taken from equilibrium position to extreme position =(TI4) '" 1
i.e., 2y' =A' or y=AI.J2(~0.7A)
sec. Now time taken to move from y =O to y=!A will be
Problem 1. If two SHMs are represented by equations 2
YI =1Osin[3", + (rrl4)] and Y, =5[sin(3<1)+./3cos(3<1)],
find the ratio oftheir amplitudes. [MNR 1990)
given by ~ = ASin( 2ltf ). i.e., sin-l(~)=e4ltt)
Solution: As Yz =5sin(31tt) +s.Ji cos(3nt) n n I
or -=- 1 i.e., (=-sec
62' 3
So if S=Acos$ and S.J3 = Asinljl
So time taken to travel from (AI2) to A = 1- (1/3) = (2/3)
i.e., if A = ~5' + (5./3)' = 10 and tan~ = 5./3/5 =./3, sec and this will be equal to the time taken for returnjoumey,
i.e., 4> = 1tI3 i.e., from A to Al2.
the above equation becomes Problem 4. A particle is executing SHM with amplitude A
Yz = Acosljl sin (31t/) + A sin~ cos3nt and has a maximum velocityv o' (a)At what displacement will
its velocity be (vo /2)? (b) What is its velocity at displacement
=Asin(3nt+/jI)
AI2?
i.e., Yz = lOsin[31tt+(n/3)]
Solution: In SHM velocity at any position y is given by
Al 10
So
,-
A-IO'
i. e.,
v=onlA2_i
Problem 2. Two particles execute SHM of the same So vwiJI be maximum when i = min = 0, i.e., Vo = Am.
amplitude and frequency along the same straight line. They
pass one another when going in opposite directions, each time So (a) Given that v = tvo = t Aro
their displacement is halfoftheir amplitude. What is the phase
difference between them? so tAro = ro~A2 - i,
Solution: In case of SHM, y = A sin(rot +$) and as here
y=A I2,so i.e., y = (./312) A ~ 0.866A
y
Asin(rot+Ijl) = Al2 (b) Given that y = Al2
or rot+~=o=sin -l(~) so v= ro../rA"""-_-(A-I-2)-,0-,,
-,,::\;---;:';~_
~1116
wI i.e., v = (./3/2)Aro = (./3/2)v o = 0.866v o
I 5
i.e., o= - lt and -It
6 6 Problem 5. A particle of mass m moves on the x-axis as
So the phase of the two Fig. 8.10 follows: Itstartsfrom rest at 1 = 0frompointx = Oand comes to
particles when they are crossing rest at t = I at the point x = L No other information is available
each other at y = Al2 in opposite directions are 0 1 = (lt/6) and about its motion at intermediate time (0< t< l~ /fa denotes
=
O2 (S1t/6). So phase difference the instantaneous acceleration of the particle, then which of
5=5, -51 = (4n/6) = (21t13) the following statements are true?
Problem 3. A particle executes SHM of amplilude 4 cm (a) a cannot remain positive for all t in the interval
and T = 4 sec. What is the time taken by it to move from OS;ts;1.
positive extreme position to half the amplitude? (b) 10.1cannot exceed 2 at any point in its path.
Solution: The equation of motion of linear SHM is y = A (c) Ia Imust be :?: 4 at some point or points in its path.
sin(rol + ~). When time is counted from extreme position y A = (d) a must change sign during the motion but no other
at t = 0, so that A = Asin(O +~), i.e.,sin~ = I or~ = rrJ2, so the
assertion can be made with the information given.
equation of motion becomes
Solution: As the particle starts from rest and stops, so a.
y = Asin[Olt + (lt/2)] = A cos rot
Now as here y =! A,! A = A cosOl/, i. e., rot = cos -I ( !)
cannot remain positive all the time in the interval S; t S; 1. So
statement (a) is correct.
°
2 2 2
y = 4COS 2( ~ t }in(lOOOt)
Further as no other infonnation is available about its
Solution:
motion, i.e., law afforce is not known, so no other assertion
can be made, i,e., statement (d) is also correct.
or y=2(1 + cos t)sin(IOOOt)
Note: This problem should not be solved by using equations of motion
[as2cos 2 = (I +cos28)]
e
as acceleration a. is not given to be constant.
or y = 2sin1000t+2sin1000txcost
Problem 6. A point particle of mass 0.1 kg is executing or y = 2 sin 1000t + sin 1001t +sin999t
SHM with amplitude of 0.1 m. When the particle passes
[as2sin A cosE = sin(A + B) + sin(A - B)l
through the mean position, its kinetic energy is 8 x 10-3 joule.
So the given expression is the resultant of3 independent
Obtain the equation of motion of this particle if the initial
harmonics.
phase of oscillation is 45°.
Solution: Given thatA=O.lm;m=O.lkg,~=45° Note: Resolving a given complex periodic motion into component
SHMs is called "Fourierallalysis".
= (1tI4) rad, so the equation of SHM will be
y=O.lsin[rot + (n/4)J .... (i) Problem 9. Two (0, a)
Now as in SHM, KE is given by linear SHMs of equal
amplitude and
K=!mro 2 (A2 _ i ) frequencies roand2roare
2
impressed on a particle (- a/~ 0) (al./'l., 0)
which according to given problem is 8 x 10- 3 J for y=O along x and y-axes 0
respectively. If the FIg. 8.11
so 8xlO-3 =!xO.lxo}(0.12 _0 2 ),
2 initial phase difference between them is (1tI2~ find the
resultant path followed by the particle.
i.e., ro =4 radls .... (ii)
Solution: According to the given problem,
Substituting the value of ro from Eqn. (ii) in (i), we get
x=asinwt and y=asin[lrot + (1t/2)]
y=0.lsin[4t + ("'4)J
Sothat y=acos2rot [assin(9+900)=cos9]
Problem 7. A linear harmonic oscillatorofforce constant
2xlO 6 Nlm and amplitude 0.01 m has a total mechanical or y = a(l- 2sin 2 rot)
energy of 160 J. Which of the following statements are (as coslrot = 1- 2sin 2 rot]
correct? Its:
or y=a[I-2(xla)'J [asx=asinrot]
(a) maximum PE is 100) (b) maximum KE is 100}
(c) maximum PE is 160} (d) minimum PE is zero or x 2 =-!a(y-a)
2
Solution: As E = 160 J, so maximum PE will be 160 J at
the extreme position. So statement (c) is correct. Now as work i. e., the resultant path is a parabola with vertex (0, a) and latus
done by restoring force in moving the mass from extreme to rectum aJ2 as shown in Fig. 8.11.
equilibrium position Note: When two SHMs at right angles to each other act simultaneously
on a particle, the path traced by the particle is called 'Lissajous
W =lAO ky(-dy) =!kA' =! x2x to' x (0.01)' =tOO! figure' and its shape depends on amplitude, frequency and phase
2 2
difference between the two simple hannonic motions.
So by Work~Energy theorem,
Ko -K E =W=100J, i.e., Ko =IOOJ, [asK E =OJ Problem 10. A block is resting on a piston which is
Now as KE is maximum at equilibrium position, so max moving vertically with a SHM of period 1.0 sec. At what
KE = 100 I, i.e., statement (b) is also correct. amplitude ofmotion will the block and piston separate? What
is the maximum velocity of the piston at this amplitude?
Note: In this problem K + U == E everywhere but max KE ~ max PE. Solution: The block is in accelerated motion up and down
Here max KE '" (E-Uo),i.e., the oscillator has PE at the
along with the piston [Fig. 8.12 (a)]; so for its motion
equilibrium position which is called 'zero poillt energy'
(i.e.,u min '" Vo '" 160-100 '" 601 -1" 0). R=m(g±a)
The block will not separate if
Problem 8. The displacement of a particle executing
R min 20, i.e., m(g-a)20
periodic motion is given by y = 4cos2(~ t) sin (1000t). Find or g2a or g 2 amax [asa $constt.l
independent constituent simple harmonic motions. or g2ro 2 A [asamax =ro 2Al, i.e., A5glw2
S
i, e., A~~ so Amax = 0.248 m ,, ' \
4 x 1tl
/0 0 '" ,
,, ' " '
, ,,
and vrnax. = Am= 0.248 x(21t/1) "" 1.56 mls ,, ,,
L/ \
/
/'
" T
\"
\
" \,
"
, "
/
,
A
E ---------- C

B
mg
(a) (b) Fig. 8.13
Fig. 8.12
Now as in case of rotational motion:
Problem 11. A block is on a horizontal slab which is 2 2
moving horizontally with a simple harmonic motion of 2
t =la = mL (d SJ[as 2
1 = mL anda = d e]
frequency two oscillations per second. The coefficient ofstatic dt 2 dt 2
friction between block and slab is 0.50. How great can the
amplitude be if the block does not slip along the slab? So from Eqns. (i) and (ii),
Solution: The block will move with the slab due to 2
mL2 d a = - mgLa d 2S g
i.e., - = - - S
friction [Fig. 8.12 (b)J and will not separate if dt 2 dt 2 L
ma:S;fL • i.e., a:$;llg [asfL=j.WIg] 2
i.e., d e = --(()2a with (I} =¥.. .... (iii)
Now as in SHM acceleration is variable and as a < Ilg, the
dt 2 L
maximum value of a(=ro 2A) must be lesser than Ilg, i, e.,
Eqn. (iii) is the standard equation of angular simple
00 2 A Sllg or A:5 (llg/oo2) harmonic motion with time period T = (21t1ro); so here

0' A O.5x9.8 =0.031 rn=3.1 em


(2 x 2.)2
To 21tt
= . . (iv)
§ 8.2 Simple Pendulum Note: (i) If a is not small, sin 0 7- 0, the motion will be oscillatory but
A point mass suspended by a massless inextensible string not simple hannonic. It has been shown that in this situation:
from a rigid support constitutes a simple pendulum. In reality
neither point mass nor massless string exist. So we can never g ~[
T==21t - i + 2i sm
2
. 20 m
-+ZZsm
224
2
13 . 4- 0m
2
+ .... 1
construct a simple pendulum strictly according to its
(where am is the angular amplitude), i.e. ,T > To and depends
definition. A small heavy spherical mass suspended from a
on amplitude.
light inextensible string constitutes an approximate simple
(ii) Also here tension 7- mgcosO but tension (-mgcosO) ==
pendulum. (mv 2; L)as the mass m moves in the arc ofa circle of radius L.
Now, when the pendulum is pulled to one side and
released, it will swing to and fro describing an are ofa circle in This is the required result and from this it is clear that:
a vertical plane (as the string is inextensible), i.e., the (l) Time period of simple pendulum is independent of
displacement involved in the motion is angular. So the motion amplitude as long as its motion is simple harmonic.
ofa simple pendulum is angular and oscillatory. Here restoring
(2) Time period of simple pendulum is also independent
torque about S is developed due to the weight of the pendulum
of mass of the bob. This is why:
as shown in Fig. 8.13, i.e.,
(a) If the solid bob is replaced by a hollow sphere of
't' "" - mgx CE = -mgLsina
same radius but different mass, time period
[negative sign is used as direction of torque is opposite to remains unchanged.
displacementJ.
(b) If a girl is swinging in a swing and another sits
Ira is small sina = a, so with her, the time period remains unchanged.
t =-mgLa .... (il (3) Time period depends on L as T cc,fL, i.e., T2 cc L. So
i.e. , restoring torque is linear, so motion will be angular simple the graph between T and L will be a parabola while
haml0nic,
between T2 and L will be a straight line. Here L is the
distance between point of suspension and centre of (b) Assuming damping to be negligible if a simple
mass ofthe bob and is called 'effective length'. This is pendulum of density p is made to oscillate inside a
why: liquid of densitycr « p), then as thrust will oppose
(a) When a sitting girl on a swinging swing stands its weight, i.e.,
up, her centre of mass will go up and so Land mg'=mg - Th or g' =g - (Vcrg/Vp)
hence T will decrease. i.e., g' = g[I-(cr / p)] [asm=VpandTh=Vcrg]
(b) If a hole is made in the bottom of a hollow sphere so g will decrease and hence T will increase.
full ofwatcr and water comes out slowly through
(c) If a simple pendulum is made to oscillate in a
the hole and time period is recorded till the sphere
freely falling lift or a satellite,
is empty, initially and finally the centre of mass
will be at the centre of the sphere. However, as g'=g-a=O [as a =gin free fall]
water drains off the sphere, the centre of mass of So T =2nfiJO = 00. This implies, that the
the system will first move down and then will pendulum will not oscillate and will remain
come up. Due to this L and hence T first increases, where left, as there will be no restoring force.
reaches a maximum and then decreases till it (d) If a simple pendulum is in a carriage which is
becomes equal to its initial value. accelerating:
(c) If the bob is suspended by a wire, due to change in
temperature, length L will change and so the time Upwards Downwards Horizontally
period. If Ll9 is the increase in temperature then as Thcnasg'::g+a Thenasg'::g-a Then as g':: ~g2 + a 2
L=Lo(I+M8)

L= 1I=(I+M8)1I2 [, +IM8] 0 T=21t~ (g +a)


L T =2lt~ (g-a)
L T =211.
L

To fi~ 2 ~(g2 + a2)


i.e.,Twill decrease i.e.,T will increasc i.e.,Twill decrease
or L-l::.!a.6.9 i.e.,
To 2
Note : (i) In case of downward accelerated motion if a> g, the
(d) If a mass M is suspended from a wire of natural pendulum turns upside down and oscillates about highest
length L and the wire stretches by LlL due to point with
elasticity, T = 211.JLI(a g)

T =2.~L:M =2.ml+ ~]
(ii) Tn case of horizontal acceleration, equilibrium position of
pendulum will not be vertical but will make an angle
0 = tan- ! (a/g) to the vertical in a direction opposite to the
From the definition of young's modulus, acceleration. See § 6.8C (I).and § 3.3 (6).
FL MgL M Mg
y = - - = -- so - = - -«1 (e) If the simple pendulum has charge q and is
AM 1tr2M L 1tr2y
oscillating in a uniform electric field E which is:

so T =2.Vg'[(I + Mg )"2
nr 2 y
Opposite to g In the direction of g Perpendicular to g
Electric force qE will Electric force qE will Electric force qE will
be opposite to the be in the direction of be perpendicular to
oZ. [[I +! Mg ](>To ) force of gravity mg, force of gravity mg. force of gravity mg.
Vg 2 nr2y
, , qE Le.,
i.e.,g'=g _ qE
(4) Time period of a simple pendulum also depends on m
I.e.,g ::g +-;n
g'::.~g2+(qElmp
acceleration due to gravity and as T ex:: (I lJi ),with So T will increase. So T will decrease, So T will decrease.
increase in g, T will decrease and vice-versa. This is
(5) In deriving the formula To =2rc,.j(Llg) we 'have
why:
(a) If a clock based on simple pendulum is taken to assumed that length of th'e pendulum L is much less
moon or a hill, g will decrease and so T will than the radius of the earth R so that 'g' always
increase. So the clock will take longer time to remains vertical. However, if length of pendulum is
complete a given number of oscillations and comparable to the fadius of earth, 'g' will not remain
hence will become slow or lose time. vertical but will be directed towards the centre of the
earth. So in the light of Fig. 8.14,
motion if the amplitude of motion is large? How does th
period of oscillation change with amplitude?
Answer: (a) The motion of a simple pendulum will h
simple harmonic only in the limit of small amplitude so tha
restoring torque mgLsinS may become linear, i.e., 't oc --e.

Note: It is a common misconcept!on that e must be small so that th


motion may be along a straight line.

(b) Ifa is not small sin a ¢S and restoring torque will no


be linear and so motion will not remain simple harmonic bu
or
will become o'>cillatory.1t has been shown that in this situatiol
ife Q is Ibe amp~li~[de o~moli:::ime p]e~od[ e ]
-To 1+ - i
Fig. 8.14
T=2n - 1+ - sm - + ....
g 2' 2 16
2.
so T=C;;=21t [II]«TO)
g - +-
1
Question VI. An astronaut on the surface of the moo)
L R finds that the period ofa simple pendulum there is much large.
than on the earth and that the pendulum continues to oscillat.
From this expression it is clear that:
for a much longer time than on the earth. What information.
(a) If L«R, (ilL)>> (IIR) so T = 21t.j(Llg) which is regarding the moon could be obtained from thes.
expected. observations?
(b) IfL»RH"'){lIL)< (lIR) so Answer: As for a simple pendulum T = 2~(Llg)
increase in time period will imply that at the moon 'g' is mucl
T = 2. fR = 2 {6.4;;lO6
Vg "V---;:o- smaller than on the earth. Furthennore, longer duration 0
oscillation at the moon implies that dissipative effects aD
negligible there, i.e., the moon has no atmosphere or if an:
=800 X 21tsec::' 83.8 minute
very thin.
And it is also the maximum time period which an Problem 12. A simple pendulum is made by attaching I
oscillating simple pendulum can have. bob of 1kg to a 5 m long copper wire ofdiameter 0.08 cm ani
(c) If L is comparable to R (say L=R), it has a certain period of oscillation. Next a 10 kg bob i.
T =2• .jRI2g ~ 1hour. substitutedfor the 1kg bob. Calculate the change in the perio(
(6) If the time period of a simple pendulum is 2 seconds, it if any. (Young's modulus for copper = 12,4 x 10 10 N I m 2 )
is called seconds pendulum as it beats seconds. So Solution: As explained in point (3) (d) of§ 8.2, we have
for a simple pendulum to act as seconds pendulum
T=2=2 • .j(Llg), i.e., L=(gl.') T=2.
Vg[(I+~2nr2y
Mg)

So at the surface of earth as g = 9.8 m/5 2 • L = I m and


for the moon so that T, - TI =2'~(!:.)X -g- (M' -M I )
g 21tr2y
LM gM 1
-= - =-
LE gE 6 or T, -TI =.j(g x L)(M, -MI)/(r'Y)

so 1 1 1: -T = .j(9.8x5)xOO-I)
LM =- LE =-m i.e.,
6 6 2 1 (0.04xlO 2)2 x12,4xlOIO
Note: Here it must be kept in mind that time period of a second 2
pendulum is alwJ\ys 2 sec when it oscillates. However, in a =. 7x9xlO- sec
satellite or freely failing lift the pendulum will not oscillate and 16x12.4
so the time period will be infinite.
i.e., T2 -TJ ='0.31 x 10-2 ='0.0031 sec
Question V. (a) When will the motion of a simple
pendulum be simple harmonic? (b) What will happen to its Problem 13. Two simple pendulums of length 1m and 16 n
respectively are both given small displacement in the saml
,
direction at the same instant. They will be again in phase after Now as in case of simple hannonic motion
the shorterpendulum has completed n oscillations. Calculate n. B=Bosinrot
Solution: In case of 'J Y So time taken by the pendulum to move from
simple '----pendulum as 0 ...... " 0 ... eqUilibrium position B to A, i.e., forB =0. when
T=2rr,.J(Llg), i.e., T=KJL, lK ...../ """'\ So =~, will be given by

t=roIsm. -I(a)
so the time period of shorter ""2......
( \ a =psinrot, i.e., p
pendulum will be small, i. e., it '... \
will complete more oscillations
"
K ......... K -------·---1
,
in the same time than the longer ~K ________ -.,.) ___________ -/_ So time taken by pendulum to move from B to A
pendu1um. So if for the first ...... / and back to B,
...... ·3 "
time the two pendulums are in
same phase when the shorter
one has completed n
,/
'.........
'2K

2K
/'
/'
I, =21=2E}n-I(~) [asoo=mJ
I
oscillations, Fig. 8.15 So time period of oscillation

i.e.,
nTs = (n -1)TU
nKF; = (n-I)K..{L;
T2 =tl +t2 +~[n+2sin-t ~)}< TI )

or nJi = (n -1)Ji6 Note: If a. =!l,T =~(Llg) [n + 2 x(nl2)] =2n~(L1g) which varifies


i,e., 3n=4 the correctness of the solution.
or n = (4/3),
Problem 15. A light rod of length l2 has a small ball of
i.e., the two pendulums will be in the same phase for the first
time when the shorter pendulum has completed (4/3) mass m2fIXed at one end and another ball ofmass ml fIXed on
oscillations. (See Fig. 8.15) it at a distance II from the free end. The rod is supported at its
end 0 and is free to rotate about a horizontal axis atO. The rod
Problem 14. A ball is suspended bya thread oflengthlat
is slightly displaced from its equilibrium vertical position and
the point 0 on the wall PQ which is inclined to the vertical by
released. Find the frequency ofits oscillations about 0 and the
an anglea. The thread with the ball is now displaced through length of the equivalent simple pendulum.
a small angle ~ away from the vertical and also from the wall.
If the ball is released, find the period of oscillation of the Solution: When this pendulum is displaced from vertical
pendulumwhen(a)~<a. (b)~>a.
by an angle B the restoring torque
't"='t"1 +'t"2 =m,gll sinS+m 2gl 2 sinB
Assume the collision on the wall to be peifectly elastic.
Solution: The motion of simple pendulum is angular i.e., , =-(mIl, + m,L,)g6 (if9issmall) . . (i)
SHM; so its equation of motion will be As restoring torque is linear, so motion (Fig. 8.17) is angular
9=90sinOOI wilh oo=~(g/L)
Q
simple hannonic. Now as
't" = Ia = (milT + mzl~ )(d z Sldt 2 )
(a) When~<a.i.e.,when
angular amplitude ~ is [as/ = milT + mzl~] .... (ii) 0
lesser than a, the ,
pendulum will oscillate
So from Eqns. (i) and (ii), :
d'9 =_ (mILl +m,L,) g6 i
with its natural :
frequency. so that dt
2
(milT + mz4) i
TI ",, 21t=21t [ .... (i) ,
d B =-(t)2S
"'$~~::
,
:
e
00 'Ii or
dl'
! mIg
(b) When ~>a, time 8 :,"------------
, (mILl + m,L, )g
taken by pendulum to Flg.8. 16 with 00 = e\
move from B to C and (mllr +m2L~)
back to B, m"
This is the standard equation of Fig. 8.17

'I =f+2'J~l =.[~r . . (ii) angular SHM with time period


T = (2"'00). So here
undeformed. Find the time period of small oscillations of the
linked pendulums, when they are deflected from their
equilibrium positions through equal displacements in the
same vertical plane: (a) in the same direction, (b) in opposite
i. e.,
direction and released.
..,......,..
L L

Note: If ml =: 0, T =: 21t~~ / g which varifies the correctness of our


solution. mg mg mg
"'9
Problem 16. A simple pendulum a/length L and mass m (a) (b) (e)
has a spring afforce constant k connected to it at a distance h Fig. 8.19
below its point ofsuspension. Find the frequency ofvibrations
of the system for small values ofamplitude. Solution: (a) When both the pendulums are displaced in
the same direction by same amount, the spring will neither
Solution: As shown in Fig. 8.18 if s
compress nor stretch, so the restoring torque on each
the pendulum is given a small angular / pendulum about the point of suspension will be due to its own
displacement e, the spring will also weight only.
Ie
stretch by y(= ktanS). So here restoring / h
l/ i. e., 1" := -mgLsin a "" - mgLe [as for small e, sin e := e]
,
torque about S will be due to both force of
gravity and elastic force of the spring, /.-y
,,'
,,
- ky
QiJ6(%6i
But as by definition

-c:=la=mL2 -d e
i. e., 2
"
, = -[mg(Lsin9) + k(h tan 9)h] dt'
Now as for smallS, tan e = sine = e mg
so 't = -(mgL + kk-)9
, Flg. S.1S so with Q:} =!I
L
i.e., restoring torque is linear, so motion is angular SHM. This is the standard equation of angular SHM with time
Now as ·p;: /Ct = mLl(d 1S/d[2) [asl=mL2] period T = (21t/00).

so mL' d'9 =-(mgL+kh')9 So here, T=21tjf,


J i.e.,
dt'
(b) When both the pendulums are displaced in opposite
or with o:} =[mgL +kh2] directions by equal amount (say y), the restoring torque on
mL' each pendulum will be due to its own weight and also by
This is the standard equation of angular SHM with elastic force of the spring which is stretched by2y(= 2Lsin9).
frequency f = (ro/2n); so here So the restoring torque on a pendulum about the point of
,-------;;- suspension will be
I=~ mgL+kh' , = -[mgLsin 9 + k(2LsinO)L] = -[mgL +2kL']9
21t mL2
But by definition 't =10. = mL2(d 29Idt 2 )
Note: If h = L,j = 2~~(f+ ;!;} This result reduces to .r == (1I2n) so
2
d 9= j!I + 2k)9 or d
2
e = _0029
~(g I L) for k -+ 0, justifying the correctness of our result. dt 2 lL m dt 2

Problem 17. Two identical simple pendulums each of


with (O'=(f+2~)
length L are connected by a weightless spring as shown in Fig.
8.19(a). Theforce constant a/the spring is k. In equilibrium,
the pendulums are vertical and the spring is horizontal and
so I, =~ = 2~~(f + 2~)<> f,)
§ 8.3 Spring Pendulum i. e. , greater the mass greater will be the inertia and so
A point mass suspended from a massless spring lesser will be the frequency of oscillations and greater
constitutes a linear harmonic spring pendulum. In reality will be the time period.
neither point mass nor massless spring exists, so we can never (3) The time period depends on the force constant kofthe
construct a spring pendulum strictly according to its definition. spring, i.e.,
A small heavy mass suspended from a light spring is an T oc (u..fk 1 or f oc..fk
approximate spring pendulum.
i.e., greater the force constant greater will be the
frequency and lesser will be the time period. [The
force constant k of a spring is inversely proportional
k
to its natural length.]
(4) If the spring has a mass M and mass m is suspended
from it, effective mass is given by
m meff =m+(MI 3) so that T=2Jt.j[m eff]lk
(See miscellaneous solved Problem 33.)
(al (bl
Fig. 8.20 (5) If two masses of mass m, and m2 are connected by a
spring and made to oscillate on horizontal surface, the
The restoring force in a spring pendulum develops due to reduced mass m is given by
eiasticityofthe spring, i.e.,
RF = ky - bi +cl .... III
-=-+ - so that T = 2~-k
m m, m2
which in the limit of small displacement becomes F = -ky, i,e.,
(See miscellaneous solved Problem 34.)
restoring force becomes linear, so motion of spring pendulum
is linear simple harmonic. However, if the displacement is
not small the motion does not remain simple hannoRic but
k
becomes oscillatory.
Now for a spring pendulum, as
d2y mg
F =-ky, i.e., m- - =-ky (al (bl
dr' Fig. 8.21
d2y (6) If a spring pendulum, oscillating in a
aSF = ma=m ]
[ dr' vertical plane is made to oscillate on a
horizontal surface, (or an inclined plane)
or i.e., 'h
WIt ro 2 =-
k time period will remain unchanged.
m However, equilibrium position for a
This is the standard equation of linear simple hannonic spring in a horizontal plane is the

.m
motion with time period T = (21t/ro). So here, position of natural length of spring as
weight is balanced by reaction
1 [Fig.8.21(b)J . While in case of vertical
T=2.JW or f=2 motion equilibrium position will be
mg
This is the required result and from this it is clear that: L + Yo with kyo = mg (Fig. 8.22). Fig. 8.22
(l) Time period of a spring pendulum is independent of (71 If the stretch in a vertically loaded
acceleration due to gravity. This is why a clock based spring is Yo then for equilibrium of mass m
on spring pendulum will keep proper time everywhere kyo = mg, Le., (mlk) =(Yolg)
on a hill or moon or in a satellite and time period of a
spring pendulum will not change inside a liquid if so
_1m
T = 21'Vk =21tV-g-
I>t
damping effects are neglected.
(2) Time period of a spring pendulum depends on the Here it must be kept in mind that time period is
mass suspended, i.e., independent of 'g' as with change in g. Yo will also
change in such a way that Yo/g[ = mJk] remains
TocJ; or constant.
(8) Composite-spring pendulum: If a spring 1 =-.L (k =2.~4000 =10Hz
pendulum is constructed by using two springs and a A 21t 'lr;;; 21t I
mass, the following three situations are possible:
(B) As the pulley is movable and string inextensible, il
mass m moves down a distance y, the pulley will move doW!
by (yI2). So the force in the spring F =k(yI2~ Now as th~
pulley is massless, F =2T, i.e., T=FI2=(kI4)y. So th(
restoring force on the mass m,
(al (bl
T=lky=k'y with k'=lk
4 4

la (k' =_1 ~ k =llA =5Hz


so =-.L
21': v-; 41t 4m 2
(C) In this situation if the mass m moves by y the pulle)
(01 will also move by yand so the spring will stretch by2y(as thl
Fig. 8.23 string is inextensible) and so T' = F = 2ky. Now as the pulley i~
massless, T = F + T' = 4ky, i.e., the restoring force on the mas~
In the light of § 3.4 (4) for these cases we have
m
respectively
T=4ky=k'y with k'~4k

1
-=-+-
1
(AI
1 k=k!+k2
(BI
k-=k1+k2
(C)
And so Ic =-21t1 fl' J¥k-=2I
-=-1
m 21t m A =20Hz
k k, k2 7
Problem 19. Two masses m\ and m2 are
SoT=21t
m(kl +k2)
kl2
T=2'~
(kl+k2)
T=2'~
(k,+k2)
suspended together by a massless spring of
spring constant k. When the masses are in
equilibrium ml is removed without disturbing the
Problem 18. Fig. 8.24 shows a system consisting of a system. Find the angular frequency and
massless pulley, a springofforce constant k = 4000Nlm and a amplitude of oscillation of m2'
block of mass m = 1 kg. If the block is slightly displaced Solution: As ml is removed, the mass m2
vertically down from its equilibrium position and released,
find the frequency a/its vertical oscillation in cases (a), (b) will oscillate and so, Fig. 8.25
and(c).
T=2nfj, i.e, ro=2;=~;2
F k Furthennore, the stretch produced by mig will set the
amplitude, i. e.,
mtg=kA i.e., A =(m\glk)
Problem 20. A body of mass 1.0 kg is suspended from /
weightless spring having force constant 600 Nlm. Anothe,
body of mass 0.5 kg moving vertically upwards hits thl
suspended body with a velocity of 3.0 mls and gets embeddet
in it. Find the frequency of oscillations and the amplitude ~
(al (bl (01 motion.
Fig. 8.24 Solution: As the oscillating mass is (m + M) while foCCl
Solution: (A) As the pulley is fixed and the string is constant of the spring is k, so
inextensible, if mass m is displaced by y the spring will stretch
by y. And as there is no mass between string and spring (as I=-.L ~ =-.L 600 = lOHz
pulley is massless)
2nV<;;;+M} 2, (1+0.51 •
F=T=ky Now by conservation of linear momentum in the collision
i. e., restoring force is linear and so motion of mass m will be mv=(m+M)V, i.e., V= mv = O.5x3 =lm1s
linear simple hannonic with frequency (m+M) (1+0.51
Now just after collision the system will have KE (k, = 1.8 N I m, k2 = 3.2 N / m and m = 200g) Is the motion
= ~ (m + M)V2«
2!
mv as collision is inelastic) at the simple harmonic?
Solution: When the block touches D, it will compress the
equilibrium position. So after collision by conservation of spring and its K.E will be converted into elastic energy of the
mechanical energy spring. The compressed spring will push the block back to D
(KE)~=(PE)~(=E) with same speed; so time taken by block to move from D
towards B and back to D will be
!(m + M)V' =!kA',i.• , A =V~(m + M)
2 2 . k t, = Tl=1t~=_{02 = ~sec
vI:; "1/I:8
=l~~~~ =2~m=5cm
2 3

Similarly time t2 taken by block in contact with spring


Problem 21. A between AC,

I~::~:~ ~ =- {02 =~sec


horizontal spring block
system of (jorce constant k)
and mass M executesSHM
'2 = T2
2 vk;
= 1t
"1/3.2 4

with amplitude A. When the Fig. 8.26 Moreover during complete oscillation between A and B,
block is passing through its equilibrium position an object of the block moves the distance CD twice with uniform velocity
mass m is put on it and the two move together. Find the new v, once from C to D and again from D to C. So
amplitude and frequency of vibration. 2£ hO.6
t3 =-V=--u-=Isec
Solution: As initially mass M and finally (m + M) is
oscillating,
T=t 1 +/2 +t3 =1t(~+~)+1=2.82sec
f- 1 /k 1' - ~
2n fii
and 1
2n vr;;;+M) Now a motion is simple harmonic only and only if
throughout the motion F = -kx.Here between C and D,F =0
so that I' - ~
Y- (m+M)
(as v = constt.); the motion is not simple harmonic but
oscillatory.

l'=f~(m7M) Problem 23. One end of y


i.e., .... (i) each of two identical springs,
each' offorce constant 0.5 Nlm,
Now by conservation of linear momentum, are attached on the opposite
Mv=(m+M)V sides ofa wooden block ofmass
and as at equilibrium position v = Am =2rtAf (as 00=21[/) 0.01 kg. The other ends of the
M2Mf = (m + M)2M'f' springs are connected to
separate rigid supports such
i,e., £= M xL that the springs are unstretched
A (m+M) I' and are collinear in a horizontal
Fig. 8.28
Which in the light ofEqn. (i) gives plane. To the wooden piece isflXed a pointer which touches a
A'=A~ vertical moving plane paper. The wooden piece kept on a
vr;;;+M) smooth horizontal table is now displaced by 0.02 m along the
line ofsprings and released. If the speed ofthe paper isO.1 mis,
Problem 22. Two find the equation ofthe path traced by the pointer on the paper
light springs of force and the distance between two consecutive mCl:fima ofthe path.
k,
constant k, and k2 an'd
Solution: Here as k=k, +k2 =0.5+0.5=1 N/m and
a block ofmass m are in
line AB on a smooth m=O.Olkg,
horizontal table such A B ro = ~klm = ~(1I0.01) = 10 radls
that one end of each C D
spring is fIXed on rigid Fig. 8.27 As the block is displaced by 0.02 m, amplitude will be
supports and the other end is free as shown in Fig. 8.27. The 0.02 ffi. So the equation of motion along the x-axis will be
distance CD between thefreeendsofthesp~ings is60cm.lfthe x = A sin(rot+ ~) =0.02sin(lOt +~) .... (i)
block moves along AB with a velocity 120 cm/sec, in between where 4J is phase constant.
the springs, calculate the period ofoscillation of the block.
But along the y~axis as paper is moving down, the pointer with ro2 = AY k
relative to paper is moving up with constant velocity 0.1 mis, m(AY +kL)
so that This is standard equation of SHM with time·period
y=vt=O.lt .... (ii) T = (2m",)
The equation of the path will be obtained by eliminating t
So T=20 m(AY +kL)
between Eqns. (i) and (ii), i. e., AYk
x =0.02sin (100y +~)
Problem 25. Two identical
Now x will be maximum when balls A and B each of mass 0.1 kg y,
sin(100y+~) = max =I, are attached to two identical
massless springs. The spring.mass P 11=+-'''''"'''!''''''''-- a
i .• , (lOOy + ~)=¥,52o,920,,,,, system is constrained to move
inside a rigid smooth pipe bent in
which implies IOOy! + tjl = rr. / 2 and lOOY2 +$ = 5rr./2
the form of a circle as shown in
so that Y2 - YI = (20/100) = 0.02 ron = 6.28 em Flg.8.3D
Fig. 8.30. The pipe is fIXed in a
Problem 24. One end of a long metallic horizontal plane. The centres of the balls can move in a circle

T
wire of length L is tied to the ceiling. The other of radius 0.06 m. Each spring has a natural length ofO.06nm
end is tied to a massless spring of spring and force constant 0.1 N/m. Initially, both the balls are
constant k. A mass m hangs freely from the free displaced by an angle 9 = n/6 radian with respect to diameter
end of the spring. The area of cross-section and PQ of the circle and released from rest. (a) Calculate the
~
the Young's modulus of the wire are A and Y ~F frequency of oscillation of the ball B. (b) What is the total
respectively. Find the time period with which
mass m will oscillate if it is slightly pulled down
and released.
Solution: As there is no mass between the Fig. 8.29
4;' energy of the system? (c) Find the speed of the ball A when A
and B are at the two ends of the diameter Po.
Solution: (a) As here two masses A and B are connected
by two springs, this problem is equivalent to the oscillation of a
spring and wire, the restoring forces in the reduced mass 111 by a spring of effective force constant keff
=
spring and the wire will be the same, i. e. , F, F 2 = F. But as given by
force constants of the spring and the wire are not equal so their m= m1m2 =0.lxO.I = 0.05kg
*
stretches will not be equal, i. e., y, Y2' However, if the mass III, +1112 0.1+0.1
m is pulled by y,
and kelT =k 1 +k2 ::O.l +O.I =0.2N/m
Y= YI + Y2 .... (i)
. Now as for a spring by Hook's law, i.e.,F = ky, So f=~t"l
2n m
=~l·20 =,!,Hz
2n 0.05 n
YI = (F1I k) .... (ii)
and for a wire by definition of Young's modulus, (b) As here one spring is compressed while the other is
stretched by same amount (say y) and balls are at rest at A and
F2L
Y=--, B. so
AY2
I 2 I 2 2
F2L £ = 2k1Y +2 k2Y =ky
i.e., Y2=- .... (Hi)
AY But from Fig. 8.30

so Y = YI + Y2 = R9, +R9 2 =2R9


=2xO.06x (n/6) =0.02nm
i.e., So £ = (0.1)(0.020)2 =40 2 x10- 5 J
(c) As at P and Q springs are unstrctched so the whole

or
2
d y { An
m dt 2 = AY+kL y
1 [asF =_m(d 2y'dt 2 )]
energy becomes kinetic of the balls A and B, i.e.,
1 v 2 = E=4n 2 xl 0-5 J
I v 2 +2'm
2'm 1 1 2 2
d 2
2
or --.l!.
2
= -(j) Y Here m, = m2=0.lkg and v, =v 2 =v
dt
So 0.1v 2 = 41t2 x 10- 5 , i.e., v =21tx 10-2 mls
Problem 26. A particle of ~ 8'..
mass m is aUached to three ..I"~ ~
~ or with (J} =3k .... (i)
M
idemical springs A, Band C each ~l1M~o~/..
of force consla,,1 k as shown in Fe F. This is the standard equation of angular SHM with time
Fig. 8.3L If the particle of mass m 0 m period T = (2moo); so here,
is pushed slightly against the ~
spring A and released, find the FAr~ T=2rr 1M
Vy;
time period ofosciIJatiolls. ~ xt • (b) In angular SHM maximum angular velocity
Solution: When the particle
of mass matO is pushed by yin the Fig. Boll (dfJ l dt) = = 60"' =60 ~(Jk IM)
direction of A, spring A will be
compressed by ywhile Band C will be stretched by y' = yeas
[similartov rnax = Awl
45°; so the total restoring force on the mass m along AO, Now as v = r(d9Idt) and here r = L, so
RF =FA +FB cos 45°+Fc cos 45 °
v
max
=~d6]
dt
=L6 0 13k
Vii
i.e, RF= ky+2(ky')cos 45°
or RF =ky+2k(ycos 4S0)cos 4S0 """
Alternative Solution to Part (a): When the rod is
i. e., F = - k' y with k'=2k displaced by 9 (in horizontal position), ME of the system will

so T = 21t rm 1m be
v-;;: =2-"'12k E=!/(d6)' +!ky'
Problem 27. A long uniform rod oJ/eng/h L and mass M 2 dt 2
is free 10 rotate in a horizontal plane about a vertical axis
But here I = ( 113)ML2 and y =LO.
through irs one end. A spring offorce constant k is connected
horizontally between one end ofthe rod (lnd afixed wall (Fig.
8.32). When the rod is in equilibrium it is parallel to the wall. so E =!"ML 2(d6)2 +!..kL29 2
6 dt 2
(a) What is the period ofsmall oscillations that result when the
rod is rotated slightly and released? (b) What will be the Now in simple harmonic motion ME is conserved, i. e.,
maximum speed oj the displaced end oj the rod, if the (dEldt) =0.
amplitude ojmotion is eO ? 2 2
!Md 0 + Ice =0 i.e., d e = --(029
3 dt 2 ' dt 2

with 00
2
= ~ which is Eqn. (i)
B
Problem 28. A diatomic molecule
has a/ol1ls of masses I1I t and 1n2 ' Th e
potential energy ofthe molecule for the
m,
., -- •
Infinit;\ ¥A
interatomic separation I' is givell by
ma~
Fig. 8.32 VCr) = -A + B(r - ro )2, where '0 is the Fig. 8.33
Solution: (a) As shown in Fig. 8.32 when the rod is given equilibrium separation and A and B are positive constants.
a small angular displacement e,
the restoring torque will be The atoms are compressed towards each other from their
developed due to elastic force of the spring, i. e., equilibrilll1l POSitiOIlS and released. What is the vibrational
.=-FL=-kyL [as F=!)') frequency of the molecule?
or t= - kL29 [as y=L9] Solution: For conservative forces, negative of potential
energy gradient is force,
But by Newton's II law for rotational motion:
2 . ,
I.e. _dV =F
t =lo.= -I ML2 -d 9 dr
3 dt 2
Given potential energy function is
so that ! ML' d '6 = - kL'6 V(r) =-A +B(r-ro )'
3 d,2 '
Hence restoring force F =- dV(r) = -2B(r - 1'0) =- 2Bx
i. e., d'6 = _ 3k 6 dr
dt 2 M Where x is displacement of reduced mass, ).l
Now we convert Ihe system to a reduced mass system. Problem 30. A small bar magnet having a magnetic
2 moment 0/9 x 10- 9 Wb-m is suspended at its centre ofgravity
Equation of mation of system is, J.l d x = -2Bx bya light torsionless Siring at a distanceof 10- 2 m venically
d,' above a long straight horizontal wire carrying a current 0/
2 I.OA. Find the frequency ofosciJIation ofthe magnet about its
or d x =_2Bx =--ro2x
dt 2 J.l equilibrium position assuming that the motion ;s undamped.
The moment of inertia of the magnet is 6 x 10-9kg-m 2.
where Soludon: The torque acting on the small bar magnet in
external magn.etic field
:.Vibrational frequency ofrnolecule f =~
2_
=-MBsin9=-MB9
[for small oscillation,9 is small, so sin9 =9]
=2. /2B B = Il 0 .2i
21[V-; Here,
4_ r
=2. .~_2-;;B~(,--m::--'--I--:+~m~,~) 2
If d 9 is the angular acceleration produced and I is M.1. of bar
2ft m,m2 d,'
Problem 29. A block a/mass M] magnet about axis of oscillation,
resting on a friclionless horizontal I d' e = -MBe=-M~O 2ie
surface is connected to a spring of dt2 4ft r
spring constant k thaI is anchored in a Ag.8.34
d 9 = _ ~o . 2iM 8 =_ro28
2
nearby wall. A block of mass M 2 is placed on the lop of the or
d/2 41t lr
first block. The coefficient of sIalic friction between the two
bodies is J.l. Assuming that the two bodies move together as a where ro 2 : ~O
-'-2iM
-
4n Ir
unit, find the period of oscillation of the system. What is the
maximum ascil/alion amplitude that permits the two bodies to So, motion is SHM with frequency of oscillation,
move as a unit? f =.!E..=.!.... ~o .2iM
Solution: Let x be the compression of spring at any 2ft 2n 4ft Ir
instant; then 1 1O- 7 x 2xlx9xlO-9
=-

or

with

So, motion is SHM with time period


~
T=21tV~
For the two blocks to move as a unit, in the extreme
position the pseudo force on M 2 should be less than maximum T, '
m!y
static frictional force. If A is the amplitude of SHM, - 1
- ---- ---~
: l ----------------
62
A -d .:. (l-d) , B
M,ro' AS llM,g FIg. 8.35
or But as 8, and 9 2 are very small
T, =T2 [asforsma1l9,cos8 = 1]
So restoring force on mass m will be
F=-[T,sin9, +T2 sin9 2 ]=-T(8, +9 2 ) [assin9=9]
But from Fig. 8.35, M'
B=-V-
9, =(y/d) and 9, = [y/(L-d)] bV
so F=-r[E+
d
y
(L - d)
]=_ d(L-d)
TL xy

i. e., restoring force is linear, so motion is linear simple


hannonic with force constant
k= TL.
d(L - d)' ,.e.,
1 (k 1 ~
1=2nV;;;=2nV~
Po

___ A___
M ,,
,
- Po
Vo ,,
VolA
M
Note: If the mass is in the middle ofthe string, d = (Ll2) so f = (ll2n)
P P ,
,j(4TlmL). (a) (b)
Fig. 8.38
(b) If a mass m is suspended from a wire of
length L, cross-section A and Young's i.e., restoring force is linear so motion of piston is
modulus Yand is pulled along the length of linear simple hannonic with force constt.

(k =.l.~BA'
the wire: Here restoring force will be L
k=BA' so1ha1 I=.l.
developed by the elasticity of the wire, i. e.,
Vo 21t V; 211. MVo
FL -YA
Y=- or F=--xy
Ay L m Note: (i) If the change is adiabatic B -+ E,. = yP and if isothermal
i.e., restoring force is linear, so motion is B ---t Ee = P.
linear simple hannonic with force Fig. 8.36 (ii) If the cylinder is vertical -P= (Po + MgIA) (where Po is
constant atmospheric pressure) while for horizontal cylinder as
weight ofpistonMg is balanced by reaction, p::: Po'
k JA, i.e., 1 =.l. (k =.l.tA
L 21tV-;;; 21t mL fBI When Restoring Force is due to Hydrostatic Pressure
or Thrust
(c) If the lower suiface of a '
cube o'side Land o· --'>
F
,/ c:::.
t=.J
Ca) Motion ofa liquid in a V-shape tube when it is slightly
'J 'J y/ m
modulus of rigidity 11 is I-''t,'---"-of depressed and released: Here it is taken for granted
fixed while fixing 'a sf that cross-section of the tube is unifonn and the liquid
is incompressible and nonviscous. Initially the level
particle ofmass m on the I

upper face, a force of liquid in the two limbs will be at the same height. If
parallel to upper face is the liquid is depressed by y in one limb, it will rise by y
Fig. 8.37
applied to mass m and along the length of the tube in the other limb. So here
withdrawn: Here restoring force will be developed the restoring force will develop ' by hydrostatic
due to elasticity, i.e., modulus of rigidity of the block, pressure difference, i. e.,
i.e., F =-6pxA=-(h, +h,)gdA = -Agd(sin +sin9,)y
F A
~= ­ or F = T\~xy
A9 L
i.e., restoring force is linear; so motion will be linear
simple hannonic with force constant

k = ~A = ~L [asA=L']
(b)
L (a)
Fig. 8.39
So that, T= 2~ = 211.~ ;L As the restoring force is linear, motion will be linear
simple hannonic with force constant
(d) If a gas is enclosed in a cylinder of volume Vo fitted k = Agd(sin9 J +sin9 2 )
with a piston of cross-section A and mass M and the
piston is slightly depressed and released: Here elastic so that
force is developed due to bulk elasticity of the gas
B,i.e.,
2 2
Note: If the tube is a U.t';1be and liquid is filled 10 a heighth,8] '" 8 2
= 90"and m '" hAd x2, so T '" 21t~(hlg).
t = la = mR 2(d
d,2
e) [ aSI = mR 2 anda = d e]
dt 2
2
(b) When a partial/y submerged floating body is slightly so mR -
2 d e =-mgRe, i. e., -d e =-<ll 29
2

depressed and released: If a body of mass m and dt 2 dt 2


cross-section A is floating in a liquid of density a with
height h inside the liquid, with
mg=Th=Ahog, i.e., m=Aho .... (i)
This in tum implies T = 2rrJro = 2rt./(R / g)
Now fromthis
equilibrium position if (b) Motion ofa ball in a tunnel through the earth:
it is depressed by y, Case I: If the tunnel is along a diameter and the ball
the restoring force is released from the surface: If the ball at any time is at
will develop due to a distancey from the centre of the earth, the {Fig. 8.42
extra thrust, i.e., (a)] the restoring force will act on the ball due to
Fig. 8.40
F =-Aagy gravitation between ball and earth. But from theory of
gravitation we know that force that acts on a particle
As restoring force is linear, the motion will be linear
inside the earth at a distance y from its centre is only
simple hannonic with force constant
due to mass M' of the earth that lies within the sphere

k" =21t~ Acrg


1'V1m
k=Aag so that T=2- m of radius y. (The portion of the earth that lies outside
this sphere does not exert any net force on the
From this expression it is clear that if density of liquid particle). So F = -(GmM'/i),
decreases, the time period will increase and vic~
versa. And also as from Eqn. (i) m=Aha,T=21t
~(hlg) where h is the height of the body inside the
liquid.
Note: (i) If the body is floating fully submerged then on pressing it
will remain at the displaced position as no net force acts on it. (al (b) (e)
(ii) If a panially submerged floating body is connected to a FIg. 8.42
venical spring and then depressed, the restoring force will be
due to both extra thrust and stretch in the spring, ButasM=(4/311tR'P and M' =(4/3).y'p,
i.e.,F '" -(ky+ Aagy) = -(k + Aag)yso that effective force
=
constant k' (k + Aag) and T '" 2rtJm/(k + Aag). I.e. , M'=M(yIR)'

Ie] When Restoring Forte is due to Gravity or Gravitation F ~ -Gm x M( L )=- GMm y
(8) Motion of a ball in a 5
i R3 R3
bowl: If a small steel
- I
r.G i. e., the restoring force is linear; so the motion is linear
baU of mass mis placed a\ SHM with force constant
at a small distance
from 0 inside a smooth
concave surface of R
\,
,
'
k= - -
GMm
R'
so that T=2 .J¥ .g'
-=2
k
-
GM
radius R (as shown in
Fig. 8.4 1) and Furthenno," as g =GMIR 2, T =2tt./(R Ig)
released, it will Which is same as that of a simple pendulum of infinite
oscillate about O. The length and is equal to 84.6 minute.
restoring torque here o mg Case II: If the tunnel is along a chord and ball is
. will be due to the force
Fig. 8.41 releasedfrom the surface: If the ball at any time is at a
of gravity mg on the
distance x from the centre of the tunnel as shown in
ball, i.e.,
Fig. 8.42 (b), the restoring force will be
or = -mg(R sin 9) = -mgRe
Now as ~estoring
torque is linear, so motion will be F' =Fsin9 =( _ GMm y)(,,) =_ GMm x
angular simple harmonic. And as by definition, RJ y R3
403

which is again linear with same force constant


k = (GmMIR 3) so that the motion is linear simple
1 -=-
sothatfL=- 1J!
21t m 21t
4qQ

harmonic with same time period

1m =2~R3
Note: (i) If x is comparable to L the restoring force will not be linear
T=2- =2n [R =84.6 minute and so the motion will not be SHM but it is still 'Oscillatory.
"Vk GM Vg (ii) If the charge q is displaced perpendicular to the line joining
the charges, the charge will not come back and its motion
Note: (i) As in SHM, Vrnax =: Am = A~(gIR) [as 002 "" gIRl, in case I will be accelerated translatory in the direction of
and II though time period is same, v rn.u will be different due displacement [Fig. 8,43 (b)J.
to the fact that in case II, A "" semichord while in case J,
A = R. So in case I, Vrnax =: R..Ji1R =: 8 kmls (= speed of a (b) Motion of a negatively charged particle between two
satellite close to the surface of earth).
equal and positively jued charged particles: If the
charged particle is displaced towards A or B it will
(ii) Ifthc ball in case I or II is dropped from a height hear with an
initial velocity u so that h '" u2/2g), the ball will reach the
move in the direction of displacement and will not
come back. So its motion along the line joining the
carth in time t:=: ~(2hlg) and on the other side will go out of
charges will be accelerated translatory [Fig. 8.44 (a)].
earth through the same distance before coming back towards
the earth. However, outside the earth F oc (-II?) and not . P q

~
FA e- Fs
(- r), so the motion does not remain simple hannonic but A o:-q~ B A~B
becomes oscillatory with time period ®- - L ---..,-x+
I'
, ® + - L' - 0 +

(==4 fIE + 4 IKsin-l[_R_]112 (a) (b)


Vg fi R+2h
Fig. 8.44
(iii) The motion of a point charge q inside a unifonnly charged
sphere can also be dealt as in case I with: .However, if the particle is.displaced perpendicular to
G = (114118 0), M -+ Q and m == q the line joining the charges by y as shown in Fig. 8.44
.. F __ __l_qQ. (b), the restoring force
gmng ,
4118 0 R3
F =-2 x _1_ qQ cose = - 2qQy [ ascose =~l
41t£0 r2 41tEor3. '
'nd

[D) When Restoring Force is Electrical


(a) Motion of a charged particle between two similar,
i.e., the restoring force is linear; so motion is linear
fued point charges: As shown in Fig. 8.43 (a), if q is SHM with force constant
given a displacement x from equilibrium position k = 2qQ sotha' IT =J... {k =J... 2qQ
along the line joining the charges, the restoring force 41t1:':oL3 21t~; 21t 41t&oL3 m
from Coulomb's law will be
F
F, Note: (i) Fromcase(a)and(b)it, is~learthati ==(..fi)/r.
(ii) If y is comparable to L, motion will be-osciiiatory but not
A ,~ B A B simple hannonic.
°1·
+®- L - , - x - q
,
(j) +
--L-O
+
(e) Motion of a charged + dQ
(a) (b) particle along the axis
Fig. 8.43
of oppositely charged
+ +
F=-(F - F )=_R[ 1 - 1 1 ring: Let a charge -q
be at a distance x from + +
,0-
p
,
B A 41t£0 (L _x )2 (L+x)2 dE
the centre 0 ofthe ring
i.e., F=--.!lfL 4Lx _ _ 4qQ xifx«L on its axis as shown in
4m:o (L2 _x 2 )2 41t£oL3 Fig. 8.45. The intensity +
at P due to element of Fig. 8.45
i. e., the restoring force becomes linear for x < L, i. e., charge dQ will be
motion is linear simple hannonic in the limit of small
displacements with force constant dE= _ I _ dQ
, 41t£0 r2
404 PHYSICS FOR COMPETITIONS - Vol. I

However, the component of dE perpendicular to the (e) Torsional vibrations ofa dipole in a uniform field: If
axis at P due to different elements of charge will a suspended dipole is rotated by a small angle 9 from
cancel each other while along the axis will add up. its equilibrium position in a unifonn field as shown in
Fig. 8.47, the restoring torque will be developed due
So E=EH =ldEcoss=l_l_dQ;o to electric force and is given by
41t£o ,2 r
l' = -[(qAEA )Lsin9 + (q BE B )Lsin9]
Here as x and r are same for all elements
=-2qEL~nS [asqAEA =qBEB =qE]
EH =_I_Qx = 1 Qx or t=-2qEL9 [as for small 9. sin 9 =9]
4m: o ,3 41tS o (R2 +x 2 )3/2
[as r = ~r(R--;':-+-x")]

So F=qE=- 1 qQx = ___ qQx qE


41re o (R 2 +x 2 )312 41tS o R J _~~"..<lJ'--+,-> E
ifx«R ::~:Oi::~o!o~:iD::D qE
• _ q •___________ J:
A
i.e., the restoring force becomes linear if x« R, so
motion is simple hannonic only in the limit of small (al (bl
amplitudes with force constant Fig. 8.47

As the restoring torque is linear, the motion is angular


k= qQ
41t£oR3
so that T=2 nJ¥-=2
k
4m:oR 3 m
qQ
simple hannonic. Now as
T =Ia =I(d'S/dt'),
(d) If a current carrying il
floating wire over A"y"iOJ;=~m!"'g==rlhB d'S
So I-=-2qLES= - pES [withp = q(2L)]
another antiparallel dt'
current carrying wire ~
~===="'o;=='==~
.12 D or
2
d 9 =_pE 9
is slightly displaced dt 2 I' i.e.,
towards it and Fig. 8.46
released: As force per unit length between two , pE
with co =-
parallel current carrying wires (dFldL)=().I.o/4rc) I
(2i1i2Ih) and repulsive if currents through them are in As this is standard equation of angular SHM with time
opposite directions, so for equilibrium of floating period T = 21t/ro, here
wire,

.... (i)
T=2'~ pE
I

Now if the upper wire is displaced by y towards the Note: If instead of electric dipole, we have a magnetic dipole of
other wire, the repulsive force will increase; so moment M in a unifonn field of induction S,
restoring force T=21t~ MB
J

F=_Il02iJi2 [_I_ _ .!.]XL [asF=dF XL ]


4. (h - y) h dL § 8.5 Superposition of Two SHMs of Same
_Ilo 2i]i2L _ Ilo 2iJi2L Frequency in the Same Direction
or F - - - y - - - - - y (ify« h)
4. h(h - y) 4. h' When a particle is superposed with more than one
hannonic forces, each force tries to move the particle in its
As the restoring force is linear, the motion will be
own direction with SHM. We call it interference ofSHM.
simple hannonic with force constant
[A] First we consider two SHMs that produce a
Ilo 2i1i2L
k=-- - so that T=2
nJ¥ -=21tt.I-==~
41tmh2 displacement ofthe particle along same line. The displacement
of the particle produced by each simple hannonic motion is
41t h2 k 1l02i}i2L
given by,
which in the light ofEqn. (i) reduces to .... (i)
Xl = Al cos cot

T=2~mh' = 2. IE and x2 = A2 cos (cot + 5) .... (ii)


mgh fi where 5 is phase difference between xI and x2'
Case (I) If 8 = 0, we say that the two motions are in phase. p'
Then the resultant motion is ... (vi) ~/ ,(ro,-~)I

X= AI COSro! + A2 cos rot = (AI + A 2 )cosrot .... (Hi)


P2 / i
Hence resultant SHM is given by ~
,
Resultant amplitude is Al + A2 , The two rotating vectors A,
x=A R sin(oot+a) .... (vii) « '
OPt and OP" corresponding to the two motions are represented 'Pi
in Fig. 8.48. o~""~,~ooc.,,~-'--~.,
t, FIg. 8.51
,, Pi x
[B) Superposition of two SHMs with the same direction
,,l P2,
,
but different frequency:
I 001'
This kind of superposition takes place when two radio
O>="'-'-~-x signals are broadcast with frequencies that are close but not
equal. Consider two oscillatory motions described' by the
equations.
XI = Al cosoolt ..... (viii)
(a) (b) x2 = A2 cos00 2 t ....(ix)
Fig. 8.48 Composition of two SHMs In phase;
A=OPi+OP2=At+~
The angle between rotating vectors OP and OPi is now z
(001 - 00 2 )t, which is not constant. ThereforeOP' does not have
Case (Ii) If B = 1f, from Eqn. (ii) constant length and does not rotate with constant angular
velocity.
x2 =A2 cos(rot + tt) = -A z cos rot
Hence the resultant motion
then resultant motion is
X=x\ +x2
x=(A t -A 2 )cosrot .... (iv)
The amplitude of motion is
which shows that the resultant motion is SHM with the same
A = [A12 + Ai +2AIA2 cos(ool -(02)11 112 ....(x)
angular frequency and resultant amplitude is
A=A\-A 2 The amplitude oscillates between the values
Hence we say that the motions are in opposition or for A=A 1 +A2 when (001-002)t=2mt
B =1t the two SHMs are in opposite phase. and A =I AI -A21 when (00\-002)t=(2n+l)1t
We say that amplitude is modulated. The frequency of the

M amplitude oscillation is expressed by

v=[ ;n 1 002
-v
----:r-'
P,
(a) (b)
00

Amplitude fluctuation or beats


)=VI 2

Amplitude variation arises when two sound sources, of


....(xi)

Fig. 8.49 Composition of two SHUs In opposition; close but different frequencies are vibrating simultaneously at
A .. OP, - OP2 =~-A2 nearby places. A listener observes a fluctuation in the intensity
ofthe sound called beats, which is due to the change in amplirude.
In general case, phase difference is
arbitrary; the resultant motion is also
SHM with the same angular frequency
00 and an amplitude given by,
P,//Ji'
(ro2- ro,)1 ,:
A

A = (A\2 + Ai + 2A J A 2 cos8)112 .... (v) A2 ' Pi


In the figure the rotating vectors A,-I:""'"
z
OPt and OP corresponding to xI and o~'-""'----x
x2 are drawn. Their resultant OP' is the Fig. 8.50 Composition 00, - 002
rotating vector corresponding to x. We of two SHMs of Fig. 8.52 Amplitude fluctuation or beats
different frequencies
use the law of vector addition to find
For Al = A2 in Eqn. (x), we get
magnitude of A. Angle of resultant OP' with OPt is given by
A=A,{2[1+cos(ro,-ro,)t]}'12
.... (xii)
we get x = Aeosrot ... .(xvii)
y=-Bcosrot .... (xviii)
which oscillates between zero and 2A!. The resultant motion is Combining these gives
given by B
y=--x .... (xix)
x=X 1 +X2 =A I cosoo1t+A 1 COS0)2t A
which is again straight line motion with amplitudes~ A 2
1 1
2:
= [2A, COS (00 1 - oo2)t]cos'2(ool +(02)t +B 2
alongRS.
=Acosk(001 +(02)t .... {xiii) Case (iii) When 03 = rrJ2
The equations are now
The motio,n can be interpreted as a hannonic motion with a
frequency of x=Acosrot .... (xx)
y= B cos(rot + 1(12)
= - Bsinrot .... (xxi)
, These equations may be combined to give
2
i
-x + - = 1 .... (xxii)
, A2 jj2
Iv which is the equation of ellipse as shown in Fig. 8.54. The
o ~
~T:'V
,
, , Y-T, t ellipse is traversed in a clockwise sense. This may be verified
Y
~ ~
,, by finding velocity of the particle at x = +A. When x = +A
:: "', ... ,,
~

.... " " , '-- , from Eqn. x = A cos rot, the x-component of velocity is
Fig. 8.53 Vx = ro A sin rot = O. Similarly from Eqn. y = A sin cot, the
y-component of velocity, V y = dyldt = -roB cosco! = -roB and
Ie] Superposition of two SlIMs in perpendicular directions:
the velocity is parallel to y-axis. Since it is negative, the point
A particle moves in a plane such that its x and y-coordinates passes through A moving downward, corresponding to a
oscillate with simple hannonic' motion. Consider a SHlvI along clockwise sense of rotation.
x-axis given by
x = A cos rot .... (xiv) Note: (i) Same ellipse is obtained if 0 = 31t12 or -n/2 but then the
motion is counter clockwise.
and the motion along y-axis by
(ii) When phase difference is ±n12 the superposition of two
y=Bcos(rot+S) .... (xv) simple harmonic motions of same frequency results in
where 8 is the phase difference between the x- y oscillations. elliptical motion.
The path of the partic~e is limited by region defined by the lines (iii) The axes of ellipse is parallel to the directions of the two
x=±Aandy=±B. motions.
We shall now consider some special cases: (iv) When A = B the ellipse transforms into circle and we have
circular motion.
Case;(l) II f;'5 =O,from Eqns. S ~:~12 B y o=:~~ a
(xiv) and (xv),
"x =' A cos rot
and y=Bcosrot
Eliminate cos rot to get ,,
B
,,
y=-x p
A -B A
which is equati0n of a straight Fig. 8.54 .s '" 240" .s = 270" 0:: 300" .s = 330" .s = 360"
line. The displacement along the line PQ is Fig. 8.55 Paths for selected phase difference when
" ' :!
A ",BandOlj =00 2
r = (x 2 + y2)112 = (A 2 + B2 )112 cos rot ....(xvi)
The resulting motion is simple hannonie with amplitude (v) For an arbitrary value of the phase difference 8, the path is
(A2+B2)1I!2 : : '('. ,1[",:':,.; ; <HI:: still an ellipse but its axes are rotated with respect to
coordinate axes for
Case (ii) If the two motions are in opposite phase, x!=Asinrot and x2=Bsin(rot+o)
03=11: resultant path of particle is given by
x
2
I 2xycosS . 2 Ii
A2+ 92 AD ",sm
(vi) When two perpendicular oscillatory motions of different
frequencies are superposed, the resultant paths are called
Lissajous figures. For these figures
[, _
.::L", ",
fy II... (aJ (bJ (eJ
If the figu re is touched tangentially by a horizontal line 8111~ Flg. a .56
II,.
points and by a vertical line at points, the frequencies of
SHM are in the inverse ratio of these numbers.

MISCELLANEOUS SOLVED PROBLEMS

Problem 31. A uniform horizontal plank is resting As the restoring force is linear, the motion will be simplc
symmetrically in a horizontal position on two cylindrical hannonic with force constant
drums. which are spinning in opposite directions about their k=/lmg
horizontal axes with equal angular velocity. The distance L
between the axes is 2L and the coejJiciem offriction between
the plank and cylinder is J.I. If the plank is displaced slightly so that T = 2-/m = h fT
from the equilibrium position along its length and released,
''{k 'I'M
show that it peiforms simple harmonic motion. Calculate also Problem 32. A mass m is connected to a spring a/mass M
the lime period ofmotion. and oscillates in SHM on a horizontal smooth sUI/ace. ' Th e
R R
force constant o/the spring is k and equilibrium length L. Find
Solution: When the
plank is situated
f .. ,-£@::= ,t~: '" (a) the KE o/the system when mass m has a speed V, (b) the
time period ofoscillation.
symmetrically on the
drums, the reactions on
the plank from the drums
will be equal and so the
A ..
mg

R~f, . I~Re
i .
B

'f-;-' " L'f\


Solution: (a) Consider a
segment of spring of length dx at a .
distance x from the fixed end (as
shown in Fig. 8.58). As the
.
~;:-1..
d,

'
,
, '
'ji •
,.
force of friction will be displacement and hence velocity of Fig. 8.58
A mg B
equal in magnitude but Fig. 8.57
different segments will be different
opposite in direction and in an oscillating spring, we assume that velocity ofthe segment
hence, the plank will be in equilibrium along vertical as well as is directly proportional to its distance from the fixed end, so
in horizontal direction. that for this segment
Now if the plank is displaced by x to the right, the dm=M dx and v==-V
reactions will not be equal. For vertical eqUilibrium of the L L
plank So Ihat KE of the segment,
RA +RB =mg .... (i)
And for rotational equilibrium of plank, taking moment dKs =21 dmv 2 =2l(M
Tdx .)(xl.V )'
about centre of mass, we have
RA(L+x)=RB(L - x) .... (ii) Ks =-1 -MV2- 1Lx 2dx= -1 -
MV2 L3 I
- x-= -MV
2
Solving Eqns. (i) and (ij), we get 2 L' 0 2 L' 3 6

RA =m~ \~x ) so KE=Km + Ks = .!mV


2
2
+.! MV2 =.!(m +! M ) V 2
6 2 3
(b) If Y is the stretch in the spring when mass M has
and RB =m{ L2:x ) velocity V, the mechanical energy of the system will be
Now as f = ~R, so friction at B will be more than at A and
will bring the plank back, i. e., restoring force here
E=KE+PE=~( m+~M )V 2 +kky2 [asPE = ~ Jry2 J
mg Now in SHM, ME is conserved, i.e., (dE/de) ='0.
1

F=-(fB -fA)=-~(RB - RA) =-~Tx


(c) As in SHM energy is conserved, i. e. ,dEldt =0. From
! (m+ !M ) 2V dV +.!.. k2y dy =0
23 dl2dl Eqn. (i), we have
lM .2V dV +lk .2x dr =o
i, e., m+!M)d'Y=_ky [asdY=VanddV = d'Y] 4 dl2 dl
( 3 dl 2 dt dt dt 2
2 or
or d Y =~2y with 0)2 = k
[m+~M]
2
dt or with 0)2 = 2k
3M
This is the standard equation of linear SHM with time
period T = (21t1co); so here This is the standard equation of SHM with time period
T =(2"'",).
T=2. [m+GH=2~m'ff
k k
with mff=m+ M
e 3
So here, T = 2~3M
2k
Problem 34. Two masses m, and m2 are connected by a
Problem 33. A solid cylinder is attached to a horizontal
massless spring so that it can roll without slipping along a spYing o//orce constant k and are placed on a frictionless
horizontal surface. The spring constant k is 3 Nlm.lfthe system horizontal surface. Show that if the masses are displaced
is released from rest at a point in which the spring is stretched slightly in opposite directions and released. the system will
by 0.25 m, find (a) the Iranslational kinetic energy and (b) execute simple harmonic motion. Calculme the frequency of
rotational kinetic energy of the cylinder as it passes the oscillation.
equilibrium position. (c) Show that under these conditions the Solution: Let the masses m, and m2 be displaced by XI
centre ofmass of the cylinder executes SHM with time period
and x 2 respectively from their equilibrium position in opposite
T =2,.j(3MI2k) directions so that the stretch in the spring will be X = Xl + x 2 .
Solution: If at any position Due to this stretch a restoring force fa will act on each mass
the stretch in the spring is x and and so equation of mass m,
will be
the velocity of centre of mass of 2
d2xl d xI k
the cylinder is V, then as nil dt 2 =-kx, i.e., dt 2 =-~x .... (i)
Fig. 8.59
U =lla', Kr =!MV'

b
2 2
and

So total mechanical energy of the system (al (bl


E =Kr +KR +U Fig. 8.60
E = lMv 2 +!My2 +lkc 2
i.e.,
242
while that for m2 will be
d'x = - la:, ,
i.e., E =lMV' +!Ia' ....(i) m2 ----f
dl
d x2
i.e. , - - = - - x
de 2
k
m2
.... (ii)
4 2
2 2
d 2x d x , d x2
According to given problem V =0 if x = 0.25.

so E =!x3 X('!)' =2.- J


i.e. ,
dt 2
-=--+--
dt 2 dt 2
.. .. (iii)

2 4 32 So substituting Eqns. (i) and (ii) in (iii),


Now at equilibrium position U = 0 [as x =0].
so lMv 2 = l.- J ' My2=,!,J
d'x
dt 2 =
{Irn! + rn1)1a
2
4 32' I.e., 8
So at equilibrium position:
(a) Translation KE =(1/2) My 2 = (1116) J (b) Rotational
or d 2x = - -k x
-
dt2 rn
with -=-+-
1
rn
1
rn l
1
rn 2

KE=(1/4)MV' =(1/32)J d 2x 2 , k
or - - = -00 X with 00 =-m
dl'
...... ,"
Which is standard equation of g'i fM of a particle of mass m We can use t:.xJg instead of kIm; here 6x denotes equilibrium
connected to a spring of force constant k as shown in Fig.8.60 extension of spring.
(b). This in turn implies that the system executes SHM and the
frequency of oscillation will be
r,-;---,-
I=J... {k =J... k(ml +m,)
2x V; 21t m}m2

Note: (i) The quantity m - m1m/(m] ,'+ m 2 ) has dimensions of mass


and is called reduced mass.
Nole: (ii) It has been assumed that load is shared equally by inclined
(ii) KE of the system will be 4mv2 with m = m1mzl(m] + m2) bar. Ifwe assume that bar remains horizontal. the stretch of
the springs 1 and 2 will be equal although the proportion of
andx=x\ +xl weight shared will be different. In this situation equivalent
dx 1 dx
2 dx spring constant for the upper supporting springs is
i,e., dt=7t+Tt' i,e., v=v1 +vl
(k j + k 2)
and the equivalent k for all the three springs is given by
Problem 35. A weightless bar is supported by two springs I = -I - + - I
from above and carries a mass m suspended from a .spring -
k k t +k2 k3
from its centre as shown in the Fig. 8.61. Calculate the natural
undamped frequency ofthe mass. Hence k '" k,k3 + k2 k 3
(k, +k2 + k3)
Solution: First we will calculate the
extension in each spring in equilibrium ..d f=-"- II
21t 'f;;;
position.
mg
1
k,
o ,
k,

= " k"lk'-,-:-
+'k,'k'-,-
Spring 3: x3=- 21t m(k, + k2 + k3)
k,
k,
mg Problem 36. A cylinder ofradius r and mass m rests on a
Spring 2: x 2 =--
2k, m curved path of radius R as shown in Fig. 8.63. Show that the
Fig. 8.61 cylinder can oscillate about the bottom position when
mg
Spring I: xl=- displaced and left to itself. Find the period of oscillation.
2kl Assume that the cylinder rolls without slipping.
The weight is equally shared by springs I and 2. The Solution: Restoring torque
displacement of the mass in equilibrium position is, acting on cylinder after a small
x +x displacement a, about an axis
x= 1 2 +x3
2 through point of contact 0 with
the curved path,
or l(mg mg ) mg 't" =-mgsinar:: -mgra
x='2 2kl + 2k2 +~
(as for small a,sina = a)
=mJ 4klk2 +klk3 +k2 k 3) Angular acceleration ofcylinder,
A
6l 4klk2k3 d'~
a=--
FIg. 8.63
dt'
Note: (i) For the system shown in equilibrium position, nI>=(R-r)e
k g ---,--
mg=kllx or /i= 6x ~=(R-r)e
r
since k
d'~ (R - r) d'e
0'
,.
ro= - =2rrv
Consequently a= - - =
de 2 r
--
dt 2
T

IJt Flg.a.62 ,
Moment of inertia about contact point,
Hence

T",21t -andf= -
g
-
21t 6x J =3mr
2
Now assume that pulley B is fixed;
k,
then extension of spring x b = ~ or x = 2xb'
,
d8+ g 8=0 Similarly if we imagine that pulley A is A
0'
dt' m(R -r) fixed, x = 2x a' But neither pulley B nor
pulley A is fixed.
B

x=2x a +2xb . . ,,(i)


Hence 00= g
From free body diagram of pulleys,
m(R -r) 2T = kbxb .... (ii)
and 2T=kaxa .... (iii) Fig. 8.67

0' T=2. m~ =.~6(Rg-r) -r)


If keq denotes ~quivalent spring constant,

-
T
=x=2x +2xb
keq a
Problem 37. In the Unstretched
,
system shown in Fig. 8.64, a :, ,Equilibrium From Eqns. (ii) and (iii),
, 0 '
~' /Posilion at

~r
disc pivoted at its centre has a x =2T and
spring attached at a distance a :[~ 'ia any time
, k ,
from centre and a mass m that

J1T
I
can rotate the disc. For small or
angular displacement calculate
the frequency ofoscillation. m
i.e., Fig. 8.68
Solution: In the '0
equilibrium position stretch of Fig. 8.64
spring isxO =a9 0 ' k{aBo)a
Torque of spring force and weight
Hence

i.e.,
,
balance each other,
ka 9 0 = mgr ....(i)

\(U
mg
Problem 39. Consider a semicircular
, a
In a slightly displaced position shell of mass m'and radius r which rolls
Equilibrium Position
equations of motion are without slipping as shown in Fig. 8.69. . CG _
Fig. 8.65
mg_- -T =ma = mra .... (ii) Use the fact that maximum kinetic energy
A
k(a,~}a
, at mean position is equal to maximum .
and Tr-ka 2 (€l o +9)=/0:. ... :(iii)' , Fig. 8.69
, 1a potential energy at extreme position to
Substituting the value of T fro~ Eqn.. find out the frequency of oscillation. .
(ii) into Eqn. (iii), mg Solution: According to 'problem, shell is in pure rolling,
we get · Fig. 8.66 contact point A can be cons'idered to be the instantaneous
m(g-ra)r-ka'(8 o +e)=/a Force diagram for a centre of rotation.
further angular
ka' displacement e
a =- a ,
(l+mr2)
where lA =ICG +m(r-a)
Comparing above Eqn. with necessary condition of SHM, i. e.,
2 =10 _ma 2 +m(r-a)2 =2mr(r-a)
CI. = -ro S,
(PE)max = mga(l-cosS)
r;;;? I r;;;?
ro=V~ - andf=21tV~' Since [(KE)max ]meanposition = [(PE) max ]extreme position
mr(r - a)ro2 = mga(l-cosS)
Problem 38. If the mass of the pulley shown in Fig. 8.67
above is small and the cord is inex'i~nsjble. find the liatural ga(l-cosS)
and (0=
frequency of oscillation of the system. r(r-a)
Solution: Let T be the tension in the cord and x a and x b
the displacements of pulleys A and B respectively.
EXERCISE

IAJ Only One Chotce is Correct (a) nmls (b) 0.5 mI,

I. Whicb one ofthe following is SHM? (CBSE 1994) (e) 5nx 10- 5 mls (d) 100nmis
(a) Motion ofa particle ina wave moving through a string 9. The displacement of a particle executing SHM is given by
fixed at both ends y = 0.2Ssin (200t) em. The maximumspeed of the partieieis:
(b) Earth spinning about its own axis (a) 200 emlsec (b) 100 cm/sec
(e) Ball bouncil'!g between two rigid vertical walls (c) 50 cm/sec (d) 0.25 cm/sec
Cd) Particle moving in a circle with uniform speed 10. A particle is executing SHM with amplitude A and has
maximum velocity VO' Its speed at displacement A ! 2 will
2. The potential energyU(x) ofa particle executing SHM is
be:
given by :
k 2 (a) (.J3'f/o/2 (b) Vo l.fi
(a) U(,) ""2 (x-a )
(c) Vo (d) Vo/4
(b) U(JI) =k,x+k2x2 +k)x 3. (Hint: Sce solved Problem 4)
II . A body of mass 5 g is executing SHM about a point 0 with
(e) Vex} =Axexp(-bx) amplitude 100 em. If its max. velocity is 100 cmlsee. its
Cd) U(x) = constant velocity will be 50 cm/sec at a distance (in cm):
3. A small creature moves with constant speed in a vertical (a) S (b) so.fi (c) SW3 (d) IW3
circle on a bright day. Its shadow formed by sun on a
12. A body executing simple harmonic motion has maximum
horizontal plane:
ace.:
(a) Describes a unifonn circle
(a) In the mean position
(b) Describes a parabola
(b) At the two extremities
(e) Executes SHM
(c) In any position
Cd) Moves in an irregular way
Cd) The question is irrelevant
4. Two SHMs are given by y\ =asin[(nI2)t+$)andY2 = b
13. A simple harmonic oscillation has got a displacement
sin [(211:1 / 3)+ $]. The phase difference between these after
lsecis: O.02mand acceleration 2.0 m/s 2 at any time. The angular

(a) n (b) n l 2 frequency of the motion is :

ec) n!4 (d) nl6 ' (a) IOrad s- 1 (b) 0.1 radls
5. How long after the beginning of motion is the displacement (e) IOOrads- 1 Cd) lrads- I
of a harmonically oscillating point equal to one half its 14. The maximum speed ofa particle executing SHM is I mls
amplitude if the period is 24 sec and initial phase is zero? and maximum acc. is I ~57 mIs 2. Its time period is:
(a) 12 sec (b) 2 see (a) 4 sec (b) I.S7 see
(c) 4 see (d) 6 sec (e) 2 sec (d) (1 II.S7) soc
[Hint; See § 8:1 (D) (a) (2)] . " 15. For a particle executing SHM the KE is given by K =Ko
6. A simple harmohic motion has an amplitude A and time cos 2 wt. The'maximum value of potential energy is:
period T. The time required by it to travel from x = A to
(a) K 0 (b) Zero
x = A ! 2is: {CBSE 1992]
(e) K 0 1 2 (d) Not obtainable
(a) TI6 (b) TI4
16. The KE and PE of a particle executing SHM with amplitude
(c) TI3 (d) TI2 A will be equal when its displacement is:
[Hint: See solved Problem 3]
(a) A.fi (b) A 12 (c) A l.fi (d) A.fii3
7. A simple hannonic motion has amp.litude A and time period
[Hint: See Question IV) ,
T. The maximum velocity will be :
17. A body executes SHM with an amplitude A. At what
(a) 4AT (b) 2A displacement from the mean position is the potential energy
T
ofthe body one-fourth of its total energy? (eRSE 1993)
(c) 2a.JA I T (d) 2nA I T
(a) A I 4 (b) A I 2
8. A particle executes SHM with amplitude 0.5 cm and (c) 3A/4 (d) Some other fraction of A
frequency 100 s - \ . The maximum speed of the particle will [Hint: See Question IV]
be:
18. A particle executes SHM with a frequency f The 26. The variation of the acceleration a of the panicle cJ\.ecuting
frequency with which its KEoscillates is: (MNR 1994) SHM with displacement y is as shown in the figure:

l~ (b)l~
(a) fl2 (b) f
(c) 2f (d) 4f
[Hint: See § 8.1 (0) (d), (3)] (a)
19. The total energy of a particle executing SHM with x_ x_
amplitude A is proportional to:
(a) A'
(c) (II A')
20. Lissajous figures refer t6:
(b) A
(d) (1/ A)
(c) ~ (d)~
(a) Lines offorce of an electric dipole
(b) Lines offorce ofa magnet in earth's magnetic field 27. Two pendulums of lengths 1.69 m and 1.44 m start
swinging together. After how many vibrations will they
(e) The path ofa fluid particle in motion in case of steady
again start swinging together?
flow
(a) 8 vibrations (b) 10 vibrations
(d) The path of a particle under the action of two
orthogonal SHM (c) 12 vibrations (d) 14 vibrations
21. The resulting motion of superposition of two simple 28. A small body of mass 1 kg is executing SHM with
hannonic motions of the same period, amplitude and phase amplitude 0.1 m and period 0.4 s. The maximum force
acting mutually perpendicular is: (CPMT 1993) acting on the body is :
(a) Circle (b) Parabola (a) 12.25 N (b) 24,75 N
(c) Straight line (d) Ellipse (c) 34.75 N (d) 44.75N
22. A particle is subjected to two mutually perpendicular SHM 29. The period of vibration ofa simple pendulum is:
such thatx =2sin rot and y = lsin [rot + (7[/4)]. The path of (a) T=2n(Llg) (b) T=2n~(Llg)
the particle will be: (eDSE 19941
(c) T=(n/2)(Llg) (d) T=2n.fi/i
(a) An ellipse (b) A straight line
(c) A parabola (d) A circle 30. A simple pendulum has some time period T. What will be
the percentage change in its time period if its amplitude is
23. A ;!article petfoons SHM along a straight line with the
decreased by 5% :
period T and amplitude A. The mean velocity of the particle
averaged over the time interval during which it travels a (a) 6% (b) 3%
distance A I 2 starting from the extreme position is: (c) 1.5% (d) It will remain unchanged
(a) A (b) A 31. A given pendulum has a spherical solid bob of aluminium.
ZI' T Its period is 1 sec. Now this bob is replaced by a spherical
(c) 2A (d) 3A solid bob of the same diameter but made of iron. Density of
T T aluminium is2.7 glccand that of iron is 9.8 glee. The period
24. A particle petfonn SHM with period T and amplitude A. of oscillation of the pendulum will now be :
The mean velocity of the particle averaged over quarter (a) (26/9) sec (b) (9127) sec
oscillation is: (c) I sec (d) 2 sec
(a) A (b) 2A 32. A girl is swinging on a swing in the sitting position. If
4T T another girl comes and sits, the period of its swing will :
(c) 2A (d) 3A (a) Decrease
T T (b) Increase
25. A horizontal platfonn is made to execute SHM of (c) Remain unchanged
amplitude a in the vertical direction. An object placed on (d) First increase and then decrease
the platfonn will lose contact with it, when the frequency of
oscillation exceeds: 33. A simple pendulum is made of a bob which is a hollow
sphere full of sand suspended by means of a wire. If all the
(a) ! Ii
nV-;;
(b) 1 Jg
21t V-;
sand is drained out, the period of the pendulum will:
(a) Increase (b) Decrease
(c) 2~ (d) 2n~ (c) Remain constant (d) Become erratic
413

34. The period of a simple pendulum is doubled when: - 42. A pendulur.l suspended from the ceiling of a train has a
(a) Its length is doubled period T when the train is at rest. When the train is
(b) Its length is halved accelerating with a uniform acceleration a, the period of
(c) The length is made four times oscillation will :
Cd) The mass of the bob is doubled (a) Increase (b) Decrease
35. A simple pendulum has a timeperiodT, when on the earth's (c) Remain unaffected (d) Become infinite
surface and Tz when taken to height R above the earth's
surface, where R is the radius of the earth. The value of
T2 IT, is: [I1T20011
(a) 2 (b) I
(c) .J2 (d) 4
36. A simple pendulum is made of a bob which is a hollow
sphere full of mercury suspended by means of a wire. If a (c) 2rt./LI g,;n 9 (d)2rt./Loos9/g
little ,mercury is drained off, the period of the pendulum will : [Hint: See solved Problem 15]
(a) Increase (b) -Decrease 44. A simple pendulum with a solid metal bob has a period T.
(c) Remain the same Cd) Become erratic The metal bob is now immersed in a liquid of density
37, A hollow metal sphere is filled with water through a small one~tenth that of the bob. The liquid is non~viscous. Now
hole in it. It is hanging by a long thread and is made to the period of the same pendulum with its bob remaining all
oscillate, Water slowly flows out of the hole at the bottom. the time in the liquid will be :
How will the period of oscillation be affected? (a) T (b) 9/lOT
[CBSEI997] (c) (.J10/9)T (d) (.J9/1O)T
(a) The period will go on increasing till the sphere is empty 45. The time period ofa simple pendulum is T. The pendulum is
(b) The period will go on decreasing till the sphere is empty charged positively and is made to oscillate while a
(c) The period will not be affected at all negatively charged plate is placed below it. The time period
(d) The period will first increase then decrease till the of the pendulum is now:
sphere is empty (a) >T (b) <T
38. The period of oscillation ofa simple pendulum of constant (c) =T (d) 00
length at a place inside a mine is : . 46. A pendulum is undergoing SHM. The velocity ofthe bob in
(a) More than it is on the surface ofthe earth the mean position is Vo' If now its amplitude is doubled,
(b) Less than it is on the surface of the earth keeping the length same, its velocity in the mean position
(c) The same as it is on the surface of the earth will be :
(d) The same as it is on the surface of the moon (a) Vo 12 (b) 21'0
39. The value of g decreases by 0.1% on a mountain as (c) Vo (d) 41V0
compared to sea level. To record proper time by means of a 47. A simple pendulum performs SHM about x=O with an
simple pendulum its length must be: amplitude A and time period T. The speed of the pendulum
(a) IncreasedbyO.l% (b) DecreasedbyO.1% atx=AI2willbe:
(c) Increased by 0.2% (d) Decreased by 0.2% (a) AIT (b) (.J3)oAIT
40. How will the period of oscillation of a pendulum hanging (c) oA(.J3)IZT (d) 30'AIT
in an ascending lift change if the lift moves with unifrom
acceleration? 48. If a simple pendulum of length L has maximum angular
(a) The period will increase displacement EI then the maximum kinetic energy of its bob
ofmassmis:
(b) The period will decrease
(c) The period will not change (a) (1/2)m(LI g) (b) (1/2) mg
L
(d) The period will first increase and then decrease
(c) mgL(I-oos 9) (d) (1/2)mgLsin 9
41. The time period ofa simple pendulum in a freely falling lift
is: 49. The phase difference between two similar pendulums A and
B is 90°.When the bob of the pendulum A has maximum
(a) Zero (b) Infinite velocity Vo' the bob of the pendulum B will be:
(c) Finite (d) None of these
414

(a) Stationary 59. -A spring pendulum has a frequency Ii; ifhalf of the above
(b) Moving with velocity (VO ) spring is used as a spring pendulum its new frequency is 12 ;
then:
(c) Moving with velocity (-Vo)
(d) Moving with velocity (VO 12) (a) /j =(.fi)f2 (b) /j=f2

50. Two simple pendulums of length 1 m and 16 m respectively (e) f2 = 2/j (d) f2 =(.fi)/j
are both given small displacements in the same direction at
60. A weightless spring which has a force constant k oscillates
the same instant. They will again be in phase after the
with frequency Iwhen a mass mis suspended from it. The
shorter pendulum has completed n oscillations where n is :
spring is cut into two halves and a mass 2 m is suspended
(a) 114 (b) 4/3 (e) 5 (d) 4 from one of the halves. The frequency of oscillation will
[Hint: See solved Problem 13] now be:
51. If the length of a simple pendulum is equal to the radius of (a) f (b) 2f
the earth, its time period will be :
(e) f 12112 (d) fx 2'12
(a) 2n,JRIg (b) 2n,JR12g
61. The vertical extension in a light spring by a weight of I kg
(c) 2rr,J2R / g (d) Indeterminate suspended from the lower end is 9.8 cm. The period of
[Hint: See § 8.2 (5) (e)] oscillation of the spring is :
52. The time period of a simple pendulum having infinite (a) 20n sec (b) (2n 11 0) sec
length is : (c) 2n/iOO sec (d) (200n) sec
(a) cQ (b) 2nJRTi 62. A force of 6.4 N stretches a vertical spring by 0.1 m. The
(e) 2n.f2.R/i (d) 2n,)R 1 2g mass that must be suspended from the spring so that it
oscillates with a periodof(nI4) sec is: (Roorkee 19901
[Hint: See § 8.2 (5) (b)]
(a) (nI4) kg (b) I kg
53. A pendulum beats seconds; the time period will be:
(e) (lin) kg (d) 10 kg
(a) 1 sec (b) 2 sec (c) 4 sec Cd) 112 sec
63. A clock S is based on oscillations of a spring and a clock P
54. The length of a second's pe.1dulum at a place where g is 980
is based on pendulum motion. Both clocks run at t~e same
cm/sec 2 is nearly: rate on earth. On a planet having the same density as earth
(a) 100 cm (b) 200cm but twice the radius:
(c) 50 cm (d) 25 em (a) S will run faster thanP
55. The length of a second's pendulum on the surface of earth is (b) P will run faster than S
1 m. The length of same pendulum on the surface of moon,
whereg is (1/6) ofthe,valueofg on the surface of earth is: .
(c) Both run at the same rate as on the earth .
(a) 36 m (b) (1136) m (d) Both run at equal rates but not the same as on the earth
(e) I m (d) (116) m 64. Two bodies M and N of equal masses are suspended from
two separate massless springs of spring constants k, and k2
56. If the length of a second's pendulum is increased by 2%,
how many seconds will it lose per day? (CPMT 19921 respectively. If. the two bodies oscillate vertically
.
such
Y
that
their maximum velocities are equal, the ratio of the
(a) 3927 s (b) 3727 s
amplitude of vibration of M to th~t of N is :
(e) 3427 s (d) 864 s
(a) k, I k2 (b) ~k, ik2
57. The natural angular frequency of a particle of mass m
attached to an ideal spring of force constant k is : (d) ~k2 I k,
(a) (kl m/ 12 (b) (ml k)1I2
65. Two springs of force constants k, and k2
(e) kim (d) mk have been arranged parallel to each other
k,
58. Two masses m1 and m2 are suspended together by a and a mass M is attached to the combination.
massless spring of spring constant k. When the masses are This arrangement is equivalent to a single
in equilibrium, m1 is removed without disturbing the
system. The angular frequency of oscillation is :
spring of force constant k given by :
(a) (k, '+k2 )1 k,k2
~ M
(a) ~klm, (b) ~klm2 (b) k,k 2 /(k,+k 2 ) Fig, '8,70

(e) 1/2,}"',/1k (d) 112,Jm;7k (e) (k, +k2)


[Hint: See solved Problem 19] (d) (k, +k2 )/2
"5
66. A mass m oscillates with SHM with frequency f = w12'1t 72. The time lJeriod and the amplitude of a simple pendulum
and amplitude A on a spring offorce constant k. Therefore: are 4 second and 0.20 metre respectively. If the
(a) The total energy of the spring is (1/2)k A 2 displacement is 0.1 m at time t ",, 0, the equation of its
displacement is represented by :
(b) The frequency is (1/2n}.j(kl m)
(a) y ~ 0,2sin(0.5!!1) (b) y~ 0,2sin (O,S,I)
(e) The maximum velocity occurs when x =0
(c) y=O.lsin(nt+x / 6) (d) y=O.lsin(O.51tl)
(d) All the above statements are correct
73 . Two blocks each of mass m
67. InFig.8.7 I,S I andS 2 are ~ arc connected with springs
identical springs. The 51 52 '
each of force conslant k as
oscillation frequency of the ~
mass is f. If one spring is shown in the Fig. 8.73. Fig. 8.73
removed,the frequency will Fig. 8.71 Initially the springs are
become: relaxed. The mass A is displaced to the left and B to the right
(a) [ (b) 2[
by the same amount and released, then the time period of
oscillation is :
(c) (./2)[ (d) [1./2
68. Two SHMs with same frequency act on a particle at right
angles, i. e. ,alongxand y-axis. If their amplitudes are equal
(a) 2.ffJ (b) 2.Jff
and the phase difference between them is (1[/2), then the
resultant is : {eRSE 19971
(c) .ffJ (d) .Jff
(a) A straight line inclined at 45° to the x-axis {Hint: After collision the blocks come to rest as moving with
(b) An ellipse with major axis along x-axis equal velocities is opposite directions. So, each mass complete
half oscillation.]
(e) An ellipse with major axis along y-axis
74. A rod of mass M and
(d) A circle
length L is pivoted al its
69. On a smooth inclined plane, k
centre and can rotate in
a body of mass m is attached
between two massless a vertical plane. Two
springs. The other ends , of springs of force o
the springs are fixed to firm constant k are
k
supports. If each spring has connected at its ends as
force constant k, the period Fig. 8.72 shown in the Fig. 8.74.
of oscillation of the body is : The time period of Fig. 8.74

(a) 2.Jff (b) 2.ff SHMis:

(.) 2-!! (b) 2~


(c) 21t~mg~n 9 (d) 2~2mg:n 9
70. A particle executes SHM. Its velocities are vI and v2 at
(c) 2~ (d) 2- 1M
"'13k
displacements xI and x2 from mean position respectively. 75. A particle is subjected to two SHMs,
The frequency of oscillation will be :
XI =Alsin rot and X 2 = A 2 sin ( rot+~)
(a) 1 [v~ +v~ ]112 (b) 1 [v~ -v~]112
21t x I2 +x22 2
2x X'2 - x I The resultant SHM will have an amplitude of: {lIT 19961
(a) (A , +A,) /2
(d) 1
21t
[xi\If -vi
-xr ]"2 (b) ~A~ +Ai
(e) ~At +Ai +..fiA 1A 2
71. A particle executes SHM iJ:l a straight line. In the first
second starting from rest it travels a distance a and in the 76. If for a particle moving in SHM there is a sudden increase
next second a distance b in the same direction. The of I % in the restoring force just as the particle is passing
amplitude of SHM is : through the mean position, what percentage change will be
(a) a-b (b) 2a - b given to the amplitude:
20' (a) 1% (b) 2%
(c) (d) a
Ja - b b (c) 0.5% (d) Zero
416

77. A particle moving with SHM from an extremity of the path 82. A small mass executes linear simple harmonic motion
towards centre is observed to be at distances x I ,x2 and x3 about a point 0 with amplitude a and period T. Its
from the centre at the ends of three successive se<:onds. The displacement from 0 at time!.. after passing through 0 is :
period of SHM is : 8
2. 2. a a
Ca) Cb) Ca) - Cc) - Cd) -"-
COS- 1(_XI_;_;_' ) co, _I(XI
- - +X,) 8 2 .J2
lx, 83. A particle executes simple harmonic motion between
2. 2. x _ -A andx:+A. The time taken by itto go fromOto A 12
Cc) Cd) is1j and to go from AI2 toA isT2 .1ben: IIIT2001)
cos-l(Xl~X3) cos-J(XI~X2) Ca) TI <T, (b) TI > T,
Cc) TI =T, Cd) TI =2T,
78. A mass M supported by a spring has a static deflection a- 84. Masses mand 3rnare attached to the two ends ofa spring of
The frequency of oscillation is :
spring constant k.lfthe system vibrates freely, the period of
Ca) 1- ("f Cb) 1- f[ oscillation will be :
'1M :at Vi
Cc)
2ft

L,fiiS Cd) 1- ~MB


Ca) .Jf (b) 2.J¥
21t 2ft g
79. A particle executes SHM starting from its mean position at (c) 7tff Cd) 2.J¥
t = Q If its velocity is ../3browhen it is at a distance b to the
85. A thin uniform rod of mass M and
A
mean position, where co "" 21t, the lime taken by the particle length L hangs from a frictionless
T pivot and is connected at the
to move from b to the extreme position on the same side is: bottom by a spring to the wall as M
S. • shown in the Fig. 8.76. L
Ca) - - Cb)

L
6ro ~ The spring constant is k. The k
Cc) ,~. Cd) • system is allowed to oscillate by
= 4", pressing end B of the rod and
80. A particle moves along a y releasing. The period of Fig. 8.76
circle of radius 5 em with a oscillation will be:
period of 6 sec in the
clockwise direction. At t = 0 Ca) T C 2~ Cb) T=2~Mg +k
the particle is at position A .1"----1-, ML
as shown in the Fig. 8.75. Cc) T=2.~ Cd) T=2.~
The expression for the SHM fii+3Mg ~~
of the x-projection of the
radius vector of the particle 86. A uniform rod oflength 2.0 m is suspended through an end
Fig. 8.75 and is set oscillating with small amplitude under gravity.
is:
The time period of oscillation is approximately:
(a) X=5COS(~t+~) (b) X"'-5Sin(~t+i) [AMU20011
Ca) 1.60, Cb) 1.80,
(c) X=5sin(~/+~) (d) X=-5cos(~t-i) Cc) 2.00 s Cd) 2.<1{),
2
81. A particle is executing a simple hannonic motion of period 87. Ifa simple harmonic motion is represented by d x + ax = Q
2s. When it is at its extreme displacement from its mean dt'
position, it receives an additional energy equal to what it had its time period is: IAIEEE 20051
in its mean position. Due to this in its subsequent motion: Ca) 2.a (b) 2• ./0.
(a) Its amplitude will change and become equal to .fi
times its previous amplitude
Cc) 2. Cd) ~
a ./0.
(b) Its periodic time will become doubled, i.e.,4 s
88. The length of a simple pendulum is increased by 44%
(c) Its potential energy will be decreased What is the percentage increase in its time period?
(d) It will continue to execute simple harmonic motion of [SEE CUPTU) 20051
the same amplitude and period as before receiving the
(a) 10% Cb) 20% (c) 40% Cd) 44%
additional energy
89. A pendulum is undergoing SHM with frequency f. What is (a) 2/ (b) //2
the frequency of its kinetic energy? (e) / /4 (d) 4/
[BHU 2005; Orissa .;rEE 2009J 96. Velocity of a particle executing v 3T
(a) //2 (b) 2/ SHM varies with time as shown 4 T
(e) 3/ (d) 4/ in Fig. 8.78. Variation of T T
90. A rectangular block of mass m and area of cross-section A displacement of the particle 4 2
floats in a liquid of density p. If it is given a small vertical from the mean position with
time will be as in : Fig. 8,.78
displacement from equilibrium it undergoes oscillation
with a time period T. Then: leDSE 2006J
Y. Y T
1
(a) T oc.JP (b) Toc-
,fA
4 2
1 1
(c) T oc;- (d) Toc-
P .In, Y 3T
91. The maximum yelocity of a particle executing simple hi-"",,",...<T'-.t (d)
harmonic motion with an amplitude 7 mm, is 4.4 mls. The 3T
period of oscillation is : [AIEEE 2006) 4 4
(a) 0.1 sec (b) 100 sec
97. Two particles A and Bare . executing SHM of same
(c) 0.01 sec (d) 10 sec frequency and of same amplitude along the same straight
92. Starting from the origin a body oscillates simple line. It is observed that maximum distance the two is .J2
harmonically with a period of 2s. After what time will its times the amplitude. Initially, phase difference between the
kinetic energy be 75% of the total energy? IAIEEE 2006] particles is then:
1 1 I 1 (a) 9QO (b) 60°
(a) '3sec (b) 12sec (c) 6sec (d) 4sec
(e) 45° (d) 30°
93. A coin is placed on a horizontal platfonn which undergoes 98. A frictionless inclined surface having inclination 30° has a
vertical simple harmonic motion of angular frequency ro. 2.5 kg mass held by a spring which is fixed at the upper end.
The amplitude of oscillation -is gradually increased. The If the mass is taken 2.5 cm up along the surface,'tension in
coin will leave contact with the platfonn for the first time: the spring reduces to zero, On releasing the mass now, its
[AIEEE 2006[ angular frequency of oscillations will be !
(a) for an amplitude of g 2 lui (a) 9 (b) 14
(b) at the highest position of the platfonn (e) 2 (d) 5
(c) at the mean position of the plat£:onn 99. Weight ofthe bob of a simple pendulum is W, Length of the
pendulum is I and it is vibrating with an amplitude A.
(d) for an amplitude of g I ui
Maximum tension in the string during oscillation will be :
94. A particle starts SHM from the mean position. Its amplitude
is a and total energy E. At one instant its kinetic energy is (a) W[I+ ~: 1 (b) W[I+ ~r
3 E.. Its displacement at that instant is :
W[I-~l
4
(e) W (d)
IWestDengalJEE 20071
a a 100. A system of masses In! and m is suspended by a ma~sless
(a) '- (b) 2 2
..[2 spring. At the instant when the system is in equilibrium.
a mass m2 is removed as a result of which the other mass
(e) (d) 3.- begins to oscillate, then:
.J3
lD
95. Two springs, of force
(a) frequency of oscillations and also the amplitude
depend on both m! and m2
(b) frequency depends on m2 only and amplitude depends
constants kJ and k2' are on m! only
connected to a mass m as Flg.8.n
shown. The frequency of (c) frequency depends on m! only and amplitude depends
oscillation of the mass is f. Ifboth k! and k2 are made four on m2 only
times their original values, the frequency of oscillation (d) frequency depends on both 11l! and m2 but amplitude
becomes: IAIEEE 2007) depends only on m2
101. A particle is executing SHM with amplitude 8 em and time 106. An object of mass mis placed on a horizontal platform. The
period 12 sec. X and Yare two positions on either side of platfonn is performing SHM along a vertical line with an
mean position and at a distance 4 em from the mean amplitude' A'. It is observed if the oscillation frequency is
position. Minimum time taken by the particle to move from more than Vo the object leaves contact with the platform,
X to Y during oscillation is : then vo> is:
(a) 6 sec
(e) 4 sec
.:' '.
(b) 3 sec
Cd) 2 sec (,) 2'(~ ) (b) 2.P;
H
"

102. Length of a spring of force constant k in its unstretched


condition is l. The spring is cut into two parts which have (e) I jg (d)
21t VA
~eir unstretched lengths in the ratio II :12 =q:l Force
constants ofthe two parts k J and k2 are then: 107. A particle is moving such that its displacement as a function
of time is given by:
(a) k(q + I),kq (b) k (q+ I) ,kq
q x (x- 6)= I-lOcos rot
Then:
(e) k(q+I),k(q+l) (d) k(q - i),k(q+l)
q q q (a) its motion is translational
(b) its motion is oscillatory but not 1?HM
103. The system shown in Fig. 8.79 is initially in
equilibrium. When the 500 gm mass is removed, (c) its motion is SHM
the system oscillates with a time period 4 sec. On (d) its motion is circular
removing the 700 gm mass also, time period of 108. The bob of a simple pendulum executes simple harmonic
oscillation will be: 900gm motion in water with period T while the period of
(a) 2 sec 700gm oscillation of the bob is To in air. Neglect frictional force of
(b) 3 sec 500gm water and given that the density of the bob is ~ x 1000
(e) 4 sec 3
Fig. 8.79
kg I m 3 . What relationship is true between T and To?
(d) 5 sec
104. A force FJ when it acts on a body causes it to execute SHM ~
(,) T ~ To (b) T~-.!l.
of time period T, whereas another force F2 when actions on 2
the same b9dy causes it to execute SHM of time period T2 . (e) T~zro (d) T~4To
If both forces act. on the body simultaneously and in a
109. A particle at the end of a spring executes simple harmonic
manner that they are always acting in the same phase, time
motion with a period T, while the corresponding period for
period of SHM will be:
another spring is T2 . If the period of oscillation with the two
Jr.,2 + r.22 springs in series is T, then:
(a) ~T.2, + r.22 (b)
T,T2 (a) T =T] +T2 (b) T2 =Tt +T}
I I I I I I
r.,2 r.22 r.,2 + r.22 (e) -=-+~ (d) --~ -- + --
(e) (d) T T, T2 T2 r,2
,
r.22
, + r.2
r,2 2 T]T2
110. A particle starts SHM from the mean position. Its amplitude
105. Velocity·time graph of a particle performing SHM is as is A and its time period is T. At some time, its speed is half
shown in Fig. 8.80. Amplitude of oscillation is then nearly: that of the maximum speed. Displacement of the particle at
v
this instant is :
+3
+2 .JiA (b) .J3A
(a) 3
+1 2
0 +__~~+O~r-~1.+0~O__ lis) (e) 2A (d) 3A
-1 .J3 .Ji
-2
Ill. Two particles are executing simple harmonic motion of
-3
same amplitude and same frequency along the same line.
Fig. 8.80 They pass each other when going in opposite directions
(a) ZOcm (b) 48 cm every time their displacement is half their amplitude. Phase
(e) 75 cm (d) 8 em difference between them is:

(a) - •3 (b) ~
4
(e) - •6 (d) 2•
3
11 2. A parti~le is executing SHM of time period 24 sec and 11 8. An object of mass 4 kg is moving along X -axis and its
amplitude 20 om. At I ""0. its displacement is +20 em. potential energy as a funct ion of x can be expressed as
Shortest time taken by the particle in moving from x= +IO U(x)= 4 (I-cos 2x)joule
cmto x ",,- !O em is:
then time period for small oscillations is :
(a) 4 sec (b) 2 sec
(c) 8 sec (d) 3 sec •
(a) - sec
2
•4
(b) - sec
11 3. A particle of mass m is attached to a spring of spring 3.
constant k and has a natural angular frequency (00' An (c) nsec (d) - sec
2
external force PCr)proportional tocos rot, ro"# (\lois applied
to the oscillator. Amplitude of the oscillator is then 119. Time period of a particle in SHM is 8 sec. At t = 0, it is at
proportional to : mean position. Ratio of the distances travelled by it in the
m first and second seconds is :
(a) (b)
(J)~. -oi m(O)~ _ co 2 ) (a) - (b) _,

I m
2 -!i,
(c) (d) I (d) _,
m (ro~ +c,}) Cii0 + ro2 (c) - -
-!i - I ..[3
114. A body of mass 1 kg is suspended from a weightless spring

X~,
120. Displacement time graph of a
having force constant 600 N/m. Another body of mass 0.5
particle executing SHM is as
kg moving vertically upwards hits the suspended body with
a velocity 3 mls and gets embedded into it. Frequency of
shown In the Fig. 8.82.
osci llation and the amplitude oflTKllion are: Corresponding force time graph of
the particle is: . Fig. 8.82
10 10
(a) - Hz, 10 em (b) - Hz, 5 em
• •
5
(c) -Hz,Scm

(d)
5
- Hz,I Ocm
• (a) l --,
~ (b)
FI~
/,
115. Fig. 8.8 1 shows an object of mass 50 I~I
gm placed on a rough horizontal __-"~
surface. The surface together with
the block on it is performing simple
".",,,,,;,,._ _

Fig. 8.81
harmonic motion along the horizontal with a time period
(c) F~, (d) FlP-,
0.5 sec. Coeffici ent of friction between the block and the
surface is !. 'Max imum amplitude of SHM 'such that the 12 1. Two blocks, each of mass m k . m

~OOOOOOO~
5
m are connected by a
object docs not slip along the surface is : (g = 10m/s 2 )
massless spring of spring
(a) 6 cm (b) 8.5 cm Fig. 8.83
constant k and the system
(c) 2.5 cm (d) 1.2 cm is placed on a frictionless surface as shown. Time period of
11 6. A simple pendulum oscillates in a vertical plane. When it system for small oscillations will be :
passes through the mean position, tension in the string is 3
times the weight of the pendulum bob. What is thc (a) 2.fff (b) 2.f¥I
maximum angular displacement of the string of the
pendulum with respect to the vertical? (0) 2.J¥ (d) 2- 1m
(a) 30· (b) 45· "V4k
(c) 60· (d) 90· 122. Time period of a spring mass system is T. If this spring is
117. Displacement of a particle executing SHM can be expressed cut into two parts whose lengths are in the ratio I : 3 and the
as some mass is attached to the longer part, the new time
period will be: [CET (J & k) 2006)
x =asin (rot +B)
At t '" 0. it is observed that the particle is at x = ~ and (a) -fJi2r ·(b) ~
2 ..[3
moving along positive x direction. Then Bcould be : ..[3r
fi fi 5n (c) (d) ..[3r
(a) - (b) - (c) - (d) ." 2
3 6 6 4
123. Four massless springs
whose force constants are '
2k,2k,k and 2k respectively
I:::im
2k2k
M
' 2k
127. A particle of mass m perfonns oscillatory motion of
amplitude A on the x-axis, about the origin. The restoring
force on the particle is given by F = -ex 3 where c is
are attached to a mass M constant and x is the instantaneous displacement. The
kept on a frictionless plane Fig. 8.84 period of oscillation is: lIlT 19931
(as shown in Fig. 8.84). If the mass M is displaced in the (a) Independent of A (b) Proportional to A 113
horizontal direction, then the frequency of the system: (c) Proportional to A -I. (d) Proportional to A - 1I2
(liT 19901 128, A particle of mass M is executing oscillations about the
(a) I~
21t 4M
(b) -'-HI
2. M
origin on the x-axis. Its potential energy is IUI= k Ixl) where
k is a positive constant. If the amplitude of oscillation is a,

(e) 1M;
21t 7M
(d) -'-lJ
2. M
then its period Tis :
(a) Proportional to lIJa (b) Independent of a
lilT 19981

124. One end of a long metallic wire of length L is tied to the (c) Proportional to Ja (d) Proportional to a)/2
ceiling. The other end is tied to a massless spring of spring 129, A partic!e of mass nI and chafgr. -q
constant k. A mass m hangs freely from the free end of the moves along a diameter of a
spring. The area of cross-section and the Young's modulus uniformly charged sphere of radius
R and carrying a total charge +Q. The
I-;::::::'-\Q
of the wire are A and Y respectively. If the ,mass is slightly
pulled down and released, it will oscillate with a lime frequency of the SHM of the
period Tequal to: lIlT 1993J particle, if the amplitude does not
exceed R, is : Fig. 8.85
(a) 2n.j(ml k) (b) 2n.jrn(YA +kL)1 YAk , -;;--
qQ qQ
(e) 2n.j(rnYA I kL) (d) 2• .JmLi AY (a) (b)
2, 4m: R 2 m 21t 41tEo m R 3
o
125, A unifonn cylinder of length L and mass M having
cross-sectional area A is suspended, with its length vertical,
from a fixed point by a massless spring, such that it is half
(e) I~
21t 4ltEonlR
(d) I ~
V
21t 4ltEoR2
submerged in a liquid of density p at equilibrium position. 130, End A ofa metallic wire of Young's
When the cylinder is given a small downward push and modulus Y, ~rea of cross-section a
released it starts oscillating vertically with small amplitude. and length / is fixed and the other end
If the fo rce constant of the spring is k, the frequency of B is tied to a horizontal, massless
oscillation of the cylinder is : lIlT 1990) rod. Two springs of spring constants
(a) ..l(k_A?g)"2 (b) ..l(k+APg)"2 k J and k2' respectively, are attached
to this rod and hung vertically down.
21t M 21t M
The other ends of the springs are tied
-,-(k+MpgL2)"2 (d) -'- k+Apg
"2 to another horizontal massless rod to
which a mass M is attached. If the
(e) 2, 2n: ( Apg )
mass is pulled down slightly and FIg.e,,,
126, A particle free to move along the x -axis has potential released, the frequency of oscillation
energy given by: of the system is:
U(x) = k[l-exp( _x 2 )] fo r -co S;x::5;+oo (a) _ kl +k2 (b)
211: k lk2M 2.
where k is a positive constant of appropriate dimensions.
I ~ k,k 2
Then, (lIT 1999(
(e) (d)
(a) At points away from the origin, the particle is in stable 21t (k l +k2 )M 2, [Ya+/ (k , +k2 )] M
equilibrium
13 1. There is a spring with natural length Lo . Two
(b) For any finite non-zero value of x, there is a force
. masses nil and m2 are conn~ted to both of its
directed away from the origin
ends as shown in Fig. 8.87. The whole system is
(c) If its total mechanical energy is kf2., it has its minimum
kinetic energy at the origin
a.
held at rest. At any time t =: nl2 is released and
system starts free fall. Initial stretched length of
(d) For small displacements from x = 0, the motion is spring before fall is L What is the displacement
m,
simple hannonic of centre of mass as a function of time? Fig. 8.87
(a) gl' (b) ~gt' one end and oscillating in a vertical plane is T2 . Amplitude
2 of oscillations in both the cases is small. Then TJ I T2 is:
(d) In ] +ln 2 x t (BCECE 2008(
m ]ln2 1
(a) - (b)
132. A particle of mass In is .J3
attached to three identical
springs A, Band C each of
(e) ~ (d)
R
force constant k as shown in
137. The x-t graph of a particle
Fig. 8.88. If the particle of undergoing, simple harmonic
mass m is pushed slightly E
motion is shown in the Fig. "'- - I(s)
against the spring A and
8.90. The acceleration of the "
released, then the time
. 1e at t = -4
partlC sec.IS:
period of oscillation would Flg.S.SS -1
3
be: Fig. 8.90

(a) 2.J¥ (b) 2~


(a) ~ 1t 2cm/s 2
(lIT 2009]
.'
(b) =-- cm/s
2
32
(e) 2.J¥ (d) 2. 1m
'13k .'
(e) _cm/s 2 (d)
-.J3.' cm/s2
133. A particle of mass In is allowed to 32 32
oscillate near the minimum of a vertical 138. A uniform rod of length Land massM is
parabolic path having the equation pivoted at the centre. Its two ends are
x 2 '" 4ay. The angular frequency of small attached to two springs of equal spring
constants k. The springs are fixed to rigid
oscillations is given by: Fig. S.S9 supports as shown in the Fig. 8.91 and

ru
(a) jgt; (b) .)2gh the rod is free to oscillate in the

(e) nJ (d)
horizontal plane. The rod is gently
pushed through a small angle €I in one
Fig. 8.91

direction and released, the frequency of oscillation is :


134. A particle executes simple harmonic oscillation with an (lIT 2009]
. amplitude a. The period of oscillation is T. The minimum
time taken by the particle to travel half of the amplitude (a) -.!.. 12k (b) -.!.. ('k
from the equilibrium position is : 21t VAl .2.VM.
. T
(UPSEE2007; AIIMS 2007; JCECE 2008( (e) 1 16k (d) -.!..p4k
(a) -
T
(b) 8
21t V'M 2. M
4 139. Two simple harmonic motions are represented by
T (d) T YI ;; 5 (sin 21tt +.J3 cos 21tt)
(e) -
12 2
135. A point mass oscillates along the x-axis according to the law Y2 =5sin( 21tt+~)
x =x o cos (cot - 1t1 4} Tfthe acceleration of the particle is
written as The ratio ofthe amplitude of two SHM's is: IDeE 2009]
a = A cos (rot + o),then: (AIEEE 2007( (a)',' (b) 1,2
(a)
,
A =xoco , o=31t/4 (e) 2,1 (d)I,.J3

(b) A = x o ' o=-'1t/4 140. If x, v and a denote the displacement, the velocity and
the acceleration of a particle executing simple harmonic
(e) A=xool, o=1t/4 motion of time period T, then, which of the following does
(d) A =xoro2 , o=-1t/4 not change with time? IAIEEE 20091

136. Time period of a simple pendulum of length I is T] and time (a) a 2T2 +41t 2V 2 (b) aTlx
period of a uniform rod of the same length I pivoted about (c) aT+21nJ (d) aTlv
422 PHYSICS FOR COMPETITIONS - Vol. I

141. A particle at the end of a spring executes simple hannonic 147. A block of mass m is fixed wall fixed wall
motion with a period t l' while the corresponding period for connected to two springs ~2k m 3k
another spring is t2 . If the period of oscillation with the two of spring constant 2k and
springs in parallel is T, then: 3k respectively, such that
the spring of spring smooth
(a) T=t 12+/
2
2 (b) T=t 1 +t2 constant 2k is compressed Fig. 8.94
(e) T -I = , 1-I +,-2 1 (d) T
-2 ,-2 -2 by 31 when the block is in its mean position. If the block is
= 1 +/ 2
displaced slightly and allowed to oscillate, the frequency
142. The displacement of two particles executing S.H.M. are will be :
represented by equation YI =4sin (10t+$), Y2 = Sees lOt.
The phase difference between the velocities of these (a) J.- fI (b) ~ {6k (e) ~ {5k (d) ~ {5k
particles is:
21t~; 21t VS;;; 21t V~ 21t V-;;;
148. The displacement of two identical particles executing SHM
(a) 4> (b) -1>

(e) .+~ (d) 4>-~


are represented by equations Xl =4sin[IOt+(~)] and
2
x2::: 5cos rot. For what value of 00 energy of both the
143. The displacement x of a particle at a time T is given by
particles is same?
x = 6sin 21. Wherexis in meier and t is in second. A simple
pendulum has the same period as the particle when the (a) 16units (b) 6units
length of the pendulum is : (c) 4 units (d) 8 units
(a) 10.0 m (b) 5.0 m 149. At t ::: 0, a particle of mass m starts moving from rest due to
~ ,
(e) 2.5 m (d) 2.0 m a force F :::Fo sin (oot)i.

p~
144. A block of mass I kg kept

E1:\£~
(a) Particle perfonns SHM about its initial position of rest
over a smooth surface is
given velocity 2 mls (b) Particle performs SHM with initial position as an
towards a spring of spring extreme position with angular frequency 00
constant I N/m at a distance 10m (c) At any instant, distance moved by the particle equals
of 10 m. Find after what Fig. 8.92 its displacement from the initial position
time block will be passing through P again (d) Initially velocity of particle increases with time but
(a) (-IO - n)sec (b) IOsec · t "" co
2n I·tb ecomes constan t
aft er tlme
(c) (1O+21t)sec (d) (1O+1t)sec
145. A thin unifonn rod is suspended in vertical A ISO. A uniform equilateral frame in vertical plane having mass
plane as a physical pendulum about point A. 3mand length at each side /is hinged at A. Two ideal mass
The time period of oscillation is TO' "Not less spring of force constant k] and k2 are attached as
counting the point A, the number n of other • shown in the Fig. 8.95. The frequency of vibration is : (0 is
points of suspension on rod such that the time centre of triangle)
period of oscillation (in vertical plane) is again B

-,~
TO' Then the value of n is: (Since the rod is thin, Fig. 8.93
consider one point for each transverse cross
section of rod)
(a) 0 (b) I (e) 2 (d) 3 B C
k,
146. A pendulum of length L has time Fig. 8.95 .'
period T for small oscillations.
1
An obstacle is placed directly
beneath the pivot so that only the
3U4
L
(a) ~(~+~+ F3g)
2" 34m 3m I
lowest one quarter of the string
can follow the pendulun;t bob
when it swings in the l~ft of its A
(b)
2"
{!'l+ +J}g)
2m
k2
3m 21
resting position as shown in the
figure. Time taken by the bob to
~(3kl +~+ F3g)
B
return the position A if it was Fig. 8.93 (e)
released from that position, is 2" 32m 2m 21
(a) T
(e) 3TI4
(b) TI2
(d) TI4 (d)
2" 22m
~(~+ 3k2
m
+F3g)2i
ANSWERS

1. (a) 2. (a) 3. (0) 4. (d) 5. (b) 6. (,) 7. (d) 8. (a) 9. (0) 10. (a) ' 11. (e) 12. (b)
13. (a) 14. (a) 15. (a) 16. (e) 17. (b) ] 8. (e) 19. (a) '0. (d) 21. (e) 22. (a) 13. (d) 14. (d)
lS. (b) 26. (e) 27. (e) 18. (b) 19. (b) 30. (d) 31. (e) 32. (e) 33. (e) 34. (e) 35. (a) 36. Cal
37. (d) 38. (al 39. (b) 40. (b) 41. (b) 41. (b) 43. Cal 44. (el 45. (b) 46. (b) 47. (b) 48. (0)
49. (a) SO. (b) 51. (b) 51. (b) 53. (b) 54. (a) 55. (d) 56. (d) 57. (al 50. (b) 59. (d) 60. Cal
61. (b) 62. (b) 63. (b) 64. (d) 65. (d) 66. (e) 67. (d) 68. (d) 69. (a) 70. (b) 71. (e) 71. (b)
73. (e) 74. (el 75. (e) 76. Ca) 77. (e) 78. (b) 79. (b) 80. (b) 81. (a) 82. (d) 83. (e) 84. (0)
85. (e) 86. (d) 87. (d) 88. (b) 89. (b) 90. (b) 91. (el 92. (e) 93. (d) 94. (b) 95. (a) 96. (0)
97. (a) 98. (b) 99. (a) 100. (e) 101. (d) 102. (el 103. (b) 104. (e) 105. (b) 106. (e) 107. (e) 108. '(e)
109. (b) 110. (b) 111. Cd) 112. (a) 113. (b) 114. (b) liS. ee) 116. (d) 117. (b) 118. (e) 119. (e) 120. (d)
121. (b) 122. (el 123. (b) 124. (b) 115. (b) 126. (d) 127. ee) 128. (a) 129. (b) 130. (d) 131. (b) 131. (b)
]J3. (e) 134. (el 135. (a) 136. (d) 137. (d) 138. (e) 139. (el 140. (b) 141. (d) 142. Cd) 143. (e) 144. (d)
145. (d) 146. (e) 147. (d) 148. (d) "9. (b) I SO. (a)

(B ) More than One Choice is Correct (a) The amplitude is 1 m


1. The motion of Halley's comet around the sun is not: (b) The amplitude is (.fi) m
(a) Periodic (b) Oscillatory (c) Time is considered from y = 1m
(c) Simple harmonic (d) Translatory (d) Time is considered from y = 0 m ,
2. The motion of a torsional pendulum is: 7. The displacement ofa particle of mass 100 g from its mean
(,) Periodic position is given by, y =0.05 sin 41t (51 + 0.4):
(b) Oscillatory (a) The period of motion is 0.1 s
(0) Linear simple hannonic (b) The maximum acc. of the particle is 10n:1m/s 1
(d) Angular simple hannonic (c) The energy of the particle is 0.051t2 joule
3. For a particle executing SHM which of the following (d) The force acting on the particle is zero when
statements holds good? displacement is 0.5 m
(a) The mechanical energy of the particle remains constant 8. If different types of pendulums are taken to the moon, the
(b) The restoring force is max.imum at extreme position time period will not change for:
(0) The restoring force is'always directed towards a fixed (a) Simple pendulum (b) Spring"penilulum
point (c) Physical pendulum (d) Torsional pendulum
(d) The velocity is minimum at the equilibrium position 9. If two orthogonal SHM of same frequency having initial
4. JfampJitude of a particle executing SHM is doubled, which phase difference of (rt l2) acts simultaneously or. a particle
of the following quantities will be doubled? free to move, the particle can move in a:
(a) Time period (b) Maximum velocity (a) Straight line (b) Parabola
(c) Maximum acceleration (d) Total energy (c) Circle (d) Ellipse
5. For a body executing SHM with amplitude A, time period 10. Which of the following functions represent SHM?
T. max velocity v max and phase constant zero, which of the (a) sin 2rot (b) sin 2 rot
following statements are correct? (c) sinro/+2cosO)t (d) sinro/+co~ 2rot
(,) AI y=(AI2), v > (v~/2) [Hint: A motion will be SHM only ifacc. oc -y 1
(b) v = (v= / 2) fo, y>( AI2) II . The (x, y) co-ordinates of the comers of a 'square platc arc
(0) Fo,,=(TI8), y>(AI2) (0,0) (L,O). (L,L) and (O,L~ The edges of the plate are ·
(d) Fo, y=( AI2), «(TI8) clamped and transverse standing waves are set up in it. If
6. If a simple harmonic motion is given by, u(x, y) denotes the displacement of the plate at the point
(x, y)at some instant of time, the possible expression(s) for .....
y = (sin rot +cos rot)m
u isfare (a =positive constant): (liT 19981
which ofthe fo llowing statements are true?
(c) The gain in kinetic energy of the second particle is ~ th
(a) acos(xxIU)cos(nyIU)
(b) asio {ru:IL)sin (Ttyl£)
(c) asio (1U"/L)sin (2n:y/L) of the initial kinetic energy of the first particle
(d) acos(2ru:IL)sin(1tylL) (d) There is a net loss in the energy of the two particles in
the collision
12. Three simple hannonic motions in the same direction
having the same amplitude a and same period are 16. Function x=Asin 2 rot+Bcos 2 rot +Csinrotcosrot
superposed.lf each differs in phase from the next by 45°, represents SHM lilT 2006)
then: [lIT 1999) (a) for any value of A,B an~ C (except C = 0)
(a) The resultant amplitUde is (1+.J2)a (b) if A·=-B;C=2B, runplitude =IB~I
(b) The phase of the resultant motion relative to the first is (c) ifA=B;C=O
90"
(d) if A = B; C:: 2B. amplitude =IBI
(c) The energy associated with the resulting motion is
17. A linear hannonic oscillatot of force constant 2x 106N/m
(3+ 2../2) times the energy associated with any single
and amplitude O.OIm has total mechanical energy of 160 J.
motion Its:
(d) The resulting motion IS ,not simple hannonic (a) Maximum potential energy is 100J
13. A particle moves in a straight line. If v is the velocity at a (b) Maximum kinetic energy is 100J
distance x from a fixed point on the line and v 2 = a _ bx 2 ,
(c) . Maximum potential energy is 160J
where a and b are constants, then:
(d) Maximum potential energy is zero
(a) The motion continues along the positive x~direction
IS. A spring mass system hanging from the
only
ceilling of ap. elevator at rest. The k
(b) The'motion is simple hannonic elevator starts accelerating upwards with
(c) The particle oscillates ~ith.a frequency equal to -;: . acce.leration a. choose the correct
alternative (s).
m

(d) The total energy of the particle is rna (a) Amplitude of oscillation is !!!E.
. k Fig. 8.97
14. A particle slides back ~ /
and forth between two (b) Amplitude of oscillation·is mg
inclined frictionless ~ ~ . . k

21t~
planes as shown in Fig.
Fig. 8.96 ..ec) Time period of oscillation is
8.96. Which of the
following statements are correct with respect to the motion
of particle? (d) Time period ofo~cillation is 21tJ3f
(a) The ,motion :is oscillatory .. ,
(b) The motion.is simple harinonic :., 19. In the given figure, the block is attached
with a system of three ideal springs·A, B, C. A
(c) If II is'the initial height, period =~ fPI The block is displaced by a sinal!" distance x k
. smefg from its equilibrium position vertically

(d) If II is the initial height, period = f! sin e


downwards and released. T represents the
time period of small vertical oscillations of
the block. Then (Pulleys are ideal)
B

15. A particle of mass mmakes a head~on elastic collision with


a particle of mass 2 m inif.ially at rest. The velocity of the
first particle before and after collision are given to be u l and
(a) T=2.P: 2k m
(b) The defonnation of the spring · A is
vI' Then which of the 'following statements is true in C
. respect of t~is collision? (2/11) times the displacement of the
block Fig. 8.98
(a) For. all values of u l ,v! willa\ways be less than u l in
.
Itlagnitude .and
u,
IVI)=}
(c) The deformation of the spring C is (1111) times ·the
displacement of the block
(d) The deformation of the spring B is ·{4/11) times the
(b) The frictional loss in kinetic energy of the first particle displacement of the block
is S/9
20. The x-coordinate of a particle moving on x~ax:is is given by
x =·3sin·l00t + Scos 2·· 50t where x is in cm ~nd· t is . time in
SIMPLE H ARMONIC MOTION 425

seconds. Which of the following is/are correct about this 5. (A) , Water in a U-tube executes SHM, the time period
motion. for mercury fi lled up to the same height in the
(a) The motion of the particle is not S. H.M. U-tube be greater than that in case of water.
(b) The,amplitude afthe S.H.M. of the particle is Scm (R) , The amplitude of an oscillating pendulum goes on
increasing. rAlIMS 2007]
(c) The ampl~tude of the resultant S.H.M. is 53 em
6. (A) , In SHM it is possible · to doubled maximum
(d) The maximum displacement of the particle from the
acceleration while keeping maximum speed
origin is 9cm
constant.
(R) , It is possible when frequency is doubled while
amplitude is halved.
1. (b), (e) and (d); 2. (a), (b) and (d); 3. (a), (b) and (e); 4. (b) 7. (A) , A small body suspended by a light spring, performs
and (e); 5. all ; 6. (b) and (e); 7. (a) and (e); 8. (b) and (d); SHM. When the entire system is immersed in a
9. (e) and (d); 10. (a) and (e); 11. (b) and (e); 12. (a) and nonviscous liquid, the period of oscillation does not
(e); 13. (b) and (e); 14. (a) and (e); IS. (a), (b) and (e); 16. change
~OO~OO~OO~OOI~OO~OOIL~ ; .OO
(R) : The angular frequency of oscillation of the particle
docs not change.
and (d).
8. (A) : A particle, simultaneously subjected to two simple
Ie] Assertion-Reason Type Questions harmonic motions of same frequency and same
amplitude, will perform SHM only if the two SHMs
(a) If both A and R " e true and R the correct
explanation of A. " are in'the same direction.
(R) : A panicle, si multaneously subjected 10 the simple
(b) If ooth A and R "e true but R is no. correct harmonic motions of same frequency and same
explanation of A. amplitude, perpendicular to each other the particle
(c) If A is true but R is false. can be in unifonn circular motion.
(d) If A is false hut R is true. 9. A block is attached with a spring and placed on a
(e) Ifboth A and R are false. smooth horizontal surface. Spring is unstretched
1. (A): An object is perfonning SHM. along a straight line. initially. Block is displaced in left direction by ax
At any extreme position, velocity is zero while and released from rest (Assume elastic collision
acceleration is non-zero. witli wall)

(R): At the extreme position in SHM, the object reverses


its direction or motion. . ~-il I
2. (A): Consider that the bob of a simple pendulum ,S a ~
hollow spherical ball. It is filled with water and a Fig. 8.99
narrOw hole is made at the'bouom. Time period of
(A) : Time period of oscillation of block depends upon
as
this pendulum will change water comes out of'the
initial displacement (ax) of block along left
bob.
direction, 'for all value 'ofax.
(R): Change of mass results in change oftime period.
(R) : If initial displacement (ax) along left direction
3. (A): Consider !tn object in SHM. It is commonly
increase~, (ax >xo ~ time period of oscillation
observed that..amplitude of oscillations reduces with
decreases.
time Il;nd the object fina lly com.es to rest at the meal}
position. In such a motion, m~chanica l energy of the 10. (A) : Two pendulums. a physical one in the form of a
object will not be conserved. homogeneous rod and a simple one, of equal mass
and length are in oscillatory motion with the same
(R): Force acting on the object described above has a
' angular amplitudes. The oscillation energy of
non-conservative component.
simple pendulum is more than that of physical one.
4. (A): As a simple pendulum oscillates, its bob has a
(R) : Oscil~ation energy of a pendulum is inversely
non-zero acceleration at the mean position which is
proponio nal to the effecti,ve length.
directed towards the point of su·sp:ension.
ANSWERS
(R) : Speed of an object in SHM is maximum at the mean
position.
1. (b) 1. (e) 3. (a) 4. (b) 5. (0) •• (a) 7. (,)
8. (d) 9. (d) 10. (, )
426 PHYSICS FOR COMPETITIONS - Vol. I

[D) Integer Type Questions lEI Match the Column


I. In the Fig. 8. 100 shown, string is light and inextensible, I. Compare for the period of small oscillation of the systems
spring is light and ideal, the pulleys arc light and in column-I and in column-II. The mass of the rod is m, the
frictionless. Initially the spring is in its natural length and length of the rod is I, and the spring constant of each spring
the string just taut. Now the system is released from rcst. is k. The rod can oscillate in vertical plane about the hinge
The maximum extension in the spring during consequent O. The springs are ideal and horizontal. Initially the rod is
. . nmgF'd vertical and the springs are undefonned. Take length OP = I
motIOn IS - k-' In n.
and the length ON = 112 . .
Match the following :

Column - [ Column - II
(a) 0 (P) T= 21t
g
J3 + 3k
m I '}j m

p k !
,I
2. AB and CD arc two identical rods each of length land mass
mjoined to fonn a cross. The moment of inertia of these two (b) 0 (q) T"" 2n
~lg + 6k
rods about a bisector of the angle between the rods (xy) in
the sameplaneofthe rods is a ml2 then the valueof24 ais: m I '}j m
C

I k
P
k !
!' 'I
(0) 0 (r) 2.
I T~
D '2 19 lk
Fig. 8.101
N
k ! - +-
21 4m
3. A system of two identical rods (L-sbaped) of mass m and I
I
m
length I are resting on a peg P shown in the Fig. 8.1 ll2.lflhe '2
system is displaced in its plane by a small angle et then
period of oscillations is given by 21t W.Find the value of (d) 0 (s) T= 21t
'13K m 12 ~3g + 3k.
k'. I k k I '}j 2m
, N 1 ••
2
p

2. Match the Column-I with Column-II lilT-JEE 20071


Fig. 8.102
4. A block of mass 2 kg is moving with speed Vo towards a Column -I Column - II
massless unstretched spring (k = 10 N/m). It is found that
(a) The object moves on (p) The object executes a
for maximum speed Vo =.J6A mis, the block compresses harmonic
the x -axis under a simple
the spring, stops at that position and does not return. conservative force in motion.
Friction coefficient at surface is ~ = ~. Value ofx is: such a way that its
x "speed" and "position",
ANSWERS satisfy V=CI~C2 _ x 2 ,
where c l and '2 are
1. 4 2. 2 3. 2 4. 5
positive constants.
SIMPLE HARMONIC MOTION 427

2~
(b) The object moves on (q) The obj ect does nOl (b) (q)
the x -axis is such a way change its direction.
k
that its velocity and its
displacement from the
origin satisfy v "" - la, I~
where k is a positive m
constant. --",.", I
(c) The object is attached (r) The kinetic energy of
to onc end of a massless
spring of a given spring
the obj ect keeps on
decreasing.
(c)

k
(r)
2ff
constant. The other end

~
of the spring is attached
to the ceiling of an
elevator. Initially every-
thing ;s at rest. The
2~
(d) (s)
elevator starts going
upwards with a k 4k
constant acceleration a.
The motion of the
object is observed from
the elevator during the k
[;--~
period it maintains this
.,."
acceleration.

(d) The object is projected (s) The object can change


from the earth's surface its direction only once.
1. a-t p; b -t q; c-t r; d-t s
vertically u wards with
GM 2. a -t p; b-t q; c -)o p; d -)o q, r
a speed 2 _ _C ,where
R, 3. a-)op;b-)o q;c-)os;d -)o r
Me is the mass ,of the [F) Linked Comprehension Type
earth and R, the
radius of the earth. " Paragraph - 1
Neglect forces from Suppose that a straight tunnel with
objects other than the frictionless interior has been drilled
earth. through earth, as shown in Fig. 8.103. 0
is the deepest point of the tunnel which,
3. Each entry in column·I shows a system consisting of a
at a depth Yo from the surface~ofearth, is
massless pulley, spring of force constant k as shown in also the mid point of the tunnel. A ball of
figures and block of mass m If the block is slightly mass 'm' is released at one end of the Fig. 8.103
displaced vertically down from its equilibrium position and tunnel. As it moves in the tunnel, its
released. Then match the period of its vertical oscillation distance from 0 at any instant is x. Mass and radius of earth
given in column-II with column-I Match them are, respectively, M and R.
Column - I Column - II Answer these questions.
1. Magnitude of gravitational force acting on the ball when it
(a)
J'-'
(P)
2nR is at a distance x from 0, as shown in Fig. 8.103, is :
(a) GMm x (b) GMm
m
. R3 [(R- YO)2 +x2 ]

k (c) Gw;m [(R _ YO)2 +x 2 ]112


R
.Inm
2. Total force Ihat acts on the oall when it is at a distance x Take mass of each ball = IOOgm
fromOis : length of dumb-bell = 50 cm
(a) G~"' . (b) GMm x torsion constant C = 5x 105 gm cm 2/s2
R R' Answer these questions.
(c) GMm (d) GMmR' 7. Time period of oscillations is :
(R - YO)2 +x2 [(R- Y O)2 +x
2
f ,
(.) 2' (b) -"-,
.J2
3. Total force acting on the ball perpendicular to the tunnel is :
Ii'
( a) GMm (b) mg
(c) 1[S (d) ~,

GMm(R O-y) 8. The system starts at rest from an initial angular


(c) zero (d) , displacement 8 = 0.2rad. Total energy of the system is :
R
(.) 10-3 ) (b) 10-4)
4. As the ball is released at rest at any end of the tunnel, it
executes Simple Harmonic Motion oftirne period: (c) W-' ) (d) 4x 10-')
9. Maximum speed of either mass during the oscillations is :
(a) 2'JGM
R (b) 2~ GM
R' (a) 12 cmls (b) 8 cmls
(c) 14cml, (d) 10eml,

(e) 27tVR
(GM (d) 2~ GM
.
R' Paragraph·3
A disc of mass mconnected with
5. Maximum velocity of the ball along the length of tunnel an ideal spring of stiffness k is
will be : released from the stretched
(a) I"G;"~'-y-o-(2R---y-
) O GM
(b) - 4 yo(R- yo)
position of spring. Then the disc
rolls down the inclined plane.
VR . R
to. Amplitude of oscillation is:
GM ~
(c) -
R'
(R - yo) (d) VII' Yo (a) mgsin 8
k Flg.8.l0S
6. Total energy of the ball when it is atadistancexfromOwill (b) 2mg,in e
be : k
GMm (b) GMmyo (c) mgsin 9
(a) - , (R - yo)
. R R' '1k
(~) GMm(2R_yo)
2R2 . . (d) 0:;; Yo (2R - yo)
(~) none of these
1'1. time period of oscillation is :
Paragraph· 1 (.) 2, 0!
ff 12m
(b) 2-
. ''tJk
Two Identical point masses ~re
a
connected by ' thin massle~s
rigid rod. This system is called a . (c) , 2~ (d) none of these
dumb-bell. It is suspended at the
mid point of the connecting rod
by a wire as shown in Fig. (,0-------- __ __-.0:,\ 12. The friction al the meen position is:
8. 104. The dumb-bell remains
suspended in equilibrium with
\:0-------- --------01 (a) 2mgsin 9
I I
(b) mgsin e

its length in a horizontal plane. Fig. 8.104


(c) mgsin 9 (d) mg tan e
The dumb·beU is given a small angular displacement in the 2 I
horizontal' plane which results in the twisting of the wire.
Paragraph. 4
When released, the dumh·bell oscillates in a simple
harmonic mimner about the equilibrium position e = 0 A particle is executing SHM along x-axis. A stationary
under a restoring t~rque that develops due to the twisting of person at origin observes the motion of the particle. He
the wire'and given by't" ::-Ce, where C is called the 'torsion plots a curve between x-coordinate of the particle and time
constant' . These vibrations are called torsion vibrations and as shown in figure. Alt = Othe velocity of man with respect
the whole system a 'torsion pendulum'. to the particle is along negative x-axis.
x(cm)
ANSWERS
8
Paragraph-) ,. (b)
..
8.5 I. (e) 3. (e) •. (d) 5. (.) 6. (d)

51-\----,'---4 Paragraph-l 7. (e) (.) 9. (d)

Paragraph-3 10. (.) 11. (c) I'. (b)


2 f--¥'------i Paragraph-4 13. (a) 14. ec) 15. (b)

O'---i- -;-+ Paragrapb-S 16. (a) 17. (a) 18. (a)


2 t, t (sec)
Ag.8.106
IG) Subje~tive Type Questions
13. The time period of the periodic motion is :
l. (a) Can a motion be periodic but not oscillatOIY?
(a) 3.0 (b) 2.5
(b) Can a motion be oscillatory but not simple harmonic?
(e) 3.5 (d) 4.0 If your answer is yes, give an example and if not,
14. The va!ue of II is: explain why?
(a) 3.0 (b) 3.75 (Ans. Yes; Uni'form circular motion, (b) Yes; When a ball is
(e) 4.25 (d) 5.25 thrown from a height on a perfeclly clastic plane surface the
motion is OScillatory but not simple hannonie as the restoring
15. The displacement of the particle from mean position as a
force F '" mgconstt. and not F C( - xl
function of time is (displacement S is positive towards +ve
x-axis) 2 . . What is the (a) distance moved by, and (b) displacement of
a body ex.ecuting simple harmonic motion in a time equal to
(a) ~=3sine;I+5:) (b) S=3Sine;t+~) its period, if its amplitude is A?
(Ans. (a) 4..4 , (b) Zero]
(e) S=3sin( 231tt+~) 3. Can a body have acceleration without having velocity?
[Ans. Yes; In SHM at extreme position velocity is zero but acc. is
Paragraph - 5 maximum = «i A]
Assume that a straight tunnel is dug across the earth (radius 4. Determine whether or not the following quantities can be in
R) passing through its centre the same direction for a simple harmonic motion:
16. Find the time a particle takes to cover the length of tunnel if (a) displacement and velocity,
it is projected into the tunnel with a speed of ..fiR. (b) velocity and acceleration,
(a) -" (R (b) • ~ (c) displacement and acceleration.
2Vg Vg [Ans. (a) Yes; when the particle is moving from equilibrium

(e) 2·l (d) .f! position to extreme position, (b) Yes; when the particle is moving
from extreme to equilibrium position, (c) No; as in SHM
displacement is always opposite 10 acceleration.]
17. Find the time a particle takes to cover the length of tunnel if 5. How are each of the following properties of a simple
it is released from a height R above the tunnel. harmonic oscillation affected by doubling the amplitude:

(a)-"~ (b).~ period, maximum velocity, maximum acceleration and


2Vg Vg total energy?

(e) 2·l (d) .f! (AOL Period remains unchanged, max.. velocity and acc. are
doubled (as tllI"WI or a lllllll ox A) and energy becomes four times (as
E ox A2)]
18. Find the time a particle lakes to cover, the length oftuonel if 6. A particle is subjected to two simple harmonic motions in
it is thrown vertically upward along the length of tunnel the same direction having equal amplitude and equal
with a speed of JER. frequency. If the resultant amplitude is equal to the
amplitude of individual motions, what is the phase
(a) ~l (b)
·l difference between the motions?
[Hint: Arl)plilude is a vector and so .A 2 = A~ ~ Ai
+ 2Al A2 cos4ll
(d) .II [ADs.2n13]
7. (a) Can we ever construct a simple pendulum strictly [Ans. T =: 21f.JLI (g+ a), (b) T : 21CJ(L/ (g a), (e) Infinite,
according to its definition? (d) T : 21C.fi I (g2 + (if4 with equilibrium position inclined
(b) Is the motion of a simple pendulum linear SHM or to the vertical by an angle a= lan - 1(a l g) opposite 10
angular SHM? acceleration]
[Hint: See § 8.2]
15. A simple pendulum of length L is suspended from the
{Ans. (a) No, (b) Angular SHM]
ceiling of a cart which is sliding without friction on an
8. Show that the maximum tension in the string of a simple inclined plane of inclination e. What will be the time period
pendulum having amplitude em' is mg (1+ e~1 } of the pendulum?
[Hint: As the motion of a simple pendulum is circular so tcns ion [Hint: Resolving' g' along and perpendicular to the plane, the
will be maximum at iowest position and will be T = (m1l2 I L) + mg. problem becomes equivalent to that of a moving carriage having
However, as motion is SHM, velocity at lowest point Ii"' Vmur. horizontal ace. a'" gsin9 with g'= geoso. So the pendulum will
= AU) = (L8",)J(g I L) '" 8",Jii. Substituting Ihis value of v in incline to the vertical at an angle ';':tan-1(alg):tan- 1
Eqn. forT we get the desired result.] (g sin 91 geosO) '" e, i.e, will become perpendicular to the plane
and T : 2n.fL/i',i.e.,T '" 21tJ(L/ geosS»)
9. A girl is swinging in a sitting position. How will the period
of swing be affected if: [Ans, T : 21rJ(L I g cos 0)]
(a) The girl stands up while swinging? 16. A simple pendulum oflength Land mass mis suspended in a
(b) Another girl of same mass comes and sits next to her? car that is moving with constant speed v around a circle of
(Hint: See § 8.2 (3) and (2) (b)J radius R. What will be the frequency of oscillation and
[ADS. (a) decreases, (b) unchanged] equilibrium position of the pendulum?
10. A hollow metal sphere is filled with water and a small hole [Hint: This problem is equivalent to that of a simple pendulum in
is made at its bottom. It is hanging by a long thread and is a cart accelerating horizontally with acceleration a'" (li I R)
made to oscillate. How will the period of oscillation change towards the centre. So the pendulum will incline to the vertical at
if water is allowed to flow through the hole till the sphere is an angle 8", tan - 1(a I g) opposite to acceleration and will have
empty? frequency f '" (l/21t)~v:'IL), i.e., f '" (I/21t.fi){g2 + a2i'4)
[Hint: See § 8.2 (3) (b))
[Ans. f '" ..!~[g2 + (v 2 I R)2 j'4 and inclined to the vertical
[Ans. Time period will first increase, reaches a maximum and 2n .y L
then will decrease.]
at an angle e .. ,an- l v' away from the centre]
_
I I. A clock with an iron pendulum keeps correct time at 20°C. Rg
How much time will it lose or gain in a day if the temp.
changes to 40°C? (y of iron = 0.00OO36f'C) 17. (a) What will be the time period ofa simple pendulum if its
length is equal to radius of earth (= 6400 km)? (b) What is
[Hint: l::..T IT: (1/2)al::..ewithct '" y 13andT : I day " 864005)
the maximum lime period which an oscillating simple
(Am, The clock will lose 10.37 sec)
pendulum can have?
12. A simpl~ pendulum with a solid metal bob has a period To'
What wlil be the period of the same pendulum if it is made
[Ans.(a)T '" 21C~2 / 2g =Ihour, (b)T '" 2n-J(R I2g) =84.6min)
to oscillate in a non-viscous liquid of density one-tenth of 18. If acceleration due to gravity on moon is one-sixth that on
the metal of the bob? the earth, what will be the length and time period of a
[An._T = To~(I0/9)J second's pendulum there?
[Ans. (1/6) m and 2 sec)
13. A simple pendulum consists of a small sphere of mass m
suspended by a thread of length I.. The sphere carries a 19. A b lock of mass 0.2 kg which slides
k
positive charge q. The pendulum is placed in a unifonn wilhout friction on a 30° incline is
electric field of strength E directed vertically upwards. connected to the top of the incline
With what period will pendulum oscillate if the by a massless spring of unstretched
electrostatic force acting on the sphere is less than the length 23.75 cm and spring constant 30"
80 N,'m as shown in Fig. 8. 107. _ ._ _ _ _ __
gravitational force?
[Ans. T : 21C.jr.L","[g:':'("QE",
' T:n::;;l)]J (a) How far from the top of the Fig, 8,107
incline does the block stop? (b) If
14. How will the period of oscillation of a pendulum be altered the block is pulled slightly down the incline and released,
if its point of suspension is; (a) moving vertically up with
what is the period of the ensuing osci llations?
acceleration a, (b) moving vertically down with
fE=lOm/s2]
acceleration a«g) (c) falling free ly under gravity (d)
moving horizontally with acceleration a? (ADS. (a) 25 cm, (b) (1tI'IO) sec)
20. A particle which is attached to a spring oscillates [Hint: Sce solved Problcm 26.J
horizontally with simple hannonic moHon with a frequency (Ans. T '" 2n.j(2m I 3k ))
of (l I lt) Hz and total energy of 10 joule. If the maximum
speed of the particle is 0.4 metre per second, what is the 27. Two blocks (m= 1.0 kg and M - 9
k
force constant of the spring? What will be the maximum kg) and a spring of force constant
potential energy of the spring during this motion? (k '" 200 N/m) are arranged on a
horizontal frictionless surface as Fig. 8.112
(MNRI991(
shown in Fig. 8.112. The
[ADS. SOO N/m and 10 joule]
coefficient offriction between the two blocks is DAD. What
21. A massless spring of force constant 100 is the maximum possible amplitude of SHM if no slipping
N /m is cut into two halves. (a) What is the
is to occur between the blocks? (g = IOm l s 2)
force constant of each half? (b) The two
halves suspended separately support a [Hint: For no slipping F < I L , i.e., a < y.g or 002A < I1g with
block of mass M. If the system vibrates at a oo2 .. k l(m+M)]
frequency of (to/x) Hz, find the value of Fig. 8.108
massM. (Ans. 20 cm]
(Ans. 200 Nlm (b) I kg] 28. A uniform rod of length L and mass
M is pivoted at its centre. It is held in k
22. A particle of mass 0.1 kg is held between two rigid supports
position by a system of springs as
by two springs offorce constant 8 N/m and 2 N/m. If the
particle is displaced along the direction of length of the
shown in Fig. 8.113. Show that when k •
springs, calculate its frequency of vibration. turned through a small angle e and
released the rod undergoes SHM
[ADS. 511t Hz] Fig. 8.113
23. Suppose that the two with a frequency: f =.l. {6k
springs in Fig. 8.109 have k, 21tfAi
different force constants kJ What will be the maximum speed of the tip of the rod?
and k 2 . Show that the [Hint: See solved Problem 27.]
frequency 1 of oscillation Fig. 8.109 (Ans. LO rrJ(3k 12m)]
of the block is then given by
29. Fill in the blanks:
f = JrJj2i-+-f2"OC
2
(a) A particle is in SHM of amplitude 2 em. At extreme
where fi and /1 are the frequencies at which the block position the force is 4 N. At the point midway between
would oscillate if connected only to spring- l or spring-2. the mean and extreme position the force is .....
24. Two springs are joined and (EAMCET (990)
connected to a mass m as (b) A body executes SHM with a period of (I In) sec and
shown in Fig. 8. 110. The an amplitude of 0.025 m. The maximum value of
surfaces are frictionless. If
the springs separately have Fig. 8.110 acceleration is ...... mIs 2 .
force constants kJ and k2' show that the frequency of (c) An object of mass 0.2 kg executes SHM along the x-
oscillation of m is: axis with frequencyof(25 I n) Hz. At the point x = 0.04
m the object has KE 0.5 J and PE 004 J. The amplitude
of oscillation is ...... m. lilT 1994}
(d) A person measures the time period of a simple
25. A bullet of mass m strikes a

--
pendulum inside a stationary lift and finds it to be T. If
m , k
block of mass M (as shown the lift stans accelerating upwards with an acceleration
in Fig. 8.111). The bullet of (g I 3~ the time period of the pendulum will be ......
remains embedded in the (EAMCET (993)
Fig. 8.111
block. Detennine (a) the
(e) A mass M attached to a spring oscillates with a period
velocity of the block immediately after collision and (b)
of 2 sec. If the mass is increased by 2 kg, the period
the amplitude and frequency of the resulting simple
increases by 1 sec. Assuming Hooke's law is obeyed,
hannonic motion.
the initial mass M was ...... kg.
[Ans. (a)mv / (m + M).(b)mv l .jk(m+ M)
(An •. (.) 2N (b) 0.4 (0) 0.06 (d) T (,/3)/2 (.) 1.6)
I"" (1 / 2ft).)k f (m + M)] 30. A particle of mass mis attached to the mid-point of a wire of
26. A particle of mass m is attached to three identical springs length Land stretched between two fixed points. TfThl" the:
each offorce constant k and making equal angles with each tension in the wire, find the frequency oflatC!'fll oscillations.
other. If the particle is pushed slightly against one of the [Aos. 1= (I I 21t).j(4T I mL)]
springs and released, find the time period of osc~l1ation.
31. A point rp,ass mis suspended at the end ofa massless wireof [Ans. T:4 ~Sin-I[_R_llf2 +4 @.;Notsimplehannonic
length L and cross-section A. If Y is the Young's modulus ~g R+2h fc-
for the wire, obtain the frequ ency of oscillations for the but oscillatory]
SHM along the vertical line. (Roorkee 20001
39. Consider the classical model of an atom such that a nucleus
[An,. f = ( II 2n).j(YA I mLl] of charge +e is unifonnly distributed within a sphere of
32. An ideal gas is enclosed in a veltical cylindrical container and radius 2 A. An electron of charge (-e) at a radial distance 1
supports a freely moving piston of mass M. The piston and the A moves inside this sphere. Find the force attracting the
cylinder have equal cross-sectionaJ area A. A1mospheric electron to the centre of the sphere. Calculate the frequency
pressure is Po and when the piston is in equilibrium the with w hich the electron would oscillate about the centre of
volume of the gas is VO' The piston is now displaced slightly the sphere. IRoorkee 1995)
from its equilibriwn position. Assuming the system is (Hint: See note (iii) in § 8.4 (C) (b).]
completely isolated from its surroundings, show that the =
(Ans. F II: 2.88 x 10-9 Nand f 0.9 x101S Hz]
pislon executes SHM and find the frequency of oscHiation.
40. Two equal charges q are kept at -a and a.along x -axis. A

[Ans. f = ~[Y(Po + ~g) v;: J'2] particle of mass mand charge (q / 2)is brought to the origin
and given a small displacement along (a) x-axis (b) along
y-axis. Describe the motion in the two cases.
33. t 0-4 kg oxygen is contained in non-conducting cylindrical
[Roorkee 19921
vessel of cross-sectional area 29.15x 10- 5 m 2 and volume
[Hint: .See § 8.4 (D) (a).]
10- 3 m 3 . It is closed by a frictionless non-condu.;:ting
piston which is free to move in a vertical direction. If the [Ans. (a) SHM with time period (2n I qN21ttoma3. (b) accelerated
piston is slightly depressed and then released, show that the translatory)
piston will execute simple harmonic motion. Find its 4 1. State whether the following statement is true or false giving
frequency. Ignore atmospheric pressure and take C p I C" reason in brief: "A ring of radius R carries a uniformly
>= 1.4 for oxygen. [Roorkee 1991) distributed charge +Q. A point charge -q is placed on the
[Ans. f". (l/21t)~CrgA I V) (lIn) Hz] axis of the ring at a distance 2R from the centre of the ring
and released from rest. The particle executes a SHM along
34. If 0.3 kg liquid of density 13.6x 10 3 kglm 3 is filled in a tube the axis of the ring."
of unifoml cross-section 1.7 x 10- 3 m 2. One limb of the [Ans. False; As here A =2R > R, the restoring force will not
tube is vertical w hile the other is inc lined to it at 60". If the be linear and so the motion is oscillatory but not SHM]
liquid in the vertical tube is slightly depressed and released, 5
42. A thin ring of radius 1 m has a positive charge' 1.0 x 10-
find the frequency of oscillation.
coulomb uniformly distributed over it. A particle of mass
[Ans. (17ht)Hz]
0.9 g and having a negative charge ofl :Ox 10- 6 coulomb is
35. A cylinder is weighed at one end so that it floats vertically placed on the axis at a distance of 1em from the centre of the
'in water with" em of its length immersed in equilibrium. If ring. Show that the motion of the negatively charged
the cylinder is pushed down and released, show that it will particle is approximately SHM. Calculate the time period of
execute SHM. Find the time period of oscillation. How will oscillation.
the frequency be affected if the cylinder is immersed in a
liquid of lower density? [Ans. T = 2tt-J4neo mR 3 I qQ = (reI 5) sec]
[Ans. T = 21t~(hl g), frequency will decrease] 43. A long horizontal wire AB, which is free to move in a
vertical plane and carries a steady current of 20 A, is in
36. A cubical body (side 0.1 m and mass 0.002 kg) floats in
equi librium at a height of 0.01 mover another parallel long
water. It is pressed and then re leased so that it oscillates
wire CD, which is fixed in a horizontal plane and carries a
venically. Find the time period.
steady current of 30 A as shown in Fig. 8.46. Show that
[Ans. T '" 2n,fCI1I ' Aag) == 0.028 sec] when AB is slightly depressed and released it executes
37. A small block is placed a little away from the centre of a simple harmonic motion. Find the p'e riod of oscillations.
concave surface of radius 2.5 m. When the block is lIlT 1994]
released, it begins to oscillate about the centre. What is the . (Ans. r '" 21t~(h l g) =00.2 seC] ,
.
period of oscillation if g = 10m /5 2 ?

~
. 44. A point particle ofmass M is attached +qM

[Ans. T = 2n.JR ' g "" (n}sec] to one end of a massless rigid


non-conducting 'rod of length L
• . E
38. A mass m is dropped in a tunnel along the diameter of earth
Another point particle', of the same -q M
from a height h (<< R) above the surface. Find the time Fig. 8.114
period of motion. Is the motion simple hannonic? mass is attached to the other end of the
. rod. The two particles carry charges +q and - q respectively.
(Hint: See note (ii) in § 8.4 (C) (b).]
This arrangement is held in a region ofa uniform electric 49. A pulley in the fonn of a circular disc of . . . . ...."'"
field E such that the rod makes a small angle 9 (say of mass m and radius r has the groove cut all
about 5") with the field direction. Find an expression for along its perimeter. A string whose one end
the minimum time needed for the rod to become parallel is attached to the ceiling passes over this
to the field after it is set free. disc pulley and its other end is attached to a
[Hint: Prove § 8.4 (D) (e), i. e., T = 2Tt~(l1 pE); Here p = qL spring of spring constant k. The other end
ofthe spring is attached to ceiling as shown
and I = 2 x M (Ll2i = ML2 / 2J '-
in Fig. 8.116. Find the time period of
Flg.a.116
[Ans.l ::: T J 4::: (rt I 2.fi),j"(M'"'Lc-,'q"El] vertical oscillations of the centre of mass
assuming that the string does not slip over the pulley.
45. A vibration magnetometer consists of two identical bar
magnets placed one over the other such that they are
mutually perpendicular and bisect each other. The time
[Ans. T =1tffffl
period of oscillation in a horirontal magnetic field is 4 sec. 50. A simple pendulum of length I has a period T seconds at a
If one of the magnets is taken away, find the period of place where g has a value go' If it is taken to a place where
oscillations aftile other in the same field. [Roorkee 19921 the temperature increases its length to (l + a) and g has the
[Hint: T::: 21t~(l / MH); initially 10 ~ 1 + 1 = 2l and value (go -b) (a) By what percentage the period is
increased? (b) In order to restore the original period, by
Mo ~ ~M2 + M2 = -tiM (as magnetic moment is a vector), so
what amount the length must be decreased?
that To = 21t~(-ti)1 I MH;
[Ans. (a) so (ago +bi) o/c.; (b) (a+.!!!.-l J
so (T ITo) ::: (1I21f4)] igo go
[Ans. T ::: (41 2114):= 3.36 sec) 51. Two small masses m! and m2 are attached to
46. A particle P moves on a circle with centre 0 and radius 2 m. the ends of a rod of negligible mass as
At time t second the radius vector OP has rotated from a shown in Fig. 8.117. The rod is pivoted so
e
fixed position OA through an angle radian where = 3 sin e that the masses and rod fonn a physical
2/. Find the velocity in the sense of increasing the speed e, pendulum. Find an expression for the period
and the magnitude of acceleration when 1 = ttl 3. of oscillation. Express your answer in tenns
of the parameters given in figure.
[Ans. velocity = -6 mls; specd '" 6 m/s; acceleration = 6J21 mls 2] Flg.8.117
mP+mf2
47: A block of mass M rests [Ans.2n I] 22]
on a frictionless (m2gi2 m\gi1)
horizontal table and is 52. Find the angular frequency of oscillation of motion of block
connected to two fixed m for small angular motion of rod BD. Consider the rod to
posts by springs (see be massless.
Fig. 8.115) having Fig. 8.115
spring constants k\ and o m
k2 rcspectively. (a) If the block is displaced slightly from
its cquilibrium position, what is the frequency of vibration? b
Suppose that the block is vibrating with amplitude A and
that at the instant that it is passing through its ,
equilibrium position, a mass m is dropped vertically on B
to the block and sticks to it. Find: Fig. 8.118
(b) the new frequency of vibration,
(c) the new amplitude of vibration. [Ans. ro = .I __"::'k,,,k,,,,?_--.,, J
m{k/+k2 (b+c)2}
[Ans.(J)=~kl +k2 ;o)' ::: k\ +k2;A'::: (~ M lAJ
M M+m M+m 53. A bob of mass M is attached to the lower end of a vertical
string of length L and cross-sectional area A. The Young's
48. A particle P of mass 2 kg is subjected to an attraction modulus of the material of the string is Y. If the bob
towards 0 proportional to OP. When P is at rest at 0 it
executes SHM in the vertical direction, find the frequency
suddcnly acquires kinetic energy 225 J and thereafter
of these'oscillations. IRoorkee20001
executes SHM with an amplitude of 5 metre. Find (a) the
period of motion, (b) the force of attraction whenOP:= 3m, .[Hin" § 8.4 (Al (bl]
(c) the rate of working of attraction after 1t1 12 second.
[Ans. (a) 2.09 second; (b) 54 newton; (c) 675 watt]
[Ans. H£"
f"' - 1 -J
2n ML
Gravitation
§ 9.1 Newton's Law of Gravitation (3) It holds good over a wide range of distances. It is
The inunortal name of Newton and Gravitation have found true from interplanetary distances to
become synonymous. Almost every one has heard the story of interatomic distances.
how young Isaac Newton, while resting under an apple tree (4) It is an action-reaction pair, i.e., the force with which
one day. was struck on the head by a falling apple. This simple one body (say earth) attracts the second body (say
incident stirred Newton to think about falling bodies and moon) is equal to the force with which moon attracts
eventually led him to the discovery of the law of gravitation. the earth. (However, accelerations will not be equal)
Studies of gravitation are today more important than ever (5) It is a two-body interaction, ** i.e, gravitational force
before. The secrets of planetary and satellite motion have now between two particles is independent of the presence
been revealed and it has paved the way for the conquest of or absence of other particles; so the principle of
space by man. Annstrong would not have earned the honour of superposition is valid, i.e., force on a particle due to
being the first man to set foot on the moon, had Newton not number of particles is the resultant of forces. due to
given us the law of gravitation. individual particles, i. e. ,
According to Newton's law ofgravitation every particle in
the universe attracts every other particle with a force directly
.........
F =F +F + ....
I 2
proportional to the product of their masses and inversely [See solved Problem 3]
proportional to the square of the distan ce between them. The
direction of this force is along the line joining the particles. (6) It is a conservative force, i.e.• work done by it is path
independent or work done in moving a particle round
Thus, the magnitude of the gravitational force F that two a closed path under the action of gravitational force is
particles 1111 and 1112 separated by a distance r exert on each zero.
other is
(7) It expresses the force between point masses. In
m
F =G ,m 2 applying it to the case of extended bodies of finite size
care should be taken in assuming that the whole mass
"
Here G, called gravitational constant, is a universal
of a body can be regarded as concentrated at its centre
of mass. [See solved Problems 4 and 5.]
constant having the value. 6.67 x 10- 11 N-m 2/kg 2 and
dimensions [M - 1L3T-2 J It defines the strength of the Note: It is worthy to note here thai Newton had his gravitational theory
ready to publish except for proving that the earth behaves as ifa!]
gravitational force between two particles, e.g., if by some
ils mass were concentrated at ils centre. It took him 20 years to
miracle G becomes one-tenth, the gravitational force will develop the calculus to the point where we would prove this fact
become (Ill 0) and we will be able to jump ten times higher by integration and only then did he publish his new famous
than the present value, i. e., will clear a high building by theory.
jumping!
Problem 1.A massMissplitintotwopartsmand(M -m),
Regarding gravitational force it is worth noting that:
which are then separated by a certain distance. What ratio
(1) It is always attract!:ve (while electric and magnetic
(mlM) maximises the gravitational!orce between the parts?
force can be attractive or repulsive).
Solution: If r is the distance between m and (M - m). the
(2) It is independent ofthe medium between the particles
(while electric and magnetic forces depend on the gravitational force will be
nature of the medium between the particles) .

.. The value of 'G' in the laboratory was first delennined by Cavendish using his 'torsion balance' .
.... Nuclear force is many-body interaction, so the principle of superposition is not valid in case of nuclear force.
F =Gm(M -m) =£(mM -m') Solution: (a) Considering an element of rod oflength dl
r2 r2 as shown in Fig. 9.2 and treating it as a point of mass (MIL)dl
For F to be maximumdFldm =Oa8 M and rare constants, situated at a distance R from P, the gravitational force due to
this element on the particle will be
i,e., ~[£(mM
dm r2
-m')]=o dF = Gm(MIL;(Rd6) aiongOP [as dl = R de]
R
i.e., M -2m=O [asGlr2 :;z!:OJ
or(ml M) =(112), i.e., thejorce.will be maximum when the two
parts are equal.
Problem 2. Two particles of equal masses move in a
circle a/radius r under the action oftheir mutual gravitational
attraction. Find the speed of each particle if its mass is m. --_x
Solution: The particles will always remain diametrically o
Fig. 9.2
opposite so that the force on each particle will be directed
along the radius. Considering the circulation of one particle, So, the component of this force along x and y-axes will be
we have dFH -dF e_GmMcos9d9
- cos-
LR
i,e., V= Wm
V4r" dFv --dF· e_GmMsin9d9
sm-
LR
Problem 3. Three equal particles each of mass mare
placed at the three corners ofan equilateral triangle ofside a. So that F =GmMf1tcos9d9=GmM[sin9t=0
xLRO LR 0
Find the force exerted by this system on another particle of
mass m placed at (a) the mid-point ofa side, (b) at the centre of and F = GmMf1tsin9d9=GmM [-cos9t
the triangle. YLRO LR 0

Solution: As gravitational force is a two body interaction, 2rtGmM


=
the principle of superposition is valid, Le., resultant force on L'
particle of mass m at P,
~ ~ ~
F = FA+F8+ Fe
-+ So,
"x
F= IF2 +F2 = F

2rtGmM
Y Y

(a) As shown in Fig. 9.1 = [asFx is zero]


L'
when P is at the mid-point
of a side, FA andF8 will i. e., the resultant force is along the y-axis and has magnitude
be equal in magnitude but (2nGmMIL')
opposite in direction so A."'__---!,-_-'"B (b) If the rod was bent into a complete circle,
will cancel each other. So m P m
Fig. 9.1 F = GmMf21t cos9d9 = O
point mass m at P will x LR 0
experience a force due to Conly, i. e.,
G 2 4G 2 and also F =GmMf21tsin8d8=O
F=Fc =G~- m =- -m alongPC y LR 0
(CP)2 (asin60 0 )2 3a 2 i.e., the resultant force on m at a due to the ring is zero.
(b) When P is at the centre of the triangle,O, the forces of Problem 5. A point mass m is at a distance x from Ihe
-> -> -> centre ofa ring ofmass M and radius R on its axis as shown in
three particles FA,Fn and Fe will be equal in
Fig. 9.3. Find the gravitational force between the two. What
magnitude and will subtend equal angles with each will thisforce be ifx» R andx« R? For what value ofx is the
other (120°), so that the resultant force on m at 0, force maximum?
~ ~ -+ ~
F =FA+Fn+Fc =0. Solution: Considering a small element of the ring (as
shown in Fig. 9.3) and treating it as a point mass, by Newton's
Problem 4. A thin rod ofmass M and length L is bent in a law of gravitation, force on the particle of mass m due to this
semicircle as shown in Fig. 9.2.(a) What is its gravitational element,
force (both magnitude and direction) on a particle with mass
mat 0, the centre ofcurvature? (b) What would be theforce on dF = G:~M = Gm(M;;nR)dl [as dM = 2~ dl]
m ifthe rod is, in the form ofa complete circle?
Now due to symmetry of the problem the compo n ~rit of subtract from this the force which the mass of the hollowed
forces perpendicular to the axis due to all elements will cancel sphere would have exerted on m. i.e.,
each other, i.e., F=GmM _GmM'
Fy=fdFy =O d
2
i
while components along kaxis will add up so that But from figure y= [d - (RI2»)
F =pdF cos9 along PO dM and as M =(413)1tR ' p, M' =(413)n(R I2)' p =(M 18)
i, e., F =GmM f dl2 (~) A dF so F =GMm _ Gm(MI8) =GMm [1 I ]
21tR r r e P
x d' [d - (RI2 )]' d' 8[1 -(Rl2d)]'
o
[ascose=~] § 9,2 Gravitational Field
In order to explain 'action at a distance' (1". e., force without
Now as for a 2given2point
12
and FI g.9.3 contact) it is assumed that a mass or mass distribution produces
ringxandr[= (R +x )1 ]will a field in space surrounding it. So the sphere of influence
be the same for all elements, surrounding a mass in which its gravitational effects are
F GmM::c I.dl GmMx [as1dl=2nR] perceptible is called the gravitationaljield of the given mass.
2nR(R2 +x2)3/2 1 (R2 +x2)3!2 r Gravitational field at a point is characterised either by a vector
function of position / , called 'gravitational intensity' or by a
Now ifx » R, F '=(GmMlx 2 ). then for a distant point ring scalar function of position V called gravitational potential.
behaves as a point mass (which is expected) and for x«R. (A) Gravitational Intensity 1*
F:::: (GmMIR 3 >X, i,e., force varies linearly with distance x. In a gravitational fie ld, at a m (Test mass)

Furthennore F will be max imum when (dFl dx)=O,i.e., point, the strength of the field /Field
called attraclion or simply field Mr . Point
A.[ GmMx
dx (R2 +x2)3/2
]=0 or ..!.[xCR2 +x 2 )-3/2]=O
dx
is defined as force experienced
per unil mass acting on a test Source'
mass supposed to be placed at Point
[as GmM :;to]
that point. Here it is taken for Fig. 9.5
i.e., [(R 2 +x 2 )- 312 _3x 2 (R 2 +x2 )-S/2 ]=O granted that test mass does not affect the mass or mass·
or x=±RIJi [as(R2+x2)-SI2:t:O] distribution producing the field. So if a test mass m at a point in
~

[Substituting this value ofx in the expression for force F, a grav itational field experiences a force F,
F =, = (2GMm/3.fiR')). ~ ~
1= Flm .... (i)
Note: From this problem it is clear that the mass of the ring cannot be Regarding gravitational intensity it is worth noting that:
supposed to be concentrated at its centre unless x» R.
(1) It is a vector having dimensions[LT- 2 }and unit Nlkg.
Problem 6. A spherical hollow is made in a lead sphere ~ ~ ~ ~
ofradius R such that its surface touches the outside surface of (2) As by definition J = ( Flm), i.e., F = ml so force on a
the lead sphere and passes through its centre. The mass a/the point mass m can be calculated from the fi eld, by
lead splJere before hollowing was M. What is the force of multiplying it by the.mass.
al/mction Ihalthis sphere would exert on a particle ofmass m
(3) If the field is produced by a point mass M and the test
which lies at a distancedfrom Ihe centre ofthe lead sphere all
mass is at a distancer from it then as by Newton's law
lhe straight line joining the centres of the sphere and the
of gravitation,
hollow (as shown in Fig. 9.4)?
Solution: As the point mass F=GMm
m is outside the lead sphere we can ,.'
assume its mass to be concentrated l = F = GM
at the centre. To calculate the m ,.'
force of attraction on the point
mass m we should calculate the (4) For an external point (r> R) a sphere (solid or hollow)
force due to the solid sphere and Fig. 9.4 behaves as whole of its mass is concentrated at its
centre, i.e.,
J
• In case of energy, intensity means energy per unit area per unit time, i.t.,1 '" EISt and is a scalar with unit W/m2 and dimensions [Mr- ].
[8] Gravitational Potential V
IOU! =-,-
GM
r
[forr>R] .... (ii)
At a point in a gravitational field potential V is defined as
negative of the work** done per unit mass in shifting a rest
mass from some reference point (usually at infinity) to the
given point, i.e.,
V = (-Wlm) .... {v)
o Regarding potential it is worth noting that:
(1) It is a scalar having dimensions [L2T -2 land unit Jlkg.
(a) (b) (e)
Flg.9.S (2) As by definition potential energy U =-w
U
It is independent of the radius of the sphere and varies So, V = m' i.e., U=mV .... {vi)
non-linearly as the inverse square of distance [ocl/r2 l.
i.e., physically potential at a point represents
(5) In case ofa spherical shell for an internal point, i.e., potential energy of a unit point mass at that point.
(r< R) field is zero, i.e.,
--> -->
lin = Ieentre =0
This all is shown in Fig. 9.8 (a)*.
.... (iii) (3) As by definition of work W = F· d r
f
(6) In case of a spherical volume So,
distribution of mass (i.e., a solid
sphere) for an internal point (r< R),
the po~ion of the sphere that lies i.e., dV=-Idr or I= - (dV/dr) .... (vii)
outside the radius r will not contribute So potential can also be defined as a scalar function
Fig. 9.7
to the field [as the field inside a of position whose negative gradient, i.e., space
spherical shell is zero]; so derivative gives intensity. From this it is also evident
I in = GAf' with M' = mass of sphere of radius r. that (a) if I is given, V can be calculated and vice
r versa; (b) negative of the slope afVlr curve gives I.
Now if M is the mass of solid sphere, (4) If the field is produced by a point mass, then as
dV = -J dr and 1= -(GMlr2) [-ve sign is used as I is
p = M= M andso M'=j, TtrJ p = .!!£.rJ
V (413)nR' 3 R' attractive]

.... {iv) V=-f[<~ldr=-G~ +C

Now here we assume the reference point to be at OC)


i.e., intensity inside a solid sphere varies linearly with
and potential to be zero there, so that C = 0,
distance from the centre. So it is minimum (Ie = 0) at
the centre and maximum at the surface (Is = GMIR 2 ). V =_GM . ••. ( VIII
"')
r
This all is shown graphically in Fig. 9.8 (b).
(5) In case of a sphere, solid or hollow, for an external
point, the sphere behaves as whole of its mass is
GM Spher)ca)-shell eM Solid·sphere
Ai -------- concentrated at the centre, i.e.,

" Vout =--r-


GM
Iforr>R] .... (ix)

i. e., V is independent of the radius of the sphere R


and varies inversely as the distance from the centre
[oc{llr)],
(a) (b)
Flg.9.S
(6) In case ofa spherical shell for an internal point (r< R)
the potential everywhere is same and equal to its value
at the surface, i. e.,
• In case of spherical shell there is discontinuity at the surface, so for internal behaviour 1$ = 0 while for external behaviour 1$ '" (GMIR 2 ).
U Negative work means force is opposite to displacement.
. Vin =- G:: =constant [forr< R] .... (x)
(ii) This problem also indicates that an astronaut in going from
earth to moon will face weightlessness when he is at a
distance of 3.42 x 105 !un from the earth. between the
[so that! = -(dVldr) = 0; this is in agreement with the earth-moon system and the engines of his spaceship are
fact that intensity inside a spherical shell is zero] inactive.
(7) In case of a solid sphere for an internal point (r< R)
potential varies non-linearly with r as Problem 9. In a certain region of space gravitational
field is given by I = -(Klr~ Taki:lg the reference point to be at
v. = - GM [3R 2 _ r'] [for r< R] ....(xi) r = ro with V "" Vo ,find the potential.
In 2R 3
Solution: We know tbat intensity is negative gradient of
From this it is clear that IVI will be maximum at the potential, i.e., 1= -(dVldr) and as here I = -(Klr), so
centre (r = O), i.e., Vcentre = (3/2) (GMIR) while at surface dV K
(r = R) it will be (GMIR), i,e.,Vcentre = (3/2) Vsurface' -= - i. e.,
dr r
Problem 7. What are the values of gravitational
attraction and potential at the suiface ofearth referred to zero or V-Vo=KlogL so V = Klog..L.+VO
potential at infinite distance? Given that the mass ofthe earth ro ro
is 6 x 10 24 kg, the radius of earth is 6400 km and
Note: In this problem we cannot assume potential to be zero at r = 0 or
G =6.67 x to - II MKSunits. r = <Xl.
Solution: According to the theory of gravitation, for an
external point a spherical mass distribution behaves as if the Problem 10. Two concentric
whole of its mass were concentrated at the centre; so shells of masses M I and M 2 are
-11 24 situated as shown in Fig. 9.9. Find
/=GM = (6.67x10 )x(6xlO )=9.8Nlkg the force on a particle of mass m
R' (6.4 x 10 6 )' when the particle is located at
(a)r=a(b)r=band(c)r=c. The
and
V=_GM =_ (6.67 x lO- ll )x(6xlO 24 ) distance r is measured from the Flg. 9.9
R (6.4 x 10 6 ) centre of the shell.
= -6.25 x 10 7 Jlkg Solution: We know that attraction at an external point
due to a spherical shell of mass M is (GMlr2) while at an
Note: This problem shows that I kg mass at the surface of earth will
internal point is zero. So:
experience a force of 9.8 Nand 6.25 x 107 J of work will be
required to shift it from the surface of earth to infinity. (a) For r = a, the point is external to both the shells; so
G-,,(M.:Jlc.,+,..:M.:.g,)
Problem 8. The distance between earth and moon is 1 A. = .:.
about 3.8 x 105 km. At what point or points wilt the a'
gravitational field strength of earth-moon system be zero? So FA =mlA =Gm(M r +M2 )la 2
Given mass of earth is 81 times the moon's mass.
(b) For r "" b, the point is external to the shell of mass M 2
Solution: In this problem, earth and moon will be treated
and internal to the shell of mass M I ; so
as point masses as they are far apart and as in case of a point
GM, ,
mass I = GMIr2, the field due to both the masses will be zero I B =--+O, i.e., FB =mIB = (GMmlb )
at I' = 00. In addition to it, the field will also be zero at a point in b'
between earth and moon where the pull of earth balances the (c) For r =C, the point is internal to both the shells; so
pull of moon. If this point is at a distance x from the earth, Ie =0+0=0, i.e., Fe "" mie = 0
GM E GM M 81 I Problem 11. The magnitudes of gravitational field at
-~= (3.8x108 _x)2 or x2 = (3.8x108 _ x)2 distances rr and r2ff'Om the centre ofa uniform sphere ofradius
R and mass M are II and 12 respectively. Find the ratio oj
[as ME =81M M ]
(/1112) if(a) rl > Rand r2 > R and (b) r1 < Rand r2 < R.
or 9(3.8 x 10 8 -x)=x
Solution: In case ofspherical volume distribution of mass,
or x ",, 3.42x10 8 m=3.42xlO s Ian
I=GM forr>R
Note: (i) The field due to earth or moon at this point will be r'
1I 24
I ", GME '" (6.67 xlO- )(6 x 10 ) = 3.42 x 10- 3 Nlkg and I""GM rforr<R
x2 (3.42xI08)2 R'
439
(a) As both 1'1 and r2 are greater than R,
I, (GMlr,2) r22
- = =-
(c) For a circle i + z2 =36, the radius of the circle =6
12 (GMlrz2 ) r,2 and so the point is outside the sphere and as for
external point V = -(GMlr)

I,
,
(b) As both r1 and 1'2 are lesser than R,
(GMIR )1'1 r,
VA + VB +VR =-(GMI6)
Now as for spheres A and B the distance of the point
1;= (GMIR 3 )r = 1'2
2 on the circle r = ~62 +22 =.J40 and the point is
outside the spheres A and B, so
Note: If'i >Randrz <R,
VA =VB =-G(MI64) / .J40
II (GM/'I2) R)
12 = (GMIR 3)rz = -"'-"
VR =_G~ -z[- 6~~1
Problem 12. A solid sphere ofuniform density and radius
4 units is located with its centre at the origin 0 ofco-ordinates.
Two spheres of equal radii 1 unit with their centres at A
=- G~ G.- 32~] = constant
(- 2,0,0) and B (2,0,0) respectively are taken out of the solid i.e.,potential at all points ofthe circle i + z2 = 36 is
leaving behind spherical cavities as shown in Fig. 9.10. Then
same, so state1J1ent (c) is true.
which of the fol/owing statements are true?
(a) The gravitationalforce due (d) For a circle i + z2 = 4, the radius of the circle = 2
to this object at the origin is zero. and so the point is inside the sphere at a distance
(b) The gravitational force at (4 -2) = 2 units from its centre so
the point B (2,0,0) is zero. V=- GM (3R2 _,2)
(e) The gravitational potential 2R'
is the same at all points on the cir.cle = ~ GM (48_4)=_22GM
y2 + z2 =36. 2x64 64
(d) The gravitational potential FIg. 9.10 So VA +VB +VR =-(22GMI64)
is the same at all points on the circle i + z2 = 4. But as for spheres A andB, the distanceofthe point on
Solution: Considering the sphere to be made of spheres A the circle r = ~22 + 22 = 2.J2 and the point is outside
andB of radius I unit and the remainder R, M =(4/3)n4 3 p and the sphere A or H, so VA = VB = -(GM 164)12.J2.
M A = M B ' = (413)<1' p = (MI64)
(a) As the force at the centre of a sphere is zero
V = { 22GM -2
R 64
GM
64x2-12
1
=-~~[22- ~]=constant
-+-+-+ --J.-+
FA+FB+FR = 0 and as FA and FB are equal and
~ ~ ~

opposite for point 0, i,e., FA + FB = 0, so FR = 0, i.e.,


i.e., potential at all points of the circle i + z2 = 4 is
statement (a) is true.
same; so statement (d) is true.
(b) As the force at a distance r«
R) from the centre of a
§ 9.3 Gravity
solid sphere, F = (GMmIR 3 )r, so F due to whole
[A) Acceleration Due to Gravity 'g'
sphere at B will be (GMm14 3 )2. So
If in Newton's law of gravitation one body (say earth) is
~ ~ ~ taken as 'reference body', the force with which reference body
FA+FB+FR =(2GMml64) (usually earth) attracts any other body towards its centre is
However, Fl1 = 0 as the point B is at the centre of called force due to gravity (of reference body) and the
sphere B and FA = [G(M 164)m14 2 Jas the point B is phenomenon 'gravity'.
outside it. So Now if the body is free to move, force of gravity (of
reference body) in accordance with Newton's II law (i.e.,
F = 2GMm_ GMm = GMm[2 _ J..] * 0 F = ma) will produce an acceleration in it. The acceleration
R 64 64x16 64 16 produced in a body by the force of gravity of reference body
i.e., statement (b) is not true.
(usually earth) is called acceleration due to gravity and is mass were concentrated at the centre, i.e. , g '= ! =
represented by 'g'. (GMlr2) [as 1 = (Flm) = g], so at the surface of earth
Regarding acceleration due to gravity, it is worth noting
g = GMIR 2 [asr=R]
that:
and for a height h above the surface of earth,
(1) It is a vector having dimensions[LT- 2 ]. SI unit mls 2 .
Its value near the surface of earth is 9.8 mls 2. g = [GMI CR +h)'] [asr = R +h]
(2) As by definition g =(Fglm) and also I =(Fo lm) so g' R2 .
-= 2' I.e., g, = g .... C·tV )
g = 1, i. e., acceleration due to gravity and
g CR + h) [I + Chi R)]'
gravitational intensity I at a point are equal in
magnitude and direction. However, it must be kept in So with increase in height 'g' decreases. However, if
mind that they are different physical quantities as one h«R

(3)
is force per unit mass while the other acceleration.
If the reference body has mass M and radius R, the
force on a body of mass m at the surface of reference
g' = g[l+ ~r i.e., g = g[l-2 ~l . ·.Cv)

body by Newton's law of gravitation will be (d) With depth: As in case of spherical distribution of
mass for an internal point,
FG = CGMmlR ') ····Ci)
g ~ I = CGMIR 3)r
and if 'g' is acceleration due to gravity of reference
body on its surface, by Newton's II law, So at the surface ofearthg = (GMIR 2) [asr = R]
Fg =mg .... (ii) and for a point at a depth d below the surface,
As Eqns. (i) and (ii) represent the same,
g, = GM CR _ d ) [asr = R - d]
GM R3
g = R2 .... (iii)

This is the relation between 'g' and 'G' and shows that .. ~ =( R~ d} ie,
g depends on reference body (i. e., MIR 2).
So with increase in depth below the swface of earth
(4) It is independent of mass, shape, size, etc., of , falling
'g' decreases and at the centre of earth it becomes
body', i.e., a given reference body produces same
zero [as for centre i.e., d = R].
acceleration in a light and a heavy falling body.
g'
Pole
(5) It is not a universal constant and depends on place,
position and planet as discussed below. ....""'._-_. g
(B] Variation in 'g'
(a) Due to shape of earth: The earth is not perfectly
spherical in shape but is an oblate sphere. It is bulged
at the equator and flattened at the poles (its equatorial
radius is about 21 Ian more than polar radius). As on
the surface g oc (lIR 2), the value of g increases from (al (b) (e)
Fig. 9.11
equator to pole, i.e., at the sUlface of earth 'g' is
minimum at equator and maximum at poles. (e) Due to rotation of earth:
The earth is rotating Pole
cD
(b) Due to change of planet: As g =(GMIR2) and M = . a, p
about its axis from west a --
(4/3)rrR 3 p:
to east. So every body on
g= GM =1 rrGpR its surface is moving in a
R' 3 circle, i. e., is in accelerated o Equator

motion. So the effective


i. e. , greater the value of(MIR 2) or pR, greater will
acceleration due to
be' g' and as with change of reference body (MIR 2) gravity (as in a lift
orpR will change, so will 'g'. accelerated down) will be Fig. 9.12
(c) With altitude: As for an external point, a spherical g'=g-a .. .(vii)
distribution of mass behaves as if the whole of its where a is the acceleration of the body towards the
centre of earth due to its circular motion.
Now for a body at P the centripetal acceleration a r = 00 2r
~2mg[~ -;]
along PO' =00 2R COSA along PO' (as r = R COS A); so
component of ar along PO wilt be GM h
2
or W2 - fJ! =2 m --
a = Q r cosA = Ro,} C05 ')... .... (viii) R' R
Substituting this value of a in Eqo. (vii),
g' =g- Ro,}COS 2 A .... (ix)
[as g = c;:: and(h! -h2 )=h]
This is the required result and from this it is clear that for a or W2 -fJ! =2mhG (~ 1tRJp ) =~1tPGmh
given 00, 'g' is maximum C"" g) whencosA. =min=O, i.e.,}., = 90", R' 3 3

[asM ~~ 1tR'pJ
i.e., at the pole and minimum (g -R(ji) when
cosA = max = 1, i.e.,"}" =0", i.e., at equator. Furthelmore for a
given point on the surface of earth (X. = canstt.) g decreases as ro
increases and vice versa. Problem 16. A body is suspended on a spring balallce in
a ship sailillg along the equator with a speed v. Show that the
Problem 13. Calculate the acceleratioll due to gravity at scale reading will be very close to Wo (l ±2mvlg) where 00 is
the sUlface of Mars if its diameter is 6760 km and mass the angular speed of the earth and Wo is the scale readillg
one-tenth that of earth. The diameter ofearth is 12742km and when the ship is at rest. Explain also the significance ofplus or
acceleration due to gravity on earth is 9.8 1111s2. minus sign.
Solution: We know that g=(GMIR 2 ) Solution: We know that at equator due to rotation of earth
g' = (g - R002), so if m is the mass of body
So, gM ~( MM)(!!L)2 ~(.!..)(12742)'
gE ME RM 10 6760 Wo ~ m(g-R"")~m[g-V:] [asV~R"'] .... (i)
2
j.e., (gM1gE)=O.35 or gM =9.8xO.35 =3 .48mJs
Now when the ship is moving along the equator with a
Problem 14. Compute the mass and density ofthe 1/100/1 if speed v, the speed of ship relative to the centre of earth will be
acceleration due to gravity on its surface is 1.62 nt/s 2 and its V ± v. The plus sign holds ifthe ship is moving in the direction
radius is 1.74 x 10 6III (G = 6.67 x 10- 11 MKS units). of motion of earth, i. e., west to east and minus if the ship is
moving in opposite direction. So the weight recorded by the
Solution: We know that g = (GMIR 2)
spring balance in the moving ship will be
, 6 ,
W~m
M ~gR ~1.62x(L74xIO) ~7.35x I022kg
So,
G 6.67 x 10 11
[g - "-.(V..:;:",V)'-.2 ]
M gR2 3g
and as p~-~ ~ --

V G(4/3).R' 4.GR
p~ 3xl.62
So,
4x3.l4x6.67xlO 11 x1.74xl0 6
:=3.3xIO J kglm 3
Problem 15. Two equal masses m alld 111 are hungfrom a
balallce whose scale pails differ ill vertical height by h.
Calculate the error in weighing, if any, ill terms of density of
earth p.
Solution: As with height 'g'
varies as ••
g'~ g ~g[I_2hJ m or w:: wo[n:2 ~v ] [as V ~ Reo]
[I +h1R]' R '. i T
h .n Here negative sign holds for the ship moving in the
and in accordance with Fig. 9.13, h1 __L ______ ________ _ direction of motion of earth, i.e., from west to east, i.e., the
h! > h2 , so fJ! will be lesser than h, body in the moving ship will weigh less if the ship is moving
W2 and from west to east, in the direction of motion of earth and more
Flg.9.13
if the ship is moving from east to west, opposite to the motion
of earth. [See also solved Question XVI in Chapter 6.]
§ 9.4 Potential Energy and Escape Velocity U= GMmh mgh [ as GM = g ] .... (viii)
rA] Potential Energy R 2 [1+(hIR)] [I + (hlR)] R2
As explained in § 4.4, potential energy is the energy Which for h« R reduces to the well known result
stored in a body or system by virtue of its configuration or its
position in a fie/d. U = mgh .... (ix)
(4) The potential energy at the centre of earth relative to
Now as for a conservative field,
00 will be given by

F=- -dU , I.e.,


. U dU=- 1'->
IU ->
r.F·dr Uc = mVc=_~GMm [asvc =lvs =_3GM] .... (X)
dr 0 0
2 R 2 R
, -> ->
i.e., U =- I F·dr+U o and is minimum* but not zero (however 'g' at centre
'.
Whenever and wherever possible we take reference point at 00
of earth is zero). Here it must be kept in mind that
Eqns. (v) to (ix) cannot be applied here as they are
and assume the PE to be zero there. valid ollly for r> R.
[BJ Escape Velocity ve
So U=-I:F.d7=-w [asl:F.ir'=W] .... (i) It is the minimum velocity with which a body should be
projectedfrom the suiface ofa planet so as to reach infinity. If
Now as by definition of potential
a body of mass m is projected with velocity v from the surface
V= -:: =~ [asu =-1: F.d1 =-w] of a planet of mass M and radius R, then as
T=.!nw z and U = -GMm E =.!mv 2 _ GMm
i. e., U=mV 2 R ' S 2 R
.... (ii)
From this it is clear that: Now if tI is the speed of body at 00, then
(I) Ifa particle of mass m! is at a distance rfroma particle Eoo = km(v')2 +o=km(v')2
of mass m2' then
-Gmm So by conservation of energy
- Gm ]
U = mIV= I 2 asV =~ .... (iii) 12GMml z.lzGMml z
r [
Zmv -~=Zm(tI) ,1.e.'2: mv =~ +2m(tI)
So in case of discrete distribution of masses
So v will be minimum when v' ~O, i.e.,
Gm Jm2 Gm2m3
U -_ 'I:.U j -_ _ [ ]
+
r12
+........
r23
(iv)
V = V . = lGM
e mm R
=~2gR [asg=GM ]
RZ
.... (xi)

(2) Ifthere is a point mass m at a distance r(> R) from the This is the required result and from this it is clear that:
centre of earth,
(I) Escape velocity is independent of the mass and
GMm
U=mV=- - - -GM] direction of projection of the body.
[ asVout = - r - ....(v)
r
(2) Escape velocity depends on the reference body.
So the potential energy of a body of mass m relative to Greater the value of (MIR) (or gR) for a planet,
00 at the surface of earth will be: greater will be the escape velocity.
U =-GMm (3) For earth as g == 9.8 mls z and R = 6400 km,
, R [as r = R] .... (vi)
ve = ~2 x 9.8 x 6.4 x 10 6 = 11.2 kmJs
and at height h above its surface will be
(4) Ifv< v e' i.e., ifbody is projected with velocity lesser
GMm
U_
h- (R+h) asr=
[Rh]
+ .... (vii) than escape velocity, it witl reach a certain height and
then may either move in an orbit around the planet or
(3) The potential energy ofa body relative to the surface may fall down back to the planet. In this situation
of earth in the light of Eqns. (vi) and (vii) will be '*
equations of motion are not valid (as g constt.) and
the problem is solved by applying conservation of
U =U h -U
,
=GMm[~- I ]
R (R+h)
mechanical energy [See solved Problems 22 and 23].
(5) If v>ve,i.e., if a body is projected with velocity
greater than escape velocity, then by conservation of
energy,
• As Us - GMmIR.Uc -U, '"' -(II2)GMmIR.
!mv' - GMm =!m(t/)' +0 Problem 19. The masses and radii ofthe earth and moon
2 R 2 are M1,R 1 and M 2 ,R 2 respectively. Their centres are at
distance d apart. What is the minimum speed with which a
.
I,e.,(v') 2 =V 2 2GM
---=v
R
2 -v 2 [ asv
e e
=~2GM]
-R- particle of mass m should be projected from a point midway
between the two centres so as to escape to infinity?
i.e., tI=~v2 -v; Solution: Potential energy of m when it is midway
between M I and M 2'
i, e., the body will move in iute ianetary or
GMt -GM,]
interstellar space with velocity (v 2 - v;). U=m(Vt +V,)=m [ --d-I-2 +---'-dl""2~
(6) A given reference body (planet) will have atmosphere -2Gm
jfthe velocity of molecules in its atmosphere [which =-d- [M t +M,l
accordin to kinetic theory of gases is given by
And as potential energy at infinity is ~ro, so work
vnm = (3RTIM) is iesserthan escape velocity. This
required to shift m from the given position to infinity,
is why earth has atmosphere [(as at earth vnm < v e)
W=0-U=2Gm(M t +M,)ld
while moon has no atmosphere (as at moon
Vnru >v e)],
As this work is provided by initial kinetic energy,
Problem 17. Three particles each o/mass mare placed at !mv2 = 2 Gm(MI +M2 ) or V=2!G(M 1 + M 2 )
the corners ofan equilateral Mangle a/side d. Calculate (a) 2 . d d 'i
the potential energyofthe system. (b) work done on this system Problem 20. Distance between the centres aftwo stars is
if the side ofthe triangle is changedfrom d to 2d. 100. The masses of these
Solution: (a) As in case of ,<:,m stars are M and 16M and
two-particle system potential energy is
given by (-Gml m2lr), so d,,-, /srf \
their radii a and 2a,
respectively. A body of
U A =U 12 +U 23 +U 31
, 11 / , '\d/ \
,
mass m is fired straight
from the surface of the
So U
A
=_3 Gmm =_3Gm
d d
A .R"_______________\ -:. B larger star towards the I
F g. 9.15
m m smaller star. What should
Fig. 9.14

__ 3Gm
,
(b) When d is changed to 2d, be its minimum initial speed to reach the sUrface ofthe smaller
star? Obtain the expression in terms ofG, Manda.
U B- - - Solution: The distance (from the smaller planet) where
2d
the gravitational pull of two planet's balance each other will be
3Gm' given by
So work done =U B -VA =~
-GMm -G(16M)m
Problem 18. What will be the acceleration due to gravity x 2 = (lOa _x)2
on the sUrface ofthe moon ifits radius were (1I4)th the radius
ofearth and its mass (lIS0)th the mass of earth? What will be i.e., x =20
the escape velocity on the surface ofmoon ifi! is 11.2kmls on So the body will reach the smaller planet due to planet's
the surface of the earth? (given that g =9.S mls2) gravitational field ifit has sufficient energy to cross the point B
(x=2a),i.e.,
Solution: As on the surface ofplanetg = (GMIR 2) 1 ,
'2 mv >m(VB-VS)
gM = MM x(RE
gE ME RM
J' =~x(4)'
SO
=!
5 but V [16GM GM 1=---g;-
65GM
s =- ~+ (100-20)
g 9.8 ,
gM =S=S=1.96m1s
and VB =_[16GM + GMJ=_20GM
SO 2a 8a
Furthermore as escape velocity ve =~(2GM/R)
So !mv2>m[65GM _20GMJ
so :; = :; x :: =~810X4= k 2 So 8a

i.e., v. =llGM Answer


i.~, v M =(v EI£o)=(1 1.2/4.47) =2.5 kmIs nun 2 a
Problem 21. A mass 6 x 10 24 kg (= mass of earth) is to be Solution: (a) As PE of the rocket at the surface of earth is
compressed in a sphere in such a way that the escape velocity (-GMmIR) and at 00, zero, so energy required for escaping
from earth
from irs suiface is 3 x 10 8 mls (equal to that of light). What
should be the radius a/the sphere? = o_ (_ G~m)=mgR
SoluHon: As v, = ~(2GMIR), R = (2GMlv;)
R = 2 x 6.67 x 10- 11 x 6 x 10 24 And as initial KE of the rocket ~ mv 2 = 2mgR is greater
(3 x 10')' than the energy required for escaping (= mgR), the rocket will
escape.
(b) If v is the velocity of the rocket in interstellar space
Note: If v<vc (=c) the particle cannot escape and as according to (free from gravitational effects) then by conservation of
theory of relativity it is nol possible to accelerate a material energy,
particle to v;?: C, so nothing can escape from such a dense
material. Such objects are known as Black Holes. A number of !m(2./iR)' -!m(~2gR)' =!mv'
black holes exist in space and even light cannot escape from 222
these. i. e., v'=4gR - 2gR or v= J 2gR
Problem 22. A projectile is fired vertically upward from Problem 25. Imagine a planet whose diameter and mass
the surface ofearlh with a velocity KV e where ve ;s the escape are both one half o/those oj earth. The day's temperature of
velocity and K < t Neglecting air resistance, show that the this p lanet surface reaches upto 800 K. Make calculation and
maximum height to which it will rise measuredfrom the centre tell whether oxygen molecules a,re possible in the almO$pher~ of
oj earth is RI{l- K2) where R is the radius ofthe earth. the planet. (Escape velocity fro m earth's surface = 11 .2 kmls,
k 1.38)( 10- 23 JIK, mass%xygen molecule S.3)( 10- 26 kg)
= =
Solution: If a body is projected from the surface of earth
with a velocity v and reaches a height h , by conservation of Solution: As v = ~2GM
energy (relative to surface of earth), , R
I 2 mgh
i. e., - mv = ;;-~::::::;-; so -=
vp
2 [I + (hIR») vE
In this problem v = KVe = K"j2gR and h = (1' - R)
i.e., Vp =V E =11.2kmls .... (i)
So !mK 22gR ::::: mg(r - R) , i.e., K 2 =(r -R) Now according to kinetic theory of gases,
2 [I+(r - R) IR) r
v =pRT = P(Nk)T
or (l-K 2 )r=R. i.e., r = RI(I - K 2 ) nTIS M Nm
Problem 23. A particle is fired vertically upwards from [asR = Nk and M = Nm]
the surface of earth and reaches a height 6400 km. Find the r -- - ;;o;'---
=
initial velocity oj the particle if R 6400 km and g at the i.e., Vnns = ~3mkT =,J~3_X.:.I..:.3.:.8_X.:.I;Oc.-_23o;;-x-,8=00 =0.79 kmls ....(ii)
sUrface ojearth is I 0 mls 2. S.3x 10 26

Solution: Ifa body is projected from the surface of earth From Eqns. (i) and (ii) it is elear that velocity of oxygen
with a velocity v and reaches a height h , by conservation of molecule in the atmosphere of planet (- 0.8 kmls) is much
energy (relative to surface of earth). lesser than escape velocity from the planet (= 11.2 kmls); so
oxygen molecules cannot escape and so the planet's
! mv2 = "C""".::""lg,:;h~ atmosphere may contain oxygen.
2 [I + (hIR»)
§ 9.5 Planets and Satellites
Here h=R =64QOkm and g =I Omls 2
IAI Planets
So v 2 =gh, i.e., v=~10 )(6400 x J0 3 = 8km1s Planets are large natural bodies rotating around a star in
Problem 24. A rocket is fired with a speed v = zJiR
near
definite orbits. The planetary system of the star sun called
solar system consists of nine planets, viz., Mercury, Venus,
the earth 's surface and directed upwards. (a) Show that it will Earth, Mars, Jupiter, Saturn, Uranus, Neptune and Pluto. Out
escape from the earth. (b) Show that ill interstellar space its ofthese planets Mercury is the smallest, closest to the sun and
speed is v = ~2gR. so hottest. Jupiter is largest and has maximum moons ( 12).
Venus is closest to Earth and brightest. Kepler after a life time
study worked out three empirical laws which govern the (2) Orbital velocity depends on the mass of central body
motion of these planets and are known as Kepler's laws of and radius of orbit. Greater the radius of orbit lesser
planetary motion. These are: will be the orbital velocity. [vo ocllJ;.]
(i) The Law of Orbits: All planets move in elliptical
(3) Close to the surface of earth r = R as h = 0
orbits with the sun at one focus.
(ii) The Law of Areas: A line that connects a planet to Vo =~G: =,JiR =)lOx6.4xI0 6 o8km/s
the sun sweeps out equal areas in equal time, i.e., the
areal velocity of the planet is always conslant. (4) Close to the surface of planet
(iii) The Law of Periods: The square of the period of
revolution ofany planet is proportional to the cube of
the semi.major axis of the orbit, i.e., T2 oc r3. .
Vo =r: = ~.ie.v, =(~)vo[asv, = l~M 1
i. e., ifthe speed of an orbiting body is made.J2 times
Newton showed that these empirical laws
followed from his law of gravitation. These laws are ofits initial velocity (i. e., velocity is increased by 41 %
found to hold equally well for satellites either natural or KE is doubled), the body will escape.
or artificial orbiting around a planet [Earth or any
Note: If the speed Vo is changed to v such thatv o < v < vt' the orbit will
other massive central body].
change from circular to elliptic [See Fig. 9.16 (b)].
[B1 Satellites
Satellites are natural or artificial bodies describing orbit (b) Time Period T: 1fT is the period of revolution,
around a planet under its gravitational attraction. Moon is a
T=2n=2nr [asvo =rooJ
natural satellite while INSAT-IB is an artificial satellite of OJ Vo
earth. We shall here now compute and discuss some physical
entities of interest for a satellite undergoing circular motion. which in the light ofEqn. (i) reduces to
(a) Orbital Velocity vo: Let a ' satellite of mass m be
T = 2~ GM
r3 = 2., t
R '{g
[as g = GM ]
R'
.... (ii)
moving in an orbit of radius r with speed Vo as shown in Fig.
9.16 (a), the centripetal force required for the motion is From this expression, it is clear that:
(mv~/r) while the gravitational force is (GMmlr2). As (1) Time period is also independent of the mass of

,
gravitation provides centripetal force,
mvo GMm
orbiting body and depends on the mass of central
body and radius of the orbit.
-r-=--'-'
r (2) T2 = (4n 2/GM)r 3 ,i.e.,T 2 ocr 3 which is Kepler's
III law of planetary motion with r -t a (semi-major
or
Vo =t~ =R~(R :h) axis) if the orbit is elliptic.
2 2
(3) As r 2 = (4n 21GM)r 3, i.e., M = [4n r 3/GT ).So by
with GM
g = - and r=(R +h) ····0) knowing the radius of the orbit and time period, we
R' can calculate the mass of central body M. This is how
:> VE Hyperbola we compute the mass of sun [See solved Problem 26]?
(4) If the satellite is revolving close to the surface of the
earth h -to, i.e.,r=R.

[R 02.~6.4 x 106 .
Vo VO<V<VE
T = 2.
Vg 10

=2nx 800s = 84.6 minute:: 1.4 hr


(a) (b)
FIg. 9.16 Note: This is the same as the time period of a simple Pf': ru:1 u1um of
infinite length [See § 8.2 (5) (b)J or time period ofSHM of a ball
From this expression it is clear that: in a tunnel through the earth [See § 8.4 (C) (b)] or time period of
(I) Orbital velocity is independent of the mass of the earth's rotation so that the weight of a body at its equator
orbiting body and is ?llways alcng the tangent to the becomes zero [Sec solved Problem 51 in Chapter 6].
orbit.
(5) Ifthere is a satellite in the equatorial plane rotating in
the direction of earth's rotation, i, e., from west to east,
then for an observer on the earth as angular velocity of
satellite will be (ros -ooE)'''' The time interval
between its two consecutive appearances overhead
will be
(a) v""l , (b) voc.!..
IT
Fig. 9.18
However, ifws =roE,T=oo,i,e" satellite will appear
stationary relative to earth, Such a satellite is called (d) Energy: As for external point, a spherical mass
'Geo.stationary Satellite' and is used for communication behaves as whole of its mass is concentrated at its centre;
purposes. The orbit of a geostationary satellite is called potential energy of the satellite
'Parking Orbit'. So fora satellite to be geostationary. Ts =24
hours which in the light of Eqn. (ii) gives r ~ 42000 km or
u=mV=_G~m [asv=_G~J .. (iv)

And as the velocity of satellite in orbit isv o, its kinetic energy


h:::. 36000 km. [See solved Problem 29]
(c) Angular Momentum L: In case of satellite motion K=.!.tmJ~=GMm
2 2r
[asvo=~GM]
r
.... (v)
angular momentum will be given by

L=mvr=mrf? [as v f? ] =
So its total energy (mechanical)
E=U +K=_GMm+GMm = _ GMm .... (vi)
r 2r 2r
i,e., .... (iii) From this, it is clear that
From this, it is clear that: (1) Kinetic, potential or total energy of a satellite depends
(1) Angular momentum of a satellite depends on both, the on the mass of the orbiting satellite and the central
mass of orbiting and central body. It also depends on body and also on the radius of the orbit.
the radius of the orbit. (2) From Eqns. (iv), (v) and (vi),
(2) In case of satellite motion, force is central, so torque
~ =-1, i.e., K=-E and ~ =2, i.e., U =2E
== 0 and hence angular momentum of the satellite is
conserved, i.e., L == constt.
(3) In case of satellite motion
as areal velocity
cIA I (r)(vdt) I
-=-
dl2dl2
=-IV •__"'!II'AS ,,,,dt
equa/IO Mil\:
i.e., kinetic energy ofa satellite in an orbit is equal to
negative of total energy while potential energy is

and as L =tmJr
so cIA
dt2m
=.£ ....(iv)
Fig. 9.17

But as L = constt., areal velocity (dA/dt) ==constant which


is Kepler's II law, i.e., Kepler's II law or constancy of areal
velocity is a consequence of conservation of angular
fp='- la)
Satellite
momentum.
Note: In case of satellite motion as L :::: mvr = constt., I.> oc (1/r). While Pe~gee ' Apogee
i.-a
according 10 Eqn. (i) orbital velocity voc(l/..Jr} These two KE • max
PE _ min ,
r--::-<f--f;;:;:C:;:;::1 KE • min
: Semi-major PE _ max
results appear to be contradictory. However, this apparent ,, '
: axis
,:
contradiction is resolved if we keep in mind that v oc (1lr) holds : ' ,
good for different points of the same orbit while v IX (IIJr) for :: :
"rmltl" rmax "
different orbits as shown in Fig. 9.18.
Ib)
FIg. 9.19

.. See § 7.1 (b).


(3) Total energy of a satellite in its orbit is negative. (2) As everything is in free fall, so objects are at rest
Negative energy means that the satellite is bound to relative to each other, i.e., if a table is withdrawn from
the central body by an attractive force and energy bclow an object, the object will remain where it was
must be supplied to remove it from the orbit to without any support.
infinity. The energy required to remove the satellite (3) Ifa glass ofwaler is tilted and glass is 'pulled out', the
from its orbit to infinity is called Binding Energy of liquid in the shape of container will float and will not
the system. i. e., flow because of surface tension.
Binding Energy = -E = GMm (4) Ifone tlies to strike a match, the head will light but the
2r stick will not bum. This is because in litis situation
convection currents will not be set up which supply
Note: If the orbitofa satellite is elliptic, (I) tbe energy E .. (-GMmf2A)
oxygen for combustion.
'" consn. with a as semi-major axis; (2) KE will be maximum
when the satellite is closest to the central body (at perigee) and (5) If one tries to perfonn simple pendulum experiment,
minimum when it is farthest from the central body (at apogee) [as the pendulum will not oscillate. It is because there
for a given orbit L .. constt., i.e. ,mvr "" constt. i.e. ,v 0: lIr] (3) PE will not be any restoring torque and so
= (E-K)will be minimum when KE::: max, i.e., the satellite is
closest 10 the central body (at perigee) and maximum when KE = T = 2.~(Ug') = '" (as g' = 0).
min, i. e., the satellite is farthest from the central body (at apogee), The condition of weightlessness can be overcome by
creating artificial gravity by rotating the satellite in addition to
(e) Weightlessness: Ifa body is in a satellite (which does
its revolution.
not produce its own gravity) orbiting the earth at a height h
above its surface, ·then as Question I. Can a satellite coast in a stable orbit in a
plane not passing through the earlh's centre? Explain your
answer.
Answer: No. In case of satellite motion centripetal force
W' = 1 is provided by gravitation between tbe earth and the satellite. If
So or W'= W
earth is not at the centre of the orbit [Fig. 9.20 (a)], i.e., if the
W [I + (hlR)]' [I + (hIR)]'
plane of the orbit is not passing through the centre of earth. the
This W' is the true weight ofthe body in lite satellite and is gravitational force F will not be directed towards the centre of
lesser than the weight on the earth. the orbit 0 ' and hence the orbit will not be stable. The
Now as the satellite is moving in a circular orbit, it has a component of gravitational force perpendicular to the plane of
radial acceleration orbit will pull the satellite till its plane passes through the
centre of the earth as shown in Fig. 9.20 (b).

i.e., it is falling towards earth's centre with acceleration a, so


apparant weight of the body in it,
Wap = m(g'- a)
where g' is the acceleration due to gravity of earth at the
la) Ib)
position of satellite, i.e. , g' = (GMlr 2 ), so that Fig. 9.20
Wap =m[GM2 _GM]=
2
O Question II. An astronaut in a satellite releases a spoon
r r out of the satellite into the space. Will the spoon falJ 10 the
i.e., lite apparent weight of a body in a satellite is zero and earth?
independent ofthe radius of the orbit. Answer: No. Due to inertia of motion the velocity of the
When the weight of a body (eitber true or apparent) spoon is equal to that of satellite. As the radius of orbit is
becomes zero, the body is said to be in the state of independent of the mass of satellite, the spoon will continue to
weightlessness. In this condition: m(lve along with the satellite in the same orbit, i. c., it will 31so
become a satellite of the earth (and will not fall to lite earth).
(I) One will find it difficult to control his movement, for
without weight he will tend to float freely. To get from Question 111. Is it possible to put an artificial satellite
one spot to the other he will have to push himself into orbit in such a way that it will always remain directly over
away from the walls or some other fixed objects. New Delhi (a place which is lIot in the equatorial pla1le).
Answer: No. A satellite wilt appear stationary only when i.e., if the earth rotates on its axis so fast that the length of day
the direction of motion and magnitude of angular velocity of becomes 1.4 hour, a body on its surface will be in 'true satellite
the satellite is equal to that of earth" i,e., the plane of the orbit orbit' .
is equatorial with time period 24 hours (from west to east) [Fig. Question VI. A satellite in low earl!! orbit experiences a
9.21 (a)]. However, New Delhi is not in the equatorial piane, small dragforcejrom the earth's atmosphere. What happens
so it will not be possible to put a geostationary satellite over to its speed because oJthis dragJorce?
New Delhi [Fig 9.21 (b)]. Answer: As due to air drag some mechanical energy of
the satellite will be converted into heat energy, there will be
loss of ME of the satellite, i.e., E = -(GMm/2r) will decrease.
This in tum implies that the radius of the orbit will decrease
and so the satellite wiil follow a sp iral path towards the earth as
shown in Fig. 9.22.
Satellite
Now as in case of satellite
motion,
PE =_ GMm KE = GMm
and
Cal r 2r
Cbl
FIg. 9.21 so with decrease in 1', PE will
Question IV. As measured by an observer on earth, decrease but KE and hence speed
would there be any difference in the periods of two satellites will increase. Increase in speed (or
each in a circular orbit near the earth's equatorial plane. but KE) inspite oj decrease ill
one moving eastward and the other westward. mechallical energy is accounted by Fig. 9.22
the fact that the rate at which PE
Answer: We know that the earth is rotating on its axis and
decreases (due to decrease ill r) is more than the rate at which
due to this rotation of earth the velocity of a point on the
mechanical ellel'gy decreases.
equator is 0.47 kmlsec from west to east. So the velocity of the
satellite moving towards east relative to earth will be V - v Problem 26. Calculate the mass oj the sun if the mean
while the velocity of the satellite moving towards west relative radius oJthe earth's orbit is 1.5 x 10 8km and G = 6.67 x 10- 11
2 2
to earth will be V + v. So the period of the two satellites as Nxm /kg .
observed by an observer, on the earth will be Solution: In case of orbital motion as v =.J(GMlr) so
(T) ,'" 21tR and (T) w," = 2nR [as T =21tR] 2.r r;:- 4n 2,.3
(V - v) (v+v) v T =-V= 21tr
VGM ' i. e., M~-­
GT'
i. e., the time period of the satellite moving towards west will be
less as compared to the satellite moving towards east. M= 4xn 2 x (1.5 x 10")3
Question V. Objects at rest 011 tile earth's sUI/ace move 6.67 x 10- 11 x(3,15xI07)2
in circular paths with a period of 24 IIolil's. Are they in 'orbit'
[asT = lyear=3J5x 10 7 s
in the sense that an earth satellite is in orbit? What would the
length of the day have to be to put such objects ill true orbit? i.e., M =2xlO 30kg
Answer: We know that the apparent weight ofa body in a Problem 27. Imagine a light plallet revolving around a
satellite is zero. However, the weight ofa body at the equator,
very massive star in a cil'cular orbit oJradius r with a period OJ
considering the motion of earth, i.e., W = meg ~ ROl 2 ) ,*0. So revolulioll T. 011 what power of 1', will the square of time
bodies at rest on the surface of the earth (though moving in period depend if the gravitational force ofattraction betweell
circular path) are not in orbit in the sense that an earth satellite the planet alld the star is proportional to 1'- 512 ?
is in the orbit.
Solution: As gravitati on provides centripetal force
Now if the body on the surface of earth is to be in 'true
= ~, =~
2
orbit' , mv i.e. , v2
r rS12 ml'3/2
W=m(g-Ro,'j=O, i.e, o>=)(gIR)
312
so that 2nr Jmr
T=-=21tr--
Sothat T = 21t = 21t
0> vii{R =21t v
2
K

i.e., r2 = 41t 11l ,.7/2; so r 2 ocr 7/2


::. 83.8 minute ::.1.4 hr K
Problem 28. Halley's comet has a period 0/76 years and Problem 30. A space-ship is launched into a circular
in 1986, had a distance ofclosest approach 10 the sun equal to orbit close to the earth's sUrface. What additional velocity
8.9 x IO lO m. What is the comet's/ar/hest distance from the sun should now be imparted to the space-ship in Ihe orbit to
if the mass o/sun is 2 x IO lO kg and G=6.67 x lO- 1l MKS overcome the gravitational pull. (Radius of earth = 6400 /em
units? and g =9.8m1s 2 )
Solution: From the problem it is se l f~evident that the orbit Solution: For orbiting the earth close to its surface,
of the comet is elliptic with sun being at one focus [See Fig.
9. 19 (b)]. Now as for elliptic orbits, according to Kepler's third
law,
mv 2
GMm
T=-,-
R
' . ,
I.e. [GM
vO=VT=...;gR =[ GM ]
as g =R 2

' J"3
T GM i. e., Vo = ~(9.8x6.4x106)=:'8kmJs
i. e., 0=
[ 4n' and for escaping from close to the surface of earth,

or
G~m +nv;. i.e. v, =l~M =~2gR [as g ~~]
=

i.e., ve=J2 x vO= 1.41 x8km1s=I1.2kmJs


= 2.7 x 10 12 m So additional velocity to be imparted to the orbiting
But in case of ell ipse, =
satellite for escaping = 11 .2 - 8 3.2 kmls
Problem 31. An artificial satellite is moving in a circular
2a =rmin + 'inax' i. e., Y
max
=2a -rmin
orbit around the earth with a speed equal t!J halfthe magnitude
so rmax = 2 x 2.7x I01 2 - 8.9x lO lO =S.3x I0 12 m ofescape velocity from the earth. (a) Determine the height of
the satellite above the earth's surface. (b) If the salellite is
Problem 29. Consider an earth satellite so positioned
stopped suddenly in its orbit and allowed to fall freely onto the
thai it appears stationary to an observer on earth and serves earth.jind Ihe speed with which it hits the surface ofthe earth.
the purpose of a fIXed relay station for intercontinental
transmission o/TV and other communications. What would be (g =9.8 ms-' and R£ =64ookm)
the height at which the satellite should be positioned and what Solution: (a) We know that for satellite motion
would be the direction of its motion? Given that the radius of
the earth is 6400 km and acceleration due to gravity on the vo =~GM =R r g [.Sg= GM and, =R+h]
surface of the earth is 9.8 mls 2.
r V(R+h) R'
Solution: For a satellite to remain above a given point on Inthisproblem Vo =~Ve =~~2gR [asv, =~2gR)
the earth 's surface, it must rotate with the same angular
velocity as the point on earth's surface. Therefore the satellite R'g 1
must rotale ill the equatorial plane from west to east with a time so R+h=2gR, i.e., 2R = h+R or h=R = 6400km
period of 24 hours.·
(b) By conservation of ME
Now as for a satellite Vo = ~(GM 1r)
0+( _G~m)=tmv' +(- G~m )
T =~:= 2nr~;M=2nr~g~' [asg=~~] , [I 1]
or v =2GM - - - [asr=R +h =R + R =2R )
6 R 2R
or r= [gR' T' ]"3 = [9.8 X (6.4 x 10 )2 x (86400)2 ]"3
41t2 41t2 or v =JG: =JiR =~lO x 6.4 x l06 =8km1s
or r;;4.23x!Q7 m=42300km Problem 32. A sky lab ofmass 2 x 10 3 kg isjirst lau.nched
So the height of the satellite above the surface of earth, from the surface ofearth in a circular orbit ofradius 2R (from
the centre of earth) and then it is shifted from this circular
h = r - R = 42300 - 6400 "36000 Ian orbit to another circular orbit of radius 3R. Calculate the
[The speed ofa geostationary satellitev o = R~(gl r) =roo minimum energy required (a) to place the lab in the jirst orbit,
(b) to shift the labfromjirst orbit to the second orbit. Given,
= 4.2 x 10 7 x 7.3 x 10- 5 = 3. 1 kmls.]
R =6400km andg = IOmls 2.
.. See § 9.5 b (5) And solved Question III.
Solution: The energy of the sky lab on the surface of earth Problem 34. Two satellites ofsame mass are launched in
GMm) GMm the same orbit round the earth so as to rotate opposite to each
Es = KE + PE=O + ( ~R =~R other. They soon collide inelastically and stick together as
wreckage. Obtain the total energy ofthe system before andjust
And the energy of the sky lab in an orbit of radius r after the collision. Describe the subsequent motion of the
E ~--nwo [asv a=~l
, + [ -GMml wreckage.
I - - -_ -GMrn Solution: In case of satellite motion energy of a satellite
2 r 2r
in an orbit is given by
(a) So the energy required to place the lab from the surface
of earth to the orbit of radius 2R. E =_ GMm
2r
E] - Es =-~~~ -[- G~rn HG~rn So the total energy of the system before collision
E j =E 1 + E2 = 2E =_ G~m
3m
i.e., llE=--xgR 2
=-3 mgR
4R 4
As the satellites of equal mass ,,
....... -----... .... ,
,
,,
10 are moving in opposite direction / ''
i.e., dE = %(2 x 10 3 x 10 x 6.4 x 10 6 ) =%(12.8 x 10 ) '
and collides inelastically, the

~\!, ~J
velocity of wreckage just after
=9.6x10 10 J
collision, by conselVation of linear
(b) As for II orbit r =3R, momentum will be
GMm GMm mv - mv =2mV, i.e., V=O , ,
En =-2(3R) =- 6i/ ' .......... _------'"
i. e., just after collision wreckage
Fig. 9.23
En -E 1 =_G~m _ ( - G:!m )=l~ G~m
comes to rest in the orbit. So
energy of the wreckage just after collision will be totally
potential and will be
Butasg=(GMIR 2 ), i.e., GM=gR 2 EF = GM(2m) =_2GMrn
r r
or till = I~ rngR = I~ (12.8 x 10 10 ) = l.l x 10 10 J
And as after collision the wreckage comes to stand still in
Problem 33. If a satellite is revolving around a planet of the orbit, it will move along the radius towards the earth under
mass M in all elliptic orbit ofsemi-major axis a, show that the its gravity.
orbital speed ofthe satellite when it is at a distance r from the Problem 35. An object weighs ION at the north-pole of
focus will be given by: the earth. In a geostationary satellite distant 7R from the
centre ofearth (ofradius R) what will be its (a) true weight, (b)
v' =GM[;-±l apparent weight?
Solution: (a) The true weight of a body is given by mg and
Solution: As in case of elliptic orbit of a satellite
mechanical energy E =- (GMml2a) remains constant, at any with height 'g' decrease;

~ =:;' =[I + (~/R)]' [as g ' =[1 + (~R)],l


position of satellite in the orbit,
t ~\" KE + PE = - GMm .... (i) So
2a
Buthere h=7R-R=6R, i.e., h1R=6
Now, ifat position r, v is the orbital speed of satellite,

KE =-I nw 2 So ~ = WE =.!Q=O.2N
2 s [I +6]' 49
\"..11\ )H\\'.\ \1,
PE=_GMrn .... (h) (b) If g' is the acceleration due to gravity of earth at the
nnd:"".i ;\~. -1.\'\\ ,
r position of satellite, the apparent weight of a body in the
' \,,\\\')"'.\')1,',','] \"i,

Sp. fNm. pql,l~. mand (ii), we have satellite will be


Wapp = m(g' - a)
But as satellite is a freely falling body, i. e., g' =a

i.e., v' = GM[~-.!.l


}' a
So Wapp = O
MISCELLANEOUS SOLVED PROBLEMS

Problem 36. If a planet was suddenly stopped in its orbit


supposed to be circular, show that it would/all onto the sun in or
a time (-./2/ 8) times the period of the planet '$ revolution.
Solution: If the mass of sun is M and radius of the planet's or [as r' = r12]
orbit is r, then as Vo = ~(GM I r),
But we are interested not in the full period of the
T = Vo
21tr /r .
= 21tr Gii' I.e., ....(i) revolution of our imaginary planet but in half the period which
V represents the length of the comet's flight one way (from the
Now if the planet (when stopped in the orbit) has velocity planet to the sun) and is equal to the time which the planet will
take to fall onto the sun.
v when it is at a distance x from the sun, by conservation of

=2~d ~)r
mechanical energy,
.!mv2
2
+(_ GMm
x
) =o_ GMm
r
i.e, t=;'

or (-:J = 2~M[r:xJ j,e, _ :=J2~M J(r:x)


Note: (i) For earth T '" 365 days, i. ~, t = 65 days. So we find that after
the sudden stand still in orbital flight, the earth would need
more than two month's to fall onto the sun.
(ii) For moon T =27.3 days so t = 5 days. So the moon will take,

0> J;dt=-J2;MJ~[tJ' only five days to fall onto the earth if it suddenly stops
moving.

Substituting x = rsin 2 e and solving the RHS, Problem 37. A body is projected vertically upwards/rom
the surface of earth with a velocity sufficient to carry it to

t=~x(~r infinity. Calculate the lime taken by it to reach height h


Solution: Ifat a distance r from the centre ofthe earth the
which in the light of Eqn. (i) reduces to body has velocity v, by conservation of mechanical energy,

t= 4~T, ie, t= ( ~)T lmv'


2
+(_ GMm)=lmv'
2 r
+(_GMm
R
)
e

Alternative Solution: or v 2 =v 2 +2GM[R - I]


Consider an imaginary comet ' R r
moving along a strongly But as v, =.j2gR and g =(GMIR')
extended flat ellipse the extreme
points of which are located on v' =2gR +2gR[(Rlr)-I]
the earth's orbit and at the
centre of the sun. The or v = ~2gRr 2 , i.e., dr = R J2i
semi-major axis of the orbit of dt .,Jr
such a comet would apparently Fig. 9.24
be half the semi-major axis of i.e., J'd -
0 1-
I
r;;::.R
JR+h \I'd
r r
the earth's orbit. So the time period of our imaginary comet T' R"o/2g .
according to Kepler's law will be given by i.e., t = ~_I_[(R + h)3!2 _ R 3I']
(T')' T' 3 R.fii
-- = - = constant
(1',)3 r3
i.e. , t =t f f[(l+ ;)'" -I]
Ox
• - Ir (-Jdx=-f .[
mr-x
orsine
12 .2 2 ] ' " r·2sin9cos9dG
" r- rsm 9

= _,.[0 (l-COS29)de= - r[ 9-.!.Sin2er = w


11 / 2 2 12 2
Problem 38. A cord of length 64 m is used to ,connect a And the equation of motion of the astronaut will be
100 kg astronaut to a space-ship whose mass is much larger
than that of the astronaut. Estimate the value ofthe tension in - -'-+T = mrro2, i.e., T=m [ rea 2
GMm -GM]
-,'
the cord. Assume that the space-ship is orbiting near earth
? r
surface. Also assume that the space-ship and the astronaut/all In the light of equation (i), above equation reduces to
on a straight line from the earth's centre. The radius of the
earth is 6400 km. T=mg[!.._(R)']=mg[(R+h) R']
R r R (R +h)'
Solution: As according to given
problem the mass of satellite M is much [asr=R +h]
greater than that of astronaut m so the
centre of rmiss of the system will be
close to satellite and as the satellite is
or T=m g[(I+ ;)-(1+ ;r]=3~t
orbiting close to the surface of earth, the
equation of motion of the system
(S + A) will be:
Fig. 9.25
Hl+;r=I-~]
G::::'M:..,.::(M:;...:+.;:m:!.) =(M +m)ROl ,
= R'
So substituting the given data,
T =3 x IOOxlOx64 =3xlO-2 N Answer
GM e =g
Rro2 =__
i.e., .... (i) 6400 x 10 3
R'
EXERCISE
[A] Only One Cboice Is Correct (a) mgR (h) 2mgR
I. The gravitational force with which the earth attracts the (e) (1I2)mgR (d) (1/4)mgR
moon: 9. When the body is moving up, the acceleration due 10
(a) Is less than the force with which the moon attracts the gravity will be:
earth (a) Downward (h) Upward
(b) Is equal to the force with which the moon attracts the (c) Sideways (d) Nil
earth 10. At sea level the value of g is minimum at:
(e) Is greater than the force with which the moon attracts (a) The equator (b) 45 0 north latitude
the earth (c) 45 0 south latitude (d) The pole
(d) Varies with the phases of the moon 11. As we go from the equator to the pol~s, the value of g:
2. The dimensional fonnula for gravitational constant is: (a) Remains the same
ICPMT 19921 (b) Decreases
(a) [M-1L'Y-'] (h) [M'L- 1y-'] ' - --- (c) Increases
(d) Decreases up to a latitude of 45 0 and ~en .i~preases
3. The value of gravitational constant in eGS system is 12. A planet has twice the values of mass and radius of earth.
Acceleration due to gravity on the surface of the planet is:
6.67x 10-8 , Its value in MKS system will be:
(a) 9.8 mis' (h) 4.9 mis'
(a). 6.67x W-' (b) 6.67x 10-7
(e) 980mls' (d) 19.6m1s'
(e) 6.67x 10-9 (d) 6.67x 10- 11
13. If the acceleration due to gravity at earth is 'g' and mass of
4. If somehow the distance between the sun and the earth is
earth is 80 times that of moon and radius of earth is 4 times
doubled, the gravitational force between them will become:
that of moon, the value of'g' at the surface of moon will be:
(a) Double (h) Half
IEAMCET 1993)
(e) Four times (d) One-fourth

inside it as shown in Fig. 9.26:


(a) is attractive and constant
(b) is attractive and depends on the
Q
5. The force between a hollow sphere and a point mass at P

·C

(a) g
(e) g/5
(h) g/20
(d) 320 g
14. The value of'g' at a particular point is 9.8 mls 2 • Suppose
the earth suddenly shrinks unifonnly to half its present size
position of the point with respect to without losing any mass. The value of'g' at the same point
centreC (assuming that the distance of the point from the centre of
Fig. 9.26 the earth does.,tlo! shrink) will now be:
(c) is zero
(a) 4.9 mis' (h) 9.8 mis'
(d) is repulsive and constant
6. Consider the earth to be a homogeneous sphere. Scientist (e) 3.1 mis' (d) 19.6m1s'
'A' goes deep down in a mine and scientist'B' goes high up 15. If the radius of earth were to shrink by 1%, its mass
in a balloon. The gravitational field measured by: remaining the same, the acceleration due to gravity on the
(a) A goes on decreasing and that by B goes on increasing earth's surface would:
(b) B goes on decreasing and that by A goes on increasing (a) Increase by 1% (b) Increase by 2%
(c) Each remains unchanged (c) Decrease by 2% (d) Decrease by 1%
Cd) Each goes on decreasing 16. Two planets have radii ~ and r1 and densities d, and d 2
7. The tidal waves on the sea are primarily due to: respectively. The ratio ofthe acceleration due to gravity on
them will be:
(a) Gravitational effect ofthe moon on the earth
(a) r,d 2 : r2d, (b) Ijd,: r1d 1
(b) Gravitational effect of the sun on the earth
2
(c) Gravitational effect of the sun and moon on the earth (c) 1j d, :rid 2 (d) 'idrr2di
(d) Gravitational effect ofthe earth on the sun 17. The value of' g' will be 1% of its value at the surface of
8. A body of mass m is moved to a height equalto the radius of earth at a height of(Re =6400 km):
the earth R. The increase in its potential energy is: (a) 6400 Ian (h) 57600 Ian
ICBSE 19901 (e) 2560 Ian (d) 64000 km
454 PHYSICS FOR COMPETITIONS - Vol. 1

18. Ace, due to gravity decreases as we go up from the surface 27. The escape velocity of a particle of mass mvaries as:
of the earth. Then in going below the surface of the earth it: (a) m' (b) m (c) mO (d) m- 1
(a) Increases (b) Decreases 28. The escape velocity of a body projected vertically upwards
(c) Remains constant (d) Decreases then increases from the earth's surface is 11.2 kmlsec. If the body is
19. If the change in the value of 'g' at a height h above the projected in a direction making 300 angle to the vertical, its
surface of the earth is the same as at a depthxbelow it, then escape velocity in this case will be: ICDSE 1994]
(both x and h being much smaller than the radius of the (a) l1.2km/s (b) 11.2x(1I2)km/s
earth): (c) 11.2(,J3/2)km/s (d) l1.2x(1/3)km/s
(a) x = h (b) x=v,
29. Escape velocity of a projectile from the surface of earth is
(c) x = 1/2h (d) x =h' about:
20. If the angular speed of the earth is doubled, the value of ace. (a) 3x 1010 cmls (b) 18600 mls
due to gravity at the north-pole: IEAMCET 19951
(c) 11.2 km/s (d) 8 km/s
(a) Is doubled (b) Is halved '
30. The ratio of the radius of earth to that of the moon is 10. The
(c) Remains same (d) Becomes zero ratio ofacc. due to gravity on the earth and on the moon is 6.
21. The variation of acc. due to gravity as one moves away The ratio of the escape velocity from the earth's surface to
from earth's centre is given by: that from the moon is:

(a)!~
(a) 10 (b) 6
(c) Nearly 8 (d) 1.66
31. Escape velocity of a body from earth is about 11 kmlsec.
Assuming the mass and radius of the earth to be about 81
,- and 4 times the mass and radius of the moon, the escape
velocity in km I sec from the surface of the moon will be:

(c)!~ (a) 0.54


(c) 11
(b) 2.44
(d) 49.5
[eDSE 19931

32. The escape velocity for a rocket on earth is 11.2 kmlsec. Its
22 . . A body weighing 20 kg on the surface of the earth is taken value on a planet where acceleration due to gravity is
to a place 6000 km below the earth's surface. Assuming the double that on the earth and diameter of the planet is twice
earth's radius to be 6000 km, the weight of the body at that that ofthe earth will be in kmlsec: ICPMT 1990)
depth is: (a) 11.2 (b) 5.6
(a) 40 kg (b) 5 kg (c) 22.4 (d) 33.6
(c) Infinite (d) Zero 33. The escape velocity from the earth is about II kmlsec. The
23. The atmosphere is held to earth by: escape velocity from a planet having twice the radius and
the same mean density as the earth is:
(a) Gravity (b) Rotation of earth
(a) 22 kmlsec (b) 11 kmlsec
(c) Magnetic field of earth (d) Winds and clouds
(c) 5.5 kmlsec Cd) 15.5 km/sec
24. Ball pen functions on the principle of:
34. The escape velocity from a spherical planet is Vo' What is
(a) Viscosity (b) Capillarity
the escape velocity corresponding to another planet of
(c) Gravity (d) Atmospheric pressure twice the radius and halfthe mean density:
25. The escape velocity from a planet of mass M and radius R is (a) (.J2)VO (b) Vo/.J2
given by:
(a) ,J"'(2G
""M"'IR
" ') (b) 2,j(GMIR) (c) 2Vo (d) 4Vo
35. Kepler's second law states that the straight line joining the
(c) ,j('lMRIG) (d) 2,j(RIGM) planet to the star sweeps equal areas in equal times. This
26. If g is the acceleralion due to gravity at the earth's surface statement is equivalent to saying that:
and ris the radius of the earth, the escape velocity for a body (a) Acceleration of the planet is zero
to escape out of earth's gravitation field is: (b) Transverse acceleration of the planet is zero
(a) .w (b) .fiir (c) Tangential acceleration of the planet is zero
(c) glr (d) rig Cd) Radial acceleration of the planet is zero
36. The Fig. 9.27 shows the Kepler's law the period of revolution of Jupiter in its orbit
motion of a planet around the will b~:
sun in an elliptical orbit with (a) 5 year (b) 7.5 year
sun at the focus. The sha4ed
(c) 12 year (d) 25 year
areas A and B are also shown
in the figure which can be Fig. 9.27 43. An earth satellite can move in an orbit, the plane of which
assumed to be equal. If t I and t 2 represent the time taken coincides with:
for the planet to move from a to band c to d respectively, (a) The plane of any great circle around the earth
then: (b) The plane of any latitude circle around the earth
(a) t1 >t2 (c) Any plane not containing the centre of earth
(b) I] <t2 (d) The plane of tropic of cancer
(e) t1 = /2 44. An artificial satellite orbiting the earth does not fall down
(d) From the given infonnation the relation between t 1 and because the earth's attraction:
t 2 cannot be detennined (a) Is balanced by the attraction of the moon
37. When a planet moves around the sun, its: (b) Produces the necessary acceleration for its motion in a
(a) Areal velocity is constant curved path
(b) Linear velocity is constant (c) Vanishes at such distances
(e) Angular velocity is constant (d) Is balanced by the viscous drag produced by the
(d) All the velocities are constant atmosphere
38. Kepler's II law is a consequence of: 45. An earth satellite is moving around the earth in a circular
orbit. For such a satellite which of the following statements
(a) Conservation of energy
is wrong?
(b) Conservation of linear momentum
(a) It is a freely falling body '
(e) Conservation of angular momentum
(b) It is moving with a constant speed
(d) None of the above
(c) It suffers no acceleration
39. Two planets are moving around the sun. The periodic
(d) Its angular momentum re:nains constant
times and mean radii of the orbits are TI , T2 and 1j, r2
respectively. The ratio ofTJIT2 is equal to: 46. If an orbiting satellite comes to stand still suddenly:

(a) (r1/r2 ) (b) (ljlr )3/2 (a) The satellite will move along the tangent
2
(b) The satellite will move radially towards the centre of
(e) (OJ / r2 )2 (d) (1 /2r2 )312 the orbit
40. Which of the following graphs represents the motion of a (c) The satellite will go to ou~er space and will be lost
planet moving about the sun? (d) The satellite will continue to move in the same orbit

(a) J,~ R3___


(b) J'L R3 ___
47. If rrepresents the r'adius of the orbit 'o f a satellite of mass //I
moving around 'a planet of mass M, the velocity of the
satellite is given by;
2
(a) v =GMlr
(c) v =GMlr
(b) v 2 =GMmlr
(d) v ,= Gmlr

(e)!~ (d)!~
48. An earth satellite of mass m revolves in a circular orbit at a
height h from the s~rface ofthe earth. R is the radius of the
earth and g is acceleration due to gravity at the surface of
the earth. The velocity of 'the satellite in the orbit is given
by,
R --- R---
41. The distances of two planets (neptune and saturn) from sun (a) gJi2/(R+h) (b) gR
are 1013 and 10 12 m respectively. The ratio of time periods (e) gR(R+h) (d) ~rgR"""2'--/(-R-
+ h-)
of the planets is: (CBSE 1993)
(a) 100 (b) IIv10 49. The orbital velocity of a body close to the surface of earth
is:
(e) v10 (d) IOv1o
(a) 8 kmls (b) 11.2 km/s
42. The mean distance of Jupiter from the sun is nearly 5.2
(c) 3x 10 8 mJs (d) 2.2 x 10 3 kmls
times the corresponding earth-sun distance. According to
456 PHYSICS FOR COMPETITIONS - Vol. I

50. An astronaut orbiting the earth in a circular orbit 120 km 58. The ratio of the kinetic energy required to be given to the
above the surface of earth gently drops a spoon out of the satellite to escape earth's gravitational field to the KE
space-ship. It will: required to be given so that the satellite moves in a circular
(a) Fall vertically down to the earth orbit just above earth's atmosphere is:
(b) Move towards the moon (a) I (b) 2
(c) Move in an irregular way and then fall down to earth (e) \ /2 Cd) Infinite
(d) Move along with the space-ship 59. A satellite is orbiting a planet at a certain height in a circular
51. The orbital speed of Jupiter is: orbit. If the mass of the planet is suddenly reduced to half,
the satellite would: [EAMCET 1993)
(a) Greater than the orbital speed of earth
(a) Continue to revolve round the planet at the same speed
(b) Lesser than the orbital speed of earth
(b) Falls freely on the planet .
(e) Equal to the orbital speed of earth
(c) Orbit the planet at lesser speed
(d) Zero
(d) Escape from the planet
52. The orbital velocity of an artificial satellite in a circular
orbit just above the earth's surface is v. For a satellite 60. The period of a satellite in a circular orbit around a planet is
orbiting at an altitude of half the earth's radius the orbital independent of:
velocity is: "IMNR 19941 (a) The mass of the planet
(a) (3/2)v (b) (,f3i2)v (b) The radius of the planet
(e) (.j2i3)v (d) (2/3)v (c) The mass of the satellite
53. Two satellites of masses ml and m2 (m l > m2 ) are revolving Cd) All the three parameters a,b and c
around the earth in a circular orbit of radii 1j and rz (~ > rz ) 61. Two satellites are orbiting around the earth in circular orbits
respectively. Which of the following statements IS true of the same radius. One of them is 100 times greater in mass
regarding their speeds v ! and v 1? IMNR 1995) than the other. Compare their periods of revolution:
(a) 100 , \ (b) 1,100
(a) VI =v l (b) VI >v l
(e) \, 10 (d) 1 ,\
(e) VI <v 2 (d) (v l' Ij )=(v 2 ' r2 )
54. If the speed of a satellite orbi.!i!!s near the surface of the 62. The period of a satellite in a circular orbit of radius R is T.
earth is changed fromVo to (.J1.5)Vo' what is most likely The period of another satellite in a circular orbit of radius
4Ris:
to happen?
(a) 4T (b) TI4 (e) 8T (d) TIS
(a) It will revolve in a circular orbit of greater radius
63. A satellite A of mass In is at a distance rfrom the centre of
(b) It will escape from the earth the earth and another satellite B of mass 2mis at a distance 2r
(c) Its orbit will change from circular to elliptic from the earth's centre. Their time periods are in the ratio:
Cd) It will remain in the same circular orbit ICBSEI9931
55. A satellite revolves round the earth in an elliptic orbit. Its (a) \ , 2 (b) \ , 4
speed:
(e) 1 , \6 (d) \ , 2.J2
(a) Is the same at all points of the orbit
64. The relay satellite transmits the T.V. programme
(b) Is greatest when it i ~ closest to the earth
continuously from one part of the world to another because
(e) Is greatest when it is farthest from the earth its: ICPMT 19901
(d) Goes on increasing or decreasing continuously (a) Period is greatcrthan the period of rotation of the earth
depending upon the mass of the satellite about its axis
56. Which is the correct relation between escape velocity V, (b) Period is lesser than the period of rotation of the earth
and orbital velocity Vo ?
about its axis
(a) V, =2Vo (b) V, = (.J2)Vo (c) Period has no relation with the period of rotation of the
(e) Vo = (.J2)V, (d) Vo =2V. earth about its axis
(d) Period is equal to the period of rotation of the earth
57. If the speed of a satellite moving around a planet is
about its axis
increased by 41.4% it will:
65. The mean radius of the earth is R, its angular speed about its
(a) Move in an orbit of greater radius
own axis is (() and the acceleration due to gravity at earth
(b) Move in an orbit of lesser radius surface is g. The cube of radius of orbit ofa 'geostationary
(c) Fall into the planet satellite' will be: [CSSE 1992)
(d) Escape from the planet
(a) (R'g/Ol) (b) (R'Ol/g) 74. Consider two satellites A and B revolving around the earth
in circular orbits with radii R A and R B' Their periods TA
(c) (Rg/Ol') (d) (R'g/Ol') and TB are 8 hand 1 h respectively. The ratio (RA. IRB ) is
66. A satellite which is geostationary in a particular orbit is equal to:
taken to another orbit. Its distance from the centre of earth (a) (8)312 (b) 8
in the new orbit is 2 times that of the earlier orbit. The time
period in the second orbit is: (c) 4 (d) (8)112

(a) 48 hour (b) 48.J2hour 75. A satellite in a circular orbit around the earth has a KE E k'
What is the minimum amount of energy to be added so that
(c) 24 hour (d) 24.J2 hour
it escapes from the earth?
67. A satellite is revolving around the earth in a circular orbit (a) E,/4 (b) E,/2
whose radius is R; work done by gravitational force in one
(c) E, (d) 2E,
revolution is:
76. Two spherical stars each of mass M with their centres
(a) Zcro (b) 2mgR 2 distant D apart revolve under mutual gravitational
(c)mgR 2 (d) mgR attraction about the point midway between their centres.
The period of revolution will be:
68. Motion of artificial earth satellites around the earth ;,
powered by:
(a) Liquid fuel (b) Solar energy
(a) .~D3
GM
(b) 2.) D3
2GM

2f[~2D3
(c) Atomic energy (d) None of these
(c) (d) .fGM
69. A man inside an artificial satellite feels weightlessness
because the force of attraction due to earth is:
GM VV'
(a) Zero at the place 77. An earth satellite is moved from one stable circular orbit to
another higher circular orbit. Which one of the following
(b) Balanced by the force of attraction due to moon
quantities increases for the satellite as a result of this
(c) Equal to the centripetal force change?
Cd) Not effective due to the particular design of the (a) Gravitational force
satellite
(b) Linear orbital speed
70. In an earth satellite moving in a circular orbit, a piece of
(c) Gravitational potential energy
metarweighing 16 g (on the earth) is weighed by a spring
balance while the metal is suspended in water. If the (d) Centripetal acceleration
relative density of the metal is 8, what weight will be 78. A satellite rotating in the same sense as earth in an
recorded? equatorial orbit has a time period of 6 hours. At a certain
(a) - 2 g (b) Zero instant, it is directly overhead an observer on the equator of
earth. It is directly overhead the observer again after a time
(c) 2 g (d) 14 g
t. The correct value of tis:
7!. The time period ofa second's pendulum inside a satellite (b) 4.8 hr
(a) 8 hr
will be:
(c) 12 hr (d) 24 hr
(a) Zero (b) 1 sec
(c) 2 sec (d) Infinite IHint: t= /s~~ ,if rotating in same sense}
E S
72. A simple pendulum has a time period Tl when on the earth's
surface and T2 when taken to a height R above the earth's 79. The radius of the earth is 6400 km. At what depth from the
surface, where R is the radius of the earth. The value of surface of the earth is the period of simple pendulum one
T2 1TJ is: IIIT2001] per cent more than that on the surface of earth?
(a) 98 km (b) 126 km
(a) 1 (b) .fi (c) 490 km (d) 49 km
(c) 4 (d) 2
80. A particle is projected vertically upwards with a speed Vo
73. In order to find time the astronauts orbiting in an earth from the surface of the earth. The maximum height reached
satellite should use: by the particle is equal to radius R of the earth. If g be the
(a) A pendulum clock acceleration due to gravity on the surface ofthe earth, then:
(b) A watch having main spring to keep it going (a) vi
<2gR (b) V02 >2gR
(c) Either a watch or a pendulum clock
(c) vi " 2gR (d) Vi " gR
(d) Neither a watch nor a pendulum clock
458 PHYSICS FOR COMPETITIONS - Vol. I

81. At what distance from the centre of the moon is the point at
which the strength of the resultant field of earth's and
moon's gravitational fields equal to zero? The earth's mass
(a) r'
Gp
(b) J3.Gp

is 81 times that of moan and the distance between centres of


these planets is 60 R where R is the radius of earth:
(e) );p (d) imp
(a) 6R (b) 4R 89. A communication earth satellite which takes 24 hours to
(e) 3R (d) SR complete one circular orbit eventually has to be replaced by
82. A satellite is moving round the earth. In order to escape it another satellite which has twice the mass of the first. If the
the velocity of satellite must be increased by; new satellite is also to have an orbital period of 24 hours,
what is the ratio of radius of the new orbit to the radius of
(a) 20% (b) 41.4%
the original orbit?
(c) 1.41% Cd) Not possible
(a) 1: 1
83. Artificial satellites can often be seen as bright stars high in
(e) ../2: 1
the sky long after sunset. What must be the minimum
altitude of satellite moving above the earth's equator for it 90. Two satellites S I and S 2 describe circular orbits of radii r
to be visible directly overhead two hours after sunset? The and 2r respectively around a planet. If the orbital angular
radius of earth is 6400 km. velocity of S I is <q what is that of S 2?
(a) 990 km (b) 640 km (a) --"C.. (b) ro../2
(e) 320km (d) I320km 2../2 3
84. The radius of earth is 6400 km and g = 9.8 mlsec 2 . If the (e) -"'- (d) ",../2
body placed at the equator has to become weightless the ../2
earth should make one complete rotation in: 91. A space probe projected from the earth moves round the
(a) 12hour (b) IAhour moon and then settles down into a steady circular orbit
(c) 6 hour (d) 24 hour round the moon's surface at a distance equal to its radius.
The rest of its rocket launch orbits on a steady circular orbit
8S. At a height above the surface of the earth equal to the radius
around the earth at a distance equal to the earth's radius,
of the earth the value of' g' (acceleration due to gravity on
from its surface. The ratio of the times taken for one
the surface ofthe earth) will be nearly:
revohltion by the space probe and the rocket launch is
(a) Zem (b) ,fi

(e) g (d) f
(givenrm = ~ ,Mm = ~; )=
4 2
(a) 1: 1 (b) -J5: 2
86. Two stars of equal mass M rotate in a circular orbit of radius
R under the influence of their mutual gravitational (e) -/3: 2 (d) 2:-/3
attraction. Choose the correct statement (s): 92. Two small bodies initially both at rest and free to move
from a distance of I m from each other, are subjected to
(a) They will have the same speed equal to ~a; only their gravitational force of attraction. They approach
each other and collide and do not separate.!n respect of this
(b) They will collide at some instant collision which of the following statements is true?
(e) They will make oscillatory motion (a) The total gravitational potential energy of the two
(d) They will have the same speed equal to ~GM
masses has increased on collision
4R (b) The total gravitational potential energy of the two
87. If the value of the acceleration due to gravity at a height h masses has decreased on collision
above the surface of the earth is the same as that at a depth d (c) The law of conservation of energy does not hold good
below the surface of the earth ami if hand d are much (d) The force of gravitational attraction vanishes when the
smaller than the radius of the earth, R, then: bodies come in contact
(a) h=Zd (b) h = d 93. If the orbital velocity of the moon is increased by 41.4% its
(e) h =% (d) h =.Jd
present value;
(a) The moon will orbit the earth with double velocity
88. The orbital period of a satellite in a circular orbit of radius r (b) The length of the moon's orbit will become double
about a spherical planet of mass M and mean density p for a (c) The moon will become a stationary satellite
low altitude orbit (r'::::.rp )will be:
(d) The moon will leave its orbit and escape into space
GRAVITATION 459

94. A body is projected vertically upwards from the surface ofa 101. Six particles each of mass m are placed at the comers of a
planet of radius R with a velocity equal to half the escape regular hexagon of edge length a. If a point mass rno is
velocity for that planet. The !Tl8ximum height attained by placed at the centre of the hexagon, then the net
the body is: gravitational force on the point mass mo is:
(b) R I 3 2
(b) _6_G"m~m~o
(a) R I 2
(a) 6Gm
(e) R I 5 (d) R I 4 a 2
a2
95. A planet of mass M and radius r is surrounded by an (c) zero (d) None of these
atmosphere o f constant density consisting of a gas of molar
102. If R is the radius of a planet, • g' the acceleration due 10
mass ll. Determine the temperature T of the atmosphere on
gravity on its surface and G is the universal gravitational
the surface of the planet if the height ofthe atmosphere is II
constant, the mean density of the planet is:
and R is the gas constant (h« r):
[Kerala PMT 20021
Rr2 I,.t GMh
(a) "Mh (b) --p;T (a) 41tG 3ru1
(b) 4gG
3gR
~hr2 3g rulg
(e) GMR (c) 4nGR (d) 12G

96. The largest and the shortest d istances of the earth from the 103. The escape velocity ofa body on an imaginary planet which
sun are 71 and 1"2' Its distance from the sun when it is al the is thrice the radius of the earth and double the mass of the
perpendicular to the major axis of the orbit drawn from the earth is (vo is the escape velocity of earth):
sun, is: [Kerala PMT 20021
'i + 1"2
(a) -4- (b) ...J!L. (a) J2i3vo (b) .{0.v o
1"1 +1"2
(e) -li13v o (d) 2/-!3 Vo
2rJr2 r, +r2
(e) (d)
1", +r2 3 104. The ratio ..!..., where g and g h are the accelerations due to
g"
97. The acceleration due to gravity on the surface of the moon gravity at the surface ofthe earth and at a height h above the
is ! that on the surface of earth and the diameter of the earth' s surface respectively, is: (SEE (UPTU) 200S)
6
moon is one-fourth that of earth. The ratio of escape
velocities on earth and moon will be:
(a) (1+ ~)' (b) (1<)'
(a)
2
.J6 (e) 3 (d) -!3
2 (e) (~)' (d) (~)'
98. The minimum energy required to launch a m kg satellite lOS. If earth is supposed to be a sphere of radius R, if g 30 is
from earth's surface in a circular orbit at an altitude of2R, value of acceleration due to gravity at lattitude of300 and g
where R is the radius of earth will be: at the equator. The value of g -g30 is: (DCE 100S)
(a) 3mgR (b) ~ mgR (a) 1. ",' R (b) 1 ",' R
4 4
(e) 2mgR (d) 4mgR (c) olR
, ,
~

99. The gravitational force in a region is given by E =ayi +axj.


Find the work done by gravitational force to shift a point
106. The acceleration due to gravity becomes (1) (g '"

mass In from (0, 0, 0) to (x o ' Yo ' zo} acceleration due to gravity on the surface of the earth) at a
(a) maxoyozo (b) maxoyo height equal to: (Karnataka CET 20071
(c) - maxoyo (d) Zero (a) 4R (b) ~
100. The ratio of the radii of the planets P t and P2 is k. TIle ratio
of acceleration due to gravity is r. The ratio of the \:scape (0) 2R (d) R

m
velocities from them will be: 2

(b) .Jk7- (e) (d) M 107. How much deep inside the earth (radius R) should a man go,
so that his weight becomes one-fourth of that on earth's
surface? IUttarakhand 20071
460

±
113. In Q. 112, the variation of force with x can be shown as:
(a) B.4 (b) !!.

~L
2
(c) 3R (d) none of these (a) (b)
4
t 08. For a satellite moving in an orbit around the earth, the ratio
of kinetic energy to potential energy is: (JCECE 2007)

(a) 2

I
(b) l
2 (c) ~L----
_x_ (d) ~;h=-!-F
_x_--=--~
(c) - (d) -!2
-!2 114. A mass M is split into two parts M' and M - M ' which are
109. Two particles of masses m1 and m2 , m1 > m2 move in then kept at certain separation. The ratio ~ such that
circular paths under the action of their gravitational
gravitational force between the parts is maximum, is:
attraction, then:
(a) they move in the same circle (.) 2I 1
(b) 4"
(b) radius of circular path of ml is more than that of rn z
(c) radius of circular path of ml is less than that of m1 (c) (d) I
8
(d) each moves along circular path of variable radius
liS. Three particles, each of mass m, are kept at the comers of an
110. Two particles of masses ml and m2 , ml > m1 move in equilateral triangle ofside 'r. Force exerted by this system
circular paths under the action of their gravitational on another particle of mass mplaced at the midpoint of any
attraction. While doing so, their separation remains side will be:
constant and equals 'r. Radius of circular path ofm2 is:
5Gm 2
,
2 (b) ~r (a) T
(a) m,
5Gm 2
(d) 3/'

116. A spherical hollow is made in a lead sphere of radius R such


111. Four particles, each afmass m, are placed at the four comers that its surface touches the outside surface of the lead
of a square of side 'a'. Force exerted by this system on sphere and passes through its centre. Mass of lead spherl!
another particle of mass 'm' placed at the midpoint of a side before hollowing is M. A particle of mass 'm' is placed at 8
of square is: distance 'd' from the centre
2 of the lead sphere and on
(.) 16Gm (b) 16Gm' the line joining the centre of --~-- ------------~ "
sJ"Sa' sJia' the sphere and the centre of
d ·1
the hollow as shown. Force
16Gm 2 (d) Zero of a"raction between the
(c) Sa' Fig. 9.29
hallowed sphere and the

-----~~----x-----~'
112. Fig. 9.28 shows a ring of mass M particle is:

'+ ' I
and radius R. A point mass M' is
placed at a distance x from the GMm
centre of ring on its axis. (a) (b)
Ag.I.28
6(1+~r
2
4d'
Gravitational force between the d

I I
ring and the mass M' is:
(a) GMM'
x' (c) GMm (d) GMm[l+ I
1- I
8(1+ ~r
2
d·'
8(1-~)'
(c) GMM' GMM'
(d) ~~~ d
(R 2 +x2) (R2 +x2 )312
117, If the density of earth is made twice without changing its and when its kinetic energy becomes K' it escapes the
radius, acceleration due to gravity on the surface of earth gravitational pull of earth, then:
will be: (take the present value as g =9.8 mls 2) (a) K' =2K (b) K' = 3K
(a) 19,6 mI" (b) 9.8 mI" (e) K'=4K (d) K' = 1.5K
125. A satellite is moving in its orbit around earth with an orbital
(e) 4.9 mI" (d) 2.45 mI"
speed 'v'. The satellite is given additional kinetic energy
118. Three particles, each of mass M, are moving in a circle
and it escapes at a certain stage. When it is escaping from
under their mutual gravitational forces such that they
the orbit, its speed is then v'. Then:
always form an equilateral triangle of side I while rotating.
Speed of each particle is: (a) v'=2v (b) v'=3v
(e) v'=-!iv (d) v' = ,{jv
(a) F~M (b) p~M 126. Suppose the gravitational force varies inversely as the nth

(e) r:' (d) r:


119. In Q. 118, time period of motion of any particle is:
power of distance. Then, time period of a planet in circular
orbit of radius R around the sun will be proP9rtionai to:
~...... n+l
(a) R n (b) R '
(a) 2~ GM
a' (b) 2~ 3GM
a' (e) R '
,-I
(d) r '
127. Given that mass of earth is M and its radius is R. A body is

(e) 2~ 5~~ (d) 2~ 5~M dropped from a height equal to the radius of earth above the
surface of earth. When it reaches the ground, its speed will
be:
120. If the earth stops rotating about its axis, the acceleration due
to gravity will be most changed at:
(a) poles (b) equator
(a) ~~m (b) ~:
(c) a place of latitude 30 (d) a place of latitude 45°
F~m
Q

(e) (d) 2Gm


121. A fully dressed astronaut canjump to a maximum height 50 R
em on the surface of earth. If average density of moon is j 128. Two objects of masses M 1 and M 2 are initially at rest at a
separation 'r'. They move towards each other under their
that of the earth and radius of moon is ~ that of earth, gravitational force. Ratio of distances travelled by M I and
M 2 when they meet is:
maximum height through which the astronaut can jump on
the surface of moon is : (a)
(a) 100 cm (b) 50 em
(c) 200 em (d) 300 em
122. What should be the angular velocity of earth about own axis
so that a person's weight at equator will be ~ of his weight 129. Eccentricity of earth's orbit is 0.0167. Ratio of maximum
5 speed in its orbit to its minimum speed is:
at poles? (a) 2.507 (b) 1.0339
(a) H (b) ~ (e) 8.324 (d) 1.000
130. A tunnel is dug along a diameter of earth of mass Me and
(e) f2i
VSii (d) ~ radius Re' Force on a particle of mass 'm' placed in the
tunnel at a distance 'r from the centre is:
123. Escape velocity of a body on the surface of earth is 11.2 (a) GM em r (b) GM em
kmls. If the mass of earth increases to twice its present R3 , R3, r
value and radius of earth becomes half, escape velocity
becomes: (c) GM e mR ; (d) GM em r
(a) 5.6 kmI, (b) 11.2 kmI, r R',
(e) 22.4 kmI, (d) 44.8 kmls 131. Three particles, each of mass m, are placed at the corners of
124. A satellite is moving in its orbit around earth. Its kinetic an equilateral triangle of side' r. Energy is given to their
energy is K. The satellite is given additional kinetic energy system so that they are now at the comers of an equilateral
triangle of side 2/. Energy given to the system in this 138. An object of mass m is kept in a cavity inside a hollow
process is: sphere of mass M and radius R such that its distance from
the centre is 'Y. Gravitational force between the sphere and
Gm' (b) Gm' the mass m is:
(a) 2a a
(a) GMm (b) GMm
3Gm' 3Gm' ,.2 R'
(c) 4a (d) 2a
(c) zero (d) GMm
132. An object is shot vertically upwards from the surface of (R -,)'
earth with a speed nv e' where ve is the escape speed and
n <1. If R is the radius of earth, maximum distance of the 139. In some region of space, gravitational field is given by -; ,
object from the centre of earth will be:
rbeing the distance and C a constant. At a certain value of
(a) R(!-n') (b) R(I+n')
r=ro , gravitational potential is V'. General expression of
(c) - -
R gravitational potential as a function of r is:
l+n2 ~ ~ I
(a) C log (b) Clog--y+V
133. If an artificial satellite is moving in a circular orbit around 7
the earth with a speed equal to halfthe magnitude of escape
(c) Clog.!: (d) C log.!.. +V'
velocity from the surface of earth, height of the satellite '0 '0
above the surface of earth is:
140. Two identical spheres are placed in contact with each other.
(a) 2R (b) R If ris the radius of each sphere, then gravitational attraction
2 between the two is proportional to:
(c) R (d) R (a) ,.2 (b) 1'4
4
134. In Q. 133, if the satellite is stopped suddenly in its orbit and (c) (d)
r2 r4
allowed to fall freely onto the earth, the speed with which it

AUS
hits the surface of earth is: 14l. A unifonn thin rod of mass m and
length I is bent into a semicircle.
(assumeg=9.8 m/5 2 andR = 6400 km) Gravitational field intensity at the
(a) 4 km/s (b) 8 km/s centre 0 is: o
Fig. 9.30
(c) 2 km/s (d) 6 km/s (a) 21tGm along AGB
135. A projectile is fired with a speed v = 2jiR from the surface I'
of earth. It escapes the gravitational pull of earth. Its speed (b) 1tOm perpendicular to A OB
in interstellar space will be: /'
(a) .Jiii (b) .j3gR (c) 21tOm perpendicular to AGB
(c) .jgR12 (d) J'2gR /'

136. Radius of earth is R and acceleration due to gravity on its (d) 1tOm along AGB
/2
surface is g. Gravitational potential on the surface of earth
is: 142. Gravitational potential on the surface of earth is V.
Gravitational potential at the centre of earth is then:
(a) +gR (b) +.¥.
R (a) V (b) lv
2
(c) -gR (d)
(c) ~V (d) zero
3
137. Radius of earth being R, gravitational potential (V) and
acceleration due to gravity (g) at any distance 'y from the 143. Fig. 9.31 shows two concentric
centre of earth such that r > R, are related as: spherical shells of masses M I and M 2
and of radii R[ and R2, respectively.
V2 -dV
(a) g=7 (b) g=-d, Gravitational potential at a point at a
distance x from centre such that
(c) g =.jVI , R[ <x <R2 will be: Fig. 9.31
(d)
-GM 2
(aJ
R, (b) _G(MI+M,) 150. Two spheres of masses m and M are situated in air and the
RI x gravitational force between them is F. The space around the
masses is now filled with a liquid of specific gravity 3.
_G(M1+ M,) -G (MI M,) Gravitational force will now be:
- +
(c)
x RI (d)
R,-
X (a) F (b) 3F

144. In Q. 143, gravitational intensity at a distance x from the (c) F (d) F


3 9
centre such tbat x > R2 is:
151. A particle of mass 10 gm is kept on the surface ofa unifonn
(a) G( Mlx:M2 ) sphere of mass 100 kg and radius 10 em. Find the work to
be done against the gravitational force between them to take
the particle far away from the sphere:
(G=6.67x 10- 11 Nm 2/kg 2 )
(a) 6.67x 10-9 J (b) 6.67x 10- 10 J

0
145. A uniform thin rod of mass m and length I is bent to fonn a (e) 13.34x 10- 10 J Cd) 3.33x 10- 10 J
circle. Gravitational potential at the centre of circle is: 152. A spherical shell is cut into two pieces ~
-2nGm along a chord as shown in Fig. 9.32. For
(a) zero (b) - -
1 points P and Q: P
-Gm .G (a) Ip>IQ
(c) (d)
.1 ml (b) I p <lQ Fig. 9.3'
146. A projectile is fired with a speed less than the escape speed. (cJ Ip =I Q = 0
Its mechanical energy is: (d) Ip =IQ.O
(a) positive at all positions [Hint:I =Ip+IQ =0]
(b) negative at all positions
153. A solid sphere ofunifonn density
(c) positive at all positions except the highest point where and radius R applies a
it is zero A
gravitational force of attraction
(d) none of the above equal to F J on a particle placed at
147. Escape speed of a projectile that is fired vertically upward a distance 2R from the centre of
is Ve' Another projectile has to be fired at 600 with the the sphere. A spherical cavity of Fig. 9.33
horizontal. Escape speed for this projectile will be: radius (Rf2) is now made in the sphere as shown in Fig. 9.33.
V V The sphere with the cavity now applies a gravitational force
(a); (b) 2V, (c): (d) V, F2 on the same particle. The ratio (F2IFJ )is: [CBSE 1993J
148. A satellite is revolving around a planet in an elliptical orbit. (a) (112) (b) (3/4)
Its closest distance to the planet is R rnin and the farthest (c) (7/8) (d) (7/9)
distance Rmax' Its angular velocity when nearest to the 154. If the gravitational force between two objects were
planet is "" then angular velocity when it is farthest from the proportional to (l/R) (and not IIR2) whereR is the distance
planet is:
between them, then a particle in a circular path would have

(a) ( R:
R )112
co
its orbital speedvproportional to;
(aJ (I/R')
(c) R
(bJ (I/R)
(d) R O
[CBSE 1994}

R )112
(c) ~
( R~, co (d) ~~ (0 155. An artificial satellite moving in a circular orbit around the
~,
earth has a total (kinetic + potential) energy EO' Its potential
149. A planet of mass M is revolving around sun in an elliptical energy is: (lIT 1997)
orbit. If dA is the area swept in a time dt, angular (a) -Eo (b) I.5E o
momentum can be expressed as: (c) 2£0 (d) Eo
(a) MdA (b) M dA 156. A planet revolves around the sun in an elliptical orbit.lfv p
dt 2 dt and va are the velocities of the planet at the perigee and
(c) 1M dA apogee respectively, then the eccentricity of elliptical orbit
(d) 4M dA
dl dl is given by:
vp va -Vp 3
(b) (a) 41tpGmR
(a)
V, v" +Vp 3 x'
(c)
Vp +Vo
vp -va
(d)
vp -Va

tip +V"
(b) 4X PGmR 3[_1 -
3 x'
I
2(2<-R)'
1
4Gmp
[Hint: 2 . a (1+e)] (c)
3(R2 +x2 )3/2
va a(I-e)

157. A binary star system consists of two stars. One star has (d) None of the above
twice the mass of the other. The star rotates about their 162. A system of binary Slars of masses In A and mB are moving
conunon centre of mass: in circular orbits of radii rA and rB respectively. IfTA and
(a) Star having the smaller mass has twice angular TB are the time periods of masses m A and 1118 respectively,
momentum compared to heavier star then, (IlT2006(
(b) Both stars have same angular momentum about centre
of mass (a) TA
Ta
"(rA )312
rB
(c) Both stars have same linear speed
(d) The heavier slar has more kinetic energy (b) TA >TB (ifrA > 18 )
158. From a large metallic sphere of radius R and mass M, a (c) TA >TB (ifmA >m n )
smaller sphere is scooped out such that the spherical hollow (d) TA "Tn
thus fonnedju st touches the larger sphere's surface on one 163. Two satellites of earth, S I and S 2' are moving in the same
side and its centre on the other side. The gravitationel field orbit. The mass of S I is four times the mass of S 2 ' Which
at a distance rfrom the centre is: one of the following statements is true?

(a) -GM["'!"r2 - 2(2r-R


I )] (b) GM[L_ I]
r2 R2
(UPSEE 2007, eDSE PMT2007(
(a) The time period of S I is four times that of S 2

(c) GM[1_.!.]
r R
(d) GM[RR H]
-r
(b) The potential energies of earth and satellite' in the two
cases are equal
(e) S t and S 2 are moving with the same speed
159. Two masses m, and m2 arc initially at rest and are separated
by a very large distance. If the masses approach each other (d) The kinetic energies ofthe two satellites are equal
subsequently, due to gravitational attraction between them, 164. Two satellites A and B go around a planet P in circular
their relative velocity of approach at a separation distance orbits having radius 4R and R respectively. If the speed of
ofdis: satellite A is 3v, then the speed of satellite B will be:
(RPMT 2007(
(b)
(a) 6v (b) 9v
(c) lv (d) none of these
165. If gE and gM are the accelerations due to gravity on the
surfaces of the earth and the moon respectively and if
160. Four identical bodies, each of mass M, revolve in a circle of Millikan's oil drop experiment could be perfonncd on the
radius R under the influence of their mutual gravitational two surfaces, one will find the ratio

(a) ~GM
3R
(b) r
attraction. The tangential speed of anyone ofthem is:
M~4 +.J2
R 4.J2
electronic charge on the moon to be:
e lectronic charge on the earth
(AIEEE 2007(

r: ~2+.J2 r: ~.J2 +~
(a) gM/g, (b) I (c) 0 (d) g,/gM
166. The escape velocity of a projectile on the earth's surface is
(c) (d) 11 .2 km s - I. A body is projected out with thrice this speed.
The speed of the body far away from the earth will be:
161. A spherical mass of radius r ::: %is taken out from a unifonn (Pb PMT2007(
sphere of radius R and mass density p. Find the force which (a) 22.4 km s -I (b) 31.7kms- t
this sphere having a cavity will exert on a mass mplaced at a
(c) 33.6 km s-1 (d) none of these
distance of x from its centre x(x > R):
167. An asteroid of mass m is approaching earth, initially at a (a) 3.80x 10 8 m Cb) 3.37x 10' m
distance of 10 Re with speed vi' It hits the earth with a 4 2
(c) 7.60x 10 m Cd) 1.90x 10 m
speed v f (Re and Me are radius and mass of earth), then:
(AIIMS 2007) 173. What will happen to the weight of the body at the south
pole, if the earth stops rotating about its polar axis?
2
(a) vI = v~ + 2Gm (1-~) [Indraprastha CET 2008)
I M eR 10 '~"
(a) No change
2 =V·2 +
(b) VI - - ( 1+-
2GMe 1) (b) Increases
I Re 10
(c) Decreases but does not become zero
(c) V f2=v 2+
i
2GM, 1)
- - ( 1-- (d) Reduces to zero
Re 10 174. Two bodies of different masses of 2 kg and 4 kg moving
2 2 2Gm(
(d) v f =v i + - - 1-- 1) with velocities 2 mls and 10 mls towards each other due to
mutual gravitational attraction. What is the velocity of their
Re 10
centre of mass? IIndraprastha CET 2008J
168. The earth moves in an elliptical orbit with the sun S at one Ca) 5 mI, (b) 6 mI,
of foci as shown in the Fig. 9.34. Its rotational kinetic
(c) 8 mls (d) zero
energy is maximum at the point: (Haryana PMT 2007]
175. A planet in a distant solar system is 10 times more massive

Ea~~ ~)c
than the earth and its radius is 10 times smaller. Given that
the escape velocity from the earth is 11 kmlsec, escape
velocity from the surface of the planet would be:
__
(AIEEE 2008)
B
Fig. 9.34 (a) 1.1 kmlsec (b) 11 kmJsec
(c) 110 kmlsec Cd) 0.11 kmlsee
Ca) A Cb) B Ce) C Cd) D
169. If g is the acceleration due to gravity on earth's surface, the 176. The ratio of radii of earth to another planet is 2/3 and the
ratio of their mean densities is 4/5. If an astronaut can jump
gain of the potential energy of an object of mass m raised
to a maximum height of 1.5 m on the earth, with the same
from the surface of the earth to a height equal to the radius R
effort, the maximum height he can jump on the planet
of the earth is: (UPSEE 2008]
is: IKerala PET 2009)
Ca) 2mgR Cb) mgR
Ca) 1 m (b) 0.8 m Ce) 0.5 m Cd) 1,25 m
1 1
(c) '2mgR Cd) 'imgR Ce) 2m

170. An artificial satellite is moving in a circular orbit around the


177. If an object of mass m is taken from the surface of earth
(radius R) to a height 2R, then the work done is:
earth with a speed equal to half the magnitude of escape
velocity from the earth. The height ofthe satellite above the [Kerala PET 2009)
earth's surface will be: IUPSEE 2008) Ca) 2mgR Cb) mgR
Ca) 6000km Cb) 5800km 2 3
(c) '3mgR Cd) 2mgR
Ce) 7500km Cd) 6400km
171. Two satellites of earth, S 1 and S 2' are moving in the same Ce) 4mgR
orbit. The mass of S 1 is four times the mass of S 2' Which
one ofthe following statements is true? [JCECE 2008} 178. At what depth below the surface of earth, the value of' g' is
(a) The time period of S 1 is four times that of S 2 the same as that ataheightofS km? {KeralaPET 2009)
(b) The potential energies of earth and satellite in the two C') 1.25 km (b) 2.5 km
cases are equal Ce) 5 km Cd) 7.5 km
(c) S 1 and S 2 are moving with the same speed Ce) 10 km
(d) The kinetic energies of the two satellites are equal 179. A body is falling freely under gravity. The distances
24 covered by the body in first, second and third minute~ of its
172. The mass of the earth is 6.00x 10 kg and thatofthe moon
22 motion are in the ratio: IKerala PET 2009J
is 7.40 x 10 kg. The constant of gravitation
Ca)I:4:9 (b) 1:2:3
G = 6.67x 10- N-m 2/kg 2 . The potential energy of the
11
Ce) 1: 3 : 5 Cd) L 5: 6
system is -7.79x 10 28 1. The mean distance between the
earth and moon is: [JCECE 2008] Ce) 1: 5 : 13
180. Work done in taking a mass from one point to another in a 188. The escape velocity of 10 g body from the earth is 11.2
gravitational field depends on: IJ &K2009) Ions -:1 . Ignoring air resistance, the escape velocity of 10 kg
(a) the end points only ofthe iron ball froni the earth will be: (DCE 2009]
(b) the path followed (a) O.01l2 km s - I (b) 0.112 Ion s-I
(c) the velocity of the mass (c) IUkms - 1 (d) 0.56kn18- I .
(d) both the length of the path and the ~d points
189. A "lift is moving .upward· with increasing speed with
181. In planetary motion, the ql)antity that r~mains conserved is: acceleration Q. The apparent weight will be: IDeE 2009)
IJ&K2009) . (a) less than the actual weight
(a) radius of the orbit (b) speed along the orbit (b) more .than the actual weight and have a fixed value
(c) angular momentum (d) angular speed (c) more than the actual weight which increases as long as
182. A satellite moving round the earth in a circular orbit of velocity increases
radius r and speed v suddenly loses some .of its energy. (d) zero
Then: IJ&K2009)
190. The density of earth in terms of acceleration due to gravity
(a) rwill increase and v will decrease (g), radius of earth (R)and universal gravitational "onstant
(b) both r and v will decrease (G) is: (OrissaJEE 2009]
(c) both r and v will increase 41tRG 31tRG
(d) rwill decrease and v will increase (a) 3g . (b) 4g
183. A body is projected up from the surface of the earth with a
velocity equal to 3/4 th of its escape velocity. If R be the (c) ...Y.
3xRG
(d) ..JL
4xRG
radius ofearth, the height it reaches is: IJ&K2009)
191. The height at which the acceleration due to gravity becomes
(a) 3R11O (b) 9RJ7 gl9 (where g = the acceleration due to gravity on the
(c) 8R15 (d) 9R15 surface.ofthe earth) in terms of R, the radius of the earth is:
184. The effect of rotation of the earth on the value of IAIEEE 20091
acceleration due to gravity is: IUPSEE 2009]
(a) g is maximum at the equator and minimum at the poles (a) 2R (b) .!!. (c) Rl2 (d) .fi R
.fi
(b) g is minimum at the equator and maximum at the poles
192. The escape velocity from the earth is 11 km/s. The escape
(e) g is maximum at both places velocity from a planet having twice the radius and same
(d) g is minimum at both places mean density as that ofearth is: IUPSEE 2009]
185. Consider a satellite going around the earth in a circular orbit
at a heightof2R from the surface afthe earth, where R is the (a) .5.5 km/s (b) )I km/s
radius of the earth. What is the speed 'of the satellite?
(c) 22 km/s (d) none of these
., IUPSC 20091
193. A body of mass m is released from a height equal to the
(a) gRI3 (b) (gR/3)I/2 radius R of the earth. What will be the velocity ofthe body
(c) (gR/2)112 (d) gR/2 when it strikes the surface of the earth? IUPSC 2009]

t 86. Two planets of masses M 1 andM 2 have satelliteso'~~asses (a) Jiii (b) .j2gR
ml and m2 respectively, revolving around them at the same (c) 2.Jiii (d) .jgRI2
radius r. The period ofthe first satellite (of mass ml ) is twice
as-that of the second. Which one of the following relations
194. The displacement y of a particle is given by
I;' is correct?\ (UPSe 2009) Y=4Cos2(~)sin(IOOOt). This expression may be
(a) 4MI =M, (b) 2MI =M,
considered to be a result of the superposition of how many
(e) M, ='2M 2 (d) miMI =m2M 2 simple harmonic motions? IAMU 2009]
187. A geostationary satellite is orbiting the earth at a height of
(a) 2 (b) 3
6R above the surface of the earth, R being the radius of the
earth. What will be the time period of another satellite at a (c) 4 (d) 5
height 2.5R from the surface of the earth? (AMU 2009] 195. If a simple pendulum of length L has maximum angular
displacement 0: then the maximum kinetic energy of bob of
(a) 6J2 hours (b) 65.5 hours massMis: [DCE2009]
(c) if) hours (d) 12 hours
IML Mg 200. The gravitational potentials of two homogeneous spherical
(a) (b)
2 g 2L shells A and B o'f same surface density at their respective
MgLsina centres are in the ratio 3 : 4. If the two shells coalesce into
(e) MgL(l-cosa) (d) single one such that surface densitY remains same, then the
2
ratio of potential at an internal point of the new shell to shell
196. A planet revolves round the sun in an elliptical orbit. The A is equal to:
linear speed of the planet will be maximum at
(a) 3: 2 (b) 4: 3
[Karnataka CET 2009]
(e) 5: 3 (d) 5: 6

A€j~C
m
201. Four similar particles of
mass m are orbiting in a
circle of radius rin the same m
,
•••••••• m
angular direction because of
B their gravitational attractive
Fig. 9.35 force. Velocity of a particle m
(a) D (b) B (e) A (d) C is given by: Fig. 9.37

[Gt+~,/2)r
197. When a body is taken from poles to equator on the earth, its
weight: IDCE 2009) (a) (b) 3~G,m
(a) increases
(b) decreases
(e) remains the same
(e) Gm(l+2,/2)
, (d) [~G;(I+;Z)r
(d) increases at south pole and decreases at north pole 202. A tunnel is dug along the diameter of the earth (Rabius R
198. A body of mass 500 g is thrown upward with a velocity 20 and mass M). There is a particle of mass m at the centre of
mls and reaches back to the surface of a planet after 20 sec. the tunneL The minimum velocity given to the particle so
Then the weight of the body on that planet is:

r:
that it just reaches to the surface of the earth is :
(Orissa JEE 2009)
(a) 2 N (b) 4 N (e) 5 N (d) IN (a)
199. Three particles P,QandR are placed as per given Fig. 9.36.
Masses of P, Q and Rare .J3 m, .J3 m and m respectively.
The gravitational force on a fourth particle S of mass m is
(b) r:
equal to: (e) p~M
T··········~r(m)
. . ......... U (d) it will reach with the help of negligible velocity .

··
· ·../ \.... 203. Maximum height reached by a rocket fired perpendicular to
the surface of the earth with a speed equal to 50% of the
. .
,/ ..J3d'" . escape velocity from earth surface is:

,/ '\'" (a) Ii (b) 16R


// 2 9
P - 3 d - -d-R
a (e) Ii (d) R
(;13m) (;13m) (m) 3 8
Fig. 9.36 204. A point P lies on the ax.is of a fix.ed ring of mass M and
radius R,at a distance 2R from its centreQ. A small particle
(a) "'3G~J2 in ST direction only starts from P and reaches 0 under gravitational attraction
2d only. Its speed at 0 will be:
(b) F3G~2 in SQ direction and F3G~2 in SU direction (a) zero (b)~
2d 2d

(c) F3G~2
2d
in SQ direction only (e) p~M (.Js-l) (d) 2GM(I
R
_
_1)
.Js
2 205. In older times, people used to think that the Earth was fiat.
(d) -.!.-13",3G",m'
2 -.2.In SQ d'lrechon
. an d .J3Gm • ST d'lfectlOn
2 In .
2d 2d Imagine that the Earth is indeed not a sphere of radius R. but
an infinite plate of thickness H. What value of H is needed 207, A point mass m and a thin f ......----~--J,1;:=~M::;
to allow the same gravitational acceleration to be uniform rod of mass M and ;'-a- .;. J-----
experienced as on the surface ofthe actual Earth? (Assume length I are located along a : :
that the Earth's density is uniform and equal in the two straight line as shown in figure Fig. 9.39
models.) what is the potential energy of gravitational interaction 0
this system?
(a) 2R (b) 4R
l l (a) _G~mln(I +7)
(e) 8R (d) R
G~m( 1+ 7)
l l GMm
P (e) - (d) - (0+112)
206. If gravitational field due to uniform
thin hemispherical shell at point P is 208, A uniform ring of mass m and radius r is
I, then the magnitude of gravitational placed directly above a uniform sphere of
field at Q is (Mass of hemisphere is mass M and of equal radius. The centre of
M, radiusR) the ring is directly above the centre of the
2R
(a) GM _ / sphere at a distance rJ3 as shown in the
2R' a Fig. The gravitational force exerted by the
Fig. 9.38
sphere on the ring wilt be:
(b) GM +/
2R' (a) GMm (b) GMm
8r' 4r' Fig. 9.40
(e) GM_/
4R (e) ,fjGMm (d) GMm
8r' 8r' ,fj
(d) 2l_ GM
2R'

ANSWERS

L (b) 1. (a) 3, (d) 4, (d) 5. (e) 6, (d) 7. (a) 8, (e) 9. (a) 10. (a) 11. (e) ", (b)
13. (e) 14, (b) lS, (b) 16, (b) 11- (b) 18, (b) 19, (b) 10. (e) 11. (c) 11. (d) 13. (a) 24, (e)
15. (a) 26, (b) 17. (e) 18. (a) 29. (e) 30. (e) 3L (b) 31. (c) 33. (a) 34. (a) 35, (b) 36, (e)
37. (a) 38. (e) 39, (b) 40. (a) 4L (d) 42. (e) 43. (a) 44, (b) 45. (e) 46, (b) 47. (a) 48, (d)
49. (a) '0, (d) 'L (b) 52. (e) 53. (e) 54. (e) 55, (b) 56, (b) 57, (d) 58, (b) '9, (d) 60. (e)
61, (d) 62, (e) 63, (d) 64, (d) OS, (d) 66, (b) 67. (a) 68, (d) 69. (e) 70, (b) 7L (d) 72, (d)
73- (b) 74. (a) 75. (e) 76, (b) 77. (e) 78. (a) 79, (b) 80, (d) 81. (a) 82, (b) 83. (a) 84, (e)
85. (e) 86, (d) 87. (e) 88. (a) 89. (a) 90. (a) 9L (b) 92, (b) 93, (d) 94, (b) 9', (b) 96. (e)
97, (b) 98, (b) 99, (b) "0, (b) 101. (e) 101. (e) 103. (a) 104. (a) lOS, (b) 106, (b) 107. (e) 108, (b)
109. (e) 110 (d) Ill. (a) 112- (b) 113. (e) 114. (a) 115. (e) 116. (c) 117. (a) 118, (d) 119, (b) "0, (b)
12L (d) 112. (e) 113. (e) 114. (a) 125. (e) 126, (b) 117. (a) 12S. (e) 129, (b) 130. (a) 13L (d) 132- (d)
133. (e) 134. (b) 135, (d) 136. (e) 137, (b) 138. (e) 139, (d) 140, (b) 141. (e) 142, (b) 143, (d) 144. (a)
145. (b) 146, (b) 147. (d) 148. (b) 149. (e) 150. (a) lSL (b) lS2, (d) 153, (d) 154. (d) 155. (e) 156, (d)
IS7. (a) 158, (.) 159. (e) 160, (b) 161 . (b) 161. (d) 163. (c) 164. (a) 165. (b) 166, (b) 167. (e) 1611. (a)
169. (e) "0, (d) 171 . (c) 172. (a) 173. (a) 174. (d) 175. (c) 176, (b) 177. (e) 178. (e) 179. (e) 180. (a)
18t: (e) 182, (d) 183, (b) 184, (b) 185. (b) 186. (a) 187. (a) 188. (e) 189. (b) 190, (d) 191. (a) 192. (c)
193. (a) '.4, (b) 195. (e) 196. (e) 197. (b) 198. (d) 199. (e) 200. (e) 101. (a) 202. (a) 203. (e) 204. (11)
205. (a) 206. (a) 207. (b) 208. (e)

[D[ More than One Choice is COl'rect (b) (F1IF2 ) = (rzllj )z if fj > Rand rz > R
I. The magnitudes of the gravitational field at distances 'j and (c) (F1IFz )=(fjlrz )if'j >R andrz >R
rz from the centre ofa uniform sphere of radius R and mass
(d) (F\IFz)=(fjlrz)liffj <R and rz <R
MareF, andF2 respectively; then: (liT 19941
(Hint: See solved Problem II)
(a) (F\IF2 )=(fjlr2 )iffj <Randrz <R
2. Inside a unifonn spherical shell: 11. Consider an attractive force which is central but is
(a) Potential is zero (b) Field is zero inversely proportional to the first power of distance. If
such a particle is in circular orbit under such a force, which
(e) Potential is constant Cd) Field is constant
of the following statements are correct?
3. In case of earth:
(a) The speed is'directly proportional to the square root of
(a) Field is zero, both at centre and infinity orbital radius
(b) Potential is zero, both at centre and infinity (b) The speed is independent of radius
(e) Potential is same, both at centre and infinity but not (c) The period is independent of radius
zero
(d) The period is directly proportional to radius
(d) Potential is minimum at the centre
12. Which of the following statement (s) is/are correct about a
4. Which of the following statements are true about planet rotating around the sun in an elliptic orbi.t ?
acceleration due to gravity?
(a) Its mechanical energy constant
(a) 'g' decreases in moving away from the centre if r > R
(b) Its angular momentum about the sun is constant
(b) 'g' decreases in moving away from the centre if r < R
(c) Its areal velocity about the sun is constant
(e) 'g' is zero at the centre of earth
(d) Its time period is proportional to y3
(d) 'g' decreases if earth stops rotating on ils axis
13. A tunnel is dug along a chordofthe earth ata perpendicular
5. Ifa body is projected with speed lesser than escape velocity:
distance!i from the earth's centre. The wall of the tunnel
(a) The body can reach a certain height and may fall down 2
following a straight line path may be assumed to be frictionless. A particle is released
(b) The body can reach a c~in height and may fall down from one end of the tunnel. The pressing force by the
following a parabolic path particle on the wall and the acceleration of the particle
varies withx(distance of the particle from the centre of the
(c) The body may orbit the earth in a circular orbit
earth) according to:
(d) The body may orbit the earth in an elliptic orbit
Pressing Pressing
6. Which of the following statements are correct about a Force Force
planet rotating around the sun in an elliptic orbit?
(a) Its mechanical energy is constant
(b) Its angular momentum is constant
(,) \J
,,' ,,
,
(b)
(c) Its areal velocity is constant
x x __A x
A
(d) Its time period is proportional to r3 x·- x _ A x_A
2 2
7. An orbiting satellite will escape if:
acceleration acceleration
(a) Its speed is increased by 41 %
(b) Its speed in the orbit is made (ili) times of its initial
value (e) (d)
(c) Its kinetic energy is doubled
(d) It stops moving in the orbit
8. Which of the following are true for a satellite in an orbit?
(a) It is a freely falling body 14. A satellite revolves around a planet in circular orbit of
(b) Its speed is constant radius R (much larger than the radius of the planet) with a
time period of revolution T. Its velocity suddenly becomes
(c) Its suffers no acceleration
zero. (If the satellite experiences gravitational force due to
(d) It does not require energy for its motion in the orbit the planet only).
9. In case of an orbiting satellite if the radius of orbit is (a) It will fall into the planet
decreased:
(a) Its KE decreases (b) Its PE decreases (b) The time of fall of the satellite is nearly 5s
(c) Its ME decreases (d) Its speed decreases
(e) The time offall of the satellite into the planet is nearly
10. If two satellites of different masses are revolving in the .fiT
same orbit, they have the same:
8
(a) Angularmomentum (b) Energy
Cd) It cannot fall into the planet so time of fall of the
(c) Time period (d) Speed satellite is meaningless
15. AparticleofmassMisata M 4. (A), Astronauts in a satellite orbiting the earth feel
distance a from surface of a weightlessness.
thin spherical shell of (R) , Weightlessness does not depend upon the distance
uniform equal mass and of the satellite from the earth.
having radius a. 5. (A), Smaller the orbit of the planer around the sun,
a
(a) Gravitational field and I" shorter is the time it takes to complete one
potential both may be Fig. 9.41 revolution.
zero at centre of the (R) , According to Kepler's third law of planetary
shell motion, square of time period is proportional to
(b) Gravitational field is zero not only inside the shell but cube of mean distance from sun.
at a point outside the shell also 6. (A), If earth suddenly stops rotating about its axis, then
(e) Inside the shell, gravitational potential may be zero the value of acceleration due to gravity will become
(d) Neither gravitational field nor gravitational potential is same at all the places.
zero insid_e the shell for default references (R) , The value of acceleration due to gravity is
independent of rotation of earth.
ANSWERS
ANSWERS
I. (a) and (b); 2. (b) and (e); 3. (a) and (d); 4.(a)and(e);S.
aU; 6. (a), (b) and (e); 7. (a) and (e); 8. (a), (b) and (d); 9. 1. (d) 1. (.) 3. (d) 4. (h) S. (a) 6. (,)
(b)and(e); 10. (e) and (d); II. (b) and (e); 12. (a),(b) and
(e); 13. (b) and (e); 14. (a) and (e); IS. (e) and (d). (D) Integer Type Questions
[C] . Assertion-Reason Type Questions 1. A solid sphere of radius 'R' and centre at A
has a cavity of radius !i and centre at B as
(a) If both A and R are true and R is the correct 2
explanation of A. shown in figure. The solid part has a
(b) If both A and R are true but R is not correct unifonn charge density 'p' and cavity has
explanation of A. no charge. The electrical potential at point Fig. 9.42
2
(e) If A is true but R is false. , A' is xpr . Considering all are in SI units. Find the value
(d) If A is false but R is true. 12Eo
(e) Ifboth A and R are false. ofx?
I. (A): An object of mass m1 and another of mass m2 2. Using the natural reference for P.E. the Gravitational
Potential at the surface of a solid sphere is k times the value
(m 2 <m1 ) are released at certain separation. The
at the centre. Find the value of 6k.
objects move towards each other under the
gravitational force between them. In this motion, 3. The period of revolution of planet A around the sun is 8
centre of mass of their system will continuously times thatofB. The distance of A from the sun isxtimes that
move towards the heavier mass m1. of B from the sun. Find x.
(R) , In a system of a heavier and a lighter mass, centre of ANSWERS
mass lies closer to the heavier mass.
2. (A) , Consider a planet in an elliptic orbit around sun. Its 1. 5 1. 4 3. 4
kinetic energy changes with time and angular
momentum remains constant.
(E) Match the Columns
(R) , Gravitational force is a central force. As a result, no
torque acts on the planet about the position of sun 1. Let V and E denote the gravitational potential and
and so angular momentum remains conserved. gravitational field respectively at a poi,}t due to certain
unifonn mass distribution described in four different
3. (A) , Geostationary satellites may be set up in equatorial
situations of column-I. Assume the gravitational potential
plane in or~its of any radius more than earth's radius.
at infinity to be zero. The value of E and V are given in
(R) , Geostationery satellites have period of revolution of
coluIlUl-II. Match the following:
24 hrs.
GRAVITATION 471

Column -I (Cause) Column - II (Effect)


(a) At centre of thin (P) E=O I. a-+ p,T; b-+ p,r; e-+ q,T; d-+ p,r
spherical shell
2. a-+ q; b -+ p; c-+ r; d-+ q
(b) At centre ofsolid sphere (q) E.O
3. a-+ s; b-+ q; c-+ r; d-+ p
(e) A solid sphere has a (,) V.O
non-concentric
IF] Linked Comprehension Type
spherical cavity. At the
centre of the spherical Paragraph - 1
cavity. Supernova refers to the explosion of a massive star. The
(d) At centre of line joining (s) V=O material in the central core of such a star continues to
two point masses of collapse under its own gravitational pull. Ifmass of the core
equal magnitude is less than 1.4 times the mass of sun, its collapse finally
results in a white dwarf star. However, if the core has a
2. A satellite of mass m is orbiting the Earth at a height h from mass greater than this, it could end up as a neutron star and
its surface. [M is mass of the earth and its radius is R.] if its mass is more than about three solar masses, the
collapse may continue till the star becomes a very small.
object with an extremely high value of density called a
'Black Hole'. Escape speed for a black hole is very large .
, "'... ...
,
---
... .... ... ,
,,
,, ,,
,, ,
Fig. 9.43
(, ~.
~ 'Nl ~_~!___\
Column -I Column - II , ,,
\,
(a) Kinetic energy of the (P) GMm
--(R+h)
-- , Black hole
,,"
satellite '... ........ ... ....... ,..'"
Fig. 9.44
(b) Potential energy of the (q) I GMm
satellite - 2(R+h)
---- Fig. 9.44 shows a black hole of radius R. Another
concentric sphere of radius R s is also shown. In fact, it is
(e) Total energy of the (,) -GMm an imaginary sphere of radius R s ' called the
satellite 2(R +h) 'Schwarzschild Radius'. It is the critical radius at which
escape speed equals the speed of light c. Nothing, even the
(d) The energy must be (s) ,)(}Mm light, can escape from within the sphere of radius Rs' So
spent to pull the satellite (R+h)
light from a black hole cannot escape and hence the
out of Ihe earth's
terminology 'black hole'.
gravitational field.
There has been astronomical evidence of a small and
3. A particle is taken to a distance r(> R) from centre of the massive object at the centre of our galaxy, the 'Milky Way'.
earth R is radius of the earth. It is given velocity V which is Suppose that there is a ring of material of diameter about 6
~
perpendicular to r . With the given values of V in column 1 light years that orbits this massive object with an orbital
the resultant path of particle in column II. Here 'G' is the speed of about 2 x 105 mls.
universal gravitational constant and' M' is the mass of the Use the given data wherever necessary and answer the
earth. questions that follow.
Column -I Column -II G =6.67x 10- 11 N_m 2/kg 2
(a) V=~GMlr (P) Elliptical Solar mass M = 2x 1030 kg
c=3x 108 mls
(b) V=~2GMlr (q) Parabolic
. I light year ",9.Sx lOt S m
(e) V>J2GMlr (,) Hyperbolic 1. Using the standard relation for the escape speed for a
planet, 'Schwarzschild Radius' ofa black hole of mass M
(d) JGMlr <V<J2GMlr (s) Circular
can be expressed as
GM GM (a) _ GMm GMm
(a) (b) (b) x
e' 2c' (x2 + R2 )112

~
'lflM (0) _ GMm GMm
(e) (d) (d)
e' (x2 +R2 )3/ 2
2. Mass (in kg) of the massive object at the centre of the Milky 7. Force acting on the particle
Way galaxy is oithe order "d GMm
(b) 1037
()h
a as a magmtu e 2 2 312 and always acts
(a) 10" (x +R )
(e) 10
43 (d) 1029 towards the centre of ring
3. Theories suggest that it is not possible fOf a single star to have
(b) has a magnitude GMm and acts towards centre
a mass of more than 50 solar masses. The massive object at the (x2 +R2 )112
centre of Milky Way galaxy is most likely to be a when the particle is to the right of ring but acts away
(a) white dwarf (b) neutron star from centre when the particle is to the left of ring
(c) black hole Cd) single ordinary star GMmx
(c) has a magnitude and always acts
4. Ifmassofe~hME =:.6x1024 kg and itsradiusRE =6400 (x2 +R2 )312
km. to what fraction of its present radius does the earth need towards the centre
to be compressed in order to become a black hole? GMmx
(d) has a magnitude and always acts
(give only the order of your answer)
(a) 10-4 (b) 10- ' towards the centre
(0) 10- 1 (d) 10- 14 8. U being the potential energy of the system, the particle is in
equilibrium
5. Which of the following systems could have a density
2
comparable to the density of black hole? (a) atx=Oand d U > Oat this equilibrium
(a) Heaviest metal dx'
(b) Diamond ,
(b) atx=Oand d ~ < Oat this equilibrium
(e) Nucleus of copper atom dx
Cd) A substance made of atoms of highest atomic number 2
(c) atx = Oand d U = Oat this equilibrium
dx'
Paragraph - 2 2
In planetary and stellar astronomy, a ring of matter is a very (d) at x =±R and d U < Oatthis equilibrium
corrunon and familiar system, e.g., Saturn's rings. Consider dx'
a unifonn ring of mass M and radius R. A particle of mass 9. Maximum value of force experienced by the particle is
'm' is released at rest at a point on the axis of ring at a GMm 2GMm
distance 'x' from the centre, as in Fig. 9.45. (a) fi2 (b) 3.JiR'
M (0) 2GMm 2GMm
3R' (d) 3.J3R'
'Y
, m
10. Let R = Sm and the particle be released at a point such that
~- - - --------------.------·x
x I
x= !OOm
(a) motion of particle will be simple harmonic
Fig. 9.45 (b) motion of particle will be oscillatory but nol simple
ham.onic
Answer these questions. (c) the particle will travel along a straight line and will not
6. Assuming that potential energy 'U' is zero when the ring oscillate
and the partir-Ie are very far apart, say, for infinite (d) the particle will move along a line in a plane parallel to
separation between the two, gravitational potential energy the plane of the ring
of their system, when the particle is on the axis at distancex 11. When the particle, released at rest on the axis at distance x,
from the centre, will be passes the centre of ring, its kinetic energy has half the
magnitude of the potential energy of the system for x = 0.
Distance from the centre of ring at which the particle is orbit of star A is R. Assuming that dimensions of the stars
released and also its total energy at half this distance is are extremely small compared to their separation, answer
(a) .flR, _ G;;m (b) J2R,_G~m these questions.
16. Consider the centre of mass as the origin. At any instant,
~ ~

(c) R, - G;;m (d) 2R ,_GMm


R position vectors of stars A and Bare RA and RB,
~ ~ ~ ~

respectively. Dot product of RA and RB, i. e., RA . RB


Paragraph 3 M
will be
A satellite is revolving in an elliptical orbit around the
earth, earth being at any focus afthe ellipse. At perihelion, (a) R' (b) -R'
.4
which is that point in the satellite's orbit at which it is
closest to the earth, it is 1200 km above the surface of earth. (c)
R' (d)
R'
At aphelion, a point in the satellite's orbit at which it is most 2 4
distant from the earth, it is 6000 km above the earth's 17. Orbital speed of star B is
surface. Given GM E = 4 x 1014 N·m 2/kg
(G is the universal gravitational constant and ME the mass
(a) ~2G: (b) t::
of carth.)
Radius of earth = 6400 km.
Answer these questions.
(c) fg
4R (d) t:
18. Energy that is required to separate the two stars to infinity is
12. Time period of the satellite is nearly 2 2
(a) -GM ( bGM )-
(a) 11200sec (b) 4500sec 2R R
(c) 9900 sec (d) 5400 sec
(c) KiM' (d) GM'
13. Ratio of speed of satellite at perihelion to its speed at R 4R
aphelion is
(a) 1.92 (b) 1.45 Paragraph - 5
(c) 1.77 (d) 1.63 Consider now another binary system of stars X of mass
14. Speed of the satellite at aphelion is nearly M x and Y of mass My. Their masses aredifTerent and they
revolve about their centre of mass. Separation between the
( ) [12GM 4800 ] 112 mI stars is R. Orbital speed of star X is 48 kmls and its distance
a . E 12400x 7600 s from the centre of mass is four times the distance of star Y
(b) [GM E 12400x 7600]112 mls from the centre of mass. Again assuming the dimensions of
the stars to be much smaller than their separation, answer
(c) [ GM E ] 112 mls the following questions.
12400x 7600 19. Consider the centre of mass as the origin. At any instant,
~ ~

position vectors of X and Yare Rx and Ry ,respectively.


(d) [2GM 4800 ] 112 mls ~ ~
E 12400x 7600 Dot product ofRx and Ry will be
15. Which of the following is correct? (a) - 0.16R' (b) -0.32R'
(a) The satellite revolves in the elliptical orbit with a
(c) -0.5R' (d) 2R'
constant speed and torque acting on it is zero
20. Orbital speed of star Y is
(b) The satellite revolves in the elliptical orbit with a
variable speed and torque actinR on it is zero (a) 60 kmls (b) 4km1s
(c) The satellite revolves in the elliptical orbit with a (c) 12 kmls (d) 16km1s
variable speed and torque acting on it is non-zero 21. Orbital time period of star X can he expressed as
(d) The satellite revolves in the elliptical orbit with a 4 R 3/2 41tR 3/2
(a) • (b) 7#~~
constant speed and torque acting on it is non-zero
~5GMy ~5GMx
Paragraph - 4 21tR
3/2
(c)
Two identical stars A and B, each of mass M, form a binary
~GMx
system. They revolve in circular orbits about their centre of
mass under the mutual gravitational force. Radius of the
22. Orbital time period of star Y can be expressed as (a) r (h) 2r
2n:R 3/2 41tR 3/ 2 (e) 3r (d) 4r
(a) (h)
~5GMy . ~G.My 28. The maximum disance from the earth's centre during
subsequent motion of the particle is :
21tR 312
(e) -'F.-~= (a) r (h) ~r
~5GMx 3
7
Paragraph - 6 (e) 3r
Two satellites S I and S 2 revolve round a planet in coplanar
circular orbits in the same sense. Their periods of Paragraph - 8
revolution are 1 hand 8h respectively. The radius of the A satellite of mass 220 kg is orbiting in an approximately
orbit of S I is 104 km. When S 2 is closest to S I answer the circular orbit 640 Jon above the surface of the earth.
following questions Suppose the satellite losses mechanical energy at the
v, average rate of 1.4 x 10 5 J per orbital revolution. Adopting
reasonabl~ approximation that force due to abnospheric
Va resistance to be neglected, the trajectory is a 'circle of
slowly diminishing radius'. (mass of earth '" 6x 1024 kg,
Sa
radius of the earth'" 6400km).
Answer the following questions.
29. The initial orbital speed of the satellite is approximately.
Fig. 9.46
(a) 6.53 km/s (b) 7.53 km/s
23. The linear speed of satellite S 2 is (e) 8.53 km/s (d) 9.53 km/s
(a) 1tx104 kmhr - 1 (b) 21tx104 kmhr- 1 30. The altitude of the satellite from surface of the earth at the
(e) 31tx104 kmhr- I (d) 41tx10 4 kmhr- I end of its 1SOO tb revolution is :
24. The linear speed of S 2 relative to S 2 is (a) 212km (h) 312km
(a) 1tx104 kmhr- 1 (b) 21tx104 kmhr- i (e) 412 km (d) 512 km
(e) 31txI04 kmhr- i (d) 41tx104 kmhr - I 31. The time period of the satellite at the end of its 1SOOtb
25. The angular speed ofS 2 as observed by an astronaut inS J is revolution is approximately:
(a) 1.55 hr (h) 2.55 hr
(a) ~rad!hr (b) 2!:rad/hr
3 4 (e) 3.55 hr (d) 4.55 hr

(e) 2! rad/hr (d) ~ radJhr ANSWERS


5
Paragraph-l I. (d) 2. (h) 3. (c) 4. (h) •• (e)
Paragraph - 7
Paragraph-2 6. (a) 7. (c) S. (a) . •• (d) 10. (h) 11. (a)
A satellite is revolving round the earth in a circular orbit of
Paragraph-3 12. (e) 13. (d) 14. (a) 15. (h)
radius 'r and velocity 'vo'. A particle is projected from the
satellite in forward direction with relative velocity Paragraph-4 16. (h) 17. (c) IS. (d)
v=U 5/ 4 - I )vo · Paragraph-5 19. (a) 20. (e) 21. (a) 22. (e)
Considering all quantities in SI units, answer the following Paragrapb-6 23. (a) 24. (a) 25. (a)
questions.
26. The absolute velocity of particle at the instant of projection Paragraph-7 26. (a) 27. (a) 28. (h)
would be: Paragraph-8 29. (h) 30. (e) 31. (d)

(a) J!.v o IG) Subjective Type Questions

(c) ~. vo I. Gravitational force between point masses m and M


separated by a distance is F. Now if a point mass 2m is
placed next to 111, what will be the (a) force on M due to 111,
27. The minimum distance from the earth's centre during
(b) total force on M?
subsequent motion of the particle is :
[Ans. (a)F(b)3F}
2. Three unifonn spheres, each having mass mand radius r, are lADS. F = 2GIlm ]
kept in such a way that each touches the other two. Find the R
magnitude of the gravitational force on any sphere due to 10. A satellite of mass 80 kg is put into an orbit 650 km above
the other two. the surface of earth. The estimated vertical velocity after
[Ans. J3Gm 2/4?J launching is 2800 mlsec at this height. To put the satellite
3. Two masses m1 and m2 are initially at rest at infinite directly into this orbit what is the magnitude · and direction
of impulse that was applied at the implementation stage?
distance apart. They approach each other due to
[Ans. 6.428xlOSkgms~ l ;e=20018'1
gravitational interaction. Find their speed of ~pproach at
the instant when they are distance d apart. II. A ring of radius R is made of thin copper wire having a
[Ans. ~[2G(ml + m 2)/d] 1 radius r and density p. Find the force F with which the
ring attracts a material particle of ma~s m placed on the
[Hint: See solved Problem 53 in Chapter 6]
axis of the ring at a distance L. At what (listance (Lmax)
4. Three identical bodies of mass M are located at the vertices
of an equilateral triangle with side L. At what speed must from the centre of the ring will this force be maximum?
they move if they all revolve under the influence of one 2rr. 2Gmpr2RL r;;;
JAns.F = 2 23/2;L max =Rlv2)
another's gravitation in a circular orbit circumscribing the (R +L)
triangle while still preserving the equilateral triangle? 12. A particle of mass m is subjected to an attractive central
[ADS. v = .j(GMIL)) force of magnitude RI r2, R being a constant. If at the
[Hint: See solved Problem 20 in Chapter 7] instant, when the particle is at an extreme position in its
5. Aparticleofmassmissituated m.-x=;jl.dX" _ I closed orbit, at a distance a from the centre of force, its
at a distance d from one end ofa - d ':' L- - speed =~RI2ma, find the other extreme position.
rod of mass M and length L as Fig. 9.47 [Ans. a/3 from centre]
shown in Fig. 9.47. Find the magnitude of the gravitational
13. A satellite of mass M is put into an elliptical orbit around
force between them.
the earth. At the point A its distance from earth is hi = 500
[ADS. F=GMmld(L + d)] km and it has a velocity v I = 30000km/hr. Find the velocity
6. If a point mass m is at a distance .x from the centre of a of the satellite when it reaches a point B at a height h2 =
spherical shell of mass M and radius R(>x\ what is the 1200 km from earth.
force between them? [ADS. v 2 =27590 kmIhr]
[Ans. Zero] 14. Two point masses M and m are placed ata distance r. Show
[Hint: In case of spherical shell for an internal point, 1 = 0 so that at a point where the gravitational field due to them is
F=ml=O]
zero, the gravitational potential
. is: -Q
r [M + m- 2.JMm].
7. A uniform solid sphere of mass M and radius b is
surrounded symmetrically by a uniform spherical shell of 15. If the force of gravity acts on all bodies in proportion to
equal mass and radius 2h. Find the gravitational field at a their masses, why does not a heavy body fall
distance (a) (3/2)b (b) (5/2)b from the centre. correspondingly faster than a light body.
[Ans. (a) 4GM19b 2, (b) 8GMI25b 2] [Ans. As g = FlmJ
8. A particle of mass m moves under the action of a central 16. Compute the mass and density of earth assuming that
force whose potential is given by acceleration due to gravity on its surface is 9.8 m/s 2 and its
VV)=Kr', (K >0) radius is 6400 km (G = 6.67x 10-11 N~m 2 Jkg 2).

(i) For what energy and angular momentum will the [ADS. M=6xl024kgandp=5.5g1cc]
orbit be a circle of radius a about the origin? 17. If the radius of the earth shrinks by one per cent, its mass
(ii) What is the period of this circular motion? remaining the same, by what per cent will the acceleration
due to gravity on its surface change?
(iii) If the particle be slightly disturbed from this circular
motion, what will be the period of small radial [Ans. Will increase by 2%]
oscillations about r := a? 18. What is the acceleration due to gravity of earth at the
surface of moon if the distance between earth and moon is
2
lADS. (i) ma ..[3k;; (ii) 4; (iii) -A-J
...;3ka ...;15ka 3.8 x 10 5 km and radius of earth is 6.4x 10 3 km?
9. Calculate the gravitational force on a mass m located at a [Ans. 0.00278 mls 2 ]
distanceR from a rod of infinite length and linear density 11.
(Ans. (a) The planet will fall into the sun along the radius (See
Note: Acceleration due to gravity at the surface of moon is about
solved Problem 36.) (b) The planet will fly in
1.62 mls 2 [about (1I6)th of acceleration due to gravity at the inter-planetary space along the tangent to the orbit]
surface of earth].
27. A satellite is revolving around a planet in a circular orbit.
19. lfearth stops rotation about its own axis, what will be the What will happen if its speed is increased from Vo to (a)
change in acceleration due to gravity on its equator? The (N)v, (b) 2v,.
radius of earth is 6.37 x 106 m and its angular speed is [Ans. (a) Orbit will become elliptic (See Problem 31), (b) the
7.27x 10-5 radls. satellite will escape as v > v ~ {ve ;; (..[2)vo)]
[Ans. Will increase by 3.4 em/s 2] 28. Consider a satellite in a circular orbit around the earth. State
20. What is the potential energy ofa body of mass mrelative to how the following properties of the satellite depend on the
the surface of earth at a (a) height h =R above its surface? radius of the orbit r: (a) speed (b) period (c) angular
(b) depth d = R below its surface? momentum and (d) kinetic energy?
[Ans. (a)imgR,(b)-4mgR] [Ans. (a) v cc r-112. (b) T Ct:- ,:312, (e) L oc ,1/2, (d) KE Ct:-,-1]
29. If a satellite is revolving around a planet of density p with
21. Calculate the gravitational potential energy of a system of n
point masses each of mass m and separated from one period T, show that the entity pT 2 is a universal constant.
another by the same effective distance r. 30. A satellite of mass mis orbitingjust above the surface of the
IHlnt: As for!.)vo point massesU '" -(rn,mj'rij) earth. Derive expressions for the orbital speed and time
period of the satellite in tenns of acceleration due to gravity
So for the system
'g' and radius R ofthe orbit,
I ~
U=-LL-
2 i~l I'"
Gm,m j
'if [
as m~~j = m:.~il [Ans. Vo =..[iii and T = 21tJ(Rlg)]
'.I l'
31. (a) A satellite is revolving in an orbit close to the earth's
surface. Taking the radius of the earth as 6.4 x 106 m
and g = 10 mls 2 , find the value of the orbital velocity
and period of revolution of the satellite.
22. Treating the earth as a symmetrical sphere of radius R = (b) What is the relationship of this orbital speed to the
6400 Ian with field 9.8 Nlkg at its surface, calculate the velocity required to send a body from the earth's
vertical speed with which a rocket should be fired so as to surface into space, never to return? IMNR 1993]
reach ·3 height 4R from the surface. [Ans. (a) Vo ;; 8 km/s; T ;; 1.4 hour,
[ADS. 10 kmls] (b)v~ ~A -= 11.2 kmls]
23. (a) Compute the escape velocity from the surface of earth
if radius of earth is 6400 Ian and acc. due to gravity at 32. (a) What is the total energy ofan artificial satellite circling
round the earth in an orbit of radius R?
its surface is 10 mls 2 .
(b) What is the escape velocity for this satellite?
(b) Ifa particle is fired vertically upwards with a speed of
IS km/s, with what speed will it move in interstellar (c) Show that the satellite obeys Kepler's law.
space? [Ans. (a)E;; - (GMml2R).(b)ve =~(2GMIR)]
[Ans. (a) 11.2 kmls, (b) 10 kmls] 33. The astronaut in a satellite orbiting the earth feels
[Hlot: v'= ~v2 -v;J weightlessness. Does the weightlessness depend upon the
distance of the satellite from the earth? If so, how? Explain
24. If the earth be at one half its present distance from the sun, your answer.
how many days will there be in a year? lilT 19961 34. If the time period of a satellite Ts is different from that of
[ADS. 129 days] earth's rotation TE and the satellite is moving in the
25. Taking moon's period of revolution about the earth as 30 direction of earth's rotation, show that the time interval
days, calculate its distance from the earth. (0 = 6.7x 10- 11 between two successive appearances of the satellite
overhead is given by
N_m 2Ikg 2 and mass of earth = 6x 10 24 kg)
[Ans. 4.08 xlO s kmJ -1 =- 1 - - 1
T Ts TE
26. What will happen to an orbiting planet ifall ofa sudden (a)
What will happen to this time interval ifTS = TE?
it comes to stand still in the orbit (b) the gravitational force
ceases to act on it? [Ans. T;; co, i.e., the satellite will appear stationary overhead]
477

Note: If the satellite is moving opposite to the rotation of earth, independent of the orbital radius while the time period T
(liT) = (IITS) + (liTE)' would be proportional to r.
40. A missile is fired radially from the surface of the earth
35. A satellite of mass m moves in a circular orbit of radius r (radius 6.4 x 106 m) at a satellite orbiting the earth. The
around a planetofradiusR. Calculate the total energy required
satellite appears stationary vertically upward from the point
to put the satellite in the orbit assuming it initially to be at rest
where the missile is launched, Its distance from the centre
and acceleration due to gravity at the surface of planet 'g'.
of earth is 25.4 x 106 m. Will the missile actually hit the
[Ans. mgR[1-(RI2r)Jl
satellite? IRoorkee 1999)
36. A satellite of mass mmoves in an elliptic orbit around a planet
[Ans. No]
of mass M, so that its maximum and minimum distances from
the planet are 1j and '2 respectively. Find the angular 41. Fill in the blanks:
momentum L of this satellite relative to the centre of planet. (a) If R is the radius of earth andg the acceleration due to
[ADS. L::: m~(2GM'ir2/('i + '2)] 1 gravity, the mass of earth is .............. (EAMCET 19931
(b) If the earth stops rotating about its axis the value of 'g'
[Hint: Apply the laws of conservation of mechanical energy and
angular momentum to the satellite] at equator will increase by .......... ..
37. A satellite is put in an orbit just above the earth's [EAMCET 1992[
atmosphere with a velocity ili times the velocity for a (c) The numerical value of the angular velocity ofrotation
circular orbit at that height. The initial velocity imparted is should be .............. radls in order to make effective
horizontal. What would be the maximum distance of the acceleration due to gravity equal to zero at the equator
satellite from the earth, when it is in the orbit? of earth.
[ADS. 2R] (d) The escape velocity on earth is 11.2 kmls. Its value for
[Hint: See solved Problem 55 in Chapter 7] a planet having double the radius and 8 times the mass
38. A satellite of mass 100 kg is placed initially in a temporary of earth is .............. mls. [EAMCEr, 1991J
orbit 800 km above the surface of earth. The satellite is to be (e) The orbital period of revolution of an artificial satellite
placed now in a pennanent orbit at 2000 km above the revolving in a geostationary offiit is ......... .
surface of earth. Find the amount of work done to move the (f) A geostationary satellite is orbiting the earth at a height
satellite from the temporary to pennanent orbit. The radius 6R above the surface ofthe earth, where R is the radius
of the earth is 6400 km. of the earth. The time period of another satellite at a
[Ans. 4.06 xlOSJ] [Roorkee 1999) height of2.5R from the surface of the earth is ............. .
hours.
39. Assuming that the law of gravitation is of the fonn
(g) According to Kepler's second law the radius~vector to
F=C Mm a planet from the sun sweeps out equal area in equal
"
Show that the speed v of a particle orbiting in a circular
interval of time. The law is a consequence of
conservation of ............. .
orbit of radius ris proportional to r -{n-\)/2 while its period [Ans. (a) (gR 2/G)(b) Rro2 (e) 1.25 x 10-3 (d) 22.4 kmls (e) 1 day
of rotation T is proportional to r{n+ \)/2 . Hence show that if (t) 6./2 (g) Angular momentum]
. . I'10rce were to vary as -I , the speed v would be
gravltatlOna ,
Fluid Mechanics
~e substances which flow are called fluids. Fluids include RD = Density of body
both liquids and gases. The science of fluids at rest is called Density of water
fluid statics while that of moving fluids hydro-dynamics.
Fluid statics includes hydrostatic pressure,jloatation, Pascal's
(6) If tn,
mass of liquid of density PI and m2 mass of
density P2 are mixed, then as:
law and Archimedes principle while hydrodynamics includes
continuity equation and Bernoulli's principle and Torricel/i 's 111 ="', + m2 andY = (m, 1p,) + (m2/P2) [asV = mlp]
theorem. This all is the subject of this chapter. In m1 +"'2 Emi
p=-=
§ 10.1 Density and Pressure V (m,lp , ) + (m, l p,) ~(m,rp;)
While dealing with fluids we are more interested in 2P,P2 .
properties that vary from point to point in the extended If m, =m2 ,p= = Hannomc Mean
PI + P2
substance rather than properties of a small piece of the
substance. This is why we talk about density and pressure (7) If V, volume of liquid of density PI and V2 volume of
rather than mass aodforee in case of fluids. liquid of density P2 are mixed, then as:
(A) Density "'=p,V, +P2V2 and V =V, +V2 [asp=ml V)
In a fluid, at a point, density p is defined as: m PlY' +P2V2 IpjV"
p=-= =- -
V V, +V2 EVi
p= lim I1m=dm
l!.V-l'OAV dV ,. If V, =V2 =V,p = (p, +P2)12 =ArithmeticMean
Regarding density it is worth noting that: (8) With rise in temperature due to thennal expansion of a
(I) In case of homogeneous isotropic substance, it has no given body, volume will increase while mass will
directional properties, so is a scalar. remain unchanged, so density will decrease, i. e.,
(2) It has dimensions [ML -3] and SI unit kglm 3 while p (mlV) Vo Vo
CGS unit glee with Po =(mlVo) =-v= VoO+yt.9)
1 glcc=IO' kglm' [asV= Vo O+yt.9»)
(3) Density of sllbstance means the ratio of mass of
substance to the volume occupied by the substance or p= Po 0 po(l-yt.9)
(I +yt.9)
while density of a body means the ratio of mass of a
body to the volume of the body. So for a solid body (9) With increase in pressure due to decrease in volume,
Density of body = Density of substance density will increase, i.e.,

whi le for a hollow body, density ofbedy is lesser than p (mlV) Vo [asp = m)
that of substance [as V body > Vsub]. Po = (m!Vo ) =-V V
(4) When inuniscible liquids of different densities are But as by definition of bulk modulus
poured in a container, the liquid of highest density
will be at the bottom while that oflowest density at the B = -Yo 6p, j. e., V = Vo [l - AP]
t.v B
top and interfaces will be plane.
(5) Sometimes instead of density we use the tenn relative
density or specific gravity which is defined as: So p=po(I-~r oPOC+B<Ip )
(B) Pressure
Ifwe fill a vessel having a holewitb a fluid. fluid flows out
of the hole. Now if we cover this bole with a plate, which
exactly fits the hole; the plate can remain. at rest only if we
apply some external force on the plate. This shows that fluid
exerts force on the plate to push it outwards. If!u is the area of
the plate and M' is the Donnal force exerted by the fluid, the
pressure at the hole
(a) (bl
p==. lim M'=dF Auld In a body Body in a fluid
..,...., As ds
Fig. 10.3
Po
-.--------------- ---------------- (5) It depends on the depth of the point below the swface
--:~~~~~~fi
-----1--------
~~~p~~~~ (h), nature of liquid (p) and acceleration due to
--------
-------
-------
-------- ---------
--------
-------
-------- gravity (g) while it is independent of the amount of
:~~~~~~,~~~~~~~~ liquid, shape of the container or cross sectional area
------- ------ ---
----------------- considered. So if a given liquid is filled in vessels of
(0) (b) different shapes to same height, the pressure at the
Fig. 10.1 base in each vessel' s will. be the same, though the
For a point at depth h below the surface of a liquid density volume or weight of the liquid in different vessels will
P. hydrostatic pressure p is given by: be different.

~.
p=po+hpg
where Po is the atmospheric pressure. The pressure difference
between hydrostatic pressure and atmospheric pressure is
called gauge,:,pressure and will be
p- Po =hpg (a) (b) (e)
Regarding pressure it is worth noting that: Fig. 10.4
(I) Even though pressure is produced by a force that has p .. = PB = Pc but W.. < WB < We
directional properties and is a vector, the pressure (6) In a liquid at same level, the pressure will be same at
itself is a scalor as its direction (always nonnal to the all points; if not, due to pressure difference the liquid
area considered) is unique and not to be specified. cannot be at rest. This is why the height of Ii quid is the
(2) It has dimensions [ML-IT- 2 ] and SI unitN/m 2 which same in vessels of different shapes containing
is specifically called pascal (Pa). Other practical units different amounts of the same liquid at rest when they
of pressure are atmosphere, bar and torr (nun of Hg) are in communication with each other.·
with
I atm=1.0l x lO s Pa=I.Ol bar = 760 torr
(3) At a point it acts in all direclions. If a pressure
measuring device is at a given point in a fluid,

m
whatever be its orientation the pressure remains the
same [Fig. 10.2].

=====-======-========-===&===
P-::::::::::::::::===== i==
:::-
==~ ~===================== 11~
P

P;,Io"
Vacuum
Also. h... _ he
- he -
FIg. 10.5
h.

(7) In case ofa fluid, p = Po + hpg; so if Po is changed by


~~ffi~~~~:$~~~~ffi~~~~-:~~~
--- ------------
- ---- ----
--- -- ------ ---. p ----
--- ---
- - --
--
Pressure
Sensor
tlpo' then the increase in pressure at a given point
Ap = Apo + A(hpg) = APO (as hpg is constant for a
Fig. 10.2
given point), i.e., if an external pressure is applied to
an enclosed fluid it is transmitted undiminished to
(4) 11 always acts normal to the fluid boundaries as the every position of the fluid and to the walls of
ability to flow makes fluid unable to sustain a
container. This is called Pascal's law.
tangential force .

• The inability of a liquid to flow from a vessel having more liquid to a vessel having less quantity of liquid when both have same level of liquid is called
'hydrostatic paradox'.
(8) In case of a given mass of an ideal gas at constant the liquid and the bottle. So the force on areaSofthe bottle wi!
temperature be F = pS = (Sls)f. Now as area S ofthe surface of the bottl<
is much larger than the area ofcork s, F may be large enough te
Va:-I or pV = constant break the bottle.
p
Question IV. Explain why an air bubble in water rise.:
This law is called Boyle's law.
from bottom to top and grows ill size?
(9) Barometer is used to measure atmospheric pressure
Answer: We know that fluids move from higher pressun
while manometer measures pressure difference, i.e ..
to lower pressure and in a fluid pressure increases with depth
so pressure at top (= po) is lesser than at the botton
(po + hpg)and so the air bubble will move from bottom to tOI
(It cannot move sideways as the pressure at same level in I
fluid is same.). Furthermore in coming from bottom to top
pressure decreases, so in accordance with Boyle's law. i.e.
p V = constant, volume V will increase, i. e., the bubble wi!
grow in size.
(a) (b) Problem 1. (a) If a room has dimensions 3 m x 4m x 5"
Barometer Manometer what is the mass of air in the room ijdensity 0/ air at NTP i
po. hpg p-po=hpg 1.3 kg / m 3 ? (b) What force does water exert on the base 0/,
Fig. 10.6
house tOllk o/base area 1.5 m2 when it isfliledwith water up t,
Question [. A king orders/ora gold crown having a mass
0/2 kg. When it arrives from the goldsmith, the volume of the a height of l 111? (Dellsityofwater is 103 kglm3 alld g= IOmls 2
crown is found to be 200 em 3. What;s your opinion about the Solution: (a) As density is defined as the ratio of mass t,
crown If the density a/pure gold is 20 x 10 3 kg/m 3? volume, i.e., p = (m IV),
m=p x V = 1.3 x (3 x 4x 5) = 78kg
Answer: The density of the crown
i.e., mass of air in the room is 78 kg which is unbelievable (a
Mass 2 33
P = = =lOxlO kglm air is assumed to be weightless) but is true.
crown Volume 200 )( 10-6
3
(b) As P-PO =hpg=lxI0 x lO=IO" N/mz
which is just half that of pure gold (= 20 x 10 3 kg/m 3).
So something is wrong with the crown. F = ~px s= 10" x 1.5 = 1.5 x 10" N
Now as density = (mass/volume), there are two i. e., the force on the base of given water tank is 1.5 x 10"/10
possibilities: 1500 kg, i. e., weight of about 15 bags of sugar.
(a) The Cfown contains impurities of light metals , Problem 2. When equal volumes "'aftWo metals are mixe
decreasing its mass for a given volume and so the together, the specific gravity 0/ alloy is 4. When equal maSSE
density. o/the same two metals are mixed together, the specific graviJ
(b) The crown contains cavities increasing its volume for a/the alloy is 3. Calculate the specific gravity 0/ each metal.
a given mass and so decreasing the density. Solution: In case of two metals
Question II. Why are sleepers used below the rails? p=(m l + m2 )/(VI +V2 )
Explain.
So when equal volumes are mixed, VI = V2 = V an
Answer: By placing s leepers below the rails cross m l = VP I and m2 = VP 2 (as P = mlV]
sectional area is increased. This in tum reduces the pressure
due to the weight of the train on the rails as, = VPI + VP 2 PI +P 2 = p = 4
P V+V i.e., 2 .... (
-=c
Force
P res sure = - -,W
"-."i"gh:::',:o:.:f:.:tr",.,,,in
and when equal masses are mixed, m l = mz = m ar
Area Effective area
VI =(m/pl)andV2 =(m/p2)
Question III. A bottle/ull a/a liqUid isfltted with a tight
cork. Explain why a slight blow on the cork may be sufficient to So P= (m +m ) ,i.e., •••• (i
break the boule? [(m/p ,) + (m/P2)]
Answer: The blow that is given to the cork exerts a Solving Eqns. (i) and (ii) for P I and P2 we find th
pressure (= Jls)on the liquid. This pressure in accordance with specific gravities of metals are 2 and 6.
Pascal's law is communicated undiminished to each part of
A
Problem 3. A liquid ofdensity Now if hi is the height of water column above A', .th,en as
p is filled in a beaker of cross in a liquid, pressure is same at all points in the same level:
seclion S 10 a height H and then a PA ' =Pc' i.e., Po + hIP ,g =Po +~P2g,

'~s
cylinder of mass m and cross or hlP, =h 2P 2; i.e., 27.2xl=2yxI3.6
section $ is made to float in it as
which on solution gives y = 1 em, i.e., mercury rises by I em
shown in Fig. 10.7. If the
from its initial level.
atmospheric pressure is Po' find Fig. 10.7
the pressure (0) at the top face A of (b) The difference of level on two sides
'he cylinder (b) at the bottom/ace C ofthe cylinder and (c) at z=h,-hz =27,2-2x l=25.2c""
the base B of the beaker. Can ever these three pressures be i.e., the water level will stand 25.2 cm higher than the mercury
equal? level in the other limb.
Solution: (a) Above the cross section A there is external
Note: The answer does not depend on the atmospherie pressure Po'
pressure due to atmosphere only.
acceleration due to gravity g or the cross section of the tube iflhc
So PA = Atmospheric pressure =Po tube is unifonn.
(b) At the point C the pressure will be due to atmosphere
and also due to the weight oflhe cylinder. i,e., Problem 5. To what height should a cylindrical vessel be
filled with a homogeneou.~ liquid to make the force with which
the liquid presses on the side of the vessel equal to the force
exerted by the liquid on the bottom of the vessel?
Note: If h is the height of cylinder inside the liquid, by concept of Solution: Consider a
hydrostatic pressure Pc must be Po + hpg so that cylindrical vessel of radius r t
Po + (mgls)=PQ+ hpg, i.e., mg=hspg _ V'pg [asV' _hs] filled with a liquidofdensitypto h 1~~~~~~~l ld
i.e., weight of cylinder is equal 10 lhe thrust, which is the a height II. If Po is the II I _________ ___ t Y
principle a/floatation and verifies the correctness of our result. atmospheric pressure,
pressure difference inside and
the 1!====o:===!1
- r-
(c) At the point B of beaker, pressure will be due to outside the base of the cylinder, Fig. 10.9
atmosphere, weight of cylinder and weight of liquid in the
beaker; so !J.Pe =(po + hpg),- Po = hpg
mg+p,uHS mg F B = i!J.PB S = Ttr hpg
PB =- Po + <:> =-Po +pgH + - "
S S Now as pressure at a depth y below the surface of liquid
inside will be (po + ypg) while outside Po' so pressl\rc ·
Note: The pressure due to the weight of cylinder al its own base
C(mg/s) is more than at the base of beaker B(mgIS) as s < S.
difference on two sides of the curved surface at depth y below
the surface will be
[fthe system is in free fall (as in a satellite), g -+0, !J.ps =(po + ypg) - Po =ypg
PA =- Pc = PB = Po [as weight =OJ So the force on a strip of curved surface of thickneSs dy.al
Problem 4. An open V-tube of uniform cross-section depth y below the surface of the liquid as shown in Fig. 10.9
contains mercury. When 27.2 cm of water is poured into one will be:
limb 0/ the tube, (a) how high does the mercury rise in the dFC =!J.ps (2nrdy) =2nrpg)'dy
other limb from its initia/level? (b) what is the difference in
levels ofliquids o/the two sides? (Pw =- 1andp Hg = 13.6units)
Fc =21':rpgj~ ydy=Ttrpgh2 .... (ii)

SolutiOD! (a) If water But according to given problem FB' = Fc; so from Eqn!i.
depresses the mercury by y, the (i) and (ii),
mercury in the other limb will rise h,
I) l______ B' -1- i. e., .
by yabove its initial level (as fluids A :~ -----~ = B hz Problem 6. The density of air in atmosphere decreases
are incompressible), so that Y -
-- A' ------- -- - - with height and can be expressed by the relation:
C
AA'=BB' = y
p = poe- Ah
Also h2 = B'B +BC Fig. 10.8 "
i.e., h2 =B'B+AA' wheJ'epO is Jhe density at sea.level. A is a constant and h is.the
height. Calculate the atmospheric pressure at sea~/eveJ.
[as Be = AA']
Assume g to be constant. (g=-9.8 mls 2 , po = 1.3 kg/~3 and
or h, = 2y [as AA' = BB' = y]
A:;: 1.2 x 1O- 4Im )
Solution: Consider dh or h' x O.28 h - 0.038 = 0 [as Po = 0.76pgJ
two levels at heights h and --+---- P1 - P
h + dh in the atmosphere or h = ~[-0.28±~(0.28)' +4xO.038 J =0. 1 m
from sea-level. If PI and h
P2 are the pressures at § 10.2 Archimedes Principle and Buoyancy
these heights respectively. Accidentally Archimedes discovered that when a body is
Sea · Level
then Fig. 10.10 immersed partly or wholly in a fluid. it is buoyed up with a
P, - PI =pgdh force equal to the weight of the fluid displaced by the body.
Now as in the atmosphere with height density and so This principle is called Archimedes' principle and is a
pressure is decreasing, if PI = P. P2 == P - dp. necessary consequence of the laws of fluid statics.
so - dp = pgdh When a body is partly or
or -dp =poe-Ahgdh (asp = po e- Ah ] wholly dipped into a fluid, the
fluid exerts force on the body due
which on integration gives to hydrostatic pressure. At any
small portion of the surface of the
p=Pog e- Ah +c body, the force exerted by the
A
fluid is perpendicular to the
But as h....., 00, p-+O,soC =0.
surface and is equal to the pressure at that point multiplied by
pog - Ah the area, The resultant of all these constant force s is called
P=A e .... (i)
upthrust or buoyancy.
At sea-level h = 0, To determine the magnitude and direction of this force
consider a body immersed in a fluid of density a as shown in
So P=P = Pog = 1.3 x 9.8 =I.06 x 10 5 N/m 2 (.. )
o A 1.2x10-4 .... 11 Fig. 10.12. The forces on the vertical sides of the body will
cancel each other. The top surface ofthe body will experience
Note: In the light of Eqn. (ii), (i) can be written as: a downward force
p=poe- Ah with Po=~ =1.06X lO~N/m2
F J =Ap, := A(hJcrg + po) [asp = hcrg+Po]
while the lower face of the body will experience an upward
force
Problem 7. An open
end wide tube is immersed F, = Ap, = A(h,ag + po)
A
vertically in mercury in 0.48 m As h2 > h, . F 2 will be greater than F" so the body will
slich a way that a length p, O.OSm experience a net upward force
-
0.05 m extends above the r?o Po : : F =F, -FI =Acrg(h, - hi)
mercury level. The open ":=:-=_-=_t_~:cH:_":_":"_~
__~_;,_,,"_,:j-_;'__=-_=- If L is the vertical height of the body
end of the tube is then - - - -- -- --- --- --
----- --- F ~ AagL = Vag [as V = AL = A(", - hl )J
Closed and the tube is
raised further by 0.43 m. i.e.,F = Weight of fluid displaced by the body.
Calculate the length of the This force is called upthrust or buoyancy and acts
Fig. 10.11
air column above the vertically upwards (opposite to the weight of the body)
mercury level in the tube. through the centre of gravity of displaced fluid (called centre
of buoyancy). .
Solution:· Considering the air in the tube with cross·
section A when the tube is open: Though we have derived this result for a body full y submerged
in a fluid , it can be shown to hold good for partly submerged
p, = po bodies or a body in more than one fluid also,
and V, = A x 0.05
Regarding upthrust or buoyancy it is worth noting that:
Now when the tube is closed and 0.43 m ofit is withdrawn (1) It is independent of all factors of the body such as its
out of mercury, if II is the length of air column above mercury
mass, size, density, etc., except the volume of the
. and P2 is the pressure of air in it, then as at same level pressure
body inside the fluid , i.e.,
is same, Po = P2 =(0.48 - h)pg and V2 =Ah.
Til IX Yin (volume of body in the fluid]
Now assuming that temperature is constant,
.This is why two bodies of different masses, shapes
p,V, = P2V2 [Boyle's law} and sizes may experience same thrust when their
i.e., Po x A xO.05 = Ah[po -(0.48 -h)pgJ volumes inside a fluid are equal.
(2) It depends on the nature of fluid, i.e., wt. of body
= ;:::::-,;:-:::;-,::-::'::;::-
Th oc a loss in wt. in water
This is why upthrust on a fully submerged body is RD = wt. of body in air
i.e.,
more in sea water than in freshwater [a sea > a FW ] wt. in air - wt. in water
(3) It depends on acceleration due to gravity, i.e., So by weighing a body in air and in water we can
Thocg detennine the relative density of the body (See solved
So if a lift is accelerated downwards with (a < g) Problem 9).
Th=Vina(g-a) Note: Ifthe loss in weight of a body in water is a while in a liquid b,
and in free fall as a = g, Th = 0 Thw=Vawg=a and ThL=VaLg -=b
(4) Due to upthrust the weight of body will decrease, i. e., :!..k. '" '!!'.J..... =loss in wt. in liquid =!!. = Wair - Wliquid

Decrease in weight } {uPthrust} OW Thw loss in wt. in water a Wair - Wwater


{ of the body = (Wo - W ) = Vinag
L (b) The volume of cavity in a body: If the weight of a
= {weight of fluid diSPlaced} body in air and water are measured, then
bylhe body Th = loss in wt. = Vag
loss in wt. w;"ir - Wwater
So, Vol. of body V
ag ag
a
- - ---- while the volume of the material of body will be
v. = mass of body in air
o density of material
G b+Th
l==dB+b 'So the volume of cavity ifanywill be (V - Vo)(See solved
Problem 10).
lal Ib) lei Idl (e) The amount ofimpurity in a given metal : lfa solid
Fig. 10.13 Weight of a body and liquid under different
situations body of mass m contains impurity y of density P I and density
of pure metal is PM' the volume of impurity and pure metal
So if Wo is the true weight of a body, the weight in a will be
fluid (called apparent weight) will be (m- y)
with Th = Vinag PM
Note: (i) When a (sinking) solid is suspended from an independent So that total volume of the body
support in a liquid [Fig. 10.13 {c)J, the weight of liquid will
increase by an amount equal /0 the decrease in weight of V =V +VM
I
=..1:.. + (m - y) .... (i)
solid (i.e., thrust) as for every action there is equal and PI PM
opposite reaction.
But if WA and Ww are weight of body in air and water
(ii) If the solid (sinking or floating) of weight a is placed or
suspended in a liquid of weight b as shown in Fig. 10.13 (d), respectively,
the reading of balance will always be Th = Vag = loss in weight = WA - Ww
(a-Th)+ (b + Th) = a+ b.
SothevolumeofbodyV = (WA - Ww)/ag ... (ii)
(5) Using Archimedes' principle or concept of thrust, we Substituting the volume of V from Eqn. (ii) in (i), y,i.e.,
can detennine: amount of impurity can be detennined (See solved Problem 11).
(a) The RD of a body: By definition, Question V. A solid cylinder is placed in a container in
RD = density of body contact with the base. When liquid is poured into the
density of water . container, none of it goes beneath the solid, which remains
weight of body closely in contact with the base. Is there a bU(! yantforce ofthe
= --;-:-"=:..::.:.:=c,-----_ solid? Explain.
weight of equal vol. of water
Answer: No; there will be no buoyant force on the
i.e., RD = wt. of body
cylinder. Actually here the cylinder (apart from its weight and
water thrust
reaction) will experience a downward force F\ = (po + hag)A
due to hydrostatic pressure as shown in Fig. 10.14 (a).
484 PHYSICS FOR COMPETITIONS - Vol. I

but the weight of water is increased by the same amount, so


that the total weight carried by him remains unchanged.
Question IX. A beaker containing water is placed on the
pan of a balance which shows a reading of M g. A lump of
sugar ofmass m g and volume u is now suspended by a thread
(from an independent support) in such a way that it is
completely immersed in water without touching the beaker
(a) (b) (e) and without any overflow of water. How will the reading
Fig. 10.14 change as time passes on?
Note: (i) However, If liquid comes in between the lower surface of Answer: Initially the balance will show the weight of M
cylinder and the base of container, the cylinder will be plus thrust. But as
. buoyant by the force F = Vag.
(ii) If the cylinder fit~ at the base ofa well in a liquid as shown in Thrust=vcrg=~xlxg [ asv=;andcr=l]
Fig. 10.14 (c) it will experience an upward force
F) = [(po + hag)A - PoA 1= hO"gA

Question VI. Explain why a soft plastic bag weighs the


fIJ =( +;} =( +; +;}
M M .. (i)

same when empty or when filled with air at atmospheric Now as the sugar dissolves, the thrust and hence the
pressure? Would the weights be the same if measured in reading ofthe balance should decrease. However, as dissolved
vacuum? sugar comes in solution so the reading of the balance should
Answer: If the weight of empty bag is Wo and the volume increase. To find which of these effects predominate, consider
the situation when half the sugar has dissolved. In this
of bag is V, when the bag is filled with air of density p at NTP,
situation the thrust reduces to (mgI2p) from (mg/p) while
its weight will increase by Vpg. Now when the bag filled with
weight of solution increases by (m/2)g (due to sugar dissolved)
air is weighed in air, the thrust of air Vpg will decrease its
so the reading of the balance will become
weight; so
W = Wo +Vpg-Vpg=Wo Uil =(M +!!:+
2p 2
m)g ""Oi)
i.e., the weight of the bag remains unchanged when it is'filled
with air at NTP and weighed in air. Finally when all the sugar is dissolved thrust will become
However, if the bag is weighed in vacuum, its weight will zero and weight of solution will increase by mg, so the reading
be Wo when empty and (Wo + Vpg) when filled with air (as will become
there is no upthrust), i.e., in vacuum an air-filled bag will
weigh more than an empty bag.
Uin = (M + =( +; +;)g
m)g M ",,(iii)

Question VII. A bucket of water is suspended from a Now as p > 1, (mI2) > (mI2p), so comparing Eqns. 0), (ii)
spring balance. Does the reading ofbalance change (a) when and (iii), we find that the reading of the balance will gradually
a piece of stone suspended from a string is immersed in the increase till all the sugar dissolves in water and finally will
water without touching the bucket? (b) when a piece ofiron or become constant equal to (M + m)g.
cork is put in the water in the bucket? Problem 8. A balloon filled with hydrogen has a volume
Answer: (a) Yes; the reading of the balance will increase of 1m 3 and its mass is 1kg. What would be the volume of the
but the increase in weight will be equal to the loss in weight of block ofa very light material which it can just lift?
the stone (Vcrg) and not the weight of stone (Vcrg)[> Vcrg as
[Density ofmaterial of block is91.3 kg/m 3 and that ofair
p>cr].
is 1.3 kglm 3]
(b) Yes; the reading of the balance will increase but the
increase in weight will be equal to the weight of iron or cork Solution: For lifting Th> W. If V is the volume of balloon
piece. and v that of load and cr is the density of air, then the above
condition implies
'Q uestion VIII. A bOy is carrying afish in one hand and a
bucketfull ofwater in the other hand. He then places thefish in (V +vjcr Ag > (M + m)g
the bucket thinking that in accordance with Archimedes' or (V+v)crA>(M+vPL) [asm=vPL]
principle he is now carrying less weight as the weight of the VcrA. - M
fish will reduce due to upthrust. Is he right?
or v< -;--"----,
(PL -cr A )
Answer: No; when he places the fish in water in the
(lxl.3 - 1) 0.3 1 l
bucket, no doubt the weight offish is reduced due to upthrust, so
v""' = (91.3 1.3) =90=300 m
Problem 9. A certain block weighs 15 N in air. It weighs Problem 12. A rod of length 6 m has a mass 12 kg. It is
12 N when immersed in water. When immersed in another hinged at one end at a distance of 3 m below water surface. (a)
liquid, it weighs 13 N? Calculate the relative density olea) the What weight must be attached to the other end of the rod so
block (b) the other liquid. that 5 m ofthe rod are submerged? (b) Find the magnitude and
Solution: By definition, RD = pia water direction ofthe force exerted by the hinge on the rod. (Specific
gravity ofrod is 0.5).
(a)8 Olor 0 y - PB
' b d RD- --_ Weight of body Solution : As shown in
a w Weight of equal vol. ofwater Fig. 10.15, the forces acting --------~--~~~
RD = Weight of body = WA on the rod are:
i.e.,
Thrust from water WA Ww (I) The weight of rod
12g N acting down-
= _ 1_5 _ =5 wards through the
15 -12 CG of the rod, i.e., Fig. 10.15
(b) As Th = loss in weight of body = Vcrg at a distance of 3 m
.. 0L ThL 15-13 2 from the hinge.
So forhqUld RD= - =--= - - = -
, aw Thw 15 - 12 3 (2) Force of buoyancy through the CG of displaced liquid
vertically upwards. As
Problem 10. A piece of copper having an internal cavity RD = Weight of displaced water
weighs 264 g in air and 221 g in water. Find the volume ofthe
Force of buoyancy
cavity. Density of copper is 8.8 glee.
Solution: As Th = Vag, the volume ofhody will be ForceOfbUOyanCy= (~)x 12g =20gN
6 05
V = Th = (264-221)g = 43cc
og lxg and acts at a distance 2.5 m from the hinge.
(3) Extra weight w at the other end ofthe rod at a distance
Now as mass of body is 264 g while the density of material
6 m from 0 acting vertically downwards.
of body is 8.8 glee, the volume afmaterial in the bouy
(4) Reaction R at the hinge at Owill be vertical (as here all
V = mass of body = 264 = 30 cc . the forces are vertical, so for horizontal equilibrium of
o density of material 8.8
therodR H =0).
So the volume of cavity = V - Vo = 43 - 30 = 13cc So for translatory equilibrium of rod,
Problem 11. A piece of brass (alloy of copper and zinc) R + Th-W-w =O
weighs 12.9 g in air. When completely immersed in water it i.e. , w-R =20g-12g=8g .... (i)
weighs .11.3 g. What is the mass of copper contained in the
And for rotational equilibrium of rod (taking moments
alloy? Specific gravities of copper and zinc are 8.9 and 7.1
about 0)
respectively.
Solution: If the alloy contains m g of copper, the amount -12g x ¥sin8 +20gx ~sin8 - w6sin9 =0
of zinc in it will be (12.9 -m) g. So the volumes of copper and
zinc will be respectively 0' w = (l4/6)g =2.33g N =2.33 kg .... (ii)
- m Substituting the value ofwfrom Eqn~ (ii) in (i) and solving
Vcu - Yz-_(12.9-m)
- and " for R, we get
Peu n PZn
R = (2.33 -8)gN = -5.67 kg
and hence, the total volume of alloy,
Negative sign implies that R is directed vertically down-
V -V ,,_
- eu+Yz m + -",(I",2.9,...-,..:mC!.)
-- .... (i) wards .
n 8.9 7.1
§ 10.3 Floatation
Now as the weight of alloy in air and water is 12.9 g and
[AI Translatory-Equilibrium
11.3 g respectively,
When a body of density P B '" and volume V is immersed in
Th = Vag = loss in wt. = (12.9 -11.3) g
a liquid of density cr, the forces acting on the body are:
or V=1.6cc [ascr=lg1cc] .... (ii)
(1) The weight of body W = mg = Vp Bg acting vertically
Substituting V from Eqn. (ii) in (i) and solving for m, we downwards through the centre of gravity ofthe body.
get m = 7.61 g.
.. Remember that density of body p8 may be different from the density of material of body p.
(2) The upthrust Th = Vag acting vertically upwards However, if meta-centre goes below CG, the couple due to
through the centre of gravity of the displaced liquid, forcesatGandB' tends to topple the floating body. [Fig. 10.17 (c)]
i. e., centre of buoyancy.
So the following three situations are possible: Th
(a) The density a/body is greater than that a/liquid
(i.e., PB >0-). In this situation as weight will be
more than upthrust the body will sink. [Fig. 10.16
(a)] ===========.
-------
----- ----
-----_.
(b) The density of body is equal to the density of ==========.::.
=
liquid (i.e., PB = 0). In this situation W Th, so (a) (b)
Fig. 10.17
the body will float fully submerged in neutral eq-
uilibrium anywhere in the liquid. [Fig. 10.16 (b)] This is why a wooden log cannot be made to float vertical
in water or a boat is likely to capsize if the sitting passengers
-- --------
------ -- -
---------- ------- -- ---
--------
-- -
---------
--- stand on it. In these situations CG becomes higher than Me
---------
---------
--------- - ---------
----
-- --- -- -- and so the body will topple if slightly tilted.
------- ---
---------
-------- -- ------
-- ------
------
-------
--------- --- ------ ---- ------ ICJ Applications
----------
---------
---------- -- -----
---------
---------- --- =========-
=========-
------
----- ---
-----
--------- --- -----
-------
--- -- --- --
--- ------ ---
--------
--------- (1) Fraction of volume outside the liquid: As in case of
---------- ------ ---- --- -- ------- a floating body W = Th
i. e., VPBg=Vjn crg or Vjn = (Pa1a)V
Fig. 10.16
(c) The density of body is lesser than that of liquid
So V,", =V-V" =V[l-P:]
(i.e., PB < cr). In this situation W < Th, so the body
will move upwards and in equilibrium will float i.e., f out V
= out =
V
[1-~]
a [See solved Problem 13]
partially immersed in the liquid such that
W=Vincrg (2) Comparison of densities of floating bodies: As for
[Vin being the vol. of body in the liquid] floatingVPB =Vina,i.e.,

or VPBg = Vincrg (asW=mg=PBVg] Yin


P, =-0'=/
V m
a
or VPB = Vincr .... (i)
From this all it is clear that: So if different bodies are floating in the same liquid,

(1) . A body will/loat in a liquid only and only ifp B S cr. PBI (fin)}
(2)
. , .. "
And when a body is floating, weight of body is equal
-=--
Pa2 (fin)2
to the upthrust, i. e., Vp Bg = Vincrg.
(3) Comparison of densities of liquids: If the same
=
(3) In case of floating as W Th, the apparent weight of
body is made to float in different liquids, then
the floating body will be zero, i. e.,
VPB = (Vin)la\ =(Vin )2 a 2
Wapp =W-Th=O
cr 1 (Vjn)2
(4) In case of floating as W=Th implies VPBg=Vincrg, or - = --
the equilibrium of floating bodies is unaffected by 0'2 (Vin )!
variations in g though both thrust and weight depend i.e., density ofliquid is inversely proportional to the volume of
on 'g'. body inside it.
IB) Rotatory-Equilibrium (4) Weighing a body: Ifa platform of mass M and cross
When a floating body is slightly tilted from equilibrium section A is floating in a liquid of density a with its height h
position, the centre of buoyancy B shifts. The vertical line inside the liquid:
passing through the new centre of buoyancy B' and initial Mg = hAag .... (i)
vertical line meet at & point M called meta-centre. If the
Now if a body of mass m is placed on it and the platform
meta-centre M is above the centre of gravity the couple due to
forces at G (weight of body W) and at B' (upthrust) tends to sinks by ythen
bring the body back to its original position [Fig. 10.17 (b)]. So + m)g = (y+ h)Acrg
(M .... (ii)
for rotational equilibrium of floating body the meta-centre Subtracting Eqn. 0) from (ii),
must always be higher than the centre of gravity of the body. mg=Aayg, i.e., Wocy . ... (iii)
FLUID MECHANICS 487

So we can determine the weight of a body by placing it on So the volume of liquid displaced by the floating ice,
a floating platform and noting the depression of the platform in VD=(M l a L ) .... (i)
the liquid by it. Now if M g ice melts completely, water formed will have
Question X. Explain why a small iron needle sinks in mass M g (as mass is conserved). Now if 0' W is the density of
water while a large iron ship floats? water, the volume of water formed will be
Answer: For floatation, the density of body must be lesser VF =(Mla w ) .... (ii)
or equal to that of liquid. In case of iron needle, the density of
needle, i.e. , iron is more than that of water, so it will sink.
°' °'
Here the liquid is water, i. e., L = w; so water displaced
by floating ice is equal to water fonned by melting of whole ice
However, the density of a ship due to its large volume is lesser and hence the level of water will remain unchanged.
than that of water. so it will float. Furthermore :
Question XI. A block o/wood isfioQting on water arOce
(a) IfO'L >O'W' (M/O' L)«MIO'w), i.e., VD<VF ,
with a certain volume V outside the water-level. The
i. e., water displaced by floating ice will be lesser than water
temperature of water is slowly raised from O°C to 20OC. How
formed and so the level of liquid in the beaker will rise.
will the volume V change with rise in temperature?
Answer: As the block of wood is floating partially (b) IfO'L<O'W' (M /O'L»(M /O'w). i.e., VD>VF
submerged, W = Vincrg. Now as with rise in temperature, water i. e., water displaced by floating ice will be more than water
contracts from ODe to 4°C (anomalous expansion) and then formed and so the level of liquid in the beaker will fa ll .
expands, the density of water will increase from ODe to 4°e and Question XIV. A beaker exactly full o/water has an ice
then will decrease. So to provide proper upthrust (= mg) for piece floating in it. As the cube melts what happens to the
floating the volumeVin will decrease from O"C to 4°e and then water level if(a) the cube contains an air bubble (b) the cube
will increase, i. e., V (volume of body outside water) will
increase from O"C to 40C and then will decrease with furth er
contains (Oa lead piece and (ii) a cork p iece. ,
Answer: (a) If M g of ice containing an air bubble is
increase in temperature. floating in water,
Question XII. A ball floats on the slIIface of water in a M=VnO'w, i.e., Vn = (M/crw)
container exposed to atmosphere. wm the ball remain [as air bubble encloses air which is weightless]
immersed at its initial depth or will it sink or rise somewhat if
Now when M g ice melts, water formed will be M g and it
the container is shifted to moon.
will occupy volume VF = (M / 0' w)' As the volume of water
Answer: We know that gravity on moon is about ( l/6)/h formed by melting of ice is equal to the volume of water
that on the earth. However, gravity has no effect on the displaced by the floating ice, the level of water in the beaker
equilibrium of floating body (as weight and thrust both will will remain unchanged.
change by same factor). Furthermore, moon has no
(b) Let the mass of ice be M g and that of embedded piece
atmosphere while earth has. On earth the weight of the ball is
m; then for floating
balanced by upthrust from water and air, i.e.,
Weight = upthrust, i.e., (M + m)=VDO'w
m = VWO'W+VAO'A .... (i)
So the volume of water displaced. by the systenl when
while on moon m = VwO' w .... (ii)
floating
VAcr A
So that Vw =Vw +-- , i.e., Vw>Vw .... (i)
aw
i. e., the volume of the ball in water at moon will be more than
Now when ice melts water fonned will be M g and so the
on the eanh, i. e., the ball will sink a little more.
volume of water fonned by melting of ice
Note: Actually moon has no atmosphere, so water will boil off VI =(Mlo w )
instantaneously. (If ice is taken to moon it will sublimate!) (i) Ifthe ice cube initially contains the lead piece then as
Question XIII, A piece of ice is floating in water. What PPb>O'W> it will sink when ice melts and so the water
witl happen to the level ofwater when all ice melts? What will displaced by it will be equal to its own volume,
happen ifthe beaker isfilled not with water but with liquid (a) i.e., V2 = (ml pPb) (asp=m/ V]
denser than water (b) lighler than water? So finally total volume occupied by water fonned by
Answer: If M g ice is floating in a liquid of density 0' L' melting of ice and lead piece
then for its eqUilibrium M m
VI +V2 = - + - .... (ii)
weight of ice = thrust, i.e., Mg = VDO' Lg Ow PPb
4SS PHYSICS FOR COMPETITIONS - Vol. I
Now as PPb > (J w' this volume will be lesser than initial Now as the pieces are sinking p > 0' w' so this volume will
by water displaced by the floating system (Eqn. i); so the level be lesser than initial water displaced by the floating system
of water in beaker will go down as the ice me/Is if the ice [Eqn.(i)]; so the level of water in the pond will go down (or
cantains a 'sinking impurity. fall).
(ii) If the ice initially contains a cork piece, then as
Note: (i) George Gamow has mentioned in his book 'Physics:
P cork < cr W the cork will float on water when ice melts. So the
I
Foundations and Frontiers' that at a scientific meeting this
water displaced by the floating cork piece will be question was put to Dr. Gamow, physicist Oppenheimer and
m=Vitt",. i.e., Vi = (m /cr w ) Noble prize-winner Fe1ilt Bloch. All three of them, not
thinking 100 carefully, gave wrong answer!
[and not its own volume (ml pcork»). (ii) tn this problem if the pieces (either sinking or floating) are
So in this case water fonned by melting of M g of ice and unloaded on the ground, the water displaced after unloading,
water displaced by floating cork, V2 '" MI C1r, will be lesser than before unloading,
, V == (M + m)/C1r ; so the level of water in the pond will fall.
""I + V'2 =--+--
M m .... (iii)
O"w Ow Question XVI. A man is sitting in a boat which isfloating
a nd as this is same as initial volume displaced by the floating in a pond. If the man drinks some waler from the pond, what
system [Eqn. 0)], the level a/water in the beaker will remain will happen to the level of water in the pond?
unchanged. .' Answer: If the man drinks m g of water from the pond, the
Question XV. A boat containing some pieces a/material weight of (boat + man) system will increase by mg and so the
is floating in a pond. What will happen to the level ofwater in system will displace m g more water for floating. So due to
the pond if on unloading the pieces in the pond, the piece (a) removal of water from pond, the water level in pond will fall
floats (b) sinks? but due to water displaced by the floating system the water
level in the pond witt rise and so the water removed from the
Answer: If M is the mass of boat and m of pieces in it,
pond is equal to the water displaced by the system; the level of
then initially as the system is floating,
water in the pond will remain unchanged.
M +m=VDO'w Question XVII. Why does a uniform wooden stick or log
i.e., the system displaces water float horizontally? If enough iron is added to one end, it will
.... (i)
float vertically; explain this also.
Answer: When a wooden stick is made to float vertically.
When the pieces are dropped in the pond, the boat will still its rotational equilibrium will be unstable as its meta-centre
=
float, so it displaces water M Via W. i.e., VI (M /0 w). = will be lower than its CG and with a slight tilt it will rotate
under the action of the couple fonned by thrust and weight in
(a) Now if the unloaded pieces floats in the pond, the
the direction oftitt, till it becomes horizontal. [Fig. 10.18 (a)]
water displaced by them
~
m=V20'w. i.e., V2 = (mIO'w) Th
So the total water displaced by the boat and the floating
pieces ' G

VI +V2= M +~ .... (ii)


O'w O'w
which is same as the water displaced by the floating system
initially [£qn. (i)]; so the level ofwater in the pond will remain
unchanged. (a) (b)
(b) Now if the unloaded pieces sink the water displaced Fig. 10.18
by them will be equal to their own volume, i.e., However, due to loading at the botton., the CG of the stick
Vz =!!.! (or log) will be lowered and so may be lower than the
p meta-centre. In this situation the equilibrium will be stable and
if the stick (or log) is tilted, it will come back to its initial
and so in this situation the total volume of water displaced by
vertical position. [Fig. 10.18 (b)J
boat and sinking pieces will be

VI +Vz =( Ow
M +!!!.)
P
....(iii) Note: Same principle applies 10 Lactometer, Nicholson Hydrometer
and many toys which always float vertically in a liquid.
Problem 13. In English the phrase 'tip of the iceberg' is Af (l + y,AT)
used to mean a small visible fraction of something that is
so y= (l+y,6T) -1~(y, -y,)AT
mostly hidden. For a real iceberg what is this fraction if the
density a/sea water is 1.03 glee and that a/ice is O.92g/cc? Problem 16. A cubical block ofiron 5 em on each side is
floating on mercury in a vessel. (a) What is the height of the
Solution: In case of floatation, weight = upthrust, i.e.,
block above mercury level? (b) Water is poured into the vessel
mg=VinO'g, i,e., Vp=;:VinO" [asp = m/V] so that it just covers the iron block. What is the height a/water
or Vin=~V so Vout=V-Vin={I-:) column?
[RD of Hg = 13.6 ond Fe = 7.2]
Solution: (a) Incase of floatation W= Th, i.e., Vp =. Vincr;
or f = V"' =(1_£.)=(1_0.92)=0.11 so if h is the height of iron block above mercury and a is the
out V cr 1.03 1.03
side of iron cube,
=0.106 or 10.6 %
Problem 14. A large block
of ice 5 m thick has a vertical
hole drilled through it floating
in the middle of a lake. What is
the minimum length of a rope :::c:::!:
required to scoop up a bucket _________________ _ lal Ibl
full a/water through the hole?
---------------- - - - Fig. 10.20
Fig. 10.19
[RD of ice =0.9] (a x a x a)p=(a -h)xa x a xa
Solution: As ice is floating, W = Th, i. e., Vpg =. Vincrg. i. e., (o-h)=a£'
Now if A is the cross-section of the block, L its thickness and °
out of L, h is inside water, V =. AL and Vin = Ah.
so ALp=hAo, h={p/o)L =0.9x5 = 4.5 m
or h=+-~ )
i.e., 4.5 m of ice will be submerged in water. So the level of
so h=5(1- 7.2 )=2.35Cm
water in the hole will be5 - 4.5 = 0.5 m below the top ofice and 13.6
hence the length of rope required to scoop up water is 0.5 m.
(b) Here upthrust is provided by both mercury and water
Problem 15. A piece of metal floats on mercury. The and if h is the height of water-level,
coefficient of volume expansion of the metal and mercury are
Vpg = Th Hg + Thw
'Y 1 and'Y 2 respectively·lfthe temperature a/both mercury and
metal are increased by an amount I1T, by what factor the i.e., axaxaxp=axa(a- .h)cr Hg +axaxhcr w
fraction of the volume of the metal submerged in mercury i.e" ap=(a-h)crHg +hcr w
changes? or h(a Hg - crW)=a(cr Hg -p)
Solution: In case of floatation W = Th, i.e., Vp = Vina.
or h=a (crHg-p) =5x(l3.6-7.2)=2.54cm
Vin P (oHg - ow) (13.6 - 1)
so J.In =- =-
V a
.... (i)
Problem 17. A block of wood floats in water with
If with change in temperature new fraction is /i~' the two-thirds a/its volume submerged. In oil the blockfloats with
factor by which the fraction of submerged volume of metal 0.90 ofits volume submerged. Find the density 0/( a) wood and
will change will be (b) oil, if density a/water is 10 3kg/m 3,
11/ = /i~ - fin _ fi~ - 1 Solution: In case of floatation W =. Th, i.e., Vp =Vincr
f fin fin
so (a) Vp =(2/3)Vo w [as Vi' = (2/3) V]
which in the light ofEqn. (i) becomes
2 2 3 kg
11/ = p' x~ - l or p= - cr w = - xlO =667-
f p a'
33 m3
(b) For oil, Vp=0.9Vcr oil
Now as with rise in temperature density varies according
or 0.9Vo,tt =(2/3Vo w) [asVp=(2/3)Vo w]
to the relation
p' = p/(l + yAT) or
01
° .,2 2 3
= - - ow = - xlO =740-
3xO.9 2.7
kg
m3
[asp = mass]
Problem 18. A glass beaker having mass 390 g and an
interior volume of500cm 3 floats on water when it is less than or
vol.
halffilled with water. What is the density ofthe material ofthe
beaker?
or
Solution: As the beaker floats in :-:-:-l~v. 500 ccJt:::-:-:::-:
water when less than half filled with ::~:: ::=-~l!!e!~ :j::=:=:
water, it will float just fully submerged :-:-::-_-_-_-_-..:-_--=-_-:-:-:-: or
when halffiUed. In this situation, ~::~::~_~a~e~~:::::::::::::::
mass of beaker + mass of water in it Fig. 10.21
or m = 14x6:::' 6 .5 kg
= Vcr 2 13
i.e., 390+250=Vxl [as" = 1glee] Problem 21. A wooden stick of length L, radius Rand
i.e., outer volume ofbeaker density p has a smail metal piece of mass m (of negligible
V =640cc volume) attach~d to its one end. Find the minimum value for
Now as inner volume of beaker is given to be 500 ce, so the mass m (in terms ofgiven parameters) that would make the
the volume of the material of beaker =640 - 500 =140 ce. But stickfloat vertically in equilibrium in a liquid ofdensityes.
as mass of beaker is 390 g, so density of material of beaker Solution: For the stick to be vertical for rotational
m 390 g equilibrium, centre of gravity should be below in a vertical1ine
p=-=-=2.79- through the centre of buoyancy. For minimum m, the two will
V 140 cc
coincide. .
Note: Density of body (beaker) will be 390/640:: 0.61 glee! Let h be the length of inunersed portion. For translatory
equilibrium,
Problem 19. A cube 0/woodsupporting200g massjust
floats in water. When the mass is removed, the cube rises by 2 Wt. of rod + mass attached = force of buoyancy
cm. What is the size a/the cube? (M + m)g = 1tR' h"g .... (i)
Solution: Ifa is the side of the cube and as cube rises 2 cm
on removing the mass, the weight of body must be equal to the
thrust provided by 2 cm height of cube of base area (a x a), i. e.,
mg=V'crg or (200xg)=(2xa2)xlxg
or a=IO.!m, i.e., the side of cube is lOcm.
Problem 20. A block a/wood weighs 12 kg and has a
relative density 0.6. It is to be in water with 0.9 a/its volume
immersed. What weight ofa metal is needed (a) if the metal is
Fig. 10.23
on the top ofwood, (b) if the metal is attached below the wood?
(RDo! metal=14) where M = rcR 2 Lp.
Solution: (a) When the metal is on the top of wood, The height of centre of mass from bottom

M +m 1 =0.9Vwcr =0.9 M cr
Pw
[as Vw = M
Pw
1 =
(M)L/2+mxO
m+M
ML
=2(m+M)
m, this should be
=M[O.9~-ll=12[O.9
For rotatory equilibrium and for minimum
m1 -1]=6kg equal to hl2.
pw 0.6
m, h ML
.... (ii)
"2 = "'2(c-'-m"'+""M"")
mlvw PwM
---
-_-_.:..-
-----:..- M
---
-:'I}:'-
---.:.-
h= ML
(m+M)
::::PM:~m2-----:
---------------
--------------
---------------
.. .. :-~-:::--~ VM ::-:---:-::::-: Substihlting for h in Eqn. (i), we get
(a) (b) (M +m)g=1tR'"g' ML
Fig. 10.22 (m+M)

(b) When the metal is attached at the bottom of wood,


(M +m,)= (0.9Vw +VM)cr
(M + m) =~ M1tR 2crL = ~1CR2 Lp ·nR 20L deep, (i.e., large cross-section) the current will be
slower and so deep water will appear to be stilJ.
III = nR 2 L.{rJP _ 1tR 2Lp
(b) When water fa lls from a
tap", the velocity of
falling water under the
action of gravity will
§ 10.4 Hydrodynamics increase with distance
(A) Principle of Continuity: According to it in case of from the tap (i. e.,v2 >V 1).
steady flow· of incompressible and nOll-viscous fluid through SO in accordance with " j A,
a tube ofnon-uniform cross-section, the product a/the area 0/ continuity equation the
cross-section and the velocity afflow is same at every point in cross-section of the water
the tube, i.e., stream will decrease
(i.e.,A 2 <A I ), j.e., the FIg. 10.25
A )( v =constant
/alling stream 0/ water
To prove it, consider a liquid flowing through a tube of becomes narrower.
non-unifonn area of cross-section as shown in Fig. 10.24. Let
(B) Bernoulli's Theorem: According to it, in case 0/
AI and A2 be the areas of cross-section at the points Band C
steady flow o/incompressible and nOll-viscousfluid through a
respectively. If vI is the velocity ofliquid at E, the volume of
tube o/non-uniform cross-section, the sum o/thepressure, the
liquid flowing per sec through the tube at B will be A1v], So if
potential energy per unit volume and the kinetic energy per
PJ is the density of liquid atB, the mass of the liquid fl owing
unit volume ;s same at every point in the tube, i.e.,
per sec through the tube at B will be PI A1v l (as p ""mass/voL).
Simi larly mass of liquid flowing per sec across C will be p + pgh +! pv2 = constt.
P2A2v2' Now s ince no liquid can cross the boundary of the 2
tube and liquid is incompressible, the liquid entering the tube To prove it consider a liquid flowing steadily through a tube
per second at B must be equal to the liquid coming out per of non~unifonn area of cross-section as shown in Fig. 10.25. If
second at C, i. e., PI and P2 are the pressures at the two ends of the tube
p\A\v\ =P2A2v2
respectively, work done in pushing the volume 6V of
incompressible fluid from point B toC through the tube will be:
or A lv 1 = A 2v 2
W =p(!'.V)=(p, - p,)!'.V .... (i)
or Av=constt.
This work is used by the fluid in two ways:
(i) In changing the potential energy of mass 6m (in the
volume.1.V) from .1.mghl to 6mgh 2 • i.e.,
, !'.U = !'.mg(h, - I.,) .... (ii)
(ii) In changing the kinetic energy from

A-
lal Ibl
FIg. 10.24
M =2I .1.m(v '2 -VI)
, .... (iii)
This equation is known as equation of continuity and
represents the conservation of mass in case of moving fluids. c
From this equation it is clear that in case of moving fluid s:
A,
( 1) The velocity offlow is independent of the nature of
liquid (assuming the liquid to be non-viscous).
(2) The velocity of flow will increase if cross-section
decreases and vice-versa. This is why:
(a) At hills, w here the river is narrow and shallow
(i.e., small cross-section) the current \.,,'i11 be
faster, while in planes where the river is wide and FIg. 10.26

• lfthe velocity offluid particles at any point does not vary with time (though it may have different values at different positions). the flow is said to be steady,
Slreamlined or laminar .
•• Here the effect of SWfaCt7lellsioll due to whieh water stream will break up into droplets has not been taken into account.
Now as the fluid is by conservation of ---- - -- ---~
- - - p ._--- --- --_ .
non·yiSCQllS,
:~~~~~~~~~~~~~~~~~~~~~~:
mechanical energy. :~~~~~~~~~__ B, __ ~~~~~~~~?:

~JJJJJJJJJ~!;~~i~ii~fJJ-
W=AU +M
i. e., (Pt - P2)LiV =tJ.mg(h2 -hi) +~Lim(V~ -vr)

or =~~~~~~~~~~~~: .PO~~~~~~~~~~~=
P, - P, = pg(h, -h,l+4p(vi-viHasp =AmIAV]
Fig. 10.29
h 1 2 1 2 (d) Magnus effect: When a spinning ball is thrown, it
or PI +pg I +'2PVj =P2 +pgh2 +'2PV2 deviates from its usual path in flight. This effect is called
Magnus effect and plays an important role in telUlis, cricket
or p+pgh+tPV2 = constant
and soccer, etc., as by applying appropriate spin the moving
This equation is the Bernoulli's equation and represents ball can be made to curve in any desired direction.
conservation of mechanical energy in case of moving fluids. If a bali is moving from left to right and also spinning
With the help of this theorem we can explain: about a horizontal axis perpendicular to the direction of
(a) Drop of pressure when fluid moves from broader to motion as shown in Fig. 10.30 (a), then relative to the ball air
will be moving from right to left.
narrower horizontal pipe: According to continuity equation: V
(
Av = constant
so where the area is small
velocity will be large and V
vice·verso.
But by Bernoulli 's (
equation for a horizontal pipe V
(i.e., h = constant) (a) (b)

p + ~ pv2 =constant E V• tUl

p,
i.e., where the velocity is large
p,
pressure will be small. Hence,
when flu id flows from broader
tP
to narrower pipe its velocity :''''-It-...l
, ,
increases and so the pressure Fig. 10.27
decreases. This all is shown in Fig. to.27.
(e)
(b) Blowing off roofs by wind Wind ~ v large so Fig. 10.30
storms: During a tornado or hurricane, ~ p p< P
when a high speed wind blows over a - jp • The resultant velocity of air above the ball will be (V + no)
straw or tin roof, it creates a low 0 while below it (V - reo) [Fig. 10.30 (b)]. So in accordance with
pressure (p) in accordance with Bernoulli 's principle pressure a bove the ball will be less than
Bernoulli's principle. (Fig. 10.28) below it. Due to this difference of pressure an upward force
However, the pressure below the will act on the ball and hence the ball will deviate from its
roof (i. e., inside the room) is still usual path ~Ao and will hit the ground at Al following the path
Fig. 10.28
atmospheric (= Po). So due to this OA I [Fig. 10.30 (c)], i.e., if a ball is thrown with back-spin, the
difference of pressure the roofis lifted up and is then blown off pitch will curve less sharply prolonging the flight.
by the wind.

.
Similarly if the
(c) Attraction between two closely parallel moving spin is clockwise, i. e..
>-_____~A ,
boats (or buses): When two boats or buses move side by s ide the ball is thrown with
in the same direction, the water (or air) in the region between top-spin, the force due ~---~------~
them moves faster than that on the remote sides. Consequently
in accordance with Bernoulli's principle the pressure between
to pressure difference
w ill act in the direction
Horizontal plane
Fig. 10.31
""'
A,
them is reduced (Fig. 10.29) and hence due to pressure of gravity and so the
difference they are pulled towards each other creating the so pitch will curve more sharply shortening the flight.
called attraction.
FLUID MECHANICS 493
Furthennore. if the ball is spinning about a vertical axis,
the curving will be sideways as shown in (Fig. lO.3 1)
producing the so called Qui swing or in swing.
(2) Greater is the
distance of the hole
----------L
-- ------ -
-=========
-- ---- ----
_=======::
from the free surface _========= V,
(C) Application of Bernoulli's Principle: of liquid greater will -========:
-=========
v,
(a) Action of Atomlser: The action of aspirator, carburettor, be the velocity of .J-~=~=~=~=~=~=~=~=~=jiJ;;;;:~v:,'
paint-gun, scent-spray or \L.rfl p efflux (i.e.,voc.[h).
insect· sprayer is based on ri m This is why liquid Fig. 10.35
Bernoulli's principle. In gushout with maximum velocity from the orifice
all these by means of which is at maximum vertical distance from the free
motion of a piston P in' a Fig. 10.32 sUI/ace of the liquid.
cylinderC high speed air is (3) As the vertical velocity of liquid at the orifice is zero
passed over a tube T dipped in liquid L to be sprayed, High and it is at a height (H - h) from the base, the time
speed air creates low pressure over the tube due to which liquid taken by the liquid to reach the base-level.
(paint, scent, insecticide or petrol) rises in it and is then blown
off in very small droplets with expelled air. l=l(Hg-h)
(b) Working of Aeroplane: This is also based on
Bernoulli's principle. The lr1arge Now during this time
, PSf7)
win~softhe~eroplaneare ~ liquid is moving
haVing tapenng as shown -)~
in Fig. 10.33. Due to this ~ ~ )) : horizontally with
constant velocity v, .l.lliiii;i;
specific shape of wings ~ ,) so it will hit the base
when the aeroplane runs, - level at a hOl;zontai Fig. 10.36
v sman, p large
air passes at higher speed I 0
Fg.1.33
distance x (called range) as shown in Fig. 10.36 such
over it as compared to its that
lower surface. This difference of air speeds above and below x =VI = ~2gh x ~[2(H - h)/g] =2~h(H - h)
the wings, in accordance with Bemoulli's principle, creates a
pressure difference, due to which an upward force called From this expression it is clear that x will be
'dynamic lift' (= pressure difference x area of wing) acts on the maximum when x 2 is maximum
plane. If this force becomes greater than the weight of the
i.e., .!L(x')=O or
plane, the plane will rise up. dh

to height H and a hole is I


(c) Velocity of Efflux: Ifa liquid is filled in a vessel up

~""~v.~rgn
---- - ---
----------
-====h
====w= _==-
H/2'
-==== ----
-- -_.-
made at a depth h below
H
-----
==== ====
the free surface of the ---------
liquid as shown in Fig. ----------
-----~ - ---

---------
-=======.:::=
-------- --
10.34, then taking the
level of hole as reference :"'x
I Mox
= H-lI
level (i.e., zero point of Fig. 10.37
potential energy) and Fig. 10.34 or H-2h=O, i.e., h=H12
applying Bemoulli's principle to the liquid just inside and
outside the hole (assuming the liquid to be at rest inside), we
get
So that x~ =2J~[H-~]=H
i.e., range x will be maximum (=H) when It = H12.
(po +hpg)+O=po +~pv2
(4) If the level of free surface in a container is at heightH
or v=~2gh from the base and there are two holes at depth II and y
below the free surface, then
which is the same speed that an object would acquire in faJljng
from rest through a distance h and is called 'velocity ofefflux' x = z,jh(H- h) and x'=2~y(H - y)
or velocity of flow. From this expression it is clear that: Now ifx = x', i.e., h(H - h)=y(H-y)
(1) The speed of the liquid coming out of the orifice is
independent of the nature and quantity ofliquid in the
i.e., i -Hy+h(H -h)=O

container or the area of the orifice. or y=~[H±(H -2h)] ,


494

i.e., y = h or (H-h) Now as Gross lift = I1p x A


4
i. e., the range will be same if the orifice is ala depth h So Gross lift = 4.1 x 10 3 x (lOx 2) =8.2 x 10 N
or (H - h) below the free surface. Now as the distance Problem 23. A horizontal pipe line carries water in a
(H - h) from top means H - (H - h) = h from the streamline flow. At a point along the pipe where the cross·
bottom, so the range is same for liquid coming out of sectional area is IDem z, the water velocity is I m/s and the
holes at same distance be/ow the top and above the pressure is 2000 Pa. What is the pressure of water at another
bottom 10.38).
poim where the cross·sectional area is 5 em 2 ?
Solution: According to continuity equation,
1 1 A1v, =A 2v Z'
10
i.e., v l =5 x1=2m1s
Now according to Bernoulli' s equation,

~A~!B I z 1 1
PI +2:PVI =Pz +2:PV2
Fig. 10.38 Fig. 10.39
2000+1x 10J xl z =P2 + lx10 3 x2z
(5) If Ao is the area of orifice at a depth y below the free 2 2
surface and A that of container, the volume of liquid
which on solution gives Pz = 500 Pa.
coming out of the orifice per second wi ll be
Problem 24. Calculate the rate of flow of glycerine of
(dVldt)=vA o =Ao~2gy [asv=~2gy] density 1.25 x 10] kg/m 3 through the conical section ofa pipe.
Due to this, the level of liquid in the container will if the radii of its ends are 0.1 m and 0.04 m and the pressure
decrease and so if the level of liquid in the container drop across its length is 10Nlm z.
above the hole changes from y to y - ely in time I 10 Solution: According to continuity equation,
1+ dl then -dV =A dy.
So substituting this value ofdV in the above equation,
Vz _ AI _ 1t X (O. I)z _ 25 .... (;j
~ - Az - rex (0.04)2 -""4
-A- o
=.
dy =A ,,2gy, '.e., I dt=- - A r:;:
I Jy - 1/, dy
and according to Bernoulli's A, A
dt Ao ...;2g
equat;on for hor;zontal tube, E=~
So the time taken for the level to fall from H to H' Iz 1z \11 \12

I =_ ~_I_ IH' y-II'dy = ~ if[-fii - Jii'] PI +'2PVl =Pl +'2PVz P2

AO J2i H Ao Vg . z 1
I.e"vl - v l = 2(PI-Pz)/p
PI
Fig. 10.40
If the hole is at the bottom of the tank, time t taken to
emptied the tank:
i.e., v~ - vf =(2 x 10)/(1.25 x 10 3
) = 16 x 10-
3 .... (H)
Substituting the value ofvz from Eqn. (i) in (ii),
t =~ ~2H [as hereH' =0]
(6.25vl)z _ vf = 16 x 10- 3, i.e., vI:::' 0.0205 mls
AO g
Problem 22. Air is stream/ngpost a horizontal aeroplane So rate of flow through the tube
wing such that its speed is 120 mls over the upper surface and 4
R = A1v 1( = AZvl) = xx {O.l)2 x 0.02 =6.28 x 10- m 3/s
90 m/s at the lower surface. If the densiiy of air is 1.3 kg/m ],
find the difference in pressure between the top and bottom of Problem 25. A non·viscous liquid of constant density
the wing. Ifthe wing is IOm long and has an average width 2 m, 1000 kglm 3 flows in a streamline
calculate the gross lift of the wing. motion along a lube of variable o
Solution: According to Bernoulli ' s equation for a cross·section. The tube is kepi
horizontal plane, i. e., PE =constant, inclined in the vertical plane as
shown in tlte Fig. 10Al The area p 5m
1 z 1 Z
PI +2:PVI =Pz +2PVz, ofcross·section ofthe tube at two
points P and Q at heights of 2 _.l,_m_-=c-,.,--,,--_'-
metre andS metre are respectively Fig. 10.41
i.e., PI - Pz ='2I P(V 'z -vI') 4xlO- 3 m l and 8xlO - 3 mz .
So PI - Pz = !X I3X(I202-90Z)
The velocity ofthe liquid at point P is 1mls. Find the work done
per unit volume by the pressure and the gravity forces as the
=4.1x10 3 N/mZ fluid flows from point P to Q.
FLUID MECHANICS 495

Solution: As gravitational field is conservative, i,e., (b) As the velocity of effiux,


W = -(J
v= ~2gh =·hxlOx6~llrn1s
dW) dU mg(h, -hi)
so ( -dV g = - - =- =-pg(h, -hi) so assuming the level of water in the tank to be constant [(t. e.,
dV V
area = 00) as it is not given] the volume coming out per second
So work done by the force of gravity per unit volume will be
dV -223
( dW)
dV g
=-pg(h, -h,)= - IO' x 9.8(5-2) R=dj'=Aov=1t{2 x lO ) xllm /s

So the volume of the water flowing through the pipe in 3


= -2.94 x 10 4 _1_ .... (i) hour
m' V =R x I = 44x 3.l4x 10-4 x3 x (60x 60) ::11150m 3
Now in case of ideal fluid motion by conservation of mass,
i.e. , Problem 27. A cylindrical tank I m in radius rests on a
platform 5 m high. Initially the tank is filled with water upto a
( dm)
dt,
=(dm)
dt,
i.e, (PAv), = (pAv),
height of5 m. A plug whose area is 10-4 m 2 is removed from
an orifice on the side of the.tank at the bottom. Calculate
or Atv 1 =A Zv 2 [asp=constant(given)]
(a) initial speed with which the water flows from the orifice
so v, = A, 3
A,tl l = 4)(10- x l =.!rnls
8x 10-' 2
.... (ii) (b) initial speed with which the water strikes the ground and
(c) time taken to empty the tank to haIfits original value
Now as work done per unit volume by pressure, (d) Does the lime to emptied the tank depend upon the height of
dW) PdV P=(P, - p,)[asdW=PdVj stand?
( -dV p =--=
dV Solution: (a) As speed of effiux is given by
But by Bernoulli's theorem, v H =~(2gh) sohere v =,!2xl0 xF I0rnls
1 2 1 2 A
PI +pgh, +2"PV' =Pz +pghz + '2pt!2 -- -----_.
-------
--------.
-------_.
-------_.
dW) 1 , ,
-------
-------
so ( dV p =(p, - p,)=pg(h, -h')+2: p(v, -v , )

Which in the light of Eqns. (i) and (ii) yields

( dW ) =2.94xl0' +1 x 10' [(0.5) ' -1')=290251


dV p 2
Problem 26. The freshwater behind a reservoir dam is
15 m deep. A horizontal Fig. 10.43
pipe 4.0 em in diameter (b) As initial vertical velocity of water is zero. so its
passes through the dam 6.0 vertical velocity when it hits the ground
m below the waler surface 15 m
as shown in Fig. 10.42 A vv =~2gh=,!2x l0 xs=10rnls
plug secures the pipe
opening. (a) Find the So the initial speed with which water strikes the ground.
Fig. 10.42
friction force between the
plug and pipe wall. (b) The plug is removed. What volume of
v=~v1 +v~ =10J2 = 14.1m1s
water flows out of the pipe in 3.0 hour? (c) When the height of water level above the hole is y,
Solution: (a) As the plug secures the pipe opening the velocity of flow will be v =J2gy and so rate of flow
force of friction between plug and pipe wall
F=A(p, -PI)
": =Aov=Ao~2gy
But PI = Po and P2 = Po + hpg or -Ady=(~2gy)Aodt [as dV =-Ady)
so F=Ahpg
which on integration gives
496 PHYSICS FOR COMPETITIONS - Vol. I

punched 01/ the vertical side of the container at a height


I=.L f!.[Jii - $1
Ao Vg h(h< H n). Determine (i) the initia/speed ofefflux o/the liquid
at the hole (jj) the horizontal distance x travelled by the liquid
1=,,1' /2[.J5-J5!21=9.2XlO'S:2.5h initially and (iil) Ihe height hm al which the hole should be
so
10-4 1/10 punched so that the liquid travels the maximum distance xm
(d) No·, as expression of t is independent of height of initiaUy. Also calculate x m.
stand. Solution: (a) (i) As for floating, W =Th
Problem 28. A cylindrical vessel filled with water uplO a Vpg = V1d1g + V2 d 2g

)P =(~L)( :)d +( ~L)( ;)2d


height a/2m stands on a horizontal plane. The side wall ofthe
or {:
vessel has a plugged circular hole touching the bottom. Find
the minimum diameter of the hole $0 thaI the vessel begins to
3 2 5
move on the floor if the plug is removed. The coefficient of i. e., p =- d+-d=-d
444
friction between the bottom of the vessel and the plane is 0.4
and total mass of water plus vessel is 100 kg. (ii) Total pressure = Po + (weight of liquid + weight of
solid)/A
Solution: From Torricelli's theorem, velocity of efflux
=~2gh i.e., H H 5 x -x L xgx _1
P = Po +-dg+ - 2dg+ -d
2 2 4 5 A
(A)
Momentum per second carried by water stream
= density )( volume coming out per second x velocity i.• , P=Po +~Hdg+!Ldg=po +!(6H+L)dg
244
=pxavxv=pav 2 (b) (i) By Bernoulli's theorem for a point just inside and
Hence force on cylindrical vessel = pa2gh outside the hole
Cylinder starts to move when reaction force is just equal to 1 2 I 2
maximum force of friction. PI +2PVl = P2 +2PV2
i.e.,
or
~Mg = pa2gh

a = JlM = O.4xlOO =0.01 m 2


i.e., Po +~ +( ~ -
dg h) 2dg = Po +k(2d)V'
2ph 2xlO J x2
, or g()H - 4h) =2v'
7C
Area of circular hole = ~ = 0.01 m 2 or v = .J"(g""'/2)-::(3"'H;--
"' -""'4 h)
(ii) As at the hole vertical velocity of liquid is zero so time
or d=~0.0~X4 =O.ll3m taken by it to reach the ground,
1 =.[rjhii)
Problem 29. A container o/large uniform cross-section
area A resting on a horizontal surface, holds two immiscible,
non-viscous and incompressible liquids 0/ densities d and 2d
So Ihat x =vt = ~~(3H - 4h) x f! =~h(3H - 4h)

each a/height (HI2)as shown in Fig. 10.44. The lower density


(iii) For x to be maximum x 2 must be maximum, i. e.,
liquid is open to the atmosphere having pressure Po' (a) A
homogeneous solid cylinder .L(x') = O
of length L(L< H 12), cross- dh
sectional area (A /S) is
or .L()Hh - 4h')=O
immersed such that it floats dh
with its axis vertical at the 3H - Sh = O,
or
liquid-liquid interface with
i. e., h =(3/S)H
length (L/4) in the denser Fig. 10.44
liquid. Determine (i) the and X =pH ()H-~H)=J.H
density ofsolid and (il) Th e total pressure at the bottom ofthe mug 24
container. (b) Ih e cylinder is removed and original
arrangemelll is restored. A tiny hole of area s (s« A) is

• The height ofSland determines the distance lit which jet of water srrikes the ground.
EXERCISE
(AJ Only One Choice is Correct 9. The principle of operation of a Brahma's press is based on:
1. Equal masses of water and a liquid of density 2 are mixed (a) Boyle's law (b) Pascal's law
together; then the mixture has a density of: (c) Dalton's law (d) Newton's law
(,) 2/3 (b) 4/3 (c) 312 (d) 3 10. A pislon of cross·sectional area 100 cm 2 is used in a
2. Pressure at a point inside a liquid does 1I0t depend on: hydraulic press to exert a force of 10 7 dyne on the water.
(a) The depth ofthe point below the surface oflhe liquid The cross·sectional area ofthe other piston which supports
(b) The nature of the liquid an object having a mass 2000 kg is:
2
(c) The acceleration due to gravity at that point (a) 100cm (b) 109 em 2
(d) The shape of the containing vessel (c) 2x 104 em 2 (d) 2x 1010 cm 2
3. Two stretched membranes of area 2 em 2 and 3 em 2 are II. As a bubble comes from the bottom of a lake to the top, its
placed in a liquid at the same depth. The ratio of the radius:
pressure on them is: (a) Increases (b) Decreases
(,) 1, 1 (b) 2, 3 (c) Doesnotchange Cd) Becomes zero
(d) 2' ,3' IHlot: See Question IV]
4. A dam for water reservoir is built thicker at the bottom than 12. An inverted (bell) lying at the bottom ofa lake 47.6 m deep
at the lOP because: has 50 em) of air trapped in it. The bell is brought to the
(a) Pressure of water is very large at the bottom due to its surface of the iake. The volume of the trapped air will be
large depth (atmospheric pressure = 70 em of Hg and density of Hg =
13.6 glcm):
(b) Water is likely to have more density at the bottom duc
to its large depth (a) 350 em) (b) 300 em)
(c) Quantity of water at the bottom is large (c) 250 em) (d) 22em 3
5. A vessel contains liquid of density [Hint: (Po + hdg)Vo = poVor V "" Vo{l+ (hdI.IHdHg))]
p as 5hown in Fig. 10.45. The
13. When a body is wholly or partially immersed in a liquid it
gauge pressure al the point Pis:
appears to lose weight. This loss of weight is equal to the
(,) hpg
wcightof:
(b) Hpg
(a) Water displaced by the body
(c) (H -h)pg
Fig. 10.45 (b) Liquid displaced by the body
(d) (H-h)pgcos9 (c) Equal volume of water
6. A tank 5 m high is half filled with walerand then is filled to (d) Equal volume of liquid
the top with oil of density 0.85 g/cm) . The pressure at the
14. Two pieces of metal when immersed in a liquid have equal
bottom of the tank, due to these liquids, is:
upthrust on them; then:
(a) 1.85 glcm' (b) 89.25 glcm' (a) Both pieces must have equal weights
(c) 462.5 g/cm 2 (d) 500 glcm 2 (b) Both pieces must have equal densities
7. Pressure is applied to an enclosed fluid. It is: (c) Both pieces must have equal volumes
(a) Increased and applied to every part of the fluid (d) Both are floating to the same depth
(b) Diminished and transmitted to the walls of the 15. If there were a smaller gravitational effect, which of the
container following forces do you think would alter in some respect?
(c) Increased in proportion to the mass of the fluid and (a) Viscous force (b) Archimedes uplift
then transmitted
(c) Electrostatic force (d) Nuclear force
(d) Transmitted unchanged to every portion of the fluid
16. A body floats in a liquid contained
and the walls of container
in a beaker. The whole system as
8. Pressure applied to an enclosed fluid is transmitted shown in Fig. 10.46 falls freely
undiminished to every portion of the fluid and the walls of under gravity. The upthrust on the
the containing vessel. This law was first fonnulated by: body due to the liquid is: Fig. 10.46
(a) Bernoulli (b) Archimedes' IUPSEE 20091
(c) Boyle (d) Pascal
(a) Zero 25. A glass bulb is balanced by a brass weight in a sensitive
(b) Equal to the weight of the liquid displaced beam balance. Now the balance is covered by a bell-jar
whieh is then evacuated; then:
(e) Equal to the weight oftbe body in air
(a) The beam wjIJ remain horizontal
(d) Equal to the weight of the immersed ponioD of the
body (b) The pan containing th.e bulb will go down
17. When a body is weighed in a liquid, the loss in its weight (c) The pan containing the bulb will go up
depends upon: 26. A m~tal ball immersed in alcohol weighs WI at O°C and W2
(b) Mass of the body at 50°C. The coefficient of cubical expansion of the metal il
Ca) Volume of the body
less than that of alcohol, assuming that the density of th(
(e) Shape of the body. (d) CG of the body
metal is large compared to tbat of alcohol, it can be showr
18. Which of the following would a hydrogen balloon find that:
easier to lift?
(a) WI > Wz
(a) I kg of water
(b) W, =W,
(b) 1 kg ofstccl
(c) WI < W2
(e) 1 kg of light packed fcather
27. The reading of a spring balance when a block is suspendet
Cd) All of the above from it in air is 60 N. This reading is changed to 40 N whel
19. A hydrogen balloon released on the moon would: the block is submerged in water. The specific gravity oflh,
(a) Climb up with an acc. of9.8 mlsec 2 block must therefore be: [MNR 1991

(b) Climb up with an ace. of9.8 x 6 m/sec 2


(a) 3 (b) 2 (e) 6 (d) 3(2

(e) Neither climb nor fa ll 28. A beaker containing water is counter poised on a balance . •
2 a finger is immersed in water so that it does not touch tho
Cd) Fall with an acc. of(9.8J6) mlsec bollom or the side of the beaker, the pan on which tho
20. An iron ball is weighed in air and then in water by a spring beaker resls will:
balance: (a) Remain in the same position
(a) lIS weight in air is more than in water (b) Rise up
(b) Its weight in water is more than in air (c) Move down
(e) Its weight is same both in air and water

-
----~
-- -----
29. A vessel with water is placed on a
(d) Its weight is zero in water weighing pan and reads 600 g. Now
21. A body weighs 40 g in air. Ifits volume is 10 ce in water, it a ball of 40 g and density 0.80 glee ------- --- --
is sunk into the water with a pin as :-:-:::-:::-
- ---- : : ~. -~-
--
:~~-::
---
will wcigh: ----- -----
-- --- ---- -
(a) 30 g shown in Fig. 10.47, keeping it -:-
-
---------- -----
- ------::--: --:_-,:
- - -- --- ----
sunk. The weighing pan will show a
(b) 40 g Fig. 10.47
reading:
(e) 50 g
(a) 600 g (b) 550 g
(d) Data are insufficient to calculate
(e) 650 g (d) 632 g
22. A block ofmelal (density 7 glee) of size 5 emx 5 em x 5 em
30. A beaker of water kept in the left pan ofa common balanc
is weighed completely submerged in water. What will be ils
is counter poised with weights in the right pan. Now a bod
apparent weight (density of water = I glcc)?
o f mass 12 g and density 3 rice is suspended inside th
(a) (6x 5x 5x 5)g (b) (4 x 4x4 x7)g beaker from an independent support and the body
(e) (7x 5x 5x 5)g (d) (4x4x4x6)g completely immersed in the water without touching tt
23. A weightless rubber balloon has 100 g of water in it. Its sides of the beaker. To restore equi librium the weight to t
weight in water·will be: added in the right pan will be:
(a) 100g (b) 200 g (a) Zero (b) 8 g
(e) 50 g (d) Zero (e) 12g (d) 4g
24. A lishennan hooks an old log of wood of weight 12 Nand 31. A beaker containing water weighs tOO g. It is placed on t~
volume 1000 em 3. He pulls the log halfway out of water. pan of a balance and a piece of metal weighing 70 g ar
The tension in the string at this instant is: having a volume or' 10 em 3 is placed inside the water in t1
beaker. The weight of the beaker and the metal would be
(a) 12 N (b) 8N
(,) ION (d) 7 N (a) 170g (b) 160 g
(e) 100 g (d) 30 g
32. A boy carries a fish in one hand and a bucket afwater in the 41. A common hydrometer rcads specific gravity of liquids.
other hand; if he places the fish in the bucket. the weight Compared to the mark 1.60n the stem, the mark 1.5 wi ll be:
now carried by him: (a) Upwards
(a) is less than before (b) is more than before (b) Downwards
(e) is the same as before (d) depends upon his speed (e) In same place
33. A body of weight WI displaces an amount of water W2. (d) May be upwards or downwards depending on
When floating: hydrometer
(a)W\>W2 (b) WI < W2 42. A raft of wood (density 600 glm 3 ) of mass 120 kg floats in
(e) WI =W2 (d) anyof(a),(b)and(e) water. How much weight can be put on the raR to make it
34. When a ship floats on water: just sink?
(a) It displaces no water (a) 120 kg (b) 200 kg
(b) The mass of water displaced is equal to the mass o(lhe (e) 40 kg (d) 80 kg
ship 43. A boat 3 m long and 2 m wide is floating in a lake. When a
(e) The mass ofwaler displaced is lesser than the mass of man climbs over it. it sinks I em further into watcr. The
the ship mass of the man is:
(d) The mass of water displaced is greater than the mass of (a) 60 kg (b) 64 kg
the ship (e) 70 kg (d) 72 kg
35. An iceberg is floalingpartially immersed insea water. If the 44. It is easier to swim in sea water than in riverwatcr because:
density of sea water is 1.03 glee and that of ice is 0 .92 glee, (a) Sea is wider than the river
the fraction of the total volume of iceberg above the level of
sea water is:
(b) Sea is deeper than the river
(e) Density of sea water is greater than that of river
(a) 8% (b) 11 %
(d) Sea is having high waves all the time
(c) 34% (d) 89%
45. When a loaded boat enters into the sea from a river, it rises
36. For a body fl oating in water the apparent weight is cqual1o:
because: INCERT 19911
(a) Actual weight of the body
(a) Therc is more water in sea than in river
(b) Zero
(b) Sea water is denser than river
(e) Weight of the body minus weight ofthe liquid
(e) There is difference of tempcrature
(d) None of the above
(d) Sea is deeper than river
37. A wooden cylinder floats vcrtieally in water with half of its
46. When a piece ofice floating in a beaker of water completcly
length immersed. The density of wood is:
melts, the level of watcr in the beaker:
(a) Equal to that of water
(a) Rises
(b) Half the density ofwater
(b) Falls
(c) Double the dcnsity of water
(c) Remains unchanged
(d) The quesiion is incomplete
(d) Rises or falls depending upon the quantity of ice and
38. Two solids A and B float in a liquid. It is obscrved that A watcr
floats with half its volume immerscd and B floats with (2/3)
[Hint: See Question Xliii
of its volume immcrscd. Compare the dcnsities of A and B:
(a) 4,3 (b) 2 ,3 47. An ice cube containing a lead piece in it is floating in a glass
of water. As icc melts the water level will :
(e) 3 , 4 (d) 1 ,3
(a) Fall (b) Risc
39. A body floats with (1 /3) of its volume outside watcr and
(c) Remain stationary (d) None of thesc
(3/4) of its volumc outside another liquid. The density of
the other liquid is: [Hint: See Question XIV]
(a) (9/4) glee (b) (4/9)glee 48. A man is sitting in a boat which is float ing in:l pOl\d. If the
man drinks some water from the pond, the level of water in
(e) (813) glee (d) (3/9) glee
the pond will ?
40. A vessel contains oil (density 0.8 glcc) over mercury
(a) Rise a little (b) Sink a little
(density 13.6 glce). A homogeneous sphere floats with half
its volume immersed in mercury and the othcr half in oil. (e) Rcmain ~tati ollary (d) Nonc ofthc:ic
The density of the material of the sphere in glce is: [Hint: Sec Questiun XVI!
(a) 3.3 (b) 6.4 (e) 7.2 (d) 12.8
500 PHYSICS FOR COMPETITIONS - Vol. I

49. A boat with wood is floating in a lake. If the wood is (c) Remains unchanged
throw n in the lake the water level will: (d) Depends on several factors
(a) Go up (b) Go down 57. A cylinder is filled with a non~viscous liquid of density d to
(e) Remain unchanged (d) None of these height ho and a hole is made at a height hi from the bottom
[Hint: See Question XV I of the cylinder. The velocity of the liquid coming outofthe
hole is:
50. A boat with scrap iron is floating in a lake. If the scrap iron
is thrown in the lake the waler level will: (a) ~2hog (b) ~2g(ho - hi)
(a) Go up (b) Go down (c) ~gdh, (d) ~dgho
(e) Remain unchanged (d) None of these
58. In a vessel of water a hole is made at a depth on.5 m from
[Hint: Sec Question XV I
the free surface. The velocity of efflux will be:
51. A ball floats on the surface of waler in a container exposed
(a) 8.4 mI, (b) 84 mI, (c) 8.4 cmls (d) 84 cmls
to atmosphere. If the container is covered and air is
compressed, the ball will: 59. A water tank on the top of a tall building feeds water in the
taps on different floors . The waler will gush out at highest
(a) Rise
speed from a tap on floor:
(b) Sink
(a) Nearest to the tank
(e) Remain immersed at Ihe same depth
(b) Farthest from the tank
(d) None of the above
(c) Near the middle of the building
52. A wooden piece floats half submerged ill a tub of water. If
(d) Speed will be same on all the floors
the system is transferred to a lift ascending with
acceleration the wooden piece will : 60. TIle volume of a liquid nowing per sec out of an orifice at
the bollom of a lank docs not depend upon:
(a) Sink a little more
(a) The height of the liquid above the orifice
(b) Rise a little
(b) The acceleration due to gravity
(e) Remain half submerged
(c) The density of the liquid
(d) Sink to the bottom
(d) The area of the orifice
53. lfin a satellite orbiting the earth, a cork is immersed in ajar
of water and released, it will: 61. A tank is filled with water to a height fI. A hole is made in
one of the walls at a depth D below the water surfacc. The
(a) Rise (b) Sink
distance x from the foot of thc wall al which the stream
(c) Remain where left (d) None of these coming out of the tallk strikes the ground is given by:
54. People living in houses far remote from a municipal watcr IMNR19921
tank often find it difficult to get water on the top floor even
(a) x=2[D(H-D)J'12 (b) x=2(gD)'12
jf it is situated lower than the level of water tank. This is
because: (c) x = 2[D(H + D)J"2 (d) Noneoflhcse
(a) There is loss of pressure when water is flowing
62. A gale blows over a house. The force due to the gale on the
(b) There is something wrong with the pipe or else water roof is:
should always reach upto the lcvel of the tank (a) Downward (b) Upward
(c) The water pipe is not laid horizontally (c) Horizontal (d) Zero
(d) The diameter of the pipe all through is not unifonn 63. Action of a paint~gun is based on:
55. An incompressible non~vi scous fluid flows steadily (a) Bernoulli's principle (b) Boyle's law
through a cylindrical pipe which has radius 2 R at point A
(c) Faraday's law (d) Archimedes' principle
and radius R at point B farther along the flow direction. If
the velocity at point A is V, its velocity at point B will be: 64. Shown figure represents vertical sections of four wing~
moving horizontally in air. In which case is the fOfe(
IMNR1 99 11
upwards?
(b) V
(d) 4V (a)~ (b)
56. Water is flowing through a pipe of unifonn eross·seclion (c) I~~I (d)
llOder constant pressure. At some place the pipe becomes
65. An aeroplane works on:
narrow; the pressure of water at this place:
(a) Archimedes' principle (b) Pascal's law
(a) Increases
(c) BernouJli's principle (d) Stoke's law
(b) Decreases
66. Magnus effect is very near to the: (c) Both v and a depend on II
(a) Magnetic field (d) Neither v nor a depends on h
(b) Electric field [Hint: The reaction of outflowing liquid == mass coming out per
(c) Bernoulli's theorem sec x velocity of container.
(d) Magnetic effect of current :. (Ahp)a == P(w·v == pa · 2gh
with a and A as area of hole and contaiI:cr
67. To get the maximum flight, a ball must be thrown as:
2~g and the time taken for the tank to be empty.
C') ~' Cb) ~
:. a ==

~ ' ~ v= at ==.[2ih I
Ce) Cd) Any ofCa), (b) and Ce) 73. A solid ball is inunersed in a liquid. The coefficients of
volume expansion with tcmpcrature of the ball and liquid
68. A balloon of total mass 1000 kg floats motionlcss over the are 4 x 10- 6 and 8x 10- 6 per °C respectively. The percentage
earth's surface. If 100 kg of sand blast arc thrown over change in upthrust when the temperature is increased by
board, the balloon starts to rise with an acceleration of: 50°C is:
Ca) 10ml,' Cb) 9.8m1,' Ca) 0.04 Cb) -D.04
(c) 1.09 mls 2 (d) with none ofthrce values Ce) 0.02 Cd) 0.08
69. A cork is submerged in water by a spring attached to the 74. Water falling vertically from the
bottom of a pail. When the pail is kept in an elevator mouth of a tap in streamline flow
moving with acceleration downwards. The length of forms a tapering column as shown in
spring: the Fig. 10.49. Which of the
following is the most correct
(a) Increases

I
(b) Decreases
explanation for this?
(c) Remains unchanged (d) None of these
(a) As the water falls down, its
70. A 25 em long tube closed at one end is lowered with the
velocity increases resulting in
open cnd down into a freshwatcr lake. If one-third of the decrease in pressure. So, it is
tube is filled with water, the distance between the surface of then compressed by the Fig. 10.49
the lake and the level of water in the tube is:
surrounding air
(a) 641 cm (b) 510 em (b) The mass of water passing through any cross-section
(c) 420 em (d) 582 em must remain constant. So in accordance with Av =
71. A hollow metallic sphere with internal and external radii r constant (as liquid is incompressible), the area of
1
and r2 respectively floats on thc surface of liquid. Thc cross-section decreases with increase in velocity
density of liquid is p! and density of material of sphere is (c) Due to surface tension effect, exposed surface area of
P2' What fraction of the spherc is inside the liquid? liquid decreases

Ca) (l_i)"2
r1 PI
Cb) (1_'r1i)!'.L
P2
(d) To attain terminal velocity, area of cross-section
decreases

75. The ~ressure of water at bottom in a lake is ~ times that at

half depth where the water barometer reads 10m. The depth
of the lake is:
72. A containcr having a hole at the (a) 15 m (b) Infinite
bottom is free to move on a
(c) 20 m Cd) 10 m
horizontal surface. As the liquid
comes out, the container moves
76. A piecc of ice floats in a vessel with water above which a
layer of lighter oil is poured. When the whole of ice melts,
in backward direction with an
which one of the following statements will be true?
acceleration 0. and finally
acquires a velocity v (when all (a) The level ofthe interface will remain thc same
Fig. 10.48
the liquid has drained out). (b) The total level of the liquids in the vessel will rise
Neglect the mass of container. The correct option out of (c) Thc total level of the liquids in the vessel will fall
the following is: while the level of interface will go up
(a) Only v depends on II (d) The level of interface as well as the total level of
(b) Only 0. depends on II liquids in the vessel will fall
502 PHYSICS FOR COMPETITIONS - Vol. I

77. Two bodies with volumes V and 2V arc equalized on a 84. A metallic sphere floats in an immiscible mixture ofwatcr
balance. The larger body is then immersed in oil of density (Pw=103 kg/m l ) and a liquid (p/=13 .5x lO l kglm 3)
d! = 0.9 glcm 3 while the smaller body is immersed in
such that its; th portion is in water and ~ th portion is in
another liquid when it is found that the equilibrium of the
balance is nol disturbed. The density of the second liquid is liquid. Density of the metal is:
then: (a) 4.5 x 103 kglm 3 (b) 4.0 x 10 3 kglm 3
(a) 2.4 glcm) (b) 1.8 glem' (c) 3.5 x 103 kglm 3
3
(d) 1.9 x 10 kgfm~
(e) 0.45 g/c~3 (d) 2.7 glem ) 85. A body is floating in a liquid contained in a vessel. The
78. The outboard motor of a smalJ boat has a propeller of whole system is falling under gravity. Upthrust on the body
diameter 200 mm. When the boat is at rest, the propeller due to liquid is:
sends back a stream of water at a speed of 10 mls. One half (a) equal to weight of liquid displaced
of the work that is being done by th~ motor is transferred to
(b) equal to weight of body in air
this water as kinetic energy. The power output of the motor
(c) equal to half the weight of object in air
is (density of water is 1000 kg/m 3): ,
(d) zero
(a) 1.25 kW (b) 6.50 kW
86. A container containing watcr has a constant acceleration ' 0'
(e) 15.7kW (d) 31.4 kW
in the horizontal direction. Free surface of water gets sloped
79. Two communicating vessels contain mercury. The with the horizontal at angle:
diameter of onc vessel is four times larger than the diameter
of the other. A column of water of height ho = 70 em is (a) eo,-I(~l (b) Sin-t~l
poured into the left hand vessel (the narrower one). How
much will be mercury level rise 'in the right hand vessel?
(Specific density of mercury = 13.6) (d) sin -ti)
(a) 0.3 cm (b) 0.7 cm
87. A vessel contains liquid of density 0' filled to a height H.
(e) 0 .1 em (d) 1.0 em The vessel is placed at rest. Avernge pressure exerted by the
80. A cylinder of height 20 m is completely filled with water. liquid on the wall is:
The velocity of efflux of water (in m s -\) through a small Hcrg
hole on the side wall of the cylinder near its bottom is: (a) 2 (b) Hcrg
[AIEEE2002[
(c) Zero (d) Hcrg
(a) 10 (b) 20 4
(e) 25.5 (d) 5 88. A cubical block of wood of specific gravity 0.5 and a chunk
81. At what speed, the velocity head of water is equal to of concrete of specific gravity 2.5 are fastencd together.
pressure head of 40 em ofHg? IRPMT 20071 The ratio of mass of wood to the mass of conercte which
(a) 10.3 mls (b) 2.8 mls makcs the combination to float with its entire volume
submerged in water is:
(c) 5.6 mls (d) 8.4 mI,
1 (b)
1
82. A person is carrying a bucket of water in one hand and a (a)
5 3
block in the other hand. After putting the block in the
bucket in which the block floats, the person carries: 3 2
(e) (d)
5 5
(a) more weight than before
(b) less weight than before 89. An object is malic of a material of density 0.4 gm/cc. It is
held 12 cm above the surface of water contained in a vessel
(c) same weight as before
and released. The depth it touches in water is:
(d) it cannot be decided
(a) 8cm (b) 6cm
83. A n object when placed in water floats with one-third of its
(c) \12 em (d) 4 cm
volume outside water. If it is placed in a liquid ' X '. it floats
90. A vessel contains an immiscible mixture of waler and a
wi th 2of its volume outside the liquid. Density of the liquid liquid of density 0. 8 gmlce. A cube of side 10 cm is placed
4
in the mixture and it is observed that the water-liquid
' X'is:
interface is at the middle of cube height. Mass of the cube
(a) ~ gmlcc (b) ~ gmlce IS:

(a) 420 gm (b) 680 gm


1
(c) 13 gmlcc (d) 1; gmlcc (c) 225 gm (d) 900 gm
91. A concrete sphere of radius R has a cavity of radius rwhich (a) 18 mls (b) 9 mls
is packed with sawdust. Specific gravities of concrete and (c) 7.2 mls (d) 14 m/,
sawdust are respectively 2.4 and 0.3 for this sphere to float
with its entire volume submerged under water. Ratio of 98. In Q. 97, an orifice has to bc so made that water coming out
mass of concrete to mass of sawdust will be: of it strikes ground at maximum horizontal range, then
(a) 4 (b) 2 (e) 3.2 (d) 1.8 height of orifice from ground will be:
92. A rectangular boat floating in water has length 4 m and (a)5m (b)7.5m
breadth 1.5 m. A person gets into the boat as a result of (c) 6 m (d) 8 m
which the boat sinks by 2 em. Mass of the person is: 99. A U-tube is partially filled with water. Oil, which docs not
(a) 80 kg (b) 100 kg mix with water, is next poured into one side until water rises
(0) 120 kg (d) 92 kg by 25 cmon the othcr slde. If the density of oil beO.8, the oil
level will stand higher than the water level by:
93. A vessel open at top contains 50 litres of water. A small
opening is made at the bottom of vessel. It is observed that 3 (a) 6.25 cm (b) 12 .50 em
litres of water comes out in time t l' the next 3 litres in' a (c) 31.25 em (d) 20 em
further time t 2 and the next 3 lilres in further time t 3' then: 100. A U-tube of unifonn cross-section is partially filled with a
(a) 11 =/ 2 =/3 (b) II </2 <13 liquid I. Another liquid II which does not mix with liquid r
(c) II >/2 >13 (d) tl :t2 </3 is poured into one side. It is found that the liquid levels of
the two sides of the tube arc the same while the level of
94. Liquid is filled to a height 'h' in a vessel whose side walls
liquid I has risen by2 em. If the specific gravity of liquid I is
are vertical. A hole is made in one ofthe side walls at depth
1.1, the specific gravity of liquid II must be:
h' below the surface of liquid such that water emerging
from hole strikes the ground at maximum horizontal (a) 1.12 (b) 1.1
distance, then: (0) 1.05 (d) 1.0
(a) h'=~ (b) h' ~"2
101. The height to which a cylindrical vessel be filled with a
4 homogeneous liquid, to make the average force with which
the liquid presses the side of the vessel equal to the force
(c) h'=~ exertcd by the liquid on the bottom of the vessel, is equal to:
3
95. A ves::;e1 contains a liquid of density 0.8 gmiee. Side walls (a) Half of the radius of the vessel
of the vessel arc vertical and its base horizontal. Two small (b) Radius of the vessel
holes are made in the vessel at heights 20 em and 40 cm (c) One-fourth of the radius of the vessel
above the base and it is observed that liquid coming out of (d) Three-fourths of the radius of the vessel
the two holes strikes ground at same position. Then height
102. Two identical cylindrical vessels with their bases at the
of a third hole to be made such that water coming out of it
same level each contains a liquid of density d. The height of
strikes ground at maximum distance is:
the liquid in one vessel is hI and that in the olher vessel is
(a) 30 em (b) 10 em (c) 25 em (d) 18.5 em h 2 . The area of either base is A. The work done by gravity
96. Water is pourcd in a tank through a cylindrical tube of area in equalising the levels, when the two vessels are connected
of cross-section A and ejecting water at a constant speed 4 IS:
mis. The tank contains a hole of area AI2 at bottom. Level (a) (hi - I., )gd (b) (hl-h,)gAd
of water in the tank will not go up beyond:
(e) (l12)(h l -h,)' gAd (d) (1/4)(1. 1 - I., )' gAd
(takeg=IOm/s 2 )
103. When a large bubble rises from the bottom of a lake to the
(a) 5.6m (b) 4.8m (c) 3.2m (d) 1.810 surface, its radius doubles. The atmospheric pressure is
97. A cylindrical tank IS
placed on a platfonn 5 m
high. Initially the tank is
r-
5m
- --- ---
- ---
------_.
-----
-- -
-- ----_.
- ---
- - -----_.
___ ___ _
cqual to that ofa column of water of height H. The depth of
the lake is:
(a) H (b) 2H
filled with water uplo a
height 5 m. An orifice is
made in the side wall of
-I-'~:"''''~'''''''''''''''~: (e) 7H (d) 8H
104. Water from a tap emerges vertically downwards with an
tank just at the bottom. initial speed of 1.0 m s-I. The cross-sectional area of the
Speed with which water
coming out of the orifice
hits ground is nearly:
'r tap is 10- 4 m 2 . Assume that the pressure is constant
throughout the stream of water and that tlie flow is steady.
The cross~sectional area of the stream 0.15 m below the tap
(takeg=lOmls 2 ) Fig. 10.50 IS: fliT 1998)
(a) 5.0 x 10-4 m 2 (b) 1.0 x 10- 5 m2 (,) Mg (b) Mg - Vpg

(c) S.Ox 10-5 m 2 (d) 2.0x 10-5 m2 (c) Mg+nR 2hpg (d) pg(V +rrR'h)

105. The pressures of water in a water pipe when the lap is 109. An L-shapcd glass tube is just inunersed in flowing water
such that its opening is pointing against flowing water. If
closed and open are respectively 3.Sx 105 N/m 2 and
the speed of water current is v, then:
3x IDs N/m 2. The velocity of water flowing through the
pipe when the lap is open is: feMe Ludhiana 20081
(a) The water in the tube
,
(a) [On1/s (b) O.Sx\0 6 m/s rises to height ~g
5
(c) 3.5x iO~ mls (d) 3x 10 mls (b) The water in thc tube
3x 10 5 3.Sx 106
rises to hcight ~
106. Water flows through a frictionless duct with a cross-section 2v
varying as shown in Fig. 10.51. Pressure p at points along
(c) The water in the tube Fig, 10.54
the axis is represented by:
~
-~ --- ... does not rise at all
(d) None of the above
""=7.~--­
Fig. 10.51 110. A water tank res ting on the floor has two small holes

[ It' _ vertically one above the other. The holes are h] em and 112
cmabove the floor. How high does water stand in the tank if
the jets from the holes hit the floor at the same point?
(')~ (b)~
(a) ~::2+:;i) (b) [(:J ~~:!)
It' I I~
(e) 2 (d) 2

""
(C)~ (d)~
"
Ill. An L-shaped tube with a small
orifice is held in a water stream
as shown in Fig. 10.55. The
upper end of the tube is 10.6 cm
107. Three tubes A, Band Care
connected to a horizontal A c above the surface of water. What
will be the height of the jet of
pipe in which ideal liquid is B
water coming from the orifice?

~~:!n~, ~ea:~d~ :~ !~: j~~~~=~~~-~:~~3--:'~__


~_~_~..::...:~~g~~~~~~~~-:~: Velocity of water stream is 2.45
mls: Fig. 10.55
junction are respectively 2 FI g.10.52
em, I em and 2 em. It can be (a) Zero (b) 20.0 cm
said: (e) 10.6 em (d) 40.0 cm
(a) The height of the liquid in the tube A is maximum 112. A metallic square plate attached
(0) Height of the liquid in the tubes A and B is same with a massless inextensible string
(c) Height of the liquid in the tubes A, Band C is same ~s ~~a~:~:sd:~o~~q~~dF~~.n~~~e:. if~ ~n~~~t~~~~6~~
(d) Height of the liquid in the tubes A and C is same the point of support is slowly raised ==:
lOS. A hemispherical portion of radius R is
removed from the bottom of a cylinder ::~i~i~~YV~~:ea~r~;it:h~;~nn:t~~~ ~n~:'=:~::~::~::~:~~~~-~~~i
of radius R. The volume of the remaining variation of tension T in the string Fig. 10.56
h
cylinder is V and its mass is M. It is against displacement of support is best represented by:
suspended by a string in a liquid of
ciensity p where it stays vertical. The
upper surface of the cylinder is at a depth
h below the liquid surface. The force on 2R
, (,) T~, (b) T~,
the bottom of the cylinder by the liquid L-=--'''-,.".-'

T~,
Ls
IS: (JIT2001j Fig. 10.53
(d) T
(e)
113. A large open tank has two holes in the wall. One is a square (a) 0.75 g/em 3 (b) 0.83g/cm 3
hole of side L at a depth y from the lOP and the other is a
(c) 1.2 g/cm 3 (d) 0.25 g/cm 3
circular hole of radius R at a depth 4 y from the top. When
the tank is completely filled with water, the quantities of ]]8. Two immiscible liquids P and Q of different densities are
water flowing out per second fr0111 both holes arc the same. contained in a wide V-tube as shown in Fig. 10.59. The
Then R, is equal to: [lIT 2000) heights of the two liquids abovc the horizontal line .xx'
which cuts the boundaty between the liquids are H p and
(a) --'-- (b) 2nL (0) L (d) -'- HQ respectively. The V-tube is transported to a planet
J2Tt 211:
where the acceleration of free fall is ~ that on the earth,
114. A small uniforn1 tube is bent into a circle of radius rwhose
p lane is vertical. Equal volumes of two immiscible liquids where the liquids do not evaporate and where the heights
whose densities afC P I and p 2 (p I > P 2 ) fill half the circle. of liquid (measured relative to XX') are hp and hQ
The angle between radius passing through interface and respectively. Which of the given statements is correct?
vertical axis is given by:

(a) tant: ] p

-1[:~ ]
(b) tan
PI

(0) "n-t[PI -P2]


PJ+P2 Fig. 10.59
(a) The liquid l.eve!s are unchanged, i.e.,lI p ::Hp and
(d) tan-I[PI +P2] Fig. 10.57
hQ "HQ
PI -P2 (b) Both liquid levels rise up so that IIp!H p '" hQIHQ
115. An aluminium wire is wound on a picce of cork of mass 10 (c) Both liquid levels rise up so that h p -ItQ = H p - H Q
g. The densities Pc' P a and P", of cork, aluminium and (d) The liquid P falls and liquid Q rises such that
water arc 0.5 x 10 3 kg/m 3 , 2.7x 10 3 kg/m 3 and Ix 10 3 hp ZHp
kglm 3 respectively. The minimum mass of aluminium wire -"--
hQ 3HQ
that should be wound on the cork so that the cork with the
wire shou ld stay completely submerged in water is: 119. A cylindrical vessel ofhcight It and base area S is filled with
(a) 24g (b) 16g (e) 36g (d) 12g watcr. An orificc of area s «S is opened in the bottom of
the vessel. Neglecting the viscosity of water detennine how
]]6. A metal ball of density 7800 kg/m 3 is suspected to have a
soon all the water will flow out of the vessel:
large number of inner cavities. It weighs 9.8 kg when
weighed directly on a balance and 1.5 kg less when
immersed in water. The fraction by volume of the cavities
(a) Time~- p! (b) Time=f~
in the metal ball is approximately:
(c) Timeo:;Ss-J2gh (d) Time",2h (f
(a) 20% (b) 30% g f;
(c) 16% (d) 11%
120. A sphere of solid material of density d has a concentric
117. The limbs ofa glass V-tube are lowered into vessels A and
spherical cavity of radius r. If it just floats in water, i. e., with
B, A containing water. Some air is pumped out through the its upper surface touching the water surface, then Rand r
top of the tube C. The liquids in the left hand limb A and the are related as:
righ t hand limb B rise to heights of 10 em and 12 cm
respectively. The density of liquid B is: (a) r3 = R3(1_~) (b) r3 =R 3(d_l)

, R'
(d) r "-~

(1+~1
2 121. A streamlined body falls through air from a height h on the
surface of a liquid. If d and D(> d) represent the densities
(Liquid)
of the material oflhe body and liquid respectively, then the
{VVate0 " ",,""
time after which the body will be instantaneously at rest, is:
Fig. 10.58
~ ~
D

L
(a) (b) _.-
125. A rectangular box containing water "~'~"'" a = 3 ml"
g d is accelerated upwards at 3 mls 2 on
an inclined plane 30° to the -,::,'::::j:':"
(c)
~~.~ (d) (2h(~)
fi D-d
horizontaL Find the slope of the free
liquid surface: 30'
Fig. 10.63
122. A homogeneous aluminium (a) 0.23 (b) _1
ball of radius,.~ 0.5 em is Jj
suspended on a weightless (c) Jj (d) 0.32
thread from onc end of a
uniform rod of mass 4.4 g. 126. A candle of diameter d is floating on a
The rod is placed on the liquid in a cylindrical container of
edge of a tumbler with diameter D(D»d) as shown in
water so that half of the ball Fig. 10.60 figure. If it is 'burning at the rate of 2
is submerged in water when the system is in equilibrium. cmlhour then the top of the candle
The densities of aluminium and water are 2.7x 103 kg/m 3
will: IAIIMS 2005J
and Ix I03 kg / m 3 respectively. The ratio of the two parts of (assume that density of wax is half
the density of the liquid)
the rod 2: Fig. 10.64
x as shown in Fig. 10.60 will
. be: (a) remain at the same height

(a) 1.5 (b) 2.2


(b) fall at the rate of 1 cmlhour
(c) 1.35 (c) fall at the rate of 2 cmlhour
(d) 3
123. A square gate of size 2m x 2m is hinged at its mid-point. A
(d) go up at the rate of 1 cmlhour
fluid of density cr fills the space to the left of the gate. The 127. Radius of an air bubble at the bottom of the lake is rand it
force F required to hold the gate stationary is: becomes 2rwhen the air bubble rises to the top surface of
the lake. If P cm of water bc the atmospheric pressure, then

: : :i:
(c) 8crg
uumuuUI m
===~=C",===,=
~~~~~~~~~~~~~~ R
tg. the depth of the lake is:
(a) 2P
(e) 5P
(b) 8P (c) 4P
IKerala PET 2005]
(d) 7P

128. A cylindrical, tank is filled with water to a level of 3m. A


hole is opened at a heightof52.5 cm from bottom. The ratio
3
F
of the area of the hole to that of cross-sectional area of the
(d) crg cylinder is 0.1. Find the square of the velocity with which
Fig. 10.61
3
water is coming out: (g = 10 mlsec 2) (lIT 2005]
,
!Hint: Take torque about hingeD,
ioogy(2dy)(y-l) == F xli
2
(a) 50 m /sec 2 2
(b) 40 m /sec 2

p~
124. A tube filled with water
129. If there were no gravity, which of the following will not be
of density p and closed
at both ends unifonnly there for a fluid? [Karnataka CET 20061.
rotates in a horizontal (a) Viscosity
plane about a vertical !--r1--1 (b) Surface tension
:' (2 -I
axis. The manometers (c) Pressure
fixed in the tube at Fig. 10.62
(d) Archimedes' upward thrust
distances Ii and r2 from the axis indicate pressures PI and
Pz respectively. The angular velocity (0 of rotation of the 130. A body is just floating on the surface of a liquid. The
tube will be: density ofthe body is same as that of the liquid. The body is
~-~ slightly pushed down. What will happen to the body?
2(P2 - PI) 2(Pz + PI)
(a) 0>= (b) 0>.= IHPPMT 20061
2 2
perl -lj ) p(/1+lj2) (a) It will come back slowly to its earlier position

(c) 0)=
t; -Pi)
P r2 'i
(d) 0>= ~t2_1l)
p r r 2 1
(b) It will remain submerged where it is left
(c) It will sink
(d) It will come out violently
131. Pressure at the bottom of tank ofwatcr is 3P, where P is
atmospheric pressure. Ifthc water is drawn out till the level
of water is lowered by one fifth, then the pressure at the
bottom ofthe tank is:
(,) 2P (b) 13PI5
IBV Pune 2006]
r
h
(e) SPI5 (d) 4PI5
132. A body of mass 120 kg and densitY 600 kg/m 3 floats in
water. What additional mass could be added to the body so
1'--::-:.-:-:::---'
FIg. 10.66
that the body willjustsink? (Orissa JEE 2006] (a) both / and h increase
(a) 20 kg (b) SO kg (b) both I and h decrease
(e) 100 kg (d) 120 kg (c) I decreases and h increases
133. A liquid X of density 3.36 glcm 3. is poured in aU-tube, (d) I increases and h decreases
which contains Hg. Another liquid Y is poured in left arm 137. The neck and bottom of a bottle are 3 cm and 15 cm in
with height 8 em. Upper levels of X and Yare same. What radius respectively. If the cork is pressed with a force 12 N
is densityofY? (DeE 20061 in the neck of the bottle, then force exerted on the bottom of
y x the bottle is: (WB (JEE) 2006]
8 Oem (a) 30 N (b) 150 N
(0) 300 N (d) 600 N
138. An open V-tube contains mercury. When 11.2 em of water
is poured into one of the anns ofthe tube, how high does the
mercury rise in the other ann from its initial level?
(Punjab CET 2006]
Fig. 10.65 (a) 0.56em (b) 1.35 cm ,
(a) 0.8 gmlcc (b) 1.2 gmlee (e) 0.41 em Cd) 2.32 em
(c) 1.4 gmlcc (d) 1.6 gmlcc 139. Water is filled up to a height h in a --~
134. A solid sphere of volume V and density p floats at the beaker of radius R as shown in the
interface of two immiscible liquids' of densities p ] and p 2 figure. The density of water is p,
respectively. Ifp I < p <P2' then the' ratio of volume of the the surface tension of water is T
and the atmospheric pressure is Po'
parts of the sphere in upper and lower liquid is:
[Kerala PET 20061
Consider a vertical section ABeD
of the water eolunm through a
T
h
(,)
P - Pl
P.2 :....p
(b)
P2 - P
P - PI
diameter of the beaker. The force
on water on one side of this section 1
by water on the other side of this
p+p] P + P2
(e) (d) section has magnitude: FIg. 10.67
P + P2 P+PI
[lIT JEE 2007[
(e) ~PIP' (,) [2PoRh +nR'pgh - 2RT[(b) [2PoRh+Rpgh' -2RT [
P 2
(e) IPo 1tR 2 + Rpgh2 - 2RTICd) iP01tR2 + Rpgh +2RTI
135. By sucking through a straw, a student can reduce the
140. A hole is in the bottom of the tank having water. If total
pressure in his lungs to 750 mm of Hg (density = 13.6
pressure at the bottom is 3 atm (I atm = 10 5 Nm -2), then
g icm 3 ). Using the straw, he can drink water fTom a glass
velocity of water flowing from hole is: (RPMT 2007]
upto a maximum depth of: [AlIMS 2006; VPSEE 2006]
(a) ,j400m,-1 (b) ,j600m,-1
(a) 10 cm (b) 75 em
(c) .J6(ims- 1 Cd) none of these
(c) 13.6 cm (d) 1.36 cm
141. Radius of one ann of hydraulic lift is four times ofmdius of
136. A wooden block, with a coin placed on its top, floats in
other ann. What force should be.applied on the narrow ann
water as shown in the figure . The distance hand lare shown
to lift 100 kg? (DeE 2007]
there. After sometime, the coin falls into the water. Then:
(a) 26.5 N (b) 62.5 N
[WB (JEE) 2006[
(e) 6.25 N (d) S.3 N
142. A glass flask having mass 390 g and an interior volume of 147. An iceberg floating in ocean. What fraction of its volume is
500 em 3 floats on water, when it is less than half filled with above the water? (Given: density of icc = 900 kg/m 3 and
water. The density of the material of the flask is: density of ocean water = 1030 kg/m 3).
(Orissa JEE 20071 90 13 10 I
(a) 103 (h) 103 (e) 103 (d) 103
(a) 0.8 glee (h) 2.8 gloo
148. A tank of cross-sectional area Ao is ...,====
(e) 1.8 gloo (d) 0.28 glee
143. A unifonnly tapering vessel is filled
filled with a liquid. A small orifice of :~~~~~~~~~
area A(A «AO) is present at the -~~~:=-:==~::-
T
~~ttl~~~i~f~nn~h~\~~o~e~~h:~::g~: ~llllllllll
with a liquid of density 900 kg/m 3 .
The force that acts on the base of the
vessel due to the liquid is: (g = 10 acceleration oftop layer of liquid at this :,
mls 2) Iindraprastha CET 20081 moment?
Fig. 10.71
(a) 3.6 N A
(a) G (h) - g
(h) 7.2 N Fig. 10.68 AO
(e) 9.0 N
(d) 14.4 N
144. An ice block floats in a liquid whose density is less than
water. A part of block is outside the liquid. When whole of
ice has melted, the liquid level will:
(Indraprastha CET 2008]
(d) Depends on H

149. A hemispherical portion of radius R is [5


removed from the bottomofa cylinder of : __:-:::-:::-::::-:::~_=
radius R. The volume of the remaining _ _ ~~

(a) rise (b) go down


cylinder is V and its mass M. It is ~ = :e.~
suspended by a string ill a liquid of _~ __ :::.:::.
(c) remain same (d) first rise then go down density p, where it stays vertical. The =~~3~::.:!:~~
145. A jar is filled with two non-mixing upper surface ofthe cylinder is at a depth - - - - - - - - --
II below the liquid surface. The force on Fig. 10.72
liquids I and 2 having densities P I and
II~IHI~I
the bottom of the cylinder by the liquid is:
P2 respectively. A solid ball, made of
a material of density P 3' is dropped in (a) Mg (h) Mg-Vpg
the jar. It comes to equilibrium in the (e) Mg-rul'hpg (d) pg(V +nR'h)
position shown in the figure.
Fig. 10.69 150. In the adjoining figure the
Which of the following is true forPI' cross-sectional area of
P2 andp 3? (AIEEE 2008( smaller tube is a and the
target tube is 20. A block of
(a) P3 < P I <P2 (b) PI >P3 > P2 mass 2 m is kept in the
(c) PI <P2 <P3 (d) PI <P3 <P2 smaller tube have same base
146. For the system shown area a as that of the tubc. Fig. 10.73
in the Fig. 10.70, the The difference between water levels of the two tubes are:
cylinder on left has a
(a) ~+...!E.
pg ap
(h) -Po- +m-
pg 2ap
mass (M)of25 kg and 5
cross-sectional area 20
(c) 2m (d) m
cm 2 and is connected ap ap
to a spring of spring
constant 1400 N/m. 151. A liquid of density p is filled in '
The piston on the right a conical vessel as shown in
has mass m (= 5kg) and Fig. 10.70 Fig. 10.74. Force exerted by
. liquid on side wall is : .: : h ..... ...... .
cross-sectional area 4 cm 2 . The minimum mass m' to be
kcpt on III so that water spills out from the left is : (g = I 0 (a) ~1tR2pgh
mls 2 ) (initially water level in both limbs is same).
(elongation of spring = 5 cm) (b) }1tR2pgh _R---+
(a) 1 kg (h) 1.4 kg Fig. 10.74

(e) 0.7 kg (d) 2.5 kg (c) 1tR2pgh


(d) 0
152. A vessel contains oil (density =0.8 glcm 3) over mercury 2n 2 n 2 r2 4n 2 n 2 r2
(,) h~ (b) 11=
(density = 13.6 glcm 3). A homogeneous sphere floats with g g
half its volume immersed in mercury and the other half in
21m 2 ,.2 4nn 2 r2
oil. The density of tile material of the sphere in g fcm 3 is: (e) h~ (d) h~ --~
g g
(a) 3.3 (b) 6.4
(e) 7.2 (d) 12.8 159. A tank is filled liquid upto height H. A small hole is made at
the botlom of tank. Let t 1 be the time taken to empty first
153. A homogenous solid cylinder of length 1
11..£ < HI2) cross-sectional area A/5 is halfand 12 the time to rest half, then -.!.. is proportional to:
I,
immersed such that it floats with its axis
vertical at the liquid-liquid interface with
(a) -fi (b) -fi~1 (e) 4.14 (d) _I
lengthLl4 in the denser liquid as shown -fi
in the figure. The lower density liquid is
160. The opcning near the
open to atmosphere having pressure Po'
Fig. 10.75 bottom of the vessel shown
Then, density D of solid is given by:
in figure has an area A. A
(a) ld (b) ~d (e) 4d (d) <L
5
disk is held against the
opening to keep the liquid,
154. A piece of copper having an intemal cavity weighs 264 g in from running out. Let FI be
air and 221 g in water. The volume of the cavity is : the net force on the disk A
Flg.10.n
(Density of copper = 8.8 glcc) applied by liquid and air in
this case. Now the disk is moved away from the opening a
(a) 43 cc (b) 30 cc
short distance. The liquid comes oul and strikes the disk
(c) 13 cc (d) 70 cc
inelastically. Let F2 be the force exerted by the liquid in
155. A metal ball of density 7800 kg/m 3 is suspected to have a F
this condition. Then _I is:
large number of inner cavities. It weighs 9.8 kg when F,
weighed directly on a balance in air and 1.5 kg less when
immersed in water. The fraction of volumes of the cavities (a) ~I (b) (e) 2 (d) l
in the metal ball is approximately: 2 I 4 ~.

(a) 20% (b) 30% 16l. In the Fig. 10.78, an ideal A

~C.B=
(c) 16 % (d) II % liquid flows through the tube,

156. Fig. 10.76 shows the top II which is of unifonn cross-

view of a cylindrical can ~=:='=~=3


mounted on a turn table. 1+--'~'4
;M~: I section. The liquid has
velocities VA and VB and
Fig. 10.78

pressures PA and PB at vertical plane (at points A and B


~e cylindrical can is filled Fig. 10.76 respectively). Which of the following is correct?
with water. At a depth h
bellow the water surface are two horizontal tubes of length (a) VA =VB,PB >PA (b) VA =VB,PA =PB
I and cross-sectional area a, with right angled bends at their (c) PA =PB (d) PA = PB ,VB >VA
ends. Ifp is the density of water, find the torque exerted on 162. A cylindrical container of cross-sectional area 100 cm 2 is
the system as the water jet emerges from the tubes. containing water upto a height of2 m. Now a cork has same
(a) 4pgh(NI)a (b) 4pghm area of cross-section and mass 5 kg is put into the container
(e) 4pghla (d) Zero and it floats in equilibrium. If a tiny hole is made on vertical
walls very close to base then the velocity of effiux will be
157. A cylindrical vessel filled with water upto the height H
(in mls) : (assume cork is freely movable down)
becomes empty in time to due to a small hole at the bottom
of the vessel. Ifwater is filled in the same vessel to a height (a) 2M (b) 5 (e) sfi (d) 10
4H it will flow out in time: 163. Water from a tap emerges vertically downwards with an
(a) 8/0 (b) 410 initial speed of 1.0 m s -1. The cross-sectional area of the
(e) 210 (d) 10 tap is 10-4 m 2. Assume that the pressure is constant
158. A liquid is kept in a vertical cylindrical vessel which is throughout the stream of water and the flow is steady. The
rotated along ils axis. The liquid rises at Ihe sides. If the cross-sectional area of the stream O. t 5 m below the lap is
radius of vcssel is r and the speed of revolution is n approximately:
rotation/sec, find the difference in height of liquid al the (a) 5.0x 10-4 m 2 (b) l.Ox 10- 5 m 2
centre of vessel and its sides. (c) 5.0x 10- 5 m 2 (d) 2.0x 10- 5 m 2
ANSWERS

1. (b) 2. (d) 3. (a) 4. (a) 5. (c) 6. (c) 7. (d) 8. (d) 9. (b) 10. (c) t 1. (a) 12. (b)
13. (b) 14. (c) 15. (b) 16. (a) 17. (a) 18. (c) 19. (d) ·20. (a) 21. (a) 22. (a) 23. (d) 24. (d)
25. (b) 26. (c) 27. (a) 28. (c) 29. (c) 30. (d) 31. (a) 32. (c) 33. (c) 34. (b) 35. (b) 36. (b)
37. (b) 38. (c) 39. (c) 40. (c) 41. (a) 42. (d) 43. (a) 44. (c) 45. (b) 46. (c) 47. (a) 48. (c)
49. (c) SO. (b) 51. (a) 52. (c) 53. (c) 54. (a) 55. (d) 56. (b) 57. (b) 58. (a) 59. (b) 60. (c)
61. (a) 62. (b) 63. (a) 64. (a) 65. (c) 66. (e) 67. (b) 68. (e) 69. (b) 70. (b) 71. (a) 72. (a)
73. (b) 74. (b) 75. (e) 76. (c) 77. (b) 78. (d) 79. (a) 80. (b) 81. (a) 82. (c) 83. (a) 84. (e)
85. (d) 86. (c) 87. (a) 88. (c) 89. (a) 90. (d) 91. (a) 92. (e) 93. (b) 94. (b) 95. (a) 96. (e)
97. (d) 9S. (a) 99. (b) 100. (b) 101. (b) 102. (d) 103. (c) 104. (c) 105. (a) 106. (a) 107. (d) lOS. (d)
109. (a) 110 (a) Ill. (b) 112. (a) 113. (a) 114. (e) 115. (b) 116. (c) 117. (b) 118. (a) 119. (a) 120. (a)
121. (d) 122. (a) 123. (b) 124. (a) 125. (a) 126. (b) 127. (d) 128. (a) 129. (d) 130. (e) 131. (b) 132. (b)
133. (a) 134. (b) 135. (e) 136. (b) 137. (e) 138. (e) 139. (b) 140. (a) 141. (b) 142. (b) 143. (b) 144. (b)
145. (d) 146. (b) 147. (b) 14S. (c) 149. (d) 150. (c) 151. (a) 152. (e) 153. (a) 154. (e) 155. (c) 156. (a)
157. (c) 158. (a) 159. (b) 160. (a) 161. (a) 162. (e) 163. (c)

[BJ More than One Choice is Correct 4. Ifa sample ofmclal weighs 210 g in air, 180 g in water and
120 g in a liquid: IMNR 1992)
1. If the system is not in free fall, which of the following
statements are true about hydrostatic pressure? (a) RD of metal is 3 (b) RD of metal is 7
(a) In a liquid points at different depths can never be at (e) RD of liquid is 3 (d) RD of liquid is (l/3)
same pressure 5. When a body of density p and volume V is floating in a
(b) In a liquid points at different depths may be at same liquid of density cr:
pressure (a) Its true weight is Vpg
(c) In different liquids points at same depth can never be at (b) Loss in its weight is Vcrg
same pressure (e) Its apparent weight is zero
(d) In different liquids points at same depth can be at same (d) Its density p is lesser than that of liquid cr
pressure
6. A piece of wood is floating in water kept in a bottle. The
(e) In different liquids points at different depths can be at bottle is connected to an air pump. When more air is pushed
same pressure into the bottle from the pump:
2. When a body is weighed in a liquid the loss in its weight is (a) The wood piece will rise a little up
equal to:
(b) The thrust of air will increase
(a) Weight of liquid displaced by the body
(c) The thrust of water will decrease
(b) Weight of water displaced by the body
(d) The total thrust will remain unchanged
(e) The difference in weights of body in air and liquid
7. In case of motion of fluid in a tube where area of cross-
(d) The upthrust of liquid on the body
section is minimum:
3. The spring balance A reads 2 kg with a (a) Velocity is max.imum (b) Velocity is minimum
A
block suspended from it. A balance Breads
5 kg when a beaker with liquid is put on the (c) Pressure is max.imum (d) Pressure is minimum
pan of the balance. The two balances are 8. The velocity ofeffiux of an ideal liquid does not depend on:
now so arranged that the hanging mass is (a) The depth of point below the free surface of liquid
inside the liquid in the beaker as shown in (b) The area of orifice
Fig. 10.79. In this situation:
(c) The density of liquid
(a) The balance A will read more than 2 kg Fig. 10.79
(d) The area of cross-section of the vessel
(b) The balance B will read morc than 5 kg
9. Which of the following represents a conservation law?
(c) The balance A will read less than 2 kg and B will read
(a) Continuity principle (b) Magnus effect
more than 5 kg
(c) Bernoulli's theorem (d) Torricelli ' s theorem
(d) The balances A and B will read 2 kg and 5 kg
respectively
10. A body totally inullersed in water is raised in the water by a
height h. The density -of the body is d and the density of
water is dO while the volume of the body is V and d > do'
Which ofthe following statements will be true? ---
- -----
------_.
------ ----- ---
--- ---
--- --- ---
--- --
---
(a) The net work done on raising the body is Vdgh
(b) The increase in the potential energy of the body is
-------
------
-----
------_
-------
-~- ---

------
-.
-------_.
---- --
=: 1 kg ~o
------
-------
----
---
---- --
--
-_.
--- ---
--- --
Vgh(d-d O) ----- -
------
---- --- :- 1 kg ~
(c) The potential energy of water is not changed by raising
the body
(d) The work done on the body by gravitational force is (a) (bJ (eJ
equal and opposite to the work done by the hydrostatic Fig. 10.82
forces so that the net work done is zero (a) The tension in the string in (b) is 10 N
II. A wall of length I supports (b) The tension in the string in (b) is 7 N
water to a height H as (c) The reading of the scale in (b) is 17 N
shown in the figure. Choose (d) The reading of the scale in (c) is 24 N
the correct statement(s). H
14. An open container having liquid
Take p as the density of
of density p moves horizontally
water. -
with acceleration a as shown in ::~
(a) The force exerted by Fig. 10.80 Fig. 10.83. Free, surface of this :-:A~'~:",:~_:~_~,,~::::_~:_~_J
water on the wall per liquid makes angle e with _:_:_:_:_:_:_:_:_
umt°l ho
engt IS l H'
2: pg horizontal. A is a point whose :::::::::::::::::
horizontal, vertical --------
and :-:--::::_:_:_:---:::_:_:
perpeddicular distance from free - - - - - - --
(b) The force exerted by water on the wall is ~ pgH2[
surface is 'I ,'2
and h. Ifpressure at
Fig. 10.83
point A is P then:
(c) The point of application of the resultant force acts ~ H
(a) P=Po +i2pg (b) P = Po +i2pa
below the free surface (c) P=Po +hp~a2 +g2 (d) None of these
(d) The point of application of the resultant force acts ~
:::::::f~I::::iI:
IS. A light cylindrical tube T
below the free surface of length I and radius r -:-::::::::S: B
containing air is inverted

~~~~~~~~~j~~~~~ i~~ ~iti


-~~---~

12. A cylindrical vessel of90 em ~~-~---­


--~----
in water (density. d). One
-======='
- --- ---
height is kept filled upto the -- ~----~.

--
--- -----
----- end of the tube is open and
brim. It has four holes 1, 2, 3'
and 4 which are respectively
-=======" 4
---- ---_ .
_~~~~~~~. 3
the other is closed. A
block B of density 2d is
---------
:-:-:-:-:-:-:-:-:-:-_ r :-_-:-:-:-:-::::-:
-----
::_::_::_::_;:_;;:_::_::_::_::_::::_::_:::;_::_::_::_::_::_::
- ------ 2
-=======.
at heights of 20 em, 30 em, ---- ---_.
--- ---
-------_.- kept on the tube as shown :::::::::::::::::::::::::=:::::"=::'::j:=
40 em and 50 em from the in the Fig. 10.84. The tube -:-:-:-:-:-:-:-:-:-:-:-:-:-:-:-::::-:---:::-
horizontal floor PQ. The Fig. 10.81 stays in equilibrium in the Fig. 10.84
water falling at the maximum position shown. (Assume
horizontal distance from the vessel comes from the hole the atmosphere pressure is to be Po)' Pick up the correct

n;,[
number marked. statement (s).
(,) 4 (b) 3
(a) The volume ofblock B is
(e) 2 (d) I
13. Three identical blocks each of mass m = 1 kg and volume
(b) The volume of block B is 21[;2 [
3x 10-4 m3 are suspended by massless strings from a
support as shown. Underneath are thrce identical containers (c) The pressure of air trapped in the tube is

3)
containing same amount ofwatcr arc placed over the scales.
In Fig. 10.82 (a), the block is completely out of the water, in Po +dg (h+
Fig. 10.82 (b) the block is completely submerged but not
(d) The pressure of air trapped in , the tube is
touching the beaker, and in Fig. 10.82 (c) the block rests on
the bottom of the beaker. The scale in Fig (a) reads 14 N. Po+dg(h+~)
J6. The vc-<;scl shown in the Fig. 10.85 has two sections ofareas (a) ap = -dg
op
(b) -=dg
of cross-section AI and A 2 , A liquid of dcnsityp fills both i!z i!z
the sections, up to a height h in each. Neglect atmospheric ap
(c) ap = 0 (d) -= dg
pressure. ax ax
21 . A circular cylinder of
radius R and height H is
filled with water to a 1-':.----_
-~~~~3:-~:-~~:-:
-- -----
height ~ II. It starts H -- --- -------- 2.H
3
rotating abo ut its axis
-----..,.--
--- -::..-----. ;::.---------
-::..--
with constantly ------------ -::.:~.

increasing angular speed. Fig. 10.87


Choose the correct
alternatives.
(a) at all speeds, shape of the free surface is parabolloid
(b) the free surface touches first the brim of cylinder and
(a) the pressure at the base of the vessel is 2hpg
then the base of the cylinder
(b) the force exerted by the liquid on the base of the vessel
(c) the free surface cannot touch the base without spilling
is 2JzpgA 2
water
(e) Ihe weighl oC the liquid is < 2hpgA2
(d) the free surface touches the brim as well as base atlhe
(d) the walls oCthe vessel at the level X exert a downward same instant
force hpg(A2 - A I )on the liquid
22. A tank o f large base area is filled
17. The weight of a balloon is measured to be WI when empty with water uplO a he ight of 5m. A - : : :_- -- -- -- :_- : : :
--- ---------
and W2 when filled with air at atmospheric pressure. The ----
---
-- -------
- ---------
------ --- ---
hole of 2 em 2 cross-section is --------------
------ - ----
weight of air in the balloon is W. Which of the following --- ------ -- -
- : : :_- -- -- -- ::_-: :
made at the bottom to allow water --
-------
---- -----
-------
-------------
is/are correct? (Assume that volume of balloon material in
negligible).
to drain out in continuous stream. ------------
For this situation, choose the Fig. 10_88
(a) W2 =WI +W (b) W2 < WI +W/2 correct statements.
(c) W z =WI Cd) WI < W2 +W/2 (a) Initially at a distance of5 m below from Ihe bottom of
18. A piece of ice is floating in liquid. What happens to the the tank, the cross-sectional area of the stream is
level of liquid when all icc melts? 1.414 cm 2
(a) Level remains same if liquid is water (b) Cross-sectional area of the stream at any distance
(b) Level will fall if liquid is water below the bottom of the tank depends upon the nature
(c) Level will rise if liquid is denser than water of liquid in the tank
(d) Level will rise if liquid is lighter than water (c) Cross-sectional area of the stream at any distance
below the bottom of the tank depends upon the area of
19. Figure shows a section of tube of varying cross-section
cross-section of the hole
area. let A I ,v] and p t respectively be cross-sectional area,
velocity of fluid (an ideal gas) and density ofliquid at I and (d) Velocity of efflux is independent of the density of the
corresponding value at 2 be A 2 ,v 2 and P 2 respectively. liquid
ANSWERS
·8 - .. "~.~.' ~'.".[-} I. (a), (c) and (e); 2. (a), (c) "nd (d); 3. (b) "nd (c); 4. (b)
~~ ~ ~OO~~ ~~~OO~~&~OO
and (d); 9. (a) and (c); 10. (b) and (c); l1.(a), (b) and (c);
Fig. 10.86 12. (a) and (b); 13. (b), (c) and (d); 14. (a) and (c); IS. (a)
(a) Alv] < A 2v 2 (b) A]v] =A 2 v 2 "nd (c); 16. (a), (b), (c) and (d); 17. (b), (c) and (d); 18. (n)
(c) A]v ]p ] =A 2v 2P 2 (d) v J =v 2 and (c); 19_ (a), (c) and (d); 20. (a) and (c); 21. (a), (b) and
20. Pressure gradient in a static fluid is represen.ted by (Z (c); 22. (a), (c) and (d).
direction is vertically upwards, and x-axis is along
horizontal, d is density of fluid):
FLUID MECHANICS 513

[C) AssertionMRcason Type Questions the difference in the height of the liquid at the eentre of the
vessel and its sides (in cm).
(.) If both A and R ",c true and R ;s the correet
3. Water is flowing in a tubc as shown in Fig. 10.9 1. Specific
explanation of A .
gravity of the liquid present in U-tube is 12. Velocity of the
(b) If both A and R
explanation of A.
".
true but R ;s not correct water at point A is ../20.2 mls. Area of cross-section at
different points are mentioned in the figure . Find the
(c) If A is true but R is false. velocity of the water (J') at 4 cm 2 outlet in mls.
(d) If A is false but R is true.
(e) Ifbolh A and R are false.
I. (A) : In laking jnto account the fact that any object which
,6C,' Ir
~
4cm'

l&C1fl' c;.~s
floats must have an average density less than that of -foA· \
water, during world war I, a number of cargo vessels
are made of concrete.
(R) : Concrete cargo vessels were filled with air.
Fig. 10.91
IAllMS 20071
4. A cylinder of unifonn cross-section is
2. (A) : Buoyancy is a conservative force.
(R) : According to Archimedes principle buoyancy is
immersed into two liquids as shown in p
Fig. 10.92. Equal volume of cylinder is
-
equal to the weight of displaced liquid. submerged in both the liquids and -
3. (A) : When a body floats such that its parts arc immersed density o f cylinder is p. Acceleration of 2p
into two immiscible liquids then force exerted by the cylinder at the shown instant isg/K. -
liquid / is of magnitude P tVlg. Where equal volume of cylinder in two
Fig. 10.92
(R) : Total buoyant force liquids. Find the value of K .
=P IVI8'+P 2V2g· 5. A cylindrical vessel of height 500 mm has an orifice (small
4. (A) : Pascal' s law is the working hole) at its boltom. The orifice is initially closed and water
principle of hydraulic lift. is filled in it upto height H. Now the top is completely
sealed with a cap and the orifice at the bottom is opened.
thrust
(R) : Pressure = - - Some water comes out from the orifice and the water level
area Fig. 10.89
in the vessel become steady with he ight of water column
S. (A) : A hydrogen fil led balloon stops rising after it has being 200 nun. Find the fall in height (in mm) of water level
attained a certain height inlhe sky. due to opening of the orifice [take atmospheric pressure
(R) : The atmospheric pressure decreases with height and = J OS N/m 2, density of water = IOOOkglm J andg ::: IOm/s 2.
becomes zero when maximum height is attained. Neglect any effect of surface tension).
A NSWERS 6. Water level is maintained in a cylindrical vessel placed on
horizontal floor up 10 a fi xed height ' H'. A tiny hole of area
, A' is punched in the side wall at a height from the bottom
J. (.) 2. (b) 3. (d) 4. (b) 5. (0)
of the vessel equal to y. The emerging stream strikes the
ground at a horizontal distance y(* 0) from cylinder. To
(D) I nteger Type Questions maintain the level in the vessel at' H', the rate (volume/sec)
I . A fixed cylindrical lank of addition of water is A ~2gH . Find n
having large cross-section
area is filled with two h p 7. A homogeneous solid cylinder of
"
liquids of densities p and .a length L(L < H / 2~ cross sectional
2p and in equal volumes area A / 5 is immersed such that it floats
as shown in the Fig. with its axis vertical at the liquid-liquid
10.90. A small hole of interface with length Ll4 in the denser
area of cross-section Fig. 10.90 liquid as shown in Fig. 10.93. The
2
a =#em is made at heighthl2 from the bottom. Find thc lesser density liquid is open to
area of cross-section of stream of liquid in em 2 just before atmosphere having pressure Po' If the FIg. 10.93
it hilS the ground. density of solid is given by kd then find
2. A liquid is kept in a cylindrical vessel which is rotated the value of 4k.
along its axis. The liquid rises at the side. If the radius of the 8. A vertical capillary with inner diameter 0.50 mm is
vessel is O.05m and the speed of rotation is 2 rev/sec. Find submerged into water so that the length of its part
protmding over the water surface is 25 mm. The radius of required for a transverse wave to travel from one end of the
curvature of the meniscus is xx 10-4 m. Find x to nearest rope to the other is ;.. , then value of n is :
integer. (Surface tension of water ::: 73x 10-3 N/m) "nO)
9. Water is being poured in a vessel at a constant volume rate
ANSWERS
of"(1", There is a small hole of area "a"at the bottom of the
tank. If the maximum level in the vessel is proportion to 1.1 2. 2 3. , 4. 2 5. 6 6. , 7. 6
8. , 9. 4 10. I 11. 8 12. 2 13. 4 14. 3
a"
- , then find the product nul. 15. 2
am
10. A cube of wood supporting 200 gm mass just floats in lEI Match the Columns
water. When the mass is removed, the cube ises by 2em.
What is the side of cube ? (in 10-1 m) 1. A cube is floating in a liquid as shown in Fig. 10.96. Match
the following:
11. There are two liquids of different densities. A solid body
can be completely submerged in liquid 1, by applying L36N I I
downward force and the same body can be completing
submerged in liquid 2 by applying 0.82 N in other liquid.
These liquids are mixed in volume proportion
----------
------- -- --
-----------
Vliquid I:Vliquid 2 = 4:x. For what value of x, the solid can be -----------
----------
- -
completely submerged in mixture liquid by applying Fig. 10.96
downward force of I N.
Column - I Column - II
12. Water is flowing in varying crosNection pipe. Area of cross·
section 1, 2 and 3 are 1 cm 2,2cm 2 and Acm 2 respectively. (a) If density of liquid (P) Increase
Water levels are shown in different vertical tubes. Speed of decreases x will
water at cross-section 3 is l mls. Value ofx is : (b) If size af cube is (q) Decrease
increased x will
(e) If the whole system is (r) Remain same
accelerated upwards x
will
(d) If density of cube is (s) First increase then
3 2 increased x will decrease
Fig. 10.94 (I) None of these
13. A small hole is made at the bottom of a symmetrical jar as
2. An ideal liquid is flowing in a tube as shown in Fig. 10.97.
shown in Fig. 10.95. A liquid is filled into the jar upto a
Area of cross-section at points A,B and C are AI' A2 and
certain height. The rate offall ofliquid is independent of the
A3 respectively (AI >A3 >A 2 )· V1,V2 ,>V3 are the
level of liquid in the jar. Then the surface of jar is a surface
velocities at the points A, Band C respectively. Match the
of revolution of the curve y::: k xn • the value of n is :

fOll0:J:~, ~:~
:::::~'::::::. ~:: h2 .............
[):::
.............. .:::::\::::.

~m ~~~~~~~ :~::::: :Z=iiiiii:r:,


Fig 1097
Fig. 10.95
Column· ] Column - II
14. Orie end ofa glass capillary tube with a radius r= o.OOScm
is immersed into water to a depth of h = 2 cm. The gauge (a) h is (P) Less than h3
pressure required to blow an air bubble of the same radius (b) h is (q) More than h
out of the lower end of the tube isxx 103 Pa. Value of x is : (e) V is (r) Less than V
(T=7xlO- 2 N/mandpo = 10 5 Pa)
(d) V, is (s) More than V
15. A uniform rope with length Land mass mis held at one end
(I) The maximum value
and whirled in a horizontal circle with angular velocity 01
amongst Ihe three
You can ignore the force of gravity on the rope. The time
velocities
515

3. The spring balance A reads 2 kg . slide it along its foundation. Also, there is a certain torque
with a block msuspended from it. tending to overrune the dam about the bottom 00'.
A balance Breads 5 kg when a
An5wer Ihe following questions.
beaker with liquid is put on the
pan of the balance. The · two I. As shown in Fig. 10.99 (a), ( I) and (2) are two rectangular
balances are now so arranged that strips I)n the wall, each having Ihe same width 'w' and the
the hanging mass is inside the same height dy. Strip (2) is at a greater depth from the
liquid in a beaker as shown in the surface of water than strip ( I).
Fig. 10.98. In this situation (g is (a) Equal force are exerted by waler on strips (1) and (2).
accelemtion due to gravity). This is because total force on the dam wall will be
Match the following: distribUlcd such that elements (strips) ofequa\ area, as
Fig. 10.98
in Ihe present case, will experience equal forces
(b) Force exerted by water on strip (I) is more than that
Column - I Column - II exerted on strip (2) because water in the upper part has
(a) Reading of balance A (P) More than 2 kg a greater tendency to move forward
(c) Force exerted by water on strip (2) is greater Ihan that
(b) Reading of balance B (q) Less than 5 kg
exerted on strip (I) because of a greater pressure
(c)
i (Tension in the string (r) Less than 2 kg difference on the two sides of the wall as the depth
increases
connecting balancing A (d) Since the water in the lower part has to support the
and the block) weight of upper water, it has no tendency to apply
(d) i (Normal reaction of (s) More than 5 kg
force on the dam wall. As a result, force exerted by
water on strip (I) is more than that exerted on strip (2)
2. Total force exet1ed by water on the dam is (take 'p' as the
the pan of balance B on
density of waler and g as acceleration due to gravity) :
the Beaker)

ANSWERS (a) ~pgWH (b) ~~PgwH


1. a~ Pi h-+ p; c-+ r; d-+ p
(c) pgwH
2. a -+ q; b--.-) p; c -t r; d -+ s,t
3. a --+ q,r; b-+ p,5; c -+ q,fj d-+ p,s 3. Net torque about the bottom (00') of the dam wall is:

[FJ Linked Comprehension Type (a) jpgwH (b)


2
~PgwH2 (c) 3
iPgwH (d) ~pgwH3
Paragraph - 1 Fig. 10.99 (b) shows a dam of height 50 m in which the
As shown in Fig. 10.99 (a), water is fill ed 10 a height 'H' upper edge of the dam runs a long the surface of water. It
implies that water behind the dam is filled to the total height
~,~
......... 6I
of the dam. ABeD is a rectangular gate in the dam which is
2 m high and 6 m wide. The gate is hinged along a
O{
~'" horizontal line through its centre as shown.
A

I
50m
D
'- 6m----.."
B

Fig. 10.99 (a)


behind the vertical face of a dam of width 'w'. Due to
pressure difference between the two sides of the dam wall, a
Fig. 10.'9 (b)
resulatant horizontal force acts on the dam that tends to
4. Torque will exerted on the gate about the hinge due to the
force that water exerts on the gate. Magnitude ofthis torque h
is : (take density of water = 10 3 kg/m 3 ; g = 10m/5 2 )
(a) 4x 104 N~m
(e) 4.5x 10 3 N-m
(b) 7.5x 103 N-m
Cd) 8x 105 N-rn
I~m
---
LII
:~~~~~~~~~
----

Paragraph - 2
Fig. 10.100 (a) shows a siphon in action. A siphon is, in
fact, very convenient for removing liquids from containers.
However, it works only if the tube is filled with liquid so
that there could be a pressure difference along the tube. If
there is no liquid, pressure at all points has the same value
and so liquid will not flow. In Fig. to. 100 (a), the liquid --- -- - -
------
B - 0 Fig. 10.100 (b)
solem
(atmospheric pressue = 10 5 N/m2, density of liquid = 1.2

1~~~i~f~i~~11
glee)
(a) 12.4 m (b) 6.5 m
(e) 7.2 m (d) 8.3 m
175 em
Paragraph - 3
Fig. 10.101 (a) shows a V-tube ofunifonn cross-section
that is open at both ends. It is partially filled with water.
Now a liquid of density' d I' is poured into the left ann. The
D liquid does not mix with water. It is observed that height of
liquid surface in the left ann is 2 em more than the height of
- --- - - water surface in the right ann, as shown in Fig. 10.101 (b).
Fig. 10.100 (8) Take density of water as 1000 kglm 3 and answer these
questions.
flowing through the siphon has a density 1.2 glee. Both the
containers are open to air and the siphon tube has a uniform
diameter. Height of point B above the surface of liquid in I ---
the upper container is 50 cm and depth of the surface of - --
-- :
liquid in the lower container below the point C is 175 cm. --
~
---- --:
--
-- :
:
---- --
----
Take g = 1000cmls 2 and answer the following questions.
5. Pressure difference between points A and Dis:
(a) zero (b) 1.0l x 105 N/m2 la) (b)
(c) 1.5x105 N/m2 (d) 4.5xl0 5 N /m2 Fig. 10.101

6. Pressure difference between points Band Cis: 9. Density of the liquid, d I is :


(a) 1.0lx 10 5 N/m2 (b) zero (a) 1200 kglm 3
(c) 1.5x 10 5 N/m2 (d) 4.5x 105 N/m2 (b) 800 kglm 3
7. Speed of the liquid as it flows out of the lower end of the (e) 1000 kg/m 3
tube is : (d) 1400 kglm 3
(a) 3 mig (b) 5 mig 10. Length of the liquid column in the left ann of the V-tube,
(e) 2.5 mig (d) 4 mig i. e.,' y' is (use density of liquid • d I' as obtained in Q. 9) :
8. Fig. 10.100 (b) shows another siphon arrangement. (a) IS em (b) 8.5 em
Maximum value of h, so that, siphon action does not stop (c) 12 cm (d) 10 em
and the liquid continues to flow, will be nearly:
11. Let us now shield (protect) the left ann
Shield ..y. 2 2
0)2 L2
(b) -Lh-ro-
r.=;J
(0)
from any air motion and blow air across 2g Dg
the lOP of the right ann with such a !~I-----h
speed that heights of lap surfaces in the
left ann and the right ann are the same,
"
---- (e)
h 3 (L+2h)2old
4gM
(d) ~L":ro2
--
-- :=
as shown in Fig. 10.102. Speed ofair,
~~------ifI
15. Difference in height in both situations:
density of air being 1.25 kglm 3 , will be
(a) depends on density of liquid but not on area of cross-
nearly:
Fig. 10.102 section of the tube
(a) 6 mI,
(b) depends on area of cross-section of the tube but not 011
(b) 12 mI, the density of liquid
(e) 18 mI, (c) depends on density of liquid and also on area of cress-
(d) 16 mI, section of the tube
12. Go back to the situation of Fig. 10.101 (b), If YOll pour (d) depends neither on density of liquid nor on the area of
water into the right ann, what should he the length of cross-section of the tube
additional water colutrul so that heights of top surfaces in Paragraph - 5
the left and the right arms afC the same? A cylindricalobjectofunifonn densityd, axial length Land
(a) 6 em uniform diameter D floats in a liquid of density 1.2 glcc.
(b) 8 em Axial length of the object is perpendicular to the liquid
surface. When the cylinder floats at rest, it is obsetved that
(e) 4.5 em
(d) This is not possible for any length of the additional j of its length is submerged inside the liquid. Take g as the
water column acceleration due to gravity and answer these questions.
Paragraph - 4 16. If the buoyant force acting on the object is 'S' and the
weight of the object is W, then, in the given situation:
A U-tube of uniform
cross-section contains a 3
(a) B = W (b) B=ZW
liquid of density 'd', T
~ -T
Diameter oflhe tube is D. h ""- h

1- -1 (d)
Height of liquid in the
left arm and also in the --€"€i'i~"f"i'fii€-f::"€-:-:"€"€"€~- 17. Density of the cylindrical object is. d
right arm is h and length L (a) >0.92 glee (b) = 0.92 glee
of horizontal portion of Fig. 10.103
(e) = 0.8 glee (d) <0.8 glee
the tube and hence the horizontal column of liquid is L.
18. Suppose that the cylindrical is slightly displaced so that it is
Consider the following situations. a distance 'x' above its equilibrium position. Net force
(A) The tube is given a uniform acceleration 'J' towards acting on it will be (all quantities are in CGS system) :
left.
(a) 0.5.n' xg (b) 0.2.n'xg
(B) The tube is mounted on a horizontal table that is made
to rotate with a uniform angular speed rowith one of the (e) ~' xg (d) 0.3.n'xg
arms (left or right) on the axis of rotation.
Take 'M' as the total mass of the liquid and 'g' as the 19. If we push down a floating object from its equilibrium
acceleration due to gravity and answer these questions. position and release it, it oscillates about the equilibrium
position. Suppose that the cylindrical object is pushed
13. Difference in height between the liquid columns in the
down from its equilibrium position by such a distance that
vertical anns (left and right) in situation (A) will be :
force of buoyancy on it will be 1.25 times its weight. The
(a) fllh (b) L'h'd/ object is then released. If perfonns Simple Hannonic
fi Mg Motion with a time period nearly equal to :

(e) L/
g
(d) LjD
gh
(a) O
·8nH (b) 1.6.H
14. DifTt:rence in height between the liquid columns in the
vertical arms in situation (8) will be :
(e) 2.H (d) 2.4.H
20. Amplitude of these oscillations will be (Ignore damping 26. When A is taken out, what will be the reading of E ?
due to fluid friction) : (a) 2.5 kg (h) 2 kg (c) 1.5 kg (d) 3 kg

(a) i4 (h) I
8 Paragraph - 8
A circular hole of radius rat
i6 I
(c) (d) 12 the bottom of 811 initially full ------
--- -------
------------
-------
- ---- ----
water container is sealed by ------
-- ----- --
----
------------
a ball of mass m and radius
Paragraph - 6
R(R > r~ The depth of the
A wooden cylinder of diameter 4r,
height h and density p/3 is kept on a
h'l A:t~~-4;-~=:t~
--
:h2
-
::=
p/3 h
water is now slowly
reduced. p is the density of
hole of diameter 2r of a tank, filled Flg.l0.l0G
Co-o-_ .-- water and V is the volume of
with water of density p as shown in
the figure. The height of the base of
H
I -
:-:-:-
:=:=tF
2,
-P:
::=: the ball inside water initially.
27. When the water level is h. the force of buoyancy exerted on
cylinder from the base of tank is H. Fig. 10.104 the ball is :
(liT-JEE 2006( (a) <Vpg - m·2pgh (b) >Vpg _ ftr 2pgh
21. If level of liquid starts decreasing slowly when the level of
(d) Vpg
liquid is at a height hi above the cylinder, the block just
starts moving up. Then, value of hi is : 28. The height of water ho at which the ball starts rising is:
(a) 2hl3 (b) 51114 (a) ~ _.....!!!....- (b) 111
ltl'2 1tT2p nr2p
(c) 5M3 (d) 5M2
22. Let the cylinder is prevented from moving up, by applying a (c) , (d) none of these
force and water level is further decreased. Then, height of ,,'
water level (h2 in figure) for which the cylinder remains in 29. When the water level is h and the ball is not rising, the
original position without application of force is : reaction force from the rim of the hole is :
(a) hl3 (b) 4hl9 (a) mg - pgV (b) mg-pgV+ftI'2pgh
(c) 2M3 (d) h 2
(c) 1IIg + pgV (d) mg-pgV-nr pgh
23. Ifheight 1i2 of water level is further decreased, then:
Paragraph - 9
(a) cylinder will not move up and remains at its original
position A dam of width w = 10m is filled with water up to height
H = 22mas shown in the figure. As we go down in the dam,
(b) for h2 =h/3, cylinder again starts moving up
pressure increases, so the thickness of the wall also
(c) for h2 = h/4, cylinder again starts moving up increases from top to bottom. There is a stony glass window
(d) . for h2 =h/5; cylinder again starts moving up of radius r = 10 cm at a height h = 2 m from the bottom as
shown in the figure. The other side of the glass window is
Paragraph ~ 7
open to the atmosphere.
Block A in the figure hangs from a spring
balance D and is submerged in a liquid C
contained in a beaker B. The weight of
beaker is I kg, the weight of the liquid is B - - -----
-------
1.5 kg. The balance Dreads 2.5 kg and
balance E reads 7.5 kg. The volume of the
-----
---------
- ------
--------
------
H
block is 0.003 m 3 . E
24. What is the density of the liquid?
::::::: '§lL-----'::=..-
w
(a) 1666.7 kglm'
Fig. 10.105 Fig. 10.107
(b) 1500kglm' 30. The net force on the glass window due to fluid pressure on
(c) 2500kglm' both sides is :
(d) 1750 kglm' (a) nkN (b) 2nkN (c) nN (d) 2nN
25. When A is taken out, what will be the reading of D? 31. The net force on the wall due to fl uid pressure on both sides
is:
(a) 7.5 kg (h) 2 kg
(c) 3.5 kg (d) 2.1 kg (a) 44000 kN (b) 12100 kN
(c) 24200 kN (d) 48400 kN
32. Find the pressure at the bottom of the dame: 37. Initial speed of the water at point P is (approximately) :
(a) 2.2x 105 Pa (b) 1.2x lOS Pa (a) 10.6 mI, (b) 36.6 mI,
5 (e) 46.6 mI, (d) 26.2 mI,
(c) 3.2x 10 Pa (d) None of these
38. The height of the water level from the bottom at whieh flow
Paragraph - 10 of water stops is :
The vessel in the figure (a) 0.7 m (b) 1.74 m
contains water as
(c) 2.0 m (d) The value does not exist
shown. The space
between water and top Paragraph - 12
of tube is filled with
Three pipes of V3~
vacuum. The base area
diameter d 1 = 4 em, \ .':w. '\. d "" 6 em
is A2 = 100 cm 2 and d 2 = 5cm and d 3 =6 ~ '\
3

the tube area A I = 10 ~-100 cm 2 em respectively 2 4


em 2 (take p w "" 1000 Flg.l0.l0S steadily deliver water ~~~ d4 ",, 9cm
to a large exit pipe of 1
kglm 3, g = lOmls 2) diameterd 4 = 9cmas
shown in Figure. The V1
/C d,,,,4cm
For this arrangement answer the following questions.
velocity of water Fig. 10.110
33. Find the total force acting on the bottom of the vessel.
through pipe 2, V2 is 5 mis and the exit flow rate
(a) 100 N (b) 150 N Q4 =45xm 3/hr. It is known that increasing Q 3 by 20%
(e) 600 N (d) 300 N
would increase Q4 by 10% [Assume all values should exist
34. What is the weight of the water? steady flow]
(a) lOON (b) ISON 39. The velocity of water through pipe I is nearly:
(e) 300 N (d) 600 N (a) 0.69 mI, (b) 7.8 mI,
35. Mark out the correct statement. (c) 11.7 mis (d) None ofthese
(a) Net force acting on the base of vessel >weightofliquid 40. Flow rate through pipe 1 is :
inside the vessel
(a) 16.8 m 3/hr (b) 451tm 3 /hr
(b) Net force acting on the base of vessel =weightofliquid
inside the vessel (c) 80 xm 3/ hr (d) None of these
(c) Net force acting on the base of vessel =weight of liquid 41 . Initially the velocity of water through pipe 4 is nearly:
inside the vessel only if there is no atmospheric (a) 6.2 mI, (b) 7.8 mI,
pressure
(e) 5 mls (d) None of these
(d) Net force acting on the base of vessel <weightofliquid

I
inside the vessel

Paragraph. 11
A sealed vessel
contains water upto
height h = 3.5 m. 4.0m _ ___ :-=:-::._
The space above the ::~ ~~ ~ ~ ~ ~ : ~-~-t1.0m
water contains - --- ___ _~-
. p
compressed aIr at Fl .10.109
pressure 4.2Po(Po = 9 Paragraph-6 (c) 22. (b) 23. (a)
Ix lOS Pa, atomospheric pressure). The vessel connected to Paragraph-7 (a) 25. (a) 26. (a)
a tube at the bottom as shown In figure. Area of
cross-section of the tube is unifonn and small as compared Paragraph-8 (c) 28. (a) 29. (b)
to the area of the vessel. Temperature of the air remains Paragraph-9 (b) 31. (c) 32. (c)
constant.
36. Initial speed of the efflux of water is (approximately): Paragraph-lO (e) 34. (b) 35. (a)
(a) 10.6 mI, (b) 36.6 mI, Paragraph-II (d) 37. (d) 38. (b)
(e) 46.6 mI, (d) 26.2 mI,
Paragraph-I2 (b) 40. (d) 41.
IGJ Subjective Type Questions 10. Lead has a greater density than iron and both are denser
than water. Is the buoyant-force on a lead object greater
1. Will 5 litre of kerosene oil weigh more in winter or in than, lesser than or equal to the buoyant-force on an iron
summer? object of the same volume?
[Ans. In winter) [Ans. Same as Til == Vag]
[Hint: See Question III in Chapter 11 11. What will be the weight ofa weightless bag when it is filled
2. Show that the density p of water of bulk modulus B at a with water and weighed in water?
depth yin the ocean is related to the density at surface p 0 by [Ans. Zero as Wo '" Til "" Vpg so Wapp '" Wo -Th '" 0]
the relation
12. Explain why a boat rises as it enters the sea from a river?
p opo[i+(poGYIB)]
(Hint: For floating W "" Th ::: Vin ag; as density of sea water is
[Hint: Proving point (8) of § 10.1 substitute /!,P = roIDi
more, the volume of boat in watcrwill decrease, i.e, the boat will
3. Indian Fakirs stretch out for a nap on a bed of nails or rise up.]
thorns. How is this possible? 13. A cylindrical block of wood of mass m and cross-section A
[Hint: As area in lying posture is maximum, the pressure due to is floating in a liquid of density 0" with its axis vertical. It is
weight will be minimum and so they are not hurt.] depressed a little and then released. Show that the moti('n of
4. Why is a fluid container more likely to burst from the base? the block is simple harmonic and find its frequency.
[Hint: As hydrostatic pressure and hence, force is maximum at
the base]
[A", [0 2~ r,:g 1
5, When you drink a liquid through a straw, you reduce the
pressure in your mouth and let the atmosphere move the
[Hint: See § 8.4 (B) (b) with J '" f]
liquid. Could you use a straw to sip a drink on the moon? 14. What angle will the free
(Ans. No; as moon has no atmosphere, there will be no surface ofa liquid subtend with
atmospheric pressure to move the liquid.1 the horizontal whcn the liquid
in a container is moving
6. Because atmospheric pressure is 10 5 N/m 2 and the area of
horizontally with constant
a person's chest is about 0.1 m 2, the force of the acceleration a as shown in Fig.
atmosphere on one's chest is about 10,000 N (E 1000 kg). 10.lll?
Why do nof our bodies collapse? Fig. 10.111
(AilS. 0= tan - l (alg)(
IAns. As we have an open system to atmosphere thc pressurc
inside and outside our body due to atmosphere is same.] [Hint: Consider a horizontal liquid cylinder in the liquid as
shown in Fig. 10.IOS. For its translatory motionFI - F2 '" ma,i.e,
7. (a) What would be the height of water column which will
CA. - pz)A ::: ALpaor(hl -/~)g = La[as p '" hpgJor(~ - h-;)IL '"
produce a pressure of one atmosphere? (a Ig), but from figure tan e == (hI -/~)IL, so tan e'" alg(
(b) Assuming that the atmosphere has the same density
15. A piece of ice is floating in a beaker filled to the brim with a
everywhere as at sea-level (= 1.3 kglm 3) and g to be liquid of density 1.5 glcc. Will there be an overflow of
constant (= 10 mls 2), calculate the height of atmosphere water as the ice melts? Explain your answer.
on the earth. [AilS. Yes]
IAns. (a) 10.31 m(b) == Skml IHlnt: See Question xnr (a)]
8. Two identical cylindrical vessels, with their bases at the 16. Does Archimedes' principle hold in a vessel in free fall?
'''same level, each contain a liquid of density p, the area of (Ans. Archimedes' principle has no meaning in this situation, as
either base is A but in one vessel, the liquid height is h t and it is a statement about buoyant force which due to condition of
in the other h2 . Find the work done by gravitational force in weightlessness docs not exist.j
equalising the levels when the two vessels arc connected. 17. A block of wood floats in a bucket of water in a lift. Will the
block sink more or less iflhe lift starts accelerating up?
fAns. tPhA(hl _/~)z]
[Ans. It will float at .the same level as equilibrium of floating
9. A U-tube is filled with a single homogeneous liquid. The body is unaffectcd by variations in g. However, thrust and weight
both depend on g and will increase.]
liquid is temporarily depressed on one side by a piston. The
piston is removed and the level of the liquid in each side 18. A solid ball of density half that of water falls freely under
oscillates. Show that the period of oscillation is rt.j(2L1 g) gravity from a height of 19.6 m and then enters water. Upto
what depth will the ball go? How much time will it take 10
where L is the tQtallength of the liquid in the tube. come again to the water surface? Neglect air resistance and
IHlnt: See (B) (a) in § 8.4; with h '" iJ2 in Chapter 8] viscosity effects in water. (g =9.8 mls 2) (Roorkcc 1991)
[Ans. 19.6mand4s]
19. The tension in a string holding a What maximum load can the balloon lift if density of air is
solid block below the surface of a 1- 1.25 kglm 3 and that of helium 0.160 kg/m 3?
liquid (of density greater than that of Ia (An •• 7700 kg)
solid) as shown in Fig. 10.11 2 is To
when the system is at rest. What will
~~llnm 26. An object hangs from a spring balance. The balance
be the tension in the string if the registers 30 N in air, 20 N when this object is immersed in
system has upward acceleration a? Fig. 10.112 water and 24 N when the object is immersed in another
liquid of unknown density. What is the density of (a) the
(Ans. T = To{l+ (alg)}1
solid and (b) the other liquid ?
(Hint: Initially for lhe equilibrium of block, we have IAns. (a) 3)( 103 kg/m 3 (b) 0.6)( 103 kg/m l )
Th~To +mg. i.e. , To",Th -mg=V(a - p)g
27. An iron casting containing a number of cavities weighs
When the lift is accelerated up, g -+ g + a so that
6000 N in air and 4000 N in water. What is the volume of
T = V (0 - p)(g + a). Therefore (TlTo) = {(g + a)lg ))
the cavities in the casting? Density of iron is 7.87 glcm 3 .
20. If two ships happen to sail parallel and adjacent to each (Ans. 0.12 m 3)
other and if they are not far away they experience a pull to
28. An ornament made of gold and copper weighs 100 g in air.
bring them together. Explain with diagram.
Its apparent weight in water is 93.6 g. Find how many g is
fRoorkee 1993, 941
gold and copper respectively in the ornament. (Specific
(Hint: See § 10.4 (8) (e»)
gravity of gold and copper is 19.3 and 8.85 respectively.)
21. Water stands at a depth H in a tank whose side walls are [Ans. Go ld ~ 80gand copper ~ 20g)
vertical. A hole is made in one of the walls at a depth h
below the water surface. Find at what distance from the foot 29. A woodenplankoflength I
of the wall does the emerging stream of water strike the m and unifonn cross-
floor and for what value of h this range is maximum? Whal section is hinged at one end
is the maximum possible range? to the bottom of a tank as
(Ans. x:=:2./h(H-h);h~HI2 ; xrrax -H )
shown in Fig. 10.113. The
tank is filled with water
[Hint: See § 10.4 (C) (e) (3») Fig. 10.113
upto a height of 0.5 m. The
22. A tank of cross-sectional area A is filled with water to a specific gravity of the plank is 0.5. Find the angle 9 that the
height H. A hole is punched in one of the walls at a depth h plank makes with the vertical in equilibrium position
below the water surface. (a) Show that the distance x from [excluding the case 9 =001.
the foot of the wall at which the resulting stream strikes the IAns. 0 = 45°1
flo or is given by x:=:2./h(H-h). (b) Could a hole be
30. A cubical block of wood 10 cm along each side fl oats at the
punched at another depth to produce a second stream that interface between an oil and water with its lower surface 2
would have the same range? Ifso, at what depth? (c) What
cm below the interface. If the heights of oil and water
is the time taken to empty the tank if a hole of area Ao is
columns are 10 cm each and the density of oil is 0.8 glcc (a)
punched at the bottom of the tank?
what is the mass of the block? (b) what is the gauge pressure
(Ans. (b) Yes, al depth (H -h)(c) t '" (A /AoN(2H/g») at the lower surface of the block?
(Hint: See § 10.4 (C) (c) (4) and (5)] (Ans. (8) 840 g (b) 980 N/m2]
23. A lank 5 m high is half filled with water and then is fill ed to 31. A ping-pong ball has a volume V and density (1/JO)lh of
the top with oil of density 0.85 glcc. What is the pressure at water. What force would be required to hold it completely
the bottom of the tank due to these liquids? submerged under water?
IAns. 4.53 x l04 N/m2) IAns. (9/10) Vawg downwards)
24. A vertical U·tube of unifonn inner cross-section contains
32. A frog in a bowl finds that he just floats without ~i nking in a
mercury in both its arms. Glycerine (density 1.3 glcc)
column of length 10 cm is introduced into one of the anns. blue-green sea (density = 1.35 glcc). If the bowl has a radius
Oil of density 0.8 glcc is poured into the other ann until the of6 cm and has negligible mass, what is the mass of frog?
upper surface of oil and glycerine are in the same horizontal (Ans. 0.61 kg)
level. Find the length of oil column. (density of mercury is 33. A rectangular air mattress has a length 2.0 m, a width 0.50 m
13.6 glee) and thickness 0.08 m. What is the maximum mass of a man
IAns. 9.61 cm) who lying on the mattress can float on water if the mass of
25. A spherical helium filled balloon has a radiusR = 12 m. The the mattress is 2.0 kg. What is the density of the mattress?
empty balloon with ils attachments has a mass 186 kg. [Ans. 78 kg and 25 kg 1m 3]
34. A metallic sphere floats in immiscible mixture of water (c) What should be the relative heights of liquid columns
(density 10 3 kg/m 3) and a liquid '(density 13.5 x 10 3 in the two tanks for equal rate of volume flow?
kglm 3) such that its (4/S)th part is in water and (1/S)th part [Ans. (a) 112; (b) 2; (c) ~/hl = 4]
in the liquid. What is the density of metal? [MNR 1994] 41. A water pipe with internal diameter of2 em carries water at
IAns. 3.5 x 10 3 kg/m 31 the ground floor of a house with velocity 2 mls and at
35. A block of wood weighing 200 g has relative density 0.8. It pressure 2x 10 5 N/m 2 . Another pipe of internal diameter 1
is to be floated in water just fully submerged. What weight em is connected to it and takes water to 1st floor,S m above
of lead is needed (a) if the lead is on the top of wood (b) if ground. What is the velocity and water pressure at 1st
the lead is attached below the wood? (RD of lead = 11.3) floor? (Take g = 10 mls 2 .)
IAns. (a)50g(b)=S5g] IAns. 8 mis, 1.2 x lOs N/m2)
36. Water flows in a horizontal pipe whose one end is closed I 2 I 2
with a valve and the reading of a pressure gauge attached to (Hint: Pa + ipVa = PF + pgh+ '2pVF and VaAa = VFAFI

the pipe is 3 x 10 5 N/m 2 . This reading ofthe pressure gauge 42. The bottom of a cylindrical vessel has a circular hole d = 1
falls to Ix 10 5 N/m 2 when the valve is opened. Calculate the cm in diameter. The diameter of the vessel D =0.5 m. Find
speed of water flowing in the pipe. (MNR 1991) the velocity with which the water level in the vessel drops
lADS. 20 mls) in tenns of the height of this level. Also detennine the
37. Water is flowing steadily through a horizontal pipe of numerical value of this velocity for the height h =0.2 m,
d 2
non-unifonn cross-section. If the pressure of water is (Ans. v=2.J2ih;v=8xlO- 4
m1sec)
4 x 104 N/m 2 at a point where the cross-section is 0.02 m 2 D
and velocity offlow is 2 mls. What is the pressure at a point 43. A cylindrical tank with a height h = 1m is filled with water
where the cross-section reduces to 0.01 m 2 ? upto its brim. What time is required to empty the tank
[Ans. 3.4 xi04 N/m 2J through an orifice at its bottom if the cross-sectional area of

38. A liquid is kept in a cylindrical vessel which is rotated the orifice is 460 that of tank?
along its axis. The liquid rises at the sides. If the radius of
IAns. 3 minuteJ
the vessel is 0.05 m and the speed of rotation is 2 revIs, find
44. A steel ball floats in a vessel with mercury. How will the
the difference in the height of the liquid at the centre of the
volume of the part of the ball submerged in mercury change
vessel and its sides.
if a layer of water completely covering the ball is poured
IAns. 2 cm)
above the mercury? If P w' P sand Pm are the densities of
[According to Bernoulli's principle P+ ~pV2 = const!. At the water, steel and mercury, find the ratio of these two
sides the speed v(=rro) is large. so the pressure is low. But the volumes in tenns ofpw'P $ and Pm'
pressure at a given horizontal level in a liquid is same, so the Ans. Vo = I- P'/Pm]
[
Iiq1!id will rise such that ~pV2 =pgll, i.e, II =(ro 2I2g)r2.J VI I-pw1ps

45. A block of ice with an area A and a height h floats in water


39. A cylindrical tank of height 0.4 m is open at the top and has
of densityp o. What work should be perfonned to submerge
a diameter 0.16 m. Water is filled in it upto a height of 0.16
the ice block completely into water if density of ice is PI?
m. Calculate how long it will take to empty the tank through
a hole of radius 5x 10-3 min its bottom. (Roorkee 1990] [Ans. W = Agh2~o-Pl )2 l
[ Ans. t= :0 ff =46.268] 46. A rubber ball with a mass M and radius R is submerged into
a liquid of density p to a depth h and released. What height
40. Two cylindrical tanks are filled with different liquids. A
will the ball jump up above the surface of water? (Neglect
small hole is made in the side of each tank at the same depth
resistance of water and air.)
h below the surface of liquid. If the hole in the tank has area
of cross-section twice that of hole in tank E,
(a) What is the ratio PII P2' the densities of two liquids, if [An., (~nR:-+l
same mass of two liquids flows out from both tanks per
unit time?
(b) What is the ratio of volumes of the two liquids flowing
out per unit time from each tank?
Properties of Matter
[In this chapter we shall study elasticity, viscosity and If the stress is normal to the surface, it is called normal
surface tension-the characteristic properties of matter. stress. The stress is always normal in the case of change in
Elasticity is associated with all the three states of matter (solid, length ofa wire or volume ofa body. The normal stress can
liquid and gas), viscosity with fluids (i, e., liquid and gas), further be compressive or tensile according as it produces a
while surface tension only with liquids.] decrease or increase in length or volume. When the stress is
§ 11.1 [A] Elasticity tangential to the surface it is called shearing or tangential
stress. If the force producing stress is inclined at an angle to
A body is said to be rigid if the relative positions of its
the normal to the surface, the stress will have both the normal
constituent particles remain unchanged when external
and tangential components as shown in Fig. 11 . 1 (c).
deforming forces are applied 10 it. The nearest approach to a
F
rigid body is diamond or carborundum. Actually no body is
perfectly rigid and every body can be defonned more or less by F
the application of suitable forces. All these deformed bodies,
however, regain their original shape or size, when the
deforming forces are removed. Th e property of matter by
virtue of which a body tends to regain its original shape and
size after the removal ofdeformingforces is called elasticity. Normal Stress Tangential Stress Force at an angle
Ifon the removal of deforming forces the body regains its (a) (b) (e)
original shape and size completely it is said to be peifectly Ag.11 .1
elastic. On the other hand if the body does not have any Since, stress is the force per unit area, its dimensional
tendency to recover its original shape and size, i.e., it retains fonnula is [ML - IT- 2] and units are dyne/cm 2 or newtonlm 2 .
the modified form it is said to be perfectly plastic. Truly
Further, it is not a scalar as at a given point for a given
speaking, there is no material which is either perfectly elastic
force it has different values in different directions (i.e., for
or perfectly plastic and the behaviour of actual bodies lies
different cross~sections). It is also not a vector as it has no
between the two extremes. The nearest approach to a peifectly
specific (i.e., fixed) direction as at a point for different
elastic body is a quartzjibre and to a perfectly plastic body is
cross~sections it has different directions. It is one of a class of
Plltty.
physical quantities called 'tensors.
[B] Stress
When extental deforming forces are applied to a body, the Note: Though both stress and pressure are force per unit area they arc
diffe rent as (i) pressure is always normal to the area while stress
body becomes distorted. Whether the distortion is large or can be either nonnal or tangential Oi) pressure on a body is
smail, various portions of the body move relative to their always compressive while stress can be compressive or tensile
neighbourers. As a result of these displacements atomic forces (iii) pressure is a scalar while stress is a tensor.
of attraction or repulsion- set up restoring forces which resist
the alteration and tend to restore the body to its original fonn. [C] Strain
The restoringforce per unit area set up inside the body is When the forces or torques acting upon a body cause
called stress and is measured by the magnitude of the relative displacements of its various parts, a change in length,
deforming force acting on Ilnit area of the body when the volume or shape is produced. The body is then said to be
eqllilibrium is established. strained.

* The atomic forces are repu ls ive if the external forcc compresses th e body and att ractive if it produces ex pansion in the body.
The relative change in dimensions or shape of a body stretching force is removed the wire does not come back to its
which is subjected to stress is called strain and is measured by natural length. Some permanent increase in length takes place.
the ratio of change in length to the original length (linear (4) If the stress is increased further, by very small
strain) or change in volume to the original volume (volume increase in it a very large increase in strain is produced (region
strain). If there is a change in shape, the strain (called AB) and after reaching point B, the strain increases even if the
shearing strain) is measured by the angle through which a line wire is unloaded and ruptures at C. In the region BC the wire
originally normal to the fIXed surface is turned. literally flows. The maximum stress corresponding to B after
F which the wire begins to flow and breaks is called breaking or
tensile strength. The region EABC represents the plastic
behaviour of the material of wire .
Note: If the plastic region between Eand B is (a) la/'ge. the material is
said 10 be ductile and can be drawn into wires; (b) small, the
material is said to bc brittle as it will break soon after the elastic
limit is crossed.

[E] Hooke's Law and Moduli of Elasticity


Linear Strain Volume Strain Shear Strain As a result of large number of experiments Prof. Hooke
(01 (bl (c) discovered that for metals within limit of proportionality·
Fig. 11.2 stress applied to a body is proportional to resulting strain, i. e.,
Thus, being just a ratio or an angle, strain IS a Stress ocStrain or (Stress/Strain) '" E "" constt. .. .. (i)
dimensionless quantity and has no units. Like stress, it is a This law is called Hooke's law and the constant of
tensor. proportionality E modulus of elasticity or simply elasticity.
Note: Linear strain in the direction offorcc is called longitudinal strain Depending on the type of stress applied and resulting strain,
while in a direction perpendicular to forcc is called lateral strain. we have following three moduli of elasticity.
(a) Young's Modulus Y
[D] Relation between Stress and Strain
Ifa body is defonned by applying forces along
If by gradually increasing the load on a vertically one dimension only, then within limits of
suspended metal wire, a graph is plotted between stress (or
load) and longitudinal strain (or elongation) we get the curve
as shown in Fig. 11.3. From this curve it is clear that:
proportionality, the ratio of linear stress either
compressive or tensile to the longitudinal strain is
called the Young's modulus of the material of the
Il

,~
(l) When the strain is Plastic Region body and is represented by the letter Y, i. e.,
small « 2%) (i.e., in region
Y "" Linear stress
OP) stress is proportional Longitudinal strain 1 l!.L
to strain. This is the region l ~
where the so called Thus, if a rod or wire of length L and
Hooke's law is obeyed. cross-sectional area A under the action of a
The point P is called limit stretching force F applied nonnally to its face
of proportionality and Strain - suffers a change M in its length, then in
Flg.11.3 equilibrium:
slope of line OP gives the
Young's modulus Y of the material of the wire. . FJ. F
Tenstle stress"" - - = -
(2) If the strain is increased a little bit, i.e., in the region area A
PE, the stress is not proportional to strain. However, the wire .. . Change in length M
and Longltudmal stram "" 0 .. II h "" -
still regains its original length after the removal of stretching ngma engt L
force. This behaviour is shown up to point E known as elastic
Y "" Tensile stress
limit or yield-point. The region OPE represents the elastic
Longitudinal strain
behaviour of the material of wire.
(3) If the wire is stretched beyond the elastic limit E, i.e., FiA FL
=-- =-- .... (ii)
between EA, the strain increases much more rapidly and if the MiL AM

* The maximum value of stress up to which stress versus strain curve is a straight line.
525

(b) Bulk Modulus B Consider a cube of material fixed at its lower face and
When a solid or fluid acted upon by a tangential forc e F at its upper surface having
(liquid or gas) is subjected to area A. The shearing stress, then, will be
change in pressure its volume
changes, but the shape remains F F
Shearing stress =!:ll
A
= F
A
unchanged. The force per unit
area, applied normally and This she3ring force causes the consecutive horizontal layers of
uniformly to the surface of the the cube to be slightly displaced or sheared relative to one
body, i.e., pressure, gives the another; each line such as AB or CD in the cube is rotated
F
stress and the change in volume FIg. 11.5
through an anglecp by this shear. The shearing strain is defined
per unit volume strain. Now, as the angle cp in radian.1: through which a line normal to a
within the limits a/proportionality, the ratio o/uniform and fixed surface has turned. For small values ofangle,
normal stress on the sUiface of a body to the volume strain is ' stram ' ' = AA'
' '='t'
called bulk modulus oflhe material a/the body and is denoted Sheanng AB =1.x
by the teller B, i.e.,
so Shear Slress
B = Volwne stress ....(v)
11 = Shear strain
Volume strain
Only solids can exhibit a shearing as these have definite
Thus, if the volume V of a body diminishes by an amount
shape.
flV when the pressure on its surface is increased uniformly by
tlp, then in equilibrium Note: Poisson discovered thai within limits of proportionality the ralio
Volume stress = Il.p o/tlre lateral strain to /ongillidinal strain is cons/ontlor a given
material. This constant in his honour is called Poisson's ralio
Volume strain =_flV and is represented by cr. It has no units and dimensions. 11 has
V been established that theoretically - \ < cr < ( 112) while
practically no substance has been found for which G is negative,
8- Ap -VAp
- AVIV -- AV .... (iii) i.e. , practically 0 < G < (112).

The negative sign shows that with increase in pressure by Regarding moduli of elasticity (Y, Band T\) it is worthy to
/:J.p, the volume decreases by /:J.V, i. e., if /:J.p is +ve, /:J.V is - ve. notc that:
By including minus sign in its definition, the bulk modulus (I) The value of moduli of elasticity is independent of the
ilselfis a positive quantity. The reciprocal of bulk modulus is magnitude of the stress and strain. It depends only on
called compressibility, i.e., the nature of the material of the body.
Compressl'b'l'
Ilty=-I =- -I - (AV) .... (' IV) (2) For a given material there can be different moduli of
8 V Ap elasticity depending on the type of stress applied and
All the states of matter possess volume elasticity. Bulk the strain resulting.
modulus of gases is very low while that of liquids and solids is (3) The moduli of elasticity has same dimensional
very high. formula and units as that of stress since strain is
(c) Modulus of Rigidity 11 dimensionless, i.e., the dimensional formula for r,B
Within limits of p roportionality, the ratio of tangential or Tj is [ML -IT-2 ) while units dynelcm 2 or
stress to the shearing strain is called modulus ofrigidity ofthe newtonlm 2 .
material olthe body and is denoted by 11 ,i.e.,
(4) Greater the value of moduli of elasticity, more elastic
_ Shearing stress
11 - Shearing strain
r
is the material. But as oc (IIM),B oc (I/AV) and
11 oc (lIcp) for a constant stress, so smaller change of
In this case the shape of a body changes but its volume shape or size for a given stress corresponds to greater
remains unchanged. elasticity.·
A:="- c' (5) The moduli of elasticity rand 11 exist only for solids
as liquids and gases cannot be deformed along one
r dimension only and also cannot sustain shear strain.
However, B exists for all states of maUer, viz., solid,
liquid or gas.
B D
Fig. 11.6

• For a rigid body .AL,AY or t. 0; so Y ,8 or T)will be 00, i.e., elasticity of(J rigid body is il/fil/ite!
(6) Gases being most compressible are least elastic while (3) Elongatioll ill a wire by its OWII weight: If a wire of
solids are most, i.e., the bulk modulus of gases is very length L and cross-section A is stretched by a force F , by
low while that for liquids and solids is very high, i. e., definition of Y,
E so1id > E liquid > Ep AL= FL
(7) Gases have two bulk-moduli, namely. isothermal AY
elasticity Ea and adiabatic elasticity E {p' It has been In case of elongation by its own weight, F (= Mg) will act
found that at a given pressure p. at CO of the wire so that length of wire which is stretched will
be (Ll2)
Ee=p
E C
and E,="IP ,
AL = (Mg)(Ll2) = MgL=pgL [as M =pAL] .... (ii)
So,that:.1=y=":'£">1 i.e., E¢»Ee AY 2AY 2Y
£9 Cv
(4) Thermal stress: If a rod is fixed between two rigid
i.e., adiabatic elasticity is greater than isothermal supports, due to change in temperature its length will change
elasticity. and so it will exert a Ilonnal stress (compressive if temp.
(8) With rise in temperature the distance between atoms increases and tensile if temp. decreases) on the supports. This
increases and so the elastic restoring force will stress is called thermal stress. As by definition,
decrease. This in tum will decrease the elasticity, i.e.,
with rise in temperature Y, B and TJ. decrease. Usually Coefficient of linear expansion a. = Ltt.e'
this temperature dependence is not taken into account
unless specified.
(9) Moduli of elasticity are three,. viz., r ,B and 11 while
T hermal strain 't = CLde

elastic constants are four, viz., Y , R, 11 and 0. =


And so Thermal stress Ya de [as Y = slressistrain] ... (iii)
Poisson's ratio a is not modulus of elasticity as it is Tensile or compressive force produced in the body
the ratio of two strains and not of stress to strain. = YAMe
Elastic constants are found to depend on each other (5) Work done in stretching a wire: In stretching a wire
through the relations: work is done against internal restoring forces. This work is
Y=3B(l - 20) and Y=2~(l+a) stored in the body as elastic potential energy or strain ellergy.
r
Eliminating 0 or between these, we get To calculate it, consider a wire of length Land
Y = 9Bll and 0 = 3B -211 cross-section A. If a force F acts along the length of the wire
3B+~ 6B+2~ and stretches it by x, then as
[F] More about Elasticity y = stress = FIA := FL
strain i l l Ax'
(I ) If the length of a wire is doubled, the longitudinal
strain will be So work done for an additional small increase dx in length,
AL = L, - L, = 2L - L=.I:=1 dW = Fdx = (YA/L)xdx
L L, L L Hence, total work done in increasing the length by M,

r
Butas Y =(stresslstrain), = stress [as strain = I] ... (i) W= f "LYA xdx=!YA (AL)'
o L 2 L
.... (iv)
So, Young's modulus is numerically equal to the stress
which will double the length ofa wire. So work done per unit volume,
(2) As for a loaded wire:
W =!
V 2
r( ALL )2 = !2 Y (strain)2 [asv := ALand strain = M
L
]
FL ,]
[ asY = AM andA=1tr
0' W ='!'stressxstrain [asYxstrain=stress] .... (v)
So, if same stretching force is applied to different wires of V 2
same material.

[as F and Y =constt.]


Also :;, =i~x"f [asV =AL]

0' W := ! F )( M = 1. load x elongation .... (vi)


i.e., greater the ratio (Llr2), greater will be the elongation. 2 2
(6) In case of compression of a fluid, as (b) as for equal volumes of water and air under the action
. m Ap AV of same pressure, water compresses less, it is more elastic than
denSlty=p=- so, - = - - air.
V P V
Question III. What .kind of elasticity is used in (a)
But by definition of bulk modulus, i.e.,
suspension bridge (b) an automobile tyre (e) an automobile
V!>p _AV = !>P
B= - - or drive shaft (d) a coil spring (e) a water lift pump (j) rubber
AV V B
heels?
Ap !>P p'-p Ap
so, -= - or --=- [asAp=p'-p] Answer: (a) In 8 suspension bridge as there is a stretch in
p B P B the ropes by the load of the bridge, the elasticity involved is
or p' = p[l +(!>PIB)] = p[l +C!>P] [as liB = C] .... (vii) linear or tens ile.
(7) In case of bending of a beam of length L. breadth band (b) In an automobile tyre as air is compressed the
thickness d, by a load M g at the middle, depression B is given elasticity involved is volume, i.e., bulk.
by (c) In transmiuing power an automobile shaft is sheared
MgL' as it rotates, so the elasticity involved is shear, i.e.,rigidity.
0 = -- .... (viii)
(d) When a coiled spring is stretched, the defonnation of
4bd'Y
the wire of the spring is in the fonn of a twisting strain so the
and for a beam of circular cross-section of radius r and length elasticity involved is shear, i.e., rigidity.
L,
(e) As in a water lift pump, the water is compressed, the
0= MgL' .... (ix) elasticity involved is volume, i.e.,bulk.
121tr 4y (t) As the shape of rubber heels changes under stress, the
(8) In case of twisting of a cylinder (or wire) of length L elasticity involved is shear, i.e., rigidity.
and radius r, elastic restoring couple perunit twist is given by Problem 1. A bar ofcross-section A is subjected to equal
4 and opposite tensile forces F at its ends. Consider a plane
c= 1tT}r
2L .... ( x) through the bar making an angle 9 with a plane at right angles
to the bar. (Fig.l1.7)
where 11 is modulus of rigidity of the material of wire,
(a) What is the
(9) In case of a rod of length L and radius r filted at one tensile stress at this plane F F
end, angle of shear $ is related to angle of twist e by the in terms ofF, A and €I?
relation
(b) What is the
L~ =rS .... (xi) shearing stress at the Fig. 11.7
Question I, Is the following statement true or false: plane, in terms of F, A
A metal rod (Young's modulus Y) has a length L and area ande?
ofcross-section A. The work done in stretching the rod by an (c) For what value of €I is the tensile stress a maximum?
amolmt llL;s =""'"--
YA(AL) '
2L
(d) For what valueof€l is the shearing stress a maximum?
Solution : (a) As tensile stress = (nonnal force/are a)
Answer: True. Follow § 11 .1(F) (5) above. and here , A N = area = (A / cos e)
Question II. In the language of Physics which is more and F N = nonnal force = F cos S
elastic (a) rubber or steel (b) air or water? 2
Answer: In Physics elasticity is defined as the ratio of So, Tensilestress= FcosQ = Fcos e
(A lcosS) A
stress applied to resulting strain, i. e.,
(b) As shear stress = (tangential force/area)
E =str~
stram and here, Area = (A /cose)
So for a given stress and Tangential force = F sin €I
E oc IIstrain So, Shear stress = F sin9 = FsinOcos O=Fsin29
(A lcosS) A 2A
i. e., a smaller change in shape or size under the action of a
same force corresponds to higher elasticity. (c) Tensile stress will be maximum, when
So (a) as for identical steel and rubber bar undertne action cos 2 9 = max, i.e., cos9 = l, i. e., 9 =00
of same force , rubber stretches more; it is less elastic than steel.
528

(d) Shearing stress will be maximum, when i. e.,


sin 29 =max, ie., sin 29 =1, i.e.,29=90°
or e '" 45° Answer (b) However, the tension in the aluminium wire will be
Problem 2. A rod AD consisting of three segments AB, TAJ = M 2g+Teu =(M2 +M1)g
BC and CD joined together is hanging vertically from a fued (/!.L) = (4+7) x 9.8 x l
support at A. The lengths of the segments are respectively 0.1 So ID
AI (22 / 7)(0.5 x 10 ')' x 7 x IO
m, 0.2 m andO.IS m. The cross-section of the rod is uniformly
I0-4 m 2, A weight ofl0 kg;s hungjrom D. Calculate the = 1.96 x 10-3 m Answer
displacements a/points B, C and D ifYAB =2.5 x 10 10 Nlm 2, Problem 4. A steel wire ofdiameterO.8mm and length 1
YBC =4 x 10 10 Nlm 2 and YCD =lx10 10 Nlm2, (Neglect the m is clamped firmly at two points A and B which are 1m apart
and in the same plane. A body is hungfrom the middle point of
weight ofthe rod.) the wire such that the middle point sags 1 cm lower from the
Solution: By definition of Young's modulus, original posilion, Calculate the mass of the body. Given that
/!.L = FL=MgL [asF=Mg] Young's modulus of the material of wire is 2 x lOll Nlm 2 ,
AY AY Solution: As shown in Fig, 11.10, for equilibrium of mass
SoforrodABI::JL = IOx9.8)(0.1 A M,
1 2.5x1010 x 10--4
= 3.92xlO- 6 m

andforrodBC~ = 1O )( 9.8 x O.2


4)(10 1° )( 10--4
= 4.90 x 10- 6 m
D. m Mg
andforrodCDM. = 1O)(9.8xO.15 Flg.11.10
3 lxlOlO x 10- 4 --'----'1' 0 Mg =2Tsin9 .... (i)
= 14.7)(10- 6 m But from figure,
So displacement of B =6L,
sin9=-tan9=(xIL) .... (ii)
=3.92 x 10-6 m 10 kg

displacement of C = /!.L I + I!.L,


Fig. 11.8 and by definition of Young's modulus,
T =YA /!.L =YA [(L' + x' )112 _ LF YAx
, .... (iii)
=8.82 x 10-6 m L L 2L2
and displacement of D =ALI + AL2 + l1L3 So substituting the values ofsin 9 and T from Eqns. (ii) and
= 23.5 x 10-6 m Answer (iii) in Eqn. (i), we get
Problem 3. Calculate the elongations
, M= -YAx-
3
Mg=2x YAx x.!, i.e., .... {iv)
of (a) a copper wire of1.4 mm diameter 2L' L gL'
and (b) an aluminium wire of I mm
diameter as shown in Fig. 11.9, if Young's Now as here2L=1 m, x=lcm=10- 2 m
modulus for copper and aluminium are and A = nr2 = 11:(0.8/2) x 10-3 f = nx (4 x 10-4)2 m 2
II x 1010N1m2 and 7xl0 10 Nlm2 Il 2
respectively. So M = 2xlO X 11:(4 x 10-4)2 x (10- )3 kg=82kg ARs.
Solution: By definition of Young's 9.8 x (112)'

modulus, 6L = ~~
ASfrOmEqn' (iV)'X=L(~;)") soe::'I=(~!)1/3
Fig. 11.9
Note:
(a) As tension in copper wire will be due to load of7 kg,
i.e., Teu =Mt g=7xgN Problem S. A wire of cross·sectional area 4 x 10-4 m2 •
So (/!.L) = 7x9.8x0.5 modulus ofelasticity2 x 1011 N m - 2and length 1m is stretched
Cu (22 17)(0.7 x 10 ')' xllxlOlD between two vertical rigid poles. A mass ofl kg is suspended at
its middle. Calculate the angle it makes with horizontal,
Solution: Follow solved Problem-4 Solution: According to given problem, for vertical
eqUilibrium of stone,
,;ne =e= [ My; J'13 Teose =mg .... (i)
and for circular motion of
e I x IO
"3 - 1 d' stone in a horizontal plane,
i.e., Answer
=[2xlO il x4xlO-4 ] - 2oora Ian
T sin e = mRoo2 .... (ii)
Problem 6. A copper wire o/negligible mass, I m, length Now as tension in the
and cross-sectional area IO -6 m2 is kept on a smooth wire is provided by its
horizontal table' with one end fIXed. A ball of mass 1 kg is elasticity,
attached to the olher end. The wire and the ball are rotating
y= TL
with an angular velocity 0/20 rad I s. If Ihe elongation in the AU
wire is 10-3 m, obtain the Young's modulus. !fan increasing
the angular velocity to 100 rad / J; the wire breaks down,
obtain the breaking stress.
so U =-,-
TL .... ( ... )
wy
111
mg
Solution: According So substituting the value Ag.11 .12
to given problem, for ofT from Eqn. (i) in (iii),
vertical equilibrium of L

(~~:~~~----!--~
ball, U = mg -;c"---
1tf2 y cosS
R=mg .... (i)
And for motion of ,,_ _ _ _ _ _ _;;;m':.;.f = _ __ _ --'O".5<.x".9"'."8"x-"0,,.8'-,'"_ _ _
mass m in a circle of 3.14)( (0.35)( 10- 3 )2 )( 7)( to lO )( 0.0872
radius r at angular Fig. 11.11
i.e., 6L = 1.668)( 10-3 m Answer
frequencyroin a horizontal plane, the centripetal force required
F = mnJ)' (ii) .... Problem 8. A sphere of radius 0.1 m and mass 8n: kg is
attached to the lower end of a steel wire of length 5.0 m and
Here this force is provided by the elasticity of the wire,
diameter 10-3 m. The wire is suspended from 5.22 m high
i.e., F =T= r; M. .... (iii) ceiling of a room. When the sphere is made to swing as a
simple pendulum, it just grazes the floor at its lowest poinl.
So, equating F from Eqns. (ii) and (iii), Calculate the velocity oflhesphere at the lowest position. Yfor
y = mrL002 =~m,>L,,'''iro,-' sleel"" 1.994 x 1011 Nlm 2 . IMNR1994]
(asr=L+U=LJ
AM. AM. Solution: As the length of
the wire is 5 m and diameter
y=lx(I)'x(20)' =4 x IO"N/m'
So, 2 x 0. 1 =0.2 m and at lowest
00 6) x (10 3) point it grazes the floor which is T 1
Further as wire breaks at O>max = 100 rad/s, at a distance 5.22 m from the 5.22m

Breaking force = mrro~x = 1x 1x (100)2 = 10 4 N roof, the increase in length of the


wire at lowest point
And as cross~section of wire is 10-6 m 2 • U=5.22-(5+0.2)
. Breaking force
Breakmg stress = ==:='==
Area
=0.02 m Fig. 11.13
So tension in the wire (due to elasticity)
=~= IOION/m2 Answer T = fA 6L =1.994 x lOll x 1t(5 x 10-4)2 x 0.02 = 199.41t N
10-6 L 5
Problem 7. A stone o/O.S kg mass is attached to one end and as equation of circular-motion of a mass m tied to a string
0/ a 0.8 m long aluminium wire of 0.7 mm diameter and in a vertical plane is
suspended vertically. The stone is now rotated in a horizontal (mv2/ r) = T - mgcos9
plane at a rale such that the wire makes an angle 0/85 0 with
So at lowest point
the vertical. Find the increase in the length of the wire.
2
(mv /r) = T - mg (ase = OJ
[Young's modulus o/aluminium =7xIO IONlm2; sin 85°=
0.9962 and cos 85°= 0.0872J But here r =s +0.02 +0.1 =5. 12 m
So (S",,'15. 12) ={I99.4x-S.x 9.S)
IlL = F f :r-Lr2dt = -F [ 1 ]L
i.e. , ~2 : (l 21x5. 12/8) : 77.44 so v:S.SmlsAnswer nYtanO poO nYtane (a+xtanO) 0

Problem 9. A loadof31.4kg issuspendedfrom a wire of or IlL " FL FL [as(a+Ltan9)=b) ....(i)


radius 10- 3 mand density~x 19 3 kglm 3·. Calculate the change M(a + Ltan9)Y MbY
in tempe(ature ofthe wire if75%ofthe work done is.converted So 1lL = 3.14x9.8x10
into heat. The Young's modulus and heat" capacity of the
. 3.14 x (9.8 x 10-4) x (5xlO 4)(2xI011)
material of the wire pre 9.S><I0 1O Nlm 2 and 490 Jlkg K
respecOvely. [asF = Mg)
Solutl(m: As work done in stretching an elastic body per "IlL.: 10-3 m = 0.001 m Answer
unit volume is given by . Further on interchanging a and b in Eqn. (i), l1L. i. e.,
W:
V
1 stress x sttain :.!. stress
2 2 Y
2
stress]
[asY.= -stram
-.
cha~g.e in length r~mains same.

. , Note: If a'" b '" r,~ =: (FLhr.?Y) which is expected.

So w=l[MKj
2 A
V
Y Problem II, A thin uniform metallic rod of length 0..5 m
llnd radiusO. 1m rotates with an angular velocity 400rad 1 sin
Now according to given Problem H: (75/100)W a horizontal plane about a vertical axis passing through one of

m~66:~x![Mg ]2 VY
its ends. Calclilate tension in the rod and the elongation ofthe
[asH=ms.6.91 rod. The density of material of the rod is 10· kg 1 m 3 and the
4 2 ,tr2
YOling'smodulusis2 x l0 11 N lm 2 .
or (PV)S69 =.L l[Mg]' V [asm=pVJ Solution: (a) Consider an
<.bw
4 21[r2 Y element of length dr at a distance r
from the axis of rotation as shown /" ---------~
--------
t_dr --.
_-+3 __
'---------- --r---~----
69 - 3[ 31.4 X9.S ] ' x in Fig. 11 . 15. The centripetal force
-8" 1txI0-6 9.SxlOlO 9x10 x 490 acting on this element will be
i.e., il9 = _1- K = 8.33 x 10- 3 K Answer dT =dmr(ll =(pA dr)rol
120 Fig. 11.15
As this force is provided by
Problem 10. A body of mass 3.14 kg is suspendedfrom tension in the rod (due to elasticity), so the tension in the rod at
one end of a wire of length 10.0 m. The radius of the wire is a distance r from the axis of rotation will be due to the
changing uniformly from 9.S x 10- 4 m at one end to 5.0x 10-4 centripetal force due to al/ elements between x = r to x = L
m at the other end. Find the change in length ofthe wire. What L
will be the change in length if the ends are interchanged? i.e., T=f pAolrdr = .!pAol[L2 -r21 .... (i)
, 2
Young's modulus of the material of the wire is 2 x 1011 Nlm 2 .
Solution: Consider an element of So here T = ~xI04 xxxlO" x (400)'[0)' -r']
length dx at a distance x from the fixed
cnd; then by definition ofY, change in
the length of the element will be =81tX I06[ l - r2 ]N
dy= Fy:; (as here L =dx] II Answer

But here A = 1Ir2 Note: The tension in the rod will not be constant but will vary from
point to point. At the free end, i.e,r"" L, it will be min = Owhile
= n(a +xtan9)2 at the other end r'" 0. it will be max == 2J'()( I06N.
So total change in length of wire
F (b) Now if dyis the elongation in the element of length dr
IIL - f Ld - F I"
<Ix
- 0 Y - nY O(a+xtanO)2
Fig. 11.14 at position r where tension is T, by definition of Young's
modulus,
To integrate it let a + x tan e = t, so that the above equation
becomes
. ~ressl
[as stram=y
• Here the tenn 'heat capacity' is used for specific heat.
which in the light ofEqn. (i) gives Problem 13. Two rods of different metals, having the
. 1 pco2 2 2 same area of cross-section A. are placed end to end between
<!Y=Z 2Y [L -r ]dr two massive walls as shown in Fig. 11.17. The first rod has a
length II' coefficient of linear expansion ex I and Young's
So, the elongation of the whole rod modulus Y,. The corresponding quantities for second rod are
~,,2 L I tv,,2 L3 12, ex 2 and Y2. The temperature ofboth the rods is now raised
1'.L=_r-_J (L2 _r2)dr= __
r-_ _
. 2Y 0 3 Y by T degrees. (a) Find the.force with which the rods act on
each other at the higher temperature in terms of the given
Hero . 1'.L=!x 10' x (400)2(0.5) ' quantities: (b)Alsojind the lengths of the rods at the higher
. 3 . 2xlOtI temperature. Assume that there is no change in the cross·
=J x 10- m
3
Answer
sectional area ofthe rods and the rods do not bend. Th ere is no
deformation of walls.
Problem 12. A Ught rod of length 200 em is suspended Solutiori: (a) Due to heating the increase in length of the
. from the ceiling horizontaliy by means a/two.vertical wires of composite rod will be,
equal/engtl! tied to its ends. One 0/ the wires is made of steel tI'.L)1 = (LI" I +L,"2)T [asI'.L = L"e]
-and i;
of cross-section 0.1c~2 'and the other of brass of and due to compressive force F from the walls due to
cr~.sec/j<?n 0.2 cm 2.. A~o;,g the rod at which distance maya elasticity, decrease in length wilt be
weight be hung to produce (0) equal stresses in both the wires (I'.L)D = -LI +-
L,]F
-
. (b) equal strains in both the wfres? Y for brass and steel are [ Y\ Y A
2
lOx 10 11 and 20
'
x 10 11 dyne/em 2 'respectively.
'
As the length of the composite rod remains unchanged the
Solution: (a) As stresses are equal, increase in length due to heating must be equal to decrease in
~ ~ .' ~ AI QI
- = - , l,e., - . = - = - or T2 =2TJ ,,,.(i)
length due to compression, i. e. ,
Al A2 T2 Ai 0.2
Now for translatory_equilibrium of the rod,
F[!:L+~]
A Y, Y
= [Llal +L:za21T
2
Tt +T2 =W .S .- - - - - - - . ' "
8 or F
= A(L,,,, +L,"2)T ....(i)
wh;ch. in the light of Eqn. (i) [(Li YI) + (L,/Y2 )]
gives 1--- 2 m - - - I

and
TI = (W/3)
T2 = (2W/3) .... (ii)
T,
(b) As initially the
length of one rod is LI and
due to heating it increases by
L, '1
Now if x is the distjulce of , (w] (I'.LI)H = (",L,T) while Rg.11.17
weight W from steel wire, for fig. 11.11 due to compression it
rotational equilibrium of rod, decreases by
T,x=T2 (2-x) or (W/3)x =(2W/3)(2-x).
i.e., x =(4/3)m so its final length
(b) As strains are equal, L; =L, +(Al;)H - (I'.LI)C =L,[I+",T - (F/AY, )]
l=l . slress]
[as stram=y
Similarly for the other rod,
Y A,Y, A2 2 L, =L, + (I'.L 2 )H - (I'.L 2 )C =L,[I+n 2T - (F/AY2 )]
1: AY 0 I 20 10" where F is given by Eqn. (i). Answer
So -L = _1_' = . x x =I, i.e., T, = T2 .... (iii)
T2 A2Y2 O.2xlOxlO" Note: In this problem the 'englh of compOsite rod re.mains unchanged,
i.e., Lj + Li = L, + L,., but that of individual rods changes, i.e.,
So for translatory equilibrium of rod, T, + T2 =W in the Li' L, .mlL; • L,.
light of Eqn. (iii) yields I
TI =T, =(WI2) .... (iv) Problem 14. Two rods of equal. cross-sections. one of
And for rotational equilibrium of rod
copper and the other of steel, are joined to form a·composite
rod of length 2.0 m. At200C' the length of the copper rod is O.S
T,x=T2 (2-x) or (W/2)x = (W/2)(2-x) m. When the temperature is raised to 120°C the length of the
i.e., x .. lm Answer composite rod increases to 2.002 m. Jjthe"'Composite ·rod is
fIXed between two rigid walls and is, thus. not allowed to
expand, it is found that the lengths of the two component rods water is 50xlO - 6latm [take g=1O mls2 and 1 aIm
also do not change with the increase in temperature. Calculate
~ IO' Nlm'].
the Young's modulus and the coefficient oflinear expansion of
steel. Given YCu = 1.3 x 1013 Nlm 2 anda Cu = 1.6 x IO-5;oC. Solution: The variation of density with pressure is given
by p' ~ p[1 + (~pIBn But as IIp ~ hpg and (I/B) ~ C, so
Solution: (a) As with increase in temperature due to p'~p(l+Chpg) or (P'-p)/p ~Chpg
thennal expansion the length of a rod changes,
i.e., L'=L(I+a.1.9) so h=.6.Px _I_ ~_I_ x I
P Cpg 100 (SO x 10-'/10') x 10 3 xlO
So for composite rod
LS +Li: ~Lc(l+aS~e)+Ls(l+aC~e) i.e" h = 2xI0 3 m=2km Answer
or Ls +Lc =(Ls +Lc)+(Lsct s +Lc C1. c )L\.€I Problem 17. A solid sphere of radius R made of a
According to given problem, material of bulk modulus B is surrounded by a liquid in a
cylindrical container. A massless piston ofarea Afloats on the
2.002 =2 + [0.5 x 1.6 x 10-5 + 1.50. S ] x 100
surface of the liquid. Find the fractional change in the radius
or 1.5Ct s =2xIO- 5 -O.8xlO-5 , i.e., as =O.8xlO-5joC of the sphere (dRIR) when a mass M is placed on the piston to
compress the liquid.
(b) When the rods are fixed between the walls and its
Solution: As for a spherical body,
length remains unchanged,
IlR I ~V
(Ms +MC)H ~(Ms +Mc)c -~-- .... (i)
R 3 V
However, as here length of individual rods also remains
unchanged, Now by definition of bulk modulus.

(MS)H ~ (Ms)c and (Mc)H~(Mc)c B=_V.6.P i.e, I ~V I =Ilp=Mg


.6.V 'V B AB
Now as (M)f{ ~La M and (M)c ~(FLlAY)
dR Mg
So Lsa s~e ~ (FLSIAYs )andLcac~e ~ (FLcIAYc ) So - ~-- Answer
R 3AB
Dividing one by the other (a c I as) = (Ys lYe)
5 Problem 18. A uniform pressure p is exerted on all sides
SoY' s_-a-Sx Yc - _ 1.6 x 10- I 3 1013
x. x ofa solid cube at temperature tOe. By what amount should the
as 0.8x10 5 temperature ofthe cube be raiseq in order to bring its volume
back to the volume it had before the pressure was applied, ij
=2.6xI0 13 N/m 2 Answer the bulk modulus and coefficient of volume expansion of the
Problem IS. Compute the bulk modulus of water ifits material are B and 'I respectively.
volume changesfrom 100 litre to 99.5 litre under a pressure of Solution: As by definition of bulk modulus
100 atmosphe(e. Compare it with that of air. B =-V(.6.pl.6.n. with increase in pressure decrease in volume
Solution: By definition of bulk modulus,
B ~-V IIp ~- 100x (100x1.013x10 )
, of the cube will be given by,

-~V~V; [as~p ~ p] .... (i)


w ~V (99.5 100)
Now with rise in temperature due to thennal expansion
= 2.026xl0 9 N/m2
volume increases, so if.6.9 is the rise in temperature,
Now as isothennal elasticity of a gas is equal to its
~V~Vy89 [asV' ~ V(l+y~e)]
pressure,
As the volume of the cube remains constant,

V; ~Vy~e, i.e., Answer


Bw C A 2.026 x 10 9
So that -~-~ Ans.
BA C w 1.013 x 10 5 Problem 19. Calculate
i.e., bulk modulus of water is very large as compared to air. theforce Fneeded to punch'a
This means that air is about 20,000 times more compressive 1.46 cm diameter hole in a
than water, i.e., the average distance between air molecules is steel plate 1.27 cm thick (Fig.
much larger than between water molecules. 11.18). The ultimate shear
strength of steel is 345
. Problem 16. Find the depth ofa lake at which the density
ofwater is I % greater than at the surface, ifcompressibility of MNlm 2 . Fig. 11.18
...

PROPERTIES OF MATTER 533

Solution: As in punching, shear elasticity is involved, the § 11.1 (A( Viscosity and Newton's Law oeViscous
hole will be punched if Force
[F~ I ]>Ultimate shear stress In case of steady flow of a flui~ when a layer of fluid slips
or tends to slip on adjacent layer in
contact, the two layers
i.e., FI1 > (Shear stress) x (Area) exert tangential force on each other which tries to destroy the
relative motion between them. The property of a flUid due to
so (Fll )min = (3.45 x 10 8 )(2xrL) [as here A = 21trLJ which it opposes the relative motion between its different
i.e., (FII)"';. = (3.45 x 10 8 )(2 x3.1 4) layers is called viscosity (or fluid friction or internal friction)
and the force between the layers opposing the relative motion
x(D.?3 x I 0- 2 )(1.27 x 10- 2 ) =200kN Answer viscous force. A briskly stirred fluid comes to rest after a short
while because of viscosity.
Pr:oblem :20. Assuming that shear stress at the base of a y
mountain is equal to the force per unil area due to its weight,
calculate the maximum possible height of a mountain on the
earth if breaking shear stress for a typical rock is30 x 10'
Nlm2 and its density 3 x 10 3 kg/ml.
,
Solution: For a mountain of height " and base area A.
dy
weight W = Ahpg. So pressure at the base due to its own
weight will be
W
p = -=hpg
x
A F'D. 11 •19.
The mountain will exist if, As a result of large number of experiments Newton found
that viscous force F acting on any layer of a fluid is directly
hpg< Breaking shear stress proportional to its area A and to the velocity gradient (dvldy) '"
h< 30 x 10
7 at the layer, i. e.,
i, e.,
3
' 3x10 x tO F ocA dv or .... (i)
dy
or (h) ... = !Okm Answer
which is nearly the height of Mount Everest! where 11 is a constant called coefficient o/viscosity or simply
viscosity of the fluid. The negative sign shows that viscous
Problem 21. AS m long cylindrical steel wire with radius force on a liquid layer acts in a direction opposite to the
2x 10-3 m is suspended vertically from a rigid support and relative velocity of flo~ of f.1uid. The Eqn. (i) is known as
car;je~ a bob of mas; 10'0 kg at the other end. it the bob gelS Newton's Jaw o/viscous/orce.
snapped, calculate the change in temperature 0/ the wire Regarding viscosity of a fluid it is worth noting thai:
ignoring radialion losses: (l ) It depends only on the nature of fluid and is
(For the steel. wire: Young's modulus =2.1xlO ll pa; independent of area considered or velocity gradient.

Density 7860kglm 3 ; specific heat capacity = 420Jlkg.K) (2) Its dimensions are [ML -IT-I] and SI units poiseuille
(PI) while CGS'unit dyne-s!cm 2 called poise (P) with
Solution : Energy stored per unit volume is given by
I PI = 10 poise
=t stress x strain (3) Viscosity of liquids is much greater (say about 100
times more) than that ofgases
When the Lob gets snapped, this stored energy is released, i. e., 11L > TlG
thereby raising temperature of wire, i. e. ,
e.g., Viscosity of water ::::' 0.01 poise
lstress x strain x volume = ms.1.B
2
while of air::::' 200).1 poise
2
or (m8) L = (pLA)sl'>e (4) In case of liquids viscosity increases with density
2AY while for gases it decreases with increase in density.
1'>6= (mg)2 = O.00457K (5) With rise in temperature the viscosity of liquids
or Answer
2pA2ys decreases while that for gases increases.

• Velocity gradient at a layer of a fluid is defi ned as the rate of change of speed with distance in II direction perpendicular to thc flow .
534 PHYSICS FOR COMPETITIONS - Vol. I

(6) With increase in pressure, the viscosity of liquids Now if a sphere is dropped in a fluid, its weight W = ing =
(except water·) increases while that of gases is (4/3)nr 3 pg acts vertically downwards, while upthrust Th=
practically independent of pressure. (4/3)nr 3crg and viscous force 6ml'v act vertically upwar4s.
(7) From kinetic theory point of view viscosity represents
transpo;t of momentum, while diffusion and
0 0:,
Initially v = and p = so the body will be accelerated down.
Because of the acceleration, the velocity will increase an4
·conduction represents transport of mass and energy
hence, viscous force. At a certain instant when viscous force F
respectively. will balance the net downward force (W - Th), acceleration
will become zero andthe.body will fall with constant velocity.
This constant velocity is called terminal velocity. So, if vr is
tenninal velocity,
6"'1rvT =W-Th=(4/3)nr 3 (p-a)g

t
v_max Th • ~ nr3o-g

VT -----------c--,,-----
Fig. 11.20
Wallv .. O
IF.6,""
(8) In case of steady flow of a liquid of viscosity 11 in a
capillary tube oflengthL and radius runder a pressure llme or Distance
difference p across it, the velocity of flow at a (b)
distance y from the axis is given by: Fig. 11.21
=....L (r2
v
4~L
- /) .... (ii)
i.e., Vr =~r2[P~0:]g ....(v)
i.e., the profile·· a/advancing liquid in a capillary is
a parabola. From expression (v) it is clear that:
Also velocity of flow is maximum along the axis (1) v T oc r2 ,i. e., tenninal velocity depends on the radius
(= pr2/411L) and minimum (= 0) along the walls (i.e., of the sphere; so if radius is made n-fold, tenninal
y=r). velocity will become n 2 times.
(9) In case of steady flow of a liquid of viscosity 11 in a (2) It depends on the density of solid. Greater the density
capillary tube oflength L and radius runder a pressure of solid greater will be the terminal velocity. This is
difference p across it, the volume of liquid flowing why when spheres of same size but of different
per second is given by densities are dropped in a fluid, sphere of greate{
density will have greater tenninal velocity.
dQ npr 4
.... (iii) (3) It depends on the nature of fluid. Greater the density
"'dt= 8~L
and viscosity of the fluid lesser will be the terminal
This is so called Poiseume'sformu[a. velocity. This is why a given sphere falls with greater
(8] Stokes Law and Terminal Velocity tenninal velocity in air than in water.
When a body moves through a fluid, the fluid in contact (4) vr ocg,i,e., tenninal velocity also depends on
with the body is rlnlgged with it. This establishes relative acceleration due to gravity 'g'. So it wilt change with
motion in fluid layers near the body, due to which viscous change of planet or in accelerated systems, e.g., due 10
force starts '(;perating. The fluid exerts viscous force on the decrease in g on moon or in a lift accelerated down,
"b'Ody to opposeHs'motion. The magnitude of the viscous force tenninal velocity witt decrease while in freely fatting
_~ depen~s oD'the 'shape a~d-, 'size , ofthebody, its speed and the system asg =0, vT=O.
:viscdsity"'o f the fluid. Stokes 'established that if a sphere of
r
radius moves with velocity v through a fluid of viscosity t}.
By measuring the tenninal velocity of a sphere of known
radius and density, the viscosity of the fluid' in which it is
the viscous force opposing the ~otion of the sphere is
falling can be found from Eqn. (v). Actually this method is
F =6nwv .... (iv) used to detennine the viscosity of highly viscous liquids. Eqn.
This law is called Stokes law. (v) was also used by Millikan to calculate the radius ofthe tiny,
electrically charged oil drops by means of which he between the horizontal layers of water. The viscosity of water
determined the charge on an individual electron. In this case, is 10-3 poiseuille.
the terminal velocity of the drops was measured as they fell in
Solution: As velocity at the bottom of the river will be
air of known viscosity. zero, velocity gradient
[C] Critical Velocity and Reynold's Number 3
If in case of steady flow of fluids the velo'city of now is dv = 18x1O =ls-1
dy 60x60x5
gradually increased, it is found that the motion remains steady
(or streamlined or laminar) upto a certain limit. If the velocity Now as the viscous force F = llA(dvldy) is tangential to
of flow crosses this limit, the fluid particles don't follow the the area,
path of their preceding ones and the motion becomes turbulent. F .
The maximum velocity upto which fluid motion is steady, is Shear stress = _ '_ I = II dv = 10- 3 xl =:1 x 10- 3 N/m 2 Ans.
A dy
called critical velocity.
Reynold through experiments established that in case of Problem 24. The velocity of a small ball of mas~' m and
density d, when dropped in a container filled with' glycerine
motion of fluids in thin tubes, critical velocity depends on the
density (p), viscosity (11) of the fluid and radius of the tube (r), becomes constant after some time. What is the viscous force
i.e., acting on the ball ifdensity ofglycerine is d2? JMNR 1993)
Solution: When the velocity of ball is constant, it is in
v , eX:.2l
rp or .... (vi) dynamic equilibrium, i.e. , forces acting on it balance each
other
where N R is a dimensionless constant called Reynold's i.e., W - Th = F(= 6mvv)
number. It is found that for steady flow N R :s; 2000. * But Th =Vd2 g = (mld l )d2 g [asV = mldd
From Eqn. (vi) it is clear that ifll =0, Vc
=0, i.e., motion is
turbulent. Viscosity thus, appears to be a property of fluid so F=mg-mg[~:l=mg[I<:l [asW=mg] Ans.
which is responsible for orderliness and steady flow.
Question IV. The density of water is more than the Problem 25. Find the viscosity of glycerine (having
density of air. Even then clouds containing water droplets density l.3glcc) ifa steel ball 0/2 mm radius (density == 8glcc)
d0n,'tfall and continue tofloat. Explain. (Roorkee 1994] acquires a terminal velocity of 4 cmls in falling freely in the
Answer: As terminal velocity v T (eX: r2), due to small size tank of glycerine.
of droplets in the cloud, is very small, the clouds fall very Solution: We know that
slowly towards earth and appear to float in the sky. 2g(p-cr)r'
2
vT = - i.e.,
Problem 22. A boat ofarea 10m jIoating on the suiface 9 ~
ofa river is made to move horizontally with a speed of 2 mls by 2
2 980x(8-1.3)x(0.2) - 146 .
applying a tangential force. If the river is 1 m deep and the So ll =- X - . pOise Answer
9 4
water in contact with the bed is stationary, find the tangential
force needed to keep the boat moving with same veloctiy. Problem 26. An air bubble of radius 1 mm is allowed to
Viscosity of water is O.Olpoise. rise through a long cylindrical column of a viscous liquid of
radius 5 em and travels at a steady rate 0/2.1 cm per sec. If the
Solution: As velocity changes from 2 mls at the surface to
density of the liquid is 1.47 g per cc,find its viscosity. Assume
zero at the bed which is at a depth of I m,
g = 980 em/sec 2 alld neglect the density of air.
Velocity gradient = dv = 2 -0 =2s- 1 Solution: Here due to force of buoyancy the bubble will
dy 1
move up and so viscous force which opposes the motion will
Now from Newton's law of viscous force, act downward and as weight of bubble is zero, in dynamic
equilibrium, Th = F,
IFI = llA dv = (10- 2 x 10-1) x 10 x 2 = 0.02 N Answer
dy 4 3
i.e., 3'1tr crg =61tT\rv T
So to keep the boat moving at same velocity, force equal to
viscous force, i.e., 0.02 N must be applied. 2crr 2 g 2 1.47x(0.1)2x980
or l1=---=-x
Problem 23. The velocity ofwater in a river is 18kmlhrat 9 vr 9 2.1
the sutface. If the river is 5 m deep, find the shearing stress
i. e., 11 = 1.524 poise Answer

* It is established from experiments that flow is laminar when Reynold's number i~ less than about 2000, whereas above about 3000 the flow of liquid is
turbulent. In the region between 200n to 3000, the flow is unstahle, i.e. , may f.hange frmn lnmi n.a c to tu~b u!ent or vice-versa.
Problem 27. Two equal drops ofwater are/ailing through Solution: The volume (quantity) of liquid flowing
air with a steady velocity v, lfthe drops coalesced, what will be through a capillary tube per second,
the new velocity?
1tPr'
Q=-=40 ... (i)
Solution: Let r be the radius of each drop. The tenninal 8~1
velocity vr ofa drop of radius r is given by
where P: pressure difference at the two ends of tube
U'(p-cr)g
~=- -~Q I: length of tube
9 ~
r: radius of tube and
Now when two drops each of radius r coalesce to fonn a
11: coefficient of viscosity of liquid.
new drop, the volume of coalesced drop will be given by
When tubes are connected in series, the amount of liquid
~ 7tH 3 =.1 1tr 3 + .11t1'3 flowing through them is equal, i.e.,
3 3 3
_ 1tPJ'4 _ 1tP2 (rl2)4
So the radius of the coalesced drop will be Q - 8~1 - 8~1 .... (ii)
R = (2)'13 r
and PI +P2 =P=pgh ....(iii)
Hence, the new tenninai velocity of the coalesced drop
From Eqn. (H),
, 2[(2)'13 r ]'(p-cr)g
vT =:9 11 .... (ii) PI
P,
=16 or P2 =16P1,
So dividing Eqn. Oi) by (i) Substituting for P2 in Eqn. (iii),
v'
-L = (2)213 or v T = (2)2/3 v [asv r =v]Answer P,+16P, = pgh or P _pgh
Vr 1 - 17

Problem 28. A spherical ball a/radius 1 x 10-4 m and .nd P, = 16pgh


density 10 kg/m 3 falls freely under gravity through a distance
4 17
h before entering .o tank a/water. Ifafter entering the water the Now, substituting for PI or P2 in Eqn. (ii),
vel9City of the, bail does not change, find h. The viscosity of 4 4
Q= n(pgh)r =.L x n(pgh)r = 4°mL/sec Answer
water is 9.8 x 10-6 N_slm 2 . 17x8~1 17 8~1 17
Solution: After falling a height h the velocity of the ball Problem 30. Spherical particles ofpollen are shaken up
will become v =~2gh. As after entering the water this velocity in water and allowed to settle. The depth of the water is
does not change, this velocity is equal to terminal velocity, 2 x 10-2 m. What is the diameter oflargest particles remaining
in suspension one hour later?
i.e., ~2gh=~rt~cr]g Density ofpollen = 1.8 x 10 3 kg m-3
Viscosity ofwater = 1x 10-2 poise and
or 2gh=[~X(W"')' (10' -IO')X9.8]'
6 Density ofwater = 1 x 10 3 kglm 3
9 9.8x 10-
Solution: For pollen particles not reaching the bottom in
or h=20x20=20AI m Answer I hour,
2x9.8
2
v::; 2 x 10. = 10-4 mls
Note: In Ihis problem viscosity ofwaler is given extremely low which 60x60 18
is usually 10- 3 N_slm 2.
Due to viscosity effects, the particles will move with
Problem 29. Water flows through a capillary tube of tenninal velocity v given by
radius r and length I at a rate of 40 mL per second, when
}- rcr3pg = ~ rcr 3crg + 6ml'v
connected to a pressure difference of h cm of water. Another
tube of the same length but radius r/2 is connected in series 2
with this tube and the combination is connected to the same r =2g(p911V- cr)
pressure head. Calculate the pressure difference across each
tube and the rate offlow of water through the combination. Substituting,11 = 1 x 10-2 poise =10-3 Pl,p = 1.8 x 10 3 kglm 3,
PAOPEATIES OF MATTEA 537

0= 1 x IOlkg;m 3 ,g =lOm/s 2 and ~= 1~; "


mis, we have
Regarding surface tension it is worth ~oting that:
(I) It depends only on the nature of liquid and is
2 9xlO-3 10-4 1 independent of the area of surface or length of line
r = 2 ' -18- , -10-(-1.8---'1-)-'-10'3 considered.
(2) It-is a scalar as it has a unique direction which is not to
r=1.77 x IO-6 m be specified.
anddiameter=2r=3.S4)( 10 6 m Answer (3) It has dimensions [MT-2] and SI unit N/m while CGS
unit dyne/cm, so that
Problem 31. A cylindrical vessel ofarea ofcross~section
A andfilled with liquid to a height a/hI has a capillary tube of I MKS unit of surface tension =10 3 (dyne/cm).
length I and radius r protruding horizontally at its bottom. If (4) Surface tension of a liquid decreases with rise in
the viscosity a/liquid is 11. densityp and g = 9.8 m/s2 ,find the temperature and becomes zero at critical temperature
time in which the level of water in vessel falls to "2' [where interface between liquid and vapours
Solution: Let h be the height of water level in the vessel at disappear).
instant t which decreases by dh in tillie 41. . . (5) The surface tension of a liquid is very sensitive to
:. Rate of flow of water through capillary tube, impurities on the surface (called contamination) and
. decreases with contamination of surface.
V =_A(dh) ....(1) (6) In case of soluble impurities surface tension may
. dt
increase or . decrease depending on the .nature of
Further, the rate of flow as given by PoiseuiJIe formula, hripurity. Usually highly soluble salt such as sodium
. . npr4 chloride increases surface tension while sparingly
V = 8~[ . .. ..(ii) . soluble salt such as soap decreases surface tension.
Question v. Compare the state ofa soap-bubble with that
From Eqns. (;) and (Ii), -A ddh =npghr' [.,' p =pgh] of a rubber balloon in followin g respects: (a) Has each
t. 8~[
surface tension? (b) Does tlte surface tension depend on area?
dt=- 8~[A dh (c) Is Hooke 's law appl!cable?
npgr 4 h Answer: (a) The soap bubble (having two free surface 'o f
a liquid) has surface tension while the balloon being made of
Required time is obtained by integrating,
rubber has no surface tension but tension due to elasticity.
t=- 8~[A J"'dh (b) Surface tension is independent of the area of the soap film
npgr 4 hI h while the tension in the balloon is proportional to stretch, j. e.,
__ 8~[A log hi change in area. (c) As in case of bubble surface tension is
Answer independent of stretch, Hooke's law is not applicable.
npgr4 e~ However, in case of balloon as tension arises due to elasticity
§ 11.3 [AJ Surface Tension and is proportional to stretch, Hooke's law is applicable.
The property ofa liquid due to which its free sUiface tries Question VI. Explain tlte following: "A thin steel needle
to Itave minimum surface area and behaves as if it were under floats on water but when a little soap solution is carefully
tension somewhat like a stretched elastic mixed with the water tlte needle sinks. "
membrane is called surface tension. A Answer: When a needle is placed gently on the surface of
small liquid drop has spherical shape, as water, as no part orit is submerged in water, it is not buoyed up
due to surface tension the liquid surface by Archimedes' principle. There is way for
tries to have minimum surface area and for to float by Archimedes' principle. A
a given volume, the sphere has minimum Since its density is greater than that of
surface area. Fig. 11.22 water so it will sink if submerged.
Surface tension of a liquid is measured It is actually kept afloat by the
by the force acting per unit length on either side of an
surface tension of water. When it is
imaginary line drawn on the free surface of liquid, the
placed on the surface of water, it
direction of this force being perpendicular to the line and Ag.11.23
depresses the surface of water slightly
tangential to the free surface of liquid. So if F is the force
due to its weight. The stretched surface due to surface tension
acting on one side of imaginary line of length L, then
exerts a restoring force, the vertical component of which can
T = (FlL) maintain equilibrium with the weight of the needle,
538

However. when a detergent is added to water its surface


tension will "suddenly decrease and the force due to surface
tension is no longer sufficient to support the weight of the
needle and so it will sink.
Note: Same is true for mosquitoes and other insects which live on the
water surface, as when they sit on the water surface, their weight 0".
,. surface
Bubble in a liquid
1· surface
Bubble in air
2· surlace
is balanced by surface tension as explained above. However, if Fig. 11.25
some kerosene oil is sprinkled on waler due to contamination of
surface, surface tension will decrease abruptly and the water W =T x 41tr2 x 2 [as bubble in air has 2-surfaces] .... (iv)
surface will no longer be able to support the weight ofthe insects
and they will sink and die due to suffocation. This surface energy becomes evident when a film breaks
scattering the liquid droplets with considerable velocity. Here
[8] Surface Energy the potential energy of the film is transfonned into kinetic
Due to the property of surface tension, the free surface of a energy of the scattered particles.
liquid is always under tension and tends to have minimum Ie) Excess Pressure
surface area. If the area of the liquid surface is increased, work Due to the property of surface tension a drop or bubble
will be done very much similar to that done in stretching a tries to contract and so compresses the matter enclosed. This in
rubber sheet. This work is stored as potential energy in the tum increases the internal pressure which prevents further
surface and the amount of this energy per unit area of the contraction and equilibrium is achieved. So in equilibrium the
surface under isothermal condition is called 'intrinsic surface pressure inside a bubble or drop is greater than outside and the
energy' or 'free su rface energy density'. difference ofpressure behveen two sides ofthe liquid surface
To derive an expression for is called excess pressure. In case of a
the surface energy consider a drop excess pressure is provided by
wire frame equipped with a hydrostatic pressure of the liquid within
sliding wire AD as shown in Fig. the drop while in case of bubble the
11 .24. If a film of liquid ' is gauge pressure of the gas confined in the
fonned it will pull the wire due bubble provides it.
to surface tension. However, as To obtain a relation between surface 0".
the film has two surfaces (upper Fig. 11.24 tension and excess pressure for a drop (or Fig. 11 .26
and lower), each surface will bubble inside a liquid), consider a drop
pull the wire parallel to itself with a force TL (as ST = of radius I' having internal and external pressures Pi and Po
force/length). So the net force of pull on the wire due to both respectively, so that excess pressure P =(Pj - po)' If the
the surfaces will be 2TL and to keep the wire at rest it is radius of the drop is changed from r to (r + dr)
to
necessary to exert a force F the right such that
Workdone = F dr= (pS)dr [asp=FISj
F =2TL .... (i)
or W=41t,.2 pdr [asS = 41t,.2] ....(i)
lethe film is to be stretched a distance I'lX to the right, work
done by the external force F will be while changc in area
W = F x Ax = (2TL) x Ax = TIlS ....(ii) llS = 41t(r + dr)2 - 41tr2 = 8ttrdr .... (ii)
where M = (L x Ax) x 2 is the total increase in area oflhe film So by definition of surface encrgy,
from both the sides. This is the desired result and from this it is
clear that:
(1) Free surface energy density = W/AS = T . This is why
T= :s 4nr 2 pdr
8nrdr
sometimes surface tension is defined as mechanical Le., p = (p; - po) = (2Tlr) .... (iii)
work done in increasing the surface area ofa liquid by However, jf instead of drop (or
unity under isothermal conditions and has units bubble inside a liquid), we have bubble
(Jim'). in air, then as the bubble has two' fr,ee
(2) Work done in forming a drop or bubble in a liquid will surfaces and if p' is the pressure between
the two surfaces, then from above for
""
be:
inner and outcr surfaces we have
W =T x 41t,.2 [as boS = 41t,.2 - O} .... (iii)
respectively, Bubble
while for a bubble in air , 2T d'
Fig. 11.27
PI - P =7 an p - Po =72T
539
Adding these two, ·IDI CapillarIty
4T Ifa tube of very narrow'
p=p·-Po=-
, r ....(iv)
bore" (called capillary) is
Eqns. (Hi)and (iv) ~· the-desired resuit and from these jt dipped in a liquid, it is found
is clear that: t11,at th~ liquid in the
capillary either ascends or
(1) Excess pressure .is inversely proportional to the radius
of bubble (or . drop), i.e" pressure inside a smaller
surrounding liqui,d. This
bubble (ardrop) is ~.igh~r than inside a larger bubble
pheI)omenon is. called
(or drop), -This is why when two bubbles of different capillarity.
sizes are put in communication with each other.. the air
In order to calculate the .
will rush from smaller to -larger bubble, so that the
height to which a liquid will
smaller will shrink while the larger Wju expand till the .
smaller bubble to droplet. . a
. rise in capillary, consider a Fig. 1 .
glass capillary of radius R
. dipped jn .wat~r as shown in Fig. 1i.30. As the meniscus is
concave and riearly spherical, ihe pressure below the meniscus
will be [Po - (2TlrHwith· Po as atmo~pheric pressure and 'r as
radius of meniscus. Now as liquid flows from higher to lower
pressure and at same level in a liquid pressure must be same
(this is because a liquid cannot sustain tangential stress), so the
liquid will ascend in the capillary till hydrostatic ,pressure of the
liquid compen~t~ for the d,ecrease in pressure, i. e.,
Fig. 11.28 2T
(2) The excess pressure in case of a drop or bubble in a po=[po_ ]+h Pg or h=2T .... (i)
r rpg
liquid is (2Tlr)andis directed from inside to outside,
But from Fig. 11.30 it is clear that radius of meniscus r is
.i.e., from concave to convex side. Thi~ result is also
related to the radius of capillary through the relation
true for meniscus of liquids, i.e., iT! case of concave
meniscus, (Rlr) =cos'6, i.e., r=Rlcosa .... (ii)
where a is the angle of contact. * So substituting the value of r
2T.I,e., PB
PA, - PB. =-, =Po -2 T(.th)
r r WI PA = Po
- from Eqn. (Ii) in (i), we get
h = 2T _ 2Tcos9 ( ... )
•.•. III
rpg Rpg
Po

A
j P
This is the desired result and from this it is clear that:
(1) The capillarity depends on the nature of liquid and
solid both, i.e., on T,p,a and R. If a>900,i.e.,
meniscus is convex, hwill be negative, i.e., the liquid
will descend in the capillary as actually happens in
case of mercury in a glass tube. However, if a = 900 ,
i. e., meniscus is plane, h = 0 and so no calpillarilly.

--
Ca) Cb)
FIg. 11.28

WHI
i.e., pressure below the meniscus is lesser than above
it by (2Tlr). wher.e -r is the radius of the meniscus. --- ---
--- ---
--
---
Similarly -in case of convex meniscus, . -- -- ----
---
---
---
---
---- ---
--
--
--- -
-------
--------
2T.
Po - PA =-,I.e., 2T
PD -= Po +- (.th)
WI PA =Po
r r I') (e)
a",900jh.O 9>90°; h--VIiI
i.e., pressure below the meniscus is more than above Fig. 11.31
it by 9Ttr) as shown in Fig. I 1.29 (b) .

• Anglc of contact is defined as the anglc_between the tangents to solid and liquid surfaces al a point of contact inside the liquid. II depends on the nature of
solid and liquid both and 'for cor)cIlye menisc11i~ it is acute while for convex obtuse.
540 PHYSICS FOR COMPETITIONS -Vol. I

(2) For a given liquid and (a) The oil in the wickofa lamp rises due to capillary
solid at a given pJace ~s action of threads in the wick.
P. T, 9 and g are constant, (b) Action of towel in soaking up moisture from the
-1=n I
1 IS .
(Fig. 11.32).
body is due to capillary action of cotton in the
hr =constt. :-:::-:::-:
--- --- -::-:-::-:-::
- - ---
-:::-::::- towel.
·i.e., lesser the radius of -:..-:..-:.. ---:...-:..- --=- -:.:-:"- (c) Water is retained in a piece of sponge on account
capiliary greater will b e 0==-~~~;.=-==~=~~:­ of capillarity.
the rise and vice-versa. 'Fig. 11.32
(Fig. 11.32)
(d) A blotting paper soaks ink by capillary action of
the pores in .the blotting paper. "
(3) Here it is important to note that in equilibrIum the
height h is independent of the shape of capillary if the (e) The root·hairs of plants draw water from the soil
radius of meniscus remains the same. This is why the through capillary action.
vertical height h of a liquid column in 'capillaries of (E) Molecular Theory of Surf~ce Tension
different shapes and sizes will be same if the radius of
The force of interaction behyee9/the molecules of the
meniscus remains the same and also the vertical
same substance is ca lled cohesive force while between
height of the liquid in a capillary does not change,
molecules of different substances is called CJdhestve/orce.
when it is inclined to the vertical. (Fig. 1~.33) .
Both the forces are attractive and have a ral).ge of about

..
lOA.

I~w~t _=="'f-~~.~
If we consider a molecul~ A .
A Q C 8
.: t:? = __.: - __.:= -: == _.:.:.:.l well inside a liquid, we find that it <"'~~~='7=0i0=~ 1
- -- --- - -- -
=:-=~ ~-
---- -. - . - . - . - : : : .
-- -- is equally attracted by the C
j: j:j:: :::-- ::::::: =:=:= ~j:" -=::::::::
- -- -- ---- --- -- ----- surrounding molecules in all ~ A _ _ ________ _
(a) (b) (e) (d) (e) directions" (within a sphere . of p -===========-
Fig. 11.33 radius lOA) and so it does not Fig. 11.35
experience any resultant force. On n ...·
Note: Keep in mind that in case of capillarity excess pressure is or near the surface of a liquid the molecules don't exist on the
balanced by hydrostatic pressure and not force due to surface free side and so a molecule (say B) experiences a resultant
tension by weight, i.e., in general (2Tlr) = hpg and not
force vertically downwards into the liquid. For this reason the
2nrT ,., mg. [2nrT .. n,.2hpg is true only for yertical cylindrical
molecules of the surface have' a tendency to go into the liquid
tubes and cannot be applied to other situations such as (a), (b), (c)
and (e).] and so the surface tends to ,contract and gives rise to surface
tension. ". ...,' ,~
(4) In case of capillary of insufficient length, i.e.,L< h, Furthermore as surface molecules like B eXP'rrience a net
the liquid will neither overflow from the upper end downward force of cohesion and have tendency to go down, so
in increasing the area of the surface more molecules are to be
shifted to the surface from inside. So mechanical work will
have to be done against the net downward force of cohesion.
This work is stored by the molecules in the surface as potential
energy caUedji-ee surface energy.
Furthennore for molecules like P in contact with the solid,
the liquid will stick to the solid surface, i.e., will wet it if the
force of adhesion A is greater than net force ofcohesion C. The
Fig. 11.34 liquid will not wet the solid surface ifC> A.
Also molecules like Q at the surface in contact with solid
like a fountain nor will it tickle along the vertical sides will experience a net force ofcohesionCRand ifadhesion A is
of the tube. The liquid after reaching the upper end
greater than cohesion C R' the resultant force will be F [Fig.
will increase the radius of its meniscus without
11.36 (a)] and as liquid cannot support tangential forceF must
changing nature such that:
be perpendicular to the surface of liquid and1so the meniscus
hr=Lr' will be concave and angle of contact acute « 90°). However, if
(5) Capillarity has large number of applications in our adhesion is lesser than cohesion, the resultant force F will be
daily life, e.g.,
This II will be max when eosS = max = I
A
So (h) = 2x75xlO"3
max 103x5xlO-4 x lO
",,0.03 m ",,3 cm Answer
A<CR A.ell Note: Historically this method was used by Quincke to determine
(a) (b) (e) the surface tension of liquids and is called 'Drop weight
Fig. 11.36 method'.

directed 'into the liquid [Fig. 11.36 (b)] and so the meniscus Problem 33. A ring F
(which is perpendicular to F as liquid cannot support is cut from a platinum
tangential force) will be convex and the angle of contact tube oJ; 8.5 em internal
obtuse (> 90°). and 8.7 em external
The behaviour ofa liquid relative to a solid depending on diameter. It is supported
horizontally/rom a pan of
forces of adhesion and cohesion in tabular fonn is given
a balance so that it comes
en,..
below: Section
in contact with the water
S. Adhesion> Adhesion = Adhesion < in a glass vessel. What is
No. Cohesion Cohesion Cohesion the sur/ace tells ion of ---.:- . ,'----.:-
~_=_-:':t.LL L,-"':::'_-:_-.:~
water if an exira 3.97 g t~~~~~~~~~~~~~~~
I. Waler l. in
container
•;, gl~s
'n
Water in a silver Mercury in a glass
container ;, .n container ;, weight is required to pull ---------------
---- -- -- - ----
example of this example of this example of this '" it away /rom water? Fig. 11 .38
situation situation situation (g =980emi,2) (MNR 1992(
,. Liquid will wet the Critical Liquid will not wet Solution : The ring is in contact with water along Its inner
solid the solid
and outer circumference; so when pulled out the total force on
3. Meniscus is concave Meniscus is plane Meniscus is convex it due to surface tension will be
4. Angle of cont8pI is Angle of cOlJ.tact is Angle of contact is F""T(21Cr1 + 21Cr2)
acute (9 < 9~O) 90" obtuse (9 >.9(0)
So, T mg [as F "" mg]
5. Pressure below the Pressure below the Pressure below the 2n(r) +r1)
meniscus ;, lesser meniscus is same as meniscus ;, more
than above ;1 by above it, than above ;1 by i.e., 1' = 3.97x980 .' = 72.13dyne/cm Answer
(2TI,), i.e.,p "" Po (2Tlr1 i.e., 3.14x (8.5 +8.7)
Le·,p""po-(2Tlr) P = Po + (2Tlr)
,. Proble~ 34. A mercury dropJ~f radius 'i' em is sprayed
6. In capillary there No capillarity In capillary there into I 06 droplets ofequal size. Calculate the energy expended
will be ascent. will be descent.
ifsurface tellsion o/mercury·;s 35 x 10-3 N~m.
Problem 32. A vessel. whose bottom has round holes Solution: Ifa drop of radius R is sprayed into ndroplets of
with diameter ofl mm is filled with water. Assuming that equal radius r, then as a drop has only one surface. the initial
suiface tension acts only at holes. find the maximum height surface area will be 4nR 2 while final area n(4w 2 ). So the
to which the water can be filled in the vessel without
increase in area
leakage. Given that sur/ace tension 0/ water is 75 x 10- 3
IlS = n(4m·2 ) _ 41tR 2
Nlm and g =lOmls 2.
T T So energy expended in the process,
Snlution: As shown in Fig. 11.37
here the vertical force due to surface' W=TAS =4.T(n,' _R2) .... (i)
tension at the
Teose x L = Tcose x 21trwill balance
hole --1;;:;::'!--- Now since the total volume of 11 droplets is the same as that
of initial drop, i.e.,
the weight mg, Le.,1tr 2hpg, i. e.,
Teose 2w=w2hpg ~1tR3 "" n[1 1Cr3 ]
or h "" (2Tcos9/prg) Fig. 11.37 or .... (;;)
• Actually surface tension ofmereury is about 4.35 x 10- 3 Nlm.
. So . subs't~tu~ing the value of rfrom Eqn. (ii) in (0, the sense of cu,yature of tlte internal film surface c,ommon to
bo'th the bubbles.
. w,,; 4,.R'T(n)"3 - I]
Soludon: Ifrl and r2 are 'the iad.ii of-smaller ~ large~
So here W=:=4x3: 14 ~ (I ~ IO-~)2 ~ 3~ x JO-3 [I02 -:- 1]' bubbles and Po is the atinosp,heric preSsure,·the pressure inside .
=4.356 x 10-3 l ' A~r : them will be .
. Problem 35; "The. lower end of a capUla.ry -tube of' . 4T.
: Pl=PO+- .an
d· P2=PO+-
4T
... :
(.)
1
diameter2.00min is dipped8.00cm below the surface a/waler . ~ ~
in a bea~er. What is .the pressure required in the tube to blow a Now as the pressure'inside the smaller ,?ubble will be more
bubble at its',end in water? ~lso calculate the exce~s pressure. than inside "the larger .bubble, so for" interface, . , .
[Sudac~ tension ~fwater ' :::: 73 x lO~3 Nlm, density of water P' PI .~ p,. .... (ii)
= 10 3 kglm 3, I atmosphere =1.01 x.IO S Pa and g=9.8mls 2 ]. Now as excess pressure .
. Solution: As the bubble ·is in water; it has only one' acts from concave to ~oilVex .
side~ the intc:rface will be r2
surface.
. . 2 . concave towards smaller
So p.p. _ P .2T · 2x7.3xIO- .146Pa bubble a~d convex towards
In out r 10'-3 larger bubble. (as ' shown iJ.1
Now ububble -is a~ a depth .o fS em in, watet, Fig. 11 .40) and if R is the
radius of interrace,
Pout == Pal + hpg p.·(4TIR) · ....(rll) FIg.'UO
So that Pi~ ,= P + POll" .; P + Pal + heg So substituting &ins. (i) .and (iii):in Eqn. (ii), we get
3
Pin =146+I ~oi x lO~ +~x 1~-2 x 1,0 x 9.8
i.e.,
=1.02 x lOS Pa . Answer
4T~4Jlll·· · ... R- · r, .
If- ,£Lrl
' - "2'
'I
I. ~~ . . - (~2: -rl )
Problem 36. The limbs ofa manometer consis,-ofuniform
So here R O.OO2 >l O·004.=·O.oo4m A~,
capillary tubes,o fradiil.4 x 10- 3 ni and7.2 x'lo--4 in. Find-out . 0.004 0.002 . ..
the correct pressure difference if the level ofthe liquid (density
Problem 38. Under iSothermal condition two soap bubbles
10 3 kglm 3, surfa~e ,tension'72 x 10- 3 Nlm) in narrower tube ofradii a and b 'coalesce to form a single bubble ofradius cJ!
stands 0.2 m above that in the broader tube. the external pressure is Po show·that sUrfaf:e tension,
Solution: If PI and Pz are the ~ressures in the broader and . Po(c 3 _a 3 _b 3 )
'I
narrower tubes of radii anei'2 respecti'vely, the pressure just T ' ='-;;--c;--;;,-'
4(a 2 +b 2 _c 2 )
below the meniscus in the respect!ve tubes will be
2T 2T p, Solution: As ex~ess pressure for a' soap bubble is (4TI,)
PI -- and P2 --. p,
and external pressure PO '
'I '2
I I
SoIhat [PI <l-[p,<~]= hPg .....J :
-f :
h -
B
so
p, • Po + (4Tlr)

Pa ={PO + :r].Pb =[PO + ~]


or 'A': .
:-:.-:." ... ..... . :. ~ ~ .~+~] -~
and Y =~1UJ3 Y, '-3
4 nb 3 and V '=~1tC3 ...•(ii)
a 3 ' e3
Now as mass is conserved,'
1-1" +.l-1b =l-1e
i.e., PaVa + PbYb = peYe '
RTa RT, · RT,
[aspy ="DT
",'
j,e.
0
II
or
= PY]
RT

As temp. is constant, ;,e.;Ta =Tb =Te, so ·~e above


expression reduces to .
. PaY" + PbVb + PeVc
Which in the light of Eqns. (i) and (ii) becomes

[PO + :rJ[~.a3 J+o + :J[~'b3 J


Problem 40. A conical
glass capillary tube of length
0.1 m has diameters 10- 3 and
A

I " "

"
B

+0 + 4n[~·c3 J
5 x 10-4 m at the ends. When it
isjust immersed in a liquid at
O°C with larger diameter in
Ltt- T
~~~ h
: :::1. 1 ::
. ~~if:
--
i. e., 4T(a 2 +b 2 - c 2 )=PO(c 3 _a 3 _b 3 ). contact withit, the /iqu,id rises
to 8 x 10-2 m in the tube. If ___ -_ ~~~f~~=""-=c!-~'l~_
____ . 2 __ ==
Po(c 3 _a 3 _ b 3 ) another cylindrical glass :_:_:__-:-:-:-:-:-::-:::-:-:-_-_-_-_-_-_-_
i. e. , T= . capillary tube B is immersed -~=_::_=_::_=_::_::_::_=_::_::::..=_=_=_=_=_=_=_
4(a 2 + b 2 _ c 2 )
in the same liquid at O°c, the -----
_-..:-_-_ -_-_-_-_-_-_-_-_-_-_-_-_-_ ---- --
- _""":..""":..-- ---:---------- ----
---.,.7"_-_-_
Problem 39. A glass capillary sealed at the upper end is liquid rises to 6 x 10- m 2
Fig. 11.42
oflength 0.11 m and internal diameter 2 x 10- 5 m. The tube is height. The rise ofliquid in the
immersedverticaliy into a liquid ofsuiface tension 5.06 x 10-2 tube B is only 5'.5 x 10- 2 m when the liquid is at 50°C, Find the
N/m. To what length has the capillary to be immersed so that nite at which the surf(:lce tellsion changes with temperature'
the liquid level inside and outside the capillwy becomes the considering the change to be linear. The density ofthe liquid is
same? What will happen to 'the water level inside the capillary (III 4) x 1-0 4 kg/m3 and angle of contact is zero. Effect of
if the seal is now broken?
temperature on density a/liquid and glass is ne~ligible.
Solution: If A is the cross-sectional
1 '.'.,.', Solution: If r is the radius of the meniscus in the conical
(L-X){~:
area of the tube and L its length, the
Po tube~ then as shown in Fig. 11,42,
initial volume of air inside it will be
VI = AL while pressure PI = Po = U1 r- r r - r
tan9 = -- l =2-- l
atmospheric pressure. =="",±b'h,=,== L-h L
Now when the tube is irrunersed in ~~~~3: ~ I' ~ 2.5 x 10-4 (5 - 2:5) x 10-4
water with its length x in water, the level ~~~~~ P2 :~~~ _¥ i.e.,
0.1 0.08
=""---~~-
0.1
of water inside and outside is same; so 11
the volume of air in the tube will be Fig. .41 i.e., rxlO 4 - 2.5 = O.2x2.5
V2 = A (L - x). Further if P2 is the pressure of gas in the tube, i.e., r=3xl0 - 4 m
2T. 2T
P2 -r
= Po' I.e., P2 = Po +--;:-
Now as capillarity is independent of the shape of tube so
at same temp. e = O°c,
Now if temperature is constant, hA rA = hBrB = (2To/pg) = constt.
PIVI =P2V2 so 'iJ "" (0.08 x 3 x 10-4 )/(6 x 10- 2 ) = 4 x 10-4 m
poAL =[PO +2:]A(L_X) or X[l+ ~; ] =L Now as from h = (2Tlrpg) for cylindrical tube,
To =hopgr =.lf6xI0 - 2 x~xlO4 x9.8X4 X IO-4J
S 22L 14
i. e., X[1 + 1.012x lOs x 1 x IO- ] = O.ll
2
2x5.06xIO 2 = 8,4 x 10- N/m
Now as for a given tube and liquid T oc h (as T = hpgrfl)
or x=O.11 = 0.01 m
11 Tso hso
- =-
If the seal is broken the pressure inside the capillary will To ho
become atmospheric, i. e., Po while capillarity will take place 2
and the rise will be So, T =5.5 x lO- x8,4xlO- 2 =7.7xlO- 2 N/m
50 6xlO 2
2
h= 2T = 2x5.06xl0- = 1.03m So rate of change of surface tension with temperature
rpg 10 S xl0 3 x9.8 assuming linearity,
However, the length of the tube outside the water is 2
I::J.T Tso -To (7.7x 8.4) x 10-
- =
0.11 - 0.01 =0.1 m; so the tube will be of insufficient length 68 50-0 50
and so the liquid will rise to the top of the tube and will stay
there with radius of meniscus, = - 1.4 x 10-2 N/mOC Answer
N~gative sign shows that with rise in temperatm';;'; surf.. ce
Answer tension decreases.
EXERCISE

[A) 'Only One C~otce is Correct (a) Length a 50 em and diameter = O.S mm
(b) Length - IOOcmanddiameter= I nun '
I . Which of the following is most elastic in the language of
physics? · (c) Length - 200 em and diameter = 2 mm
(a) Air (b) Hydrogen (d) Length = 300 cm and diameter = 3 nun
(c) Water (d) Mercury 9. A copper wire of length 0.9 m and cross-section 1.0 mm 2
2. Which of the following substances has the highest value of is stretched by a load of 1.0 kg, Young's mOdulus for
Ihe Young's modulus? ICPMT 1993) copper is 1.2x 101' N m-2 andg = 10ms -2. The extension
(a) Sleel (b) Rubber of the wire in mm is :
(c) Wood (d) Plastic (al 0.013 (b) 0.Q75
3, The diagram 11.43 represents (e) 0.11 (d) 0.13
b e c'
the applied force F per unit area
10. A tensile force of 2x 105 dyne doubles the length of a
. with the corresponding change
in length x per unit length F rubber cord of cross-s.ectional area 2 cm2 . The Young's
produced in a thin wire of modulus of rubber is :
uniform cross-seclion. If the o L-_-,,-_ __ (a) 4x 105 dynelcm2 (b) Ix lOS dynekm 2
curve is plotted, the region in x-
which Hooke's law is obeyed. Fig. 11.43 (c) 2x 105 dynelcm 2 (d) Ix 104 dyne/cm2
the region in which the material acts like a viscous liquid II. An elongation of 0.1% in a wire of cross-sectional area
and the elastic limit respectively are represented by: 10-6 m2 causes a tension of lOON. The Young's modulus
(a) ob.cc·.c (b) oc,cc' ,c is :
(c) ob,bc.c (d) oa.cc'.c· (a) 1012 N/m2 (b) 10" N /m2
4. The diagram 11 .44 shows the T, (c) 1010 N/m2 (d) 10' N/m'
change x in the length of a thin 1 12. When a certain weight is suspended from a long uniform
uniform wire caused by the
T, wire its length increases by one cm. If the same weight is
application of stress F at two F
suspended to another wire of the same material and length
different temperatures Tl and
T2 . The variations shown suggest but having a diameter half of the first one. the increase in its
that : length will be :
Fig. 11.44 (a) 0.5 em (b) 2 em
(a) T, >T2
(b) T, <T, (c) 4 cm (d) 8 em
(e) T, -T, 13. A wire of length Land radius ris fixed at one end and force
F applied to the other end produces an extension y. The
5. ~imensional formula for modulus of elasticity is:
extension produced in another wire of the same material of
(a) [MLr'J (b) [ML- 'T-'J
length 2L and radius 2r by a force 2F is :
(e) [ML-'T-' J (d) [ML-'r' J (a) y (b) 2y
6. The Young's modulus ofa wire is numerically equal to the (e) yl2 (d) 4/y
stress which will: 14. Two wires of the same radius and material have their
(a) Not change the length of the wire lengths in the ratio I : 2. If these are stretched by the same
(b) Double the length ofthe wire force the strain produced in the two wires will be in the ratio:
(c) Increase the length by 50010 (CPMT 19931
(d) Change the area of cross-section of the wire to half (a) I : I (b) I : 2
7. A wire can be broken by applying a load of 20 kg wt. The (e) 2: I (d) I: 4
force required to break the wire of twice the diameter is:
IS . A bar of iron of length I m and I cm2 cross-section is
(a) 20 kg WI. (b) 5 kg wI. heated from O"C to 100De. The coefficient of linear
(e) 80 kg wt. (d) 160kgwt. expansion of iron is 10- 5 rc.
The increase in its length is:
8. The following four wires are made of the same material.
(a) I em (b) 0.1 em
Which of these will have the largest extension when the
same tension is applied? ICPMT 1990) (e) om em (d) 0.001 em
16. A metal rod (Y = 2x 10 12 dyne/sq. em) of coefficient of 24. The isothennal bulk modulus of a perfect gas at pressure P
linear expansion 1.6x 10-5 per °C has its temperature raised is equal to:
by 20°C. The linear compressive stress to prevent the (a) P (b) Ply
expansion of the rod is : (c) P'y (d) p, .
(a) 2.4x 10 8 dyne/sq. em (b) 3.2x 108 dyne/sq. em 25. The isothennal bulk modulus of a perfect gas at atmos-
(e) 6.4x 10 8 dyne/sq. em Cd) 1.6x 10 8 dyne/sq. em pheric pressure is:
17. If a bar is made of copper whose coefficient of linear (a) 1.03.x 105 N/m2 (b) 1.01x 10 6 N/m2
expansion is one and a half times that of iron, the ratio of ec) 1.03x 1010 N/m2 (d) 1.03x 1011 N/m2
force developed in the copper bar to the iron bar of identical
26. A beam of metal supported at the two ends is loaded at the
lengths and cross-sections, when heated through the same
centre. The depression at the centre is proportional to:
temperature range (Young's modulus of copper may be
taken to be equal to that of iron) is: (a) y2 (b) Y
(a) 3/2 (b) 2/9 (c) I/Y (d) IIY2
(c) 914 (d) 419 27. The Poisson's ratio cannot have the value:
18. Energy stored per unit volume in a stretched wire is: (a) 0.7 (b) 0.2 (c) 0.1
ICPMT19931 28. What is the relation between Y,B and 11 for isotropic

(a) ~ Load x strain , (b) Load x strain


material? (CPMT 1993]
2 3BY 9B~
(a) ~= (9B+Y) (b) ~= (4B+Y)
(c) Stress x strain (d) -ItressS '
x stram
2 9BY (d) Y = 9B~
19. A unifonn metal rod of2 mm 2 cross-section is heated from (c) ~ = (9B-Y) (3B+~)
O°C to 20°C. The coefficient of linear expansion ofthe rod is 29. For a given material, the value ofY is 2.4 times that ofrt. Its
12x 10-6fe. Its Young's modu1us of elasticity is lOll N 1m2 . Poisson's ratio is: IEAMCET 1992)
The energy stored per unit volume of the rod is: (a) 2.4 (b) 1.2
IMNRi9921 (c) 0.4 (d) 0.2
(a) 2880 J/m 3 (b) 1500 Jim' [Hint: Y", 211(1 - a)]
(c) 5760 J/m 3 (d) 1440 Jim' 30. For the same cross-sectional area and for a given load, the
2
20. A wire (Y = 2x 1011 NI m )has length 1 mandarea 1 mm 2 ; ratio of depressions for the beam of square cross-section
the work required to increase its length by 2 mm is : and circular cross-section is:

(a) O.4J (b) 40J (a) 11::3 (b) " I


(c) 3:. (d) I:.
(c) 4 J (d) 400 J
31. The property due to which a material can be hammered into
21. A rubber ball is taken to a 100 m deep lake and its volume
thin sheet is called:
changes by 0.1 %. The bulk modulus of rubber is nearly:
(a) Ductility (b) Malleability
(a) Ix 10 6 N/m2 (b) Ix 10 8 N/m2
(c) Brittleness (d) Elasticity
(c) Ix 10 7 N/m2 (d) Ix 109 N/m2
32. Two wires A and B of the same material and of same length
22. For gases: are stretched between two fixed supports. Wire A is thicker
(a) Isothennal elasticity is greater than adiabatic elasticity than B. If their initial temperatures and tensions are same,
(b) Adiabatic elasticity is greater than isothennal elasticity then due to. increase in temperature:
(c) Both the elasticities are equal (a) Both TA. and TB decrease equally
23. A given quantity of an ideal gas is at pressure P and (b) TA. decreases more than TB
absolute temperature T. The isothennal bulk modulus of the (c) TB decreases more than TA.
gas is: [liT 1998] (d) Both TA. and TB remain unchanged
(a) 3p (b) P 33. A liquid disturbed by stirring comes to rest after some time
3 due to its property of:
3 (a) Surface tension (b) Viscosity
(c) -P (d) 2P
2 (c) Molecular attraction (d) Low compressibility
34. The coefficient of viscosity 11 of a liquid can be defined r
43. A ball of mass' m' and radius' is released in viscous
from the equation F =f\A (dvl dr)whereF is the tangential liquid. The value onts tenninal velocity is proportional to :
force between two liquid layers of area A. The layers are at (a) (1 / r)only (b) ml r
a distance dy apart and dv is the difference in their
(c) (ml r)"2 (d) manly
velocities. Suitable unit for dv I dx is :
(a) ms- ! (b) 5 - 1 44. Thc tenninal velocity of a small sized spherical body of
radius r falling vertically in a viscous liquid is givcn by the
(e) Nm- 2 (d) N- 1m2 following proportionality: (CPMT 1990)
35. The dimensional fonnu la for coefficient of viscosity :
(a) rx II r2 (b) rx I I r
(CPMT 19921
(c) rx r (d) rx r2
(a) (MLT-') (b) (ML- ' T- ')
45 . The tenninal velocity v of a small stccl ball of radius r
(d) [ML'r') falling under gravity through a column of a viscous liquid
of coefficicnt ofviscosityT'( dcpends on mass of the ball m,
36. The Poiseuille is the unit of:
acceleration due to gravity g. coefficient ofviscosityll and
(a) Pressure (b) Friction radius r. Which of the following relations is dimensionally
(e) Viscosity (d) Surface tension correct ? (CDSE 1992)
37. Which of the following has greatest viscosity? (a) vrxmgr/ ll (b)vox:mgTJr
(a) Hydrogen (b) Air (c) vocmg/111 Cd) v ocr;mg I r
(e) Water (d) Ammonia 46. A small steel ball falls through water more slowly than
38. With increase in temperature the viscosity of: through air primarily bccause of:
(a) Both gases and liquids increases (a) Surface tension effect
(b) Both gases and liquids decreases (b) Smaller density difference
(e) Gases increases and ofliqu ids decreases (c) Viscous force
(d) .Gases decreases and of liquids increases (d) The value of' g' being lesser in water
., .
39. The profile of advancing liquid in a tube is a: 47. A small spherical solid ball is A
B
(a) Straight line (b) Circle dropped in a viscous liquid. Its
journey in the liquid is best
(c) Parabola (d) Hyperbola C
described in the figure drawn '_8~
40. Liquid flows through two capillary tubes connected in by :
series. Their lengths are Land 2L and radius rand 2r ~ D
(a) Curve A
respectively. The pressure difference across the first and
second tube are in the ratio: (CPMT 1991) (b) CurveB Distance -
(c) Curve C FIg. 11.45
(a) 8 (b) 2
(c) 4 (d) 1/8 (d) Curve D
[Hint: (dQ I d/)1 = (dQ I d1h,i.e. .(1fI1'i4 1 8'lL,)= ("P2ri 18'1Lz) J 48. A small spherical solid ball is
dropped from a height in a A
41. The viscous force on a small sphere ofradiusR moving in a viscous liquid. Its journey in the B
fluid varies as : [CPMT 1993J liquid is best described in the
Fig. 11.46 by the:
v c
(a) ocR' (b) oc R D
(a) Curve A
(c) oc (l l R) (d) oc(ll R)'
42. If a small sphere is let fall vertically in a large quantity of a
(b) CurveD ,-
Fig. 11 .4S
(c) CurveC
still liquid of density smaller than that of the material of the
sphere: (d) CurveD
(a) At first its velocity increases, but soon approaches a 49. Tenninal velocity of water drops depends upon the:
constant value (a) Radius of drop (b) Charge of drop
(b) It fall~ with constant velocity all along from the very (c) Temp. ofdrop (d) Velocityoflight
beginning 50. Rain drops fall from a height under gravity; we observe that :
(e) At first it fa lls with a constant velocity which after (a) Their velocities go on increasing until they hit the
some time goes on decreasing ground but the velocity with which the drops hit the
Cd) Nothing can be said about its motion ground differs with the radius of the rain drop
(b) Their velocities go on increasing until they hit the 58. The surface tension of a liquid:
ground, velocity being independent ofthe radius of the (a) Increases with area
dmp
(b) Decreases witlJ area
(e) They fall with a tenninal velocity which is independent
(c) Increases with temperature
of the radius of the rain drop
(d) Decreases with temperature
(d) They fall with a tenninal velocity which depends upon
the radius of the rain drop 59. When there is no external force, the shape ofa small liquid
drop is detennined by :
51. The velocity of a falling rain drop attains limiting value
because of: (a) Density of liquid (b) Viscosity of air
(a) Upthrust ofaie (c) Surface tension (d) Elasticity of liquid
(b) Viscous force exerted by air 60, Small liquid drops assume spherical shape because:
(e) Surface tension effect (CPMTI991(
(d) Air current in atmosphere (a) of the action ofabnospheric pressure from the sides on
the liquid
52. A small drop of water falls from rest through a large height
h in air, the final velocity is : (b) of the action of gravitational force
(a) Almost independent of.Jh (c) the liquid tends to have minimum surface area due to
surface tension
(b) Proportional to .Jh
(d) of the friction of air
(e) . Proportional to h
61. Water flows in a continuous stream down a' vertical pipe
(d) Inversely proportional to h whereas it breaks into drops when failIng fre'ely because of:
53. Two capillary tubes of same radius rbut oflengths ' ] and /2 (a) ViscositY (b) Surface' iensio~
are fitted in parallel to the bottom of a vessel. The pressure
(c) Abnospheric pressure (d) Hydrostatic pressure
head is P. What should be the length lofthe single tube that
can replace the two tubes, so tha't the rate of flow is same as 62, An iron needle slowly placed on the surface of water floats
before: in it because: (MNR 1993]
11/2 (a) When inside water it will displace more water than its
(,) 1= /1 + I z (b) / 0 - - weight .
II + I z
(b) The density of material of needle is less than that of water
1 1 1
(e) 1= - + - (d) /0 - - (c) Of surface tension
II Iz II + I z
Cd) Of its shape
54. Two capillary tubes of same length I but radii 1j and rz 63. When kerosene is sprinkled on the surface of a pond
are fitted in parallel to the bottom of 1;1 vessel. T4~ pressure mosquitoes can no longer remain sitting over it because:
head is P. What should be the radius rofthe single tube, that
(a) Oil reduces the S.T., so liquid membrane is no longer
can replace the two tubes, so that the rate of flow is same as
able to support them
before:
(b) Mosquitoes are repelled by the smell of the oil
(,) r=rl +rz (b) r=Ij2 +rz2
, , ,
(e) r =rl +rz
1 1 1
(d) - =- + -
(c) Swimming is no tonger pleasant to the mosquitoes
Cd) The question is irrelevant
r r1 r, 64. The surface tension ofaliquid is 5 N/m. Ifafilm is held on a
55. Free surface of a liquid has a tendency to contract and ring of area 0.02 m Z , its total surface energy is about:
minimise its surface area. This tendency is due to: (,) 5x 10- ' J (b) 2.5x 10-' J
(a) Viscosity (b) Elasticity
(c) 2x 10- 1 J (d) 3x 10- 1 J
(c) Surface tension (d) Friction
65. The amount of work done in increasing the size of a soap
56. Unit of surface tension is:
film lOx 6cm to 10 x 10 cm is (S.T. = 30x 10- 3 N/m):
(a) newtonlm 2 (b) electron volt/cmz (a) 2.4x lO- z J (b) 1.2x 10-2 J
(c) joule/m (d) joule x nun
(e) 2.4x W-, J (d) 1.2x 10-4 J
57. The surface tension of a liquid is 70 dyne/cm. It may be
66. The amount of work done in forming a soap bubble (S.T.
expressed in M.K.S. system as:
= 30x 10-3 N/m) of radius 5 em is :
(a) 70 N/m (b) 7x lO- z N/m
(a) 1.88x 10- 3 J (b) 1.88x 101 J
(c) 7x 102 N/m (d) 7x 103 N/m
(c) 1.88x 10-1 J (d) 1.88x 10 3 J
67. A spherical soap bubble has a radius r. The surface tension 75. Excess pressure inside a soap bubble is :
of the soap film is T. The energy needed to double the (a) ocllr (b) ocr
diameter of the bubble at the same temperature is:
(c) oc-Jr (d) Independent ofr
[EAMCET 19921
76. An air bubble of radius r in water is at a depth h below the
(a) 24ny2r (b) 4nr2r water surface at some instant. If P is atmospheric pressure
(c) 2rrr2r 2
(d) 121tr r and d and T are the density and surface tension of water
68. A soap bubble (surface tension 30 dyne/em) has a radius of respectively, the pressure inside the bubble will be:
2 em. The work done in doubling its radius is: IRoorkee 1990]
(a), Zero (b) 2261 erg (a) P+hdg-(4Tlr) (b) P+hdg+(2Th)
(c) 1135.5 erg (d) 9043 erg (c) P+hdg-(2Th) (d) P+hdg+(4Tlr)
69. The energy needed in breaking a drop of radius R into n 77. Find the difference of air pressure between the inside and
drops of radius r is : outside ofa soap bubble 5 mm in diameter ifsurface tension
(a) (4nr 2 n-41tR 2 )T is 1.6N/m: [CPMT1992]
(b) (4nR2 -41tr 2 )/T (a) 2560 N/m' (b) 3720 N/m'
(c) 1208 N/m' (d) 10132 N/m'
(c) [(4/3)nr' -(4/3)nR'IT
78. A water drop is divided into eight equal droplets. The
(d) (41tR 2 -n4nr2 )/T pressure difference between the inner and outer side of the
70. A spherical liquid drop of radius R is divided into eight big drop will be: (CPMT 1991)
equal 9~~pl,ets. If-surface tension is T, then work done in the (a) Same as for smaller droplet
process will be: [CPMT 1990,93J (b) (112) of that for smaller droplet
(a) 2nR'T (b) 3nR'T (c) (1/4) of that for smaller droplet
(c) 4nR'T (d) 2nRT' (d) Twice that for smaller droplet
71,. ·A drop of liquid of diameter 2.8 mm breaks up into 125 79. The pressure just below the meniscus of water :
,i,dentieai 4rops. The change in energy is nearly (ST = 75 (a) Is greater than just above it
dyne/cm):
(b) Is lesser than just above it
(a) Zero (b) 19 erg
(c) Is same as just above it
(c) 46 erg (d) 74 erg
(d) Is always equal to atmospheric pressure
72. Two water droplets merge with each other to fonn a larger
droplet. In thi~ process: 80. The quantity on which the rise of liquid in a capillary tube
does not depend is :
{a) Energy,is Ii.bery.t~d ,."
(a) Density of the liquid (b) Atmospheric pressure
(b) Energy is absorbed
(c) Radius of the capillary (d) Angle of contact
(c) Energy is neither liberated nor absorbed
81. Ifthe angle of contact is 0° the shape of meniscus is :
(d) Some mass is converted into energy
(a) Plane (b) Parabolic
73. A soap bubble assumes a spherical surface. Which of the
given statements is wrong? (c) Cylindrical (d) Hemispherical
(a) The soap film consists of two layers of molecules back 82. Two capillary tubes of different diameter are dipped in
to back water. The rise of water is:
(b) The bubble encloses air inside it (a) Greater in tube ofsmaJler diameter
(c) The pressure of air inside the bubble is less than the (b) Greater in tube of larger diameter
atmospheric pressure; that is why the atmospheric (c) Same in both
pressure has compressed it equally from all sides to (d) Zero in both
give it a spherical shape.
83. On putting a capillary tube in a pot filled with water the
(d) Because of the elastic property of the film, it will tend level of water rises upto a height of 4 em in the tube. If a
to shrink to as small surface area as possible for the tube of half the diameter is used, the water will rise to the
volume it has enclosed.
height of nearly :
74. The pressure difference inside and outside a soap bubble is: (a) 2 em (b) 5cm
(a) TIR (b) 2TIR (c) 8 em (d) 11 cm
(c) 4TIR (d) TI2R
84. A very narrow capillary tube records a rise of 20 em when (a) 0.1 m (b) 0.2 m
dipped in water. When the area of cross-section is reduced (e) 0.98 m (d) Full length of tube
to one-fourth of the fonner value, water will rise to a height 92. Water from inside the earth rises through the trunk of a big
of: tree to leaves high up. The main reason for this is:
(a) 10 em (b) 20 em (a) Capillary action (b) High viscosity of water
(e) 40 em (d) 80 em (c) Gravitational force (d) Evaporation of water
85. A capillary tube of radius r can support a liquid of weight 93. Kerosene oil rises in the wick of a lantern because of:
6.28)( 10-4 N.lfthe surface tension of the liquid is 5x 10-2
(a) The diffusion of oil through the wick
N/m. the radius of the capillary must be:
(b) The buoyant force of air
(a) 2.5>< 10-4 m (b) 1.5x 10-3 m
(c) The surface tension
(c) 2.0x 10-4 m (d) 2.0x 10- 3 m
(d) The gravitational pull of the wick
86. Water rises in a capillary tube to a certain height such that
94. One end of a towel dips into a bucket full of water and the
the upward force due to surface tension is balanced by other end hangs over the bucket. It is found that after some
75 >< 10-4 N, force due to the weight of the liquid. If the time the towel becomes wet. It happens:
surface tension of water is 6>< 10-2 N/m, the inner (a) Because viscosity of water is high
circumference of the capillary must be:
(b) Because of the capillary action of cotton threads
(a) 1.25)( 10- 2 m (b) 0.50x 10- 2 m
(c) Because of gravitational force
(e) 6.5)( 10- 2 m (d) 12.5)( 10- 2 m (d) Because of evaporation of water
87. Water rises in a capillary to a height H when the capillary is 95. Consider a liquid contained in a vessel. The liquid-solid
verticaL If the same capillary is now inclined t(l the vertical, adhesive force is very weak as compared to the cohesive
the vertical height of water level in it will : force in the liquid. The shape of the liquid surface near the
(a) decrease solid shall be: [MNR 1994]
(b) increase (a) Horizontal (b) Almost vertical
(c) will not change (c) Concave (d) Convex
(d) may increase or decrease depending on the angle of 96. Which of the foll owing relations is correct connecting thc
inclination elastic constants?
88. Water rises to a height of 16.3 em in a capillary of height 18 9 3 1 9 3 I
(a) - : -+ - (b) - : -+ -
cm. If the tube is cut at a he ight of 12 cm: K y ~ K ~ Y
(a) Water will come as a fountain from the capillary 931 913
(e) - :-+- (d) -:-+ -
(b) Water will stay ataheightof12 cm in the capillary tube y ~ K y ~ K
(c) The height of water in the tube will be 10.3 cm
97. Water is flowing from a reservoir through two tubes of
(d) Water will flow down the sides of the capillary rube length I m and 16 m respectively fixed at the bottom of
89. When a capillary is dipped in mercury the level of mercury reservoir. The diameters of their bores ar:! 4 mm and 8 rom
in the tube will be: respectively. The rate of flow of water in these tubes will be
(a) Higher than in container in the ratio of:
(b) Lower than in container Ca) I : I (b) I: 4
(c) Equal to that in container Ce) I : 8 (d) 1:16
(d) Will not be there in the capillary 98. A small wooden ball, an oil drop and an air bubble each of
radius r are moving up in a liquid of viscosity 11. The
90. A capillary is dipped vertically in a liquid. The level in the
tenninal velocities of there three, i. e., v w' Vo and va
capillary will be same as outside the capillary ifthe angle of
respectively will be such that :
contact is: [CPMT 1993J
(a) v II' = v o = v a (b) vII' <vo <va
(a) Zero (b) 90"
(c) Acute (d) Obluse (c) V w >v o >va (d) vII' >va > vo
91. In a surface tension experiment with a capillary tube water
99. Turbulent flow occurs when the Reynolds number is above:
rises upto 0.1 m. If the same experiment is repeated on an Ca) 1000 (b) 500
artificial satellite which is revolving around the earth, water (e) 3000 (d) 5000
will rise in the capillary tube upto a height of: 100. Two g lass plates having a little water in between cannot be
[Roorkee 1992) easily separated because of:
(a) viscosity (b) at. pressure ~V I
(b) - o c -
(e) surface tension (d) friction V B
101. Sp4"it,an,d water are mixed in a glass cylinder such that the (d) -~V oc B-2
of
density the mixture becomes equal to density of olive oil. V
If a drop of olive oil is added to this mixture then oil will: 108. Water is flowing through a pipe of constant cross-section.
(a) Spread on the surface of mixture At some point the pipe becomes narrow and the cross-
(b) Take a spherical shape inside the mixture section is halved; the speed of water is:
(e) Take a spherical shape on the surface of mixture (SEE (UPTU) 2005(
(d) Floats o~ the surface of mixture as distorted drop (a) reduced to zero
102. The load versus (b) decreased by a factor of 2
elongation graph for Load (c) increased by a factor of2
four wires of the same (d) unchanged
material is shown in the B
109. n droplets of water each of radius r coalesce to fonn a big
'Fig" 11.47.' The thinnest drop of radius R. If the energy released during coalesc~nce
wire is represented by goes into heating the drop, rise in temperature will be: (take
the fine: T as surface tension of water and J as mechanical
(a) OC Elongation equivalent of heat)
(b) OD
(e) OA
Fig. 11.47
(a) !:(~
J ,
_J.)
R
(b) 3T(~_J.)
J , R
(d) OB
103. The steel raHway track -is to be stopped from expanding at
(e) !:(~+J.)
J , R
.. the joints due'to:temperature in,creases. If the area of cross~
110. Vessels A and B contain water. Soap is added to water in
'section of the track is IOOcm 2 then the elastic temperature vessel A and sugar is added to water in vessel B, then:
increases 20°C to 40°C, then work done per unit length will
(a) surface tension of solution in A is less than that of
be(lfa~1.2x w-'rq (PET(Raj.)2001( water and surface tension of solution in B is also less
. (a) 57.6 I (b) 14.4J than that of water
(e) 28.8 I (d) 7.2 I (b) surface tension of solution in A is more than that of
104. A wire suspended vertically from one of its ends is water and surface tension of solution in B is also more
stretched by attaching a weight of 200 N to the lower end. than that of water
The weight stretches the wire by 1 mm. Then the elastic (c) surface tension of solution in A is more than that of
ene~gy stored in the wire is: IAIEEE 20031 water and surface tension of solution in B is less than
(a) 0.21 (b) 101 that of water
(e) 20 I (d) 0.1 J (d) surface tension of solution in A is less than that of
105. A wire fixed at the upper end stretches by length 1 by water and surface tension of solution jn B is more than
applyi~g a force F. The work done in stretching is: that of water
(AIEEE 2004( Ill. Fig. 11048 shows a capillary tube
dipped in water. If P is the
FI
(a) (b) FI atmospheric pressure, Px the pressure
2 at x and Py the pressure at y:
F (a) Px =Py =P
(e) 2F1 (d)
21 (b) Px <P,Py >P
106 . . The mass of water that rises in capillary tube of radius R is Fig. 11.48
(c) Px <P,Py = P
M. The mass of water that rises in tube of radius 2R is:
(d) Px=P,Py>P
(CPMT2002(
112. A capillary tube is dipped in water. Water rises in the
(a) M (b) MI2 capillary to such a height that the upward force because of
(e) 2M (d) 4M surface tension is balanced by lAx 10- 3 newton weight of
107. For a constant hydraulic stress on an object, the fractional water. If surface tension of water is 70x 10- 3 N/m, inner
cha~ge in the object's volume ( 6:) and its bulk modulus circumference of capillary is :
(a) 2cm (b) 4cm
(B)arerelatedas: [AIIMS2005] (c) 1.2 cm (d) 3.4 em
551

113. If the radius ' "of capillary tube dipped in water is changed, 119. For a liquid. shear modulus is :
the height' h' 10 which water rises in the capillary also (a) infinity (b) zero
changes. Variation of h with rwill be as in :
(c) non-zero and finite (d) none of these
120. Increase in length ofa wire on stretching is 0.025%. Ifits
poisson's ratio is 004, percentage decrease in diameter is :
t (a) 0.03 (b) 0.02
h (c) om (d) 0.05
C
121. Breaking stress ofa material is 106 N/m 2 . If the density of
0 material is 3x to 3 kglm 3 • what should be the length of this
r_ material of unifonn cross-section so that it breaks by its
Fig. 11 .49 own weight? (take g "" 10 mIs 2)
(a) B (b) C (c) A (d) D (a) 3.33 m (b) 33.3 m
114. A spherical object falling in air attains a terminal speed 15 (c) 0.33 m (d) None of these
mls. Tenninal speed of this object when il falls in vacuum 122. Young's modulus of a wire of length I and radius r is Y.
will be:
Length of the wire is reduced to! and radius also reduced to
(a) 15 mls 2
(b) less than 15 mJs
~. Young's modulus will now be:
(e) more than IS mls 2
Y
(d) there is no terminal speed in this situation (a) - (b) .!: (c) 2Y (d) Y
115. A needle which is not wetted by water is 10 em long and is 4 2
placed on the surface of water. It is obsclVed that the needle 123. Fig. 11.50 shows a wire of length / and of slightly non-
sinks if its weight is more than Woo then Wo is : (take unifonn cross-section. Its radius at one end is R] and at the
surface tension of water ;: 70 dyne/em and g = 10 m/s 2) other, it is R2
(a) 1.8 gm wt. (b) 2.2 gm wt.
Rm--IT\R,
(c) 1.4 gm wt. (d) 3.6 gm wt.
1~
116. X and Y are two capillary tubes with lengths JX and Iyand
with radii rx and ry respectively. When a pressure difference I' I 'I
P is maintrunoo between the ends of X, the mte of flow of Fig. 11 .50
water through X is 10 cc/sec. X WId Y are now connected in The wire is stre.ched by forces F and F applied along the
series WId the same pressure difference P is maintained across length in opposite directions and nonnal to the ends. Y
the combination. If Ix =2ly WId rx ='Y' rate of flow of being the Young's modulus, extension caused in the wire is
water through the combinttion will be : nearly :
_
(a) 3 cd",c (b) 52 cdsec
7 (a) ,,"
~1~
nYR]R2
(c)
20
"3 cc/sec (d) ~ cclsec FlY
4
117. In Q. 116, if IX"" Iy and rx = Zry rate of flow of water
through the combination will be : 124. Stress that develops in a body when the body is defonned, is:
() to cc/sec (b) ~ cclsec (a) scalar
a 17 9
(b) vector
(c) ~ cc/sec 25
(d) - cc/sec (c) neither a scalar nor a vector
4 4 Cd) always nonnal to area
118. For a perfectly rigid body:
125. A wire is stretched by hanging a weight from its end and
(a) Young's modulus is infinity and Bulk modulus is zero develops a longitudinal strain a 1. Y being the Young's
(b) Young's modulus is zero and Bulk modulus is infinity modulus, elastic potential energy of the wire per unit
(c) Young's modulus is infinity and Bulk modulus is also volume can be expressed as :
infinity Yo' 2 Yo'
(a) Yo 3Ycr
(d) Young's modulus is zero and Bulk modulus is also (b) 2 (c) 2 (d) 4
2
zoeo
126. Rods X and Y are of same material and of same length but 133. One end of a unifonn wire of length L and of weight W is
their radii 8re rx and ry respectively. Each rod is rigidly anached rigidly to a point in the roof and a weight WI is
fixed at one end and twisted by applying same torque at free suspended from its lower end. If S is the area of cross-
end. As a result, twist produced in rod X is ct; x and that section of the wire, the stress in the wire at a height (3L14)
from its lower end is: (liT 1992)
produced in Y is 4Ir' then 41 X is:
~y (a) WI I S (b) [WI +(W I 4)) I S
(e) [W, + (3W 14 ))1 S (d) (WI + W) I S
" "
.
(a) .L (b) L 134. An aluminium and steel wire of same length and cross-
"~y "X section are attached end to end. The compound wire is hung
from a rigid support and a load is suspended from the free
(e) .L (d) 'y end. Y of steel is C20n) times of aluminium. The ratio of
4 4
ry rx increase of length of steel wire to aluminium wire is:
127. Bulk modulus of water is 2x 10 5 N/m 2 . Change in pressure [EAMCET 1993]
required to increase density of water by 0.1% is: (a) 20: 3 (b) 10:7
(a) 2x 109 Nini 2 (b) 2x 108 N/m2 (e) 7: 20 (d) I: 7
(c) 2x 106 N/m2 (d) 2x 104 N/m2 135. When an elastic material with Young's modulus Y is
subjected to stretching stress S the elastic energy stored per
128. Young's modulus of iron is 2x lOll N/m2 and interatomic
unit volume of the material is:
spacing in iron is 3x 10- 10 N/m.lnteratomic force constant
is: (a) YS 1 2 (b) S'(Y 1 2)
(a) 60 N/m (b) 180Nlm (e) S'I2Y (d) S I 2Y
(e) 120N/m (d) 3 Nlm 136. The upper end ofa wire of radius 4 mm and length 100 em
129. Length of an elastic string is' 0' metre when the tension is 4 is clamped and its other end is twisted through an angle of
newton and ' b' metre when the tension is 5 newton. Length 30"; theangle ofshearis: INCERT 1990)
in metre wlten the tension is 9 newton is : (a) 12' (b) 0.12' (e) 1.2' (d) 0.012'
(a) 4 ~ -5b (b) 5b-4a 137. A unifonn rod of mass m, length 1.., area of cross-section A
(e) 9b-9a (d) a+b and Young's modulus Y hangs from a rigid support. Its
elongation due to its own weight will be:
130. Nonnal density of gold is p and its bulk modulus is K.
Increase in density of a lump of gold when a pressure P is (a) mgL (b) mgL
applied unifonnly on all sides is : AY 2AY

(a) pP (b) P (e) 2mgL (d) Zero


K pK AY
K 138. A unifonn rod of'mass m,' lengthL, area of cross-section A is
(e) (d) pK
rotated about an axis passing through one of its ends and
pP P
perpendicular to its length with constant angular velocity (J)
131. A body of mass 10 kg is attached to a wire oflength 0.3 m. in a horizontal plane. IfY is the Young's modulus of the
Its breaking stress is 4.8x 10' N/m 2. Area of cross-section material of rod, the increase in its length due to rotation of
of the wire is 10-6 m 2 . What is the maximum angular rod is:
velocity with which it can be rotated in a horizontal circle? moo2L2
(a) 2 radls (b) I radls (a) """A7.
y'-
(e) 4 radls (d) 8 radls 2
m00 L2
132. A cube at temperature O"e is compressed equally from all (e) =,..::...
3AY
sides by an external pressure P. By wltat amount should its
139. A large open tank has two holes in the wall. One is a square
temperature be .raisea to bring it back to the size it had
hole of side L at a depth y from the top and the other is a
before t'te external pressW'C was applied. Bulk modulus of
circular hole of radius R at a depth 4 y from the top. When
material of cube is K and the coefficient oflinear expansion
the tank is completely filled with water, the quantities of
is a.:
water flowing out per second from both holes are the saine.
(a) 3K P
Then R is equal to: (liT 20001
P (b) 3Ka
L L
(c) 3;0., (d) .!..-
(a).J2ii (b) 2nL (e) L (d) 2.
. Ku
140. Two soap bubbles of different radii are in communication Y2 respectively are fixed between two rigid massive walls.
with each other: The rods are heated such that they undergo the same
(a) Air flows from the larger bubble into the smaller one increase in temperature. There is no bending of the rods. If
until the two bubbles are of equal size Ct l and Ct 2 are in the ratio 2 : 3, the thennal stresses

(b) The size of the bubbles remains the same developed in the two rods are equal provided Y1: Y2 is equal
to: (BHU (Mains) 2008]
(c) Air flows from the smaller bubble into the larger one
and the larger bubble grows at the expense of the (a) 2:3 (b) 1:1 (c) 3:2 (d) 4 : 9
smaller one 147. A 20 cm long capillary tube is dipped in water. The water
(d) The air flows from the larger into the smaller bubble rises upto 8 cm. If the entire arrangement is put in a freely
until the radius of the smaller one becomes equal to falling elevator the length of water colunm in the capillary
that ofthe large one and ofthe large one equal to that of tube will be : (AIEEE 2005]
the smaller one (a) 4 cm (b) 20 cm (c) 8 cm (d) 10 cm
141. If twa soap bubbles of radii 'i and r2 (> r1 ) are in contact, 148. A wire elongates by 1rum whcn a load W is hanged from it.
the radius of their common interface is: If the wire goes over a pulley and two weights Weach are
hung at the two ends, the elongation of the wire will be (in
(a) r,+r2 (b) (r1 +r2 )2
mm): [AIEEE2006[
(d) )'lr2 (a) 0 (b) H2 (c) 1 (d) 21
142. A soap bubble is blown slowly at the end of a tube by a 149. If the tenninal speed of a sphere of gold (density = 19.5
pump supplying air at a constant rate. Which one of the gmlcc) is 0.2 mls in a viscous liquid (density = 1.5 gm/cc),
following graphs represents the correct variation of the find the tenninal speed ofa sphere of silver (density = 10.5
excess of pressure inside the bubble with time? gm/cc) of the same size in the same liquid: (AIEEE 2006]

l~ l~
(a) 0.1 mI, (b) 0.2 mI,
(a) (b) (c) 0.4 mI, (d) 0.133 mI,
150. Two capillaries of length Land 2Land of radii Rand 2R are
I- I- connected in series. The net rate of flow of fluid through

l~ l~
them will be : (given ratc of the flow through single
(c) (d) capillary X = nPR 4 / &,L) (DCE 2005)
(a) 8/9 X (b) 9/8 X (c) 5/7 X (d) 7/5 X
I- I-
151. A hole is in the bottom of the tank having water: If total
143. If there is no change in the volume of wire on stretching,
pressure at the bottom is 3 attn (1 attn = 10 5 Nm- 2), then
then Poisson's ratio for the material of wire is:
velocity of water flowing from hole is : (UPCPMT 2006]
(a) -I (b) 0
(a) "400m, - 1 (b) "600m,-1
(0) 0.5 (d) .0.25
144. An oil drop of density p is floating half immersed inside a (c) .J6oms- 1 (d) none of these
liquid of density cr. If the surface tension of the liquid be T, 152. A body of density Dl and mass M is moving downward in
then the radius R of the oil drop is : glycerine of density D 2 . What is the viscous force acting 011
(a) .}3T/g(2p - cr) (b) ~ it? (Orissa JEE 2006)
V~
(a) Mg(I-~:) (b) Mg(I-~:)
(c) ~ (d) None of these
V~ (c) MgD, (d) Mg D2
145. Density of lead having bulk modulus K at one atmospheric 153. Water flows steadily through a horizontal pipe of variable
pressure is Po' The density of lead when pressure increased cross-section. If the pressure of water is P at a point where
byMo is: flow speed is v, the pressure at another point where the flow
Kpo speed is 2v, is: (take density of water as p)
(a) (b) Kpo
K-Mo K+Mo [J & K CET 2006[
Jpv2 pv2
(c) (d) Kpo (a) p - - (b) p - -
K +2APo
2 2
K - 2APo
Jpv2
146. Two rods of different materials having coefficients of (c) p - - - (d) p_pv2
thennal expansion (11 and Ct 2 and Young's moduli Y1 and 4
154. Blood is flowing at the rate of200cm 3/sec in a capillary of 160. Water is flowing through a horiwntal pipe of varying
cross-sectional area 0.5 m2. The velocity of flow, in cross-section. If the pressure of water equals 2 cm of
nun/sec is: [Kerala PMT 20061 mercury, where the velocity of the flow is 32 cms- 1, what
(a) 0.1 (b) 0.2 (c) 0.3 (d) 0.4 is the pressure at another point, where the velocity of flow is
(e) 0.5 65cms- l ? [OrissaJEE2007)
155. Sixty four spherical rain drops of equal size are falling (a) 1.02 cm ofHg (b) 1.88 cm ofHg
vertically through air with a terminal velocity 1.5ms- i , If (c) 2.42 cm ofHg (d) 1.45 cm ofHg
these drops coalesce to fonn a big spherical drop, then 161. A wire of length L and area of cross-section A is stretched
terminal velocity of big drop is: fWB JEE 2006J through a distance x metre by applying a force F along
length, then the work done in this process is: (Y is Young's
(a) 8 rus -I (b) 16 rus- 1
modulus of the material) [West Bengal JEE 2007)
(c) 24 ms- 1 (d) 32 ms- 1
(,) ~(ALen) (b) (AL)(YL)m
156. When a body falls in air, the resistance of air depends to a
great extent on the shape of the body. Three different
shapes are given. Identify the combination of air resistances (c) 2(A.L)(YL)GJ (d) 3(A.L)(YL>(i)
which truly represents the physical situation. (The cross-
sectional areas are the same) (Punjab CET 2006J 162. Spherical balls of radius R are falling in a viscous fluid of
viscosity 11 with a velocity v. The retarding viscous force
R R acting on the spherical ball is: IUPSEE 2007)
(a) directly proportional to R but inversely proportional to v
(b) directly proportional to both radius R and velocity v
(c) inversely proportional to both radius Rand velocity v

W
(1) Disc
w
(2) Ball
L
(3) Cigar shaped
(d) inversely proportional to R but directly proportional to
velocity v
163. A frame made of metallic wire enclosing a surface area A is
Fig. 11.51 covered with a soap film. lethe area of the frame of metallic
wire is reduced by 50%, the energy of the soap film will be
(a) 1<2<3 (b) 2<3<1
changed by: (UPSEE 2007)
(e) 3<2<1 (d) 3 < I < 2 (a) 100% (b) 75%
157. A metal plate of area 10 cm 2 rests on a layer of oil 6 rom
3
(c) 50% (d) 25%
thick. A tangential force of 10-2 N is applied on it to move
164. A 5 m aluminium wire (Y = 7x 101ON/ m 2 )of diameter 3
it with a constant velocity of 6 cm s -I . The coefficient of
rom supports a 40 kg mass. In order to have the same
viscosity of the liquid is: fWB JEE 2006)
elongation in a copper wire (Y = 12x 10 10 N 1m2) of the
(a) 0.1 poise (b) 0.5 poise
same length under the same weight, the diameter should be
(c) 0.7 poise (d) 0.9 poise
in mm: [RPMT 2007)
158. The speeds of air-flow on the upper and lower surfaces of a
(a) 1.75 (b) 2.0 (c) 2.3 (d) 5.0
wing ofan aeroplane are VI and v 2 respectively. If A is the
cross-sectional area ofthe wing and' p' is the density of air, 165. The cylindrical tube ofa spray pump has a cross-section of
then the upward lift is: [EAMCET 2006) 8 cm 2 • one end of which has 40 fine holes each of area 10-8
I I m 2 , If the liquid flows inside the tube with a speed ofO,15
(a) '2pA (vI -v 2 ) (b) '2pA (vI +v 2 ) m min - I , the speed with which the liquid is ejected through
the holes is : IKarnataka CET 2007)
I 2 2
(d) 2pA (v, +v2) (a) 50 ms -I (b) 5 ms- I
159. A layer of glycerine of thickness 1 rom is present between a (c) 0.05 ms- I (d) 0,5 ms- I
large surface area and a surface area ofO.l m 2 . With what
166. The radius R of the soap bubble is doubled under isothennal
force the small surface is to be pulled, so that it can move
condition. If T be the surface tension of soap bubble, the
with a velocity of 1 mfs? (Given that coefficient of viscosity
required surface energy in doing so is given by:
=0.07 kg-m- I s -I) {Orissa JEE2007)
(Pb. PMT 20071
(a) 70N (b) 7N
(a) 32nR'r (;') 241tR 2 r
(c) 700 N (d) 0.70 N
(c) 8nR'r (d) 4nR 2r
167. A sphere of radius R is gently dropped in a liquid of 173. The excess of pressure inside the first soap bubble is three
viscosity 11 in a vertical unifonn tube. It attains a tennina! times that inside the second bubble. The ratio of volume of
velocity lJ. Another sphere of radius 2R when dropped into the first 10 that of the second bubble is: (Pb. CET 2008)
the same Ijquid will attain a terminal velocity: (a) I : 3 (b) I : 9
IJ & K (CET) Z0081 (c) I : 27 (d) 9: I
(a) v (b) 2v 174. Two wires of same material and radius have their lengths in
(c) 4v (d) 9v ratio 1 : 2. If these wires are stretched by the same force, the
168. A capillary tube is taken from the earth to the surface of strain produced in the two wires will be in the ralio:
moon. ruse of the liquid column on the moon (acceleration IUPSEE 20091
due to gravity on earth is 6 times that on moon) is: (a) 2: I (b) L I
IEAMCETZ0081 (c) I : 2 (d) I: 4
(a) six times that on the earth's surface 175. A sphere of radius rand density P is dropped under gravity
(b) 1/6 that on the earth's surface through a fluid of viscosity T\. If the average acceleration is
(e) equal to that on the earth's surface
half of initial acceleration, what is the time required to
attain terminal velocity? IUPSC 20091
(d) zero
169. A liquid drop of radius R breaks into 64 tiny drops each of
(a) 4pr' i (9\t) (b) 9p" /(4~)
radius r.lfthe surface tension of Iiquid isT, then the gain in (c) 4p,' I (lit) (d) p"/~
energy is : (BHU Z008} 176. A vessel contains oil (density 0.8 g cm-3 ) over mercury
(a) 481tR 2T (b) 121tr2T (density 13.6 gcm-3). A homogeneous sphere floats with
(c) 9(mR'r (d) 1921tR'r half volume immersed in mercury and the other half in oil.
170. A copper wire and a steel wire of the same diameter and The density of the material of the sphere in gem -3 is:
length aTe joined end to end and a force is applied which IAMUZ0091
stretches their combined length by I cm. Then the two (a) 12.8 (b) 7.2
wires will have : (Manipal (Med.) 2008) (c) 6.4 (d) 3.3
(a) the same stress and strain 177. A wire of length Land radius a rigidly fixed at one end. On
(b) the same stress but different strains stretching the other end of the wire with a force F, the
(c) the same strain but different stresses increase in ils length is I. If another wire of same material
(d) different stresses and strains but of length U and radius 2a is stretched with a force 2F,
the increase in its length will be: (OrissaJEE 2009)
171. A spherical solid ball of volume V is made of a material of
(a) 1/4 . (b) I
density PI' It is falling through a liquid of density
p 2 (p 2 < PI} Assume that the liquid applies a viscous (c) 1/2 (d) '}j
force on the ball that is proportional to the square of its 178. The Poisson's ratio ofa material is 0.5. Ifa force is applied
speed v, i.e.• F vW:ous ::=.lc.v 2 (k > 0) The terminal speed of to a wire of this material, there is a decrease in the cross-
sectional area by 4%. The percentage increase in the length
the ball is : (A1EEE 2008)
is: IWB (JEE)Z009]
(a) Vg(p, -p,) (b) Vgp, (a) 1% (b) 2%

r:'
k k (c) 2.5% (d) 4%
(d) Vg (P, -p,) 179. Two spheres of equal masses but radii 71 and 72 are allowed
(c) to fall in a liquid of infinite colunm. The ratio of their
k
172. A water drop is divided into 8 equal droplets. The pressure
tenninal velocities are: fWD PEE) 2009)
difference between the inner and outer side ofthe big drop (a) I (b) rl :72
will be: (BHU (Screening) 2008)
(d) ,f\:F,
(a) same as for smaller droplet
I 180. The excess pressure inside a spherical drop of radius 70f a
(b) - of that for smaller droplet liquid of surface tension Tis : IDCE 2009)
2
(a) directly proponional to rand inversely proportional to T
(c) '4I of that for smaller droplet (b) directly proportional to T and inversely proportional to r
(d) twice that for smaller droplet (c) directly proportional to the product ofT and 7
(d) inversely proportional to the product ofT and 7
181. A metallic rod oflength I and cross-sectional area A is made (a) 2F L (b) 4F L
ofa material of Young's modulus Y. If the rod is elongated YA YA
by an amount y, then the work done is proportional to :
(c) .!...L (d) 3F L
IDCE2009] YA 2YA
(a) y (b) I1y 187. A ring is cut from a platinum F
(c) Y' (d) I1y2 tube with 8.S cm internal and
8.7 cm external diameters. It is
182. Two wires are made of the same material and have the same supported horizontally from
volume. However wire 1 has cross-sectional area A and the pan of a balance so that if
wire 2 has cross-sectional area 3A. If the length of wire 1 comes in contact with water in
increases by !J.x on applying force F, how much force is a glass vesseL Ifan extra 3.97
needed to stretch wire 2 by the same amount? g wt. is required to pull it out
IAIEEE 2009] of water, then surface tension
(a) F (b) 4F of water will be (in dyne/cm) :
(c) 6F (d) 9F (a) 76.81 -----
.- -----------
----------------
--- ------ -- ---
-- --------------
._--------------
.---- --------- ----
183. Two solid pieces, one of steel and the other of aluminium (b) 75.22
when inunersed completely in water have equal weights. Fig. 11.54
(c) 74.75
When the solid pieces are weighted in air:
(d) 72.13
[Karnataka CET 2009)
188. The ' two graphs represent the interaction force (F) and
(a) the weight of aluminium is half the weight of steel
potential energy (U) of a diatomic molecule in tenus of the
(b) steel piece will weigh more inter atomic distance r. The graphs are not drawn to the scale.
(e) they have the same weight Which of the following expressions is true?
(d) aluminium piece will weigh more F U
184. The stress required to double the length of a wire of
Young's modulus Y is: IJ&K2009]
(a) Y/2 (b) 2Y
, d
(c) Y (d) 4Y
185. A s~ll ;olid sphere of radius R made of
a , material of bulk . modulus B is (,) (b)
surroUnded by an incompressible liquid Fig. 11_55
in a cylindrical container. A massless
piston of area A floatS on the surfaces of (a) a=c (b) a=d
the liquid. Find the magnitude of Fig. 11.52
(c) b=c (d) b =d
frictional change in the radius of the sphere (dR / R )when a 189. The work done in stretching a wire by 0.1 mm is 4 1. The
mass M is placed slowly on the piston to compress the work done in stretching another wire of same material, but
liquid. with double the radius and half the length by 0.1 mm is:
Mg (b) Mg
(a) 3AB (a) 16J (b) 32J
AB
(c) 64 J (d) None
(c) 3Mg (d) none of these
AB 190. A solid cylinder of mass m. length t and
area of cross-section A is placed vertical
186. A unifonn elastic rod of Elastic rod
as shown in the figure. If Young's
cross-section are A, F~ ~3F modulus is Y then strain energy stored in
natural length L and \\\\\\\\\\\\,~\
the cylinder is :
Young's modulus Y is Fig. 11.53
placed on a smooth horizontal surface. Now two horizontal m2 g 2 J Fig. 11.56
(a) 3AY (b)
,,) forces (ofmagnitudeF and 3F) directed along the length of
rod and in opposite direction act at two of its ends as shown. m2 g 2 t
After the rod has acquired steady state, the extension of the (c) 2AY (d)
rod will be :
P ROPERTIES OF MAnER 557

191. A thin wire ring of 3 em radius float on the surface of a length 2.0 m. The extension produced in this composite
liquid. The pull required to raise the ring before the film wire, when it is loaded with a mass of 200 kg is
breaks 3.14)( 10- 3 N more than its weight. The surface .r
(YSteel = 2x 10" Nt m2 O:lpper '" Ix 1011 Nt m2 , g ;: 10
tension of the liquid (in N/m) is : mls 2 ) (weight of the composite wire is negligible)
(.) 8.3x 10-2 (b) 8.3x 10-' (0) 0.5 mm (b) I mm
(b) 9.0 x 10-' (d) 9.8x W, (e) 2mm (d) 4mm
192. A composite wire ofunifonn cross-section of 5.5x 10- 5 m 2
consists ofa steel wire of length 1.5 m and a copper wire of

ANSWERS

I. (d) 1. {a} 3. (a) 4. (0) , . (b) 6. (b) 7. (e) 8. (0) •• (b) 10. (b) II. (b) n. (e)
13. (a) 14. (a) IS. (b) 16. (e) 17. (a) 18. (d) 19. (a) 20. (a) 1I. (d) 22. (b) 23. (b) 14. (a)
25. (a) 16. (e) 27. (a) 28. (d) 29. (d) 30. (e) 31 . (b) 32. (b) 33. (b) 34. (b) 35. (b) 36. (e)
37. (e) 38. (e) 39. (e) 40. (a) 41. (b) 42. (a) 43. (b) 44. (d) 45. (e) 46. (e) 47. (e) 48. (e)
49. (a) 50. (d) 51. (b) 52. (a) 53. (b) 54. (e) 55. (e) 56. (b) 57. (b) 58. (d) 59. (c) 60. (c)
6 .. (b) 62. (e) 63. (a) 64. (c) 65. (c) 66. (a) 67. (a) ... (d) 69. (a) 70. (c) 71. (d) 72. (a)
73. (cl 74. (c) 75. (a) 76. (b) 77. (a) , 78. (b) 79. (b) 80. (b) 81. (d) 82. (a) 83. (e) 84. (c)
85. (d) 86. (d) 87. (e) ... (b) 89. (b) 90. (b) 91. (b) 92. (a) 93. (e) '4. (b) 95. (d) 96. (c)
97. (a) ... (b) 99. (a) 100. (c) 101. (b) 102. (c) 103. (a) 104. (b) 105. (a) 106. (c) 107. (b) 108. (c)
10•. (b) 110 (d) 111. (c) 112. (a) 113. (c) 114. (d) 115. (c) 116. (c) 117. (a) 118. (c) 119. (b) .. UO. (c)
121. (b) 122. (d) 113. (a) 114. (c) 125. (b) 126. (d) 127. (c) 128. (a) 129. (b) 130. (a) 131. (c) 132. (b)
133. (c) 134. (c) 135. (cl "6. (b) 137. (b) 13S. (e) 139. (a) 140. (cl 141. (c) 141. (b) 143. ecl 144. (cl
145. (a) 146. (e) 147. (b) 14S. (c) 149. (a) 150. (a) 151. (a) 151. (a) 153. (a) 154. ~j) ISS. (e) 156. (c)
157. (a) ISS. (cl 159. (b) 160. (b) 161. (a) 161. (b) 163. (e) 164. (c) ,.,. (b) 166. (b) 167. (e) 16S. (a)
169. (d) 170. (b) 171. (a) 172. (b) 173. (c) 174. (b) 175. (a) 176. (b) 177. (b) 178. (d) 179. (c) 180. (b)
l SI. (c) 182. (d) 183. (d) 184. (c) 18S. (a) 186. (a) 187. (d) 188. (b) 189. (b) 190. (b) 191 . (b) 191. (b)

(B] More than One Choice is Correct (e) Its temperature dependent while friction not
I. Regarding elasticity which of the following statements are (d) Its independent of area like surface tension while
correct? frict ion depends
(a) Rubber does nOI obey Hook's law 4. When an air bubble moves up from the bottom of a lake:
(b) Elasticity· can be different for tensile and compressive (a) Its velocity increases and becomes constant
stress (b) Its velocity decreases and becomes zero
(e) Elasticity is independent of temperature (c) Its acceleration increases and becomes constant
(d) Poisson's ratio is a modulus of elasticity (d) Its acceleration decreases and becomes zero
2. When a wire is sttetched to double its length: S. Which of the fo llowing statements are not correct about a
(a) Its radius is halved soap bubble?
(b) Sttain is unity (a) Work done in formi ng the bubble of radius Rand
2
surface tension T is 41tR T
(e) Stress is equal to Young's modulus
(d) Young's modulus is equal to twice the elastic energy (b) Work done in doubling the radius of a bubble of radius
per unit volume R and surface tension Tis 241tR2T
3. Viscous force is somewhat like friction as it opposes th~ (e) Pressure inside a bubble is greater than inside a drop of
motion and is non-conservative but not exactly so, because: the same radius and liquid
(a) Its velocity dependent while friction not (d) Pressure inside a bubble is lesser than outside it
(b) Its velocity independent while friction is

• For bone E 1cnsiie > ECOII'pR3Ii"" while for concrete Eco~ve > E tenoi1c
558 PHYSICS FOR COMPETITIONS - Vol. I

6. When a drop of liquid splits up into a number of drops: 13. Pick out the wrong statement from the following.
(a) Volume increases (b) Area increases (a) Viscosity depends upon the nature of the liquids
(e) Energy is liberated (d) Energy is absorbed (b) Viscosity of liquids is much greater than of gases
7. Excess pressure can be (Tl' 1R) for: (c) Viscosity involves transport of mass
(a) Spherical drop (d) In case of liquids, viscosity decreases with decrease in
(b) Spherical meniscus density
(e) Cylindrical bubble in air 14. Blood flows in a aorta of radius 9 nun at 30 cm/s. Although
the cross-sectional area of a capillary is much smaller than
(d) Spherical bubble in water
that of the aorta, there are many capillaries, so their total
8. If a liquid rises to the same height in two capillaries of the cross-sectional area is much larger. All the blood from the
same material at the same temperature: aorta flows into the capillaries and the speed of flow
(a) The weight of liquid in both capillaries must be equal through the capillaries is 1 mmls. Then:
(b) The radius of meniscus must be equal (a) Volume flow rate is nearly 4.58litres/min
(e) The capillaries must be cylindrical and vertical (b) Volume flow rate is nearly 1.63x to- S ro 2 / s
(d) The hydrostatic pressure at the base of capillaries must (c) The total cross-sectional area of the capillaries is
be the same nearly 763 cm2
9. No capillarity will take place if: (d) The total cross-sectional area of the capillaries is
(a) The liquid is at its b.pt. nearly 458 cm 2
(b) The liquid is at its f.pt. 15. Two'light wires A andB shown in the fi gure are
(e) The angle of contact is 0" made of the same material and have radii rA and
(d) The angle of contact is 90" 'a respectively. The block between them has a m
mass m When the force F is mg 1 3, one of the
10. With rise in temperature which of the following forces can
wires breaks:
never increase?
(a) A breaks if rA = ' B
(a) Elastic Fig. 11.58
(b) A breaks if rA < 2rB
(b) Friction
(c) Either A or B may break if rA < 2rB
(c) Foree due to surface tension
(d) The lengths of A and B must be known to predict which
(d) Viscous
wire will break
II. If for a liquid in a vessel force of cohesion is twice of 16. Which of the following statements is/are correct?
adhesion:
(a) adhesive force helps us in writing
(a) the meniscus will be convex
(b) cohesive force keeps mercury away from container
(b) the liquid will wet the solid
(c) separation of plates in contact with water is difficult
(c) the angle of contact will be obtuse due to adhesive force
(d) there will be capillary descent (d) separation of plates in contact with water is difficult
12. A steel rod of length 6 m and due to cohesive force
diameter 20 nun is fixed between 17. A spherical solid body is dropped inside a vast expanse of
. I
two rigid supports as shown in Fig. viscous liquid of large depth and of coefficient ofviscosity
11.51. Y .. 2xt0 6 N/cm 2 • 1'\. The density ofthe solid is greater than of the liquid. The
6m time taken by the body to attain the 900..1. of the steady state
(1 = 12x 10-61 °C and change in
fig. 11.57
temperature is 80°C. velocity is dependent on :
(a) Stress in the rod is 1920 N/cm 2 when the ends do not (a) density of the liquid (b) density of the solid
yield (c) diameter of the sphere (d) coefficient of viscosity
(b) Stress in the rod is 1581 Nlcm 2 when the ends do not 18. The tangential force F (viscous force) required to move a
yield fluid layer at a constant speed v (along +ve x -axis). When
the layer has an area A (x-z plane) and is located a
(c) Stress in the rod is 1800 Nlcm 2 when the ends do not
perpendicular distance yfrom an inunobile surface. Then F
yield
is proportional to :
(d) Stress in the rod is 1581 Nlcm2 when the ends yield by
1 nun (.) 1 (b) 1 (c) v (d) A
y z
PROPERTIES OF MAnER 559

plank. The area of the end face is 10 cm2 and young's


modulus is 2.lxI0" N/m 2 . If the strain produced in the
I. (a) and (b); 2. (b). (e) and (d); 3. (a) and (e); 4. (a) and (d);
direction oflhe force isxx 10-7, then find the value ofx.
5. (a) and (d); 6. (b) and (d); 7. All; 8. (b) and (d); 9. (a) and
4. A uniform rod. is rotating ahout one of its end with a
(d); 10. (a). (b) and (e); 11. (a). (e) and (d); 12. (a) and (d);
13. (e); 14. (a) and (e); 15. (a) and (b); 16. (a). (b) and (d);
17. (b). (e) and (d); 18. (a). (e) and (d)
I,
constant angular velocity win gravity free space. The radio
strain produced in two halves is k (> Then find the value
of20k.
II
(C) Assertion-Reason Type Questions 5. A 10nggJass capillary tube of radius ris placed horizontally
(a) If both A and R are true and R is the correct and filled with water (angle of contact for water - glass -
explanation of A. 0°). If the tube is made vertical, then the length of water
(b) If both A and R are true but R is not correct column that remains in the capillary is 11T where' T' is the
explanation of A. rpg
(e) If A is true but R is false. surface tension of water, p is density of water. Find 11
(d) If A is false but R is true.
(e) If both A and R are false. I. 3 2. 5 3. 2 4. 4 5. 4
1. (A); A bubble comes from the bottomofa lake to Ihe lOp.
(R) : Its radius increases. IAIIMS 2008J
[E] Match the Columns
2. (A): In the proportionality limit the intennolecular forces I. A tube is inverted in a mercury vessel as shown in Fig.
are strictly conservative in nature. 11 .59. P is the pressure inside the tube above the mercury
I- level as shown.
(R): In the proportionality limit the body regains its ___ P
r--::.
natural length completely after removal of all
deforming forces.
3. (A): In the absence of wind rain drops from higher h
clouds move with nearly uniform speed in vertical
direction near the earth surface. ---
--- - ------
-- t=~
-- -- ---
(R): The viscous drag on the rain drops falling from 1':---------
---------
--------
higher clouds is proportional to the speed of the rain Flg.1U59
drop.
Match the following:
ANSWERS
Column· J Column - II
I. (b) 2. (d) 3. <a)
(a) If P is increased, then (P) increases
height h
[D] Integer Type Questions
(b) If P is increased, then (q) decreases
I. An air bubble of radius 1 mm is allowed to rise through a pressure at O.
long cylindrical colunm of a viscous liquid of radius 5 cm
and travels at a steady rate of2.l cm per sec. If the density (e) If P is increased, then (,) remains same
pressure at 1 em above
of the liquid is 1.47 g I cc. Its viscosity is nearly %poise. 0
Then find the value of n. Assume g = 980 cm / sec 2 and (d) If the vessel is moved (s) increases or decreases
neglect the density of air. upwards with depending on the size
acceleration (a<g~ of the tube
2. A spherical ball of radius 3.0x 10-4 m and density
then height h
104 kg I m3 falls freely under gravity through a distance
H=nx324m before entering a tank of water. If after 2. A light rod with unifonn cross-section oft 0-4 m2 is shown
entering the water the velocity of the ball does not chang..:, in the Fig. 11.60. The rod consists of three different
findn. Viscosity of water is lOx 10-6 N_s/m2 ,g = IOmts 2 . materials whose lengths are 0.1 m, 0.2 m and 0.L5 m and
3. A uniform elastic plank moves due to a constant force whose Young's modulii are 2.5x 1010 NI m2 , 4x 1010
Fo = 84 N distributed unifonnly over the end face of the NI m2 and Ix 10 10 N I m 2 respectively.
560 PHYSICS FOR C OMPETITIONS - Vol. I

A 4. With regard to dependence of quantities given in Column-I


0.1 V_ 2.Sx 1010 on the quantities mentioned in Column-II, match the
B ColunUl-I and Co lumn-II.
0.2 Y_ 4x10 10
C
0.15 Column - I Column - II
, "10 10
o V.
(a) Young's modulus of a (P) Depends on
10 kg substance temperature
Fig. 11.60
(b) Bulk modulus of a (q) Depends on length
Assume that the string connecting the blocks and rod does substance
not elongate. Match the following:
(c) Modulus of rigidity of a (,) Depend on area of
Column ~ I C olumn - II substance cross-section
(a) Displacement of point (P) 2. (d) Volume o f a substance (,) Depends on the nature
B (approx) (in 10- 6 m) of material

(b) Displacement of point (q) 9 5. Consider a wire of length I, cross-sectional area A and
C(approx.) (in 10-6 m) Young's modulus Y. Match Column-I with Column-II.

(0) Displacement of point (,)


D (apprax) (in 10- 6 m)
• Colu mn - ] Column -II

(a) If the wire is pulled at (p) Young's modulus Y


(d) Displacement of the (,) 12 its ends by equal and
block (approx) (in 10- 6 opposite forces 0
m) magnitude F so that it
undergoes an
3. A andD are two blocks which are held together on a smooth
elongation x, according
horizontal table by applying equal and opposite forces F,
to Hooke's law, F:= lex,
on its two ends as shown in Fig. 11.61. Area of
cross-section of face a,b ,c,d is A. The interface at where k is the force
e,j,g,h is sufficiently rough so that there is no sliding. constant. Force
(Assume that force F is such that Ronnal aftable on block A constant (k) of the rod
is equal to weight of A and normal of table on block B is will depend on
equal to weight of B).
(b) Let us suspend the 'Yire (q) elongation (x)
vertically from a rigid
support and attach a
mass m at its lower end.
If the mass is slightly
pulled down and
released, it executes
SHM of a time period
Fig. 11 .61
that will depend on
Match the following:
(c) If the given wire is (r) length (I)
Column - I Column - II fixed between two rigid
supports and its
(a) Longitudinal stress at (P) F temperature ;,
face abed A increased thermal stress
(b) Longitudinal stress at (q) Fcos 2 e that develops in the rod ,
face efgh will depend on
A
(d) Work done ;n (,) ",ea of cross-seclion
(c) Transverse stress at (,) Fsin 29
faceabcd stretching the wire to a (A)
2A
length I + x will depend
(d) Transverse stress at (,) zero on
face efgh
PROPERTIES OF MAnER 561

(b) average intennolecular separation in the surface layer


increases; average intcrmolecular force decreases, so
I. a-+ q; b---tr;c-tr; d---tq
the surface tension will decrease.
2. a-+ r; b-t q; c-+ p; d -+ P (c) average intermolecular separation in the surface layer
3. a-+ p; b-t q; c-+ s; d-+ r decreases; average intermolecular force decreases; so
the surface tension will decrease.
4, a-+ p,S; b-+ p,~; c -+ p,s; d-+ p,q,r
(d) average intermolecular separation in the surface layer
5, a-+ p, c,S; b-+ p,r,S; c-+ p; d-+ p,q,r,s increases; avemge intcnnolecular force increases; so
the surface tension will decrease.
IPI Linked Comprehension Type
2. Magnitude oflhe surface tension force acting on any leg of
Paragraph - 1 the insect will be nearly
Water fonns drops owing to a tendency of its surface to (a) 3.6x 10- 6 N (b) 5.8x W-' N
contmct. Surface of water acts like a stretched membrane
by virtue of an cffect called 'Surface Tension', For a small (c) 4.5x 10-4 N (d) 2Ax 10- 5 N
quantity of water, the effect of surface tension is greater 3. Mass of the insect, assuming that each leg bears an equal
than the weight of water; its surface contracts so as to weight, isnearly (takeg = IOm/s 2 )
achieve the smallest surface area. This implies a spherical
(a) 180 mg (b) 120 mg
shape, The phenomenon of surface tension is common to all
liquids. The nattlre of this phenomenon can be understood (c) 100 mg (d) 216 mg
on the basis of mo lecular forces. Consider a liquid with its 4. Component of the surface tension force acting on any leg
surface open to the atmosphere. Some molecules of the that is ineffective in balancing the weight is nearly
liquid w ill escape from the surface and exist as 'vapour' (a) 5.7x 10-.5 N (b) l A x 10- 5 N
above the surface. In fact , molecules on the surface are in a
dynamic slate. In equilibrium, some molecules are leaving (e) 2.7x 10-6 N (d) 3.9x 10--4 N
the surface at any time to convert to the vapour state and an Paragraph - 2
equal number of molecules convert from the vapour state to
the liquid state on the surface. Consequently !tie number of Poiseuille's equation for the flow rate relates the flow rate
molecules in the surface layer is smaller than in the layers to the coefficient of viscosity besides other parameters. Let
lying below. Average separation between the surface us consider the situation regarding flow of blood in major
molecules is slightly greater and attractive forces between arteries of the body. Given that coefficient of viscosity of'
them have a tendency to pull them together. This explains blood at the body temperature is 4x 10- 3 N_s/m 2 .
how the surface behaves like a stretched membrane and Answer the following questions.
also the origin of surface tension. Inside the liquid, average 5. Consider a major artery of radius 004 em. It carries blood at
intennolecular force is zero. Surfacc tension depends on a flow rate 5 eels. Using Poiseuille's equation, the pressure
tempemture. As temperature is increased, exchange rate of difference per metre will be nearly (take densi ty of mercury
molecules between the liquid and the vapour also increases. ~ 13.6 glee)
You are familiar with the quantitative de fi nition of surface (a) 9.6mmof Hg (b) 3.2 mm of Hg
tension as the force acting per unit length on an imaginary
(c) 1.5mmof Hg (d) 8.6 mm ofHg
line that sepHrates adjacent sections of the surface of the
liquid. It is possible, due to surface tension, that tiny objects 6. If average value of arterial pressure is nearly 100 mm of
of density more than the density of water CHn float on the mercury, the answer obtained in Q.5 suggests that
surfHce of water. (a) viscous effects are significant in major arteries, so
Consider an insect with e ight legs standing on water. The Bernoulli 's equation is not applicable
end of each leg has a spherical shape of radius I mm. (b) viscous effects are insignificant in major arteries, so
Surface of water makes an angle 600 with the vertical at its Bernoulli's equation is applicable
contact with the leg and surface tension of water at the (c) it is not possible to judge the significance of viscous
given temperature is 72 dyne/cm. effects
Answer the following questions. (d) viscous effects arc significant in major arteries;
I . As temperature is increased Bernoulli 's equation will be applicable because it has
nothing to do with viscosity
(a) average intennolecular separation in the surface layer
decreases; average intermolecular force increases; so 7. The maxi mum speed of fl ow is at the centre of the artery
the surface tension will increase. 'aorta' and it is nearly 36 cm/s. Using Bernoulli equation, it
562 PHYSICS FOR COMPETITIONS - Vol. I

can be inferred that (take average arterial blood pressure as 12. For given F, maximum elongation oflhe bar will occur if
100 mm o(Hg and density of blood s:: 103 kg / ml) (a) the forces act perpendicular to the rectangular cross·
(a) changes in the velocity of blood cause significant section of the two ends
changes in blood pressure (b) the forces act along the rectangular cross·section at the
(b) changes in the velocity of blood do not cause any two ends
significant changes in blood pressure (c) the forces are inclined at 45 0 with the rectangular
(e) effect of changes in the velocity of blood on the blood cross·section at the two ends such that they are also
pressure cannot be e$limated opposite
(d) changes in the velocity of blood cause sign ificant (d) the forces are inclined at 300 with the rectangular
changes in blood pressure in fat persons cross·section at the two ends such that they are also
opposite
8. Average blood pressure in the leg of a standing person 1 m
below the heart will be nearly (take average blood pressure Paragraph. 4
at the level of heart as 100 mm of Hg; density of blood In Fig. 11 .63, AB is a 2 m long rod of negligible weight. It is
2
=10 3 kg / m 3 ;g= IOm /s ) supported at the ends by wires X and Y of equal length.
(a) 144 mm orHg (b) lOOmmofHg Young's modulus for wire X is 2x Wi ' N/m2 and its area
(e) 92mmofHg Cd) [74 mm ofHg of cross·section 3 mm 2 . For wire Y, Young's modulus is
lAx 1011 N/m2 and its area of cross-section is 6 mm 2 • A
Paragraph - 3 mass M is suspended at any point along the rod.
Fig. 11.62 shows a bar of uniform rectangular cross·seelion
of area S. Equal and opposite tensile forces are applied at
the ends of the bar. Each force has a mag nitudeF. ABCDis , y
a plane through the bar and inclined at angle 41 with the
plane perpendicular to the length of the bar.
B
A B
A

F F
Fig. 11.63
Answer the following questions.
13. Considering all possible positions of suspension of mass M
o along the rod
Fig. 11.62
(a) restoring forces in wires X and Y arc always equal
Answer the following questions. (b) restoring forces in wires X and Y may be unequal
9. Nonnal stress at the plane which is inclined with the plane (c) restoring force in wire X will be greater than the
perpendicular to the length of the bar is restoring force in wire Y
(a) F (b) Feos~ (d) restoring force in wire Y will be greater than the
S S restoring force in wire X
2
Feos $ F 14. Distance ofa point on the rod from A, such that the mass M
(e) S (d)
Scos4I suspended from it will rcsult in equal stresses in wires X
and Y, is
10. Shear stress on the plane which is inclined with the plane
perpendicular to the length of the bar is (a) 8/5 m (b) 7/6 m
Fsin.$ F (e) 4/3 m (d) 2/3 m
(a) s (b) Seos~ 15. Distance ofa point on the rod from A, such that the mass M
suspcnded from it will result in equal strains in wires X and
(c) Ftan$ Fsin 2$
(d) Y, is
S 2S
8 7
II . Shear stress on the said plane will be maximum for 41 = (a) - m
5
(b) lim
(a) 90" (b) 45· 4 2
(e) 30" (d) O· (e) - m (d) - m
3 3
PROPERTIES OF MATTER 563
Paragraph ~ 5 4nwT\R4 nco'lR4
(a) (b)
A sphere of radius 0.1 m and mass T 'Zf'
81tkg is attached to the lower end of a 4
2xCt>l)R
stccl wire of length 5 m and diameter (e) (d) 16n:w'lR 4
10- 3 m. The wire is suspended from 5.22 m T T
5.22 m high ceiling of a room. When 21. A disc rotating with angular velocity W is placed on a
sphere is made to swing, it just grazes viscous liquid of thickness T. Find the angle rotated by the
the floor at its lowest point, due to the disc before it comes to rcst. [Viscosity = TJ, mass of disc ,.,
elongation. Fig. 11.64 M, rndius of disc = R]
y for steel is 2x 10" Nt m 2• g '= 10m / 52 4(o)oTM (b) 2m oTM
(a)
Based on the above infonnation, answer the followin g 'I1 XR2 ~ml2
questions. Ct>oTM. woTM
(eJ (d)
16. Find the extension of wire when the sphere is al lowest ~ml 2 2T\1fR2
point. (Approximately)
ANSWERS
(a) 0.2 m (b) 0.02 m
(e) 2 m (d) 0.002 m Paragraph-I I. (b) 2. (e) 3. (a) •. (d)
17. Find the tension in the wire at mcan position during Paragraph-2 .5. (e) 6. (b) 7. (bJ •• (dJ
oscillations. (Approximately) Paragraph-3 9. (e) 10. (d) 11 . (b) 12. (a)
(a) 100 nN (b) 20hN Paragraph4 13. (b) 14. (e) 1.5. (b)
(e) 3007tN (d) 7.cro
Paragraph-S 16. (bJ 17. (b) 18. (b)
18. Find the velocity of the sphere when it is at lowest point.
(Approximately) Paragrap h-6 19. (0) 20. (b) 21. (e)

(aJ .,J12.8m1, (b) .rn


mI,
[G) Subjective Type Questions
(e) .,J1I4.08 mI, (d) .J5i2 mI,
1. What kind of elasticity is utilised in:
Paragraph - 6 (a) suspension fibre of galvanometer
Consider a disk of mass tn, radius R lying (b) bending of beam
on a liquid layer of thickness T and
(c) cutting a piece of paper
coefficient of viscosity '11 as shown in the
Fig. 11.65. .----------. (d) propagating mechanical waves in a fluid

Answer the following questions.


------------

Fig. 11.65
(e) in blowing a balloon
[Ans. (a) shear (b) linear (e) shear (d) bulk (e) bulk}
2. Draw a schematic graph of the applied force verSlIS
19. The coefficient of viscosity varies as 11 =1l0X (x measured
resulting e longation of a metallic wire. Show the elastic
as shown in the figure) at the given instant the disk is
floating towards right with a velocity v as shown. Find the limit on this graph. What feature of this grnph is related to
the value of Young's modulus of the material of the wire?
force required to move the disk slowly at the given instant.
[Hint: See § 11.1 (D)]
21l0R2V 8TJoR 2U
(a) T (b) T 3. A metallic wire is stretched by suspending a weight to it. If
cr is the longitudinal strain and Y its Young's modulus of
elasticity, show that the elastic potential energy per unit
(bJ volume is given by (112)Ycr 2 .
20. The torque required to rotate the disk at a constant angul ar [Hint: See § 11.1 (F) (5)]
velocity cogivcn the viscosity is unifonnly '11. 4. Explain, why a small spherical ball falling in glycerine soon
attains a constant velocity?
(Hint: See § 11 .2 (8)]
5. Explain, what will happen when two soap bubbles of
different rndii are in communication with each other?
IRool'kee 19901
Fig. 11.66
[Hint: See § [1.3 (C)(I)l
564 PHYSICS FOR COMPETITIONS - Vol. I

6. Show that in case of cylindrical drop excess pressure will [Hint: Sec solved Problem 8)
bc(T l r} [Ans. v = 3.76m/s]
[Hint: Repeat § 11.3 (C) with S = 2wl. and l!.S '" 21Cru-L] 14. A slightly tapering wire of length L and radii a and b is
7. Fill in the blanks - subjected to two equal and opposite stretching forces F
(a) The stress required 10 double the length of a wire of along its length. IfY is the Young's modulus of material of
Young's modulus Y is ......... .... IEAMCET 19901 wire, calculate the extension produced.
[Hine: See solved Problem 10]
(b) The longitudinal strain in a metal bar is 0.05. If the
Poisson's ratio for the material is 0.25, the lateral strain [Ans. 6L = FLJ1tabY)
is ............ . 15. A wire with a length of2Lis stretched betWeen two fixed
(c) A wire ofiengthLandcross-sectional area A is made of points 2L distance apart. A body with a mass M is
a material of Young's modulus Y. If the wire is suspended exactly from tbe middle point of the wire. The
stretched by an amount x, the work done is .. .......... . wire sags by a small amount making an angle 9 with the
horizontal on each side. If the cross-sectional area is A and
(d) If for a metal Y :::: 6.6x 10 10 N /m 2 and B = Il x 1010
2 Young's modulus Y, prove that
N/m , Poisson's raliowill be ...... tEAMCET 19931
B= [Mg I AYj i13
IAns. (a) Y (b) 0.0 125 (e) YAx 2 12L (d) 0.4]
[lIInt: See solved Problem 4 with note.]
8. When a wire of 0.4 em diameter is loaded with 2S kg weight
16. A steel wire of cross-sectional area 0.5 mm is held
2
the length of wire 100 em is found to be expanded to 102
cm. Calculate the Young's modulus of the material of wire? between two fixed supports. If the tension in this wire is
[An,. 9.8 x 109 dyn~cm 2] negligible and it is just taut at a temperature of 20°C,
determine the tension when the temperature falls to O°C.
9. A force ofl06 N /m2 is required for breaking a material. If (Y =2)( 1011 N/m2 and 0. = 12)( 10- 6 r q
the density of the material is 3 )( IOJ kglm 3 , then what
should be the minimum length of the wire made of the same [Ans. 25.2 NJ
material so that it breaks by its own weight? 17. Two rods of different materials but ofcqual cross-section
[Hint: For brcaking(mg 1 A) > BS with m = ALp] and length (1 .0 m each) are joined to makc a rod of length
2.0 m. The metal ofone rod has coefficicnt oflinear thermal
[Ans. 34 m]
expansion 1O- 51"C and Young's modulus 3)( 10 10 N/m 2 .
10. An Indian rubber cord L m long and area of cross-section The other metal has the values 2x 10- 5JOC and 10 10 N/m2
A m 2 is suspended vertically. Density ofrubbcr is Dkglm ) respectively. How much pressure must be applied to the
and Young's modulus of rubber is Y N/m2 . Calculate the ends of the co mposite rod to prevent its expansion when the
extension in the rubber cord under its own weight. temperature is raised by 100°C? (Roorkee 19911
[Hint: § Il.l (F) (3)] [Hint: See solved Problem 13]
(Ans. DgLl I 2 y] (Ans. p = FIA .. 2.25 x 101 N/m 2]
II . Find the natural length of a rod if its length is LI under 18. In Fig. 11 .67, PQ represents a
tcnsion T) and ~ under tension T2 within limits of uniform bar of mass 10 kg
elasticity. supported horizontally by three
ABC
[Ans. (I,T2 - ~1j) 1 (T2 -1j)] uniform wires A,B and C ofthe
same length attached symmetri-
12. A simple pendulum is made by attaching a bob of I kg to a
calJy to thc bar. The wires A
Sm long COpper wire of diameter 0.08 em and it has a
and C are of copper 1 sq. mm in p a
certain period ofoscillation. Next a 10 kg bob is substituted
cross-section and B of iron 2 Fig. 11 .67
for the I kg bob, calculate the change in period if any.
sq. mm in cross-section . If the Y for copper and iron is
[Y = 12.4 )( 10 10 N/m2]
Ix 101[ and 2)( 10[ 1 N/m 2 respectively, find the tension in
[Hint: See solved Problem 12 in Chaptcr 8]
each wire.
(Ans. AT = 0.0031 5]
[Hint: The bar will remai n horizontal if(t.L>cu = (M' )Fe a nd for
13. A sphcre of radius 10 em and mass 25 kg is attached to the
its equilibrium 2FCU + FFc "" lOkg]
lower end of a steel w ire which is supported from the
[An s. To. = (5/3 ) kg wt. and TFc = (20/3) kg wt.]
ceiling of a room. The point of SUPP()r1 is 521 em above the
floor. When the sphere is sct swinging as a simple 19. A sphercconlracts in volume by O.O t % when subjected toa
pendulum its lowest point just grazes the floor. Calculate the normal uniform pressure of 100 atmosphere. Calculate the
velocity of the ball at its lower position. (Y = 2)( lOll N/m 2 ; bulk modulus of its material in COS unit.
12
unstretched Icngth of the wire = 500 cm and its radius is (-1ns. 10 dyne/cm2l
0 .05 cm)
20. Find the density of sea water at a depth where the pressure much work will be done? (Surface tension of water = 72
is 9.8x 10 7 dync/cm 2 in excess of the pressure at the dyne/cm)
surface. The density of water at the surface is Iglcc and the [Ans. 144 erg]
compressibility of water is SOx 10-6latmosphere. 27. Calculate the amount of energy evolved ~hen n identical
(1 aIm = 106 dyne/cm 2 ) droplets of a liquid of surface tension T merge into one.
[Hint: See solved Problem 16] [Ans. 41(1.2/1T [ I - (lI/1f3]]
[Ans. 1.005 glee) 28. What would be the pressure inside a small air bubble of
21. Find the force which must act tangentially over the surface radius 0.01 mm situated just below the surface of water?
of a body of surface area 18 cm 2 in order to produce a shear The surface tension of water is 75x 20- 3 N/m and
of 2". Crt = 9x lOll dyne/cm2) I atmosphere = 1.01x 10 5 N/m 2 .
[Ans. 5.65 xlOll dyne] [Ans. 1.16 x lOs N/m2 ]
22. A highly rigid cubical block A of small massM and side Lis 29. Two separate air bubbles of radii 'i and 'i formed of the
fixed rigidly on another cubical block B of the same same liquid come together to fonn a double bubble as
dimensions and of low modulus of rigidity 11 such that the shown in Fig. 11.40. Show that the radius of their interface
lower face of A completely covers the upper face of B. The is given by
lower face of B is rigidly held on a horizontal surface. A
r= -'i'i-
small forcc F is applied perpendicular to one of the side
r2 -rj
faces of A. Calculate the time period with which A will
execute oscillations when the force is withdra\\-ll. What angles will be fonncd betweenlhe films at the points
of contact?
lIlT 19921
[Hint: See § 8.4 [AJ (c)] [Hint: See solved Problem 37. Further for equilibrium at the point
of contact as forces are equal, the angle between them will be
[Ans. T = 21t,JM I ..,L ] equal.]
23. What is the Poisson's ratio of the material ofa wire whose [Ans. 120°]
volume remains constant under an external nonnal stress? 30. Two spherical soap bubbles coalesce. IfV is the consequent
[Hint: V '" rcR 2L; differentiating, find (J = -( (:) / ( ~~) ] change in volume of the contained air and S the change in
the total surface area, show that
[Ans. 0 '" 0.5] 3PV +4ST=0
24. In Millikan experiment an o il drop, in absence of any wherc T is the surface tension of the soap bubble and P the
electric field, is seen to be falling with a constant speed of atmospheric pressure.
0.04 crn/s. If the dcnsity of the oil is 0.98 glcc and viscosity [Hint: Repeat solved Problem 38 with Po",P; V ",(4 / 3)1t
of air is 1.8x 10-4 poise, calculate the radius of the drop.
(a 3 + b3 - c3)and S '" 41t(a 2 + b 2 _ c2»)
Neglect the force of buoyancy due to displaced air.
31. A U-tube whose ends are open and whose limbs are vertical
[Ans. r '" 1.84 x 10-4 em)
contains oil of specific gravity 0.8 and surface tension 28
25. An air bubble of diameter 2 mm rises steadily through a dyne/cm. If one limb has a diameter of2.2 mm and the other
solution of density 175x]03 kg/m 3 at a rate 3.5 crn/s. of 0.8 mm, what is the difference in level of the oil in the
Calculate the coefficient of viscosity of the solution. The two limbs?
weight of the air bubble is negligible.
[Hint; Repeat solved Problem 36 with PJ = P2 '" po]
[Ans. I J.3 em]
[Hint: 'See solved Problem 26]
[Ans. 1.1 poise]
26. A water film is made between two parallel wires of length
20 em and at a distance of 0.5 em from each other. If the
distance betwecn the wires is increased by I mm, how
Vibrations and Waves
§ 12.1 Vibrations offer resistance to the motion. As a result of this thl
When a body capable of vibrating in the absence of any r.lechanical energy of the body gradually decreases. The bod~
dissipative force under the action of a linear restoring force therefore vibrates with a gradually decreasing amplitude am
executes simple hannonic motion with a single natural finally comes to rest. This type of motion is calle(
frequcncy* depending upon its dimensions and elastic or some damped-oscillatOlY and vibrations damped. The motion of,
other constants [as discussed in detail in Chapter 8], the period pendulum in a liquid, the motion of the coil of (\ ballisti(
of motion is independent of amplitude and the body vibrates galvanometer, oscillatory motion of charge in L- C-R circuit, \(
indefinitely with constant amplitude. Oscillations of a and fro motion of a metallic strip in a magnetic field 0
pendulum, tuning fork or string all in vacuum can be cited as vibration of electric and magnetic fields in an eiectromagneti(
examples, of this type of motion. The vibrations of electric and wave propaga~ing through metals are examples oflhis type.
magnetic fields in an electromagnetic wave propagating in free
space is the best possible example of this type. This type of ~y

vibrations are called free or undamped vibrations and are


analytically represented by
d 2y (.)
m--=-ky with solution y "" asin[oot+$] .... (i}
dt 2 \

(.)

(h)
Fig. 12.2
In case of damped vibrations if we assume that dissipativ,
Fig. 12.1 force is non-periodic and varies linearly with velocity (whicl
However, when forces other than linear restoring force, is usually the case). the differential equation of simpl,
such as dissipative, non-periodic or periodic are impressed on harmonic motion of this case will become:
the body executing SHM (i.e., free vibrations) the motion of d 2y dy asv= dy
m- -"=-ky-y- [
the body gets modified and depending on the nature and type dt2 dt dt
of the impressed force the resulting vibrations can be classified
in the next four types: (where y is called damping constt.) which for light dampin:
i.e., (b=yl2m)< 00 0 on solution yields
IAI Damped Vibrations
When dissipative forces such as friction or viscous force y=Ae-b1sin(ooDt+$) with ooD=~(oo6-b2) .... (ii
acts on a body executing SHM (i. e., free vibrations), the forces
'" Ifa body vibrates with more than one natural frequency (i. ~ ,higher harmonics) simultaneously its motion will not be SHM but periodic and will ~
dealt with Fourier-theorem.

r
From this equation it is clear that in case of dampcd d'y dy ..
m-~ =-ky-y-+Fo smWf
vibrations: dt2 dt
(1) The body vibrates with a frequency wD which is
slightly lesser than the natural frequency of the body On solving above equation, we get:
y = A sin (wt -$) ... (iii)
"0'
F
(2) Amplitude and hence mechanical energy E(oc A2) With A ::: 0
2
m[(w~ - ( 0 )2 +(ywlm)2]112
both decrease exponentially with time, i. e.,
AD = A e-bi an dE D='Oe
E -2bt I
'hE o=2'mwo
WIt 'A' ... (iv)
[8] Maintained Vibrations
Ifin the presence of dissipative force a body is maintained From this equation it is dear that in case of weakly
vibrating with its natural frequency and constant amplitude by damped forced vibrations:
supplying energy to it from some external non-periodic source (I) The hody vibrates with the frequency ofapplied forci!.
of energy, the vibrations of the body are called maintained. 00 (and not with its natural frequency (0 ),
0
Though the source of energy in this case is non-periodic, (2) The amplitude isfinite and constallt. It depends on the
periodicity is imposed upon it by the vibrating body. It frequencies of applied force, body and damping.
therefore supplies energy to the vibrating body periodically Lesser the difference in frequencies and lesser the
and makes up the loss due to dissipative forces. In ~h, damping, greater will be the amplitude of vibration.
main
. spring supplies external energy ,
~ ,."..
, 1...Jl (3) The resulting displacement ofth!! body is not in phase
m such a way that the balance wheel ~. I 'F =u
with the applied force. It will lag or lead the applied
moves as if it were undamped. Other -:: =- ~ i
examples of maintained vibrations arc I
1; f.
force as the frequency of applied force is lesser or
greater than the natural frequency of the body.
electrically maintained tuning fork or
electronic oscillator (in which the
valve maintains the oscillation of
? f
~
' However, if damping is absent,
A = _-,F"o'---,~ and
charge in L-C circu it). In case of ~ m(w~ - (0 2 )
maintained vibrations: Fig. 12.3
IDJ Resonant Vibrations
(l) The body vibrates with its natural frequency and
constant amplitude in presence of damping. It is a special case of forced vibrations. If a body is set
(2) The external source of energy is non-periodic and
compensates for the energy loss due to dissipative
equal to the natural
frequency of the body,
I
vibrating by an external periodic force whose frequency is

forces by feeding energy to the vibrating body


i.e., 00 = 00 0 , the
through its periodicity.
amplitude of vibration A r
[C] Forced Vibrations increases at each step
When a system capable of vibration is subjected to an and becomes very
external periodic force of different frequency, the motion in the large* (theoretically 00
early stages is irregular. The system tends to vibrate with its for no damping). Such
natural frequency while the impressed force tries to impose its vibrations are called
own frequency upon the system. During this tussel the sympathetic vibrations
amplitude rises and falls several times but ultimately the system or resonant vibrations
vibrates with the frequency of force (and not with its own and the phenomena
natural frequency). Such vibrations are calledforced vibrations. resonance.
ro = roo .
For example, if we press the stem of a vibrating tuning fork Fig. 12.4 shows the Fig. 12.4
against a table or speak in a microphone, the table or diapbram variation of (displace-
of microphone vibrates with the frequency of tuning fork or ment) amplitude with frequency for different values of
speech respectively (and not with its own natural frequency). damping. From these cUlves it is clear that smaller the
If a periodic force F = Fo sin wt is applied to a body of damping, taller and narrower is the resonance peak. Such
natural frequency 00 0 in presence of small damping, the resonance is called sharp resonance.
modified differential equation of motion for this case will be:

.. At resonance (i.e., OJ '" roo)' displacement ampli tude is vel)' large but not maximum. The theory shows that it is maximum for frequency wh ich is slightly
lesser than (!lo[ '" (ro~ - 2b 2 )!12]. Actually at resonance the amplitude of velocity is maximum.
Furthermore, for weak damping at resonance the phase of Problem 2. r{the amplitude afvelocity ofa particle acled
displacement lags behind the force by (n/2) [as on substituting on by aJorce F = Fo coswt along x-axis is given by:
ro = roo with y ;t; 0 in Eqo. (iv), we get $ = tan - 100 = 7tl2]. 1
Vo = 2
The phenomenon of resonance has wide range of (aw _bw + c)112
applications in various fields of physics, e.g., find the condition oJresol/ance and resanantJrequency.
(a) If a tuning fork is vibrated, another object in the Solution: At resonance Vo will be max ~ 00
neighbourhood (having frequency equal to vibrating
tuning fork) begins to vibrate automatically due to So aw 2 - bw+c= min-tO .... (i)
resonance.
(b) If a vibrating tuning fork is held near the mouth of a i.~. , L(aw 2 - bw+c)=O
dw
resonance column and level of water in it adjusted, a
loud sound will be heard due to resonance when the 2aro-b=O,
natural frequency of the air column (body) in the tube i.e. , resonance frequencyro = (bI2a) .... (ii)
becomes equal to that of the tuning fork (force). Further substituting the value ofw from Eqn. (ii) in (i), we
(c) Ifa vibrating tuning fork is held near a stretched string get
and the length of the string is adjusted, vibrations of
large amplitude will be produced in the string due to i. e., b 2 '" 4ac .... (m)
resonance when its natural frequency becomes equal
to that of the tuning fork (force). which is the required condition of resonance.
(d) Soldiers are not allowed to march in steps while Alternative solution: At resonance Vo ~ 00.
passing over a suspension bridge. This is because if
incidentally the frequency of their march coincides so aro 2 -bro+c=O
with the natural frequency of the bridge, due to
resonance, the bridge will vibrate violently and may
i. e., w=[b±~(b2 - 4ac) JI2a .... (i)
collapse. Now as at resonance ro is single valued, which is possible
(e) When we tune a particular station on our transistor or only if
T.V., wc make the frequency of our set equal to that of b 2 -4ac=O, i. e.,
the dcsired station so that due to resonance it catches
the desired station. Without the phenomenon of which is the required condition of resonance, and so at
resonance, radio or T.V. would be useless as they will resonance ro =(bI2a) [substituting b 2 = 4ac in Eqn. (i)]
then receive only a very weak jumble of different
frequencies .
§ 12.2 Waves
(AJ Physical Concept
Note: In all these examples if 00 "" 000, the vibrations produced in the (a) What a wave is
body will be forced.
A wave is a disturbance which propagates energy (and
Question I. Can a great singer cause a glass object to momentum)from one place 10 the other without the transport
shatter by his singing? Explain with reason. ofmatter. It is well spread over a region of space without clear
cut boundaries. It cannot be said to bc localised here or there. It
Answer: Yes. If the frequency of his singing coincides
is hard to think of any mass being associated with a wave.
with the natural frequency of the glass object, the glass~object
Moreover, quantities like amplitude, wavelength, frequcncy
will vibrate violently due to resonance and may get shattered.
and phase are used to characterise a wave which have no
Problem 1. If the amplitude of a damped-oscillator meaning for a particle. The most important characteristic of a
becomes one-third in 2 minutes, what will be the amplitude of wave that differentiates it from a particle is diffraction.
oscillation as compared to initial one after 6 minutes? Diffraction is a convincing prooJoj wave-nature*.
Solution: In case of damped oscillations, amplitude
decreases exponentially with time, i.e., Note: Encroachment or spreading of waves around the edges of
apertures or opaque obstacles is called diffraction and can be
A = Aoe- bt , i.e., (AIAo) = e- bt

so 1 = e- bx2
3
and -.!L = e- bx6
Ao
-:v1[ --- --- Diffraction

~ =(e-2b) 3= [~J3 Particles Obstacle Screen Waves


Ao 3 27 Fig. 12.5
* Diffraction ofelectrons by metals in Davison- Germer or Thomson experimenrsestablished the wave nature of electron and hence existence of mailer waves.
explained only on the basis of wave-nature. Diffraction hikes waves. Waves on springs or sound waves in air are examples of
place when wavelength of wave is comparable to the dimensions
longitudinal waves.
of diffracting objects.

Waves can be one, two or three dimcnsionai according to Vibration


( )
the number of dimensions in which they propagate energy. )
Waves moving along strings are onc·dimensionat. Surface Wave motion
waves or ripples on water are two dimensional, while sound or
light waves travelling radially out from a point source are (8) (b)
3·0. Fig. 12.7
(b) Mechanical and Non-mecha nica l Waves The transverse or longitudinal nuture 0/a wave is decided
A wave mayor may not require a medium for its by 'polarisation' as transverse wave can be polarised while a
propagation. The waves which don 't require medium/or their longitudinal wave cannot be. This is shown diagramatically in
propagation are called non-mechanical, e.g., light, heat Fig. 12.8.
,.
(infrared) and radio waves are non-mechanical as they can
propagate through vacuum. In fact all e lectromagnetic waves (~H+t~
.\ r,;;:
(EMW) such as y-rays, X -rays or microwaves are
non-mechanical. On the other hand the waves which require Unpo!arised
medium/or thcirpropagation are called mechanical waves. In (a) (b)
the propagation of mechanical waves elasticity and density of Fig. 12.8
the medium play an important role. This is why mechanical
Note: In the opinion of the author till today all nOIl-me&hOlli&oiwoves
waves sometimes are also referred 10 as elastic waves. Waves are Irallsverse. This in tunl implies that:
on strings and springs, seismic waves or sound waves arc (i) If a wave is longitudinal it is mechanical, but if a wave is
familiar examples of mechanical waves. mechanical it mayor may not be longitudinal.
One cannot listen his companion on the moon, or sound (ij) If a wave is non-mechanical it is transverse, but if a wave is
from the sun does not reach the earth because there is no transverse it mayor may not be non-mechanical.
medium for its propagation. (d) Mechanical Waves in Different Media
Note: Apart from mechanical (elastic) and non-mechanical A mechanical wave will be transverse or longitudinal
(electromagnetic) waves there is also another kind of waves depends 'on the nature of medium and mode of excitation.
called 'matter waves'. These represent wavelike properties of
particles and arc govemed by the laws of quanlum physics. In strings mechanical waves are always transverse, that
too when the string is under a tension. [See Question III]
(c) Transverse and Longit udinal Waves In gases and liquids mechanical waves are always
Mechanical waves are further divided into two types: longitudinal, e.g., sound waves in air or water. This is because
(i) Transverse waves: If the particles of the medium fluid s cannot sustain shear.
vibrate al right angle to the direction of wave motion or energy In solids mechanical waves (may be sound) can be either
propagation the wave is called transverse wave. These are transverse or longitudinal depending on the mode of
propagated as crests and troughs. Waves on strings are always excitation. The speeds of the two waves in the same solid arc
transverse. different (longitudinal waves travel faster than transverse
waves), e.g., if we strike a rod at an angle as shown in Fig. 12.9
Vibration C C C C (a), tlie waves' in the rod will be transverse while if the rod is
Wave motion struck at the side as shown in Fig. 12.9 (b) or is rubbed with a
T T T cloth the waves in the rod will be longitudinal. In the case ofa
(a) (b) vibrating tuning fork the waves in the prongs are transverse
Fig. 12.6 while in the stem are longitudinal. Furthennore, in case of
seismic waves produced by earthquakes both S (shear) and P
(ii) Longitudinal waves: If the particles of a medium (pressure) waves are produced simultaneous ly which travel
vibrate in the direclion of wave motio n, the wave is called through the rock in the crust at different speeds [u S 'Z 5 km/s
lo ngitudinal. These arc propagated as compressions and while vp "::: 9 kmls]. S-waves arc transverse while P-waves
rarefactions and also known as pressure or compressional longitudinal.
~C C c ,C R eR e produced. This in tum implies that longitudinal waves cannot
~ ~I -+ , -+ be propagated along a string (So the waves in a string are
T T always transverse, that 100 when the siring is under tension. If
Transverse waves longiluclinal waves
tension in the string is zero, transverse mechanical waves will
(a) (b) also not propagate as then v = J(TI m) = 0].
IB} Analytical Concept
To describe the motion of a particle in one·dimcnsional
space only. a function of one variable such as f (I) is sufficient,
e.g., the motion of a particle moving along x·axis is given by
x = ut + ~ at 2, i.e.,x = J( t). However, to describe a wave of

any type mathematically functions which depend on two


variables space and time, i. e.,J(x, t) are required. It has been
shown analytically that any function of space and time which
satisfies the equation,
B d 2 y. _ I d 2 Y
Ripples
Fig. 12.10 dx Z - v 2 dt Z
Question II. Distinguish between sound and radio waves represents a wave, e.g. , functions y = A sin rot or y = A sin kx
ofthe samejrequency (sayI5 kHz). don't satisfy the above equation, so don 't represent waves,
Answer: Following are the points that distinguish a sound while functions A log (at + bx), A sin (rof - lex), A sin kxsin rot
wave from a radio wave: or {Asin (W/-h)+Bcos(rot+kx)] satisfy the above
equation, so represent waves.
(I) Sound waves cannot travel through free space while
radio waves can. It has also been established that if in the wave-functions
space time variables appear in the combination of (at + bx),
(2) The speed of sound in air is 332 mls while of radio
the function represents a travelling wave, i.e., the equation ola
waves 3 x 10 8 mls. travelling wave is of the form
(3) With rise in temperature the velocity of sound y = F(at + bx) .... (ii)
increases while that of radio waves does not change.
negative sign between t and x implies that the wave b,
(4) Sound waves can be detected by the human ear if its travelling along posjtive x-axis and vice-versa, i.e.
frequency lies between 20 Hz-20 kHz while radio
y = ~(ax + bt), Y = (ax - bt)2, Ae-B(x-Vl)l ,A sin (ax - bt) Z
waves of any frequency cannot be.
(5) Sound waves can never be polarised (excluding the y=acos 2 (CIlt-la:) or y = acos.1.rotsin (rol-kx) represent!
transverse mode in solids) while radio waves can travelling waves while y=Asin(4x z _ 9t z ), y =Asir
always be polarised.
(ax 2 + bt)or y =cosla:sinrotdon' t represent travelling waves
Question III. Explain why (a) transverse mechanical
[However, they may represent a wave if they satisfy Eqn. (i).]
waves cannot be propagated in liquids and gases while (b)
If a travelling wave is a sin or cos fun ction·'" of (at - bx
waves on strings are always transverSe.
or (at + bx), the wave is said to be harmonic or plant
Answer: (a) To transmit a transverse mechanical wave
progressive wave. Here we shall limit ourselves to I-D plant
the medium must be elastic so as to provide a restoring force
progressive wave which in iI's most general fonn is given by
when acted on by shearing stress. But liquids and gases flow
when acted on by shearing stress, i. e., they cannot sustain shear y=Asin(rot+kx+¢t) .... (iii'
stress to provide restoring force and so cannot transmit From Eqn. (iii) it is clear that a set offourparameters A.¢t,
transverse mechanical waves. ro and k completely describes a plane progressive wave.
(b) Longitudinal waves are pressure waves, i. e., they are (1) As the maximum value of sin or cos functions can bl
transmitted as compression and rarefaction in a medium. Now I, A represents the maximum value of wave-functiOl
as the string is non·stretchable so it can neither be compressed and is called the amplitude of the wave.
nor stretched, i. e. , in it compression and rarefaction cannot be
• Here yis used to rep resent wave function, i.e, yis a funetion of space and time and is not a y eo-ordinate .
•• Periodic functions such as tan (w- lex) arc not uscd in physics for describing a wave motion as thcsc become co for some value of (001- kx) during
period while in a wave displacement is always finite.
(2) The constanl$ is cai ledphasecollstantor initial phase So the wave will repeat itself if y= y, i.e.,
and enables us to find the position from where time is x' =x + (21tlk) as sin {9 ±2x) =sin€! .
considered. If at t ""O,x ""0,, will be zero which is Now as the distallce after which the wave repeats
usually the case with a wave and implies that in wave itseJfis called wavelength A, so
motion time is considered when the wave was at the
origin. Henceforth we shall assume $ "" 0 and the A=x'_x=2x i.e. , k = 21& ... .(v)
k ' '-
wave is travelling along positive x-axis unless stated
otherwisc. k is called propagation constant or wave vector and
has unit (radlm). The constant k or wavelength A
(3) As the wave at a given position at time t' [Fig. 12.1 I
depends on the nature of the medium (as same source
(a)) w;1I be
will produce waves of different wavelengths in
y "" A sin (wt ' - kx) different media) and also on the source producing the
So the wave will repeat itself if y= y, i.e., waves (as in a given medium sources of different
t' = I + (21t1ro) as sin (8 + 21t) =sin8. frequencies will produce different wavelengths).
Now as the time after which a wave repeats itseJf is (5) If the shape of the wave does not change as the wave
called time period. i.e., propagates in a medium, with increase in t,x will also
T=t' -t =(27t1w) increase in such a way that
Further as the rate at which the wave repeats itseJf is oot - kx = constant .... (vO
called its frequency f(with units Hz) so The argument ofharmonic function (rot - kx) is called
phase of the wave and is constant if the shape of the
J=.l=~ ····Ov) wave remains unchanged.
T 2.
orro = 2xf.w is called angular frequency (with units Further if we consider two points at positions XI and
x2 on a wave at a given instant, then
rad/s). Here it is worth noting that ro,f or T are the
characteristics of the source producing the wave and $1 =oot-kx l and $2 =rot - kx 2
are independent of the nature of the medium in which so $, -$, =k(x, -x, )

[ask=~'l
the wave propagates.
i. e., . ... {vii)
x. COIlS" .
:-- T.. ; .. f--: From this it is clear that if Ax = A, 6.~ =21T, i.e., a path
y difference A corresponds to a phase change of (2x)
~_--!-¥"--_.'..f __L,L3T/2 rad.
(6) As the phase of a plane progressive wave· is constt.,
i.e.,
Displacement 01 a particle roi - kx =constant
al different instants
the so called wave or phase velocity will be given by
lal
t .. constt. v=-
dx w 2>if
= - "" - - = j')... ....{viii)
dt k (27t1,-)
and depends only on the nature of the medium in
which the wave propagates and is independent of the
source producing the waves.
Now if in a medium, waves of different wavelengths
Position 01 different particles
travel with same velocity the medium is said to be
at same instant in a string non-dispersive and energy will also be transmitted at
Ibl same speed, i. e., v. However, ifin a medium velocity
Fig. 12.11 of different wavelengths is different (e.g., for light in
water or glass), the medium is said to be dispersive
(4) As the wave at a given time at position x' [Fig. 12.11
and energy is not transmitted with wave or phase
(b)) wm be velocity but with group velocity,
y = A sin (rot - lui)
(iii) The coefficient of t gives angular frequency
v = dro* = v - '}.. dv .... {ix) 00[;: 2TCf: (2TCIT)].
g dk <fA.
(iv) The coefficient of x gives propagation constant or wave
which is less than wave or phase velocity. number k(: 2nl)').
(7) As a plane progressive wave propagating aiong (v) The ratio of coefficient of t to that of x gives wave or phase
positive x-axis with t = 0 at x = 0 is given by velocity, i.e.,v ~ (oolk) and is constant for a given medium.
(vi) Panicle velocity will be obtained by differentiating y with
y=Asin(rot-kt) respect to t, i.e., uPa :: (dyldt) = -u (slope of ylx curve) and
so the velocity of a particle on it will be is not conStl. wi th maximum value = Aoo.
(vii) When a given wave passes from one medium to another its
vPa = ~ =Arocos (rot - fa) .... (x) frequency does not change, so from v = fA
{VI )/(v2 ) '" (A] )1(1.. 2),
Sutas (viii) In case of vibrations of string (in its fundamental mode)
cos (ro, - kx) =~ {l - sin ' (ro, - kx)} = ~l - (y/ A)' "' ;: 2 [length of sIring].
(ix) While using equation v '" f'}.., be careful to decide which oul
so .... {xi) ofv,fand A. is constant (See solved Problems 6, 11 and 14).

It w ill be maximum when Question I V. The wave functions, y=A~(x - tJt),


y2 : min=O
y = (x + vt)3 ,y = A(x - vl)or Y "" A log (x + vt)thollgh are of
i.e., (vpa)mu =Aro .... (xH) the/orm y = I(x ± vt), but not useful in wave motion. Explain
Furthermore, the slope of the wave will be why?
dy = - Ak cos(wt-kx) Answer: In dealing with wave motion we usually assume
dx that the wave function is continuous, single valued, harmonic
which in the light of Eqn. (x) yields and finite. No doubt the given wave functions are continuous
and single valued, are not useful in wave motion as these are
dy =-Ak vPa = _ vPa [asv = ~k] neither harmonic nor finite for all valuesofxand t [asx -+ 00 or
dx AID v
t -+ 00, f(x, t) -+ 00]'
i.e., VPa = -v x (slope) .... (xiii) Problem 3. The equation for the displacement of a
i.e., particle velocity at a given position and time is stretched string is given by

-~]
equal to negative ofrite product ofwave velocity with
slope of the wave at that pOint. y= 4 Sin 21t[ _t_
0.02 100
(8) A plane progressive wave (either transverse or
where y and x are in cm and t in sec. Determine the (0)
longitudinal, mechanical or non-mechanical) in the
direction in which wave is propogating'(b) amplitude (c) time
light of above can be written in many forms such as:
period (d)frequency (e) angular f requency (f) wavelellgth (g)
y=Asinlro, - kx] ....{a) propagation constant (h) velocity of wave (i) phase constant
or y= A~n2_lft - (x/A )] and (j) the maximum particle velocity.
las ro = 2_/ and k = (2"')' )] Solution: Comparing the given equation with the gene ral

or Y=Asin21t[~-~] [a sf = lIT]
wave equation-

Y = Asin 2it[~-~+<jl ]
. T)'
y= Asin(wt -kx+ <jl ), i.e.,
or y=Asink[vl - x] [asin(a),v = ro/k}
or y= A sin ro [t - (xlv)] [as in (a), klro= IIv] we find that:

Note: In solving numerical problems related to equation of plane


(a) As there is negative sign between t and x terms, the
progress ive wave remember that: wave is propagating alollg positive x-axis.
(;) If tho s;gn "'!wee"I ..d x " nos ;, ",g" ; ~ tho w". ;s (b) The amplitude of the wave A = 4cm = 0.04 m.
propagating along positive x-axis and vice-versa.
(c) The time period of the wave T ""0.02 s = (1 /50) s.
(ii) The coefficient of sin or cos function, i.e.,A gives the
amplitude of the wave while its argument (rot + Icc:) denotes (d) The frequen cy of the wave f = (l IT) =50 Hz.
phase. (e) Angular frequency of the wave ro = 21tf = lOOn radls.

dk dk
d
'" -doo .. - (vk);:v+k -dv · -dJ.. _v+k_
d"A dk
. dU(-'<)
-
rf).. k 2
(f) The wavelength of the wave A = 100cm = I m. i. e., the wavelength of transverse waves propagating on string
(g) The propagation constt. [= wave vector = (2rrJA) is 2 m. Now as the frequcncy of wire is given to be 330 Hz, so
=21trad/m.] from v = fA velocity oftransverse waves along the wire will be
(h) The velocity of wave v = fA = 50 x I = 50 mls. v wire = 330 x 2 = 660 mls
0) The phase constt., i.e., initial phaseljl =0. i.e., for transverse mechanical waves propagating along the
0) The max particle velocity (vpa)max =Aro=O.04x wire,
I 001t = 41t mls. f=330Hz, A=2m and v=660mls
Problem 4. An observer standing at sea coast observes 54 (b) Here vibrating wire will act as source and produce
waves reaching the coast per minute. If the wavelength of the sound, i.e., longitudinal waves in air. Now as frequency does
waves is 10m, find the velocity. What type of waves did he not change with change in medium so f = 330 Hz and as
observe? velocity in air is given to be = 330 mls so from v = fA
Solution: As 54 waves reach the shore per minute, Aair = (v ai /f)=(330/330)=lm
f ~ 54 ~1-Hz i. e., for sound (longitudinal mechanical waves) in air produced
60 10 by vibrations of wire (body),
And as wavelength ofwa':es is 10 m f=330Hz, A=lm and v = 330rn/s
9
u=fA=~xI0=9m1s Note: From tltis problem it is clear that with change in medium velocity
10 and wavelength change while frequency remains constant, i.e.,in
The waves on the surface of water are combined this problem as f = constt. v oc A.:
transverse and longitudinal called 'ripples'. In case of surface
Problem 7. A progressive wave offrequency 500 Hz is
waves the particles ofthe medium move in elliptical paths in a
vertical plane so that the vibrations are simultaneously back travelling with a velocity of360 mls. How far apart are two
and forth and up and down. (See Fig. 12.10) points6000utofphase? JMNR 19931
Solution: We know that for a wave v = fA,
Problem 5. A light pointer fIXed to one prong ofa tuning
fork touches a vertical plate. The fork is set vibrating and the
So
plate is allowed to fall freely. 8 complete oscillations are
counted when the plate falls through 10 cm. What is the
Now as in a wave path difference is related to phase
frequency afthe tuningfork?
,, difference by the relation,
Solution: Time taken by plate to
fall 10 em freely under gravity
,( j phase difference.6.1jl = 21t (path difference Ax)
l.
t~ {2h ~ ~2xI0 ~!s Here, phase difference.6.1jl =60° = (rrJI80) x 60 = (rt/3) rad
Vii 980 7
So path difference Ax = ~ (.6.~)
And as in this time, i.e., (1/7) s 8 2n
oscillations are recorded on the plate,
the number of oscillations per sec., =0.72x~=O.l2m
2n 3
i.e., frequency of the tuning fork will Fig. 12.12
be=7x8=56Hz Problem 8. A transverse wave is travelling along a string
from left to right. Fig. 12.13[0] below represents the shape af
Note: This experiment is caned 'falling plate' method and in the the string (snap shot) at a given instant. At this instant (a)
laboratory is used to detennine 'g' if f is given and vice~versa. which points have an upward velocity? (b) which points will
have downward velocity? (c) which points have zero velocity?
Problem 6. A sonometer wire 100 cm in length has a
(d) which points have maximum magnitude of velocity?
fundamental frequency of330 Hz. Find (a) the velocity af
propagation of transverse waves along the wire and (b) the Solution: For a wave vPa = -v x (slope)
wavelength of the resulting sound in air if velocity ofsound in i. e., particle velocity is proportional to the negative ofthe slope
airis330mls. of ylx curve; so
Solution: (a) In case of transverse vibration of string for (a) For upward velocity vPa = positive, so slope must be
fundamental mode: negative which is at points D, E and F.
L=(A/2), i.e., A =2L=2xl=2m (b) For downward velocity vPa = negative, so slope must
be positive which is at A, Band H.
c Further amplitude is the maximum value of wave function
which will be when
(x+2t) 2 ",, 0

So A "" Ymax "" l~ = 2


Problem 10. The amplitude of a wave disturbance
propagating in the positive x direction is given by

y y"" 1 at t ",, 0 and Y "" I at t o:= 2s


[I+x'] [I+(x - I)']
where x and yare in m. The shape of the wave disturbance
does not change during the propagation. What is the velocity
ofthe wave?
Solution: In telms of power-function a pulse is given by
y= a
G b + (x + vt)2
(b)
A comparison of given equations with the above reveals
Fig. 12.13
(x =F VI) 2 == (x _ 1)2 for 1=2 s
(e) For zero velocity slope must be zero which is at C
andG. i.e., vi = l for t=2s
i.e., v = (1/2) = 0.5 m/s
(d) For maximum magnitude of velocity Islopel :::; max
which is at A and E. Note: In this question the word 'amplitude' is misused for
Problem 9. A travelling wave pulse is given by 'displacement' as amplitude is the maximum value of
displacement and is constant which is here I as (x + vt)2 ~
Y= 10 mi nimum "" 0.
5 + (x + 2t)2
Alternative solution: At I = 0
In which direction and with what velocity is the pulse
propagating? What is the amplitude o/pulse? y=- 1- i. e.,
I +x 2 '
Solution: A wave pulse is a disturbance localised only in
a small part of space at a given instant [as shown in Fig. 12.14]
andatt = 2 Y= 1 , i.e.,
[I + (x - I) ' ]
x -x 1
So v :: 2 I = - - = 0.5m1s
12 II 2-0

Ie) Velocity of Transverse Mechanical Waves


, Velocity ofa transverse wave propagating along a string
having tension T and mass per unit length m, is given by
Fig. 12.14
and its shape does not change during propagation. Though a v = )(Tlm)
pulse can be represented by exponential or trigonometrical To prove this consider a small segment of the pulse on a
functions also, it is usually expressed by the fonn string of length !!.L fonning an arc of the circle of radius r as
shown in Fig. 12.15.
y= a Al
b + (x + vt) 2 --0 -- ,, --x 9--
, ,, , to ,,
Comparing the above with the given pulse we tind that T
,,, ,, , , T

f(x+vt)=(x + 2t)' ,, 0 ,,,


,, ,
,,
i. e. , the pulse is travelling along negative x-axis with velocity
2 mls.
....... _----
Fig. 12.15
A force equal to tension in the string p~lIs this segment Nowasv =f').. ---------------------
------------------
tangentially at each end. The horizontal components of these
forces cancel while the vertical components add to fonn a
and f = 750Hz, L --

VA = 2x750=1500mls
restoring force,
i. e., A. = 2m,f = 750Hz
F =2Tsin8
andv = 1500mls
But heresin8 =8 and 28 = (Mlr) Fig. 12.16
(b) Now as in case of a
So F = T(29) = T(l!.Llr) .... (i) wire under tension
Now as the string element is moving with velocity v in a
arc of circle it has a centripetal acceleration towards the centre V= iT so vA =~TA , i.e., vB =1500~TB
of the circle
V; VB TB TA

Furthennore if m is the mass per unit length of the string,


....(ii) or VB =1500~: =1500~g[1-g"/Pl [asg'={l-~)l
the mass of element
.6.m = m / L .... (iii)
or = 1500~1-~
Va = 1000 mls

Now from Newton's II law for the element, Further as here A. = constt. = 2 m so from v = fA,
F = (Am)a
fa = Va = 1000 =500 Hz
So substituting the values of F,.6.m and a from Eqns. (i), A. 2
Oi) and (iii) in the above equation,
i.e., in this situation,

T[ ":J = (mAL) x [v; ] '- = 2m, f = 500Hz and v=lOOOmls


Note: Here}.,. == constant; so f and v will change according to the
i.e., .... (iv) relation vex: fwith v a:..fi .

This is the required result and from this it is clear that the
Problem 12. A wire of mass 9.8xIO-3 kg per metre
velocity of a transverse wave propagating along a string
passes over a frictionless pulley fIXed on the top ofan inclined
depends only on the characteristics of the string (T and m) and frictionless plane which makes an angle of 30° with the
is independent of the frequency of the propagating wave. horizontal. Masses M 1 and M 2 are tied at the two ends ofthe
Now if A is the cross-section of the wire, m = pA wire. The mass Ml rests on the plane and the mass M 2 hangs
vertically downwards. The whole system is in equilibrium.
so v=MJm [as:=stresss]u(v) Now a transverse wave propagates along the wire with a
velocity of 100 mls.Find the value of masses M J and M 2 .
Furthermore it has also been established that if a (g=9.8mls')
transverse mechanical wave instead of propagating along a
Solution: For equilibrium of M I along and perpendicular
string, is propagating through solid media (e.g., S-seismic
waves in rocks) its velocity is given by to the plane we have respectively:
M l gsin9=T and M1gcos8 = R ....(i)
v = ~(~/p) u •• (vi)
And for equilibrium of
where 11 is modulus of rigidity and p density of the solid. M 2,
Problem 11. A wire of uniform cross-section is stretched M 2 g =T .... (ii) T
between two points I m apart. The wire is fIXed at one end and a Now as the velocity of
weight of9 kg is hung over a pulley at the other end produces a wave on a string is given
fundamental frequency of750 Hz. (a) What is the velocity of by L _ __ _ _ _- ' M,g
transverse waves propagating in the wire? (b) If now the
suspended weight is submerged in a liquid ofdensity (519) that v = ~, Fig. 12.17

of the weight, what will be the velocity and frequency of the


waves propagating along the wire? .
I.e., T =v 2m
Solution: (a) In case of fundamental vibrations of string and as here V = 100 mls and m =9.8 x 10- 3 kg/ill
("AJ 2) = L, i.e.,').. =2x 1 = 2m.
T = (lOO)2 x9.8x10- 3 = 98N .... (iii)
576

Substituting the value ofT f~om Eqn. (iii) in Eqos. Oi) & (i), Solution: (a) As the string has mass
M 2 = (TIg) = (98/9.8) = IO kg and it is suspended vertically, tension in it
will be different at different points. For a
M, = (TlgsinO) = 98/[9.8 x (1/2)] =20 kg
point at a distance x from thc free end,
Problem 13. A copper wire is held at the two ends by tension will be due to the weight of the
rigid supports. At 30"C, the wire is just taut, with negligible string below it. So if M is the mass of string L
tension. Find the speed a/transverse waves in this wire at IO°C if oflength L, the mass oflengthxofthe string
Y = 1.3 x lOll Nlm 2, a = 1.7 x IO - 5/ oe andp =9 x 10 3kg/m 3 . will be (MIL)x.
Solution: We know that in case of transverse wave
propagating along a string, .. T=[: x]g

v=~=,ffi So v- iT - / Mgx -,Jg;


v-;;; ~ L(MIL)
.... (i) Fig. 12.19

Here tension in the string is produced by thennal stress


Here x = 0.5 m
i. e., T =Y(Thstrain) = Ya..1.9
A
[asa= ilL ]
LM
so x ;;-9."8 - 2.2 I nus
v = .J""O.-CS"'
(b) From part (a) it is clear that the tension and so the
so V = ~ = ~ [as p = A:J velocity of the wave is different at different points. So if at
point x the wave travels a distance dx in time dt,
5
1.3 x 10" x1.7xlO- x (30 - 10) =70m/s v = dx or ,Jg; = dx [from Eqn. (i)]
V= dt dt
9x 10 3
Idt=1 dx, le., t= _I_ I"x-1I2dx
Problem 14. A uniform rope aflength 12m and mass 6kg
hangs vertically from a rigid support. A block of mass 2 kg is
or
,Jg; £0
attached 10 the free end of the rope. A transverse pulse of i.e., t =2~(Llg) ... (ii)
wavelength 0.06 m is produced at the lower end of the rope. Hence L = 2.45m so t=2~(2.45/9.8) - lsec
What is the wavelength of the pulse when it reaches the top of
the rope? § 12,3 Sound Waves
Solution: As the rope has a mass and a rAI Audible, Infrasonic and Ultrasonic Waves
mass is also suspended from the lower end, the Mechanical waves can be transmitted in all the three states
tension in the rope will be different at different of matter namely solids, liquids and gases. In liquids and gases
points. these are always longitudinal while in solids these can be
Now as

'L =
va
r
v = ~(T/ m)

TB
= f"(6~+-c2cc)g- = 2
V 2g
Purse
either transverse or longitudinal, e.g., mechanical waves on
strings are always transverse while seismic waves propagate
through earth in both modes, i.e., transverse, (S-waves) and
longitudinal (P-waves). According to their frequencies
longitudinal mechanical waves are divided into the
2 Kg
or frAT]=2 [asv=fA] following three categories:
[f. A• FIg. 12.18
(a) Audible or Sound Waves: These are longitudinal
Here fT = fa as frequency is the characteristic of the mechanical waves which lie within the range of sensitivity of
source producing the waves. human ear typically 20 Hz to 20 kHz. These are generated by
vibrating bodies such as tuning fork, vocal cords, stretched
So AT = '))..0 ",,2xO.06=0.12 m
strings or membranes.
Note: In this problcm f = consll., so v and Awill change according to (b) Infrasonic Waves: The longitudinal mechanical
the relation v 0: Awith v o:...fi. waves whose frequencies lie below 20 Hz are called
'lnfrasonics'. Infrasonic waves are created by earthquakes,
Problem 15. A uniform rope of mass 0.1 kg and length (P-waves), volcanic eruptions, ocean waves and elephants and
2.45 m hangs from a ceiling. (a) Find the speed of transverse whales. Pcriodic motion such as of a pendulum at frequency
wave in the rope at a pointO.5m distantfrom the lower end, (b) lesser than 20 Hz also produces these waves.
Calculate the time taken by a transverse wave to travel the full
(c) Ultrasonic Waves! Longitudinal mechanical waves
length of the rope. (g =9.8mls2 ) having frequencies greater than 20 kHz are called ultrasonic
VIBRATIONS AND WAVES 577

waves. Though the human ear cannot detect these waves, This is tum implies that:
certain creatures such as mosquito. fish, dog and bat· show (a) If we hear a distant momentary sound through a
response to these. These waves can be produced by the high metallic pipe we will hear it twice, first in metal
frequency vibrations of a quartz crystal under an alternating and then in air(asvmetal > vair)provioed the time
electric field (piezo-electric effect) or by the vibrations of a interval between two sounds is greater than
ferromagnetic rod under an alternating magnetic field (1/1O)s (persistence o{hearing).
(magnetostriclion-effect). Ultrasonics are used for navigation
(b) As for sound Vw > v A while fOf light Vw < VA
underwater (SONAR), in a~oustical rpicroscope~ giving much water is rarer than air for sound··· and denser for
better resolution than optical microsc;opes, in ultra-sonography light. This is why in travelling from air to water a
(i.e., in photographing or scanning soft tissues of body). in
beam of sound bends away from the normal while
bloodl~s s~ery and wave t~erapy. These are also used to
towards the nonnal (Fig. 12.20).
repel mosquitoes or attract fishes (ultrasonic buzzers). Ught
In order to avoid confusion and trouble, unless stated
otherwise, in this chapter '~nd hen~eforth the tenn sound AIr Rarer
wave·· is used for mechanical 'wav'es propagating in air with -----
frequency in the range 20 to 20 kHz. Furthermore, as velocity
of sound (and also of ultrasonics or infrasonics) inair at NTP is
=====
-=Water ~

332 mis, from V= fl.. we find that the wavelength of sound


waves lies in the range 1.66 cm to 16.6 m (while for ultrasonics (a) (b)
A. < 1.66 cm and for infrasonics A. > 16.6 m) , Fig. 12.20

(8] Velocity of Sound (2) In case of propagation of sound in solids· u • (rods)


When a sound wave travels through a medium such as air, elasticity involved is Young's modulus while for
liquids and gases bulk modulus, i. e.,
water or steel, it will set particles of medium into vibrations as
it passes through them. For this to happen the medium must
possess both inertia, i. e., mass density (so that KE may be
stored) and elasticity (so that PE is stored). These two
vSolid = 5 while v F1uid =~ .... (ii)
This is why
properties of matter determine the velocity of sound, i.e.,
velocity of sound is the characteristic of the medium in which 10
20 x l0 -S096m/s
waves propagate. Analytically it has been shown that velocity 7.7x 10)
of sound in a medium of elasticity E and densityp is given by
9
v=~(E/p) .... (i) 2.lxl0 ::: 1450m!s
and VWater =
10 3
Note: Mechanical waves travelling in solids as transverse waves are
called shear-wares and their velocity is given as u" J(TJ / p) (3) Newton's Formula: The fonnula for velocity of
where TJ is modulus of rigidity and p is density of solid. For sound in air was first proposed by Newton. He
transverse mechanical waves propagating along a string, assumed that when sound propagates through air
v=../(J'lm) temperature remains constt. (i.e., the process is
isothermal) so
From this expression it is clear that: B =E, =P
(1) As solids are most elastic while gases least, i.e.,
Vair =~(Plp)
ES >EL> EG
AtNTPforairP=I.OI x IDS N/m2 andp == 1.3 kg/m)
So velocity of sound is maximum in solids and
minimum in gases (as E» p), e.g., 5
so V '= 1.0l x l0 =279m1s
V steel > V water > vair air 1.3
~SOOOm/s ~ ISOOm/s ~330m/s

• Bats not only detect but also produce ultrasonics.


•• Sound being a mechanical wave, in gases and liquids it is always longitudinal but in solids it can be either longitudinal or transverse.
••• The concept of rarer and denser media for a wave is througb the velocity or propagation (and not density). lesser the velocity, denser is said 10 be tbe
medium and vice-rersa .
•••• Irthe solid is an unbounded medium, the velocity ofiongitudinal waves is given by
B + 1"\/3
v=--.
P
However, the experimental value of sound in air is i.e., velocity of sound in a gas is of the order of rms
332 mls which is higher than given by Newton's speed of gas molecules and lesser than it (v::::: v rms )'
fannula. (7) As velocity of sound in a gas according to Eqn. (iv) is
(4) Laplace Correction: In order to remove the
discrepancy between theoretical and experimental v=r::
. v~lues ' of
, , velocity of sound, Laplace modified velocity of sound in case of gases at constant
Newton's fannula assuming that propagation of
temperature depends on' the nature of gas, i.e., its
sourid in air is an adiabatic process, i.e., atomicity (r) and molecular weight. Ligh~er the gas
• I·,
B F E =y*P greater will be velocity, e: ~.,
v=~(tPlP)
_'j.

,Sothat
1; r;-c-;;; ! ,
.... {iii) v H= ' -
-
Yo
N'32
2
,,:",~ ["""
, ' " 5
7]
asYH=YO' =-
i.e., p =vlAl x279=331.3m1s [asYair =1.41]
or 'v
=4v I
~' j
which is in good agreement with the experimental H O
value (332 In!s). This in tum establishes that sound This is why velocity of soood ih hydrogen is greater (4
propagates adiabatically through gases. times that in oxyg,en gas).
(5) The 'velocity of sound in air at NTP is 332 mls which (8) Effect of temperature: In a gas
is much lesser than that of light and radio waves
(=3 x 10 8 mls). This implies that:
v=~Y~T
r.
(a) Ifwe set our watch by the sound ofa distant siren So for a given gas, j. e., rand M = constt.,
it wiU 'be slow. , v oc ~T(in K),
(b) If we record the time in a race by hearing the i.e., with rise in temperature velocity ofsound in a gas
sound from the starting point it will be lesser than increases. So:
actual.
(a) The velocity ofa given gas will become ntimes if
(c) In a cloud-lighting, though light and sound are temperature is changed from T to T' such that
produced simultaneously but as c> v, light
precedes thunder. nv =
v
fT',
fr i.e., T' = n 2T [temperature in K1
.
(6) As in case of gases

v=~=~:; =ff
(b) When change in temperature is smail,
Vi fT' 273 + t [I t ]112
Vo =Vr= 273+0= +273
v=~YPV [ asp =~mass
[ tOe]
i.e., ]
mass volume Le, v'=v o 1+ 546 =[332+0,6IUOC)]mls

or v = ~Y~T [asPV =~11 i.e., for small temperature variations at DoC, the
mass velocity of sound changes by 0.61mls when
or V= ~r RT [as)! = mass ] .... (iV)
temperature changes by IOC"''''.
M mol.wt.M (9) Effect of Relative Humidity: With increase in
And from kinetic theory of gases, humidity, density of air decreases; so in the light of

v= =~(3RT/M) v=~(yP/p)

So v: =H, i.e., v=[tf2 v~


we conclude that with rise in humidity velocity of
sound increases. This is why sound travels faster in
humid air (rainy season) than in dry air (summer) at
the same temperature. "'''''''

... r - C plCv and has values (5/3), (7/5) and (4/3) for mono, dia and polyatomic gases respectively.
...... If the initial temperature is not O°C, Vo .. 332 mls and so the result will change. [See solved Problem 19 (b).]
......... In case of humid helium and humid hydrogen, velocity of sound is less than in dry He, H2 as their densities increases with h'.unidity.
VIBRATIONS AND WAVES 579

(10) Effect of Pressure: As velocity of sound (b) Calculate the ratio of the speed of sound in neon to

v=J% =N =~Y::
that in water vapours at any temperature. (Molecular weight
of neon =2.02 x lO- 2 kg/mol and for water vapours
=1.8x 10-z kg/moT)
so pressure has no effect on velocity ofsound in a gas
as long as temperature remains constant. This is why Solution: The velocity of sound in air is given by
in going up in ,the atmosphere, though both pressure
and density decrease, velocity of sound remains
constant as long as temperature remains constant.
v=J! =
N=~Y~T
Furthermore it has also been established that all other (a) In terms of density and pressure,
factors such as amplitude, frequency, phase, loudness.
pitch, quality, etc., hi ve practically no effect on
vH = PH x p,;, =~p,;, [as Pnir = PH]
velocity ofsoiind. j.,.
Vair PH Pair PH '
' Velocity of sound in air is measured by resonance tube or
Hebb 's method while in gases by Quinke 's tube. Kundt '$ tube
is used to delennine velocity of sound in solid, and or gases.
or v H =V·
'"
xt';'
PH
=332 XN
6
1
=1328 mls
(b) In terms of temperature and molecular weight,
Problem 16. A plew-electric quartz plate of thickness
0.005 m is vibrating in resonant conditions. Calculate its
fundamental frequency if for quartz, Y = 8 x 10 10 Nlmz and
p=2.6Sxl0 J kg/mJ.
Solution: We know that for longitudinal waves in solids,

V=
5 - ,
P
so V=
8xl010
2.65x10
Furthennore, for fundamental mode of plate,
J
=S .5x10 3 mls

Problem 19. (a) Filld the speed ofsound in a mixture ofl


OJ2)=L, so A.=2x5xlO- J = 10-z m mole of helium and 2 mole of oxygen at 27"C. (b) If the
temperature is raised by I K to 300 K. find the percentage
Butas v=jA.. i.e. , f::::(vn..)
change in the speed of sound in the gaseous mixture.
so 1=[5.5xl0'110-'] =5.5xlO' Hz=550kHz (R =8.3 1JlmoIK).
Problem 17. Determine the change in volume of6 lUres of Solution: (a) In tenns oftemperature. the speed of sound
alcohol if the pressure is decreased from 200 cm of Hg to 75 is given by
cm. (Velocity ofsound in alcohol is 1280m/s, density ofalcohol
.... (i)
:::: 0.81 g/cc, density ofHg:::: 13.6g!cc and g = 9.81 m/i )
Solution: For propagation of sound in liquid Now here as I mole of helium is mixed with 2 mole of
v=J(B /p), i.e, B =v'p oxygen,

But by definition B = -V(APIIlV) M . =n l M I +n ZM 2 =lx4+2 x 32


nux nl+n Z 1+2
So - V ~ =v'p, I.e., 6V = V~":)
= 68 x IO-J kwmol .... (ii)
3
Here I1P =h,pg - h,pg =(75 -200) x 13.6 x 981 Further as helium is monatomic [C y = (312)R} while
= - 1.667 x 10 6 dyne/em Z oxygen is diatomic (C y = (S/2)R), so for mixture
So 6V =(6 x 10' )(1.667 x 10 6 ) =0.75 CC nlC y + nzCV,
0.81 x (1.280 x 10')'
(C ) . =
Yll'lx , ,
I
lI+n
,

Problem 18. (a) Speed ofsound ;n air is 332 m1s at NTP. =I x (3 /2)R +2 x (5/2)R =UR
What will be the speed of sound in hydrogen at NTP if the 1+2 6
density of hydrogen at NTP is (1116) that of air?
13 19
(Cp)mix =(Cy )mix +R =r;R + R =r; R [asC p -Cy =R]
[A ssumePair /PH =I.} (MNR 19951
C p 19 6 19 Note: For a hannonic-progressive wave as from § '12.2 (B) (7) .
(y)"" =C=6"R x 13R =13 .... (iii)
v dy =_VPav'
dx
p =_Bdy = B("-"']
dx v
So substituting the values of M and y from Eqns. (ii) and
(iii) in (i) with T = 300 K and R = 8.31 Jlmol K, we get i.e, pressure in a sound wave is equal to the product of elasticity
of gas with the ratio of particle speed to wave speed.
v=
19
- 8.3lx300
x ---'-"-''-''-'',
13(68/3 x to 3) But from Eqn. (i) t = -Akcos (oot - kx) ., " .

= 400,9 mls So p=AkBcos(wt-kx)


(b) As v=~(yRTIM), for. given gas, i.e., P=POcos(oot-kx)
~v=lLlT =.!.x-1-xlOO=O.167% with Po = ABk ....(iv)
v 2 T 2 300 . From Eqns. (i) and (iv) it is clear that; ·
Note: So change in velocity of sound by 10 change in temperature at (I) A sound wa;t.~ may ~be fr considered as .either a
27°C =300 K is =400.9/600= 0.668 mls and not 0.61 mls at O°C displacement wave y,= Asin (wt - kx) or a pressure
as in § 12.3 (B) 8 (b). wave p '" Po c~s (oot - kx).
" , (2) The pressure wave is 900 out of phase with respect to
Ic) Displacement ofPr~ssure Waves displacement wave, i.e., displacem~t will be
A sound wave (i.e., longitudinal mechanical wave) can be maximum when pressure is minimum and vic~-v,ersa.
described either in terms of the longitudinal displacement This is shown in Fig. 12.22.
suffered by the particles of the medium (called
displacement-wave) or in terms of the excess pressure
generated due to compression and rarefaction (called
pressure-wave).
Consider a sound wave travelling in the x-direction in a
medium. Suppose at time I, the particle at the · undisturbed
position x suffers a displacement y in the x-direction. The
displacement wave then will be described by
-lex) .... (i) Fig 12.22
(3) The amplitude of pressure wave,
Po =' ABk = AkfYV2 '[asv=~Blp]
or Po =pvAoo [asv=wlk] .... (v)
X X+II.X
(4) As sound-sensors (e.g., ear or mike) detect pressure
Fig. 12.21
changes, description of sound as pressure-wave is
Now consider the element of medium which is confined preferred over displacement wave.
within x and x +!:J.x in the undisturbed state. If S is the [D) Energy, Power and Intensity of Sound
cross-section, the volume element in undisturbed state will be Ifa sound wave given by y= A sin (oot - kx)is propagating
V '" S !:J.x. As the wave passes, the ends at x and x +!:J.x are through a medium, the particle velocity will be
displaced by amounts yand y +.6.y so that increase in volume
of the element will be .6. V = S .6.y. This in tum implies that vPa = d: = Awcos (wt - kx)
volume strain for the element under consideration,
So if p is the density of the medium, kinetic energy of the
il.V S il.y il.y
V = S il.x = il.x .... (ii) wave per unit volume will be

So corresponding stress, i.e., excess pressure = ,!,p[d


2 dt
Y J2 =.!.p002 A2 cos 2 (oot - kx)
2
P=B[-il.V] [ asB= - Vil.P =-v L ] and its maximum value will be equal to energy perunit volume
V il.V il.V
[as (KE)max = (PE) max = E], i.e., energy density U. So
t>y
0' p=-B -
il.x
[from Eqn, (ii)] .... (iii)
U =t pA2 ro 2 .... (i)
VIBRATIONS AND WAVES 581

So the energy associated with a volume S ax will be I


or SLI -SL, =1010g ...l .... (ix)
'" 1 ,ro
~=Ut.V=2pA 's tu .... (ii) I,
e.g., two sounds whose intensity ratio is 2 differ inSL by 10 log
So, power (rate of transmission of energy) will be 2 = 3 dB. Here it must be kept in mind that' ratio ·Of two

p =~ =~pv(l)2A2S [as~ =v].... (iii) intensities corresponds to difference in their sound level (and
not ratio).
Now as Intensity· is defined as average energy Note: While solving problems related to intensity of sound along with
transmitted per unit nonnal area per sec., i. e., power per unit the above, also remember that:
area, so
/lEP)
l = - =- =-pvro A
22
.... (iv)
(i) As intensity, I ", ::1 whi leU '" ~,
SM S 2 So !... '" llE xSllL",v [asll.V =St.LandM= v]
U SlI.l l!.E l:JI
Further as in case of sound wave displacement amplitude
or intensity I (energy flux) =U x v
is related to pressure amplitude through the relation
=Energy density xvelocity .... (x)
Po =pvA~ so
(ii) With increase in distance from the source the tota l energy or
1 =!pvco2[ Po ]2 =! p~ ....(v)
power transmitted remains the same but intensity decreases.
For an isotropic point source o f power P, intensity I at a
2 pvro 2 pv
distance r from it will be /" - - ---~~
/' ',-
Eqos. (iv) and (v) give intensity of sound in tenDS of
displacement and pressure amplitude respectively and
I=f= 4~ (as S=4nr2] .... (xi):' P r \
: 'JI
according to these for a given source and medium Nowas fora given medium and source, \ Source ,I
2 '- /
locA 2 (or p~) .... (vi) I ex: A .... (xii) ',, __ ____ /
So from Eqns. (xi) and (xii), Fig. 12.23
Note: In ¢8se of vibrating string, as pS will represent mass per uni t Al oc (ll? ),i.e, for spherical waves
length m, so from Eqn. (iii) the average rale of transpon of I ex: (II?) and A «(lIr)
energy. i.e , power transmitted by a vibrating string will be
(iii) In case of electromagnetic waves (e.g. ,light or radio waves),
P =.!mvrol A2 m = mass
2
with
length
... (vii)
I '" ~ (E8)
flo
with §.. ""
B
c and c,.-r.L:-
. ,rflo&o
The 81 unit of intensity is W1m 2 . However, as human ear
Problem 20. The faintest soulld the human ear can detect
responds to sound intensities over a wide range, i, e., from
at afrequency of l kHz (for which the ear is most sensitive)
10- 12 W/m 2 to 1 W/m 2 • so instead of Specifying intensity of
corresponds to an intensity ofabout 10- 12 Wlm 2(the so called
sound in W /m 2 , we use a logarithmic scale of intensity called
threshold of hearing). Determine the pressure.. amplitude and
the sound level defined as
maximum displacement associated with this sound assuming
SL =10 log [:.1 .. (viii) the density of air = 1.3 kglm 3 and 'IIe/ocity of sou.nd in air
=332ml s.
where 10 is the threshold of human ear, i.e.,IO- 12 W/m 2 . The Solution: As Po =;jlx2pu
,---;,;;-- -- -- \ .. , \'
sound level defined in this way is expressed in decibel (dB). A So PO =~(l0 12» (2)( 1.3 )(332 =2.94 )( 10- 5 N/m f
sound of intensity 10 has an SL = I 0 log (10 1 10 ) = 0 dB while
sound at the upper range of human hearing called threshold of Now as Po = pvooA
."". ".-
pain has a intensity of 1 W1m 2 or a SL = 10 logOIl 0- 12 ) = 120 Po 2.94)( 10- 5 . 0-11
So A=- - = = 1.1 )( 1 m
dB. pvro 1.3 x 332 x (21t )( 10 3 )
We also use dB as a relative measure to compare different From Ihis it is clear that the ear is a extremely sensitive
sounds with one another, rather than with reference intensity; detector of sound waves as it can respond to amplitudes about
as for two intensities I I and 1 2 , (I / 1O)th the size of an atom! .
II 12
SLI-SL, =lOlog--lOlog- Note: See also unsolved Problem 33 for upper limit.
10 10

• Some books and authors use lhe words ' energy flux ' or power dens ity for intensity.
Problem 21. What is the maximum possible sound level in Problem:4. A dog while barking delivers about I m W oj
dB ofsound waver in air? Given that density ofair =1.3 kglm 1, power. If this power is uniformly distributed over a
v =332 m/s and atmospheric pressure P = 1.01 x lOs N 1m2. hemispherical area, what is the sound level at a distana of 5
m? What would the sound level be if instead ofl dog. 5 dogs
Solution: For maximum possible sound intensity, start barking at the same time each delivering I mWofpower?
pressure amplitude of wave will be equal to atmospheric
Solution: As power is distributed uniformly in a
pressure, i. e.,
hemisphere, intensity at a distance of 5 m from the source will
Po "" p=l.Olx lOs N/m2 be
, ,,
1 .!'.9.-=(1.0IxI0) =l.lSxlO7W/m' 1= P = P
3
----'1"-0-_... 637 WI '
So S (112)41[r 2 = 2x 1[x52 ==. j.l m
2pv 2 x1.3x332

SL = IOlogL =IOIOg~ =190dB so SL = 10 log L = 10 log 6.37 x 10"


10 10- 12 10 (10''')
Problem 22. (0) The power ofsound/rom the speaker ofa I. e, SL = I O(log 6.37 +610g 10] = IO[O.SO +6]
radio ;s 20 m W. By turning the knob of volume control the i. e., SL =68dB
power of sound is increased to 400 mW. What is the power Ifthere are 5 dogs barking at the same time and same level,
increase in dB as compared to original power? 12 =51 1,
(b) How much more intense is an 80 dB sound than a 20 I 51
dB whisper? So SI-; -SL, = lOlog-1. =lOlog - '
I, II'
Solution: (a) As intensity is power per uni.t area, for a
given source P oc I, so i. e., SI-; = SL, + 10 log 5
SI-; - SL-, = 10 log (/,/1,) or SI-; =6S+lO x O,7 = 75 dB
Problem 25. An observer is at a distance of one metre
i, e., ilSL = 10 log P, = 10 log 400 from a point /ight source whose power output is 1 leW.
P, 20
Calculate the magnitude of electric and magnetic fieldJ
assuming that the source is monochromatic, it radiateJ
i,e., ';SL = I O[log 20]= 13 dB
uniformly in all directions and that at the point ofobservation
(b) By definition of sound level, it behaves like a travelling p{ane wave. Given that (j.l o/41t)
SL, -SL, =lOlog(/,II,) =10 - 7 Hlmandc:::::3x10 8 mls. (MNR199S]
So SO - 20 =10 10g(/,/I,) Solution: By definition of intensity,
or 6= 10g(I,II,), 3 3
I=P =~ 10 =10 W/m'
i,e., (/,11, )=10' S 41t1'2 41t x 12 41t
Problem 23. A window whose area is 2m 2 opens on a Now in case of electromagnetic waves, as
street where the street noise result in an intensity level at the I= - I EB and E
-=C
window of 60 dB. How much 'acoustic power' enters the ~o B
window via sound waves? Now ifan acoustic absorber is fitted
at the window, how much energy from street will it collect in so I= _I_ ExE, i.e., E=~IlJ.oc
~o c
five hours?
Solution: By definition SL == 1010g (1110 )

So 101~gL=60 or L=10 6 and B=E = 100.fj =_1 x 10" =5.57xlO,7 Web/m '
10 10 c 3xlO' .fj
i.e., 1 == (10- 12 x 10 6 ) = 10-6 W/m 2= 1 j.lW/m 2 7
Note: (i) As B .. ~, H .. 5.77xI0-7 =0,46 AIm
So P = IS ::.: lxlO- 6 x2=2j.lW 4nx10-
(ii) The peak values of fields will be
and E::::: P X t =2x 10-6 x 5 x 60x60 = 36x 10- 3 J
Eo s: (..fi)E and BO:= (..fi)B [as E .. Eo/./2)
VIBRATIONS AND WAVES 583

Problem 26. A wave pulse starts propagating in (b) Pitch: It is the sensation received by the ear due to
+x-direction along a non-uniform wire of length 10m with frequency and is the characteristic which distinguishe~ a shrill
mass per unit length given m = rno + ax and under a tension of (or sharp) sound from a grave (or flat) sound. As pitch depends
100 N. Find the time taken by the pulse to travel from the on frequency, higher the frequency higher will be the pitch and
lighter end (x =0) to the heavier end. (rno = 10- 2 kgrmand shriller will be the sound. Regarding pitch it is worth noting
0. =9 x 10-' kglm') that:
Soludon: Velocity of transverse wave in a string, (I) The buzzing ora bee or humming of a mosquito has
high pitch but low loudness while the roar of a lion

V=~=~=~mo:ax has large loudness but low pitch.


(2) Due to more hannonics usually the pitch of female
~rm-o-+-ox
- dx =..fi d, voice is higher than male.
(3) In a record player if a record of45 RPM is played at 78
Integrating within proper limits, RPM, pitch will increase and sound will become
f~ ~mo +oxdx =..fi f~d' shriller. tftbe same record is played at 33 RPM, pitch

[2(m o ;o.OX)'12 I =..fi, Note:


will decrease and so sound will become grave.

In a record player, volume control controls the loudness.

(4) In a tape·recorder or TV, bass and trcble refer to low


0' ~ (mo +o.L)'12 -mi12 J=..fi, and high pitch respectively. So at bass (or woofer on),
30. low pitch, i.e., grave sounds such as of ' tabla' or
'dholak' become loud while at treble, high pitch, i. e.,
, =_2_(mo +aL)312 -mJI2]
30...fi shrill sounds such as ofjlute or 'ghoonghroo' become
predominant.
2
(5) Due to Doppler effect frequency and hence pitch of
3x9xlO- J xlO sound appears to be different from the actual onc.
((10-' +9x 10-' x 10)31' _(10- ')'12 J (c) Quality (or Timbre) :
It is the sensation received by
= 2 ((10-' +9x lO-') 3I' - IO -' J
27 x 10- 2 the ear due to 'waveform '. Two

=2.335 sec Ans.


sounds of same inten~ity and
frequency as shown in Fig.
,
[EI Characteristics of Sound 12.24 will produce different
• :T
Sound is characterised by the following three parameters:
(I) Loudness: It is the sensation received by the ear due to sensation on the ear if their
wavefonns are different. Now .
Y~--lL+-"""-+'C;-_.L.,.
j'
intensity a/sound. Experimentally Weber-Fechner established as wavefonn depends on a 1 •t
that- overtones present, quality of 1_ _-' T
LocK logl sound depends on number of
Fig. 12.24
i.e., greater the amplitude of vibration, greater will be the overtones, i. e., hannonics
intensity I (cc A 2) and so louder wilt be the sound as in a shout present and their relative intensities. The dependence of
quality on phase is controversial. Regarding quality it is worth
and lesser the intensity, feeble will be the sound as in
whispering. noting that:
(I) We can recognise a person (without ' s~eing) by
The loudness being the sensation, depends upon the
listening to his sound as it has a definite quality:
sensitivity of the li.stener's ear. Therefore, loudness ofa sound !I '
of given intensity may be different for different listeners. (2) If same note is played on different instruments say
Similarly ~o sounds of equal intensity but different frequency sitar and veena at same loudness it produces different
may not appear to be equally loud even to the same listener sensation on the ear due to their different quality.
because the sensitivity of the ear is different for different (3) Sometimes it becomes difficult to recognise a' person
frequencies. by listening to his sound on telephone or'tape due to
The unit of loudness is phon which is equal to the intensity poor quality of sound.
level in dB of equally loud sound ofl kHz (for which tbe ear is
most sensitive].
EXERCISE

[AJ O:nly Og.~ Choice is Correct (c) Temperature is too low during night and high during
day
1, BeUs are m~e. of metal and not of wood because:
(d) There are too many craters on the surface of the moon
(a) The thepnaJ cond\.\ctivity of metal is greater than that
of wood 9. Wbich of the following statements is correct? (CPMT 19931
(b) The, density of the metal is greater than th~t of wood (a) Both sound and light waves in air are longitudinal
(e) The sound.is not cond~cted by metals but is radiated (b) Both sound and light waves in air are transverse
Cd) Wood dampens vibrations while metals are elastic (c) Sound waves in air are transverse while light
longitudinal
2. The vibrations taking place in the diaphragm of a
microphone will be : (d) Sound waves in air are longitudinal while light
transverse
(a) Free vibrations
10. It is possible to distinguish between transverse and
(b) Damped vibrations
longitudinal waves by studying the property of:
(e) Forced vibrations
(a) Interference (b) Diffraction
Cd) Electrically maintained vibrations
(c) Reflection (d) Polarisation
3. In the case of sustained forced oscillations the amplitude of
11. Which of the following phenomena cannot take place with
oscillations: (CPMT 1990J
sound waves in air?
(a) Decreases linearly
(a) Refraction (b) Interference
(b) Decreases sinusoidally
(c) Diffraction (d) Polarisation
(c) Decreases exponentiall.y
12. Sound waves in air cannot be polarised because:
Cd) Always remains constant
[CPMT19931
4. Two sources of sound are in resonance when:
(a) Their speed is small
(a) They, look alike
(b) They require medium
(b) They are situated at a particular distance from each
other (c) These are longitudinal
(c) They produce the sound of same frequency (d) Their speed is temperature dependent
(d) They are excited by the same exciting device 13. Mechanical waves (sound waves) in a gas is :
5. When a tuning fork is vibrated, another in the (a) Transverse
neighbourhood begins to vibrate. This is due to the (b) Longitudinal
phenomenon of: (e) Neither _transvet:Se nor longitudinal
(a) Gravitation (b) Newton's III law (d) Either tJ'a?sverse or longitudinal
(c) Resonance (d) Consonance 14. Elastic waves in solid are: , '
6. Which of the following cannot travel through vacuum? (a) Transverse
(a) Light waves (b) Heat waves (b) Longitudinal
(c) X-rays (d) Sound waves (e) Either transverse or longitudinal ..
7. A big eXRlosion on the moon cannot be heard on the earth (d) Neither transverse nor longitudinal
because:
15. Transverse elastic waves can propagate:
(a) The explosion produces high frequency sound waves
(a) Both in a gas and a metal
which are inaudible
(b) In a gas but not in a metal
(b). Sound waves reql.!-ire a material medium for
propagation (e) In a metal but not in a gas
,
(c) Sound waves are absorbeq in the atmosphere of moon (d) Neither in a gas nor in a metal
(d) Sound waves are absorbed,i1J. earth's atmosphere 16. Transverse sound waves cannot.travel in :
8. An astronaut cannot hear his companion. at the surface of (a) An iron rod
the moon b~9ause : (b) Hydrogen gas
(a) Produced frequencies are above the audio frequencies (e) A stretched nylon string
(b) There is no medium for sound propagation (d) Lubricating oil
17. Mechanical waves on the surface of a liquid afC : 28. Three progressive waves A,B and C are shown in Fig.
(a) Transverse 12.25. With respect to wave A:
(b) Longitudinal l!.__ A C
(e) Torsional
(d) Both transverse and longitUdinal (or ripples)
18. Which of the following properties of a wave does not
change with a change in medium?
(a) Frequency (b) Wavelength
(e) Velocity (d) Amplitude FIg•• 2.25
19. Of the following properties of a wave, the one that is (a) The wave C lags behind in phase by x / 2 and B leads by
independent of the other is its: . /2
(a) Amplitude (b) Velocity (b) The wave Cleads in phase by xandB lags behindlby x
(e) Wavelength (d) Frequency (c) The wave C leads in phase byx/ 2andB!lags behind by
20. The relation between frequency n, wavelength A and . /2
velocity of propagation v of a wave is: (d) The waveClags behind in phase by Kand B,leads by 1t
(a) n=VA (b) n= ')./v 29. Which one of the following does not represent a travelling
(c) n-vlA (d) n-1/v). wave?
2 1. The relation between frequency f. wavelength A and (a) y=Y./(X-VI) (b) Y=Y m sink(x+vt)
velocity o~ propagation v of a wave is : (c) y= Ym log (x-vt) ·(dj y -/(x' _VI' )
(a) v).-/ (b) V l v-I
(HI.t: A travelling wave is of the fonn F(ax± btl)
(c) v//).-I (d) / I v+)./v-I
30. Which of the following , expressions is thaLof a simp1,e
22. A boat at anchor is rocked by waves whose crests are 100m hannonic progreSsiye wave?
apart and whose velocity is 2S mlsec. These waves strike
the boat once every: (a) Asincot (b) A sin rot eos kt
(a) 2500 sec (b) 0.25 sec (c) A.n(Ol/-a) (d) Acosk(x)
(c) 1500 sec (d) 4 sec a
[HlnC : A hannonie progressive wave is sin or cos function of
(~± bI))
23. The wavelength of sound in air is 10 cm. Its frequency is:
31. The equation of a transverse wave is given by Y = 10 sin
(a) 330 cycle per sec (b) 3.3 kilocycle per sec x(OOlx-21) where yand x are. in cm, and! 1 in ,sCc. Its
(c) 30 megacyclc per sec (d) 3xl09 cycle per sec frequency is :
24. The wavelength of sound from,a tuning fork oil frequency (a) 10 Hz (b), 2I;h,,,., . ,,'" '."
330 Hz is nearly: .,. (c) I Hz , ,(d) O.OtHz
(a) 100 em (b) 10 em, 32. The equation of a transyene wavc .is1givcn by y .. 20 sin
(c) ~ cm (d) 330em n(002x-21) where yand X arc, in etn!and '1, in~scc. The
2S. A . ~ource
of frequency 500 Hz emits wav~s of w!\velength .,.
wavelength in".em will be: IMNR
. 1990)
0.2 m. How long does it take the w.ave to travel}09.m? (a) 50 (b) .100 ,(c) ,290 , (d) ~ I
(a) 70 sec (b) 60 sec 33. A simple h~o~ic. waye, ~avingr ~pli~d~, 4 . ~~ time
(c) 12 se~ (d) 3 sec period T is represented'by the equation y = Ssm,. (I + '4 )m.
The values of A (in'm)andT(ins) are: ., (MNR1991)
26. Two sound waves are respectively;
(a) A -5:T-2 (b) . A -IO:T b l' '
y =asin (oot -kx)apd y =b cos (6)t -kx)
(c) A -5:T-1 (d) A -10:T-2
The phase difference between the two waves is:
34. A transverse progressive wave is given by t1I:e equation
00 ./2 (b) ~/4 (x and Y being in cm); .
(c) • (d) 3./4 y = 2cos x(Q5x-200t,V
27. The relation between phase difference and path .difference Which (If the following statements-is true? '
;" IMNR 19951
(a) A.=2emand v =400 cmls
(a) &~_2·lIx (b) M-2W1x (b) ). ... 2emandA-2em
).
(e) J..=4cmand!=lOOHz
(d) &~--lIx •
). (d) A-2cmand/-200Hz
~5. The velocity of sound in air is nearly: 45. The velocities' of sound in an ideal gas at temperatures Tj K
(a) 332 mI, (b) 332 milel, andT2 Karev j andv 2 respectively. If the root mean square
velocities of the same gas at same temperatures are c1 and
(c) 332 ftI, (d) 3.32 kmI,
c2 • then:
36. Ultrasonic, infrasonic and aUdibJe. waves travel through a
medium with sp.,eeds v u' 11 I and v a respectively; then: (a) c2 =ct (v 2 !v 1 ) (b) c2 =ct (.,JV;TV;)
.j ' ,'
(a) Vi =V a =V ri : (b) Vu >Va >V/
(e) Vu <va <V/\ (d) va :SV u =V j
46. If Co and c denote the sound velocity and root mean square
37. In which medium has sound maximum velocity? velocity of molecules of a gas:
(a) In gases (b) In liquids (a) Co >c (b) Co =c
(ch In solids (d) In vacuum (c) Co =c(y/3)1I2 (d) , Co and care not related
38. Of the material mentioned below the speed of sound is
47. The velocity of sound in a gas is 300 mls. The root mean
,Iargestin: ""'J';"
squart.l velocity of the molecules is ofthe or~er of:
., .(a) . Water, ·', (b) Steel
(a) 4000 mI, (b) 40 mig
(e) Vacuum Cd) Air
{c) 4 mls (d) 400 mls
39:,' Transverse wav'es are generated in uniform steel wires A
".! andrB by atta,ching their free 'ends 'to a vibrating source of 48. Velocity of sound is ,measured in hydrogen and oxygen
frequency 500 Hz. The diameter of wire A is half that of gases at a given tempera~e. The ratio of the two velocities
-wire B,and the :tension ,o fwire A is half that of the wire B. will be :
The velocities of waves in wires A and B are in the ratio: (a) 1: 4 (b) 4: I
It, .' I \ (r.' 1 .,
(a) I: 2 (b) 2: I . (c) I: I (d) 32: i
(c) I tT2 " (d) .J2 ! I ." 49. The velocity of sound in a gas is:
40: TIle velociiyof souna in any;~a§'
,
dep~nds upon~
'/'
,i (a) Indep;lJ.dell! oftemperature on absolute scale
(b) Proportional to square root oftemperature
• ' • -'" . il , ;.[CBSE 1992) '

(a) Intensity (b) Amplitude (c) Inversely proportional to square root of temperature
I ,',
(c) Density and elasticity (d) Volume and temperature (d) Proportional to' square of temperature
, ., " ." .f I .
41. Velocity of sound in a gas is proportional to: 50. The temperature at which speed of sound in air becomes
double of its value at 27°Cr: {MNR 1990]
n (a) Square r9,ot,of isothxrmal elasticity J,
I

(a) 54'C (b) 327'C


-(b) : Adiabatic'elasticity IJ··' ! '
(c) 927'C (d) -123'C
(c) Square root of adiabatic elasticity
51. ~he speed of sound in air is : (CRMT 1993)
(d) Isothermal e'l~stici_ty
(a) ocpressure of air (b) ocsquare 'ofpressure
42. If the volume elasticity of fresh water and sea water are
:.- \I assumed to be the same, it is necessary that for the velocity (c) oc~presstire . (d) independent of pressure
Ofsoilnd lobe the same: [;<1: I. [' I ' ! I , "
52. The speed of'soUnd in a gas at NTP is 300 mlsec. If the
tih; I f ll: 'I};, .: , ,/.]
.- (a) Fresh water must be at a higher temperature . pressure ' is increased foUr times, without change in
" J" Il' "1 , I" Irl, tteinpetature, the velocity of sound will be: '
(b) Sea water must be at a higher temperature
of:
.' (cr
l •
Botlih&U~t rbe at f~~~ t~in 'er 'fute'c ':,: ,,!.t ,· (' /'
'! ,itt" . ,n.)' ,).', ,' ,., Il,.iii,:;,,:,,· , ); \ ,',
f" (a) 150 mI, ".10 (b) 300 mI,
il '1!(41 ..li~ish water lJ\q~\ ~ve high.~rl refractive index . (c) 600 mI, (d) 1200 mI, .
43. The Lap'lace'~ . corr~ction in the expre~sion for the velocity 53. The velocity of sound in air is'not'affected by'chang~s in:
of sound gi~eJ?,by Ne~,n is needed b~cause.sound waves: (a) Moisture contehts of'ihe ai~
". A~~ ~Nl~9-g\~9t~al 'I ,~, •. ·'tl)" 'r (b) The temperatUre' of the air
(b) Propagate isothermally 1 I'" "".': (c) The atmospheric pressure
(c) Propagate,adiabatically. (d) The composition of air
(d) Are oHong wavelengths I'. " '! • )./ ]; . d 54." Wh'ich of
, • (J I ,
the' following state'ments is wrong?
44. The speed of a longitudinal wave in a gas is given by : (a) Changes in air temperature have no effect on the speed
(a) v=.jp/d (b) v=(II.y).jpld of sound
(b) Changes in air pressure have no effect on the speed of
(c) v=.jyp/d '(d) v ·=.jp/yd
sound
(e) The speed of sound in water is higher than in air 64. Waves of displacement amplitude a and frequency ootravel
(d) The speed of light in water is lesser than in air in air with the same velocity. Which of the following waves
55. Ifat a place the speed ofasound wave of frequency 300 Hz has the highest intensity?
is V, the speed ofanotber wave of frequency 150 Hz at the (a) a =lOx lO-4 m,oo=SOOradls .
same place will be: [CPMT 1993) (b) a.,. 2x lO-4 m,ro =2000radls ,!
(a) V ' (b) V/2
,(c) a = 20xlO~ m,ro=200radls Ii
(c) 2V (d) 4V
Cd) a = 20x 10-4 m, 00= llS radls
56. The loudness of sound depends upon:
65, Decibel is:
(a) Amplitude (b) Pilch
(a) A musical instrument
(e) Velocity (d) Wavelength ' .'
(b) A measurement of sound level
57. The bells ofa college or temple are made oflarge size. It is I, ..

for:
(c) A musical note "
, r, .," "
:1'
(d) The wavelength of noise
(a) Producing sound of high -pitch
66. The intensity level of sound A is 2 be} greater than that ofB.
(b) Producing loud sound
How many times more intense is the sound A than B?
(e) Producing sound of high quality
(a) 10 ' (b) 100
(d) Show
, (c) 1060 (d) 2
58. The intensity of a sound wave gets reduced by 20% on
67. The power of a sound from the sp,e*.~'R f a rad\o, is 20
passing through a slab. The reduction in intensity on
passage through two such consecutive slabs is: milliwatt. By turning the knob of t~~ -v;?~~me co~trol the
power of the sound is increased to 400 milliwatt. The power
(a) 4()OIo (b) 36% 'increase in decibel as compared to the original power is:
(e) 30% (d) 50% (a) 13 dB (b) 10 dB ' ,.,
59. The intensity ofa harmonic W8ve :
(a) Depends on its frequency and not 'amplitUde
[CPMT 1993)
Cc) 20 dB (d) B" :sqQ,fl ':"::\'1'
68, The shrillness ,(pitch) ofa sound note depends
, upon:
,
(b) Depends on its amplitUde and not frequency .. (AFMCI994)
(c) Depends upon both its frequency and amplitude (a) Amplitude (b) Frequency
(d) Depends neither on frequency nor on its amplitude (c) Wavelength (d) Velocity , /'.,
60. If the intensity ratio of two waves is 4 : I, the ratio of their 69. Ifin a gramophone a music record is made to twn faster. the:
amplitude is: [CBSE 1993) . ".' I' , ".
(a) Intensity increases (b) Pitch increases
(a) 2: 1 (b) 1: 2 I',. "
(c) Timbre changes (d) Pitch decreases
(c) '4 : 1 (d) 1: 4 ,i, ;". ;J" .... ,;". " ... " "'''\ ,.,;/,r,1 ,/,J!... ,~, _... I,U I
70, The voi~e, of .~ lion IS ~ifferent fro~ th~t(,of a mosquito
61. If the prC!~ureamplitude in a sound wave is tripled then the becaus~ : 'J, t:
intensity of the sound is increased by a fa~.~or:
, (a) The two animals have different size
(C~MT 1992) ., . '
(b) The two vo~ces trarel with different velC?Ci~es, ,
(a) 3 (b) 6
(c) Thesounds, have r~ifferentpitch fIl')ll~ / ' ,
(c) 9 (d)
,
.Ji (d) Th~ ~oun~s .~*ve ~i,£f~ren,t !l~~ 'J ,'O,~,:., r. ,'"1;-
62. A .is singing a note and at the ,same time.II is singing a note 71, The quality of a tone:
with
,.
exactly ont;-eighth'I,the, frequency of the note of A. The
energies of the two sounds are equal. The amplitude of the (a) Decreases with loudness
note orB is: (b) ,Vanes directly as pitch .J ,,·to .,

(a) Same as that of A. (b) Twice that of A (c) Depends on the overtones that are present there
(c) Four times that of A Cd) Eight times that of A (d) Varies inversely as the frequency , '..., "

63. The amplitude of sound is doubled and the frC!quency is 72, It is possible to ,recognize a person by hearing his voice
reduced to one-fourth. The intensity of sound at the same even ifhe is hidden behind a sQlid walt Thisds due to the
point will be: ICBSE 19921 fact that his voice:
,I' , I", ",
(a) Increased by a factor of2 (a) Has a definite pilch
(b) Increased by a factor of 4 1\"1~, (b) Ha~j ll d,yfinitc;, fluality
(c) Decreased by a factor of2 (c) Has a definite loudness
(d) Decreased by a factor of 4 (d) Can penetrate the wall
73. The same note being played on sitar and veena differs in :
(a) Pitch
(a) Hz <h)H;
(b) Quality
(e) .J42 (d) 51
(e) Both pitch and quality 5 5
, ,
(d) Neither pitch nOf quality 82. Three progressive waves A,B and C are shown in the Fig.
74. In an orchestra, the musical sounds of different instruments 12.26.
are distinguished from one another by which of the B A C
following characteristics? (CBSE 1993)
(a) Pitch (b) Loudness
(e) Quality (d) Overtones
75. The speed of sound waves having a frequency of 256 Hz
compared with the speed of sound wave that has frequency
512Hz is:
Fig. 12.26
(a) Hatfas great ~) The same
(e) Twice as great Cd) Four times as great With respect to A, the progressive w,ave, :
76. Which of the following equation represents hannonic [HaryaDa PMT 2007)
wave? [eDSE 1994) •
(a) B lags by "2 and C leads by "2 •
(aj' y=Acos(rot-kx) . (b) y=Asinro/
J"
(e): y=Apos~
I I
Cd) y=Asin(at-bx"'!"c) (b) B lags by 'It and C leads by 1t
77. , The intensity of; a plane progressive wave of frequency 1 •
(c) B leads by - and C Jags by -
2 2

kHz is 10- 10 watt/m 2 .,Ifthe density of air is 1.3 kg/m 3 and
the speed ofl so~d itS equal to 330 mis, then pressure (4) B leads by n:and C Ijlgs ,by 1t
amplitude of the wave is ; 83. Oxygen is 16 times heavier than hydrogen. Equal volumes
(a) 3x 10-5 N/m2 (b) 3x 10-4 N/m2 of hydrogen and oxygen are mixed. Ratio of speed of sound
(d) 3x 10-3 N/m2
to
in the mixture that in hydrog~n is ;
'(c) '3x 10--6, N/m2
78. The variation of the speed of sound with temperature is
greatest in : '
(3),[8 (b) If
. (a) 'metals (b) gases (ej . (2 (d) . {ll
(c) solids (d) ' liquids '117 '117
79. A string in a musical ins!J'Ument i~ SO em long and its 84. Sound wave of wavelength 60 em in a medium is refracted
fundamental tTCquency is ' 800 Hi. 'if a frequency of 1000 into air. If spe~d ~f so'und in the medium is 5000 nvsec,
Hz is to be produced, then required length of string is : wavelength of the wave in air is nearly: (take speed of
sound in air ils 330 mls) "
.., (AJIMS ZOOZ( , L

(a) 4cm (b) 2cm


(a) . 62Sem (b) • 40 em
(e) 6 em (d) 1 em
(e) 50 em . (d) 375 em
.. I '
80. A transverse wave propagating' along x-axis is represented 85. A siren emits sound Jfintensity 1.6 W/m 2 and frequency
by: ' . 1000 Hz. Speed of sound in air is 330 mls arid density of air
is '1.3 kgtm 3 . Ampli'tu'de o'fsbund emitted by the siren is
y(x, t )=8.0sin (D.Snx-4n:t -n:1 4) nearly: ' ') .
wherexis in metre and t is in second. The speed ofthe wave
is: (eDSE 2006)
(a) 2.2x 1O~6 m (b) 3.8x 10-6 m "
(a) 41tmls . (b) 05.mls (c) , 7.2;1O":5 ,~ ' ,' (d) i :4xlO- m
5

,;. (c) , w'4 1m1s ,(d) . 8 mI, 86. Sound carried by: air from a sitar to a listen~r is a wave of
8·H The' ratio'ofthe velocity bfsound in hydrogen (.., = 71 5) to . which of the following types?

that in helium ( y = ~) at the same t~mp~rature is :


(a) Longitudinal stationary
(b) Transverse progressive
[Kama'taka CET 2007) (c) Transverse stationary
(d) Longitudinal progressive
87. A hospital uses an ultrasonic scanner to locate tumours in a 94. The waves produced by a motor boat sailing in water are:
tissue. Operating frequency of the scanner is 4.2 MHz. The (AIIMS 2004(
speed of sound in a tissue is 1.7 mls. Wavelength of sound
in tissue is close 10 : (a) longitudinal
(b) transverse
(a) 4>< 10-4 m (b) 8x 10-4 m
3 (c) longitudinal and transverse
(c) 4x 10- m (d) 8x 10-3 m , , "
(d) stationary
88. Two waves can be expressed as x::::\a' ~i~( rot+~) and 95. A wave equation is
y"" IO.!4 sin (601 + It)
:t = acosoot. Phase difference between the two waves is: , ,
(Oriss. JEE2002; CBSEPMT 2004( where x and y are in metres and t is in sec.. UIl.ieh
,i ~ "
of the
following statements is correct?
(a) -•
3
(b)!! I'
6

(c) -
2
(d) • ' .
(a) Frequency of wave is 50 1t h~rtz " .,
(b) 'Tne wave ' travels with ~ velocitY. 400 ~mls in the
'L
89. The ratio of phase veloci~ to group velocity for
non-dispersive media is : negative direction of x-axis '
(a) I : 1 (b) I : 2 (e) Wavelength of the waves is 1tmetre
(c) 2: 1 (d) 1 : 4 (d) All ofthe above
90. A sound wave travelling with a velocity V in a medium A 96. A string consists of two parts attached at x = 0. The right
reaches a point on the interface lof medium A and medium part of the string (x > OYhas mass}l r per unit length and the
B. If the velocity in mediumB is 2V, the angle of incidence leftpartofthestring(x<O)hasmaSs}l,perunitlength. The
tension in the string is T. If a wave of un it amplitude travels
for total internal reflection of wave is :
along the left part of the string, what is the'amplitude of the
(a) > 15° (b) > 30° wave that is transmitted to the right'part of the string?
(e) > 45° (d) > 90'
(a) 1 (b) ---,2~
91, With the propagation of a longitudinal wave through a
1+ r;;
material medium, the quantities transmitted in the direction
of propagation are: V;;
(a) energy 2[ii;' [ii;' -I
(b) linear momentum and mass
(c) energy and linear momentum
(b) V;; (d) V;;
1+ ~ ~+I
(d) energy, linear momentum and mass
92. Two wave pulses (the shape of one being inverted with
V;; V;;
, respect to the other) travel in opposite direction on a string 97. A flat disc has - regularly space holes along a circle
to approach each other. What will happen to them? concentric with the disc. The disc is rotated about an axis
passing through its centre and perpendicular to its plane and
(lAS 1996(
an air nozzle is directed against the disc. Everytime a hole
(a) They will collide and vanish after collision passes the nozzle, a stream of air is released and a wave
(b) The pulses will pass through each other without any pulse is generated. If the number of holes in the disc is 80
change in their shape and it is rotating at 1200 rev/min, frequency of sound
(c) The pulses will reflect, that is, the pulse going towards produced will be:
right will move to left after collision and vice-versa (.) 96000 Hz (b) 1600 Hz
(d) The pulses will pass through each other but their (c) 80 Hz (d) 4800 Hz
shapes will be modified
98. Speed of sound in a mixture of 1 mole of helium and 2
93. A wave travels unifonnly in all directions from a point moles of oxygen at 27°C is:
source in an isotropic medium. The displacement of the (a) 800 mi. (b) 600 mi.
medium at any point distance r from the source may be
(c) 1200 mi. (d) 400 mi.
represented by: (A is a constant representing strength of
source) 99. In Q. 97. if the temperature is increased by lCOC, percentage
change in the speed of sound in the gaseous mixture is:
(a) ~sin (kr - rot) (b) A..[;sin (kr-rot)
.;; (a) 1% (b) 0.334%

(c) Arsin (kr-rot) (c) 0.167% (d) 2%


(d) ,JA;'sin (kr-oot)
100, A speaker produces sound of power 30 mW. Using an 108. A very weakly damped hannonic oscillator is acted upon by
_, amplifier the power of sound is increased to 600 mW. an external periodic force and is executing SHM in the
Increase of power in dB is : steady state. The phase of the oscillator with respect to that
(a) 13 dB (b) 18 dB of the driving force at resonance is :
(c) 8 dB , (d) 2,6 dB . (a) • (b) . / 2
I "
101. Equation ora wave is represented by
"
<', '"
(c) . / 4 (d) Zem
109. The amplitude of velocity of a particle is given by
Y=104~in(1001- :o)m. .; "
Ym =Yo I (aro 2 -boo+c); the condition for single resonant
then the velocity of wave will be: frequency is: ,
'(0)"1000.nt. " ,,<b) 100 mI, ' (a) ' b 2 '< 4ac lj (bj b 2 > 4ac
(e) zero (d) 4 mls ",,, (c) b 2 ... 4ac (d) b' = " ,
,!n I,' ~I oJ I , ,., ". ,i
102. A uniform rope of mass M is suspended from a rigid lID. The ampHtude of~elocity of a particle acted on by a force,
IiI '!supJ{brt: A.,¢ave'~ulse i~ sct up 'it the iower ~d. ihen the o F cos rot, along the x-direction is given by;
pulse : ': ,,"I, '
I
x=--;=~~=
(a) travels with decreasing speed " ~(aCJi -bro+c)
(b) travels with increasing speed
( '. . " .",' " wherea,b and care constants and b 2 :<!::4ac. For what value
(e) travels With con s~t speed
'. ! i' 1 of rodoes the resonance occur?
(4) ~ot travel t~r~)Ugh ,the rope,
(a) 0)=0 (b) O)= b l 2a
103. A string ofJength I is stretched by 1130 and transverse
waves in the string are found (0 travel at a speed v o. Speed (c) oo=b l a (d) O) =C

of transverse wav,es when the string is stretched by 1/ 15 III . A sound wave in air can be represented as;
will be: y =5x 10- 2 sin [100t -5OxJ m
(a)
v.2 (b) 2v, where t is expressed in sec and x in m and y represents
displacement. The phase velocity of the wave is :
(c) 2.J2v, (d) .J2v, (a) 2 mi. (b) 3 mi.
104, Velocity of sound in a mixture of gases is 320 mls. Pressure (c) 5000 mi. (d) 0.5 mi.
of the mixture is now made 9 times keeping the temperature 112, If the equation of a progressive wave is given by
same. Velocity of sound in the given mixtur'e wi(( now be:
(a) 960 mi. (b) 2880 mi. Y= 4 sinn
' [, x
'5-'9+6 n]
(c) 160 mi. (d) 320 mi.
, ' , then which of the following is correct? ICBSE 1993)
105, AmpHtude of a wave represented by the displacement
(a) v=5cmls (b) ),= 18m
, I, ' I 'II b '
equation y.= fa ,Sin rot ± Jb cos rot WI e: (d) j=50Hz
(c) A - 0.04 cm
113, The displacement y(in em) produced by a simple hannonic
(a) ~O+b (b) .fa - .fb wave is given by;
ab . ab
y = (1O / n).;n (2lJOO>u - >fX 117)
(c) .fa +.fb (d) o+b The period and maximum velocity of the particle in the
ab ab medium respectively will be given by :
106, Presswe is increased by 1 ahnosphere and temperature also
(a) 10-3 s and 330 mls
increased by lOC, velocity of sound :
(a) increasesby61 mls (b) decreases by 61 mls (b) 10-4 sand 200 mls
(c) increases by 0.61 mls (d) decreases by 0.61 mls (c) 10-3 sand 200 mls
(e) remains constant (d) 10-4 sand 330 mi.
107, The amplitude of a damped oscillator becomes half in 1 min. 114, A transverse wave is described by the equation
The amjlHtude after 3 min will be (1/ x) times the original y = Yo sin 2n(ft -xl;\') The maximum particle velocity is
value, where x is : equal to four times the wave velocity if:
(a) 2x 3 (b) 2' (a) ),= ny, / 4 (b) ),=ny, / 2
(c) 3' (d) 3x 2' (c) ),="y. (d) 1.=2ny.
liS. The equation of a wave is given by y =IOSin[ ~~ + 0.} If ' 123. In a sinusoidal wave, the time required for a particular point
to move from maximum displacement to Zero displacement
the displacement is 5 em at t = 0, then the total phase at is 0. 175. The freqUeDCY of the wave is : ,. . lBVP 2008)
t = 7.5 s will be:
(a) (2. / 3)rad (b) (2. IS)rad
(.) 1.47 Hz
(e) 0.73 Hz
..
(b) 0.36 Hz
' (d) 2.94 Hz
.
(e) (./3)rad (d) (./2)rad 124. Fundamental frequency orla sonometer wire is 11. If the
116. A wave is represented by the equation length and diameter of thb.i wir~ are doubled keeping the
;'Y=7sin( 71tt-QMX+~)
tension same, then the neJ fundamental frequency is:
,"" !, (UPSEE 2009(
where x is in metres and t in seconds. The'speed of Waves is: 2n " • 11
(a) ../2 ',;.,> (b) 2../2
,II d' [AIEEE2004jCBSE 'PMT20061
(a) 1751tmls (b) 49.mI, ,(c) ..fin I (d) hdn~ ofthes~ ' ;'1
49
(e) - mI,

(d) 0.28. mI,
. - 125. A progressive wave in a medium is represented by the
, .... equation y c o.1sin( IOru - tim )where yand x are in cm
11 7. Velocity of sound is generally greater in solids than in gases •
because: . IBCECE ~006J and t iii s~onds. Tlie wavele~gth and velocity ofthc' w{ive
(a) density of solids is low but the elasticity is high is: ., I ' I .IAMU209?)
(b) density of solids is high but the elasticity is low 5
(a) - m, 31.4 mls , (b) ,4.4 m, 22 mls
(e) density of solids is high but the elasticity of solids is II , iV
very high
(e) 2.2 m, I I mls (d) '.'..' m, 22 mls ,.~

(d) both density and elasticity of solids are low .5 ,


118. When sound waves travel from air into water, which of 126. Angle between wave velocity and particle, velocity qf a
these remains constant? [Orissa JEE 2006; Manipal2006) longitudinal wave is : t· (Orissa JEE 2009J
,. ·.1' .
(a) velocity (b) frequency (a) 90' (b) W
"
(e) wavelength (d) all of these (e) 0° (d) 120'

119. A stone thrown into still water, creates a circular wave 127. A motor cycle starts from rest and accelera.tes along a
pattern moving radially outwards. If r is the distance straight path at 2 mIs 2 . At the starting point of the motor
measured from the centre of the panern, the amplitude of cycle there is a stationary electric siren. IHow farlhas the
the wave varies as: [AIIMS 2006) motor cycle gone when the driver hears
lf2 1 , - the
, , frequency ofthe
(a) r- (b) r- siren at 94% ofits value when the motor cycle was at rest?
(c) r- 2 (d) r-J12 (Speed of sound = 330 ms -\)
" .
(AIEEE 20091
120. The speed ofa wave on a string is 150 mlswhen the tension (a) 49 m (b) 98 m
is 120 N. The percentage increase in the tension in order to
(e) 147 m (d) 196 m
raise the wave speed by 20% is : [BCECE 2006)
128. Sound waves transfer: [Karnataka CET 20091
(a) 44% (b) 40% (c) 20% (d) 10%
(a) only energy not momentum
121. If wave y =Acos(rot+ kx) is moving along x-axis, the
shapeofpulseatt '" Oandt =2s: (DCE2007) (b) energy
(a) are different (b) are same (c) momentum
(e) may not be same (d) none of these (d) both energy and momentum
122. Two waves are given by Yl = eos (4t-2.:c) and 129. In a sinusoidal wave, the time required for a particular point

YI =sin(4t-2.:c+~} The phase difference between Y2


to move from maximum displacement to zero displacement
is 0.14 second. The frequency of the wave is:
and Yt is: IKerala PMT 20071 )Kerata PET 2009)
(a) 0.42 Hz (b) 2.7S Hz ,
(a) n:
4
(b) •
4
(e)
3n
(d) ~
(e) 1.79 Hz (d) 0.S6 Hz
4 2
3. (e) 3.5 Hz
(e)
2
130. Ai. string of mass m and length I is hanging from ceiling as 131. String A has length I,radius of cross section' r, density of
shown in the Fig. .12.27. Wave in string move upward. v A material p and is under tension T. String B has all these
and-v s are the speed of wave at A and B respectively. Then quantities double those of string A. If fA and Is are the
vB is : corresponding fundamental frequencies of these vibrating
string, then:
B (a) fA = 2f. (b) fA =4f.

,.
.
31
A
I~
(e) f. =2f A (d) fA =f.
132. A transverse wave travels along the Z-axis. The particles of
the medium must move:
(a) . along the Z-axis (b) al,ongJ,he X-axis .
'Fig. 12.27 (c) along ,the Y-axis , (d) in the Xl' plane
(a) .,f3v A (b) ,v A (e) <v A (d) .J2v A
." '.,'.
ANSWERS

I. (d) 2. (e) 3. (d) 4. (e) 5. (e) 6. (d) 7. (b) 8. (b) 10. (d) 11. (d) 12. (e)
,•• (d)
13. (b) 14. (e) 15. (e) 16. (b) 17. (d) 18. (a) 19. (a) 20. (e) 21. (b) n. (d) 23. (b) 24. (a)
25. (d) 26. (al 17. Ca) 28. <a> 29. (d) 30. (e) 3J. (e) 32. (b) 33. Ca> 34. (e) JS. (a) 36. (a)
37. (e) 38. (b) 3•• (d) 4e. (e) 41. (e) '2. (b) '3. (e) ... (e) 45. (a) 46. (e) '7. (d) 48. (b)
4'. (b) SO. (e) 51 . ' (d) 52. (b) 53. (e) 54. (a) 55. (a) 56. (a) 57. (b) 58. (b) 59. (e) 60. (a)
61. (e) 61. (d) 63. (d) 64. (a) 65. (b) 66. (b) 67. (a) ... (b) 6•. (b) 70. (e) 71. (e) 72. (b)
73. (b) 74. (e) 75. (b) SO. '(d)
76. (d) 77. (b) 78. (b) 7•. (d) 81. (c)
,81. (c) 83. (c) 84. (a)
85. (d) 1 86 . (d) 87. (a) 88. (a) 89. (a) '0. (b) 91. (e) 92. (b) 93. (d) 94. (c) 95. (e) 96. (a)
'7. (b) '8. (d) ... (e) 100. (a) tOI. (a) 10'. (b) 103. (d) 10'. (d) 105. (a) 106. (c) 107. (b) 108. (b)
109. (e) 110 (b) Ill. (a) 112. (b) 113. (e) 114. (b) 115. (a) 116. (a) lt7. (c) 118. (b) 119. (a) 110. (a)
121. (b) 122. (b) 113. (a) 124. (d) 125. (h) 126. (e) 127. (b) 128. (d) 119. (c) 130. (a) 131. (b) 132. (d)

~ ~(d2:ll
(B) More than One ChoIce is Correct 2
[Hint: Wave functions satisfy the equation d ""
of
L Amplitude vibrations relmiins'constant in case of: dx v dt
. c'a) Free vibriiions (b) Damped vibrations 5. Which of the following represent a travelling wave?
(c) Maintained vibrations (d) Forced vibrations (a) ye A ~(x-vt) (b) y=Aeos(ax +bl)
2. In case of forced vibrations :
(e) y=A log(x-vt ) (d) y=f(x'-v,')
(a) Displacement varies with the frequency of force
(b) Displacement is not in phase with force [Hint: Travelling wave is ofthc form F(ax ± bt)]

(c) Amplitude decreases exponentially with time 6. Which of the following do not represent a travelling
(d) Amplitude becomes 00 if the frequency of force . hannonic wave?
coincides with that of body (a) A(ax - bl) (b) Atan(6)I-kx)
3. At resonance: (c) Acos 2 (vt-x) (d) A cos hsin rot
(a) The amplitude of displacement is max. (Hint: Travelling harmonic wave is sin or cos function of
(b) The amplitude of velocity is max. (ax± bt»)
(c) Displacement lags the force by (n/ 2) 7. Which of the following waves represent a travelling
'(d) Displacement leads the force by (n / 2) hannonic wave?
4. The displacement of particles in a string stretched in the x· (a) Asin(cot-Io:)-Asin(rot+kx)
direction is represented by y. Among the following (b) Asin(rot-kx)+Bsin{rot-kx)
expressions for y, those describing wave motion are:
(c) Asin(rott-kt)+Asin(ro2t-kt)
(a) coslo:sin(l)t (b) k 2 x2 _ro2 t 2
(d) Asin(col - kt)+Bsin3(rot-kx)
(c) cos 2 (10:+(0/) (d) cos(kx2 _(02 t 2)
VIBRATIONS AND WAVES 593

8. A wave is represented by the equation; (c) Its practical unit is decibel


y = A sin (101tx+ 15nt + (nI3)] (d) Its practical unit is phon
where x is in m and t in s. The expression represents! 16. As a wave propagates: (lIT 1999)
IIITl9901 (a) the wave intensity remains constant for a plane wave
(a) A wave travelling in posilivex-direction with velocity (b) the wave intensity decreases as the inverse of the
1.5 mls distance from the source for a spherical wave
(b) A wave travelling in negative x-direction with velocity (c) the wave intensity decreases as the inverse square of
1.5 mls the distance from the source for a spherical wave
(c) A wave travelling ;n negative x -direction with (d) total intensity of the spherical wave over the spherical
wavelength 0.2 m surface centred at the source remains constant at all times
(d) A wave travelling ;n positive x -direction with 17. y(x, t )=o.8/[(4x+5t)2 +5] represents a moving pulse,
wavelength 0.2 m
where x, yare in metre and t in second. Then: lilT 1999)
9. A wave equation which gives the displacement along y-
direction is given by; (a) pulse is moving in +x -direction
(b) in 2 second it will travel a distance of2.5 m
• y=1O-4 sin(60t+2x)
(c) its maximum displacement is 0.16 m
where x and yare in m and t in s. This represents a wave:
(d) it is symmetric pulse
(a) travelling with velocity of 30 mls in the negative x-
direction 18. In a wave motion y == asin (k:c-Ol t ), ycan represent :
(b) of wavelength (n)m lilT 19991
(e) offrequency(30In)Hz (a) electric field (b) magnetic field

(d) of amplitude 10-4 m travelling along negative x-axis (c) displacement (d) pressure

10. A transverse sinusoid;:ll wave of amplitude a, wavelength ~. 19. A travelling wave pulse is given by y = 6 2 where
and frequency f is travelling on a stretched string. The 2+(x+3t)
maximum speed of any point on the string is wI 0, where v symbols have their usual meanings; x, yare in metre and t is
is the speed ofpropagalion of the wave. If a:::: 10-3 m and in second. Then:
v = 10 ms - ', then ).,and fare given by: (liT 1998J (a) The pulse is travelling along +ve x -axis with velocity
3m/s
(a) )..=2n:xlO- 2 m (b) A=IO- 3 m
(b) The pulse is travelling along -ve x -axis with velocity
(0) 1"IO'/(2n)Hz (d) 1"10' Hz 3m/s
11. Which of the following waves are mechanical? (c) The amplitude of the wave pulse is 3 m
(a) Torsional waves· (b) Microwaves (d) The pulse is a synunetric pulse
(c) Seismic waves (d) Ripples 20. A transverse wave is travelling along a stretched string
12. Transverse mechanical wave can travel in : from right to left. The figure shown represents the shape of
(a) Iron rod
the string (snap shot) at a given instant. At this instant:
(b) Hydrogen gas
(c) Water (d) Stretched string y
wave motion
13. Which of the following phenomena can lake place with
sound (in air)? A~B~C_D~~E____~__-+ X
(a) Interference (b) Diffraction
F"-~"H
(c) Polarisation (d) Scattering G
14. The velocity of sound is affected by change in: Fig. 12.28
(a) Temperature (b) Medium (a) the particles at A,B and Hhave upward velocity
(c) Pressure (d) Wavelength (b) the particles atD,E and F have downward velocity
15. Which of the following statements are correct about (c) the particles at C,E and Ghave zero velocity
intensity of sound? Cd) the particles at A and E have maximum velocity
(a) 1t depends only on amplitude of wave
(b) It depends both on amplitude and frequency of wave
distance of the mass from the left end of the rod in cm is
(takeg=IOms-2 ):
1. (a), (e) and (d); 2. (a), (b) and (d); 3. (b) and (e); 4. (a)
~~~~~~~~d;~~~~ ~ ~~
(e); 9. aU; 10. (a) and (e); 11. (a), (e) and (d); 12. (a) and
(d); 13. (a), (b) and (d); 14. (a) and (b); 15. (b) and (e); 16.
(a), (e) and (d); 17. (b) and (e); 18. aU; 19. (b), (e) and (d);
20. (a), (b) and (d)
40 em
ICI Integer Type Questions Fig . 12.30
1. A stretched string of length'/' is excited by plucking the ANSWERS
string at right 'angles to its length at a point which is a
1. 4 2. 2 3. 4 4.4 S. 2 6. S
distance of'-!.. from one end and stopping the vibration at
4 [D] Match the Columns
the centre of the string with a finger. Find the number of
1. Let us consider the following diagram in which
antinodes for the vibration.
a block of mass M is being supported by a
2. Two pulses are created on the rope at points A and B unifonn rope of mass I kg and length 10 meter.
respectively as shown in the diagram. A pulse is created at the bottom of the rope and
The rope is rotating about end A where as end B is free. If it reaches the top. In column I the value of Min
SA and S B are the distances covered by pulse A and pulse kg is given and in column II time (in sec) after
8 respectively in rope frame before they cross each other. If which the pulse reaches the top is given. Match
S
k = ---4..., find the value ofCk - l)2.
them Fig 1231 .
SB Column - I Column - II
3. At a distance ar20 m from a point source, the loudness is 30 (a) I (P) 2(2-.fl )
dB. After a minimum distance of x(soJiO )meter the sound
is inaudible. Value of x is: (b) 2 (q) 2(.J5-2)
4. A radio receiver is set up on a mast in a middle of calm lake
(e) 3 (r) 2(2+.fl)
to track a ratio signal from a satellite orbiting the earth. As
the satellite rises above horizon, the intensity of the signal (d) 4 (,) 2(./2- 1)
varies periodically. The intensity is maximum when the
satellite is 9 1 = 30 above the horizon and then again at 2. Consider a situation (i) that two sound waves, YI = (112m)
9 2 = 60 above the horizon, Wavelength of the satellite sin 504n(t -xl 300) and Y2 = (0.6m)sin 490n(t - x 1300)
signal is x( 3~) meter. The receiver is 4 m above lake
are superimposed, Consider another situation (ii) that two
sound waves, Y t =(o.2m)sin 504'1t(I - X/300) and Y2 =
surface . Value ofx is : (o.4 m)sin 504 'It(t +x / 3001 are superimposed. Match the
Column.I with Column . II .
5, As shown in Fig. 12.29 string P
PQR is stretched by force Column - I Column - ((
F = (3-lOkt) N, where k is a
(a) In situation (i) (p) Stationary waves ace
constant and t is time in second, R formed
At time t = 0, a pulse is generated L = 1m
at the end Pofthe string. For the (b) In situation (ii) (q) There will be the
F=[3·k(10)t) N
value of k (in N/s) if the value of phenomenon of 'Beats'
Fig. 12.29
force becomes zero as the pulse (e) When two waves 0 (r) Amplitude of the
reaches point Q. same frequency and resultant wave will vary
6. A unifonn ho rizontal rodof length40 cmand mass 1.2 kg is amplitude and periodically with
supported by two identical wires as shown in Fig. 12.30.A trevelling in opposite position
mass of 4.8 kg is placed on the rod so that the same tuning directions superimpose
fork may excite the wire on left into its fundamental
vibrations and that on right into its first overtone? The
Y N
(d) If the intensity of sound (s) Amplitude of the
alternately increases resultant wave will vary
and decreases periodically with time x
periodically as a result
of superposition of
waves of slightly R
different frequencies Fig. 12.32

Answer the foll owing questions.


ANSWERS
5. For the point P
1. a-+ $; b-+ r; c-+ p; d-+ q (a) Transverse velocity ;, negative and transverse
2. a-+ q, s; b-+ p,c; c-+ p,c; d .... q,s acceleration is positive
[E] Linked Comprehension Type (b) Transverse velocity ;s positive and transverse
acceleration is negative
Paragraph - 1 (c) Transverse velocity ;s positive and transverse
A child, playing with a long rope, ties one end and holds the acceleration is zero
other. The rope is stretched taut along the horizontal. The (d) Transverse velocity is zero and transverse acceleration
child shakes the end he is holding. up and down, in a is positive
sinusoidal manner with amplitude to em and frequency 3
Hz. Speed of the wave is 15 mls and, at I = 0. displacement 6. For the point Q
at the child's end is maximum positive. Assuming that there (a) Transverse velocity ;s negative and transverse
is no wave reflected from the fixed end, so that the waves in acceleration is positive
the rope are plane progressive waves, answer the following (b) Transverse velocity ;, positive and transverse
questions. acceleration is negative
(also assume that the wave propagates along the positive x- (c) Transverse velocity ;s negative and transverse
direction) acceleration is also negative
I. A wave function that describes the wave in the given (d) Transverse velocity ;s positive and transverse
situation is acceleration is also positive
(a) y"(0.lm)oos [(2 rad / m)x-(12.5 ..d / s)/ ) 7. For the point R
(b) y " (0.1 m)cos[(1.26 rad I m)x-(IS.S ..d I s)t) (a) Transverse velocity is negative and transverse
(c) y = (O.lm)sin [(1.5 rad I m)x - (IO rad I s)t] acceleration is positive
(d) y = (0.1 m)sin [( 1.5 rad / m)x - (4 rad / s) t] (b) Transverse velocity is zero and transverse acceleration
2. Equation of displacement ofa point 2.5 m from the child's is positive
end can be expressed as (c) Transverse velocity is negative and transverse
(a) y =--(O.! m)cos(1 8.8 rad / s)t acceleration is also negative
(b) y=(O.1 m)cos(12.5rad /s)t (d) Transverse velocity is positive and transverse
acceleration is also positive
(c) y=(O. 1 m)sin(4rad /s)t
8. For the point S
(d) y"-{O.I m)s;n(IOrad /s)/
(a) Transverse velocity is negative and transverse
3. Phase difference between the child's end and a point 2.5 m
acceleration is positive
from the child's end will be
(b) Transverse velocity is zero and transverse acceleration
(a) " (b) 3. (c) - •(d) • . is positive
2 2 4
(e) Transverse velocity is negative and transverse
4. Velocity of the child's end at t = 0 is acceleration is also negative
(a) 3 mls (b) 4.5 mls (d) Transverse velocity is positive and transverse
(c) zero (d) 12.5 mls acceleration is also positive
Paragraph - 2 9. For the point L
A sinusoidal wave is passing along a string in the positive x (a) Transverse velocity is zero and transverse acceleration
direction. yis the displacement orany point on the string and it is is positive
perpendicular to the direction of propagation of the wave. Fig. (b) Transverse velocity is zero and transverse acceleration
12.32 shows the view of the wave at a given time, say t = o,P,Q,R is negative
etc. are different points on the string.
(e) Transverse velocity is negative and transverse 15. In the questions above, weight of the rope has been
acceleration is zero neglected as compared to the suspended weight. However,
(d) Transverse velocity is positive and transverse if we also account for the weight of the rope, the speed of
,acceleration is also positive the wave at the top of the rope will be
10. For the point M (a) 60 mls (b) 64.8 mI,
(a) Transverse velocity is negative and transverse (c) 62.6 mls (d) 68.4 mls
acceleration is positive Paragraph. 4
(b) Transverse velocity is negative and transverse
One end of a long rope is tied to a ftxed vertical pole. The
acceleration is also negative
rope is stretched horizontally with a tension 8 N. Let us
(e) Transverse velocity is positive and transverse consider the length of the rope to be along X -axis. A simple
acceleration is also positive harmonic oscillator at x =0 generates a transverse wave of
(d) Transverse velocity is positive and transverse frequency 100 Hz and amplitude 2 em along the rope. Mass
acceleration is zero of a unit length of the rope is 20 gmlm.
11. ForthepointN Ignoring the effect of gravity, answer the following
(a) Transverse velocity is zero and transverse acceleration questions.
is negative 16. Wavelength of the wave is
(b) Transverse velocity is negative and transverse (a) 50 em (b) 20 em
acceleration is positive (c) 8 cm (d) 32 em
(c) Transverse velocity is negative and transverse 17. Assuming that the oscillator has its maximwn negative
acceleration is also negative displacement at t =0, wave equation (function) for the
/ (d) Transverse velocity is positive and transverse
acceleration is negative
wave can be expressed as y =
(a) -{0.02 m) cos [8x(radlm) x- tOOx(rad/s) t]
Paragraph - 3 (b) (0.02 m) cos [10. (radlm) x - 200. (radls) tJ
One end ofa 60 m long rope of mass 1.8 kg (c) (0.02 m) cos [lOx (radlm) x- 200x(radls) t -x]
is tied to a rigid horizontal support held (d) -{0.02 m) sin [8x(radlm) x- 100x(radls) t]
high above the ground. The rope hangs 18. Which of the following is correct?·
vertically and kept taut by a weight' W' (a) The wave propagates with a ftxed speed and any
suspended at its lower end. A person jerks particle of the mediwn vibrates with the same fixed
the lower end of the rope sideways in a speed.
sinusoidal maIUler and a transverse wave of (b) The wave propagates with a ftxed speed but any
frequency 2 Hz and amplitude to cm w particle of the medium vibrates with a variable speed.
passes along the rope such that there are 2 Fig. 12.33 (c) The wave propagates with .a variable speed but any
cycles of the wave in the total length of the particle of the medium vibrates with some fixed speed.
rope.
(d) The wave propagates with a variable speed and any
Neglecting the weight of the rope as compared to the particle of the medium also vibrates with a variable
suspended weight Wand with g = 10m! s 2, answer the speed.
following questions. 19. Maximum magnitude of transverse acceleration of any
12. Mass corresponding to the suspended weight W is point on the rope will be nearly
(a) 14.2 kg (b) 10.8 kg (a) 7888m1s' (b) 8244m!s'
(e) 6.6 kg (d) 8.4 kg (e) 9277 m!s' (d) 3333 m!s'
13. Maximum rate at which energy is put into the wire, i.e., the 20. Tension in the given rope remaining the same, if a simple
maximum instantaneous power is nearly hamlOnic oscillator of frequency 200 Hz is used instead of
(a) 6.4 W (b) 1.5 W the earlier oscillator of frequency 100 Hz
(c) 4.5 W (d) 2.8 W (a) Speed of transverse waves in the rope will be doubled;
14. Average power will be nearly wavelength will not change
(a) 3.2 W (b) 2.25 W (b) Speed of transverse waves in the rope will become
(c) 1.4 W (d) 0.75 W half; wavelength will become one· fourth
VIBRATIONS AND WAVES 597

(c) Speed of transverse waves in the rope will become four Paragraph - 6
times; wavelength will be doubled A sinusoidal wave is propagating in negative x -direction in
(d) Speed of transverse waves in the rope will not change; a string stretched along x -axis. A particle of string alx::: 2m
wavelength will become half is found at its mean position and it is moving in positive y
direction at t = 1 sec. The amplitude of the wave, the
Paragraph ~ 5
wavelength and the angular frequency of the wave are 0.1
A rope is attached at one end to a fixed vertical pole. It is meter, tt l 4 meter and 4tt rad/sec respectively.
stretched horizontally with a fixed value of tension T.
Answer the following questions.
Suppose, at t = ~ a pulse is generated by moving the free
end of the rope up and down once with your band. The 25. The equation of the wave is
pulse arrives at the pole at instant t. (a) y =O,1 sin [4n(r-I)+8(x - 2)]
Ignoring the effect of gravity. answer the following (b) y-O,I sin [(r-I) - (x-2)]
questions. (c) y =O,1 sin [4n(r -I) - 8(x-2)]
21. If you move your hand up and down once by the same (d) y=O, 1 sin [n(r-I)+x+2]
amount but do it more rapidly. say, twice as fast as in the 26. The speed of particle at X " 2 m and t = 1sec is
earlier case,
(a) Q21tmls (b) U6ttmls
(a) Time taken for the pulse to reach the pole will increase
and it will be doubled (0) U4nmls (d) °
27, The instantaneous power transfer through x = 2 m and
(b) Time taken for the pulse to reach the pole will decrease
and it will become half t = U2S sec is
(e) Time taken for the pulse to rt;ach the pole will not (a) 10 lis (b) 4n J/s
change 3
(d) Time taken for the pulse to reach the pole will reduce
by four times
(0)
2.
"311' (d) °
22. If you move your hand up and down once but to a greater
distance and in the same amount of time
(a) Time taken for the pulse to reach the pole will increase
(b) Time taken for the pulse to reach the pole will not
change
(c) Time taken for the pulse to reach the pole will decrease
(d) Time taken for the pulse to reach the pole may increase
or decreasr.
23. If you use a string of same length but of greater mass
(a) Time taken for the pulse to reach the pole will not
change [F) Subjective Type Questions
(b) Tirrie taken for the pulse to reach the pole will increase I. Which type of oscillations are produced when (a) an
(c) Time taken for the pulse to reach the pole will decrease electric field vibrates in a light wave propagating in glass,
(b) a metallic strip oscillates in a magnetic fi eld, (c) the
(d) Time taken for the pulse to reach the pole may increase
pendulum in a clock oscillates, (d) the diaphragm of a
or decrease .
microphone or loudspeaker vibrates, (e) you tune your TV
24. In all the questions above, we have considered a fixed value set to catch a desired station?
of tension. However, if tension in a given rope is increased [Ans. (a) Free (b) Damped (c) Maintained (d) Forced and (e)
and a pulse is generated as described, Resonant]
(t:.) Time taken for the pulse to reach the pole may increase 2. Given below are some examples of wave motion. (A) State
or decrease which waves are mechanical and which are non-mechanical.
(b) Time taken for the pulse to reach the pole will not (B) State whether the wave is transverse, longitudinal or
change combined or either. (a) Waves produced in a pipe
(c) Time taken for the pulse to reach the pole will increase containing a liquid by moving a piston back and forth. (b)
(d) Time taken forthe pulse to reach the pole will decrease Ultrasonic waves in air produced by a vibrating quartz
crystal. (c) Seismic waves produced by an earthquake. (d)
598 PHYSICS FOR C OMPETITIONS - Vol. 1

Waves produced by the motion of a boat sailing on water. surface of earth. Will the velocity change during
(e) TV waves travelling from transmitter to receiver. (f) propagation? Prove your statement.
Light waves travelling from sun to earth. [Ans. In going up density and pressure both decrease; however as
[Ans. (A) TV and light waves are non-mechanical while all others v ... J'IRT / M, the velocity will remain constant as long as
arc nlcchanical. (8) (a) longitudinal (b) longitudinal (e) both temperaNre remain constant.]
transverse and longitudinal but separately (d) combined
12. USe the fonnula v = ~yP I p, to explain why the speed of
transverse and longitudinal (e) transverse (t) transverse]
sound in air (a) is independent of pressure and (b) increases
3. What type of mechanical waves do you expect to exist in (a)
with temperature.
vacuum, (b) air, (c) inside water, (d) rock, (e) on the surface
of a liquid? 13. Write an expression for specd of mechanical waves in (a)
[Ans. (a) no wave (b) longitudinal (e) longitudinal (d) either string (b) rod (c) liquid (d) gas and (e) rock when
transverse or longitudinal or both separately (e) ripples, i.e., propagating as (i) transverse wave (ii) longitudinal wave.
combined transverse and longitudinal]
[An" (.) f!
V;;; (b) II (,) Vp
Vp II (d) fp
fYP
4. Is it possible to have longitudinal waves on a string? A
transverse wave on a steel rod?
[Ans. (a) No; See Question III (b) Yes; See § 12.2 (A) (d)]
(e)(i)~ (ij)JB+ Tl/3]
p
5. The source of energy of sun is fusion of hydrogen which 14. Show that the slope of a transversely vibrating string at any
provides energy in the fonn of heat, light and sound. point on it is numerically equal to the ratio of the particle
Explain why sound from sun does not reach earth while speed at the point to the wave speed on the string.
heat and light do?
IMNR 19931
[Ans. Sound is a mechanical wave and so requires a medium for
its propagation and as there is no medium between sun and earth, (Hlnl: See solved Problem 13 in Chapter I]
sound cannot reach the earth. Heat and light are non-mechanical 15. Calculate the temperature at which the speed of sound in a ir
waves, so do not require medium for propagation and reach the becomes double of its value at O"C.
earth. ) [Ans. 81!)OC1
6. The speed of sound waves in air depends on temperature 16. Velocity of sound in a tube containing air at 27"'C and at a
but that of light does no~. Why? pressure of76 cm ofHg is 300 mls. What will its velocity
[Ans. Sound is mechanical wave and for it v '" ,JyRT ! M while be when the pressure is increased to 100 cm of Hg and the
light is non-mechanical (electromagnetic) wave and for it temperature is kept constant?
,.1/ J"o'ol [Ans. Same 300 mls)
7. Light waves can be polarised while sound waves cannot be. 17. Defining the terms energy density and energy flu x in
Why? reference to a wave, show that:
[Ans. Light waves arc transverse while sound waves are Energy flu x = Energy density x Wave velocity
longitudinal] [Hint: See § 12.3 (d) with note (i)]
8. Give an evidence in support of the fact that (a) sound is a 18. Each of the fo llowing properties of sound listed in column
wave (b) sound is a mechanical wave (c) sound waves are A primarily depends on one of the quantities in column B.
longitudinal. Write down the matching pairs from the two columns.
[Ans. (a) diffraction (b) requires medium for propagation (c)
cannOI be polarised) ColumnA Column B
9. When sound travels fro m a ir into water, does the frequency
Pitch Wavefonn
of the wave change, the wavelength, the speed?
(Ans. No; Yes; Yes) Quality Frequency
10. Explain why: Loudness Intensity
(a) Velocity of sound is generally greater in solids than in
(Ans. Pitch~ Frequency; Quality -+ Wavefonn and Loudness -+
gases.
Intensity]
(b) 'The velocity of sound in oxygen is lesser than in
19. In view of the characteristics of sound, distinguish between
hydrogen.
the roar of lion and humming of mosquito.
11. Write down Laplace fonnula for velocity of sound in an
[Ans. Roar has large loudness but low pitch while humming has
elastic medium. The air gets thinner as we go up in the low loudness but high pitch)
atmosphere. Sound waves propagate upwards from the
20. Why is it sometimes difficult to recognise the speaker's 26. If at t :=: 0 a travelling wave pulse on a string is described by
voice over the telephone? the function:
[Ans. Due to poor quality of sound) 6
21. Fill in the blanks:
Y = -lx02~+3-J

(a) A normal ear cannot listen to mechanical waves of what will be the amplitude and wave function representing
frequency exceeding ............ Hz which are called the pulse at time t, if the pulse is propagating along positive
... ....... .. waves. x -axis with speed 4 mls?
(b) Phase difference between waves y: a sin (rot - la)and [Hint: For motion of pulse along positive x-axis replace x by
y "" acos (rot -la) is ............ IEAMCET 1992J x -vtwith v = 4 mls in the wave function.]
(c) A body vibrating with a certain frequency sends waves [Ans. Amplitude = 6/3 = 2 m; y= 6!1(x-4t)2 + 3]]
of wavelength IS cm in medium A and 20 cm in
27. A man seeing a lightning starts counting seconds, until he
mediumB. If the velocity of waves in A is 120 mis, that
hears thunder. He then claims to have found an
inBwillbe ............ mls. IEAMCET1991J approximate but simple rule that if the count of second is
(d) A point source emits sound equally in all directions in a divided by an integer, the result directly gives, in km, the
non-absorbing medium. Two points P and Q are at a distance of the lightning source. What is the integer?
distance of9 m and 25 m respectively from the source. [Velocity of sound in air = 330 mls]
The ratio of the amplitudes of the waves at P and Q is [Hint: See solved Problem 2 in Chapter I]
[Ans.3]
(e) The term woofer (or bass) refers to ............ frequency
28. From a cloud at an angle of30" to the horizontal we hear the
part of musical sound.
thunder clap 8 s after seeing the lightning flash. What is the
(0 The quality of a note is determined by ............ present height of the cloud above the ground if the velocity of
in it. IEAMCET 1992J sound in air is 330 mls?
[Ans. (a) 20,000, ultrasonic (b) rrJ2 (c) 160 (d) 25/9 (e) low (1) [Ans. 1.320 Ian]
overtones]
29. A load of 20 kg is suspended by a steel wire as shown in
22. State whether the following statements are true or false Fig. 12. Velocity of w'aves when rubbed with a resined
giving reasons in brief: cloth along the length is 20 times the velocity ofthe waves
(a) A wave is represented by the equation y:=: o.S sin in the same string when it is plucked. Find the area of cross-
(IOt+x)m. It is a travelling wave propagating along section of the wire if Y for steel is 19.6x 10 10 N/m2 and
positive x -direction with velocity 10 mls. g:9.8m/s 2 .
(Roorkee 1995) [Hint:vT =..JTlm ",.j(TlpA) andv L ",.j(Ylp)]
[Ans. False, as the wave is propagating along negativex-axis with [Ans. 0.004 cm 2l
speed 10 mls]
30. A blast gives a sound of intensity 0.80 W 1m2 and frequency
(b) The ratio of the velocity of sound in hydrogen gas
I kHz. If the density of air is 1.3 kglm 3 and speed of sound
(y:7/S)to that in helium gas ('y~S/3) at the same
in air is 330 mls find the amplitude of the sound wave.
temperature is (51)/5.
[Ans. 9.7 x 10-6 m]
[Ans. False, as vH 1vHc '" (..J42)1 5] 8 2
31. A sound has an intensity of 5x 10- W/m . What is the
23. A wave is expressed by the equation y: o.Ssin 1t(0.Olx-3t), sound level in decibel?
where yandxare in m and tins. Find the speed of propagation. [Ans. 47 dB]
[Ans. 300 mls] 32. A certain sound level is increased by an additional 30 dB.
24. The equation of a travelling wave is: Find the factor by which (a) its intensity increases and (b)
y '" 0.07sin (l21tx - 50On:t) its pressure amplitude increases.
where the distances are in m and time t in s respectively. [Ans. (a) 103, (b).J103 ~ 32]
Calculate the wavelength and velocity of the wave. 33. The loudest sound, that the ear can tolerate at a frequency of
IMNR1995J I kHz corresponds to intensity of I W/m2 (the so called
[Ans. A:=: (l/6)m and v ", 41.7 mls] threshold of pain). Determine (a) the pressure amplitude (b)
displacement amplitude and (c) sound level. The density of
25. Write down the equation for a wave propagating with air is 1.3 kglm 3 and speed of sound in air is 330 mls.
velocity 330 mls and having frequency 110 Hz. The
[Hint: See solved Problem 20 with SL ::; 1010g(1/10-12) ", 120dB]
amplitude is 0.05 m.
[Ans. y:=: 0.05 sin 21t[llOt ± (x 13) + 4> D [Ans. (a) Po =30 N/m2 (b) A = 1.1 x 10-5 III and (c) 120 dB]
34. A line source emits a cylindrical expanding wave. maximum value of the displacement at any time and place?
Assuming tbat the medium absorbs no energy find how the (Take the density of air to be 1.29 kgfm 3 .)
intensity and amplitude of the wave vary with distance r (Ans. (a) 0 (b) 3.67 x 10- 7 m)
from the source.
36. A conveyer belt moves to the right with a speed v = 300
[ADS.l oc (1 1r)and A <x:: (ll..rr)]
mlmin. A very fast man puts pies on the belt at a rate of 20
per minute and they are received at the other end by a pie
Note: As for a cylindrical wave power will be related to intensity by
eater:
the relation
(a) lfthe pie eater is stationary, find the spacing A.between
P "" I (area) '" 121trl
pies and frequency f with which they are received by
So I a:: (I I r)and I ac: A2 ,..4 IX (1 / ,J;}. Further as equation of the sta~ionary pie eater.
plane progressive waves is y '" A sin (rot - kx). the equation of (b) The man now walks with a speed of30 mlmin towards
a hannonic-expanding cylindrical wave will be
the receiver while continuing to put pies on the belt at a
y::: (B I.[,.)sin (rot -kr) rate of20 per minute. Find the spacing of the pies and
where B is a constant of proportionality having dimensions the frequency with which they are received by the
fLY' l stationary pie eater.
(c) Repeat your calculations fora stationary man and a pie
35. A typical loud sound wave with 8 frequency of I kHz has a eater who moves towards the man at 30 mlmin.
pressure amplitude of about 10-4 atm. (a) At t .. o, the [AilS. (a) 15 m, 20 pie/min; (b) 13.5 m, 22.2 pie/min; (c) 15 m, 22
pressure is maximum at some point Xl' What is the pie/min.]
displacement at that point at I::: 0 ? (b) What is the
Superposition of Waves
§ 13.1 Reflection and Refraction ~f waves i.e., the phase of djsplacement wave changes by 1t in
When sound waves are incident on a boundary between case ofreflection from a denser medium. fued end or
two media, a part of energy of incident waves returns back into rigid support. while in case of reflection from rare
the initial medium (reflection) while the remaining is partly medium or free end there is no inversion of wave or
absorbed and partly transmitted into the second medium phase change..... The transmitted wave is never
(refraction). In case of refl ection inverted. This is shown in Fig. 13.2 (For proof see
Question I.)
Rare

Yt .. A.,sfn
(rol- k:X)

Transmitted
~9flected wave
wave
Ag.13.1
(1) The frequency of the wave remains unchanged, i.e.,
wf =oor =00/ =00 (al (bl
(2) The incident ray, reflected ray, nonnal and refracted Fig. 13.2
ray are always in the same plane. Echo can be cited as an example of reflection of sound
(3) In case of reflection, fro m a distant object such d - -- - -I

Angle of incidence i = Angle of reflection r


(4) In case of refraction,
as hill or cliff. If there is
sOllnd reflector at a::;
a
~So="~m:.~====~===1
distance d from the source, Detector
sin; = vi (V i = velocity in the first medium
the time interval between Reflector
sin t VI original sound and its echo Fig. 13.3
at the site of source will be
VI =ve!ocity in the second medium]
(5) In case of reflection from a denser· medium or rigid t=!t+!t=2d
v v v
support or fixed end there is inversion of reflected
displacement wave, i.e., if the incident wave is given Now as persistence of ear is (1110) s, echo of a sharp or
by momentary sound (such as clap or gun-fire) will be heard if
y = Ai sin (rot - kt) t >l. or 2d>1.. ie, d > v
10 v 10' . , 20
the reflected wave will be
In case of articulate sound, it has been found that a person
Y = -Ar sin (wt + kt) = Ar sin (oot + kt + 1t)
can pronounce ~o as to be heard distinctly at most 5 syllables

• The concept ofrare and denser medium for a wave is through its speed (and not density ofmcd ium), e. g., water is rare for $ound and denser fOTlight than
air as for sound t1j" > v A while for lighl Vw < V A'
on In case of refl ection of longitudinal pressure waves (§ 12.3 (c)l reverse is the case, i.e, it suffers a phase change of 1t from a free or open end and no
change in phase from rigid boundaries.
602 PHYSICS FOR COMPETITIONS - Vol. I

per sec; so the minimum time interval between the original (3) As Ar will be negative if v 2 < VI' i.e., in case of
sound and its echo is (1/5) s for monosyllabic sounds, (2/5) 5 reflection from a del/ser medium (or rigid boundalY
for hisyllabic sounds and so on. or fIXed end ) there is a phase change of1t
Furthennore, multiple or successive echoes will be Problem 1. A man standing in front of a mountain at a
produced when there are two or more reflectors. If successive certain distance beats a dnlm at regular intervals. The
echoes take place at regular interval of time it is called drumming rate is gradually increased and he finds that the
harmonic echo. echo is not heard distinctly when the rate becomes 40 per
Question I. Finding the amplitude of reflected and minute. He then moves nearer to the mountain by 90 m alld
lransmilted displacement waves from a plane boundary at finds the echo is again not heard when the drumming rate
normal incidence, discuss the change in phase ofreflected and becomes 60per minute. Calculate (a) the distance between the
transmilted waves if any. mountain and tlte initial position ofthe man, (b) the velocity of
sound.
Answer: Suppose the incident wave of amplitude Ai and
Solution: Initially as the drumming rate is 40 per minute,
frequency ro is propagating along positive x-axis with velocity
i.e., time interval between two successive drum beats is
vI ·i,e., (60/40), i.e., (3/2) s, and as the echo is not heard distinctly, the
Yj = AI sinoo[t-(xlv 1}] .... (i) time taken by sound to travel to the hill and return back is (3/2)
Now as on reflection frequency does not change and for s. So, if d is the distance between man and hill and V the
Donnal incidence, the reflected wave will move opposite to velocity of sound,
incident wave (i.e., along negative x.axis), but in same
medium, so if Ar is ils amplitude, il will be given by
Yr =Arsinoo[t+(xlv l )] .... (ii)
Transmitted wave will move in the direction of incident
al-'
=;:::: 2 d -,:.;;;J;/!!1
::::::=::,

wave with same frequency in the other medium with speed v 2 Fig. 13.4
and so if At is its amplitude, it will be given by
Y, =At sinoo[t - (x l v 2 )J ...• (iii)
!!+!!=2d =1 .... (i)
v v v 2
Now as wave is continuous so at the boundary x::; 0, When the man moves 90 m towards the hill, the distance
continuity of displacement requires between him and the hill will be (d - 90) and as the echo is not
Yj+Yr=Yt for x =O heard again at drumming rate of 60 per minute, i.e., for time
Substituting Eqns. (i), (ii) and (iii) in the above with x = 0 interval (60/60) = I sec, so
and simplifying, we get ·2(d -90) = I .... (ii)
Ai + Ar = A, .... (iv) v
Also at boundary the slope of wave wiU be continuous, Solving Eqns. (i) and (ii) ford and v, we find (a) d =270 m
i.e., (b)v=360mls.
ProhJem 2. An engine approaches a hill witlt a cOl/stam
for x=O speed. Whe... it is at a distance of 0.9 km it blows a whistle.
whose echo is heard by the driver after 5 sec. 1/ the speed 0)
which in the light ofEqns. (i), (ii) and (iii) gives sound in air is 33\.1 mis, calculate the speed of the engine.
-A 00 A ro - A ro Solution: If the sj'ed of the engine is V, the distance
_'-cosrol + - '- cos rot ::; - ,- coso>,
VI VI v2 travelled by the engine in 5 sec will be 5V. And hence, the
distance travelled by sound in . the hill and comin~
i.e., Ai -Ar = (v l /v 2 )A , ... (v)
back to the moving driver = 900 + =(1800 - 5V).
Solving Eqns. (iv) and (v) for Ar and A,. we get
2v
andA= 2A
I VI + v2 I
These are the required results and from these it is clear that Fig. 13.5
in case of displacement waves:
So time interval between the original sound and its echo
(1) As At is always positive whatever be VI and v 2 ' the
phase oftrans milled wave a/ways remains unchanged. 1800 -5V =t=5s (given)
330
(2) As Ar will be positive only ifv2 > vI' i.e., in case of
reflection from a rare medium (or free end) there is The above equation on solution gives V = 30 mls.
no change in phase.
SUPERPOSITION OF WAVES 603

Problem 3. A person standing between two parallel hills, Note: This problem is an cxample of harmonic echo as here echoes are
fires a gun. He hears the first echo after 1.5 s and the second produced at regular interval (of2 s), i.e. ,at time 2 s, 4s, 6 s, 8 S, .. .
after 2.5 s. If the speed of sound is 332 mls. Calculate the SO in case ofhannonic echo if the time interval between original
distance between the hills. When will he hear the third echo? sound and I echo is T, the II echo will occur at 2T while 3rd at 3T
Solution: Let the person P be at a distance x from hill HI and so on. And in echoes at (3T),2x(3T),3 x (3T), sound from
both hills will reach simultaneously.
and y from H 2 as shown in Fig. 13.6. The time interval
between the original sound and echoes from HI andH 2 will be Problem 5. A road runs midway between two parallel
respectively, rows ofbuildings. A motorist moving with a speed 0136 kmlhr
2x sounds the horn. He hears the echo one second after he has
tl =-v and sounded the horn. Find the distance between the two rows of
buildings. When will he hear the echo a second time?
Solution: As the motorist is travelling along the middle,
he will hear echoes from both the sides simultaneously at PI'
P2, etc., as shown in Fig. 13.7. According to the given problem,
Fig. 13.6 PPI =Vxt=[36x158]xl=10m
So the distance between the hills,
A B
x + y= ¥[tl + t2 ]= 3;2 [1.5 +2.5]=664m 77 7)1'2
;;y/!
TEl 7 VI 57 7· -1
''';1.." Pl ~::J/ii-r·''';1.." P2 ~:r/

Now as I echo will be from HI after time tl while II echo


p"!
I n, OI
/ ~ ',/
v. ~ : 6~-~:;;;><'~,~ L I ., Lr /><~"~
",/"'f' ,~/"'f'
~,~

~,
from H2 after time t 2, III echo will be produced due to ! 'I' "Ii 7 1'
reflection of sound ofI echo fromH 2 orofH echo fromH I , i.e., e 0
Fig. 13.7
t3 =tl +t2 =1.5+2.5 = 4 sec.
Now as the time taken by reflected sound to reach P1 from
i. e., III echo will be produced after 4 sec and in it sound from
Pis 1 sec.
both I and II echoes will reach simultaneously.
PA+API = 330xi=330m
Note: This problem is an example of multiple or successive echoes so PA=(330/2)=165m [asLi=b,PA=APd
which are not hannonic, as t) = 1.5 s,t2 '" 2.5 sand t3 = 4 s.
In right·angled triangle PDA,
Problem 4. A rifle shot is fired in a valley formed between
OA =~(PA)' _(PO)'
two parallel mountains. The echo p'om one mountain is heard
afler2s and the echo from the other mountain is heard2s later. or OA =~(165)2 - (10/2)' =164.92 [as PO = PP/2]
Ifvelocity of sound is 360 mis, (a) What is the width of the
valley? (b) Is it possible to hear the subsequent echoes from So the width of the road
the two mountains simultaneously at the same point? If so, AC = 2x OA = 2x 164.92 =329.84 m Answer
after what time? From the figure it is also clear that he will hear successive
Solution: (a) LetP be the position of source of sound and echoes from both sides simultaneously at regular intervals of 1
listener at distances x and yfrom hills HI and H2 as shown in sec., i. e., the 2nd echo will be heard after 2 s and the 3rd after 3
Fig. 13.6. If HI is closer to P, it will produce I echo so that s from the starting point.
(2xl v) = 2sec, i.e., x =v .... (i) Question II. State whether thefollowing statement is true
and as II echo is heard 2 slater, i. e., time interval between or false, giving reason in brief'
original sound and II echo (from H 2) is 2 + 2 = 4 s, so "A plane wave ofsound travelling in air is incident upon a
(2ylv) = 4sec, i.e., y=2v .... (ii) plane water surface. The angle of incidence is 60 D• Assuming
Snell's law to be valid for sound waves, it follows that the
So from Eqns. (i) and (ii) the width of the valley,
sound wave will be refracted into water away from the
x+ y=3v=3x360=1080m normal. "
(b) III echo will be produced by reflection of sound of I Answer: From Snell's law,
echo from H 2 or of II echo from HI' i. e.,
sin i = v A :::. 330 = 022
t3 =1 1 +t2 = 2+4=6s sint Vw 1500 .
i. e., after 6 s (from original sound) we will hear echo from both
the mountains simultaneously at same position. or sin i = 0.22 x sin (t)
Now as for refraction into water, Y = Ao + Al sin rot + Az sin2rot + .. .
L(t)max ::: 90°, i.e., (sint)max =1 + B] cos rot + B2 cos2rot ~ .. .
So (sini)max = 0.22, i.e., (i)max =sin- I (O.22):::'13°
o
As here i is given to be '60°, i.e., i> (i)max' so the sound
wave will not be refracted into water but it will be totally
reflected back into the air from the air-water boundary. Hence,
the given statement is wrong.
(s) (b)
Note: In this problem i(= 60°) is greater than ic (= imu = 13°) and Fig. 13.8
sound is moving from denser to rare medium, so actually total The resultant complex wave will be periodic wit.
internal reflection wjll take place at the boundary and no sound
periodicity T = (2rrJro). Its shape will depend on the number (]
will enter into the water.
harmonics present and their frequencies and amplitudes. Th
§ 13.2 [AJ Principle of Superposition effect of such a sound wave to the ear is pleasant and so th
sound is called musicaL
So far we have considered a single mechanical wave
propagating in a medium, However, if two or more waves Y] =Asin2rot=~A[I-COS2rot]
arrive simultaneously. the particles of the medium are
Y2 = B cos 3 rot = ~B [cos3rot +3cosrot]
subjected to two or more simultaneous displacements and a
and
new wave is produced. This phenomenon of intermixing oftwo
or more waves to produce a new wave is called superposition are simplest examples of complex periodic waves wit
ofwaves. periodicity (27t12co)and (27t1ro) respectively. (y] and Y2 -are nc
In case of superposition of waves the resultant wave harmonic waves but the result of superposition of harmoni
function at any point is the algebraic sum of the waves.)
wave-functions of individual waves, i. e., Fourier has shown that any periodic function can b
Y=Y1 +Y2 +Y3 + ... represented as the sum oflarge number of sinusoidal waves c
This principle is catled principle of superposition and carefully chosen frequencies and amplitudes, e.g., a comple)
holds good as long as the amplitude of the wave is not (00 periodic wave called square wave as shown in Fig. 13.8 (t
large, i. e., the restoring force is proportional to displacement. with the help of Fourier analysis is given by
This principle can be applied to many types of waves including
y= 4A [sin rot + !sin3rot + IsinSwt + ... ]
wave pulse, waves on strings, surface water waves, sound n J 5
waves and light and radio (i.e., electromagnetic) waves. In this i. e., if a number ofwav~s with frequencies in the ratio of 1: 3
chapter we shall apply this principle to hannonic waves which 5 ... and amplitudes in the ratio (111): (1/3) : (1/5) propagat
under specific conditions gives rise to complex waves, simultaneously, their superposition will give rise to a comple
inteiference, beats and stationary waves. periodic 'vave whose shape is square and time period (27t1ro
(B) Complex Waves Fig 13.9 (3) shows the waves for first three 'component wave
If in a medium a number of waves are propagating,by and their resuaant, while Fig. 13;9 (b) shows the resultar
principle of superposition the resultant will be, wave for 15 tem.~.
Y= YI + Y2 + Y3 + ... Resultant Curve
Curve to be
or y = I,[ro, (I -x/v)] + 12 [ro 2 (t - x/v)] + ... y 1,,
/"'y'-'
,..;
, ..... analy"d
,-,
Now if the component waves are not harmonic, or ifin _\_l_ ,\ T T
different harmonic their frequencies do not bear a definite ,,
relationship among themselves, the resultant complex wave '.' .1
will have no definite regularities, i.e., will not be periodic. In
case of sound the sensation of such waves is not pleasant to the (s) (b)
ear and is called noise. [Fig. 13.8 (a)] Fig. 13.9

However, if the frequencies of component harmonic § 13.3 Interference* of Sound Waves


waves are integral mUltiples of some frequency, say ro, then
When two or more sound waves arrive at a poir
taking (t -xlv) ~ t,
simultaneously almost along the same line, the resultar
* Here we are assuming the waves to be displacement waves, but same will be valid for longitudinal pressure waves with YI '" PI with Al = PlO and Y2 = J
with 142 = P20·
intensity of sound at that point is different from the sum of From Eqn. (viii) it is clear that the resultant intensity at P
intensities due to each wave separately. This modification of is not just the sum of intensities due to separate waves
intensity due to superposition o/two or more WQves is called (J I + 12 ) but different and depends on phase difference +
interference. (given by Eqn. (Hi)l, i.e., position of the point P. So:
Let the two (a) IntenSity will be maximum where
waves each of
frequency co angular
from Sl[==~=~'=::::::;::::d p cos+ = max = +1
sourcesS, andS 2 reach 0 ---------------------- i.e., +
=O,2n,41t, ...
the point P as shown in
'I,.b'll
or += 2M with n = 0,1,2, .... .... (ix)
Fig. 13.10. If initially 82
the waves are in the
or ~.1t(6x)=21U1 [as from Eqn. (iii),. =~1t Ax]
same phase and have Fig. 13.10
or !:lx=n')" with n=O,I,2,... .... (x)
amplitudes Al and A z respectively,
Y, =A,sin [rot-la] And I rrsx =(1 1 +12 +w,)
.... (i)
and Y2 =Azsin[rol-k(x+ru)] i.e.,
or Y2 =A 2 sin(mt-kx-cp] .... (ii) i.e., intensity will be maximum at those points where path
with ~ =k<lx =~n(<lx) [aSk=~nJ .... (iii) difference is integral multiple of wavelength')... and maximum
intensity is greater than the sum of two intensities (II +1 2 ),
So the resultant wave at P by principle of superposition These points are called points of constructive interference or
will be interference maxima.
y = Y, + Y2 =Al sin(w/ -h) + A2 sin(wt -lex -cp) (b) Intensily will be minimum where
or y={A1 +Azcosq,]sin(rot-kt)-[Azsin$]cos(rot-h:) cos+=min=- I
Now if AI + A z cosq, = Acos9 i.e., ~ = rr, 3rr, 5rr, ...
and A z sin$ =Asine .... (iv) or ~ =(2n-l)n with n=I,2,3,... .... (xii)
the above equation will become or ~ (<lx) = (2n -I)n [as from Eqn. (iii),$ = ~n <lx ]
Y= Asio (rot -1a)cos9 - A cos (oot - kt)sin9
or y=Asin(ro/-la -9) .... (v) or <lx = (2n -1»).12 with n = I, 2, ... .... (xiii)
(as sin (A - B) =sin A cosB - cos Asin B] And I min =(1 1 +12 -2~1/2)
with A = [A12 + Ai + 2AIA2 cos~ J1I2 .... (vi) i.e.. 1m: = (..[1; ~..p;)2 oc: (AI - A2)2 .... (xiv)
[bysquaringandaddingAsine and A cose in Eqn. (iv)J i.e., intensity will be minimum al those points where path
and 9=tan- I(A 2 sin+ / (A I +A 2 cos+») .... (vii) differe1lce is odd integral multiple of (A/2) and minimum
intensity is lesser than the sum of two intensities (II + J 2)'
[by dividing Asine by A cose in Eqn. (iv)]
These points are called points of destructive interference or
From Eqn. (v) it is clear that in case of superposition of interference minima.
two waves of equaf frequencies propagating almost in the
From (a) and (b) it is clear that in case of interference:
same direction, resultant is a hannonic wave of same
frequency ooand wavelength (A = 27t1k) but amplitude A [given (I ) All maxima are equally spaced (as path difference
by Eqn. (vi)]. Now as from Eqn. (iv) of§ 12.3 (D), intensity of between two consecutive maxima is A.) and equally
a wave loud [IlilaX =(..[1; +..[i;) 21 Same is also true for
minima with I min = (..[1; ..... ..[i;)2. Also interference
maxima and minima are alternate as for maxima
So the intensity of resultant wave !:lx = 0, A..2A., etc., while for minima!:lx = (A.I2). (3')... /2).
I = K[Af + Ai + 2AI A2 cos .] [from Eqn. (vi)] etc. This all is shown graphically in Fig. 13. 11 .

or .... (viii)
1=1 1 + / 2 +2(~/IJ2 ) cos 4>
Here, II =1 and 12 =4/,
So I = 1+ 41 + 2.J41 x I cos~ = 51 + 41 cos~
So (a) For~
=0 1 =51 + 41 x 1=91 [as cosO =I]
Phase Diff.O 1f 21t 311 411 Sir 6n: _ <jo
(b) For~ = (ttf2) 1=51+4/ x O=51 [as cos(ttf2) = OJ
Path OiH. 0 m " 3A1l 2). 5)./1. 3).. !J. x and(c) For~=. 1=51+4/(-1) ~ 1 [as cos (1t) =-1]
Fig. 13.11 Problem 7. Two speakers connected to the same source
(2) IIf'W( = (Jj; +Fz)2 = (AI +A2)2 with ~ = Al2
offixedfrequency are placed2.0 m apart in a box. A sensitive
microphone placed at a distance of4.0 m from their mid-point
Jmin (Ft _ .[i;)2 (AI - A2)2 12 Ai along the perpendicular bisector shows maximum response.
So if/I and] 2 or Al and A2 are given (/ rna/I min lean The box is slowly rotated till the speakers are in line with the
be calculated and vice-versa. From the above it is also microphone. The distance between the mid-point of the
clear that if I] = / 2 =1o, speakers and microphone remains unchanged. Exactly 5
maximum responses are observed in the microphone in doing
1= = (F, +F,)2 = 4/ 0 this. Calculate the wavelength of the sound wave.
and Imjn =(Fo -Fa)2 =0 Solution: As shown in Fig. 13.12 (a) initiallyS 1M = S2 M,

i.e., in maxima intensity will be 4-times that ofa single Ax = 0, i. e.,at M there is zero order maxima.
5,
wave (/ 0 ) while intensity of minima is zero if the
inte!ferillg waves are ofequal inlensities.
(3) In interference the intensity of maxima
exceeds the sum of individual intensities + by
an amount 2~1112 while of minima (""I, - V I Z)2
<.,fi; + ji:;)2
ffJ J?!--
fT
.
2m!O
,I : - - 4 m -
s, <a)
)
M
e:>

Fig. 13.12
-2m-
&.!

SI
• •
O!-4m-

(b)
M
---- ~ --- -- -i!>

lacks(!, + /2 )by thesameamount2~1II2 (shown in According to the given problem on rotation of speakers
Fig. 13.11). about 0 when SI and S2 are in line with microphone M as
shown in Fig. 13.12 (b), 5 maximum responses are observed,
Hence, we conclude that in interference energy is
neither created nor destroyed but is redistributed. i.e., S2M -SIM =51.. or S2S1 =5/...
(4) Here we had assumed that the two waves from Sl and i.e., A. =215=OAm [asS I S 2 = 2m]
S2 start in the same phase. Hence, at P they have a Problem 8. In a large room a person receives direcl
constant phase difference $ = (2n/'A )ilx, developed sound waves from a source 120 m away from him. He alst;
due to different paths traversed by them. Such waves receives waves from the same source which reach him, beinE
are said to be 'Coherent' and produce sustained reflectedfrom the25 m high ceiling at a point halfway betweer.
interference effects. However, if there is an initial them. For which wavelengths will these two sound wave~
phase difference between the waves $ 0 then $ = $ 0 + interfere constructively?
(2n/'A)6.x and if $o is not constant and varies rapidly Solution: As shown in Fig. 13.1 3 for reflection from thf
and randomly with time, at P sometimes constructive
ceiling:
and sometimes destructive interference will take c
= ~(lmax + /min) =(/1 + 12 )

S~P
place so that, l av and
hence, no interference effect is observed. Such waves
are called 'Incoherent',
So for observing illteljerence effects waves must be Source Detector
coherent. Fig. 13.13
Problem 6, Two sources afintensities I and 41 are used in path SCP =SC +CP =2SC [as L;= Lr,SC = CPJ
an inreljerence experiment. Find the intensity at points where
the waves from the two sources superimpose with a phase or path SCP=2~60' +25' = 130m
difference of(a) zero, (b) (n/2) and (c) 1t So path difference between interfering waves along path~
Solution: In case of interference of two waves of SCP andSP,
intensities II and 12 with phase difference $, <u=130-120=IOm
Now for constructive interference at p. I oc At = max (or min), i.e.,cos 2 21tfA t = 1 (or 0)
~=nA. i.e., 10 =1If.. or cos 21ifAI=±1 or zero
or ).= 10 with n= I.2, 3 ... i. e.. 2-IA t = O. "2,, ... or (""2). (3""2). (5""2) •...
n
· 123135
i.e.,). = IO m, 5 m, (10/3) m and so on. I. ~ t = O. 2/A • 2/A • 2/A •... or 41 ' 41A • 41A
A
Note: As here the detector of sound is ear which responds to pressure SO time interval between two consecutive beats
variations and so in this problem we have considered
interference of sound as pressure waves (and not displacement lit = In - tn_I"" (l/2fA )
waves) which do not suffer a change in phase 'when refleeted So beatjrequency, i.e.. numberofbeats per sec will be
from a 'rigid boundary' (but suffers a phase change cflt when
reflected from a 'free end' as in organ pipes), This is in contrast 1 2(/1 - 1,)
to the reflection of displacement waves which suffers a phase
Ib =M =2/A = 2 =(/, - 1,)
change of1t from a rigid boundary and no change in phase from a
'free end'. So due to interference where we get pressure maxima,
i.e .. beat frequency is equal to the difference 01
there will be displacement minima and vice-versa, j.e., if the frequencies of two interfering waves.
detector instead of ear is displacement sensor, or wave, light
instead of sound, at the same position we shall get displacement
,,1 •

• t. 1s
minima for the above mentioned wavelengths. •
! . 1•
Wave 9' amplitude ~ and frequer;ICY 17
§ 13.4 Beats •

•• •

When two sound waves of nearly equal (but not exactly
equal) frequencies travel in same direction, at a given point
,,1 ,• :•
:•
'•
i t- 1s

due to their superposition, intensity alternately increases and :
• • • •
decreases periodically. This periodic waxing and waning 01 : (b)
••
Wave?, amplitude ~ and Irequ~ 19
, , ,
sound at a given position ;s called beats. •• •• •• •

• : I :
Consider two sound waves of frequencies 11 and 12 •

:•
propagating simultaneously in the same direction. For
simplicity if we assume that their amplitudes are equal, at
,1 •





,
, !· ' ,
t _1s
position x =0 "• , , , , "•••
Yl = Asin27if,1 and Y2 =A sin 21if21 i , I
:• •I
. C +sm
N ow as sm . D =2' sm (C + D) cos~
,,
(C:...,-,.:D=-c) •
2 2 (c) Resultant Wave of amplitude 2A and frequency 18
Fig. 13.14
y=2Acos21t(f1 ~ /2)tsin21t(f1 +/2 )t (4) Here we have assumed that AI = A2 == A
2 2
or y= 2A cos 21tIA tsin21ifav t .... (i) So that lmax oc(Amax)2 oc 4A2

with 1 =/, - 1, and 1 =/ , +1, ....(ii) and 1 min OC (Amin)2 = 0


A 2 av 2
i. e., lmax =4/0 and l min =0
or y = Ab sin 21tfav t with Ab =2A cos (21tIA t) .... (iii) However, if Al :t A21 then as in interference,
From this it is clear that:
lmax oc (AI +A2)2 and l min oc(A 1 - A 2 )2
(I) The resultant wave is a harmonic progressive wave of
frequency lav. i.e., (f, +/2 )12 and amplitude Ab
which is periodic in time. (Shown in Fig. 13.14 (c)] So that
1,= _[A, +A,]2 _[F. +F,j2
(2) As Ab = 2Acos21tIAt, amplitude 01 resultant wave
Innn - Al - A2 -.[i; - JI;
varies periodically as frequen cy; (5) Here the intensity of sound at a given point is not
IA =(/, - 1,)12«1,.[=(/, +I,)/2J constant but varies periodically with time, i.e.,
and as Icos 91max = I and Icos91min = 0 interference is not sustained. And as the persistence of
ear is about (1110) s, beats will be detected by the ear
IAmaxl=2A and IAmin l=O
only ifbeat frequency
{This is shown by dotted curve in Fig. 13.14 (c). ]
(3) A beat, that is maximum or minimum intensity, will
Ib = I, - fz < 10 Hz
occur when
So if fb > 10, beats produced will be > 10 but heard at which the amplitude ofresultant waves varies, (c)frequency
zero (as there will be continuous sound of intensity at which beats are produced? Find also the ratio ofmaximum
1 av = (l max + I min )/2 = II + 12 instead of waxing and minimum intensities of beats.
and waning of sound). Solution: Comparing the given wave equation with
(6) If II ~I, =1, IA =0 and I" =1 Y= A sin (rot - /a) = Asin ro[t - (xlv)] [as klro = lIv]
So that y=2Asin2nl[t - (xlv)], We find that here
i.e., y=2Asin(cot-kx) Al = 0.3 and ro l =21tf1 =596tt, i.e., f1 =298 Hz
i.e., the resultant wave will be of frequency f and and A2 = 0.5 and ro 2 = 21tf2 =60411:, i.e., f2 =302Hz
amplitude 2A and so I oc4A 2,i.e,1 =410 which is
So (a) The frequency of the resultant wave
constant and so beats will disappear and sound of
=11 + I,,, (298+302) =300Hz
uniform intensity will be produced. Musicians use I av 2 2
this in tuning their instruments. They sound their
instruments against a standard source and tune it until (b) The frequency at which amplitude of resultant wave
beats disappear (sound ofunifonn intensity is heard). varies:
In this situation the frequency of their instruments is (298 - 302) = 2 Hz
equal to that of standard source. 2
Note: While solving problems related 10 beals remember that: (c) The frequency at which beats are produced
(i) Ifb,,1 fr'qu,nor com" out to bo ""tio",1 do,', ""d it off Ib =21A = 11 - I, = 4Hz
to nearest integer as over a proper time interval this (d) The ratio of maximum to minimum intensities of beat
fractional frequency will convert into an integer, e.g., ifbeat
frequency is 4.2 Hz, then in 5 sec, 21 (and not 20) beats will
be produced and in lst, 2nd, 3rd and 4th sec 4 beats will be
[~ = (AI +A2 )' = (0.3+0.5)' =64=16
heard while in the 5th sec beats heard will be 5 (and not 4). ["" (A , _ A )' (0.3 _ 0.5)' 4
2
(ii) Tuning fork is a source of sound of single frequency and
Problem 10. A tuningfork A produces 4 beats with tuning
frequency ofa tuning fork ofann length L and thickness d in
the direction of vibration is given by fork B offrequency 256 Hz. When A is filed beats are found to
occur at shorter intervals. What was its original frequency?
.~... f=[:']v= :,~ ["v=~l Solution: As tuning fork A produces 4 beats with B of
where Y is the Young's modulus and p the density of the
frequency (fB =)256 Hz, the frequency of A, fA will be
material of the tuning fork. Furthennore laadingarwaxing a fA. =fB ±4=256±4, i.e., fA =252Hz or 260Hz
tuningfark increases its inertia (and not dimensions) and so
Now on filing due to decrease in inertia frequency of A
decreases itsfrequency whilefiling a tuning/ark decreases
its inertia and so increases its frequency. wiU increase and occurrence of beats at shorter duration means
increase in beat frequency; so if
Question III. As in sound, can also in light beats of two
light sources 'o f .nearly equal frequencies be observed? IA =252Hz IA =2601Iz
Explain your answer. 256 - I A =4Hz IA -256=4Hz
Answer: No. To obseti',~ beatt' by two light sources, at a and so with increase in fA and so with increase in fA
given point the phase difference between the sources should beat frequency will decreases. beat frequency will increase.
change regularly and slowly. As emission of light from atoms
and as on filing beats frequency increases the frequency of A
is a random and rapid phenomenon, the phase difference at a
before filing was 260 Hz.
given point due to two independent light sources will change
rapidly and randomly and instead of observing beats we shall Problem 11. A tuning fork of unknown frequency when
get uniform intensity, I av = (l max + I min )/2 = II + 12 , sounded with another offrequency 256Hz gives 4 beats per sec
and when loaded with a certain amount of wax it is again
Note: Like sound, the phenomenon of beats takes place in radio waves found to give 4 beats. Find the unknown frequency.
giving rise to modulation. In case of light too beats can be
observed if the light sources are LASER beams of nearly equal Solution: Let the frequency of unknown tuning fork be f.
frequencies as LASER gives coherent beam of light. As it gives 4 beats with another of frequency 256 Hz,
Problem 9. If two sound waves, YI =0.3sin5961t f=256 ± 4, i.e., f = 252Hz or 260Hz
[t - x/330]and yz = 0.5 sin 6041t[t -x/330]are superimposed, Now on loading due to increase in inertia frequency f will
what will be the (a )frequency ofresultant wave, (b)frequency decrease, so if
f =252Hz f = 260 Hz (b) As I oc 4A2 COs Z n(W 2 - WI )/, it will bc 2A2 when
256- f =4Hz f - 256=4Hz 2A2 = 4A2co~/ n(w2 - (0
1)1
and so with decrease in f beat and so with decrease in! beat
frequency will increases. frequency will decrease. i.e., cosn(w 2 -WI)t=±(lIJi)

From the above it is clear that iff = 252 Hz, on loading the or n(w 2 -Wl)t**=~, 31t, 5;, ...
beat frequency will increase and can never be 4 again. 4
However, if f = 260 Hz, on loading the beat frequency will I
i.e., t =- --''- ,
3
, 5
,...
decrease as long as f remains greater than 256 Hz. If due to 4(00 2 -WI) 4(00 2 - WI) 4(w 2 -WI)
loading its frequency becomes lesser than 256 Hz, the beat
SO time intcrval betwecn two succcssive positions for
frequency will increase again and will be 4 Hz when it has
decreased to a value (256 - 4) = 252 Hz. So the original ,
which intensity remains:S; 2A 2 as shown in Fig. 13.15,
frequency of unknown tuning fork is 260 Hz (and after loading
it becomes 252 Hz).
Problem 12. Two radio stations broadcast their
programmes at the same amplitude A and at slightly different
frequencies w I and 00 2 respectively, where 00 2 - WI = 10 3 Hz.
A detector receives the signals from the two stations
~
1
simultaneously. It can only d~fect signals of intensity> 2A 2 • 4
Fig. 13.t5
(a) Find the time-interval between sllccessive maxima of the
intensity of the signal received by the detector. (b) Find the
time for which the detector remains idle in each cycle of the
intensity of the signal. Problem 13. There are three sources of sound of equal
Solution: If the detcctor is at .. = 0, the two radio-waves at intensities with frequency 400,401 and 402 Hz. Wha! is !he
the sitc of detcctor in accordance with given conditions (i. e., beatfrequency heard if all are soullded simultancou:·;ly?
Al = A2 = A andf] =00 1 andh =(0 2 ) will bc Solution: As intensities are equal , amplitudes of waves
YI = A sin 21£wT I Y2 = A sin 21£w zt
and will be equal and if for simplicity wc consider the waves at
So by principle of superposition, °
x = with 401 = f, YI = Asin21t(f -1)t; Y2 = A sin2nJI and
1) = Asin2n(f +1)/
Y = y\ + yz = A sin 21£00 1t + A sin 21£00 2 t
. D = 2'
. C + Sill + D) cos (C - D) So by principle of superposition,
B ut as Sill sm (C
2 2 y=Asin2n(f -1)t + Asin2njt + Asin2n(f + 1)t
(ro - ro) (ro +ro ) Taking first and last terms together,
y =2Acos2n 2 1 tsin21£ 1 2 t
2 2 y =2A cos 21t:t sin 21£jt + A sin2n/f
(ro + ro ) (ro - ro) or y = A [2cos21tt + 11sin21tji
or y=A'sin21£ I 2 t with A' =2Acos21£ 2 I t
2 2 or y=:oA'sin2rift with A'=A[I +2cos21t:t]
So that .... (i) So / oc (A')z oc A2 (I + 2cos21tt)z .... (i)

(a) So / will bc maximum when For I to be max or min,


2 d/ e, .!L(I
COS n(002 - ool)t=max = l, i.e., cos1£(w z -w])t = ±1 dt =0, i
.. dt + 2eos2nt)2 = 0
or 1£(00 2 - (01)t = 0,n,2tt, .... , i. e., 2{l + 2eos 21Ct)(2sin 2nt) x 21£ = 0
i.e., t=O, [I/(ro, - ro,)J,[2/(ro2 -ro,)], ...
So time interval between two consecutive maxima (as
i.e. , either sin21£t == O or 1+ 2cos21£{ = °
So if 1 +2eos21C1 = 0, i.c., cos 21tt = (-l i2)
~hown in Fig. 13.13).
or 2nt*** =2nll± (2nI3) , .... ,\vithn=O, 1, 2.. .

.. In this problem the exa miner has used ro for f.


n 0" '" 2JTII ± 0 with 0 '" 1t/4 and (31t) /4. and n '" 0, 1,2, ..
n", 0" ", 2JTI1 ± Owithll"'0, I.Z, ...
it represents a wave. However, as it is not ofthe fonn
i. e., .... (ii)
F (ax ± bl), the wave is not travelling and so is called
standing or stationary wave.
and for these values of t [i.e.,cos2nt=(-1I2)1, I =0, i.e.,
intensity is minimum. (2) The amplitude of the wave
and if sin 2m = 0, i. e., 21tt = mt with n = 0, I, 2, ... A$ =2Acoskx
I 3 is not constant but varies periodically with position
i.e., t =0, 2,1,2'2, ... .... (iii) (and not with time as in beats). ~
J [from Eqn. (i)] will be (3) The pointsJorwhich amplitude is minimum are called
2 2 2 2 nodes and for these
9A ,A ,9A ,A " ..
coskx =0, i. e.,
i.e., intensity is maximum (with two different values).
So from Eqns. (ii) and (iii) it is clear that in one second we
get two minima (and two maxima of different intensities) and i.e.,
hence beat frequency (i.e., number of beats per sec) is two.
This all is shown graphically in Fig. 13.16. i.e., in a stationary wave nodes are equally spaced and the
spacing between two adjacent nodes is Q.12) with Amin =0.
Also from Egn. (i) for nodes, displacement y = 0 for all values
oftime (as A$ = 0), i.e., nodes are permanently at rest (though
they are not physically clamped).

I - >.12 -"""II-, '" -'''1'- >.12


2A N",...-- -----
o AN AN ......, N AN
/ Segment
X -",--
Fig. 13.16 3AJ4 A 5>.12
§ 13,5 Stationary Waves '" Fig. 13.17

When two waves of same frequency and amplitude travel (4) The points Jor which amplitude is maximum are
in opposite directions at same speed, their superposition gives called antinodes and for these,
rise to a new type of waves called stationary waves or coskx = ±I, i.e., kx=O, 1t', 21t', 31t', ...

x = O,~,~,3;,...
standing waves.
Let the two waves of same amplitude and frequency
i.e,
1t
[ask = 2A. J
travelling in opposite directions at same speed be i. e., like nodes, antinodes are also equally spaced with
YI = Asin(rot-kx) and Y2*= A sin (rot + kx) spacing (A.I2) and Amax = ±2A. Furthennore, nodes
Then by principle of superposition, and antinodes are alternate with spacing (A/4). This
all is shown graphically in Fig. 13.17. In the figure,
y= y, + Yz = A[sin (rot - kx) + sin (rot + kx)]
profiles of the stationary wave (i. e., curve between x
Nowassm
. C +sm
. D = 2'
sm
(C +
2
D)
cos
(C -2 D) and y for constant I) at instants t=O,t=(TI4) and
t = (3TI4) are shown.
So y = 2A cos kx sin rot (5) The nodes divide the medium into segments (or
or y=A$sinrot with A$=2Acoskx .... (i) loops). All the particles in a segment vibrate in same
phase, but in opposite phase with the particles in the
This is the required result and from this it is clear that: adjacent segment. Twice in one period all the particles
(1) As this equation satisfies the wave equation, pass through their mean position simultaneously with
2 2 maximum velocity (Ai))), the direction of motion
d y I d Y
-- =--- being reversed after each half cycle. This all is shown
dx 2 v 2 dt 2 in Fig. 13.18.

* If Y2 is the reflected displacement wave from a rigid boundary then it wil! bc [-A sin (rot + h)] as there is a phase change of"/(.
611

t __N~-r__~N__~~N~ in a segment (elastic PE + KE), always remains the


I It - 0
,) ,,
, ,, " T
"
:
- .-- , -- --- ----- , - - - -- ----- , __ _ _ oJ t- 4
,, , ,
, ,, ,,
,) r---~:---L--_+:--~---4:---j l a ~
, , , Elastic PE. max _ E
, , J 3T
-. - - , --------.- ,----- -- - -- - , - -- - , t - 4'" Kinetic energy. min '"' 0
,,, ,,, ""
"" la)
, ,
, , " I_T
t v _ max_A$ro
:
: 5T
---- .---------- ---------- , . --- ~ t := "4
~
,
• _ ___ 4,_ _ ___ ___ ___ ,__ _____ _ _ __ , ____ _
~

x_o }J4 3}J4 j}J4 Elastic PE _ min _ 0


Ag. 13.18 Kinetic energy. max '"' E
Ib)
(6) Standing waves can be transverse or longillldinal. Fig. 13.20
e.g., in strings (under tension) if reflected wave exists,
the waves are transverse-stationary, while in organ (8) In standing wave if the amplitudes of component
pipes waves are longitudinal-stationary. In case of waves are not equal, then asA min ",O(Y)N = Amin
longitudinal waves as pressure and displacement sin rot, i.e., node will not be pennanently at rest and so
waves have a phase difference of (1d2) [§ 12.3 (C) some energy will pass aeross the node and the wave
(2»), at nodes where displacement is min pressure will will be partially standing. (Fig. 13.2 1).
be max while at antinodes where displacement is max
pressure will be min, i.e., in case of longitudinal- IA'~,
stationary waves, nodes are points of max pressure
(min displacement) while antinodes of minimum
~ Flg. 13.21
pressure (max displacement).
In such situations we estimate the extent to which the
2A Displacement wave resultant wave is standing by the tenn sta nding wave
ratio defined as
SWR "" _A_
= _, =~(:.:A-'-t~+~A:!2~)
Amin (A I - A2 )
[as AmaJ: = At +A 2 and Amin = AI - Az l
So that for a progressive wave SWR (min) "" I as =
, ... _--_ ... ,
Pressure wave
-, A2 = 0) while for perfectly standing wave SWR =
(max) = 00 (as AI = A Z)' T he value of SWR for all
other waves will lie between these limits (i. e. , I and (0).
Fig. 13.19 Question rv. In what respect does a stalionary wave
(7) As in stationary waves nodes are pennanently at rest, differ from a progressive wave?
so no energy can be transmitted across them, i.e., Answer: Following are the basie differences between a
energy of one region (segment) is confined in that stationary and a progressive wave:
region. However, this energy oscillates between (I) A stationary wave does not transmit energy from one
elastic potential energy and kinetic energy of the part of space to the other while a progressive wave .
particles of the medium. When all the particles are at docs.
their extreme positions K.E is minimum while elastic
(2) In a stationary wave different particles (in a loop)
PE is max [as shown in Fig. 13.20 (a)J and when all
have different amplitudes (between 0 and 2A) but
the particles (simultaneously) pass through their mean
sarno phase relative to each other while in a
position KE will be maximum while elastic PE
progressive wave all particles have same amplitude A
minimum [Fig. 13.20 (b)] . The total energy confined
612 PHYSICS FOR COMPETITIONS- Vol. I

but different phase relative to each other (in a where x andy are in em and t in sec. (a) What is the maximum
wavelength span). displacement al x = 5 cm? (b) Where are the nodes located
(3) In a stationary wave all particles pass through their along the string? (e) What is the velocity of the particle at
mean position simultaneously but with different x ~ 7.5 em and t = 0.25 s? (d) Write down the equations of
component waves whose superposition gives the above wave.
speeds (A sOl), while in a progressive wave all particles
pass through their mean position successively but Solution: (a) For x =5. Y = 4sin (51t115) cos (96m)
with same speed (Aro). or y=2.ficos(961tt)
(4) Stationary waves, either transverse or longitudinal, So y will be max. when
are represented by nodes and antinodes with spacing
cos(961tt)=max. = 1, i.e., (Ymax)r-=-5 =2J3cm
(JJ2) between two consecutive nodes or antinodes
while progressive waves, either transverse or (b) At nodes amplitude of wave is zero,

4sin[~] = 0 ~ =0.1t,21t.31t ....


longitudinal, ere represented by crests and troughs
with spacing Q.) between two consecutive crests or i.e.. or
troughs.
Sox = 0, 15.30.45,60cm [asicngthofstring=60cmJ
(5) In stationary waves nodes arc permanently at rest
(c) As y= 4sin (m/IS)cos (961tt)
while in progressive waves no point is pennanently al
rest. dy =
dl
-4Sin[~]sin
15
(961(t) x (96Jt)
Problem 14. FiJI in the blanks: (a) The standing wave,
y = 2A sin kx cosrot in a closed organ pipe is the resultant oJthe So thc velocity of the particle atx = 7.5 cm and t =0.25 s,
superposition of y , ::::: Asin(<ot - lex) and Y2 = ... v pa = - 3841tsin (7 .5n:1IS)sin (96n:10.2S)
(b) Sound waves offrequency 660 Hzfall normally on a i.e. , vpa =-384nx l xO= 0
peifectly reflecting wall. The shortest distance from the wall at (d) As 2sinAcos B = sin(A+B)+sin(A -B)
which the air particles have maximum amplitude of vibration
is ... m(v =330mls). So y=4sin[~}OS(961t1)
Solution: (a) As 2sin A cos B =sin (A + B) +sin (A - B)
so y =2A sin kx cos <ot = A sin (/0: +(0/) + A sin (kx - <ot) +n( ~ +96.1 )+ Sin(~ -96nt)]
Now as by principle of superposition,
y=YI +Y2 with y, = Asin(rot-kx)=-Asin(kx - w/) or y = 2sin[961t1 + ~] -2Sinl961tt - ~ ]
so Y2 =Y-YI = Asin(rol+ kx) +2Asin (/o:-ro/)
i.e., Y2 ::=Asin(rot+kx) - 2Asin(rot-kx) [as sin (-a) = -sin 9]

Note: Usual answer to this problem is Y2 = -A sin(wt + b~ No doubt i.e., y= YI + Y2 with Y1 ::=2sin[961tt + ~]
this represents the reflected wave, but as explained above in the
opinion of au thor its superposition with YJ will not provide the
given resultant wave. and Y2 = - 2sin[961ti - ~]
(b) As the wall is rigid so at the wall there will be node Problem 16. A metallic rod of length 1 m is rigidly
(displacement). Now the point where amplitude is maximum is clamped at its mid-point. Longitudinal stationary waves are
an antinode, so minimum dislance from the wall where particle set lip in the rod ill such a way that there are two nodes on
displacement is maximum is the distance between a node and a eUher side of the mid-point. The amplitude of an aminode is
consecutive antinode. i.e., Q..14). But as here 2 x 10~6 m. Write the equation a/motion at tl point 2 cmfrom
v=330m/s and f=660Hz the mid-point and those of cOl/stituent waves in the rod.
i.e , ;\.=(v ll)=(3301660) =0.5oo (Y =2 x lO" N lm2 al/dp = 8 x 10 3 kgl m 3 )
the required distance is (1/4) (0.5) =0.125 m Solution: In rods, like strings, clamped point is a node
Problem 15. The vibrations of a string oj length 60 cm while the free end anti node; so the situation in accordance with
fIXed at both ends are represellted by the equation, given condition is as shown in Fig. 13.22.

y= 4Sill[~ }OS(961tt) 3>E3>doE3>c2...


x", Ll2
Fig. 13.22
Now as distance between two consecutive nodes is AI2 Problem 17. The following equations represent
while between a node and an antinode is A/4 transverse waves-

4X[~J+ 2m =L, i.e., A = 2;I =004m .... (i)


and
zl = Acos(kx-rot),
z3 "" Acos(kY - Olt)
22 ;;: Acos(kx+oot)

AlsoasY = 2xl0!1 N/m2 and p := 8x10 3 kg/m 3 IdentifY the cWlbination (a) of the waves which will
produce standing waves (b) a wave travelling in the direction

If
2xlOil making an angle 0/45 0 with the positive x and y-axis. In each
V= - = =5000m/s
casefind the position at which the resultant intensity is always
P 8 x 10 3
zero.
So fromv = j).,f = (viA) =(50001004) = 12500 Hz .... (ii)
Solution: (a) The standing waves are produced due to
Now ifincident and re~ected waves along the rod are Yl = superposition of two waves of equal amplitude and frequency
Asin(rot-kx) and Y2 =Asin(rot+kx+$), resultant wave travelling in opposite direction with same speed. Here
will be: amplitude, frequency and velocity of all waves are equal but z\
Y = YJ + Y2 = A[sin(rot-kx)+sin(rot+kx+$)] is travelling along positive x-axis, z2 along negative x-axis and
z) along positive y-axis, so superposition of z\ and z2 will
. C
Butassm +SIO sm (C + D) C08 ",(
. D = 2' -
C'-c-,.:D,-,)
produce stationary waves,
2 2

Y=2ACOS[kx + H+" +%] i.e., z;;o; z \ +z2 ;;o; Acos(cot - la)

But ascos C +cosD ;;O;2COS[ c; D


+ Acos(cot + kx)
}os[C; DJ
Now as free end of the rod is an antinode, i.e., amplitude is
max at x = 0, so that SO z ;;o; 2A cos kxcosoot
[ascos(--a) ;;o; cos9]
Now as amplitude of standing wave is (2A cos kx),
COS[k x o+~J =maX = l, i.e., ¢I=O 2 2
I = A , = 4A cos Ia
2

and A,ll3X =2A =2x 1O-6 m (given) So 1 will be zero ifcos 2 Ia = 0


So y=2 x 10- 6 cos kxsinwt i. e.,

0' y = 2 10 - 6 COS [~1t x


x ]sin 2nfl i. e.,

[aSk = ~1tandro = 21tf ] (b) Since the waves z\ and z) are travelling with same
speed, v = (colk) = fA along positivexand y-axis respectively.
Above equation in the light ofEqns. (i) and (ii) reduces to (v)x=(v)y' i.e. , (dxldt) = (dyldt) or y=x+C
y = 2 x 10- 6 COS 5m sin 2S0DOnt ... {iii) So the superposition of z \ and z) will give rise to a wave
propagating along y =x + C. i. e., in a direction 45" inclined to
Now as for a point 2 em from the mid-point positive x and y-axis. So the required wave.
x = (0.50 ±0.02),
z=zl + z) = Acos(cot - la)+ Acos(oot-ky)
y =2 x 10- 6 cos51t(O.50 ± 0.02) sin 250001tt
This is the required result.
But cos C +cos D =2COS [C ;D }o{ C ;D ]

Nowas 2cosAsinB =sin (A + B) - sin (A -8)


So z = 2Acosk (X; Y)cOS[rot-k (X;Y)J
the resultant wave y = 2 x 10- 6 cos (5ru:)sin (25000m) can be
written as As the amplitude of this wave
6
y:= IO - [sin (Sm + 250001[/) -sin (5ru - 250001[/)] A =2Acos k(x- y). 1 cccos2 k(x- y)
s 2' 2
i.e., y = 10- sin [250001[1 + 51tX] + 10-6 sin [250001tt - 51tX]
6

[as sin (-0) = - sin 8] So forI =0, cos 2 k(x-y) = 0


2
or y = Y! + Y2 with Y1 = 10-6 sin [25000nl + 51tX] . k(x - y) _ n 3n 5n _ n 3n 5n
l.e·,--2- - -2'2.2.··· or x- y -k. k ' k '····
and Y2 = 10-6 sin [25000rrl - 5m]
representing the required constituent waves. So [aSk = ~"J
Problem 18. Tlte displacement oflhe medilln! ;n a sound opposite directions at same speed, treating the incident wave
wave is given by the equatioll as superposition of two waves of amplirudes 0.8A and 0.2A ,
y= A cos (ax + bl) we get
where A, a allli b are positive conslanls. The wave;s reflected y =O. 8A(cos(bt + ax) - cos (bt - ax )] + O.2A cos (bt + ax)
by all obstacle situaled at .t :.:: 0. The intensity of the reflected But as cOSC - COsD=2sinC;DsinD~C
WO\lC is 0.64 limes that of the incident wave. (0) What is the
.wl/ve/eng/I! alldfrequency of the incident wave? (b) Write the y = -1. 6A sin axsin bt + O.2A cos (bt + ax)
equation jor the reflected wave. (e) In the resultant wave
formed ajier reflectioll, find the maximum and minimum i.e., y = )'Sla + YPro
values of the particle speed ill llle medium. (d) Express the with )Isla =- 1.6 sinaxsinbt and YPro =O.2Acos(bt+ax)
resultant waves as a SIlpel1Josilion of a stallding wave and a Now as antinodes arc points of maximum displacement in
travelling wave. Whaz are the positions o/the antinodes ofthe a stationary wave, so for antinodes,
standing wave? What is the direction of propagation of the sin(ax) = max =± !
travelling wave?
i. e.,
Solution: (a) Comparing the given equation with standard
equation of travelling wave, i. e., And as there is positive sign be/ore x in the equation 0/
y ::: Asin(rot + h+~) progressive wave, the wave is travelling along negative x-axis,
A. A,C». band k. a and $ • (./2) i.e., in the direction a/inCident wave.

w /:~ = ~ a~ . A=2~=?n
21t 21t k a Note: In this problcm AI ~ A , so the wave is not perrectly standi ng,
2
i.e. , here lit nodes(poin t o f minimum displacement), amp lilude is
(b) As in case of reflection of displacement wave fro m a
mi nimum (AI - A2 )but not zero. So nodes are not permancntly
hard bollI/dUly there is a phase change of nand as intens ity of at rest and energy will escape through thcm. This a ll is in
reflected wave is accordance wit h § 13.5 (8) wit h SWR '" 1.810.2 =9.
I A'
i.e. , ..L= 0.64 or _r =0.64 § 13.6 Vibrations of Strings
/0 A2
When a string capable of vibrating (i. e., is under tension)
or AI' = 0.8A is set into vibration, transverse hannonie waves will propagate
So the equation of reflected wave which will move along it. If the length of the string is finite, reflected wave will
opposite to incident wave will be also exist and so due to multiple reflections standing waves of
Y2 =O.8Acos (bt - ax t n) = -O.8A cos(bt - ax) large amplitude called 'resonant standing waves' will be
produced, i.e., the waves in a lallt string 0/finite length are
(c) In a wave the velocity of particles executing SHM is
transverse stationmy.
given by
The string will vibrate in such a way that fixed (i.e.,
v = wJA2 -l clamped) pOinls a/the SIring are nodes, as the string at thesc
In a standing wave amplitude varies from point to point, so points is not free to move while the point o/plucking or free
particle velocity will be Illax when amplitude is max (i.e., at end is all anlinode as here displacement will be maximum Fig.
ant inodc) and y= min = O,i.e. , particle is passing through its 13.23.
Fixed
mea n position so that e,.
(v AN )1J'I3X =roA nl3X = b(A +0.8A) = 1.8bA Free
And the partic le velocity wi ll be minimum whe n
y = max ::: A (whatever it is), i.e., at extreme position and w
'" ---- --------
2 2
vlll,in =ro..JA _A = 0
(a) (b)
Note: In this problcm as amplitude is different at different positions so Fig. 13.23
'J'rL1X is diffcrent at different positions, being max at antinodes
Now as minimum distance between two nodes is (A 12), so
where A;; Anli'l~ ;; A1 + A2 [so (v N )ma~ ::: 1.8bA] and min at
in a laut string fixed at both e nds only those standing waves
nodes where A ;; A min;; Al - A2 and so (v N )min '" O.2bA.
will be possible for which
(d) As stationary wave is. the result of superposition of 1- 2A 31-
L =- or L=- or L ="2''''
two waves of equal amplitude and frequency travelling in 2 2
Antinodes Freque- Wavel-
Harmonic Mode Nodes
or loops ocy ength
First Fundamental 1 2 / (2L1 1)
I-Harmonic II-Harmonic Second Iovert(\nc 2 3 2/ (2L12)
Third If ovcl'hmc 3 4 3/ (2L13)
la) Ib)
.... .... .. .. ... .... ....
N N.... ___ N.--N nth (n-I)th (n+l) '1/ (2LI ,)
, overtone "
I---L- -o-I Regarding frequency of a vibrating string it is worth
noting that:
III-Harmonic
(I) As a string has many natural frequencies (all integral
Ie) multiples of fundamental frequency), so when it is
Fig. 13.24 excited with a tuning fork (or a vibrating body), the
nA 2L. string will be in resonance with the given body if any
i.e., L=-
2 or A. /I =n- WIth n = 1,2,3, ... .... (i) one of its natural frequencies coincides with that of
the body.
i.e., in a string a/length L only those standing waves can exist
for which A. = (2L/I), (2L12), (2U3), ... and maximum possible (Z) AsforaS1ring/ = (lIZL)(~Tlm)
wavelength of standing wave is 2L (a) f oc (11 L) if T and m are constant
Now as in case of waves V= fA, i.e.,! = vi')... and for (b) f oc ..fi if Land m are constant
waves along a string v = .JCT/ m), the possible frequencies of
(c) foc(lI.Jm) jf TandLareconstant
vibration of the string in the light of Eqn. (i) will be:
These laws of vibration of string are known as
f!l=2~~ with n=1,2,3,... .... (ii) Mersenne's laws of vibration of string and according
to these the frequency of a string can be changed by
I.e. , in case of vibrations of strings, number of natural changing its length, tension or mass per unit length.
frequencies are possible and if we take (3) If M is the mass of a string of length L, m = (M I L)

Z~ ~, I, = nl
So 1 iT 1 IT tiT- .... (iv)
1= ....(iii) I = zL'{;;; = ZL'{(!iiL) = ::i'tML
i.e., I, =1, 12 =ZI, 13 =3/,··· Also if the radius of string is r and its density p,
So in case of vibrations of strings higher frequencies are m = 1tr2p.
integral multiples off [given by Egn. (iii)1, i.e., forms a
harmonic series. SO I=l..~ T,
2L 1tr2p
i.e., 1=_1
2Lr
/T .... (v)
V-;;p
The frequency will be minimum when n = min = I,i.e.,
(4) If the string is vibrating in nth hannonic, its frequency
I
nun
=/\=I=l.. iT
2L V;;
will be nf, the number of loops in the string or
antinodes will be n, while total number of nodes
In this situation the string will vibrate in one loop with (including two at the ends) will be (n + I), e.g., in case
nodes at the ends and antinode in the middle as shown in Fig. of 3rd hannonic of a vibrating string, frequency of
13.24 (a). The lowest possible frequency with which a string vibration will be 3f, antinodes will be 3 while total
can vibrate is called its fundamental frequency and nodes = 3 + 1= 4 and A = (2Il3).
corresponding mode ofvibration* ,fundamental mode orfirst (5) In case of vibrations of composite string (i.e." string
harmonic. The higher possible frequencies are called made up by joining two strings of different lengths,
cross-sections and densities) having same tension
overtones with f2 being second hannonic or first overtone and
through out, the joint is a node while lowest common
f3 third hannonic or second overtone. This all is given below
fundamental frequency of the string will be
in a tabular form:
fe =n,f, =n2f2 .... (vi)
Here higher hannonics will be integral multiples of
common frequency f c'
.. Standing wave panern.
616 PHYSICS FOR COMPETITIONS - V ol. I

Note: While solving the problems related 10 vibrm ions of strings. keep
fund amental freque ncies in the ratio of 1 : 3 : 4, the vibrating
in mind: length should be in the ratio 1 : (1/3): (1 /4), i.e.,
(i) T he mode of vi bmlioo is taken 10 be fundamental unless L\ :1-,: 1-, :: 12: 4 :3
sin ted otherwise. FUr1 hcr vibrations of rods or siring free at
If common fac tor is x.
Olle end can Ix dealt similarly by tak ing the point ufplucking
o r free end as an anti nodc wh ile POill1 of tOl/ching or Ilx + 4x + 3x = 114 = length of the string
cl::nnping as a node. i.e., 19x = 114 or x =6
(ii) 111l€l'val means the ralio of two frequencies (and not their
So LI =t 2x 6 = 72cm, L,.,. =4x6 =24cm
difference). If the interval is I, h '" ii. i.e., the two vibrating
bodies are said 10 he illllnison. Ifin'eIVal is 2, 12 is said to be and L3 =3x6= 18em
(II/ nc/ave h'gfiern nd so if interval is 2" '/2 will be II-octave Problem 20. A sonometer wire fixed at aile elld has a
higher. Similtlrly /I-octave lower means 12 ~ fil2" ,e.g., solid mass M hm:gingfi"om its other end to produce tensiOIl ill
3-octavc higher Illc:ms 12 ", 23Ii = 8Jj while 3-octave it. It is fOl/lld tliat 70 em length of the wire produces u certain
3 fimdamcntalfrequency when plucked. When the seme mass M
tower means .12 '" .Ii /2 c fi 18.
is hanging ill water completely submerged ill it, it is found thm
(iii) Tension in II wire (sIri ng) may be due to differe nt ca\lse ~. Ifi l the length o/the wire 11m· to be changed by5cm in order that if
is due to a suspendC11 mass M,T == Mg and if the suspended
mass of d~nsity p is submerged in a liquid of density 0",
will produce the same ji/l1damelllal frequency. Calculate the
T '" /JIg'wi thg' = g{I - (a/p) ].lfT is due to elasticity of wire density of the material of mass M.
T '" YA(6UL){ns Y '" TL I A6L) and if strain (AUL) ;s due 10 Solution: In Ihis problem ten~ion is provided by the
ihenna l expansion T "" YAa 60(a5 a =t.L IL MJ.,.
suspended weight , i. e., T = Mg,
Question V. Will it be possible 10 monitor the
temperature ofa wire by measuril1g its vibrational frequency ?
so 1= I
2x O.70
~Mg
m
.... (i)
Explain. Whcn the suspended mass is submcrged in water
Answer: Yes, In case of a wire suspended vertically from
a rigid support and loaded at the free end, the frequency of the T'= Mg' = Mg[l-~1 L
wire will be given by o 0
and as tcnsion dccreases, to L_-"-_ _ C=--__
1 =£
2L
IT =E~
V;; T
2 ML produce same frequcncy length of
wire must also be decreascd, i.e.,
Now for a given wire, mass of wire and tension in it L' = 0.70 - 0.05 = 0.65 m. r.;-,;----;--;-;-; Fig. 13.25
rcmains constant so
I Mg[l- (a l p)]
K .... (ii)
with 1 =2xO.65 m
1 =.JL
Equating value off from Eqns. (i) and (ii) and simplifying,
61 1 6L
1= - 2L 0.65]2 =
[ 0.70
l -~orp = 196 x a = 7.26x 10 3 kg/m}
P 27
Now as with change in temperature change in length is
givcn by Problem 21. The Jimdamental frequency of a SOl/ometer
wire inCreases by 6 Hz if its tension is jllcreased by 44%
6L = La~e rasa. =.6.LlL .19]
keeping the length COllstalll. Find the change in the
So N
- = - -I Cl.19 6e=-~ 61 fundamental freqllency oJlhe sonometer wire. whenlhe leng th
I 2 ' a I afthe wire is increased by 7.0% keeping the originaltellsion ill
$0 iftherc is fract iona l increase in frequency temperature the wire.
will decrease and vjce~versa, i.e., by measuring vibrational Solution: In case of vibration of a string, fun damental
frcqu~ncy we can monitor the temperature of the wire. frequen cy is given by
Problem t 9. A sonometer wire has a length of 114 cm
hetween two fixed ends. Where should two bridges be placed
I=~IT
2L v-;;;
[§ 13.6]
so (IS (() divide the wire into three segments whosefundumental
So if length of given wire is kept constant,
ji-equellcies are ill the ratio 1 : 3 : 4 ?
Solution: In case of a given wire under specific tension, (f'If) =(T' I T)1I2
fundamental frequency of vibration f 0:: (I/L). So for having and as here f' = f + 6 and T' = T + 0.44T "" 1.44T.
(f + 6) ~ ~1.44T i.e., f ~30Hz Further,
f T
Now if keeping the original tension (T), the length of .... (ii)
given wire is changed,
.... (iii)
/"
-~-~-
1 1 [as/"~/+0.20/~1.20~
f I" 1.20
Now, k cc.! (as k =ro!v, assuming that frequency of wave
/"~ 30 ~ 25Hz v
so
1.2 in both strings is same)
and hence I1f ~ f" -.r ~ 25 - 30 ~ -5 Hz
k2oc-
1 and k '" 1
i.e., fundamcnUiI frequency will decrease by 5 Hz. 32 1 SO

U2 - ~)
Problem 22. A long wire PQR is made by joining two
wires PQand QR ofequal radii. PQ has length 4.8 m and mass
O.06kg. QR has lellgth2.56m and mass 0.2 kg. The wire PQR is A, ~(l 1 )X3.5 ~1.5em
under a tellsion 0180 N. A sinusoidal wave-pulse of amplitude ~ + ~

3.5 em is sellt along the wire PQfrom the end P. No power is


80 32 ·
dissipated during the propagation of the wave-pulse. 1
2x--
Calculate: A ::: 80 x 3.5 =2.0cm Answer
(a) the time taken by the wave-pulse to reach the other
end R of the wire and
, (1 + 1)
80 32
(b) the amplitude of the reflected and transmitted Alternative solution of part (b): The energy transferred
wave-pulses after the incident wave-pulse cross the join! Q. by the wave-pulse through the wire is given by,
Solution: (a) Velocity of Q R
transverse wave in stretched P E=.!.(eo 2 m)A2 ::: 1. ).tIA 2 eo 2
4.Sm 2.S6m 2 2
string is given by, v = ~T!m. Fig. 13.26
where ).t : mass per unit length of string.
Foe PQ, T ~ SON, /II = 0.06 kglm The average rate of energy, i. e., average powcr transferred
4.S is,
and forQR, T=80N, III = 0.2 kg/m - E 1 2 2
2.56 P=-= - ).tvA (0
t 2

Velocity of wave in PQ, VI -


_~80X4.8 -- 80m!s The conservation of power at the junction Q gives,
0.06 Pi =Pr +p/
and velocity of wave in QR, v 2 -_~SOx2.56 --32 m!s I A2 2 1 A2 2 1 A 2eo2
2).t\v\ i eo ::: 2).t\V\ reo + '2).t2 v 2 I
0.2
:.Time taken by pulse to reach fr.om P to R, 2 2 2 .... (i)
or ).t\v\(A j -Ar )=).t2 v 2 A /
t = 4.8+2.56",,0.14s The division of amplitude at Q gives,
SO 32
Ai == Ar + At
(b) The equation ofincidenl wave is
Substituting for AI in' Eqn. (i), we have
Yj=Aisin(eot-k1x) with A i =3.5cm
The equation of transmitted and reflected wave at Q will ).I.\v\ (A? - A;) =).I.2v2 (Aj - Ar)2
be
or
Ai + Ar 1l2 V 2
.... (ii)
Y/ = A t sin(oot-k 2x) Ai - AI" Il\v\
and Yr =-Arsin(ro!+kix) The velocity of transverse wave-pUlse in a stretched string
{Here phase change of 1C takes place due to reflection at is given by,
denser media.]
At point Q, (Yj )x"' o = (Yt )x=o - (Yr ) x=o v~Jr
.... (i)
So, equation (ii) can be written as,
Ai + A, =
Ai Ar
t 2 = "0-0.2-0
flJ
1-=2-0.5-'
0.06/4.8
6
As the frequency of the
wave in both strings (A
vibrating with p loops and B
vibrating with q loops) must
O.75m
B

I + (A/Aj) be the same, so Fig. 13.27


or :-~:-'-': = 2.5
l-(A/A i )
AI' 3
- P~
2lA
-
m
=-
21BA
qtf -
mB
Solving, we get -=-
Aj 7 r;;; _fA (P;;
p _ fA
3 3 q-IB V-;;;-I;VPn
AI' ="7A( =7x3.5=1.5 em

and
= 0.3 ~6.3 =1
AI =0 Ai - AI' =3.5-1.5 0.75 2.8 5
=2.0cm Answer
So, p=3,q=5
Problem 23. A wire of density 9 x 10 3 kglm3 is stretched No. of antinodes = p + q
between two clamps 1 m apart and is subjected to an extension =3+5=8 Allswer
of 4.9 x 10-4 m. What will be the lowest frequency of
transverse vibrations in the wire? (Y = 9x 10 10 Nlm2) Note: (i) The frequency cannot be calculated as tension is not
provided.
Solution: In case of fundamental vibrations of a string, (ii) Total no. of nodes including the two at the ends will be = 9.

/=2~~=2~$ [asm=pA] Problem 25. A steel wire of length 1 m, mass 0.1 kg and
uniform cross-sectional area JO- 6 m 2 is rigidly fIXed at both
Here tension is due to elasticity of wire. ends. The temperature of the wire is lowered by 20°C. If the

So T =YA[M]
L
[as Y =
stram
stce~ = AM
TL 1 transverse waves are set up by plucking the string in the
middle, calculate the frequency of the fundamental mode oj
vibration. (Y = 2 x JOl l N lm2 ando. = 1.21 x JO- 51°C)
and hence, f=~~YAxI!.L=~l ~Y IlL Solution: In case of vibrations of string, when plucked at
2LpAL2LpL
the middle, fundamental mode of vibration will be produced;
Substituting the given values, so
/=_1_ 9xlOIO x4.9xlO-4
=35Hz Answer / = ~ iT
2xI · 9x103xI 2LV;
Problem 24. Two metallic strings A and B of different Here tension is produced by thennal stress, i. e.,
materials are connected in series forming a joint. The strings T = A (thennal stress) [as stress = TIA}
have similar cross-sectional area. The length ofA is IA =0.3m or T = YA (thennal strain) [as Y = stress/strain}
andthatofB is Is =0.75 m. One end ofthe combined string is
or T=YAa6.9 [asM=La ~e]
tied with a support rigidly and the other end is loaded with a
block ofmass m passing over africtionless pulley. Transverse So /=~~YAa ~e
waves are set up in the combined string using an external 2L m
source of variable frequency. Calculate (i) the lowest Substituting the given_values,
frequency for which standing waves are observed such that the
ll
joint is a node and (ii) the total number of antinodes at this Y =_1_ 2xJO xJO 6 x 1.21xJO 5 x20 =llHz
frequency. The densities of A and Bare 6.3 x 10 3 kg m -3 and 2xl (0.1/1)
2.8 x I 0-3 kg m -3 respectively.
Problem 26. An aluminium wire of cross-sectional area
Solution: The frequency of transverse waves in a 1 x 10- 6 m 2 is joined to a copper wire of the same cross-
stretched string is given by
section. This compound wire is stretched on a sonometer,
n= p iT pulled by a weight oflO kg. The tota/length of the compound
2IV;' wire between the two bridges is 1.5 m of which the aluminium
where the string is vibrating with p loops, T = tension in the wire is 0.6 m and the rest is the copper wire. Transverse
string and m = mass per unit length ofthe string. vibrations are set up in the wire by using an external force 0)
vuriable!requency. Find the lowest frequency ofexcitation/or would cause an occurrence of5 beats per sec, when both wires
which standing waves are fonl/ed, such that the join! in the vibrate together?
wire is (I node. What is the total number of nodes observed at Solution: As here tension IS changing, fundamenta l
this frequency excluding the two at the ends of the wire? The frequency
density of alumillillm j.~ 2.6 x 10 3 kg/fIl 3 and that of copper 1
1.0401 x 10 4 kg/m 3 .
1=-"-- {T =KJ'i with K=--
2L'r;;; 2L.r;;;
Solution: As the lolallength of the wire is 1.5 m and out "'I 1 AT AT 2Af
of wh ich LA == 0.6 m, so the length of copper wire j=2T' i.e., -=--
T 1
I.e = 1.5 - 0.6 = 0.9 m. The tension in the whole wire is same
=
(= Mg 109 N) and as fundamental frequency ofvibratioll of so AT =2>< _5- x100 = 2%
string is given by . T 500

{T d"T
1 Problem 28. A string 120 cm ill length sustains a standing
!=2Lf,;; =2L VPA [asm = pA] wave, with the points of string at which the displacement

So fA = ~ 1{p5
2LA
--
PA A
and le =_I-t 2Lc PeA
.. ..(i)
amplitude is equal to Ji mm being separated by 15.0 em. Find
the maximum displacem ent amplitude. Alsofind Ihe harmonic
corresponding 10 this wave.
Solution : From Fig. 13.29
Now as in case of composite wire, the whole wire will
vibrate with fundamental frequ ency points A,B,C,D and E are having
equal displacement amplitude.
j==/lAfA =ncfc .... (ii)
Further, x E -xA. =A = 4x J5=60cm Fig. 13.29
Substituting the values offA andfe from Eqn. (i) in (ii),
As ,- = 21 = 2 x I20=60
~ T ""~ T II n
2xO.6 Ax2.6x 10 3 2xO.9 Axl.040 lx104
1I = 2 x 120=4
"A 2 {2.6 2 I I 60
i.e.,
lie ""3ViOA =3 x '2=3 So, it corresponds to 4th hannonic.
So that for fundamental frequency of composite string, Also, distance of node from A = 7.5 cm as distance between
nA =1 and ne =3,i.e., aluminium string will vibrate in first Band C = IS cm and node is between them. Taking node at
hannonic (one loop), i.e., fundamental mode while copper origin, the amplitude of stationary wave can be written as,
string in thi rd harmonic (3 loops), i.e., II overtone as shown in a=Asinkx
Fig. 13.28.
Here a =-J2 mm· k",,21t :; 21t and x=7.5cm
, ), 60
AI
'I' CU 'IN
N N N N
Ji = A sin(21t x 7.5) = A sin 2!
60 4
<> <)
Hence, A ",,2 mm
Problem 29. Twoforks A and B when sounded together
:;t{~ produce 4 beal/s. Thefork A is in unison with 30cm length ofa
10 Kg sonometer wire and B is in unison with 25 em length of the
Fig. 13.28 .same wire at the same tellsion. Calculate thefrequellcy of the
I=IA =3/e forks. [MNR 1993]
This in tum implies that total number of nodes in the string Solution: As in case of vibrations of string under specific
will be 5 and so number of nodes excluding the nodes at the tension foc(IIL),i.e ../=(KIL). As forks A and B are in
ends",,5-2 = 3. unison (i.e., frequencies are equal) with 30 cm and 25 cm
length ofa given sonometer wire respectively,
and I=IA
1
=-2
06 IO x 9.8 = 161.8 Hz(=3/e )
x. 10 6x2.6x 103 K K . IA 25 .... ( i)
fA =30 and fa = 25' I. e., fs =30
Problem 27. Two identical sonometer wires have a
fundamental frequency of500 Hz when kept under the same Further as the forks A and B produce 4 beats when
tension. What fractional increase in the tension of one wire sounded together,
But from Eqn. (i) it is clear that fA < f B Substituting this value of / in Eqn. (ii), K = 10 which i[
So Is - I A =4 the light ofEqn. (i) implies
Solving Eqn. (i) and (ii) for fA aod/B ' we get It = 10
J....
2LV; or
fA = 20Hz and f8 =24 Hz
Problem 30. A siring under a tension of 129.6N produces Substituting the given data,
10 beats per sec when it is vibrated along with a tuning fork.
When the tension in the string is increased to 160 N, it sounds d= ----~~~----~~
(2 x 0.5)2 x 10 2 x xx (0.5 x 10 3)2
in unison with the same wning/ork. Calculate the fundamental
frequency ofthe tuningfork. [MNR 1994) = 40 x 10 3 kglm 3 = 12.74 x \0 3 kg/m 3
Solution: As here the tension in the wire is changed, so its •
fundamental frequency § 13.7 Vibrations of Organ Pipes
Is = J.. {T =KJ'i with K=_I_ An organ pipe is a cylindrical tubc of uniforn
2L V;'; 2L..{; cross-section in which a gas (usually air) is trapped as !
column (1". e., bounded medium). One end 0/ an organ pipe i.
Now with increase in tension, K..fi will increase and
always open while the other may be closed or open giving ris.
beats are decreasing to zero when T = 160 N (as unison means to closed end or opel! end organ pipe respectively. (Ifboth tho
the frequencies are equal). ends of an organ pipe are closed it becomes a cavity.)
I - K,J129.6=10 .... (i) As an organ pipe contains gas (air), the waves in it ar.
and I - K,J160 =0 .... (ii) longitudinal. And as the medium is bounded the incident wav.
travelling along the axis ofthe tube will be reflected back fron
Substituting the va lue of K from second equation in fi rst,
the other end resulting in stationary waves, i.e., file mecllanica
I - I l29.6 = 10 0' 1-0.91 = 10, i.e., I = 100 Hz waves in all organ pipe are always longitudinal-stationary.
160 Now if in an organ pipe reflection is from closed ent
Problem 31. A metal wire ofdiameter I mm is held on two which is rigid (i.e., hard boundary), the closed end wi ll be :
knife edges separated by a distance of50cm. The tension in the node as particles at node are permanently at rest and i
wire is 100 N. The wire vibrating with its /undamentd reflection is from open end (i. e., free end or soft boundary
frequency and a vibrating tuning/ark togetherproduce 5 beats where particles are free to move, there will be an antinodE
per sec. The tension in the wire is then reduced to 81 N. When i.e.,in an organ pipe, closed end is a/ways a node whilefre
the two are excited, beats are heard again at the same rate. end an antinode.
Calclilate (a) the frequency a/the lark and (b) the density 0/ [AJ Closed End Organ Pipe
the material of the wire. Tn case of closed end organ pipe, as closed end will alway
Solution: As here only tcnsion in the vibrating string is be node while free end antinode and minimum distanc
changed, so betwecn a node and antinode is QJ4). only those longitudinal
stationary waves can exist for which length of the pipe
f = J.. {T =KJ'i with K=_l_ .... (i)
s 2L V;;; 2LJ; L =1:. L=3< L=5< ...
4 4 4
Now ifthe frequency of tuning fork is/, then accordi ng to
given problem, i.e., ). =4L ). = 4L ). = 4L ...
1 3 5
I-KJ'i=5 0' KJ'i-1=5
f - K..JT = 5
If on decreasing the tension, beats will
increase and can never be 5 again. However, if K..fi - / = 5 IE L L
with decrease in tension, beats will decrease, become zero (b)
(al
(when K..fi = f) and will increase whenK..fi become< /. So

1pc:>C
according to given problem,
K.flOO - 1=5 and I-K.J8i=5
i.e., 10K =I+5 and 9K = 1-5 .... (ii) L

1+5 IO (el
So
1 - 5 =9" 0' I =95Hz Fig. 13.30
This is shown in Fig. 13.30. Now as v= JI..-, the frequency (1) Maximum possible wavelength is 2L.
of vibration of air column in the pipe will be (2) Fundamental frequency is fo(=vI2L) and higher
I, =....!:..
4L
or 12 =4L
3v 13 = 4L
or 5v
'"
harmonics are all integral multiples offundamental
frequency. So here like strings fixed at both ends first
So if I, =lc (=vI4L); 12 =3/c 0' 13 =5Ic '" hannonic is fundamental, second harmonic will be
first overtone and so on.
i. e., in case of closed end organ pipe:
(3) The mode of vibration of open end organ pipes are
(1) Maximum possible wavelength is 4L.
identical with that of a rod clamped in the middle as
(2) Fundamental frequency is f c (= vI4L) and higher shown in Fig. 13.33.
harmonics are only odd multiples of fundamental -g -g
frequency, i.e., even hannonics arc absent. So first • =:::><:::= •
harmonic is fundamental, third harmonic will be first
overtone (as II hannonic is absent) and so on.
! l !
-g Air - co!umn

ilk
5 -
"0

8-~
IE
Clamped

Rod
Free end
{CJ Discussion
(a)
Open end organ pipe
Fig. 13.33
(b)
Rod clamped at the middle

(a) (b) From all this it is clear that:


Fig. 13.31 (I) In case of closed end organ pipe higher hannonics are
(3) The modes a/vibration of closed end organ pipe are odd integral multiples of fundamental frequency Uc)
identical to that of a rod clamped at one end with the while for open end all integral multiples of
difference that in organ pipes waves are always fundamental frequency If,), 1
longitudinal while in rods these can be longitudinal or
transverse.
{BI Open End Organ Pipe
L L
In case of open-end organ pipe at both ends there will be
antinodes and as minimum distance between two consecutive
antinodes can be Q.,/2), only those stationary waves will exist
for which the length of the pipe
j j
), ), fc .d fo" 2f
L =- or L = 2- oc
2 2 (a) (b)
Fig. 13.34
i.e., ),=2L
I (2) Asic = (vI4L) and I, = (vI2L)
So for identical organ pipes but one with one end
closed while the other open.
L L
I
o
=~
2L
=2(~)=2/c
4L
(a) (b)
i.e., if a closed organ pipe is converted into open its
frequency will be doubled and if open is converted
into closed its frequency will be halved.
L
(3) If an open pipe is half submerged in water, it will
(c) become a closed organ pipe oflength half that of open
Fig. 13.32 pipe as shown in Fig. 13.35. So its frequency will
This is shown in Fig. 13.32. Now as v = f')..., the frequency become,
of vibration of air column in the pipe will be
v ~ ~
I, = 2L 12 = 2L I, = 2L '"
i.e., equal to that of open pipe, i.e., frequency will
Soif I , =lo(=vI2L); 12 =2/, 13 =3/ 0
,,, remain unchanged.
i.e .. in case of open end organ pipe: (4) Due to finite momentum of air molecules in organ
pipes reflection takes place not exactly at open end
but somewhat above it; so in an organ pipe antinode is i.e, the frequency of an organ pipe will remain unchanged if
not fonned exactly at free end but slightly above it, the ratio of speed of sound in it to its length remains constant.
Le., at a distance e = O.6r (called end correction or [See Question X]
Rayleigh correction) with r being the radius of pipe. Question VI. Discuss the reflection of longitudinal,
So for closed organ pipe L ~ L +O.6rwhile for open pressure wave from closed and open end in an organ pipe with
pipe L ~ L + 2 x 0.6 r (as both ends are open) so that
emphasis on change in phase if any.
Answer: In case of reflection from a rigid boundary when
Ie = 4(L:O.6r)
a longitudinal compression reaches the boundary, it will press
the boundary with maximum pressure and will be reflected
back as compression, as the boundary is not free to move. This
1L in tum implies that in case ofreflection ofpressure waves from
a rigid boundary the boundary will be (displacement) node
and there wU/ be no change in phase.
Now if the reflection of sound is from an open end the
pressure at the end will be equal to atmospheric pressure and
will not change unless the pressure of the whole surrounding
(a) (b) atmosphere is changed.
Fig. 13.35 Therefore, pressure at the open end always remainspo
and f = v (atmospheric pressure), i.e., minimum, i.e., it is a
o 2(L+1.2r) (displacement) antinode. The attempt by the wave to compress
This is why for a given v and L narrower the pipe the air at the open end causes rarefaction, which travels back in
opposite direction. Thus, longitudinal pressure wave ;s
higher will the frequency or pitch and shriller will be
the sound. reflected from an open end with a phase change of1t
Question VII. Where will a person hear maximum sound
(5) For an organ pipe (closed or open) if v = constant,
at (displacement) node or antinode?
loc! Answer: Perception of sound is due to pressure variations
I-
and as at node displacement is minimum, pressure will be
i.e., loc -LI [asA. ce L] maximum while at antinode as displacement is maximum,
pressure will be minimum. So sound will be maximum at
So with decrease in length of vibrating air column, displacement nodes (which is actually pressure-antinode).
i. e., wavelength (A. cc L), frequency or pitch will Question VIII. Two organ pipes of same length open at
increase and vice-versa. This is why the pitch both ends produce sound of different pitch if their radii are
increases gradually as an empty vessel fills slowly. different. Why?
(6) As for a given length of organ pipe')." = constant Answer: Due to end correction, the frequency of an open
I ocv organ pipe depends on its radius according to the relation,
So (a) With rise in temperature as velocity will increase
1= v
(v oc Jl\ the pitch will increase. [Change in 2[L + L2rJ
length with temperature is not considered unless So the pitch will be different for different radii and
stated.] narrower the pipe higher will be the pitch.
(b) With change in gas in the pipe asvwill change, so Question IX. Explain clearly why the quality of sound
f will change (v oc ~rl M). from an open pipe is different than from a closed pipe ofsame
(c) With increase in moisture as v will increase, so fundamental frequency.
the pitch will also increase. Answer: Quality of sound basically depends on number
(7) For an organ pipe if f = constant of overtones present. In case of closed organ pipe overtones
are only odd multiples of fundamental frequency while in case
vocl-
of open pipe they are all integral multiple of fundamental
or vocL [.sv= II-J frequency. So the number of overtones in open pipe are more
f v
= - = constant than in closed pipe and so the quality of sound in the two pipes
I- will be different.
Question X. Can we ever construct an organ pipe whose (c) What are the maximllm and minimum pressure at the
frequency does not change with temperature? If ftO, why? If open end of the pipe?
yes, under what condition? (d) What are tlte maximum and minimum prC'ssure at the
Answer: In case of an organ pipe f ocvlL So freq uency closed end ofthe pipe?
will not change witb temperature, if vlL is independent of Solution: (a) In case of closed organ pipe as fundamental
temperature. frequency is (vf4L) and only odd harmonics are present,
Now as v and L both depend on temperature, so f will be second overtone will mean fi fth harmonic and so
independent of temperature if
VI v V L2
f = ~~ = 440 Hz (given)
- =-2 or -2= -
LJ L2 VI L, L = 5x330 =!1 m
and hence
oc .,fi and 4x440 16
Now as V La = L.(I +a9)
(b) In tenns of pressure as at the position of displacement
So antinode there is pressure node and vic~versa, the variation of
273 + 0, 1+«9, pressure amplitude of standing pressure waves along the
length of the column with x = 0 at its open end will be
Simplifying the above equation with the help of binomial
theorem. we get P =L\posinla = L\posin(~1tx) (~k =~1t)
1+! (9, - 9,) =1 +<1(9, - 9,), i.e., <1 = (11546) Fe
2 273 Now as for second overtone L = (5/4)?.., so at the middle,

..
So the frequency of organ pipe will be independent of L 5
x =- =-A.
temperature varia/ions iflhe coefficient a/linear expansion of 2 8
its material is (IIS46)/°C.
A N
,
1
Problem 32. A tuning/ark having!requency 0/340 Hz is
vibratedjust above a cylindrical tube. The height ofthe tube is
120cm. Water is slowly poured in. What is the minimum Iteight
=
I L
LI2
!

~N I
of water requiredfor resonance? (v 340mls)
Solution: As the tuning fork is in resonance with air
column in the pipe closed at one end,

f=n:L with n=I ,3,5, ... Displacement wave pattern Pressure wave pattem
(al (b)
So length of air column in the pipe Fig. 13.36

p =llposin~{~). )=/lposin(%1t)
L = nu =n340 x I00 = 25ncm with n = I ,3,5, ...
4f 4 x 340 and hence

i.e., L=25cm , 7Scm, 125 cm, ...


i.e., !>Po
Ipl =Apo x1- = - Answer
Now as the tube is 120 cm, so length of air column must be
lesser than 120 cm, i.e., it can be only 25 cm or 75 cm. Further Ji Ji
if h is the height of water filled in the tube, (e) For free end as x= O, p ~ O,j. e., the amplitude of
L +h= 120cm or h= 120 -L pressure wave is zero (as it is a node), so
So h will be minimum when L = max = 75 Pmax = Pmin = Po ± O= Po Answer
(d) For closed endx + (5 /4))., so the amplitude of pressure
(h) min = 120- 75 = 45cm Answer
wave
Problem 33. The air column in a pipe closed at one end is
made to vibrate in its second overtone by a tuning fork of
frequency 440 Hz. The speed of sound in air is 330 ms - 1. End
IP I= I~p,Sin ~'(%A )1 =!>p.
corrections may be neglected. Lei Po denote the mean so P rnax =Po + llpo =
and Pmin Po - Ilpo Answer
pressure at any point in the pipe and IlPO the maximum Problem 34. A tube ofcertain diameter alld length 48 cm
amplitude ofpressure variation. is open at both ends. ltsfundamentalfreqllellcy ofresonance is
(a) Find the length L of the air column. foun d to be 320 Hz. The velocity of sound in air is 320 mls.
(b) What is the amplitude of pressure variation at the Estimate the diam eter of the tube. One end of tlte wbe is /lOW
middle of the column? closed. Calculate the frequency ofresonance for the tube.
Solution: Due to end correction the frequency of an open tuningfork. Another pipe of the same length but open at both
end organ pipe is given by, ends is filled with air and it also resonates in its fundamental
with the same tWlingfork. Calculate the velocity of sound at
/, _ v = v [ase=O.6r] WC in the gas. Given that the velocity ofsound in air is360mls
0 - 2[L+2e] 2[L+2xO.6r]
at 30°C where the experiment is peifonned.
So substituting the given data,
Solution: If f is the frequency of tuning fork then
320 = 320 x 100 or r:!Q cm according to given problem at 30"C,
2[48 + t.2r] 6
1=lc and 1=/0
So d = 2r=2x(10/6)=3.J3 em
VG VA
Now when one end is closed, i. e, Ic =10 and -
4L
=-
2L
/. - v = 320x 100 = 163.3 Hz i.e., v G =2v A ",,2x360=720mls
c - 4(L + 0.6r) 4[48 +0.6x (10/6)]
Now as in case of sound v oc JT
Problem 35. For a certain organ pipe, three successive
273+30 - l03
resonance frequencies are observed at 425,595 and 765 Hz so -(vG)T
--_ -
(vo)O
-- - -273
273
respectively. Taking the speed ofsound in air to be 340 mls. (a)
Explain whether the pipe is closed at one end or open at both
ends. (b) Determine the fundamelltal frequency and length of i. e., (vG)o = (vG )TJ273 =720 x 0.95 = 684 mI,
303
Ihepipe. IMNR 19911
Solution: (a) The given frequencies arc in the ratio Problem 38. A 8 is a cylinder of length 1 m fitted with a
,hinflexible diaphragm C at middle and two other thinjlexible
425: 595: 765, i.e., 5: 7: 9
diaphragms A and B at the ends. The portions AC and BC
and as clearly these are odd integers so the given pipe is closed contain hydrogen ali(I oxygell gases respectively. The
at one end. diaphragms A and B are set illta vibrations of the same
(b) From part (a) it is clear that the frequency of 5th frequency. What is tlte minimum frequency of these vibrations
hannonic (which is 2nd overtone) is 425 Hz. for which diaphragm C is a node? Under the conditions ofthe
So 425=5fe or fe =85Hz experiment the velocity of sound in hydrogen is 1100 ntIs and
oxygell300 mls.
Further as fe=....!!...' L'i=~= 340 = lm
4L 4/c 4x85 Solution: As diaphragm C is a A C B
node, A and B will be antinodes (as in
Problem 36. The first overtone of an open organ pipe an organ pipe either both ends are H2 O2
beals with the first overtone ofa closed organ pipe with a beat antinodc or one end node and thc other
frequellcy of2.2 Hz. The fundamelllal frequency of the closed antinode), i.e., each part will behave as
orgall pipe is 110Hz. Filld the lengths ofthe pipe. Fig. 13.37
a closed end organ pipe so that
Solution: For closed organ pipe fe = (211-1)~ with v 1100
4Lc 1/1 =H- = - - =550Hz
4LH 4 x 0.5
n = I,2, ...
vo 330
While for open organ pipe fa =n~ with 11 = 1,2, ...
And 10 = - = ~- = 150Hz
4Lo 4 x 0.5
2Lo
As the two fundamental frequencics are different, th(
According to given problem
system will vibrate with a common frequency f c such that
v
fe = - - = 110
. 330
l.e. , Lc =-~=0.75m
4Le 4xl1O fe = II Hf lf =lIofo

Furthcnnore i.e.,
liN
- =- fo
= -ISO = -3
110 fll 550 11
~-~=22mm=m~22(M~=~=m)
4Le 2Lo Lo 4Lc
i.e. , the 3rd hannonic of hydrogen and 11th hannonic 0
oxygen or 9th harmonic of hydrogen and 33rd hannonic 0
oxygen will have sl'lme frequency. So the minimum commOI
i.e., LO = 330 ~ Lo ::: 1.0067 m or 0.9937 m frequency
330+ 2.2
Problem 37. A pipe of length 1.5 m closed at one end is f= 3x550 or Ilx150 = 1650 Hz
filled with a gas and it resonates ill its fundamental with a (as 6th hannonic ofH and 22nd of 0 will not exist.
Problem 39. A 'pop' gun consists of a tube 25 em fong But as with decrease in temperature beats decrease, first
closed at one end by a cork alld at the other end by a tightly choice is not pennissible; So
Jitted pis/on. The piston is pushed slowly in. When the pressllre K~(273 +;I)- 1=4, i.e, 1 +4=I SK ....(i)
rises to one and halflimes the atmospheric pressure, Ihe cork
is violently blown out. Calculate the frequency of the ' pop ' and · K~(273+16)-I=I, i.e., 1+1 =17K .... (ii)
caused by its ejection. (v = 340m15)
So dividing Eqn. (i) by (ii),
Solution: Assuming the cross-section to be A and
I +4 IS
--=- or f=50Hz
compression to be isothennal (as the process is slow), from
1+1 17
PIV1 = P2 V2
Problem 42. A string 25 em long and having a mass of2.5
P >jo 25x A =(312) x P x L x A. i.e. , L = (50/3)cm
I - - 25 em - - - I
g is under tension. A pipe closed at one end is 40 em long.
Now after the ejection When the string is set vibrating in its first overtone and the ajr
of cork, for oscillating air in the pipe in its fundamental frequency, 8 beats per sec are
node will be at piston (rigid heard. It is observed that decreasing the tension in the string
boundary) while anti node decreases the beatfrequen cy. Ifthe speed ofsound in air is 320
will be at the open end and mls,find the tension in the string.
as minimum distance
Soludon: For first overtone (i.e., second hannonic) of the
between node and antinode is (A / 4).
1 . . 5 0 .le · A=-m
2 string with M = mL = mass 0rf.:.5tn.:.·",n"g:",-_.,--
so -=
4 L = -cm
3 ' · ., 3
1 =2. iT =~ T = T = 40.,fi
and hence I =£=340x3 =510Hz
s 2L V;;; ML 2.5 x 10 3 x 0.25
A 2 and for fundamental of closed organ pipe,
Problem 40. An under-water swimmer sends a sound
I.e = ~= ~ =200Hz
signal to the surface. If it produces 5 beats per second when 4L 4 x 0.4
compared with the funda mental tone ofa pipe of20 cm length So ·200 - 4o..fi = S or 40.,fi - 200 = S
closed at one end, what is the wavelength of sound in water?
Now as decreasing the tension, decreases the b¥at
(Takevwater = 1500mlsandv Air =360mls.)
frequency, first choice is not pennissible.
Solution: If J is the frequency of sound (which is
independent of medium), according to given problem So 4o..fi = 208, i.e, T =[2::]' = (5.2)' ~27N
1 =le±5.
But as Ie =~ = ~ =450Hz Problem 43. A closed pjpe and an open,pipe sounding
4L 4xO.2 together produce 5 beats per sec. If the length 0/the open pipe
So 1 =(450 ± 5)Hz is 30cm,find by how much the length ofthe closed pipe must be
changed to bring the two pipes in unison? [v=330mlsj.
Now as v =fl., i.e., A =vll
Solution: According to given,problem',
VwVw
So Aw = Iw = / [aslw =IA = f] Ie - 10 =5, i.e.,. Ie =10. ±5

1500 But fe =~ and fo =~ = 330 =550Hz


i. e. , AW = 450 ±5' i.e., A. w =3.30m or 337m 4Le 2L 2 x 0.30

Problem 41. A column of air at 51°e and a tuning fork so ~ = 550±5


produce 4 beats per sec when sounded together. As the 4Lc
temperature of the air column is decreased, the number of L - 330 m 330 , m
i. e., or
beats per sec tends to decrease and when the temperature is e- 4x555 4 x 545
16°C the hVo produce I beat per sec. Find thefrequency ofthe
or Le =14.8648 cm or 15.1376 cm
tuning fork.
Now the two pipes will be in unison when
Solution: As for a given column of air f oc v and v o:: ..fi
L
so Ie = K.,fi i.e., LC =3.=15
2
cm
If the frequency of fork is f,
So if the frequency of closed pipe is greater than open
I-K.,fi =4 or K.,fi - 1=4 pipe, its length should be increased by 15 - l4. 8648 = 0.1352
em. And if the frequency of closed pipe is lesser than open pipe
its length should be decreased by 15.1376 - 15 = 0.1376 em.
f -f[v-(+v
Ap -
v-O ) ]_f[_V
- v-uS
] "'V s
S --+v D
s
§ 13.8 Doppler Effect
i.e., apparent Irequency or pitch will increase as lAp> I.
When a source and detector of sound separated by a large
distance are at rest relative to the medium (usually air) the However. if the source moves away from the detector 0. e.,
detector (usually listener) receives the same number of waves recedes from it) v s will be opposite to v so v s --+ -v s
per sec as that emitted by the source. However, if there is
motion of source andlor detector relative to the medium along
f =f v-(-v
v-O ) _f[_V ] +-Vs
. Ap
s v+vS S --+v D
the line of sight·, the number of waves received per sec is
usually different from that emitted and the frequency or pitch i.e. , apparentlrequency or pitch will decrease as lAp < f.
of the source appear to be different. This apparent change in (B) Detector Alone in Motion
frequency or pitch due io motion of source and detector In this situation v s = 0 and detector is moving away from
relative to the medium along the line ofsight is called Doppler the source (i.e., receding from it). v D is in the assumed
Effect.
direction ofv sov D --+ +v D
Let us consider the case when both source and detector are
moving along the same line with detector ahead, with speeds
Us and UD respectively relative to the medium air, at rest as
fA' =t[V-v(:~D)]=/[V-:D ]
shown in Fig. 13.39.
i.e.. the apparen.tlrequency or pitch decreases as lAp < f
In one second the first wave emitted by the source at S
However, if the detector is moving towards source (i.e.,
=
travels a distance v x I v and reaches A. Meanwhile the
approaching it) v D will be opposite to v, so V D --+ -VD
source S moves a distance v s and reaches S' emitting Ith
wave. Hence ifvs < v, the I waves emitted by the source in _ [V-(-VD)]_-f [V+VD]
fA -I
one sec are contained in a distance S~ = v - v s . p v-O v

~::;1'J ljl v::;: ~vo-ci-)--Iljl i.e., the apparentlrequency or pitch increases as lAP> I .
[C] Source and Detector Both in Motion
5 5' ADD' 8
Fig. 13.39 Ifthe source and detector both move towards each other,
Now the first wave received by the detector at D covers a i.e., approach each other. Vs will be in the assumed direction
distance v in one sec and reaches B meanwhile the detector ofv while v D opposite tovso that v s --+ +v s and v D --+ -v D
itself covers a distance v 0 and reaches D' where it receives the
last wave of a second, Hence. if v 0 < v the waves received by
f =f[V- (-1JD)]=f[V+vD] +-vs vD...
Ap v-(+u s ) v-vs S .... v D
the detector in one second are contained in a distance
D'B = v-v o ' which shows maximum increase in frequency or pitch.
But as waves contained i':l a distance v - v S = I . the However, if the source and detector both move away from
number of waves contained in a distance v - v 0 will be each other, i.e., recede from each other, vp will be in the
=fx (v-vD) assumed direction of v while Vs opposite to v so that
(v-v s) v D .... +v D andvs --+-v s

which is the apparent frequency lAp as perceived by the f -f[V-(+V D)]_ f[V-VD]
detector. Ap - v-(-vs) - v+vs
which shows maximum decrease in Irequency or pitch .
....(i)
All cases of Doppler effect can be written in a single
, fonnula
In this fonnula direction of sound from source to observer
is taken to be positive, so speeds v D and v s are positive if
these are in the direction of sound (fromS taD) and negative if fA, =f[~::~] .... (ii)

opposite. This all will be clear from t~e following particular


where v s ,v 0 and v are the speed of source, detector and
cases:
sound relative to medium respectively (and ifthe medium is at
[A] Source Alone in Motion rest, relative to earth). The upper sign on Vs (orv D ) is used
In this situation v D = 0 and source is moving towards the when the source (detector) moves towards the detector
delector 0. e.. approaching it). vs is in the assumed direction of (source) while lower sign is used when it moves away.
v,sovs--++vs
• The line joining the source to the detector.
SUPERPOSITION OF WAVES 627

Discussion (b) If one is at the centre ofa circle


(I) There is always an increase in frequency or pitch if while the other is moving on it v
source moves towards detector or detector moves with uniform sp·eed. In·· this
towards source or both move towards each other situation component of u along 0
while a decrease infrequency ifeither or both move the line of sight, i.e., radius, will
away. The change in frequency or pitch depends on be u oos900=0' so
, [v±O·j ·.. ;: .. FI~. ~3.41
speeds of source and detector and not on distance
between them, e.g., if an engine is approaching a
1'=/ v+o " / \
stationary listener at constant velocity, increase in (c) !fboth are at rest and wind blows at speed w. In
pitch, by Eqn. (ii) will be same when the engine is this situation,
either at a distance of 1 km or 10 m from the listener.
However, intensity will be different in the two cases I' = I[(V ±W)+O] = I
(v±w)+O
asl oc (lfr2).
(5) Doppler effect, i.e., change infrequency depends on
(2) We have derived Eqn. (ii) assuming that the motion is the fact that whether the source is moved towards the
along the line joining the source and detector (line of detector or the detector is moved towards the source,
sight). If the motion is along some other direction, the e.g., if a source is moved towards a stationary detector
components ofvelocities along the line joining source with a speed u,
and detector are considered in Eqn. (ii) for v sand
v D' e.g., if at any instant
the line joining the
_source
.. e
VI I, =/[_v]
v-u
moving source and "'0
o.
&
[asv D =Oandv s =uinEqn.(ii)1 .... (iv)
stationary detector makes and if a detector is moved towards a stationary source
an angle 9 with the with a speed u,

=/[V:U]=/[I+~]
direction 0; motion of Detector
source, v s -+ v s cos 9 Fig. 13.40 I,
and so I Ap = I [--v"--~l
v-vscos8
.... (iii) [asv D =uandv s =OinEqn.(ii)]
From Eqns. (iv) and (v) it is self-evident that f t
.... (v)
~ f 2.
In such situations f Ap is not constant and depends on However, it u« v,
9 and may be greater, equal to or lesser thanf as9< = I, = I [ 1 ] = 10 _ (ulvW'
or>900. I-(ulv) ',. . '
(3) If the medium is at rest relative to earth, v s' V D and v
will refer to the speeds of source, detector and sound =/[1+;;]=1, ",,(vi)
relative to earth which is usually the case. However, if
i.e, in the limit when speed ofsource and detector are
the medium (air) itself starts moving with respect to
much lesser than that of sound. v, the change in
given frame of reference (say earth), appropriate
frequency becomes independent of the fact whether
changes must be made in Eqn. (ii), i.e., if wind blows
the source is moved or the detector.
at a speed wfrom tl)e source to the detector, v -+ v + w
(6) In case of reflection of soundfrom a reflector, the
and if in opposite direction (i. e., from detector to
detector will receive two notes, one directly .from the
source) v -+ V - w.
source and the other from the reflector (which also
(4) There will be no Doppler effect, i.e., no change in acts as a source). The frequencies of the two notes will
frequency: depend on the position and state of motion of source,
(a) If source and detector both move in same detector and reflector. If the two frequencies are
direction with same speed, i.e., if v s = v D = u slightly different, the superposition of these waves
from Eqn. (ii), will produce beats and the beat frequency can be used
-')u u-+ to detennine the speed of the reflector (or source).
I["-=.!':] I
I' = v-u = S -+v D This principle is used in sonar and radar to detennine
the speed of moving objects, e.g., if the source is at
rest and a reflector is moving towards the source with
speed u, the reflector (acting as detector D\) will
'hear' the frequency IA =/[l+~] and t. =/ [l-~] . . (x)
11 =f[V:UJ=f[l+;] [as source is at rest which are same as in case of sound (for u« v).
So at low speed Doppler effect in light and sound is
and detector is moving towards the source]. governed by the same formulae.
Now this frequency will be reflected from th~ Now as for electromagnetic waves c= fA and c is
reflector (which now acts as source and is moving independent of relative motion between source and
towards the detector D 2); so the frequency received observer, the above forml,1lae in terms 0tA. wilt become
back at the site of source from the reflector

=/l[~] A: =i[I+~J . and A: =f[' - ~]


I,
v-u
AA =A[l+;rl and J.. =){l - ~rl
-~----- -;)..---------
' 4"" or R
- --. 0
~ ______ ~------------- I Using binomial theorem again and simplifying, we
f,
gel

AA-A= -¥A and AR-A=+~A


Fig. 13.42 i.e., (aA)A =-¥>. and
which in the light of the above equation becomes
I, =[_v ]x J~] =/[V+u]
v-u JL v v-u
or ....(xi)

i.e .• ~n case of approach frequency increases while


Ifu«v, wavelength decreases, i.e., shift aA is towards blue
(A >.
end of the spectrum A = -.6.)..) while in case of
recession frequency decreases and wavelength
increases, i.e.. shift is towards red end
' So beal frequency, Q. R=A +.6J..).This effect allowed astronomers to
4f=I, _I =/[2:] or u=¥[;r] ... (';;i)
determine the speeds of stars and galaxies relative to
the earth by studying the wavelength (or frequency)
of radiations (light) coming from them.
(7) Doppler effect holds not only for sound (mechanical
(8) When speed of detector
waves) but also for electromagnetic waves (non-
or source becomes
mechanical waves) including microwaves. radio-
equal to or greater than
waves and visible light. However, as electromagnetic
that of sound. Doppler
waves do not require a medium for their propagation s
formula (U) does not
and the motion of source relative to detector or of
apply (as it was derived
detector relative to source represents same physical
by assuming v D and
situation (as speed of light is independent of relative
v s < v). For example if Fig. 13.43
motion between source and observer), the formulae
v D > v and the detector
are different from that of sound. Here when either
is moving away from the source, the sound will never
source or detector or both are in motion, only two
reach it and ifv s > v the source gets ahead of the wave
cases are possible, viz., of approach and recession and
for these apparent frequency is given by, in its direction of motion as shown in Fig. 13.43.

IA =1 Jc+v
c-v
and IR:I ~[c-v]
c+v
.. (iX)
When the speed of source (v s ) is greater than the
speed of sound (v) the speed is called supersonic
speed and the ratio (vs lv) Mach number. In this
where c is the velocity of light and v is the relative situation a conical wavefront of high energy pressure
speed of approach Qr recession. For v« c, the above waves [with source at its apex and semi cone angle =e
fonnulae with the help of binomial theorem reduces
sin - 1(vlv s )=sin -1(llMach No.)] called 'shock-
to:
waves' is continuously produced and when we
intercept it, a loud bang of sound called sonic boom
/, = f v , i.e., 50=~, i.e.,
is heard which can break windows and even cause v+v s v+vs
damage to buildings. Here it is worthy to note that
So adding the two Eqns.,
shock waves are produced not only when source
3fo .
crosses the sound barrier (a misconception) but are 2V= 100' I.e., I =200
} =66.67 Hz Answer
generated continuously as long as Vs > v.
An interesting analogy of shock-waves is the V- Problem 46. A whistle offrequency 540 Hz rotates in a
shaped bow-waves produced by the motion of a boat circle of radius 2 m at an angular speed of15 rad/sec. What is
when its speed exceeds the speed of the surface water the lowest and highest frequency heard by a listener, a long
waves. distance away at rest with respect to the centre of the circle
Similarly, when high speed charged particles (v =330m/s)? Can the apparent frequency by ever equal to
(electrons) move through a transparent medium actual?
(water) with speed greater than that of light in that Solution: In case of circular motion v = rro, so here
medium, i.e., Vs > V but< c, a shock or bow wave of Vs =2xI5=30mls 0
light called Cerenkov-radiation is produced.
Vs
,
and as detector is at :
Problem 44. A railway engine moving with a speed of60
mls passes infront ofa stationary listener. The real frequency
rest, £~~~:'_O!____________ ~_ ------L-----
:0
c
of the whistle is 400 Hz. Calculate the apparent frequency :,,
heard by the listener (a) when the engine is approaching the B v,
listene/~ (b) when the engine is moving away from the listener.
(v 340ml,)
= (MNR 1995(
So frequency will Fig. 13.44
be minimum when the source is at B and moving away from
Solution: We know that in case of Doppler effect, the listener

lAp =/[~:::] I
nun
=/[_V_]=540[
v+vs
330
330 + 30
]=495HZ

Here listener, i.e., detector is at rest"i.e., v D = 0 and frequency will be maximum when source is at D and

=/[v~Vvs]
approaching the listener.
lAp
I~ =/[_V_]=540[
v-v
330 ]= 594HZ
330-30
So, (a) when the engine is .approaching the listener, s
Further when source is at A or C, speed of source along
IA =/[_v_]=400[ 340 ]=485.7HZ line of sight v s cos 90° = 0

IA =1-v-=1=540Hz
v-v s 340-60
Le.,
(b) when the engine is receding from the listener, P v±O

IR =/[_v_]=400[
v+vs
340 ]=340HZ
340+60
i. e., apparent frequency will be equal to actual frequency when
the source is moving perpendicular to the line joining the
listener to the centre of circle, i. e., A and C.
Note: Change in frequency as the engine passes by (i.e., flies past), Problem 47. A source of sound is moving along a
N = f A - fR =485.7 - 340 = 145.7 Hz. circular orbit of radius 3 m with an angular velocity of 10
radls. A sound detector located jar away from the source is
Problem 45. When a train is approaching the observer, executing linear simple harmonic motion along the line BD
the frequency ofthe whistle is 1OOcps while when it has passed with amplitude BC = CD = 6 m. The frequency ofoscillation of
the observer, it is 50 cps. Calculate the frequency when the the detector is (5/n)per sec. The source is at the point A when
observer moves with the train. the detector is at the point B. If the source emits a continuous
Solution: According to Doppler effect in case of sound wave of frequency 340 Hz, find the maximum and the
approaching of source, minimum frequencies recorded by the detector (velocity of
sound = 330m ls~
/, = f __v_, i.e., 100=~, i.e., v-v s =E Solution: Time period of circular motion T =(2rrJoo)
v-v s v-v s 100
= (2rrJIO) is same as thatofSHM, i. e.,T = (lIf) = (nlS), so both
while in case of recession of source, will complete one periodic motion in same time. Furthennore
as source is moving on a circle, its speed
Vs ::::rro=3xlO=30mls Here as SD=~12+D.62 = 1.166 mile
N
se
cos9 = - = - - =0.857
1
SD 1.166
A
--------------.----~----~ So speed of source along the line of sight,
B-C 0
8
vs -)0 v S cose = 60 xO60 x 0.857 =0.019 mile/s
M

is
and as .defectot' executing SHM,
Fig. 13.45
So i.p =f[~V_]=400[ 0.2 J=442HZ
v-v s 0.2 - 0.019
. v 'D '=ro'~A2 - i = lO~62 -l Problem 49. Two tuning forks with natura/frequencies
i.e., (vD)max =60mls when y=O 340 Hz each move relative to a stationaJY observer. One fork
moves away from the observer, while the other moves towards
i. e., detector is at C. Now in the case of Doppler effect,
him at the same speed. The observer hears beats offrequency3

fAp =f[::~~] Hz. Find the speed ofthe tuningfork. (Velocity ofsound in air
= 340m/s)

So f Ap will be maximum when both move towards each other. Solution: As observer is at rest,

, finax =f[V+VD] wlthv D= max


., .. , v -vs
fAp =f[ v
V+ Vs
]
. > " . ""':" :" "' : <~;;;~ ' . '-,.,,:,- .' -;.
i. e.; the_ siftir<:eh afM,and detector ate and moving towards B. If speed of tuning fork is u,

fA f[-V-J
, ' • '-,' ," " " ..'< , '(' .• ' .c_, , ,.. . •

=f[~-J
so .. ; :.,'" " ;,"
,:~

f ·' =340[330+60J=442HZ v-u


= wh;le h v+u
~ 330-30 ' Now as beat frequency is 3 Hz, so fA and fR are very
close which is possible only if u« v. So using binomial
. ~irh~larly. I,~.p; ,will, h.e mi,nim~m when both are moving
theorem,

. . ';~:~:~i~9~::e~;[~::~] MthvD =nmx fA =f[t-;;r' =f[l+~J


i.e."the source is at N and detector ate but moving towards D,
so ' and fR =f[t+~r =f[t-;;J
f· =340[330-60J=255HZ So beat frequency
nun . 330+30
, Problem 48. A locomotive approaching a crossing at a ~f=fA -fR
=f[2:J
sRe?4pl;.)~ :'!ll/e,'s!hr, :~ourzds:a whistle 'offrequency 400 Hz
.- w.hen'tjniM'[r:oiJt;lhe crqssing. ' There I~no wind and the speed i.e., u=~[~fl=340[_
3 J = 1.5m1S Answer
ofsound.fn air is 0,200 mile/so What frequency is heard by an 2 f 2 340
observer 0.60 mile from the crossing on the straight road
Note: If one tries to solve this problem without making above
which ~rosses the railroad at right angles? approximation, calculations become complicated and time
Solution: The situation is shown in Fig. 13.46. consuming (due to quadratic Eqn. in 1/). Verify it yourself.
, . ' 1 mile .' ' '
C Problem 50. A sonometer wire under tension of 64 N
vibrating in its fundamental mode is in resonance with a
vibrating tuning fork. The vibrating portiol/ of the sonometer
wire has a length of 10 em and mass 1g. The vibrating tuning
fork is now moved away from the vibrating wire at a constant
speed alld an observer standing near the sonometer hears one
beat per sec. Calculate the speed with which the tuningfork is
moved, if the speed ofsound in air is 300m/s. IMNR 1990]
Fig. 1'" 10
'

~ 'E:j
Solution: As the frequency of a vibrating string o ..
--<-
Is =2~~=~& [asm=~J " ~

1 la) Ib) Ie)


so Is =-
2
64 ] = 400Hz Fig. 13.47
10 3 xlO \
[B] Source between the observer and the wall
Now, as initially sonometer wire is in resonance with
In this situation [shown in Fig. 13.47 (b)] as observer is at
tuning fork, the frequency of tuning fork
rest and source is moving away from him so he will hear
1=ls=400Hz frequency

=/[-V-]=256[~J=252.2HZ
When the tuning fork is moved away from the observer
standing near the sonometer at a constant speed u the apparent I, v+vs 330+5
frequency of tuning fork will be
and as the source is approaching the wall, the frequency of
IR =/[_V
V+U
J sound reaching the wall

AslR is producing beats withl,fR is nearly equal to I, J, =/[_ V_]= 259.9HZ [from ease (A)]
i. e., u« v so that v - vs
As this frequency will be reflected by the stationary wall
towards the stationary observer, the frequency of reflected
sound heard by the observer will be 12 ,
Sobeatfrequency 6/ =1-IR =/[~] So beat frequency = fz - fl =259.9 -252.2 =7.7 Hz
[C] If the observer and source are moving together towards
and substituting given data, the wall

u:V [~] =300[ 4~OJ =0.75m1s


In this situation [shown in Fig. 13.47 (c)] as source and
Answer
observer are moving in same direction with same speed, the
frequency of source heard by the observer,
Problem 51. A source of sound oflrequency 256 Hz is
moving rapidly towards a wall with a velocity 015 mls. How I, =/ [V- VD]=1 =256HZ [asv D=v s ]
many beats per sec will be heard if sound travels at a speed of v-v s
330mls? . and as source is approaching the stationary wall, the frequency
Solution: The observer will receive two sounds, one of sound reaching the wall
directly from the source and the other reflected from the wall
and the two will produce beats at the site of the observer. In 1>1 [_ V_ ] [asvD =0] .... (iv)
this problem, position of observer is not specified; so the
v - vS
question is open to following three possibilities: Sound of this frequency will be reflected by the wall
[A] Observer between the source and the wall (source at rest) towards the observer which is moving towards
In this situation [shown in Fig. 13.47 (a)] as source is the wall (source). So the frequency 12 heard by the observer
moving towards the observer, so he will hear after reflection from the wall
' ..
I , =/[_V_] =256X[ 330 J=259.9HZ .... (i) 12 = IA [V+vD J
V 'I' ,I
....
, (v)
v - vs 330-5
This is also the frequency 'heard' by the wall towards SubstitutinglA from Eqn. (iv) in (v),
which source is moving.
I, = I[V +vs] .
Now as the wall (source) and observer both are at rest, the v-vs
frequency of reflected sound heard by the observer 12 = 1\ =
259.9 Hz. i.e., I = 256[330 + 5J =263.9 Hz .... (vi)
, 330 - 5 ,
So Beat frequency = 12 - f\ == 0
So beat frequency
6f = I, - I , = 263.9 - 256 = 7.9 Hz Answ"
632
- Wind
Problem 52. A band playing music at a frequency f is
movillg towards a wall 01 a speed VB- A motorisl is following
the band with a speedv m. Ifv is the speed o/sound. obtain an
expr~sionfor the beatjrequency heard by the motorist.
S.olutioh: . The situation is shown in Fig. 13.48. As the Fig. 13.49
motorist (observer) is following the band (source), he will hear
two frequencies one directly -from the band while the other IA =/[_V
v-vs
]
reflected. from
. the wall.
~\" .
__. . :, ..:~ fi " . Now as w ind is blowing fro m source to observer

I ; MI 'm.2.~,. I· v--tv+w,
!' IA =1 (v +w)-vs
(v+w)
..L ,.r-f ~ 't~ ~
Substituting the given data,.
'Observer Source W.II
Fig. 13A8 I =580[ 1200 + 40 ] = 599.3 Hz
Takirig the direction of sound from source to observer 10 A (1200+40)-40 .
be positive, the frequency of the sou,nd heard directly by the (b) If x is the required distance from the hill, the distance
motorist from the band, moved by the train will be (I -x ) and hence the time taken by
the train to travel this distance is (1- x )/40.
II =/ [V- (-Vml]=/[V+Vm]
v- (-v ) v+v8
....(i) In this time sound travels a distance I km a t speed (1200 +
B
40) and comes back a d istance x at speed (1 200 - 40); so
. Now _a~ I,h e freq~ency of sound reaching the wall towards
which the band (source) is' moving, 1- x =_I_+~, i.e., x=29 km = 933,3 m
40 1240 11 60 31

.
F= I[_V
v-v
] ....(ii) Now the e ngine will act as observer and hill as source; so
o the frequency heard by the moving observer towards the
" The~ riequ~n!;y of refle'cled sound from the wall, heard by stationary source will be
motoris( (ob~rver) who is moving towards the wall
(stationary source) will be /, =IA[V+vVD ]
I, =f'[V+vm]=/[v+Pm]
v v-vB
[from Eqn. (ii)] But in this situation as w ind is blowing opposite to the
directio n of motion of sound,
So the beat frequency heard by the motorist v--+v-w

61 = I, -II = I[V+vm] _1[_V+-- ,V.'J!.m] so


_ A [(V-W)+VD
I,-I ]
v-vB V+V8
(
v-w)
-I (v+v
-
2
m) x 2VB Answer
= 5993 x 1200 = 620Hz
. 1160 .
Answer
(v -vi)
Problem S4. Th e wavelength of light comillg from a
Pr~blem 53. A train approaching a hill at a speed of 40 distant galaxy isfollnd to beOS % more than that comillgfrom
, kmIh! sounds a whistle of frequency 580 Hz when it is at a a source 011 earth. Calculate the velocity of the galaxy.
': 't , ' " , ' .'distqlice ofl kmfrom 'the hill. A wind with speed 40 km/hr is
Solution: As wavelength has incre~s e d the galaxy is
~ ·(~':'>'~~': ~'bi.$~j~k}k~~~~ direction ~Imotion "0/ the tra·in. Find (0) the receding from the earth, Fmther as,
.. ffeii~i/ncyoithewhistle as heard byan'observeronthe hill, (b)
the dis tance Irom tlie hill at which the echo from the hill is .6,). = E.!.. i,e., Do!.. =.!::
c ' A. c
heard by the driver and its frequency. (Velocity ofsound in air
= 1200kmlhr)
Solution: (a) For observer at rest (on hill) and source
v=c[~' ] =3 XIO ' x[~~~]
[en.g in.e] ,! poving towards the observer, =1.5 x 10 6 mls
Problem 55. An astronaut is approaching the moon. He Due to motion of111edium, velocity of sound is increased if
sends out a radio signal offrequency 5 x 10 9 Hz andfinds out velocity of medium is in the direction of sound velocity.
that 'thefi'equency shift in echo received is 10 3 Hz. Find his Vs =v+vw
speed of approach. When source is in motion and observer stationary,
Solution: If the astronaut (source) at speed v is appro- frequency is given by
aching the moon, the frequency 'heard' by the moon will be, f' o f (v+vw)
Ilof~C+v .... (i) (v + vw) - vb
c-v Here v b is velocity of boat and v 11' is velocity of water stream.
Now this frequency will be reflected back by the moon On substituting numerical values, we get
(now source) towards the astronaut which is observer and
- f' 100696 Hz
0
moving towards the moon. So the astronaut will hear a
frequency (b) In this case au blows opposite to sound; hence
apparent frequency
- I I ~C+v
f 2- -- .... (ii)
c-v f' o f (v-v")
(v - va -Vb)
Substituting the value of fl from Eqn. (i) in (ii)
The velocity of sound in air
f; of[~~~]~/[I+2:] (asv« c]
v0 ~Y RT x"'8."3;-'1x"2"'9:;-3 0 344
0 -"I".4'::' ~s
f2 - 1 M 28.8xlO 3
- f - oc '
2v
ie, VO} [j] On substituting numerical values, we get

So substituting the given data


f' 0103039 Hz
3 Problem 57. A source S emitting ~ ~
v = 3XIO'[ 10 ]0 30mlS sound of 300 Hz is flXeO on block A
2 5 x 10 9 which is attached to the free end of a · SA A B Sa
spring SA as shown in the Fig. 13.50. Fig. 13.50
Problem 56. A boat is travelling in a river with a speed of
The detector D fIXed on block B attached to free end ofspring
10 m/s along the stream flowing with a speed of 2m/s. From S B detects this sound. The blocks A and B are simultaneously
this boat, a sound transmitter is lowered into the river through displaced towards each other through a distance ofl.0 m and
a rigid support. The wavelength of the sound emitted fi'om the then left to vibrate. Find the maximum and minimum
transmitter inside the water is 14.45 mm. Assume that frequencies ofsound detected by D ifthe vibrationalfrequency
attenuation of sound in water and air is negligible. of each block is 2 Hz.
(a) What will be the frequency detected by receiver kept Solution: Apparent frequency is maximum when both
inside the river downstream? source and detector approach each other.
(b) The transmitter and the receiver are now pulled up V+VD
into the air. The air is blowing with a speed of5 m/s in the Vmax =V Vs
Vo .... (i)
direction opposite the river stream.
Determine the frequency of the soulld detected by the Frequency is minimum when source and detector both
receiver. recede from each other.
(Temperature ofthe air and wate,. = 20°C; Density ofriver v-v
v . = D V .... (ii)
water = 10 3 kg/m 3; Bulk modulus of the water = 2.088 x 10 9
mm v+vs 0

Pa; Gas constant R = 8.31 J/mol-K; Mean molar mass of air where v = velocity of sound, v D = velocity of detector, vs =
3
=28.8 x 10- kg/mol; C p,1I! / Cu,m for air = 1.4). velocity of source; Vo = frequency of source.
Solution: Velocity of sound in water is given by Herev s =v D =roA,
v JB/p
0
Maximum velocity of A and B = roA = 41t mls = 12.56 inls
Now substituting numerical values in Eqns. (i) and (ii)
9
V o 2.088 x 10 = 1445111/ S
v 0 [340 + 12.56]x300=323HZ
10 3 max 340 - 12.56
Frequency in still water =.!!. = 10 5 Hz v . = ( 340-12.56)X300 = 278.6HZ Answer
l. mm 340 + 12.56
EXERCISE
{A) Only One Choice is Correct (,) 10: 8 (b) 9: I
1. Echoes are produced due to: (c) 4: I (d) 2: I
(a) Reflection (b) Refraction 8. Beats are the result of:
(c) Resonance (d) Beat production (a) Diffraction
2. The elastic constant and density of sea water are (b) Destructive interference
respectively 2.156x 10 9 newton m- 2 and I.Ix 10 3 kglm 3 . (c) Constructive and destructive interference
In a sonar experiment it was found that the acoustic echo (d) Superposition of two waves of nearly equal frequencies
from the sea bed is heard after 2 sec. Thedepthofthesea is:
9. Two tuning forks are sounded together and beats are heard.
(a) 1.40 km (b) 1.96 km • We can infer that:
(c) 2.80 km (d) 1.50 km (a) The pitches ofthe two notes are nearly the same
3. A man standing between two cliffs hears the first echo of a (b) The intensities of the two notes arc nearly the same
sound after 2 sec and the second echo 3 sec after the initial
(c) The qualities of the two notes are nearly the same
sound. If the speed of sound be 330 mis, the distance
(d) The velocities of the two sounds are nearly the sam
between the two cliffs should be: 7
(a) 1650 m (b) 990 m 10. Two adjacent piano keys are struck simultaneously. The
notes emitted by them have frequencies fi and 12 , The
(c) 825 m (d) 660 m
number of beats heard per sec is:
4. A man standing between two cliffs claps his hands and
(a) 112 (fj - 12 ) (b) 1/2 (fj + I,)
starts hearing a series of echoes at intervals of2 sec. Since
the speed of sound in air is 340 mfs, the distance between (c) (fj - I,) (d) 2(/1 - I,)
the cliffs must be: (NCERT 19921 11. If a tuning fork of frequency 512 is sounded with a
(a) 340 m (b) 680 III
vibrating siring of frequency 505.5 the beats produced per
sec will be :
(c) 1020 m (d) 170m
(a) 6 (b) 7
5. To demonstrate the phenomenon of interference we require:
(c) 6.5 (d) any of these
(a) Two sources which emit radiations of the same
frequency ~ 12. If two tuning forks of frequencies 512 Hz and 532 Hz are
sounded together, beats' produced and heard are
(b) Two sources which einit radiations of nearly the same respectively:
frequency
(a) 20and20 (b) 20 and 0
(c) Two sources which emit radiations of the same
frequenc~ and have a definite phase relationship
ec) Oand20 (d) OandO
(d) Two sources which emit radiation of different 13. A source x of unknown frequency produces 8 beats with a
frequencies source of250 Hz and 12 beats with a source of270 Hz. The
frequency of source x is:
6. Out of the given waves (A), (8), (C) and (D),
(a) 258 Hz (b) 242 Hz
y=Asin (kt-rot) ... (A)
(c) 262 Hz (d) 282 Hz
y=Asin(rot - kt) ... (B)
14. A luning fork whose frequency is given by the
y=Acos(h+rot) ... (C) manufacturer as 512 Hz is being tested with an accurate
y=Acos(rot-kt) ... (D) oscillator. It is found that the fork produces a beat of2 Hz
emitted by four different sources S]>S2,S3 and S4 when the oscillator reads 514 Hz but produces a beat of 6
respectively; perfect interference phenomena will be Hz when the oscillator reads 510 Hz. The actual frequency
observed in space under appropriate conditions when: of the tuning fork is:
(a) S 1 emits (A) and S 2 (8) (,) 508 Hz (b) 512 Hz
(b) S2 emits (C) andS 4 (D) (c) 516Hz (d) 518Hz
(c) S3 emits (B) andS 4 CD) 15. When two tuning forks A and B are sounded together x
Cd) S 4 emits (B) and S 3 (C) beatls are heard. Frequency of A is n. Now when one prong
of fork B is loaded with a little wax, the number of beatls
[Hint: Perfect interference means 1 min '" 0 and Imax "" 4/ 0J
decreases. The frequency offorkB is: (MNR 1993]
7. The two waves having intensities in the ratio I : 9 produce
(a) II+X (b) II-X
interference. The ratio of the maximum to the minimum
intensities is equal to: (c) lI_x 2 Cd) n - 2x
16. A tuning fork offrequency 100 Hz, when sounded together (a) Resonance (b) Harmonics
with another tuning fork of unknown frequency, produces 2 (c) Beats (d) Stationary waves
beats per sec. On loading the tuning fork whose frequency
24. Which of the following represents a standing wave?
is not known and sounded together with a tuning fork of
frequency 100 Hz produces one beat, then the frequency of (Roorkee 19971
the other tuning fork is : (a) y = Asin(oot - kx)
(a) 102 (b) 98 (b) y=Asinkxsin(oot-9)
(c) 99 (d) 101 (c) y=Ae-bxsin(oot - kx+u)
17. A fork produces 6 beats per sec with another fo rk of (d) y~(ax+b)sin(OJt - kx)
frequency 384. If the prongs offirst fork are slightly filed 4 25. Ifa wave is ~epresented
by the following equation;
beats per sec are produced. The frequency of the first fork
21tX. 21Wt
after filing is: y=Acos-,m - -
1- 1-
(a) 390 H, (b) 378 H,
then it is a :
(c) 380 Hz (d) 388 H,
(a) Progressive wave
18. There arc three sources of sound of equal intensities with
frequencies 400, 401 and 402 vib/s. If all are sounded (b) Stationary wave
together, the number of beats heard per sec is : (c) Longitudinal progressive wave .
(a) 0 (b) I (d) Transverse progressive wave
(c) 2 (d) 3 26. In case of stationary sound waves in air:
19. The number of beats produced per sec by the vibrations (a) Each air particle vibrates with the same amplitude
Xl "" A sin (3201tl)andx 2 = A sin (326ri:t)is: [CPMT 1993] (b) Amplitude is maximum for some particles and
(a) 6 (b) 4 minimum for some others
(c) The particles do not execute periodic motion
(c) 3 (d) 2
20. Two vibrating tuning forks producing progressive waves (d) The particles do not vibrate at all
given by; 27. In a stationary wave all particles are:
Yj ",,4sln (50Ont) Y2 =2sin (506nt) (a) At rest at the same time twice in every period of
are held near the ear of a person. The person will hear: oscillation
(a) 3 beatsls with intensity ratio between maxima and (b) At rest at the same time once in every period of
minima equal to 2 oscillation
(b) 3 beats/s with intensity ratio between maxima and (c) Never at rest at the same time
minima equal to 9 (d) Never at rest at all
(c) 6 beatsls with intensity ratio between maxima and 28. Energy is not transferred by: IMNR 1990; CPMT 1990)
minima equal to 2 (a) Transverse progressive wave
(d) 6 beats/s with intensity ratio between maxima and (b) Longitudinal progressive wave
minima equal to 9 (c) Stationary wave
21. Two waves of wavelength 2 m and 2.02 m respectively, (d) Electromagnetic wave
moving with the same velocity, superpose to produce 2
29. The distance between two consecutive nodes in a Kundt's
beats per sec. The velocity of the wave is: ICPMT 19911
tube is:
(a) 400.0 m/, (b) 402.0 mls
(a) )14 (b) )12
(c) 404.0 mI, (d) 406.0 mI,
(c) 1- (d) 3)14
22. A source of frequency fgives 5 beats/s when sounded with
30. The distRnce between a node and successive antinode in a
a source of frequency 200 Hz. The second harmonic of
stationary wave of wavelength A. is :
source gives 10 beats/s when sounded with a source of
(a) 1- (b) )J2
frequency 420 Hz. The value of fis: ICBSE 1994]
(c) )14 (d) 21-
(a) 200 H, (b) 210 H,
31. A sound source of frequency 170 Hz is placed near a wall.
(c) 205 Hz (d) 195 H,
A man walking from the source towards the wall finds that
23. Two waves of the same frequency travelling in the same there is periodic rise and fall of sound intensity. If the speed
medium but in opposite directions if superposed give rise of sound in air is 340 mis, the distance separating the two
to: adjacent positions of minimum intensity is: [MNR 1992)
(a) (I/Z) m (b) (3/Z) m 39. Standing waves can be obtained in an air column even if the
(e) 1 m (d) Z m interfering waves are:
3Z. The equation of stationary wave in a stretched string is (a) of different pitches
given by y = 5sin (ru: 1 3)cos (4Orct)wherexand yare in em (b) of different amplitudes
and t in sec. The separation between two adjacent nodes is: (e) of different qualities
[CPMTI990[ (d) moving with different velocities
(a) 1.5 em (b) 3 em 40. A string of5.S m length has a mass of 0.035 kg. If tension in
(e) 6 em (d) 4 em the string is 77 N, then speed ofa wave on the string is:
33. A wave represented by the equation y ::: acos (kx - wt) is [CRSE 1992J
superposed with another wave to fann a statiomllY wave (a) 77 mls (b) 10Z mI,
such that the pointx = Ois a node. The equation for the other
(c) 110m's (d) 164 mI,
wave is:
(a) asin(kx+w() 41. The linear density of a vibrating string is l.3x 10-4
(b) - acos (kx+wt)
kg/m. A transverse wave is propagating on the string and is
(e) -acos(kx-rot) (d) -asio (,b--wt)
described by the equation r=0021 sin (x+30t) where x
(Hint: Given equation can give rise stationary waves only with (a) and yare measured in metre and t in sec. The tension in the
or (b). Now for x '" 0 to be node, amplitude of stationary wave string is :
must be a sine function of x, which is possible only with (b)J
(a) O.IZ N (b) 0.48 N
34. A wave of frequency 100 Hz travels along a string towards
(e) 1.20 N (d) 4.80 N
its fixed end. When this wave travels back, after reflection a
node is fonned at a distance of 10 cm from the fixed end. 4Z. A unifonn string of length L and mass M is fixed at both
The speed of the wave (incident and reflected) is : ends ~nder tension T. Then it can vibrate with frequency
given by the fonnula: [CPMT 1992]
[CRSE 1994[
(a) 5 mls (b) 10 mI, (a) J ~ ~JT (b) J~~ iT
(e) ZO mI, (d) 40ml,
2 ML VAi
2L

35. Stationary wave is represented by; (e) J ~ ~ iT (d) J~~ 1M


y =: A sin (lOOl)eos (00 Ix) zVAi zVLf
where yand A are in mm, t in sec andx in m. The velocity of 43. A sonometer wire of density d and radius r is held between
the~ave is: [CBSE 1994[ two bridges at a distance L apart. The wire has a tension T.
The fundamental frequency of the wire will be :
(a) 1 mI, (b) 10' mI,
(c) 104 mls (a) J~_I IT (b) J~.': r;;;i
(d) not derivable
2LrV;d 2L Vr
36. What type of waves are produced in a sitar wire?
J ~_I IT J~~ ~
(a) Transverse-progressive
[CPMTI992[ (e)
2L1' v;r (d)
2L Vr
44. A sitar wire is replaced by another wire of same length and
(b) Longitudinal-progressive material but of triple radius. If the tension in the wire
(c) Transverse-stationary remains the same, the frequency will become:
(d) Longitudinal-stationary (a) Nine times (b) Three times
37. In a resonance tube we get: (c) One-third (d) One-ninth
(a) Stationary-longitudinal waves 45. If the tension in a sonometer wire is inereased by a factor of
(b) Stationary-transverse waves four, the fundamental frequency of vibration changes by a
(c) Progressive-longitudinal waves factor of:
(d) Progressive-transverse waves (a) 4 (b) (114)
38. At antinodes : (e) Z (d) (l/Z)
(a) Pressure is maximum and velocity is minimum 46. The tension of a piano wire is 16 kg weight. What must be
the change in tension so as to produce a tone an octave
(b) Pressure is minimum and velocity is maximum
lower?
(c) Pressure and velocity both are maximum
(a) 8 kg weight (b) 32 kg weight
(d) Pressure and velocity both are minimum
(c) 12 kg weight (d) 4 kg weight
47. The frequency of the note produced by plucking a given (b) Always an antinode
string increases as : (c) Someti mes a node and sometimes an antinode
(a) The length of the string increases (d) Neither a node nor an antinode
(b) The tension in the string increases 56. In a long cylindrical tube the wave level is adjusted and the
(c) The tension in the string decreases air column above it is made to vibrate in unison with a
(d) . Both the tension and length increase proportionately vibrating tuning fork kep t at the open end. Maximum sound
is heard when the air column lengths arc equal to :
48. A stretched wire of length 60 em vibrates with a
fundamental note of frequency 256 Hz. If the length of the !: 3l.
(,) !:4 '2'4 (b) !: l. 3l.
wire'is reduced 10 15 em while the tension is kepi constant, 2' '2
the fundamental frequency of the wire will be:
(e) !: 3l. 5A (d) l. 3l. 5l.
(,) 64 (b) 256 2 '2'2 4'4'4
(e) 512 (d) 1024 57. A glass tube of 1.0 m length is filled with water; the water
49. A stretched wire of length 114 em is divided into three can be drained out slowly at the bottom of the tube. If a
segments whose frequencies are in the ratio 1 : 3 : 4; the vibrating tuning fork of frequency 500 Hz is brought at the
length of the segments must be in the ratio: upper end of the tube and the velocity of sound is 330 mis,
then the total number of resonances obtained will be:
(,) 18:24:72 (b) 24 :72: 18
IMNR19941
(e) 24: 18 :72 (d) 72 :24 : 18
(,) 4 (b) 3
50. A stretched string oflength L, fix ed at both ends, can sustain
stationary waves of wavelength Agiven by: (e) 2 (d) I
(,) l.-n'/2L (b) l.-L'nn 58. For a resonance tube the air columns for the first and second
resonance differ by 3 1.5 cm. The wavelength of the sound
(e) l. -2Ll n (d) l. - 2Ln waves in the tube is :
51. The speed of transverse waves in a stretched string is 700 (b) 63.0 om
(a) 31.5 cm
cmls. lfthe string is 2 m long, the frequency with which it
(c) 126.0 cm (d) 252.0 em
resonates in fundamental mode is : [CPMT 1993)
(,) (7/2) Hz
59. A tuning fork of frequency 500 Hz is sounded on a
(b) (7/4) Hz
resonance tube; the first and second resonance is obtained
(e) (14) Hz (d) (2n) Hz at 17 cm and 52 cm. The velocity of the sound in Ill/s is:
52. U nlike a laboratory sonometer, a stringed instrument is (,) 170 (b) 350
seldom plucked in the middle. Supposing a sitar string is
(e) 520 (d) 850
plucked at about ( 1/4 th) ofits length from the end, the most
prominent harmonic would be : 60. In a resonance tube experiment the first resonance is
obtained for 10 em of air column and the second for 32 cm;
(a) Eighth (b) Fourth
the end correction for this apparatus is equal to:
(c) Third (d) Second
(a) 0.5 em (b) 1.0 em
53. A tuning fork of frequency 480 Hz produces to beats per
(e) 1. 5 em (d) 2 em
sec when sounded with a vibrating sonometer string. What
must have been the frequency of the string if a slight 6 I. The speed of sound in air is Vi the fundamental frequency of
increase in tension produces fewer beats per sec than the air column in a pipe of length Lclosed at one end is :
before? ICPMTI9931
(,) 460 Hz (b) 470 Hz (,) j-(vI4L ) (b) j-(vl2L)
(e) 480 Hz (d) 490 Hz (e) j:(3vI 4L) (d) j-(v l L )
54. A tuning fork vibrating with a sonometer having 20 cm 62. If the velocity o f sound in air is 320 mfs, the frequency of
wire produces 5 beats per sec. The beat frequency does not the fundamen tal note emitted by a tube oflength I m closed
change if the length of the wire IS changed to 21 cm. The at one end is: lIlT 1990)
frequency ofthctuning fork must be : IMNR 1991) (,) 80 Hz (b) 240 Hz
(,) 200 Hz (b) 210 Hz (e) 320 Hz (d) 400 Hz
(e) 205 Hz (d) 2 15 Hz 63. A hollow metallic tube of length L and closed at one end
55 . A stationary wave is set up in a resonance air column of a produces resonance with a tuning fork of frequency f. The
glass tube partially filled with water by holding a tuning entire tube is heated carefully so that length changes by I if
fork near the open end; the open end of the tube is: the change in velocity V of the sound is v. The resonance
(a) Always a node will now be produced by tuning fo rk whos\!' flcquelll.:Y is :
(a) (V+v) / 4(L+/) (b) (V-v) / 4(L-1) (a) Double (b) Half
(e) (V + v) / 4(L - I) (d) (V-v) / 4(L+I) (c) Same (d) None of these
64. When a tall empty jar is placed below a water tap from 73. A cylindrical tube, open at both ends, has fundamen tal
which water flows QUI slowly into the jar producing a sound frequency' f' in air. The tube is dipped vertically in water
note, the pitch of the nole will: so that half of it is in water. The fundamenta l frequency of
(a) G radually rise (b) Gradually fall the air column is now: (lIT 1992; CBSE 1997)
(c) Remain unchanged Cd) None of these (a) / 12 (b) 3/14
65. If oil of density higher than water is filled in place of water (e) / (d) 2/
in a resonance tube, its frequency will : 74. If the fundamental frequency ofa pipe closed at one end is
(a) Increase 512 Hz, the fundamental frequency of a pipe of the same
(b) Decrease dimensions but open at both ends will be:
(e) Remain unchanged (a) 1024Hz (b) 512Hz
(d) Depend on the density of material of tube (e) 256 Hz (d) 128 Hz
66. In an open organ pipe the fundamental note is produced 75 . An open organ pipe has fundamental frequency of300 Hz.
when its lengt.h is: The first overtone of this organ pipe is the same as the first
overtone of a closed organ pipe. The length of the closed
(a) )../4 (b) . )../ 2
organ pipe is:
(e) 3)../ 4 (d) )..
(a) 10 cm (b) 4 1 cm
67. The velocity of sound in air is 330 m1s. The fundamental
(c) 82 cm (d) 185 cm
frequency of an organ pipe open at both ends and of length
0.3 m will be: (MN R 1993) 76. A closed o rgan pipe and an open organ pipe have their first
overtones identical in frequency. Their lengths are in the
(a) 200 Hz (b) 550 Hz
ratio:
(e) 330 Hz (d) 275 Hz
68. A pipe open at both ends gives frequencies which are:
ICPMT 1990, 921
77. A sound wave is passing through an air column during the
(a) Only even multiples of fundamental frequency consequent compressions and rarefactions:
(b) Only odd multiples of fundamental frequency (a) Boyle's law is obeyed
(e) All integral multiples of fundamental frequency (b) Total amount of heat remains constant
(d) All fractional muiliples of fundamental frequency (e) Density of air remains constant
69. Air is blown at the mouth of a tube of length 25 em and (d) Bulk modulus of air oscillates
diameter 2 em open at both ends. If the velocity of sound is
330 mis, the sound 'emitted by the tube will have all the 78. An organ pipe vibrates in fundamental resonance with the
frequencies in the group: medium first as air, then hydrogen and then methane. The
velocities of sound in the three media are different, owing
(a) 660, 1320,2640 Hz (b) 660,1000,3300 Hz
to which in the three cases:
(e) 332,660, 1320 Hz (d) 330,990, 1690Hz
(a) The frequency changes
70. An open pipe of length 33 cm resonates to a frequency of
(b) The wavelength changes
1000 Hz. Is it the:
(c) The wavelength and frequency both change
(a) Fundamental frequency of the pipe
(d) The wavelength and frequency both remain invariant
(b) First harmonic ofthe pipe
79. The apparent change in the pitch o fa sounding body due to
(c) Second harmonic of the pipe
relative motion of the body and an observer is called:
(d) Fourth harmonic of the pipe
(a) Doppler effect (b) Huygens' law
71. Ifni is the resonance frequency ofa pipe open at both ends
(c) Newton's law (d) Galileo law
and n 2 the resonance frequency of a pipe open at one end
only and both are in the fundamental mode and the pipes are 80. An engine is approaching a stationary observer. It sounds a
of the same length, then: whistle. Indicate whether:
(a) n 1 =211 2 (b) n 1 =n2 (a) The observer hears the same frequency
(c) 2111 ="2 (d) 3n] =4n 2 (b) The observer hears a slightly higher frequency
72. An open pipe produces fundamental note. All of a sudden (c) The observer hears a slightly lower frequency
one of its ends is closed. If again fundamental note is (d) The observer hears the same frequency but of different
emitted, the frequency of the note will be: quality
639
81. A whistle giving OUI 450 Hz approaches a stationary
observer at a speed of 33 mls. The frequency heard by the
observer in Hz is: (liT 1997)
(aj 409 (bj 429
(cj 517 (dj 500
82. A tuning fork of frequency 90 is sounded and moved I, fb I,
towards an observer with a velocity equal to one-tenth the Fundamental
velocity of sound; the note heard by the observer will have a frequency
frequency: Fig. 13.51

(a) 100 (bj 90 (a) 1,2, I (b) I ,2,2


(cj 80 (d) 900 (c) 2, I ,2 (dj 2,2, I
83. A source of frequency 150 Hz is moving in the direction of 90. Two wave trains, with intensities I and 2I respectively
a person with a velocity of 110 mls. The frequency heard by arrive and superpose at a point A with opposite phases. The
the person will be (v;;; 330 mls) : amplitude of the superposed wave at A at that instant will
(a) 225 Hz (bj 200 Hz be:
(c) ISO Hz (d) 100 Hz (aj 51 (bj Jj
84. An object producing a pitch of 400 Hz approaches a (cj )(3 - 2-/2)1 (dj (-/2+Irf/
stationary person in a straight line with a velocity of 200
mls. Velocity of sound is 300 mls.The person will note a 9). A string is under tension so that its length is increased by..!.
change in frequency. as the'object flies past him equal to : n
of its original length. The ratio of fundamental frequency of
(aj 1440 Hz (bj 240 Hz longitudinal vibration and fundamenlal freque ncy of
(cj 1200 Hz (d) 960 Hz transverse vibration will be:
85. The difference between the apparent frequency o f a source
of sound as perceived by an observer during its approach
(aj '" I (b) .r., I
(c) I:.r;; (d) I: "z
and recession is 2% of the natural frequency ofthe source.
If the velocity of sound in air is 300 mis, the velocity of the 92. Two waves represented by,
source is : Yt = IOsin 200Ont,
(a) 6 mI,
(cj 1.5 mI,
(b) 3 mI,
Yz = 20sin ( 200Ont+~)
(dj 12 mI,
86. A radio wave of frequency 840 MHz is sent towards an arc superimposed at any point at a particular instant. The
aeroplane. The frequency of radio echo has a frequency 2.8 amplitude of the resultant wave is:
kHz more than the original frequency. The velocity of the (a) 200 (bj 30
aeroplane is:
(aj 3 kml' (bj 2 kml,
ICPMT J990J
(cj IOJs (dj IOn
93. A column of air at 57°C and a tuning fork produce 4 beats
(c) 4 kmI, (dj 0.5 kmI, per second when sounded together. When the temperature
(Hint: See solved Problem 55.) of the air column is gradually decreased to 9"C, the two
87. When an aeroplane moves at speeds higher than the velocity produce I beat per second. The frequency of the tuning fork
of sound in air, a loud bang is heard. This is because: .is:
(a) It explodes (aj 63.5 Hz (bj 36.5 Hz
(b) It produces shock wave which is received as the bang (cj 60 Hz (d) 30 Hz
(c) Its wings vibrate so violently that the bang is hearcd 94. A racing car moving towards a cliff sound its hom. The
(d) The normal engine noise undergoes a Doppler shift to driver observes that the sound reflected from the cliff has a
generate the bang p itch Olle octave higher than the actual sound of the hom. If
88. When an object moves with supersonic speed, the ratio of v is the velocity of sound, then the velocity of the car is:
the specd of object to that of sound is called: (aj vl2 (bj vl-/2
(a) I-number (b) Magic number (cj vl4 (dj vl3
(c) Rynolds number (d) Mach number
95. The fi rst overtone of a closed organ pipe oflength II has the 102. 41 tuning forks are arranged such that every fork gives 5
same frequency as the seoond hannonic of an open pipe of beals with the next. The last fork has a frequency that is
I double of the first. Frequency of the first fork is :
length 12 , The ratio ..lis:
I, (a) 400 (b) 205
(a) I (b) 0.75 (e) 2 10 (d) 200
(e) 1.25 (d) 1.5 103. A whistle emits sound waves of frequency 2000 Hz. These
sound waves reach a point' X' through two different paths
96. If '\ and 12 are the lengths of air column for two consecutive
such that path difference is 20 cm and there is silence at this
resonance position when a tuning fork of frequency f is
point.' Y' is a successive point at which also there is silence
sounded in a resonance tube, then velocity of sound in air is :
but the path difference is 35 cm. Speed of sound is:
(a) 2(1, -/,)f (b) 4/,f
(a) 340 mI, (b) 300 mI,
(e) 4/~f (d) I, ~/, (e) 380 mI, (d) 332m1s
104. A tuning fork of frequency 480 Hz produces 10 beats per
97. As beals are produced by the superposition of waves of second when sounded with a vibrating sonometer string.
slightly different frequencies: What must have been the frequency of the string if a slight
(a) frequency of the resultant wave is the difference of increase in tension produces fewer beats per second than
frequencies of superposing waves before? IAIEEE 2003)
(b) frequency of the resultant wave is the sum of (a) 480 Hz (b) 490 Hz
frequenc ies of superposing waves (d) 460 Hz
(e) 470 Hz
(c) frequency of the resultant wave changes with lime
lOS. Consider ten identical sources of sound all giving the same
(d) frequency of the res ultant wave and the frequency at frequency but having phase angles w hich are random. If the
which amplitude oscillates are unequal average intensity of each source is 10 , the average of
98. As beals are produced by th~ superposition of waves of rcsuhant intens ity I due to all these ten sources will be:
slightly different frequencies :
(a) /'/0 (b) /.100/ 0
(a) phase difference between waves at any given point
(e) 1.10/0 (d) I . M l o
does not change with time but it can be different at
different points 106. There arc three sources of sound of equal intensities with
(b) phase difference between waves at any given point frequencies 400, 401 and 402 Hz. What is the beat
does not change with time and it is also the same at all frequency heard ifaH are sounded simultaneously?
points (a) 3 (b) 2
(c) phase difference between waves at any given point (e) I (d) 0
changes with time
107. A tuning fork gives 4 beats with 50 cm length of a
(d) phase difference is zero at position where intensity of sonometer wire. If the length of wire is shortened by 1 em,
sound is minimum the number of beals is still the same. The frequency of the
99. Two sound waves superimpose to produce beats. Thesc fork is:
waves can be described as (a) 404 Hz (b) 396 Hz
YI '" 5sio 36On(t _...2....)
240
(e) 384 Hz (d) 400 Hz
108. Two organ pipes give 4 beats when sounded together at
and Y2 ",3sin 3521t(/--=-)
240
21°C. Calculate the number of beats at 121"C:
(a) 3.9 beats/second (b) 4 beals/second
Frequency of the resu ltant wave fonned is: (c) 4 .6 beats/second (d) 2.8 beals/second
(a) 178 Hz (b) 204 Hz · 109. A person carrying a whistle emitting continuously a note of
(e) 224 Hz (d) 148 Hz 212 Hz is runni ng towards a reflecting surface with a speed
100. In Q.99, frequency at which amplitude of resultant wave 18 kmlhr. Speed of sound in air is 345 mJsec. Number of
oscillates is: beats heard by him are:
(a) 6 (b) 120 (e) 2 (d) 4 (a) 6 (b) 4 (e) 8 (d) 3
101. In Q.99, the ratio of maximum and minimum intensities of 110. If seventh hannonic is set up in a string fixed on both ends,
sound when the beats are produced is: number of nodes and antinodes fonned are:
(a) 8 (b) 16 (e) 12 (d) 4 (a) 8, 7 (b) 7,7 (e) 8.9 (d) 9,8
SUPERPOSITION OF WAVES 641

Ill. A racing car moving towards a cliff sounds its hom. The If speed of sound in air is 340 mis, the distance in metre
driver observes that the sound reflected from the cliff has a separating two successive positions of minimum intensity
pitch one octave higher than the actual sound of the hom. If IS :
v is the velocity of sound, velocity of the car is : (.) I (b)
(a) v (b) ~ 2
2 .,fi
(e) ~ (d) 2
2
(e) v (d) ~
4 3 120. A standing wave having 3 nodes and 2 antinodes is formed
112. Equation of a stationary wave can be expressed as between two atoms having a distance 1.21 A between them.
Wavelength of the standing wave is :
y = 8sin cos 20nr
1tX
4 (.) 2.42 A (b) 1.21 A
Here x and yare in em and t in sec. (e) 3.63 A (d) 6.05 A
Amplitude of any of the progressive waves which 121. Vibrations of a string of length 100 em fixed at both ends
superimpose to form the given stationary wave is: are represented by the equation
(a) 8em (b) 4 cm y = 8Sin(;) cos (50nt)
(e) 6 em (d) 10 em
113. In Q.112, time period of any component progressive wave here x and yare in cm and I in seconds.
is : Maximum displacement at x "" 5 em is :
(a) to sec (b) 1 sec (a) 5.6 em (b) 7.2 em
(e) 0.1 sec (d) 3,ec (c) 4.4 em (d) 3.8 em
114. In Q.112, wavelength of any component progressive wave 122. In Q.121, antinodes are located at:
is: (.) 0,20,40,60,80,100 em
(a) 8 em (b) 6 em
(b) 10,30, 50,70,90 em
(e) 9 em (d) 12 em
(c) 20, 40,60,80 em

(a) 60 em/sec
,
11 5. In Q.112, speed of any component progressive wave is :
(b) 100 cin/sec
(d) none of the above
123. In Q.121, speed of the particles atx =IOcm and at t =0.1 sec
(e) 80cmlsec (d) 120cmlsec is :
11 6. In a stationary wave: 5
(a) strain is maximum at antinodes (a) zero (b) - eml'
2
(b) strain is maximum at nodes 2
(c) - cmls (d) 4 mI,
(c) strain is minimum at nodes 5
(d) strain is constant throughout 124. Displacement of a particle executing periodic motion is
ll7. At a certain instant, a stationary transverse wave is found to ·given by
have maximum kinetic energy. The appearance of string at y = 4cos 2 (t 12)sin (1000/)
that instant is :
This expression may be considered to be a result of
(a) sinusoidal with amplitude A 12
superposition of:
(b) sinusoidal with amplitude A
(a) two waves (b) three waves
(c) sinusoidal with amplitude A 1 3
(c) four waves (d) five waves
(d) straight line
125. Four simple hannonie vibrations given by
118. A stationary wave can be represented as
Yl = 8cos wI, Y2 ... 4cos(wt+1t/2)
y = asin (IOOt)cos (O.Olx )
Y3 "" 2cos(wt+n\ Y4 =cos(wt+31t/2)
where a and yare in millimetre, I in second and x in metre.
are superposed on each other. Resulting amplitude and
Velocity of wave is:
phase are respectively:
(.) 10 mI, (b) 10' mI,
(c) 10-2 mls (d) 10' mi.,
(a) J45 and tan -I ~ (b) 145 and tan - 1 ~
11 9. A sound source of frequency l70 Hz is placed near a wall.
A man walking from the source nonnally towards the wall
(c) ...fi5 and tan -I ..!. (d) .J7s ",d t.n - 1 ~
3
finds that there is a periodic rise and fall of sound intensity.
126. Two waves given by 134. X is an organ pipe closed at one end and vibrating in its
fundamental mode and Y is an organ pipe open at both ends
y\ ""Asio 21t evt _ x )
A and vibrating in its second overtone. Both pipes are in
resonance with the same tuning fork. Ratio of length of X to
and Y2 =Bsin 21t[(vt_x)+x'] that ofY is:
A
1 (b) 1
superimpose on each other. (a) -
3 2
Amplitude of resultant wave for x' = ')",/6 is : 1
~) A+B ~) A-B (e) - (d) 4
6
(e) ) A' +8' - AB (d) ,;rA
-O;'-+-B-O;2-+-A-B 135. Two closed organ pipes A and B have the same length but A
127. A tuning fork of frequency 200 Hz is in unison with a is wider than B. When thez vibr~te in the fundamental
sonometer wire. Number of beats heard pcr second when mode, their frequencies are A and B, then:
the tension is increased by one per cent is : 00 l=i ~) l<i
(a) 1 (b) 2 00 l>i ~) 1.i
(e) 4 (d) ~ 136. Frequencies ofthree tuning forks A,B and Chave a relation
2 nA >" B > "e' When the forks A and B are sounded
128. If an open organ pipe is sounded with a tuning fork having together, the number of beats produced is III • When A and C
frequency 256 Hz, resonance occurs at 35 em and 105 em. are sounded together, the number of beats produced is 11 2 ,
Velocity of sound is: Then the number of beats produced when Band Care
(a) 360 mig ~) 540 mig
sounded together is :

(e) 512m1s (d) none of these


129. Standing waves arc produced in 10m long stretched string.
If the string vibrates in 5 segments and wave velocity is 20 (c) liZ -n l (d) n1 -n z
mis, frequency is: 137. Fundamental frequency of a sonometer wire carrying a
(a) 2 Hz (b) 4 Hz block of mass lkg and density 1.8 gmlcc is 260. When the
block is completely immersed in a liquid of density 1.2
(e) 5 Hz (d) 10 Hz
gmlcc, then the new frequency will be:
130. A sonometer string and a tuning fork when sounded
together give 6 beats/sec, whether the length of sonometer (a) 150Hz (b) 300Hz
string is 95 cm or 100 em. Frequency ofthe tuning fork is : (e) 450 Hz (d) 75 Hz
(a) 262 Hz (b) 256 Hz 138. Linear density of a vibrating string is 10-4 kg/m. A
(c) 260 Hz (d) 234 Hz transverse wave is propagating on the string which is
described by the equation
131. Speed of sound in air is 333 mls. The fundamental frequency
of an open pipe is 333 Hz. The second overtone of the open y = 0.02sin (x + 30t)
organ pipe can be produced with a pipe of length: where x and yare in metres and t is in seconds.
(a) I.Om (b) l.5m Tension in the string is :
(e) 2.0 m (d) 0.5 m (a) 0.09 N (b) 0.36 N
132. Two wire of the same material and radii rand 2r (e) 0.9 N (d) 3.6 N
respectively are welded together end to end. The 139. When a vibrating tuning fork of frequency 512 Hz is held
combination is used as a sonometer wire and kept under above the mouth of a resonance tube of adjustable length,
tension T. The welded point is midway between' the two the first two successive positions of resonance occur when
bridges. When stationary waves are set up in the composite the lengths of the air columns are 15.4 cm and 48.6 em
wire, the joint is a node. Then the ratio of the number of respectively. Then the velocity of sound is:
loops formed in the thinner to thicker wire is :
(a) 512(48.6+ 15.4)cm/see
(a) 1: 2 (b) 1: 1
(b) 512(48 ..6~ 15.4)cm/sec
(e) 4:1 (d) 1.:4
(e) 2x 512(48.6 - 15.4) crrvsec
133. Standing waves are:
(d) 2x 512(48.6 + 15.4) cmlsec
(a) always transverse
140. In Q. 139, diameter of resonance tube is nearly:
(b) transverse or longitudinal
(a) 2em (b) 4cm
(c) always longitudinal
(c) 1.5 em (d) 3.2 em
(d) neither transverse nor longitudinal
141. A pipe closed at its bottom and half filled with water is 149 Bullets are fired at regular intervals of 14 s from an
vibrating in the fundamental mode. If, instead of water, it is annoured car A moving towards car B with a velocity of
half filled with another liquid of density greater than water, 108 kmlhr. At what intervals will an officer seated in B
frequency of fundamental mode will: and dashing with a speed of 72 km/hr towards A hear the
(a) Increase (b) decrease bullet shots (velocity of sound = 330 mls)?
(c) remain same (d) become zero (a) lOs (b) 12s
(c) 14 s
142. Fundamental frequency ofa closed pipe is 220 Hz. Ifith of ~
4 150. Two pulses in a stretched
the pipe is filled with water, the frequency of the first string whose centres are
~
overtone of the pipe now is :
(a) 220 Hz (b) 440 Hz
initially 8 cm apart are
moving towards each other
as shown in the figure. The
- -8cm-
Fig. 13.52
(e) 880Hz (d) 1760Hz
speed of each pulse is 2 cm/s. After 2 sec, the total energy of
143. A glass tube 1.52 em long and open at both ends is
the pulses will be: IIIT2001)
immersed vertically in a water tank completely. A tuning
(a) Zero
fork of600 Hz is vibrated and kept at the upper end of tube
and the tube is gradually raised out of water. Total number (b) Purely kinetic
of resonance heard before when the tube comes out ofwatcr (c) Purely potential
is: (take speed of sound, v = 330 mls) (d) Partly kinetic and partly potential
(a) 12 (b) 6 151. The ends ofa stretched wire oflength Lare fixed atx = Oand
(e) 8 (d) 4 x = L. In one experiment, the displacement of the wire is
144. Frequency of a vibrating wire is n. If tension is coubled, Y1 = A sin (nxl L)sin rot and energy is E1 and in another
density is halved and diameter is doubled, new frequency experiment its displacement is Y2 =sin (2nxl L)sin 2rot
will be : andenergyisE 2 . Then: IIIT2001]
(a) 2n (b) 4. (a) E2 =E 1 (b) E, =2E 1
(c) n (d) nl.fi (c) E2 =4E I (d) E, = 16E 1
145. Two wires made up of the same material are of equal 152. Two vibrating strings of the same material but lengths Land
lengths but there radii are in the ratio I: 2. On stretching 2Lhavc radii 2rand rrespectively. They are stretched under
each of these two strings by the same tension, the ratio the same tension. Both the strings vibrate in their
between the fundamental frequencies is: fundamental modes, the one of length L with frequency VI
(a) 2:1 (b) 1:2 and the other with frequency v2 . The ratio VI J v2 is given
(e) 4:1 (d)l:4 by: lIlT 20001
(a) 2 (b) 4
146. An organ pipe closed at one end has fundamental frequency
1500 Hz. Maximum number of overtones generated by this (e) 8 (d) 1
pipe which a normal person can hear is: 153. A string of length 0.4 m and mass 10-2 kg IS tightly
(a) 9 (b) 13 clamped at its ends. The tension in the string is 1.6 N.
(e) 12 (d) 6 Identical wave pulses are produced at one end at equal
intervals of time, !::.t. The minimum value of /).1 which
147. In an experiment it was found that a sonometer in its
allows constructive interference between successive pulses
fundamental mode of vibration and a tuning fork gave 5
is : lIlT 19981
beats when length of wire is 1.05 metrc or I metre. The
velocity of transverse waves in sonometer wire is : (a) 0.05 s (b) 0.10s
(a) 400 mls (b) 210 mls (c) 0.20 s (d) 0.40 s
(e) 420m/s (d) 450m/s 154. A 30 cm long string with linear density 0.650 g/m 3 is
148. Two tuning forks A and B are sounded together and 8 beats placed ncar a loudspeaker that is fed by an audio oscillator
per second are heard. A is in resonance with a 32 cm air of variable frequency having range 500 Hz to 1500 Hz. It is
column closed at one end and B is in resonance when length found that the string is set into oscillation only at the
of air column is increased by 1 cm. The frequencies offorks frequencies 880 Hz and 1320 Hz. The tension in the string
A and Bare: is :
(a) 264 Hz, 256 Hz (b) 272 Hz, 264 Hz (a) 90.6 N (b) 45.3 N
(c) 528 Hz, 520Hz (d) 520Hz, 512 Hz (e) '8 N (d) 135.9N
155. In a sonometer wire, the tcnsion is maintained by 162. A train moves towards a stationary observer with speed 34
suspending a 50.7 kg mass from the free end of the wire. mls. The train sounds a whistle and its frequency registered
The suspended mass has a volume of 0.0075 m3 . The by the observer is fi. If the train's speed is reduced to 17
fundamental frequency of the wire is 260 Hz. If the mis, the frequ ency registered is 12 , If the speed of sound is
suspended mass is completely submerged in water, the 340 mis, then the ratio!J... is: (liT 20001
fundamental frequency will become: I,
(,) 240 Hz (b) 220 Hz
(,) !.II (b) ~ (c) 2 (d) 12
(c) 230 Hz (d) 280 Hz 19 2 18
156. The frequency of the sound ofa car hom as perceived by an 163. In hydrogen spectrum the wavelength ofH (l line is 656 om,
observer towards whom the car is moving differ.; from the whereas in the spectrum of a distant galaxy, the Ha line
frequency of the hom by 2.5%. Assuming that the velocity wavelength is 706 nm. Estimated speed of the galaxy with
of sound in air is 320 mis, the velocity of ear is: respect to the earth is : (lIT 1999]
(,) 8 mls (b) 800 mls 8
(a) 2x 10 mls 7
(b) 2x 10 mls
(c) 7.5 mI, (d) 6 mls
(c) 2x 106 mls Cd) 2x 105 mls
157. A man is watching two trains, one leaving and the other
164. The fundamental frequency of a longitudinal vibration of a
coming with equal speeds of 4 mls; if they sound their
rod clamped at its cenlre is 1500 Hz. Ifthe mass of the rod is
whistles each of natural frequency 0(240 Hz, the number of
96 g the increase in its lotallenglh produced by a siretching
beals heard by the man (velocity of sound in air = 320 mls)
force of IO kilogram weight will be :
will be equal to :
(a) 0.011 cm (b) 0.096 cm
(,) 6 (b) 3 (c) Zeco (d) 12
(c) I em (d) 0.1 em
158. A train blowing its whistle moves with a constant velocity v
away from an observer on the ground. The ratio of the 165. A closed organ pipe which emits a fundamental note of 11
natural frequency of the whistle to that measured by the hertz has a wide hole made at one-third of its length from
observer is found to be 1.2. If the train is at rest and the the closed end. The lowest frequency of note that will now
observer moves away from it at the same velocity, this ratio be emitted will correspond to :
wouldbegiven by: lIlT 19931 (a) !!. hertz ( b) !!. hertz
(,) 0.51 (b) 1.25 2 3
(c) L52(d) 2.05 (c) 311 hertz (d) 2n hertz
159. Light shows many properties like those of sound. A star, 166. A source S emits h' •
which emits a line of wavelength 5000 A is receding from electromagnetic waves which
you in the line of sight with a velocity (1 / 100) of speed of are detected by the detector D
light. The wavelength ofthis line as observed by you will be : kept at a distance of:!.. from
(,) 4900 A (b) 4950 A 4
the source. The waves directly
(c) 5050 A (d) 5100 A rect:ivcd from Sand reflectcd
160. If n , ,n 2 and n 3 arc fundamental frequencies of the three by a layer d above the ground
segments into which a string is divided by balancing a are found to be in phase. If the
number of bridges below it, then the original fundamental
o 4 s
layer moves up by a distance Fig. 13.53
frequency of the string is given by : h "" O.O l d, the intcrfering
(a) ..In =.,J;;; +Fz +Fz waves are once again in phase. The wavelength of the
waves from the source is about :
(b) 11 = 11,+11 2 +11)
(,) 0.04 d (b) 0.08 d
I I I I
(c) - " " - + - + - (c) 0.02 d (d) 0.01 d
11 111 112 11)
167. In a standing wave pattern obtained in a tube filled with
(d) - I =-+-+-
I I I iodine, due to vibrations of frequency 800 cycle/sec, the
..In.,J;;;FzFz distance between eleven consecutive nodes is found 10 be I m,
when the temperature of iodine vapours is 3520(:. If the
161. There are Ihree sources of sound of equal intensity with
temperature is 127OC. the distance between consecutive nodes
frequencies 500, 502 and 504 Hz respectively. The number
is:
of beats heard per second is: [Dhanbad 19941
(a) 0.08 m (b) 0.072 m
(a) Zero (b) Two
Cc) Four (d) Eight (c) 1.25 m (d) ~127 xo'lm
352
SUPERPOSITION OF WAVES 645
168. A progressive sinusoidal wave of amplitude 2.5 em, (a) A hears a note of lower frequency
wavelength 30 em and frequency 30 Hz is superposed on a
(b) Number of beats heard by A is higher than that heard
similar wave travelling in the opposite direction to fonn a
by B
standing wave pattern. The magnitude of the maximum
· 0 f a partie
. 1e at x = -)em
. .IS : (c) The number of beats heard by B is 37 in 5 second
ve 1OClty
. 2 (d) Both A and B will hear the same number of beats every
(a) 900 emlsec (b) 942 emlsec second
(c) 10 emlsec (d) zero 174. Three resonant frequencies of a string are 90, 1SO and 210
169. A mass m is attached to the free cnd of a sonometer wire. Hz. If the length of the string is 80cm. What is Ihespeed of
the transverse wave in the string ?
The fundamental frequency of the wire is found to be 500
cycle/sec when the mass is in air. It becomes 480 cycle/sec (.) 45 ml, (b) 75 mI,
when the mass is immersed in a liquid and 460 cycle/sec (c) 48 mI' (d) 80 mI,
when the mass is immersed in water. The specific gravity of 175. A 40 cm wire having a mass 3.2 g and area of cross-section
the liquid is :
I mm 2 is stretched between two fixed supports 40.05 cm
(.) - _~(~5OO~)~'~ (b) 500-480 apart. In its fundamental. note the wire vibrates with a
(480)' _ (460)' 500 -460 frequency 220 Hz. The Young's modulus is :
(a) 1.98)(1011 N/m2 (b) 2.2xlO II N/m2
(c) ,,(5~0~0)c,'-=-~(4~8::o0)~' (500)' - (460)'
(d) (c) 3.96)( 101I N/m2 (d) 3.2)( 1011 N/m 2
(500)' -(460)' (500)' - (480)'
176. A railway engine whistling at a constant frequ ency moves
170. A uniform rod AD of length 0.4
with a constant speed. It goes past a stationary observer
metre and mass 1.2 kg is suspended
beside the railway track. Which of the following graphs
by two identical wires from a
best represents the variatiCin of frequency of the sound (1/)
ceiling as shown in Fig. 13.54.
Where should a mass of 4.8 kg be heard by the observer with time (I)?
suspended from the rod so that a
tuning fork of frequency 256 Hz is
in resonance with the first wire at A A '--- - - - - - - - ' B
'l fi Fig. 13.54
(a) "In,t (b) "IL, t
W h I e its Irst overtone resonates
with the second wire at B?
(a) 5 em from A
(b) 35 em from B
(c) "1/\ , t (d) "I A , t

177. Two observers A and B start from the pointO to move due E
(c) 20cmfromA and due W respectively at time t == 0. with the same cQnslant
(d) At the centre of mass of rod speed u mis, when a source, situated d metre due S of their
171. An open organ pipe of length L vibrates in its fundamental starting point 0 emits a continuous note of frequency 1/
mode. The pressure variation is maximum : hertz. The velocity of sound isv mls and there is no wind. In
respect of the sound received by the two observers, which
(a) At the two ends
of the following statements will be true?
(b) At the mid point of the pipe inside
(a) The apparent frequencies of the note as heard by the
(c) At distances U4 from either end inside the pipe two observers will be the same and given by
(d) At distances US from either end inside the pipe
172. A sounding body emitting a frequency of t SO Hz is dropped
from a height. During its fall under gravity it crosses a
n' =n[1 ~u' :.' 1
balloon moving with constant velocity of 2 m s - lone (b) The apparent frequency of the note as heard by A will
second after it started to fall. Find the number of beats heard be greater than that heard by B
by observer in the balloon at the moment he crosses the (c) The apparent frequency of the note as heard by A will
body. (Velocity of sound in air is 300 mls) be smaller than that heard by B
(.) 12 (b) 6 (d) The frequencies of the note heard by observers A and B
(c) 8 (d) 4 will be the same as the real frequency n hertz
173. Two persons A and B each carrying a source of sound of 178. A string of length f is stretched by I I 10 and the velocity of
frequency 500 Hz are standing a few metre apart. A starts transverse waves along it is V. When it is stretched by 1/20
moving towards B with a velocity of 4.5 mls. If the speed of (assuming Hooke's law is applicable), the velocity of
sound is 300 mis, which of the following statements are true? waves will become:
646 PHYSICS FOR COMPETITIONS - Vol. I

(a) V (b) .j2v 18S. Two organ pipes both closed at one end, have lengths I and
1+ 6/. If the velocity of sound in air isv, then the number of
V
(c) -/2 (d) 2V beats per second is: [BHU Mains 2007]
v v
(a) (b) 21
179. Two sound waves with wavelengths 5.0 m and 5.5 m 41
respectively, each propagate in a gas with velocity 330 mls.
We expect the following number of beats per second: (c) ~M (d) ~M
4/' 112
ICBSE20061
186. Two tuning for:':s w hen sounded together g ive 8 beats/so
(a) 12 (b) 0
When A is sounded with air column of length 37.5 em
(c) I (d) 6 closed at one end resonance occurs in its fundamental
180. A whistle producing sound waves of frequencies 9500 Hz mode. B gives reson'ance with air column oflength 38.5 em
and above is approaching n stationary person with speed and closed at one end in its fundamental mode. The
vms-I , The velocity of sound in air is 300 III s - I . If the frequencies of tuning forks are: [BHU Mains 2007]
person can hear frequencies uplO a maximum of 10,000 Hz, (a) 300 Hz, 300 1·lz (b) 300 Hz, 308 Hz
the maximum value ofv upto which he can hear the whistle (c) 308 Hz, 308 Hz (d) 30S Hz, 300 Hz
is: IAIEEE20061
187. An engine moving towards a wall with a velocity 50 m s -I
(a) 15ms - i (b) 30 m s- 1 emits a note 1.2 kHz. Speed of sound in air is 350 m s-I.
(e) lsnms- I (d) (IS/-/2)m,-' The frequency of the note after reflection from the wall as
heard by the driver of the engine is :
181. A string is stretched between fixed points separated by 75.0
em. It is observed to have resonant frequencies of 420 Hz [Karnataka CET2007]
and 315 Hz. There are no other resonant frequencies (a) 2.4 kHz (b) 0.24 icHz
between these two. Then the lowest resonant frequency of (c) 1.6 kHz (d) 1.2 kHz
this string is: [AIEEE 20061
188. A glass tube is open at both the ends. A tuning fork 01
(a) 1050 Hz (b) 10.5 Hz frequency f resonates with the air column inside the tube.
(c) 105 Hz (d) 1.05 Hz Now the tube is placed vertically inside water so that hall
182. A massless rod is suspended by two A•.,.·................, C thc length of Ihe nlbe is filled with water. Now the ail
identical strings AB and CD of column inside the tube is in unison with another fork 01
cquallength. A block of mass In is frequency l' . Then: [Karnataka CET 2007]
, u'pended from point 0 such that B ' ......f'..: :( •.;:-",1 . D (a) 1'" I (b) 1'"4/
BO is equal to 'x'. Further, it is _
observed that the frequency of 1st
... x (c) /' " 2/ (d) 1'"1
m . 2
hannonic (fundamental frequency) Fig. 13.55
in AB is equal to 2nd hannonic 189. A wave travelling along positive xMaxis is given by y "'" A
frequency in CD. Then,length of BO is : lilT 20061 sill (w/-kr} If i\ is reflected from rigid boundary such Iha'
80% amplitude is re flected , then equation ofrcflected wav(
(a) LiS (b) 4LiS is: IUttarakhand 2007J
(c) 3Li4 (d) Li4 (8) y=Asin(rol + kr) (b) y"",-0.8Asin(rot+kr)
183. When two tuning forks (fork I and fork 2) are sounded (c) y "", O.8Asin(rot+kx) (d) y"A,in(oot + QSkx)
simultaneously, 4 beats per second are heard. Now, some 190. The difference between the apparent frequency of a sourct
tape is attached on the prong of the fork 2. When the tuning of sound as perceived by the observer during its approact
forks arc sounded again, 6 beats per second are heard. If the and recession is 2% of the frequency of the source. If Iht
frequency of fork I is 200 Hz, then what was the original speed of sound in air is 300 ms - 1, the velocity of the sourc(
frequencyoffork 21 [UPSEE20071
IS: IPbPMT2007:
(a) 200 Hz (b) 202 Hz
(a) 1.51\1s - 1 (b) I2m s - i
(c) 196 Hz (d) 204 Hz
(c) 6 m s - 1 (d) 3ms- 1
A n observer moves towards a stationary source of sound,
with a velocity one-fifth of the velocity of sound. What is 19 1. A tuni ng fork of frequency 340 Hz is vibrated just abov!
the percentage increase in the apparent frequency? the tube of 120 em height. Water is poured slowly in th,
tube. What is the minimum height of water necessary fo
IUPSEE20071
the resonance? (speed of sound in air = 340 m/s)
(a) zero (b) 0.5%
IPbPMT2007
(c) 5% (d) 20%
(a) 45 em (b) 30 e:n (c) 40 cm Cd) 25 cm
192. An open pipe is suddenly closed with the result that the 199. In order to double the frequency of the fundamental note
emitted by a stretched string, the length is reduced to ~ th of
second overtone of the closed pipe is found to be higher in
frequency by 100 Hz than the fi rst overtone of the initial 4
pipe. Fundamental frequency of the open pipe will be; the original length and the tension is changed. The factor by
[PET (Kerala) 20071 which the tension is to be changed, is:
Ca) 100 Hz Cb) 150 Hz (CMC Ludhiana 20081
Ce) 200 Hz Cd) 300 Hz 3
Ca) - Cb) ~ Ce) ~ Cd) 2
193. A uniform wire of linear density 0.004 kg/m 3 when
8 3 9 4
stretched between two rigid supports with a tension 200. A closed organ pipe oflength 20 cm is sounded with tuning
fork in resonance. What is the frequency of tuning fork?
3.6x 10 2 N, resonates with a frequency 420 Hz. The next
harmonic frequency with which the wire resonates is 490 Cv~332m1s) (JIPMERZ0081
Hz. Length of the wire in metres is; [Pb. PMET20081 Ca) 300 Hz Cb) 350 Hz
Ca) 1.41 Cb) 2.14 (c) 375 Hz Cd) 415 Hz
Ce) 2.41 Cd) 3. 14 201. An engine is moving 011 a circular path of radius 100 m with
194. Two tuning forksP andQwhen set vibrating, give4 beats/s. a speed of 20 m/s. What will be the frequency observed by
Ifa prong of the fork P is filed, the beats are reduced to 2/s. an observer standing stationary at the centre of circular path
when the engine blows a whistle of frequency 500 Hz?
What is the frequency of P, if that ofQis 250 Hz?
IBVPZ0081
IDCE20081
(a) Morc than 500 Hz (b) Less than 500 Hz
Ca) 246 Hz Cb) 250 Hz
(c) 254 Hz
Ce) 500 Hz (d) No sound is heard
Cd) 252 Hz
202. A car sounding its hom at 480 Hz moves towards a high
195. Two waves represented by y=asin(wt-la:) and
wall at a speed of20 m/s. If the speed of sound is 340 mis,
y = acos (rot -la:) are superposed. Resultant wave will
the frequency ofthe reflected sound heard by the girl sitting
have an amplitude; [UPSEE 2008}
in the carwill be closest to: rUPSEE2009j
Ca) a Cb) .J2n Ce) 2n Cd) zem (a) 540 Hz Cb) 524 Hz
196. Two strings of the same material and the same area of Ce) 568 Hz Cd) 480 Hz
cross~section are used in sonometer experiment. One is
203. A tuning fork produces 4 beats per second when sounded
loaded with 12 kg and the other with 3 kg. Fundamental
with a sonometer wire of vibrating length 48 cm. It
frequency of the first string is equal 10 the first overtone of produces 4 beats per second also when the vibrating length
the second string. If the length of the second string is 100 is 50 cm. What is the frequency of the tuning fork?
cm, length of the first siring is: IEAMCET 20081
IAMU20091
(a) 300 em (b) 200 em
Ca) 196 Hz Cb) 284 Hz
(c) 100 cm Cd) 50 em
Ce) 375 Hz Cd) 460 Hz
197. A wave travelling along the x ~axis is described by the
204. In a closed organ pipe, the 1st resonance occurs at 50 cm. At
equation y(x,t)= o.005cos (ax - Pi} If the wavelength and
what length of pipe, the 2nd resonance will occur?
the time period of the wave are 0.08 m and 2.0 s,
(OrissaJEE 2009J
respectively, then a. and J3 in appropriate units are:
(a) 150 cm Cb) 50 em
IAIEEE Z0081
(c) 100 cm Cd) 200 em
Ca) u ~ 25.00a,~ ~ a Cb) n ~ 0.08,~ ~ 2.0
a a 205. Three successive resonant frequencies of a string arc 90,
ISO, a~d 210 Hz. If the length of the string is 80 em, the
a
Cd) u ~ 12.50a,~ ~ ~ speed of the transverse wave in the string is :
2.0
Ca) 96 mI, (b) 48 mls
198. While measuring the speed of sound by perfOiming a
Ce) 24 mI, Cd) 32 mls
resonance column experiment, a student gets the first
resonancc condition at a column length of 18 COl during 206. To decrease the fundamental frequency of stretched string
winter. Repeating the samc cxpcriment during summer, she fixed at both ends one might:
measurcs the column length to be x cm for the second (a) Increase its tension
resonance. Then: IAIEEE 2008) (b) Increase its wave velocity
(a) 18> x (b) x> 54 (c) Increase its length
(c) 54>x>36 (d) 36 > x> 18 (d) Decrease its linear mass density
207. A wire having a linear density of 0.05 g1cm is stretched 209. A string of mass 111 is fixed at both ends. The fundamental
between two rigid supports with a tension of 4.5x 107 dyne. tone oscillation are excited in the string with angular
It is observed that the wire resonates at a frequency of 420 frequency roand max.imum displacement amplitude A. Find
Hz. The next higher frequency at which the same wire the total energy contained in the string.
resonates is 490 Hz. The length of the wire is : . 1
7
(a) _mro Z A 2 (b) ..!.moi A2
(a) -m (b) 1m 2 4
15
15
(c) .!.mrozA z (d) .!.mro2 A2
(c) 3m (d) -m 6 8
7 210. There is a set of four tuning forks, one with the lowest
208. A roller skater carrying a portable stereo skates at constant frequency vibrating al 550 Hz. By using any two tuning
speed to an observer at rest. Which of the following forks at a time, the following beat frequencies are heard: I,
accurately represents how the frequency perceived by the 2,3 , 5. 7, 8. The possible frequenc ies of the other three
observer changes with time as skater approaches and then forks arc (in Hz) :
recedes the observer? (a) 552,553,560
(b) 557,558,560

i~ (b)J~
(c) 552, 553, 558
(d) 551, 553, 558
(a)
I 2 11. A conveyor belt moves to the right with speed v '" 300
time In/min. A pie-man puts pies on the belt at a rate of 20 per
time
minute while walking with speed 50 mlmin towards a

~~ J~
receiver at the other end. The frequency with which they are
, , received by the stationary receiver is :
(c)
I (d) (a) 26.67/minute (b) 30 Iminute
(c) 22.22 /minute (d) 24/minute
time time

ANSWERS

t. <a> 2. (a) 3. (c) 4. (e) 5. (e) 6. (a) 7. (e) 8. (d) 9. (a) 10. (e) 11. (e) 12. (b)
13. (a) 14. (e) 15. (a) 16. (a) 17. (e) 18. (e) 19. (e) 20. (b) 21 . (e) 22. (e) 23. (d) 24. (b)
25. (b) 26. (b) 27. (a) 28. (e) 29. (b) 30. (e) 31. (e) 32. (b) 33. (b) 34. (e) 35. (d) 36. (e)
37. (a) 38. (b) 39. (b) 40. (e) 41. (a) 42. (a) 43. (a) 44. (e) 45. (e) 46. (e) 47. (b) 48. (d)
49. (d) 50. (e) 51. (b) 52. (d) 53. (b) 54. (e) 55. (b) 56. (d) 57. (b) 58. (b) 59. (b) 60. '(b)
61. (a) 62. (a) 63. (a) 64. (a) 65. (e) 66. (b) 67. (b) ... (0) 69. (a) 70. (e) 71. (a) 72. (b)
73. (e) 74. (a) 75. (b) 76. (e) 77. (b) 78. (a) 79. (a) SO. (b) 81. (d) 82. (a) 83. (a) 84. (d)
85. (b) 86. (d) 87. (b) 88. (d) 89. (e) 90. (c) 91. (b) 92. (e) 93. (b) 94. (d) 95. (b) 96. (a)
97. (d) 98. (e) 99. (a) 100. (e) 101. (b) 102. (d) 103. (b) 104. (e) 105. (e) 106. (b) 107. (b) J08. (e)
109. (e) 110 (a) 111. (d) 112. (b) lB. (e) 114. (a) 115. (e) 116. (b) 117. (d) 118. (b) 119. (b) 120. (b)
121. (a) 122. (b) 123. (a) 124. (b) 125. (b) 126. (d) 127. (a) 128. (a) 129. (e) 130. (d) 131. (d) 132. (a)
133. (b) 134. (e) 135. (b) 136. (o) 137. (a) 138. (a) 139. (e) 140. (b) 141. (e) 142. (e) 143. (b) 144. (e)
145. (a) 146. (d) 147. (b) 148. (a) 149. (b) 150. (b) 151. (e) 152. (d) 153. (b) l:i4. (b) ISS. (a) 156. (a)
157. (a) 158. (b) 159. (e) 160. (e) 161. (e) 162. (d) 163. (b) 164. (a) 165. (e) 166. (e) 167. (a) 168. (e)
169. (e) 170. (a) 171. (b) 171. (a) 173. (e) 174. (e) 175. (a) 176. (b) 177. (a) 178. (e) 179. (d) 180. (a)
181. (e) 182. (a) 183. (e) 184. (d) 185. (e) 186. (d) 187. (e) 188. (a) 189. (b) 190. (d) 191. (e) 192. (e)
193. (b) 194. (a) 195. (b) 1%. (e) 197. (a) 198. (b) 199. (d) 200. (d) 201. (e) 202. (a) 203. (a) 204. (a)
205. (a) 206. (e) 207. (d) 208. (b) 209. (b) 210. (d~ 211. Cd)
[B) More than One Choice is Correct 8. Velocity of sound in air is 320 mls. A pipe closed at one end
has a length of 1 m. Neglecting end correction, the air
I. In which case will there be no change in phase of column in the pipe can resonate for sound of frequency:
displacement-wave?
lilT 19901
(a) Wave propagating from denser to rarer medium
(a) 80 Hz (b) 240 Hz
(b) Wave propagating from rarer to denser medium
(c) 320 Hz (d) 400 Hz
(e) Wave is reflected from a denser boundary
9. An air column in a pipe. which is closed at one end, is in
(d) Wave is reflected from a rarer boundary resonance with a vibrating tuning fork of frequency 264
2. In case ofinterfcrence oftwo waves each of intensity Jo. Hz. If u =3JO mis, the length of the column in em is :
the intensity at a point of constructive interference will IMNR 19901
be:
(a) 3 1.25 (b) 62.50
(a) 4/0 for coherent sources (c) 93.75 (d) 125
(b) 2/0 for coherent sources
10. In case of Doppler effect for sound:
(e) 410 for incoherent sources
(a) Change in frequency is independent of distance
Cd) 210 for incoherent sources berween source and observer
[Hint: See point (4) § 13.3} (b) Change in frequency depends on the fact that source is
3. The equation y = A sin 2 (oot -kx)represenIS a wave: moving towards the observer or observer is moving
towards the source
(a) With amplitude A and frequency (ro/2n)
(c) Frequency heard will change if wind starts blowing
(b) With amplitude (A I 2) and frequency (00/ n)
between stationary source and observer
(e) Which is hannonic-progressive
(d) Passenger s itting in a moving train will hear different
Cd) Which is complex-periodic pitch than produced by the engine of the same train
4. Which of the following waves will give rise to stationary II. If a satellite moving in an elliptic orbit relative to the eanh
waves? transmits radio-signals towards earth, the frequency
(a) Asin(w/-kx), Asin(wt+kx) received by a tracking station on the eanh:
(b) Asin(wt+kx),-Asin(wr +kx) (a) Can be higher (b) Can be lower
(c) Asin(wr+kx),-Asin(wt-kx) (c) Can be equal (d) Cannot remain constant
(d) Asin 2 (wt - kx),Bsin J(wt -fa) 12. If the shift in a star light is towards red end:
S. In case of superposition of waves (at x = q): (a) The star is approaching the eanh
YI =4sin(1026nt) and Yz = 2sin(lO I41tl ) (b) The star is receding from the earth
(a) The frequency of resulting wave is 510 Hz (c) The apparent frequency is lesser than actual
(b) The amplitude of resulting wave varies at frequency 3 Hz (d) The apparent wavelength is lesser than actual
(c) The frequency of beals is 6 Hz 13. Which of the following statements are tnte about a
stationary wave?
(d) The ratio of maximum to minimum intensity is 9
(a) Different particles move with same amplitude
6. Two identical straight wires are stretched so as to produce 6
beats per sec when vibrating simultaneously. On changing (b) All particles between two successive nodes reach their
extreme positions together in phase
the tension slightly in one of them the beat frequency
remains unchanged. Denoting by TJ ,T2 , the higher and the (c) Displacement and velocity nodes coexist
lower initial tensions in the strings, then it could be said that (d) Velocity nodes and pressure nodes coexist
while making the above changes in tension: lIlT 19911 14. Standing waves can be ~roduccd: lilT 19991
(a) Tz was decreased (b) T] was increased (a) On a string clamped at both the ends
(c) T2 was increased (d) T] was decreased (b) On a string clamped at one end and free at the other
7. At displacement-nodes : (c) When incident wave gets reflected from a wall
(a) Displacement is minimum (d) When two identical waves with a phase difference ofn
(b) Pressure is max imum are mov ing in the same direction
(c) Sound is maximum I S. A simple hannonic progressive wave is incident nonnally
Cd) Particle speed is minimum on a reflecting surface and gets reflected normally back
with the reflected amplitude 80% of the incident wave.
When the incident wave and refl ected wave are superposed, whi stles moving towards the right with a speed of25 mls.
then we have: The velocity of sound in air is 350 m/s. Assume there is no
(a) No energy transfer takes place along the direction of wind. Then which of the following statemenlS are true?
propagation of direct wave (a) The apparent frequency of whistle B as heard by A is
(b) Energy transfer in the direction of direct wave 444 Hz approximately
(c) Energy transfer in the direction of reflected wave (b) The apparent frequency of B as heard by the observer
is 469 Hz approximfllely
(d) Positions of maximum and minimum amplitudes
alternately on the path of superposition (c) The difference in the apparent frequencies of A and B
16. If two progressive waves represented by as heard by the observer is 4.5 Hz
y\ = asin (WI - lce)and Y2 = acos (WI -kx)aresuperposed, (d) The apparent frequencies of the whistles of each other
the resultant wave at x has amplitude: as heard by A and B are the same
(a) a (b) 2. 21. Two notes A and B, sounded together, produce 2 beats per
sec. Notes Band C sounded together produce 3 beats per
(c) .J2a (d) Zew sec. The notes A and C separately produce the same number
[Hlnl: Here the phase difference between two waves travelling in ofbcats with a standard tuning fork of 456 Hz. The possible
Ai
the same direction is It 12. Apply Rl ;;: A;l + 2A, A2 cosO) frequency ofthe note B is :
17. Two elastic waves represented respectively by the (,) 228 Hz (b) 456 Hz

equations Yl = aSin ( (Ot + 1t7) and Y2 =2a Sin ( rot -1t7) (c) 456.5 Hz (d) 458.5 liz
22. The equation of a transverse wave travelling on a string is
superimpose at a point A to produce an intensity maximum y "" 2cos [tt( o.5x - 200t)l where x and yare in centimetre
at that point. Which of the following statements will be true and t is in sec. Which of the following statements are fal se
in respect of this superposition? in respect of this wave?
(a) The intensity maximum at A will be (measured in (a) The velocity of the wave in the string is 4 mls
terms of the intensity of the first wave as unit) 9 units (b) The phase difference between two points on the wave
(b) The amplitude of the superposed wave at the point A separated by a distance of 0.5 m is 1t /2
will have a minimum of magnitude a (c) The two consecutive points on the wave which have
(c) There will always be a node at the point A their displacements numerically equal but opposite in
sense will be separated by the distance of 2 em
(d) Taking 1= 0 at the instant when the intensity is
(d) The displacement of the point distant 100 cm from the
maximum at A, the minimum at A will occur at I =!...,
4 source will be zero at t = lOOs
where Tis the period of either wave 23. A string is under tension and stationary vibrations are
18. An organ pipe closed at one end resonates with a tuning produced in it A ,B ,C ,D are positions of consecutive nodes:
fork of frequenc ies 180 Hz and 300 Hz. It will also resonate (a) The distance AC =~
with tuning forks of frequencies: 2
(.) 360 Hz (b) 420 Hz (b) The phase difference between a vibrating point on AB
(c) 480 Hz (d) 540 Hz and another on BC is zero
19. The apparent frequency of a sound wave as heard by an (c) The amplitudes of vibration of different points
observer is 10% more than the actual frequency. If the between A and the midpoint of A and B are different
velocity ofsound in air is 330 mls: (d) When every point of AB is having maximum
(a) The source may be moving towards the observer with a displacement then every point of BC is having
velocity of30 ms- I maximum displacement in opposite direction
(b) The source may be moving towards the observer with a 24. A string is fixed at both ends and transverse wave wid.
amplitnde ao are excited choose the correct statement.
velocity of 33 m s-I
(a) Energy of oscillations on the string is directly
(c) The observer may be moving towards the source with a
proportional to tension in the string
velocity of30 ms - I
(b) Energy of oscil lations in II th overtone will be equal to
(d) The observer may be moving towards the source with a ,, 2 times of the in first overtone
velocity 003 ms- I
(c) Average kinetic energy of string (over one period) is
20. Two whistles A and Beach have a frequency of500 Hz. A is half of the oscillation energy
stationary and B is moving towards the right (away from A)
(d) Average kinetic energy of string (over one period) is
at a speed of 50 mls. An observer is between the two
full of the oscillation energy
25. The vibrations ofa string ofJenglh 60 COl fixcd at both ends (CJ Assertion-Reason Type Questions
are represented by the equation y = 4sin (961rt) ,
1tx cos
(a) If both A and R a" true and R is the correct
15
explanation of A.
where x and yare in cm and t in seconds. Then:
(a) maximum displacement of a particle at a point at x = 5 (b) If both A and R are true but R is nol correct
explanation of A.
cmis4cm
(b) third node is fonned at 30 cm from one end of the string (c) If A is true but R is false.
(c) the velocity ofa particle at x = 7.5 cm and t = 0.25 s is (d) If A is fa lse but R is true.
zero (e) Ifboth A and R are false.
(d) equations of the component waves whose 1. (A): Two sound sources placed close to each other emit
superpoSitIOn gives the above equation are sound waves of same frequency. Sound waves from

YI = 2sin (~ - 96)'[/ )and Y2 = 2sin( ~ + 96m ) the two superimpose while propagating forward.
This will result in a non·unifonn distribution of
26. A car moves towards a hill with speedVC ' It blows a horn of intensity of sound.
frequency fwhich is heard by an observer following the (R): Superposition of sound waves of same frequency
car with speed Vo' The speed of sound in air is V : and travelling in opposite directions results in the

(a) The wavelength of sound reaching the hill is '7 2. (A):


formation of stationary waves.
In case of beats, intensity of sound at some positions
v-v in space remains maximum and, at others, it remains
(b) The wavelength of sound reaching the hill is __C_ minimum.
J (R): Beats are fonned due to superposition of sound
(c) The wavelength of sound of hom directly reaching the waves of unequal frequenc ies.
observer is __O
_
v +v
3. (A): Sound pmdueed by an open organ pipe is more
J melodious than that produced by a closed organ
(d) The beat frequency observed by the observer is pIpe.
2VC (V +VO )J (R): Air can flow in a better way in an open organ pipe.
V
2
-V2 4. (A): Two tuning forks having frequencies 410 Hz and
27. When beats are produced by two progressive waves of 524 Hz are kept close and made to vibrate. Beats
nearly the same frequency, which one of the following will not be heard.
(is/are) incorrect? (R): Sound waves superimpose only when the
(a) The particles vibrate simple hannonically, with the frequencies of superposing waves are equal or
frequency equal to the difference in the component nearly equal.
frequencies 5. (A): A sound source S emitting waves of frequency 512
(b) The amplitude of vibration at any point changes simple Hz moves towards a stationary listener with a
harmonically with a frequency equal to the difference constant speed v. Apparent frequency of waves as
in the frequenc ies of the two waves found by the listener is 520 Hz. If now the same
(c) The frequency of beats depends upon the position, source is placed at rest and the listener moves
where the observer is towards the source with constant speed v (same as
(d) The frequency of beats changes as the time progresses the speed of source in the earlier case), apparent
frequency as found by the listener will be 520 Hz.
ANSWERS
(R): Doppler effect depends 011 the value of relative
speed. If magnitude of relative speed in any two
I . (a), (b) and (d); 2. (a) and (d); 3. (b) and (d); 4. (a) and
situations is same, magnitude of Doppler effect will
(c); 5. All; 6. (c) and (d); 7. All; 8. (a), (b) and (d); 9. (a)
also be the same.
and (c); 10. (a) and (b); 11. All; 12. (b) and (c); 13. (b) and
(c); 14. (a) and (c); 15.(b)and(d); 16.(c);17.(a), (b)and ANSWERS
(d); 18. (b) and (d); 19. (a) and (d); 20. (c) ond (d); 21. (a),
I. (b) 2. (d) 3. (c) 4. (c) 5. (e)
(c) and (d); 22. All; 23. (c) and (d); 24. (a) and (c); 25. (b),
(c) and (d); 26. (b)and (d); 27. (a), (c) and (d).
[DJ Integer Type Questions 2. Column - I shows different sets of standing waves in a
string of length L whose ends are fixed or free according to
1. S I and S 2 are two coherent A B
currents sources of radiations
separated by distance 100.25
-
• •
100.25 A
FIg. 13.55
respective figure and Column - II shows possible equations
for them where symbols have usual meaning.
,... where A, is the wave length Column - I Column - II
of radiation. S I and S 2 in phase by ~. A and B arc two (a) /'-..... x=L (P) A sm'Lcoswt
. 1tX
2 x-o ~
y=
points on the linejoi~ing S I and S 2 as shown in Fig. 13.55.
The ratio of amplitudes of source S 1 and S 2 are in ratio 1 : (b) ~ (q)
y = A sm 2ru:
. -cos WI
2. The ratio of intensity at A to that of B ( ;; ) is 1 : x, find
x-o x=L L

the value of x.
2. A cart is moving towards a fixed target with speed ofv m/s.
(e)
I===-;;-L
x=o
(e) 1tX
Y = A cos -cos wt
2L

A gun mounted on cart fires bullets at the rate of 5 per


second, If number of the bullets received at target per
second is 6 then find the value of 11 if bullets are moving
(d)
P----JX=L
x=O
ru
(s) y = A cos -cos
L
wt

with 11 v.
3. Referring to the above question, match the Column-I and
Column-II.
1. 9 2. 6
Column - I Column - II
IE) Match the Columns (a) Frequency of resultant (p) O.8m
wave in solution (i) in

I
l. String of length i is fastened
between two supports as shown
in the Fig. 13.56. The end of the
string can be made free or fixed x = 0 x= 1
I Hz will be

(b) Maximum amplitude of (q) 250


the resultant wave m
at x=O and x=l. A travelling Fig. 13.56 situation (ii) will be
wave, Yi = Asin(kx-wt) is generated at x=o' It gets
reflected from the other end. The reflected wave is given by (e) In situation 0), in a time (e) 4
Yr =Asin(kx+wt+~) These two waves superimpose to of 1 sec, intensity of
from standing wave whose equation is represented as sound at a point will
y = f(x,t~ Symbols have their usual meanings. Match become maximum on
Column - I with Column - II. how many occasions?

(d) Ratio of maximum and (s) zero


Column - I Column - II minimum intensities in
(a) Boundary is free alx = 0 (p) ~ · n situation (ii) will be
and fixed at x = i
4. Match the Column-J and Column-II.
(b) Boundary is fixed at (q) ~.O
x=Oandatx=i Column - [ Column - II

(e) Boundary is free atx "" 0 (e) k=- nn (a) Consider a string that is (P) These are longitudinal
n belongs to fixed at both ends. If stationary waves
andatx=l 1 '
integer the string is plucked,
stationary waves will
(d) Boundary is fixed at (s) k = (2ft - I) ,II be setup in the string.
belongs
x""Oand free atx=! 21
(b) In organ pipes, (q) Fundamental frequency
to integer
stationary waves can be of the vibrations will
(t) Y = 2A sin kxcos W( setup in the air column also depend on
temperature
SUPERPOSITION OF WAVES 653

(e) Frequency of the firs 1«) Fundamental frequency


3. With one end of the string at x = 0. positions of the
overtone in a pipe 01 of the vibrations will antinodes can be exp ~cssed as :
length 40 em, that is also depend on •5
(a) x=n-em, where II ",0. 1,2 •. ..
open at both cds. will be humidity
(take speed of sound in 2.
(b) x=lI-em, wheren =0, 1,2, ...
air" 330 mls) 5
(d) On the surface of (s) 825 Hz •
ec) X=II-cm, where 1/'" I . 3, 5, ...
motion of moon, 5
frequency
fundamental
of the
note

(d) x=lI-cm,wherell""I,3,5, .. .
10
emitted by a pipe of 4. Amplitude of simple hannonic motion of a point on the
length 20 em that is string that is located at x = 1.8 cm will be :
open at both ends, will (a) 3.3 em (b) 6.7 cm (c) 4.9 cm (d) 2.6 cm
be
5. Fig. 13.57 (c) shows the standing wave pattern at t =Odue
to superposition of waves given by Yt and Y2 in Fig. 13.57
(a) and (b). In Fig. 13.57 (c), N is a mode and A anantinode.
1. a-+ p ,Sj b-,lo p,r,l; c-+ q,r; d-+ q,s,t At this instant say t = 0, instantaneous velocity of points on
the string
2. a ...... q;b-+p;C-+T;d-+s )

3. a-+ q; b-+ p; c-+ r; d-+ r y,


4. a-+ q; b-+ p,q,fj c-+ s; d -+ does not match
(a)

IF) Linked Comprehension Type •


Paragraph - 1 y,
Consider a standing wave fonned on a string. It results due
to the superposition of two waves travelling in opposite (bl
direetions. The waves are travelling along the length of the A
string (x -direction) and displacements of elements on the
string are along the y -direction. Individual equations of the
two waves can be expressed as
Yt ::::6(cm) sin [5 (rad/cm)x -4 (rad/s)t]
Y2 '" 6(cm) sin [5 (rad Icm)x+4 (rad / s)t] A A
Here x and yare in cm. (e)
Answer the following questions. FIg. 13.57
I. Maximum value of y~position coordinate in the simple (a) is different for different points
hannonic motion of an element of the string that is located (b) is zcrO for all points
at an antinode will be :
(e) changes with the position of the point
(a) ±6cm (h) ±8em
Cd) is constant but not equal to zero for all points
(c) ± 12 cm (d)±3em
2. Ifone end of the string is atx =0. positions of the nodes can Paragraph - 2
be described as ; A vertical pipe open at both ends is partially submerged in

(a) x=ll-cm.whercll=0.1.2 •...
5
water. A nming fork of unknown frequency is placed near
the top of the pipe and made to vibrate. The pipe can be
moved up and down and thus length of air column in the
2.
(b) x=n cm.whcren"'0. 1. 2•... pipe can be adjusted. For definite lengths of air column in
S the pipe, standing waves will be setup as a result of
n superposition of sound waves travelling in opposite
(c) x:=n cm. wheren=I ,3, 5, ...
S directions. Smallest value of length of air column, for

(d) x=n 10 em, where,,'" 1,3,5, '"
which sound intensity is maximum. is 10 em.
[take speed of sound, v ,., 344 mls)
Answer the following questions. 12. How far is the person from the pole when he hears a
6. The air column here is closed at one end because the surface minimum in sound intensity a second time?
ofwaier acts as a wall. Which of the fo llowing is correct? (a) 5.6 m (b) 7.8 m
(a) At the closed end of the air column, there is a (e) 12.4 m (d) 17.6 m
displacement node and also a pressure node 13. As the person walks toward the pole, the total number of
(b) At the closed end of the air column, there is a times that the person hears a minimum in sound intensity
displacement node and a pressure antinode will be :
(e) At the closed end of the air column, there is a (a) 2 (b) 8
displacement antinodc and a pressure node (e) 4 (d) 6
(d) At the closed end of the air column, there is a 14. At some instant, when the person is at a distance 4 m from
displacement antinode and also a pressure antinode the pole, the wave function (at the person's location) that
7. Frequency of the tuning fork is : describes the waves coming from the lower speaker is
(a) 1072 Hz (b) 940 Hz y=Acos(kx - rot)
(e) 860 Hz (d) 533 Hz where A is the amplitude
8. Length of air column for second resonance will be: ro = 21tv with v = 680 Hz (given)
(a) 30 em (b) 45 em and k = 21t
(e) 20 em (d) SO em ~
9. Length of air column for third resonance will be : wave function (at the person's location) that described
(a) 30 em (b) 45 em waves coming from the upper speaker can be expressed as :
(e) 20 cm (d) SO em (a) y=Acos(kx -rot + 1t)
10. Frequency of the second overtone is: (b) y=Acos(kx-rot + 1t /2)
(a) 3400 Hz (b) 2500 Hz (e) y=A eos(kx - rot+21t)
(e) 4300 Hz (d) 1720 Hz (d) Y= ACOS( kx-rot+ 3;)
Paragraph - 3
An oscillator offrequency 680 Hz drives two speakers. The Paragraph - 4
speakers are fixed on a vertical pole at a distance 3 m from As shown in the Fig. 13.59, a vibrating tuning fork of
each other as shown in the Fig. 13.58. A person whose frequency 512 Hz is moving towards the wall with a speed
height is almost the same as that of the lower speaker walks 2 nVs. Take speed of sound v = 340 mls.
toward the lower speaker in a direction perpendicular to the
pole. Assuming that there is no reflection of sound from the
ground and speed of sound is v = 340 mls.
Y_4rn1S
I
Fig. 13.59
Answer the following questions.
3m
15. Suppose that a listener is located at rest between the tuning

1 Person approaches
along this di ~ection
fork and the wall. Number of beats heard by the listener per
sec will be :
(a) 4 (b) 3
(e) 0 (d) I
Fig. 13.58 16. If the listener is at rest and located such that the tuning fork
Answer the following questions. is moving between the listener and the wall, number of
beats heard by the listener per sec will be nearly:
J I. As the person walks toward the pole, his distance fro m the
pole when he fi rst hears a minimum in sound intensity is (a) 0 (b) 6
nearly: (e) 8 (d) 4
(a) 14.6m (b) 17.9m 17. If the listener, along with the source, is moving towards the
(c) 10. 1 m (d) 22.4 m wall with the same speed, i.e., 2 mis, such that the source
remains between the listener and the wall, number of beats frequency (highest frequency - lowest frequen cy) is thus
heard by the listener per sec will be: 320 Hz. The speed of sound in still air is 340 mls.
(a) 4 (b) 8 (e) 0 (d) 6 lilT JEE 20071
18. If the listener, along with the sourcc, is moving towards the Answer the fo llowing questions.
wall with the same speed, i. e., 2 mis, such that he (listener) 23. The speed of sound of the whistle is:
remains between the source and the wall. number of beats (a) 340 mls for passengers in A and 310 mls for passengers
heard by him will be : inB
(a) 2 (b) 6 (b) 360 m/s for passengers in A and 310m/s for passengers
(e) 8 (d) 4 inB
19. If the sound waves produced by the tuning fork can be (c) 310 mls for pas:>{;Jlgers in A and 360 mls for passengers
expressed as Y =0.2 (cm) sin (Io; -oot), where k = 2J't and inB
). (d) 340 mls for passengers in both the trains
ro=2n!(f=512 Hz), maximum value of amplitude in a 24. The distribution of the sound intensity of the whistle as
beat wilt be : observed by the passengers in train A is best represented by:
(a) 0.4 cm (b) 0.6 em
(c) 0.8 cm
Paragraph ~ 5
(d) 0.2 em
(b)l~
Two waves Yj =A cos(0.51lX - IOO1tt) f1 12 F
and Y2 = A cos (046nx - 921tt)
are travelling in a pipe placed alongx -axis. 11IT20061
Answer the following questions.
20. Find the number of times intensity is maximum in time
interval of I sec:
25. The spread of frequency as observed by the passengers in
(a) 4 (b) 6
train B is :
(e) 8 (d) 10
(a) 310 Hz (b) 330 Hz
21. Find wave velocity of louder sound :
(e) 350 Hz (d) 290 Hz
(a) lOO mis (b) 192m1s
(e) 200 mls (d) 96 mls Paragraph -7
22. Find the number of times Yj + Y2 = Oat x = 0 in I sec: A hannonic transverse wave is travelling along a stretched
string towards positive x -axis. It reflects at its end at 0
(a) 100 (b) 46
which is free to oscillate. Assume that the amplitude of
(e) 192 (d) 96 incident and reflected waves are equal. The incident wave
Paragraph ~ 6 equation is given as Y = 4sin (a-3t\ where yis in em, xis
in meters and I is in seconds.
Two trains A and B are moving with speeds 20 mls and 30

' \ U~y.. _,
mls respectively in the same direction on the same straight
track. with B ahead of A. The engines are at the front ends. L
-
The engine of train A blows a long whistle. "-.../ 0

Fig. 13.61

Answer the following questions.

J L
Ij 12 Frequency
26. The reflected wave equation is :
(a) y=4sin(2y +3t) (b) y=-4sin (1\,+3t)
(c) y ::: 4eos(lx+3t) (d) y= -4cQ.'.(2\'+3J)
Fig. 13.60
27 . Equation of the resultant wave fonned after superposition
Assume that the sound of the whistle is composed of of incident and reflected waves is:
components varying in frequency from" = 800 Hz 10 (a) y::: 8sin 2\"cos 3t (b) y "" lxcos 3t
12 = 1120Hz, as shown in the Fig. 13.60. The spread in the (c) y = 8cos2ysin3f (d) y= 2\'sin 3r
656 PHYSICS FOR COMPETITIONS - Vol. 1

28. Find the least distance from the reflecting end 0 at which (ii) 3sin (5x-o.5t) + 4cos (5x-o.5t)
node is fonned : (iii) y =4cos 3 (5x-o.5t)
"4
(8) - meters
4
"
(b) -em [Ans. (a) (ii), (b) (i), (e) (iii)]
8. Specify the equation which may represent the phenomenon
"
(c) - meters
2
(d) "2
-em of (a) beats, (b) interference and (c) standing waves (i) 2A
cos (6.rol 2) t cos (<0/ - kx\ (ii) 2A cos kxsin <0/, (iii) 2A cos
W 2)sin (oo,-kx+ 0)
[Hlnl: In ca.se of interference, beats and stationary waves, the
amplitude of resultant wave is constant, periodic in time and
periodic in poSition respectively.]
[Ans. (a) (O, (b) (iii), (c) (ii)]
9. Which of the following two waves on superposition can
give rise to (a) heats, (b) stationary waves, (e) interference?
y\ =Acos 21t[fil+(x! A.)]
Y2 =Acos21t[fit+(xJl.. 1 )+1t]
Y3 = Aeos21t[J2f+(xJl.. z )]
:lIld Y4 =Acos21t[J2f-(x ! A. 2 )]
IGJ Subjective Type Questions (Ans. (a) Yt and >'3, (b) Y3 and Y4' (e) Yt and >'2J
I. A harmonic wave is given by 10. What factors detennine lite pitch of a tuning fork?
y ::: o.Ssin 167t [/ -(x 140)] (Ans. Fora tuning forkJ3-F '" {d I LZ')../Yip, so the pitch ofruning
If for normal incidence its amplitude falls to 60% on fork depends on the thickness of prong in the direction of
reflection from a plane boundary, write the equation of vibNltion (f 0: d~ length of the prong f 0: (I I LZ'j, elasticity of
reflected wave if the boundary is (a) rigid, (b) soft. 1n.1terial of fork (f 0: Jf) and density of material of fork
[Ans. (a) y = -O.3 sin 16n[ / + (x! 40n (f « II .JP)I
(b) y = n3 sin 16,,[ t + (x 140))) II. State the laws of transverse vibration of string and define
2. Docs the law of conservation of energy hold good in case of interval.
interference of waves? Explain. [Hint: Interval mcans ratio of two frequencies.J
[Hlnl: See § 13.3 (3)J 12. What is the maximum possible wavelength of standing
[Ans. Yes] waves in a I m long string (a) ifit is fixed at both ends, (b) if
3. Two identical travelling waves moving in the same its one end is free while the other fixed, (c) if it is plucked in
direction aTC out of phase by $ radians. What is the the middle when fixed at both ends, (d) ifit is touched in the
amplitude of the combined wave in terms of the common middle when clamped at both ends?
amplitude A of the two combining waves? [Ans. (a) 2 m, (b) 4 m, (c) 2 m, (d) 1 m]
[Ans. 2A cos(q,/2)] 13. In case of vibration of two taut strings, strings are identical
4. Show that the resultant of two travelling waves of the same in all respects with length of one say B being 4 times that of
amplitude and frequency that travel in opposite d irection is A. Which frequencies of A will match with those ofB iffirst
a stationary wave. eight hannonics are considered?
5. What are beats and how can they be produced? Find the (Ans. First andsccond hannonics of A matc h with 4th and 8th of B
expression for beat frequency . respectively.)

6. Distinguish between interference and beals. 14. Sketch standing wave patterns for fundamental and second
overtone for the vibrations of air column in a tube (a) closed
(Ans. In beats at a given position, intensity varies periodically
with time with periodicity T "'- [I I (Ji - iz)]while in interfercnce at one end, (b) open at both ends.
at a given time, intensity varies periodically with position with 15. If oil of density higher than water is used in a resonance
periodicity (A).] rube, how will the frequency change?
7. Given below are some functions of x and t which represent [Ans. The frequency will not change as in resonance tube the air
the displacement of transverse or longitudinal elastic column vibrates and not the liquid. Here liquid surface acts as a
waves. Sttlte whieh of these may represent (a) harmonie movable rigid boundary for reflection of sound waves whose
travelling wavc, (b) standing wave and (e) eomplex position can be changed by increasing or decreasing the amount of
periodic wave liquid in the tube.]

(i) y = 2cos3xsin(IOt)
SUPERPOSITION OF WAVES 657

16. If the fundamental frequency of an organ pipe is 150 Hz, the depth of the sea ifin a sonar experiment acoustic echo
what is the frequency of (a) second harmonic if it is closed from the sea-bed is heard after 2 sec?
at one end, (b) second harmonic if it is open at both ends, (Ans. 1.4 Ion]
(c) second overtone ifit is closed at one end and Cd) second 24. What is the minimum di stance of sound reflector fro m a
overtone if it is open at both ends? person for hearing echo of his own bi-syllabic sound ifhe
[AilS. (a) 2nd hamlonic docs not exist, (b) 300 Hz, (c) 750 Hz, (d) can utter at most 5 syllabies per sec and the velocity of
450 Hz) .sound is 330 ntis?
17. Does the change in frequency due to Doppler effect depend [Ans, 66 m)
(a) on distance between source and observer, (b) on the fact 25. A sound wave of 40 cm wavelength enters the tube as
that souree is moving towards the listener or listener is shown in Fig. 13.62. What must be the smallest radius r
moving towards the source? such that a minimum will be heard at the detector?

rL:&
[Ans. (a) No (it is a misconception), (b) In case of sound yes,
while in case of light no]
\ 8. A whistle emitting a sound of frequency 440 Hz.is tied to a
string of 1.5 m length and rotated with an angular velocity $ ==:::::...:~~~=
Fig. 13.62
D
of20 rad s -I in the horizontal plane. Calculate the range of
frequencies heard by an observer stationed at a large [HlDt: For min l!. '" (2n - I)(A 12) with l!. '" n:r - 2r]
distance from the whistle. [lIT 1996J [Ans. 17.5 em]
[Hint: Follow solved Problem 46] 26. A set of25 tuning forks is arranged in a series of decreasing
(Ans. l nm == 484 Hz and Inrin = 403.3 Hz) frequencies. Each fork gives 3 beats with the succeeding
one. The fi rst fork is the octave of the lasl. Calculate the
19. A 2000 Hzsiren and a person arc both at rest with respect to frequency of the first and the 16th tuning fork.
earlh. What frequency does the person hear if the wind is
[Hint: In ease of A.P., II til term .. a + (n -1)d]
blowing at 12 ntis (a) from source to observer and (b) from
observer to source? [Ans, 144 Hz and 99 Hz]
[Hint: Sec § 13.8,4 (c») 27. You are given four tuning forks; the lowest frequency of the
fork is 300 Hz. By striking two tuning forks at a time, 1, 2,
[Ans. (a) 2 kHz, (b) 2 kHz1
3, 5, 7 and 8 Hz beat frequencies arc heard. What are the
20. Sound waves of frequency f arc sent at speed v from a possible frequencies of the o ther three forks?
stationary transmitter and arc received back at the site of
(Ans. 30 1, 303 and 308 Hz)
transmitter from a distant object moving towards the
28. You have five tuning forks that vibrate at different
transmitter with speed II. Show that the frequency of the
frequ encies. By using the forks two at a time, what is the (a)
reflected waves received by the transmitter will be given by
minimum number and (b) maximum number of different
fA f["2"]
p
=
v-u
beat frequencies that you can produce?
(Hint: (a)n!l llx(lI-r)! (b) II!/r!x (lI-r)! ]
while beat frequcncy due to superposition of transmitted [Ans. (a) Four, (b) Ten)
and reflected waves will be
29. A I m long steel wire has mass 20 g and is stretched with a
force of 800 N. What arc the frequencies of its fundamental
mode of vibration and the next three higher modes?
(Ans. 100,200,300 and 400 Hz]
Note: This is thc principle of sonar and radar discussed in § 13.8 (7).
30. A string is stretched between fi xed supports separated by
2 1. What will you conclude if the spectral line of wavelength 75.0 cm. It is observed to have resonant frequencies of3 15
4000 A in the spectrum of light from a star is found to be and 420 Hz and no other resonant frequencies between
displaced from its normal position towards red end of the these two. (a) What is the lowest resonant frequency for this
spectrum by 2 A? string? (b) What is the wave speed for this string?
[Ans. The star is receding from earth with speed of ISO kmls.} [Ans. (a) 105 Hz, (b) 157.5 Il"lIs)
22. What do you think will happen if a source moves at a speed 3 1. In a sonometer wire, the tension is ma intained by
greater than that of sound? W ill Doppler fonnu la hold? suspending a 50.7 kg mass from the free end of the wire.
[Ans. Shock waves are produced. No, as it is valid only ifvs < v] The suspended mass has a volume of 0.0075 m 3 . The
fundamental frequency of vibration of the wire is 260 Hz.
23. The elastic constant and density of sea water are What will be the fundam ental freq uency if the mass i£
respectively 2.2x 109 N /m2 and 1.1 x 10 3 kg/m 3 . What is completely submerged in water?
650

What will be the fundamental frequency if the mass is 40. A column of air and a tuning fork produce 4 beats per sec
completely submerged in water? when sounded together. The tuning fork gives the lower
(Hine: See solved Problem 20) note. The temperature of air is 15"<:. When the temperature
fall s to IO"C the two produce 3 beats per sec. Find rhe
(Ans. 240 Hz]
frequency of the fork. IMNR 19921
32. The length of a sonometer wire is 0.75 m and density
(Hint: See solved Problem 41)
9x 103 kg/m 3 . Itean bear a stressor &lx 10 8 N/m2 without
exceeding the elastic limit. What is the fundamental (Ans. I 10.6 Hz]
frequency that can be produced in the wire? (lIT 1990) 41. A man is travelling along a straight line joining two
[Ans. 200 Hz)
stationary identical sources each of frequency 1 kHz. Find
his speed ifhe hears 5 beats per sec. Velocity of sound '"
33. A wire of density 9 g/cm 3 is stretched between two clamps llO mi•.
tOO em apart. While subjected to an extension of 0.05 em,
[Hlnt:See solved Problem 49]
what is the lowest frequency of transverse vibrations in the
wire, assuming Young's modulus of the material to be [Ans. 0.825 mls]
9)( 1011 dyne/cm 2? 42. State whether the following statement is true or false giving
reason in brief: "A source of sound with frequency 256 H2
[Ans. 2SJi =35.35 Hz] is moving with a velocity v towards a wall and an observer
34. A long string of length 1.5 m is made of steel. The tension in is stationary between the source and the wall. When the
it produces an elastic strain of 1%. Calculate its observer is between the source and the wall he will heal
fundamental frequen cy jf density and elasticity of material beats."
of the wire are 7.7x 10 3 kglm 3 and 2.2x lOll N fm2 (Hint: See ease (A) in solved Problem 51J
respectively. IMNR 1992) (Ans. FalseJ
IAns. 178.174 HzJ 43. A siren emitting a sound of I kHz moves away from 8
35. Two wires of radii rand 2r respectively are welded together stationary observer towards a cliff at a specd of 10 mfs.
end to end. This combination is used as a sonometer wire Calculate the frequency of the sound echoed off the clift
and is kept under tension T. The welded point is midway (speed of sound = 330 mfs). Will there be any beal
between the two bridges. What would be the ratio of the frequency?
number of the loops fonned in the wires such that the joint {lIlnt: See case (B) in solved Problem 5 I]
is a node when stationary vibrations are set up in the wire?
[Ans. 12 - 1031.0 Hz; N::: 60.4 Hz (as fi ::: 970.6 Hz) but thl
[Hint: See solved Problem 26]
human ear will not be able to hear beats of this frequency.]
[Ans. I : 2]
44. A bus is moving towards a huge wall with a velocity of!
36. The speed of sound in air is 340 mls. Calculate the m/s. The driver sounds a hom of frequency 200 Hz. What il
fundamental frequency and the first and second harmonics the frequency of the beats heard by a passenger of the bus i:
for a pipe 85 cm long and open at both ends. the speed of sound in air is 342 mis? lilT 19941
[Ans. 200 Hz, 200 Hz and 400 Hz]
[Hint: See case (C) in solved Problem 51]
37. A well with vertical sides and water at the bottom resonates [Ans. 5.9 Hz]
at 7.00 Hz and at no lower frequency. The air in the well has a
45. How fast would you have to go through a red light to have i
density of 1.10 kglm J and bulk modulus of1.33x lOs N /m 2 .
appear green if the wavelengths of red and green light art
How deep is the well?
respectively 620 nm and 540 nm? Is it possible to achievi
(Hint: Treat the well as a closed organ pipe.] thi s speed on earth?
IAns. 12.4 mJ (Ans. 4 . lxl0~ kmls, which is about 3650 times the eSoCll!"
38. A tunnel leading straight through a hill grealiy amplifies velocity from earth (11.2 kmfs)! So there is no question 0
tones at 135 and 138 Hz. Find the shortest Icngth of the achieving it on the earth.]
tunnel if velocity of sound in air is 330 mls. 46. Microwaves which travel with the speed of light an
(Hint: Treat the tunnel as open organ pipe.] reflected from a distant aeroplane approaching the waVI
[Ans. 55 mJ source radar. It is found that when the reflected waves arl
39. Weak back~ground noise from a classroom set up the beat against the waves radiated from the source, the bea
fundamental stationary wave in a card~board tube of length frequency is 990 Hz. If the microwaves are 0.1 m il
80 cm with two open ends. What frequency do you hear wavelength, what is the approaching speed of the aeroplane~
from the tube (a) if you jam your ear against one end? (b) if [HInt: Repeat solved Problem 55 to prove v = (c 12)[6/1 f]
you move your ear away enough so that the tube has two I I
open cnds? Take v ;;; 320 mls. Now as c= /').., v '" ZAN = '2 xO.1 x990 - 49.5 mls]
[Ans. (a) 100 Hz, (b) 200 Hz] [Ans. 49.5 mls]
47. A source of sound revolving in a circle of radius 15 m is them varies linearly from T] to T2,? The velocity of sound
emitting a signal of frequency 200 Hz. It completes one propagation in air is equal to U = a.,.,fi, where a. is a
revolution in 3 seconds. Calculate the maximum and constant.
minimum frequencies of the signal heard at a point 30 m 21
from the centre of the circle. (Speed of sound = 330 mls) IAn.,I= r. r . 1
a(v'T] + v'T2 )
(Roorkee 2000]
55. A source of sonic oscillations with frequency Vo and a
[Hint: Follow solved Problem 46.]
receiver are located on the same nonnal to the wall. Both
[Ans. 221.0 Hz, 182.6 Hz] the source and the receiver are stationary and the wall
48. Two airplanes, one flying east and the other due west. arc recedes from source with velocity u. Find the beat
on a near collision course separated by 15 km when the frequency registered by the receiver. The velocity of sound
pilot of onc plane travelling at 900 km/hr, observes the is equal to v.
other on his Doppler radar. The radar unit emits 2u,
[Ans.~J
electromagnetic waves of frequency 3x 1010 Hz. The radar u+v
readout indicates that the other plane's speed is 750 kmlhr. 56. A source emitting sound of frequency 180 Hz is placed ,
Detennine the frequency received by the pilot's radar. infront ofan obstacle at a distance of2 m from it. A detector
[Ans. 4f = 9.2 x 104 Hz, f =30.000926 GHzJ is also placed infront of the obstacle at the same distance
49. The driver of a car travelling at 100 kmlhr towards a cliff from it. (i) Find the minimum distance between the source
briefly sounds the horn. Exactly one second later he hears and the detector for which the detector detects a maximum
the echo and notes that its frequency is 840 Hz. How far of sound. (ii) How much farther to the right must the
from the cliff was the car when the driver sounded the hom obstacle be moved if the two waves are to be out of phase by
and what is the frequency of the horn'? 180<>7 (Speed of sound in air = 360 mls)
[Ans. 183 m, 713 Hz] [Ans. (i)x = 3 m; (ii) 0.625 m]
50. Ocean waves move toward the beach with a speed of 8.9 57. Receiver R] and R2 and also
mls and a crest to crest separation of 15.0 m. You are in a source S producing sound at a Rj $ R2
small boat anchored off the shore. You now lift anchor and frequency of Vo = 1000 Hz are
head out to sea at a speed of 15 mls. What is the frequency arranged near a stationary wall in
of the waves you observe? the order indicated in the figure . Fig. 13.63
The receivers are stationary, while
[Ans. 1.59 Hz]
the source moves towards wall ataspeedofu= 8.5 mls. The
51. A string with a mass density of 4x 10-3 kglm is under a speed of sound is v = 340 mls. (a) Which of the receivers
tension of 360 N and is fixed at both ends. One of its will register beats? (b) What is the frequency of these
resonance frequencies is 375 Hz. The next higher beats'?
resonance frequency is 450 Hz : [Ans. (a) Only receiver Rl ; (b) .6.v= 50 Hz]
(a) What is the fundamental frequency of the string'?
58. A string 25 cm long and having a mass of 2.5 g is under
(b) Which hannonics are the ones given'? tension. A pipe closed at one end is 40 cm long. When the
(c) What is the length of the string'? string is set vibrating in its first overtone and the air in the
[Ans. (a) 75 Hz; (b) 5th, 6th; (c) 2 m] pipe in its fundamental frequency, 8 beats per second are
52. A 50 cm long wire fixed at both ends vibrates with a heard. It is observed that decreasing the tension in the string
fundamental frequency fo when the tension is 50 N. If the decreases the beat frequency. If the speed of sound in air is
tension is increased to 60 N, the fundamental frequency 320 mis, find the tension in the string.
increases by 5 Hz and a further increase in tension to 70 N [Ans. 27.04 N]
results in a fundamental frequency of (fo + 9.6) Hz. 59. Two sources of soundS ] andS 2 p
Detennine the mass of the wire. vibrate at same frequency and are
[Ans. 9.1 gJ in phase (Fig. 13.64). The
53. Three successive resonance frequencies in an organ pipe intensity of sound detected at a e a
are 1310, 1834 and 2358 Hz. (a) Is the pipe closed at one point P as shown in the figure is $j $2
end or open at both ends? (b) What is the fundamental e
10 , If equals 45°, what will be Fig. 13.64
frequency? (c) What is the length of the pipe? Take velocity the intensity of sound detected at
of sound 340 mls. this point if one of the sources is switched off?
[Ans. (a) Closed at one end; (b) 242 Hz; (c) 0.32 mJ [Ans·Jo /4]
54. How long will it take sound waves to travel the distance I =
60. A source of sonic oscillations with frequency Vo 1700 Hz
between the points A and B if the air temperature between and a receiver are located at the same point. At the moment
t =0 the source starts receding from the receiver with the same tuning fork is 11; when it is immersed in a liquid
constant accelerationf1. = 10 mIs 2, Assuming the velocity the length is '2' Calculate the specific gravity of the solid
of sound to be equal to v = 340 mis, find the oscillation and that of the liquid.
frequency registered by the stationary receiver at t = lOs I P-P
after the start of motion. [Ans. ds = -'--2,dl ""
, I-I]
2--1-]
I-I]
[Ans.13.14Hz]
64. A wire when stretched by the weight of a solid, gives a
61. A detector receives waves from three sources of fundamental frequency v, when the temperature is changed
frequencies 498 Hz, 500 Hz and 502 Hz respectively. The
by llT. Find the change in frequency oftransverse vibration
!l!llplitude of each wave is A :
of the wire if the temperature falls down by llT, if Young's
II,' . (i) Find the beats frequency detected by the receiver if it modulus of the wire is Y and the coefficient of linear
can measure intensities greater than 4A 2. expansion is a..
(ii) Also, find the actual beats frequency. [Ans. v = (v +J} ~nU~T ]
(iii) What is time for which the detector remains idle in one n 21 p
second? 65. A wire when stretched by the weight of a solid, gives a
[Ans. (i) 2 Hz; (ii) 4 Hz; (iii) 3: 51 frequency ofV; when the solid is immersed in water it gives
3 a frequency y, and when immersed in liquid it gives a
,
62. An" observer standing at the bed
. . of a railroad hears the frequency o-ry z. Calculate the specific gravity of the solid
whist1~ of.the locomptiye of tqe tr~in that ~shes past him. and that of the liquid.
, y{~en..~he train is approaching rthe observer, the frequency vZ v 2 _v 2
of the 'whistle sound is vI' while when it has I!assed the [Ans. ds = -'--2; d f =--z--t]
V -v] v - v]
observer, the frequency is v 2' Determine the sp~~d of the
"
train and find the whistle frequency when the observer 66. A source of oscillation S is fixed to "'5 T;;; st -W]' rn!
i moves together with the train. the bed of a river whose water v
v -v ' I flows with a velocity v; upstream
,
[Ans. Speed of Irainvr '" _'_ _2 v, where v is the velocity of
and downstream there are fixed R, • S
• R2

v] +v2 .
2vv (also to the river bed) two receivers
sound,-'-'.J R, and R z (Fig. 13.65). The source JUS! .a)j£d las X 242
v] +v2
generates oscillations whose Fig. 13.65
63. A certain length' of a wire when loaded with the weight ofa frequency is vO' What frequencies do receivers RI and R z
solid vibrates in unison with a tuning fork. When the weight register?
is immersed in water the length of the wire resonating with [Ans. Vo for both receivers.]

-, I;

"
:, .

,.
"
and Expansion
§ 14.1 Temperature temperature is about 10 8 K (in fusion test reactor)
Temperature is defined as degree a/hotness or coldness of while lowest 10-8 K (achieved in 1990 through
a body and its measurement thermometry. nuclear spin cooling). Theory has established that 0 K
Regarding temperature it is worth nOling that: can never be achieved practically.
(1) Temperature is one of seven fundamental quantities (8) Production and measurement of very low temperature
with dimensions [9]. It is a scalar physical quantity (below boiling point of liquid nitrogen, i.e.• 77 K) is
with SI unit kelvin (K). called 'cryogenics' while measurement of very high
(2) When heat is given to a body and its state does not temperatures (> 1000 K) is called 'pyrometry'.
change, the temperature of body rises and if heat is (9) Temperature of the core of sun is 10 7 K while of its
taken from a body its temperature falls , i.e., surface 6000 K. Nonnal temperature of human body
temperature can be regarded as the effect of calise is 310.15 K (= 37°C = 98.6°F) while NTP implies
'heal',
273.15 K (= O"C = 32'F).
(3) Temperature detennines the direction of flow of heat,
as in thermal conduction, heat flows from a body at § 14.2 Scales of Temperature
higher temperature (9 H) to a body at lower Though SI unit of temperature is kelvin which is defined
temperature (9 l.) till both acquire the same common as (1/273.16)th part of thermodynamic temperature of triple
e e e e
temperature such that L < < II similar to the point of water (temperature at wi)ich ice, water and water
flow of water from higher level 10 lower level or vapours co-exist). There are various temperature scales such as
charge from higher potential to lower potential, i. e., Celsius. Fahrenheit, Reaumur, Rankine, etc.
temperature determines the thermal slate of a body In nearly all countries of the world the Celsius scale
whether it can give or receive heat. (fonnerly called centigrade scale) is used for aU popular and
(4) Two bodies are said 10 be in thermal equilibrium if conunercial and most scientific measurements. Conversion
and only if Ihey are al same temperature. In this from one scale to the other may be made by using the identity:
Celsius Fahrenheit Kelvin New
situation heat in the two bodies mayor may not be
equal. BP of water Tu
(5) According "to kinetic theory of gases, temperature, a
macroscopic physical quantity (as it can be perceived
T,
and measured directly) is a measure of average
translational kinetic energy of a molecule .
(microscopic physical quantity), i. e., •n
TK oc (KE)molccule
(6) Although the temperature of a body can be raised
without limit, it cannol be lowered without limit and
theoretically limiting low temperature is. taken to be 273.16
zero of the Kelvin scale (i. e., no negative temperature FP otwater 273.15
on Kelvin scale is possible).
(7) Though when the universe was created some 10 10 Different Scales of Temperature
Fig. 14.1
year ago, its temperature was about 10 39 K which at
present is about 3 K. The highest possible laboratory
• For Rellumur scale TL - Oaud Tu - 80 while for Rankine scale TL - 492 and Tu ·672
Tc -0 TF -32 TK -273.15 Tz -TL of thermometric property atO K, 273.16 K and TK K is
.... (i)
)()() -0 = 212 -32 = 373.15 -273.15 = Tu - TL D, X rr and X respectively, linear variation of
thermometric property with temperature requires:
[where Z is any temperature scale· with lower and upper fixed
O=aO+b; 273.16=aX tr +b and TK =aX +b
points TL and Tu respectively].
TK X
The change in temperature on different scales will be So that --=--,
related by 273.16 X,.,.
6.Tc =ATF "" aTK ::: dTz
....(ii) i. e., ....(iv)
TK = (27l.l6)[..£]K
100 180 100 Tu - TL X T,
Here it is worth noting that: (4) It has been found experimentally that temperature
(1) In case of Celsius or Fahrenheit scales a given recorded by thermometer depends on thermometric
temperature is expressed by °C or OF respectively substance used and its thermometric property. So
while change in temperature by C" or P, e.g., 20°C different types of thermometers or even same type
means a temperature of 20 degree on Celsius scale thermometers with different thermometric substances
while 20c0 will mean that temperature on Celsius give slightly different values of same temperature.
scale (whatever it was) is changed by 20 degrees. We This difference in readings is smallest among constant
don't use a degree mark in reporting kelvin volume gas thermometers using different gases. This
temperatures, i. e., it is written as 273 K and not is why constant volume gas thermometer is used as
273°K. Distinction in nomenclature is made between standard thermometer and its scale gas scale. So for it,
temperature and temperature difference. Thus we say
that boiling point of nitrogen is 77 K and temperature TK =273.16["!"'-] K ....(v)
PT,
ofa body is raised (or lowered) by 77 K.
(2) Formerly in old thermometry, two arbitrarily fixed Further it has also been established that the variation
points ice and steam points (viz., f.pt. and b.pt. of in reading of constant volume gas thermometers
water at I atm.) are taken to define the temperature reduces as we reduce the amount of gas (and hence
scale, e.g., in Celsius scale f.pt. of water is assumed to pressure) and in the limit PTr , tending to zero all
be ODe while b.pt. l000c and the temperature interval constant volume gas thermometers give the same
between these is divided into IOOequal parts. So if the reading, i.e., temperaNce scale defined as
thermometric property at temperatures OOC, lOire
and Tc De is X 0' X 100 and X respectively, the linear TK = lim (273.16)["!"'-]K .... (vi)
pr,. ...o PTr
variation'" of thermometric property with temperature
requires: coincides with ideal gas scale or absolute
O=aX O +b; 100=aX"100 + b and Tc = aX" +b thermodynamical scale of temperature which is
independent of property of any individual substance.
So that Tc-O= X-Xo
100-0 X 100 -Xo Question I. The absolute temperature (KelVin scale) TK
is related to temperature TC on Celsius scale by the relation,
or Tc =
x-x 0 x 1000e .... (iii) Tc =TK -273.15
X 100 -=Xo
Why do we have 273.15 in this relation and not273.16?
The thennometric property X may be length of a liquid Answer: As triple point of water on Celsius scale is
in a capillary (mercury thermometer), pressure of a gas O.OlOC (and not DOC) and on Kelvin scale 273.16 and the size
at constant volume (constant volwne gas thennometer), of degree on the two scales is same, so
volume of a gas at constant pressure (constant pressure
Tc -0.01 =TK -273.16
gas thermometer) or resistance of a given platinum wire
(platinum resistance thermometer). or Tc =TK -273.15
(3) Nowadays in modern tbermometry instead of two Problem 1. At what temperalllre. ifany, do the/ol/owing
fixed points only one reference point is chosen" pairs 0/ scales give the same reading: (a) Celsius and
which is triple-poinl 0/ water (temperature at which Fahrenheit. (b) Fahrenheit and Kelvin and (c) Kelvin and
ice, water and water-vapours co-exist) and has been Celsius?
assigned arbitrarily a value 273.16 K. So if the value
·J'",mx+c
.. The other is itself 0 K where the value of thermo metric property is assumed to be zero.
e
Solution: If the temperature is at which the reading of
So 28.887
TK =273.16x 11.000 = 717.32 K
two scales coincides, then from
Tc -0 TF -32 TK -273.15 Now as Tc = TK - 273.15
-100
-= 180 100 Tc =717.32-273.15=444.17°C AnsweJ'
(a) 9 9-32
100 =""""i80'
.
I. e.,
9= - 40 § 14.3 Thermometers
A thennometer is an instrument used to measi.lre
j, e., reading of Celsius and Fahrenheit scale coincides at - 40°, temperature of a body. It works by absorbing some heat from
ie the body, so the temperature recorded by it is lesser than actual
(b) 9 -32 = 9 -273.15 9 =574.6
180 100" " unless the body is a constant temperature bath. Following are
i. e., reading of Fahrenheit and Kelvin scale coincides at 574.6", some of the so many existing thermometers in COnTInon use
nowadays.
() 9-2739 h·h· ·bl
clOD = 100 W Ie IS not pOSSt e (AI Liquid Thermometers
These are based on thennal expansion of liquids in a
So reading of Celsius and Kelvin scale can never coincide.
narrow tube which rise with temperature according to the
Answer relation,
Problem 2. In a scientific book that describes a
L = Lo(l + uTc )
temperature scale called Z. boiling and freezing points of
water are referred a565° Z and -15'2 respectively. (a) To what In liquid thermometers mercury is preferred over other
temperature on Fahrenheit scale would a temperature -95OZ liquids as its expansion is large and uniform and it has high
correspond? (b) What temperature change on the Z scale thermal conductivity and low specific heat. Also it can
would correspond to a change 0/40" on Celsius scale? measure temperature over a wide range from - 50°C (f.pt.) tn
350°C (b. pt.). Upper limit of range of mercury thermometer
Soludon: (a) If Tz is the temperature on Z scale
can be raised up to 550°C by filling nitrogen in space over
corresponding to TF on Fahrenheit scale, mercury under pressure (which elevates its b.pt.) while lower
TF -32 Tz - (-15) range can be extended to -80°C by replacing mercury by
212 - 32 = -;65=-_-(i-
_-:-:15"") alcohol.
or (TF - 32)/180 = (-95 + 15)/80 (B) Gas Thermometers
or TF =-ISO+32 =- 14soF These are based on the fact that for a given mass of an
ideal gas,
(b) In case of change of temperature as,
P oc TK if V = constt. [constt. volume gas thermometer]
.1.Tc .1.Tz SO
.1.Tz = - x40 = 32Zo Answer or V oc TK ifP =constt. [constt. pressure gas thermometer]
100-0 65 - (- 15)' 100
In case of constant volume gas thermometer according to
Problem 3. A constant volume gas thermometer shows old and modern thermometry we have respectively,
pressure readings of50 cm and 90 cm of mercury atO°C and P-Po p
1OO"C respectively. What is the temperature on gas scale when Tc = xlOO°C or TK =273.l6-K
the pressure reading is60cm ofmercury? (MNR 1991) P iOo -Po PTr

Solution: In terms ofFP and BP of water, where Po ,p]{)o ' PTr and P are pressure of gas at temperatures
p-p oDe, IOOoe, triple point of water and unknown temperature
Tc = 0 xlOO°C respectively keeping the volume constant. Regarding gas
PlOo -Po
thermometers it is worth noting that:
So T = 60 - 50 x 100°C = 25°C Answer (1) These are more sensitive than liquid thcnnometers as
c 90-50 expansion of gases is more than that of liquids.
Problem 4. The resistance of a platinum resistance (2) These are most accurate and are used to calibrate
thermometer isfound to be II.OOOohm when dipped in a triple other thermometers.
point cell. When it is dipped in a bath, the resistance isfound to
(3) These have normal range from -~OO°C to 500°C.
be 28.S87 ohm. Find the temperature of the bath in °C on
Upper and lower limits can be extended to l600°C
platinum scale.
and -268°C by replacing hydrogen by nitrogen and
Solution: In terms of triple point of water, helium respectively.

TK = [273.16~l
R
K
(4) These are large and cumbersome and take enough
time to record a given temperature.
T,
(C] Resistance Thermometers filament. These are primarily based on 'Stefan's law' of
These are based on the fact that resistance of metals varies thennal radiations, viz .•
with temperature according to the relation- R = eATi
R =Ro(l+aTc) In these thennometers intensity of radiations is measured
where (J. is the temperature coefficient of resistance. Usually and using the above fOimula the temperature of the body is
platinum is used in resistance thermometers as it has high determined. These thennometers are usually employed to
melting point and for it a is constant and large. In case of measure temperature higher than IOOO°C (- MP of gold). The
resistance thermometer, temperature of sun is measured by 'pyro-helio-meter' .
R- R R K IF] Vapour-Pressure Thermometers
Tc = 0 x 100°C or TK = 27J.16 -
RiOO Ro RT, These are used to measure very low temperatures and are
[Old thermometry] [Modem thermometry] based on the fact that saturated vapour pressure P of a liquid
depends on temperature according to Ihe relation,
These thermometers in conjunction with resistance
measuring devices such as 'Callendar and Griffth bridge' logP = a+bTK + ...£..
usually measure temperatures from - 200°C to 1200°C. TK

Note: Nowadays for measuring low temperature 'germanium' The range of these thermometers is from 120 K to 0.7 1 K
(semiconductor) resistance thermometer is used whose for different Iiquid~vapours.
resistance increases with decrease in temperature. This
thennometer is used in the range 4 K to 77 K. Note: Temperatures below 1 K are measured by 'magnctic
thermometers' which are based on Curie-I::.w according to
ID] Thermoelectric Therm&meters which magnetic susceptibility of a paramagnetic salt varies
inversely as its absolute temperature, i.e. ,
These are based on "Seebeck effect" according to which
when two distinct metals are joined to form a closed circuit
rK oc(C/xJ
called thermocouple [as shown in Fig. 14.2 (a)] and difference Question II. State and explain three reasolls why water
in temperature is maintained between their junctions, an e.m.f. is considered unsuitable/or use in thermometers?
is developed. The e.m.f. is calted thermo~e.m.f. and if one Answer: Water is unsuitable for use in thermometers
junction is at O°C it varies with temperature as,
because (i) Due to anomalous behaviour its expansion is not
e* = aTe +bTJ unifonn. so the temperature scale will not be uniform. (ii) Due
to large specific heat and low thermal conductivity it will not
where a and bare constants.The variation of thenno~e.m. f.
respond quickly to change in temperature. (iii) Its freezing
with temperature is shown in Fig. 14.2 (b). e.m.f. is measured
point is O°C while boiling point lOO°Cj so the temperature
by a potential measuring device such as 'potentiometer'.
range over which it can be used will be small, i.e., from O°C to
t Neutral
Temperature IOO°C. (iv) It is transparent (so invisible), sticks to glass wall
Cu Cu
and has high rate of evaporation.
Question III. Explain why (a) the glass bulb a 0/
Temperature mercUly thermometer is long (cylindrical and IIOt spherical)
of Inversion
T,
and thin; (b) Clinical thermometer cannot be sterilized by
Temperature _ boiling.
Answer: (a) The rate of flow of heat through conduction
(a) (b)
is given by.
Fig. 14,2
Thermocouple is used to measure temperature in the linear dQ = KA (9 H -9 L )
part of EMF versus temperature cUlVe and has nonnal range
dt d
- 200°C to 1600°C. It has low thermal capacity and high In order that a mercury thermometer inspite of having low
thermal conductivity, so can be used to measure quickly thennal conductivity (K) of glass may quickly conduct heat
changing temperature and temperature of tiny objects. from body to mercury, the glass bulb containing mercUlY is
IE] Pyrometers made long (so that its area is increased) and thin (so that d is
decreased).
As the word 'pyro' means fire, pyrometers are used to
measure very high temperature such as of a furnace or glowing

• It is maximum ( 1.2 mY/CO) for antimony and bismuth.


(b) Clinical thennometer is usually calibrated to read E2 and hence the average distance between them from r, to r2 .
from 95°F to 110°F while the boiling point of water is 212°F. Due to this increase in distance between atoms, the matter as a
So on sterilization by heating due to thennal expansion of whole expands. Had the potential energy curve been
mercury ill the capillary, the capillary of the thermometer will synunetrical, no thennal expansion would have taken place in
burst. spite of heating!
Problem 5. The temperature coefficient ofresistance ofa
Note: Certain substances like rubber contract on heating. In these
wire is 12.5 x IO-4/C o. At300K the resistonceofa wire is l ohm. substances molecules are fonned by intertwined and crosslinked
At wltat temperature in K will the resistance ofsame wire be2 chains of atoms and atoms vibrate both longimdinally and
Iransversely. As the temperature increases, the amplitude or
ohm?
transverse vibrations increases and so the average distance
Solution: We know that variation of resistance with between the plancs of atoms decreases; hence the solid on
temperature is given by, heating will contract.

R =RO(I+aTC) Furthermore as intennolecular forces (responsible for


Now as 300 K = 300 - 273 = 27°C, average distance between atoms) is maximum in solids while
So I = Ro(l+27a.) and 2=R o(l +a.Tc ) minimum in gases, the expansion of solids is least while of
or TC = 54+(l/a)=54+(l1l2 .5x l0-4) =(54+800)OC gases highest. Solids can expand in I-D (linear expansion),
2-D (supeificial expanSion) and 3-D (volume expansion) while
So TK =854+273=1 127K AI/SIVer liquids and gases usually suffer change in volume (i.e., 3-D)
Problem 6. The thermoelectriC powers of Pt, Nt' and Fe only. Here it is worthy to note that:
are - 4, - 21 alld +12 microvolt per Co respectively. What will (1) In case of linear expansion, the coefficient of linear
be the magnitudes of the e.m/. 's of PI-Ni alld Ni-Fe expansion is defined as fractiorial increase in length
thermocouples with jUl/ctions at DOC alld 100°C? in
per CO rise tempera,~re, i.e:,
Solution: When a thennocouple is used in linear range ",,*=Mx_1_ (.)
with one of jts junctions at DoC, its e:m.f. - L llT .... 1

e=aTC The coefficients of superficial expansion ~ and


where a is the difference in thermoelectric powers of the two vo~ume expansion yare defined in a similar way, i. e.,
metals and Tc is the temperature of olher j unction in OC. So:
p = dA )( _1_ and y = 6V x _1_ .... (ii)
(a) For Pt-Ni thennocouple as A llT V llT
a =-4-(-21)=17~ VIC' a, [3 and y all have dimensions [9-'], units per CO (not
e = 17 (f.lV/CO) x 100 Co = 1.7 mV 0c) and depend only on the nature of material.
(b) For Ni-Fe thermocouple as (2) From definitions ofa , [3 and y, i.e., Eqns. (i) and (ii), it
a = 12 - (-21) = 33 ~V/C' ." is clear that increase in length, area and volume with
e= 33 (~tV/CO)x 100 Co =3. 3 mV Altswer rise in temperature tlT will be respectively:
§ 14.4 Thermal Expansion I>.L = La llT; M = AP 6T
When matter is and 6V=Vy6T ....(iii)
heated without change . and so final length L'(= L + tlL), .area A' and volume
+ V"will be:
in state, it usually
expands. According to
o ----------------..:--------
----------i---------- T2 L' ;L(1+0. 6T); ·f",A(l+p6T)
atomic theory of matter, r2 ~
and
, V' = V(l+y6T) .... (jv)
asymmelly in potential E El T,
i.e., change of final value of length, area or volume
energy curve is
depends on its initial value, change in temperature
responsible for thermal Eo
and nature of material.
expansion as with rise
in temperature say from r_ (3) 'As area is 2-D while volume 3-D, So

~' =[~r ~ =[~r


T, toT2 the amplitude of Fig. 14.3
vibrat ion and hence energy ofaloms increases from E, to £2 and

• Acnlally it is the average valuc orcocfficienl orlincar expansion over the temperature range rrom T.f::. to {T + 6T)'C and so coefficient ortinear expansion
III temperature T will be gillcn by
( l '" lim .! IJ./. '" .! til.
IJ.T~oLIlT !.dT
But L'=L(l+,,/J.T), i.e., (L'IL) =(I +,,/J.T) [(tIT) - (lIT')] oscillations. Now as measurement of
time is through the counting of oscillations, so in time
So (A'IA) =(I +" /J.T)' and (V'/V) =(I +" /J.T)'
t, a clock with time T' will lose time by
or A'=A(I+20./J.T) and V'=V(I+l<x/J.T)
Comparing these expressions for A' and V' with that ,[, ,] [T' ]
/J.I=T T-T' =11"-1
ofEqn. (iv), we find:
p =20. and y =3" However, as for a simple pendulum T =21t(Llg)1I2
i.e., (,,11)= (1312)= (yI3) T' = (L' = L(I+a/J.9) =I+!,,/J.9
or " : p : y :: I : 2 : 3 ....(v) T VI: L 2
i.e., ratio ofa,~and y is 1 : 2: 3. I
So .6.t = -u .6.6 t .... (vii)
(4) If two strips of equal ~, Tl T, 2
length but of different ~ From this it is clear that:
metals are placed on each eu ,"~ Fe
(a) The clock witt lose time, i.e., will become slow if
other and riveted, the ~ 9' > 9 and witt gain time, i. e., will become fast if
single strip so fonned is $ 9'< 9 with.6.9 =9' - 9.
C[S811ed , 'bllm4e4ljaUThie . strif ~ (b) The gain or lose in time is independent of time
ee F Ig. • • 18 stnp
(.) (b) period T and depends on the time interval t.
has the characteristic
property of bending on Fig. 1"." (7) When a rod whose ends are rigidly fixed such as to
heating due to unequal linear expansion of the two prevent expansion or contraction undergoes a change
metals. The strip will bend with metal ofgooalera on in temperature, due to thermal expansion or
outer side, i, e., convex side. This strip finds its contraction, a compressive or tensile stress is
application in auto-cut or thermostat in electric developed in it. Due to this thermal stress the rod will
heating circuits. It has also been used as thermometer exert a large force on the supports. If the change in
by calibrating its bending. temperature ofa rod of length L is.6.9 CO, then
(5) If a scale gives correct reading at temperature 9, at
temperature 9' (> 9) due to linear expansion of scale,
Thennalstrain= i =a.6.9 [asa =.0.:: ;9]
the scale will expand and scale reading will be lesser So Thennal stress = Ya .6.9 [as Y = stress/strain]
than true value [Fig. 14.5 (b)j so that,
or ForceF=YAa.6.9 [asstress=FIAl ... (viii)
True value = Scale reading [I +a (9' -9)]
(8) Thermal expansion of an isotropic object may be
i.e., TV=SR[I+,,/J.9j with /J.9=(9'-9) ....(vi) Imagined as a photographic enlargement. So if there
However, if9' < 9, due to contractions of scale, scale is a hole A in a plate C (or cavity A inside a body C),
reading wi1l be more than true value; so true value will the area of hole (or volume of cavity) will increase
be lesser than scale reading and will still be given by when body expands on heating, just as if the hole (or
Eqn. (vi) with /J.9 (=9' - 9) negative. cavity) were solid B of the same material. Also the
expansion of area (or volume) of the body C will be
independent of shape and size of hole (or cavity), i. e.,
will be equal to that of D .

• to 8tO'>0
TV_SR TV>SR
(a) (b)
fig. 14.5
(6) If a pendulum clock keeps proper time at temperature
Expansion ot Expansion of
9, at temperature 9'(>9) due to linear expansion, A. Expansion ot B C • Expansion ot 0
length of pendulum and hence time period will Fig. 14.6
increase. Due to this increase in time period, in a
given time interval (say t), the pendulum will exe<:ute (9) In case of expansion of volume Vofa liquid
(tiT,) oscillations instead of (tIT) and so will lose Vi. =V(I+YL/J.9)
while for volume V of container, this range is negative). This behaviour of water in the
Vi =V(I+Y s 69) range DoC to 4°C is called anomalous expansion and
So the change in volume of liquid relative to a rises due to the fact that water has three types of
container, molecules, viz,. H 20, (H20h and (H 20)3 having

Vi -vs =V(YL -Ys)69 different volume per unit mass [called specific
volume = (l/p)] and at different temperature their
i. e., Il VApp. = Vy App. tlS with y App. = YL - Ys .",(ix) properties in water are different.
From expression (ix) it is clear that:
(a) If YL > Ys' YApp. > D, so tlVApp. will be positive, 1
i. e., level of bquid in the container will rise on
~
,"
heating.
(b) Ify L < y S' 'Y App.< 0, so tlVAPP . will be negative, -E •••
0

:g ~

i.e., level of hquid in the container will fall on
heating. o'c 4'C
'" o'c 4'C
Temperature _ Temperature _
(c) IfYL='YS (=3a s ),'YApp.=O, sotlVA . will be
zero, i. e., level of liqUId in the conf~iner will (al (bl
remain unchanged on heating. FIg. 14,7
(10) If m mass of a solid or liquid, at a given temperature, Variation of specific volume and density of water
occupies a volume V, the density p will be (m/V), Now with temperature is shown in Fig. 14.7 (a) and (b)
if the temperature is increased by de Co, the mass will respectively, From these graphs it is clear that at 4°C
remain unchanged while due to thermal expansion density of water is maximum while its specific
volume will increase and become V' = Vel + 'Y Ae), so volume (volume/mass) is minimum.
that density, '
Note: During winter when the water at the surface of a lake cools below
, m m P .... (x) 4°C by cool air, it expands and becomes lighter than waler below.
p =V'=V(1+yM)=(1+yM) Therefore the water cooled-below 4°C stays OD the surface and
freezes when the temperature of surrounding falls below O"C.
i. e., with rise in temperature density decreases, Thus the lake freezes first at the surface and water in contact with
(11) The thrust on V volume of a body in a liquid ofdensity ice has temperature O"C while at the bottom oflhe lake 4°C (as
density of water at 4°C is maximum) and fish and other aquatic
cr is given by. animals remain alive in this water.
Th=V"g
Question IV. The difference between lengths ofa certain
Now with rise in temperature by Ae Co, due to
brass rod and of a steel rod is claimed to be constant at all
expansion, volume of body will increase while
temperatures. Is this possible?
density of liquid will decrease according to the
relations: Answer: If LB and Ls are the lengths of brass and steel
V'=V(1+Y s 69) and "'=,,/(1+YL69) rods respectively at a given temperature, then the lengths ofthe
So the thrust will become, rods when temperature is changed by AS Co will become,

Th' = V'cr'g L"=L.(1+,,.M) and Ls ~Ls(1+"S69)


Th' =V'cr'g = (l+y s 69) So that L" -Ls =(L. -Ls)+(L.". -Ls "s)A9
Th V"g (I+YL69) SO (LB- Ls) will be equal to (LB - Ls) at all
temperature if,
Now as 'YS<'YL,Th'<Th,i,e., with rise in
L.". -Ls"s =0 [as69~OJ
temperature thrust also decreases. This in tum implies
that if a body is weighed in a liquid at different or (L.ILs ) = (" s I".)
temperatures, with rise in temperature due to decrease i. e., the difference in the lengths of the two rods will be
in thrust its weight (= Wo - Th) will increase. independent of temperature if the lengths are in the inverse
ratio of their coefficients of linear expansion,
(12) Nonnally on heating matter expands and contracts on
cooling, In case of water, water expands on heating if Question V. There are two spheres of same radius and
its temperature is greater than 4°C. In the range O°C to material at same temperature but one being solid while the
4°C water contracts on heating and expands on other hollow. Which sphere will expand more if(a) they are
cooling (i. e., its coefficient of volume expansion in heated to the same temperature, (b)same heat is given to them?
Answer: (a) As thennal Problem 8. The alternate discs of iron and carbon,
expansion of isotropic solids is
similar to true photographic
enlargement, expansion of a
cavity is sarneas ifithad been a
solid body ofthe same material,
i.e. ,
o
Fig. 14.8
having same area of cross-section, are cemented together to
make a cylindrical conductor whose temperature coefficient of
resistivity is zero. Ifthe change in temperature in two alternate
discs is the same, determine the ratio oftheir thickness and the
ratio of heat produced in them. The resistivity of iron and
carbon at 20°C are 1><10- 7 and 3xl0- 5 O-m and their
"v = Vy "e temperature coefficient of resistance are 5 x 10- 3 and
As here V, y and ,1,9 are same for both solid and hollow -7.5 x 10-4 per OC, respectively. Neglect thermal expansion.
spheres treated (cavity); so the expansion afboth will be equal. Solution: According to given problem,
(b) If same heat is given to the two spheres due to lesser M ,,(R F +Rc) .
mass, rise in temperature of hollow sphere will be more [as u=--= =0, I.e., !::.RF+!::.Re=O
R"e (RF+Rc)"e
.6.9=Qlmc] and hence its expansion will be more [as
"v=Vy"ej. or (RFa F +R e a e )6.9=0 [as!::.R=Ra6.9]
Question VI. Stale whether the following statement is RF _ U c _ (-7.5x1O- 5 ) _ 3
troe or false giving reason in brief "A barometer made of a
So Re - - a - - 5x10 3 - 20
F
very narrow tube is placed at normal temperature and'
pressure. The coefficient of volume expansion of mercury is But as R = P (II S) and here S is same, so
1.8xlO-s/co and that of the tube is negligible. The PFiF "" .l., i.e., IF = 1..x3 x l0-5 ",, 45 Answer
temperature ofmercury in the barometer is now raised by I C~ Pele 20 Ie 20 Ix 10-.7
but the temperature of the atmosphere does not change. Then Also as discs are in series, i. e., current I is same,
the height ofmercury in the tube remains unchanged. "
HF RF 3
Answer: In case of barometer, -- = -= - Answer
He Re 20
P=hdg
Problem 9. The brass scale of a barometer gives correct
Here as temperature of atmosphere is constt., atmospheric
reading at O°c. Coefficient of linear expansion of brass is
pressure P will remain constant, so
2.0>; 10-5ICo, The barometer reads 75 cm at 27°C. What is the
hd =constt. [asg = constt.]
atmospheric pressure at 27°C?
Now as with rise in temperature density of mercury d will
decrease, h i.e., height of mercury colunm will increase and Solution: We know that in case of scale reading,
True value = scale reading [1 + a 6.9]
hence the given statement is false.
.
Note: As here hd ==conslt. (h'l h) = (did') Here SR = 75 cm;
i.e., (h'lh) = (1+ YHg l!.8) (asd'=dl (1+y Hg l!.8)) and M =(9'--9) =27-0=27C'
i.e.', h'=h(1+'YHgl!.9) or 6h=h'-h=hYHg68;tO So Truevalueh = 75(l + 2xlO-5 x 27) =75.04 em Answer
Problem 7. A steel~scale is to be prepared such that the Note: The pressure at O°C will be given by
"
millimetre 'intervals are to be accurate within 5 x 10-4 mm at a hodo=hd=P, i.e., lJo=h(dldo )
certain !emperature. Determine the maximum permissible Butas d=du/ (l +y Hg l!.8),
temperature variation during the ruling of the millimetre hoe h withh=hs (1+cx.M)=75.04cm
marks ifa steel = 13.22 x 10 - 61c o (1+y Hg l!.8)

Solution: We know that in case of linear expansion


,
Problem 10. A steel wire AD of length 85 em at 10°C is
M=Lu"e. flXed rigidly at points A and B in an aluminiumframe as shown
HereM< 5 >< lO-4 mm;L = lrrunanda = 13.22 x 1O- 6/Co in Fig, 14.9. Ifthe temperature ofthe system is raised to 110°C,
what extra stress wjlJ be produced in the wire relative to
So "e= M = (<5xl0-') aluminium frame. Assume that coefficient of linear expansion
La lx13.22x10 6
for ,aluminium and steel are 23 x 1O-6/c o and II x 1O-6 /c o
i. e., 6.9< 37,83 Co
respectively and Young's modulusfo>' steel is 2 x 1011 Pa.
or ("e)~ =37.83C' Answer
669

Solution: As on heating Problem 13. A glass flask of volume one litre at OOC' is
aluminium frame will expand Aft--:c_S_'_"_'_W_",..'_-fl' B filled level full of mercury at· this temperature. The flask and
more than the steel wire, thermal Aluminium-Frame mercury are now heated to 1000C'. How much mercury will
strain in steel wire relative to spill out if coefficient of volume expansion of mercury is
aluminium frame will be, Fig. 14.9 1.82 x 10- 4/ c o and linear expansion of glass is 0.1 x 10-4/c o
(WL) = (o.AL -o.s)M [asM = Lo.l>ej respectively? (MNR 1994)
i.e., 6
Thermal strain = (23 - 11) x 10- x (110 - 10) Solution: In case of thermal expansion of liquid, change
=1.2xlO-3 in volume of liquid relative to container is given by

But as Thermal stress = Y x strain [asY =stress/strain] l>V=V(YL -Ys)l>9


Here V =1 litre = 1000cc and Ys =3u G =0.3 x 10-4/c o
So Thermal stress = 2x 10" x 1.2 x 10- 3
=2Ax 10 8 Pa Answer So l>V = 1000(l.82-0.3)x1O-4 x (100-0)
Problem 11. A thin rod of negligible mass and area of =lS.2ce Answer
cross-section 4 x 10- 6 m2, suspended vertically from one end, Problem 14. A one litre glass flask contains some
has a length of0.5 m at 100°C. The rod is cooled to O°C, but mercury. It is found that at different temperatures the volume
preventedfrom contracting by attaching a mass at the lower ofair inside the flask remains the same. What is the volume of
end. Find (i) this mass and (ii) the energy stored in the rod. mercury in this flask if coefficient oflinear expansion of glass
is 9xl0-6/Co while volume expansion of mercury is
Given for the rod, Young's modulus = 10" Nlm2,
Coefficient oflinear expansion = 10~5 K-' and g = 10 mls 2. J.8 x 10-4/c o?
Solution: If V is the volume of flask, VL of mercury and
Solution: Strain = ~ =u 6.8 = 10~5 x 10 2 = 10-3
VA of air in it,
So Stress = YxStrain = lO!1 x 10- 3 =10 8 (N/m 2 ) V=VL+VA
F Mg A Now as with change in temperature volume of air remains
Bu' Stress=-= - :. M=-(Stress) constt., the expansion of mercury will be equal to that of the
A A g
whole flask, i.e.,
6
M=4xl0- x10 8 = 40kg Answer L1V=6.VL
10
or VYG6.9 = VL YL6.9 [as6.V=Vy6.8]
Further as in case of elasticity
Here V = llitre = 1000 cc and Y G =30. G = 27 x 10-6/Co
Energy I
~;="'c= - stressxstram =- xl
. I 08 xlO - 3 =-I xI OS -' 'J
Volwne 2 22m3 So VL = (1000x27xlO-6/1.8xlO-4)=ISOcc

and as Volume of rod = 4x 10-6 x.!. =2x 10-6 m 3 Problem IS. A sinker of weight Wo has an apparent
2 weight 1fJ when weighed in a liquid at a temperature T, and W2
So Energy=.!. x lOS x2x 10- 6 =0.1 J Answer when weighed in the same liquid at a temperature T2. The
2 coefficient of cubical expansion of material of sinker is Ys'
Problem 12. A pendulum clock with a pendulum made of What is the coefficient of volume expansion of the liquid?
invar (a = 0.7 x 10-6/ c o) has a period of0.5 s and is accurate Solution: We know that weight of a body .in: a liquid is
at2S°C. Ifthe clock is used in a country where the temperature given by
averages 3SoC, what correction is necessary at the end of a WApp. "'" Wo - Th with Th = Vag
month (30 days) to the time given by the clock? i.e., Wo - WApp. = Vag
Solution: As explained in § 14.4 (6), in time interval t the So according to given problem,
clock will become slow (or will lose time) by, Wo -1fJ = V,O' ,g and Wo - W2 = V20" 2g
llt ='!'oo 6.8 Wo -1fJ V,O" 1
2 So =--
Wo - W2 V20" 2
'" = l x (7 x 10- ) x (30 x 86400) x (35 - 25)
So, 7
2 Now as V2 = V,(1 +Ys,1,9) and a 2 =0",/(1+Y L L19)
= 9.1 s Answer
0_ (Wo -W,) __ (I + YL A9 ) m, =p,v, and m2 =P2v2
~ M=(T,-T,)
(Wo-W,) (1+Y s A9)
with
PI
_ m, ._v1
=_ .... (i)
which on simplification provides Pz m2 VI

Ify g' is coefficient of volume expansion of bulb,


rr= [Wo-W,]
Wo -W,
Ys+
(W,-W,)
(WO -W,)(T, -T,)
Answer
v,
=v,[I+y.(I, -I,)J .... (ii)

Problem 16. A sphere ofdiameter 7 em and mass 266.5 g and


pi
-=[I+YR(I, -I,)J .... (iii)
floats in a balh of liquid. As the temperature is raised, the P,
sphere begins 10 sink at a temperature of 35°C.lfthe density of
the liquid is 1.527 g/cm J at OOC,jind the coefficient of cubical Substituting for v 2 and £.L from eqns. (ii) and (Hi) in (i),
VI P2
expansion of the liquid. Neglect the expansion ofthe sphere.
Solution: In case of floating, [I+YR(I, -I,)= -
m, J l+y.(I, -I,) ... (iv)
Weight of body =Thrust, i.e., mg=Vinag m,
As the sp~ere just sinks at 35°C, Here, y g =30. g =27xlO-6 /oC
Th _ V., ag
ml =IS6.25-S7.S =98.75g
Th _ Va' g
m, = 156.25 - 58.5y = 97.68g
tl = IS°C,t2 =52°C
Substituting in (iv),
[I + YR (52 -15)J = 98.75 [I +27 x 10- 6 x (52 -15)J
----------------
-----
-- -

--
-

-- -. --
-

----------------
,----=
. • . . .----
... . -
---.
---- -

. . . ..
-- -- %

== == - .
.

"====== =mg ======:


---r---====--
97. 68
At Temp. T < 3500 At Temp. T _ 3500 Solving, Answer
Fig. 14.10
Problem 18. An aluminium cube of side 20 em floats on
m=Va' [asVin =V] mercury. How much farther will the block sink when the
temperature rises from 300 K to 350 K? Given that the density
or a'=!!!.= m = 266.5 =1.483g1cm 3 ofaluminium and mercury at 300 K are 2.7 and 13.6 glee while
V (4/3),,' (4/3)_(7/2)'
the coefficient of volume expansion of mercury and linear
Now as a'= cr , (l+y1l8)=E. expansion of aluminium are 1.8 x 1O-4/co and 23 x 1O- 6/C (
(l+yA9) 0' respectively.
i.e., I +y (35 - 0) = (1.527/1.483) Th " VInO'g Th' _ VI~ (/ 9

or 0.044 - 85 x 10-4 /oC Answer


Y~1.483x35- .
L v
Problem 17. A loaded glass bulb weighs 156.25 g in air. ::: 1_J:;;;:,o;L~ h::
- <:.: :<:. : :':
When the bulb is immersed in a liqUid at temperature I SOC. it ~ ::<:.::: .

weighs 57.5 g. On heating the liquid, the apparent weight ofthe


bulb becomes 58.57 g. Find Ihe coefficient ofreal expansion of
the liqUid. (Given, coefficient of linear expansion oj glass --------------
_ ... _ .... ==== . ..
T _ 350K
=9 x lO-6 j o C ). Rg.14.11
Solution: When the bulb is immersed in liquid, the loss in Solution: If side of cube, density of liquid (mercury) and
its weight is due to upthrust of liquid and is equal to weight of length of the floating cube inside the liquid are L,a,li and L',
liquid displaced. a', h' at 300 K and 350 K respectively, by principle oj
Let m, and m2 represent the loss in weight of bulb at lower floatation.
and higher temperatures I, and 12 respectively; VI and V z the
[wilhm=Vp=pL'J .... (i:
volumes of bulb and Pl' P2 the densities afthe liquid at these
temperatures respectively. Then, i.e., hL'o =h'(L')'o' [asVin =hA=hL2]
• In density change of liquid, only real expansion of liquid is involved. The expansion of container docs not affect the density of liquid in any way.
a/the liquid in A and D measured from the base line are 52.8
em and 51 em respectively. Determine the coefficient of
thermal expansion ofthe liqUid.
Solution: As the liquid in the horizontal tube PQ is at
rest, gauge-pressure at P and Q will be equal. So if Po is the
atmospheric pressure,
So Ah(=h'-h)=h(YL -2<%s)xl>9 Po + (pgh A )" - (pgh.), =Po + (pghD ), - (pg"c)"
Also from Eqn. (i), i.e., m=Vino. we have i.e., p,[hD +h]=p,,[hA +h] [ash. =hc =h,given]
J
pL =hL'", i.e, h=[£]L=[2.7 ]X20=3.970cm or .!l= h+hA
" 13.6 P9S h+hD
So Ah = 3.970[1.8 x 10-4 -2x23 x 10--6)(350-300) = 49 + 52B = 101.8 = 1.018
49+51 100
=2.66xlO-2 em Answer
Problem 19. The apparatus shown in the Fig. 14.12 ~x[I+95Y]=1.018
[1+5y] Po [
asp - Po
(I +y A9)
1
consists 0/ four glass columns
p Q
connected by horizontal 0.018
or
sections. The height 0/ two A B O y= (95-5xI.018)
central columns Band Care 49 95" 5": 95" S-
cm each. The two outer columns :::El.",.",--",.!t:..,Jt--"",--",.t.L = 0.018 =2x 1O-4;co Answer
Fig. 14.12 89.91
A and D are open to the
atmosphere. A and C are maintained at a temperature of 95°C The coefficient of volume expansion of the liquid is
while the columns Band D are maintained at SOC. The height 2x 10-4;co.
EXERCISE

[A] Only One Choice is Correct 12. The temperature of a subslan-.::c increases by 27 Co, On the
Kelvin scale this increase is equal to: ICPMT 19931
1. The scale of temperature on which the temperature are only
positive is : (a) 300 K (b) 2.46 K
(a) Fahrenheit (b) Celsius (c) 27 K (d) 7 K
(c) Kelvin (d) Reaumur 13. A Centigrade and a Fahrenheit thennometer are dipped in
boiling water. ,The water temperature is lowered until the
2. Celsius is the unit of:
Fahrenheit thennometer registers 1:40°, Wha.t is the fall in
(a) Temperature (b) Heat temperature as re~istered by the Centigrade thcnnometcr '!
(e) Specific heat (d) Latent heat ICBSE 19921
3. On the Celsius scale the absolute zero of'temperature is at: (a) 30' (b) 40'
(a) O'C (hi -32'C (c) 60' (d) 80'
(c) IOO'C (d) -273.15'C 14. Mercury thennometers can be used to measure temperature
4. The correct value of O°C on the Kelvin scate is : upto, ICBSE 19921
(a) 273.15 K (b) 273.16 K (a) 100'C (b) 212'C
(c) 273 K (d) 273.2 K (c) 360'C (d) 500°C
5. Triple point temperature of water is: 15. Mercury boils at a temperature of367~C; however mercury
(a) 273.16 K (b) 373.16 K thennomcters are made which can measure upto 500°C.
This is don~ by:
(c) 273K (d) 273.15K
(a) Maint,aining vacuum above the Hg column in the stem
6, The temperature in the Fahrenheit scale corresponding to
of the thennometer
253 K is: lIAS 1996]
(b) Filling nitrogen gas at low pressur~ above mercury
(a) -4'F (0) 4'F (c) 12'F (d) 36'F " ,
column
7. Oxygen boils at -183°C. This temperature is
(c) Filling nitrogen gas at high pressure above mercury
approximately: (CPMT 1992]
colunm
(a) 215'F (b) -297'F
(d) Filling oxygen gas at high pressure above mercury
(d) 361'F
column
8. Recently, the phenomenon of superconductivity has been
16. Two thennomcters are constructed in the same way except
observed at 95 K. This temperature is nearly equal to:
that one has a spherical bulb and the other a cylindrical
ICPMTI9901 bulb; which one will respond quickly to teinperature
(a) -288'F (b) - 146'F changes?
(c) -368°C (d) +178'C (a) Spherical bulb thennometer
9. The reading of Centigrade thennometer coincides with that (b) Cylindrical bulb thennometer
of Fahrenheit thermometer in a liquid. The temperature of (c) Both equally
the liquid is :
(d) Either
(a) - 40'C (b) O'C
[Hint: See Question IU]
(c) 100°C (d) 300°C
17. Two thcnnomcters are used to record the temperature of a
[Hint: See solved problem I (a)] room. If the bulb of one is wrapped in wet hanky:
10. The Fahrenheit and Kelvin scales of temperature will give (a) The temperature recorded by both will be same
the same reading at: [MNR1992)
(b) The temperature recorded by the wet-bulb thennometer
(a) - 40 (b) 313 will be greater than that recorded by the other
(c) 574.25 (d) 732.75 (c) The temperature recorded by dry-bulb thermometer
[Hint: See solved problem I (b)] will be greater than that recorded by the other
11. The freezing point on a thermometer is marked as 20° and (d) None of the above
the boiling point as 150°. A temperature of 60°C on this 18. The standard scale of temperature is:
thermometer will be read as :
(a) The mercury scale
(a) 40' (b) 65'
(b) The gas scale
(c) 98' (d) 110'
(c) The platinum resistance scale
[Hint: See solved problem 2]
(d) None of the above
THERMOMETRY AND EXPANS,oN 673

19. In 11 constant volume gas thermometer, the temperature of a (c) First increases and then decreases to zero
bath is measured by: (d) First increases, becomes maximum and then decreases
(a) Keeping the mass of the gas constant and by noting the to zero and finall y reverse its direction
change in volume 27. The temperature range that can be measured by
(b) Keeping the mass ofthe gas constant and by noting the thermocouple thermometer is: (CPMT 1992}
change in pressure (a) -200"C '0 160O"e (b) -260"C to 80O"e
(e) Keeping the volume of the gas constant and by noting (e) - 26O"C to 500"C (d) - 300"C to 800"C
the change in pressure
28. The; temperature of the sun is approximately: ICPMT 1993]
Cd) Keeping the mass and volume of the gas constant and
(a) 1000 K (b) 7000 K
by noting change in pressure 6
(e) 10 K (d) 'Ox to K
20. Gas thennometers are more sensitive than liquid
thermometers because: 29. For measurements of very high temperature say around
5000°C(ofsun), one can use: (CPMT 1991]
(a) Gases expand more than liquids
(a) Gas thermometer
(b) Gases do not easily change their state
(b) Platinum resistance thermometer
(c) Gases are much lighter
(e) Vapour pressure thermometer
(d) Gases arc easy to obtain
(d) Pyrometer (Radiation thermometer)
21. The temperature range measured by hydrogen gas
thcnnometer is : 30. Pyrometer is an instrument used to measure :
(a) -200"C to 800"C (b) -200°C to 500QC (CPMTI993)
(e) - 260"C to 800"C (d) -26O"C to 500"C (a) High temperatures (b) Low temperatures
22. The temperature coefficient of resistance of wire is (c) Specific heat (d) Latent heat
12Sx 10-4 lco. At 300 K the resistance ofthe wire is 1 ohm. 31. The study of physical phenomena at low temperatures
The resistance will be 2 ohm at : (below liquid nitrogen) is called: ICPMT 1992)
(a) 1154 K (b) 1100 K (a) Refrigeration (b) Radiation
(e) 1400 K (d) 1127K (c) Cryogenics (d) Pyrometry
[Hint: See solved problem 5J 32. When a metal rod is heated it expands because:
23. In a resistance thermometer, the resistances al OGC and (a) The size of its atoms increases
1000c are 6.74 and 7.74 ohm respectively. The temperature (b) TIle distance among its atoms increases
corresponding to 6.53 ohm resistance is: (e) Atmospheric air rushes into it
(a) +53"C (b) +21"C (d) The actual cause is still unknown
,
(e) - 53"C (d) c 21"C 33. Expansion during heating:
24. Which instrument will you most conveniently employ to (a) Or-curs only in solids
measure a temperature of 400°C? [CPMT 1990]
(b) Increases the weight of a material
(a) Vapour pressure thermometer
(c) Decreases the density ofa material
(b) Radiation pyrometer
(d) Occurs at the same rate for all solids and liquids
(c) Resistance thermometer
34. Two rods oflengthsLI and L2 are made of materials whose
(d) Constant volume gas thermometer coe~ficient of linear expansion are ul and u 2 . If the
25. No other thennometer is as suitable as platinum resistance difference between the two lengths is independent of
thennometer to measure temperature in the entire range of: temperature:
(a) O"C to toO'C (b) 100"C '0 1500"C (a) (L,Il'2)"(n,'n,) (b) (L,IL,)=(n,'n,)
(e) - 50"C to 350"C (d) -200"C to +600"C (e) 4n, "Lin, (d) n~r, =nlL,
26. A thermoelectric thermometer is made o f copper·iron
35. When a strip made of iron (u J ) and copper u 2 (> ( 1 ) is
thenn~ouple. One junction is placed in melting ice and the
heated:
other end is placed in a bath, whose temperature is
continuously increased from O"C to 600°C. The current in (a) Its length does not change
the galvanometer: (b) It gets twisted
(a) Remains constant (c) It bends with iron on concave side
(b) Constantly goes on increasing (d) It bends with iron on convex side
36. An iron tyro is to be fitted onlo a wooden wheel 1.0 m in 45. During severe winter in the low temperature zones of the
d iameter. The diameter o fthe tyre is 6 nun smaller than that world, the superficial parts of the lakes are frozen leaving
of the wheel. The tyre should be healed so that its water below. That the freezing at the bottom is prevented
temperature increases by a minimum of (coefficient of because :
volume expansion of iron is 3.6x IO-s ICj : (a) The conductivity of ice is low
(a) 167 c- (b) 334 Co (b) The watcr has large latent heat affusion
(e) SOO c- (d) 10OOc- (c) The water has large specific heat
37. If the length of a cylinder on heating increases by 2%, the (d) The temperature of the earth at the bottom oflake is high
area of its base will increase by : (CPMT 19931
46. A composite rod made of copper (a = I.Sx 10-~ K - i ) and
(a) 0.5% (b) 2%
steel (a " 1.2x 10- 5 K- 1 ) is heated; then it: [DCE 20051
(c) 1% (d) 4%
38. When a copper ball is heated, the largest percentage (a) bends with steel on concave side
increase will occur in its: IEAMCET 19921 (b) bends with copper on concave side
(a) Diameter (b) Area (c) does not expand
(e) Volume (d) Density (d) data is insufficient
39. Water is filled in a glass beaker at room temperature and the 47. A beaker is completely filled with water at 4°C. It will
beaker is heated. The level ofwaler in the beaker: overflow if: IAFMC 2005]
(a) Rises (a) heated above 4°C
(b) Does not change at all (b) cooled below 4"C
(c) First rises and then falls (c) both heated and cooled above and below 40(:
(d) First falls and then rises respectively
40. On heating a liquid of coefficient of cubical. expansion y in a (d) none of the above
container having cocfficient of linear expansion (y / 3) the 4S. The volume of a metal sphere increases by 0.24% when its
levcl ofthe liquid in thc containcrwill: IEAMCET 1993J temperature is raised by 40"C. The coefficient of linear
(a) Rise (b) Fall expansion of the metal is ... .. ..... fOe. [Kerala PET 2005]
(c) Remain stationary (d) It is difficult to say (al 2x 10- ' (b) 6x 10-'
41. When water is heated from O"C to I (fC, its volume: (e) 18x 10-' (d) 1.2x W,
(a) Increases (el 2.lx 10- '
(b) Decreases 49. A vertical column 50 cm long at 50°C balances another
(c) Remains unchanged column of same liquid 60 cm long at IOO°e. Coefficient of
(d) First decreases, then increases absolute expansion of the liquid is ;
(a) O.OOOSfC (b) 0.002fC
42. Water has maximum density at:
(e) 0.OO02f C (d) O.OOS fC
(a) O'C (b) 32'F
50. Absolute coefficient of expansion of a liquid is 10 times the
(el - 4'C (d) 4°C
volume coefficient of expansion of the vessel. Ratio of
43. A beaker is filled with water at 4°C at one time; the
absolute and apparent expansion of liquid is :
temperature is increased by a few °C above 4°C and at
another time it is decreased by few"C below 4"C. One shall (a) 10 (b) ~
observe that : IEAMCET 1992J 9 9
(a) The level remains constant in each case (e) .!!l (d) 3
3 10
(b) In case I water overflows while in II case its level
comes down 5 1. A unifonn solid sphere of copper is rotating about a
diameter with an angular speed ro. Its temperature is
(c) In case II water overflows while in I case its level
increased by SOOC. aeu being the coefficient of linear
comes down
expansion ofCu, new angular speed of the sphere wilt be:
(d) Water overflows in both the cases
44. The water surface in a lake is just going to frceze. What is (a) ~~w,:- (b) __00",---
1+ 80a cu 1+ 160aeu
the temperature of water at the bottom?
(al O'C (b) 4°C (e) 00(1+ 80acu) (d) _-,=00, : -
1+180a eu
(c) Less than 'O°C (d) More than 4°C
52. X is a uniform wire oflength Ix and radius rx ' Y is another (a) only the block is heated
uniform wire of same material and length /;y and radius r . (b) both block and pin arc heated together
Both wires are heated from 2O"C to 100"C. If Ix =1 y b~t
(c) bot!l block and pin are cooled together
rr <ry:
(d) only the pin is cooled
(a) X expands more than Y
59. Two metal strips that constitute a thermostat must
(b) X expands less than Y
necessarily differ in their:
(e) X expands as much as Y
(a) length
(d) none of the above
(b) mass
53. X is a uniform wire of length Ix and radius rx' Y is another
(c) resistivity
unifonn wire of same material and length I y and radius r y .
Both wires, initially at same temperature, are given equal (d) coefficient of linear expansion
amounts of he a!. If Ix = IY but Yx < ry : 60. Coefficient of apparent expansion of mercury in a glass
(a) X expands more than Y vessel is l53x 10-61"C and in a steel vessel is 144 x 10-6I"C.
(b) X expands less than Y If a for steel is l2x 1O-6I"C. then that of glass is:
(c) X c:xpands as much as Y (a) 35 x 10-'/ oC (b) 6x 10--6/ ,,<:
(d) none of the above (e) 27xI0--6/oC (d) 9xI0-'/oC
61. Coefficients of apparent expansion of a liquid when it is
54. In Q.53, if rx =1, expansion of X and Y will be in ratio: detennined using two different vessels A and B are '11 and
" 4
'12 respectively. If the coefficient ofHnear expansion of the
vessel A is a, coefficient ofl inear expansion ofvesselB is :
(a) '1 1'12 (b) YI -Y2 +a
55. A metal ball immersed in alcohol weighs WI at OOC and w 0.('1 1 +'(2) 3
2
at 59"C. Coefficient of cubical expansion of metal is less
than that of alcohol. Assuming that the density of metal is (e) Y, - Y, (d) Y, +Y,
large compared to that of alcohol, it can be shown that: 3a 3a
(a) WI > w2 (b) WI == w2 62. Coefficients of linear expansion of an isotropic solid along

(d) w2 ==-
w, three rectangular axes in the solid are ax.a ), and a: .
Coefficient of cubical expansion of the solid for small
2
change in temperature can be expressed as: (assume
56. A rod of length 20 em is made of melal A. It expands by cx x' a y and a z to be small quantities)
0.075 em when its temperature is raised from O°C to 100°C.
(a) ax +a + cx z (b) axcx ya z
Another rod of a different metal B having the same length
expands by 0.045 em for the same change in temperature. A
third rod of the same length is composed of two parts, one
(c) ,
a 2 +cx 2 +a 2,
, (d) ·noneofthese

of metal A and the other of metal B. This rod expands by 63. A rod X of length I and l' ~ t oefficient of linear expansion a
0.060 em for the same change in temperature. In Ihe is joined in series with Cilwlber rod Y of length U and of
composite rod, the portion made of metal A has length: coefficient of linear expamh)n 2a. Average coefficient of
linear expansion oftbe comrosile rod is:
(a) 12 em (b) 15 em
(a) 2.25 a (b) 1.67 a
(c) 10 em (d) 8 em
(e) 3.33 a (d) 1.5 a
57. Two straight metallic strips each of length land thickness d
make a bimetallic strip. Their coefficients of linear 64. A brass sphere is heated to give it a small increase of
expansion are a and a'. The bimetallic strip is heated temperature. Percentage change will be maximum in its:
through a temperature .6.9 and it bends into an arc whose (a) density (b) diameter
radius can be expressed as : (c) volume (d) mass
d 65. A one litre glass flask contains some mercury. It is found
(a) (a'--a)M (b) d(a'-a)AS
that at different temperatures, the volume of air inside the
flask remains the same. Volume of mercury in the flask is :
(e) d(a'+n)AO (d) d
(a'+a).6.9 (Given a glass ==9x 10- 6 P C and YHg ==1 .8x 10-4 r C)
58. At some temperature T. a bronze pin is a linle large to fit (a) 100 cm3 (b) 120 em'
into a hole drilled in a steel block. The change in
(c) 150 cm 3 (d) 200 cm 3
temperature required for an exact fit is minimum when:
676

66. A brass wire 1.8 m long at 27°C is held taut with negligible 73. A liquid withcoefficientofvolumeexpansionyis filled ina
tension between two rigid supports. Diameter of the wire is container of material having the coeffici ent of linear
2 mm, its coeffi cient of linear expansion, expansion a.; if the liquid overflows on heating :
(lBrass = 2)( 10-5 fOe and its Young's modulus, (a) y-3a (b) y>3a
Ynrass = 9. ))( 10IO N /m2 . If the wire is cooled to a (e) y < 3a (d) y>3a,
temperature _39°C, tension developed in it is nearly : 74. The apparent coefficient of expansion of a liquid when
heated in a copper vessel is C and when heated in a silver
(,) 380 N (b) 420 N vessel is S. If A is the coefficient of linear expansion of
(e) 510 N (d) 125 N copper, the coefficient of linear expansion of silver is :
67. • A' is a steel rod oflength fA and ' B' a copper rod of length [EAMCET 1991 1
lB' Values of IA and IB such that fA -18 =5 em at all C+S -3A (b) C+3A - S
temperatures, are: (a) 3 3
(Giveno. eu =1 .7 x IO- 5 / c C,(1Steel =1.lxlO-s/0C) S+3A - C C+S+3A
(e) 3 (d)
(a) 21.4 em, 16.4 em (b) 10.8 em, 5.8 em 3
(c) 14.2 em, 9.2 em Cd) 32.4 em, 27.4 em [Hlnl: Given thate '" y - 3.4 andS = '( - 3x; equatey from these
68. It is required to prepare a steel metre scale such that the to find x]
millimetre intervals ate to be accurate within 0.0005 mm at 75 . A solid ball made of material having coefficient of volume
a certain temperature. Maximum temperature variation expansion 8x 1O-61"C is immersed in a liquid having
allowable during the rulings of millimetre marks is: coefficient of volume expansion 3x to- 6I"C. The
(Ct Stccl =13.22)( 10-6 f>C) percentage change in upthrust when temperature is
increased by IOO"C is :
(a) 37.8°C (b) 45.4°C
(a) 0.05% (b) - 0.05%
(e) JO.2°C (d) 62.6"C
(c) 0.5% (d) 0. 11%
69. A rod of length 40 em has the ccefficient of linear
76. A metal wire of length I and radius r is fixed between rigid
expansion Ct l = 6x 1O- 6I"C. The other rod has coefficient
supports. Initially it is just taul. Now, due to decrease in
of linear expansion, (12 = 4 x I 0- 61"c. If the difference in temperature, the tension developed in the wire :
their lengths at all temperatures remains the same, the (a) oc I (b) oc~
length of the other rod is: I
(a) 26 cm (b) 32 cm 1
(d) oc -
(e) 60 em (d) 80 em r'
70. A copper disc with a central hole is heated. The diameter of 77. The relation between the volume and temperature of a
the hole ; IEAMCET 1994) sample of water in the range O°C to IOO°C is best
(a) Increases represented by :
(b) Decreases
(c) First decreases, then increases
(d) Remains unchanged
I
71. A solid ball of metal has a spherical cavity inside it. If the
(a)
~
ball is heated, the volume of the cavity will :
(a) Increase (b) Decrease Temp. - Temp. -
(c) Remain unchanged (d) Have its shape changed

~
72. There are two spheres of same material and radius. One is
solid and the other is hollow. If they are heated to the same
temperature the expansion of: I (d) ~
(a) Solid sphere is more (e)
~
(b) Hollow sphere is more
(c) Solid and hollow spheres are equal Temp._ Temp. -
(d) Solid is outwards while that of hollow inwards
78. Which of the following curves represents variation 01
density of water with temperature best?
83, The volume expansibility of brass and iron are

(a) il / 54x 10-6 K- 1 and 36x 10- 6 K- 1 respectively. They are


supported between rigid walls. They are heated so that they
have the same rise oftemperarure and there is no bending. If
Temp. _ Temp. _ the same thennal stress is developed in them, the ratio of
Young's modulus of brass and iron will be:

(e)il~ (d)fl~
(a) 2 : 3
(e) I: I
(b) 3: 2
(d) 4: 9
84. A glass bortleofvolume 100 cc at O"e is filled with paraffin
Temp. _ at 2O"C. If the density of paraffin at O"C is 0.8 glee,
Temp. _
"I paraffin =I Ox 10-4 fOC and the coefficient of linear
79. Three metal rods of the same length A
expansion of glass is lOx 10- 6 rc, the mass of paraffin is :
and area of cross- section fonn an
equilateral triangle as shown in Fig. (a) 78.45 g (b) 78.48 g
14.13. D is the mid-point of side BC. (e) 81.6 g (d) 80 g
If AD is independent for small change 85. AU-tube coUlains mercury with onc limb at OOC and the
m temperature, then( ct I is the other at 1WC. The heights of the mercury columns are 60
coefficient of linear expansion for rod B 0 C em at O°C and 62 em at 100°C. The coefficient of volume
BC and (12 for rods AB and AC) : Fig. 14.13
expansion of mercury per °c is:
(b) a, =a,2- (a) 2/100 (b) 2/60
(e) 2162 (d) 3.33x 10-4
(c) 0.
2 =(1, (d) 0., =40. 2
86. A barometer having a metallic scale reads a height Ho at
[Hint: ADZ = /2 - ~ -[l(l + ctzt)r - [~(1 + ct1t)r. expanding temperature ErC. The true scale is calibrated at O°C. If a is
the coefficient of linear expansion of the metal and y the
binomially and negtecting (l2 and higher order terms, keeping coefficient of cubical expansion of mercury, the correct
AD2 independent of t.] pressure H corr is :
80. The moment of inertia of a body is I and its coefficient of (b) H (I-JaG)
(a) H (1+ y(I)
linear expansion is a, If the temperature of the body rises by o (l+a9)
a small amount llT. then change in its moment of inertia • (1+«0)
about the same axis is : (e) H (1+«0) (d) H (I- aO)
a lilT , (I+ye) 0(1-300)
(a) al~T (b) 2al~T (e) 4al~T (d)
2 87. A metallic sphere (the coefficient of linear expansion
I:J Ur 2
[Hint:l=mr :.-= -=2aATJ a=2x1O- 6 per°C) of radius 10 cm is heated at the rate of
I , 2"<: per second. Its volume will increase at the rate of:
81. When a block of iron floats in mercury at O"e, a fraction kj (a) 6xlO-Ocm 3/s ' (b) 12 x lO--6 cm 3/s
of its volume is submerged, while at the temperature 6QOC,
a fraction k2 is seen to be submerged. If the coefficient of (c) loox 10- 3 cm 3 /s (d) 8nx ]0-3 cm 3Is
volume expansion of iron is "I Fe and that ofmercwy is "IHg' 88. An aluminium cube 10 em on a side at O°C is heated from
k lOoC to 30°C. The change in its density is given by
then the ratio-I can be expressed as: lIlT 200lJ
k, [Given: Coefficient of volume of expansion of aluminium
1+6QyFe 1-6OyFe = 7.2x 10- 5re, Density of aluminium al OOC = 2700
(a) (b) kglm') :
l+6OyHg 1+6Oy Hg
(a) - 3.87 kglm' (b) -38.7 kglm'
1+6OyFe 1+ 6OyHg
(e) (d) (e) 4.5 kglm' (d) 1.83 kg/m'
I - MYH, 1+ 6OyFe
89. Two unifonn brass rods A and B of length I and 2/ and radii
82. When a rod is heated but prevented from expanding, the 2r and r respectively are heated to the same temperature.
compressional forte developed is independent of: The ratio of the increase in the volume of A to that ofB is:
(a) Length of rod IAMU20091
(b) Area of cross-section of rod (a) I: I (b) I: 2
(c) Material of rod (e) 2: I (d) I: 4
(d) Change in temperature
678

90. A platinwn resistance thennometcr is constructed which reads (c) bend with steel on convex side
O°C at ice point and 100°C at steam point. The resistance of (d) none
platinum coil varies with t as R/ = Ro (1+ oJ +~(2). If f P 95. The volume of a metal ball increases by 0.18% when its
denotes temperature on the platinum resistance thennomctcr temperature is increased by 30°C. What is the coefficient of
and t denotes temperature on mercury scale. Then resistance linear expansion of metal?
as the function of t p will be : (a) 2x 10- 5f'C (b) 3x 10-' I"C
(a) Ro[l +al p +px( IOO)t p l (c) 6x 10- 5FC (d) 1.8x IO- ' I"C
(b) Ro[l +ptp+a x (IOO)tpl 96. 1be coefficienl of apparent expansion of a liquid when
(e) Ro[l+13tp+w p J detennined using two different vessels A and B are y \ and 1 2
(d) Ro[Pt p +alpl respectively. If the coefficient oflinear expansion of vessel A
91. A barometer reads 75.0 em on a steel scale. The room is a, the coefficient of linear expansion of vessel B is :
temperature is 30°C. The scale is correctly graduated for (a) 0:)'\Y2 (b) Y\ -Y2
O°C. The coefficient of linear expansion of steel is )'\ +12 2a
a. :: 1.2x 10-5fOe and the coefficient of volume expansion
(c) )'\ -)'2 + a
of mercury is r = 1.8x 10-4 re. The correct atmospheric (d) 1 \ - )'2 +(1
3 3
pressure will be :
97. Two unifonn rods of equal length (10 ) and equal masses
(a) 74.89 em (b) 76.89 em have coefficient of linear expansion (1 and 2a are placed in
(e) 70.89 em (d) 72.89 em contact on a smooth horizontal surface as shown in
92, On a X temperature scale, water freezes at - 125.0° X and Fig.14.14. The temperature of system is 9 cC. Now the
boils at 375.0° X. On a Y temperature scale water freeze s temperature is increased by 6.9"C. The junction of the rods
at -70.0° Y and boils at - 30.0° Y. The value of will shift from its initial position by :
temperature on X scale equal to the temperature on of 10 10
50.0° Y on Y-scale is :
(a) 455.0° X (b) _ 125.0° X
Initial temperature = eoe
(e) 1375.0° X (d) 1500.0° X Fig. 14.1 4
93. If He ,HK and H F are heat required to raise the IOa6.9 10a6.9
(a) - - (b) loMe (e) loMe (d) - -
temperature of one gram of water by one degree in Celsius,
2 3 64
Kelvin and Fahrenheit temperature scales respectively then
98. The coefficient of linear expansion of an inhomogeneous
(a) HK. >Hc >HF (b) HF >He >HK
rod changes linearly from <11 to (12 from one end to the
(c) HK = He >HF (d) HK =Hc =HF other end of the rod. The effective coefficient of linear
94 . Two identical rectangular strips one of copper and other of expansion ofthe rod is :
steel are rivened together to form bimetallic strip
(<<copper > a slccI } On heating this strip will : (a) <1 1 + (12
(a) remain straight
(c) ~a. 1a. 2
(b) bend with copper on convex side

ANSWERS

I. (0) 2. (a) 3. (d) 4. (a) 5. (a) 6. (a) 7. (b) 8. (a) 9. (a) 10. (e) 11. (e) IZ. (e)
13. (b) 14. (e) IS. (e) 16. (b) 17. (e) 18. (b) 19. (d) 20. (a) 21. (b) 22. (d) 23. (d) 24. (e)
25. (d) 26. (d) 27. (a) 18. (b) 29. (d) 30• . (a) 31 . (e) 32. (b) 33. (e) 34. (b) 35. (e) 36. (e)
37. (<I) 38. (e) 39. (d) 40." (e) 41. (d) 42. (d) 43. (d) 44. (b) 45. (I) 46. (a) 47. (e) 48. (b)
'9. (d) SO. (a) 51. (b) 52. (e) 53. (a) 54. (b) 55. (c) 56. (e) 57. (a) 58. (a) 59. (d) 6<). (d)
61. (b) 62. (a) 63. (b) 64. (e) 65. (e) 66. (a) 67. (e) 68. (a) 69. (e) 70. (a) 71. (a) 72. (e)
73. (b) 74. (b) 75. (a) 76. ee) 77. (d) 78. (d) 79. (d) 80. (b) 81. (a) 82. (a) 83. (a) 84. (b)
85. (d) 86. (e) 87. (c) 88. (a) 89. (e) 90. (a) 91. (a) 92. (e) 93. (0) 94. (b) 95. (a) 96. (d)
97. (e) 98. (b)
679

[BI More than One Choice is Correct 9. A vessel is partly filled with liquid. When the vessel is
cooled to lower temperature the space in the vessel
I. Which of the following statements are not true ?
unoccupied by the liquid remain constant. The volume of
(a) Size of degree is smallest on Celsius scale
the liquid (VL) and the volume of the vessel (VV ), the
(b) Size of degree is smallest on Fahrenheit scale coefficient of the cubical expansion of the material of the
(c) Size ofdegree is equal on Fahrenheit and Kelvin scale vessel (Yv ) and of the liquid (YL ) are related as:
(d) Size of degree i~ equal on Celsius and Kelvin scale (a) YL >Yv (b) Yr <Yv
2. Reading of temperature may be same on:
(c) 2l:.. =Vv (d) "tv = VL
(a) Celsius and Kelvin scale "IL VL h Vv
(b) Fahrenheit and Kelvin scale
10. When the temperature of the copper coin is raised by 80°C,
(c) Celsius and Fahrenheit scale its diameter increases by 0.2%.
(d) All the three scales (a) percentage rise in the area of face is 0.4%
3. To measure a temperature say around 400°C which of the (b) percentage rise in the thickness is 0.4% .
following thermometers can be used most conveniently?
(c) percentage rise in volume is 0.6%
(a) Gas thermometer (b) Mercury thermometer
(d) coefficient of linear expansion of copper is
(c) Platinum thermometer (d) Thermocouple 025x lO-4;oC
4. Which ofthe following statements are true ?
II. A spinning wheel A is brought in contact with another
(a) Rubber contracts on heating wheel B initially at rest. Because of friction at contact, the
(b) Water expands on freezing second wheel also starts spinning. Which of the following
(c) Water contracts on heating from 0 to 4°C energies of the wheel B increase?
(d) Water expands on heating from 4 to 40°C (a) Kinetic (b) Total
5. If a, p and y are coefficients of linear, superficial and (c) Mechanical (d) Internal
volume expansion respectively then: 12. Choose the wrong statements from the following two
(a) (pla)=(1/2) (b) (P/y)=(213) spheres made of the same material have same diamete~.
(c) (yla)=(3/l) (d) (pla)=(y/p) One sphere is hollow and the other is solid. If they are
heated through the same range of temperature:
6. Due to thermal expansion, with rise in temperature:
(a) the hollow sphere will expand more than the solid
(a) Metallic scale reading becomes lesser than true value
sphere
(b) Pendulum clock becomes fast
(b) the solid sphere will expand more than the hoHow
(c) A floating body sinks a little more sphere
(d) The weight ofa body in a liquid increases (c) both spheres will expand equally
7. Two identical beakers are filled with water to the same level (d) the hollow sphere will not expand at all
at 4°C. If one say A is heated while the other B is cooled,
13. In the plot ofoC - OF (x- ygraph) :
then:
(a) slope is 9/5 (b) yintercept is 32°F
(a) Water level in A will rise
. . 160
(b) Water level in B will rise ()
c xmterceptls - - (d) x intercept is 32°C
9
(c) Water level in A will fall
(d) Water level in B will fall ANSWERS
8. A bimetallic strip is fonned out of two identical strips, one
of copper and the other of brass. The coefficients of linear
1. (a) and (c); 2.(b)andCc); 3. (c) and (d); 4.All;5.(b) and
expansion of the two metals are a c and a B' On heating, (c); 6. (a), (c) and (d); 7. (a) and (b); 8. (b) and (d); 9. Ca) and
the temperature of the strip goes up by I:J.T and the strip (d); 10. (a), (c) and (d); 11. All; 12. (a), (b) and (d); 13. (a),
bends to form an arc of radius of curvature R. Then R is : (b) and (c)
lIlT 19991
Ic) Assertion-Reason Type Questions
(a) Proportional to I:J.T
(b) Inversely proportional to I:J.T (a) If both A and R are true and R ;, the correct
explanation of A.
(c) Proportional to laB -a C I
(b) If both A and R are true but R ;, not correct
(d) Inversely proportional to la B - ac I
explanation of A.
(c) If A is true but R is false. (c) Liquid level drops with (I) y=3a
(d) If A is false but R is true. respect to container
(e) Ifboth A and R are false. (d) Liquid level remains (,) y>3a
I. (A): Given any two objects, the one with the higher same with respect to
temperature contains more heat. ground
(R): Integral energy of an object is function of absolute
2. Apply your understanding of thennal expansion to match
temperature.
Column~I and Colum-II.
ANSWERS
Column - [ Column - II
1. (d) (a) A scale that measures (p) a (coefficient of linear
length gives correct expansion)
(D) Integer Type Questions reading at temperature
9. At a higher
1. If the volume ofa block of metal changes by 0.12% when temperature €I', due to
heat is changed from 40°C to 60°C, find the linear the linear expansion of
expansion coefficient of the metal. (in to-s/K) the scale, the scale
2. In the shown planar frame made of thin unifonn rods, the reading will be
length of section AB and EF is I, and its, thermal linear different from the true
coefficient of expansion is a"
The length of section CD is value. Fractional
12 and its thennallinear coefficient of expansion is 0. 2 ' CB change in the measured
and DE are of same length having thermal linear coefficient length, due to thennal
a expansion of the scale,
of expansion is 0. 2 ' It is given that ~2 =3.5. Points A,S,E
al will depend on (assume
insignificant expansion
and F reside on same line, that is, sections AS and EF
of the actual length
I
overlap. Then find -the ratio of.1.., for which the distance being measured)
II
between end A and end F remains the same at all
(b) Suppose that a (q) 9' -9
pendulum clock keeps
temperatures:
proper time at
D[ temperature 9. At a
higher temperature 6',
A E due to linear expansion,
Fig. 14.15 length-' of pendulum
changes and so also the
time period. If an actual
time duration t ;s
1.22.7 measured by the clock
at the temperature 6',
(E1 Match the Columns fractional loss of time
1. Whenever a liquid is heated in a container, expansion in will depend on
liquid as well as container takes place. Ify is the coefficient
(c) If temperature of a rod (I) Coefficient of volume
of volume expansion of liquid and a is coefficient of linear
is increased from a to expansion of the liquid
expansion of container
a', thennal strain in it
Column - [ Column - II will depend on

(a) Liquid level rises with (P) y=2a (d) If a rod is floating on a (s) Young's modulus
respect to container liquid and the temp-
erature ;, increased
(b) Liquid level remains (q) y<3a from €I to a', upthrust
same with respect to will change and it will
container depend on
3. Fig. 14.16 shows three spheres X ,Y and Z. These are of the ANSWERS
same material and at same temperature. X is a solid sphere
1. a-+ s; h-+ r; c-+ p,q; d -+ p.q
of radius R, Y is an extremely thin-walled hollow sphere of
radius R and sphere Z, also of radius R,has a cavity ofeadius 2. a -+ p,q; h-+ p,q; c-+ p,q; d -+ p,q,r
RJ3, as shown. Match the Column-I with Column-II. 3. a -+ p,q.r; h-+ p,q,r; c-+ does not match; d -+ does not match
4. a -+ s; h-+ q; c-+ p.q; d -+ q

X
0 Y
FIg. 14.16
Z
IF) Linked Comprehension Type
Paragraph - 1
In a constant volume gas thermometer, the physical change
involved is the variation of preswre of a fixed volume of a
gas with temperature. The thennometer can be calibrated
by using the ice and the steam points of water. However, in
Column - I Column - II
1954, a new scale based on two fixed points was adopted.
(a) If same amount of heat (P) X The first reference temperature on the new scale is the
is given to each sphere, absolute zero and the second reference temperature is the
expansion of which ' triple point of water' . Triple point ofwaler is a temperature
spheres will be such that the three phases of water (ice, water and vapour)
different from each coexist in equilibrium at a definite pressure.
other? A student using a constant volume gas thermometer
measures the pressure at the triple point of water (0.0 1DC) to
(b) If the spheres are (q) Y be 4.5x 104 N/m2 and the pressure at the boiling
heated to a common
temperature of water (100"C) to be 6x 104 N/m 2. Using
temperature, expansion
this infonnation and assuming that the pressure varies
of which spheres will
linearly with temperature, answer the following questions.
be the same?
I. The Celsius temperature at which pressure of the gas is zero
(0) As the temperature is (r~ Z will be nearly:
increased, sphere Z will (a) - 273'C (b) - 300'C
expand but its cavity (0) -267'C (d) -312'C
size
2. Celsius temperature of absolute zero as detennined by the
(d) For a given amount of (s) Decreases student will be nearly:
heat · supplied to each (a) -267'C (b) -312'C
sphere, increase of (0) -273'C (d) -300'C
temperature will be the
3. Pressure of a given amount of gas is directly proportional to
same 'for which the temperature on the Kelvin scale. Which of the following
spheres?
is correct?
4. Match Column-! and Column·II, (a) This statement is strictly valid for the gas in the
thennometer being used by the student
Column -] Column -II (b) This statement is not strictly valid for the gas in the
thermometer being used by the student
(a) Bimetallic strip (P) Radiation from a hot
body
(c) For the given thennometer, strict validity of this
statement cannot be determined
(b) Steam engine (q) Energy conversion (d) This statement will become more and more valid as the
gas pressure increases
(0) Incandescent lamp (r) Melting
4. Using the given tbennometer, the pressure measured by the
(d) Electric fuse (s) Therr"al expansion of student when the Celsius temperature is 50"C will be :
solids (a) 4.45x 104 N/m2 (b) 4.85x 104 N/m2
(c) 5.25x 104 N/m2 (d) 5.75x 104 N/m2
5. So as to observe the effect of nature of gas, the student between two rigid supports A and B. A B

~,, " '~'~"i'X2.1


perfonns measurements with thennometers having
different gases. He will find that: distance thefrom
Initially, XI the support'
left end A'isand
of the rod at . _ _~.
(a) measurements are almost independent of the nature of the right end at distance x 2 from the
gas if the gas pressure is low and the temperature is support B. Temperature of the rod is
well above the temperature at which the gas liquefies now increased by llT so that the rod
expands and its length increases. Let Fig. 14.17
(b) measurements are almost independent of the nature of
gas if the gas pressure is low and the temperature is I!.T be large enough that increase in
below the temperature at which the gas liquefies length, ifthe supports A and B were not there, will be more
than XI + x 2 . However, the presence of rigid supports A and
(c) measurements are almost independent of the nature of
gas if the gas pressure is high and the temperature is B will prevent any increase in length beyond xI +X2 . Stress
below the temperature at which the gas liquefies in the rod can be expressed as :
(d) measurements are almost independent of the nature of
gas if the gas pressure is high and the temperature is
(a) r(Xl :X2 +allT) (b) AYCt..6T

well above the temperature at which the gas liquefies


(c) Y XI +x2
Paragraph. 2 L
If the ends ofa rod are rigidly clamped so as to prevent any (Here L is the initial length of rod; other symbols denote
actual expansion or contraction and then the temperature of physical quantities already described above.)
the rod is changed, the rod will develop tensile or 9. Let us now consider an object that is kept under pressure
compressive stresses, also called 'thennal stresses'. These and not allowed to expand on increasing the temperature. If
stresses could sometimes be adequate to cause pennanent the temperature is increase by llT, increase in the pressure
strain or could also break the rod. Concrete highways are will be :
always left with gaps between the sections in order to (a) yBIIT (b) aBlIT
provide for the expansion or contraction of concrete. (e) aBpllT (d) YBpllT
Similarly, if the temperature of an object kept under
(Here a is the coefficient of linear expansion and 'I is the
pressure is increased but the object is not allowed to
coefficient of volume expansion; B is the bulk modulus and
expand, the pressure will also increase. Assuming a p the density of the material, I!.T is the increase of
negative sign for a compressive stress and a positive sign temperature.)
for tensile stress, answer the following questions.
6. Consiqer a rod rigidly clamped at its ends. As the Paragraph - 3
temperature is increased: A copper rod of length II and a steel rod of length 12 , both
(a) the rod develops a tensile stress having the same area of cross-section, are placed end to end
between two rigid supports X and Y such that their system
(b) the rod develops a compressive stress just fits between the supports with no initial stress in the
(c) ~he rod develops a tensile stress if the material of rod rods. Temperature of the rods is now increased by llT. If
has a high value of a (coefficient of linear expansion) YCu is the Young's modulus of copper and a eu its
and a compressive stress if the material of rod has a low coefficient of linear expansion, while Y St is the Young's
value ofa modulus of steel and a St its coefficient of linear expansion,
(d) it cannot be predicted whether the rod will develop a answer the following questions.
compressive stress or a tensile stress (Assume that area of cross-section does not change and the
7. Again consider a rod rigidly clamped at its ends. As the rods do not bcnd. Also assume that there is no defonnation
temperature of the rod is increased or decreased, thermal of the supports.)
stress in the rod can be expressed as : 10. As the temperaturc is increased:
(a) YLa./).T (b) y"AIIT (a) the length of the composite rod (copper + steel) does
(e) -Ya./).T (d) -YLa./).T not change but that of each individual rod changes
(b) the length of the composite rOd does not change and
(Here L is the length of the rod and A its area of
that of each individual rod also does not change
cross-section; Y is the Young's modulus of the material of
rod and a its coefficient of linear expansion; llT is the (c) the length of the composite rod increases and that of
change of temperature, i. e. • final temperature - initial each individual rod also increases
temperature.) (d) the lcngth of the composite rod decreases; length of
onc of the two rods increases while that of the other
8. Now consider that the rod is not fixed at the ends so that it is
decrcascs
free to expand or contract. The rod is kept horizontally
THERMOMETRY AND EXPANSION 683
II. As the temperature is increased: 5. What is the value of (a) O°F on Celsius scale? (b) 0 K on
(a) the stress in the steel rod is more than the stress in the Fahrenheit scale?
copper rod [Ans. (a) -17.8°C (b) 459.67°P]
(b) the stress in the steel rod is less than the stress in the 6. The highest and lowest temperature recorded on earth are
copper rod 136°F (at Azizia, Libya in 1922) and -12TF (at Vostok
(c) the stress in the steel rod and that in the copper rod are station, Antarctica in 1960). Express these extremes in cc.
equal [Ans. 57.7°C and -88.3°C]
(d) the stress in the steel rod will be more than that in the 7. At what temperature is the Fahrenheit scale reading equal to
copper rod if its length is greater than the length of the (a) twice (b) half, of Celsius?
copper rod and vice-versa
[Ans. (a) 160° (b) (320113) = 24.6°J
12. As the temperature is increased, stress in the steel rod can
8. A Fahrenheit thennometer registers 110° while a faulty
be expressed as :
Centigrade thennometer registers 44°. What is the error in
(a) YStaSt6.T (b) the latter?
(c) [/la cu ,:12(J.SI jYSt.6.T(d) [Ans.0.667°C]
9. A faulty thennometer read.s 5°C in melting ice and 99°C in
steam. Find the correct temperature in of when this faulty
thermometer reads 52°C.
13. As the temperature is increased by I:J.T, length of the copper
[Ans. 122°P]
rod will be :
10. The Reaumur temperature scale is almost obsolete, but is
(a) I, (b) 1,(I+a","T)
still used to some extent in France. On this scale freezing
(c) lI[l+aCu.6.T-~j
AY",
(d) ',(l+(a cu -a St )6.T] point and boiling point of water are 0° and 80° respectively.
What Celsius temperature corresponds to 32°R?
(HereF is the magnitude offorce with which the copper rod [Ans.40°C]
acts on the steel rod and A is the area of cross-section.) II. Explain why a column of mercury in a thennometer first
descends slightly and then rises when it is heated"?
ANSWERS
[Ami: Due to heating first the bulb expands and so the column of
Paragraph-l 1. (b) 2. (d) 3. (b) 4. (oj 5. (a) mercury descends and then both bulb and mercury expand and
expansion of mercury is more than glass, the mercury column
Paragraph-2 •. (b) 7. (c) 8. (d) 9. (a)
begins to rise.]
Paragraph-3 10., (a) 11. (c) 12. (d) 13. (c)
12. Fill in the blanks:
(a) No negative temperature is possible on ...::; .. :.... . scale.
(G) Subjective Type Questions
(b) The size of degree on temperature scale is smallest for
1. What is minimum possible temperature? Is there also a ............. scale.
maximum possible temperature?
(c) Correct'value ofO°C on Kelvin scale is ............ .
[Ans. 0 K; No]
(d) For a given thennoCouple the temperature at which
2. 'Distinguish between °C and Co. thenno-EMF is maximum is called .......... temperature.
[Ans. °C is used to specify a given temperature while C~ to (EAMCET 1993)
represent change in temperature on Celsius scale]
[Ans. (a) Kelvin (b) Fahrenheit (c) 273.15 (d) Neutral]
3. If your doctor tells you that your temperature is 310 degree
13. Explain why (a) rubber contracts on heating (b) sometimes
above absolute zero, should you worry? Explain your
answer. water pipes burst in severe winters.
[Ans. 310 K '" 37°C = 98.6°F which is normal temperature of [Ans. (a) See note in § 14.4, (b) On freezing water expands and
human body, so one should not worry.] due to expansion large force acts and pipes may burst]

4. Many medicine labels inform the user to store below 86°F. 14. Water expands when it freezes . Can we define a coefficient
Why? of volume expansion for freezing process?
[Ans. 86°F ~ 30°C. Above this temperature the chemical [Ans. No; coefficient of expansion is defined as change in volume
compounds in the medicine may change due to thennal per unit volume per Co rise in temperature and as freezing takes
decomposition or chemical reactions making the medicine place at constant temperature I1T -+ 0, r -+ wji.e., it cannot be
ineffective or injurious.] defined.]
15. A steel surveyor's tape is exactly correct at 20"C. On a day contact with the lower surface of ice ? (b) at the bottom of
the temperature is 45°C, a distance ofS7.13 m is measured. the lake?
Is the true distance more or less than this value? [Ans. (a) O°C, (b) 4°C]
[Ans. More; See § 14.4 (5)] 26. Explain how fish and other aquatic animals remain alive in
16. Two identical strips one of copper and the other of steel are a lake even when it freezes at the surface.
riveted together to fonn a bimetallic strip. What will [Hint: See note in § 14.4 (12)]
happen on heating? 27. A block of wood is floating on water at ooe with a certain
(Ans. The strip will bend; See § 14.4 (4)] volume V above water level. The temperature of water is
17. If a brass plate with a hole in it expands, does the hole get slowly raised from ODe to 20De. How will the volume V
larger or smaller? change with rise in temperature ?
[Ans. Larger; See § 14.4 (8)] [Ans. First increase and then decrease; See question XI in Chapter
18. A brass disc fits snugly in a hole in a steel plate. Should you 10.]
heat or cool the system to loosen the disc from the hole? 28. What should the length of a steel and copper rod be at ODC
[Ans. As a B > uFe' on heating brass will expand more than steel so that the length of the steel rod is 5 cm longer than the
and so disc will become more tight, so to loosen it from the steel copper rod at any temperature? [0. S = L2x 10- 5leo and
plate we must cool the system.] o.Cu = 1.6x 1O-5ICO]
19. A solid ball of metal has a spherical cavity inside it. What
[Hint: See Question IV with (ls)-} leu)= 5 em]
will happen to the volume of cavity if the ball is heated?
[Ans. Increase; See § 14.4 (8)J [Ans. (Ls) '" 120 em and (leu) = 15 em]
20. Does the change in volume of a body when its temperature 29. A clock which keeps correct time at 30°C has a pendulum
is raised, depend on whether the body has cavities inside? made of iron. How many seconds will it gain or lose per day
[Ans. No; See § 14.4 (8)] when the temperature falls to OOe? [(1Fe '= 12x 10-6 leo]
21 . Will five·litre of benzene weigh more in summer or in [Ans. Gain 15.55 s]
winter? 30. A steel girder is 50 m long and has a croSHectional area
[Ans. Winter; See Question III in Chapter I] 250 cm 2 . What is the force exerted by the girder when
22. A long cylindrical vessel having a linear coefficient of heated from 5°C to 25 De ?
expansion (1 is filled with a liquid upto a certain level. On [o.s = 11x1O- 6/CoandYs =2xlO ll Pal
heating, it is found that the level of the liquid in the cylinder
remains the same. What is the coefficient of volume [Ans. 11 x 105 NJ
expansion of the liquid? 31. A composite rod is made by joining a copper rod end to end
[Ans. 3(l; See § 14.4 (9)] with a second rod of different material, but of same
23. A metal ball inunersed in alcohol weighs WI at ooe and W2 crosHection. At 25°C the composite rod is 1 m in length, of
at sODe. The coefficient of cubical expansion of the metal is which the length of copper rod is 30 cm. At 125°C, the
less than alcohol. Assuming that density of the metal is length ofthe composite rod increase by 1.91 nun. When the
large compared to that of alcohol, find which ofW! and Wz composite rod is not allowed to expand by holding it
is greater? between two rigid walls, it is found that the lengths of the
constituents do not change with rise in temperature. Find
[Hint: According to § 14.4 (11), (Th'ITh) = [(1+ "fsM) I
the Young's modulus and coefficient of linear expansion of
(I+YLl!.ElH But as Th=Wo-W and "fL >Ys
[CWo - W2){ (Wo - W])]< I, i.e. , W2 > WI] the second rod if o.Cu = 1.7x 10-5leo and YCu = l.3x 10]1
24. A piece of metal floats on mercury. The coefficient of N/m 2 .
volume expansion of metal and mercury are y 1 and y 2 [Hint: See solved Problem 14 in Chapter 11.]
respectively. If the temperatures of both mercury and metal [Ans. C( = 2.0 xlO- s{CO and Y = 1.1 x 10]] Pal
are increased by an amount !J.T, by what factor does the 32. A composite bar of length L = LIf. + LB is made from a bar
fraction of the volume of the metal submerged in mercury of metal A and length LIf. • attached to a bar of metal Band
changes? [lIT 1991] length LB' Show that the effective coefficient of linear
[Ans. ('(2 - Y]) l!.T; See solved Problem 17 in Chapter 10.] expansion 0. for this bar is given by
25. The top ofa lake is frozen. Air in contact is at-15°C. What o.=[u,4LIf. +uBLB]/L
do you expect the maximum temperature of water (a) in [Hint: L'= L(I+ ct6.e) = LA + Ls]
33. Two rods of different materials but having the same length 35. Show that when the temperature of a liquid in a barometer
.r.. are arranged end to end between fixed rigid supports. The changes by de and the atmospheric pressure is constant, the
L L height h changes by!J.h = 'Yh de, where 'Y is the coefficient of

I Vb Ct1
7 I,, Y2,CX:!
IT volume expansion of liquid. Neglect the expansion of the
glass tube.

I 7 m I
Fig. 14.18
T+.6.T
[Hint: See note in Question VI.]
36. Show that for a substance of density p and coefficient of
volume expansion 'Y, when temperature changes by Lie,
density changes by:
temperature is T and there is no initial stress. The rods are
~p=4'p~e
heated, so that their temperature increase by ll.T. Show that
[Hint: See § 14.4 (10) and find Ap '" (P' -p)]
the rod interface is displaced upon heating by an amount,
Y CX Y 37. Show that with risf; in temperature by Lie, a pendulum
M=[Ct 1 t- 2 2]UT clock, in a time interval t will lose time by:
Y, +Y2 I
Lit = - (lJ' Lie
[Hint: See problem 13 in Chapter 11 with ~ = L,. and 2
- I,) m (L, - L!,)]
t.L = (L{ where a. is the coefficient of linear expansion of the
34. A unifonn pressure P is exerted on all sides of a solid cube pendulum.
at temperature re. By what amOWlt should the temperature
[Hint: As T = 21t(L/ g)lf2; (IlL I L) = ..!.wMI(as IlL = La Ae), i.e.,
of the cube be raised in order to bring its volume back to the 2
value it had before the pressure was applied? Given the in time T clock loses time by AT, so in the interval twill lose time,
coefficient of volume expansion of cube is 'Y and bulk I
AT (t I T) = - alAe or See § 14.4 (6)]
modulus of elasticity B. 2
[Hint: See solved Problem 18 in Chapter 11.]
Heat and Calorimetry
§ 15.1 Heat mechanical energy, i.e., work can be converted into
The energy associated with configuration and random heat but whole of heat can never be converted into
motion of the atoms and molecules within a body is called work.
internal energy. The part of this internal energy which is (4) Through a large number of experiments Joule and
transferred from one body to the other due to temperature others have shown that when mechanical energy
difference is called heat, i.e., heat is energy in transit which is (work) is converted into heat, the ratio of work done
transferred from one body to the other due to temperature (W) to heat produced (Q) always remains the same and
difference between them. Regarding heat it is worth noting constant, i.e.,
that: (WIQ) = consU. = J or W = JQ .... (i)
(1) As it is energy, it is a scalar, with dimensions The constant J is called mechanical equivalent of heat
[ML2T-2 ] and SI unit joule while practical unit and has value 4.2 J/cal (more precisely 4.186 J/cat).
Here it must be noted that J is not aphysical quantity but
calorie (cal) or British thermal unit (Btu). These units a conversion-factor which merely expresses the
are related to each other through the relation: equivalence between joule and calorie, i. e.,
1 cal=4.186J=3.96xlO- 3 Btu 1 calorie = 4.2 joule
IfW and Q are in the same units, J = 1.
Note: (i) The unit calorie is defined as amount of heat required to
raise the temperature ofl g of water through leo (more (5) When heat is given to a body, usually its temperature
specifically from 14.5 10 15'soC) while Btu as heat required increases. However, it is also found true that
to ra ise the temperature of I pound of water through I po temperature of a body may not change when it is
(from 63 to 64°F). heated (as in melting of solid or boiling of a liquid,
(ii) The 'Calorie' with capital C used in nutrition (to specify i.e., in change of state) or its tempflrature may
energy value of food contents) is written as 'Cal' and referes increase without heating [as in shaking a liquid in a
to kilocalorie, i.e.,
thennos-flask or compressing a gas in a cylinder, i.e.,
I Cal'" I kcal '" 103 cal by converting work (ME) into heat (internal energy)).
(2) In a body heat is stored as internal energy, heat unlike (6) The heat lost or gained by a system depends not only
temperature, pressure, volume or internal energy is on the initial and final states, but also on the path
not an intrinsic property ofthe body or system. It has taken by the process, i.e., heat is path dependent· and
meaning only as long as it describes the transfer of is taken to be positive if the system aqsorbs it and
energy into or out of the body or system, i.e., we negative ifreleases it.
cannot say that a body contains certain amount of heat Problem 1. How many calories of heat will
but that body can transfer certain amount of energy as approximately be developed by a 210 watt bulb in 5 minute?
internal energy under specified conditions. Solution: 210 watt means 210 J/s [as watt=joulelsec], so
(3) As heat is a fonn of energy it can be transfonned into heat generated by the bulb in 5 minute,
others and vice-versa, e.g., 'Thermocouple' converts =2IO(J/s)x(5x60s)=63xIO' J
heat energy into electrical while a 'resistor' electrical
into heat energy. Friction converts mechanical energy Now as 4.2 J is equivalent to 1 cal of neat, the heat
into heat while heat-engine converts heat-energy into generated by the bulb in 5 minute in cal will be,
mechanical. Here it is worth mentioning that whole C!f = (63 x 10 3 )/4.2 = 15 x 10 3 cal = 15kcal Answer

... i.e., heat required to produce a given change in temperature ofa body depends on external conditions such as pressure or volume.
§ 15.2 Specific Heat Capacity c In CGS units*"", In SI units, i. e.,
Specific heat
When heat is given to a body and its temperature i. e., callg Co J/kgK
increases, the heat required to raise the temperature of unit Ice (solid) 0.5 2100
mass of a body through 1 Co (or K) is called specific heat Water (liquid) 1 4200
capacity or simply specific heat of the material ofthe body. If Steam (gas) 0.47 1970
Q heat changes the temperature of mass m by !:J.T:
(6) If a substance is undergoing change of state which
c' =JL
m"'T .... (.)I takes place at constant temperature [called isothennal
change), specific heat:
Regarding specific heat it is worth noting that:
c=L=JL=oo [as~T=O]
(1) Its dimensions ar~· tL2T-2e-l] while SI units J/kg K ml1T mxO
and practical units callg CD, i. e., specific heat ofa substance at its meltingpoint or
(2) It depends on nature of material of body. Dulong and boiling point or isothermal change is infinite.
Petit has found that for elemental solids (with few (7) If the temperature of a body changes without transfer
exceptions such as carbon) of heat with the surroundings (adiabatic change) as in
(Atomic weight) x (sp. heat) = 6(callCO) shaking a liquid or compressing a gas,
=constant .... (ii)
c=L=_O_=o [asQ=O]
So, heavier the element lesser will be the specific ml1T ml1T
heat, i.e.,
i.e., specific heat of a substance, when it undergoes
< c eu < cAl
eHg adiabatic change, is zero.
(3) Specific heat of a substance also depends on (8) Specific heat of a substance can also be negative.
temperature (particularly at low temperatures) the Negative specific heat means that in order to raise the
variation of specific heat with temperature for water is temperature, a certain quantity of heat is to be
shown in Fig. 15.1 (a) while for metals in Fig. 15.1 withdrawn from the body. Specific heat of saturated
(b). This temperature dependence of specific heat is water vapours is negative.
usually neglected.
(9) When specific heats are measured, the values

1>
1 obtained are also found to depend on the conditions
of the experiment. In general measurements made at
gl.008 . '. .
constant pressure are different from those at constant
~ 1.004
... volume. For solids and liquids this difference is very
j 1.000 small and usually neglected. ,The specific heat of
~ 0.996 L~_ _~_ _ -+- gases are quite different under constant pressure
20 40 60 80 100 100 200 300 400 500 condition (cp ) to constant volume . condition (cv)'
Temp. in °C _ _ Temp. lnK _ _
[For details see § 17.4(C)]
(a) Water (b) Metals
(10) As by definition c = (Q/mI1T), heat required to change
Fig. 15.1
the temperature of m g ofa substance through l1T:
(4) Specific heat is found to be maximum for hydrogen
Q = mc"'T .... (ii)
(3.5 caVg CO) then for water (1 caVg CO = 4200 J/kg
And as I1T = (Q/mc), greater the specific heat of a
K). For all other substances specific heat is lesser than
substance lesser will be the change in temperature for
1 caVg Co and is minimum for radon and actinium
a given mass when same heat is supplied. Now as
(~O.022 caUg CO).

(5) Specific heat also depends on the state of substance,


is
specific heat of water very large (1 callg C», by
absorbing or releasing large amount of heat its
i.e., solid, liquid or gas, e.g., specific heat of solid temperature changes by small amounts. This is why,
copper will be different from that of liquid copper. In it is used in hot water bottles oras coolant in radiators.
case of water: This is also how the sea moderates the climate of
nearby coastal land .
.. Actually it is the average specific heat over the temperature range from T to T + llT and specific heat at a temperature Twill be
p= • lim .lllQ i.e., c= .. Q !.d
Q1' ....omtJ.T' IIIdT
.... Also expressed as kcal/kg Co.
(11) Molar Heat Capacity: If instead of unit mass we Solution: (a) As thermal capacity is defined as the heat
consider one mole of a substance, the heat required to required to change the temperature ofa given sample by lCD,
change the temperature of one mole of a substance
TC - Q - 300J 15-.L
through leo (or K) is called molar heat capacity or - lie - (45 -25j"C Co
mola~ specific heat and is represented by C. If the
molecular weight of a substance is M: (b) As specific heat capacity means thennal capacity per
unit mass and here m = 25 g = 25 x 10-3 kg
C = Me = ~ [as c = ~ and 1.1 = !!!...] ... (iii)
~lIT mllT M c = TC = 15(1/CO) =600 - J -
m (25xlO 3)kg kg Co
Its SI units are (J/mol K) while dimensions
[ML'r'e-I~-I]. (c) As molar heat capacity means thennal capacity per
mol and here ~ =(25/50) =0.5 mol
(12) Thermal Capacity: If instead of unit mass we
consider the whole body, (of mass m), the heat e= TC 15(1/CO) =30 _ 1_
~ 0.5 mol mol Co
required to raise the temperature ofa given body by I
C"is called its thermal capacity·, i.e., [Also C =Me = (50 x 10- 3 kg/mol) x (600 J/kg CO)
Theonal capacity = me = IlC = (Q/llT) .... (iv)
= 30 llmol CO] Answer
Theonal capacity of a body depends on the mass and
Problem 3. At very low temperature, the molar heat
nature of body. It has units (11K) or caliCo and
capacity of rock salt varies with temperature according to
dimensions [ML 2T - 2 - l]. a Debye's relation, C = k (TA:! D)3 with k =2000 (Jlmol K) and
(13) Water-Equivalent: If thennal capacity of a body is
expressed in terms of mass of water it is called eD =300K.
water~equivalent of the body, i.e., water-equivalent (a) How much heat is required to raise the temperature of
of a body is the mass ofwater which when given same 2 mol ofrock salt from 10 to 50 K?
. amount of heat as to the body, changes the (b) What is the mean molar heat capacity in this range?
temperature of water through same range as that of (c) What is the molar heat capacity at 50 K?
the body, i.e., Solution: (a) If dQ heat is given to a sample at
W=(mxe)g .... (v) temperature T and its temperature changes by dT, by definition
The unit of water~equivalent W is g while its of molar heat capacity, i.e., C = (dQ/~ dT)
dimensions (M]. Units and dimensions of some dQ=~CdT, i.e., Q=~ICdT
physical~quantities used in heat are given below in a
tabular fonn
Units
or Q=~f' k[.I..]'
T] e dT=A[T,'
,m3
_ TI4]
D ~D
Sl. Sym~
Physical qu.antity Dimensions CGS
No. bol SI
(practical) or Q = 2x2000[50' -I0'] = !Q(625-1)=231.1 1
4(300)3 27
1. Heat Q [ML2r- 2 ] joule calorie

,. Specific-heat e [L2r- 2a- 1 ] JlkgK caVgCO


(b) As mean molar heat capacity is defined as,
C =L so C= 23111 2.9 _1_
[ML2T2a- l,... -I ~~T' (2 mol)(50 - 1O)K mol K
3. Molar sp. heat C JlrnolK caVmol eo
(c) As according to given problem molar heat capacity at
4. Latent heat L [L2T2] J/kg caVg temperature T is given by

5. Thennal capacity TC [ML2 r- 2a- 1 ] JIK CaVeD


C = k[e:f
6. Water-equivalent W 1M] kg g

Problem 2. A certain substance has a mass of50 g/mol. so for, T=50K,C=2000 [ 50]3 =9.26 11K Answer
When 300J ofheat is added to 25 g ofsample of this material, 300 rna
its temperature rises from 25 to 45OC. Calculate (a) thermal Problem 4. In order to save energy suppose you decide to
capacity (b) specific heat capacity and (c) molar heat capacity boil water for your tea by shaking it in a thermos flask.
ofthe sample. Suppose you use (112) kg of tap water at 200C and waterfalls
• Here the word 'capacitY is misleading as it is used for heat required per degree change in temperature and not the total amount of heat a body can hold.
HEAT AND CALORIMETRY 689

0.4 m in each shake and you make 30 shake each minute. constant temperature. In the reverse process, (freezing and
Neglecting any loss of heat by the flask, calculate how long condensation) heat is released at constant temp. (viz., FP and
must you shake the flask so that water begins to boil? Given CP)
that g = 10 m/s 2 and specific heat a/water is 4200Jlkg. The amount of heat required to change the slale of unit
Solution: Mechanical energy lost by water per shake mass of a substance at a constallt temperature is called the
=mgh = (112) (10) (0.4) =2) heat of tr.ansfo rmation or latent heat (symbol L). So, if Q heat
changes the state of mass m of a substance at constant
As number of shake per sec is (30/60)
temperature, heat oftransfonnation or latent heat
=
ME lost per sec = 2 x (112) 1 Jls
L={Qlm) i.e., Q=mL .... (i)
Now heat required to boil (112) kg water of 20°C
Regarding latent heat it is worth noting that:
Q = meAT = (1/2) (4200) 000 -20) = 168 kJ
(1) It has SI unit (J/kg) while practical unit (caVg) and
As energy is supplied to the water at the rate of I J/s, time
dimensions [L 2T -2]. It depends on nature and state
taken for boiling water of substance.
= 168000) = 168000 = 194=2 da Answer (2) The beat oftransfonnation during melting or freezing
I (J/s) 24 x 6Ox6O· y is called the beat of fusion (LF ) while during boiling
Problem S. A lead bullet just melts when stopped by an or condensation is called heat of vaporisation (Lv)'
obstacle. Assuming that 25 % of heat is absorbed by the For water'
obstacle,find the minimum velocity of the bullet if its initial
temperature is 27°C. (Melting point of lead = 3270(;; Specific Latent Heat In eGS units cal/g In Sl units Jlkg
=
heal of lead 0.03 callg CO; Lalenl heal offusion of lead 6 = FusionLF 80 333x 103
ca/Ig alld J = 4.2J/caf) Vaporisation Ly S4{) 22S6x 103
Solution: Ifmass of the bullet is m g, heat absorbed by it
to raise its temperature from 27°C to 327°C (3) Heat of Evaporation: In case of evaporation, liquid
x
mcl1T = m 0.03 x (327 - 27) = 9 mcal converts into vapours (gaseous phase) at all
And heat required by the bullet to melt temperatures and heat required to change unit mass of
liquid into vapours at a given temperature is called
mL=m x 6 =6mcal
heat of evaporation at that temperature. Due to large
So, total heat required by the bullet heat of evaporation, water on sprinkling, produces
Q, = (9m +6m) =15mcal=(15mx 4.2)J [aslcal=4.2J] cooling in a closed room.
Now when bullet is stopped by the obstacle loss in its (4) Heat of Sublimation: If a solid such as camphor (or
mechanical energy ice in vacuum) directly converts into gaseous phase at·
all temperatures the process is called sublimation and
ME=1(mxlO-3)v2J [asmg= m x lO-3 kg]
2 heat required to change unit mass of solid directly
into vapours al a givell temperature is called heat of
As 25% of this energy is absorbed by the obstacle, the
sublimation at that temperature.
energy absorbed by the bullet
(5) Hoar Frost: If vapours by releasing heat, directly
Q = 75 x 1mv2 x10 - 3 = ~mv2 xl0-3 J comes to solid state without passing through liquid
2 100 2 8
phase, the process is called' Hoarfrost' . Fonnation of
Now the bullet will melt if snow by freezing of clouds (ie., water vapours) in low
Q 2 ~QI temperature zones of atmosphere can be cited as an
example of 'hoar frost ' . (Which is reverse of
i. e., ~mv2 )( 10-3 ~15mx4.2
8 sublimation)
(6) In case of change of state, the distance between
Le., v~.J(4x4.2)xI02 or vmin = 410mls Answer
molecules changes, so work will be done against
§ 15.3 Heat of Transformation intennolecular forces . If the molecules come closer to
When heat is given to a solid or liquid usually temperature eacb other (as in condensation or freezing) energy
rises. However, ifheat is given to a solid at its melting point or will be released and if molecules move apart as (in
to a liquid at its boiling point, the solid will change into liquid boiling or melting) energy will be absorbed,
and liquid into gas without change in temperature. Thus, ice (7) In melting or boiling heat supplied is used to increase
melts (at O°C) and water boils (at 100°C) by absorbing heat at a the Internal potential energy of the substance (and
690

also in doing work against external pressure) while Q1 =S x A x t=Sxnx (2.5)' x (20) = (392.5)Scal
internal kinetic energy (oc T) remains constant. So,
internal energy of steam at lOO oe is more than that of And heat required to melt 109 of ice,
water at IOO°e. This is why steam bums are more Q2 = ML=10x80=800cal
painful than that caused by boiling' water or more heat But according to given problem, Q, = Q2' i, e.,
is required to convert a given mass of water into steam 392.5S =800 or S =2.04caVcm 2min Answer
(at 100°C) than to raise its temperature to lOoDe.
(8) Clausius and Clapeyron has shown that the effect of Note: The constant S is called 'solar constant' and its value in 81 units
pressure on boiling point of liquids and melting pohlts is 1.4 kW/m2.
of solids (TK ) is related to latent heatL (caVg) through
Problem 8. An earthen pitcher loses I g of water per
the relation:
minute due to evaporation. Ifthe water equivalent ofpitcher is
dP = JL .... (ii) 0.5 kg and the pitcher contains 9.5 kg of water, calculate the
dT TK(VF -V[) time required for the water in the pitcher to cool to 28°C from
where VF and VI are final and initial specific volumes its original temperature of 30"C. Neglect radiation effects.
(cc/g). From this expression it is clear that · Latent heat ofvaporisation ofwater in this range oftemperature
(a) ' In case of boiling 15 volume of vapours VF is is 580 cal/g and specific heat of water is 1kcallkg CO.
always greater than volume of liquid VI' (dPldT) Solution: As water equivalent of pitcher is 0.5 kg, i.e.,
is positive, i.e., bpt. of every liquid rises (or pitcher is equivalent to 0.5 kg of water, heat to be extracted
elevates) with increase in pressure. from the system of water and pitcher for decreasing its
(b) In case of melting, for substances like wax and temperature from 30°C to 28°C:
sulphur which expands on melting VF > VI' m.pt. Q 1 = (m + W)cl>T
rises (or elevates) with increase in pressure. = (9.5 + 0.5) kg (I kcaVkg CO) (30 - 28tC =20 kcal
However, for substances like ice and mbber
And heat extracted from the pitcher through evaporation
which contracts on melting VF < VI so, (YF - VI ) in t minutes:
will be negative, i.e., m.pt. is lowered (or
depressed) with increase in pressure. Melting of Q2 = mL =[dm x t]L =~ x (tInin) x580 cal =580x tcal
ice under pressure and its resolidification when dt mm g
pressure is released is called 'regelation'. According to given problem, Q2 = Q" i. e.,
Problem 6. A substance of mass M kg requires a power 580xt = 20x10 3 or t=34.5minute Answer
input of P watt to remain in the molten state at its melting
point. When the power is turned oJ!, the sample completely Problem 9. 0.75 g of petroleum was burnt in a bomb
solidifies in time t sec. What is the latent heat offusion of the calorimeter which contains 2 kg of water and has a water
substance? equivalent 500 g. The rise in temperature was 3 Co. Determine
the calorific value ofpetroleum.
Solution: As by supplying power P at m.pt. the substance
is in steady state, the rate of loss of heat by the molten metal at Solution: Heat absorbed by water:
its m.pt. Q1 = mcI1T = 2000xlx3=6kcal
= rate of heat supply = P watt And heat absorbed by calorimeter
Now when supply is switched off, the substance Q 2 = Wcfl.T =500x 1x 3 =1.5 kcal
completely solidifies in time t, so, the total heat released by the So, total heat absorbed
substance at melting point, Q1 = (P x t) J.
Q=QI +Q, =6+1.5=7.5kcal
But in case of change of state
As this heat is produced by combustion of 0.75 g of
Q, =(MxL)J petroleum,
According to given problem Q1 =Q2' i.e., . Q 7.5 kcal kcal
Calonficvalueoffuel=m= 0.75g =IO Answer
P[=ML or L=(PtIM) Answer T
Problem 7, Rays from the sun are focussed by a lens of
diameter 5 cm on to a block of ice (L = 80 kcal/kg) and 109 of Note: Calorific-value of fuel is defined as heat produced by
ice is melted in 20 minutes. Calculate the heat received from combustion of unit mass of fuel and is usually detennined by
'bomb shell calorimeter'.
the sun on I cm 2 per minute.
Solution: If S calorie of heat is received from the sun per Problem 10. 0.05 kg of ice at -20"C is to be converted
cm 2 per minute, the heat collected by the lens of radius 2.5 cm into steam at 100°C. The ice is first heated with a 420 watt
in 20 minutes, heater for 5 minute. After this, it is heated with an infra-red
HEAT AND CALORIMETRY 691

lamp of A. = 10,000 Ii for 23 minute and 20 second at an § 15.4 Principle of Calorimetry


efficiency 0[50% . Find the rate at which the photons are
When two bodies (one being solid and other liqllid or both
striking the ice when healed with infra-red lamp.
being liquid) at different temperatures are mixed, heat will be
(Planck's COI/SII. = 6.62 x 1O-34 J _s ; c = 3 x 10 8 mls; transferred from body at higher temperatUre to a body at lower
specific heat of ice at - 20OC =SOD cal/kg; sp heat of water =I temperature till both acquire same temperature. The body at
kcal/kg; latent heat affusion ofice = 80kcal/kg and lalent heat higber temperature releases heat while body at lower .
afvaporisation ofwater= S42kcal/kg) (MNR 1994J temperature absorbs it, so that:
Solution: Heat absorbed by ice to change its temperature Heat lost = Heat gained,
from -20oe to O°C i. e., principle of calorimetry represents the law of
Q, = mcl>T = O.OS x O.S x [0 - (-20)] =O.S kcal conservation ofheat energy.
Heat absorbed by ice at O°C to convert into water at O°C While, using this principle always remember that:
Q2 = mL=0.05x80 = 4kcal (I) Temperature of mixture (1') is always ~ lower
Heat absorbed by water to change its temperature from temperature (T1) and :S higher temperature (TH)' i. e.,
O°C to 100°C TLSTsTH
Q3 = IIlclJ.T =O.OS x I x [100 - 0] = 5 kcal i. e., the temperature of mixture can never be lesser
Heat absorbed by water at 1000C to convert it into steam at than lower temperatures (as a body cannot be cooled
IOQ"C below the temperature of cooling body) and greater
Q4 = IIlL = 0.05 x 542 =27.1 kcal than higher temperature (as a body calUlot be heated
above the temperature of heating body). Furthermore
So, total heat absorbed by ice al - 2O"C in converting into
steam at I OO"C usually rise in temperature of one body is not equal to
the fall in temperature of the other body though heat
Q =0.5 + 4 +S +27. 1 = 36.6 kcal gained by one body is equal to the Heat lost by the
or Q = 36.6x10 3 x4.2= IS3 .72kJ [as I cal=4.2J] other.
Now as heat supplied by 420 W heater in 5 minute: (2) When temperature of a body changes, the body
QH = 420(1/,)(S x 60,)= 126kJ releases heat if its temperature falls and absorbs heat
when its temperature rises. The heat released or
So, heat supplied by Infra-red lamp:
absorbed by a body of mass m is given by:
Q-QH = (IS3.72 - 126)kJ=27.72kJ .... (i)
Q=mcllT
Now if n photons (each of energy hv) strike per sec, the
where c is specific heat of the body and!:J.T change'in
energy of incident photons in time t:
its temperature in Co or K.
E = n x(hv)xt = lIx~Cxt [asv=iJ (3) When state of a body changes, change of state takes
place at constant temperature [m.pt. or b.pt.] and heat
Here t = 23 min 20 sec = 23 x 60 + 20 = 1400s released or absorbed is given by,
and ).,=10000xlO- 10m=1O- 6 m Q=mL
So, E = (/I x 6.62 x 10- 34 x 3 x 10 8 x 1400) x (1110-6 ) where L is latent heat. Heat is absorbed if solid
converts into liquid (at m.pt.) or liquid converts into
= 278.04xnxlO- 18 J vapours (at b.pt.) and is released if liquid converts into
Now as efficiency of the lamp is 50010, i.e., half of this solid or vapours converts into liquid.
energy is used to heat the water, i. e., (4) No cbange ofstate due to same temperature: Iftwo
Q,.
= (EI2) =(II2) x 278.04 x nx 10- 18 1 ....(ii) bodies are al same temperature, no transfer of heat
between them can take place. If water in a beaker is
So, from Eqns. (i) and (ii) placed in a bath of boiling water, the temperature of
(1I2)x278,04xnxlO- 18 =27.72x10 3 J water in the beaker will increase till it reaches lOODC.
3 However, the water in the beaker will never boil as
i. e., n=2 x27.72><10 heat required for change of state will not be
278.04 x 10- 18 transferred from water bath to the water in the beaker
as both are at same temperature (100°C). Similarly if
Answer we put the beaker containing water in melting ice, the
water in the beaker will cool to O°C but will never (d) In the region CD temperature is constant, so it
freeze. represents change of state, i. e., boiling with
(5) No change in temperature due to insufficient heat: boiling point T2 . At C all substance is in liquid
If we put some ice in hot water, the temperature of state while atD in vapour state and between C and
mixture will be higher than m.pt. of ice only and only D partly liquid and partly gas. The length 0/ line
CD is proportional to latent heat 0/ vaporisation,
ifheat'released by hot water in attaining the m.pt. of
i.e.,
ice is more than heat required to bring the ice to O°C
and then melt the whole ice. Ifthis heat 'is insufficient Lv oc Length of line CD
to melt the whole ice, the temperature of system will [In this region specific heat oc (lltan 0) = 00]
be m.pt. of ice with some ice remaining unmelted. (e) The line DE represents gaseous state of substance
[See solved Problems 17 and 18J with its temperature increasing linearly with time.
(6) Heating curve: If to a The reciprocal of slope of line will be
proportional to specific heat or thennal capacity
solid, heat is supplied ci. T, b,~~.________F,Boi~
.--ig
given mass (m) of a
! "c",. of substance in vapour state.
at constant rate P and a E T m.p\. B8t i (7) In calorimetry we use different types of calorimeters
~ 1 A-Melli"",
, ,, ,,i
graph is plotted to determine specific heat, e.g.,
, ,
between temperature ~ , ,
, (a) Bunsen ice calorimeter is used to detennine
o " latent heat of ice.
and time, the graph is
as shown in Fig. 15.2
lime- "
Fig. 15.2 (b) Regnault's calorimeter is used to detennine
and is called heating specific heat of solids or gases (at constant
curve. From this curve it is clear that: pressure).
(a) In t~eregion OA temperature of solid is changing (c) Nernst's vacuum calorimeter is used to
with time so, determine specific heat of solids at very low
Q=mcs l1T temperature.
or PM=mcs l1T [asQ=PM] (d) Joly's differential calorimeter is used to
But as (I1Tll1t) is the slope of temp/time curve determine specific heat of gases at constant
volume,
Cs oc (l/slope of line OA)
(e) Callendar and Barne's calorimeter is used to
i.e., specific heat (or thermal capacity) is
find specific heat of liquids and gases (at constt.
inversely proportional to the slope of temp/time
pressure).
curve.
(f) Bomb shell calorimeter is used to determine
(b) -In the region AB temperature is constant, so it
calorific value of fuels.
represents change of state, i. e" melting of solid
with melting point 1'1 . At A melting starts and at B Problem 11. The temperatures o/equal masses o/three
all solid is converted into liquid. So between A different liquids A, Band C are 12°C, 19°C and 28"C
and B substance is partly solid and partly liquid. respectively. The temperature when A and B are mixed is 16°C
If LF is the latent heat offusion and when Band C are mixed is 23°C. What would be the
temperature when A and C are mixed?
P(t, - I.) .
Q=mLF orLF = [.sQ=P(I, - I.)] Solution: In accordance with principle of calori~etry:
m ',:.
when A and B are mixed
or LF oc Length oflin~. 4.~ "
mcA (16-12)=mcB (19-16), i.e., cA =(3/4)cs
i.e., Latent heat offusion is proportional to the
length o/lineo/zero slope. [In. this region specific and when Band C are mixed
heat oc (l/tan 0) = 00] mc,(23-19)=mcc (28-23), i.e., Cc =(4/5)c,
(c) In the region BC temperature of liquid increases Now when A and C are mixed if T is the conunon
so specific heat (or thermal capacity) of liquid temperature of mixture:
will be inversely proportional to the slope of line mCA (T -12) = mcc (28 - T)
BC, i.e., Substituting cA and Cc from above,
CL oc (l/slope of line BC) (3/4)(T -12) = (4/5) (28 - T)
Which on solving gives, T = 20.25°C Answer
Problem 12. The reading of a thermometer of mass According to given problem Q2 =Q" i.e.,
0.0550 kg and specific heat 0.837 kJ/kg K is IS.OOC. It is then
600mS =6oo x 10 3 , i.e., mS = l x I0 3 g = lkg Answer
completely immersed in 0.300 kg o/water and it comes to the
same final temperature as the water. If the thermometer reads Problem 15. How many gram ofice at-14OC is needed to
44A°e. What was the initiailemperalure o/water? (sp. heat of cool2oog of water[rom 250C to IOOC? (cicc := O.5 cal/g CO alld
water = 4.2kJlkg K) LF [or ice =80 callg)
Solution: When thermometer is immersed in hot water, it Solution: In cooling 200 g of water from 25°C to 10DC
takes some heat from the water and the whole system acquires heat to be extracted from water,
thermal equilibrium. Hence, temperature rcad by the Q\ = (mc6T)w =200. I. (25- 10)=3000 cal
thermometer is somewhat lower than initial temperature of
And heat absorbed by m g ice at -14OC to convert into
water. 1fT is initial temperature of waler, heal released by hot
water of IOOC,
water:
Q 2 = (mcllT)jce + mL + (mcllT)w ,
Q\ = 0.300. 4.2. IO J (T - 44.4) = I260(T - 44.4) J
i.e., Q, =m[0.5{0- (-14)) J +80 +1 (10 - 0)J = 97mcal
While heat absorbed by thermometer: According to given problem,Q2 ::. Qt ' i.e.,
Q, = 0.055.0.837. IO J (44.4 - 15) = 1353.43 J
97m := 3000, i. e. , m=3Ig. Answer
So by principle of calorimetry, i.e. , Q j =Q 2
Problem 16. The temperature of100 g o[wate~ is to be
T -44.4 ={1353.431l 260) or T=45.5°C Answer raisedfrom 240(; to 900(' by adding steam to it. Calculate the
Problem 13. An electrically heating coil was placed ill a muss ofthe steam required for this purpose,
calorimeter containing 360 g of water at IO"C. Th e coil Solution: If m is the mass of steam, by principle of
consumes energy at the rate 0[90 watt. The water equivalent calorimetry
o[the calorimeter and the coil is 40 g. Calculate, what will be 100.1 x (90 - 24) = m540 + m (100 - 90)
the temperature of the water after 10 minutes? (J = 4.2
i. e., 550m =loo x 66, i. e., m=12g Answer
joule/cal)
Problem 17. Ice at OOC is added to 200 g o[water initially
Solution: Energy supplied by the heater to the system in
at70°C in a vacuumflask. When 50 g ofice has been added and
10 minute: has all melted. the temperature o[ the flask and cOlltents is
Q\ =P . ,=(90J/,).(l 0 . 60 ,) =54kJ 400('. When a further 80 g of ice has been added and has ail
i.e, Q\ =(54/4.2)kcal=12857cal [a, 4.2 J = 1 calJ melted, the temperature of the whole becomes 10OC.
Now if T is the final temperature of the system, energy Neglecting heat lost to the surroundings, calculate the [atellt
absorbed by it to change its temperature from lOOC to roc heat o[[usion ofice. (sp ecific heat of water is I callg CO)
Q, = (m+W)c6T=(360+40) x l .(T- IO)cal Solution: If L is the latent heat of ice and W is the water
equivalent of flask, according to principle of calorimetry
According to given problem, Q, = Q2' i. e.,
ML + McllT = (m + W)cllTwater
400(T - lo) = 12857 or T =42.l4°C Answer
i. e., 50[L + I. (40 -O)J= (200 + W).l. (70 -40)
Problem 14. In an industrial process 10 kg of water per
hour is to be heated from 200C to 80°C. To do this, steam at i. e., 5L =3W+400 .... (i)
1500C is passed fro m a boiler into a copper coil immersed in Now the system contains (200 + SO) g of water at 40"C so
waler. The Sleam condenses ill the coil and is returned 10 the when further 80 g of ice is added:
boiler as water at90OC. How mallY kg ofsteam is required per 80[L+1 x (lO-O)J = (250 + W) x I x (40 -1 0)
hour? (specific heat of steam· = 1 callg CO alld latent heat of
i. e., 8L=3W +670 .... (ii)
steam = 540 callg)
Solving Eqns. (i) and (ii),
Solution: Heat required by lo kg water to change its
temperature from 200c to 8<Y'C in one hour: L = 90(caUg) and W =(50/3) g AII~wer
Problem 18. 5 g of water at 300(' and 5 g of ice at ~20"C
Q\ =[mc6 TJ w ",,, =(l0.IO J )(l)(80-20)=600kcal
are mixed together in a calorimeter. Find the final
If III g is the mass of steam condensed per hour, the heat temperature of the mixture. Assume water equivaient of
released by steam in converting into water of 90"C calorimeter, to be negligible, sp. heat of ice and water are 0.5
Q2 =mscs (150-100) +ms Lv + mscw(100 -90) and 1cal/g CO alld lalent heal of ice is 80 callg.
i.e., Q2 = m8 [1 x 50+540+lxIOJ=600ms cal Solution: Here ice will absorb heat while hot water will
release it. So if T is the final temperature of the mixture, heat
[as Cs =Cw = I caVg 0c]
given by water
~~~~~~~~~~
• Actually specific heat of steam is 0.47 caVg CO.
Q 1 =mct.T=5xl x (30-T) .... (i) Problem 20. A mixture of250 g of water and200 g ofice
And heat absorbed by ice atO°C is kept in a calorimeter which has water equivalent of50
Q, =5x(1/2)[0-(-20)]+5x80+5xl(T-0) .... (ii) g. !f200 g of steam at 100°C is passed through the mixture,
calculate the final temperature and weight of content of the
So, by principle of calorimetry Q1 =Q2' i.e., calorimeter. Given that specific heat of ice and water are 0.5
150 - 5T=450 +5T, i.e., T=-30°C and I cal/g CO while latent heat 80 and 540 callg respectively.
Which is impossible as a body c~ot be cooled to a Solution: If 200 g of steam condenses the heat released
temperature below the temperature of cooling body. The by it will be
physical reason for this discrepancy is that the heat remaining Q1 =200x540=108kcal
after changing the temperature of ice from - 20°C to OC>C is
insufficient to melt the whole ice and so the system acquires a Now as heat required by ice and water to change into water
temperature of O°C with some ice left unmelted and we are at 100°C
taking it for granted that heat is transferred from water at oGe to Q, = 200x 80 + (200 + 250 + 50) x 1 x (100 -0) = 66 kc.1
ice at aGe so that temperature of system drops below O°C. As Q2 < Q1' whole of steam will not condense. If m is the
However, as heat cannot flow from one body (water) to the mass of steam condensed
other (ice) at same temperature (O°C), the temperature of mx540 = 66kcal or m= l22.2g
system will not fall below O°C. So,finally the system will contain:
Note: Here the water in cooling to m.pt. of ice, i.e. , O"C can provide 250 + 200 + 122.2 = 572.2 g of water
5xix30=150 cal of heat. Out of this ISO cal of heat and 200-122.2=77.8 g of steam, all at 100°C Answer
5 xO.5 x20 == SO cal will be used to heat ice from-20°C to O°C so
the remaining heat == 150 - 50 = lOOcal will melt only (100/80)= Problem 21. A solid material is supplied with heat at a
1.25 g ofice out ofS gpresent. So thefinal state afsystem will be constant rate. The temperature of the material is changing
5 -1.25 =3,75 gafice and 5 + 1.25 == 6.2S g afwaterat O°C. with the heat input as shown in Fig. 15.3. Study the graph
carefully and answer the following questions:
Problem 19. A lump of0.10 kg of ice at -IOOC is put in
(a) What do the horizontal regions AB and CD represents ?
0.15kgofwaterat20OC. How much water andice will befound
. in the mixture when it has reached in thermal equilibrium? (b) !fCD = 2AB, what do you infer?
(specific heat of ice = 0.50 kcal/kg while its latent heat 80 (c) What does the slope of DE represent?
kcallkg) (d) The slope of OA > the slope of Be. What does this
Solution: The heat which 0.15 kg of water can release indicate?
when its temperature is changed from 20°C to O°C: Solution: (a) The given curve is the heating-curve of a
Q1 =mwcwATw =0.15x(lxlO')x(20-0) solid. In the horizontal portions
AS and CD as temperature of E
=3000c.1 .... (i) substance remains constant when t
----------,,-,,-------'ID
Now as heat absorbed by 0.10 kg of ice at -10°C to it is heated, so these represents ~ C
increase its temperature to O°C, change of state. The portion AS ~ B
Q2 = micecicellTice =0.10 x (0.5 x 10 3 ) x [0 - (-10)] represents conversion from solid
to liquid while CD from liquid to o'---c------
=500 cal vapours. Heat Input - -
So, remaining heat = Q 1 - Q 2 =3000 -500 =2500 cal Fig. 15.3
(b) In case of change of
Now as latent heat of ice is 80 kcal/kg, the remaining heat stateQ = mL
will melt only i.e., length of line cc L [as Q ;;: length of line]
(2500/80 xlO') = 0.03125 kg of ice
So, L v = CD =2 [as CD =2AS , given]
So, the ice remaining = 100 - 31.25 = 68.75 g LF AS
and total water = 150 + 31.25 = 181.25 g i. e., Latent heat of vaporisation of substance is double that oj
The temperature of the system will be ODC. Answer fusion.
Note~ This is a problem of insufficient heat as remaining 2500 cal can (c) As along DE temperature of substance which is in
melt only 31.25 g of ice out of 100 g present. Ifhere we directly vapour state is increasing with heating, the slope ofline will be
apply the principle of calorimetry we shall find temperature of (dTldQ) .nd as
mixture lesser than GOC which is absurd as explained in Problem
18. dQ = mcdT; dT= ___ = 1
dQ mc Th. Cap.
i. e., slope ofline DE represents reciprocal ofthermal capacity Problem 23. The specific heat of a substance varies with
of vapour state of the given substance. temperature T as C = ATl + BT cal/gOC, with temperature in
(d) As slope of OA> slope of Be and OA represents degree kelvin. Calculate the amount ofheat required to heat50 gof
heating of solid while Be of liquid, substancefrom27°C to 57°e. Alsofind the time taken if the heat is
supplied by a heater of resistance 200 ohm operating on 200 V.
_ 1_ >_1_ [ass!ope= 1 =_1_] [Given A =2.5 x 10- 3 callg eC)3,' B = 12 x 10- 2 callg(0C) 2.]
(mc)s (me)L Th.eap. me
Solution: Amount of heat required to change temperature
or Cs < cL > i.e., the specific heat of solid is lesser than that of of m g of substance by dT,
liquid.
c, dQ=mcdT = m(AT 2 + BT)dT
Problem 22. Two bodies t100 : : : : : in: : :
() 80 --:1-;--:--:---:8-;;-- ' --:-I: C1
ofequal mass m are heated at a " 60 __ foe
On integrating between the given limits, the amount of
uniform rate under identical f-; 40 .J +_-l~1: : ;6 1: ++
--1---l-- •
heat re'quired,
conditions. Their change in ~
temperatures are shown F
20 - .. : --t--+-+++++ Q = mJ330(AT2 +BT)dT = m[AT3 +BT1 ] 330
0123456789
graphically in Fig. 15.4(a) 300 3 2
Time - 300
What are their melting points? Fig. 15.4
3
=50[2.5 1O- (330 3 - 300 3 )+6xlO-'(330' - 300')]
X
(b) What is the ratio of their
latent heats? (e) What is the ratio a/their specific heats ? 3
Solution: (a) Horizontal portion of temperature versus
time curve represents the fact that temperature is constant
= 429075 cal
,
during heating, which is characteristic of change of state. So, Let t is the required time; then ~ = Q
the horizontal portion represents change ofstate and constant RJ
temperature melting point. Therefore, melting point of 1 = QRJ = 429075 x 200 x 4.2
or
substance I and 11 are respectively 400C and 60OC. V' (220)2
(b) If heat is supplied at constant rate, P, Q = P At and as
during change of state, Q = mL, so, mL = P f1t = 7446.7 s =2 hr. 4 min. AI/SIVer
Problem 24. An ice cube ofmass 0.1 kg at O°C is placed in
i.e., L= [: ] .6.t::'::~(lengthofline) an isolated cOl/tainer which is a1227OC. The specific heat S of the
contailler varies with temperature T according to the empirical
Here P = constt. and
m l = ml . relalion, S = A + BT, where A = 100 callkg-K and -B = 2 x 10- 2
LI A,B, 7- 4 3 callkg-K 2 . If the final temperature of the container is 27OC,
So, - = - - = --=- delermine the mass of the container. (Latent heat offusion of
Ll A1Bl 6 - 2 4
water = 8xl0 4 callkg,· sp. heatofwater = 103 callkg-K).
i. e., the ratio of latent heat affusion ofthe two substances are
in the ratio 3: 4. Solution: Heat lost by container
300
(c) In the portionOA andBC the substance is in solid and =- ) soomc(A+BT)dT
liquid state respectively and its temperature is changing:
6.Q = mc6.T and 6.Q = P 6.t = -me [ AT + BT2 ] 300 = 21600m
c
So, ~~ = :c or Slope = :c [as~~ = SIOPC] Heat gained by ice = mL + mC 6.T
2 500

As here ml = ml and P = constant, = O.l x 8x104 + 0.lxI0 3 >:027


(cn/cI ) = (Slope) l/(Slope) II = 10700 cal
So,for solid state of substances: From principle of calorimetry
(cn/c,) s = (40/4)/(60/2) = (I/3) Heat lost by container = Heat gained by ice
While for liquid state of substances: 21600mc = 10700 cal
(cn/c,) L = (20/2)/(40/2) = (1/2) or me = 0.495 kg Answer
i.e., the ratio of specific heat of two substances is in the ratio
Note: Speeific heat of container is temperature dependent, so we have
(l/3) if they are in solid state and (lI2) if they are in liquid
to calculate heat lost for a small temperature changc dT and then
state. integrate it from initial temperature to final temperature.
EXERCISE

(AJ Only One Choice is Correct 13. Which of the following has the highest specific heat?
1. The natural direction of heat flow between two reselVoirs (a) Water (b) Mercury
depends on their: (c) Kerosene (d) Copper
(a) Temperature (b) Volume 14. Which of the following has highest specific.heat?
(e) Pressure (d) Nature (a) Hydrogen (b) Water
2. Conversion of heat into electrical energy can be achieved (c) Copper (d) Mercury
by: 15. A liquid is being converted into vapours at its b.pt.; the
(a) Transistor (b) Voltameter specific heat of the liquid at this temperarure will be :
(e) Thennocouple (d) Photoelectric cell (a) Zero (b) Infinite
3. Which device can convert all electrical energy into heat? (c) Positive (d) Negative
(a) Radiator (b) Converter 16. Specific heat of a gas undergoing adiabatic change is :
(e) Generator (d) Resistor (a) Zero (b) Infinite
4. How much heat energy in caVsec will be obtained from 1.5 (c) Positive (d) Negative
kW heater? 17. The specific heat of metals at low temperature is :
(a) 1.5 kcal (b) 357 cal (a) Proportional to T (b) Proportional to T2
(e) 735 cal (d) 573 cal (c) Proportional to T3 (d) Independent ofT
5. Ifa 5 kg body falls to the ground from a height of30 m and
18. During melting process, heat given to a body is utilised in :
if all its mechanical energy is converted into heat, the heat
produced will be : (a) Increasing temperature
(a) 350 cal (b) 150 cal (b) Increasing density
(e) 60cai Cd) 6cal (c) Increasing the average distance between molecules
6. 420 joule of energy supplied to 10 g of water will raise its (d) Decreasing the mass of the body
. temperarure by nearly: 19. When vapour condenses into liquid: ICPMTI9901
(a) 1 C" (b) 4.7 C" (c) 10 Co (d) 42 Co (a) It absorbs heat
7. A 2 g bullet moving with a velocity of200 mls is brought to (b) It liberates heat
a sudden stoppage by an obstacle. The total heat produced (c) Its temperature increases
goes to the bullet. If the specific heat of the bullet is 0.03 (d) Its temperarure decreases
caVg Co, the rise in its temperarure will be:
20. A test tube containing water is placed in boiling water so
(a) 158.0 Co (b) 15.80 Co that its mouth remains outside the boiling water. Check the
(c) 1.58 C" (d) 0.1580 C" wrong statement:
8. The SI unit of heat is: (a) Water in the test tube will soon begin to boil
(a) calorie (b) Btu (b) Temperature of water inside the rube will rise
(c) kilocalorie (d) joule (c) Convection currents will be set-up in the water inside
9. The unit of specific heat is: the tube
(a) cal (b) caVg Cd) Volume of water inside the rube will increase
(c) caVgCO (d) caVC" 21. 1 g of ice at O°C is converted into steam at 100°C. The
10. Th~nit of latent heat is: amount of heat required to do so is :
(a) joule (b) joulelkg (a) 80 cal (b) 536 cal
(c) joule/k.g K (d) jouieIK (c) 616 cal (d) 0.716 kcal
II. The calorific value of fuels is detennined by: 22. Spheres of equal masses of iron, copper and silver are
heated to the same temperature and are placed on paraffin.
(a) Bunsen calorimeter (b) Nemst's calorimeter
Which sphere will melt maximum paratpn ?
(c) Joly's calorimeter (d) Bomb calorimeter (a) Iron
12. Specific heat ofa substance depends on its:
(b) Copper
(a) Mass (b) Volume
(c) Silver
(c) Temperarure Cd) Nature
(d) All will melt equal paraffin
23. The ratio of densities of two bodies is 5: 6 and their specific 32. IflO gofice at O°C is mixed with 10 g of water at 10°C, the
heat in the ratio 3: 5. The ratio of their thermal capacities final temperature t is given by:
per unit volume is : (a) IOx80 > 10(1O- 1)+ 10(1 - 0)
(a) 2S: 18 (b) I: 2 (b) I ' S"C
(e) 2: I (d) I : I (e) "O'C
24. Liquids A and B are at 30"C and 20oe. When mixed in equal (d) lOx 80+ 1O(t -0) - 10(10 - 1)
masses, the temperature ofthe mixture is found to be 26°C.
33. I g ofice at ooe is added to 5 g of water at lODe. If the latent
Their specific heats are in the ratio of:
heat is 80 callg, the final temperature of the mixture is:
(a) 3: 2 (b) I : I
(a) S'C (b) O'C
(e) 2: 3 (d) 4 : 3
(d) None of these
25. A liquid of mass m and specific heat c is heated to a
34. 1 g of ice is mixed with 1 g of steam. After thennal
temperature T. Another liquid of mass (ml2) and specific
equilibrium is achieved, the temperature of the mixture is:
heat 2eis heated to a Icmperaturc2T. If these two liquids are
mixed, the resulting temperature of the mixture is : (a) 100'C (b) SS'C (e) 7S'C (d) O'C
(a) (213) T (b) (8/S) T 35. 200 g of a solid ball at 200C is dropped in an equal amount
of water al 8<rC. The resulting temperature is 6O"C. This
(e) (3/S) T (d) (312) T
means that specific heat of solid is: {lAS 1996]
26. 20 g of boiling water is poured into an ice-cold brass vessel
(a) One fourth of water (b) One halfofwater
(specific heat 0.1 caUg Cj of mass 100 g. The resuiling
temperature is : (c) Twice of water (d) Four times of water
(a) 66.WC (b) 6.66'C 36. Equal masses of three liquids A,B andChave temperatures
lODe, 2soe and 40"C respectively. If A andB are mixed, the
(e) 0.66'C (d) SO'C
mixture has a lemperature of I SOC. IfBand C are mixed, the
27. Which one of the fo llowing would raise the temperature o f mixture has a temperature ofJO"C. If A and C are mixed, the
20 g of water at 300C most, when added to water? mixture will haye a temperature of:
(Specific heat of copper = 0.1 callg CO) (a) 29'C (b) 2S'C
(a) 20 g of water at 40°C (e) 20'C (d) 16'C
(b) 80 g of water at 27°C 37. The factor not needed to calculate the amount of heat lost or
(c) IOOOgofcopperat25°C gained when there is no state change is:
(d) 4g ofwater at 80"C (a) Weight (b) Specific heat
28. A 10 kg iron bar (specific heat 0.11 callg CO) at 80°C is (c) Relative density (d) Temperature change
placed on a block of ice. How much ice melts ?
38. Two liquids of equal volume are thoroughly mixed. If their
(a) l.l kg (b) 10 kg specific heats areC] ,C2 , temperatures 9 1 ,9 2 and densities
(e) 16 kg (d) 60 kg d] ,d z respectively, the final temperature of the mixture is:
29. When 0.93 watt-hour of energy is supplied to a block of ice C19, +Cz9 z (b) d l C I 9 1 +d l Cl 9 2
(a)
weighing 10 9 it is found that :
dICI+~~ ~~ +~~
(a) Half of the block melts
(e) d ICI 9, + d 2 C 2 9 2 _d.cl9:J1~+--,d",2c:9:"-2
(d) ..,
(b) The entire block just melts
d 19, +d 2 9 2 C19] +C2 9 2
(c) The entire block melts and the water attains a
temperature 4 QC 39. Two moles of helium, four moles of hydrogen and one mole
(d) The block remains unchanged of water vapour form an ideal gas mixture. The molar
specific heat at constant pressure of the mixture is
30. 10 g of ice cubes at O"C are released in a tumbler (water
(neglecting vibrational degrees of freedom):
equivalent 55 g) at 40°C. Assuming that negligible heat is
taken from the surroundings the temperature of water in the (a) 2R (b) 16 R (e) lOR (d) 23 R
tumbler becomcs nearly (L'" 80 callg) : 3 7 3 7
(a) 31'C (b) 22'C 40. Carbon dioxide is a linear, synunetric, triatomic molecule.
Neglecting vibrational degrees of freedom, the ratio of
(e) 19"C (d) O'C C
31. One kg ice al O"C is mixed with l kg of water at 1O"C. The specific heats, 'Y ", 1 is:
resulting temperature will be : C"
(a) Between O°C and 10°C (b) Greater than 10DC (a) -
7
(b) ~ (e) 2 (d) ~
(c) Lesser than O°C (d) Equal to O°C S 3 7 3
41. Calculate the amount of heat (in calories) required to 50. A source of heat supplies heat at a constant rate to a
convertS goficeatO°Cto steam at IOQoe: (DPMT 2005) substance. Fig. 15.5 shows the variation of temperature of
(a) 3100 (b) 3200 the substance with the heat supplied. The slope of segment
(c) 3600 (d) 4200 DE represents :
42. 109 of ice at DOC is mixed with 100 g of water at sooe. t
E

What is the resultant temperature of mixture?


• ;C'----/o

(a) 31.2'C (b) 32.S'C


[AFMC200S]
iE
A
B

(c) 36.7'C (d) 3S.2'C ~o'----------------


Heatinpul_
43. If the height ofNiagra fall is 50 m, the difference between
Fig. 15.5
the temperature afwater at the top and the bottom of the fall
is : (a) The latent heat of vaporisation
(a) 117 C' (b) 11.7C' (b) The specific heat of vapour
(c) 1.17 C' (d) 0.1l7 C' (c) The thennal capacity ofthe vapour
44. A piece of ice of mass 50 kg is pushed with a velocity of 5 (d) The reciprocal of the thermal capacity of the vapour
mls along a horizontal surface. As a result of friction
51. A block of ice at -10DC is slowly heated and converted to
between the piece and the surface, the piece stops after
steam at 100"C. Which of the following curves represents
travelling 25 m; the ice melted in this process is:
the phenomenon qualitatively? (lIT 2000)
(a) 50 kg (b) 50 g
(c) 7.S6 g (d) I.S6 g
45 Steam burns are more painful than those caused by boiling
water at the same temperature because:
(a) Steam contains more heat than the same amount of
water at the same temperature. When steam condenses
(a)
}bc=
Heatsupp._
(b)
}~ Heatsupp._

}~ }~
it gives out this extra laterit heat
(b) Hot air is also mixed up with steam (c) (d)
(c) Steam produces some chemical effect on the skin to
cause pain Healsupp._ Healsupp._
(d) The question is irrelevant as steam burns are equally 52. A substance of mass m kg requires a power input of P watt
painful as those caused by boiling water to remain in the molten state at its melting point. When the
46. The sprinkling of water reduces the temperature of a closed power is turned off, the sample completely solidifies in time
room because: t sec. The latent heat of fusion of the substance is :
(a) The temperature of water is less than that of the room (a) Pm (b) PI
(b) The specific heat of water is high m
(c) Water has large latent heat of vaporisation I
(c) m (d) Pm
(d) Water is a bad conductor of heat PI
47. What extinguishes a fire most quickly? 53. Equal volumes of water and alcohol when put in similar
calorimeters take 100 sec and 74 sec respectively to cool
(a) Cold water (b) Boiling water
from 50"C to 40DC. The thennal capacity of each
(c) Hot water (d) Ice calorimeter is numerically equal to the volume of either
4S. 109 of ice at - 20DC is dropped into a calorimeter containing liquid. The specific gravity of alcohol is O.S. If the specific
109 of water at 10DC; the specific heat of water is twice that heat capacity of water is 1 cal/g, the specific heat capacity
of ice. When equilibrium is reached, the calorimeter will of alcohol will be :
contain : (a) 0.6 cal/gCD (b) O.S caVgC'
(a) 20 g of water (b) 20gofice (c) 1.6 cal/gC D (d) 1.8 cal/gCD
(c) lOgiceand lOgwater (d) 5giceand15gwater 54. 10 kg of ice at _5 DC is kept in large, thermally insulated
49. Steam is passed into 54 g of water at 30"C till the container. 1 kg of steam at 1l0DC is quickly let into the
temperature ofthe mixture becomes 90"C. If the latent heat container which is instantly closed. The final temperature
of steam is 536 cal/g, the mass of the mixture will be : of the contents of the container when thermal equilibrium is
(a) SO g (b) 60 g (c) 50 g (d) 24 g reached is :
[Using: specific heat of ice = 2100 J/kg K, specific heat of 59. In the definition of calorie, one calorie is the heat required
water = 4200 J/kg K, latent heat of ice = 136x 105 Jlkg, to raise the temperature of 1 g of water through 1°C in a
latent heat of stea m = 225x 106 JIIeg, specific heat of steam certain interval of temperature and at certain pressure. The
= 1930 llkg KJ temperature inlerval and the pressure are: lilT 2005)
(a) 50"C (b) 10000C (a) I 3.5°C to 14.5°C and 76 nun of Hg
(e) O"C (d) 27'C (b) 14.SoC to 15.5°C and 760 nun of Hg
55. The figu re on right shows a 6("C) (c) 6. 5°C t07.soCand76 mmof Hg
cooling curve for a substance ~ (d) 98.5°C to 99.5"C and 760 nun of Hg
which, starting as a liquid. 60 __ I :' 60. A kcttle with 2 litre water at 27°C is heated by operating
eventually solidifies. The specific \
coil heater of power I kW. The heat is lost to the
heat capacity of the liquid is
atmosphere at constant rate 160 J/see, when its lid is open.
20x 10 3 J/kg K. The rate of 10 20 30 t (min)
In how much time will water be heated to 77°C (specific
temperature drop just before Fig. 15.6
heat of water = 4.2 kl/kg)? )IlT 2005]
so lidification at 60"C is 5 K/min. The latcnt fusion o f the
(a) 6 min 2 sec (b) 14 min
substance is :
(c) 8 min 20 sec (d) 7 min
(a) 1.0 x 105 JJkg (b) 1.5x 105 J/kg
6 1. A 2 kg copper block is heated to 500°C and then it is placed
(c) 2x 10 5 J/kg (d) 3.6 x lOs Ilkg
on a large block of ice at O°C, Jfthe specific heat capacity of
56. Some mass of ice is kcpt in an aluminium container at copper is 400 J/kgCOand latent heat of fusion of water is
-40°c' Heat is added to the system at a constant rate . The
3,5 x lOS J/kg, the amount of ice that can melt is :

9('C)tf-
graph showing the variation in temperature ofthe system as
a function of time would be : (Kerala PET 2006]
7 8
.('C) ~ (a) - kg (b) - kg
(a) 0 I(S) (b) 0 I(S) 8 7
7
-40 -40 (d) - kg
5
O(OC)~ .(OC)~ 7
(e) 0 I(S) (d) 0 I(s) (e) l kg
-40 -40
62. mg of a gas of molecular weight M is flowing in an isolated
57. A metal sphere of radius r and specific heat S is rotated tube with velocity V. If the gas flow is suddenly stopped, the
about an axis passing through its centre at a speed of n rise in its temperature is :
rotations per second. It is suddenly stopped and 50% of its (y - ratio of specific healS, R - universal gas constant, J -
energy is used in increasing its temperature. Then thc rise in mechanical equivalent of heat) IEAM CET 2006]
temperature o f the sphere is :
(a) my'(r- I ) (b) my'(r - I )
2tt 2 n 2 r2
(a)
5S
1MRJ
(d) MY
2RJ
,r
5(1trll)2 2RJ
(d) 14S
63. The following figure represents the temperature versus
58. Liquid oxygen at 50 K is heated to 300 K at constant time plot for a g iven amount of a substance when heat is
pressure of I atm. The rate of heating is constant. which of supplied at a fixed rate and at a constant pressure. Which
the fo llowing graphs represcnts the variation o f part o f lhis plot represent a phase change? IBCECE 2006]
temperaturc w ith time? lilT 2004]

T,mp'L T ,mPL
I

t
Temp. b
d
,
(a) (b)
e
Time Time
• llme -

T,mp, c T'mp ~
Fig, 15,7

(.) a toband etoJ (b) dlo eand e to J


(e) (d) (e) btocandctod (d) btocanddtoe

11m, 11m,
64. The temperature of equal masses of three different liquids 70. Two bodies at different temperature are mixed in
A,B, and C are 12°C, 19°C and 28°C respectively. The calorimeter. Which of the following quantities remains
temperature when A and B are mixed is 16°C and when B conserved?
and C are mixed is 23°C. The temperature when A and C are (a) sum of temperature of two body
mixed is: IWB (JEE) 2007]
(b) total heat of two bodies
(a) I8.2'C (b) 22'C
(c) total internal energy of the two bodies
(c) 20.2'C (d) 24.2'C
(d) internal energy of each body
65. 19 g of water at 30°C and 5 g of ice at - 20°C are mixed
71. A copper block of mass 3.35 kg is heated to a temperature
together in a calorimeter. What is the final temperature of
of 500°C and then placed in a large block of ice. What is the
the mixture? Given specific heat ofice=0.5 calg -1 (c o )-1 maximum of ice that can melt? Specific heat of copper
and latent heat of fusion of ice "" 80 cal g - 1. = 390J kg -1 (CO) -1 and heat of fusion ofwater= 3.35x 10 5
fWD (JEE) 2009] Jkg- I .
(b) -5'C (c) 5'C (d) IO'C (a) 1.95 kg (b) 2.25 kg
66. A resistor and a capacitor are connected in series with an (c) 2.50 kg (d) 3.05 kg
a.c. source. If the potential drop across the capacitor is 5 V
72. Two solid bodies of equal mass m initially at T "" 0"C are
and that across resistor is 12 V, the applied voltage is:
heated at a unifonn and same rate under identical
IKarnataka CET 2009) conditions. The temperature of the first object with latent
(a) 13 V (b) 17 V heat ~ and specific heat capacity in solid state C1 changes
(c) 5 V (d) 12 V according to graph 1 on the diagram. The temperature of the
second object with latent heat ~ and specific heat capacity
67. 0.1 m 3 of water at 80°C is mixed with 0.3 m 3 of water at
in solid state C 2 changes according to graph 2 on the
60°C. The final temperature of the mixture is:
diagram. Based on what is shown on the graph, the latent
IKarnataka CET 2009) heats LJ and L2 , and the specific heat capacities C 1 and C 2
(a) 65'C (b) 70'C in solid state obey which of the following relationships?
(c) 60'C (d) 75'C
68. When hot liquid is mixed with a cold liquid, the
temperature of mixture:
(a) first decreases then becomes constant
(b) first increases then becomes constant 2 3 4 5 6 789
11me -
(c) continuously increases Fig. 15.8
(d) is undefined for some time and then becomes nearly
constant (a) I, >L,:C 1 <C, (b) LI <L,:C 1 <C,
69. The ratio of the specific heat capacity to molar heat capacity (c) LI >L,:C 1 >C, (d) I, < L,:C 1 >C,
ofa body: 73. 4 g of steam at 100°C is added to 20 g of water at 46°C in a
(a) universal constant container of negligible mass. Assuming no heat is lost to
surrounding, the mass of water in container at thennal
(b) depend on mass of the body
equilibrium is : Latent heat of vaporization = 540 callg
(c) depend on molecular weight of body (Specific heat of water = 1 callg -CO)
(d) is dimensionless (a) I8g (b) 20g (c) 21 g (d) 22g

ANSWERS

1. (a) 2. (e) 3. (d) 4. (b) 5. (.) 6. (e) 7. (a) 8. (d) 9. (e) 10. (b) 11. (d) 12. (d)
13. (a) 14. (a) 15. (b) 16. (a) 17. (e) 18. (c) 19. (b) 20. (a) 21. (d) 22. (a) 23. (b) 24. (a)
25. (d) 26. (a) 27. (d) 28. (a) 29. (b) 30. (b) 31. (d) 32. (e) 33. (b) 34. (a) 35. (b) 36. (d)
37. (e) 38. (b) 39. (d) 40. (a) 41. (c) 42. (d) 43. (d) 44. (d) 45. (a) 46. (c) 47. (b) 48. (e)
49. (b) SO. (d) 51. (a) 52. (b) 53. (a) 54. (e) 55. (e) 56. (b) 57. (a) 58. (c) 59. (b) 60. (e)
61. (b) 62. (b) 63. (d) 64. (c) 65. (e) 66. (a) 67. (a) 68. (d) 69. (e) 70. (c) 71. (a) 72. (a)
73. (d)
(8) More than One Choice is Correct 10. Which of the following statements are true ?
). Which of the following statements are not true ? (a) With increase in pressure, boiling point of water
increases
(a) Heat is a macroscopic physical quantity
(b) Heat cannot be stored in a body
(b) Heat is intrinsic property ofa body
(e) With increase in pressure melting point of ice
(e) Heat is stored in a body as internal energy decreases
(d) Heat is path independent
(d) Vapour can be directly converted into solid
2. Which of the following statements are true?
11. Increase of pressure:
(a) Whole of work can be converted into heat (a) Always increases the boiling point ofa liquid
(b) Whole ofheal can be converted into work
(b) Always increases the melting point ofa solid
(e) Temperature ofa body can be raised without heating it (c) Increases the melting point of solids which expand on
(d) Temperature of a body may not change when it is melting
heated
(d) Always decreases the melting point of a solid
3. Thennal capacity ofa body depends on:

T.mp~
12. Heat is supplied to a certain
(a) The beat given homogeneous sample of matter at a
(b) The temperature raised uniform rate. Its temperature is
(e) The mass of the body plotted against time as shown in Fig.
15.9. Which of the following TIm.
(d) The material of the body Fig. 15.9
conclusion can be drawn?
4. Specific heat of a substance can be :
(a) Its specific heat capacity is greater in solid state than in
(a) Finite (b) Infinite
liquid state
(e) Zero (d) Negative
(b) Its specific heat capacity is greater in liquid state than
S. Molar heat capacity of a gas does not depend on: in solid state
(a) Its temperature (c) Its latent heat of vaporization is greater than its latent
(b) Its molecular weight heat of fusion
(c) Its atomicity (d) Its latent heat of va poriution is smaller than its latent
(d) The conditions under which heat is supplied heat of fusion
6. With increase in pressure on a substance : ANSWERS
(a) b.pt. may be elevated (b) b.pt. may be depressed
(c) m.pt. may be elevated (d) m.pt. may be depressed 1. (b) and (d); 2. (a), (c) and (d); 3. (c) and (d); 4. All; S. (a)
7. We can get vapour state of a substance through the pr<;)Cess and (b); 6. (a), (c) and (d); 7. (a), (b) and (c); 8. (b), (c) and
of: (d); 9. (.), (b) and (c); 10. All; 11. (a) and (c); 12. (a) and (c).
(a) Boiling (b) Evaporation
(c) Sublimation (d) RegeJation Ic] Integer Type Questions
8. When two samples at different temperatures are mixed, the I. Equal masses of three liquids A ,R and Chave temperatures
temperature of the mixture can be : IO"C, 2SoC, 40°C respectively. If A and B are mixed, the
(a) Lesser than lower or greater than higher temperature mixture has a temperature of 1SoC. If Band C are mixed the
(b) Equal to lower or higher temperature mixture has a temperature of 30°C. If A and C are mixed,
the mixture will have a temperature (xx S)OC, then find the
(c) Greater than lower but lesser than higher temperature
value of.
(d) Average of lower and higher temperatures
2. In an industrial process 10 kg of water per hour is to be
9. When mg of water at IO"C is mixed with mg of ice at ODC, heated from 20°C and SO°C. To do this, steam at lSO"C is
which of the following statements are false? passed from a boiler into a copper coil immersed in water.
(a) The temperature of the system will be given by the The steam condenses in the coil and is returned to the boiler
equation. mx 80+ mx Ix (T -0) .. mx Ix (IO-T) as water at 90"C. How many kg of steam are required per
(b) Whole of ice will melt and temperature will be more hour 7 Specific heat of steam - I kcallkg C". Latent heat of
than O"C but lesser than 1O"C steam'" S40 kca! kg -t .
(c) Whole ofice will melt and temperature will be O"C 3. If there are no heat losses, the heat released by the
(d) Whole of ice will not melt and temperature will be ODC condensation x g of steam at 100°C into water at 100°C
converts yg oficeatO°C into water at !Oa°e. The ratio yl x IE) Linked Comprehension Type
is :
Paragraph - 1
4. 2 litre of liquid at 10°C is mixed with 7 litre of same liquid
An icc cube of mass 0.1 kg of O°C is p laced in an isolated
at 20o e. Consider the linear increase in volume of liquid container which is 227°C. The specific heat capacity S of
with increase in temperature and neglect all kinds of heat the container varies with temperature T according to the
losses from liquid to surrounding. After thennal empirical relation S = A + BT, where A = 100 cal/kg - K
equilibrium is reached, find the net volume of liquid (in and B = 2x 10- 2 cal/kg - K2 . If the final temperature of the
litre), (Temperature coefficient of volume expansion of container is 27°C, latent heat of fusion of water is 8x 104
liquid = 0.000035 OC- 1) cal/kg, specific heat of water is 10 3 cal/kg-K.
5. Water of mass m2 "" 1 kg is contained in a copper Answer the following questions.
calorimeter of mass m1 = 1 kg. Their common temperature
1. Heat gained by the ice will be :
t =lO"C. Now a piece of ice of mass m3 = 2 kg and
temperature is -iloe dropped into the calorimeter. (a) lOx 10 3 cal (b) 10700 cal
Neglecting any heat loss, find the final temperature of (c) 700 cal (d) 1700 cal
system (in cq. [Specific heat of copper = 0.1 kcal/kg Co, 2. The mass oftbe container will be :
specific heat of water = 1kcal/kg Co, specific heat of ice = (a) 0.495 kg (b) 0.125 kg
0.5 kcal/kg Co, latent heat of fusion of ice '" 78.7 kcal/kg] (c) 0.3 kg (d) 0.9 kg
6. 2 kg of ice at - 20°C is mixed with 5 kg of water at 20°C in 3. If the coefficient of linear expansion for ice varies with
an insulating vessel having a negligible heat capacity. temperature as a (T} If Lo is initial length of the edge of ice
Calculate the final mass of water remaining in the cube at 10 and the temperature of ice changed from TO to
container. Given specific heat capacities of water and ice T(To >T)tben:
are 4.186 kJ K- 1 and 2.092 kJ K- 1 kg - 1 respectively.
Latent heat of fusion of ice == 334.7 kJ kg -I . (in kg) (a) L=Lo/'U(T)dT (b) L=Lo [I +J.U(T)dT]
7. The earth receives 8.4 J/minute/m2 energy from the sun. If
4
the solar energy in lOx 10 m 2 area is concentrated in a cup
field with 100 g water at O°c. The time the water will boil in (c) L=Lo [1- J. U(T)dT] (d) L>Lo
sec. is :

Paragraph - 2
A source of heat supplies heat at a constant rate to solid
1. 2 2. 1 3. 3 6. 6 7. 3
cube. The variation of temperature of the cube witb heat
supplied is shown
[D] Match the Columns
F
1. Match the Column-I with Colunm-II. o
E
Column -] Column - II

(a) Principle of calorimetry (P) Conversion of energy


Heat supplied -
(b) Specific heat (q) Phase transfonnation Fig. 15.10

(c) Latent heat (,) Vigorous solidification 4. The portion DE of graph represents conversion of:
(a) solid into liquid (b) liquid into vapour
(d) Super cooling (,) Maximum fo,
(c) solid into vapour (d) vapour into liquid
hydrogen
5. The reciprocal of the slope of the portion EF of the graph
(t) Heat gained is equal to shown in figure represents:
heat lost (a) latent heat affusion
(b) latent heat of vaporization
ANSWERS
(c) thennal capacity of liquid
1. a-+t;b-+s;c-+q;d-+r
(d) thermal capacity of the vapour
6. In the figure it is observed that DE:::: 3BC. This means that: (c) How a sea moderates the climate of nearby coastal land?
(a) the thennal capacity of the vapour is 3 times that of (d) Why steam bums are more painful than those caused
liquid by boiling water at same temperature?
(b) the specific heat of vapour is 3 times that ofliquid 7. Which extinguishes a fire more quickly, hot or cold water?
(c) the latent heat of vaporization of the liquid is 3 times [Ans. Hot water; being at higher temperature is converted into
the latent heat of fusion of liquid steam in lesser time (which by Slopping combustion extinguishes
(d) the latent heat of fusion of the solid is 3 times the latent the fire).]
heat of vaporization of the liquid 8. Can water in a beaker be made to boil by placing it in a bath
of boiling waler?
ANSWERS {Ans. No; the water in the beak.er will be heated to lOO"C bul will
Paragraph-1 1. (b) 2. (a) 3. (b) not boil as for boiling required heat (mL) will not be transferred
from the bath to the beaker as both are at same temperature
r.ragraph-2 4. (b) S. (d) 6. (e) 100°C.]
9. Can water be made to boil without heating?
IF] Subjective Type Question. [Ans. Yes; by reducing pressure over it, as with decrease in
1. What is wrong with the statement "body at higher pressure b.p!. is lowered and ifprcssure is decreased to the extent
temperature contains more heat"? that b.pt. becomes equal to the room temperature, the water will
start boiling without heating.]
[Ans. Heat is the energy in transit so a body cannot contain heal.
Actually heat in a body is stored as internal energy.] 10. Can you account for the fact that "in case of ice floating in
water (or placed on a table) more ice melts from the base
2. Does the mass ofa body change when it is heated or cooled?
than from sides or top''?
[Ans. Yes; in heali ng (or cooling) a body absorbs (or releases)
{Ans. The pressure at the basc[po + (mg I s)]will be more Ihan on
energy and in accotdancewilhE= m~. i. ~ .m - (£ 1dl), the mass
sides or top (Po), so melting point at base is lowered resulting in
will increase (or decrease). However, due 10 large value of speed
of light c, the change in mass is too small to be detected more melting.]
experimentally. (Rumford actually performed experiments to 11. What is the effect of pressure (a) on boiling point of a
detect the change in mass which failed.)] liquid? (b) on melting point ofa solid?
3. Distinguish between calorie and Calorie. [Ani. (8) Always elevates (b) may elevate or depress depending
(Ans. calorie is the unit of heat energy used in physics while on the nature of solid. For solids such as ice which contracts on
Calorie is the unit ofenergy value of foods used in nutrition and melting, it is depressed while for solids such as sulphur and wax
1 Cal = I kcal - 1000 caL] which expand on melting it elevates.]
4. Can temperature of a body be raised without heating it? Is 12. Can ice be made to sublimate? Is the reverse of sublimation
the converse also true, i. e., the temperature of a body may also possible?
not change when it is heated? [Ans. Yes; by reducing pressure over it so that external pressure
[Ani. Yes; in adiabatic change, e.g., in compressing a gas or becomes equal 10 vapour pressure of solid, e.g., ice will sublimate
on moon (where external pressure = 0). The reverse of
shaking a liquid. Yes; in isothermal change, e.g., in melting or sublimation, i.e. ,conversion ofvapours directly into solid is called
boiling·1 'Hoar frost' and freezing of clouds 10 form snow is an example of
5. Can the specific heat ofa substance be (a) zero, (b) infinite it. Here it is worthy to note that the main difference berween ice
and (c) negative? and SIlOIl' is thefact that ice resultsfrom conversion o/Iiquid into
[Ans. (a) Yes; as c- (Q I m.6.T), forc - Oeither Q = Oor.6.T .. 00. solid while S IIOW from conversion of vapours illto solid.]
But as.6.T cannot be 00, Q '" 0. i.e., the change must be adiabatic, 13. Ex.plain why two pieces of ice stick to each other if they are
e.g.,dwingcompressionofagasor sbakingofa liquidllT ;11: Obut pressed against each other and released?
Q '" Oso c= O. (b) Yes; forc =00 either Q _ 00 or.6.T = 0. But as (Ani, Oue to regelatioll .)
Q 7' 00, AT = 0. i.e., the change must be isothennal, e.g., during
melting of a solid or boiling of a liquid 6Q ;11: 0 but AT = 0, so 14. You are given a closed E
C 00, (c) Yes; negative specific heat will mean that with increase metal container with an
II:

in temperature (6T will be positive), heat will be released (Q will unknown substance. Could t ~Cl-_ _...JID
be negative). This actually happens in case of saturated vapours, you detennine whether the ~
i.e. • specific heat ofsaturated vapours is negative.] substance was liquid, gas or {E
6. Explain: crystalline solid? You are
(a) Why water is used in hot water bottles or as coolant in allowed to carry any thermal 0"------- - - -
experiment, you wish on it. TIme -
radiators? The heat capacity of the Fig. 15.11
(b) Why sprinkliug of water reduces the temperature of a container is known and is independent of temperature.
closed room?
[Ans. By supplying heat at constant rate and plotting a graph calorimeter if the ice is to be changed to water and heated to
between temperature and time, (Fig. 15.11) we shall note the 40°C? [Heat of vaporization of water = 540 caVg; heat of
number of horizontal lines. If these are two the substance was fusion of ice = SO callg and specific heats of copper and ice
solid, if one, liquid and ifnone, gas.]
are 0.1 and 0.5 callg CO respectively.)
15. Fill in the blanks: [Ans.4.615g)
(a) Assuming equal masses of ice and water in contact, the 21. A child (of mass 30 kg and sp. heat I callgCO) running a
temperature of water needed to just melt the ice is temperature of 10l oP is given an antipyrin (a medicine that
lowers fever) which causes an increase in the rate of
(b) 300 g of water at 25°C is added to 100 g of ice at oDe; evaporation of sweat from the body. If the fever is brought
the final temperature of the mixture is .......... . down to 98°P in 20 minutes, what is the extra average rate
(e) A substance of mass M kg requires a power input of P of evaporation caused by the drug? (Latent heat of
watts to remain in the molten state at its melting point. cvaporation of water in the given range of temp. , is 500
When the power source is turned off, the sample cal/g.)
completely solidifies in time t seconds. The latent heat [Hint: Here I:J.T = 101 - 98 = 3 F' = (513) CO and if (dm I dt) g of
offusionofsubstance is ........... (lIT 1992) water evaporates per minute Mcl:J.T = (dm l dl) x(t in min) x(L)]
[Hint: See solved Problem 6.J [Ans.5 glminuteJ
(d) The variation of temperature of a material as heat is 22. How much steam at lOoDe will just melt 3.2 kg of ice at
given to it at a constant rate as shown in Fig. 15.11. The - 10°C ifspecific heats of ice, water and steam are 0.5, 1 and
material is in solid state at point 0; the state of the 0.47 caVg Co respectively while latent heats of fusion and
material at the point P is .......... . vaporization are 80 and 540 caVg respectively?
[ADS. (a) BODC (b) ODC (c) (Pt ! M) Cd) partially solid and partially [ADS. 425 gj
liquid.] 23. Ifa mass of ice at O°C is mixed with an equal mass ofwater,
16. Water falls through a height of250 m. Assuming that whole what is the temperature of the mixture if the initial
of the energy due to the fall is converted into heat, calculate temperature of water was (a) lOO°C? (b) 10°C? How much
the rise in temperature of water. [g = 9.8 mJs 2 and specific ice has melted in each case?
heat of water is 4.2 kJlkg K) [Ans. (a) 10°C and whole, (b) DoC and only (l/8)th]
[ADS.O.5B°C] 24. A closed thennally insulated vessel contains 100 g of water
17. A lead bullet at 11 ODe strikes a steel plate and melts. What at O°C. If the air from this vessel is rapidly pumped out,
was its minimum speed if the heat produced is shared intensive evaporation will produce cooling and as a result
equally between the plate and the bullet? of this water will freeze. How much ice will be fonned by
this method? If latent heat of fusion is 80 callg and of
{Specific heat and latent heat oflead are 0.03 caVg CO and 5
evaporation 560 callg.
cal/g re spectively while its melting point is 327°C]
[Hint: rfm g ice is formed, mLF = (lOO-m)Ly]
[Ans. 440m/s]
IS. A refrigerator converts 50 g of water at 30De into ice at [Ans. 87.5 g]
-20°C in one hour. Determine the quantity of heat removed 25. When a small ice crystal is placed into supercooled water, it
per minute. [Specific heat of ice and water are I and 0.5 begins to freeze instantaneously. (a) What amount of ice is
caVg CO respectively and latent heat of fusion for ice is 80 fonned from 1 kg of water supercooled to SOC? (b) What
cal/g.] should the temperature of the supercooled water be for all
[Ans. 100 caVmin] of it to be converted into ice?
[Ans·Ca) 100 g, (b) - 80°C]
19. At a place where intensity of solar radiation is 600 W/m2 a
small solar cooker of reflecting diameter of 0.6 m and 26. In an insulated vessel, 0.05 kg steam at 373 K and 0.45 kg
efficiency 40% is used to heat up water. How long would it of ice at 253 K are mixed. Then, find the final temperature
take to completely boil off 0.5 litre of water initially at ofthe mixture. (lIT 20061
20°C? [Specific and latent heats of water are 4.2 kJlkg K Given 4-usion =SO caVg = 336 JIg, Lvaporization = 540 cal/g
and 2256 kJ/kg] ~ 2268 Jig,
[ADS. 5.3 hours]
S ice = 2100 J/kg K = 0.5 caVg K
20. If 20 g of ice in a 109 copper calorimeter is originally at and S water = 4200 J/kg K = I callg K
- 30°C. How much steam at 100°C must be condensed in the
(Ans. DoC]
Heat Transfer
[Heat can be transferred from one place to the other by entering the rod at one end becomes equal to that leaving the
three different processes namely, C{mdUClion, convection and rod at the other end.
radiation. Cd'nduction usually takes place in solids, ICJ Law of Conduction
convection in fluids (i.e., liquids· and gases) and no medium is In steady state heat passing through a bar of length Land
required for radiation. In this chapter we shall discuss these cross-section A in time t when its ends are at temperatures eI
modes one by one in detaiL]
and 92 « 9, );is~g=i=ve=n=b=Y=:=::..!:l'::===~~
§ 16.1 Heat Conduction
[AJ Process e, (r) A ----+> Q ()e,
In conduction the molecules ofthe body transfer heatfrom
a place at higher temperature to a place aJ lower temperature Fig. 16.1
without actually moving in the body. According to kinetic (9 -9 )
theory, the molecules of a solid are in a state of oscillation Q=KA I 2 t .... (i)
about their fixed positions of equilibrillm; thus when a bar is.
L
heated at one end, the molecules at' 'the heated end absorb So rate of flow ofheat will be
energy. Therefore, they vibrate about their equilibrium dQ __ -KA d9 .... ( n
.. )
position more violently and interact·· with the molecules in dt dx
the next section of the rod. Thus they transfer a part of their The quantity (de/dx) is called temperature gradient
energy to neighbouring molecules putting them in more (minus sign indicates that with increase in x, temperature 9
violent vibrations. These new molecules in tum transfer decreases) and the constant K depends on the nature of metal
energy by interaction to the molecules in the next section and and is called coefficient of thennal conductivity or simply
so on. Thus_t1!~ heat energy i ~ passed along the rod from thermal 'conductivity and · is a· measure of the ability of a
molecule to' molecul.e without changing their equilibrium substance to conduct heat through it. The dimensions of
position along the tiar. coefficient of thennal conductivity are [MLT - J 9 - I J while its
IB) Steady State SI units are W/mK and practical units calfcrn sec Co or caVm
Ifa bar ofunifonn cross-section is heated at one end, each sec CO.
cross-section of the bar receives heat by conduction from the ID) Discussion
adjacent cross-section towards the heated end. This heat is (I) In general solids are better conductors than liquids···
used in three ways; a part is absorbed by the cross-section to and liquids are better conductors than gases. Metals are much
increase its temperature, another part escapes out from the better conductors than non-metals. This is because metals
sides of the cross-section in the fonn of radiation and the third contain 'free' electrons which behave like molecules of gas
is conducted to the next cross-section. Thus. the tempera11lre of and helps in transferring energy from hot to cold end due to
each cross-section increases and after some time a state is their large thennal speeds [v <x:.fi].
reached when temperature at each cross-section of the bar
(2) Thermometric Conductivity
becomes constant though different for different cross-sections.
This state of the bar is called steady state. In steady state no The ratio of thennal conductivity to thennal capacity pcr
heat is absorbed by the bar and if radiation and convection unit volume o f a material is ~ Called thermometric
losses from the sides are minimised and neglected, the heat conductivity or diffusivity, i. e.,

• Mercury though liquid is heated by conduction and not by convection.


•• In liquids and gases the interaction is molecular collision. in non-metals lattice vibration and in metals both lattice vibration and free electron collbion.
.... Ifliquids and gases are heated from the top (so thaI convection is nOI possible) they transfer heat (from top to bottom) by conduction.
"I----
SH-·,··~-II"
K
D=- .... (Hi)
pc
'Itis a measure of rate of change of temperature (with time) _H,
when the body is not in steady state. (b)
(3) Wiedemann-Franz Law Fig. 16.2
Wiedemann and Franz have shown th",t at a given
temperature T, the ratio of thenna! conductivity to electrical
or °1 -0, =H(R t +R,), i.e., H=(OI-O,)/(R I +R,)
(° -0,) .
conductivity is constant, i.e., or H= 1 With R = R r +R 2 .... (viii)
R
-K =constt. .... (iv) i.e., in series the total thermal resistance is equal to the sum of
crT
individual resistances.
i. e., a substance which is a good conductor of heat (e.g., silver)
(b) If the two rods are in parallel as shown in Fig. 16.2 (b):
is also a good conductor of electricity and a substance which is
a bad conductor of heat (e.g., porcelain) is also a bad conductor HI =(° 1 -0,) .and H, =(° 1 - 0,)
ofeiectricity. RI R2
(4) Ingen Hauz's Experiment But here H = H 1 +H2
Ingen Hauz's has shown that if a number ofidentical rods
of different metals are coated with wax and one of their ends is 50 that H= (Ot -0,) with ~= _
, +_R2,_ . . (ix)
put in boiling water, then in steady state the square oflength of R R Rl
the bar over which wax melts is directly proportional to the From Eqns. (viii) and (ix) it is evident that flow of,heat
thennal'conductivity of the metal, i. e., through rods in series and paraUel is an~logous to the flow of
current through resistances in series and parallel. This analogy
~ = constt. .... (v)
is of great importance in solving complicated problems of heat
L
conduction. [See solved Questions III and IV and Problems
(5) Thermal Resistance R (4), (5), (6) and (7).]
The law of thennal conduction (2) Growth of Ice on Ponds
(° -0,) AO
Q = KA 1 L t canbewrittenasH=R .... (vi) VVhen atmospheric
temperature falls below O°C
with (Q/t)=H and (° 1 -O,)=M the water in the Jake will
which is analogous to Ohm's law (I = VIR) with start freezing. Let at any
L time t, the thickness of ice in
R =- ".,(vii) the lake be y and
KA
atmospheric ,temperature ) s
The quantity'R is called thennal resistance· in analogy to
--a°C. The temperature of
electrical resistance (LlaA) and has unit (KIW). Greater the water in contact with the Fig. 16.3
thermal resistance, better will be the insulation and poorer
lower surface of ice will be zero, If A is the area of the lake,
will be the conduction.
heat escaping through ice in time dt,
lEI Applications
(1) Rods in Series and Parallel dQI = KA[O-(-ll)]dt
y
(a) If two rods are in series as shown in Fig. 16.2 (a), the
heat flowing per sec through them will be Now due to escaping of this heat if dy thickness of water in
contact with lower surface of ice freezes,
- (Ot -0) d H _-,-
(0--;:-_0,'-'.)
H 1- an 2- dQ, =mL=p(dyA)L [asm=pV=pAdy]
RI R2
But as dQI =dQ2' the rate of growth of ice will be
But here, i.e., in series HI H 2 H = = dy K9 ,
So 9 1 - 9 = HR 1 and 9 -9 2 =HR 2 -=-x- ....(x)
dt pL y
and so time taken by ice to grow a thickness y,
t=pLl'ydy=!PL y' , ... (xi)
(a) K9 0 2K9

• Some authors and books use the term thermal resistance or R-value for (LIK).

, .
From Eqn. (xi) it is clear that time taken to double and cross-section 1t[(2R)2 _ R 2)=31tR 2 will conduct heat
triple the thickness will be intbe ratio tl :/2 :13: :1 2 :2 2 :3 2, simultaneously; so total heat flowing per sec will be
i,e., 'I : '2 : '3 : : 1 : 4 : 9 and the time intervals to change _dQ = _dQ_, + _dQ_, _ :.:
K:!.:,1tR:::...-'C;(6:!.:,_-",6,,-,)
thickness from 0 to y, from yto2yand so on will be in the ratio
dldldl L
41, "~I, : 41, :: 0' -0'): (2' -I'): (3' -2'),i... ,M,: 41,
K,31tR'(6, -6,)
: 41, :: I : 3 : 5. + L . ... (i)
Quesdoa I. Explain why in summer a block o/metal feels
hotter than a block a/wood while in winter re~rse_is the cose, Now if the equivalent conductivity is K,
although the temperature ofboth is same either in summer or
in winter (whatever it is} At what temperature would the two
dQ =K 41tR '(6,-6,) [~SA=7t{2R)'] .... (ii)
dl L
feel equally hot or cold?
So from Eqns. (i) and (ii) we have,
Answer: On touching a body. the feeling of hotness
depends not only upon the temperature but also upon the 4K=K, +3K" i.e, K = (K, +3K,)/4
conductivity. The conductivity of metals is much higher than Alternative Answer
that afwood. In summer room temperature is more than that of This problem is equivalent to two thennal resistances of
the human body (3']OC). Soon touching heat will flow from the equal length L and cross~sections 1tH 2 and n[(2R)2 _ R 2]
_block to our body. Due to large thennal conductivity metals
= 31tH 2 in parallel so that,
will conduct while wood will not and as heat flows from higher
temJ?Crature to lower, the metal will appear hotter than wood. _1_ = _1 + _1_ with R =J,...
Req Rl R2 KA
In winters room temperature is lower than that of our
body, soon touching heat will flow from our body to the block. Now if K is equivalent conductivity of rod of length Land
However, due to large thermal conductivity of metal heat will radius 2R, i. e., cross-section 41tH 2
flow from our body to metal and so it wilt appear colder than
wood. K41tR' K,1tR' K,l1tR' K
L = L + i.e., K = c:.;,_+.:.3"KCL
L '
However. if the temperature of room, i.e. blocks is 3'r'C 4
(equal to that of our body) no transfer of heat will take place Question IV. Four square pieces o/insulalion, o/two
between our body and the block (whatever be their thermal different materials all with same thickness and area A, are
conductivities) and so both will appear equally hot or cold. available to cover an opening ofarea 2A. This can be done in
Quetdon II. Two rods A and B are ofequal length. Each either ofthe two ways shown in Fig. 16.4. Which arrangement
rod has its end at temperatures T, and T2. What is the (a) or (b) would ensure lesser heatflow if Kl ::F- K2?
condilion that will ensure equal rates offlow of heat through
the rods A and B?
ADswer: We know that in case oCheat conduction (dQldt)
=KA.(O, - 8 2 )/L, so for equal rates of flow,
dQ, =dQ" i.e., KAAA =(6,-6,) =K.A. (6, -6·, )
dt dt LA La
or (AA1A.)=(K.IKA) [as LA =L.=Lgiven]
i.e., the ratio ofcross-sectional area ofthe two rods must be in (0) (b)
the inverse ratio oftheir thermal conductivities. Fig. 16.4
QuestloD III. A cylinder ofradius R made ofa material of Answer: If the thennal resistance of piece of conductivity
thermal conductivity Kl is surrounded by a cylindrical shell 0/ Kl isR I while that ofK2 isR 2 , for case (a),2Rl and2R2 will
inner radius R and outer radius 2R made of a material of be in parallel.
the""!al conductivity K 2. The two ends 0/the combined system _1- = _1_ + _I_
are maintained at two different temperatures. There is no loss So
Ra 2R) 2R2
-0/ heat across the cylindrical sut/ace and the system is in
steady slale. What is the effective thermal conductivity of the i.e., R =2R,R, =HM
system? , (R, +R,)
Aoswer: In this situation a rod of length L and area of while for case (b) (R 1 + R2 ) and (R, + R2 )will be in parallel.
cross-section 1tR 2 and another of same length Land
fusion of ice and vaporisation of water are 80 callg and 540
AM
callg respectively. (MNR 1992]
Solution: If the point is at a distance x from water at
Now as arithmetic mean is greater than hannonic mean, if
100°C, heat conducted' to ice in time t,
R! =f. Ri ' thennal resistance in case (b) is higher than in case (a).
So case (b) would ensure lesser heat flow as compared to (a). Q . "KA(200 - 0)xt
Ice (1.5-x)
Problem 1. The only possibility ofheat flow in a thennos
Steam x I.S-x)
flask is through its cork which is 75 cm 2 in area and 5 em
, ::::::::
thick. Its thermal conductivity is 0.0075 (ca/km sec CO). How
iO,ng will 500 g of ice ~ttOOC !n the thermos flask take to melt ,. Ice
Water "
,.1'
into water at QOC? The outside temperature is 40°C and latent at 100°C at ooc . ';.\. '." \
heat of ice is 80 cal/g. Fig. 16.5
··t
Solution: The heat entering the flask through conduction So ice melted by this heat
, ·1 ' J : I i i.'
by the cork in time t sec will be, _. ,I
"i' m. =,Qice = KA (2QO -9) x lt
Q\ = KA (8\ -8,) t = 7.5 x 10- 3 x 75 x (40-0) t Ice Lp 80 (1.5-x)
L . 5
Similarly heat conducted by the rod to the water at 100°C
= 4.5 x t cal in time t,
. '

And heat required to melt the ice in the flask, =KA(200-100)t


Qwater
Q 2 =mL = 500 'x 80 = 40x10 3 cal X

~ccording
to giv,ep problem, Q\ ':" Q2> i. e., So steam formed by ~his heat

4.5xt=40xlO J or t=8.9x10 3 sec=2.47hr Answer m = Qwater =KA(200 - 100)t


steam L . 540 x ,x
Problem 2. A room at 20°C is heated by a heater of V
resistance 20 ohm connected to 200 volt mains. The According to given problem mice = m steam , i. e~,
temperature is uniform throughout the room and the heat is , 200 ' = ' 100 ~; t x=.2.m"~1O.34cm
transnHtted throug~ a glass window of are~ " 2
"tm d im 80(1.5 - x) 540x, 58
"
thickness 0.2 cm. Calculate the temperature outside. Thermal i. e., 200°C temperature must be maintained at a distance 10.34
cond/fctivity of glass is 0.2 callm C" sec and mechanical em from water at 100°C.
equivalent ofleat is 4.2J/c.al.
. Problem 4. Two identical rectangular rods of metal are
Solution: Ire
is the temperature of outside, heat passing welded end to end as shown in Fig. 16.6 (a) and 10J of heat
per sec through the glass window, • flows through the rods in 2 min. How long would it take fdr'30J"
dQ = KA (9\ - 9,) ~,0.2 x 1x (20 -8) cal =\ 00(20 - 9) ofheat toflow through the rods if they are welded as show..n in
dt L _.~ 0.2xlO 2 sec I Fig. 16.6 (b)?
And heat produced ~er sec by the heater in the ryom Solution: Ifthe thermal resistance of each rod is R, in case
2 . 2 i
(a) the rods are in series; so
P = ~ Joule = 1:...... cal = 200 x 200 = 476.2 cal RS = R+R=?R
'- R sec RJ sec 20 x 4.2 r sec
, . So rate of flow of heat in steady state will be
Now as the temperature of the room is constant, the heat
dQ A9 10 (100-0)
produced per sec by heater must be equal to the heat conducted --:it =
Rs' i.e., "2=:= 2P .... (i)
th ~ough. t~e glass windo":,",, ·l.

i.e." lOQ(20-9)=476.2 or 9 = 15.24°C Answer


14~,~~,,~'{00i···:······:···!:itD
'1'6o~b ' .•'" .............. ......: ..~~~ .. 1,
Problem 3. One end of a copper rod of uniform cross-
section and of length 1.5 m is kept in contact with ice and the \' \ .. (a) \ ,",,'
other end with water at 100°C:. At what p~i.nt along its length
should a temperature of200OC be maintained so that in steady
state, the mass of ice melting be equal to that of the steam
produced in the same interval of time? Assume that the whole
system is insulated from the surroundings. Laten: heat of (b)
'" 'Fig. 16.6
709

In case (b) the rods are in parallel; so


I 1 ' 1 . R R
T=R+R' I.e., P=2 T:: 1IKl + TzK 2 and, K:: 'KK
12=HM
p K, +K z K l +K2
So rate of flow of heat in this situation will be
dQ_i19 . 30_(100-0) ( .. )
dt - Rp' I.e., t - CRf2) .... 11 Problem 6. Three cylindrical rods A, Band C of equal
lengths and equal diameters are join~d in series as shown in
Substituting the value of R(= 10) from Eqn. (i) in (ii), we Fig. 16.& Their thermal conductivities ar~ 2K, K and 0.5 K
get! =1.5 minute. Answer respectively. In steady state, if the free ends of rods A and C
Problem 5. Two plates each of area A" thickness L1 and are at 100°C andO°C respectively, calculate the temperature at
L 2'> and thermal conductivities K\ and K2 respectively are the two junction points. Assume negligible loss through the
joined to form a single plate of thickness (L1 + L 2 ). If the curved surface. What will ' be the equivalent thermal
temperatures of the free surfaces are TJ and T2 , calculate (a) conductivity? [MNT 1993]
L T" L Too L
rate of flow of heat (b) temperature of interface and (e)
equivalent thermal conductivity. "'
100oC ~~
2K
B
K
I ",·;:tC..
0.5 K
! IOoc

Solution: (a) If the thennal resistances of the two plates Fig. 16.8
are R I and R2 respectively, then as the plates are in series,
Solution: As the rods are in series,R eq =R A +RB +Rc
Rs = R 1 +R2 =!:L+~
AK, AK2
[aSR =!:.....]
KA , with
'
R = (LlKA)

i,e., R =-L + -L +-L - =7L- ,.,,(i)


and so H=dQ =tl9 = (T, - T2 ) = ACTl -T2 ) eq 2KA KA 0.5 KA 2KA
dt R RI +R2 [!l+ L2 ] And h;nce H = dQ = i19 = (100 - 0) = 200KA
, .((1 K2 , dt R (7L12KA) 7L

(b) If T is the ~~c;E~~~ Now in series, rate of flow of heat remains same, i, e.,
common temperature Tl Ir T, H=HA =HB =HC'
of interface, then, as in So for rod A,
series the rate of flow
of heat remains same, [~n = [~~} ie, (100-~B)2KA = 2°~f
i.e.• H =HI (=H 2 ). sp
TI -T2 TI -T ,· Fig. 1~.7 or
, 'TAB = 100 - (100/7) = (600/7) = 87.7 o C
=- -
R "-" And for rod C, "
I
'T = T j R 2 + T2RI dQ] =[dQ]" i.e., (TBC -0) x O.5KA =200KA
i. e... ' [ dt C dt . L 7L
,, (R, +R,) " , '
or TBC =(40017) =57.1 °C
Furth~pnore if Keq is equivalent thermal conductivity,
or
T= [i, L, ] , R
eq
=L+L+L
KA
OJ
7L
2KA
[from Eqn. (i)]

_KI + K2
i.e., Keq =(617)K '" Ans;ver
(c) If K is the equivalent conductivity of composite slab,
I Problem 7. A double-pane window used for insulating a
i.e., slab of thickness L1 + ~ and cross~sectional area A. then room therr;lally from outside, consists oftwo glass sheets each
as in series of area 1m 2 and thickness 0.01 m separated by a 0.05 m thick
stagnant air space. In the steady state, the room-glass
inteifaqe and t~e glass-outdoor interface ar,e atuc,onstant
temperatures of 27°C andO°C respectively, Calculate the rate
Le., [aSR= ~J of heat flow through the window pane: lis~ find the
temperatures of other ·' interfaces, Given, thermal
conductivities of glass and air as 0,8 and 0.08 Wm-IK- l
respectively.
Solution: In case ofthennal conduction as Now as in 1his bridge [(PIQ) = (RIS)], so the bridge is
balanced, i.e., the temperature of junctions C and D is equal
dQ =KA "9 ="9 and the rod CD becomes ineffective as no heat will flow
dt L R
through it. .
with R=~ Now as the thennal resistance of the bridge between
KA junctions B and E is

~R) ~2R)'
Alr
K.O.08 R: (R + (2R i .• , RBE =}R
E
The total resistance of bridge between A andE will be
R,,=R AB +RBE =2R + (4/3)R = (10/3) R
_" , _ So the net rate of flow of heat through the bridge will be
16.9
"". dQ "9 (60 - 10) 15
R = l:~=-.L[0.01 x2+ 0.05] dt =R" = (1O/3)R =]1
" KA A 0.80 0.08
Now ifTB is the temperature at B,
and as here
dQ] _ "9 _ 60-TB
R I 5 26
=-+-=- [ dt AB RAB 2R
'" 40 8 40
dQ = "9 = (27 -O)x 40 =41.5 W Q 60-TB 15
and hence
dt R 26
Answer BUI[d ] =dQ, i.e.,
2R =R' i.e.,
dt AB dt
Now if eland e 2 are the temperatures of air in contact Also atB
with glass in the room and outside as shown in Fig. 16.9.
(27-9,)
41.5 = 0.80 x I =:,~'-'
om
[~~L =[~~L +I~~t
i. e., !1 =30 - Tc + _30,",-~T:1D,-
and 41.5 =0.80 x I (9 2 -0) R R 2R
om
and as Tc =TD =T, 30=3(30-T),
Solving these for a1 and a2 we get
9 1 =26.48°C and 8 2 =0.52°C Answer i. e., TC =TD =T=20°C Answer
Problem!J, A cylindrical block of length 0.4 m and area
Problem 8. Three rods of material x and three rods of
material yare connected as shown in Fig. 16JO(a} All the of cross...rection 0.04m2 is placed coaxially on a thin metal
rods are ofidentical length and cross~sectional area.lftheend disc of mass 0.4 kg and same cross~sectjon. The upper face oj
A is maintained at 600C and the junction E at IDOC, calculate the cylinder is maintained at a constant temperature of400 K
the temperature of junctions H, C and D. The thermal and the initial temperature of the disc is 300 K. If the thermal
conductivity ofx is 0.92 cal/em s CO and that oly is 0.46 cal/em conductivity ofthe material ofthe cylinder is 10 WlmK and the
s CO. specific heat ofthe material ofthe disc is600J/kg K, how long
C

6COC
A 2R B
OR
2R
C

2R
10;C
will it take for the temperature ofthe disc to increase to 3S0K?
Assume for purpose ofcalculation the thermal conductivity oj
the disc to be very high and the system to be thermally
insulated except for the upper face ofthe cylinder.
Solution: If
temperature of disc at time
the 0.04 m2
"-
D D a,
t is the heat conducted by Cylinder
(al (bl
the cylinder to the disc in
time dt O.4m
1
Fig. 16.10
Solution: Treating the given network of rods in tenns of
thennal resistance Rx and R y with
_
dQ-KA 0L
(9 -9) .
dt.. .. (I)
j 1°
Rx = AxO.92
Land R =
y
L [asR = AK
Ax0.46
L] This heat increases the
temperature of disc of
.... Dh,e

specific heat c and mass m Fig. 16.11
so that if R y =2R x =2R frome toa + da.
711

So dQ=mcd9 .... (ii) Problem 11. A 2 m long wire of resistance 4 ohm and
So from Eqns. (i) and (ii), diameterO.64 mm is coated with plastic insulation ofthickness
0.06mm. When a current 015 amperejIows through the wire,
KA (9 0 -9) = dt = med9 find the temperature difference across the. insulation in steady
L
state ifK = 0.16 x 10-2 cal/cm CO sec.
j,e., dt= meL de Solution: Consider a coaxial cylindrical shell of radius r
KA (9 0 9) and thickness dras shown in Fig. 16.13. The radial rate of flow
Integrating the above between limits, 9 = a1 to e = e2' of heat through this shell in will be

t = meL [-log (9 _ 9)]92 = meL !Oge[9 0 -9 1]


KA 0 9] KA 9 0 -9 2
Substituting given data:
1=0.4><600 x 0.4 10 [400-300J = 24010 2
lO x 0.04 ge 400-350 ge
i.e., t =240 x 0.7 = 168sec =2 min 48sec
Fig. 16.13
Answer
H= dQ =-KA d9
Problem 10. A point source ofheat ofpower P is placed dt dr
at the centre of a spherical shell of mean radius R. The
[Negative sign is used as with increase in r,a decreases.)
material ofthe sheil has thermal conductivity K. Calculate the
thickness of the shell if temperature difference between the Now as for cylindrical shell A =2nrL,
outer and inner sUifaces oflhe shell in steady state is T. H = -2nrLK d9
Solution: Consider a dr .
concentric spherical shell of or bdr =_ 21tLKf92da
radius r and thickness dr as fa r H 9 1
shown in Fig. 16.12. The radial
which on integration and simplification gives:
rate of flow of heat through this
shell in steady state will be, H = dQ = 2nKL(9, -9,) .... (i)
dt log,(bla)
H= dQ =-KA d9
dt dr /'R (5)' x4
[Negative sign is used as Here H= 4.2 = 4.2 =24caVs, L=2m = 200cm
with increase in r,e decreases.]
Fig. 16.12 r, =(0.64/2) mm =0.032 em
Now as for spherical shell
and r 2 ~rl +d=O.032+0.006=O.038cm
A =41[r 2
24x log,(38/32)
H=_4nr 2K d9 or fb dr =_ 41tKf92d9 So (9, - 9,) = ---='-------'------,-
So 2x3.l4x200xO.16xIO 2
dr a r2 H 91
= 24x 2.3026[log lO 38 - log lO 32]
which on integration and simplification gives
3.14xO.64
dQ 41Mb(9, -9,)
H=-=K .... (i)
dt (b - a) or (9, _ 9,) = 55X[1.57-1.50]02°C Answer
2
Now in steady state as no heat is absorbed, rate ofloss of
heat by conduction is equal to that of supply, i.e., H = P, and § 16.2 Convection
here Convection requires a medium and is the
process in which heat is transferred/rom one
and a=-b=R place to the other by the actual movement 0/
So Eqn. (i) becomes, heated substance (usually fluid). When a
fluid in a container is heated, the fluid at the i..=::::;\iiiii;;;;:=CI
p = 41tR2KT bottom gets heated due to con~uction
(b a) through the bottom of the container. Its
41tR 2 KT density decreases and consequently it rises
Fig. 16.14
i. e., thickness of shell (b - a) = p Answer up. At the same time denser cool fluid will
712 PHYSICS FOR COMPETITIONS - Vol. I

move down and thus convection currents are set up in the upwards and so the liquid in the tube will move clockwise, i. e.,
liquid and the whole fluid is heated by actual movement of fromB 10 A. [Fig. 16.15].
mass. This type of convection which results from difference in § 16.3 Radiation
densities is'called natural convection. However, if a heated
The process by which heat is transferred directly from one
fluid is forced to move by a blower, fan or pump, the process is
called forced convection. body to another, without requiring any medium is called
radiation. It is by radiation that the heat from the sun reaches
It is found that rate of heat convection from an object is the earth or a hot object hung in vacuum cools.
proportional to the temperarure difference (~9) between the
To explain heat transfer by radiation it is now universally
object and convective fluid and the contact area A, i.e.,
accepted that all bodies at all temperatures and at all times are
dQ] =hA /l9 emitting energy. This energy is called radiant energy and is in
[ the fonn of electromagnetic waves. The electromagnetic
dt convection
where h is a constant of proportionality called convection waves of wavelength ranging from 0.0001 mm to 1 mm are
coefficient ind depends on the properties of fluid such as or
called infra red radiation ' heat radiatio·n. When a body
density, viscosity, specific heat and thermal conductivity, etc. absorbs more heat radiation'than emitted, its temperature rises
and it is said to be heated by radiation. However, if a body
Regarding convection it is worthy to note here that:
emits more radiation than absorbed, its temperature falls and it
(1) Natural convection takes heat from bottom to top is said to be cooled by radiation.
while forced convection in any direction.
.Regarding radiation it is worth noting that:
(2) In case of natural convection, convection currents
(1) Radiation is thefastest mode ofheat transfer from one
move warm fluid upwards and cool fluid downwards.
place to another ' as in this mode heat energy is
This is why heating is done from base while cooling
from top. propagated at speed of light, i. e., 3 x 10 8 mls.
(3) Natural convection cannot take place in a gravity-free (2) As all bodies radiate energy at all temperatures and at
region such as freely falling lift or orbiting satellite. all times, radiation from a body can never b~'stopped
but can be minimised.
(4) Natural convection plays an important part in
ventilation, in changing climate and weatber and in (3) It does not affect the medium through which it passes,
fonning land and sea b're~'zes and t~ade winds. (i.e., which is ' transparent to it), e.g., when heat
radiation is converged by a lens on a body which is
(5) The forced convection of blood in our body by the
opaque to it, the body is heated and not the lens.
pump (heart) helps in keeping the temperature of the
body constant. (4) Rough and dark (i.e., black) surfaces are good
Question V. Why felt is used for thermal insulation in
absorbers while shining and smooth sUrfaces are
preference to air? good reflectors of heat radiation.
(5) Glass and water vapours have the property of
Answer: No doubt air is a better bad conductor of heat
transmitting shorter wavelength heat radiations
than felt but it transfers more heat from one place to the other
through them while reflecting longer ones.
by convection (in spite of its low thennal conductivity).
(6) Heat radiations are invisible .and, like light, travel in
On the other hand, in felt the actual movement of air is
straight lines, cast shadow, affect photographic plates
stopped by trapping it in between its fine fibres and
and can be reflected by mirrors and refracted by
consequently convection is not possible, making it a better
lenses.
thennal insulator than air.
(7) It is detected by thennocouple, thennopile and
Question VI. State whether the
following statement is troe or false, --. '- D
---------"-
--',
--
-
-. radiometer, etc. The spectwm of heat radiation is
produced and studied with the help ofrock salt prism.
giving reason in brief "Water in a :
- ---- There are a number of laws related to radiation such as
closed tube is heated with one arm : ¥-

vertically placed above the lamp. Water A' : ---.-_


-_ _:- B Kirchhoff's law, Newton's law, Stefan's law, Wien's
..-
will begin to circulate along the'tube in displacement law and Planck's law. Each of these laws deals
counter clockwise direction. " with one or the other aspect of radiation and are discussed in
the forthcoming articles of this chapter. '
Answer: The statement is false, as
on heating the liquid at A will become Fig. 16.15 Question VI. Heat is gcne,:,ated continuously in an
Iigh~er and will rise up. This will push the liquid in the tube electric heater but its temperature becomes constant after
some time. Why?
HEAT TRANSFER 7'3

Answer: Initially due to absorption of heat the [BI The Law


temperature of the heater rises. When the temperature of the According to Kirchhofrs law the ratio of emissive power
beater becomes higher than that of the surroundings it starts to absorptive power is same for all surfaces at the same
losing its energy by radiation and convection. And when the temperature and is equal to the emissive power ofa perfectly
rate of loss of this heat becomes equal to the rate of heat black body at that temperature.
production in the heater, steady state is reached and the
temperature of the heater becomes constant. Thus if a and e represent the absorptive and emissive
power of a given surface while E and A for a perfectly black
§ 16.4 Kirchhoff's Law body, then according to the l<iw,
[AI Some Definitions Related to the Law e E
a=A ....(i)
(a) Perfectly Black Body: A body which absorbs all the
radiations incident on it is called ~a~k
But for a perfectly black body, A = 1; so
a perfectly black body. A perfectly
black body maintained at a !.=E .... (ii)
suitable temperature absorbs •
radiations of all wavelengths. It If emissive and absorptive powers are considered for a
cannot be realised in practice. Fig. particular wavelength A,
16.16 shows an artificial e,
black-body due to Fery. Platinum
black and surfaces coated with Fig. 16.16 ., = E)..
- .... (iii)

lamp black are good approximations of a perfectly black body Now since, E" is constant at a given temperature,
as these absorb 98% of radiations incident on them. Since a according to this law if a surface is a good absorber of a
perfectly black body neither reflects nor transmits any particular wavelength it is also a good emitter of that
radiation, it will always appear black whatever be the colour of wavelength. This in tum implies that a good absorber is a
the incident radiation. good emitter (or radiator)
(b) Absorptive Power a: Absorptive power ofa surface is
Ie) Applicatlons
defined as the ratio of the radiant energy absorbed by it in a
given time to the total radiant energy incident on it in the same (I) Sand is rough and black, so it is a good absorber and
time. As a perfectly black body absorbs all radiations incident hence iri deserts, days (when radiation from sun is
on it, the absorptive power of a perfectly black body is incident on sand) will be very hot. Now in accordance
maximum equal to unity. Absorptive power has no units and with KirchhoWs law, good absorber is a good emitter,
dimensions. so nights (when sand emits radiation) will be cold.
This is why in deserts days are hot and nights cold.
Note: The above definition refers to radiations of all wavelengths.
Sometime it becomes necessary to consider radiation of a
(2) When a shining metal ball having some black spots on
panicular wavelength. In such situations the so called spectMlI its surface is heated to a high temperature and is seen
absorptive pown is defined as the ratio of the radiant energy of in dark, the black spots shine brilliantly and the
a given wavelength absorbed by a given surface in a given time to shining ball becomes dull or invisible. The reason is
the total radiant energy of that wavelength incident in the same that the black spots on heating absorb radiations and
time on the same surface within a unit length range. It is
so emit these in the dark while the polished shining
represented by a~ and is related io absorptive power athrough the
relation part reflects radiations and absorbs nothing and so
does not emit radiations and becomes invisible in the
dark.
(3) When white light is passed through sodium vapours
(c) Emissive Power e*: For a given surface it is defined and the spectrum of transmitted light is seen, we find
as the radiant energy emitted per sec per unit area of the two dark lines in the yellow region. These dark lines
surface. However, if we consider emissive power ofa surface are due to abSClrption of radiation by sodium vapours
for a particular wavelength instead of all wavelengths, it is which it emits when heated. This is in accordance
called spectral emissive power and is represented bye" and is with Kirchhoffs law, i. e., a good emitter is a good
related to emissive power ethrough the relation e = J~ el,.dA. absorber.

Emissive power of a surface depends on its nature and Note: The sodium vapours which absorb radiation from while light will
glow yellow by the emission of absorbed radiation as good
temperature. It has units W /m 2. It is maximum for a perfectly absorber is a good emitter.
black body and minimum for a smooth shining white body .
• Actually it is the intensity of emilted radiation and some authors call it "radi(Jncj' R also.
714 PHYSICS FOR COMPETITIONS - Vol. I

(4) Fraunhofer lines are dark lines in tbe spectrum of the PI =eA aT 4
sun and are explained on the basis ofKirchhorrs law.
When white light emitted from the central part of the , and is receiving energy by absorbing radiations at a rate
sun (photosphere) passes through its atmosphere P2 =eAaTo4
(chromosphere) radiations of those wavelengths will
So net rate ofloss of energy by the body through radiation,
be absorbed by the gases present there which they
usually emit (as a good emitter is a good absorber) P=P, -P, =eAa(T' -To') .... (iv)
resulting in dark lines in the spectrum of the sun. Now ifm is the mass of the body and cits specific heat, the
(5) A red piece of glass appears red as it reflects red and rate ortoss of heat at temperature T must be
absorbs all other radiations incident on it. So if a piece dQ mc-dT
of red glass is heated to red hot it will reflect red and -= .... (v)
dt dt
bence in dark will be invisible. However, ifit is heated
to incandescence [white hot (> 800°C)], -it will reflect and if the body is cooling by radiation, Eqns. (iv) and (v) must
red end and will absorb all others, so when seen in represent the same,
dark it will glow with emission of radiations which it
i.e., mc dT ::::eAa[T4 -T04]
has absorbed, i.e., it will glow with emission of dt
radiations complementary to red (or white deficient in
red), i. e., cyan (or bluish). or dT = eAa [T' -7:'] .... (vi)
dt me 0
Similarly, if a blue glass is heated to red hot, it
will absorb red and so in the dark will appear red. i. e., when a body cools by radiation tbe rate of cooling depends
However, if it is heated to incandescence it will reflect on:
blue end and so in dark will glow with emission of (I) Na/ure of radialing surface, i.e. , greater the
radiation compiementry to blue (or white deficient in emissivity, faster will be the cooling.
blue), i.e., yellow (or reddish). (2) Area of radiating surface, i.e., greater the area of
§ 16.5 Stefan's Law radiating surface, faster will be the cooling.
According to it the radiant energy emitted by a perfectly (3) Mass of radiating body, i.e., greater the mass of
black body per unit area per sec 0.e. , emissive power or radiating body slower will be the cooling.
radiancy or intensity of black body radiation) is directly (4) Specific heal of radiating body, i.e., greater the
proportional to the fourth power of its absolute temperature, specific heat of radiating body slower will be the
i.e., cooling.
or ....(i) (5) Temperature of radiating body, i.e., greater the
temperature of radiating body faster will be the
where a is a constant called Stefan'S constant having
cooling.
dimensions [MT - 3e-4] and value 5.67 x IO-8 W/m2K4 .
(6) TemperalUre of surroundings, i.e., greater the
If the body is not a perfectly black body, temperature of surroundings slower will be the
R =eaT4 .... (ii) cooling.
where e is called emissivity* or relative emittance and has (8) Newton's Law ofCooting
value 0< e< 1 depending on the nature of surface. It has no Ifin case of cooling by radiation the temperature T of body
units and dimensions. is not very different from that of surroundings, i.e. ,T = To + llT
So the energy radiated per sec by a body of area A will be Then as T4 - T04 =[(To + 6T)4 - T04]
given by:
P=RA=eAaT 4 .... (iii)
So if the temperature of a body is doubled, the energy
or T'-TO' = TO'[[I+~~r -I]
radiated per sec by the body will become sixteen times.
Applications of the Law, or T' - To' =To'[I +4~~ -I] (usingBinomialtheorem)
(A) Cooling by Radiation
If a body at temperature T is in an environment of or T4 - T04 == 4Ti llT
temperature To « T), the body is losing energy by emitting
radiations at a rate So in the light of above, Eqn. (vi) reduces to

• It is different from emissive p?wer (which is here represented by R).


HEAT TRANSFER 715

dT = eAa 47:' AT intensity of solar radiation at ,


.. ..------............,
".

dt me 0 the surface of earth (called / ,


AT=Ae solar constant S) will be given
But as

So
= (e -eo)Oe

~ =K(e-e o)
1

CooNng curve
of a Body

90 ----- ----------------.
by:
S=~= 41tR aT4
4rtr 2
2
4nr 2
('
,
\
E i--:----J;
Sun /
,
......'
ie,T=l(;),]"4
" ' ...
K = eAo 4~o' ( .. ) TImet -
~ ......... _---_ .....
~ ....
with ~. ... VII
Fig. 16.17
me (ix) Fig, 16.18
i,e.• the rale of cooling is proportional to temperature
difference between the body and its surroundings provided But as r = 1.5 x 10 8 km, R = 7 x 10 S lan,
the temperature of the body is not very different from the
surroundings. This law is called Newton's law ofcooUng and S=2 cal =1.4 kW
2
is a'particular case of SteJan's law. cm min m2
From Newton's law of cooling it is clear that:
and
(1) Greater the temperature difference between the body
and its surroundings greater will be the rate of

~
T=(1.5XI0 8 ) , x
cooling.
1.4xIQ3
]"4 ::::'5800K
(2) Ife =e O' (de ldl) =0 7 x lOS 5.67x lO- 8
Answer
i.e.• a body can never be cooled to a temperature
lesser than its surroundings by radiation. This result is in good agreemer.t with the experimental
a a
(3) If a body cools by radiation from I to 2°C in time I , value of temperature of sun, i. e. , 6000 K. The difference in the
then taking two values is attributed to the fact that sun is not a perfectly
black body.
de =(e, -e,) and e =9 = (e, +e,)
dt t av 2 Question VII. State whether the statements given below
are troe or false giving reason in brief'
Newton's law of cooling, i.e., Eqn. (vii) becomes
(a) "Black body radiation is wltite",
tel -,e')]=K[(e, +2 e,) -eo] ....(viii) (b) "Emissivity of a body is equal 10 its absorptive
[ power"·
This form of the law helps in solving numerical Answer: (a) Troe; white radiation includes all possible
problems related to Newton's law of cooling, wavelengths. Since a black body absorbs radiation of all
(4) Newton's law of cooling can be used to compare the wavelengths, so on being heated to a suitable temperature it
specific heats of two liquids as : if equal masses of two will emit radiation of all wavelengths, i. e., black body
liquids having same surface area and finish cool from radiation is white.
same initial temperature 8, to same final temperature (b) According to Kirchhoffs law,
9 2 with same temperature of surroundings, i. e., 8 0 in .!=E a=~
time intervals tt and t2 respectively, then according to a A' 'e ,
•.. E [as A =1] .... (,.)
Eqn, (viii),

- =K2
tt c)
- or - =- II [ .
as from Eqn. (vU),Ka;- I] Now according to Stefan's law,
.£ = R Body =EaT 4 =E ....(ii)
~ ~ ~ ~ e
E R Blactbody O'T 4
(C) Temperature of tbe SUD
So from Eqns. (i) and (ii) for a body,
If R is the radius ofthe sun and T its temperature, then the
energy emitted by the sun per sec through radiation in Emissivity (E) =absorptive power (a)
accordance with Stefan 'slaw will be given by So the given statement is true.
4
P =eAaT =41tR 1aT4 ",.(viii) Note: Here e is used for emissivity to differentiate it from emissive
[Sun is a perfectly black body as it emits radiations of all powere.
wavelengths and so for it e= 1.] Question Vlll. A sphere, a cube and a thin circular plate
In reaching earth this energy will spread over a sphere of all made of the same material and having the same mass are
radius r (= average distance between sun and earth); so the initially heated to 200OC. Which of these objects ~ilI cool
716 PHYSICS FOR COMPETITIONS - VOl. I

fastest and which one slowest when left in air at room So e= 5 .


temperature? Give reason. 5.67xlO 'x(5x I0-4)x(10 3 )'
Answer: When a body cools by radiation, according to I
Stefan's law, =-=0.18 An~er
5.67
dT = eAcr [T 4 _ T04] Problem 13. A solid copper sphere '(density p and
dt me
specific heat c) of radius r at an initial temperature 200 K is
Here, m,e. e.T and To are same fOT all bodies. suspended inside a chamber whose walls are at almost 0 K.
So (dTldl) <XC Are. A What ;s lhe time required for the temperature of Ihe sphere to
Now as fora given mass area of plate is maximum while of drop to 100 K?
sphere is minimum, the disc will cool fastest while sphere Solution~ According to Stefan's law the rate of loss of
slowest. energy due to radiation ,.
Question lX. A solid sphere of copper of radius R and a P ~ eAcrT' . ..:.(i)
hollow sphere of same material of inner radius r and outer ,~, .. -
, . .
Now if the rate ofchange of temperature of a body of mass
radius Rare hea/ed to the same temperature and allowed to
m and specific heat c is (detdt), the rate of loss of heat will be
cool in the same environmem. Which of them starts cooling
faster? dQ d9 dT
"dt =-mcdi =-me,'dt [asd9=dT] .... (ii)
Answer: When a body cools by radiation, according to ,
Stefan's law: As Eqns. (i) and Cit) represents the.same
, quantity of energy
_mc dT =ooAT 4
dl
As radiation takes place from outer surface, here all (Negative sign is used as with ti~e, temperature decreases)
factors except m are same so Here m = (4/3) nr J p and A =47tr2
3
(dTldt)H = "'s = (4/3)7tR p = R3 >1 or - f' dt = rpc {'tiT
3 3 3 3 So
(dTldt)s mH (4/3).(R _r )p (R _r ) o 3ecr T4
100 ,~
i. e., the hollow sphere will cool. at a faster rate than the solid
one. or t = rpc [_1_] : :
7rpc x 10-6 second Answer
900 T J 200 72ro
Question X. State whether thefollowing statement is troe
or false: "Two spheres ofthe same material have radii 1m and Problem 14. A solid body X ofheat capacity C is kept in
4 m and temperatures 4000 K and 2000 K respectively. The an atmosphere whose temperature is TA = 300 K. At time t = 0
energy radiated per sec by theftrst sphere is more than that by the temperature of X is TO = 400 K. It cools according to
the second sphere. " Newton's law ofcooling. At time t l , its temperature isfound to
be 350K.
Answer: According to Stefan's law, energy radiated by a
At this Ume(ll \ the bodyXis connected toa Jarge box Yat
body per sec is given by
atmospheric temperature TA , through a conducting rod oj
P= ecrAT 4 =ecr4ttR 2T 4 [ashereA =4ttR 2] length L, cross-sectional area A'and'thermal conductivity K.

So !l.=(lm)2
P2 4m
x(4000K)' =1
2000K
The heat capacity of Y is so large th(lt any 'variation in its
temperature may be neglected. The cross-sectional'area A 0)
the connecting rod is small compared 10 the surface area ofX
i. e., both the spheres radiate energy at the same rate. So the Find the temperature ofX at time t = 3t l .
given statement is false. Solution: As in cooling, the temperature of bod)
Problem 12. A body which has a swface area 5.00cm 2 decreases with time.
and a temperature of7270(; radiates 300joule of energy each dQ de
-=-mc - = -C-
de
[asmc = C,given]
minute. What is its emissivity? (Stefan-Boltzmann constt. dt dt dt
8 2 4
CJ =5.67xIO- Wlm K ). and according to Newton's law of cooling,
Solution: According to Stefan-Boltzmann law, energy ~~ =BCG - Go) with 9 0 :::: temperature of surroundings
radiated per sec by a body
P = eaAT4 So -c de = B(e - 300) [ase o = 300K. given]
He«, P = (300/60) lis = 5 lis, T = 273 + 727 ~ I 000 K dt
or f'400 (9 _dO300) _Bf'dl ie
Co' , ' ,
10 (0 - 300) = _ B
8'(400-300) c ' _d6dl =K(6-6 0)
i.e., a =300 + LOOeXp( - ~ t) or I~ dt=.!.Je2- de or t = .!.log [€II-eo ]
K 91 (9-9 0 ) K e e -eo
2
and as for t = Ii'S =350;B = [C log e2lrd.
1 [60-IOJ
So here 7=-loge 1 [40-IOJ
- - andals07=-loge - -
Now when the body X is connected to a body Y (of K 40-10 K 9-10
infinite thennal capacity and at temperature 9 0) through a
conducting rod of length L, crossMsection A and thermal 108[50J=
30 IOg[~J
9- 10 0' ~= 30
3 (9-10)
conductivity K, the body X will lose heat not only by radiation
but also by conduction. And so in this situation, i. e, Answer

~~=(~n +(~7).. ,
Problem 16. A liquid takes 5 minute to cool from 80°C to
50°C. How much time will it take to cool from 60"C to 300('?
The temperature ofthe surrounding is200('. (MNR 1996)
i.e, -C d9 = KA (O -0 0 ) + B(9 -9 )
dl L 0 Solution: According to Newton's law of cooling:

or de =-(0-9 0 )[KA +BJ =_(O .-3oo)[KA + IOg, 2] (9; ~92 )=K[(9 1 ;9 2 -9 0 ) ]


dl . CLC CL I,
or IBF de
31
_DJ ] dt with D = [KA + IOge 2 ] So that ( 80~50)=K[(80;50 -20)]
350 e - 300 . '1 CL II

or 1 (O F - 300) and (60~30 )= K[( 60;30)-20]


og, (350 -300) =- 2VI"
Solving these for t, we get t = 9 minute Answer
L e., 6F =;00+50eXP[-21{ ~ + 1~~,,2)] Answer § 16.6 Wien's Displacement Law ./
When a body is heated it emits radiations of . all
Problem 15., A body cools in 7 minute from 600C 10 40OC. wavelengths. However, the intensity of radiations Of different
What will be its temperature after the next 7 minute? Th e wavelengths is different. According to Wien the product of
temperature ofsurroundings is 1~oc. [MNR 1992) wavelength corresponding to maximum intensity of radiation
Solution: According to N~wton's Jaw of cooling. and temperature ofthe body ill Kelvin is constant, i. e.,
Am T = b = constant .... (i)
> [6, ~62 ]=K[(61 ;9 2)~'eo y' The constant b is called Wiell 's constant and has value
2.89x 10-3 m-K. This law is of great importance in
So that ,[60; 40J =K[(60;40 )- 10] , astrophysics as through the analysis of radiations coming from
a distant star, by finding). m' the temperature of the star
T(= bl)' m) is detennined, e.g .• analysis of the solar spectrum
j, e., K=-.14L .... (i) shows that for solar radiation).. m = 4753 A; so temperature of
sun in accordance with Wien's law must be:
Now if after cooling from 40"C for 7 minute the 3
temperature of the body becomese, according to Newton's law 7: = ~= 2,89x 10- =6080K
of cooling, ' Sun Am 4.753 x 10 7
'.
Question XI. A blackened platinum wire, when
. [40 -9J=K[( 40 +9 )- 10]
7 2 gradually heated, appears first dull red, then blue and finally
which in.theii~t, of Eqn: (i), i.e. : K = P/1::l) gives :
white. Why?
Answer: According to Wien's law,
[40;9] =1~ [20;9J AmT =oonsn., i.e., A m =(oonsn.)ff
So when a black body is heated, ). m decreases, i. e., with
i.e., 160-49 =20 +6 or 9=28"C Answer rise in temperature the maximum intensity of radiation emitted
Alternative Solution: According to Newton's law of gets shifted towards the shorter wavelengths. So the colour of
cooling, the heated object will change from that of longer wavelength
(red) to that of shorter (blue) and when the temperature is wavelengths in the tbennal spectrum ofa black body radiation.
sufficiently high and all wavelengths are emitted, the colour The results obtained are shown in Fig. 16.19. From these
will become white (incandescent). curves it is clear that:
Question XII. On a winter night you feel warmer when T,
clouds cover the sky than when the sky is clear. Why?
Answer: In the day heat radiations coming from the sun
are absorbed by the earth while in the night the earth radiates
this heat. Now as according to Wieo's law A. m T = eaRstt. t the
temperature of hot earth is much lesser than that of sun (6000
K); the wavelength of radiations emitted by the earth will be of
much longer wavelength than that incident on it.
Now as clouds (and glass) have the property of passing
radiations of shorter wavelength and reflecting longer ones,
the heat emitted from the earth will not pass through the clouds
and thus, heat energy emitted by the earth is trapped between
the earth and the cloud causing wanner nights. (Had there been

.-
no clouds, the energy emitted by the earth will be radiated in
the sky causing cool nights.)

Not~: Similarly, if sunlight ~nters a glass room the glass acts as a one 111
way gate, i.e, it allows the heat of shor1er wavelengths (coming FIQ.18.19
from the sun) to enter the room and does not allow the longer (I) At a given temperature energy is not unifoll1l1y
wavelength radiations emitted by the heated objects in the room
to escape 'from it (by reflecting them). In this way energy is distributed among different wavelengths.
trapped in the glass house and the room will be gradually wanned (2) At a given temperature intensity of heat radiation
up. This effect is called 'green house effect' as it is used to keep increases with wavelength, reaches a maximum at a
the plants in wann atmosphere in winters by placing them in a particular wavelength and with further increase in
glass bouse exposed to sun. wavelength it decreases.
Problem 17. Two bodies A and B have thermal (3) With increase in temperature wavelength AIII
emissivi/ies 0/0.0 1 and 0.81 respectively. The outer surface corresponding to most intense radiation decreases in
areas 0/ the two bodies are same. The two bodies emit total such a way that
radiant power at the same rate. The wavelength AS A III x T =constt. [Wien's Law]
corresponding to maximum spectral radiancy in the radiation (4) For all wavelengths an increase in temperature causes
from B is shifted from the wavelength corresponding to an increase in intensity.
maximum spectral radiancy in the radiationfrom A by 1.00~m
(5) The area under the curve = I RAd').. will represent the
If the temperature 0/A is 5802K, calculate (a) the temperature
ofB and (b) wavelength AS' total intensity of radiation at a particular temperature.
This area increases with rise in temperature of the
Solution: (a) According to Stefan's law the power
body. It is found to be directly proportional to the
radiated by a body is given by:
fourth power of absolute temperature of the body, i.e.,
P=eaAT 4
R=IR.d~<xT4 [Stefan's Law]
, According to the given problem, PA =Ps withA A =A B ,
18] Planck's Law
So that eAT} =eBT;, i,e., 0.01x(5802)4 =0.81(TB )4
In order to explain the distribution of energy in the
or TB =(113)(5802) = 1934 K Answer · spe_ctrum of black body radiation Max Planck assumed' that
(b) According to Wien' s displacement law electromagnetic radiations are not emitted or absorbed
continuously by a body but in discrete bundles or packets of
~ATA =~·BT., i.• , ~. =(580211934)~A
energy called quanta. Later on these quanta of energy were
i.e., AB =31'-..04 andalso A8 -AA =lllm(given) called 'pbotons'. The energy associated with a photon is given
So ~. -(I/3V-. =1~m , i.e., ~B = 1.5~m by,
§ 16.7 Planck's Radiation Law E=hv ....(i)
[A) Distribution of Energy in the Spectrum of Black Body where v is the frequency of radiation and h Planck's constant
Langley first and later on Lummer and Pringsheim [=6.6x 10- 34 l-s]
investigated the distribution of energy amongst the different
On the basis of this quantum theory of radiation Planck SOME RADIATION CONSTANTS
has shown that energy distribution in the spectrum of black
body radiation is given by.
S. Constant Symbol Value Dimensions
No.
2
R _21thc I ....(ii) I. Solar consu. l.4x 101 Wfm2
S [Mr')
). -~ [et:!'f),J;T -I]
2. Stefan's constt. a 5.67x 10-8 W/m2K4 [Mr'e-4]
This law is called Planck's radlation law and is in
excellent agreement with experimenta I observations both for 3. Wien's constt. b 2.89x 10-3 mK. [Le]
short and long wavelengths.
4. Planck's constt. h 6.63x 10-34 J-s [ML"r']

MISCELLANEOUS SOLVED PROBLEMS

Problem 18. A wire a/length l.Om and radiuslQ-3 m is Solution: According to given problem in the light of
carrying a heavy current and is assumed to radiate as a black Wien's displacement law and Stefan's law we have
body. At equilibrium its temperature is 900 K while that ofthe respectively,
surroundings is 300 K. The resistivity of the material of the 9000T, =). 2 T2 .... (i)
wire at 300 K is n2 x 10-8 ohm-m and its temperature
and (P21P,) = (T2I T,)4 = 8 1 ....(ii)
coefficient o/resistance is 7.8 x 1 0-3/C~ Find the current in
Substituting the value ofT2 =3T1 from Eqn. (ii) in (i), we
the wire. [Given Stefan 's constant ==5.68 x-IO- 8 Wlm2 K4] get
Solution: According to Stefan's law power radiated by a 9000T, =).(3T,), i.e, ).2 =3000A
black body is given by, SO the energy of photon corresponding to 3000 A,
p= dE ="A[T 4 - To4 J=,,2lVL[T 4 -To'] E=hv=hc=6.62xlO-34 x 3 x l01
dt
A. 3xlO-7
i.e., P = (5.68 x 10-8 )(2nx 10- 3 x 1)[(900)4 - (300)4]
19
= 73.6nW ....(i) =6.62x I0- =4.14eV ....( iii)
1.6xlO- 19
Now if / is the current in the resistance R,
Now as according to Bohr's theory, energy ofan electron
p =/ 2R =12[pLlAJ
in hydrogen atom is given by
or Z2 13.6
E =-Rch - = --eV
2
i.~, P =12 (n 2 x 10-8 )[1 + 7.8 x 10-' x (900 - 300)] n2 71

xllnx (10-3 )2 So excitation energy between n = 2 and n 3 levels of =


hydrogen atom,
i.e., P == 5.681tx 10-2 /2 .... (ii)
According to the given problem Eqns. (i) and (ii) represent
M = E, -E = 13.6[_1 __
2 2 22 3
I]
the same.
5 .
, = 36 x 13.6=1.8geV .. .. (iv)
So A ......'
Problem 19. The peak emission/rom a body at a certain But according to given problem potential equivalent to
temperature occurs at a wavelength 0/9000 A. On increasing this energy, i.e., 1.89 eV is sufficient to stop most energetic
its temperature the total radiation emiUed is increased to 81 photoelectrons; the maximum kinetic energy of emitted
times. At the initial temperature when the peak radiation from photoelectrons will be 1.89 eV.
the black body is incident on a metal sUrface it does not cause Now as in case of photoelectric effect,
any photoemission from the surface. After the increase 0/ '(KE)= =hv -e<l>, i.e, e<l>=hv-(KE) ...
temperature the peak radiation from the black body causes
photoemission. To bring these photoelectrons to rest, potential So substituting the values of hv and (KE )mu. from Eq.DS.
equivalent to the excitation energy between the n == 2 to 71 == 3 (Iii) and (iv) in the above,
Bohr levels of hydrogen atom is required. Find the work e<l>=4.l4 -1.89 =2.25 eV A"""",
function of the metal. [h == 6.62 x 1O- 34 J..s and c =3 x 10 8m/s]
EXERCISE

IA) Only One Choice is Correct (c) Paper is thin


(d) Paper is good radiator ofheat
• I . Heat is transferred most rapidly by the process of :
10. During severe winter in the low temperature zones of the
(a) Conduction (b) Convection
world. the superficial parts of the lakes are frozen. leaving
(c) Radiation (d) Combustion water below. The freezing at the bottom is prevented
2. The high thermal conductivity of metal is due to free because:
electrons. The relevant electron property is :
(a) The conductivity of ice is low
(a) Its being charged (b) The water has large specific heat
(b) Its high average energy (c) The water has large latent heat affusion
(c) Its high average thennal speed (d) The temperature of the earth at the bottom of the lake is
(d) Its low volume high
3. Under steady state the temperature ofa body: II . Which of the following combinations of properties would
(a) Increases with time be mostdesirable for a cooking pot ?
(b) Decreases with time (a) High specific heat anc3; low thermal conductivity '
(e) Does not change with time and is same at all points of (b) Low specific heat and high thermal conductivity
the body (c) High spe<:ific heat and high thermal conductivity
(d) Does not cbange with lime but is different at different (d) Low specific heat and low thermal conductivity
cross-sections of the body
12. If K and (J respectively are the thennal and electrical
4. A metallic rod is continuously heated at its two ends. the
conductivities of a metal at absolUie temperature T, then:
flow of heat through the rod does not depend upon :
K K
(a) The area of cross-section of the rod (a) _ ... constant (b) - = constant
oT 0
(b) The mass of the rod
(c) I( =constant Cd) ~,.. constant
(c) Time T Kl'
(d) The temperature gradient 13. According to Ingen Hauz's experiment the thermal
5. The quantity of heat which crosses unit area of a metal plate conductivity K and length Lofthe metal rod upto which the
during conduction depends upon: wax melts are related as:
(a) The density of the metal K'
K (b) - = constant
(b) The temperature gradient perpendicular to the area (a) - - constant
L L
(e) The temperature to which the metal is heated . K
(c) - = constant (d) KL=constant
Cd) The area of the metal plate
L'
6. The coefficient of thermal conductivity of a metal depends
14. Two ends of rods of length L and radius r of the same
on:
material are kept at the same temperature. Which of the
(a) Temperature difference between the two sides fo llowing rods conducts most heat? (CPMT 1993)
(b) Thickness of the metal (a) L=50cm,r= l cm (b) L=IOOcm,r=2cm
(e) Area of the plate (c) L=25cm,r=0,5cm (d) L= 7Scm,r= 1.5 cm
(d) None of the above IS. Heat is flowing through two cylindrical rods of the; same
7. The SI unitofthermal conductivity is: (CPMT 19931 material. The diameters of the rods are in the ratio I : 2 and
1
(a) Js- mK- 1 (b) J s m- K- 1
I their lengths are in the ratio 2 : I. If the temperature
difference between their ends is the same, then the ratio of
the amounts of heat conducted through them per unit time
8. In which case does the thermal conductivity increase from will be:
left to right? (8) I : I (b) ,2 :· 1
(8) AI, Cu, Ag (b) Ag, Cu, AI
(c) I : 4 (dj ' I : 8
(c) Cu, Ag, AI (d) AI, Ag, Cu
16. When a coil of copper is kept at a certain dis~nce above a
9. Water can be heated in a cup of paper because: flame, the candle keeps burning. On placing' the coil over
(a) Paper is good conductor of heat the fl ame it is extinguished. This happens because:
(b) Paper has high specific heat
(a) The coil conducts away heat very quickly due to its 24. A calorimeter full of hot water is hung in vacuum. It will:
high conductivity which reduces the temperature of the (a) Cool by conduction (b) Cool by convection
flame to a value below its ignition temperature
(c) Cool by radiation (d) Not cool at all
(b) The coil prevents the setting up of convection current
25 , A body which absorbs all the radiations incident over it is
in air
called a :
(c) The coil reduces the amount of oxygen necessary for
(a) Black body (b) Perfectly black body
burning
(c) Good absorber (d) Good emitter
(d) The coil reduces the radiation losses
26. A black body emits:
17. Water is usually heated by:
(a) Radiation of all wavelengths
(a) Conduction (b) Convection
(b) No radiations
(e) Radiation (d) All these processes
(c) Radiations of only one wavelength
18. In natural convection, a heated portion of a liquid moves
because: Cd) Radiations of selected wavelengths
(a) Its molecular motion becomes aligned 27. The best laboratory'~pProximation to an ideal black bOdy is:
(b) Of molecular collisions within it (a) A lump of charcoal heated to high temperature
(e) Its density is less than that of the surrounding fluid (b) A glass surface'coated with coaltar
"
(d) Of currents of the surrounding fluid (c) A metal coated with a black dye
19. It is hotter at the same distance over the top of a fire than it is (d) A hollow enclosure blackened inside with soot and
on the side of it mainly because: having a small hole
(a) Heat is radiated upwards 28. "A good absorber of heat is a good radiatorofheat also" is:
(b) Air conducts heat upwards (a) Stefan's law (b) Kirchhoff's law
(c) Convection takes more heat upwards (c) Planck's law (d) Wien's law
(d) Conduction, convection and radiation all contribute 29. Which of the following statements is wrong?
significantly in transferring heat upwards (a) Rough surfaces are better radiators than smooth
20. Ventilators are provided at the top of room: surfaces
(a) To bring oxygen for breathing (b) Highly polished mirror-like surfaces are very good
radiators
(b) So that sunlight may enter the room
(c) Black surfaces are better absorbers than white ones
(c) To maintain convection currents to keep the air fresh in
the room (d) Black surfaces are better radiators than white ones
(d) To provide an outlet for carbon dioxide 30. Four identical copper cylinders are pa.inted; if they are all
21. heated to the same temperature and left in vacuum which
While measuring the thennal conductivity of a liquid we
will cool most rapidly? (MNR 1992]
keep the upper part hot and the lower cool so that:
(a) Convection may be stopped (a) Painted shiny white (b) Painted rough black
(c) Painted shiny black (d) Painted rough white
(b) Radiation may be stopped
(c) Heat conduction is easier downwards 31, In deserts, days are quite hot while nights quite cold
because:
(d) It is easier and convenient to do so
(a) A good reflector is a good emitter
22. A composite slab consists of two plates of thicknesses L1
(b) A good emitter is a good absorber
and L2 and thennal conductivities K 1 and K 2'.The cross-
sectional areas are equal. The equivalent thennal (c) A good absorber is a good reflector
conductivity is: (d) A good emitter is a good absorber and good reflector
(a) K,K2(~+L2) (b) K2~ +K2L, 32. A slab of ice is one half covered with black cloth and the
K2~ +K,L2 K,K 2 (~+L2) other half with ,white cloth. This is then placed in sunlight.
After some time the pieces of cloth are removed. Which of
(c) ~L2 (K , + K 2 ) (d) K2~ +K,L2 the following statements is correct? .
(K2~ + K,L2 ) ~L,(K, +K,) (a) Ice has melted equally under both the pieces
23. Sun's heat reaches us by: (b) More ice has melted under white cloth
(a) Conduction (b) Convection (c) More ice has melted under black cloth
(c) Radiation (d) Scattering (d) It will depend on the medium in which ice is placed
33. A polished metal with rough black spot on it is heated to (c) The walls radiate heat which the block of ice absorbs
ahout 1400 K and quickly taken to a dark room. Which one till the ice is changed into water and both acquire the
of Ihc,following ~tatements will be true? same temperature. Thereafter neither radiates any heat
(a) The spot will appear brighter than the plate (d) Ice radiates heat only after it has been melted into water
(b) The spot will appear darker than the plate 42. If the temperature of the sun is doubled, the rate of energy
(e) The spot and plate will be equally bright received on earth will be increased by a factor of:
(d) The spot and plate will not be visible in dark {eDSE 19931
34. A piece afred glass when heated in dark to red hot state will (a) 2 (b) 4
appear to be : (e) 8 (d) 16
(a) White (b) Red 43. A black body at a high temperature T K radiates energy at
(e) Green (d) , Invisible the rate E wattJm 2; when the temperature faUs to (T / 2) K
35. Fraunhofer lines ir the spectrum of sun are explained by: the radiated energy will be: (MNR 1993; DCE 20091
(a) Wien's law (b) Planc~'s law (a) EI4 (b) EI2
(e) Newto~'s law )~) Kirchhofrs law (e) 2E (d) EI 16
36. The total radialio,,: emitted by ,8 perfectly l?lack body is 44. The temperature of a body is increased from 27 uC to 12'rC.
proportional to: II(\,S 19961 The radiation emitted by it increases by a factor of:
(a) Temperature on ideal gas scale (liT 19901
(b) Fourth root oftemperarure on ideal gas scale (a) (256181) (b) (15/9)
(e) Fourth power oftemperarure on ideal gas s~ale (e) (4/3) (d) (12127)
(d) Square of temperature on ideal gas scale 45. Two spherical black bodies of radii RI and R2 having
37. Energy E radiated per sec per unit area of a black body is surface temperatures TI and T2 respectively radiate the

r
given as ~ = crT4 , (J is : same powers; then RI / R z is equal to :
(a) The density of the body
(b) Velocity of radiation (a) (;J (b) (~
(c) Stefan's.constant
(d) Universal gravitational constant
38. The unit of Stefan's constant is :
(e) (~ )' (d) (;:)'

(a) N·msee- I (K)-4 (b) wattm- 2 (K)--4 46. The spectra of a black
body at temperatures
(e) J_sm- 2 (K)-4 "Cd) ergs- 1m-3 (K)-4 327"C and 627"C are
.· d ' J'
39. A metal piece is heated up to T K; the temperature of the shown in the Fig. 16.20.Jf ~7"C
surrounding is t K. The heat loss to the surrounding due to Al and A2 be the areas
327'C
radiation is proportional to : under the two curves
respectively, the value of T-
(a) (T - t)' (b) T' _I' Fig. 16.20
A 2 lA l is:
(e) (T _ I/ 14 (d) (a) 81 . (b) 9
40. Check the correct statement: 16 4
(a) A body at O"C :mits no heat energy 27 16
(e) 8 (d)
(b) A body at-absolute zero emits no heat energy 81
(c) Heat energy emitted by a body at 200"C is twice the 47. According to Newton's law of cooling (provided the
amount of energy emitted at IOOOC difference of temperature is small) the rate of loss of heat is
proportional to :
(d) Two bodies baving the same temperature when placed
near each other neither emit any heat energy nor absorb (a) The excess temperature
41. A block of ice is placed ins!de a closed roOm; then: (b) The square of the excess temperature
(a) Only the walls of the room radiale heat energy which is (c) The cube of the excess temperature
absorbed by the ice, but ice does not radiate any heat (d) The fourth power of the excess temperature
(b) The walls oflhe room and the block of ice radiate heat 48. A sphere, a cube and a thin circular plate all made of the
all the time same mass and finish are heated to a temperature of200°C;
which of these objects will cool slowest when left in air at 56. As the temperature of a black body increases, the
room temperature ? wavelength of the emitted radiations of maximum intensity:
(a) The sphere ICPMTI993]
(b) Thecube (a) Increases
(e) The circular plate (b) Decreases
(d) All wilt cool at the same rate (c) Remains unchanged
49. A ball A has twice the diameter as another ball B of the same (d) Depends on the material of the black body
material and with same surface finish. A and B are both 57. According to Wien's displacement law: (CPMT 1993)
heated to the same temperature and allowed to cool (a) A. m =constant (b) A.mT=constaat
radiatively; then :
(c) A. m T2 ::::constant (d) A.~T:::: constant
(a) Rate of cooling of A is same as that of B
(b) Rate of cooling of A is twice that of B 58. Thep intensity of radiation emitted by the SWl has its
(e) Rate of cooling of A is half that of B maximum value at a wavelength of510 nm and that emitted
by the North stu- has the maximum value at 350 run. If
(d) Rate of cooling of A is four times that of B these stars behave like black bodies, then the ratio of the
50. Newton's law of cooling is used in the laboratory for surface temperatures of the SWl and the North Star is:
detennining : IIITl997]
(8) Specific heat of gases (b) Specific heat of liquids
(a) 1.46 (b) 0.69
(e) Latent heat of gases (d) Latent heat of liquids
(e) 1.21 (d) 0.83
51. A bucket full afhot water is kept in a room and it cools from 59. A black body is at a temperature of2800 K. The energy of
75°C to 70Ge in Tl minute, from 70°C to 65°C in T2 minute
radiation emitted by this object with wavelength between
and from 65°C to 600C inT3 minute; then: [CBSEI99S)
499 nm and 500 run isU I , between 999 nm and 1000 nm is
(a) TI =T2 =T3 (b) TI <T2 <T3 U 2 and between 1499 run and 1500 nm isU 3 . The Wien's
(c) TI > T2 > T3 (d) TI < T2 > T3 constant b = 2.SSx 10- 6 nmK. Then: [lIT 1998)
52. A body in a room cools from 90°C to SO°C in 5 minute. The (a) U I =0 (b) U, =0
time taken to cool from 70"Cto 600C in: (CPMT 1992) (c) U I >U2 (d) U 2 >UI
(a) 5 minute 60. Cloudy nights are warmer than stary ones because:
(b) Less than 5 minute (a) Clouds absorb heat in the day and supply it in thenighf
(c) More than 5 minute (b) Clouds reflect back heat radiations to the earth
(d) Less or more than 5 minute depending on the nature of (c) Heat of the earth's atmosphere increases in the
the liquid presence of clouds
53. In"a room where the temperature is 30°C a body cools from (d) The question is irrelevant
61°C to 59°C in 4 minute. The time taken by the body to 61. The colour ofa star is an indication of its:
cool from 51°C to 49°C will be: (MNR 1991)
(a) Weight (b) Distance
(a) 4minute (b) 6minute (c) Temperature (d) Size
(c) 5 minute (d) S minute 62. The temperature of the surface of the sun can be inferred
54. A pan filled with hot food cools from 50.O"C to 49.9OC in from a study of the :
5 sec. How long will it take to cool from 40.0OC to 39.9°C
(a) Solar flares (b) Solar spots
if the room temperature is 30"C? [CBSE 1994)
(c) Solar corona (d) Solar spectrum
(a) 2.5s (b) lOs
63. The spectral energy distribution of the sun (temperature
(e)20s (d)5s 6050 K) has a maximum at 4753 A. The temperature of a
55. If there are two bodies at different temperatures, the star for which this maximum is at 9506 A, is:
wavelength of radiation (in which maximum radiation (a) 6050 K (b) 3025 K
energy is distributed):
(e) 12100 K (d) 24200 K
(a) Will be greater for the body at higher temperature
64. The meaning of the Planck's hypothesis about the black
(b) Will be greater for the body at lower temperature body radiation is the relation between:
(c) Has no relation with temperature, so will be same for
(a) Frequency and velocity (b) Energy and amplitude
both the bodies
(c) Energy and frequency (d) Colou~ and frequency-
65. Which of the following curves represents spectral 72. Two spheres of the same material have radii 1 m and 4 m
distribution of energy of black body radiation? and temperatures 4000 K and 2000 K respectively. The

(a) '1~ (bd l1C ratio of the energy radiated per second by the first sphere to
that by the second is:
(a) 1;1
(0)4;1
(b) 16;1
(d) 1;9
[AIEEE 2002)

73. Infrared radiation,is detected by: (AIEEE 2002)

l'l - (d)l~
(a) spectrometre (b) pyrometer
(0) E ~ (c) nanometer -j (d) photometer
74. Which of the following is more close to a black body?
(a) Black board paint (b) Green leaves
66. A spherical black body with a radius of 12' em radiates 450 (c) Black holes ' (d) Red roses
W power at 500 K. If the radius were h~lved and the 75. Two rods are of same material and have same length and
temperature doubled, the power radiated in watt would be:
'. . (lIT 1991 (II)J
area. Heat.1.Q flows through them in 12 minutes, when they
are joined side by side. If now both the rods are joined in
(a) 225 (b) 450 parallel, then the same amount of heat .1.Q will,flow in:
(0) 900 (d) 1800 ,_ ._ (BHU 2005J
67. The power radiated by a black body is P and.it radiates (a) .24 min (b) 3 min
maximum energy around the "wavelength A. o . Now the (c) 12 min (d) 6 min
temperature of the black body is chang'ed so that it radiates 76. A hot ana a cold body are kept in vacuum separated from
maximum energy around wavelength A. o / 2 The power each other. Which of the following will cause decrease in
radiated by it will now increase by a factor of: temperature of the hot body? [AFMC ~005]
. (a) 2 (b) 16 (a) Radiation
(0) 8 (d) 64 (b) Convection 'p .. "
68. A black body at a temperature of 1640 K has the
wavelength corresponding to maximum emission equal to
1.75 ' ,... Assuming the moon to be' a perfectly black body,
the temperature of the moon,: if the wavelength
corresponding to maximum emission is 14.35 Ii is:
[Kerala PMT 2002)
(a) 100 K (b) 150 K
,;, ,
(0) 200 K _co (d) 250'K
69. According to Newton's law of cooling, the rate of cooling
of a body is proportional to (.1.a)lI, where .1.a is the
difference of th~ temperature of the body and the
. surroundings, and n is equal to: [AIEEE 2003; DCE 2009)
(a) two (b) three
(c) four , (d) one
70. In the Ingen Hauze's experiment the wax melts upto lengths
10 and 25 em on two identical rods of different materials.
The ratjo of thermal conductivities of the,two materials is:
(a) I; I Cb) I; 2
(MPPET 2002J ",
(0) 3; 2 , i. . (d); 2 ; 3
(a) I; 6.25 ", (b) 6.25; I
80. A 2 em thick slab of commercial thennocOle, 100 cm 2 in
/ (0) 1;.J2s (d) I; 2.5
cross-section 'and having thennal conductivity 2x 10-4 cal
n--AJ~Jack..body is at a temperatt.i:n~ 300 K, It emits'imergy at a sec- I, em'!;! (C' ),, 1 h~'insulating regions differing by l00"C.
. rate, which is proportional to: (AIIMS 2002)
) The quantity of heat flowing through it in a day will be:
Ca) 300 (b) (300)3
(a) 20.4 kea! (b) 43,2 ICcal
(0) (300)2 (d) (300)4 (e) 86.4 koal (d) 63.6 koal
HEAT TRANSFER 725

81. One end of a copper rod of length 1.0 m and afea of cross- 86. Two rods of same length and areas of cross-section A 1 and
section 10- 3 m2 is immersed in boiling waler and the other A 2 have Iheir ends maintained al same temperalUre. If K 1
end in ice. If the coefficient of thenna! conductivity of and K 2 are their thermal conduclivities, c t and c 2 are the
copper is 92 caVm-s-CO and the latent heat of ice is 8x 10 4 specific heats of their material and Pl' P2 their densities,
cal/kg, then the amount ofice which will melt in one minute then for the rate of flow by conduction through them to be
equal : .
is: [MNR 1994 1
• Al Kl c2 P2
(a) 9.2x 10-"kg (b) 8x 10' kg (a) - : -- -
A2 c l PI K2
(e) 6.9x 10-' kg (d) S.4 x 10- ' kg
82. Two cylinders of the same diameter. one of 1000C (e) -A , : -.K::.;:'c:!C'p,:,!...
iron and the other of silver"are placed in c1o~e I~&~e' A2 K2c2P2

contact as shown in Fig. 16.21. If the thennal "!\.J, 87. A m~tal bar of ,l~ngth I is covered ""ith some non-
A
conductivity of silver is 11 times that of iron
the" temperature of ! the interface A is A, conducting material to prevent heat losses from its surface
and the two ends are maintained at steady temperatures 9,
approximately: "
0"C and 9 2 « 9 1 ) In the steady state, the temperature 90fthe
(al" 91.7"C (b) 8O'C Fig. 16.21 bar above the surrounding at distance x from the hot end
(e) 50°C (d) 8.3°e varies as shown in the graph;

!~,
83. A wall has two layers A and B, each made of a different
material. Both the layers have the same thickness. The (a) (b)
thennal conductivity of the material of A is twice that of B;
x_
if under thermal equilibrium the temperature diffe~ence
i. across the wall is 36"C, the temperature difference across

the layer A is :
(a) ' 6°C (b) 12°C
(e) 18°C (d) we [Hint: Heat crossing thrciugh any cross-section is same lind so the
84. Two identical plates of metal are welded end to end as temperature gradient.]
shown'ih Fig. 16.22 (a); 20 ~I of heat flows;'through it in 4
88.· If in Q. 87 l the rod is not covered with' insulating material.
minute. If the plates are welded as shown in Fig. 16.22 (b),
the correct variation- of temperature with distance from hot
the same amount of heat will fl ow through the plates in :
.( . . end is expressed by ; )
. (a) graph (a) ." '.' (b) graph (b) i
.- ,.r. 'p." • It \ (e) . graph (e) (d) graph.(d) " "
.. , (a)
,'t , " [Hint: In steady state, 0 = 9ot'~, dO = ~90e"""")
~)' .. dx .
89. Three ro~s made of the same ~ 9O'C
'"f~ ~ . ~~. ~., .. material and having the same
cross-section have joined as shown O"C
(b)
FIg. 16,22
,. in the Fig. 16.23. Each rod is of
same length. The left and right 9O"C
(a) 'I minute (b) 2 minute
ends are kept at O"C and 9O"C Fig. 1~ .23
respectively. The temperature of the j unction of the three
(c) 4 minute (d) ' 16 minute
" ,1 rods will be: [liT 2001)
85. A cylinder of radius R made of a material of thermal
conductivity K l Is su~ounded by a cyli~drjcal shell of
(a) we (b) we
(e) 30°C (d) 20'e
iqner radius R and outer radius 2R made o:f aI?aterial of
thennal conductivity !G2' The two ends of the combined 90. A solid object of mass M is made from a material of specific
system Are maintained at two different temperatures. There heat capacity C and specific latent heat of fu sion L and of
is no loss of heat across the cylindrical surface and the very high thennal conductivity enters the atmosphere from
system is in steady state. The eff!!Ctive thennal conductivity outer space, its temperature being below its melting point
. by I1T. Because of atmospheric friction it absorbs energy at
of the system is; [CPMT 1990J
a constant rate k. The time before the solid becomes
(a) K, +K, (b) (K, +3K, ) / 4 completely molten is given by :
(e) K,K, /(K , + K,) (d) (3K,+K,)/ 4
726 PHYSICS FOR COMPETITIONS - Vol. I

(a) M(C+L)A T (b) M(CAT+L) (a) the hollow sphere will cool at a faster rate for all values
R R ofT

MAT~C+L)
(b) the solid sphere will cool at a faster rate for all values of
(e) M (C + L)ATR
(d) T
9'1. A metallic block A of square cross~section of area a, length (c) both spheres will cool at the same rate for all values of
d and thennal conductivity K I is in contact with two blocks T
(d) both spheres will cool at the same rate only for small
Band C each of area of cross-section ~. length ~ and
values ofT
2 2
thennal conductivitiesK 2 andK), respectively. and placed 96. Fig. 16.25 shows three rods of z
one on top oftbe other (see Fig. 16.24). If the left side of A
is at temperature T2 and the right side of Band C is at
s:tme metal and of same area of
cross· section. These rods fonn A
temperature T, (T2 > T} \ the effective thermal conductivity
of the combination is :
the sides of a right angled
isosceles triangle. Points X and x~v
FIg. .us
T, Yare maintained at constant
T,/' /' /' zr
temperature and T, respectively. Assuming heat transfer
B
K, by conduction only, temperature of point Z in steady state
K, /' will be :
K, (a) T (b) .f'2T
A C
/'
, d_ (e) T 12 (d) .f'2T
d •• J2+1
2
Fig. 18.24 97. Fig. 16.26 shows a conduction rod of non unifonn cross·
K2 +K J 2 - section, The ends of the rod are maintained at constant
(a)K 1 + 2 (b) j (K, +K2 +K,) temperatures l00"C and O°C respectively, At steady state,
temperatures of P,Q,R and S are Tp,TQ,TR and Ts
respectively, then:
(e) -;;,-KC!,:;;K£2K::.,,:;--
KI +K 2 + K 3 '\!::-:-:-r-_
92. The thickness ofice in a lake is 10 em and its temperature is
- IO"C. The thennal conductivity of ice ;;; 0.004 calcm- I
.~ \.L___p~iJia
! __--_R~!tjS----JD_~
;-
I
, _

sec-I oc- I. Densityoficc c 0.92 g/ee, latent heat of ice ::::: 80 FIG·16.2t
caVg. The time taken for thickness.of ice to be doubled is :
(a) Tp -TQ =T. -Ts (b) Tp -TQ >T. -Ts
(a) 10 6 second (b) 2.751< lOs second
(c) Tp -TQ <TR -Ts (d) none of these
(e) 104 second (d) 9.2x lOS second 98. Which of the following statements is true about th(
93 . A sphere, a cube and a disc all of the same material and of · radiation emitted by human body?
same volume are heated to 600"C and left in air. Which of (a) Radiation is emitted only during the day
these will have the lowest rate of cooling?
(b) Radiation emitted is in the infrared region
(a) cube
(c) Radiation emitted lies in the ultraviolet region anc
(b) d;sc
hence is not visible
(c) sphere (d) Radiation is emitted during the summers and abso~
(d) all will have the same rate of cooling during winters
94. The coefficients of thermal conductivity of copper, 99. A thin wire of length I and surface area A is heated to I
mercury and glass are respectively kc ,km and k such that temperature T, Its electric resistance is R, emissivity of it:
kc > k m > kg . If the same quantity of heat is t~ flow per surface is e, How much electric current should bl
second per unit area of each and corresponding temperature maintained through the wire so as to maintain it:
gradient are X c. X m and X g : temperature at T ? Assume the temperature of surrounding:
(a) X c c:.Xm =Xg (b) X c < Xm < Xg to be negligible compared toT, Use Stefan's constant as a :
eA"'T 4
(c) X c >X m > Xg (d) Xm < X c <Xg (a) ~ (b) (eaT' 1R)'12
95. A solid sphere and ~ hollow sphere of same material and

(eA~T,)"2
size arc heated to same temperature and allowed to cool in (eAaT4 )"2
the same surroundings. If the temperature difference (e) R (d)
between each sphere and its surroundings is T, then:
100. Four spheres P,Q ,R and S are of same radius but made of 105. The energy spectrum of a black body exhibits a maximum
different metals. Their densities are in the ratio 3 : 4 : 6 : 5 around a wavelength Ao. The temperature of the black body
and specific heals are in the ratio 2 : 5 : 3 : 6. These are is now changed such that the energy is maximum around a
initially kept at same temperature and placed in the same wavelength 3Ao 14. The power radiated by the black body
surroundings. The sphere which has the slowest rate of will now increase by a factor ~f:
cooling is : IKarnataka CET 2002]
(a) P (b) Q (a) 64/27 (b) 256/S1
(e) R (d) S (e) 413 (d) 1619
lOt. A sphere and cube made of same material and having same 106. Three discs A,B and C having radii 2m, 4m and 6m
surface area are heated to the same temperature and kept in respectively are coated with carbon black on their outer
the same surrounding. The ratio of their initial rates of surfaces. The wavelengths corresponding to maximum
cooling will be : intensity are 30Q nm, 400 nm and 500 nm respectively. The

(a) H'I (b)


power radiated by th~m are QA ,Qs and Qc respectively:
(IIT2004(

(e) H'I (a) QA is maximum


(c) Qc is maximum
(b) Q8 is maximum
(d) QA ""Q s =Qc
102. A rod of length / and of a uniform cross~sectional area A is 107. A system consists of two concentric spheres of radii r] and

made of a material ofnon~uniform thermal conductivity. Its r2 kept at temperatures T] and T2 respectively. The radial
thermal conductivity is temperature dependent and varies rate of flow of heat in substance between the two concentric
spheres is proportional to: ' " ' [AIEEE 2905]
as K "" !!.., where B is a constant. If ends of the rod are . ,

T (a) . ,IVl !o·


maintained at constant temperatures T, and T2 with T\ > T2, " .<r2 -Ij)
rate of flow ofheat in steady state will be :
(c) rz -.Ii Cd} ' Iri !'2 I .! '

;A/lOg[~~)
:1

(a) 'V2 " ' I ; r1


lOS. A sphe~ical body of emissivity e = 0'6and area A and placed
(e) BA
I
IOg[T,)
T,
inside a perfectly black body is maintained at temperature
T. then energy radiated per second will be : lIlT 2005]
103. i The plots of intensity' vs. (a) E=Q4crAT', ·. (b) E =QScrAT'
wavelength for three black (e) E=Q6crAT' (d) E = lOcrAT'
bodies at temperatures 'D' J; t, q

T1 ,T2 and T3 respectively


109. The graphs shown in the Fig. 19. ~~ " rj~p~~,~ent en~rgy
density.E" versus Afor three sources Suri, welding arc <).nd
are as shown. Their
temperatures are such T-
Fig. 16.27
.. ..
tungsten filament. For' Amax . correct combination will be :
,
(lIT 20051
that, (liT 20001
(a) T, >T2 >T3
(c) T2 >T3 >T\
(b) T, >T3 >T2
(d) T3 >T2 >T] ..' ~
104. Five rods of same
dimensions are arranged
as shown in Fig. 16.28.
They have thermal con~
ductivities k, ,k2 ,k3' A
C

B
Aa
.
-' Fig. 16.29
t

k4and k s' When points (a) l~ tungsten, ~~ weldil1g arc, 3~ Sun


A and B are maintained
at different temperatures, (b) l~ Sun, 2~ tungsten, 3~ welding arc
no heat flows through o (c) l~ Sun, 2, welding are, 3~ tungsten .
Fig. 16.28
the central rod if: (d) l~ welding arc, 2- Sun, 3~ tungsten
(Karnataka CET 2002] 110. A black body at 122rC emits radiations with maximum
(a) k,k4 =k zk 3 (b) k] ""k 4 andk 2 = k J intensity at a wavelength of sooO.A. If the temperature of
k, k2 the body is increased by lOOO°C, the maximum intensity
(e) - =- (d) k,k2 = k J k 4 will be observed at leDSE 2006]
k4 k3
(a) 4000 A (b) 5000 A (e) 6000 A (d) 3000 A
Ill. Assuming the sun to be a spherical body of radius R at a 117. The temperature of the two outer surfaces of a composite
temperature of T K, evaluate the total radiant power, slab, consisting of two materials having coefficients of
incident on earth, at a distance rfrom the sun: thermal conductivity K and 2K and thickness x and 4x,
(Where TO is the radius of the earth and (J is Stefan's respectively are T2 and Tl (T2 > T\} The rate of heat
constant.) IAIEEE2006) transfer through the slab, in a steady state is
rgR 2 crr4 A (T -T )K)
R2 aT 4
(
2x \ I, with/equalto: IUPSEE2007)
(a) 4.,.' (b) "
4m;2R2 crT 4 m;2R2 crT 4
(e) 0 (d) =0:,.:-"--
r2 r2
112. Three objects coloured black, grey and white can withstand
hostile conditions upto 2800°C. These objects are thrown
into a furnace where each of them attains a temperature of Fig. 16.30
2000°C. Which object will glow the brightest?
(,) 1 (b) 112
(AIIMS 2006)
(e) 2/3 (d) 1/3
(a) white object
118. Assuming Newton's law of cooling to be valid, the
(b) black object
temperature of body changes from 60 ce to ~,oce in 7 min.
(c) grey object Temperature of surrounding being loce. Find its
(d) all glow will equal brightness temperature after next 7 min. [BHU Mains 2007}
113. In which mode of transmission, heat waves travel along (,) 24°C (b) 20°C
strengths lines with the speed oflight? [WB (JEE) 2006) (e) WC (d) 28°C
(a) Natural convection (b) Forced convection
119. The surface temperature of the sun which has maximum
(c) Thennal convec.tion (d) Thermal radiation energy emission at 500 nm is 6000 K. The temperature of a
114. An ideal black body at room temperature is thrown into a star which has maximum energy emission at 400 run will
furnance . 1t is observed that: (WD (JEE) 2006] be: IKarnataka CET 2007]
(a) it is the darkest body at all times (a) 8500 K (b) 4500 K
(b) it cannot be distinguished at all times (e) 7500 K (d) 6500 K
(c) Initially it is the darkest body and later it becomes 120. Two identical rods AC and CB made of two different metals
brighte,st having thermal conductivities in the ratio 2 : 3 are kept in
(d) Initially it is the darkest body and later it cannot be contact with each other at the end C as shown in the Fig.
distinguished ,H
16.31. A is at 10QoC and Bis at 25 cC..Then thejunctionCis
115. Ari ice box made of styrofoam (thermal conductivity "" 0,01 at: (Karnataka CET 2007}
Jm- 1 s-l K':"\) is used to keep liquids cool. It has a total A C B·

wall area including lid ofO.S m 2 and wall thickness of2.0


cm. A bottle of water is placed in the box and filled with ice. Fig. 16.31
If the outside temperature is 30°C the rate of flow of heat
(,) WC (b) 60°C
into the box is: (in J-s -1) IWD (JEE) 2007)
(e) 75°C (d) 50°C
(a) 16 (b) 14
121. Which of the following ci;-c:ular rods, (given radius rand
(e) 12 (d) 10
length I) each made of the same material and whose ends
116. Assuming the sun to have a spherical outer surface of radius
are maintained at the same temperature will conduct most
r, radiating like a black body at temperature tOC, the power
heat? (JCECE 2007(
received by a unit surface, (normal to the incident rays) at a
distance R from the centre of the sun is: IUPSEE 2007) (a) r=2ro;J=21o (b) r = 2ro ;I=lo
4nr 2 0't4 r 20'(t + 273)4 (c) r=ro;l=lo (d) r=roil = 21o
(,) (b) '--'=~'- 122. Two circular discs A and B with equal radii are blackened.
R2 41tR2
They are heated to same temperature and are cooled under
100 2 r2 O't 4 identical conditions. What inference do you draw from
(e)
R' their cooling curves? IRPMT 2007}
where O'·is the Stefan's constant.
HEAT TRANSFER 729

A. (c) more than 12 minute but less than 24 minute


(d) more than 24 minute
R
B 128. A planet having average surface temperature To at an
1 average distance d from the sun. Assuming that the planet
receives radiant energy from the sun only and it loses
radiant energy only from the surface and neglecting all
other atmospheric effects we conclude:
- ('-'01
Fig. 16.32 (CMC Ludbiana 20081
(a) To f£d 2 (b) To 0: d-2
(a) A and B have same specific heats
(b) specific heat of A is less (c) To 0: d" 2 (d) To oc d- 1f2
(c) specific heat of B is less 129. A body cools from 50°C to 49°C in 5 s. How long will it
(d) nothing can be said take to cool from 40°C to 39°C? Assume temperature of
123. Two rods of copper and brass having the same length and surroundings to be 30"C and Newton's law of cooling is
cross-section are joined end to end. The free end of the valid; IBVP 20081
copper rod is at O"C and of the brass rod is at 100"C in (a) 2.5 s (b) 10 s
steady state. Ifthennal conductivity of copper is 4 times of (c) 20 s (d) 5 s
that of brass, find the temperature at the junction of two
130. Two rods of equal length and diameter have thennal
rods. (MPPMT 1001, Karna.aka CET 2007)
conductivities 3 and 4 units respectively. Ifthey are joined
(a) 20°C (b) 40 'C in series, the thennal conductivity of the combination
(c) 60 'C (d) 80 'C would be, IWB (lEE) Z0t91
124. Two walls of thickness d l and d 2 and thermal (a) 3.43 (b) 3.5
conductivities K I and K 2 are in contact. In the steady state, (c) 3.4 (d) 3.34
if the temperatures at the outer surface are TJ and T2• the
Ill. If temperature of a black body increases from _73°C to
temperature at the conunon wall is: (AIEEE 1007)
327°C. then ratio of emissive power at these two
(a) _K-,-I~-",_+_K,..2,-T.,-2 (b) K]T,d Z +K 2Tzd1 temperatures is: (Orissa JEE 20091
d, +d2 K,d 2 +K 2d, (a) 27 , I (b) 81 , I
(d) KJTJd t +K zT2d 2 (0)1,27 (d) 1,81
KId, +K 2d 2 132. The amount of heat energy radiated by a metal at
temperature Tis E. When the temperature is increased to 3T,
125. A black body emits radiation of maximum intensity for the energy radiated is : IKarnataka CET 10091
wavelength of soooA when the temperature of the body is
(a) 81E (b) 9E
l22rC. If the temperature of the body is increased by
lOOO"C, the maximum intensity would be observed at : (c) 3E (d) 27E
ICMC(VeUore) Z0081 133. The requirement for heat conduction to take place in a solid
(a) loooA (b) 2000A
j" IJ & K Z0t91
(a) density gradient (b) uniform density
(c) 3000 A (d) 4000 A
(c) temperature gradient (d) uniform temperature
(0) 5000 A
134. A unifonn rod of length L at room temperature To just fits
126. A body cools from 8O"C to 64°C in S min and same body
between two walls also at room temperature To . as shown in
cools from 8O"C to S20C in 10 min. What is the temperature
Fig. 16.33 (a), Now the left wall is maintained at room
of the surrounding? IDUMET (Med.) 2008)
temperature To and right wall i§ maintained at temperature
(a) 24°C (b) 22 °C 27"0 as shown if Fig. 16.33 (b).
(c) 28'C (d) 25'C
127. Ice starts freezing in a lake with water at O"C when the
atmospheric temperature is - I O"C. If the time taken for Icm
~
l
(!o g;- l
I!To
of ice to be fonned is 12 minute, the time taken for the (s) (b)
thickness of the ice to change from 1 cm to 2 cm will be: . Rg. 18.33

IBHU (Scree.lng) Z0081 After the rod has achieved thennal steady state, the
(a) 12 minute variation of tension in rod shown in Fig (b) as a function of
distance x from left end is best represented by :
(b) less than 12 minute
730 PHYSICS FOR COMP ETITIONS - Vol . I

(a)
T,
L L _, x=L
(b) T

x =l
pcr unit time corresponding to each segment respectively then :
E

(e) T~, (d) T~, Al A2 • .A3 ),


,.L x=L Fig . 16.34

135. Three separate segments ofcquaJ area Al ,A 2 and A) are (a) "2 > /I[ > 11) (b) 113 > 111 >n 2
shown in the energy distribution curve of a black body (c) 111 ::::11 = n) Cd) 113 > 11 2 >n [
2
radiation, Ifill ,11
2 and II) are ll11mberof photons emitted

ANSWERS

1. ee) 2. (e) 3. (d) 4. (b) 5. ·(b) 6. (d) 7. (d) 8. (a) 9. ee) 10. (a) 11 . (b) 12. (a)
13. (e) 14. (b) IS. (d) 16. (a) 17. (b) J8. (e) 19, (e) 20. (e) 21. (a) 22. (a) 23. (e) 24. (e)
25. (b) 26. (a) 27. (d) 28. (b) 29. 1(b) 30. (b) 31. (b) 32. (e) 33. (a) 34. (d) 35. (d) 36. (e)
37. (e) 3•. (b) 39. (b) 40. (b) 41. (b) 42. (d) 43. (d) 44, (a) 45. ee) 46. (a) 47. (a) 48. (a)
49. (e) 50. (b) 51. (b) 52. (e) 53. (b) 54. (b) 55. (b) 56. (b) 57. (b) 58. (b) 59. (d) 60. (b)
, .; , .,',
61. ee) 62. (d) 63. (b) M. (e) 65. Ca) 66. (d) 67. (b) 68. (e) 69. (d) 70. (a) 71. (d) 72. (a)
73. (b) 74. ee) 75. (b) 76. (a) 77. (a) 78. (e) 79. (d) 80. (e) 81. (e) 82. '(d) '3. (b) 84. (a)
'5. (b) 86. (d) 87. (b) 88. (a) '9. (b) 90. (b) 91. (d) 92. j b) 93. (e) 94. (b) 95. (c) 96. (b)
97. (e) 9.: ' (b) 99. (d) 100. (d) 101. (a) 102. (c) 103 ... (b) , !04. (a) 105. (b) to<!., (b) 107. (a) 10 •. (d)
109.
, (a) 110 (d) Ill. (d) 112. (b) 113. (d) 114. (c) 115. (c) 11 6. (d) 117. (d) 118. (d) 119. (c) 120. (a)
IZ I. (b) 122. (b) 123. (a) 124. (b) 125. (c) 126. (a) 127. (d) 128. (d) 129. (b) 130. (a) 131. (d) 132. (a)
133. (c) 134. (d) 135. (d)

I (BI More than O'ne Choice is Correct · 6. In accordance with Kirchhoff's law:
,. ,, " .',
(a) Bad absorber is bad emitter
'I. For transmission of heat from one place to the other
medium is required in : " (b) Bad absorber is goo&reflector
(a) Conduction (b) Convection (c) Bad reflector is good emitter
(c) Radiation (d) All of these (d) Bad emitter is good absorber
2. A body can be cooled by : 7. If 'a' red glass is heated and seen in dark, it will:
(a) Conduction (b) Convection (a) Appear red (b) Appear green
-. (e) Radiation (d) All of these (e) Appear white (d) Become invisible
3. In steady state: 8. The energy radiated by a body depends on:
(a) Temperature does not change with time (a) Area of body (b) Na~ure of surface
(b) All parts ofthc"body are at same temperature (c) Mass of body (d) Temperature of body
(c) There is no flow ofhcat 9. The rate of cooling ofa body by radiation depends on:
(d) There is no absorption of heat (a) Area or body
4. Which of the following terms reprcsent the same with (b) Mass of body
reference to radiation? (c) Speci fic heat of body
(a) Emissivity (b) Emissive power (d) Temperature of body and surroundings
(c) Radiancy (d) Intensity 10. If a solid sphere and a hollow sphere of the same materia:
5. Which of the foll owing have no units and dimensions? and radius are heated to the same temperature:
(a) Emissive power (b) Absorptive power (a) Both will emit equal amount of radiation per sec in th(
(c) Emissivity (d) Radi ancy beginning
HEAT TRANSFER 731

(b) Both will absorb equal amount of radiation per sec in (c) The power radiated by the body changes by a factor
the beginning . 256/81
(e) The initial rate of cooling will be same for both the (d) The power radiated by the body changes by a factor of
spheres 81/256
(d) The two spheres will have same temperature at any IS. A black body is at a temperature of2880 K. The energy of
instant radiation emitted by this object between wavelength 4990
11. Two identical objects A and B are at temperatures TA and A and 5000 A isUI , between 9990 A and 10000 AisV2 and
between 14990 A and 15000 A isU3 . The Wien's constant
TB respectively. Both objects are placed in a room with
perfectly absorbing walls maintained at a temperature T b=2.88xI0-3 mK,then:
(TA > T > Ta ). The objects A and B attain the temperature T (a) V 2 >UI (b) V 2 >U3
eventually. Select the correct statements from the following: (c) VI =U3 <V2 (d) VI <U2 <V3
(lIT 1993( 3
[Hint:J.. :::!!..=288xlO- 10,000 A, so maximum rad iation
(a) A only emits radiation, while B only absorbs it until m T 2880
both attain the temperature T corresponds to wavelength around 10,000 A]
(b) A loses more heat by radiation than it absorbs, while B 16. In a dark room with ambient temperature To, a black body is
absorbs more radiation than it emits, until they attain kept at a temperature T. Keeping the temperature of the
the temperature T black body constant (at T), sun rays are allowed to fall on
the black body through a hole in the roof of the dark room.
(e) Both A andB only absorb nl.diation, but do not errtit it,
Assuming that there is no change in the ambient
, until they attain the temperature T temperature of the room, which of the following
(d) Each object continues to emit and absorb radiation statement(s) is/are correct? lIlT 20061
even after attaining the temperature T (a) The quantity of radiation absorbed by the black body in
12. A heated 'body emits radiation which has maximum unit time will increase.
intensity at frequency vm . If the temperature of the body is (b) Since emissivity:::: absorptivity, hence the quantity of
doubled : radiation emitted by black body in unit time will
(a) The maximum intensity radiation will be at frequency increase. ' >

2 vm (c) Black body radiates more energy in unit time in the


(b) The maximum intensity radiation will be at frequency .visible spectrum.
(112) vm (d) The ref1e~ted. energy in unit ,time by:the black body
,If' 1"
a
(c) The total emitted energy will increase by factor 16 remains same.
(d) The total emitted energy will increase by a factor 2 17. A black body emits radiation at the rate P when its
13. Two bodies A and B have thennal emissivities of 0.01 and temperature is T. At this temperature the wavelength at
0.81 respectively. The outer surface areas of the two bodies which the radIation has'maximum"int ens iW ·is·Xo ' If' at
are equal. lpe ~o bodies emit, total radiant power at the another temperature T' the P9wer radiated is P' and
1.
same rate. The wavelength A8 corresponding to maximum wavelength at maximum intensity is ~ then:
spectral radiancy in the radiation from B is shifted from the 2 '
wavelength corresponding to maximum spectral radiancy
in the radil!tion from A, by 1.00~m If the temperature of A
(,) 1'.21' (b) 1'=E
2
is 5802 K: (lIT 1994( P
(a) The temp of B is 1934 K (0) P'·16P (d) p'= ~
16
(b) 1.8 =L5pm 18. There is a rectangular metal plate in which two cavities in the
(c) The temp of B is 1160 K shape of rectimgie and circle are made, as shown with
(d) The temp of B is 2901 K dimensions. P and Qare centres ofthese cavities. On heating
"
the plate, which of the following quantities increase?
[Hint: See solved Problem 17]
14. The maximum spectral radiancy of a black body b
corresponds to a wavelength A. If the temperature is now
changed so that the maximum spectral radiancy now a
corresponds to wavelength 3A/4, then:
Q
(a) The new temperature is 4/3 times the old temperature
(b) The new temperature is 3/4 times the old temperature Fig. 16.35
PHYSICS FOR COMPETITIONS- Votl

(a) area (ttr2 ) of circular cavity 3. (A): While measuring the thermal conductivity of liquid
experimentally, the upper layer is kept hot and the
(b) area(ab)ofrectangUlarcavity
lower layer is kept cold.
(e) Distance (PQ) between centre of both cavities
(R): This avoids heating of liquid by convection.
(d) width (b )ofrectangular cavity
. IAIIMS 2007)
19. Grapbs give the temperature along an x·axis that extends
4. (A): The temperature of the surface of· the sun is
directly through a wall consisting of three layers with
approximately, 6000 K. FIf we i¥.e a big lens and
different, nonzero and finite coefficient of thermal
), focus the sun rays, we,can!produce a temperature of
conductivities. The air temperature on both sides of the wall
8000 K.
are different and uniform. Out of the situations as shown by
the graph, which is/are impossible? (R): This highest, temperature can be produced
T T according to 'second law of thermodynamics.
,, IAIIMS 20071
: 'ANSWERS
(a) (b)
~::-'::,-=--'-- x ~::-'::-'-=--'--x ..'
ABC ABC 1. (b) 2. (d) ' 3. ,(.) . •. (0)
T T
ID) IntegerType Q~estlons
"
1. An insulated container is filled with a mixture of water and
(c) (d)
ice at t c ""ooe. Another container is filled with water that is
L'-;--';:-'-;:-'_x '-'-::-'::"::-,--x =
continuously boiling at 1h 100 ,o e in a series of
ABC ABC
'experiments,,_the containers are connected.by various thick
rods that pass through ,the walls of container. The rod is
ANSWERS
insulated in such a way that there is no heart loss to
1. (a) and (b); 2.AII; 3. (a) and (d); 4. (b). (c) and (d); 5. (b) surrClunding. In experiment l,,acopper,rod js'llsed, and the
ice melts in TJ ",,2 min. In experiment 2, a s!eel rod of same
and (c); 6. (a), (b) and (c); 7. (b) and (d); 8. (a), (b) and (d);

You might also like